You are on page 1of 1314

t

.me/
Exam_
Sakha_
Off
ici
al
2022-23
EDITION

National Defence Academy &


Naval Academy

More than
8000 MCQs
Complete Coverage of Syllabus
Chapterwise Division of
Previous Years’ Questions

Compiled & Edited by


Arihant ‘Expert Team’

ARIHANT PUBLICATIONS (INDIA) LIMITED


ARIHANT PUBLICATIONS (INDIA) LIMITED
All Rights Reserved

© Publisher
No part of this publication may be re-produced, stored in a retrieval system or by any means,
electronic, mechanical, photocopying, recording, scanning, web or otherwise without the written
permission of the publisher. Arihant has obtained all the information in this book from the
sources believed to be reliable and true. However, Arihant or its editors or authors or illustrators
don’t take any responsibility for the absolute accuracy of any information published and the
damage or loss suffered thereupon.
All disputes subject to Meerut (UP) jurisdiction only.

Administrative & Production Offices


Regd. Office
‘Ramchhaya’ 4577/15, Agarwal Road, Darya Ganj, New Delhi -110002
Tele: 011- 47630600, 43518550

Head Office
Kalindi, TP Nagar, Meerut (UP) - 250002
Tel: 0121-7156203, 7156204

Sales & Support Offices


Agra, Ahmedabad, Bengaluru, Bareilly, Chennai, Delhi, Guwahati,
Hyderabad, Jaipur, Jhansi, Kolkata, Lucknow, Nagpur & Pune.

ISBN 978-93-25797-96-3
PO No : TXT-XX-XXXXXXX-X-XX
Published by Arihant Publications (India) Ltd.
For further information about the books published by Arihant, log on to
www.arihantbooks.com or e-mail at info@arihantbooks.com
Follow us on
CONTENTS
NDA/NA Solved Paper 2021 II 1-29 19. Three Dimensional Geometry 218-235
NDA/NA Solved Paper 2021 I 30-58 20. Limits, Continuity and Differentiability 236-252
NDA/NA Solved Paper 2020 I & II 1-30 21. Differentiation 253-268
NDA/NA Solved Paper 2019 II 1-29
22. Application of Derivatives 269-283
NDA/NA Solved Paper 2019 I 1-27
NDA/NA Solved Paper 2018 II 1-32 23. Indefinite Integrals 284-294
NDA/NA Solved Paper 2018 I 33-62 24. Definite Integrals 295-304
NDA/NA Solved Paper 2017 II 1-14 25. Areas Bounded by Regions 305-312
26. Differential Equations 313-326
MATHEMATICS 27. Vector Algebra 327-344
1. Set Theory 3-12 28. Statistics 345-367
2. Relations and Functions 13-29 29. Probability 368-385
3. Complex Numbers 30-45 30. Miscellaneous 386-388
4. Binary Numbers 46-51
5. Sequences and Series 52-66 GENERAL ENGLISH
1. Spotting the Errors 391-433
6. Quadratic Equations and Inequalities 67-82
2. Vocabulary 434-450
7. Permutations and Combinations 83-92
3. Antonyms 451-460
8. Binomial Theorem 93-101
4. Synonyms 461-470
9. Logarithm 102-106
5. Sentence Improvement 471-480
10. Matrices 107-114
6. Sentence Completion 481-491
11. Determinants 115-132
7. Jumbled Sentences and Paragraphs 492-505
12. Measurement of Angles and
8. Comprehension 506-514
Trigonometric Ratios 133-150
13. Inverse Trigonometric Functions 151-158
GENERAL SCIENCE
14. Height and Distance 159-167
PHYSICS
15. Properties of Triangles 168-177
1. Measurement, Motion and Force 517-530
16. Coordinate System and Straight Lines 178-195
2. Work, Energy and Power 531-536
17. Circle 196-204
3. Centre of Mass and Rotational Motion 537-542
18. Conic Section 205-217
4. Gravitation 543-549 9. Oxidation, Reduction and
5. General Properties of Matter 550-557 Electrochemistry 697-703

6. Heat and Kinetic Theory of Gases 558-566 10. Non-Metals and Their Compounds 704-721
7. Thermodynamics 567-573 11. Some Important Chemical Compounds 722-734
8. Optics 574-590
BIOLOGY
9. Oscillation and Waves 591-602
1. Diversity in Living World 735-746
10. Electrostatics 603-610
2. Cell and Cell Division 747-755
11. Current Electricity 611-621
3. Constituents of Food (Biomolecules) 756-761
12. Magnetic Effects of Electric Current and
4. Structural Organisation of Plants and Animals 762-776
Magnetism 622-633
5. Plants Physiology and Reproduction 777-785
13. Nucleus and Radioactivity 634-639
6. Human System -I 786-796
14. Modern Physics 640-646
7. Human System-II 797-803
CHEMISTRY 8. Health and Diseases 804-812
1. Physical and Chemical Changes 647-649 9. Economic Importance of Biology 813-819
2. Elements, Mixtures and Compounds 650-655 10. Ecology, Biodiversity and Environment 820-828
3. Laws of Chemical Combination and
Gas Laws 656-660
GENERAL STUDIES
4. Concept of Atomic, Molecular and
1. History 831-913
Equivalent Masses 661-667
2. Geography 914-986
5. Atomic Structure and Radioactivity 668-676
3. Indian Polity 987-1020
6. Periodic Classification of Elements 677-684
4. Indian Economy 1021-1046
7. Chemical Bonding 685-689
5. General Knowledge 1047-1086
8. Acids, Bases and Salts 690-696
NDA /NA
National Defence Academy/Naval Academy

SOLVED PAPER 2021 (II)


PAPER I : Mathematics
1. If x 2 + x + 1 = 0, then what is the 3. If log10 2, log10 (2x − 1), log10 (2x + 3) 5. If p = (1111 … up to n digits), then
value of x 199
+x 200
+x 201
? are in AP, then what is x equal to? what is the value of 9 p 2 + p ?
(a) − 1 (b) 0 (a) 0 (b) 1 (a) 10n p (b) 2 p ⋅ 10n
(c) 1 (d) 3 (c) log 2 5 (d) log 5 2 (c) 10n p − 1 (d) 10n p + 1
Ê (c) Given that, log10 2, log10(2 − 1), Ê (a) Given that,
x
Ê (b) Given that,
x + x + 1=
2
0 ...(i) log10(2 x + 3) are in AP. p = (1111... upto n digits)
∴x 199
+ x 200 + x 201 = x 199(1 + x + x 2 ) ∴2 log10(2 x − 1) = log10 2 + log10(2 x + 3) = 1 + 10 + 10 2 + K + 10 n−1
= x 199 × 0 log10(2 x − 1)2 = log10 2(2 x + 3) 1 (10 n − 1)
=
= 0 ⇒ 2 2x + 1 − 2 ⋅ 2 x = 2 ⋅ 2 x + 6 10 − 1
⇒ (2 x )2 − 4(2 x ) − 5 = 0  n−1 a( r n − 1)
2. If x , y , z are in GP, then which of Q a + ar + ar + K + ar
2
=
the following is/are correct?
Let 2x = y
 r − 1 
⇒ y2 − 4 y − 5 = 0
1. ln(3x ), ln(3y ), ln(3z ) are in AP. 10 n − 1
( y − 5) ( y + 1) = 0 ⇒ p=
2. xyz + ln( x ), xyz + ln(y ), ⇒ y = 5 or y = − 1
9
xyz + ln(z ) are in HP. ⇒ 9 p = 10 n − 1
(Ignore because 2 x cannot be negative)
⇒ 9 p + 1 = 10 n
Select the correct answer using the ⇒ y = 5 ⇒ 2x = 5
code given below. ⇒ 9 p2 + p = 10 n ⋅ p
x = log 2 5
(a) 1 only Hence, option (c) is correct. ∴ Hence, option (a) is correct.
(b) 2 only
(c) Both 1 and 2 4. Let S = {2, 3, 4, 5, 6, 7, 9}. How many 6. The quadratic equation
(d) Neither 1 nor 2 different 3-digit numbers (with all 3x 2 − (k 2 + 5k ) x + 3k 2 − 5k = 0 has
digits different) from S can be made real roots of equal magnitude and
Ê (a) Given that x, y, z are in GP. which are less than 500? opposite sign. Which one of the
⇒ y 2 = xz ...(i)
(a) 30 (b) 49 following is correct?
(1) If log( 3x ), log( 3 y), log( 3 z) are in AP (c) 90 (d) 147 5
(a) 0 < k <
Then, 2 log( 3 y) = log( 3x ) + log( 3 z) Ê (c) Let S = {2, 3, 4, 5, 6, 7, 9} 3
9 y 2 = ( 9xz) ⇒ n(S ) = 7 3
(b) 0 < k < only
Three digit number less than 5
9 y 2 = ( 9xz)
3 5
500 = (c) < k <
y 2 = xz 5 3
Hence, statement (1) is correct. 3 6 5 (d) No such value of k exists.
Hence, we can say if x, y, z are in GP. = 3 × 6 × 5 = 90
Ê (d) Since, we2 know that if a quadratic
∴log x, log y, log z are in AP. ∴Option (c) is correct. equation ax + bx + c = 0 has real
⇒ xyz + log x , xyz + log y, xyz + log z Note Hundreds digit can be filled with roots of equal magnitude and opposite
are in AP. 3 choices that are 2, 3, 4. sign.
Hence, Statement (2) is wrong. Similarly, tens digit can be filled Then, b= 0 ...(i)
with 6 ways and unit digit can be and product of roots < 0 ...(ii)
∴Option (a) is correct.
filled with 5 ways.
2 NDA/NA Solved Paper 2021 (II)

In the given quadratic equation, q = − 2 p = − 2(1) 2. From 1; a, b, c are in AP.


3x 2 − ( k 2 − 5k )x + 3k 2 − 5k = 0 q = −2 ∴ b = a + d , c = a + 2d
a = 3, b = − ( k 2 + 5k ), c = 3k 2 − 5k ∴Option (b) is correct. where, d is common difference.
By Eq. (i), b=0 a b c ∴ ( b + c )2 = ( a + d + a + 2d )2
⇒ − ( k 2 + 5k ) = 0 9. If ∆ = d e f = (2 a + 3d )2
⇒ k ( k + 5) = 0 (c + a ) = ( a + 2d + a )4 = (2 a + 2d )4
4
g h i
∴ k = 0, − 5 ( a + b )2 = ( a + a + d )2 = (2 a + d )2
By Eq. (ii), Product of roots < 0 then what is Here, (c + a )4 = ( b + c )2.( a + b )2
c 3d + 5g 4a + 7 g 6g So,( b + c )2, (c + a )2,( a + b )2 are not in G.P.
<0
a 3e + 5h 4b + 7h 6h equal to? Hence, option (a) is correct.
3k 2 − 5k
⇒ <0 3 f + 5i 4c + 7i 6i 1 a
3 11. If A =  ,
(a) ∆ (b) 7∆
⇒ k ( 3k − 5) < 0 0 1
(c) 72∆ (d) − 72∆
∴ 0< k<
5
a b c where a ∈ ù, then what is
3 A 100 − A 50 − 2A 25 equal to?
Ê (d) Given, ∆ = d e f
From (i) and (ii) no such values of k exists.
g h i (a) − 2I (b) − I
Hence, option (d) is correct. (c) 2 I (d) I
3 d + 5g 4a + 7 g 6g where I is the identity matrix.
7. If an = n (n !), then what is
= 3 e + 5h 4b + 7 h 6h  
a1 + a 2 + a 3 + … + a10 equal to? 1 a
Ê (a) A =  0 1 a ∈ ù
3 f + 5i 4c + 7 i 6i  
(a) 10! − 1 (b) 11! + 1
(c) 10! + 1 (d) 11! − 1 3 d + 5g 4a + 7 g g The sequence for given matrix A is
= 6 3 e + 5h 4b + 7 h  1 na 
Ê (d) Given, an = n( n!) h
An =
= ( n + 1 − 1) ( n!) 3 f + 5i 4c + 7 i 0 1 
i  
= ( n + 1)n! − n! By C1 → C1 − 5C 3, C 2 → C 2 − 7C 3  1 100a 
∴A − A − 2 A =
100 50 25
= ( n + 1)! − n! 0 1 
3d 4a g 
∴ a1 = 2 ! − 1! = 6 3e 4b h  1 50a   1 25a 
a2 = 3 ! − 2 ! − −2
0 1  0 1 
3f 4c i    
... ... ... ... ... ... d a g  1 − 1 − 2 100a − 50a − 50a 
a10 = 11! − 10 ! =
=6× 3× 4 e h By C 2 ↔ C1 0 − 0 − 0 1− 1− 2 
b  
∴a1 + a2 + a3 + ... + a10 f c i − 2 0 
= 2 ! + 1! + 3 ! − 2 ! + 4 ! − 3 ! + K+ 11!− 10 ! = = − 2I
 0 − 2
= 11! − 1!
a d g  
= − 72 b e h By R ↔ C
Hence, option (a) is correct.
= 11! − 1
−b a−b−c
c f i
∴Option (d) is correct. a
a b c
8. If p and q are the non-zero roots of 12. If − a b − a + b − c − kabc = 0
= − 72 d e f = −72 ∆
the equation x 2 + px + q = 0, then −a −b −a−b+c
g h i
how many possible values can q
have? Hence, option (d) is correct. (a ≠ 0, b ≠ 0, c ≠ 0)
then what is the value of k?
(a) Nil (b) One 1 1 1
10. If , , are in HP, then (a) − 4 (b) − 2
(c) Two (d) Three
b+c c +a a+b (c) 2 (d) 4
Ê (b) Given quadratic equation Ê (a) Given that,
x 2 + px + q = 0 which of the following is/are correct?
1. a, b, c are in AP a −b a− b−c
and roots are p and q (non zero)
−a − a + b − c − kabc = 0
− coefficient of x 2. (b + c )2 , (c + a )2 , (a + b )2 are in GP. b
∴ Sum of roots = −a −b −a−b+c
coefficient of x 2 Select the correct answer using the
p+ q = − p ...(i) code given below. (a ≠ 0, b ≠ 0, c ≠ 0)
constant term (a) 1 only (b) 2 only R1 → R1 + R 2
∴ Product of roots = (c) Both 1 and 2 (d) Neither 1 nor 2 0 0 − 2c
coefficient of x 2
pq = q ...(ii) Ê (a) Given that, −a b − a + b − c − kabc = 0
⇒ pq − q = 0
1
,
1
,
1
are in HP. −a −b −a − b + c
b+c c + a a+ b
q ( p − 1) = 0 − 2c[( − a ) ( − b ) − ( − a )b ] − kabc = 0
⇒ b + c, c + a, a + b are in AP.
Q q ≠ 0⇒ p − 1= 0 − 2c(2 ab ) − kabc = 0
⇒ ( a + b + c ) − ( b + c ), ( a + b + c )
p=1 − kabc = 4abc
− (c + a ), ( a + b + c ) − ( a + b ) are in AP.
From Eq. (i) ⇒ k=−4
⇒ a, b ,c are in AP.
p+ q = − p Hence, option (a) is correct.
NDA/NA Solved Paper 2021 (II) 3

8n + 7 16. If sinθ and cosθ are the roots of the 19. Suppose 20 distinct points are
13. What is ∑ i n equal to, equation ax 2 + bx + c = 0, then placed randomly on a circle. Which
n =1 which one of the following is of the following statements is/are
where i = − 1? correct? correct?
(a) − 1 (b) 1 (a) a2 + b 2 − 2 ac = 0 1. The number of straight lines that
(c) i (d) − i (b) − a2 + b 2 + 2 ac = 0 can be drawn by joining any two
8n + 7
(c) a2 − b 2 + 2 ac = 0 of these points is 380.
Ê (a) Let S = ∑ in
2. The number of triangles that can
n=1 (d) a2 + b 2 + 2 ac = 0
be drawn by joining any three of
S = i + i 2 + i 3 + ... + i 8n + 7
Ê (c) Given, equation ax + bx + c = 0 …(i)
2
these points is 1140.
 ( i )8n + 7 − 1  i 4( 2n +1) + 3 − 1
=i  =i   QRoots are sin θ and cos θ Select the correct answer using the
 i −1   i −1  ∴ sin θ + cos θ = −
b code given below.
 i 3 − 1 a (a) 1 only (b) 2 only
=i   [Q i 4 n + r = i r ] and sin θ ⋅ cos θ =
c
(c) Both 1 and 2 (d) Neither 1 nor 2
 i −1 d
− i − 1 − i 2 − i On squaring both sides, we get Ê (b) Given, that there are 20 distinct
=i  = points on a circle and we have to draw a
 i − 1 
2
i −1 sin2 θ + cos 2 θ + 2 sin θ cos θ =
b
straight line by joining any two of these
2
1− i a points.
= = −1 c b2
i −1 1+ 2 = 2 Hence, number of straight lines
a a 20 × 19
14. If z = x + iy , where i = − 1, then = 20C 2 = = 190
a( a + 2c ) = b 2 2
what does the equation ⇒ a 2 − b 2 + 2 ac = 0 ∴ Statement (1) is wrong.
zz + | z | 2 + 4(z + z ) − 48 = 0 ∴Option (c) is correct. and Number of triangle
represent? 17. If C (n , 4 ), C (n , 5) and C (n , 6) are in 20 × 19 × 18
= 20C 3 = = 1140
(a) Straight line AP, then what is the value of n? 1× 2 × 3
(b) Parabola (a) 7 (b) 8 (c) 9 (d) 10 ∴Statement (2) is correct.
(c) Circle
Ê (a) C 4 ⋅ C 5 and C 6 are in AP.
n n n
(d) Pair of straight lines Hence, option (b) is correct.

z = x + iy ⇒ 2 ⋅ nC 5 = nC 4 + nC 6 20. How many terms are there in the


Ê (c) Given, 21
2 n! n! n!
∴ z = x − iy = +  a2 b2 
5 !( n − 5)! 4 !( n − 4)! 6 ! ( n − 6)! expansion of  2 + 2 + 2
∴ z + z = 2x  b a 
2 2( n!)
and z = x 2 + y2
5( 4 !)( n − 5) ( n − 6)! where a ≠ 0, b ≠ 0 ?
∴ zz +| z|2 + 4( z + z) − 48 = 0 n!  1 1  (a) 21 (b) 22
= +
( x + iy)( x − iy) + x 2+ y 2 + 4(2 x ) − 48 = 0 4 !( n − 6)!  ( n − 4) ( n − 5) 6 × 5  (c) 42 (d) 43
21
x 2 + y 2 + x 2 + y 2 + 8x − 48 = 0 2 1 1  a2 b2 
2 x 2 + 2 y 2 + 8x − 48 = 0 = + Ê (d)  2 + 2 + 2 
5( n − 5) ( n − 4) ( n − 5) 30 b a 
x 2 + y 2 + 4x − 24 = 0
2n − 8 − 5 1  a 2
21
42
= b
=  + 
a b
which represents circle.
5( n2 − 9n + 20) 30 ⇒   +   b a
Hence, option (c) is correct.  b a 
30(2 n − 13) = 5n2 − 45n + 100
15. Which one of the following is a 5n2 − 105n + 490 = 0
Since, we know that number of terms in
the expansion of ( a + b )n = n + 1
square root of 2a + 2 a + b , 2 2
n2 − 21n + 98 = 0
Hence, total number of terms
where a, b ∈ ú ? ( n − 14) ( n − 7 ) = 0 = 42 + 1 = 43
(a) a + ib + a − ib n = 14 or n = 7 ∴Option (d) is correct.
∴ Option (a) is correct.
(b) a + ib − a − ib 21. For what values of k is the system
(c) 2a + ib 18. How many 4-letter words (with or of equations 2k 2 x + 3y − 1 = 0,
(d) 2a − ib, where i = − 1 without meaning) containing two
7 x − 2y + 3 = 0, 6kx + y + 1 = 0
Ê (a) 2 a + 2 a + b
2 2 vowels can be constructed using
consistent?
only the letters (without repetition)
= 2 a + ib − ib + 2 a 2 − i 2b 2 of the word ‘LUCKNOW’? 3 ± 11 21 ± 161
(a) (b)
= ( a + ib ) + ( a − ib ) (a) 240 (b) 200 10 10
+ 2 ( a + ib ) ( a − ib ) (c) 150 (d) 120 3± 7 4 ± 11
(c) (d)
10 10
= ( a + ib + a − ib )2 Ê (a) In LUCKNOW, there are 2 vowels
Hence, square root of
and 5 consonants. Ê (b) Given equations,
∴4 letter words = 5C 2 ⋅ 2C 2 ⋅ 4 ! 2 k 2x + 3 y − 1 = 0
2a + 2 a + b =
2 2
a + ib + a − ib = 10 × 1 × 24 = 240 7x − 2 y + 3 = 0
∴Option (a) is correct. 6kx + y + 1 = 0
4 NDA/NA Solved Paper 2021 (II)

For consistency, determinant formed by 24. If sin( A + B ) = 1 and Ê (c) Given equation,
the equations π x 4 − 10x 2 + 9 = 0
2k2 3 −1
2sin( A − B ) = 1, where 0 < A, B < ,
2 Let y = x 2
7 −2 3 =0 then what is tan A : tan B equal to? ∴ y 2 − 10 y + 9 = 0
6k 1 1 (a) 1 : 2 (b) 2 : 1 ( y − 9) ( y − 1) = 0
2 k 2 ( − 2 − 3) − 3 (7 − 18 k ) (c) 1 : 3 (d) 3 : 1
⇒ y = 9 or y=1
− 1 (7 + 12 k ) = 0 Ê (d) Given, sin( A + B) = 1 x 2 = 9 or y=1
− 10k 2 − 21 + 54k − 7 − 12 k = 0 and 2 sin( A − B) = 1
x 2 = 9 or x 2 = 1
− 10k 2 − 42 k − 28 = 0 π
0 < A, B< x = ± 3, x = ± 1
2
5k 2 − 21k + 14 = 0 ∴ Sum = | 3 | + | − 3 | + | 1| + | − 1|
π
21 ± 441 − 280 Q sin( A + B) = 1 = sin
k= 2 =8
10 π Hence, option (c) is correct.
⇒ A+ B= ...(i)
21 ± 161 2
k= 27. Consider the expansion of (1 + x )n .
10 2 sin( A − B) = 1
Hence, option (b) is correct. 1 π Let p , q , r and s be the coefficients
⇒ sin( A − B) = = sin
2 6 of first, second, nth and (n + 1)th
22. The inverse of a matrix A is given
π terms respectively. What is
− 2 1  ⇒ A − B= ...(ii)
by  3 1 6 ( ps + qr ) equal to?
 2 − 2 Now, adding Eq. (i) and Eq. (ii), we get (a) 1 + 2 n (b) 1 + 2 n2
π π 4π (c) 1 + n2 (d) 1 + 4n
What is A equal to? 2A = + =
2 6 6
1 2   1 − 2 Ê (c) Given, (1 + x )
n
(a)   (b)   π π
 3 4 − 3 4  A = ,B=
3 6 In the above expansion, ( r + 1)th term
1 2  − 1 2  π π
(c)  (d)  ∴ tan A : tan B = tan : tan = nC r x r
 
Tr +1
 3 − 4  3 4 3 6 ∴ T1 = nC 0x ° , T2 = nC1x ′
− 2 1  = 3:
1
= 3:1 p=1 ...(i)
Ê (a) A= 3 1 3
 2 −  ∴ n
C1 = q
2 ∴Option (d) is correct.
n= q ...(ii)
∴ | A | = ( − 2 )  −  − = − ≠ 0
1 3 1
 2 2
25. Consider a regular polygon with Tn = C n − 1x
n n− 1
,
2
10 sides. What is the number of
1
A11 = − , A12 = −
3 Tn + 1 = nC nx n
triangles that can be formed by
2 2
joining the vertices which have no ∴ r=n ...(iii)
A 21 = − 1, A 22 = − 2 common side with any of the sides ∴ s=1 ...(iv)
 − 1 − 1 of the polygon? ∴ ( ps + qr ) = 1 ⋅ 1 + n ⋅ n = 1 + n2
∴adj A =  2 
(a) 25 (b) 50
 3  = (1 + n2 )
 − − 2  (c) 75 (d) 100
2 Hence, option (c) is correct.
 − 1 − 1 Ê (b) Given number of sides ( n) = 10
1 1  2 
∴A =−1
adj A = 3π
| A| 1  3
− −

− 2
Number of triangles which have no 28. Let sin − 1 x + sin − 1 y + sin − 1 z =
2  2  common side with any of the sides of the 2
 1 1 n ( n − 4) ( n − 5) for 0 ≤ x , y , z ≤ 1. What is the value
polygon =
= 2 2  = 1 2  3!
3    of x 1000 + y 1001 + z 1002 ?
2   3 4
 2  ∴Number of triangles
(a) 0 (b) 1
Hence, option (a) is correct. 10 (10 − 4) (10 − 5)
= (c) 3 (d) 6
23. What is the period of the function
6 −1 −1 −1 3π
10 × 6 × 5 Ê (c) Let sin x + sin y + sin z =
f ( x ) = ln(2 + sin 2 x )? = 2
6 Which is only possible when,
π
(a) (b) π (c) 2 π (d) 3π = 50 π π
2 sin− 1 x = , sin−1 y = ,
Hence, option (b) is correct. 2 2
Ê (b) f( x ) = ln(2 + sin2 x ) π
26. Consider all the real roots of the sin−1 z =
Q Period of sin2 x is π. equation x 4 − 10x 2 + 9 = 0. 2
and f( π + x ) = log {2 + sin2 ( π + x )} ⇒ x = 1, y = 1, z = 1
What is the sum of the absolute
= log {2 + sin2 x } ∴ x 1000 + y1001 + z1002
= f( x )
values of the roots?
(a) 4 (b) 6 = 1 + 1+ 1= 3
Hence, period of ln(2 + sin2 x ) = π
(c) 8 (d) 10 Hence, option (c) is correct.
∴Option (b) is correct.
NDA/NA Solved Paper 2021 (II) 5

29. Let sin x + sin y = cos x + cos y for 31. What is the value of the following? Ê (c) Given that first term of AP = p
 x y   3  3  ⇒ a= p
all x , y ∈ ú. What is tan  +  cot sin − 1   + cot − 1    …(i)
 2 2   5   2  Where, a denotes first term.
equal to? 6 7 and a3 = q , a5 = 3
(a) (b)
(a) 1 (b) 2 17 16 ⇒ a + 2d = q ...(ii)
(c) 2 (d) 2 2 16 17 a + 4d = 3 ...(iii)
(c) (d)
7 6 ∴ pq = a( a + 2d )
Ê (a) Given that,
sin x + sin y = cos x + cos y ∀ x ∈ ú  − 1 3 + cot − 1 3  = ( 3 − 4d ) ( 3 − 4d + 2d )
Ê (a) cot sin 
x +  x − y  5 2
2 sin 
y = ( 3 − 4d ) ( 3 − 2d )
 cos    
 2   2   2
= 9 − 18 d + 8 d 2
  1 −  3   
x + y  x − y
= 2 cos   ⋅ cos   ⇒ cot cot −1
  5 
+ cot − 1 
3 Let f = 9 − 18d + 8d 2
 2   2    
3 2 f ′ = 0 − 18 + 16d
   
x + y −  
2 sin  x y 
5
 = − 18 + 16d
 ⋅ cos    
 2   2  For maxima and minima
⇒ =1  1 − x2 
 x + y x − y f′ = 0
2 cos   ⋅ cos   [Q sin− 1 x = cot − 1  
 2   2    ⇒ − 18 + 16d = 0
 x 
x + y
tan x y  xy − 1  18 9
 = 1 or tan +  = 1 and cot x + cot − 1 y = cot − 1  
⇒ d = = .
 2  2 2 16 8
 x + y 
Hence, option (a) is correct. Now, f ′′ = 16 (Positive)
 3 
⇒ cot cot −1   + cot −1  
4 9
  2   So, f will be maximum at d = .
 0 2  3
30. Let A =  
8
− 2 0   4 × 3 − 1 
 
Hence, option (c) is correct.

and (mI + nA )2 = A , where m, n are ⇒ cot cot − 1  3 2  34. Consider the following statements
  4 + 3  in respect of the roots of the
positive real numbers and I is the   3 2 
identity matrix. What is (m + n ) 1 6 equation x 3 − 8 = 0
equal to? ⇒ =
1
17 17 1. The roots are non-collinear.
(a) 0 (b) 6 2. The roots lie on a circle of unit
2
3 Hence, option (a) is correct. radius.
(c) 1 (d)
2 32. Let 4 sin 2 x = 3, where 0 ≤ x ≤ π. Which of the above statements
 0 2 is/are correct?
Ê (d) Let A =  − 2 0  and ( mI + nA ) = A What is tan3x equal to?
2
(a) 1 only (b) 2 only
  (a) − 2 (b) − 1 (c) Both 1 and 2 (d) Neither 1 nor 2
where, I is Identify matrix (c) 0 (d) 1
Ê (a) x3 − 8 = 0
1 0  0 2 Ê (c) Given that 4 sin x = 3, 0 ≤ x ≤ π
2
QmI + nA = m + n ⇒ ( x − 2 ) ( x 2 + 2 x + 4) = 0
0 1 − 2 0
    3
∴ sin2 x = x = 2, 2ω, 2ω 2
m 0   0 2 n 4
= + − 1 + 3i
 0 m  − 2 n 0  π 2π Where, ω =
    ⇒ sin x =
3
= sin or sin
2 3 3 2
 m 2 n
= π 2π Hence, roots are non-collinear and will
− 2 n m  ∴ x = ,
  lie on a circle of 2 unit radius.
3 3
∴( mI + nA ) = A
2
3π Hence, option (a) is correct.
 m 2 n  m 2 n  0 2  ∴ tan 3x = tan
=
− 2 n m  − 2 n m  − 2 0
3 35. Let the equation sec x ⋅ cosec x = p
     = tan π = 0 have a solution, where p is a
 m2 − 4n 2   0 2 2π
4mn
= Also tan 3x = tan 3  
positive real number. What should
 − 4mn m2 − 4n2   − 2 0   3  be the smallest value of p?

= tan 2 π = 0 1
⇒ 4mn = 2 and m2 − 4n2 = 0 (a) (b) 1
∴Option (c) is correct. 2
1
mn = and m = ± 2 n (c) 2
2 33. Let p , q and 3 be respectively the (d) Minimum does not exist
When, m = 2 n first, third and fifth terms of an AP.
1 Let d be the common difference. If Ê (c) sec x ⋅ cosec x = p
(2 n) ( n) = 1
2 the product ( pq ) is minimum, then ⇒ =p
1 sin x ⋅ cos x
n = ± ⇒m = ± 1 what is the value of d?
2
2 3 ⇒ =p
(a) 1 (b) 2 sin x cos x
1 3
∴ m + n = 1+ = 8
2 2 9 9 2
(c) (d) = p
Hence, option (d) is correct. 8 4 sin 2 x
6 NDA/NA Solved Paper 2021 (II)

Where, sin 2 x ∈ [− 1, 1] ⇒ We cannot say A = B 41. Let ABC be a triangle.


If sin 2 x = 1 e.g., if A = φ and B = {1, 2} If cos 2A + cos 2B + cos 2C = − 1,
Then p = 2 will be the smallest value. ⇒ A − B = φ and A ≠ B then which one of the following is
Hence, option (c) is correct. ∴Option (a) is correct. correct?
36. For what value of θ, where 38. Consider three sets X , Y and Z (a) sin A sin B sin C = 0
π having 6, 5 and 4 elements (b) sin A sin B cos C = 0
0 < θ < , does sin θ + sin θ cos θ
2 respectively. All these 15 elements (c) cos A sin B sin C = 0
attain maximum value? are distinct. Let S = ( X − Y ) ∪ Z . (d) cos A cos B cos C = 0
π π How many proper subsets does S
(a) (b)
have? Ê (d) Given that, ABC is a triangle and
2 3
π π cos 2 A + cos 2 B + cos 2C = − 1
(c) (d) (a) 255 (b) 256
4 6 (c) 1023 (d) 1024 ⇒ 1 + cos 2 A + cos 2 B + cos 2C = 0
 2 B + 2C 
Ê (b) Let P = sin θ + sin θ ⋅ cos θ Ê (c) Given, n( X ) = 6, n( Y ) = 5, n( Z ) = 4 ⇒ 2 cos 2 A + 2 cos  
 2 

dP
= cos θ + cos 2 θ − sin2 θ S =(X − Y ) ∪ Z
2 B − 2C 
dθ Since, all 15 elements are different. ⋅ cos   =0
 2 
n( X − Y ) = 6
dP
For maxima-minima = =0 Hence,

and n(S ) = 6 + 4 = 10 ⇒ 2 cos 2 A + 2 cos( B + C )
cos θ + cos 2 θ − sin2 θ = 0 ⋅ cos( B − C ) = 0
⇒ Number of proper subsets of S
cos + cos 2 θ − 1 + cos 2 θ = 0
= 2 10 − 1 {Q A + B + C = 180}
2 cos 2 θ + cos θ − 1 = 0
= 1024 − 1 ⇒ 2 cos 2 A + 2 cos (180° − A )
(cos θ + 1)(2 cos θ − 1) = 0
= 1023 ⋅ cos( B − C ) = 0
1
⇒ cos θ = − 1 or cos θ = ∴Option (c) is correct.
2 ⇒ 2 cos 2 A − 2 cos A ⋅ cos( B − C ) = 0
π 39. Consider the following statements ⇒ 2 cos A [cos A − cos( B − C )] = 0
⇒ θ= π or θ=
3 in respect of relations and functions.
⇒ 2 cos A[cos(180° − ( B + C )]
π
θ = π can be neglected as θ ∈  0,  .
1. All relations are functions but all
 2 functions are not relations. − cos( B − C )] = 0
π 2. A relation from A to B is a ⇒ − 2 cos A[cos( B + C ) + cos( B − C )] = 0
∴ θ=
3 subset of Cartesian product  B +C + B − C
⇒ − 2 cos A  2 cos
Hence, option (b) is correct. A × B.  2
37. Consider the following statements 3. A relation in A is a subset of B+ C − B + C 
⋅ cos  =0
in respect of sets. Cartesian product A × A . 2 
1. The union over intersection of Which of the above statements are
− 4 cos A ⋅ cos B ⋅ cos C = 0
sets is distributive. correct?
(a) 1 and 2 (b) 2 and 3 ⇒ cos A ⋅ cos B ⋅ cos C = 0
2. The complement of union of two (c) 1 and 3 (d) 1, 2 and 3 ∴ Option (d) is correct.
sets is equal to intersection of
their complements. Ê (b) Since, we know that relations can be 42. What is the value of the following
function iff every element has unique determinant?
3. If the difference of two sets is image.
equal to empty set, then the two cos C tan A 0
Hence, first statement is wrong.
sets must be equal. sin B 0 − tan A
If R : A → A then R ⊆ A × A
Which of the above statements are 0 sin B cos C
and if R : A → B then R ⊆ A × B
correct ?
(a) 1 and 2 (b) 2 and 3 Hence, 2nd and 3rd statements are (a) − 1
(c) 1 and 3 (d) 1, 2 and 3 correct. (b) 0
∴Option (b) is correct. (c) 2 tan A sin B sin C
Ê (a) Since, we know that distributive
property for sets A, B and C. 40. If log10 2 log 2 10 + log10 (10x ) = 2, (d) − 2 tan A sin B sin C
A ∪ ( B ∩ C ) = ( A ∪ B) ∩ ( A ∪ C ) then what is the value of x? cos C tan A 0
and ( A ∪ B)′ = A ′ ∩ B′ (a) 0 (b) 1
(By De Morgan’s Law)
Ê (b) Let ∆ = sin B 0 − tan A
(c) log 2 10 (d) log 5 2 0 sin B cos C
U Ê (b) Given that,
A B ∆ = cos C [0 + sin B tan A ]
log10 2 ⋅ log 2 10 + log10(10)x = 2
− tan A [sin B cos C − 0]
1
log10 2 × + x log10 10 = 2 = tan A sin B cos C
log10 2
− tan A sin B cos C
1+ x = 2
∴∆ = 0
C ⇒ x =1
Hence, option (b) is correct.
Also, if A − B = φ ∴Option (b) is correct.
NDA/NA Solved Paper 2021 (II) 7

43. Suppose set A consists of first 250 25K 2 + 1 + 10K 48. If i = − 1, then how many values
= 1 + 5K
natural numbers that are multiples 16
does i − 2n have for different n ∈¢?
of 3 and set B consists of first 200 25K 2 + 10K + 1 = 80K + 16
even natural numbers. How many ⇒ 25K 2 − 70K − 15 = 0 (a) One (b) Two
elements does A ∪ B have? (c) Four (d) Infinite
5K 2 − 14K − 3 = 0
(a) 324 (b) 364 5K − 15K + K − 3 = 0
2 Ê (b) Given that, i = − 1
(c) 384 (d) 400 5K ( K − 3) + 1( K − 3) = 0 To find ( i )− 2n
Ê (c) Given that, A consists of first 250 ( K − 3) ( 5K + 1) = 0 Let i = r(cos θ + i sin θ)
natural numbers that are multiple of 3. 1 π
⇒ K = 3 or − ⇒ r = 1, θ =
∴ A = {3, 6, 9, 12, ..., 750}, 5 2
n( A ) = 250 π π
Hence, option (c) is correct. ∴ i =  cos + i sin 
Set B consists of first 200 even natural  2 2
numbers.
46. Consider the digits 3, 5, 7, 9. What −2n
π π
∴ B = {2, 4, 6, 8, ..., 400}
is the number of 5-digit numbers ∴ ( i )−2n =  cos + i sin 
formed by these digits in which  2 2
∴ A ∩ B = {6, 12, ..., 750} − 2 nπ 
each of these four digits appears? = cos   − 2 nπ 
Q n( A ∪ B) = n( A ) + n( B) − n( A ∩ B)  + i sin 
(a) 240 (b) 180  2   2 
= 250 + 200 − 66 (c) 120 (d) 60 = cos( nπ) − i sin( nπ)
n( A ∪ B) = 384
Ê (a) Given digits are 3, 5, 7, 9.  − 1; if n is odd
Hence, option (c) is correct. = ( − 1)n = 
Since, the number of ways to find 5-digit  1; if n is even
44. Let Sk denote the sum of first k numbers = 5! ∴Option (b) is correct.
S 30 but using 3, 5, 7, 9 every time one-digit
terms of an AP. What is a b c
S 20 − S10 will be repeated. 49. If x = ,y = ,z = ,
Hence number of 5-digit numbers with b−c c −a a−b
equal to? 5!
digit 3 repeated = then what is the value of the
(a) 1 (b) 2 2!
(c) 3 (d) 4
following ?
Number of 5-digit numbers with digit 5 1 −x x
Ê (c) Let’s take first K terms are first K repeated =
5!
natural numbers. 2! 1 1 −y
K ( K + 1) 1 1
∴ SK = Number of 5-digit numbers with digit 7 z
2 5!
repeated = (a) 0 (b) 1
30( 31) 2! (c) abc (d) ab + bc + ca
S 30 2
Consider = Number of 5-digit numbers with digit 9 a b c
S 20 − S10 20(21) 10(11)
− 5! Ê (a) Given, x = , y= ,z=
repeated = b −c c−a a−b
2 2 2! 1 −x x
930 ∴Total 5-digit numbers
= =3 ∴ 1 1 −y
310 5! 5! 5! 5!
∴Option (c) is correct. = + + + 1 z 1
2! 2! 2! 2!
5 × 4 × 3 × 2 ! R 2 → R 2 − R1 and R 3 → R 3 − R1
= 4 × 
45. If the roots of the equation
 1 −x
4 x 2 − (5k + 1)x + 5k = 0 differ by  
x
2!
= 0 1+ x − y − x
5 × 4 × 3 × 2 !
unity, then which one of the = 4 ×   0 z+ x 1− x
following is a possible value of k?  2! 
(a) − 3 (b) − 1 = 240 = (1 + x ) (1 − x ) − ( − y − x ) ( z + x )
(c) −
1
(d) −
3
Hence, option (a) is correct. = 1 − x 2 + x 2 + ( y + z)x + yz
5 5  b c   a 
47. How many distinct matrices = 1+  +   
Ê (c) Given equation, c − a a − b  b − c
exist with all four entries taken from
4x 2 − ( 5K + 1)x + 5K = 0 ...(i)  b c 
{l, 2}? +  × 
Let the roots are α and β. c − a a − b
(a) 16 (b) 24
− ( − ( 5K + 1))
α +β = (c) 32 (d) 48  ab − b 2 + c 2 − ac   a 
4 = 1+    
5K + 1 Ê (a) Given digits are 1, 2.  ( a − b ) (c − a )   b − c
= a b  
4 Let matrix = + 
bc

c d  − a) ( a − b )
5K    (c
α ⋅β =
4 QEach entries can filled with 2 ways. ( b − c ) ( a − b − c )a
= 1+
Given that, α −β =1 Therefore, number of distinct matrices ( a − b ) (c − a ) ( b − c )
bc
(α + β) 2 − 4αβ = 1 =2 ×2 ×2 ×2 +
(c − a ) ( a − b )
2 = 16
 5K + 1 − 4 5K  = 1 ( a − b ) (c − a ) + a 2 − ab − ac + bc
    Hence, option (a) is correct. =
 4   4  ( a − b ) (c − a )
8 NDA/NA Solved Paper 2021 (II)

ac − a 2− bc + ab + a 2− ab − ac + bc If Eq. (i) passes through (2, 1) C = ( x , y)


=
( a − b )(c − a ) ⇒ 4 + 1 + 4g + 2 f + c = 0
=0 4g + 2 f + c + 5 = 0 ...(iv) B(2, 2)
Hence, option (a) is correct. On solving Eqs. (ii), (iii) and (iv)
50. Consider the following in respect of Eqs. (ii) – Eq. (iii)
the matrix ⇒ 14g − 34f + 4 = 0
A C
1 1 1 ⇒ 7 g − 17 f + 2 = 0 ...(v) (0, 0) (x, y)
A = 1 1 1 Eq. (iv) − Eq. (iii)
 
1 1 1 ⇒ 8g − 16f − 80 = 0 Q∆ABC is equilateral triangle.
1. Inverse of A does not exist g − 2 f − 10 = 0 ...(vi) ∴ AB = BC
2. A 3 = A Eq. (v) − 7 × Eq. (vi)
⇒ (2 − 0)2 + (2 − 0)2
− 3f + 72 = 0
3. 3A = A 2
f = 24 = (2 − x )2 + (2 − y)2
Which of the above are correct ?
From Eq. (vi)
(a) 1 and 2 only (b) 2 and 3 only ⇒ 8 = 4 + x 2 − 4x + 4 + y 2 − 4 y
(c) 1 and 3 only (d) 1, 2 and 3 g = 2 f + 10
⇒ x + y 2 − 4x − 4 y = 0
2
...(i)
1 1 1 g = 58
1 1 1 and AB = AC
Ê (c) Given matrix, A = From Eq. (iv) c = − 4g − 2 f − 5
 
1 1 1 ⇒ (2 − 0)2 + (2 − 0)2
= − 232 − 48 − 5
Q A = 11 ( − 1) − 11( − 1) + 11
( − 1) = 0 c = − 285 = ( x − 0)2 + ( y − 0)2
∴ A − 1 doesn’t exist. ∴ Centre = ( − g , − f ) ⇒ 8 = x 2 + y2 ...(ii)
1 1 1 1 1 1 = ( − 58, − 24) and AC = BC
Now, A 2 = 1 1 1 1 1 1
   ∴Option (d) is correct.
⇒ x 2 + y 2 = ( x − 2 )2 + ( y − 2 )2
1 1 1 1 1 1
52. If r is the radius of the circle, then
 3 3 3 ⇒ x 2 + y 2 = x 2 + y 2 − 4x − 4 y + 8
which one of the following is correct?
=  3 3 3 = 3 A ⇒ x + y=2 ...(iii)
  (a) r < 10 (b) 10 < r < 30
 3 3 3 (c) 30 < r < 60 (d) r > 60 From Eqs. (ii) and (iii)
 3 3 3 1 1 1 8 = x 2 + (2 − x )2
Ê (d) Since, the centre of the above circle
and A 3 =  3 3 3 1 1 1 x + 4 + x − 4x = 8
2 2
= ( − 58, − 24)
  
g = 58, f = 24 and c = − 285 x 2 − 2x − 2 = 0
 3 3 3 1 1 1
2 ± 4+ 8
 9 9 9 ∴ Radius = g 2 + f 2 − c x =
=  9 9 9 ≠ A 2
  = ( 58)2 + (24)2 − ( − 285) 1± 3
 9 9 9 =
= 3364 + 576 + 285 2
Hence, option (c) is correct.
= 4225 Hence, third vertex atleast one irrational
r = 65 unit. coordinate.
Directions (Q.Nos. 51 and 52)
Consider the following for the next two ∴Option (d) is correct. ⇒ Area will also be irrational.
questions that follow. Hence, option (c) is correct.
A circle is passing through the points Directions (Q.Nos. 53 and 54) 54. The difference of coordinates of the
(5, − 8), (− 2, 9) and (2,1). Consider the following for the next two
third vertex is
questions that follow.
51. What are the coordinate of the The two vertices of an equilateral triangle (a) 0 (b) 3
centre of the circle? are (0, 0) and (2, 2). (c) 2 2 (d) 2 3
(a) (− 2, − 50) (b) (− 50, − 20)
53. Consider the following statements. 1± 3
(c) (− 24, − 58) (d) (− 58, − 24) Ê (d) Since, x =
1. The third vertex has atleast one 2
Ê (d) Given that, circle is passing through
the points ( 5, − 8), ( − 2, 9) and (2, 1). irrational coordinate. and y=2 − x
Let the equation of circle be 2. The area is irrational. 1± 3
Which of the above statements y=2−
x 2 + y 2 + 2 gx + 2 fy + c = 0 ...(i) 2
is/are correct?
If Eq. (i) passes through ( 5, − 8) 3± 3
(a) 1 only y=
∴ 25 + 64 + 10g − 16f + c = 0 (b) 2 only 2
⇒ 10g − 16f + c + 89 = 0 ...(ii) (c) Both 1 and 2 1+ 3 3− 3
If x = ,y=
If Eq. (i) passes through ( − 2, 9) (d) Neither 1 nor 2 2 2
4 + 81 − 4g + 18f + c = 0 Ê (c) Let vertex of A = ( 0, 0) ∴ | x − y| = 2 3
− 4g + 18f + c + 85 = 0 …(iii) B = (2, 2 ) Hence, option (d) is correct.
NDA/NA Solved Paper 2021 (II) 9

Directions (Q. Nos. 55 and 56) Directions (Q. Nos. 57 and 58) On solving Eqs. (i) and (ii)
Consider the following for the questions Consider the following for the next two x = 2, y = 1
that follow. questions that follow. ∴ A = (2, 1)
The coordinates of three consecutive The equations of the sides AB, BC and CA On solving Eqs. (ii) and (iii)
vertices of a parallelogram ABCD are of a triangle ABC are x − 2 = 0 , y + 1 = 0 y = − 1, x = 6
A(1, 3), B(− 1, 2) and C(3, 5). and x + 2y − 4 = 0 respectively. ∴ c = ( 6, − 1)
55. What is the equation of the 57. What is the equation of the altitude ∴Circumcentre will be mid-point of
diagonal BD? through B on AC? 2 + 6 1 − 1
AC =  ,  = (4, 0)
(a) 2 x − 3y + 2 = 0 (a) x − 3 y + 1 = 0 (b) x − 3 y + 4 = 0  2 2 
(b) 3x − 2y + 5 = 0 (c) 2 x − y + 4 = 0 (d) 2 x − y − 5 = 0 ∴Option (a) is correct.
(c) 2 x − 3y + 8 = 0
Ê (d) Equation of AB ⇒ x − 2 = 0 ...(i)
(d) 3x − 2y − 5 = 0 Directions (Q. Nos. 59 and 60)
Equation of BC ⇒ y + 1 = 0 ...(ii)
Ê (c) Given, vertices of parallelogram are Equation of AC ⇒ x + 2 y − 4 = 0 ...(iii) Consider the following for the next two
A = (1, 3), B = ( − 1, 2 ), C = ( 3, 5) questions that follow.
A
D C(3, 5) The two ends of the latus rectum of a
parabola are (− 2, 4) and (− 2, − 4).
x+2y–4=0
x–2=0 D 59. What is the maximum number of
parabolas that can be drawn
through these two points as end
B y+1=0 C
A(1, 3) B(–1, 2) points of latusrectum?
On solving Eq. (i) and Eq. (ii), we get (a) Only one (b) Two
ABCD is a parallelogram, then
x = 2, y = − 1 (c) Four (d) Infinite
Mid-point of AC = Mid-point of BD ∴ B = (2, − 1) Ê (b) The maximum number of parabolas
1 + 3 3 + 5   −1 + x 2 + y 
⇒  ,  = ,  Slope of AC =
− coefficient of x that can be drawn through
 2 2   2 2  coefficient of y
−1 + x 4 1
⇒ = ⇒x = 5 m1 = −
2 2 2
2 + y 8 −1 −1
and = ⇒y=6 ∴Slope of altitude BD = = =2
2 2 1
m1 −
∴ Point = ( 5, 6) 2
∴ Equation of BD, ∴Equation of altitude BD drawn from B
on AC having slope 2.
where B = ( − 1, 2) and D = ( 5, 6)
y + 1 = 2( x − 2 ) These two points as end points of
6−2 latusrectum = two
y−2 = ( x + 1) y + 1 = 2x − 4
5 − ( − 1) ∴Option (b) is correct.
⇒ 2x − y − 5 = 0
4 60. Consider the following statements
y − 2 = ( x + 1) Hence, option (d) is correct.
6 in respect of such parabolas
6 y − 12 = 4x + 4
58. What are the coordinates of
circumcentre of the triangle? 1. One of the parabolas passes
⇒ 2x − 3y + 8 = 0 through the origin (0, 0).
(a) (4, 0) (b) (2, 1)
∴Option (c) is correct. (c) (0, 4) (d) (2, − 1) 2. The focus of one of the parabolas
lies at (− 2, 0).
56. What is the area of the Ê (a) Slope of line AB Which of the above statements
parallelogram? − coefficient of x −1
⇒ = =∞ is/are correct?
3 coefficient of y 0
(a) 1 sq. unit (b) sq. units (a) 1 only (b) 2 only
2 0
5 Slope of line BC = − = 0 (c) Both 1 and 2 (d) Neither 1 nor 2
(c) 2 sq. units (d) sq. units 1
2 ∴Angle between AB and BC
Ê (a) Let parabola-1 passes through
origin.
Ê (c) The vertices of parallelogram are =
∞ − ( 0) π
A(1, 3), B( − 1, 2), C( 3, 5) and D(5, 6). 1 + ∞ ⋅ ( 0) π (2,4)
∴Area = | Area of ∆ABC+ Area of ∆ACD| π
1 ⇒ tan θ = ∞ θ=
= | 12 ( − 5) − 1( 5 − 3) + 3( 3 − 2 )| 2
2 (0,0)
Q∆ABC is right angled triangle.
1
+ | 1( 5 − 6) + 3( 6 − 3) + 5( 3 − 5)| ∴Circumcentre will lie on Hypotenuse
2
AC i.e. x + 2 y − 4 = 0 at mid point. (–2,–4)
1
= |−3 − 2 + 3 + ( −1) + 9 − 10 | Equation of AB : x − 2 = 0 ...(i)
2
Equation of AC : x + 2 y − 4 = 0 ...(ii) It’s equation will be
= 2 sq. units
Equation of BC : y + 1 = 0 . ..(iii) y 2 = − 4ax
∴Option (c) is correct.
10 NDA/NA Solved Paper 2021 (II)

Whose leading points of latusrectum will 4 2 66. Let a, b and c be unit vectors such
Hence, −=−
be ( − a, 2 a ) and ( − a, − 2 a ) 6 3 that a × b is perpendicular to c.
∴ a=2 where ( − ) indicates externally division. If θ is the angle between a and b,
∴ Focus = ( − 2, 0) Hence, option (c) is correct. then which of the following is/are
Hence, option (a) is correct. 64. The number of spheres of radius r correct?
61. The locus of a point P ( x , y , z ) which touching the coordinate axes is 1. a × b = sinθ c
moves in such a way that z = 7 is a (a) 4 (b) 6 2. a ⋅ ( b × c) = 0
(a) line parallel to X-axis (c) 8 (d) infinite Select the correct answer using the
code given below.
(b) line parallel to Y-axis Ê (c) Since, we know that the number of
(c) line parallel to Z-axis spheres of radius r touching the (a) only (b) 2 only
(d) plane parallel to xy-plane coordinate axes is 8. (c) Both 1 and 2 (d) Neither 1 nor 2

Ê (d) Since, point moves in a plane z = 7 Hence, option (c) is correct. Ê (c) Given, that a , b and c be unit vectors
such that a × b is perpendicular to c.
which will be parallel to xy-plane. 65. ABCDEFGH is a cuboid with base
Z
angle between a and b = θ
ABCD. Let A (0, 0, 0), B(12, 0, 0),
∴ a × b = a b sin θ ⋅ c
Z=7 C (12, 6, 0) and G (12, 6, 4 ) be the
{Qa × b is the vector perpendicular to a
vertices. If α is the angle between
and b}
AB and AG. β is the angle between
= 1 ⋅ 1 ⋅ sin θ ⋅ c
x AC and AG, then what is the value
= sin θ c
of cos 2α + cos 2β?
Since, a, b and c are lying on the same
40 64
(a) (b) plane.
y 49 49
∴ a ⋅ (b × c ) = 0
Hence, option (d) is correct. 120 160
(c) (d) Hence, option (c) is correct.
49 49
62. Consider the following statements 67. If a + 3b = 3$i − $j and
1. A line in space can have Ê (b) Given, ABCDEFGH is a cuboid.
infinitely many direction ratios. QAngle between AB and AG = α 2a + b = $i − 2$j , then what is the
2. It is possible for certain line that d.r’s of AB = (12 − 0, 0 − 0, 0 − 0) angle between a and b?
the sum of the squares of = (12, 0, 0) π
(a) 0 (b)
direction cosines can be equal to 6
d.r’s of AG = (12 − 0, 6 − 0, 4 − 0)
sum of its direction cosines. π π
= (12, 6, 4) (c) (d)
Which of the above statements 3 2
is/are correct? a1a2 + b1b 2 + c1c 2 $ $
(a) 1 only (b) 2 only ∴ cosα = Ê (d) Given, a + 3b = 3 i − j ...(i)
a12 + b12 + c12 a22 + b 22 + c 22
(c) Both 1 and 2 (d) Neither 1 nor 2 and 2a + b = $i − 2 $j ...(ii)
144 + 0 + 0
= Eq. (i) × 2 – Eq. (ii)
Ê (c) Since, we know that A line in space
can have infinitely many direction ratio 12 2 12 2 + 6 2 + 4 2
(2a + 6b ) − (2a + b ) = 2( 3$i − $j)− ( $i − 2 $j )
and also it is possible for certain line that 144 6
cos α = = 5b = 5$i
the sum of the squares of direction 12 × 14 7
cosine can be equal to sum of its ∴ b = $i
direction cosines. Now, d.r’s of AC = (12, 6, 0)
d.r’s of AG = (12, 6, 4) From Eq. (i)
For example, (1, 0, 0) is the direction
cosines for X-axis. 144 + 36 a = ( 3$i − $j ) − 3b
∴ cos β =
∴ l = 1, m = 0, n = 0, then 180 × 14 a = 3$i − $j − 3$i
l 2 + m2 + n2 = 12 + 0 2 + 0 2 a = − $j
180 180
Hence, option (c) is correct. = =
180 × 14 14 ∴ a ⋅ b = a b cos θ
63. The xy-plane divides the line where θ is the angle between a and b.
∴cos 2α + cos 2β
segment joining the points
= 2 cos 2 α − 1 + 2 cos 2 β − 1 ( − $j ) ⋅ ( $i ) = 1 ⋅ 1 ⋅ cos θ
(− 1, 3, 4) and (2, − 5, 6).
 6 2  180  2  0 = cos θ
= 2    + 
(a) internally in the ratio 2 : 3
  −2 π
(b) internally in the ratio 3 : 2 7  14   ∴ θ=
 2
(c) externally in the ratio 2 : 3
= 2 
(d) externally in the ratio 2 : 1 36 180  Hence, option (d) is correct.
+  −2
 49 196  68. If (a + b ) is perpendicular to a and
Ê (c) Since, we know that xy-plane divides
the line segment joining the points (x 1, 72 90 magnitude of b is twice that of a,
=+ −2
 z  49 49 then what is the value of
y1, z1) and (x 2, y2, z2 ) in the ratio  − 1  ⋅
 z2 
=
162 − 98 64
= ( 4a + b ) ⋅ b equal to?
A B 49 49 (a) 0 (b) 1
(–1,3,4) (2,–5,6) Hence, option (b) is correct. (c) 8| a |2 (d) 8| b|2
NDA/NA Solved Paper 2021 (II) 11

Ê (a) Given, a + b is perpendicular to a . 71. If y = (1 + x ) (1 + x 2 ) (1 + x 4 ) (1 + x 8 ) an + b n


Ê (c) Given, nlim where a > b > 1
∴ (a + b ) ⋅ a = 0 dy →∞ an − b n
(1 + x ), then what is
16
at x = 0

2
+ b ⋅a = 0  b 
n
a n 1 +   
a dx
equal to?  a
a ⋅ b = − | a |2 ⇒ lim   Q b < 1
b =2 a
(a) 0 (b) 1 n→ ∞ n  a 
and n  b
(c) 2 (d) 4 a 1−  
∴ ( 4a + b ) ⋅ b = 4a ⋅ b + b ⋅ b   a  
Ê (b) Given, y = (1 + x ) ( 1 + x ) (1 + x )
2 4
= 4( − a
2
)+ b
2
1+ 0
∴ = =1
2
(1 + x 8 )(1 + x 16 ) 1− 0
= − 4a + (2 a ) 2
dy
∴ = (1 + x ) (1 + x 2 ) (1 + x 4 ) ∞
∴  b =0
=−4 a
2
+ 4 a
2 dx
(1 + x 8 ) ⋅ (16x 15 )  
 a
=0 + (1 + x )(1 + x ) (1 + x ) ( 8x )(1 + x )
2 4 7 16
Hence, option (c) is correct.
Hence, option (a) is correct. + (1 + x ) (1 + x 2 ) ( 4x 3 ) (1 + x 8 ) (1 + x 16 )
  x
69. Let a , b and c be three vectors such + (1 + x ) (2 x ) (1 + x 4 ) (1 + x 8 ) (1 + x 16 ) 75. Let f ( x ) = 1 + 2k , 0 < x < 2
a , b and c are coplanar. Which of + (1) (1 + x 2 ) (1 + x 4 ) (1 + x 8 ) (1 + x 16 )  kx , 2≤ x < 4
the following is/are correct? ∴
dy
= 0 + 0 + 0 + 0 + 1= 1 If lim f ( x ) exists, then what is the
x→2
1. (a × b ) × c is coplanar with dx x = 0
a and b Hence, option (b) is correct.
value of k?
2. (a × b ) × c is perpendicular to (a) − 2 (b) − 1
72. If y = cos x ⋅ cos 4 x ⋅ cos 8x , then (c) 0 (d) 1
a×b 1 dy π
Select the correct answer using the what is at x = equal to? 1 + x ; 0 < x < 2
code given below. y dx 4 Ê (d) Let f( x )  2 k
 kx ; 2≤x<4
(a) 1 only (a) − 1 (b) 0 (c) 1 (d) 3
(b) 2 only Q lim f( x ) exists
Ê (a) Given, y = cos x ⋅ cos 4x ⋅ cos 8x x →2
(c) Both 1 and 2 ⇒ lim f( x ) = lim ( kx )
(d) Neither 1 nor 2 ∴log y = log cos x + log cos 4x x → 2− x → 2+
+ log cos 8x
lim  1 +
Ê (c) Given that a, b, c are coplanar. x 
 = lim ( kx )
n → 2−  2 k  x → 2+
On differentiating w.r.t ‘x’.
⇒ a ⋅ (b × c ) = 0 = b ⋅ (c × a )
1 dy 1 1
= c ⋅ (a × b ) ⋅ = ( − sin x ) + 1+
2
= 2k
y dx cos x cos 4x
⇒ [a b c ] = [b c a ] = [c a b ] = 0 2k
1 2
Q (a × b ) × c = − c × (a × b ) ( − 4 sin 4x ) + ( − 8 sin 8x ) = 2k − 1
cos 8x 2k
= − [(c ⋅ b )a − (c ⋅ a )b ]
= − tan x − 4 tan 4x − 8 tan 8x 2 = 4k 2 − 2 k
= − [[c b a ] − [c a b ]]
4k − 2 k − 2 = 0
2
= 2 [c a b ] = 0  1 dy  π
∴  = − tan− 4 tan π 2k2 − k − 1 = 0
Hence, {(a × b ) × c} ⋅ (a × b ) = 0  y dx  at x =
π 4
4 − 8 tan2 π (2 k + 1) ( k − 1) = 0
⇒ (a × b ) × c is perpendicular to a × b.
= − 1− 0 − 0= − 1 1
and coplanar with a and b. ⇒ k = 1 or k = −
Hence, option (a) is correct. 2
Hence, option (c) is correct. Hence, option (d) is correct.
70. If the position vectors of A and B 73. Let f ( x ) be a polynomial function
76. Consider the following statements
are ( 2 − 1) $i − $j and $i + ( 2 + 1)$j such that fof ( x ) = x 4 . What is f ′(1)
in respect of f ( x ) = | x | − 1 :
respectively, then what is the equal to? 1. f ( x ) is continuous at x = 1.
(a) 0 (b) 1 (c) 2 (d) 4
magnitude of AB? 2. f ( x ) is differentiable at x = 0.
(a) 2 2 (b) 3 2 Ê (c) Given, f( x ) be a polynomial such that Which of the above statements
(c) 2 3 (d) 3 3 fof( x ) = x 4 is/are correct?
$ $
Ê (c) Given that, OA = ( 2 − 1) i − j To find f ′ (1) = ? (a) 1 only (b) 2 only
Q fof( x ) = x 4 ⇒ f( x ) = x 2 (c) Both 1 and 2 (d) Neither 1 nor 2
and OB = $i + ( 2 + 1) $j
∴ f ′(x ) = 2 x Ê (a) Given, f( x ) = | x | − 1
∴ AB = OB − OA
⇒ f ′ (1) = 2 × 1 = 2 Since, modulus function is continuous.
= (1 − 2 + 1)$i + ( 2 + 1 + 1)$j Hence, option (c) is correct. ⇒ f( x ) is continuous at x = 1
AB = (2 − 2 )$i + ( 2 + 2 )$j an + bn and| x | is not differentiable if x = 0
74. What is lim
− bn
n → ∞ an ∴f( x ) = | x | − 1 is not differentiable at
∴| AB | = (2 − 2 )2 + (2 + 2 )2 x =0
where a > b > 1, equal to?
= 4+ 2 − 4 2 + 4+ 2 + 4 2 (a) − 1 (b) 0
Hence, statement (1) is correct and (2) is
false.
= 12 = 2 3 (c) 1
(d) Limit does not exist Hence, option (a) is correct.
Hence, option (c) is correct.
12 NDA/NA Solved Paper 2021 (II)

[x ]
77. If f ( x ) = , x ≠ 0, Ê (b) Given that, f( x ) = 1 − sin x Ê (c) Given function,
[x ] y
Since, the range of sin x is [− 1, 1].
where [ ] denotes the greatest − 1 ≤ sin x ≤ 1 (0, 5) f(x)=x2+1
integer function, then what is the − 1 ≤ − sin x ≤ 1
right-hand limit of f ( x ) at x = 1? 1 − 1 ≤ 1 − sin x ≤ 1 + 1
(a) − 1 (0, 2)
0 ≤ 1 − sin x ≤ 2
(b) 0 (0, 1)
(c) 1 ∴Range = [0, 2 ]
(d) Right-hand limit of f(x )at x = 1does Hence, option (b) is correct. x
not exist 80. What is the slope of the tangent of
Ê (c) Given that, f( x ) =
[x ]
,x ≠ 0 π f( x ) = x 2 + 1
y = cos − 1(cos x ) at x = −
in (1, 2)
| x| ?
4 ⇒ y = x2 + 1
[x ]
= lim (a) − 1 (b) 0 x 2 = ( y − 1)
x → 1+ | x| (c) 1 (d) 2 Which is the equation of parabola with
x = 1 + h, where h → 0 −1
Ê (a) Given that, y = cos (cos x ) vertex ( 0, 1).
[1 + h] 1 x =1
∴ lim = = =1 Since range of cos − 1 x is [0, π ]. At
h→ 0 [1 + h] | 1 + 0 | f(1) = 12 + 1 = 2
∴ y = cos −1 (cos x ) = − x,
Hence, option (c) is correct.
if x ∈ ( − π, 0) f(2 ) = 2 2 + 1 = 5
78. Consider the following statements π Hence, maximum value of the function in
Q x =− (1, 2 ) is 5 and minimum value is 2.
in respect of the function. 4
 1 ∴ y=−x
Hence, option (c) is correct.
f ( x ) = sin  2  , x ≠ 0.
x  dy 83. If f ( x ) satisfies f (1) = f ( 4 ), then
⇒ = −1 4
1. It is continuous at x = 0, dx what is ∫ f ′ ( x ) dx equal to?
if f (0) = 0.
1
∴Slope of tangent = − 1
2 (a) − 1 (b) 0
2. It is continuous at x = . Hence, option (a) is correct.
(c) 1 (d) 2
x 81. What is the integral of f(1) = f( 4)
Which of the above statements Ê (b)
f ( x ) = 1 + x 2 + x 4 with respect to 4
is/are correct? ∴ ∫ f ′ ( x )dx = [f( x )]14 = f( 4) − f(1)
(a) 1 only (b) 2 only x 2?
1
(c) Both 1 and 2 (d) Neither 1 nor 2 x 3
x 5
= f(1) − f(1) = 0
(a) x + + +C
π
 1 3 5
Ê (b) Given that, f( x ) = sin 2  , x ≠ 0 x3 x5 84. What is ∫ 2 e ln(cos x )dx equal to?
x (b) + +C 0
At x = 0, 3 5 (a) − 1 (b) 0
x4 x6 (c) 1 (d) 2
LHL lim sin 2  (c) x 2 + + +C
1 π π
x → 0− x  4 6 2 2
Ê (c) Let I = ∫ e ∫ (cos x )dx
4 6
x x ln(cos x )
dx =
= value in between − 1 and + 1 (d) x 2 + + +C
2 3 0
π
0
RHL lim sin  2 
1 π
x → 0+ x  Ê (d) Given function, f( x ) = 1 + x + x
2 4
= [sin x ]02 = sin
− sin 0 = 1
2
= value in between − 1 and + 1 ∴Integral of f( x ) w.r.t x 2. Hence, option (c) is correct.
∴ Limit doesn’t exist ⇒ f( x ) is not = ∫ (1 + x 2 + x 4 ) ⋅ 2 x dx
continuous at x = 0. 85. If ∫ 1 − sin 2xdx = A
= ∫ (2 x + 2 x 3 + 2 x 5 )dx
At x =
2
, sin x + B cos x + C , where
π x4 x6 π
= x2 + + +C 0 < x < , then which one of the
π
lim sin 2  = sin =
1 1 2 3 4
x→
2 x  4 2 Hence, option (d) is correct. following is correct?
π
 2  1 1 82. Consider the following statements (a) A + B = 0 (b) A + B − 2 = 0
Also f  = sin = (c) A + B + 2 = 0 (d) A + B − 1 = 0
 π  2 
2
2 in respect of the function
  f ( x ) = x 2 + 1 in the interval (1, 2). Ê (b) Given that,
 π
∴f( x ) is continuous at x =
2 1. The maximum value of the ∫ 1 − sin2 xdx = A sin x + Bcos x + C ,
π function is 5. π
where 0 ≤ x ≤ .
Hence, option (b) is correct. 2. The minimum value of the 4
function is 2. Let
79. What is the range of the function
I = ∫ cos 2x + sin2x − 2 sin x ⋅ cos x dx
f ( x ) = 1 − sin x defined on entire Which of the above statements
is/are correct?
real line?
(a) 1 only (b) 2 only
I= ∫ (cos x − sin x )2 dx
(a) (0, 2) (b) [0, 2] π
(c) Both 1 and 2 (d) Neither 1 nor 2 {Q cos x > sin x when 0 < x < }
(c) (− 1, 1) (d) [− 1, 1] 4
NDA/NA Solved Paper 2021 (II) 13

I = ∫ (cos x − sin x ) dx 88. What is the differential equation of 91. Consider the following statements
I = sin x + cos x + C B
y=A− ?
in respect of the function
1
= A sin x + B cos x + C x f (x ) = x + ⋅
∴ A = 1, B = 1 (a) xy2 + y1 = 0 (b) xy2 + 2 y1 = 0 x
(c) xy2 − 2 y1 = 0 (d) 2 xy2 + y1 = 0 1. The local maximum value of
∴ A + B − 2 = 1+ 1− 2 = 0
Ê (b) Given, y = A −
B f ( x ) is less than its local
Hence, option (b) is correct. x minimum value.
86. What is the order of the differential On differentiating w.r.t ‘ x ’ 2. The local maximum value of
= 0 − B  − 2  = 2 f ( x ) occurs at x = 1.
equation of all ellipses whose axes dy 1 B
are along the coordinate axes? dx  x  x Which of the above statements
(a) 1 (b) 2 x2
dy
=B is/are correct?
(c) 3 (d) 4 dx (a) 1 only (b) 2 only
Ê (b) Since, the equation of ellipse On differentiating again w.r.t ‘x’. (c) Both 1 and 2 (d) Neither 1 nor 2
x2 y2 d 2y 1
+ 2 =1 x2 ⋅ +
dy
⋅ (2 x ) = 0 Ê (a) Given, f( x ) = x +
a 2
b dx 2 dx x
1 2
∴There are 2 variable a and b. ⇒ x ( xy2 + 2 y1 ) = 0 f ′(x ) = 1 − and f ′′( x ) = 3
x2 x
∴Order of the differential equation = 2 ⇒ xy2 + 2 y1 = 0
For critical points f ′ ( x ) = 0
Hence option (d) is correct. Hence, option (b) is correct.
1
1− 2= 0
87. What is the degree of the π  x
89. What is ∫ ln  tan  dx equal to?
x
differential equation of all circles 0  2 x =±1
touching both the coordinate axes At x = 1, f ′′( x ) = 2 > 0
1
in the first quadrant? (a) 0 (b) (c) 1 (d) 2
2 ⇒ f( x ) is minimum at x = 1
(a) 1 (b) 2 π ⇒ f(1) = 2
 x
(c) 3 (d) 4 Ê (a) Let I = ∫ ln  tan  dx ...(i) At x = − 1 f ′′( x ) = − 2 < 0
2
Ê (b) If r be the radius of circle. 0
⇒ f( x ) is maximum at x = − 1
π
 π − x 
I = ∫ ln  cot 
Since, the circle touching both the
  dx ⇒ f( − 1) = − 2
coordinate axes in the first quadrant.   2 
0 Hence, statement (1) is correct and (2) is
y π false.
 x 
I = ∫ ln  cot    dx ...(ii)
 2  ∴Option (a) is correct.
0

r Adding Eq. (i) and Eq. (ii) 92. What is the maximum area of a
(r, r)
π rectangle that can be inscribed in a
  x
+ ln  cot   dx
∫ ln  tan 2 
r x
2I = circle of radius 2 units?
x  2 
0
(a) 4 sq. units (b) 6 sq. units
π π
= ∫ ln tan ⋅ cot  dx = ∫ ln(1) dx
x x (c) 8 sq. units (d) 16 sq. units
∴Centre = ( r, r ) and radius = r  2 2  Ê (c) Let x and y be the length and
0 0
∴Equation of circle 2I = 0 breadth of rectangle respectively.
( x − r )2 + ( y − r )2 = r 2 ∴ I=0
D C
x 2 + y 2 − 2 xr − 2 yr + r 2 = 0 …(i) Hence, option (a) is correct. 2
2 x + 2 yy′ − 2 r − 2 ry′ = 0 90. Where does the tangent to the curve O y
r(1 + y′ ) = x + yy′ y = e at the point (0, 1) meet X -axis?
x 2
x + yy′ (b) (− 1, 0) A B
r= (a) (1, 0) x
1 + y′
(d)  − , 0
1
(c) (2, 0) In ∆ABC,
Putting the value of r in Eq. (i)  2 
x 2 + y 2 = 16
( x + yy′ ) ( x + yy′ ) Ê (b) Given curve, y = e
x
x 2 + y2 − 2 x −2y ⇒ y = 16 − x 2
1 + y′ 1 + y′ dy
∴ = ex
2 ∴Area of rectangle, A = xy
 x + yy′  dx
+   =0  dy  A = x 16 − x 2
 1 + y′    = e0 = 1
 dx  at(0, 1) dA x
(1 + y′ )2 x 2 + (1 + y′ )2 y 2 − 2 x ( x + yy′ ) = 16 − x 2 + (− 2 x )
∴ Equation of tangent at (0, 1). dx 2 16 − x 2
(1 + y′ ) − 2 y( x + yy′ ) (1 + y′ )
y − 1 =   16 − x 2 − x 2 16 − 2 x 2
dy
+ ( x + yy′ )2 = 0 ( x − 0) = =
 dx  at(0, 1)
(1 + y′ ) ( x + y ) − 2( x − y) ( x + yy′ )
2 2 2 16 − x 2 16 − x 2
y − 1= x
(1 + y′ ) + ( x + yy′ )2 = 0 For maximum A,
Since, ( − 1, 0) satisfies above equation.
Hence, the degree of the differential dA
=0
Hence, option (b) is correct.
equation is 2. dx
14 NDA/NA Solved Paper 2021 (II)

16 − 2 x 2 98. If the function f ( x ) = x 2 − kx is


x
Ê (a) Let y1 = e and y2 = e
e x
⇒ =0 ⇒ 16 − 2 x 2 = 0
16 − x 2 dy1 x dy monotonically increasing in the
∴ = ee ⋅ e x , 2 = e x
⇒ x2 = 8 ⇒ x =±2 2 dx dx interval (1, ∞ ), then which one of
dy1 ee ⋅ e x
x
the following is correct?
−4x . 16 − x 2 − (16 − 2 x 2 )
x
⇒ = = ee (a) k < 2 (b) 2 < k < 3
1 dy2 ex
( −2 x ) (c) 3 < k < 4 (d) k > 4
d 2A 2 16 − x 2 ∴Option (a) is correct.
Now, = Ê (a) Let the function f( x ) = x − kx is
2
dx 2 16 − x 2 95. What is the condition that monotonically increasing in (1, ∞ ).
− 4x (16 − x 2 ) + x (16 − 2 x 2 ) f ( x ) = x 3 + x 2 + kx has no local
= ⇒ f ′(x ) ≥ 0
(16 − x 2 )3/ 2 extremum? ⇒ 2x − k ≥ 0
− 3x (16 − x 2 ) (a) 4k < 1 (b) 3k > 1 ⇒ k ≤ 2 x in (1, ∞ ) at lower value
=
(16 − x 2 )3/ 2 (c) 3k < 1 (d) 3k ≤ 1 at x =1
Ê (b) Given that, f( x ) = x + x + kx
3 2
 d 2A  − 3(2 2 )(16 − 8) k<2
 2 = (Negative)
 dx  at x = 2 2 (16 − 8)3/ 2 Qf( x ) has no local extremum. Hence, option (a) is correct.
⇒ f ′(x ) ≠ 0 99. What is the area bounded by
∴ y = 16 − (2 2 ) = 2 2 2
⇒ 3x 2 + 2 x + k ≠ 0 y = [ x ], where [⋅] is the greatest
Area = 2 2 × 2 2 = 8 sq. units integer function, the X -axis and the
for no extremum, D < 0
Hence, A is maximum at x = 2 2. lines x = − 1.5 and x = − 1.8 ?
⇒ (2 )2 − 4( 3) ( k ) < 0
(a) 0.3 sq. unit (b) 0.4 sq. unit
93. What is ∫
dx ⇒ 4 − 12 k < 0
equal to? (c) 0.6 sq. unit (d) 0.8 sq. unit
x ( x + 1)
2
3k > 1
1  x2  Ê (c) Given, y = [x ]
∴Option (b) is correct.
(a) ln  2  +C
2  x + 1
Y
96. If f ( x ) = 2x , then what is
 x2  f ′( x )
(b) ln  2  + C 10
∫2
–1.8 –1.5
dx equal to?
 x + 1 f (x )
–2 –1
3  x2  X
(c) ln  2  +C (a) 4ln2 (b) ln4 1 2
2  x + 1 (c) ln5 (d) 8ln2 –1
1  x 2 + 1 Ê (d) Given, f( x ) = 2
x
–2
(d) ln   +C
2  x 2  10
f ′(x )
∴ ∫ 2 = [ln 2 ]2
dx = [ln f( x )]10 x 10
−1.5 − 1.5
∫ x (x 2 + 1)
dx
Ê (a) Let I= f( x )
2
= [x ln 2 ]210 ∴ Area = ∫ [x ] dx = ∫ (− 2 ) dx
−1.8 − 1. 8
1 A Bx + C = 10 ln 2 − 2 ln 2
Q = + 2 = − 2( x )−− 1.5
x ( x 2 + 1) x x +1 = 8 ln 2 1.8

∴Option (d) is correct. = − 2( − 1.5 + 1.8) = − 0.6


(by using partial fraction) ∴ Area = 0.6 sq. unit
0
Ax 2 + A + Bx 2 + Cx
1
x ( x + 1)
2
=
x ( x + 1)
2
97. If
∫− 2 f ( x ) dx = k , then Hence, option (c) is correct.
0 100. The tangent to the curve x 2 = y at
1 = ( A + B)x 2 + Cx + A ∫− 2 | f ( x ) | dx is (1, 1) makes an angle θ the with the
⇒ A + B = 0, C = 0, A = 1
(a) less than k positive direction of X -axis. Which
∴ B= − A= −1 (b) greater than k one of the following is correct?
1 x  (c) less than or equal to k π π π
∴I = ∫ = ∫ − 2
dx
 dx (a) θ < (b) < θ<
x ( x 2 + 1) x x + 1 (d) greater than or equal to k 6 6 4
0
π π π π
1
= ln x − ln( x 2 + 1) + C Ê (d) Given, ∫ f( x )dx = k (c) < θ< (d) < θ <
2 −2
4 3 3 2
Ê (d) Given, curve y = x
1 1 2
= (2 ln x ) − ln ( x 2 + 1) + C 0
2 2 To find ∫ f( x ) dx dy
= 2x
1 −2
= (ln x 2 − ln( x 2 + 1)) + C dx
2 Let f( x ) = x dy
= 2 × 1= 2
1  x2  0
= ln 2  +C 0
x2 dx at(1, 1)
2  x + 1 ∴ ∫ x dx =  2  = − 2 = k ⇒ tan θ = 2
−2  − 2
Hence, option (a) is correct. π π
0 0 Q tan = 3 = 1.732 and tan = ∞
94. What is the derivative of e e x with ∴ ∫ x dx = − ∫ x dx 3
π π
2
−2 −2
respect to e x ? ∴ <θ<
x = − (− 2 ) = 2 ≥ k 3 2
(a) ee (b) e x Hence, option (d) is correct.
x ∴Option (d) is correct.
(c) ee e x (d) ee x
NDA/NA Solved Paper 2021 (II) 15

101. Consider the following relations for 2 2 Which of the above statements
(a) (b)
two events E and F. 5 3 is/are correct?
1 1
1. P ( E ∩ F ) ≥ P ( E ) + P ( F ) − 1 (c) (d) (a) 1 only (b) 2 only
3 4 (c) Both 1 and 2 (d) Neither 1 nor 2
2. P ( E ∪ F ) = P ( E ) + P ( F ) + P ( E ∩ F ) 1 3
3. P ( E ∪ F ) ≤ P ( E ) + P ( F ) Ê (d) Given, P( A ) = , P( B) = , P(C ) = p Ê (c) Two regression lines are
2 4
Which of the above relations is/are 3x − 4 y + 8 = 0, 4x − 3 y − 1 = 0
QProbability that the problem can not be
correct? for finding the regression line of y on x
solved = P( A ) ⋅ P( B ) ⋅ P(C )
(a) 1 only (b) 3 only 3x − 4 y + 8 = 0
=  1 −   1 −  (1 − p)
1 3
(c) 1 and 3 only (d) 1, 2 and 3
 4 y = 3x + 8
2  4
Ê (c) Let E and F be two events. y =   x + 2
3
1 1 ...(i)
Then, P( E ∪ F ) = P( E ) + P( F ) − P( E ∩ F ) = × (1 − p)  4
2 4
...(i) 1− p and the regression line of x on y:
=
or P( E ∪ F ) ≤ P( E ) + P( F ) 8 4x − 3 y − 1 = 0
P (E ∪ F ) ≤ 1 ∴ Probability that the problem can be 4x = 3 y + 1
− P (E ∩ F ) ≥ − 1 solved 3
x = y+
1
...(ii)
⇒ P( E ) + P( F ) − P( E ∪ F ) ≥ P( E ) + P( F ) − 1 = 1 − Probability that the problem cannot 4 4
⇒ P ( E ∩ F ) ≥ P( E ) + P( F ) − 1 be solved Hence, option (c) is correct.
29 (1 − p)
Hence, option (c) is correct. = 1− 107. Consider the following statements.
32 8
102. If P ( A / B ) < P ( A ), then which one 1− p 29
1. The coefficient of correlations
of the following is correct? = 1− 3
8 32 r is
(a) P(B| A) < P(B) (b) P(B| A) > P(B) 1− p 3 4
(c) P(B| A) = P(B) (d) P(B| A) > P( A) = 2. The means of x and y are 3 and 4
8 32
 A respectively.
Ê (a) If P   < P( A ) 1− p=
3
B 4 Which of the above statements
P( A ∩ B) is/are correct?
P   =
A 1
Q < P( A ) ∴ p=
 B P( B) 4
(a) 1 only (b) 2 only
P( A ∩ B) (c) Both 1 and 2 (d) Neither 1 nor 2
⇒ < P( B)
Hence, option (d) is correct.
P( A ) 105. In a cricket match a batsman hits a Ê (a) Since, regression line of y on x
3
P   < P( B) six 8 times out of 60 balls he plays. ⇒ y= x +2
B

 A What is the probability that on a
4
3
ball played he does not hit a six? ∴ b xy =
Hence, option (a) is correct. 4
2 1 2 13 and regression line of x on y
103. When the measure of central (a) (b) (c) (d)
3 15 15 15 3 1
tendency is available in the form of ⇒ x = y+
mean, which one of the following is Ê (d) Since, the batsman hits a six 8 times 4 4
out of 60 balls. 3
the most reliable and accurate ∴ b yx =
measure of variability? The batsman could not hit sixes in (60-8) 4
(a) Range
balls. ∴Coefficient of correlations
(b) Mean deviation ∴ Probability that on a ball played he 3 3
52 r= b xy × b yx = ×
(c) Standard deviation does not hit six = 4 4
(d) Quartile deviation 60
3
13 r=
Ê (c) When the measure of central p= 4
15
tendency is available in the form of mean Means of x and y are nothing but the
Hence, option (d) is correct.
then, we know that Standard Deviation is solution of regression lines
the most reliable and accurate measure 3x − 4 y + 8 = 0
of variability. Directions (Q. Nos. 106 and 107)
Consider the following for the questions and 4x − 3 y − 1 = 0
Hence, option (c) is correct.
that follow. 3x − 4 y = − 8 ...(i)
104. A problem is given to three Two regression lines are given as 4x − 3 y = 1 ...(ii)
students A , B and C, whose 3x − 4y + 8 = 0 and 4x − 3y − 1 = 0 Eq. (i) × 4 − Eq. (ii) × 3
probabilities of solving the problem 12 x − 16 y = − 32
1 3 106. Consider the following statements.
independently are , and p, l. The regression line of y on x is 12 x − 9 y = 3
2 4 3 7 y = 35 ⇒ y = 5
respectively. If the probability that y = x +2
4 ∴ 4x = 1 + 3 × 5
29
the problem can be solved is , 2. The regression line of x on y is x =4
32
3 1 ∴Statement (2) is wrong.
then what is the value of p ? x= y+
4 4 Hence, option (a) is correct.
16 NDA/NA Solved Paper 2021 (II)

th
Directions (Q. Nos. 108 and 109) Marks Frequency Median = 
5 + 1
 term = 3 rd term
Consider the following for the questions 40-45 8  2 
that follow. =x +2
45-50 9
The marks obtained by 60 students in a ∴Mean > Mode and Median < Mean
certain subject out of 75 are given below. 50-55 6
Hence, correct option is (a).
55-60 4
Marks Number of students
60-65 2 112. The mean of 10 observations is 5.5.
15-20 4 If each observation is multiplied by
20-25 5 Ê (b) Highest frequency is given for class 4 and subtracted from 44, then what
25-30 11 25-30. is the new mean?
30-35 6 ∴ Model class will be 25-30. (a) 20 (b) 22
35-40 5 ∴ l = 25, h = 5 (c) 34 (d) 44
∴ f1 = 11, f0 = 5, f2 = 6
40-45 8 Ê (b) Given that, the mean of 10
f1 − f0
45-50 9 ∴ Mode = l + ×h observation is 5.5.
2 f1 − f0 − f2 x + x 2 + K + x 10
50-55 6 Q Mean = 1
11 − 5 10
55-60 4 = 25 + ×5
22 − 5 − 6 x 1 + x 2 + K + x 10
60-65 2 5.5 =
6 275 + 30 10
= 25 + ×5= 10
108. What is the median? 11 11 ∴ ∑ x i = 55
(a) 35 (b) 38 = 27.73 i =1
(c) 39 (d) 40 Hence, option (b) is correct. Also, given that new observations are
Ê (c) 110. What is the mean of natural obtained by multiplying by 4 and
numbers contained in the interval subtracting from 44.
Marks Frequency Cumulative
frequency [15, 64]? Hence, new mean = 44 − 4 × 5.5
(a) 36.8 (b) 38.3 = 44 − 22 = 22
15-20 4 4
(c) 39.5 (d) 40.3 Hence, correct option is (b).
20-25 5 9
25-30 11 20
Ê (c) Mean of natural numbers contained 113. If g is the geometric mean of 2, 4, 8,
in [15, 64]. 16, 32, 64, 128, 256, 512, 1024, then
30-35 6 26 = Cf 15 + 16 + 17 + K + 64 which one of the following is
=
35-40 5 31 50 correct?
64 14
40-45 8 39
∑n − ∑r (a) 8 < g < 16
(c) 32 < g < 64
(b) 16 < g < 32
(d) g > 64
n=1 r =1
45-50 9 48 =
50-55 6 54
50 Ê (c) Given that, geometric mean of
64 × 65 14 × 15 2, 4, 8, 16, 32, 64, 126, 256, 512, 1024 is
55-60 4 58 −
= 2 2 g.
60-65 2 60 50 2 × 4 × 8 × 16 × 32 × 64
∴ g = 10
N = 60 2080 − 105 × 128 × 256 × 512 × 1024
= = 39.5
50 1
N
∴ = 30 ⇒ model class will be 35 − 40. Hence, option (c) is correct. = (2 1 + 2 + 3 + K + 10 )10
2
1 11
∴lower limit ( l ) = 35 111. For the set of numbers g = (2 55 )10 ⇒ g = (2 ) 2
h = 40 − 35 = 5 x , x , x + 2, x + 3, x + 10 where x is a
natural number, which of the ∴ 2 5 < g < 2 6 ⇒ 32 < g < 64
N
− C. f Hence, option (c) is correct.
∴ Median = l + 2 ×h following is/are correct?
f 1. Mean > Mode 114. If the harmonic mean of 60 and x is
30 − 26 2. Median> Mean 48, then what is the value of x?
= 35 + × 5 = 39
5 Select the correct answer using the (a) 32 (b) 36
Hence, option (c) is correct. code given below. (c) 40 (d) 44
109. What is the mode? (a) 1 only
(b) 2 only Ê (c) Given, harmonic mean of 60 and x is
(a) 27.27 (b) 27.73 48.
(c) Both 1 and 2 2 ab
(c) 27.93 (d) 28.27 Q H=
(d) Neither 1 nor 2
a+ b
Marks Frequency
Ê (a) Given data x, x, x + 2, x + 3, x + 10 2 × 60 × x
15-20 4 where x ∈ N. 48 =
60 + x
20-25 5 → f0 x + x + x + 2 + x + 3 + x + 10
∴Mean = 2880 + 48x = 120x
25-30 11 → f1 5
5x + 15 72 x = 2880
30-35 6 → f2 = =x + 3
5 x = 40
35-40 5 ∴ Mode = x Hence, option (c) is correct.
NDA/NA Solved Paper 2021 (II) 17

115. What is the mean deviation of first Since, the sum of the digits 7 3
Ê (d) Given, P(not A ) = , P(not B) =
10 even natural numbers? = 1 + 3 + 7 = 11 10 10
(a) 5 (b) 5.5 which is not divisible by 3.  A
P  =
3
0  B  14
(c) 10 (d) 10.5 ∴P (number of divisible by 3) =
3! P( A ∩ B)
Ê (a) Mean deviation of first 10 even P   =
B
∴ ...(i)
natural numbers =0  A P( A )
2 + 4 + 6 + K 20 Hence, option (a) is correct. 7
Since, mean ( x ) = Q P(not A ) =
10 118. What is the probability that the 10
2(10 × 11) roots of the equation x 2 + x + n = 0 ∴ P( A ) = 1 −
7
=
3
= = 11
20 10 10
are real, wheren ∈ N and n < 4? 3
∴Mean deviation 1 P( not B) =
| 2 − 11| + | 4 − 11| + | 6 − 11| (a) 0 (b) 10
4 3 7
+ K + | 20 − 11| 1 1 ∴ P( B) = 1 − =
= (c) (d) 10 10
10 3 2
P( A ∩ B)
9 + 7 + 5 + 3 + 1+ 1+ 3 + 5 + 7 + 9 P   =
A
Ê (a) Given, equation x + x + n = 0,
2
=  B
10 P( B)
where n ∈ N, n < 4
=5 3 P( A ∩ B)
∴ n ∈ {1, 2, 3} =
Hence, option (a) is correct. 14 7
Since, above equation is quadratic.
10 10 10
116. If ∑ x i = 110 and ∑ x i2 = 1540, So, for each value of n, we have two
roots. ∴ P( A ∩ B) =
3
i =1 i =1 20
then what is the variance? ∴Total number of roots = 6 3
(a) 22 (b) 33 When n = 1  B  20 1
Eq. (i) ⇒ P   = =
(c) 44 (d) 55 x2 + x + 1 = 0  A 3 2
10
− 1 ± 1 − 4 − 1 ± 3i 10
Ê (b) Given, ∑ x i = 110 x= =
Hence, option (d) is correct.
i =1 2 2
when n = 2,
10 120. Seven white balls and three black
∑xi = 1540
2
and x2 + x + 2 = 0 balls are randomly placed in a row.
i =1
−1 ± 1 − 8 − 1 ± 7 i What is the probability that no two
Σx i2  Σx i 
2 ⇒ x = =
Q Variance = −  2 2 black balls are placed adjacently?
 n 
n and n = 3, x + x + 3 = 0
2
7 8
1540  110 
2 (a) (b)
= −  − ± 11i 15 15
 10  ⇒ x =
10 2 11 13
(c) (d)
= 154 − 121 There are no real roots. 15 15
= 33 0 Ê (a) There are 10 balls among which 7
∴P (roots are real) = = 0
Hence, option (b) is correct. 6 are white and 3 are black.
Hence, option (a) is correct. ∴Number of ways to arrange 10 balls
117. 3-digit numbers are formed using
= 10!
the digits 1, 3, 7 without repetition 119. If A and B are two events such that
of digits. A number is randomly 7 3 If we put the balls in such a way that no
P ( not A ) = , P ( not B ) = and two black balls are placed adjacently.
selected. What is the probability 10 10
that the number is divisible by 3? 3 ∴Number of arrangements = 7 ! × 8P3
P ( A | B ) = , then what is P ( B | A ) 7 ! × 8!
1 14 ∴ P=
(a) 0 (b)
3 equal to? 5 ! × 10 !
1 1 11 9
(c) (d) (a) (b) 6×7 7
4 8 = =
14 14 9 × 10 15
1 1
Ê (a) Let 3-digit numbers using the digits (c) (d) Hence, option (a) is correct.
without repetition 1, 3, 7 are 3!. 4 2
PAPER II : English Language and General Studies

Part A (English Language)


Directions (Q. Nos. 1-10) Each item 8. His aversion is known to all. 15. The thorax is the upper part (a) /of
in this section consists of a sentence (a) hospitality (b) hostility your body from the neck down (b) /
with an underlined word followed by (c) humility (d) humbleness to the bottom of the ribs and
four words (a), (b), (c) and (d). Select diaphragm. (c) / No error (d)
Ê (b) ‘Aversion’ refers to a strong dislike or
the option that is nearest in meaning to disinclination towards something. Ê (b) Part (b) contains the error. Replace
the underlined word. Hence, ‘hostility’ is its correct synonym. ‘from’ with ‘extending from’ to make the
1. He is essentially a lowbrow person. 9. To be able to decipher something is sentence error free.
(a) coarse (b) proud wonderful. 16. The practical uses of astrology (a) /
(c) passionate (d) pathetic (a) decode (b) encode have seldom been seriously (b) /
Ê (a) ‘Lowbrow’ means not highly cultured (c) simulate (d) animate studied outside India. (c) /
and lacking in refinement. Hence, No error (d)
‘coarse’ is its correct synonym.
Ê (a) ‘Decipher’ means to convert a coded
text or signal into normal language.
Ê (d) The given sentence is error free and
2. His nostalgia is deep. Hence, ‘decode’ is its correct synonym. grammatically correct.
(a) excitement (b) longing 10. It is the opportune time to think 17. She collected (a) / some armful of
(c) happiness (d) ability about investment in the real estate medicines and (b) / poured them
Ê (b) ‘Nostalgia’ means a sentimental sector. down a drain. (c) / No error (d)
longing for a period in the past. Hence, (a) honorary (b) appropriate
option (b) is the correct answer. (c) difficult (d) unsuitable
Ê (c) Part (c) contains the error. Replace
‘a’ with ‘the’ to make the sentence error
3. His truancy is detrimental. Ê (b) ‘Opportune’ time means a time that free.
(a) loyalty (b) integrity is suitable for doing something. Hence
‘appropriate’ is its correct synonym.
18. In many countries, the United
(c) honesty (d) absenteeism Nations (a) / or other
Ê (d) ‘Truancy’ refers to the problem or Directions (Q. Nos. 11-20) Each internationally organised (b) /
situation of children being absent from peacekeeping forces are trying to
school regularly without permission. question in this section has a sentence
with three parts labelled (a), (b) and (c). keep the peace. (c) /
Hence, ‘absenteeism’ is its correct No error (d)
synonym. Read each sentence to find out whether
there is any error in any part. If you Ê (b) Part (b) contains the error. Replace
4. He is a maleficent person. find no error, your responses should be ‘or’ with ‘and’ to make the sentence error
(a) generous (b) cunning indicated as (d). free.
(c) criminal (d) friendly 19. Early cities rely on food and water
11. Nursery classes (a)/ for the children
Ê (c) ‘Maleficent’ means working or (b) / starts on April every year. (c) / (a) / from the surrounding
productive of harm or evil. Hence, No error (d) countryside, (b) / but today cities
‘criminal’ is nearest in meaning to often depend on distant sources.
maleficent. Ê (c) Part (c) contains the error. Replace
‘on’ with ‘in’ to make the sentence error (c) / No error (d)
5. His solemnity is celebrated. free. Ê (a) Part (a) contains the error. Replace
(a) greed (b) desire 12. The pollution level is very (a) / high ‘early cities rely’ with ‘earlier cities relied’
(c) trust (d) dignity in our city, and (b) / becomes a to make the sentence grammatically
correct.
Ê (d) ‘Solemnity’ refers to the state of health hazard. (c) / No error (d)
being serious and dignified. Hence 20. Climate change, whether it is (a) /
‘dignity’ is its correct synonym. Ê (c) Part (c) contains the error. Replace
‘becomes’ with ‘has become’ to make natural or human-induced, (b) / is
6. His bounty is limitless. the sentence error free. stressful for the people. (c) / No
(a) benevolence (b) gallantry 13. Indian is a country of linguistic (a) / error (d)
(c) nepotism (d) chivalry and cultural diversity with (b) / Ê (c) Part (c) contains the error. Replace
Ê (a) ‘Bounty’ means generosity. Hence, multiple ethnic and social groups. ‘for’ with ‘to’ to make the sentence error
‘benevolence’ is its correct synonym. (c) / No error (d) free.
7. The Holocaust was experienced by Ê (d) The given sentence is error free. Directions (Q. Nos. 21-30) Each
millions. 14. Why does the lapse rate (a) / effect question in this section consists of a
(a) celebration (b) destruction the tendency (b) / of air to rise ? (c) sentence with an underlined word
(c) construction (d) beautiful / No error (d) followed by four words (a), (b), (c) and
Ê (b) ‘Holocaust’ refers to destruction that Ê (b) Part (b) contains the error. Replace (d). Select the option that is opposite in
occurred on a massive scale. Hence, ‘effect’ with ‘affect’ to make the sentence meaning to the underlined word.
option (b) is the correct answer. grammatically correct.
NDA/NA Solved Paper 2021 (II) 19

21. His opinion is lopsided. 29. Amit is a dogmatic person. The correct sequence should be :
(a) partial (b) crooked (a) assertive (b) amenable (a) PRQS (b) RPQS
(c) unequal (d) balanced (c) bold (d) rigid (c) SPQR (d) QRPS
Ê (a) PRQS
Ê (d) ‘Lopsided’ means not equally Ê (b) ‘Dogmatic’ means of a person
balanced. Hence, ‘balanced’ is its completely certain of their bailiffs and 35. like mine (P) / even to someone
correct antonym. expecting other people to accept them. with (Q) / a tin ear (R) / her singing
Hence, ‘amenably is its correct antonym sounded pretty melodious (S)
22. His work is praiseworthy.
as its means willing to accept what
(a) admirable (b) condemnable The correct sequence should be
someone says or does without arguing.
(c) commendable (d) creditable (a) PRQS (b) RPQS
30. Sachin is very fickle in his behaviour. (c) SPQR (d) QRPS
Ê (b) ‘Praiseworthy’ means deserving (a) stable (b) capricious
approval and admiration. Hence, Ê (d) QRPS
‘condemnable’ meaning blamable is (c) mercurial (d) vacillating
36. The sorry state of affairs in the
opposite in meaning to praiseworthy. Ê (a) ‘Fickle’ means frequently changing modern factories (P) / the
one’s loyalties and affections. Hence,
23. His deeds had retrograde results . ‘stable’ is its correct antonym. Government of India (Q) / under
(a) progressive (b) negative the British was pro-capitalist (R) /
(c) retreating (d) reverse Directions (Q. Nos. 31-40) Each of though it took some half-hearted
Ê (a) ‘Retrograde’ means to revert to an the following items in this section consists steps to mitigate (S)
earlier and inferior conditions. Hence, of a sentence, the parts of which have The correct sequence should be :
‘progressive’ meaning forward-moving is been jumbled. These parts have been (a) PRQS (b) QRSP
its correct antonym. (c) SPQR (d) QPSR
labelled P, Q, R and S. Given below each
24. He always offers palatable sentence are four sequences namely (a), Ê (b) QRSP
solutions. (b), (c) and (d), You are required to 37. of extreme poverty among its people
(a) acceptable (b) agreeable re-arrange the jumbled parts of the (P) / economic policies was the
(c) unacceptable (d) pleasant sentence and mark your answer carefully. prevalence (Q)/a major characteristic
Ê (c) ‘Palatable’ means acceptable or 31. symbolic of peace, and prosperity of British rule in India (R) / and the
satisfactory. Hence, ‘unacceptable’ is its (P)/the Gupta Empire in India is net result of British (S)
correct antonym. (Q)/represented through The correct sequence should be :
25. His views on the subject are numismatic archaeological (R) / (a) PRQS (b) QRSP
microscopic. and literary evidences of multiple (c) RSQP (d) QPSR
(a) broad-based (b) minute kinds (S)
The correct sequence should be Ê (c) RSQP
(c) precise (d) fine
(a) QPRS (b) QSRP 38. you can’t judge (P) / very
Ê (a) ‘Microscopic’ means concerned with (c) PSQR (d) RSQP intelligent, but (Q) / a book by its
minute details. Hence, ‘broad-based’ is
opposite in meaning to ‘microscopic’. Ê (a) QPRS cover (R) / he doesn’t look (S)
32. the transportation system so that it The correct sequence should be :
26. He is always obdurate in his (a) SQPR (b) QRSP
behaviour towards other people. (P) / and not just the affluent
section of society (Q) / provides (c) RSQP (d) QPSR
(a) flexible (b) callous
(c) insensible (d) obnoxious
mobility to everyone (R) / there are Ê (a) SQPR
many ways to restructure (S) 39. membership and partisanship as
Ê (a) ‘Obdurate’ means stubbornly The correct sequence should be :
refusing to change one’s opinion. well as (P) / can be found in the
(a) PQRS (b) RQPS decline in party (Q) / evidence of a
Hence, ‘flexible’ is its correct antonym.
(c) SPRQ (d) QRPS crisis in party politics (R) / in the
27. Mohan always had profound Ê (c) SPRQ rise of antiparty groups and
respect for Sohan. movements (S)
33. and accurately displays spatial (P) /
(a) deep (b) extreme The correct sequence should be :
relationships between landforms
(c) sincere (d) superficial (a) RQPS (b) QRSP
and water bodies (Q) / the globe
Ê (d) ‘Profound’ means very great or shows Earth’s spherical shape (R) / (c) RSQP (d) QPSR
intense. Hence, ‘superficial’ is its correct and comparative distance between
antonym. ‘Superficial’ means Seeming to
Ê (a) RQPS
locations (S)
have a particular quality, although this is 40. government, market and voluntary
not true or real.
The correct sequence should be : organisations (P) / mixture of
(a) PQRS (b) RPQS contributions from (Q) / the
28. We were living in turbulent times. (c) SPQR (d) QRPS
provision of public goods (R) / and
(a) destructive (b) unstable Ê (b) RPQS services can be regarded as a
(c) calm (d) stormy
34. the function of the capillaries (P) / complex (S)
Ê (c) ‘Turbulent’ means involving a lot of and other material to all the cells in The correct sequence should be
sudden changes, arguments ore (Q) / is to take nutrients, oxygen (R) (a) RQPS (b) QRSP
violence. Hence, ‘calm’ is its correct / the body and to take away their (c) RSQP (d) QPSR
antonym.
waste material (S) Ê (c) RSQP
20 NDA/NA Solved Paper 2021 (II)

Directions (Q. Nos. 41-50) Given 44. To level with (c) To become dysfunctional
below are some idioms/phrases followed (a) To be honest (d) To become superlative
by four alternative meanings for each. (b) To be on a plain Ê (c) The idiom ‘be out of action’ means
Choose the most appropriate answer (c) To cheat someone broken or not working. Hence, option (c)
from among the options (a), (b), (c) or (d). (d) To take revenge given its correct meaning.
41. Once-over Ê (a) The phrase ‘to level with’ someone 48. Learning from the school of hard
(a) To do something quickly means to tell someone the truth, knocks
(b) To do something elaborately especially when it may be unpleasant. (a) Learning things from experience
(c) To do something perforce Hence, ‘To be honest’ gives its correct (b) Learning things from others
(d) To do something for others meaning. (c) Learning things from environment
Ê (a) The phrase ‘once-over’ means to 45. Show your hand (d) Learning things through difficult
review, inspect or examine something in (a) To tell people about the past experiences
a quick fashion. (b) To tell people about the future Ê (d) The idiom ‘leaning form the school of
Hence, option (a) is the correct (c) To tell people about them hard knacks’ means leaning thing
answer. through difficult experiences.
(d) To tell people about your plans
42. A bag of bones 49. Raise some hackles
Ê (d) The idiom ‘show your hands’ means
(a) An extremely fat person to allow people to know about your (a) To annoy
(b) An extremely thin person intentions. (b) To make someone happy
(c) A wealthy person Hence, option (d) is the correct (c) To do a great task
(d) A healthy person answer. (d) To sit idle
Ê (b) The idiom ‘a bag of bones’ means a 46. To pull up Ê (a) The idiom ‘raise some hackles’
very thin person or animal. means to annoy or upset someone.
Hence, option (b) is the correct (a) To lift heavy weights
(b) To check oneself Hence, option (a) is the correct answer.
answer.
(c) To shorten the distance 50. A man of letters
43. Muddy the waters (d) Too short of destination (a) An excellent artist
(a) To spread dirt around
Ê (b) The phrase ‘pull up’ means to life (b) A scholar in literature
(b) To make a situation easy
cense someone to stop ore pause: (c) An outstanding sportsman
(c) To make one’s life happy
(d) To make the situation complicated check. So, ‘To check’ oneself given its (d) A known novelist
correct meaning.
Ê (d) The idiom ‘muddy the waters’ Ê (b) ‘A man of letters’ means a man,
means to make a situation more 47. Be out of action usually a writer, who knows a lot about
literature. Hence, ‘A Scholar in literature
complicated or confusing than it was (a) To get punishment
gives its correct meaning.
before. (b) To get reward

Part B (General Studies)


51. Girth of stem of a plant increases Alleles contribute to the organism's Ê (d) Buoyancy or upthrust, is an upward
due to division of cells in phenotype, which is the outward force exerted by a fluid that opposes the
(a) apical meristem only appearance of the organism. Some weight of a partially or fully immersed
(b) lateral meristem only alleles are dominant or recessive. object. In a column of fluid, pressure
(c) apical and intercalary meristem increases with depth as a result of the
53. Which one of the following viruses
(d) both apical and lateral meristem weight of the overlying fluid.
used to be responsible for highly
Ê (b) The girth of the stem of a plant infectious disease smallpox? 55. Weight and mass of an object are
increases due to the division of cells in (a) Adenovirus (b) Variola virus defined with Newton’s laws of
lateral meristem only. Lateral meristems (c) Aichi virus (d) Coxsackie virus motion. Which among the
are known as secondary meristems following is true?
because they are responsible for Ê (b) Variola virus was used to be
responsible for the highly infectious (a) Weight is a constant of
secondary growth or increase in stem
disease smallpox. proportionality
girth and thickness.
The disease is spread through
(b) Mass is a constant of
52. Different varieties of the same gene person-to-person contact, most often
proportionality
are called from inhalation of air droplets expelled
(c) Mass is not a constant of
(a) Genotypes (b) Sib pairs through the infected person. The
proportionality
(c) Alleles (d) Isomers disease was eradicted worldwide in
(d) Weight is a universal constant.

Ê (c) Different varieties of the same gene 1976 through vaccination. Ê (b) The weight of an object is defined as
are called alleles because an allele is a the force of gravity on the object and
54. Buoyancy is a/an may be calculated as the mass times the
variant form of a gene. Some genes
(a) upward pressure acceleration of gravity, w = mg. Since
have a variety of different forms, which
(b) downward pressure the weight is a force, its SI unit is the
are located at the same position, or
(c) downward force newton. Thus mass is a constant of
genetic locus, on a chromosome.
(d) upward force proportionality.
NDA/NA Solved Paper 2021 (II) 21

56. Fundamental laws of physics require Ê (a) One-third of the individual resistance Ê (c) Cobalt therapy is the medical use of
(a) conservation of energy and because if three or more resistors each gamma rays from the radioisotope
non-conservation of charge with the same value are connected in cobalt-60 to treat conditions such as
(b) conservation of charge and non- parallel, then the equivalent resistance cancer. Beginning in the 1950s,
conservation of linear momentum
R
will be equal to where R is the value of cobalt-60 was widely used in external
(c) conservation of charge and n beam radiotherapy (teletherapy)
non-conservation of energy the resistor and n is the number of machines, which produced a beam of
(d) conservation of energy, momentum individual resistances in the combination. gamma rays that was directed into the
and charge patient's body to kill tumor tissue.
60. Rutherford’s alpha-particle (α)
Ê (d) Fundamental laws of physics require scattering experiment was 64. To protect steel and iron from
conservation of energy, momentum and responsible for the discovery of rusting, a thin layer of which one of
Charge. With respect to classical which one of the following? the following metals is applied?
physics, conservation laws include (a) Electron (b) Proton (a) Magnesium (b) Zinc
conservation of energy, mass (or (c) Aluminium (d) Lead
(c) Atomic Nucleus(d) Neutron
matter), linear momentum, angular
momentum, and electric charge. Most Ê (c) Rutherford’s alpha-particle experiment Ê (b) Zinc metal present on the surface of
conservation laws are exact, or absolute, was responsible for the discovery Of iron and steel forms a thin protective
in the sense that they apply to all Atomic Nucleus. Ernest Rutherford's layer of basic zinc carbonate due to the
possible processes. most famous experiment is the gold foil reaction between zinc, oxygen, CO 2 and
experiment. A beam of alpha particles moisture in the air. The standard
57. Work is said to be one Joule when a was aimed at a piece of gold foil. Most reduction potential of Zinc is less than
force of alpha particles passed through the foil, iron, or Zinc is more reactive than Iron.
(a) 4 N moves an object by 25 cm but a few were scattered backward. This Hence, Iron will not undergo corrosion.
(b) 2 N moves an object by 1 m showed that most of the atom is empty 65. Cinnabar is an ore of which one of
(c) 1 N moves an object by 1 cm space surrounding a tiny nucleus.
the following?
(d) 1 N moves an object by 50 cm 61. What is the maximum number of (a) Copper (b) Zinc
Ê (a) One joule is defined as the amount of electrons in the M-Shell? (c) Mercury (d) Manganese
work done when a force of one newton is (a) 6 (b) 8
exerted through a distance of one meter. Ê (c) Cinnabar is a toxic ore of mercury,
(c) 18 (d) 32 composed of mercury sulfide (HgS). It is
Thus force of 4 N moves an object by 25
cm resulting in 1 joule of work. Ê (c) Maximum number of electrons in the the only important ore of mercury. It has
M-Shell is 18. Each shell has a fixed a bright red color that has caused people
58. A uniform motion of a car along a number of electrons. The formula to to use it as a pigment, and carve it into
circular path experiences determine the number of electrons in a jewelry and ornaments for thousands of
(a) a change in speed due to a shell is 2(n)2. years in many parts of the world.
change in its direction of motion For the M shell principle, the quantum
66. Imagine a current-carrying straight
(b) a change in velocity due to a number is 3.
change in its direction of motion
conductor with magnetic field of
So, the maximum number of electrons is
(c) a change in momentum due to no lines in anti-clockwise direction.
= 2(3)2
change in its direction of motion Then the direction of current is
Maximum number of electrons =18.
(d) a constant momentum due to a determined by
change in its direction of motion 62. Chlorine occurs in nature in two (a) the Right-Hand Thumb rule and it
Ê (b) A uniform motion of a car along with isotopic forms of masses 35 u and would be in the downward
a circular path experiences a change in 37 u in the ratio of 3 : 1 direction.
velocity due to a change in its direction respectively. What is the average (b) the Left-Hand Thumb rule and it
of motion. atomic mass of the Chlorine atom? would be in the downward direction.
Since velocity is a vector, changes in (c) the Right-Hand Thumb rule and it
(a) 36.1 u (b) 35.5 u
direction constitute changes in velocity. would be in the upward direction.
(c) 36.5 u (d) 35.1 u
A change in velocity is known as (d) the Left-Hand Thumb rule and it
acceleration. The change in velocity due
Ê (c) The average atomic mass for an would be in the upward direction.
element is calculated by summing the
to circular motion is known as centripetal masses of the element's isotopes, each Ê (c) The right-hand thumb rule is used to
acceleration. find the direction of the magnetic field
multiplied by its natural abundance on
around a current carrying a straight
59. Three equal resistors are connected Earth.
conductor. Thus, a current-carrying
in parallel configuration in a closed P x + P2x 2
Average atomic mass = 1 1 straight conductor with the magnetic
electrical circuit. Then the total P1 + P2
field of lines in the anti-clockwise
resistance in the circuit becomes ( 35 × 1) + ( 37 × 3) direction would be in the upward
= = 36.5 u
(a) one-third of the individual 3+1 direction according to the right-hand
resistance thumb rule.
(b) two-third of the individual 63. Which one of the following
resistance elements’ isotopes is used in the 67. The device used to produce electric
(c) equal to the individual resistance treatment of cancer? current is known as
(d) three times of the individual (a) Iodine (b) Sodium (a) motor (b) generator
resistance (c) Cobalt (d) Uranium (c) ammeter (d) galvanometer
22 NDA/NA Solved Paper 2021 (II)

Ê (b) The device used to produce electric Ê (b) Dingko Singh was an Indian boxer Novak Djokovic defeated Stefanos
current is called a Generator. Electric who won the gold medal at 1998 Asian Tsitsipas in the final, 6–7, 2–6, 6–3, 6–2,
generator, also called a dynamo, is any Games in Bangkok. He was from the 6–4 to win the Men's Singles tennis title
machine that converts mechanical North-eastern Indian state of Manipur. at the 2021 French Open.He is from
energy to electricity for transmission and He was awarded the Padma Shri by the Serbia and is currently (as of November,
distribution over power lines to domestic, Government of India in 2013. 2021) the No.1 Ranked tennis player in
commercial, and industrial customers. the world.
Generators also produce the electrical 72. Who among the following is the
power required for automobiles, aircraft, lone Indian Cricketer included in 76. Economist Rebeca Grynspan, who
ships, and trains. the ICC Hall of Fame special recently became the first woman to
inductions to mark the inaugural ICC be appointed as the head of the
68. Myopia is a defect in human vision
World Test Championship Final? United Nations Conference on
where an image of a
(a) Dilip Vengsarkar Trade and Development
(a) nearby object is focused beyond
the retina. (b) Ravi Shastri (UNCTAD), hails from
(b) nearby object is focused before the (c) Mohinder Amarnath (a) Germany (b) Poland
retina. (d) Vinoo Mankad (c) Costa Rica (d) Mexico
(c) distant object is focused before the Ê (d) Vinoo Mankad is the lone Indian Ê (c) Economist Rebeca Grynspan, who
retina. Cricketer included in the ICC Hall of recently became the first woman to be
(d) distant object is focused beyond Fame special inductions to mark the appointed as the head of the United
the retina. inaugural ICC World Test Championship Nations Conference on Trade and
Development (UNCTAD), hails from
Ê (c) Nearsightedness (myopia) is a Final. The BCCI commemorated Vinoo
common vision condition in which you Mankad's contribution to Indian cricket, Costa Rica. UNCTAD is a permanent
can see objects near to you clearly, but naming the domestic Under-19 One day intergovernmental body established by
objects farther away are blurry. It occurs tournament as the Vinoo Mankad Trophy. the United Nations General Assembly in
when the shape of your eye causes light 1964. Its headquarters are located in
rays to bend (refract) incorrectly,
73. Which one of the following nations Geneva, Switzerland. It also have offices
focusing images in front of your retina is not a permanent member of G7? in New York and Addis Ababa.
instead of on your retina. (a) India (b) Canada
(c) France (d) Italy 77. ‘Belt and Road Initiative’ is a
69. Tyndall effect is a phenomenon of foreign policy initiative of
(a) scattering of light by the colloidal Ê (a) India is not the permanent member (a) China (b) USA
particles of G7. The Group of Seven is an
inter-governmental political forum (c) Canada (d) Japan
(b) refraction of light by the colloidal
particles consisting of Canada, France, Germany, Ê (a) Belt and Road Initiative’ is a foreign
Italy, Japan, the United Kingdom and the policy initiative of China. It is also known
(c) dispersion of light by dust particles
United States. as One Belt One Road or OBOR and is a
(d) refraction of light by dust particles
global infrastructure development strategy
Ê (a) Tyndall effect, also called Tyndall 74. Operation Olivia, an initiative to adopted by the Chinese government in
phenomenon, scattering of a beam of protect Olive Ridley turtles, is 2013 to invest in nearly 70 countries and
light by a medium containing small undertaken by international organizations.
suspended particles. For e.g., smoke or (a) Indian Navy
dust in a room, which makes visible a (b) Indian Coast Guard
78. Which one of the following is the
light beam entering a window. The effect (c) Ministry of Environment Forest and theme of the International Day of
is named for the 19th-century British Climate Change Yoga 2021?
physicist John Tyndall, who first studied (d) Ministry of Earth Sciences (a) Yoga at home and yoga with family
it extensively. (b) Yoga for climate action
Ê (b) Operation Olivia, particles. an (c) Yoga for well-being
70. Twinkling of stars is primarily due initiative to protect Olive Ridley turtles, is
undertaken by Indian Coast Guard. (d ) Yoga for peace.
to the atmospheric
(a) refraction (b) reflection Operation Olivia was started by the Ê (c) Yoga for well-being is the theme of
(c) polarisation (d) dispersion Indian Coast Guard (ICG), first in early the International Day of Yoga 2021. The
1980s. This operation helps in protecting International Day of Yoga has been
Ê (a) The twinkling of a star is due to the Olive Ridley turtles every year when they celebrated annually on 21st June since
atmospheric refraction of starlight. The start nesting along Odisha coast for 2015, following its inception in the United
starlight, on entering the earth’s breeding in months of November to Nations General Assembly in 2014. Yoga
atmosphere, undergoes refraction December. is a physical, mental and spiritual
continuously before it reaches the earth. practice which originated in India.
The atmospheric refraction occurs in a 75. Who among the following is the
medium of gradually changing refractive winner of French Open 2021 Men’s 79. Kenneth Kaunda, who died recently
index. Singles Final? at the age of 97, was the former
71. Dingko Singh, an Asian Games gold (a) Novak Djokovic President of
medallist, who died recently, was (b) Rafael Nadal (a) Brazil (b) Mexico
(c) Stefanos Tsitsipas (c) Zambia (d) South Sudan
associated with which one of the
(d) Daniil Medvedev
following games? Ê (c) Kenneth Kaunda, who died recently
(a) Swimming (b) Boxing Ê (a) Novak Djokovic is the winner of the at the age of 97, was the former
French Open 2021 Men's Singles Final. President of Zambia. He was the first
(c) Archery (d) Gymnastics
NDA/NA Solved Paper 2021 (II) 23

President of Zambia from 1964 to 1991. Ê (c) White colour precipitate is obtained Ê (d) Photosynthesis is a process used by
He was at the forefront of the struggle for by passing CO 2 (carbon dioxide) gas plants and other organisms to convert
independence from British rule. Zambia, through lime water. Carbon dioxide light energy into chemical energy that,
in southern Africa, is a landlocked reacts with limewater (a solution of through cellular respiration, can later be
country of rugged terrain and diverse calcium hydroxide, Ca(OH)2, to form a released to fuel the organism’s activities.
wildlife, with many parks and safari white precipitate (appears milky) of Here, light energy is directly used to split
areas. calcium carbonate, CaCO 3. Adding more water.
80. The maiden Indian Navy - carbon dioxide results in the precipitate
dissolving to form a colourless solution
89. How are evergreen plants with
European Union Naval Force woody stems having naked seed
of calcium hydrogencarbonate.
(IN-EUNAVFOR) Exercise (2021) classified?
was conducted in 85. Which one of the following pairs of (a) Angiosperms
(a) Gulf of Mannar elements is liquid at room (b) Monocotyledons
(b) Gulf of Aden temperature and at normal pressure? (c) Pteridophytes
(c) Gulf of Khambhat (a) Gallium and Bromine (d) Gymnosperms
(d) Gulf of Aqaba (b) Mercury and Bromine
(c) Gallium and Mercury Ê (d) Gymnosperms are woody plants,
Ê (b) Indian Navy - European Union Naval either shrubs, trees, or, rarely, vines
Force (IN-EUNAVFOR) Exercise (2021) (d) Gallium and Caesium (some gnetophytes). They differ from
was conducted in Gulf of Aden. Along Ê (b) The two liquid elements at room flowering plants in that the seeds are not
with Indian Navy, other naval forces are temperature and at normal pressure are enclosed in an ovary but are exposed
from Italy, Spain and France.The naval Mercury (symbol Hg and atomic number (naked seeds) within any of a variety of
exercise included advanced air defence 80) and Bromine (symbol Br and atomic structures, the most familiar being
and anti-submarine exercises, tactical number 35). Mercury is the only metal cones.
manoeuvres, Search & Rescue, and that is a liquid at room temperature.
other maritime security operations.
90. Which one of the following tissues
86. Which one of the following is known as basic packing tissue
81. The pH value of Milk of Magnesia methods can be used to separate and found" in xylem and phloem?
is approximately anthracene from a mixture of salt (a) Collenchyma (b) Parenchyma
(a) Zero (b) 7 and anthracene? (c) Sclerenchyma (d) Vessels
(c) 10 (d) 14 (a) Distillation (b) Sublimation
Ê (b) Parenchyma forms the bulk of plant
Ê (c) pH value of Milk of Magnesia is (c) Evaporation (d) Chromatography ground tissue, where they may be
approximately 10. It is a base and is
Ê (b) Sublimation is the conversion specialised to function in
used to neutralize the excess acid in the between the solid and the gaseous photosynthesis, storage, or transport.
stomach. phases of matter, with no intermediate Parenchyma is integral to vascular
82. Which one of the following liquid stage. Thus, Sublimation can be tissue, where it provides a route of
compounds is used in ‘black and used to separate anthracene from a exchange for materials within and
mixture of salt and anthracene. between the xylem and the phloem.
white’ photography?
(a) AgF (b) AgBr 87. Shoots of plant show upward 91. Which one from among the
(c) AgCI (d) Ag 2SO 4 movement and it can be designated following planets is largest in size?
Ê (b) AgBr (Silver Bromide) is used in to be (a) Earth (b) Venus
black and white photography. Its use in (a) Negatively phototropic (c) Mars (d) Mercury
black and white photography is an (b) Positively chemotropic Ê (a) Size of the planets in order from
example of Photochemical reaction. (c) Positively hydrotropic smallest to largest is Mercury, Mars,
Silver chloride is sensitive to light and (d) Negatively geotropic Venus, Earth, Neptune, Uranus, Saturn,
breaks down to bromide form metallic and Jupiter. Thus, Earth is the bigger in
silver, which appears black. Ê (d) The tendency of plant stems and
other parts to grow upwards. 'This is size as compared to Mercury,Mars and
83. What is the number of water called negative geotropism because the Venus.
molecules present in a Ferrous plant is growing away from the force of 92. Match List I with List-II and select
Sulphate crystal? gravity. A negative geotropism is a the correct answer using the codes
turning away from the earth, such as by
(a) 1 (b) 3 given below the lists :
a plant stem that grows upward.
(c) 5 (d) 7
List I List II
Ê (d) 7 water molecules are present in a 88. Which one of the following (Place) (Feature)
Ferrous Sulphate crystal known as the statements about the process of
A. Mumbai 1. Queen of the
water of crystallization. The colour in photosynthesis is correct ? Arabian Sea
ferrous sulphate is due to the presence (a) Chemical energy is converted into
of water molecule in it. light energy. B. Visakha- 2. Biggest port of
patnam India
84. What is the colour of the (b) Carbon dioxide is oxidised to form
carbohydrate. C. Chennai 3. Land-locked
precipitate obtained by passing harbour
(c) Water molecule splits into
CO 2 gas through lime water?
hydrogen and oxygen. D. Kochi 4. Oldest port on the
(a) Green (b) Blue (d) Light energy is directly used to split Eastern Coast
(c) White (d) Brown water.
24 NDA/NA Solved Paper 2021 (II)

Codes Ê (d) The factors affecting the occurrence 99. Which one of the following is not a
A B C D A B C D and distribution of ground water includes writ?
(a) 2 4 3 1 (b) 2 3 4 1 eight criteria; land use, soil, slope, (a) Mandamus (b) Habeas Corpus
(c) 1 3 4 2 (d) 1 4 3 2 rainfall, elevation, lithology, lineament (c) Certiorari (d) Severability
density and drainage density.
Ê (b) The correct matching is Ê (d) Severabilty is not a writ. There are 5
A-2,B-3,C-4,D-1. Thus amount of precipitation (rainfall),
writs mentioned under Article 32 viz,
Mumbai Port is India's largest port by Rate of evaporation and the infiltration of
Habeas Corpus; Mandamus; Prohibition;
size and shipping traffic. Located in the water to the ground (drainage
Certiorari; Quo-Warranto. Under these
West Mumbai on the western coast of density) are relevant factors. Distance
Writs, Judiciary can protect the rights of
India, the Mumbai Port is situated in a from sea has no bearing on distribution
the aggrieved parties.
natural harbor. of ground water.
100. Which one of the following is not a
Visakhapatnam port is the land locked 96. The Constitution (35th Amendment)
port in India and the only major port of
part of Fundamental Rights?
Act of 1974 is related to which one
Andhra Pradesh. It is India's third largest (a) Right to education
of the following States?
state-owned port by volume of cargo (b) Right to establish educational
(a) Mizoram institutions by minorities
handled and largest on the Eastern
(b) Sikkim (c) Right to be conferred with titles
Coast.
(c) Nagaland (d) Right against untouchability
Chennai is the oldest port on the eastern (d) Arunachal Pradesh
coast of India.It is formerly known as Ê (c) Right to be conferred with titles is not
Madras Port, is the second largest Ê (b) 35th amendment to the constitution a fundamental rights as Article 18(1)
container port of India, behind Mumbai's is related to the state of sikkim.It gives abolishes all titles. It prohibits the State
Nhava Sheva. The port is the largest one effect to the wishes of the people of to confer titles on anybody whether a
in the Bay of Bengal. Sikkim for strengthening Indo-Sikkim citizen or a non-citizen.
co-operation and inter-relationship, the Military and academic distinctions
Kochi port is known as the 'Queen of
Bill seeks to amend the Constitution to are, however, exempted from the
Arabian Sea’ as it was an important
provide for the terms and conditions of prohibition.
trade center for spices on the western
association to Sikkim with the Union.
coast of India from the 14th century. 101. Which one of the following is not a
97. Which one of the following best function of the Constitution of India?
93. Which one of the following is a
crater lake in India? describes the electoral system of
(a) To ensure participation of good
(a) Lonar lake
India? people in politics.
(b) Sambhar lake (a) First-Past-the-Post System (b) To guarantee a set of rights to
(c) Chilika lake (b) Proportional Representation citizens.
(d) Vembanad lake (c) Mixed System (c) To define the power of the different
(d) General Ticket organs of government.
Ê (a) Lonar Lake, also known as Lonar (d) To create conditions for a just
crater, is a notified National Geo-heritage Ê (a) First past the post system is the
Monument situated in the state of electoral system of India. In a society.
Maharashtra and is a crater lake. It was first-past-the-post electoral system , Ê (a) To ensure participation of good
created due to a meteorite. It is the only voters cast their vote for a candidate of people in politics is not a function of the
major hoverback in the basalt. their choice, and the candidate who constitution of India as it provides
receives the most votes wins (even if the everyone an opportunity as a freedom to
94. Fine-grained bed of ephemeral lake top candidate gets less than 50%, which participate in democratic politics.
in a desert is also known as can happen when there are more than Rights of the citizens are mentioned in
(a) Playa (b) Oasis two popular candidates. Part III, Separation of powers of
(c) Drumlin (d) Natural levee
98. Which one of the following is a executive, judiciary and legislature is
Ê (a) Playa is a fine-grained bed of non-justiciable right? also defined in the constitution and
ephemeral lake in a desert. A playa is a Directive principles of state policy
dry, vegetation-free, flat area at the (a) Right to adequate livelihood
mentions provisions for a just society.
lowest part of an undrained desert basin. (b) Right against exploitation
It is a location where ephemeral lakes (c) Right of accused 102. Which one of the following
form during wet periods, and is underlain (d) Right to life and personal liberty statements about biodiversity is not
by stratified clay, silt, and sand, and correct?
commonly, soluble salts. Ê (a) Right to adequate means of
livelihood under Article 39 is a (a) The term ‘biodiversity’ was coined
95. Which one of the following factors non-justiciable right mentioned in part IV by Walter G. Rosen in 1986.
does not affect the distribution of as Directive principal of state policy. (b) The term ‘biodiversity hotspots’
was coined by Norman Myers in
groundwater? Right against exploitation (Article 23),
1988.
(a) Amount of precipitation Right of accused( Article 20) and Right
(c) The regions having richest
(b) Rate of evaporation to life and perosnal liberty (Article 21)
biodiversity are called ‘biodiversity
(c) Ability of the ground surface to are all fundamental rights enshrined in
hotspots’.
allow water to infiltrate into the part III of the Constitution and are (d) More than 100 hotspots of
groundwater system justiciable in nature under Article 32 of biodiversity are identified in the
(d) Distance from the sea the Indian Constitution. world.
NDA/NA Solved Paper 2021 (II) 25

Ê (d) Statement (d) is incorrect. There are Ê (c) The lithosphere includes the crust 110. Who among the following
36 biodiversity hotspots in the world. and, below, the uppermost layer of the described the Directive Principles of
These regions have very high level of mantle; it floats on the weaker State Policy as the novel feature of
species richness or high level of asthenosphere.The lithosphere is the
the Constitution of India?
biodiversity concentration. The term outer solid shell of the Earth. As the
biodiversity was given by walter G. Rosen cooling surface layer of the Earth's
(a) Jawaharlal Nehru
in 1986, while ‘Biodiversity Hotspot’ term convective system, the lithosphere (b) Rajendra Prasad
was joined by Norman Myers in 1988. thickens over time. (c) S.N. Mukherjee
(d) B.R.Ambedkar
103. The Earth’s atmosphere is mainly 107. Which one of the following
heated by which one of the following? countries has the maximum time Ê (d) Dr. B.R. Ambedkar described the
Directive Principles of State Policy as the
(a) Short wave solar radiation difference from Greenwich Mean novel feature of the Constitution of India.
(b) Reflected solar radiation Time (GMT) ? Directive Principle of State Policy's
(c) Long wave terrestrial radiation (a) India (b) Nepal (“DPSP”) are a set of social and
(d) Scattered solar radiation (c) Sri Lanka (d) Bhutan economic obligation imposed on the
Ê (c) Earth’s atmosphere is mainly heated Ê (d) Bhutan (GMT+6) has the maximum
Government (Union & State) to establish
by Long wave terrestrial radiation.The time difference from the Greenwhich a welfare society.
insolation received by the earth is in the Mean Time. India (GMT+5:30), 111. Cornea in human eye
form of shortwaves. The earth after being Nepal (GMT +5:45) and Sri Lanka
heated itself becomes a radiating body
(a) is a light sensitive screen.
(GMT+5:30) are all behind Bhutan and (b) is a muscular diaphragm.
and it radiates energy to the atmosphere closer to GMT.
in the long waveform. (c) contains blood vessels.
108. Which one of the following is not a (d) is composed of proteins and cells.
104. Which one of the following is the feature of the Constitution of India?
correct sequence of layers as we Ê (d) Cornea in human eye is composed
(a) It provides a set of basic rules. of protein and cells. The cornea is made
move from the Earth’s surface (b) It specifies the power of the up of cellular and acellular components.
upwards? Government. The cellular components include the
(a) Troposphere, Stratosphere, (c) It is the supreme law of the land. epithelial cells, keratocytes, and
Thermosphere, Mesosphere (d) It specifies the supremacy of the endothelial cells. The acellular
(b) Troposphere, Stratosphere, judiciary. component includes collagen and
Mesosphere, Thermosphere glycosaminoglycans. The epithelial cells
(c) Thermosphere, Mesosphere, Ê (d) Primacy of judiciary is not one of the are derived from epidermal ectoderm.
provisions of the Indian constitution. It
Stratosphere, Troposphere
(d) Stratosphere, Mesosphere,
provides a set of rights in Part III of the 112. Power of a lens of focal length 25
constitution, it also provides a basic set cm is
Troposphere, Thermosphere
of rules in the form separation of powers (a) + 2.5 Dioptre (b) + 3 Dioptre
Ê (b) Moving upward from ground level, between different organs of the (c) + 4 Dioptre (d) + 5 Dioptre
the correct sequence of atmospheric government and is the supreme law of
layers are the troposphere (upto 12 km ), the land and nothing in contravention of Ê (c) Power of a lens of focal length 25 cm
stratosphere (located between the constitution can be done in the is +4 Dioptre.
12-50 km), mesosphere (located country. First we will convert the focal length in
between 50-80 km), thermosphere meters.
109. Which one of the following
(80-700 km) and exosphere (located ⇒ 25 cm = 0.25 m
between 700-10000 km). statements about the Attorney
Now, substituting the value of focal
General of India is not correct?
105. Which one of the following is not a length in the formula of power of lens we
(a) He has the right of audience only in get
soil forming factor?
(a) Parent material
the Supreme Court of India. P = 10.25 m −1
(b) He shall receive such remuneration ⇒ P = 4 m −1
(b) Topography as the President may determine.
(c) Climate m-1 is also expressed as diopters or D
(c) He shall be qualified to be
(d) Human habitation appointed as a Judge of the As it is a convex lens so its power is
positive. Thus, the power of the lens is
Ê (d) Human habitation is not a soil Supreme Court.
forming factor. The whole soil, from the (d) He shall give advice to the +4D.
surface to its lowest depths, develops Government of India on all legal 113. What is the total number of
naturally as a result of five factors. The matters. chambers in the stomach of
five factors are: 1. parent material,
2. relief or topography, 3. organisms
Ê (a) Statement (a) is not correct. domestic animals like cattle,
The Attorney General for India is the buffalo, goat and sheep ?
(including humans), 4. climate, and
Indian government's chief legal advisor, (a) Four (b) Two
5. time.
and is its principal Advocate before the (c) One (d) Three
106. Which one of the following best Supreme Court of India.
describes the Lithosphere? The Attorney General has the right of Ê (a) There are four chambers in stomach
of domestic animals like cattle, buffalo,
(a) Upper and lower mantle audience in all Courts in India as well as goat and sheep. Ruminant stomachs
(b) Crust and core the right to participate in the have four compartments: the rumen, the
(c) Crust and upper mantle proceedings of the Parliament, though reticulum, the omasum and the
(d) Lower mantle and core not to vote. abomasum.
26 NDA/NA Solved Paper 2021 (II)

114. Intestinal bacteria are main source 118. Reverberation is a phenomenon According to formula,
of which one of the following associated with a 1
K = mv 2
vitamins? (a) multiple refraction of sound. 2
1
(a) Vitamin E (b) Vitamin C (b) multiple reflection of sound. 100 = × 2000 × v 2
(c) Vitamin B12 (d) Vitamin A (c) single refraction of sound. 2
(d) single reflection of sound. 100 = 1000 × v 2
Ê (c) Intestinal bacteria are main source of
Vitamin B12. Vitamin B12 is readily Ê (b) Reverberation is a phenomenon v 2 = 100
absorbed in the last part of the small associated with a multiple reflection of v = 10.0 m/s
intestine (ileum), which leads to the large sound. Reverberation is the Hence, the correct answer is 10.0 m/s
intestine. However, to be absorbed, the phenomenon of persistence of sound
vitamin must combine with intrinsic after it has been stopped as a result of 122. Which one of the following ions is
factor, a protein produced in the multiple reflections from surfaces such not iso-electronic with F − ?
stomach. as furniture, people, air, etc. (a) O 2− (b) Na +
115. Which one of the following hormones These reflections build up with each (c) Ne (d) N−
reflection and decay gradually as they
is responsible for the development of
are absorbed by the surfaces of objects
Ê (d)

N negative is not iso-electronic with
female sexual characters? F because Isoelectronic species are
in the space enclosed. known as atoms or ions that have the
(a) Prolactin (b) Estrogen
119. Which among the following is true same number of electrons.
(c) Oxytocin (d) Progesterone
for propagation of sound waves? In isoelectronic species, there the
Ê (b) Estrogen is a hormone responsible number of electrons would be equal but
for development of female sexual (a) Sound can travel in vacuum and it
the elements would be different. In other
characters. It acts on organs in the body is a transverse wave in air.
words, ions and atoms which are having
(including the breast and uterus) and as (b) Sound cannot travel in vacuum
equal numbers of electrons are called
a chemical messenger in the brain, and it is a longitudinal wave in air.
the isoelectronic species.
controlling key aspects of the (c) Sound can travel in vacuum and it
reproductive system, including childbirth is a longitudinal wave in air. 123. What is the total number of
and lactation, and aspects of human (d) Sound cannot travel in covalent bonds in methanol?
behaviour. vacuum and it is a transverse (a) 3 (b) 4
wave in air. (c) 5 (d) 6
116. Spherical mirror formula relating
an object distance ‘u’, image Ê (b) Statement (b) is true. Sound cannot Ê (c) There are total number of 5 covalent
travel in vacuum and it is a longitudinal bonds in methanol as carbon is attached
distance ‘v’ and focal length of wave in air. Sound propagates through to 3 H and 1 OH with 4 covalent bonds.
mirror ‘f ’ may be applied to a plane air or other mediums as a longitudinal O and H in OH are bonded with 1
mirror when wave, in which the mechanical vibration covalent bond. So there are total 5
(a) focal length goes to infinity. constituting the wave occurs along the covalent bonds.
(b) focal length goes to zero. direction of propagation of the wave.
(c) image distance goes to zero. 124. Which one of the following is the
120. A tennis ball is thrown in the chemical formula of Plaster of Paris?
(d) image distance goes to infinity.
vertically upward direction and the 1
Ê (a) Spherical mirror formula relating an ball attains a maximum height of (a) CaSO 4 ⋅ H2O
object distance u, image distance v and 2
20 m. The ball was thrown (b) CaSO 4 ⋅ 2H2O
focal length of mirror f may be applied to
approximately with an upward (c) CaSO 4 ⋅ 5H2O
a plane mirror when focal length goes to
infinity. For plane Mirror focal length is velocity of (d) CaSO 4 ⋅ 4H2O
infinite. Thus power = 1/focal length (a) 8 m/s (b) 12 m/s
Ê (a) Plaster of paris is calcium sulphate
results in power being Zero. (c) 16 m/s (d) 20 m/s hemihydrate. Its chemical formula is:
1
117. Nuclear energy is generated by Ê (d) In this question CaSO 4 . H 2O. It is prepared by heating
H (max) = 20m 2
(a) nuclear fission and its expression gypsum to a temperature of 100°C in a
was proposed by Einstein. and g = 10 m/s 2.
kiln; it loses 3/4th of its water of
(b) nuclear fission and its expression According to formula crystallisation and forms plaster of paris.
was proposed by Rutherford. u2
(c) nuclear fusion and its expression Height(max) = 125. The unit of the ratio between thrust
g
was proposed by Bohr. and impulse is same as that of
(d) nuclear fusion and its expression Hence, the answer will be 20 m/s. (a) frequency
was proposed by Heisenberg. 121. An object of mass 2000 g possesses (b) speed
Ê (b) Nuclear energy is generated by 100 J kinetic energy. The object (c) wavelength
nuclear fission and its expression was must be moving with a speed of (d) acceleration
proposed by Rutherford. A nuclear (a) 10.0 m/s (b) 11.1 m/s Ê (a) Thrust is force. Impulse is the integral
reactor is driven by the splitting of atoms, (c) 11.2 m/s (d) 12.1 m/s of a force, F, over the time interval, t, for
a process called fission, where a particle which it acts.
(a ‘neutron’) is fired at an atom, which Ê (a) In this question,
Thus the ratio of thrust i.e F and Impulse
then fissions into two smaller atoms and Mass (M) = 2000 g
would give time inverse which is
some additional neutrons. Kinetic energy (K) = 100 J equivalent to Frequency.
NDA/NA Solved Paper 2021 (II) 27

126. Which one of the following figures 129. Which one of the following is not a (b) It promoted a systematic study of
correctly shows the path of a ray of conservative force? India’s past through Sanskrit
light through a glass prism? (a) Frictional force sources.
(c) It promoted a systematic study of
(b) Electric force
India’s past through Persian
(c) Gravitational force
sources.
(a) (b) (d) Spring force
(d) It promoted a systematic study of
Ê (a) Frictional forces are not conservative India’s past through Western
forces because the amount of work done sources.
by friction depends on the path. One can
associate a potential energy with a Ê (a) Statement a is correct. Tattvabodhini
Patrika promoted a systematic study of
(c) (d) conservative force but not with a
India’s past in the Bengali language.
non-conservative force. A conservative
On 6 October 1839 Debendranath
force is one for which the work done is
Tagore established Tattvaranjini Sabha
independent of path.
Ê (a) Figure a correctly shows the path of which was shortly thereafter renamed the
a ray of light through a glass prism as it 130. A negative work is done when an Tattwabodhini (Truth-seekers) Sabha.
shows the rays moving towards normal applied force F and the The objective of Tattvabodhini Sabha
while going from rarer to denser medium corresponding displacement S are was to encourage religious inquiries and
and away from normal while moving (a) perpendicular to each other. disseminate the essence of Upanishads.
from denser to rarer medium. (b) parallel to each other. 133. In which one of the following years
127. When a light beam falls on a (c) anti-parallel to each other. did the British demarcate a large
triangular glass prism, a band of (d) equal in magnitude. area of land as Damin-i-koh for
colours is obtained. Which one of Ê (c) A negative work is done when an settling the Santhals?
the following statements is correct applied force and the corresponding (a) 1810 (b) 1793
in this regard? displacement S are anti-parallel to each (c) 1885 (d) 1832
(a) Red light bends the most, as the other. The work W that a force F does on
an object is the product of the Ê (d) It was in the year 1832 that the British
refractive index of glass for red demarcated a large area of land as
light is greatest. magnitude F of the force, times the
Damin-i-Koh for settling the Santhals.
magnitude d of the displacement, times
(b) Red light bends the most, as the Damin-i Koh is in the area of Sahebganj,
the cosine of the angle θ between them.
refractive index of glass for red Pakur, and Godda districts in the
In symbols, W = fd cos θ.
light is lowest. Jharkhand. British persuaded the local
(c) Violet light bends the most, as the So, if angle is 180 degree, W will be tribe of Rajmahal Hills i.e. Santhals to live
refractive index of glass for violet negative. in the foothills of Rajmahal by giving land
light is greatest. 131. Which of the following statements to them. “Damin-i-Koh” is a Persian term
(d) Violet light bends the most, as the about the Indian Councils Act of that means 'skirts of the hills'.
refractive index of glass for violet
1861 is/are correct? 134. The Tropic of Cancer does not pass
light is lowest.
1. It enlarged the Governor through which one of the following
Ê (c) Each beam of light, with its own General’s Council for the States?
particular wavelength (or color), is
slowed differently by the glass.The light
purpose of making laws. (a) Manipur (b) West Bengal
waves are refracted as they enter and 2. The Governor General was not (c) Gujarat (d) Jharkhand
leave the prism. The shorter the authorised to increase the Ê (a) The tropic of cancer does not pass
wavelength of the light, the more it is number of members. through the state of Manipur.The Tropic
refracted.Violet light is refracted the most Select the correct answer using the of Cancer passes through eight states in
as it has the shortest wavelength codes given below: India: Gujarat (Jasdan), Rajasthan
causing the coloured light to spread out (a) Only 1 (b) Only 2 (Kalinjarh), Madhya Pradesh (Shajapur),
to form a spectrum. (c) Both 1 and 2 (d) Neither 1 nor 2 Chhattisgarh (Sonhat), Jharkhand
(Lohardaga), West Bengal
128. The image of an object formed by a Ê (a) Statement 1 is correct. Indian (Krishnanagar), Tripura (Udaipur) and
plane mirror is Councils Act of 1861 enlarged the
Mizoram (Champhai).
(a) erect, real and larger. Governor General’s Council for the
(b) erect, virtual and same size. purpose of making laws. For the 135. Which of the following pairs of
(c) inverted, virtual and same size. executive functions of the Council, a fifth crop and product is/are correctly
(d) inverted, real and smaller. member was added. Now there were five matched?
members for home, military, law,
Ê (b) The image formed by a plane mirror 1. Food crop Ragi
revenue, and finance. (A sixth member
is always virtual (meaning that the light for public works was added in 1874.) 2. Cash crop Jute
rays do not actually come from the 3. Plantation crop Coconut
image), erect (upright), and of the same 132. Which one of the following Select the correct answer using the
shape and size as the object it is statements about Tattvabodhini codes given below:
reflecting. Patrika is correct?
(a) Only 1 (b) 2 and 3
A virtual image is a copy of an object (a) It promoted a systematic study of (c) 1, 2 and 3 (d) Only 3
formed at the location from which the India’s past in the Bengali
light rays appear to come. language. Ê (c) All are correctly matched.
28 NDA/NA Solved Paper 2021 (II)

‘Ragi’ also known as Finger Millet, is a Select the correct answer using the 142. Which one of the following
robust, popular food crop and code given below: statements about the ishtahars
marvelous grain crop in India. It is called (a) Only 1 (b) Only 2 issued ‘during the Revolt of 1857 is
dryland crops. Jute is also known as the (c) Both 1 and 2 (d) Neither 1 nor 2 correct?
golden fiber and is India’s major cash
crop. Jute fiber is obtained from the Ê (c) Both 1 and 2 are correct.In 1875, a (a) They glorified the Muslim rule in
committee was appointed to inquire the India.
inner bark of the jute plant. It is used in
conditions of all factories .After effect of (b) They glorified the co-existence of
making gunny bags, mats, ropes, yarn,
this committee , factory act 1881 was different communities under the
carpets and other artifacts.
adopted. The act prohibited the Mughal Empire.
The term plantation crop refers to those employment of children under the age of (c) They glorified the message of
crops which are cultivated on an 7. Children between the ages of 7-12 Islam.
extensive scale in contiguous areas, were to work for a maximum of 9 hours. (d) They glorified the role of Queen
owned and managed by an individual or It was passed during viceroyship of Lord Victoria.
a company. The crops include tea, Rippon.
coffee, rubber, cocoa, coconut, arecanut, Ê (b) Statement b is correct. Ishtahars
oil palm, palmyrah and cashew. 140. Which of the following statements glorified the co-existence of different
about Mahatma Gandhi’s communities under the Mughal Empire.
136. Which one of the following anti-untouchability campaign is/are During the revolt only a few
coalfields is not located in Jharkhand? correct? proclamations and ‘ishtahars’
(a) Jharia (b) Ramgarh (notification) were issued by rebel
1. The All India Harijan Sevak
(c) Deogarh (d) Umaria leaders to propagate their ideas and
Sangh was founded for this persuade people to join the revolt.
Ê (d) Umaria Coalfield is located in Umaria purpose. Proclamation issued by rebel leaders
district in the Indian state of Madhya
2. The campaign was to root out appealed to all sections of the
Pradesh in the valley of the Umrer River,
a tributary of the Son River. untouchability, since it did not population irrespective of cast and
enjoy the sanction of the Hindu creed. The rebellion was seen as a war
137. Which one of the following is the Shastras. in which both Hindus and Muslims were
longest parallel of latitude? Select the correct answer using the equally to lose or gain. It was remarkable
(a) Tropic of Cancer codes given below: that during the uprising, the religious
(b) Tropic of Capricorn (a) Only 1 division between Hindus and Muslims
(c) Arctic Circle was hardly noticeable despite the
(b) Only 2
(d) Equator attempt of the British government.
(c) Both 1 and 2
Ê (d) Equator is the longest parallel of (d) Neither 1 nor 2 143. During the Industrial Revolution,
latitude. The Equator is at 0°, and the
Ê (a) Statement 1 is correct. In 1932, who among the following designed
North Pole and the South Pole are at 90° Gandhi founded the Harijan Sevak the 'flying shuttle loom?
north and 90° south, respectively. The Sangh as part of his efforts to eradicate (a) Samuel Crompton
Equator is the longest circle of latitude the concept of untouchability from (b) Edmund Cartwright
and is the only circle of latitude which India’s caste system. (c) John Kay
also is a great circle.
Statement 2 is incorrect-Untouchability (d) Richard Arkwright
138. The periodic rise and fall of ocean did enjoy the sanction of shastras as it is
Ê (c) John Kay was an English inventor
water in response to gravitational based on the caste-based system whose most important creation was the
forces is called mentioned as varna system in Rig Veda flying shuttle, which was a key
dividing society into four classes. contribution to the Industrial Revolution.
(a) Current (b) Waves
(c) Tides (d) Tsunami 141. The founders of the Paramhansa Through this machine one person could
Mandali founded in Maharashtra operate a shuttle across a very wide
Ê (c) The periodic rise and fall of ocean loom, which greatly increased the rate of
water in response to gravitational forces believed in which one of the
cloth production.
is called Tides. Tides are long-period following?
waves that move through the oceans in (a) Social distancing along casts lines 144. Match List I with List II and select
response to the forces exerted by the (b) Glorification of caste system the correct answer using the codes
moon and sun. Tides originate in the given below the lists :
(c) Forbidding widow remarriage
oceans and progress toward the
coastlines where they appear as the
(d) One God and in breaking caste rules List I (Dam) List II (State)
regular rise and fall of the sea surface. Ê (d) The founders of the Paramhansa A. Hirakud 1. Gujarat
Mandali founded in Maharashtra B. Panchet 2. Bihar
139. Which of the following statements believed in One God and in breaking
about the first Indian Factory Act C. Kosi 3. Odisha
caste rules.
passed in 1881 is/are correct? D. Ukai 4. Jharkhand
Paramahansa Mandali was a secret
1. The Act dealt primarily with the socio-religious group, established in Codes
problem of child labour. 1849, in Bombay and is closely related A B C D A B C D
2. The Act laid down that children to Manav Dharma Sabha which was (a) 3 4 2 1 (b) 3 2 4 1
between 7 years and 12 years of founded in 1844. It was started by (c) 1 2 4 3 (d) 1 4 2 3
age would not work for more Durgaram Mehtaji, Dadoba Pandurang,
and a group of his friends.
Ê (a) The correct matching is A-3, B-4,
than 9 hours a day. C-2, D-1.
NDA/NA Solved Paper 2021 (II) 29

Hirakud Dam Project is a multipurpose Contrary to staple and food crops, cash 1. It was opposed by Ambalal
scheme intended for flood control, crops form a small part of Indian Sarabhai.
irrigation and power generation. The agriculture and are entirely sold and 2. Its 20 points included demands
dam is built across river Mahanadi at exported. Indian agriculture is
for civil liberties and adult
about 15 km upstream of Sambalpur subsistence-based, dependent on
town in the state of Odisha. Monsoon rains and 1.25 billion people
suffrage.
are dependent on only 2.4 percent of Select the correct answer using the
Panchet dam is built across the
arable land in the world. codes given below :
Damodar River in Dhanbad District's
Panchet area in Jharkhand. (a) Only 1
147. Which one of the following is not a (b) Only 2
Koshi High Dam, is a multipurpose specified Sub-Mission of the
project proposed to be constructed on (c) Both 1 and 2
National Livestock Mission?
the Saptakoshi River of Nepal. The (d) Neither 1 nor 2
(a) Livestock development
project is primarily aimed to control
(b) Pig development in North-Western Ê (b) Option 2 is correct. The Karachi
floods in south-east Nepal and northern Resolution was passed by the Indian
region
Bihar of India. National Congress at its 1931 Karachi
(c) Fodder and feed development
Ukai dam is the largest multipurpose (d) Skill development, technology session.
project so far completed in Gujarat state. transfer and extension Elections on the basis of Universal Adult
The Ukai dam is located across Tapi Franchise and Basic civil rights of
River near Ukai village of Fort-Songadh Ê (b) Pig development in the north-western
region is not a specified Sub-Mission of freedom of speech, Freedom of Press,
taluka in Surat district. Freedom of assembly, Freedom of
the National Livestock Mission. National
145. Which one of the following features Livestock Mission scheme is association were included in the 20
is the result of erosion and implemented with the following three points put forward in the resolution.
deposition work of a river? Sub-Missions: Sub-Mission on Breed 150. Which of the following statements
(a) Pothole (b) Oxbow lake Development of Livestock & Poultry, about the Law Commission headed
(c) Levee (d) Rapid Sub-Mission on Feed and Fodder
by Lord Macaulay is/are correct ?
development, Sub-Mission on Extension
Ê (a) Pothole is the result of erosion and and Innovation. 1. It attempted to codify the laws.
deposition work of a river. Pothole is a 2. It was opposed to uniform
cylindrical, bowl-shaped, or irregular 148. Who among the following wrote
system of Courts.
hollow that is usually deeper than wide. the famous text ‘A Discourse on
Political Economy’? Select the correct answer using the
It is formed in the rocky bed of a stream
codes given below:
by either the grinding action of sediment (a) Montesquieu (b) Voltaire
whirled around by stream eddies or the (c) Rousseau (d) Adam Smith (a) Only 1
force of fast flowing water. Potholes (b) Only 2
usually have spirally grooved surfaces.
Ê (c) Jean-Jacques Rousseau wrote the (c) Both 1 and 2
famous text ‘A Discourse on Political (d) Neither 1 nor 2
146. Which one of the following is Economy’ in 1775. A revolutionary and
not a feature of agriculture in controversial work, this book is a classic Ê (a) Option 1 is correct. Lord Macaulay
of political theory and a key source of headed the first such Law Commission
India? established in 1834 under the Charter
democratic ideals.
(a) Subsistence agriculture Act of 1833. It recommended
(b) Pressure of population on 149. Which of the following statements codification of the Penal Code, the
agriculture about the resolution adopted in the Criminal Procedure Code, and a few
(c) Dependence upon Monsoon Karachi session of the Indian other matters.
(d) Predominance of cash crops National Congress with regard to The statement (2) is incorrect as it
Ê (d) Predominance of cash crops is not a fundamental rights and economic supported the idea of uniform system of
feature of agriculture in India. policy is/are correct? courts.
NDA /NA
National Defence Academy/Naval Academy

SOLVED PAPER 2021 (I)

PAPER I : Mathematics
1. The smallest positive integer n for 3. If ∆ is the value of the determinant and we know that
which a1 b1 c1 n
C 0 + nC1 + n C 2 + ... + nC n = 2 n
n2 ∴ nC1 + nC 2 + … + nC n = 2 n − nC 0
1 − i  a2 b2 c 2
  =1 = 2n − 1
1 + i  a3 b3 c 3
Hence, option (b) is correct.
where i = −1, is then what is the value of the
(a) 2 (b) 4 (c) 6 (d) 8
following determinant? 5. If a + b + c = 4 and
n2 pa1 b1 qc 1 ab + bc + ca = 0, then what is the
1− i 
Ê (a)   = 1, where i = −1 pa 2 b 2 qc 2 value of the following
1 + i 
pa 3 b 3 qc 3 determinant?
n2
1− i 1− i a b c
 ×  =1 (p ≠ 0 or 1, q ≠ 0 or 1)
1 + i 1 − i  b c a
(a) p∆ (b) q∆
n2 (c) ( p + q ) ∆ (d) pq∆
 1 + i 2 − 2i  c a b
  =1 a1 b1 c1
 1− i
2
 (a) 32 (b) − 64
Ê (d) Given, a2 b2 c 2 = ∆ (c) − 128 (d) 64
n2
 1 − 1 − 2i  a3 b3 c 3
  =1 Ê (b) Let
 1+ 1  pa1 b1 qc1 a b c
⇒ (− i ) n2
= (− i )4 ∴ pa2 b 2 qc 2 ∆ = b c a
pa3 b 3 qc 3 c a b
⇒ n2 = 4
a1 b1 c1 a+ b+c b c
n=2
= p ⋅ q ⋅ a2 b2 c 2 = a+ b+c c a
Hence, option (a) is correct.
a3 b3 c 3 a+ b+c a b
2. The value of x, satisfying the
= pq∆ (by C1 → C1 + C 2 + C 3 )
equation log cos x sin x = 1, where
1 b c
π Hence, option (d) is correct.
0 < x < , is = (a + b + c ) 1 c a
2 4. If C 0 , C 1, C 2 , ..., C n are the
π π π π 1 a b
(a) (b) (c) (d) coefficients in the expansion of
12 3 6 4 (1 + x )n , then what is the value of (To take common a + b + c from C1)
π 0 b−c c − a
Ê (c) logcos x sin x = 1, where 0 < x < C 1 + C 2 + C 3 + ... + C n ?
2 = (a + b + c ) 0 c − a a−b
(a) 2 n (b) 2 n − 1
⇒ (cos x )1 = sin x ⇒ cos x = sin x 1 a b
(c) 2 n − 1 (d) 2 n − 2
⇒ tan x = 1 ⇒ tan x = tan π / 4 (by R1 → R1 − R 2, R 2 → R 2 − R 3 )
Ê (b)Q(1 + x) = C 0 + C1 x + C 2
n n n n
⇒ x = π/4 = ( a + b + c ) [( b − c )( a − b ) − (c − a )2 ]
Hence, option (c) is correct. x + … + Cnx
2 n n

= ( a + b + c ) ( ab − b 2 − ca + bc
NDA/NA Solved Paper 2021 (I) 31

− c 2 − a 2 + 2ca ) (a) 0 (b) 1 (c) 2 (d) 3


1 1 1
= − ( a + b + c ) ( a + b + c − ab − bc − ca )
2 2 2
Ê (b) + +…+
log 2 n log 3 n log100 n
= − ( a + b + c ) [( a + b + c )2 − 3 ( ab + bc + ca )]
= log n 2 + log n 3 + log n 4 + ... + log n 100
= − ( 4) [16 − 0] = − 64.
= log n(2 ⋅ 3 ⋅ 4 ⋅ 5…100)
6. The number of integer values of k, for which the equation = log100! (100 !) [Q n = 100 !]
2sin x = 2k + 1 has a solution, is =1 [Q log aa = 1]
(a) zero (b) one
Hence, option (b) is correct.
(c) two (d) four
Ê (c) Given, 10. If z = 1 + i , where i = −1, then what is the modulus of
2 sin x = 2 k + 1 2
z+ ?
Q − 1 ≤ sin x ≤ 1 ⇒ − 2 ≤ 2 sin x ≤ 2 z
− 2 − 1 ≤ 2 sin x − 1 ≤ 2 − 1 (a) 1 (b) 2 (c) 3 (d) 4
− 3 ≤ 2k ≤ 1 Ê (b) z = 1 + i , where i = −1
−3 1
≤ k ≤ ⇒ − 1⋅ 5 ≤ k ≤ 0 ⋅ 5 2 2 2 (1 − i )
2 2 z+ = (1 + i ) + = (1 + i ) + ×
z (1 + i ) (1 + i ) (1 − i )
∴Integer values of k = − 1, 0
2(1 − i )
Hence, option (c) is correct. = (1 + i ) + =| 1 + i + 1 − i |=| 2| = 2
2
7. If a1, a 2 , a 3 , ..., a 9 are in GP, then what is the value of the Hence, option (b) is correct.
following determinant?
ln a1 ln a 2 ln a 3 11. If A and B are two matrices such that AB is of order n × n ,
then which one of the following is correct?
ln a 4 ln a 5 ln a 6
(a) A and B should be square matrices of same order.
ln a 7 ln a 8 ln a 9 (b) Either A or B should be a square matrix.
(a) 0 (b) 1 (c) Both A and B should be of same order.
(d) Orders of A and B need not be the same.
(c) 2 (d) 4
Ê (a) Let first term and common ratio of GP are a and r respectively. Ê (d) Given that, order of matrix AB = n × n
log a1 log a2 log a3 log a log ar log ar 2 If we take A n× p and Bp × n, then AB will be of order n × n.
∴ log a4 log a5 log a6 = log ar 3
log ar 4
log ar 5 So, orders of A and B need not be the same, is correct.
log a7 log a8 log a9 log ar 6 log ar 7 log ar 8 Hence, option (d) is correct.

log a log a + log r log a + 2 log r 12. How many matrices of different orders are possible with
= log a + 3 log r log a + 4 log r log a + 5 log r elements comprising all prime numbers less than 30?
(a) 2 (b) 3 (c) 4 (d) 6
log a + 6 log r log a + 7 log r log a + 8 log r
Ê (c)QPrime numbers less than 30 = {2, 3, 5, 7, 11, 13, 17, 19, 23,
[Qlog mn = log m + log n]
29}
log a log r log r
⇒ Number of elements = 10
= log a + 3 log r log r log r
∴Possible order of matrices with
log a + 6 log r log r log r 10 elements = 10 × 1, 1 × 10, 2 × 5, 5 × 2
(by C 2 → C 2 − C1 and C 3 → C 3 − C 2) ∴Nubmer of matrices of different order = 4
=0 [QC 2 = C 3] Hence, option (c) is correct.
8. If the roots of the quadratic equation x 2 + 2x + k = 0 are p q
13. Let, A =
real, then r s
(a) k < 0 (b) k ≤ 0 where p , q , r and s are any four different prime numbers
(c) k < 1 (d) k ≤ 1 less than 20. What is the maximum value of the
Ê (d) Given quadratic equation, determinant?
x2 + 2 x + k = 0 … (i) (a) 215 (b) 311 (c) 317 (d) 323
p q
Since, roots are real Ê (c) A = r s , prime numbers less than 20
⇒ D ≥ 0 ⇒ b 2 − 4ac ≥ 0
= {2, 3, 5, 7, 11, 13, 17, 19} ⇒ A = ps − rq
(2 )2 − 4(1) ( k ) ≥ 0 ⇒ 4 ≥ 4k ⇒ k≤1
For maximum values of A, p and s must be maximum and r and q
Hence, option (d) is correct. must be minimum.
9. If n = 100!, then what is the value of the following? Then, p = 17, s = 19, r = 2, q = 3
1 1 1 1 ∴ A = 17 × 19 − 2 × 3
+ + + ... + = 323 − 6 = 317
log 2 n log 3 n log 4 n log100 n
Hence, option (c) is correct.
32 NDA/NA Solved Paper 2021 (I)

x 1 3
14. If A and B are square matrices of order 2 such that
det( AB ) = det( BA ), then which one of the following is correct? Ê (c) Given, 0 0 1 =0 … (i)
(a) A must be a unit matrix 1 x 4
(b) B must be a unit matrix − 1 ( x 2 − 1) = 0 ⇒ x 2 − 1 = 0 ⇒ x 2 = 1
(c) Both A and B must be unit matrices
⇒ x = ± 1 ⇒ 1 − x2 = 0
(d) A and B need not be unit matrices
∴ x =1
2
Ê (d) A2 × 2 and B2 × 2 are two matrices
x = + 1, − 1
and| AB| = | BA | ⇒| A || B| = | B|| A |
Hence, option (c) is correct.
1 2  −2 1 
Let A =  , B =  3 / 2 then,| AB| = | BA |
 3 4   − 1 / 2  19. What is the value of the following?
Hence, we can say A and B need not be the unit matrices.
tan 31° tan 33° tan 35° ...tan 57 ° tan 59 °
(a) − 1 (b) 0 (c) 1 (d) 2
Hence, option (d) is correct.
Ê (c) tan 31º⋅ tan 33º⋅ tan 35º … tan 57º⋅ tan 59º
15. What is cot 2x cot 4 x − cot 4 x cot 6x − cot 6x cot 2x equal = tan 31º ⋅ tan 33°⋅ tan 35º … x tan 45° x K tan( 90° − 33° )
to? ⋅ tan( 90° − 31º )
(a) − 1 (b) 0 (c) 1 (d) 2 = tan 31°⋅ tan 33°⋅ tan 35°… cot 35° ⋅ cot 33°⋅ cot 31°
Ê (c)Qcot 6 x = cot(2 x + 4 x) = (tan 31°⋅ cot 31° ) ⋅ (tan 33°⋅ cot 33° ) ⋅ (tan 35°⋅ cot 35° ) …
cot 2 x ⋅ cot 4 x − 1  cot A cot B − 1 = 1⋅ 1⋅ 1 … = 1
cot 6 x = Qcot ( A + B) =  Hence, option (c) is correct.
cot 2 x + cot 4 x  cot A + cot B 
1 x x +1
⇒ cot 6 x ⋅ cot 2 x + cot 6 x ⋅ cot 4 x = cot 2 x ⋅ cot 4 x − 1
∴ cot 2 x ⋅ cot 4 x − cot 4 x ⋅ cot 6 x − cot 6 x ⋅ cot 2 x = 1 20. If f ( x ) = 2x x ( x – 1) x ( x + 1)
Hence, option (c) is correct. 3x ( x – 1) 2( x – 1)( x – 2) x ( x + 1)( x – 1)
3 then what isf ( − 1) + f (0) + f (1) equal to?
16. If tan x = − and x is in the second quadrant, then what
4
is the value of sin x ⋅ cos x ? (a) 0 (b) 1
6 12 6 12 (c) 100 (d) − 100
(a) (b) (c) − (d) −
25 25 25 25 1 x x+1
Ê (d) Given, Ê (a) f( x) = 2x x( x − 1) x( x + 1)
−3 3 x( x − 1) 2( x − 1) ( x − 2 ) x( x + 1) ( x − 1)
tan x = and x is in the 2nd quadrant.
4
1 −1 0 1 0 1
Let perpendicular be 3k and base be 4k, then f( − 1) = − 2 2 0 = 0 ⇒ f( 0) = 0 0 0 = 0
Hypotenuse = ( 3k )2 + ( 4k )2 = 5k 6 12 0 0 4 0
3 −4 1 1 2
sin x = and cos x =
5 5 f(1) = 2 0 2 =0
3  − 4  − 12 0 0 0
∴sin x ⋅ cos x = ×   =
5  5  25
∴f( − 1) + f( 0) + f(1) = 0 + 0 + 0 = 0
Hence, option (d) is correct. Hence, option (a) is correct.
17. What is the value of the following? π
21. The equation sin −1 x − cos −1 x = has
 7 π   5π  6
cosec   sec  
 6  3 (a) no solution (b) unique solution
4 4 (c) two solutions (d) infinite number of solutions
(a) (b) 4 (c) − 4 (d) − −1 −1 π
3 3 Ê (b)Qsin x − cos x = … (i)
6
7 π  5π   π  π
Ê (c) cosec   ⋅ sec   = cosec  π +  ⋅ sec  2 π − 
 and we know that sin− 1 x + cos − 1 x =
π
6 3 6  3 … (ii)
2
π π
= − cosec ⋅ sec = − 2 × 2 = − 4 Adding Eqs. (i) and (ii), we get
6 3
π π 2π
Hence, option (c) is correct. 2 sin− 1 x = + ⇒ 2 sin− 1 x =
6 2 3
18. If the determinant −1 π
⇒ sin x =
x 1 3 3
π 3
0 0 1 =0 x = sin =
3 2
1 x 4 Hence, the given equation has a unique solution.
then what is x equal to? Hence, option (b) is correct.
(a) − 2 or 2 (b) − 3 or 3 (c) − 1 or 1 (d) 3 or 4
NDA/NA Solved Paper 2021 (I) 33

22. What is the value of the following? (a) 0 (b) 1 ladder, the elevation of the top of
(c) 2 tanθ (d) 2 cot θ
(sin 24 ° + cos 66° ) (sin 24 ° − cos 66° ) 2
the flagstaff is 75°. What is the
(a) − 1 (b) 0 (c) 1 (d) 2 1 + tan2 θ 1 − tanθ  height of the flagstaff ?
Ê (a) − 
Ê (b) (sin24º + cos 66º )(sin24º − cos 66º ) 1 + cot θ  1 − cot θ 
2 (a) 12 m (b) 9 m
2 (c) ( 6 + 3 ) m (d) ( 6 + 3 3 ) m
= (sin24º + cos 66º )  
{sin( 90° − 66º ) − cos 66º } 1 + tan2 θ  1 − tanθ  Ê (d) Let AC be a vertical flagstaff.
= − 
[Qsin( 90° − θ) = cos θ] 1 1 − 1  ∴ CD = 6 m, BD = 6 m
1+
= (sin24º + cos 66º ) (cos 66º − cos 66º ) tan2 θ  tanθ  ∠CBD = 75°
2 C
= (sin24º + cos 66º )( 0) = 0  1 + tan2 θ   tanθ(1 − tanθ)
= tan2 θ − 
Hence, option (b) is correct.  tan2 θ + 1  tanθ − 1 
6m 15°
23. A chord subtends an angle 120° at = tan2 θ − tan2 θ = 0
the centre of a unit circle. What is Hence, option (a) is correct. D
6m
the length of the chord?
(a) 2 − 1 units (b) 3 − 1 units 26. What is the interior angle of a h
75°
(c) 2 units (d) 3 units regular octagon of side length 2 cm?
π 3π 3π 3π 60°
B
Ê (d) Given, radius of the circle = 1 unit (a)
2
(b)
4
(c)
5
(d)
8
A
Let AD = h meter
Ê (b) Given, length of side of regular
octagon = 2 cm In ∆ABC
O QSum of interior angles of octagon 90 + 75 + ∠C = 180° [Q sum of interior
= ( 8 − 2 ) × 180º angle of triangle is 180°]
1 120° 1
= 6 × 180° ∠C = 15°
[Qsum of interior angles of polygon In ∆BCD,
A B
= ( n − 2 ) × 180º ] BD = CD ⇒ ∠BCD = ∠CBD = 15º
6 × 180º ∴ ∠ABD = 75º − 15º = 60º
∠AOB = 120º ∴Interior angle =
8 h 3 h
In ∆ABD, sin 60º = ⇒ =
By using cosine rule, 3π 6 2 6
= 135º =
OA 2 + OB2 − AB2 4 h= 3 3m
cos 120º = … (i)
2 ⋅ OA ⋅ OB Hence, option (b) is correct. ∴Height of the flagstaff = ( h + 6) m
Let AB = x unit, OA = 1 unit, OB = 1 unit = ( 3 3 + 6) m
27. If 7 sin θ + 24 cos θ = 25, then what is
From Eq. (i),
the value of (sin θ + cos θ)? Hence, option (d) is correct.
− 1 1 + 1 − x2 26 6 31
= ⇒ − 1 = 2 − x2 (a) 1 (b) (c) (d) 29. The shadow of a tower is found to
2 2 ⋅ 1⋅ 1 25 5 25 be x metre longer, when the angle
⇒ x 2 = 3 ⇒ x = 3 unit
Ê (d) Given, 7 sinθ + 24cos θ = 25 of elevation of the sun changes
Hence, option (d) is correct. Since, we know that if from 60° to 45°. If the height of the
24. What is (1 + cot θ − cosecθ) a sinθ + b cos θ = c tower is 5(3 + 3 ) m, then what is x
(1 + tan θ + sec θ ) equal to? then b sinθ − a cos θ = a2 + b 2 − c 2 equal to?
(a) 1 (b) 2 (c) 3 (d) 4 (a) 8 m (b) 10 m
Q 7 sinθ + 24 cos θ = 25 … (i)
Ê (b) (1 + cot θ − cosec θ) ∴24 sinθ − 7 cos θ = 7 + 24 − 25 2 2 2 (c) 12 m (d) 15 m
(1 + tanθ + sec θ) Ê (b) In the given diagram,
24 sinθ − 7 cos θ = 0 … (ii)
 cos θ 1  sinθ 1  AB represents the position of tower,
= 1 + −  1 + +  Eq. (i) × 7 + Eq. (ii) × 24
 sinθ sinθ   cos θ cos θ  where h = 5( 3 + 3 ) m
49 sinθ + 168 cos θ = 175 A
 sinθ + cos θ − 1  sinθ + cos θ + 1
=    576 sinθ − 168 cos θ = 0
 sinθ   cos θ 
625 sinθ = 175
5(3+√3)m

(sinθ + cos θ) 2 − 12
= sinθ =
175
=
7
sinθ ⋅ cos θ 625 25
sin2 θ + cos 2 θ + 2 sinθ cos θ – 1
= 7 
2
24
sinθ cos θ ∴cosθ = 1 −   =
 25  25 45° 60°
1 + 2 sinθ ⋅ cos θ − 1
= =2 7 24 31
D x C y B
sinθ ⋅ cos θ ∴sinθ + cos θ = + =
Hence, option (b) is correct. 25 25 25
2 Hence, option (d) is correct. CD = x m
 1 − tan θ 
1 + tan 2 θ In ∆ABC,
25. What is −  28. A ladder 6 m long reaches a point
1 + cot θ  1 − cot θ 
2
tan 60º =
5( 3 + 3 )
⇒ 3=
5( 3 + 3 )
6 m below the top of a vertical
equal to? BC BC
flagstaff. From the foot of the
34 NDA/NA Solved Paper 2021 (I)

∴ BC = 5( 3 + 1) m Ê (b) Let AB = m, AC = n Ê (c) A = {1, 3, 5} and B = {2, 4, 7}


In ∆ABD,
BC = m + n + mn
2 2 Since, number of elements are same in
5( 3 + 3 ) both the sets.
tan 45º = A
BD ⇒ A and B are equivalent sets.
5( 3 + 3 ) If A = {1, 5, 9}, B = {1, 5, 5, 9, 9}
⇒ 1=
BD Which is nothing but B = {1, 5, 9}
∴ BD = 5( 3 + 3 ) m m n
Since, elements are same in A and B
Since, x = BD − BC ⇒ A and B are equal sets
x = 5( 3 + 3 ) − 5( 3 + 1) Hence, option (c) is correct.
x = 5( 3 + 3 − 3 − 1) B √m2+n2+mn C
36. Consider the following statements
x = 10 m
By using cosine rule, 1. The null set is a subset of every
Hence, option (b) is correct.
AB2 + AC 2 − BC 2 set.
cos A =
30. If 3 cos θ = 4 sin θ, then what is the 2 AB ⋅ AC 2. Every set is a subset of itself.
value of tan( 45° + θ )? m2 + n2 − m2 − n2 − mn 3. If a set has 10 elements, then its
7 7 ⇒ cos A = power set will have 1024
(a) 10 (b) 7 (c) (d) 2 mn
2 4 elements.
−1
⇒ cos A = ⇒ A = 120º Which of the above statements are
Ê (b) If 3cos θ = 4sinθ 2
3 sinθ correct?
⇒ = ∴∠B + ∠C = 180 − ∠A
(a) 1 and 2 only (b) 2 and 3 only
4 cos θ [Q sum of interior angle is 180°]
3 (c) 1 and 3 only (d) 1, 2 and 3
⇒ tanθ = = 180º − 120º
4 Ê (d) Since we know that null set is a
∠B + ∠C = 60º subset of every set and every set is a
tan 45º + tanθ
∴tan( 45° + θ) = Hence, option (b) is correct. subset of itself.
1 − tan 45°⋅ tanθ
34. What is the area of the triangle If n( A ) = 10
3
1+ ∴n( P( A )) = 2 10 = 1024
= =44+ 3
=7
ABC with sides a = 10cm, c = 4cm
1 − 1×
3 4−3 and angle B = 30°? ∴all the given statements are true.
4 (a) 16 cm2 (b) 12 cm2 Hence, option (d) is correct.
Hence, option (b) is correct. (c) 10 cm2 (d) 8 cm2
π 37. Let R be a relation defined as xRy if
31. tan −1 x + cot −1 x = holds, when Ê (c) Given, a = 10 cm and only if 2x + 3y = 20, where
2
(a) x ∈ R c = 4 cm x , y ∈ N . How many elements of
(b) x ∈ R − ( − 1, 1) only ∠B = 30º the form ( x , y ) are there in R ?
(c) x ∈ R − { 0} only A (a) 2 (b) 3
(d) x ∈ R − [− 1, 1] only (c) 4 (d) 6
−1 −1 π Ê (b)Q xRy ⇔ 2 x + 3 y = 20
Ê (a) Since, tan x + cot x = where, x, y ∈ ¥
2 c b
for all x ∈ R. 20 − 2 x
Q y=
Hence, option (a) is correct. 3
1 All ordered pair which satisfies the given
32. If tan A = , then what is cos2A 30°
7 B C relations are (1, 6), (4, 4), (7, 2).
a
equal to? ∴ R = {(1, 6), ( 4, 4), (7, 2 )}
24 18 12 6 1
(a) (b) (c) (d) QArea of triangle = ac sin( ∠B) ∴ n( R ) = 3
25 25 25 25 2
Hence, option (b) is correct.
1 1 1 1
Ê (a) tan A = = × 10 × 4 × sin 30º = × 40 ×
7 2 2 2 38. Consider the following statements
1 − tan2 A 1 − (1 / 7 )2 = 10 sq cm 1. A function f :¢ → ¢, defined by
∴ cos 2 A = =
1 + tan2 A 1 + (1 / 7 )2 Hence, option (c) is correct. f ( x ) = x + 1, is one-one as well
49 − 1 48
as onto.
= = 35. Consider the following statements 2. A function f : ¥ → ¥, defined by
49 + 1 50
1. A = {1, 3, 5} and B = {2, 4, 7 } are f ( x ) = x + 1, is one-one but not
24
cos2 A = equivalent sets. onto.
25
2. A = {1, 5, 9 } and B = {1, 5, 5, 9, 9 } Which of the above statement(s)
Hence, option (a) is correct. are equal sets is/are correct?
33. The sides of a triangle are m, n and Which of the above statements (a) 1 only
is/are correct? (b) 2 only
m + n + mn . What is the sum
2 2
(c) Both 1 and 2
(a) 1 only (b) 2 only
of the acute angles of the triangle? (c) Both 1 and 2 (d) Neither 1 nor 2 (d) Neither 1 nor 2
(a) 45° (b) 60° (c) 75° (d) 90°
NDA/NA Solved Paper 2021 (I) 35

Ê (c) Statement I 2. The sum of z and its conjugate is 4(α 2 + α )


⇒ β=
f :¢ → z a real number. −2
f( x ) = x + 1 Which of the above statement(s) β = − 2α 2 – 2α
Let f( x1 ) = f( x2 ) is/are correct?
Hence, option (a) is correct.
(a) 1 only (b) 2 only
⇒ x1 + 1 = x2 + 1
⇒ x1 = x2
(c) Both 1 and 2 (d) Neither 1 nor 2 43. If one root of 5x 2 + 26x + k = 0 is
Ê (c) Let z = x + iy
⇒ f is one-one in¢. reciprocal of the other, then what is
and every element of co-domain has its z = x − iy the value of k?
pre-image in domain. ∴ z − z = x + iy − x + iy = 2 iy which is (a) 2 (b) 3 (c) 5 (d) 8
⇒ f is onto. an imaginary number. Ê (c) Given quadratic equation
Statement II ⇒ Statement-1 is correct. 5 x 2 + 26 x + k = 0 … (i)
f : ¥→ ¥ Also, z + z = x + iy + x − iy = 2 x which
Let α and β be the roots.
f( x ) = x + 1 is real.
1
⇒ Statement-2 is correct. According to question, β =
Let f( x1 ) = f( x2 ) α
x1 + 1 = x2 + 1 Hence, option (c) is correct. k
QProduct of roots =
⇒ x1 = x2 41. What is the modulus of the complex 5
⇒ f is one-one in ¥. number i 2n + 1( −i )2n − 1, where n ∈ N k
α ⋅β =
But there is no element in ¥ such that 5
and i = −1? 1 k k
f( x ) = 1 ⇒ α⋅ = ⇒1 =
(a) − 1 (b) 1 (c) 2 (d) 2 α 5 5
Hence, f is not onto on ¥. 2n + 1
Ê (b) Let z = i ( − i )2n − 1, where n ∈¥ ⇒ k=5
Given statements are correct.
Hence, option (c) is correct. = ( i )2n( i ) ( − i )2n( − i )−1 Hence, option (c) is correct.

 i  44. In how many ways can a team of


39. Consider the following in respect of = ( i 2n )( −1)2n.( i 2n )  
 −i  5 players be selected from 8 players
a complex number z.
= ( i 4 n )( −1) = ( i 4 )n.( −1) so as not to include a particular
1. (z −1 ) = (z )−1 player?
= − 1 = − 1 + 0i
2. zz −1 = | z | 2 ∴ | z| = 1
(a) 42 (b) 35
(c) 21 (d) 20
Which of the above is/are correct? Hence, option (b) is correct.
(a) 1 only (b) 2 only Ê (c) Given that there are 8 players among
(c) Both 1 and 2 (d) Neither 1 nor 2 42. If α and β are the roots of the which one particular player is there.
equation 4 x 2 + 2x − 1 = 0, then Hence, number of ways to select 5
Ê (a) Let z = x + iy players = 8 − 1C 5
which one of the following is
z = x − iy 7 ×6
1 x + iy correct? = 7C 5 = = 21
( z)− 1 = = 2 (a) β = − 2α − 2α (b) β = 4α − 3α
2 2 1× 2
x − iy x + y2
(c) β = α 2 − 3α (d) β = − 2α 2 + 2α Hence, option (c) is correct.
1 x − iy
Also, z− 1 = = 2 Ê (a) Given quadratic equation
x + iy x + y2 45. What is the coefficient of the
4 x2 + 2 x − 1 = 0 … (i) middle term in the expansion of
x + iy
( z− 1 ) = = ( z)− 1
x 2 + y2
If α, β are the roots of Eq. (i), then these (1 + 4 x + 4 x 2 )5 ?
value will satisfy the given equation.
(a) 8064 (b) 4032
∴Statement 1 is correct. 4α 2 + 2α − 1 = 0 … (ii) (c) 2016 (d) 1008
| z| = x + y
2 2
and 4β 2 + 2β − 1 = 0 … (iii)
Ê (a) (1 + 4 x + 4 x )
2 5

⇒ | z| = x + y
2 2 2
From Eq. (i), = {(1 + 2 x )2} 5 = (1 + 2 x )10
( x − iy) −2
But zz− 1 = ( x + iy) Sum of roots = ∴Total number of term in the expansion
x 2 + y2 4
of (1 + 2 x )10 = 10 + 1 = 11
x 2 + y2 −1
= = 1 ≠ | z|2 α +β=  11 + 1
x 2 + y2
2 ∴ Middle term =   th term
−1  2 
∴Statement 2 is wrong. β= −α
2 = 6 th term
Hence, option (a) is correct. On putting the value of β in Eq. (iii), T6 = T5 +1 = 10C 5 (2 x )5
2
40. Consider the following statements −1  = C 5 × 2 5 × x5
10
4 − α  + 2β − 1 = 0
in respect of an arbitrary complex  2  ∴Coefficient of middle term = C5 ⋅25
10

number z. 1  10 × 9 × 8 × 7 × 6
4 + α 2 + α  − 1 = − 2β = × 2 5 = 8064
1. The difference of z and its 4  1× 2 × 3 × 4 × 5
conjugate is an imaginary
1 + 4α 2 + 4α − 1 = − 2β Hence, option (a) is correct.
number.
36 NDA/NA Solved Paper 2021 (I)

5
Ê (b) If f( x + 1) = x − 3 x + 2
2
46. What is ⇒ [2 a + ( 5 − 1)d ]
2
C (n , 1) + C (n , 2) + ... + C (n , n ) equal Let x + 1 = y
1 5
to? = × [2 a6 + ( 5 − 1)d ] ⇒ x = y − 1 or x → x − 1
4 2
(a) 2 + 2 2 + 2 3 + ... + 2 n ∴f( x ) = ( x − 1)2 − 3( x − 1) + 2
[Qsum of n terms of AP,
(b) 1 + 2 + 2 2 + 2 3 + ... + 2 n
n
S n = [2 a + ( n − 1)d ] = x2 + 1 − 2 x − 3 x + 3 + 2
(c) 1 + 2 + 2 2 + 2 3 + ... + 2 n − 1
2
(d) 2 + 2 2 + 2 3 + ... + 2 n − 1 = x2 − 5 x + 6
4(2 × 2 + 4d ) = 2 a6 + 4d
Ê (c) C ( n, 1) + C ( n, 2 ) + … + C ( n, n) 16 + 16d = 2 a6 + 4d
Hence, option (b) is correct.
= C1 + C 2 + C 3 + … + C n
n n n n
16 + 16d = 2( a + 5d ) + 4d 52. If x 2 , x, − 8 are in AP, then which
{Q C 0 + C1 + C 2 + … + C n = 2 }
n n n n n
16 + 16d = 2 a + 14d one of the following is correct?
= 2 n − nC 0 = 2 n − 1 16 + 16d = 2 × 2 + 14d (a) x ∈ { − 2} (b) x ∈ { 4}
2d = − 12 ⇒ d = − 6 (c) x ∈ { − 2, 4} (d) x ∈ { − 4, 2}
Now, we shall solve the option to check
10
Ê (c) If x , x, − 8 are in AP, then
2
whether sum is 2 n − 1 or not. ∴ S10 = [2 a + (10 − 1)d ]
2
Let’s take 2 x = x2 − 8
= 5[2 × 2 + 9( − 6)]
S = 1 + 2 + 2 2 + 2 3 + … + 2 n − 1 which = 5[4 − 54] ⇒ x − 2x − 8 = 0
2

forms a GP. S10 = − 250 ⇒ ( x − 4) ( x + 2 ) = 0


where a = 1 Hence, option (b) is correct. x ∈ { − 2, 4}
2
r= =2>1 49. Consider the following statements Hence, option (c) is correct.
1
a( r n − 1) 1. If each term of a GP is multiplied 53. The third term of a GP is 3. What is
Q S = by same non-zero number, then the product of its first five terms?
r−1
the resulting sequence is also a (a) 81
( − 1)
12 n
GP. (b) 243
∴ S = = 2n − 1
2 −1 2. If each term of a GP is divided by (c) 729
same non-zero number, then the (d) Cannot be determined due to
Hence, 2 n − 1 = nC1 + nC 2 + … + nC n
resulting sequence is also a GP. insufficient data
∴Option (c) is correct.
Which of the above statements Ê (b) Given
47. What is the sum of the coefficients is/are correct? a3 = 3
of first and last terms in the (a) 1 only (b) 2 only Q a3 = ar 2 in GP [Qan = ar n − 1 in GP]
expansion of (1 + x )2n , where n is a (c) Both 1 and 2 (d) Neither 1 nor 2 ar 2 = 3 … (i)
natural number? Ê (c) Let us take a GP. To find a1 ⋅ a2 ⋅ a3 ⋅ a4 ⋅ a5
(a) 1 (b) 2 a, ar, ar 2, … is in GP.
= a( ar ) ( ar 2 ) ( ar 3 ) ( ar 4 )
(c) n (d) 2 n ⇒ ak, akr, akr 2, … will also be in GP
= a 5r10 = ( ar 2 )5 = 3 5 = 243
Ê (b) Expand (1 + x) by using binomial
2n
where, k is non-zero number.
expansion 1 Hence, option (b) is correct.
If k = , m ≠ 0
= 2nC 0 x 0 + 2n
C1 x + 1 2n
C2x 2
m 54. The element in the ith row and the
a a a j th column of a determinant of
+…+ 2n
C 2n x 2n
⇒ , r, r 2, … will also be in GP.
∴The coefficient of first and last term of
m m m third order is equal to 2(i + j ). What
Hence, both statements are correct. is the value of the determinant?
the expansion
50. How many 5-digit prime numbers (a) 0 (b) 2 (c) 4 (d) 6
= 2nC 0 + 2nC 2n
= 1+ 1= 2 can be formed using the digits 1, 2, Ê (a) Given,
3, 4, 5 if the repetition of digits is aij = 2( i + j )
Hence, option (b) is correct.
not allowed? ∴a11 = 2(1 + 1 ) = 4 , a21 = 2(2 + 1) = 6
48. If the first term of an AP is 2 and (a) 5 (b) 4 (c) 3 (d) 0 a12 = 2(1 + 2 ) = 6 , a22 = 2(2 + 2 ) = 8
the sum of the first five terms is a13 = 2(1 + 3) = 8 , a23 = 2(2 + 3) = 10
Ê (d) Given digits are 1, 2, 3, 4, 5
equal to one-fourth of the sum of Since, the sum of digits a31 = 2( 3 + 1) = 8, a32 = 2( 3 + 2 ) = 10,
the next five terms, then what is the = 1 + 2 + 3 + 4 + 5 = 15 is divisible by 3. a33 = 2( 3 + 3) = 12
sum of the first ten terms? ⇒ Every 5 digit number formed by the 4 6 8 2 3 4
(a) − 500 (b) − 250 given digits will be divisible by 3. ∆ = 6 8 10 = 2 ⋅ 2 ⋅ 2 3 4 5
(c) 500 (d) 250 ⇒ There is no prime number. 8 10 12 4 5 6
Ê (b) Given, first term of an AP ( a) = 2 Hence, option (d) is correct.
= 8 [2(24 − 25) − 3(18 − 20) + 4(15 − 16)]
1
and a1 + a2 + a3 + a4 + a5 = 51. If f ( x + 1) = x 2 − 3x + 2, then what ∆ = 8 [ − 2 + 6 − 4]
4
( a6 + a7 + a8 + a9 + a10 ), where is f ( x ) equal to? ∆ =0
an = a + ( n − 1)d (a) x 2 − 5x + 4 (b) x 2 − 5x + 6 Hence, option (a) is correct.
(c) x 2 + 3x + 3 (d) x 2 − 3x + 1
NDA/NA Solved Paper 2021 (I) 37

55. With the numbers 2, 4, 6, 8, all the Diagonals of a parallelogram bisect Ê (b) Let A = ( − 4, 2 )
each other.
possible determinants with these
O is mid-point of AC. A(–4, 2)
four different elements are
 −3 + 5 4 + 2 
constructed. What is the sum of the ⇒ Coordinate of O  ,  or
 2 2 
values of all such determinants?
(a) 128 (b) 64 (c) 32 (d) 0 (1, 3)
O is mid-point of BD.
Ê (d) Given numbers are 2, 4, 6, 8.  x + 0 y − 4
∴We can form determinant of order 2. ⇒ Coordinate of O is  , 
 2 2 
Number of determinats − A′(4, –2)
 x y 4 
= 4 × 3 × 2 × 1 = 24 or  , 
2 2 
Let’s observe some determinants image point
Therefore, compare the coordinate of O
2 6 6 2 A′ = ( 4, − 2 )
= 8 − 48 = − 40 , = 40 ⇒
x
=1⇒ x=2
8 4 4 8 2 ∴Mid-point of
2 8 6 4 y− 4  − 4 + 4 2 + ( − 2 )
= 8 − 48 = − 40 , = 40 and = 3 ⇒ y = 10 AA′ =  ,  = ( 0, 0)
6 4 2 8 2  2 2 
Hence, the fourth vertex is (2, 10). −2 −2
4 8
= 8 − 48 = − 40 ,
8 4
= 40 Slope of AA′ =
4 − ( − 4)
6 2 2 6 58. If the lines y + px = 1 and
− 4 −1
4 6 8 2 y − qx = 2 are perpendicular, then = =
= 8 − 48 = − 40 , = 40 8 2
8 2 4 6 which one of the following is
correct? Since, AA′ and mirror line are
Hence, we can see that we are getting a perpendicular.
pattern where each determinant value (a) pq + 1 = 0 (b) p + q + 1 = 0
(c) pq − 1 = 0 (d) p − q + 1 = 0 Slope of line mirror
will be neutralised by other value.
−1 −1
Hence, sum of the values of all Ê (c) Given y + px = 1 … (i) = = =2
Slope of AA ′ − 1 / 2
determinants = 0 y − qx = 2 … (ii)
Hence, option (d) is correct. Equation of a line is y − y1 = m ( x − x1 )
Eqs. (i) and (ii) are perpendicular
∴ Equation of a line mirror is
56. What is the radius of the circle ⇒ m1 ⋅ m2 = − 1 where m1 and m2 are the
y − 0 = 2 ( x − 0)
4 x 2 + 4y 2 − 20x + 12y − 15 = 0? slope of Eqs. (i) and (ii)
⇒ y = 2x
− coefficient of x
(a) 14 units (b) 10.5 units and m =
coefficient of y Hence, option (b) is correct.
(c) 7 units (d) 3.5 units
− p − (− q ) 61. Consider the following statements
⇒ × = −1
Ê (d) Given equation of circle 1 1 in respect of the points ( p , p − 3),
4 x + 4 y − 20 x + 12 y − 15 = 0
2 2
⇒ − pq = − 1 (q + 3, q ) and (6, 3)
15
⇒ x 2 + y2 − 5 x + 3 y − =0 ⇒ pq − 1 = 0 1. The points lie on a straight line.
4
Hence, option (c) is correct. 2. The points always lie in the first
On comparing with quadrant only for any value of p
x 2 + y 2 + 2 gx + 2 fy + c = 0 59. If A , B and C are in AP, then the and q.
−5 3 − 15 straight line Ax + 2By + C = 0 will
g = , f = ,c = Which of the above statement(s)
2 2 4 always pass through a fixed point.
is/are correct?
∴Radius = g 2 + f2 − c
The fixed point is (a) 1 only (b) 2 only
(a) (0, 0) (b) ( − 1, 1)
(c) Both 1 and 2 (d) Neither 1 nor 2
=
25 9 15 7
+ + = = 3.5 unit (c) (1, − 2 ) (d) (1, − 1)
4 4 4 2 Ê (a) Given points are A( p, p − 3),
Ê (d) Given A, B, C are in AP. B(q + 3, q ) and C( 6, 3)
Hence, option (d) is correct.
⇒ 2B = A + C As, Points lies on a straight line,
57. A parallelogram has three ⇒ A − 2B + C = 0 … (i) so slope of AB = slope of BC
consecutive vertices ( − 3, 4 ), (0, − 4 ) On comparing A − 2 B + C = 0 q − p+ 3 3−q
and (5, 2).The fourth vertex is =
with the given line Ax + 2 By + C = 0, q + 3− p 6−q − 3
(a) (2, 10) (b) (2, 9) we get x = 1, y = − 1  y2 − y1 
(c) (3, 9) (d) (4, 10) Hence, line Ax + 2 By + C = 0 will pass Qslope of a line = 
 x2 − x1 
Ê (a) through (1, − 1)
Hence, option (d) is correct. 1= 1
D (x, y) C(5, 2)
∴Statement 1 is correct.
60. If the image of the point ( − 4, 2) by But it’s not necessary that the collinear
O a line mirror is (4, − 2), then what is points lie in the first quadrant only.
the equation of the line mirror? ∴Statement 2 is wrong.
(a) y = x (b) y = 2 x
A(–3, 4) B(0, –4) Hence, option (a) is correct.
(c) 4 y = x (d) y = 4x
Let the fourth vertex be D ( x, y)
38 NDA/NA Solved Paper 2021 (I)

62. What is the acute angle between Ê (d) Given point is ( 3 tanθ, 2 sec θ) 67. If l , m, n are the direction cosines of
the lines x − 2 = 0 and ⇒ x = 3 tanθ, y = 2 sec θ the line x − 1 = 2(y + 3) = 1 − z ,
3x − y − 2 = 0 ? x y
= tanθ, = sec θ then what isl 4 + m 4 + n 4 equal to?
(a) 0° (b) 30° (c) 45° (d) 60° 3 2 11 13
(a) 1 (b) (c) (d) 4
l1 : x − 2 = 0 Q sec 2 θ − tan2 θ = 1 27 27
Ê (b)
l2 : 3 x − y − 2 = 0 y2 x2 Ê (b) Given line is
− = 1 which represents conjugate
− Coefficient of x 4 9 x − 1 = 2( y + 3) = 1 − z
∴Slope of line l1, m1 = Hyperbola. x − 1 y + 3 1− z
Coefficient of y ⇒ = =
−1 ⇒ a 2 = 9, b 2 = 4 2 1 2
= =∞ x − 1 y − ( − 3) z − 1
0 a2 ⇒ = =
∴ e = 1+ 2 1 −2
The line l1 is parallel to Y-axis or b2
perpendicular to X-axis. ∴Direction ratios are <2, 1, − 2 >
9 13
= 1+ = ∴Direction cosines are
− 3
∴Slope of line, l 2, m2 = = 3 4 4 2 1
−1 13 < , ,
e = 2 2 + 12 + ( −2 )2 2 2 + 12 + ( − 2 )2
The line l 2 makes an angle 60° from 2
−2
positive X-axis. Hence, option (d) is correct.
2 2 + 12 + ( − 2 )2
∴Angle between l1 and
65. Consider the following with regard 2 1 −2
l 2 = 90º − 60º = 30º to eccentricity (e ) of a conic section ∴l = ,m= ,n=
3 3 3
Hence, option (b) is correct. 1. e = 0 for circle 4 4 4
2  1  −2 
63. The point of intersection of 2. e = 1 for parabola ∴l 4 + m4 + n4 =   +   +  
 3  3  3 
diagonals of a square ABCD is at 3. e < 1 for ellipse 16 + 1 + 16 33 11
the origin and one of its vertices is = = =
Which of the above are correct? 81 81 27
at A( 4, 2). What is the equation of (a) 1 and 2 (b) 2 and 3 Hence, option (b) is correct.
the diagonal BD? (c) 1 and 3 (d) 1, 2 and 3
(a) 2 x + y = 0 (b) 2 x − y = 0 68. What is the projection of the line
Ê (d) Since, we know that circle has
(c) x + 2 y = 0 (d) x − 2 y = 0 eccentricity 0 segment joining A(1, 7, − 5) and
Ê (a) Since, diagonal BD passes through and parabola has eccentricity 1 B( − 3, 4, − 2) on Y-axis?
the origin O( 0, 0). (a) 5 (b) 4 (c) 3 (d) 2
and ellipse has eccentricity e < 1
D C
and hyperbola has eccentricity e > 1. Ê (c) A = (1, 7, − 5) and B = ( − 3, 4, − 2 )
Hence, option (d) is correct. ∴Direction ratios of
AB = < ( − 3 − 1), ( 4 − 7 ), ( − 2 + 5) >
(0, 0) 66. What is the angle between the two = < – 4, − 3, 3 >
lines having direction ratios 6, 3, 6 ⇒ a = − 4, b = − 3, c = 3
and 3, 3, 0 ? Direction cosines of Y-axis = < 0, 1, 0 >
A (4, 2) B l = 0, m = 1, n = 0
π π
(a) (b)
0−2 1 6 4 ∴Projection of AB on Y-axis
QSlope of OA = =
0−4 2 π π =| al + bm + cn|
(c) (d)
3 2 = | − 4 × 0 + ( − 3) × 1 + 3 × 0 | = 3
QOA and OB are perpendicular to each
other Ê (b) Direction ratios of line l1 = < 6, 3, 6 > Hence, option (c) is correct.
−1 −1 a1 = 6, b1 = 3, c1 = 6
∴slope of OB = = 69. What is the number of possible
slope of OA 1 / 2 Direction ratios of line l 2 = < 3, 3, 0 >
values of k for which the line
⇒ a2 = 3, b 2 = 3, c 2 = 0
= −2 joining the points (k , 1, 3) and (1, − 2,
a1a2 + b1b 2 + c1c 2
∴Eqs. of BD having slope – 2 and Qcosθ = k + 1) also passes through the point
passes through (0, 0) a12 + b12 + c12 a22 + b 22 + c 22 (15, 2, − 4 )?
y − 0 = − 2 [ x − 0] 6×3+ 3×3+ 6×0 (a) Zero (b) One (c) Two (d) Infinite
=
[Q Equation of a line 6 + 32 + 62 ⋅ 32 + 32 + 0
2
Ê (c) Let A = ( k, 1, 3), B = (1, − 2, k + 1)
⇒ y − y1 = m ( x − x1 )]
27 and C = (15, 2, − 4)
⇒ 2x + y = 0 ⇒ cosθ =
9×3 2 Since, line AB passes through C also.
Hence, option (a) is correct.
1 π Hence, points A, B and C are collinear.
⇒ cos θ = = cos
64. If any point on a hyperbola is 2 4 k 1 3
(3 tan θ, 2sec θ), then what is the π ∴ 1 −2 k+1 =0
∴ θ=
eccentricity of the hyperbola? 4 15 2 −4
Hence, option (b) is correct.
(a)
3
(b)
5
(c)
11
(d)
13 k( 8 − 2 k − 2 ) − 1( − 4 − 15k − 15)
2 2 2 2 + 3(2 + 30) = 0
NDA/NA Solved Paper 2021 (I) 39

6k − 2 k 2 + 19 + 15k + 96 = 0 72. Consider the following statements Which of the above statements are
2 k 2 − 21k − 115 = 0 which is quadratic in respect of a vector c = a + b, not correct?
equation. (a) 1 and 2 (b) 2 and 3
where | a | = | b| ≠ 0
⇒ k has two values. (c) 1 and 3 (d) 1, 2 and 3
1. c is perpendicular to (a − b ).
Hence, option (c) is correct.
2. c is perpendicular to (a × b ). Ê (d) Statement I
70. The foot of the perpendicular Let a and b are unit vectors
Which of the above statements
drawn from the origin to the plane i.e. |a| = |b| = 1
is/are correct?
x + y + z = 3 is (a) 1 only (b) 2 only a × b = |a||b| sin θ n$
(a) (0, 1, 2) (c) Both 1 and 2 (d) Neither 1 nor 2 |a × b| = |a||b| sin θ
(b) (0, 0, 3) = sin θ ∈ [−1, 1]
Ê (c) c = a + b where a = b ≠ 0
(c) (1, 1, 1) Therefore, statement I is incorrect.
Consider, c ⋅ (a − b ) = (a + b ) ⋅ (a − b )
(d) (− 1, 1, 3) Statement II
= |a|2 − | b |2 = |b|2 − |b|2 = 0
Ê (c) Let the foot of the perpendicular Let a and b are unit vectors
drawn from the origin to the plane ⇒ c is perpendicular to (a − b ). i.e. a ⋅ b = |a||b| cos θ
x + y + z = 3 be ( a, b, c ). Also, c ⋅ (a × b ) = (a + b ) ⋅ (a × b ) = cos θ ∈ [−1, 1]
O (0, 0, 0) = a ⋅ (a × b ) + b ⋅ (a × b ) Therefore, statement II is also incorrect.
= 0 + 0= 0 Statement III
⇒ c is perpendicular to (a × b )
Let a = $i and b = $j
Normal Hence, option (c) is correct.
⇒ |a| = 1,|b| = 1
73. If a and b are two vectors such that |a + b| = | $i + $j| = 2
A
(a, b, c) | a + b| = | a − b| = 4, then which
|a − b| = | $i − $j| = 2
one of the following is correct?
Direction ratios of the plane = < 1, 1, 1 > (a) a and b must be unit vectors ax − xa
(b) a must be parallel to b
76. If lim =− 1
∴Direction ratios of OA and normal will x→ax a − aa
be in the same ratio. (c) a must be perpendicular to b
then what is the value of a?
a−0 b−0 c−0 (d) a must be equal to b
(a) − 1
∴ = = (b) 0 (c) 1 (d) 2
1 1 1 Ê (c) Given,|a + b| = |a − b| = 4 ax − xa
⇒ a = 1, b = 1, c = 1 Ê (c) xlim = −1
⇒|a + b|2 = |a − b|2 →a xa − aa
∴ A = (1, 1, 1)
|a|2 + | b |2 + 2a ⋅ b ax − xa 0 
Hence, option (c) is correct. ⇒ lim = –1  form
= |a| + | b | − 2a ⋅ b
2 2 x→ a xa − aa 0 
71. A vector r = a$i + b$j is equally ⇒ 4 a ⋅b = 0 ⇒a ⋅b = 0 By using L’ Hospital rule,
inclined to both x and y axes. If the ⇒ a must be perpendicular to b. a x loge a − ax a − 1
lim =–1
magnitude of the vector is 2 units, Hence, option (c) is correct. x→ a ax a − 1 − 0
then what are the values of a and b a a loge a − a ⋅ a a − 1
respectively?
74. If a , b and c are coplanar, then what ⇒ = −1
1 1 1 1 is (2a × 3b ) ⋅ 4 c+ (5b × 3c) ⋅ 6a equal a ⋅ a a− 1
(a) , (b) , a a(loge a − 1)
2 2 2 2 to? ⇒ = −1
(c) 2 , 2 (d) 2, 2 (a) 114 (b) 66 (c) 0 (d) − 66 aa
$ $ Ê (c) Given that, a , b and c are coplanar ⇒ loge a = − 1 + 1
Ê (c) r = ai + bj
⇒ [a b c ] = 0 … (i) ⇒ loge a = 0
| r| = a2 + b 2 = 2 ∴ a = e0 = 1
∴(2 a × 3 b ) ⋅ 4 c + ( 5 b × 3 c ) ⋅ 6 a
Y
= 2 ⋅ 3 ⋅ 4 [a b c ] + 5 ⋅ 3 ⋅ 6 [b c a ] ∴ a=1
= 24 [a b c ] + 90 [a b c ] {Q[a b c ] Hence, option (c) is correct.
r b = [b c a ]}
77. A particle starts from origin with a
45° = 24 × 0 + 90 × 0 = 0
X velocity (in m/s) given by the
ai Hence, option (c) is correct. dx
equation = x + 1. The time (in
75. Consider the following statements dt
Since, r is equally inclined from X-axis
1. The cross product of two unit second) taken by the particle to
and Y-axis.
vectors is always a unit vector. traverse a distance of 24 m is
Hence, r makes 45º from the X-axis.
2. The dot product of two unit (a) ln24 (b) ln 5
∴a = | r |cos 45º and b = | r |sin 45º vectors is always unity. (c) 2 ln 5 (d) 2 ln 4
1 1
a=2 × , and b = 2 × 3. The magnitude of sum of two dx
2 2 Ê (c) = x + 1
unit vectors is always greater dt
a = 2 and b = 2 than the magnitude of their dx
difference. ⇒ = dt
Hence, option (c) is correct. x+1
40 NDA/NA Solved Paper 2021 (I)

a
d2
On integrating both sides
dx
R.H.S. Let I = ∫− a g ( x) dx At x = 1, f ′( x) = − 6 < 0
dx 2
∫ x + 1 = ∫ dt a
⇒ I= ∫− a f( x) dx … (i) ∴At x = 1, f ′( x ) is maximum.
ln( x + 1) = t + c … (i) b b Hence, option (c) is correct.
[Q ∫ f( x ) dx = ∫a f ( a + b − x ) dx ]
Since, at t = 0, distance ( x ) = 0 a
83. A 24 cm long wire is bent to form a
∴ln( 0 + 1) = 0 + c a
I= ∫− a f( − x ) dx … (ii) triangle with one of the angles as 60°.
0=c What is the altitude of the triangle
∴ln( x + 1) = t Adding Eqs. (i) and (ii), we get
a having the greatest possible area?
At x = 24 m 2I = ∫− a [ f ( x ) + f( − x )] dx (even function)
(a) 4 3 cm (b) 2 3 cm
t = ln(24 + 1) = ln25 = ln 5 2 a
⇒ 2 I = 2 ∫ [f( x ) + f( − x )] dx (c) 6 cm (d) 3 cm
t = 2 ln 5 0
a Ê (a) C
Hence, option (c) is correct. I= ∫0[ f ( x ) + f( − x )]dx = L.H.S
60°
f (a − x ) ⇒ g ( x ) = f( x ) and
78. What is ∫
a
dx equal a b
0 f ( x ) + f (a − x ) Hence, option (a) is correct.
to?
(a) a (b) 2 a 81. What is the area bounded by B c A
(c) 0 (d)
a y = 16 − x , y ≥ 0 and the X -axis?
2
Given, a + b + c = 24
2
(a) 16 π sq. units (b) 8 π sq. units ⇒ c = 24 − ( a + b )
a f( a − x )
Ê (d) Let I = ∫0 dx … (i) (c) 4 π sq. units (d) 2 π sq. units a2 + b 2 − c 2
f( x ) + f( a − x ) Again cos C =
x→ a − x Ê (b) Shaded portion in the diagram 2 ab
represents the area bounded by a2 + b 2 − c 2
a f( a − a + x ) ⇒ cos 60° =
I=∫ dx y = 16 − x 2 , y ≥ 0 and X-axis. 2 ab
0 f( a − x ) + f( a − a + x )
y 1 a2 + b 2 − c 2
a f( x ) ⇒ =
I= ∫0 f ( a − x ) + f( x )
dx … (ii) (0, 4) 2 2 ab
⇒ ab = a 2 + b 2 − c 2
Adding Eqs. (i) and (ii), we get
a f( a − x ) + f( x ) x ⇒ ab = a 2 + b 2 − [24 − ( a + b )]2
2I = ∫ ⋅ dx (–4, 0) (4, 0)
0 f( a − x ) + f( x ) ⇒ ab = a 2 + b 2 − 576
a (0, –4) − ( a + b )2 + 48 ( a + b )
2I = ∫0 1⋅ dx
⇒ ab = a 2 + b 2 − 576 − a 2 − b 2
2 I = x |a0 Put y = 0, then 16 − x 2 = 0
− 2 ab + 48 ( a + b )
2I = a − 0 ⇒ x=± 4
a 4 ⇒ 3ab − 48( a + b ) = − 576
I=
2
∴ Required area = ∫− 4 16 − x 2 dx
⇒ ab − 16( a + b ) = − 192
4
⇒ ab − 16a = 16b − 192
Hence, option (d) is correct. =2 ∫0 16 − x 2 dx
x +x
3 2 ⇒ a( b − 16) = 16( b − 12 )
79. What is lim equal to? x 16 x
4
16( b − 12 )
x → −1 x 2 + 3x + 2 =2  16 − x 2 + sin−1 ⇒ a=
 2 2 4  0 b − 16
(a) 0 (b) 1 (c) 2 (d) 3
8π 1
= 2 [0 + 8 sin−1 1] = 2 × Again ar( ∆ABC ), A = ab sin C
x + x3
0
2
 2 2
Ê (b) Given, xlim =  from
→ –1 x2 + 3 x + 2  0  = 8 π sq units 1 16( b − 12 )b
= × × sin 60°
By using L′ Hospital rule Hence, option (b) is correct. 2 b − 16
3 x2 + 2 x 1 16( b − 12 )b 3
lim = 82. The curve y = − x 3 + 3x 2 + 2x − 27 = × ×
x→ –1 2x+ 3 2 b − 16 2
has the maximum slope at 4 3( b 2 − 12 b )
3(–1)2 + 2(–1) 3–2 =
= = =1 (a) x = − 1 (b) x = 0 b − 16
2(–1) + 3 –2 + 3
(c) x = 1 d) x = 2 dA
Hence, option (b) is correct. ∴ =4 3
Ê (c) Given that, y = − x + 3 x + 2 x − 27
3 2
db
80. If ∫ [ f ( x ) + f ( − x )]dx = ∫ g ( x )dx
a a
dy  (2 b − 12 )( b − 16) − ( b 2 − 12 b ) ⋅ 1
0 −a Slope = = − 3 x2 + 6 x + 2  
then what is g ( x ) equal to? dx  b − 16 
(a) f( x ) (b) f( − x ) + f( x ) ∴f ′ ( x ) = − 3 x 2 + 6 x + 2 Maximum value for A
(c) − f( x ) (d) None of these For maxima/minima of f ′( x ). dA
=0
d db
Ê (a) Given that, [f ′ ( x )] = − 6 x + 6 = 0
a a dx 4 3
∫0[ f ( x ) + f( − x )] dx = ∫− a g ( x) dx ⇒ 6x = 6 ⇒ x = 1

b − 16
[2 b 2 − 32 b − 12 b

If g ( x ) = f( x ) +192 − b 2 + 12 b] = 0
NDA/NA Solved Paper 2021 (I) 41

⇒ b 2 − 32 b + 192 = 0 (sec x − tan x ) 1


I= ∫ sec 2 x − tan2 x dx =
x
[cos(ln x ) − sin(ln x )]
⇒ ( b − 24) ( b − 8) = 0
(sec x − tan x ) At x = e,
⇒ b = 24, 8 I= ∫ 1
dx dy 1
= [cos(lne ) − sin(lne )]
when, b = 24
= ∫ sec x dx − ∫ tanx dx dx e
16(24 − 12 ) 16 × 12
a= = = 24 1
24 − 16 8 I = log| sec x + tan x | − log| sec x | + C = [cos 1 − sin1]
e
and c = 24 − (24 + 24) = −24 Hence, option (d) is correct.
[Qlne = 1]
It is impossible, Hence, option (a) is correct.
86. What is ∫
dx
when, b = 8 equal to?
16( 8 − 12 ) 16( −4)
sec (tan −1 x )
2
89. If x = e t cos t and y = e t sin t , then
a= = =8 −1
(a) sin x + c (b) tan x + c−1
8 − 16 ( −8) dx
(c) sec −1 x + c (d) cos −1 x + c what is at t = 0 equal to?
and c = 24 − ( 8 + 8) = 8 dx dy
∴So triangle will be equilateral Ê (b) Let I = ∫ sec 2(tan− 1 x) (a) 0 (b) 1 (c) 2e (d) − 1
3
Ê (b) Given that, x = e cos t , y = e sint
t t
∴ Height = (side) dx
2 I= ∫1+ 2
tan (tan −1
x) dx d d t
3 ∴ = et cos t + cos t e
= × 8 = 4 3 cm [Q sec x = 1 + tan x ]
2 2 dt dt dt
2 = et ( − sint ) + cos t ⋅ et
dx
84. If f ( x ) = e | x | , then which one of the = ∫1+ x2 dy
= et
d d
sint + sint ⋅ et
following is correct? −1
dt dt dt
I = tan x+C dy
(a) f ′ ( 0) = 1 (b) f ′ ( 0) = − 1 = et cos t + et sint
Hence, option (b) is correct. dt
(c) f ′ ( 0) = 0 (d) f ′( 0) does not exist
dx dx / dt
Ê (d) Given that, f( x) = e
| x| 87. If x + y = 20 and P = xy , then what ∴ =
dy dy / dt
 ex ; x ≥ 0 is the maximum value of P ?
⇒ f( x ) =  − x (a) 100 (b) 96 dx et (cos t − sint )
; x< 0 =
e (c) 84 (d) 50 dy et (cos t + sint )
LHD at x = 0,
Ê (a) Given, x + y = 20 At t = 0,
f ( 0 + h) − f ( 0 ) dx cos 0º − sin 0º 1− 0
f ′ ( 0 − ) = lim ⇒ y = 20 − x ∴ = =
h → 0− h
P = xy dy cos 0º + sin 0º 1 + 0
−h
e −e 0
 dx 
= lim P = x(20 − x ) =1
h → 0− h  
P = 20 x − x 2  dy  t =0
(by using L’ Hospital rule)
dP Hence, option (b) is correct.
e− h ∴ = 20 − 2 x
= lim − = −1 dx
h → 0− 1 dP
90. What is the maximum value of
For maxima/minima, =0 sin 2x ⋅ cos 2x ?
RHD at x = 0, dx
1
f ( 0 + h) − f ( 0 ) 20 − 2 x = 0 (a) (b) 1 (c) 2 (d) 4
f ′ ( 0 + ) = lim 2
h → 0+ h ⇒ x = 10
eh − e0 Ê (a) Let y = sin2 x ⋅ cos 2 x
= lim d 2P
At x = 10, 2 = − 2 < 0 1
h→ 0 +
h dx y= [2 sin2 x ⋅ cos 2 x ]
2
eh ∴P is maximum at x = 10
= lim = e0 = 1 1
y = sin 4 x
+
h→ 0 1 ⇒ y = 20 − 10 = 10 2
QLHD ≠ RHD ∴Maximum value of P = xy Since, we know that
∴f ′( x ) does not exist at x = 0 = 10 × 10 = 100 −1 1 1
− 1 ≤ sin 4 x ≤ 1 ⇒ ≤ sin 4 x ≤
Hence, option (d) is correct. Hence, option (a) is correct. 2 2 2
1
88. What is the derivative of ∴Maximum value =
85. What is ∫
dx 2
equal to? sin(ln x ) + cos(ln x ) with respect to x
sec x + tan x Hence, option (a) is correct.
(a) ln(sec x ) + ln|sec x + tan x| + c at x = e ?
cos 1 − sin1 sin1 − cos 1 91. What is the derivative of e x with
(b) ln(sec x ) − ln|sec x + tan x| + c (a) (b)
e e
(c) sec x tan x − ln|sec x − tan x| + c cos 1 + sin1
respect to x e ?
(c) (d) 0 xe x ex xe x ex
(d) ln|sec x + tan x| − ln|sec x| + c e (a) e
(b) e
(c) e
(d)
dx ex x x ex e
Ê (d) Let I = ∫ Ê (a) Let y = sin(ln x) + cos(ln x)
Ê (a) Let y1 = e and y2 = x
x e
sec x + tan x
dy 1  1
(sec x − tan x ) ∴ = cos(ln x ) ⋅ +  − sin(ln x ) ⋅  dy1 dy
I=∫
1
× dx dx x  x ∴ = e x , 2 = exe − 1
(sec x + tan x ) (sec x − tan x ) dx dx
42 NDA/NA Solved Paper 2021 (I)

dy1 ex xe x Ê (b) Given, f( x) = sin x 98. Which one of the following


∴ = = e
dy2 exe − 1 ex From the graph of sin x differential equations has the
Hence, option (a) is correct. y general solution y = ae x + be − x ?
92. If a differentiable function f ( x ) d 2y d 2y
3π (a) + y=0 (b) − y=0
f (x ) + 1
3 π 2
dx 2 dx 2
satisfies lim = − , then O x d 2y
x → −1 x 2 − 1 2 π + y=1
dy
− y=0
2π 5π 3π (c) (d)
2 2 dx 2 dx
what is lim f ( x ) equal to? −x
x → −1 Ê (b) Given, y = ae + be
x
3
(a) − (b) − 1 dy
2  π ∴ = ae x − be − x
We can see that f( x ) increases in  0,  dx
(c) 0 (d) 1  2 d 2y
f( x ) + 1 −3 π   5π  = ae x + be − x = y
Ê (b) Given, x →
lim = and decreases in  , π  and  , 3π. dx 2
−1 x2 − 1 2 2   2 
d 2y
f( x ) + 1 ⇒ Statement-1 is wrong and Statement-2 ⇒ − y=0
Q lim has denominator 0 at dx 2
x→ −1 x2 − 1 is correct.
x= −1 Hence, option (b) is correct.
Hence, option (b) is correct.
⇒ lim f( x ) + 1 = 0
x→ −1
95. What is the domain of the function 99. What is the solution of the
⇒ lim f( x ) = − 1 following differential equation?
x→ −1 f ( x ) = 3x ?
Hence, option (b) is correct.  dy 
(a) ( − ∞, ∞ ) (b) ( 0, ∞ ) ln   + y = x
93. If the function (c) [0, ∞ ) (d) ( − ∞, ∞ ) − { 0}
 dx 
a + bx , x < 1 (a) e x + e y = c (b) e x + y = c
Ê (a) Given, f( x) = 3
x

 (c) e − e = c
x y
(d) e x − y = c
f ( x ) =  5, x =1 QWe know that, domain of exponential
 
dy
b − ax , x > 1 function is ( − ∞, ∞ ). Ê (c) Given, ln  + y = x
 dx
∴Domain of 3 x = ( − ∞, ∞ )
is continuous, then what is the  dy 
⇒ ln  = x − y
value of (a + b )? Hence, option (a) is correct.  dx 
(a) 5 (b) 10 96. If the general solution of a ⇒
dy
= ex − y
(c) 15 (d) 20 differential equation is dx
 a + bx ; x < 1 y 2 + 2cy − cx + c 2 = 0, where c is an dy e x
 ⇒ =
Ê (a) Given that, f( x) =  5 ; x=1
arbitrary constant, then what is the dx e y
 b − ax ; x > 1 ⇒ e ydy = e xdx
 order of the differential equation?
Q f( x ) is continuous. (a) 1 (b) 2 (c) 3 (d) 4 On integrating both sides,
⇒ f( x ) will be continuous at x = 1 ∫ e dy = ∫ e dx
y x
Ê (a) Given that, y + 2cy − cx + c = 0
2 2

lim f( x ) = f(1) = lim f( x )


x → 1− x → 1+
Since, the above equation contains only ey + c = ex ⇒ ex − ey = c
one variable constant.
lim ( a + bx ) = 5 = lim ( b − ax ) Hence, option (c) is correct.
x → 1− x → 1+ Hence, order of the differential 2
a+ b=5=b−a equation = 1 100. What is ∫ e (2 ln x + ln x )dx equal to?
⇒ a+ b=5 Hence, option (a) is correct. x4 x3
(a) +C (b) +C
Hence, option (a) is correct. 4 3
97. What is the degree of the following 5 5
94. Consider the following statements 2x x
differential equation? (c) +C (d) +C
in respect of the function 5 5
d 2y + ln x 2 )
f ( x ) = sin x x = 1+ Ê (d) Let I = ∫ e
( 2ln x
dx
2
1. f ( x ) increases in the interval dx (ln x 2 + ln x 2 )
(a) 1 = ∫e dx
(0, π).
(b) 2 2 2 2

2. f ( x ) decreases in the interval = ∫ e 2ln x dx = ∫ eln( x )


dx = ∫ x dx
4
(c) 3
 5π 
5
x
 , 3 π .
(d) Degree is not defined I= +C
 2  d 2y
5
Ê (a) Let x = 1 + 2 Hence, option (d) is correct.
Which of the above statement is/are dx
correct? 2
d y  d 2y
1 101. Consider the following measures of
(a) 1 only ⇒ x2 = 1 + ⇒  2  = x2 − 1 central tendency for a set of N
dx 2
 dx 
(b) 2 only numbers
(c) Both 1 and 2 ∴Degree = exponent of highest order
1. Arithmetic mean
(d) Neither 1 nor 2 derivative = 1
2. Geometric mean
Hence, option (a) is correct.
NDA/NA Solved Paper 2021 (I) 43

Which of the above uses/use all the 105. The following table gives the Ê (c) Mean of 73, 85, 92, 105, 120
data? frequency distribution of number of 73 + 85 + 92 + 105 + 120
x=
(a) 1 only (b) 2 only peas per pea pod of 198 pods 5
(c) Both 1 and 2 (d) Neither 1 nor 2 475
1 2 3 4 5 6 7 =
Ê (c) Since, we know that the measures of Number of peas 5
central tendency are Mean, Median and
Frequency 4 33 76 50 26 8 1 x = 95
Mode. ∴ Sum of deviations from their mean
Where Arithmetic Mean and Geometric What is the median of this = (73 − 95) + ( 85 − 95)
mean are the type mean. distribution? + ( 92 − 95) + (105 − 95) + (120 − 95)
Hence, option (c) is correct. (a) 3 (b) 4 (c) 5 (d) 6 = − 22 – 10 − 3 + 10 + 25 = 0
102. The numbers of Science, Arts and Ê (a) Hence, option (c) is correct.
Commerce graduates working in a Number of Frequency Cumulative 108. Let x be the HM and y be the GM of
company are 30, 70 and 50 Peas frequency two positive numbers m and n. If
respectively. If these figures are 1 4 4 5x = 4y , then which one of the
represented by a pie chart, then 2 33 37 following is correct?
what is the angle corresponding to 3 76 113 (a) 5m = 4n (b) 2 m = n
Science graduates? 4 50 163 (c) 4m = 5n (d) m = 4n
(a) 36° (b) 72° 5 26 189 Ê (d) Given, two positive numbers are m
(c) 120° (d) 168° and n.
6 8 197
Ê (b) The ratio of Science, Arts and 7 1 198 ∴ H.M. of m and n =
2 mn
Commerce graduates m+ n
= 30 : 70 : 50 = 3 : 7 : 5 Σf = 198
2 mn
x= … (i)
∴Angle corresponding to Science N 198 m+ n
3 QN = 198, = = 99
graduates = × 360º 2 2
G.M. of m and n = mn
3+7 + 5 N N 
th term +  + 1 th term y = mn … (ii)
=
3
× 360º = 72º 2 2 
Median = Q 5x = 4y
15 2
 2 mn 
Hence, option (b) is correct.
=
99th term + 100th term 5  = 4 mn
2  m + n
103. For a histogram based on a 3+ 3
= =3 Squaring both sides, we get
frequency distribution with unequal 2
2  5mn 
class intervals, the frequency of a   = (2 mn )
2
∴ Median = 3
class should be proportional to  m + n
Hence, option (a) is correct.
(a) the height of the rectangle 25m2n2
⇒ = 4mn
(b) the area of the rectangle 106. If M is the mean of n observations m + n2 + 2 mn
2
(c) the width of the rectangle x 1 − k , x 2 − k , x 3 − k , ..., x n − k ,
⇒ 25mn = 4m2 + 4n2 + 8mn
(d) the perimeter of the rectangle where k is any real number, then
[Q m ≠ 0, n ≠ 0]
Ê (b) Since, we know that for a histogram, what is the mean of
based on a frequency distribution with x 1, x 2 , x 3 , ..., x n ? ⇒ 4m2 + 4n2 − 17 mn = 0
equal intervals, the frequency of a class (a) M (b) M + k ⇒ 4m2 − 16mn − mn + 4n2 = 0
is proportional to height of the rectangle (c) M − k (d) kM
and for a histogram based on frequency ⇒ 4m( m − 4n) − n( m − 4n) = 0
distribution with unequal intervals, the Ê (b) Given that, ⇒ ( m − 4n) ( 4m − n) = 0
frequency of a class is proportional to Mean of x1 − k, x2 − k, x3 − k, … , ⇒ m = 4n or n = 4m
Area of the rectangle. xn − k
Hence, option (d) is correct.
Hence, option (b) is correct. ( x − k ) + ( x2 − k )+… + ( xn − k )
∴M = 1
n 109. If the mean of a frequency
104. The coefficient of correlation is ( x1 + x2 + … + xn ) nk distribution is 100 and the
independent of M = −
n n coefficient of variation is 45%, then
(a) change of scale only x1 + x2 + … + xn
M + k= what is the value of the variance?
(b) change of origin only
n (a) 2025 (b) 450
(c) both change of scale and change of
origin ∴ Mean of x1, x2, x3, … , xn = M + k (c) 45 (d) 4.5
(d) neither change of scale nor change Hence, option (b) is correct. Ê (a) Since, we know that
of origin Coefficient of variation
107. What is the sum of deviations of the
Ê (c) Since, we know that coefficient of σ
variate values 73, 85, 92, 105, 120 (CV ) = × 100 … (i)
correlation is independent of both x
change of scale and change of origin. from their mean?
Where σ is standard deviation and x is
Hence, option (c) is correct. (a) − 2 (b) − 1
mean.
(c) 0 (d) 5
44 NDA/NA Solved Paper 2021 (I)

Given, x = 100 and CV = 45% observations is 65. What is the mean Since, we know that if
σ of the discarded observations? Geometric mean of x1, x2, x3, … xn is G.
From Eqs. (i), 45 = × 100
100 (a) 250 ⇒ Geometric mean of x12, x22, x32, … xn2
⇒ σ = 45 (b) 125 is G 2
∴ Variance = σ 2 = ( 45)2 = 2025 (c) 120
Geometric mean of fx12, fx22, … , fxn2 is fG 2.
(d) Cannot be determined due to
Hence, option (a) is correct. insufficient data ∴Required geometric mean = 3(10)4
110. Let two events A and B be such that Ê (b) Given, mean of 12 observations = 75 = 3 × 10000 = 30000
P ( A ) = L and P ( B ) = M . Which one and M. Hence, option (c) is correct.
12 12
of the following is correct? 5 1
L+ M −1 ∑ xi ∑ xi 115. If P ( A ∪ B ) = , P ( A ∩ B ) = and
(a) P( A| B) < ∴ x=
i =1
⇒ 75 =
i =1
1 6 3
M 12 12 P ( A ) = , then which of the
L+ M −1 12 2
(b) P( A| B) >
M
⇒ ∑ xi = 900 … (i) following is/are correct?
i =1 1. A and B are independent events.
L+ M −1
(c) P( A| B) ≥ Let observations x11 and x12 is discarded 2. A and B are mutually exclusive
M 10
events.
(d) P( A| B) =
L+ M −1 ∑ xi Select the correct answer using the
i =1
M then mean = = 65
10 code given below.
Ê (c) Given, P( A ) = L, P( B) = M 10 (a) 1 only (b) 2 only
Q P( A | B) =
P( A ∩ B) ∴ ∑ xi = 10 × 65 = 650 … (ii)
(c) Both 1 and 2 (d) Neither 1 nor 2
i =1
P( B) 5 1
From Eqs. (i) Ê (a) Given, P( A ∪ B) = , P( A ∩ B) =
P( A ) + P( B) − P ( A ∪ B) 6 3
P( A | B) = 12
P( B) ∑ xi = 900 P( A ) =
1
i =1 2
L + M − P( A ∪ B)
P( A | B) = 10 1 1
⇒ P( A ) = 1 − =
P( B) ⇒ ∑ xi + x11 + x12 = 900
2 2
i =1
∴ P( A ∪ B) = L + M − P(.B) P( A | B) ∴ P( B) = P( A ∪ B) − P( A ) + P( A ∩ B)
⇒ 650 + x11 + x12 = 900 5 1 1 4 2
Q 0 ≤ P( A ∪ B) ≤ 1 = − + = =
⇒ x11 + x12 = 250 6 2 3 6 3
⇒ L + M − P( B) ⋅ P( A | B) ≤ 1 250
∴ Mean of x11 and x12 = = 125 Q If A and B are independents, then
⇒ P( B) ⋅ P( A | B) ≥ L + M − 1 2 P( A ∩ B) = P( A ) ⋅ P( B)
L+ M −1 Hence, option (b) is correct.
P( A | B) ≥ [QP( B) = M ] QP( A ∩ B) =
1
M 3
113. If k is one of the roots of the equation
Hence, option (c) is correct.
x ( x + 1) + 1 = 0, then what is its 1 2 1
and P( A ) ⋅ P( B) = × =
111. For which of the following sets of other root? 2 3 3
numbers do the mean, median and (a) 1 (b) − k ⇒ Statement-1 is correct.
mode have the same value? (c) k 2 (d) − k 2 If A and B are mutually exclusive, then
(a) 12, 12, 12, 12, 24 P( A ∪ B) = P( A ) + P( B)
Ê (c) Given, quadratic equation 5
(b) 6, 18, 18, 18, 30 x( x + 1) + 1 = 0 Q P( A ∪ B) =
(c) 6, 6, 12, 30, 36 6
x2 + x + 1 = 0 … (i) 1 2 7
(d) 6, 6, 6, 12, 30 ∴ P( A ) + P( B) = + =
Since, we know that ω, ω are the roots2
2 3 6
Ê (b) For option (a), Mean of Equation when ⇒ Statement-2 is wrong.
12 + 12 + 12 + 12 + 24
= − 1 + 3i − 1 − 3i Hence, option (a) is correct.
5 ω= and ω 2 =
2 2
= 14 ⋅ 4 ≠ mode (12) 116. The average of a set of 15
⇒ If one of the roots of Eqs. (i) is k, then
observations is recorded, but later it
For option (b), Mean other root will be k 2.
is found that for one observation,
6 + 18 + 18 + 18 + 30 Hence, option (c) is correct.
the digit in the tens place was
= = 18
5 114. The geometric mean of a set of wrongly recorded as 8 instead of 3.
Mode = 18, Median = 18 observations is computed as 10. The After correcting the observation,
Hence, for the data 6, 18, 18, 18, 30, geometric mean obtained when the average is
Mean = Mode = Median = 18 each observation x i is replaced by 1 10
(a) reduced by (b) increased by
Hence, option (b) is correct. 3x i4 is 3 3
10
112. The mean of 12 observations is 75. If (a) 810 (b) 900 (c) reduced by (d) reduced by 50
3
two observations are discarded, then (c) 30000 (d) 81000
Ê (c) Let unit digit for wrongly recorded
the mean of the remaining Ê (c) Given that, geometric mean of a set observation = b
of observations = 10
NDA/NA Solved Paper 2021 (I) 45

When tens digit is 8, then number 5 25 But mode is the number of highest
(a) (b)
= 10 × 8 + b = 80 + b 36 36 frequency.
25 25
When tens digit is 3, then number (c) (d) ∴ x should be 15
54 216
= 10 × 3 + b = 30 + b Hence, option (d) is correct.
⇒ One observation is recorded Ê (d) For a binomial distribution mean
= np and variance = npq 120. If A and B are two events such that
(( 80 + b ) − ( 30 + b )) more while 3 5
calculating average.
Where p is probability of success and P( A ) = and P ( B ) = , then
q is the probability of unsuccess and n is 4 8
Hence, after correcting the observation,
number of observations. consider the following statements
the average will be reduced by
{( 80 + b ) − ( 30 + b )}
2
∴Given, np = , npq =
5 1. The minimum value of P ( A ∪ B )
= 3 9 3
15 is .
2 5
50 10 ⇒ ⋅q = 4
= = 3 9
15 3 2. The maximum value of P ( A ∩ B )
5
Hence, option (c) is correct. q = 5
6 is .
5
8
117. A coin is tossed twice. If E and F ∴ p= 1− q = 1−
6 Which of the above statements
denote occurrence of head on first
1 is/are correct?
toss and second toss respectively, = (a) 1 only
then what is P ( E ∪ F ) equal to? 6
(b) 2 only
2
(a)
1
(b)
1 Q np = (c) Both 1 and 2
4 2 3
(d) Neither 1 nor 2
3 1  1 2
(c) (d) ⇒ n  =
 6 3 3
4 3
Ê (c) Given, P( A ) =
4
Ê (c) Given that, a coin is tossed twice. ⇒ n=4
5
∴ S = { HH, HT, TH, TT} Q p( X = x ) = nC x pxq n − x P( B) =
8
Given, E be the event of occurrence of ∴P( X = 2 ) = 4C 2 p2q 4 − 2 QP( A ∪ B) = P( A ) + P( B) − P( A ∩ B)
head on first toss and F be the event of
occurrence of head on second toss.  1  5
2
25
2
and P( A ∩ B) = P( A ) + P( B) − P( A ∪ B)
=6×  ×  =
∴ E = { HH, HT}  6  6 216 Here P( A ∪ B) will be minimum if
n( E ) 2 1 Hence, option (d) is correct. P( A ∩ B) is maximum and vice-versa.
∴ P( E ) = = =
n(S ) 4 2 Since, minimum value of P( A ∩ B) is zero
119. If the mode of the scores 10, 12, 13, and maximum value of P( A ∩ B) is
2 1
F = {TH, HH}, P( F ) = = 15, 15, 13, 12, 10, x is 15, then what minimum ( P( A ), P( B)).
4 2
is the value of x? ⇒ maximumP( A ∩ B)
Q E ∩ F = { HH}
(a) 10 (b) 12  3 5
∴ P( E ∩ F ) =
1
(c) 13 (d) 15 = minimum  , 
4  4 8
∴ P( E ∪ F ) = P( E ) + P( F ) − P( E ∩ F )
Ê (d) Given, observations are 5
=
10, 12, 13, 15, 15, 13, 12, 10, x 8
1 1 1
= + − Q Mode = 15
2 2 4 ⇒ Statement-2 is correct.
1 3 Also, minimum value of P( A ∪ B) is
= 1− = Scores Frequency
4 4 maximum ( P( A ), P( B))
10 2
Hence, option (c) is correct. ∴Minimum value of
12 2  3 5
118. In a binomial distribution, the mean P( A ∪ B) = maximum  , 
13 2  4 8
2 5 3
is and variance is . What is the 15 2 =
3 9 4
probability that random variable Since, frequency of all other numbers is ⇒ Statement-1 is correct.
X = 2? same as frequency of 15. Hence, correct option is (c).
46 NDA/NA Solved Paper 2021 (I)

PAPER II English Language and General Studies


Part A (English Language)
Directions (Q. Nos. 1-10) Each organisation or cause. Hence, ‘backer’ 12. The sky is boundless.
question in this section consists of a is its correct synonym. (a) high (b) vast
sentence with an underlined word (c) expansive (d) finite
7. Arrogance is a hallmark of his
followed by four words. Select the Ê (d) ‘Boundless’ means unlimited or
nature.
option that is nearest in meaning to the immense. Hence, ‘finite’ meaning
(a) concern
underlined word and mark your (b) unpretentiousness
limited in extent or size is its correct
answer. antonym.
(c) conceit
1. Her smile was contagious. (d) simplicity 13. The sky is clear today.
(a) arrogant (b) disrespectful Ê (c) ‘Arrogance’ describes to someone (a) bright (b) opaque
(c) sarcastic (d) catching who has an exaggerated sense of their (c) cloudless (d) blue
own importance. Hence, ‘conceit’
Ê (d) ‘Contagious’ means infectious. Ê (b) The antonym of clear is ‘opaque’,
meaning excessive pride in oneself is its which means unclear or hard to be
Hence, ‘catching’ is its correct
correct synonym. seen through.
synonym.

2. Her dynamic nature impressed 8. She undertook a reconnaissance of 14. I have a fascination for deep waters.
everyone. the entire issue. (a) dark (b) light
(a) revaluation (b) ratification (c) dangerous (d) shallow
(a) enduring (b) attentive
(c) investigation (d) regularisation
(c) evolutionary (d) jealous Ê (d) The antonym of deep is ‘shallow’,
Ê (c) ‘Dynamic’ refers to process or Ê (c) ‘Reconnaissance’ refers to military which means of little or no depth.
observation of a region to locate an
system characterised by constant 15. My boss has been too generous.
enemy or ascertain strategic features.
change, activity or progress. Hence, (a) stingy (b) rough
Hence, ‘investigation’ is its correct
‘evolutionary’ is nearest in meaning to (c) evil (d) hostile
synonym.
dynamic.
9. My daughter is my replica. Ê (a) The antonym of generous is ‘stingy’,
3. She was lamenting her destiny. which means miserly.
(a) pride (b) clone
(a) celebrating (b) bemoaning
(c) blaming (d) making
(c) love (d) original 16. Spring is a time of plenty.
(a) ugliness (b) scarcity
Ê (b) ‘Lamenting’ means to express regret Ê (b) ‘Replica’ means an exact copy or
model of something. Hence, ‘clone’ is (c) roughness (d) dryness
or disappointment about something.
Hence, ‘bemoaning’ is its correct
its correct synonym. Ê (b) The antonym of plenty is ‘scarcity’,
synonym. which means shortage or lack of
10. The sardonic nature of her stories something.
4. Under his leadership the company made her stand out among the
grew in an organic manner. contemporary writers. 17. He is an industrious workman.
(a) compassionate (b) insightful (a) active (b) productive
(a) natural (b) speedy
(c) mocking (d) comic (c) lazy (d) disloyal
(c) unusual (d) disciplined
Ê (c) ‘Sardonic’ means grimly mocking or Ê (c) ‘Industrious’ means hard-working
Ê (a) ‘Organic’ means characterised by and diligent. Hence, ‘lazy’ is its correct
gradual or natural development. Hence, cynical.
antonym.
‘natural’ is its correct synonym.
Directions (Q. Nos. 11-20) Each 18. Plants grow in abundance here.
5. His charm lies in his oratory. question in this section consists of a (a) shrivel (b) stretch
(a) deceit sentence with an underlined word (c) spread (d) enlarge
(b) eloquence followed by four words. Select the
(c) looks
option that is opposite in meaning to
Ê (a) The antonym of grow is ‘shrivel’,
(d) nobility which means to wrinkle or shrink.
the underlined word and mark your
Ê (b) ‘Oratory’ means the art or practice of answer. 19. She rarely comes here.
formal speaking in public. Hence, (a) seldom (b) never
‘eloquence’ is its correct synonym. 11. He found her extremely attractive (c) always (d) frequently
and charming.
6. She is a patron of art and culture. Ê (d) ‘Rarely’ means not often or seldom.
(a) unnatural Hence, ‘frequently’ meaning regularly or
(a) entrepreneur
(b) modern habitually is its correct antonym.
(b) admirer
(c) repulsive
(c) critique
(d) backer
(d) disapproving 20. She is a rather crooked woman.
(a) polite
Ê (d) ‘Patron’ refers to a person who gives Ê (c) ‘Attractive’ means pleasing or
agreeable. Hence, ‘repulsive’ meaning (b) generous
financial or other support to a person, (c) straightforward
arousing intense distaste is its correct
antonym. (d) happy
NDA/NA Solved Paper 2021 (I) 47

Ê (c) ‘Crooked’ means dishonest or 26. Turn topsy-turvy other people don’t have problems too.
insincere. Hence, ‘straightforward’ (a) To completely change something (c)
meaning honest and frank is its correct (b) To completely evaluate something No error
antonym. (c) To enjoy yourself greatly (d)
(d) To exhaust yourself completely
Directions (Q. Nos. 21-30) Given Ê (c) The use of ‘too’ in the sentence
Ê (a) The phrase ‘turn topsy-turvy’ means shows that a further point with a similar
below are some idioms/phrases to be in utter confusion or disorder.
followed by four alternative meanings meaning is being added. Hence,
Hence, ‘to completely change
remove ‘don’t’ to make the sentence
to each. Choose the response (a), (b), (c) something’ best expresses the meaning
grammatically correct.
or (d), whichever is the most of the phrase.
appropriate meaning and mark your 32. If you say that someone
answer.
27. A clarion call
(a)
(a) A trumpet call
21. Overstep the mark (b) An intimidating voice you admire has feet of clay,
(a) To tell people how successful you are (c) A strong request (b)
(b) To step into someone else’s areas of (d) An urgent order you mean that they have hidden faults.
expertise Ê (c) The idiom ‘a clarion call’ means a (c)
(c) To upset someone by doing/saying strong request for something to No error
more than you should happen.
(d) To do something in an excited way (d)
Ê (c) The idiom ‘overstep the mark’ means
28. Fire in the belly Ê (d) The given sentence is grammatically
to upset someone by going beyond (a) Fear and hatred correct.
what was allowed. Hence, option (c) is (b) Powerful ambition
(c) Love and dedication 33. He refused to change
the correct answer.
(d) Lethargy and indifference (a)
22. Palsy-walsy friends Ê (b) The idiom ‘fire in the belly’ means a
his decision;
(a) Good friends strong determination to succeed. (b)
(b) Friends who help each other in Hence, option (b) is the correct answer. he refused it point out.
difficult situations
(c) Friends by choice and not by chance 29. A hunky-dory situation (c)
(d) Unfriendly (a) There are serious issues among No error
people (d)
Ê (a) The phrase ‘palsy-walsy friends’
(b) There are no problems and people
means good friends. Ê (c) ‘Point out’ is used incorrectly in the
are happy sentence. It should be replaced by
23. Open a Pandora’s box (c) There is war and bloodshed all over ‘point blank’, which means in a blunt
(a) To do something that causes a lot of (d) There is no work, only enjoyment and direct manner.
new problems that you did not expect
Ê (b) The idiom ‘a hunky-dory situation’
(b) To do something out of compulsion means a very satisfactory and problem 34. The importance of trade in Mughal
(c) To do something beyond expectation free situation. Hence, option (b) is the times reinforced
(d) To do something out of the box, that correct answer. (a)
brings award and ceremonies for you
30. Give somebody a leg up the cultural definition of wealth as
Ê (a) The idiom ‘open a Pandora box’
means to begin or introduce something (a) To pull someone down something
that leads to many other problems. (b) To deceive and betray someone (b)
Hence, option (a) is the correct answer. (c) To help someone for their livelihood
(d) To help someone to be successful comprising of movable property.
24. Pull your socks up (c)
(a) To get well-dressed for the occasion
Ê (d) The idiom ‘give somebody a leg up’
means to help someone/achieve No error
(b) Improve your work or behaviour something and become successful.
(c) To speak in an honest way without (d)
hesitation Ê (c) The use of ‘of’ after ‘comprising’ is
(d) To be in control of an organisation,
Directions (Q. Nos. 31-40) Each
invalid, as ‘comprise’ means consist of.
often secretly question in this section has a
Hence, remove ‘of’ to make the
sentence with three underlined parts
Ê (b) The idiom ‘pull your socks up’ labelled (a), (b) and (c). Read each
sentence grammatically correct.
means to improve your behaviour or
work to achieve something. sentence to find out whether there is 35. In the nineteenth century,
any error in any underlined part. If (a)
25. To get under somebody’s skin you find no error, your response most traditional scholars
(a) To deceive someone should be indicated as (d).
(b) To admire someone (b)
(c) To annoy someone 31. You don’t have tried to stay clear from the imperial
(d) To support someone (a) Government.
Ê (c) The idiom ‘to get under somebody’s a monopoly on suffering;
(c)
skin’ means to irritate or upset someone. (b) No error
(d)
48 NDA/NA Solved Paper 2021 (I)

Ê (c) The preposition ‘from’ is used Ê (a) Replace ‘when’ with ‘while’ to make 45. The first film on Tagore was such a
incorrectly. It should be replaced by ‘of’ the sentence error free and success that now they are going to
to make the sentence correct. To ‘stay grammatically correct.
make a ……… .
clear of’ means to avoid someone or
something. 40. I have the opportunity to study (a) serial
(a) (b) (b) sequence
36. He began his discussion by pointing in American next year. No error (c) sequel
over (d) sequential
(c) (d)
(a) Ê (c) Sequel
that men and women Ê (d) The given sentence is grammatically
correct. 46. The United Nations had ……… 2020
(b) as the International Year of Plant
had different biological functions. Directions (Q. Nos. 41-50) Each of the Health.
(c) following sentences in this section has a (a) ruled
No error blank space and four words or group of (b) ordered
(d) words given after the sentence. Select (c) foretold
the word or group of words you consider (d) declared
Ê (a) ‘Pointing over’ is used incorrectly in
the sentence. It should be replaced by most appropriate for the blank space Ê (d) Declared
‘pointing out’, which means to say and mark your answer.
something to make someone aware of
47. My brother is ……… punctual, but
a situation.
41. There was a time when West he is late today.
Germany was a distinct ……… . (a) normatively (b) primarily
37. Though he is poor, but he is (a) policy (b) polity (c) normally (d) basically
(a) (b) (c) abstract (d) hierarchy
Ê (c) Normally
honest. No error Ê (b) ‘Polity’ means an organised society;
(c) (d) a state as a political entity. Hence, it is 48. My son is very ………; he trusts
appropriate to fill the blank. everyone.
Ê (b) ‘But’ should not be used with (a) fallible (b) gullible
‘though’. Hence, remove ‘but’ to make 42. I was ……… with the film; I had (c) sensible (d) credible
the sentence grammatically correct. expected it to be better.
Ê (b) ‘Gullible’ means easily persuaded to
38. My mother has been doing (a) disappointed (b) disappointing believe something. Hence, it is
(c) annoying (d) prejudiced appropriate to fill the blank.
(a) (b)
everything for the family. Ê (a) Disappointed 49. Mahatma Gandhi was a lover of
(c) 43. It was a ……… experience. humanity and a ……… believer in
No error Everybody was shocked. the goodness of human nature.
(d) (a) terrified (b) horrified (a) staunch (b) powerful
Ê (b) Replace ‘has been doing’ with ‘does’ (c) terrifying (d) denouncing (c) cheerful (d) hopeful
to make the sentence error free and Ê (c) Terrifying
grammatically correct.
Ê (a) Staunch
44. Elephants ……… when they perceive 50. I wish I ……… her before we met.
39. When learning to dive it is important danger. (a) know
(a) (b) (a) trumpet (b) frolic (b) have known
to relax in between and take breaks. (c) whine (d) sing (c) knew
(c) (d) known
Ê (a) The sound that elephants make is
No error called ‘trumpet’. Hence, it is appropriate Ê (c) Knew
(d) to fill the blank.

Part B (General Studies)


51. Consider the following statements (a) 1, 2 and 3 (b) 2 and 3 Ê (a) Density of water at 4°C is 1000kg/m 3
(c) 1 and 2 (d) Only 1 3 3
about light year. or 10 kg/m . It is the maximum density
1. Light year is a unit for Ê (d) Light year is a unit for measurement of of water. The ratio of mass and volume of
measurement of very large very large distances. It is used to express substance is known as density.
distances. the distances of celestial bodies.
53. Which of the following pairs of
2. Light year is a unit for 52. Which one of the following physical phenomena and the
measurement of very large time regarding density of water at discoverer is/are correctly matched?
intervals. atmospheric pressure is correct? 1. James Chadwick : Photoelectric
3. Light year is a unit for (a) Density of water at 4°C is 1000 kg/m3 . effect
measurement of intensity of light. (b) Density of water at 0°C is 1000 kg/m3 . 2. Albert Einstein : Neutron
Which of the statements given (c) Density of water at 0°C is 100 kg/m3 .
3. Marie Curie : Radium
above is/are correct? (d) Density of water at 4°C is 10 kg/m3.
NDA/NA Solved Paper 2021 (I) 49

Select the correct answer using the 57. The composition of gases in 62. Which of the following sets of
codes given below. exosphere is elements has the same valency?
(a) 1, 2 and 3
(a) helium and hydrogen (a) Na, Mg, Ca (b) Na, Mg, Al
(b) 1 and 2
(b) neon and oxygen (c) Mg, Ca, K (d) Mg, Ca, Ba
(c) 2 and 3
(c) neon and hydrogen
(d) Only 3
(d) helium and neon Ê (d) Mg, Ca, Ba sets of elements has the
same valency. The valency of an
Ê (d) Photoelectric effect by Albert Ê (a) The exosphere is mainly element is determined by the number of
Einstein According to Einstein, light is
composed of extremely low densities of valence electron in the outermost shell
made up of little packets, at first called
hydrogen and helium; and the heavy of an atom of an element.
quanta and later photons. When a
molecules nitrogen, oxygen and carbon
photon hits the electrons, it gives
dioxide. 63. Which one of the following is the
electrons enough energy to escape lowest possible temperature?
from the surface of the metal. This 58. Which one of the following is not (a) 0° celsius
explains the behaviour of light striking used as a raw material in the (b) − 73° celsius
the metal. manufacture of glass? (c) − 173° celsius
• Neutron by James Chadwick The (a) Soda (d) − 273° celsius
neutron is a subatomic particle, (b) Alumina
symbol n, which has a neutral charge, Ê (d) The lowest possible temperature is
(c) Borax the absolute zero. At this temperature,
and a mass slightly greater than that (d) Gypsum
of a proton. no more heat can be remove from a
• Radium by Marie Curie Radium is a Ê (d) The correct answer is gypsum. Glass body. Absolute zero is precisely
is made from natural and abundant raw equivalent to −27315
. degrees celsius
chemical element with the symbol Ra
and atomic number 88, also known materials (soda, alumina and borax, etc.) on celsius scale.
as the alkaline earth metals. Pure that are melted at very high temperature
to form a new material glass.
64. Numerically two thermometers, one
radium is silvery-white, but it readily
reacts with nitrogen on exposure to in Fahrenheit scale and another in
air, forming a black surface layer of 59. In electrolytic refining of copper, Celsius scale shall read same at
radium nitride. the electrolyte is a solution of (a) − 40° (b) 0°
(a) acidified copper chloride (c) − 273° (d) 100°
54. LED (a semiconductor device) is an (b) acidified copper sulphate
abbreviation that stands for (c) potassium chloride Ê (a) The relation between the Fahrenheit
and Celsius scale is given by the
(a) Licence for Energy Detector (d) sodium sulphate
equation. Therefore, Celsius and
(b) Light Energy Device
Ê (b) In the electrolytic refining of copper Fahrenheit scale coincide at
(c) Light Emitting Diode (electrolyte is CuSO 4 solution + − 40°.
(d) Lost Energy Detector dil. H 2SO 4 ) or acidified copper ( ° F − 32 ) ° C
=
Ê (c) LED is an abbreviation that stands sulphate. Pure Cu cathode and impure 180 100
for Light Emitting Diode. It is a Cu anode. Metallic impurities having
Now, let us assume that Fahrenheit and
semiconductor light source that emits lower oxidation potential than Cu are
Celsius scale coincide at x°, therefore
light when current flows through it. deposited as sludge.
( ° x − 32 ) °x
⇒ =
55. The statement “friction force is a 60. Solder is an alloy of 180 100
contact force while magnetic force (a) Cu and Sn (b) Fe and Zn On solving, we get
is a non-contact force” is (c) Pb and Sn (d) Ag and Zn 5
⇒ ( ° x − 32 ) =° x
(a) always true Ê (c) Solder is a mixture of lead (Pb) and 9
(b) true only at 0°C tin (Sn). Tin-lead solder is the most Therefore, x = −40°
(c) a false statement popular in solder paste printing and it is
(d) Either true or false depending upon mainly composed of tin and lead, with 65. The image we see in plane mirror is
the temperature of the surroundings same trace metal element like (a) real and thus can be photographed
Ê (a) Friction force is a contact force antimony (Sb). (b) virtual and nearer than the object
because it works between the contact (c) virtual and is laterally inverted
surface of two bodies and magnetic
61. Which one of the following (d) The real but cannot be photographed
statements about dihydrogen (H 2 ) is
force is a non-contact force. Ê (c) The image in a plane mirror is virtual,
not correct? erect same size and laterally inverted.
56. Which one of the following is the (a) H 2 is lighter than air and insoluble in For a plane mirror, the size of the image
chemical formula of hypobromous water. is equal to the size of the object.
acid? (b) H 2 is inert at room temperature due to
(a) HBrO 4 highH H bond dissociation enthalpy. 66. Which one of the following colours
(b) HOBr (c) H 2 reacts with alkali metals at high may be obtained by combining
temperature to yield metal hydrides.
(c) HBr green and red colours ?
(d) HBrO 3 (d) A mixture of NO 2 and H 2 is known as
(a) Blue (b) Magenta
Syngas.
Ê (b) Hypobromous acid is a weak, (c) Pink (d) Yellow
unstable acid with chemical formula Ê (d) Statement (d) is incorrect because
syngas or synthesis gas is a fuel gas Ê (d) Yellow colour is obtained by
HOBr, where the bromine atom is in the combining green and red colours. It is a
+1 oxidation state. It is also called mixture consisting primary of hydrogen,
secondary colour.
‘bromanol’ or ‘hydroxidobromine’. carbon monoxides.
50 NDA/NA Solved Paper 2021 (I)

67. Which of the following are the F1-particles which are the site of (a) Experimental leaf 1
Electron Transportation System (ETS) of (b) Experimental leaf 2
primary colours of light? (c) Control leaf
aerobic cellular espiration.
(a) Yellow, red and green (d) All the leaves will dry up
(b) Blue, red and green 71. Squamous epithelial cells are found simultaneously
(c) Violet, red and yellow in the inner lining of
(d) Indigo, violet and green Ê (b) Experimental leaf 2 will dry up last,
(a) oesophagus because in dicot plants, stomata are
Ê (b) Primary colours of light are blue, red (b) small intestine generally found in lower surface of
and green. This is the essential method (c) ducts of salivary gland leaves and applying vaseline/vegetable
used to create the precepting of a (d) kidney oil will close these stomata, thus
board range of colours.
Ê (a) Squamous epithelial cells are found preventing the water loss through
68. According to the new cartesian sign in the inner lining of the oesophagus. transpiration (evaporation of water in
Simple squamous epithelium is a single the form of vapour form leaf surface)
convention, which one of the
layer of flat scale-shaped cells, which and drying process will take a lot more
following is correct in respect of the are helpful in transportation of food from time compare to other leaves.
1 1 1
formula = + , where symbols buccal cavity to stomach.
75. Which one of the following
f v u
72. Transformation of meristematic statements is not correct for light
have their usual meanings? cells into specific permanent tissues rays ?
(a) It applies only to spherical mirrors.
occurs by the process of (a) Light travels at different speeds in
(b) It applies only to spherical lenses.
(a) cell differentiation different media.
(c) It applies to spherical mirrors as well
(b) cell division (b) Light travel at almost 300 million
as spherical lenses.
(c) cell multiplication metres per second in air.
(d) It is an invalid formula.
(d) cell regeneration (c) Light speeds down as it leaves a
Ê (a) For a spherical mirror, the formula water surface and enters the air.
showing the relationship between the Ê (a) Transformation of meristematic cells (d) Light speeds up as it leaves a glass
into specific permanent tissue occurs
distance of the object(u), the distance surface and enters the air.
by the process of cell differentiation. In
of the mirror (v) and the distance of the speed of light in air is 3 × 10 8 m/s
this process, cells formed by Ê The
focus ( f ) is called the mirror formula, and the speed of light is different in
meristematic tissues take up a specific
which is as follows different media.
role and lose the ability to divide. As a
1 1 1
= + result, they form a permanent tissue 76. A glass prism splits white light into
f v u taking up a permanent shape, size and
different colours. This phenomenon
69. Movement of materials to different function.
is called dispersion of light by
parts of cytoplasm and nucleus is 73. The gaseous product of a process in prism. Which one of the following
generally carried out by plants is a requirement for another statements is correct?
(a) Ribosomes vital process that releases energy. (a) Red light will deviate the most and it
(b) Mitochondria is because of the reflection of light.
(c) Lysosomes
Given below are four combinations
of the process and product. Identify (b) Violet light will deviate the most and it
(d) Endoplasmic reticulum is because of the refraction of light.
the correct answer.
Ê (d) The movement of materials to (a) Respiration and nitric oxide
(c) Red light will deviate the most and it
different parts of the cytoplasm and is because of the refraction of light.
nucleus is generally carried out (b) Transpiration and water vapour (d) Violet light will deviate the most and it
(c) Photosynthesis and oxygen is because of the reflection of light.
Endoplasmic Reticulum (ER). although
(d) Germination and carbon dioxide
Golgi bodies have main role in the Ê VIBGYOR (increasing order of
function of intracellular transporation. Ê (c) In plants, oxygen gas is produced as wavelength). (Frequency decreases in
ER is the largest membrane-bound by-product of photosynthesis which is increasing order).
organelle in eukaryotic cells and required by aerobic cellular respiration
Refractive index is inversely
performs a variety of essential cellular to produce energy (ATP).
proportional to wavelength. So, violet
functions, including protein and lipid Photosynthesis occurs in chloroplast
light will deviate the most.
synthesis and biosynthesis of cell and while aerobic cellular respiration occurs
nuclear membrane. in mitochondria. 77. A current of 1.0 A is drawn by a
70. In mitochondria, ATP synthesising 74. In a dicot pot herb, filament of an electric bulb for 10
chemical reactions take place in the vaseline/vegetable oil was applied minutes. The amount of electric
(a) outer membrane on the upper surface of one leaf charge that flows through the
(b) matrix (experimental leaf 1) and on the circuit is
(c) inner membrane (a) 0.1 C (b) 10 C
lower surface of another leaf
(d) DNA of mitochondria (c) 600 C (d) 800 C
(experimental leaf 2).
Ê (c) In eukaryotes, Adenosine Vaseline/Vegetable oil was not Ê (c) Given, I = 1 A,
Triphosphate (ATP) synthesis reaction t = 10 min = 10 × 60 s = 600 s
occurs on the inner membrane of
applied on the control leaf. The Q
plant was deliberately not watered ∴ I=
mitochondria which sides toward matrix t
compartment. It contains the ATP for several days. Which leaf will dry ⇒ Q = It ⇒ Q = 1 × 600
synthesis complex or oxisome or up last? ⇒ Q = 600 C
NDA/NA Solved Paper 2021 (I) 51

78. Which one of the following process that uses hydrogen gas to (a) displacement reaction
change a liquid vegetable oil into a hard (b) endothermic reaction
formulae does not represent (c) decomposition reaction
spreat. This process stabilises the oil
electrical power? and prevents spoilage from oxidation. (d) exothermic reaction
(a) I 2R (b) IR 2
(c) VI (d) V 2 / R 83. Which one of the following Ê (d) The formation of slaked lime
(calcium hydroxide) Ca(OH) 2 when
Ê (b) Electric power is the rate per unit materials is not an allotrope of water is added to lime (CaO) is
time at which electrical energy is carbon? exothermic.
transferred by an electric circuit, (a) Diamond (b) Graphene CaO( s ) + H 2O ( l ) → Ca(OH) 2 (s)
V2 (c) Fly ash (d) Fullerene [Evolution of heat]
P = I 2R = = VI
R Ê (c) Diamond, graphene and fullerene 88. Which one of the following is not a
Its SI unit is watt. are three allotropes of pure carbon. Fly
ash is not a allotrope of carbon. bio-mass energy source?
79. The sound created in a big hall Hence, option (c) is correct.
(a) Wood
persists because of the repeated (b) Nuclear reactor
reflections. The phenomenon is 84. Which one of the following (c) Gobar gas
reactions does not result in the (d) Coal
called
(a) reverberation (b) dispersion evolution of hydrogen gas? Ê (b) Nuclear reactors are sources of
(a) Reaction of zinc metal with dilute nuclear energy, not bio-mass energy.
(c) refraction (d) diffraction
sulphuric acid solution Wood, gobar gas and coal are the
Ê (a) The sound created in a big hall (b) Mixing water to Plaster of Paris organic or bio-mass sources of energy.
persists because of the repeated (c) Heating zinc metal with sodium
reflection. The phenomenon is called hydroxide solution
89. Twinkling of stars is due to
reverberation. (d) Reaction of potassium metal with (a) particular frequencies of the starlight.
water (b) reflection of starlight from the oceanic
80. When light is scattered by a surface.
molecule and the frequency of the Ê (b) When plaster of Paris is mixed with (c) atmospheric refraction of starlight.
water, it gets converted into gypsum. (d) magnetic field of earth.
scattered light is changed, this
1
phenomenon is called i.e. CaSO 4⋅ H 2O + H 2O Ê (c) The twinkling of a star is due to
2 atmospheric refraction of starlight. The
(a) Rayleigh scattering → CaSO 4 ⋅ 2H 2O,
(b) Raman effect starlight on entering the earth’s
(c) Photoelectric effect Hence, it doesn’t give hydrogen gas. atmosphere, undergoes refraction
(d) Rutherford scattering 85. Which one of the following acids is continuously before it reaches the earth.
The atmospheric refraction occurs in
Ê (b) When light is scattered by a predominantly found in tomatoes?
molecule and the frequency of the the medium of gradually changing
(a) Acetic acid (b) Tartaric acid
scattered light is changed, this refractive index.
(c) Oxalic acid (d) Lactic acid
phenomenon is called Raman effect. 90. Which one of the following cannot
Ê (c) Oxalic acid a chemical compound
81. Which one of the following that occur naturally in almost every be the unit of frequency of a sound
statements about the cleansing plant to some degree including fruits, wave?
action of soap is not true?
vegetables and grain plants. The (a) dB (b) s − 1 (c) Hz (d) min− 1
amount of oxalic in tomatoes is about
(a) The oil and dirt gets collected in the
50 mg per 100 g. Ê (a) We know that,
centre of the micelle. 1
Frequency =
(b) Soap micelle scatter light. 86. Which one of the following Time period
(c) Soaps are ammonium salts of long conclusions could not be derived 1
chain carboxylic acids. =
(d) Soap forms insoluble precipitates from Rutherford’s α-particle Second or minute
with the calcium and magnesium ions scattering experiment? Apart from this, the SI unit of frequency
in hard water. (a) Most of the space in the atom is is hertz (Hz).
empty.
Ê (c) Detergents are ammonium or (b) The radius of the atom is about 10 5 91. ‘Beats’ is a phenomenon that occurs
sulphonate salts of long chain
times the radius of the nucleus. when frequencies of two harmonic
carboxylic acids. Hence, option (c) is
(c) Electrons move in a circular path of waves are
not true. Soap is a mixture of sodium
fixed energy called orbits.
salts of various naturally occurring fathy (a) equal
(d) Nearly all the mass of the atom
acids. (b) far apart
resides in the nucleus.
(c) multiples of each other
82. Hydrogenation of vegetable oils Ê (c) The concept of electrons move in a (d) nearly same
using nickel catalyst is an example of circular path of fixed energy called orbits
was put forward by Bohr and not derived Ê (d) When two sound waves of
(a) substitution reaction approximately equal frequency are
(b) elimination reaction from Rutherford’s scattering experiment.
produced simultaneously, the intensity
(c) addition reaction Therefore, option (c) is incorrect.
of the resultant sound wave increases
(d) free-radical polymerisation
87. Reaction of quick lime (CaO) with or decreases with time.
Ê (c) Hydrogenation of vegetable oils water to produce slaked lime This change in the intensity of sound is
using nickel catalyst is an example of called beats.
addition reaction. Hydrogenation is a
(Ca(OH)2 ) is an example of
52 NDA/NA Solved Paper 2021 (I)

92. Light waves are incident on an produce pollen tube. This pollen tube Ê (c) Preparation of plaster of Paris
elongated in style and reaches ovary to doesn’t require the use of NaCl. It is
air-glass boundary. Some of the
release male gametes in embryo sac of simply obtained by heating gypsum
light waves are reflected and some ovule. (CaSO 4 ⋅ 2H 2O) at 120°C in a rotary
are refracted in the glass. Which kiln.
one of the following properties is 96. If human blood is placed in a 2%
the same for the incident wave and detergent solution, what will 101. Which one of the following
the refracted wave? happen to the RBC? Harappan sites was a specialied
(a) Speed (a) The RBC will shrink. centre for making shell object ?
(b) Direction (b) The RBC will swell and become turgid. (a) Lothal (b) Balakot
(c) Brightness (c) The RBC will swell and burst. (c) Amri (d) Kot Diji
(d) Frequency (d) The RBC will lyse.
Ê (b) Among the given options, Harappan
Ê (d) Frequency of refracted and reflected Ê (a) If human blood is placed in a 2% sites specialised centre for making shell
wave never changes because the detergent solution, the RBC will shrink. object is Balakot and the other site is
frequency does not depends on the Detergent solution is a hypertonic Nageshwar. Shell objects like bangles,
medium. solution compared to red blood cells, ladles and inlay were made at these
hence RBCs are, if placed in it, the cell specialised centres.
93. Which one of the following will shrink due to exosmosis, i.e.
statements is true for a simple movement of water from its higher 102. Which one of the following was not a
harmonic oscillator? concentration (RBC) to lower part of the dhamma of King Ashoka ?
(a) Force acting is directly proportional concentration (Detergent solution) (a) Honouring the king
to the displacement from the mean through cell membrane (SPM) of RBCs. (b) Tolerance of religions other than
position and is in same direction. one’s own
97. The major source of vitamins and (c) Respecting Brahmanas
(b) Force acting is directly proportional
to the displacement from the mean minerals for vegetarians is (d) Promoting the welfare of his subjects
(a) black gram and wheat
position and is in opposite direction.
(b) rice and mustard Ê (a) Honouring the king was not a part of
(c) Acceleration of the oscillator is the Dhamma of king Ashoka. Dhamma
constant. (c) vegetables and fruits
of king Ashoka established the idea of
(d) The velocity of the oscillator is not (d) soya bean and milk
paternal kingship. He regarded all his
periodic. Ê (c) Vegetables and fruits are major subjects as his children and believed it
Ê F = − k x (SHM equation) source of vitamins and minerals for the king’s duty to look after the welfare
vegetarians. of the subjects. Ashoka’s philosophy
where, x = displacement from mean
state that everybody should serve
position and F = acting force. 98. If a ray of light enters from a rarer
parents, revere teachers, and practice
94. During seed germination, the part of medium to a denser medium at zero ahimsa and truthfulness. He asked
the embryo which grows into root is angle of incidence, it would everyone to avoid animal slaughter and
(a) reflect back sacrifice. He advocated tolerance
(a) radicle (b) plumule
(b) go straight towards all religions and sought
(c) cotyledon (d) epicotyl
(c) turn towards right conquest through Dhamma and not
Ê (a) During seed germination, the part of (d) bend at 45º war.
the embryo which grows into root is
called radicle. It is the first part of a Ê (b) When light falls normally on the 103. Which of the following statements
interface of two medium, then light
seedling (a growing plant embryo) to about Saguna Bhakti traditions
travels in straight line. So, the refraction
emerge from the seed during the is/are correct?
angle is zero.
process of germination to form
embryonic root of the plant which grows 99. Mirage is an illustration of 1. Saguna Bhakti traditions focus
downward in the soil. (a) only dispersion of light
on the worship of specific deities
(b) only reflection of light such as Vishnu or his avatars.
95. In a typical flower, germinating (c) only total internal reflection of light 2. In Saguna Bhakti traditions,
pollen grains pass through several (d) both refraction and total internal Gods and Goddesses are
parts of the gynoecium before they reflection of light conceptualised in
reach the ovule. A list of the parts Ê (d) A mirage is a naturally occurring anthropomorphic forms.
of gynoecium is given below in optical phenomenon in which light rays Select the correct answer using the
different combinations. Choose the bend via refraction to produce a codes given below
combination that represents the displaced image of distant objects or (a) Only 1
the sky. (b) Only 2
correct sequence of pollen tube
(c) Both 1 and 2
pathway/journey. 100. Common salt (NaCI) is not used as (d) Neither 1 nor 2
(a) Style, Stigma, Ovary a raw material for perparation of
(b) Stigma, Style, Ovary which one of the following Ê (c) Both the statements about Saguna
(c) Pistil, Stigma, Ovary Bhakti are correct. Saguna Bhakti has
compounds ? form, attributes and quality. Saguna
(d) Ovary, Pistil, Style
(a) Bleaching powder saints worshipped the anthropomorphic
Ê (b) Pollen grains gather on the surface (b) Baking soda manifestations of the divine being,
of stigma of gynoecium through (c) Plaster of Paris particularly Vishnu or his avatars. The
pollination and get germinated to (d) Washing soda
NDA/NA Solved Paper 2021 (I) 53

Bhakti saints emphasised on two ways 106. Which one of the following is the plates are often not shown on major
of imaging the nature of the God viz. plate maps. For purposes of this list, a
correct meaning of ziyarat ?
Nirguna and Saguna. Nirguna is the minor plate is any plate with an area
(a) Pilgrimage to the tombs of sufi saints.
concept of a formless God, which has less than 20 million km 2 but greater
(b) The practice of revenue farming
no attributes or quality. Saguna has than 1 million km 2. They are Somali
(c) The death anniversary of a sufi Shaikh
form, attributes and quality. (d) A form of Islamic divorce Plate, Nazca Plate , Indian Plate ,
Amurian Plate , Sunda Plate, Philippine
104. At which one of the following Ê (a) In Islam, ziyara or ziyarat is a form of Sea Plate, Okhotsk Plate, Arabian
places was a Shiva temple not pilgrimage to sites associated with Plate, Yangtze Plate, Caribbean Plate ,
constructed under the patronage of Muhammad, his family members and Cocos Plate , Caroline Plate ,Scotia
the Chola rulers? descendants, his companions and other Plate, Burma Plate, New Hebrides
venerated figures in Islam such as the Plate.
(a) Chidambaram
prophets, Sufi auliya, and Islamic
(b) Thanjavur
(c) Gangaikonda Cholapuram
scholars. 110. Advantage(s) of tectonic activity in
(d) Naneghat Iceland include(s)
107. Keppel Island is completely bleached
1. Source of natural geothermal
Ê (d) Shiva Temple was not constructed at mainly due to the expansion of
Naneghat under the patronage of the (a) Starfish
energy
Chola rulers. Naneghat, is a mountain (b) Blue whale 2. Creation of new land
pass in the Western Ghats range (c) Octopus 3. Attraction of tourists
between the Konkan coast and the (d) Sea horse Select the correct answer using the
ancient town of Junnar in the Deccan codes given below.
plateau. Chola empire was not
Ê (a) Keppel Island is completely bleached
mainly due to the expansion of starfish. (a) Only 1
extended to this region therefore no The population of starfish is growing (b) 2 and 3
temple was build in the region. unconditionally due to which starfish are (c) 1 and 3
able to eat coral; a polyp that builds (d) 1, 2 and 3
105. Which of the following statements
about the Deccan Riots Commission limestone reef on which the coral Ê (c) The tectonic plates whose turbulent
community lives. Keppel Island is interactions formed Iceland, are the
is/are correct? located in the southern Great Barrier reef. Eurasian tectonic plate and the North
1. The Commission did not hold Coral bleaching is the process when American tectonic plate. Spanning the
enquiries in the district which corals become white due to various Mid-Atlantic Ridge, Iceland emerged
were not affected. stressors, such as changes in as a result of the divergent, spreading,
2. The Commission did record temperature, light or nutrients. boundary between these two plates
statements of ryots, sahukars and the activity of Iceland´s own
108. Which one of the following rivers is hotspot or mantle plume.
eye- witnesses. not a tributary of river
Select the correct answer using the Brahmaputra? 111. The process whereby certain
codes given below. minerals absorb water, expand and
(a) River Manas
(a) Only 1
(b) River Kameng change is called as
(b) Only 2
(c) River Mahananda (a) Hydration
(c) Both 1 and 2
(d) River Subansiri (b) Oxidation
(d) Neither 1 nor 2
(c) Hydrolysis
Ê (c) In 1875, peasants of Maharashtra in Ê (c) Among the given options, Mahananda
river is not a tributary of river (d) Carbonation
some parts of Pune and Ahmednagar
districts revolted against increasing
Brahmaputra. Tributaries of Brahmaputra: Ê (a) The process whereby certain
left-Lhasa River, Nyang River, Parlung minerals absorb water, expand and
agrarian distress. The Deccan Riots of
Zangbo, Lohit River, Dhansiri River, change is called as Hydration. A good
1875 targeted conditions of debt
Kolong River/ right-Kameng River, Manas example of hydration is the absorption
peonage (kamiuti) to moneylenders.
River, Beki River, Raidak River, Jaldhaka of water by anhydrite, resulting in the
The rioters’ specific purpose was to
River, Teesta River, Subansiri River. formation of gypsum. Hydration
obtain and destroy the bonds, decrees,
The Mahananda River is a expands volume and also results in
and other documents in the possession
trans-boundary river that flows through rock deformation.
of the moneylenders. The movement
the states of Bihar and West Bengal, and
also got support from the Poona 112. Which one of the following is the
Sarvajanik Sabha co-founded by M G Bangladesh. It is an important tributary of
the Ganges.
longest Latitude?
Ranade. However, the Government of
(a) 90 degree Latitude
India pressurised Bombay to enquire 109. Which one of the following is not a (b) 23-5 degree Latitude
into the matter for this the Deccan Riots (c) 0·0 degree Latitude
Commission was set up which
minor plate?
(a) Cocos plate (d) 66·5 degree Latitude
presented a report to the British
Parliament in 1878. The commission (b) Nazca plate Ê (c) The longest latitude is 0° latitude,
(c) Caroline plate known as the Equator which divides
did’nt hold inquiries in areas unaffected
(d) Antarctic plate the Earth into two equal halves, the
by it and also did’nt recorded the
statements of royats, sahukars and Ê (d) Antarctic plate is not a minor plate. northern hemisphere and two southern
eye-witnesses. There are 15 minor plates, these smaller hemisphere.
54 NDA/NA Solved Paper 2021 (I)

113. If it is 12.00 Noon in India, on 116. Which one of the following Ê (c) Muhammadan Anglo-Oriental
which meridian will it be 7:00 am of statements about the Ain-i-Akbari College was founded in 1875 by Sir
Syed Ahmad Khan, initially as a primary
the same day? is not correct?
school, with the intention of taking it to a
(a) 7.5 degree E. Longitude (a) It was written by Abu’l Fazl.
college level institution, known as
(b) 7.5 degree W. Longitude (b) It is a part of a larger work called
Muhammadan Anglo Oriental Collegiate
(c) 75 degree E. Longitude Akbar Nama.
School. Sayyid Ahmad Khan was a
(d) 75 degree W. Longitude (c) It describes the Mughal Empire as
having a diverse population and a great believer in religious toleration. He
Ê (d) If it is 12:00 Noon in India, on 75 composite culture. believed that all religions had a certain
degree, W. Longitude it will be 7:00 am underlying unity which could be called
(d) It was later revised by Sadullah Khan
of the same day. practical morality. Hindus, Parsis, and
on the orders of Shah Jahan.
Current time in India = 12:00 PM Christians had contributed freely to the
Time at required meridian = 7:00 AM
Ê (d) Abu’l Fazl worked on the ‘Akbar funds of his college whose doors were
Nama’. The Akbar Nama is divided into
Difference in time = 5 Hours, and this open for all the Indians.
three books: The first book dealt with
time at meridian (7:00 AM) is 5 hours Akbar’s ancestors, the second 119. Basket-of-eggs topography is
behind. Hence, it is west direction. recorded the events of Akbar’s reign related to
As we know the Earth takes 24 hours, ,the third is the Ain-i Akbari. Badshah (a) Drumlins (b) Eskers
for one complete rotation, so Nama was written by Abul Hamid Lahori (c) Cirques (d) Moraines
24 hours = 360° was later revised by Sadullah Khan on
360 the orders of Shah Jahan. Ê (a) Basket of eggs topography is related
1 hour = = 15°, thus the Earth to Drumlins. Drumlins which is a
24 117. Swami Dayanand Saraswati depositional landform formed by
rotates 15 degree in/hour glaciers. They are forms of rounded
1. was opposed to the worship of
In 5 hours = 5 × 15° = 75° idols of Gods and Goddesses. hummocks resulting from the
Hence, at 75° West longitude, the time deposition of glacial till which look like
2. regarded the Vedas as infallible. inverted boat or spoon
will be 7:00 AM.
3. had met and had discussions with
114. Who among the following was Ishwar Chandra Vidyasagar. 120. Which one of the following planets
considered to be the preceptor of Which of the statements given has the highest density?
Mirabai ? above are correct? (a) Mercury (b) Venus
(a) 1, 2 and 3 (c) Jupiter (d) Earth
(a) Dadu (b) Raidas
(b) 2 and 3
(c) Ramanand (d) Surdas Ê (d) Among the given options, Earth has
(c) 1and 3 the highest density. Earth is the fourth
Ê (b) Guru Raidas, a low caste leather (d) 1 and 2 smallest of the planets. Earth is the
worket was the preceptor of Mirabai. He
was low class leather worker. One of Ê (a) All the statements are correct. densest planet because it is made of
Swami Dayanand Sarasawati was a dense material and it is heavy planet.
the main principle of her philosophy
philosopher, social leader and founder The Average Density of Earth is 5.5
was that one should abandon the
of Arya Samaj. He was opposed to the grams/cm 3.
comforts of life and devote fully to her
God for attainment of peace and worship of idols of Gods and 121. Point of Origin of Earthquake wave
salvation. Goddesses as he believe that God has
is known as
no shape and no form. Hence,
115. Consider the following statements statement 1 is correct. Swami (a) Epicentre (b) Focus
about the Mahanavami Dibba (c) Photosphere (d) Seismic Zone
Dayanand regarded vedas as eternal
1. It was the name of a giant box of and infalliable. Hence, statement 2 is Ê (b) The focus is the place inside Earth's
sweets distributed at the correct. crust where an earthquake originates.
He met Ishwar Chandra Vidyasagar in The point on the Earth's surface directly
Mahanavami festival.
Calcutta and had discussions with him. above the focus is the epicenter.
2. It was the name of a massive
platform with a base covered Hence, statement 3 is correct. 122. The maximum depth of Lithosphere
with relief carvings. 118. Consider the following statements. is found in the
Which of the statements given 1. Muhammadan Anglo-Oriental
(a) Pacific Ocean
above is/are correct? (b) Siberian Plain
(a) Only 1 (b) Only 2
College was founded at Aligarh (c) Patagonian Desert
(c) Both 1 and 2 (d) Neither 1 nor 2 by Sayyid Ahmad Khan. (d) Himalayan Mountains
2. Sayyid Ahmad Khan was a great Ê (a) The maximum depth lithosphere is
Ê (b) Mahanavami Dibba was the name of believer in religious toleration,
massive platform with a base covered found in the pacific ocean. Earth’s
with relief carvings was the King’s and Hindus, Paris and Christians lithosphere, which constitutes the hard
palace in Vijayanagara though there is had contributed to the funds of and rigid outer vertical layer of the
no definite evidence. The Mahanavami his college. Earth, includes the crust and the
Dibba had a very impressive platform Which of the statement(s) given uppermost mantle. Mariana Trench is
known as “the audience hall”. It was above is/are correct? located in the western Pacific Ocean, it
surrounded by high double walls a (a) Only 1 (b) Only 2 is the deepest oceanic trench on Earth
street running between them. (c) Both 1 and 2 (d) Neither 1 nor 2 and the deepest part of the Earth.
NDA/NA Solved Paper 2021 (I) 55

123. A large body of magmatic material celebrate as ‘Independence Day’. The (a) Washington Conference
declaration was passed due to the (b) San Francisco Confernee
that cools in the deeper depth of the (c) Second Continental Congress
breakdown of negotiations between
Earth’s crust and develops in the leaders of the freedom movement and (d) First Continental Congress
form of large domes is known as the British over the question of Ê (c) The United States Declaration of
(a) Batholiths (b) Lacoliths dominion status for India. Independence is the pronouncement
(c) Lopoliths (d) Phacoliths adopted by the Second Continental
127. Which one of the following
Ê (a) A large body of magmatic material provides for the complete equality
Congress meeting in Philadelphia,
that cools in the deeper depth of the Pennsylvania, on 4th July, 1776.
Earth’s crust and develops in the form of men and women in India?
of large domes is known as Batholiths. (a) Articles 14 and 15 of the Constitution 130. Who among the following was the
Forms from cooled magma deep in of India head of the Government that was
Earth's crust. These areas are exposed (b) Fifth Schedule of the Constitution of overthrown by the Bolsheviks in
to the surface through the process of India the 1917 Revolution?
erosion accelerated by continental uplift (c) The Indian Independence Act
(a) Alexander Kerensky
acting over many tens of millions to (d) Article 20 of the Constitution of India
(b) Prince Lvov
hundreds of millions of years. Ê (a) Article 14 The State shall not deny to (c) Grand Duke Sergei
any person equality before the law or (d) Tsar Nicholas II
124. Which one of the following Oil the equal protection of the laws within
Refineries is not located in Assam? the territory of India. Ê (d) Tsar Nicolas II was the head of the
Government that was overthrown by the
(a) Tatipaka (b) Numaligarh Article 15 secures the citizens from Bolsheviks in the 1917 Revolution. The
(c) Bongaigaon (d) Digboi every sort of discrimination by the State, Russian Revolution was a period of
Ê (a) Among the given options, Tatipaka on the grounds of religion, race, caste, political and social revolution that took
Oil Refinery is not located in Assam, it is sex or place of birth or any of them. place in the former Russian Empire and
located in Andhra Pradesh. It was However, this Article does not prevent began during the First World War. The
established in 2001. It is ONGC’s first the State from making any special Russian Revolution was a series of two
ever oil refinery to produce provisions for women or children. revolutions: the first of which overthrew
petro-products. the imperial government and the
128. Which of the following statements second placed the Bolsheviks in power.
125. Where and when did Mahatma about the non-permanent member that Caused the Tsar of the Russian
Gandhi make his first public of the Security Council of the Empire, Nicholas II, to step down from
appearance in India on coming back United Nation is/are correct? his throne.
from South Africa after two 1. Their total number is now 10 but
decades? 131. Which one of the following is not a
was originally only 6.
(a) Champaran in 1917 form of non-permanent member of
2. They are elected for a term of the Security condensation?
(b) Lucknow in 1916
(c) Banaras Hindu University in 1916
two years only. (a) Dew (b) Fog
(d) Ahmedabad in 1918 Select the correct answer using the (c) Frost (d) Sleet
codes given below.
Ê (c) Gandhi's first public appearance in (a) Only 1 Ê (d) Condensation is the change of the
India was on the occasion of the (b) Only 2 state of matter from the gas phase into
opening ceremony of the Banaras (c) Both 1 and 2 the liquid phase, and is the reverse of
Hindu University in February 1916, (d) Neither 1 nor 2 vaporisation. The word most often refers
which was distinguished by the to the water cycle. Sleet is Rain and snow
presence of many magnets and princes Ê (c) The UNSC is composed of 15 mixed is precipitation composed of rain
and of the Viceroy himself. He also members, 5 permanent and 10 and partially melted snow.
passed critical comments on the non-permanent. Five permanent
Princes, as a result many princes members: China, France, the Russian 132. Which one of the following clouds
walked out. Federation, the United Kingdom, and is a rain-bearing cloud?
the United States and Ten (a) Cumulus cloud (b) Stratus cloud
126. When and where was the demand non-permanent members: Elected for (c) Nimbus cloud (d) Cirrus cloud
for “Purna Swaraj” or complete two-year terms by the General
Assembly. Each year, the General Ê (c) Nimbus cloud is a rain-bearing
independence made by the Indian cloud. Nimbus clouds are dark clouds
Assembly elects five non-permanent
National Congress? as they carry a substantial quantity of
members (out of ten in total) for a
(a) Bombay, 1885 water droplets. Nimbus comes in union
two-year term. The ten non-permanent
(b) Lahore, 1929 with other cloud forms that bring
seats are distributed on a regional
(c) Kheda, 1917 substantial precipitation.
basis. As stipulated in rule 144 of the
(d) Bombay, 1942
rules of procedure, a retiring member is 133. Tn which one of the following
Ê (b) The Indian National Congress, on not eligible for immediate re-election. countries is intensive subsistence
19th December, 1929, passed the
‘Purna Swaraj’ – (total independence) 129. At which of the following was the agriculture not predominantly
resolution – at its Lahore session. A American Declaration of practised?
public declaration was made on 26th Independence adopted on 4th July, (a) India (b) Japan
January, 1930 – a day which the 1776? (c) Canada (d) Indonesia
Congress Party urged Indians to
56 NDA/NA Solved Paper 2021 (I)

Ê (c) Intensive subsistence agriculture is (b) The Preamble states the objects of Ê (b) An Outward Direct Investment (ODI)
not predominantly practised in Canada. the Constitution of India, is a business strategy in which a
Intensive farming is the kind of farming (c) The Preamble is enforceable in a domestic firm expands its operations to
in which farmers grow their crops, fruits, Court of Law. a foreign country. For example, some
and vegetables on a small piece of land (d) A Republic refers to the people as the companies will make a green field
source of all authority under the
using simple tools. investment, which is when a parent
Constitution.
company creates a subsidiary in a
134. The Headquarters of South-Eastern Ê (c) Preamble is not enforceable in a foreign country.
Railway is located at court of law. A preamble is an
(a) Bilaspur (b) Secunderabad introductory statement in a document 142. ‘Exercise Desert Knight - 21’ is a
(c) Kolkata (d) Bhubaneswar that explains the document’s bilateral air exercise between the
philosophy and objectives. The Indian Air Force and the Air Force
Ê (c) The Headquarters of South-Eastern
Railway is located at Kolkata (Gardern preamble basically gives idea of the of which one of the following
Reach). It is the administrative and following things/objects: Source of the
countries?
purchase office of the entire zone. Constitution, Nature of Indian State,
(a) USA (b) France
Statement of its objectives, Date of its
(c) Britain (d) Israel
135. Bharatmala Pariyojana is related to adoption.
(a) interlinking of Northern and Southern Ê (b) Indian Air Force (IAF) and French Air
Indian rivers in a garland shape. 139. Which one of the following is not a and Space Force conducted a bilateral
(b) networks of National Highways in power of the Speaker of the Lok Air exercise, Ex Desert Knight-21 at Air
India. Sabha ? Force Station Jodhpur from 20th to 24th
(c) interlinking of all cities of India (a) Speaker shall preside over the House January, 2021. The exercise is unique
through Railways. of the People. as it includes fielding of Rafale aircraft
(d) interlinking of all industrial regions of (b) Speaker will cast vote in the first by both sides and is indicative of the
India through pipelines. instance in the House. growing interaction between the two
(c) Speaker will have power to maintain premier Air Forces.
Ê (b) Bharatmala Pariyojana is an
umbrella program for the highways order within the House of the People.
(d) Speaker can adjourn the House or 143. Tableau of which one of the
sector envisaged by the Ministry of
Road Transport and Highways. To suspend the meeting till there is a following States/Union Territory
optimise the efficiency of freight and quorum. was adjudged best in the Republic
passenger movement across the Ê (b) Speaker does not cast vote in the Day Parade, 2021 ?
country by bridging critical first instance in the house. Speaker can (a) Ladakh
infrastructure gaps through effective only cast his vote when both the (b) Uttar Pradesh
interventions. government and the opposition get (c) Tripura
equal votes on a particular law or a (d) Uttarakhand
136. The natural vegetation which motion.
covers maximum geographical Ê (b) Uttar Pradesh's tableau depicting the
140. Which one of the following is Ram Temple at 2021 Republic Day
areas of India is parade in New Delhi was adjudged the
(a) Tropical deciduous forests included in Article 51A (Part IV A) best and awarded by Union Minister
(b) Tropical thorn forests of the Constitution of India? Kiren Rijiju.
(c) Montane forests (a) Fundamental Duties
(d) Tropical evergreen forests (b) Suspension of Fundamental Rights 144. In the recently concluded elections
(c) Special Powers of Governors in December 2020,
Ê (a) Tropical deciduous are a variety of (d) Writs
temperate forest ‘dominated’ by trees Faustin-Archange Touadera has
that lose their leaves each year. They Ê (a) The idea of Fundamental Duties is won a second term in office as
are found in areas with warm moist inspired from the Constitution of Russia. President of one of the following
summers and cool winters. These were incorporated in Part IV-A of
countries. Identify the country.
the Constitution by the 42nd
137. River Beas, flowing from Himachal Constitutional Amendment Act, 1976 on
(a) Central African Republic
Punjab, joins the river (b) Republic of South Africa
the recommendations of Swaran Singh
(c) Republic of Ghana
(a) Indus (b) Setluj Committee. Listed in Article 51-A of the
(d) Republic of Mozambiqu.
(c) Chenab (d) Ravi Constitution.
Ê (a) In recently concluded elections in
Ê (b) River Bears, flowing from Himachal, 141. As per the data up to November, December 2020, Faustin—Archange
Punjab, joint the river Settuj. The Beas
River rises in the Himalayas in central
2020, released by the Union Finance Touadera has now a second term in
Ministry, which one of the office as President of central African
Himachal Pradesh, and flows for some
following countries ranks 1 in Republic.
470 km to the Setluj River in the Indian
state of Punjab. terms of ODI (Outward Direct 145. Which one of the following
138. With regard to the Constitution of Investment) for the year 2020 - 21 ? statements is not correct in respect
(a) USA of the ‘Legion of Merit’ award?
India, which one of the following (b) Singapore
statements is not correct? (a) This award is conferred by the
(c) Mauritius President of the United States of
(a) The words - Socialist and Secular, (d) United Kingdom America.
were not originally part of the
Constitution.
NDA/NA Solved Paper 2021 (I) 57

(b) This award was conferred to the Ê (d) A large-scale Tri service Military among the teams from the Ranji
Prime Minister of India in December, ‘Exercise Kavach’ conducted in Trophy.
2020. January 2021 under the aegis of the
(c) This was also awarded to the Prime Andaman and Nicobar Command
149. Which country has replaced
Minister of Australia, Scott Morrison. (ANC), the only Joint Forces Command Maldives to host the 2023 Indian
(d) It is the highest civilian award of the Ocean Island Games?
of the country. The exercise involves
United States of America. (a) Madagascar (b) Sri Lanka
synergised application of maritime
Ê (d) Legion of Merit award is a military surveillance assets, coordinated air and (c) India (d) Mauritius
award of the United States Armed maritime strikes, air defence, submarine
forces that is given for exceptionally
Ê (a) Madagascar replaced Maldives to
and landing operations. host the 2023 Indian Ocean Island
meritorious conduct in the performance Games. It is a quadrennial multi sport
of outstanding services and 147. Recently a state of emergency has
event from Indian ocean Islands
achievements. This award was been declared in which one of the nations. The games was created by
confused to the Prime Minister of India following countries? international Olympic Committee in
in December, 2020. It was also (a) Maldives (b) Bhutan 1977.
awarded to the Prime Minister of (c) Nepal (d) Myanmar
Australia, Scott Morrison. 150. Identify the correct reason, out of
Ê (d) State of emergency was declared in the following, about Claire Polosak
146. Which one of the following Myanmar. It was declared on
statements is most appropriate 1st February, 2021. The state of for being in the news.
emergency was declared for one year. (a) She has been honoured with the
about ‘Exercise Kavach’ ? Pulitzer Prize.
(a) It is a military exercise of the Indian 148. Which one of the following teams is (b) She recently won a Grand Slam
Army only. the winner of the Syed Mushtaq Ali championship.
(b) It is a joint military exercise involving (c) She became the first female match
Trophy, 2021?
the Indian Army and the Indian Navy official to conduct a men’s cricket test
only. (a) Uttar Pradesh (b) Punjab
(c) Tamil Nadu (d) Baroda match.
(c) It is a joint military exercise involving (d) She has been conferred with the
the Indian Army, the Indian Navy and Ê (c) The Indian State of Tamil Nadu won Gandhi Peace Prize.
the Indian Air Force only. the Syed Mushtaq Ali Trophy, 2021.
(d) It is a joint military exercise involving It is a domestic Twenty 20 cricket
Ê (c) Claire Polosak was in the news
the Indian Army, the Indian Navy, the because she became the first female
championship in India, organised by match official to conduct a men’s
Indian Air Force and also the Indian
the Board of control for cricket in India, cricket test match.
Coast Guard.
NDA/NA Solved Paper 2020 (I & II) 1

NDA /NA
National Defence Academy/Naval Academy

SOLVED PAPER 2020 (I & II)


PAPER I : Mathematics
1−i i 
1. If matrix A =  where ⇒ 20 − ( n + 2 ) = n − 2
1 − i  Ê (a) The general term in the binomial
 −i 10
⇒ 20 = n + 2 + n − 2
expansion of  2 − x 
2
is
i = − 1, then which one of the x  ⇒ 2 n = 20
following is correct? 10 − r ⇒ n = 10
C r  2 
2
(a) A is hermitian Tr +1 = 10
( − x )r
x  5. For how many values of k, is the
(b) A is skew-hermitian
(c) ( A )T + A is hermitian −20 + 2r +
r
 0 k 4
= C r (2 )10 − r x
10
( −1)r
matrix − k 0 − 5 singular?
2
(d) ( A )T + A is skew-hermitian  
For independent of x, put
Ê (c) We have r 5r − k k − 1
−20 + 2 r + = 0 ⇒ = 20
1 − i i 
A= 
2 2 (a) Only one (b) Only two
 −i 1 − i  ⇒ r=8 (c) Only four (d) Infinite
1 + i −i  ∴ T8 + 1 = 10C 8(2 )10 − 8 ( −1)8
Now, A =  Ê (d) The condition for singular matrix is
 i 1 + i  10 × 9 0 k 4
= × 22 × 1
1 + i i  2 ×1 − k 0 −5 = 0
( A )T = 
 − i 1 + i  = 180 −k k −1
Now, consider Expanding along R1, we get
3. If (1 + 2x − x 2 )6 = a 0 + a1x + a 2 x 2
1 + i i  0 − k( k − 5k ) + 4( − k 2 + 0) = 0
X = ( A) + A = 
T

 −i 1+ i  + K + a12 x 12 , then what is a 0 − a1


⇒ 4k 2 − 4k 2 = 0
1 − i i  + a 2 − a 3 + a 4 − K + a12 equal to?
+  ⇒ 0 = 0, ∀k ∈ R
 −i 1− i  (a) 32 (b) 64
Hence, for infinite values of k, given
 2 2i  (c) 2048 (d) 4096
= matrix is singular.
 Ê (b) We have,
 −2 i 2 
(1 + 2 x − x 2 )6 = a0 + a1 x + a2 x 2 6. The number (1101101 + 1011011)2
 2 −2 i 
X = 
can be written in decimal system as
2 i 2  + ....+ a12 x 12
(a) (198)10 (b) (199)10
 2 2i  Put x = − 1 both sides, we get (c) (200)10 (d) (201)10
( X )T =   = X 5
 −2 i 2  (1 − 2 − 12 )6 = a0 − a1 + a2 − ....+ a12 Ê (c) Now, (1101101)2 = 1 × 2 + 0 × 2
6

Hence, X = ( A )T + A is a hermitian ⇒ a0 − a1 + a2 − .....+ a12 = ( −2 )6 = 64 + 1 × 2 4 + 1 × 2 3 + 0 × 2 2 + 1 × 21


matrix. + 1 × 20
4. If C (20, n + 2) = C (20, n − 2), then
2. The term independent of x in the what is n equal to? = 64 + 0 + 16 + 8 + 0 + 2 + 1
10 = ( 91)10
 2  (a) 18 (b) 25
binomial expansion of  2 − x  and
x  (c) 10 (d) 12
(1011011)2 = 1 × 2 6 + 1 × 2 5 + 0 × 2 4
is equal to Ê (c) Given, C(20, n + 2) = C (20, n − 2 )
⇒ C (20, 20 − ( n + 2 )) = C (20, n − 2 ) + 1 × 2 3 + 1 × 2 2 + 0 × 21 + 1 × 2 0
(a) 180 (b) 120
(c) 90 (d) 72 [Q C ( n, r ) = C ( n, n − r )] = 64 + 32 + 0 + 8 + 4 + 0 + 1 = (109)10
2 NDA/NA Solved Paper 2020 (I & II)

∴ (1101101 + 1011011)2 Also given, AB = C i i2 i3


= (1101101)2 + (1011011)2 x + y y  2   3 Ê (d) Let ∆ = i
4
i 6
i8
⇒  2x =
= ( 91)10 + (109)10 = (200)10  x − y  −1 2 
  i 9
i 12
i 15
7. What is the value of 2 x + 2 y − y  3 i −1 − i
⇒  4x − x + y  = 2 
1 1     = 1 −1 1
log 5 1024 − log 5 10 + log 5 3125?
10 5 2 x + y  3 i 1 −i
⇒  3 x + y = 2 
(a) 0 (b) 1 (c) 2 (d) 3     [Q i 2 = − 1, i 3 = − i , i 4 = 1]
1 1
Ê (a) log 5 1024 − log 5 10 + log 5 3125 On equating the corresponding
= i ( i − 1) + 1 ( − i − i ) − i (1 + i )
10 5 elements, we get
1 1 2 x + y = 3 and 3 x + y = 2 [Expanding along R1]
= log 5 2 − log 5( 5 × 2 ) + log 5 5 5
10
10 5 ⇒ x = − 1 and y = 5 = i 2 − i − 2 i − i − i 2 = − 4i
=
10 5
log 5 2 − [log 5 5 + log 5 2 ] + log 5 5  −1 + 5 5  4 5
∴ A= =  a h g  x
2 × − 1 −1 − 5  −2 −6
10 5
[Q log mn = log m + log n] 13. Let A =  h b 
f and B =  y ,
  
∴The determinant of matrix A is
= log 5 2 − [1 + log 5 2 ] + 1 [Q log m m = 1] 4 5
 g f c   z 
=0 A = = −24 + 10 = −14
−2 −6 then what is AB equal to?
8. If x = logc (ab ), y = loga (bc ),  ax + hy + gz
z = logb (ca ), then which of the . ≤ x ≤ 4.5, then which one of
10. If 15
the following is correct? (a)  y 
following is correct?  
(a) (2 x − 3) (2 x − 9) > 0  z 
(a) xyz = 1  ax + hy + gz
(b) (2 x − 3) (2 x − 9) < 0
(b) x + y + z = 1
(c) (2 x − 3) (2 x − 9) ≥ 0 (b)  hx + by + fz 
(c) (1 + x )−1 + (1 + y)−1 + (1 + z)−1 = 1  
(d) (2 x − 3) (2 x − 9) ≤ 0  
(d) (1 + x )−2 + (1 + y)−2 (1 + z)−2 = 1
z
. ≤ x ≤ 4.5
Ê (d) We have, 15  ax + hy + gz
Ê (c) We have, x = logc ( ab ) (c)  hx + by + fz 
y = log a( bc ) 3 9  
⇒ ≤ x≤ ⇒ 3≤ 2x≤ 9
z = log b (ca )
2 2  gx + fy + cz 
⇒ (2 x − 3) (2 x − 9) ≤ 0 (d) [ax + hy + gz hx + by + fz
Now, 1 + x = logc c + logc ( ab )
= logc ( abc ) 11. Let S = {1, 2, 3, K}. A relation R on gx + fy + cz]
1 + y = log a( abc ) S × S is defined by xRy if Ê (c) Now,
and 1 + z = log b ( abc ) 1 a h g   x
loga x > loga y when a = . Then
Now, (1 + x )−1 + (1 + y)−1 + (1 + z)−1 2 AB =  h f   y
b
   
= [logc ( abc )]−1 + [log a( abc )]−1 the relation is  g f c   z
(a) reflexive only  ax + hy + gz
+ [log b ( abc )]−1
=  hx + by + fz 
(b) symmetric only
=
1
+
1
+
1 (c) transitive only  
logc ( abc ) log a( abc ) log b ( abc ) (d) both symmetric and transitive  gx + fy + cz 

=
log c
+
log a
+
log b Ê (c) We have, S = {1, 2, 3, ....} 14. What is the number of ways in
log( abc ) log( abc ) log( abc ) and log a x > log a y
which the letters of the word
 log n  1
a = ∈ ( 0, 1)
Q log m n = log m 
Here, ‘ABLE’ can be arranged so that the
  2
vowels occupy even places?
log c + log a + log b log( abc ) ∴ log a x > log a y ⇒ x < y
= = =1 (a) 2 (b) 4 (c) 6 (d) 8
log( abc ) log( abc ) Now, x R x ⇒ x < x which is not
possible. So it is not reflexive relation. Ê (b) In a given word ‘ABLE’ Vowels are
9.  x +y y   2 Now, x R y ⇒ x < y
{ A, E}.
Let A =  , B =  − 1 1 2 3 4
 2 x x − y    But y |< x, so it is not symmetric relation.
As, vowel occupy even places, so two
3 Now, x R y and y R z vowels occupy the places 2 and 4.
and C =  . If AB = C , then what is ⇒ x < y and y < z ⇒ x < z ⇒ x R z
2 Therefore, the number of ways of
Hence, it is transitive relation only. occupying the vowels in even places is
the value of the determinant of the
2!.
matrix A? 12. What is the value of the
Now, we have two consonants and
(a) − 10 (b) − 14  i i 2 i 3
these consonants occupy the odd
determinant i 4 i 6 i 8 where
(c) − 24 (d) − 34
places 1 and 3. Therefore, the number of
x + y y  
 9 12 15 
Ê (b) Given, A =  2 x x − y ways of occupying the consonants in
  i i i 
odd places is 2!.
2   3
B =   and C =  
i = −1? ∴Total number of ways = 2 ! × 2 !
 −1 2  (a) 0 (b) − 2 (c) 4i (d) − 4i =2 ×2= 4
NDA/NA Solved Paper 2020 (I & II) 3

15. What is the maximum number of Ê 16. (c) Also given, Hence, for real values of tan A, K cannot
lie between  , 3 .
n( Y ) 4 1
points of intersection of 5 =
n( Z ) 5 3 
non-overlapping circles?
(a) 10 (b) 15 (c) 20 (d) 25 16 + 18 + 17 + b 4
⇒ = Directions (Q. Nos. 21 and 22) Read
90 5
Ê (c) The maximum number of points of the following information and answer
intersection of 5 non-overlapping circles ⇒ 51 + b = 72
the two items that follow.
⇒ b = 72 − 51 ABCD is a trapezium such that AB and CD
= Selection of two circles × 2 = 21 are parallel and BC is perpendicular to
[Q Two intersecting circles Ê 17.(d) Now, them. Let ∠ADB = θ, ∠ABD = α , BC = p
cut at two points] n( X ) + n( Y ) + n( Z ) − n( X ∩ Y ) and CD = q.
5×4 − n( Y ∩ Z ) − n( X ∩ Z ) + n( X ∩ Y ∩ Z )
= C2 × 2 =
5
× 2 = 20
2 ×1 = n( X ∪ Y ∪ Z ) 21. Consider the following
= a + 12 + 18 + 16 + b + 17 + c 1. AD sin θ = AB sin α
Directions (Q. Nos. 16-18) = a + b + c + 63 2. BD sin θ = AB sin (θ + α )
Consider the following Venn diagram, Which of the above is/are correct?
where X, Y and Z are three sets. Let the = a + b + 43 + 63 [Q c = 43]
(a) 1 Only (b) 2 Only
number of elements in Z be denoted by = a + b + 106 (c) Both 1 and 2 (d) Neither 1 nor 2
n(Z) which is equal to 90. Ê 18. (a) Complement of X Ê (c) We have, ∠ADB = θ, ∠ABD = α,
= p + b + c + 17
X Y = p + b + 43 + 17 [Q c = 43] BC = p and CD = q
= p + b + 60 D q C
a 16 b
Directions (Q. Nos. 19 and 20) Read θ
18 p


12 17 the following information and answer

90°
the two items that follow.
c tan 3A α
Let = K , where tan A ≠ 0 A B
Z tan A
1 1. In ∆ABD, use Sine rule,
and K ≠ . sinθ sinα
3 =
16. If the number of elements in Y and AB AD
Z are in the ratio 4 : 5, then what is 19. What is tan 2 A equal to? ⇒ ADsinθ = ABsinα, which is correct.
the value of b? K + 3 K −3 2. In ∆ABD, ∠A = π − (θ + α )
(a) (b)
(a) 18 (b) 19 (c) 21 (d) 23 3K − 1 3K − 1 Use Sine rule in ∆ABD,
3K − 3 K + 3 sin A sinθ
17. What is the value of (c) (d) =
n ( X ) + n (Y ) + n (Z ) − n ( X ∩ Y ) K −3 3K + 1 BD AB
sin( π − (θ + α )] sinθ
− n (Y ∩ Z ) − n ( X ∩ Z ) tan 3 A ⇒ = …(i)
Ê (b) Given, =K BD AB
+ n ( X ∩ Y ∩ Z )? tan A
⇒ ABsin(θ + α ) = BDsinθ,
(a) a + b + 43 (b) a + b + 63 3 tan A − tan3 A
= =K which is correct.
(c) a + b + 96 (d) a + b + 106 (1 − 3 tan2 A ) tan A
Hence, both statements are
18. If the number of elements ⇒
3 − tan2 A
=K
correct.
belonging to neither X, nor Y, nor Z 1 − 3 tan2 A 22. What is AB equal to?
is equal to p, then what is the ⇒ K − 3K tan2 A = 3 − tan2 A ( p2 + q 2 ) sin θ
(a)
number of elements in the ⇒ K − 3 = tan A( 3K − 1)
2 p cos θ + q sin θ
complement of X? ( p2 − q 2 ) cos θ
K −3 (b)
(a) p + b + 60 (b) p + b + 40 ⇒ tan2 A = p cos θ + q sin θ
(c) p + a + 60 (d) p + a + 40 3K − 1
( p2 + q 2 ) sin θ
Solutions (16-18) (c)
20. For real values of tan A, K cannot q cos θ + p sin θ
Given n( Z ) = 90 lie between ( p2 − q 2 ) cos θ
⇒ 12 + 18 + 17 + c = 90 1 1 (d)
(a) and 3 (b) and 2 q cos θ + p sin θ
⇒ c = 90 − 47 = 43 3 2
X Y
1
(c) and 5
1
(d) and 7 Ê (a) In right angle, ∆BCD,
5 7 ∠B = 90° − α
a 16 b Ê (a) For real values of tan A, K lies when BD = p2 + q 2
K −3
18 ≥ 0 and 3K − 1 ≠ 0 CD
12 17 3K − 1 and sin B =
1 BD
⇒ ( K − 3) ( 3K − 1) ≥ 0 and K ≠ q
c 3 ⇒ sin( 90° − α ) =
1 p2 + q 2
Z ⇒ K < and K ≥ 3
3 [Q ∠B = 90°−α ]
4 NDA/NA Solved Paper 2020 (I & II)

q π = − 2 sin 30° sin18°


⇒ cosα = ⇒ 2B = − A
p2 + q 2 2 1 5 −1
π = −2 × ×
BC ⇒ A + 2B = 2 4
and cos B = 2
BD 1− 5
=
⇒ cos( 90° − α ) =
p 25. What is sin 3x + cos 3x + 4 sin 3 x 4
p +q2 2
− 3 sin x + 3 cos x − 4 cos 3 x equal 29. Consider the following statements:
p
⇒ sinα = to? 1. If ABC is a right-angled triangle,
p2 + q 2 (a) 0 (b) 1 right-angled at A and if
(c) 2 sin 2 x (d) 4 cos 4x 1
From eq. (i), sin B = , then cosec C = 3.
Ê (a) sin 3 x + cos 3 x + ( 4sin x − 3sin x)
3
sin( π − (θ + α )) sinθ 3
=
BD AB + ( 3 cos x − 4 cos 3 x ) 2. If b cos B = c cos C and if the
BD sinθ triangle ABC is not right-angled,
⇒ AB = = sin 3 x + cos 3 x − sin 3 x − cos 3 x = 0
sin(θ + α ) then ABC must be isosceles.
26. The value of ordinate of the graph Which of the above statements
p2 + q 2 sinθ
= of y = 2 + cos x lies in the interval is/are correct?
sinθ cos α + cos θ sinα (a) 1 Only (b) 2 Only
(a) [0, 1 (b) [0, 3] (c) [− 1, 1] (d) [1, 3]
(c) Both 1 and 2 (d) Neither 1 nor 2
[Q BD = p +q ]
2 2
Ê (d) We know that, 1
p + q sinθ
2 2 −1 ≤ cos x ≤ 1 Ê (b) 1. We have, sin B =
= 3
q p ⇒ −1 + 2 ≤ 2 + cos x ≤ 1 + 2 C
sinθ + cos θ
p2 + q 2 p2 + q 2 ⇒ 1≤ y ≤ 3
∴ y ∈ [1, 3]
( p2 + q 2 )sinθ
=
q sinθ + pcos θ 27. What is the value of
cos 17 ° − sin 17 ° 8 cos 10° ⋅ cos 20° ⋅ cos 40° ?
23. If tan θ = , then
cos 17 ° + sin 17 ° (a) tan 10° (b) cot 10°
what is the value of θ? (c) cosec 10° (d) sec 10°
B A
(a) 0° (b) 28° (c) 38° (d) 52° Ê (b) 8cos 10° cos 20° cos 40°
AC 1
Ê (b) We have, sin10° ⇒ =
= 8 cos 10° cos 20° cos 40° × BC 3
cos 17 ° − sin17 ° sin10°
tanθ = ⇒ AC = k and BC = 3k
cos 17 ° + sin17 ° 4(2 sin10° cos 10° ) cos 20° cos 40°
=
1 − tan17 ° sin10° Use pythagoras theorem in ∆ABC,
= 4 sin20° cos 20° cos 40°
1 + tan17 ° = AB = ( BC )2 − ( AC )2
sin10°
[Divide numerator and = ( 3k )2 − ( k )2
denominator by cos17 °] Q2 sin A cos A = sin2 A
2(2 sin20° cos 20° ) cos 40° = 9k 2 − k 2
⇒ tanθ = tan( 45° − 17 ° ) =
sin10°
 tan 45° − tan17 °  = 8k 2
Q tan( 45° − 17 ° ) = 1 + tan 45° tan17 °  2 × sin 40° cos 40°
=
  sin10° = 2 2k
⇒ tanθ = tan28° sin 80° sin( 90° − 10° ) BC
= = Now, cosec C =
⇒ θ = 28° sin10° sin10° AB
cos 10° 3k 3
= = cot 10° = = , which is not correct.
24. A and B are positive acute angles sin10° 2 2k 2 2
such that cos 2B = 3 sin 2 A and 2. Suppose we consider ∆ABC is an
3 sin 2A = 2 sin 2B. What is the
28. What is the value of isosceles triangle.
cos 48° − cos 12°?
value of ( A + 2B )? A
π π π π 5 −1 1− 5
(a) (b) (c) (d) (a) (b)
6 4 3 2 4 4 c b
5+1 1− 5
Ê (d) We have, cos 2 B = 3sin A
2
(c) (d)
2 8
and 3 sin2 A = 2 sin2 B
2 sin2 B 3 sin2 A Ê (b) cos 48° − cos 12 ° B C
∴ = 48° + 12 °   48° − 12 ° 
cos 2 B 3 sin2 A = − 2 sin  sin  ∴ ∠B = ∠C
 2   2 
sin2 B 2 × sin A cos A Also we have, b cos B = c cos C
⇒ 2 =
cos 2 B sin2 A  C + D 
Q cos C − cos D = − 2 sin 2   ⇒ b cos C = c cos C
⇒ tan2 B = cot A   [put B = C ]
 C − D 

π
tan2 B = tan − A  sin  ⇒ b = c , Which is correct.
2    2  

NDA/NA Solved Paper 2020 (I & II) 5

30. Consider the following statements Ê (a) We have, (sin3 θ + cos 3 θ)

1. If in a triangle ABC, A = 2B and a sin x + b cos x = c


2 2
t3 − t5 − (sin5 θ + cos 5 θ)
Now, =
2 t5 − t7 (sin θ + cos 5 θ)
5
b = c , then it must be an On dividing both sides by cos x, we get
obtuse-angled triangle. a tan2 x + b(1) = c × sec 2 x − (sin7 θ + cos 7 θ)
2. There exists no triangle ABC ⇒ a tan x + b = c(1 + tan x )
2 2
(sin θ − sin θ) + (cos 3 θ − cos 5 θ)
3 5

with A = 40° , B = 65° and =


[Q sec θ − tan θ = 1]
2 2
(sin5 θ − sin7 θ) + (cos 5 θ − cos 7 θ)
a
= sin 40° cosec 15°. ⇒ tan2 x( a − c ) = c − b sin3 θ(1 − sin2 θ) + cos 3 θ (1 − cos 2 θ)
c =
c−b sin5 θ (1 − sin2 θ) + cos 5 θ (1 − cos 2 θ)
Which of the above statements ⇒ tan2 x = …(i)
is/are correct? a−c
sin3 θ cos 2 θ + cos 3 θ sin2 θ
(a) 1 Only (b) 2 Only d −a =
32. What is equal to? sin5 θ cos 2 θ + cos 5 θ sin2 θ
(c) Both 1 and 2 (d) Neither 1 nor 2
b−d
2 2 sin2 θ cos 2 θ (sinθ + cos θ)
Ê (d) 1. We have, in ∆ABC, (a) sin y (b) cos y =
A = 2 B and b = c (c) tan2 y (d) cot 2 y sin2 θ cos 2 θ (sin3 θ + cos 3 θ)
Ê (c) We have, b sin y + a cos y = d
2 2
⇒ Angles opposite to equal sides are sinθ + cos θ t1
= =
equal. On dividing both sides by cos 2 y, we get sin3 θ + cos 3 θ t3
∴ ∠C = ∠B b tan2 y + a(1) = d (sec 2 y)
Also, A = 2 B = 2C
35. What is t 12 − t 2 equal to?
⇒ b tan2 y + a = d (1 + tan2 y) (a) cos 2θ (b) sin 2θ
In ∆ABC, ∠A + ∠B + ∠C = 180°
[Q sec 2 θ − tan2 θ = 1] (c) 2 cos θ (d) 2 sin θ
⇒ 2C + C + C = 180°
Ê (b) t 1 − t 2 = (sinθ + cos θ)
2 2
180° ⇒ tan2 y ( b − d ) = d − a
⇒ C = = 45°
4 d −a − (sin2 θ + cos 2 θ)
⇒ = tan2 y …(ii)
⇒ B = 45° and A = 90° b−d = sin2 θ + cos 2 θ + 2 sinθ cos θ
Thus, it shows that ∆ABC is not an 2
33. What is
p
equal to? − (sin2 θ + cos 2 θ)
obtuse angle triangle.
q2 = 2 sinθ cos θ = sin2θ
Hence, statement 1 is incorrect.
(b − c ) (b − d ) ( a − d ) (c − a )
2. We have A = 40°, B = 65° (a) (b) 36. What is the value of t 10 where
(a − d ) (a − c ) ( b − c ) (d − b )
C θ = 45°?
(d − a ) (c − a ) (b − c ) (b − d ) 1 1 1
(c) (d) (a) 1 (b) (c) (d)
75° ( b − c ) (d − b ) (c − a ) ( a − d ) 4 16 32
b a
Ê (c) Now, t 10 = sin θ + cos θ
10 10
Ê (b) We have,
p tan x = q tan y = (sin 45° )10 + (cos 45° )10
40° 65°
c On squaring both sides, we get. [Put θ = 45°]
A B
p2 tan2 y 10 10
=
= 
1 
+ 
In ∆ABC, 1 
q 2 tan2 x  
 2  2
∠A + ∠B + ∠C = 180° (d − a ) / ( b − d )
=
= 2 5  = 4 =
⇒ 40° + 65° + ∠C = 180° 1 1 1
(c − b ) / ( a − c )
⇒ ∠C = 75°  2  2 16
[Q from eq. (i) and (ii)]
Use sine rule in ∆ABC, (d − a ) ( a − c )
a c = Directions (Q. Nos. 37-39) Read the
= ( b − d ) (c − b ) following information and answer the
sin 40° sin75°
( a − d ) (c − a ) three items that follow.
a
= sin 40° cos ec 75°, =
(d − b ) ( b − c ) Let α = β = 15°.
c
Hence, Statement 2 is incorrect. 37. What is the value of sin α + cos β?
Directions (Q. Nos. 34-36) Read the 1 1 3 3
following information and answer the (a) (b) (c) (d)
Directions (Q. Nos. 31-33) Read the 2 2 2 2 2 2
following information and answer the three items that follow.
Let t n = sin n θ + cosn θ Ê (d) sinα + cos β
three items that follow.
t3 − t5 =  cos β  × 2
Let a sin 2 x + b cos2 x = c, 1 1
sinα +
34. What is equal to?  2 2 
b sin 2 y + a cos2 y = d t5 − t7
and p tan x = q tan y t1 t3 = 2 (sinα cos 45° + sin 45° cos β )
(a) (b)
t3 t5 = 2 (sin15° cos 45° + sin 45° cos 15° )
31. What is tan 2 x equal to? t t [Q α = β = 15° ]
c−b a−c (c) 5 (d) 1
(a) (b) t7 t7 = 2 sin(15° + 45° )
a−c c−b
3 3
(c)
c−a
(d)
c−b Ê (a) We have, = 2 sin 60° = 2 × =
t n = sinn θ + cos n θ 2 2
c−b c−a
6 NDA/NA Solved Paper 2020 (I & II)

cot α cot β − 1
38. What is the value of Ê (c) Given, sin x + sin y = cos y − cos x Now, cot(α + β ) =
cot α + cot β
sin 7α − cos 7β? ⇒ 2 sin
x + y  x − y
 cos   2 −1 1
1 1 3 3  2   2  = =
(a) (b) (c) (d)
3 3
2 2 2 2 2 2 y + x  y − x
= − 2 sin  sin 
Ê (d) sin7α − cos 7β  2   2  44. The roots α and β of a quadratic
x − y  x − y equation, satisfy the relations
= 
1
sin7 × 15° −
1
cos 7 × 15°  2 ⇒ cos   = sin 
 2 2   2   2  α + β = α 2 + β 2 and αβ = α 2β 2 .
x − y
= 2 (sin105° cos 45° − cos 105° sin 45° ) ⇒ tan  =1
What is the number of such
= 2 sin(105° − 45° )  2  quadratic equations?
= 2 sin 60° 41. If A is a matrix of order 3 × 5 and B
(a) 0 (b) 2 (c) 3 (d) 4
Ê (d) Given, α + β = α + β
2 2
= 2 ×
3
=
3 is a matrix of order 5 × 3, then the
2 2 order of AB and BA will and αβ = α 2β 2
39. What is sin (α + 1° ) + cos (β + 1° ) respectively be ⇒ αβ(1 − αβ ) = 0
(a) 3 × 3 and 3 × 3 ⇒ αβ = 0 and αβ = 1
equal to? (b) 3 × 5 and 5 × 3
(a) 3 cos 1° + sin 1° Consider α + β = α 2 + β 2
(c) 3 × 3 and 5 × 5
1 = (α + β )2 − 2αβ
(b) 3 cos 1° − sin 1° (d) 5 × 3 and 3 × 5
2
1 Ê (c) Given, matrix A is of order 3 × 5 and When αβ = 0,
(c) ( 3 cos 1° + sin 1° )
2 matrix B is of order 5 × 3. α + β = (α + β )2 − 2 × 0
1 ∴Order of matrix AB = [ A ]3 × 5 [B]5 × 3 ⇒ (α + β ) = (α + β )2
(d) ( 3 cos 1° + sin 1° )
2 = [ AB]3 × 3 ⇒ (α + β ) [1 − (α + β )] = 0
Ê (c) sin(α + 1° ) + cos(β + 1° ) and order of matrix BA = [B]5 × 3 [ A ]3 × 5 ⇒ (α + β ) = 0
= sin(α + 1° ) + cos(α + 1° ) [Q α = β ] = [BA ]5 × 5 and α + β =1
= sinα cos 1° + cos α sin1° 42. If p 2 , q 2 and r 2 (where p , q , r > 0) It implies that when αβ = 0, gives two
+ cos α cos 1° − sinα sin1° quadratic equations.
are in GP, then which of the
= cos 1° (sinα + cos α ) When αβ = 1
following is/are correct?
+ sin1° (cos α − sinα ) α + β = (α + β) 2 − 2αβ
3 1. p , q and r in GP.
= cos1° × ⇒ (α + β ) = (α + β )2 − 2(1)
2 2. ln p , ln q and lnr are in AP.
Select the correct answer using the ⇒ (α + β )2 − (α + β ) − 2 = 0
+ sin1°  sinα  2
1 1
cos α − code given below ⇒ [(α + β ) − 2 ] [(α + β ) + 1] = 0
 2 2 
(a) 1 Only (b) 2 Only ⇒ α + β = 2, − 1
 3 (c) Both 1 and 2 (d) Neither 1 nor 2
Q sinα + cos α =  Again it implies that, when αβ = 1, gives
 2 2 2 2
Ê (c) 1. Given, p , q and r in GP. two quadratic equations.
3 ∴ q2 = p2r 2 ⇒ q 2 = pr Hence, there are total four such
= cos 1° + 2 sin1° (sin 45° cosα …(i)
2 quadratic equations formed.
⇒ p, q and r in GP, hence Statement 1
− cos 45° sinα )
is correct. 45. What is the argument of the
3
= cos 1° + 2 sin1° sin( 45° − α ) ln p + ln r ln pr lnq 2 1−i 3
2 2. Now, = = complex number , where
2 2 2
3 1+i 3
= cos 1°+ 2 sin1° sin 30° [Q α = 15° ] [From eq. (i)]
2
lnq i = − 1?
3 2 =2 = lnq
= cos 1° + sin1° 2 (a) 240° (b) 210° (c) 120° (d) 60°
2 2 ⇒ ln p, lnq and ln r are in AP, which is 1− i 3 1− i 3
3 1 correct statement. Ê (a) Let z = ×
= cos 1° + sin1° 1+ i 3 1− i 3
2 2 Hence, both statements are correct.
(1 − i 3 )2
=
1
( 3 cos 1° + sin1° ) =
2 43. If cot α and cot β are the roots of 12 − ( i 3 )2
the equation x − 3x + 2 = 0, then
2
1 − 3 − 2 i 3 −2 − 2 i 3
40. If sin x + sin y = cos y − cos x , = =
π what is cot (α + β ) equal to? 1+ 3 4
where 0 < y < x < , then what is 1 1 −1 − i 3
2 (a) (b) (c) 2 (d) 3 =
2 3
 x − y 2
tan   equal to? Ê (b) Given, cot α and cot β are the roots of − 3 /2
 2  Now, tanθ =
y
=
x 2 − 3 x + 2 = 0. x −1 / 2
1
(a) 0 (b)
2 Now, sum of roots, cot α + cot β = 3 = 3 = 60°
(c) 1 (d) 2 and product of roots, cot α cot β = 2
NDA/NA Solved Paper 2020 (I & II) 7

1 q r 3−1 2
Since, given complex number lies in IIIrd ∴ adj A = A = A ,
quadrant. D = ( p + q + r) 0 r−q p− r
Which is incorrect statement.
∴ arg( z) = 180° + 60°= 240° 0 p−q q − r Hence, only Statement 1 is correct.
46. What is the modulus of the complex = ( p + q + r ) [1{( r − q ) (q − r ) 51. The centre of the circle ( x − 2a )
cos θ + i sin θ − ( p − r ) ( p − q )}] ( x − 2 b ) + ( y − 2c ) ( y − 2d ) = 0 is
number , where
cos θ − i sin θ = ( p + q + r ) [rq − r 2 − q 2 + qr (a) (2 a, 2c ) (b) (2 b, 2d )
(c) ( a + b, c + d ) (d) ( a − b, c − d )
i = − 1? − { p2 − pq − rp + rq }]
1 3 = ( p + q + r )[− p − q 2
2 Ê (c) Given equation ( x − 2 a) ( x − 2 b )
(a) (b) 1 (c) (d) 2 + ( y − 2c ) ( y − 2d ) = 0 is a circle,
2 2 − r 2 + pq + pr + rp]
cos θ + i sinθ whose end points of a diameter are
Ê (b) Let z = =
−( p + q + r )
[2 p2 + 2q 2 A(2 a, 2c ) and B(2 b, 2d ).
cos θ − i sinθ
2 Now, centre of circle
cos θ + i sinθ
⇒ z = + 2 r 2 − 2 pq − 2qr − 2 rp] = mid point of diameter.
|cos θ − i sinθ| ( p + q + r) 2 2 a + 2 b 2c + 2d 
= [( p + q 2 − 2 pq ) =  , 
cos 2 θ + sin2 θ 2  2 2 
=
cos 2 θ + sin2 θ + (q 2 + r 2 − 2qr ) + ( r 2 + p2 − 2 rp)] = ( a + b, c + d )
1
=1 = − ( p + q + r ) [( p − q )2 52. The point (1, − 1) is one of the
2
vertices of a square. If 3x + 2y = 5 is
47. Consider the proper subsets of {1, 2, + (q − r )2 + ( r − p)2 ]
the equation of one diagonal of the
3, 4}. How many of these proper Since, p, q and r are distinct. square, then what is the equation of
subsets are superset of the set {3}? ∴ D< 0
the other diagonal?
(a) 5 (b) 6
49. What is the sum of the last five (a) 3x − 2 y = 5 (b) 2 x − 3 y = 1
(c) 7 (d) 8
coefficients in the expansion of (c) 2 x − 3 y = 5 (d) 2 x + 3 y = − 1
Ê (c) The proper subset of {1, 2, 3, 4} are (1 + x )9 when it is expanded in Ê (c) We have point (1, − 1) is one of the
{},{1},{2},{ 3},{ 4},{1, 2},{2, 3},{ 3, 4},
ascending powers of x? vertices of a square.
{1, 3},{1, 4},{2, 4},{1, 2, 3},{2, 3, 4}, D C (1, −1)
(a) 256 (b) 512 (c) 1024 (d) 2048
{1, 2, 4},{1, 3, 4}
Ê (a) (1 + x) = C 0 x + C1 x + ....
9 9 9 9 8
The superset of the set { 3} in the given

3x
+
proper subset are + 9C 5 x 4 + 9C 6 x 3 + 9C 7 x 2 + 9C 8 x1 + 9C 9

2y
=
{ 3},{2, 3},{ 3, 4},{1, 3},

5
∴The sum of last 5 coefficients in the
{1, 2, 3}, {2, 3, 4}, {1, 3, 4} expansion of (1 + x )9 is
Hence, number of proper subset of the A B
superset of the set { 3} is 7. Here, we see that point (1, − 1) does not
9
C 5 + 9C 6 + 9C 7 + 9C 8 + 9C 9 satisfy the diagonal equation
48. Let p , q and r be three distinct 9× 8×7 × 6 9× 8×7 3 x + 2 y = 5. Therefore we consider
= +
positive real numbers. If 4× 3×2 ×1 3×2 ×1 point (1, − 1) of the other vertex of
 p q r 9 ×8
diagonal.
+
+ 9+1
D = q r p, then which one of 2 ×1
We know that, diagonal of a square are
  perpendicular to each other.
 r p q = 126 + 84 + 36 + 9 + 1 = 256
Let perpendicular equation of
the following is correct? 50. Consider the following in respect 3 x + 2 y = 5 is 2 x − 3 y + c = 0…(i)
(a) D < 0 (b) D ≤ 0 of a non-singular matrix of order 3. This equation passes through the vertex
(c) D > 0 (d) D ≥ 0 C (1, − 1)
1. A (adj A ) = (adj A ) A
Ê (a) We have, ∴ 2(1) − 3( −1) + c = 0
2. | adj A | = | A |
p q r
Which of the above statements ⇒ 2 + 3+c=0
D= q r p is/are correct? ⇒ c = −5
r p q (a) 1 Only (b) 2 Only Put c = − 5 in eq. (i), we get
(c) Both 1 and 2 (d) Neither 1 nor 2 2 x − 3y − 5 = 0 ⇒2 x − 3y = 5
Applying C1 → C1 + C 2 + C 3
p+ q + r q r Ê (a) 1. By using the property of adjoint of 53. Let P ( x , y ) be any point on the
square matrix.
D= p+ q + r r p ellipse 25x 2 + 16y 2 = 400. If Q (0, 3)
A(adj A ) = (adj A ) A = A I
p+ q + r p q
Hence, Statement 1 is correct. and R (0, − 3) are two points, then
1 q r 2. By using the property of adjoint of what is ( PQ + PR ) equal to?
= ( p + q + r) 1 r p square matrix, if matrix A is of order n, (a) 12 (b) 10 (c) 8 (d) 6
1 p q then
adj A = A
n−1 Ê (b) Given, equation can be rewritten as
Applying R 2 → R 2 − R1 and x2 y2
R 3 → R 3 − R1 + =1
Here, order of given matrix A is 3. 16 25
8 NDA/NA Solved Paper 2020 (I & II)

Here, a 2 = 16, b 2 = 25, b > a In right angled ∆OAP,


1 58. If the foot of the perpendicular
t drawn from the point (0, k ) to the
16 a2 AP 2
Eccentricity, e = 1 − 2 = 1 − sinθ = ⇒ sinθ =
b 25 OP OP line 3x − 4y − 5 = 0 is (3, 1), then
9 3 ⇒ t = 2OP sinθ …(i) what is the value of k?
= = (a) 3 (b) 4 (c) 5 (d) 6
OA
25 5 and cosθ =
OP Ê (c) Now equation of line AP, which is
Foci = ( 0, ± be ) =  0, ± 5 ×
3
 = ( 0, ± 3) 1 2 perpendicular to the given line
 5 t
3 x − 4 y − 5 = 0 is
∴The given points ( 0, 3) and R( 0, − 3) are ⇒ cosθ = 4
OP 4x + 3y + c = 0 …(i)
the foci of given ellipse.
1 P (0,p)
PQ + PR = 2 b = 2 × 5 = 10 (2 × OP sinθ)2
⇒ cos θ = 4
54. If the circumcentre of the triangle OP
[From Eq. (i)]
formed by the lines x + 2 = 0,
⇒ cos θ = ΟP sin2 θ 3x–4y–5=0
y + 2 = 0 and kx + y + 2 = 0 is A (3,1)
( − 1, − 1), then what is the value of k? ⇒ OP = cos θ cosec θ 2

Since, it passes through ( 3, 1)


(a) − 1 (b) − 2 (c) 1 (d) 2 56. Under which condition, are the ∴ 4 × 3 + 3 × 1+ c = 0
Ê (c) Here, we see that triangle formed by points (a, b ), (c , d ) and (a − c , b − d ) ⇒ c = − 15
given lines is right angled triangle.
y collinear? Put c = − 15 in Eq. (i), we get
(a) ab = cd (b) ac = bd
C
(c) ad = bc (d) abc = d 4 x + 3 y − 15 = 0
x=–2 kx+y+2=0 Also, this equation passes through the
Ê (c) The condition for the points point P ( 0, k ).
x′ x ( a, b ), (c, d ) and ( a − c, b − d ) are
collinear, is ∴ 4( 0) + 3( k ) − 15 = 0
y=–2 15
A B a b 1 ⇒ k= =5
c−a d 1 =0 3
y′
Therefore, circumcentre of right angled −c b −d 1 59. What is the obtuse angle between
triangle lies on the hypotenuse of Apply R 2 → R 2 − R1 and R 3 → R 3 − R1 the lines whose slopes are 2 − 3
triangle. a b 1 and 2 + 3?
∴Point ( −1, − 1) satisfy the equation of c−a d − b 0 = 0 (a) 105° (b) 120° (c) 135° (d) 150°
line kx + y + 2 = 0
−c −d 0
⇒ k( −1) − 1 + 2 = 0 ⇒ k = 1 Ê (b) Given m1 = 2 − 3 and m2 = 2 + 3
Expanding along C 3, we get
55. In the parabola, y 2 = x , what is the 1[(c − a ) ( −d ) + c(d − b )] = 0 ∴The obtuse angle between the line
length of the chord passing through ⇒ −cd + ad + cd − cb = 0 having slopes m1 and m2 is
the vertex and inclined to the ⇒ ad = cb m − m2
tanθ = 1
X -axis at an angle θ? 1 + m1m2
(a) sin θ ⋅ sec 2 θ (b) cos θ ⋅ cosec 2 θ
57. Let ABC be a triangle. If D (2, 5) and
2 − 3 − (2 + 3)
(c) cot θ ⋅ sec 2 θ (d) 2 tan θ ⋅ cosec 2 θ E (5, 9 ) are the mid-points of the =
sides AB and AC respectively, then 1 + (2 − 3 ) (2 + 3)
Ê (b) We know2 that, any point on the
−2 3 −2 3
parabola y = 4ax is P( at , 2 at )
2 what is the length of the side BC? = =
(a) 8 (b) 10 (c) 12 (d) 14 1 + ( 4 − 3) 2
We have equation of parabola is y = x 2

Ê (b) Now, length of DE = − 3 = − tan 60°= tan(180° − 60° )


1
Here, a = = ( 5 − 2 )2 + ( 9 − 5)2 = ( 3)2 + ( 4)2 θ = 120°
4
∴Any point on the parabola y 2 = x is = 9 + 16 = 25 = 5 sq units. 60. If 3x − 4y − 5 = 0 and
P  t 2, t 
1 1 We know that, if D and E are the mid 3x − 4y + 15 = 0 are the equations
4 2  points of AB and AC, then length of DE of a pair of opposite sides of a
y is half of BC. square, then what is the area of the
1 2 1
P 4 t ,2 t A
square?
(a) 4 sq units (b) 9 sq units
1
t (2,5) D E (5,9)
(c) 16 sq units (d) 25 sq units
2
q
x′ x Ê (c) Now, distance between two parallel
O 1 2 A 15 + 5 20
4
t lines = =
( 3)2 + ( −4)2 9 + 16
B C
∴Length of BC = 2 × DE [Q Distance between two
y 2 =x c 2 − c1
y′ = 2 × 5 = 10 sq units. parallel lines = ]
a2 + b 2
NDA/NA Solved Paper 2020 (I & II) 9

=
20
=
20
=4 ∴The coordinate of Q is (2, 0, 0). 66. If a$ is a unit vector in the xy-plane
25 5 ∴The perpendicular distance PQ making an angle 30° with the positive
3x–4y–5=0
D C = (2 − 2 )2 + ( 3 − 0)2 + ( 4 − 0)2 X-axis, then what is a$ equal to?
= 0 + 9 + 16 = 25 = 5 units 3 $i + $j 3$i − $j
(a) (b)
2 2
63. If a line has direction ratios $i + 3 $j $i − 3 $j
< a + b, b + c , c + a >, then what is (c) (d)
2 2
the sum of the squares of its
A 3x–4y+15=0 B Ê (a) Given a$ makes an angle of 30° with
direction cosines? X-axis. So, angle between a$ and $i is 30°.
It is clear that, side of square = distance
(a) ( a + b + c )2 (b) 2 ( a + b + c ) y(j)
between two lines.
(c) 3 (d) 1
∴ Side = 4
∴Area of square = (Side)2 = ( 4)2 Ê (d) Given, direction ratio of a given line
is < a + b, b + c, c + a > â
= 16 sq units. ∴ Direction cosine of given line is 60°
a+ b
61. What is the length of the diameter < , 30°
of the sphere whose centre is at ( a + b )2 + ( b + c )2 + (c + a )2 x(i)
O
(1, − 2, 3) and which touches the b+c
, Q a b = |a||b| cosθ
plane 6x − 3y + 2z − 4 = 0? ( a + b )2 + ( b + c )2 + (c + a )2 ∴ a$ ⋅ $i =|a$|| $i| cos 30°
(a) 1 unit (b) 2 units c+ a
> 3
(c) 3 units (d) 4 units ⇒ a$ $i = 1 × 1 ×
( a + b ) + ( b + c )2 + (c + a )2
2
2
Ê (d) Now, radius of sphere Now, sum of the square of direction [Q a$ is a unit vector|a$| = 1]
= Length of perpendicular drawn from
cosine Similarly, a$ makes an angle of 60° with
centre to the tangent 2
  Y-axis. So, angle between a$ and $j is 60°.
6(1) − 3( −2 ) + 2( 3) − 4 a+ b
= = 
∴ a$ . $j =|a$|| $j|cos 60°
( 6)2 + ( −3)2 + (2 )2  ( a + b )2 + ( b + c )2 + (c + a )2 
 
1 1
6+ 6+ 6−4 2 = 1× 1× =
= =
14
=
14
=2  b+c  2 2
36 + 9 + 4 49 7 +  
 ( a + b )2 + ( b + c )2 + (c + a )2  3$ 1$ 3$i + $j
  ∴ a$ = i + j=
∴Diameter of sphere 2 2 2
2
= 2 × radius of sphere  c+ a 
+   67. Let A be a point in space such that
= 2 × 2 = 4 units  ( a + b )2 + ( b + c )2 + (c + a )2 
  |OA | = 12 , where O is the origin. If
62. What is the perpendicular distance ( a + b )2 + ( b + c )2 + (c + a )2 OA is inclined at angles 45° and 60°
=
from the point (2, 3, 4) to the line ( a + b )2 + ( b + c )2 + (c + a )2 with X -axis andY -axis respectively,
x −0 y −0 z −0 =1
then what is OA equal to?
= = ? (a) 6$i + 6$j ± 2k$
1 0 0
(a) 6 units (b) 5 units 64. Into how many compartments do (b) 6$i + 6 2 $j ± 6 k$
(c) 3 units (d) 2 units the coordinate planes divide the (c) 6 2 $i + 6$j ± 6 k$
space? (d) 3 2 $i + 3$j ± 6k$
Ê (b) Given equation of line is
x−0 y−0 z−0 (a) 2 (b) 4
= = = λ(say) (c) 8 (d) 16 Ê (c) Let OA = x i + y j + zk
$ $ $
1 0 0
Ê (c) The coordinate planes divides the Where x = 12 cos 45° and y = 12 cos 60°
Any point on the line is Q( λ, 0, 0)
P (2,3,4) spaces into 8 compartment. ∴OA = 12 cos 45 ° $i + 12 cos 60° $j + zk$
65. What is the equation of the plane = 12 ×
1 $ 1
i + 12 × $j + zk$
which cuts an intercept 5 units on 2 2
12 $
the Z-axis and is parallel to = i + 6$j + zk$ …(i)
2
A Q B XY-plane? 144
(a) x + y = 5 (b) z = 5 ⇒ |OA| = + 36 + z2
x–0 y–0 z–0 2
1
=
0
=
0 (c) z = 0 (d) x + y + z = 5
⇒ 12 = 108 + z2
Now, Dr’s of PQ are (2 − λ, 3, 4) Ê (b) Plane parallel to xy plane is z = k…(i)
Since, it intercept 5 units on Z-axis. On squaring both sides, we get
Since, PQ is perpendicular to AB.
∴Point ( 0, 0, 5) satisfy Eq. (i), we get 144 = 108 + z2
∴ (2 − λ )1 + 3 × 0 + 4 × 0 = 0
[Q a1 a2 + b1b 2 + c1c 2 = 0] 5=k ⇒ z2 = 36 ⇒ z = ± 6
⇒ 2 −λ=0 Put k = 5 in Eq. (i), we get Put z = ± 6 in Eq. (i), we get
⇒ λ =2 z=5 OA = 6 2 $i + 6$j ± 6k$
10 NDA/NA Solved Paper 2020 (I & II)

x + x2 + x3 − 3
68. Two adjacent sides of a 71. What is lim equal Ê (b) We have,
parallelogram are 2$i − 4 $j + 5k$ and x→1 x −1 
2/ 3
to?  dy  
2
dy
i$ − 2$j − 3k$ . What is the magnitude (a) 1 (b) 2
(c) 3 (d) 6 k
dx
= ∫ 1 +  dx   dx
 
of dot product of vectors which x + x2 + x3 − 3
Ê (d) We have, lim On differentiating both sides w.r.t., x, we
represent its diagonals? x→1 x−1
get
(a) 21 (b) 25 (c) 31 (d) 36  0 form 2/ 3
  d 2y  dy 
2
Ê (c) We have, 0  k = 1 +   
dx 2
  dx  
D C 1 + 2 x + 3 x2 
= lim
x→1 1 On cubing both sides, we get
[by using L’ Hospital’s rule] 3 2
 d 2y  dy 
2

= 1 + 2(1) + 3(1)2 k 3  2  = 1 +   
 dx    dx  

= 1+ 2 + 3 = 6
A B d 2y
Here, highest order derivative is 2
72. The radius of a circle is increasing dx 2
AB = 2 $i − 4$j + 5k$
at the rate of 0.7 cm/sec. What is and whose degree is 3.
and BC = $i − 2 $j − 3k$ the rate of increase of its sin x log (1 − x )
Now, AC = AB + BC 75. What is lim equal
circumference? x→0 x2
= (2 $i − 4$j + 5k$ ) + ( $i − 2 $j − 3k$ ) (a) 4.4 cm/sec (b) 8.4 cm/sec
to?
1
= 3$i − 6$j + 2k$ (c) 8.8 cm/sec (d) 15.4 cm/sec (a) − 1 (b) Zero (c) − e (d) −
dr e
and BD = AD − AB = BC − AB Ê (a) We have, = 07 . cm/sec sin x log(1 − x )
= ( $i − 2 $j − 3k$ ) − (2 $i − 4$j + 5k$ )
dt Ê (a) xlim
→0 x2
Now, circumference of circle, C = 2 πr
= − $i + 2 $j − 8k$ sin x 1− x
On differentiating w.r.t. t, we get = lim × lim log
x→ 0
x x→ 0 x
Now, dot product of diagonals, dC dr
= 2π = 1 × ( −1) = − 1
AC.BD = ( 3$i − 6$j + 2k$ ).( − $i + 2 $j − 8k$ ) dt dt
22 Q lim sinθ = 1 and lim log (1 − x ) = −1
= − 3 − 12 − 16 = − 31 =2 × × 07. cm/sec  θ→ 0 θ x→ 0 
7 x
∴Magnitude of dot product of diagonals.
= | AC ⋅ BD| = |−31| = 31 = 4.4 cm/sec 76. If f ( x ) = 3x 2 − 5x + p and f (0) and
x4 − 1 x3 − k3
69. If |a × b|2 + | a ⋅ b |2 = 144 and 73. If lim = lim 2 , f (1) are opposite in sign, then
x→1 x − 1 x →k x − k2
which of the following is correct?
| a | = 4, then what is | b | equal to? where k ≠ 0, then what is the value (a) − 2 < p < 0 (b) − 2 < p < 2
(a) 3 (b) 4 (c) 6 (d) 8 (c) 0 < p < 2 (d) 3 < p < 5
2 2 of k?
Ê (a) We have, a × b + a ⋅ b = 144 2 4 8 Ê (c) We have,
(a) (b) (c) (d) 4
⇒ |a|2|b 2|sin2 θ + |a|2|b|2 cos 2 θ = 144 3 3 3 f( x ) = 3 x 2 − 5 x + p
2 2 x4 − 1 x3 − k 3 Now, f( 0) = 3( 0)2 − 5( 0) + p = p
⇒ a b (sin2 θ + cos 2 θ) = 144 Ê (c) lim = lim 2
x→1 x − 1 x→ k x − k 2
2 and f(1) = 3(1)2 − 5(1) + p
⇒ ( 4)2 × b (1) = 144
4 −1 ( x − k ) ( x 2 + k 2 + kx )
⇒ 4(1) = lim = 3− 5+ p= p−2
[Qsin2 θ + cos 2 θ = 1] x→ k ( x − k) ( x + k)
Since, f( 0) and f(1) are opposite signs.
144
2  xn − an 
⇒ b = =9 = na n − 1  ∴ f( 0) f(1) < 0
16 Q xlim
→a x − a
  ⇒ p × (p − 2) < 0
b =3
x 2 + k 2 + kx 0< p< 2
⇒ 4 = lim
70. If the vectors a = 2$i − 3$j + k$ , x→ k ( x + k)
77. If e θ φ = c + 4θφ, where c is an
b = $i + 2$j − 3k$ and c = $j + pk$ are ⇒ 4=
k2 + k2 + k2
arbitrary constant and φ is a
coplanar, then what is the value of p? k+ k
function of θ, then what is φ dθ
(a) 1 (b) − 1 (c) 5 (d) − 5 3k 2 4×2 8
⇒ 4= ⇒ k= = equal to?
Ê (b) The condition of three vectors to be 2k 3 3
(a) θ dφ (b) − θ dφ
coplanar is a . (b × c ) = 0
74. The order and degree of the (c) 4θ dφ (d) − 4θ dφ
2 −3 1
differential equation Ê (b) We have,
⇒ 1 2 −3 = 0
e θφ = c + 4θφ
2
0 1 p   dy 
2 3
= ∫ 1 +
dy
k    dx are On differentiating w.r.t θ, we get
⇒ 2(2 p + 3) + 3( p + 0) + 11
( − 0) = 0
  dx   dφ dφ
 e θφ  θ + φ = 0 + 4 θ + φ
dx
⇒ 4p + 6 + 3p + 1 = 0  dθ   dθ 
⇒ 7p + 7 = 0
respectively

p = −1
(a) 1 and 1 (b) 2 and 3 ⇒ (θe θφ − 4θ) = 4φ − φe θφ
(c) 2 and 4 (d) 1 and 4 dθ
NDA/NA Solved Paper 2020 (I & II) 11

dφ φ ( 4 − e θφ ) 80. Let y = 3x 2 + 2 . If x changes from By using integration by parts, we get


⇒ =
dθ −θ ( 4 − e θφ ) I1 = x ∫ cos xdx − ∫  ( x )∫ cos xdx  dx
d
10 to 10.1, then what is the total  dx 
⇒ φ dθ = − θdφ change in y?
= x sin x − ∫ 1 × sin xdx
78. If p ( x ) = ( 4e )2 x , then what is (a) 4.71 (b) 5.23 (c) 6.03 (d) 8.01
= x sin x + cos x
Ê (c) Change in y,
∫ p ( x ) dx equal to? δy = f(101
. ) − f(10)
and I2 = ∫ sin x ⋅ cos xdx
p( x ) p( x ) Put sin x = t ⇒ cos xdx = dt
(a) + C (b) +C = 3 × (101. )2 + 2 − [3(10)2 + 2 ]
1 + 2 ln 2 2 (1 + 2 ln 2 ) t 2 sin2 x
∴ I2 = ∫ tdt = = +c
2 p( x ) p( x ) = 308.03 − 302 2 2
(c) +C (d) +C
1 + ln 4 1 + ln 2 = 6.03 [Q put t = sin x]
Ê (b) We have, 81. If f ( x ) = sin x , where x ∈ R, is to sin2 x
∴ I = x sin x + cos x + +c
p( x ) = ( 4e ) 2x
x 2
be continuous at x = 0, then the
∴ ∫ p( x )dx = ∫ (4e ) dx 84. What is the domain of the function
2x

value of the function at x = 0 f ( x ) = cos −1 ( x − 2)?


= ∫ 4I
2x 2x
.e dx (a) should be 0
II
(a) [− 1, 1] (b) [1, 3] (c) [0, 5] (d) [− 2, 1]
(b) should be 1
By using integration by parts, we get −1
(c) should be 2 Ê (b ) We know domain of cos x is [−1, 1].
∫ p( x)dx = 4 ∫ e dx
2x 2x
(d) cannot be determined ∴ Domain of cos −1( x − 2 ) is
sin x
− ∫  ( 4 2x ) ∫ e 2xdx dx Ê (b) We have, f( x) =
d
−1 ≤ x − 2 ≤ 1
 dx  x
⇒ −1 + 2 ≤ x ≤ 1 + 2
Since, f( x ) is continuous at x = 0
4 2x e 2x e 2x ⇒ 1≤ x ≤ 3
= − ∫ 4 2x (log 4)2 × dx ∴ f( 0) = lim f( x )
2 2 x→ 0 Hence, domain of the function is [1, 3].
sin x
4 2x .e 2x = lim =1
= − log 4 ∫ 4 2x .e 2x dx x→ 0 x 85. What is the area of the region
2
4 2x e 2x 82. The solution of the differential enclosed between the curve y 2 = 2x
⇒ ∫ p( x)dx = 2 − log 4 ∫ p( x)dx equation dy = (1 + y 2 ) dx is and the straight line y = x ?
4 2x .e 2x 1 2
(a) y = tan x + c (a) (b) 1 (c) (d) 2
⇒ (1 + log 4) ∫ p( x )dx =
2 (b) y = tan ( x + c ) 2 3
(c) tan−1 ( y + c ) = x Ê (c) Given curve y = 2 x is a parabola
2
4 2x .e 2x
⇒ ∫ p( x)dx = 2(1 + log(2 )2 ) + C (d) tan−1 ( y + c ) = 2 x having vertex ( 0, 0) and open right side.
Ê (b) We have, dy = (1 + y )dx
2
p( x ) Given straight line y = x, which passes
= +C dy through origin.
2(1 + 2 log 2 ) ⇒ = dx
1 + y2 The point of intersection of line y = x and
79. What is the value of curve is ( x )2 = 2 x
On integrating both sides, we get
π
dy ⇒ x( x − 2 ) = 0 ⇒ x = 0, 2
4 ∫ 1 + y2 = ∫ dx + c Put x = 0, then y = 0
∫ ( tan x + tan x ) dx?
3
Put x = 2, then y 2 = 2 × 2
0 ⇒ tan−1 y = x + c
1 1 y = tan( x + c ) y=±2
(a) (b)
4 2 ∴The point of intersection of given curve
(c) 1 (d) 2 83. What is ∫ (e log x + sin x ) cos x dx is O( 0, 0) and A(2, 2 ).
π /4 y y=x
Ê (b) Let I = ∫0 (tan3 x + tan x ) dx equal to? A (2,2)
π /4 sin2 x B
(a) sin x + x cos x + +c
= ∫0 tan x. (tan x + 1) dx
2
2
π /4 sin2 x
= ∫0 tan x sec 2 x dx (b) sin x − x cos x +
2
+c x′ x
sin2 x O
[Q sec x − tan x = 1]
2 2
(c) x sin x + cos x + +c
Put tan x = t 2
sin2 x y2=2x
⇒ sec 2 xdx = dt (d) x sin x − x cos x + +c
2 y′
π
∴ Upper limit, t = tan =1
Ê (c) Let I = ∫ (e + sin x ) cos x dx
log x
4 Now, area of shaded region OABO
Lower limit, t = tan 0 = 0 = ∫ x cos x dx + ∫ sin x cos x dx = ∫ ( y2 − y1 )dx
2
= I1 + I2 = ∫0( 2 x − x ) dx
1
1 t 2 
∴ ∫0t dt =  2 
I=
Where, I1 = ∫ x cos xdx 2
 0  x 3/ 2 x2 
and I2 = ∫ sin x.cos x dx = 2 − 
1 2 1
= (1 − 0 ) =
2
Now, I1 = ∫ x cos xdx  3/2 2 0
2 2 I II
12 NDA/NA Solved Paper 2020 (I & II)

 2 2 3/ 2 x 2 
2
xn − 1 Integration I become a rational, if
= x − 
= ∫ x n( x n + 1)dx 8 − 4k = 0
 3 2 0
[Multiply numerator and k=2
 2 2 3/ 2 2 2 
= 2 − − 0 − 0 denominator by x n − 1] 91. Consider the following statements
 3 2  Put x n + 1 = t
8 2 for f ( x ) = e − | x | :
= −2 = nx n − 1dx = dt
3 3 1. The function is continuous at
∴ I =∫
dt x = 0.
86. If f ( x ) = 2x − x 2 , then what is the n(t − 1)t 2. The function is differentiable at
value of f ( x + 2) + f ( x − 2) when 1 1 1  x = 0.
n  ∫ t − 1 ∫ t dt 
x = 0? = dt −
 Which of the above statements
(a) − 8 (b) − 4 is/are correct?
1
(c) 8 (d) 4 [log t − 1 − log t ] + c (a) 1 Only (b) 2 Only
n
Ê (a) We have, f( x) = 2 x − x (c) Both 1 and 2 (d) Neither 1 nor 2
2
1 t −1 −x
Now, f( x + 2 ) + f( x − 2 ) = [2( x + 2 ) = log Ê (a) We have, f( x) = e
n t
− ( x + 2 )2 ] + [2( x − 2 ) − ( x − 2 )2 ] e − x , x≥ 0
Q log m − log n = log m  = x
= [2 x + 4 − ( x + 4 + 4 x )]
2
 n  e , x< 0
y
+ [2 x − 4 − ( x 2 + 4 − 4 x )] 1  x n + 1 − 1
= log   +c
 x +1 
n
= [ − x 2 − 2 x ] + [ − x 2 + 6 x − 8] n
y=ex y=e–x
= − 2 x2 + 4 x − 8 [put t = x n + 1]
x′ x
At x = 0, 1  xn 
= log  n  +c
f( x + 2 ) + f( x − 2 ) = − 2( 0)2 + 4( 0) − 8 n  x + 1
= −0+ 0− 8 89. What is the minimum value of y′
= −8 | x − 1 |, where x ∈ R?
It is clear from the graph that f( x ) is
87. If x my n = am + n , then what is
dy (a) 0 (b) 1
(c) 2 (d) − 1 continuous everywhere. But f( x ) is not
dx
equal to? differentiable at x = 0, due to sharp
 x − 1, x≥1
(a)
my
(b) −
my
Ê (a) Let y = x − 1 =  −( x − 1) , x < 1 shape.
nx nx  Hence, Statement 1 is correct but
mx ny y
(c) (d) − y=–(x–1) y=x–1 Statement 2 is incorrect.
ny mx
m+n 92. What is the maximum value of
Ê (b) We have, x y = a
m n
sin x ⋅ cos x ?
On taking log both sides, we get x′ x 1
(a) 2 (b) 1 (c) (d) 2 2
log x m + log y n = ( m + n)log a 2
mlog x + nlog y = ( m + n)log a Ê (c) Let f( x) = sin x cos x
On differentiating both sides w.r.t. ‘x’ we y′ 1
= × 2 sin x.cos x
get 2
m n dy The graph of a given function is shown
+ =0 [Multiply numerator and
x y dx above. From the graph it is clear that the
denominator by 2]
minimum value of given function is 0.
dy − my 1
⇒ = = sin2 x
dx nx 90. What is the value of k such that 2
88. What is ∫
dx
equal to? 3x 2 + 8 − 4k We know that maximum value of sin2 x
integration of with
x ( x + 1)
n
x is 1.
1 1
1  xn  respect to x, may be a rational ∴ f( x )max. = × 1 =
(a) ln  n  +c 2 2
n  x + 1 function?
3x + 3− x − 2
(a) 0 (b) 1 (c) 2 (d) − 2 93. What is lim equal to?
x→0
 xn +
1 (a) 0 x
(b) ln   +c 3 x 2 + 8 − 4k (b) − 1
 x 
n
Ê (c) Now, I = ∫ dx
x (c) 1
 x 
n
= ∫  3 x + − k  dx
(c) ln  n  +c 8 4 (d) Limit does not exist
x +1  x 
x 3x + 3− x − 2  0
1  x n + 1 Ê (a) xlim  form 

(d) ln   +c →0 x 0
n  xn  3 x2
= + 8 log x − 4k log x + C 3 x log 3 − 3 − x log 3 − 0
dx 2 lim
Ê (a) Let I = ∫ x( x n + 1) 3 x2
x→ 0 1
I= + ( 8 − 4k ) log x + C [by using L’ Hospital’s rule]
2
NDA/NA Solved Paper 2020 (I & II) 13

= 3 0 log 3 − 3 0 log 3 (a) 1 Only (b) 2 Only 99. What is the solution of the
= log 3 − log 3 (c) Both 1 and 2 (d) Neither 1 nor 2  dy 
differential equation ln   = x ?
=0 Ê (c) 1. We have f( x) = ln( x)  dx 
y
94. What is the derivative of tan −1 x (a) y = e x + c (b) y = e − x + c
y=ln(x)
(c) y = ln x + c (d) y = 2 ln x + c
with respect to cot −1 x?
(a) − 1 (b) 1 x′ x Ê (a) Given differential equation is
ln   = x ⇒
1 x dy dy
(c) 2 (d) = ex
x +1 x2 + 1  dx  dx
−1 −1 y′ ⇒ dy = e xdx
Ê (a) Let f( x) = tan x and g ( x) = cot x
It is clear from the graph that f( x ) is On integrating both sides, we get
On differentiating w.r.t ‘x’, we get
increasing in the interval ( 0, ∞ ).
1 1 y = ex + c
f ′( x) = and g ( x ) = − 2. We have f( x ) = tan( x )
1 + x2 1 + x2 y 100. Let l be the length and b be the
d 1 y=tan x
f( x ) breadth of a rectangle such that
1 + x2
∴ dx = = −1 l + b = k . What is the maximum
d 1
g ( x) − area of the rectangle?
dx 1 + x2 x′ x
k2 k2
(a) 2 k 2 (b) k 2 (c) (d)
95. The function u( x , y ) = c which 2 4
–π π
satisfies the differential equation 2 y′ 2 Ê (d) We have, l + b = k
x (dx − dy ) + y (dy − dx ) = 0, is It is clear from the graph that f( x ) is
Now, area of rectangle A = lb
(a) x 2 + y 2 = xy + c π π A = l( k − l )
(b) x 2 + y 2 = 2 xy + c increases in the interval  − ,  .
 2 2 On differentiating w.r.t, ‘l’, we get
(c) x 2 − y 2 = xy + c dA
(d) x 2 − y 2 = 2 xy + c Hence, both statement are correct. = ( k − l ) + l( 0 − 1) = k − 2 l
dl
Ê (b) We have, 98. Which one of the following is
For maximum, put
dA
=0
x (dx − dy) + y (dy − dx ) = 0 correct in respect of the graph of dl
⇒ xdx + ydy − ( xdy + ydx ) = 0 1 k
y= ? k − 2 l = 0, l = ,
⇒ xdx + ydy − d ( xy) = 0 x −1 2
On integrating both sides, we get (a) The domain is { x ∈ R | x ≠ 1} and the d 2A
Now, 2 = − 2 < 0
x2 y2 c range is the set of reals dl
+ − xy = (b) The domain is { x ∈ R | x ≠ 1}, the
2 2 2 k
range is { y ∈ R | y ≠ 0} and the Therefore A is maximum at l =
x 2 + y 2 = 2 xy + c graph intersects y-axis at ( 0, − 1)
2
(c) The domain is the set of reals and the ∴Value of maximum area,
96. What is the minimum value of k2
A =  k −  =   =
range is the singleton set {0} k k k k
 π (d) The domain is { x ∈ R | x ≠ 1} and the 2  2 2 2
3 cos  A +  where A ∈ R ? 4
 3 range is the set of points on they
(a) − 3 (b) − 1
y-axis 101. The numbers 4 and 9 have
1 frequencies x and ( x − 1)
(c) 0 (d) 3 Ê (b) We have, y = …(i)
π x−1 respectively. If their arithmetic

Ê (a) Let f( x) = 3cos  A +  The domain of given graph is mean is 6, then what is the value of
3
{ x ∈ R : x ≠ 1} x?
We know that, the minimum value of
π 1 (a) 2 (b) 3 (c) 4 (d) 5
cos  A +  is −1. Consider y =
x−1
 3 Ê (b) Given, arithmetic mean = 6
∴Minimum value of f( x ) = 3 × ( −1) 1 4 × x + 9 × ( x − 1)
⇒ x − 1= ∴ =6
y x + ( x − 1)
= −3
1 1+y
97. Consider the following statements: ⇒ x = + 1= ⇒ 4 x + 9 x − 9 = 6(2 x − 1)
y y
1. The function f ( x ) = ln x ⇒ 13 x − 9 = 12 x − 6
Here, we see that x is not defined for ⇒ x= 3
increases in the interval (0, ∞ ). y = 0.
2. The function f ( x ) = tan x ∴Range of given curve is { y ∈ R : y ≠ 0} 102. If three dice are rolled under the
increases in the interval Also graph intersect Y −axis, put x = 0 in condition that no two dice show the
 π π Eq. (i), we get same face, then what is the
− , .
 2 2 y=
1
= −1
probability that one of the faces is
0−1 having the number 6?
Which of the above statements 5 5 1 5
is/are correct? Hence, graph intersect Y-axis at point (a) (b) (c) (d)
(0, −1). 6 9 2 12
14 NDA/NA Solved Paper 2020 (I & II)

Ê (c) Let S = Total sample space 104. The sum of deviations of n number Ê (c)
= 6 3 = 216 of observations measured from 2.5 Marks Number of students Difference
E = Event of getting no two face have is 50. The sum of deviations of the Physics Mathematics P−M
same number. i.e. all three faces have same set of observations measured (P) (M)
distinct numbers. from 3.5 is − 50. What is the value 10-20 8 10 8 − 10 = 2
∴ n( E ) = 6 × 5 × 4 = 120 of n? 20-30 11 21 11 − 21 = 10
and F = Event of getting one of the face (a) 50 (b) 60 (c) 80 (d) 100 30-40 30 38 30 − 38 = 8
have 6 number. Ê (d) We have, 40-50 26 15 26 − 15 = 11
n( F ) = (1 × 5 × 4) × 3 = 60 Σ( xi − 2.5) = 50 50-60 15 10 15 − 10 = 5
and n( E ∩ F ) = 60 and Σ( xi − 3.5) = −50 60-70 10 6 10 − 6 = 4
P( F ∩ E )
P   = ⇒ Σxi − (2.5)n = 50
F …(i)

 E P( E ) and Σxi − ( 3.5)n = − 50 …(ii)
The largest difference between number
of students under Physics and
n( F ∩ E ) / n(S )
= On subtracting Eq. (ii) from Eq. (i), we Mathematics is 11, which lies in the
n( E ) / n(S ) get n = 100 interval 40-50.
60 / 216 60 1
= = = 105. A data set of n observations has 107. Consider the following statements:
120 / 216 120 2
mean 2M, while another data set of 1. Modal value of the marks in
5 1
103. If P ( A ∪ B ) = , P ( A ∩ B ) = and 2n observations has mean M. What Physics lies in the interval 30-40.
1 6 3 is the mean of the combined data
P ( not A ) = , then which one of 2. Median of the marks in Physics
2 sets?
3M 2M 4M
is less than of marks in
the following is not correct? (a) M (b) (c) (d) Mathematics.
2 2 3 3
(a) P ( B) = Which of the above statements
3 Ê (d) Given, x1 = 2 M is/are correct?
(b) P ( A ∩ B) = P ( A )P( B)
x2 = M (a) 1 Only
(c) P ( A ∪ B) > P( A ) + P( B) (b) 2 Only
(d) P (not A and not B) and n1 = n and n2 = 2 n
(c) Both 1 and 2
= P (not A ) P(not B) ∴Mean of the combined data (d) Neither 1 nor 2
5 n x + n2 x2
Ê (c) We have, P( A ∪ B) = , = 1 1 Ê (a)
6 n1 + n2
1 Marks Number of Cumulative
P( A ∩ B) = n × 2M + 2n × M
3 = students Frequencies
n + 2n
1 Physics Mathemat Physics Mathemat
and P(not A ) = 4nM 4
2 = = M (P) ics (M) ( cf1 ) ics ( cf2 )
3n 3 10-20 8 10 8 10
1 1
⇒ P( A ) = 1 − =
2 2 20-30 11 21 19 31
Directions (Q. Nos. 106-108) Read the
(a) Q P( A ∪ B) = P( A ) + P( B) − P( A ∩ B) following information and answer the 30-40 30 38 49 69
5 1 1
⇒ = + P( B) − three items that follow. 40-50 26 15 75 84
6 2 3 50-60 15 10 90 94
5 1 1 Marks Number of Students
⇒ P( B) = − + 60-70 10 6 100 100
6 2 3 Physics Mathematics
5−3+2 4 2 1. The maximum frequency in
= = = , 10-20 8 10
6 6 3 Physics is 30, which lies in the
20-30 11 21
Which is correct. interval 30-40.
30-40 30 38 Hence, modal value lies in the
1 2 1
(b) P( A ) ⋅ P( B) = × = 40-50 26 15
2 3 3 interval 30-40, which is correct
50-60 15 10 statement.
= P( A ∩ B) which is
correct. 60-70 10 6 2. Cumulative frequency of Physics is
1 2 7 N1 = 100
(c) Now, P( A ) + P( B) = + = 106. The difference between number of N 100
2 3 6 Now, 1 = = 50
5 students under Physics and 2 2
and P( A ∪ B) = N
6 Mathematics is largest for the Just greater than 1 is 75, which
∴P( A ∪ B)|> P( A ) + P( B), which is 2
interval
incorrect. lies in the interval 40-50.
(a) 20-30 (b) 30-40
Hence, option (c) is not correct. (c) 40-50 (d) 50-60 Here, cf1 = 49, f1 = 26,
h = 10, l = 40
NDA/NA Solved Paper 2020 (I & II) 15

Q Median of Physics, Md 1 Ê (b) Now, mean of given observations, 112. The arithmetic mean of 100
N1 − 6− 5− 4 − 1+ 1+
− cf1 4 observations is 40. Later, it was
=l+ 2 ×h + 5+ 6 found that an observation ‘53’ was
f1 x=
8 wrongly read as ‘83’. What is the
50 − 49
= 40 + × 10 =0 correct arithmetic mean?
26
10 Now, standard deviation, σ (a) 39.8 (b) 39.7 (c) 39.6 (d) 39.5
= 40 + = 40 + 0.38
26 Σ( xi − x )2 Ê (b) Correct arithmetic mean
= 100 × 40 − 83 + 53
= 40.38 n =
100
Cumulative frequency of Mathematics is ( − 6 − 0)2 + ( − 5 − 0)2 + ( − 4 − 0)2 4000 − 30 3970
N2 = 100 = = = 397
.
+ ( −1 − 0)2 + (1 − 0)2 + ( 4 − 0)2 100 100
N 100
Now, 2 = = 50 + ( 5 − 0)2 + ( 6 − 0)2 113. A husband and wife appear in an
2 2 =
N 8 interview for two vacancies for the
Just greater than 2 is 69, which lies in
2 6 + 5 + 4 + 1+ 1+ 4 + 5 + 6
= same post. The probability of the
the interval 30-40. 8 1
Here, cf2 = 31, f2 = 38, h = 10, 32 husband's selection is and that of
= = 4=2 7
l = 30 8 1
Median of Mathematics ( Md 2 ) the wife's selection is . If the
N2 110. If Σx i = 20, Σx i2 = 200 and n = 10 for 5
− cf2 events are independent, then the
=l+ 2 ×h an observed variable x, then what is
f2 the coefficient of variation? probability of which one of the
50 − 31 (a) 80 (b) 100 (c) 150
(d) 200 11
= 30 + × 10 following is ?
38 σ 35
Ê (d)Q Coefficient of variation, = × 100 (a) At least one of them will be selected
190 x
= 30 + (b) Only one of them will be selected
38 Now, standard deviation,
(c) None of them will be selected
= 30 + 5 = 35 σ =
1
n Σxi2 − ( Σxi )2 (d) Both of them will be selected
Here we see that median marks of n
Physics is greater than median marks of 1 Ê (a) Now, the probability of husband’s
= 10 × 200 − (20)2 selection,
Mathematics. Hence, statement 2 is 10
1
incorrect. 1 P( H ) =
= 2000 − 400 7
10
108. What is the mean of marks in 1 6
1 ⇒ P( H ) = 1 − =
Physics? = 1600 7 7
10
(a) 38.4 (b) 39.4 (c) 40.9 (d) 41.6 and the probability of wife’s selection,
40
= =4
Ê (c) 10 P(W ) =
1
Marks Mid Number of µ i = µ i fi Σx 5
and mean, x = i 1 4
value students in x i − 45 n ⇒ P(W ) = 1 − =
Physics (fi ) 10 20 5 5
= =2 Now, probability of none of them is
10-20 15 8 −3 −24 10
selected
20-30 25 11 −2 −22 4
∴Coefficient of variation = × 100 = 200
2 = P( H ∩ W ) = P( H ) P(W )
30-40 35 30 −1 −30
6 4 24
= × =
40-50 45 26 0 0 111. What is the probability that 7 5 35
50-60 55 15 1 15 February of a leap year selected at Now, probability that at least one of them
60-70 65 10 2 20 random, will have five Sundays? is selected
Total Σfi = 100 Σfi µ i = −41 (a)
1
(b)
1 = 1 − P(None of them is selected)
5 7 24 11
2 = 1− =
Here, A = 45, h = 10 (c) (d) 1 35 35
Σf µ 7
∴ Mean = A + i i × h 114. A dealer has a stock of 15 gold
Σfi Ê (b) In a leap year, February has 29 days.
In out of 29 days, there are four weeks coins out of which 6 are
( −41)
= 45 + × 10 and one day extra. In out of the four counterfeits. A person randomly
100
weeks, there exist four Sunday. In one picks 4 of the 15 gold coins. What is
= 45 − 41. = 40.9 extra days, it may be Sunday or the probability that all the coins
109. What is the standard deviation of the Monday or Tuesday or Wednesday or picked will be counterfeits?
Thursday or Friday or Saturday.
observations − 6, − 5, − 4 , − 1, 1, (a)
1
(b)
4
Probability of getting five sundays in the 91 91
4 , 5, 6 ? month of February =
1 6 15
(c) (d)
(a) 2 (b) 2 (c) 2 2 (d) 4 7 91 91
16 NDA/NA Solved Paper 2020 (I & II)

Ê (a) Total number of gold coins = 15 (c) Y = 112


. X −5 1. P( A ∪ B) = 1 − P( A) + P( A ∩ B)
(d) Y = 017
. X + 5.8 = 1 − 0.6 + 0.4
and number of counterfeits coins = 6
∴Probability of getting all four coins are Ê (b) We have, = 0.8, which is incorrect.
6 X = 65, Y = 67, P(B ∩ A )
C
counterfeits = 15 4 2. P(B / A ) =
C4 σ x = 2.5, σ y = 3.5 P( A )
6×5 and r( X , Y ) = 0.8 P( A ∪ B)
=
2 ×1 The equation of regression of Y on X is P( A )
= σ
15 × 14 × 13 × 12 Y − Y = r y (X − X) 1 − P( A ∪ B)
σx =
4× 3×2 ×1 1 − P( A )
6×5×4×3 3.5 1 − [P( A ) + P( B) − P( A ∩ B)]
⇒ Y − 67 = 0.8 × ( X − 65)
= 2.5 =
15 × 14 × 13 × 12 1 − P( A )
⇒ Y − 67 = 112
. ( X − 65)
1 1 1 − [0.6 + 0.5 − 0.4]
= = ⇒ Y = 112
. X − 72.8 + 67 =
7 × 13 91 ⇒ Y = 112
. X − 5.8 1 − 0.6
1 − [07. ] 0.3
115. A committee of 3 is to be formed 118. Consider a random variable X = =
0.4 0.4
from a group of 2 boys and 2 girls. which follows Binomial distribution = 075
.
What is the probability that the 1
committee consists of 2 boys and 1 with parameters n = 10 and p = . Which is incorrect.
5 Hence, both statements are incorrect.
girl? Then ,Y = 10 − X follows Binomial
2 1 120. Three cooks X, Y and Z bake a
(a) (b) distribution with parameters n and
3 4
3 1 p respectively given by special kind of cake and with
(c) (d) 1 2 respective probabilities 0.02, 0.03
4 2 (a) 5, (b) 5,
5 5 and 0.05, it fails to rise. In the
Ê (d) Total number of persons = 2 + 2 = 4 3 4
(c) 10, (d) 10, restaurant where they work, X
∴Probability of getting a committee of 3 5 5
bakes 50%, Y bakes 30% and Z
2
C 2 × 2C1 1 × 2 1 1
persons = 4
= = Ê (d) Given, B( X , n, p) = B( X , 10, ) bakes 20% of cakes. What is the
C3 4 2 5
proportion of failures caused by X?
∴ B( X , n, p) = nC X pX (1 − p)n − X 9 10 19 28
116. In a lottery of 10 tickets numbered 1 X 10 − X (a) (b) (c) (d)
= 10C X   1 − 1 
to 10, two tickets are drawn 1 29 29 29 29
 
 5  5 Ê (b) Let A = Bake X-type of cake
simultaneously. What is the
probability that both the tickets put p = 1  B = Bake Y-type of cake
 5 
drawn have prime numbers? and C = Bake Z-type of cake
1 1 2 1 10 − X X
C10 − X    1
4 50
(a) (b) (c) (d) = 10
  Then, P( A ) = 50% = ,
15 2 15 5  5  5 100
Ê (c) Total number of lottery tickets = 10 [Q nC r = nC n − r ] P( B) = 30% =
30
,
The prime number from 1 to 10 are 100
Y 10− Y
= 10C Y    
4 1 20
{2, 3, 5, 7} P(C ) = 20% = ,
 5  5 100
∴Probability of drawing two prime
[Q Y = 10 − X ] P   = 0.02 , P   = 0.03
number tickets X X
4×3 = B( Y , n, p)  A  B
2 ×1 and P   = 0.05
4 4 X
C2 Where, n = 10 and p =
= 10 =
C 2 10 × 9 5 C 
2 ×1 By using Baye’s theorem,
119. If A and B are two events such that
4×3 P ( A ) = 06. , P ( B ) = 05 P( A ) P  
2 . and X
= =
10 × 9 15  A  A
P ( A ∩ B ) = 0.4, then consider the P  =
 X
P( A ) P   + P( B) P  
X X
117. Let X and Y represent prices (in `) following statements
 A  B
of a commodity in Kolkata and 1. P ( A ∪ B ) = 09 .
+ P(C ) P  
X
Mumbai respectively. It is given 2. P ( B / A ) = 06 . C 
that X = 65, Y = 67, σ X = 25 ., Which of the above statements 50
× 0.02
σ y = 35
. and r ( X , Y ) = 08.
. What is is/are correct?
= 100
the equation of regression of Y on (a) 1 Only (b) 2 Only 50 30 20
× 0.02 + × 0.03 + × 0.05
X? (c) Both 1 and 2 (d) Neither 1 nor 2 100 100 100
(a) Y = 0175 X −5 Ê (d) We have, 100
. 1 10
. = = =
(b) Y = 112
. X − 5.8 P( A ) = 0.6, P( B) = 0.5 and P( A ∩ B) = 0.4 . + 0.90 + 100
100 . 2.9 29
NDA/NA Solved Paper 2020 (I & II) 17

PAPER II English Language and General Studies


Part A (English Language)
Directions (Q.Nos. 1-10) Each item in this section consists 10. The oceans carry a huge amount of heat from the tropics
of a sentence with an underlined word followed by four to the high latitudes.
words. Select the option that is opposite in meaning to the (a) significant (b) major (c) tiny (d) dormant
underlined word and your response accordingly.
Ê (c) The antonym of ‘huge’ is ‘tiny’.
1. He nodded absently throughout the meeting.
(a) capably (b) alertly (c) agitatedly (d) dreamily Directions (Q.Nos. 11-20) Each of the following items in
this section consists of a sentence(s), the parts of which have
Ê (b) The given word means carelessly. Hence, from the given been jumbled. These parts have been labelled as P, Q, R and
options the correct antonym is alertly which means carefully.
S. Given below each sentence are four sequences namely (a),
2. I fully believe that the cornerstone of good policy is an (b), (c) and (d). You are required to rearrange the jumbled
electorate that is educated on the national issues. part of the sentence and mark your response accordingly.
(a) cerebral (b) enlightened (c) ignorant (d) erudite
11. several years ago, course on climate change at Texas
Ê (c) The antonym of ‘educated’ is ignorant. P Q
3. For important medical decisions, even finding a doctor you A & M University Professor Andrew Dessler created an
trust is not enough. introductory for freshman and sophomores
(a) significant (b) trivial (c) basic (d) probable R S
(a) P R Q S (b) Q R P S (c) S Q R P (d) P Q R S
Ê (b) The antonym of ‘important’ is trivial meaning ‘of little value’.
4. Planets move in their orbits. Ê (a)
(a) push (b) rotate (c) stall (d) flow 12. I realize that solving the climate change problem
Ê (c) The antonym of ‘move’ is stall meaning ‘to stop’. P
than solving will be much harder
5. Temperature is a measure of internal energy of an object Q R
and is frequently expressed by physicists in units of
the ozone depletion problem
Kelvin.
S
(a) found (b) told (c) distributed (d) concealed
(a) P R Q S (b) Q R P S (c) S Q R P (d) P Q R S
Ê (d) The given word ‘expressed’ means to convey. Hence, its
antonym will be concealed meaning 'to hide'. Ê (a)
6. People argue about why Venus is so much warmer than 13. although the temperature of this layer of the
the Earth. P
(a) friendlier (b) colder (c) wilder (d) heavier when directly comparing the satellite
Q
Ê (b) The antonym of ‘warmer’ is ‘colder’. measurements of temperature
7. Scientists are concerned whether the oceans and land R
biosphere will take up as much carbon in the future as atmosphere should generally track the surface temperature,
they presently do. we must be careful
(a) worried (b) indifferent (c) curious (d) puzzled S
Ê (b) The antonym of ‘concerned’ is indifferent meaning ‘without any (a) P R Q S (b) P S Q R (c) S Q R P (d) P Q R S
concern’.
Ê (b)
8. The biggest debate among scientists today is about cloud 14. the element heats up, eventually reaching high temperatures
feedback. P Q
(a) contend (b) moot (c) wrangle (d) agreement
glowing like a dark orange that radiates the visible range,
Ê (d) The given word ‘debate’ means to argue. Hence, its antonym R S
will be agreement meaning ‘to agree to a thought’. (a) P Q S R (b) Q P S R (c) S Q R P (d) P R S Q
9. The Earth’s climate sensitivity is conventionally defined as Ê (d)
the equilibrium temperature increase caused by a doubling
15. two people argue about why Venus is so much warmer
of carbon dioxide.
P
(a) imperviousness (b) willingness
(c) responsiveness (d) closeness to the Sun, so it absorbs more solar energy.
Q
Ê (a) The given word ‘sensitivity’ means responsiveness. Its The second argues that it’s because Venus has a thick,
antonym will be imperviousness meaning ‘something that will not
be affected or will not respond’. R
18 NDA/NA Solved Paper 2020 (I & II)

greenhouse-gas rich atmosphere than the Earth. The first 22. He was known for his gentle disposition.
argues that it’s because Venus is closer (a) harmful (b) amiable (c) cunning (d) adjusting
S Ê (b) The given word gentle means kind and friendly. Therefore, from
(a) P S Q R (b) Q R P S (c) S Q R P (d) P Q R S the given options, 'amiable' meaning friendly is its near synonym.
Ê (a) 23. The harder we kick, the better the ball bounces back.
16. are now used by more than a thousand firms (a) recoils (b) deflates (c) inflates (d) ascends
P Ê (a) The given phrase ‘bounces back’ means ‘to come back to the
and are growing in popularity original position’. Therefore, from the given options, ‘recoil’ means
Q the same as bounces back.
in the United States and Europe gain sharing-plans 24. I plan and execute.
R S (a) desire (b) debate (c) accomplish (d) discard
(a) P R Q S (b) Q R P S (c) P R S Q (d) S P R Q
Ê (c) The given word ‘execute’ means to accomplish.
Ê (d) 25. He listened to me with equanimity.
17. adversity without succumbing (a) carelessly (b) excitedly
P (c) patiently (d) half-heartedly
to the clouds of doubt and jealousy Ê (c) The given phrase ‘with equanimity’ means carefully and
Q patiently.
between friends which is subjected 26. The whole thing was a futile exercise.
R (a) costly (b) pointless (c) indecisive (d) successful
to both prosperity and the essay ’Of Friendship’ by Francis
Ê (b) The given word 'futile' means 'useless'. Hence, from the given
Bacon celebrated the intimacy options, pointless is it's near synonym.
S
(a) P R Q S (b) Q R P S (c) S R P Q (d) P Q R S 27. He was too cunning for her.
(a) capable (b) fit (c) intelligent (d) dodgy
Ê (c)
Ê (d) The given word ‘cunning’ means deceitful. From the given
18. friends without which the world is options, dodgy also means deceitful and hence is its near
P synonym.
make friends and a person wills to want true 28. This decision is crucial for me.
Q (a) momentous (b) natural (c) primitive (d) inept
it is miserable solitude that compels a person to
Ê (a) The given word ‘crucial’ means important. From the given
R options, ‘Momentous’ is its near synonym.
nothing other than a place of wilderness
S 29. The media went into a frenzy about the case.
(a) silent (b) creative (c) berserk (d) wrong
(a) P R Q S (b) Q R P S (c) R Q P S (d) P Q R S
Ê (c) Ê (c) The given phrase ‘into a frenzy’ means ‘to go crazy or mad
position’. From the given options, ‘berserk’ means the same as the
19. takes the longest that never started the job to finish phrase.
P Q R S 30. He has been yearning to go home.
(a) P R Q S (b) R Q P S (c) S Q R P (d) P Q R S (a) declining (b) demanding (c) begging (d) hankering
Ê (b) Ê (d) The given word ‘yearning’ means 'intense longing for
20. to what you can create control, shift something. Therefore, from the given options, ‘hankering’ means
the same as the word.
P
your energy instead of worrying about what you cannot Directions (Q.Nos. 31-40) Each item in this section has a
Q R S sentence with a missing preposition. Select the correct
(a) P R S Q (b) Q R S P
preposition from the given option and mark your response
(c) S Q R P (d) R S Q P
accordingly.
Ê (d)
31. Simulations of the 20th century by climate models that
Directions (Q.Nos.21-30 ) Each item in this section consists exclude the observed increase ……… greenhouse gases fail
of a sentence with an underlined word/words followed by to simulate the increase in temperature over the second
your words. Select the option that is nearest in meaning to half of the 20th century.
the underlined word and mark your response accordingly. (a) of (b) in (c) by (d) to
21. She got the divorce within no time. Ê (a) Preposition ‘of’ is used when we need to refer to a thing.
(a) detachment (b) breaking down 32. In extremely poor societies, children can be put to work
(c) annulment (d) punishment
……… a young age and are therefore a source of income.
Ê (a,c) The given word ‘divorce’ means annulment or detachment. (a) in (b) on (c) by (d) at
NDA/NA Solved Paper 2020 (I & II) 19

Ê (d) Preposition ‘at’ is used when we use Ê (c) ‘by walk’ must be replaced by 'on
a common time phrase. Ê (b) Preposition ‘on’ is used to refer to foot' to make the sentence
surfaces such as land or water body. grammatically correct.
33. People who are averse ……… hard
work, generally do not succeed in 40. He visits the needy to relieve them 45. What is the time in you watch?
life. ……… their sufferings and poverty. (a) (b) (c)
(a) to (b) about (c) at (d) on (a) from No error
(b) off (d)
Ê (a) Preposition ‘to’ is used when we (c) of
indicate a receiver of an action.
(d) on
Ê (c) ‘in’ must be replaced by ‘by’ to make
the sentence grammatically correct.
34. I have known her ……… a long time. Ê (c) Preposition ‘of’ is used to indicate a
(a) since (b) for belonging of somebody.
46. The price of my dress
(c) at (d) before (a)
Directions (Q.Nos. 41-50) Each item in is higher than that of yours. No error
Ê (b) Preposition ‘for’ is used to indicate
the length of time. this section has a sentence with three (b) (c) (d)
underlined parts labelled as (a), (b) and Ê (d)
35. I accepted the offer ……… certain (c). Read each sentence to find out
conditions. whether there is any error in any 47. My friend’s daughter-in-laws
(a) on (b) in
underlined part and indicate your (a) (b)
(c) by (d) within have come. No error
response against the corresponding letter
Ê (a) Preposition ‘on’ is used to indicate a i.e., (a) or (b) or (c). If you find no error, (c) (d)
state of something.
your response should be indicated as (d). Ê (d)
36. She is a woman ……… humble 41. This building comprises of six houses, 48. I prefer seafood
origin. (a) (b) (a) (b)
(a) off (b) of
(c) from (d) within
three parking lots and one basement. than Mediterranean food. No error
c (c) (d)
Ê (b) Preposition ‘of’ is used when we No error
need to refer to a characteristic of a Ê (c) 'Than' must be replaced by 'to' to
thing/ person. (d) make the sentence grammatically
correct.
37. There is no cure ……… the common Ê (d)
cold. 42. I look forward to meet you. No error 49. We were discussing about the issue.
(a) for (b) of (c) to (d) on (a) (b) (c) (d) (a) (b) (c)
No error
Ê (a) Preposition ‘for’ is used to indicate a Ê (b) For the sentence to be correct ‘meet’
connection with somebody/something. must be replaced by ‘meeting’. (d)
38. I ran ……… John yesterday, and it 43. I cannot cope up with this presure. Ê (b) ‘About’ must be removed to make
the sentence grammatically correct.
was a pleasant surprise. (a) (b) (c)
(a) by (b) off (c) beside (d) into No error 50. Dispose off the garbage
(d) (a) (b)
Ê (d) Preposition ‘into’ is used to a point
wherein you hit someone or something. from the shelves. No error
Ê (b) ‘up’ must be removed from the (c) (d)
39. My grandmother enjoyed boating sentence to make the sentence
……… the lovely lake.
grammatically correct. Ê (a) ‘Off’ must be replaced by ‘of’ to make
the sentence grammatically correct.
(a) in (b) on 44. I came to this place by walk. No error
(c) beside (d) within (a) (b) (c) (d)

Part B (General Studies)


51. New Zealand is considered part of people who inhabit the islands of Hawaii island chain, formerly known as
which one of the following island Polynesia are termed Polynesians ‘Sandwich islands’’, are an archipelago
groups? sharing many similar traits including of 137 islands.
(a) Micronesia (b) Melanesia language family, culture and beliefs.
Melanesia islands (Literal meaning
52. Which of the following statements
(c) Polynesia
‘Islands of Black People’), a Sulo-region
with regard to Pradhan Mantri
(d) Hawaii Island chain
of Oceania extending from New Guinea Krishi Sinchayee Yojana (PMKSY)
Ê (c) New Zealand is island country in the islands to Arafura sea and to Tonga. is/are correct?
South Pacific ocean. It is considered as
Micronesia island is a Sulo region of 1. PMKSY was launched during
the South westernmost part of
Oceania, composed of thousands of 2015-16.
Polynesia. Polynesia is a sub-region of
small islands in the Western Pacific 2. The basic aim of PMKSY is to
Oceania made up of more than 1000
ocean. enhance physical access of water
islands scattered over the Central and
southern Pacific ocean. The indigenous
on farm.
20 NDA/NA Solved Paper 2020 (I & II)

3. One of the main objectives of • In India, the First Five Year Plan was Codes
PMKSY is to achieve introduced in 1951, which was based A B C D
on Harrod-Domar model and focused (a) 3 1 4 2
convergence is investments in
primarily on the development of (b) 3 4 1 2
irrigation at the field level. primary sector, especially agriculture (c) 2 4 1 3
Select the correct answer using the and irrigation. On, 1st January, 2015 (d) 2 1 4 3
codes given below. planning commission (which was
(a) Only 1 (b) 1 and 2 responsible for formulating five year Ê (b) The term economic planning is used
(c) 2 and 3 (d) All of these to describe the long-term plans of the
plan) was replaced by NITI Aayog
(National Institution for Transforming Government of India to develop and
Ê (d) Pradhan Mantri Krishi Sinchayee India). The twelfth Five Year Plan co-ordinate the economy with efficient
Yojana (PMKSY) was launched during utilisation of resources.
(2012-17) would be called the last five
2015-16. It will be implemented across
year plan of India. First Five Year Plan (1951 to 1956) Its
the country with an outlay of ` 50,000
objective was Rehabilitation of
crore in five years. 55. The call for Garibi Hatao was refugees, rapid agricultural
The basic aim of PMKSY is to enhance incorporated in development to achieve food
physical access of water on farm by (a) Fourth Five Year Plan self-sufficiency in the shortest possible
providing assured irrigation, reducing (b) Fifth Five Year Plan time and control of inflation.
wastage of water and improving water (c) Sixth Five Year Plan
Fifth Five Year Plan (1956 to 1961) Its
use efficiency. (d) Seventh Five Year Plan
objective was removal of poverty and
To achieve its objectives of achieving Ê (b) Garibi Hatao (Remove poverty) was attainment of self-reliance.
convergence of investments in irrigation the theme and slogan of Indira Gandhi’s
Eleventh Five Year Plan (2007 to 2012)
at the field level. It amalgamated 1971 election campaign.The slogan and
Its objectives were Rapid and inclusive
ongoing schemes such as Accelerated the proposed anti-poverty programme
growth; Empowerment through
Irrigation Benefit Programme (AIBP), that came with it were designed to give
education and skill development;
Integrated Watershed Management Gandhi an independent national
Reduction of gender inequality and
Programme (IWMP) and on-farm water support, based on rural and urban poor.
Environmental sustainability.
management. It was part of the Fifth Five-Year Plan
Twelfth Five Year Plan (2012 to 2017)
Hence, option (d) ‘All of these’ is the (1975-79). The Fifth Five-Year Plan laid
Its objectives were "faster, sustainable
correct answer. stress on employment, poverty
and more inclusive growth". It proposed
alleviation (Garibi Hatao) and justice.
53. According to Article-79 of the The plan also focused on self-reliance
a growth target of 8%; Raised
Constitution of India, which of the agriculture output to 4%; Manufacturing
in agricultural production and defence.
sector growth to 10% and services to
following is/are described as a part In 1978, the newly elected Morarji Desai 9% respectively.
of Parliament of India? government rejected the plan.
1. The House of the People The objectives of Fourth Five Year Plan 57. Which one of the following
2. The Council of States (1969-74) was growth with stability and Directive Principles was inserted by
3. The President of India progressive achievement of the Constitution (42nd
Select the correct answer using the self-reliance. Amendment) Act, 1976?
codes given below. The basic objective of Sixth Five Year (a) The State shall minimise inequalities
(a) Only 1 (b) 1 and 2 Plan (1980-85) was economic in income
(c) 2 and 3 (d) All of these liberalisation by eradicating poverty and (b) Equal justice and free legal aid
achieving technological self-reliance. (c) Promotion of co-operative societies
Ê (d) As per Article-79 of the Constitution (d) Provision for early childhood care
of India, the Council of the Parliament of The objectives of Seventh Five Year
the Union consists of the President and Plan (1985-90) includes the Ê (b) To promote equal justice and to
two Houses known as the Council of establishment of self-sufficient provide free legal aid to the poor
States (Rajya Sabha) and the House of economy, opportunities for productive (Article-39A) was inserted by the
the People (Lok Sabha). employment and upgradation of Constitution 42nd Amendment Act,
technology. 1976.
54. The Five Year Plan was first The 42nd Amendment Act of 1976
launched in 56. Match List I with List II and select added four new Directive Principles to
a. China (b) USSR the correct answer using the code the original list.
(c) India (d) Bhutan given below the lists: Others are
Ê (b) Among the given options, Five Year List I (Major Objective) List II (Five • To secure opportunities for healthy
Plan was first launched in erstwhile Year Plan) development of children (Article-39).
USSR (1928-1932) by Joseph Stalin • To take steps to secure the
(Soviet Ruler), under ‘New Economic A. Faster and more 1. First
inclusive growth participation of workers in the
Policy’. management of industries
• The First Five Year Plan in China was B. Faster, more inclusive 2. Fifth (Article-43A).
started in 1953 and ended in 1957. and sustainable growth
• To protect and improve the
• The First Five Year plan in Bhutan C. Correction of 3. Eleventh environment and to safeguard forests
was started in 1961 and ended in disequalibrium caused and wildlife (Article-48 A).
1966, and also it was fully funded by by Second World War
• Article 38(2) of Directive Principles of
India. D. Attaining self-reliance 4. Twelfth State policies says that State shall
NDA/NA Solved Paper 2020 (I & II) 21

strive to minimise the inequaliting of This committee was created when Lord (d) Tourism and jobs : a better future for
income, status, facilities, Birkenhead, Secretary of State of India all
opportunities etc. asked the Indian leaders to draft a Ê (d) The theme of World Tourism Day’
• Article 43-B (inserted by 97th Constitution for the country. The report, 2019 was 'Tourism and Jobs: A Better
Amendment, 2011) deals with the which demanded a Dominion Status for Future For AII’.
promotion of co-operative societies. India was considered by the Congress. The World Tourism Day which observed
• Article 45 of DPSPs says that state The main idea of the Nehru report was
shall endeavour to provide early globally on 27th September is hosted
that India would be given Dominion by a different country every year. India
childhood care and education for all status. The Nehru draft report also
children until they complete the age has been selected as the host country
defined the citizenship and for celebration of World Tourism Day
of six year.
Fundamental Rights. 2019 by United Nations World Tourism
• Directive principles of State Policies
Tej Bahadur Sapru (TB Sapru) was an Organisation (UNWTO). This is for the
(DPSPs), enshrined in Part IV (Article
36 to 51) of Constitution, with the Indian freedom fighter, lawyer, politician very first time that India hosted the day
basic idea that the ‘State’ should and the leader of the liberal party in in its National Capital, New Delhi.
keep these principles while framing British ruled India. • ‘Sustainable Tourism’ was the theme
laws, policies, ordinances etc. for World Tourism Day, 2017
60. Which one of the following
58. A common High Court for two or statements about Jaipal Singh is not • ‘Tourism and the Digital
Transformation’ was the theme for
more States can be established by correct? World Tourism Day, 2017.
(a) a law passed by the Parliament (a) He was a member of the Constituent
(b) an order of the Supreme Court of Assembly 63. Which one of the following
India (b) He founded the Adivasi Maha Sabha countries has hosted the Army
(c) an order of the President of India (c) He was the captain of the first Indian Exercise TSENTR, 2019?
(d) an amendment to the Constitution of National Hockey Team
(a) Russia (b) China
India (d) He campaigned for a separate State
(c) Kazakhstan (d) Kyrgyzstan
of Chhattisgarh
Ê (a) A common High Court for two or Ê (a) Armies of India, Pakistan took part in
more state can be established by a law Ê (d) Jaipal Singh Munda (1903-1970) the Army Exercise TSENTR, 2019 that
passed by the Parliament. As per the was a politician, prolific writer and
was conducted by Russia from 9th to
Constitution of India, Articles-214-231 sportsman. He was the member of the
23rd September, 2019. It was the first
deals with the provisions of the High Constituent Assembly which debated
time that forces from India and Pakistan
Courts in India. At present, we have 25 on the new Constitution of the Indian
were participated in multilateral exercise
High Courts in the country, which Union and who spoke for the first time
together since airstrikes conducted by
includes 7 common High Courts. in the Assembly (on 19th December,
Indian force, deep in Pakistan’s Balakot
These are Bombay High Court, Calcutta 1946) and owned up proudly to his
district on February 2019. Earlier armies
High Court, Madras High Court, ‘tribal heritage’. He captained the Indian
of India and Pakistan had met last year
Gauhati High Court, Kerala High Court, field hockey team to clinch gold in the
as part of military exercise conducted
Punjab and Haryana High Court and 1928 Summer Olympics in Amsterdam.
by Shanghai Cooperation Organisation
Jammu and Kashmir High Court, Singh became President of Adivasi
(SCO) also held in Russia.
Mahasabha in 1939.
Article-217 deals with the appointment
of judges. 61. What is ‘Tikki Mausi’ in the context 64. Which one of the following
Article 231 of our Constitution states of Malnutrition? countries is called the ‘country of
that Parliament by law establish (a) A specially packaged food item winds’?
common high courts for two or more (b) A mascot (a) India
states or for two or more states and a (c) Name of a scheme (b) China
Union Territory. This provision was (d) Name given to the healthcare (c) Denmark
brought by Seventh Amendment Act, providers (d) Germany
1956.
Ê (b) ‘Tikki Mausi’ is the mascot unveiled by Ê (c) Denmark is called ‘Country of
Winds’. Denmark ranks first in the world
the Odisha government and UNICEF.
59. In the year 1928, a committee of to produce electricity from wind.
Congress leaders drafted a It aims to spread awareness about
nutrition of children and women. 65. Which one of the following is the
Constitution for India. The Through state level communication and oldest scientific department of
Committee was headed by awareness generation, this mascot is Government of India?
(a) Mahatma Gandhi expected to bring out changes in the (a) Department of Biotechnology
(b) TB Sapru behaviour of common people and (b) Survey of India
(c) Motilal Nehru make them aware about child and (c) India Meteorological Department
(d) Jawaharlal Nehru (d) DRDO
women’s nutrition and development.
Ê (c) In the 1928, a committee of Ê (b) Survey of India, The National Survey
Congress leaders drafted a Constitution 62. Which one of the following was the and Mapping Organisation of the
for India. It was headed by Motilal theme of the World Tourism Day, country under the Department of
Nehru. It was prepared by a committee 2019? Science and Technology, is the Oldest
of All Parties Conference chaired by (a) Sustainable tourism Scientific Department of the
Motilal Nehru with Jawaharlal Nehru as (b) Tourism and the digital transformation Government of India. It was set-up in
its Secretary. There were a total of 9 (c) Tourism : responding to the challenge 1767 and has evolved rich traditions
members in this committee. of climate change over the years.
22 NDA/NA Solved Paper 2020 (I & II)

The Department of Biotechnology is an 68. Which one of the following was the Apart from the Indian Navy, the navies
Indian Government department, under of Indonesia, Singapore, Sri Lanka and
official mascot of Khelo India Youth
the Ministry of Science and Technology Thailand participated in the inaugural
responsible for administrating
Games, 2020? edition.
development and commercialisation in (a) Vijaya (b) Yaya
the field of modern biology and
(c) Rongmon (d) Ammu 71. Chalk and marble are different
biotechnology in India. It was set-up in Ê (a) Jaya the black buck and Vijay the forms of
1986. tiger are the mascots for Khelo India (a) Calcium hydrogen carbonate
Youth Games.The third Khelo India (b) Calcium carbonate
The India Meteorological Department is
Youth Games was held from 10th (c) Calcium acetate
an agency of the Ministry of Earth
January, 2020 and 22nd January, 2020 (d) Sodium carbonate
Sciences of the Government of India.
in Guwahati (Assam) India. The event Ê (b) Chalk and marble are different forms
It is the principal agency responsible for
witnessed 20 national level of calcium carbonate.
meteorological observations, weather
multidisciplinary grassroots games The chemical formula of calcium
forecasting and seismology. It was
played among the age groups of carbonate is CaCO 3. In Earth it is
founded in 1875.
under-17 and under-21 categories. produced by the sedimentation of
The Defence Research and Maharashtra defended its title as it shells of fossilised fishes, shells one
Development Organisation is an topped. million of years.
agency under the Department of
Defence Research and Development in 69. In January, 2020, a passenger It is a precursor of quick lime and
Ministry of Defence of the Government aircraft crashed in Iran soon after slaked lime.
of India, charged with the military’s taking off from Tehran’s Imam 72. The number of maximum electrons
research and development, Khomeini airport killing about 170
headquartered in Delhi. It was founded
in N Shell is
people onboard. The airplane (a) 2
in 1958.
belongs to (b) 8
66. ‘Naseem-Al-Bahr’ is a bilateral (a) Qatar Airways (c) 18
naval exercise between India and (b) Ukraine International Airlines (d) 32
(c) Singapore Airlines
(a) United Arab Emirates Ê (d) The maximum number of electrons
(b) Iran (d) Cathay Pacific in N shell is 32. Total electrons in other
(c) Saudi Arabia Ê (b) A Ukraine International Airlines (UIA) shells are as follows
(d) Oman flight 752 (PS752) crashed shortly after K shell – 2 L shell – 8
Ê (d) Naseem-AI-Bahr naval exercise is a taking off from the Iranian capital M shell – 18 N shell – 32
bilateral exercise between India and Tehran’s Imam Khomeini airport on 8th
Oman. January, killing all 176 passengers and 73. Vinegar is also known as
crew members on board. (a) Ethanoic acid (b) Nitric acid
India and Oman conducted 12th edition
This flight was shot by the Iranian (c) Sulphuric acid (d) Tartaric acid
of bilateral maritime exercise
Naseem-AI-Bahr in Mormugao Port, Islamic Revolutionary Guards Corp. Ê (a) Vinegar is also known as Ethanoic
Goa in January 2020. This shoot down occurred, five days acid. It’s chemical formula is
after US President Donald Trump CH 3COOH. It contains 5-8% acetic acid
Naseem-AI-Bahr (or sea breeze) is a
launched a drone strike that killed by volume.
naval exercise between the Indian Navy
Iranian General Qusem Soleimani.
and the Royal Navy of Oman (RNOV), 74. A liquid is kept in a glass beaker.
being conducted since 1993. Qatar airways is the state owned flag
Which one of the following
‘AL Nagan’ is a joint military exercise carrier airline of Qatar.
statements is correct regarding the
between Indian and Royal Army of Cathy Pacific airways limited is the flag
carrier airline of Hongkong.
pressure exerted by the liquid
Oman.
Singapore airlines is the flag carrier
column at the base of the beaker?
‘Ex Eastern Bridge’ is another bilateral
(a) The pressure depends on the area of
exercise between Indian Airforce and airline of Singapore.
the base of the beaker
Royal Air Force of Oman.
70. MILAN, a multilateral naval (b) The pressure depends on the height
67. Koneru Humpy excels in which one exercise, 2020 was hosted by which of liquid column
(c) The pressure does not depend on the
of the following sports? one of the following cities? density of the liquid
(a) Boxing (a) Port Blair (b) Kochi (d) The pressure neither depends on the
(b) Table Tennis (c) Visakhapatnam (d) Panaji area of the base of the beaker nor on
(c) Chess
(d) Billiards
Ê (c) MILAN 2020-a multilateral naval the height of liquid column
exercise scheduled to be conducted by
Ê (b) Statement (b) is correct regarding
Ê (c) Koneru Humpy is an Indian chess the Indian Navy from 18th to 28th the pressure exerted by the liquid
player who is also reigning world rapid March, 2020, at Visakhapatnam, has column at the base of the beaker the
champion. In 2002, she became the been postponed taking into pressure depends upon the density (d),
youngest woman ever to achieve the consideration the safety of all height (h) of liquid column and also on
title of Grandmaster at the age of 15 participants and travel restrictions acceleration due to gravity (g).
years, 1 month, 27 days, beating Judit imposed by the spread of COVID-19'.
p = hdg
Polgar’s previous record by 3 months. MILAN was first held in 1995.
NDA/NA Solved Paper 2020 (I & II) 23

75. Which of the following statements poisonous gases and biological 80. According to Census of India, 2011,
warfare.
is not correct regarding the travel which one among the following is
• Rio de Janerio (Brazil) is famous for
of sound waves? the least populated State in India?
United Nations Conference on
(a) Sound waves can travel through Environment and Development (a) Maharashtra (b) Madhya Pradesh
water (UNCED), also known as Earth (c) Odisha (d) Punjab
(b) Sound waves can travel through air
(c) Sound waves can travel through steel
Summit or Rio Summit in 1992. Ê (d) Among the given options, Punjab
(27,704,236) is the least populated
(d) Sound waves can travel through 78. Krishna Raja Sagara Dam/Reservoir state, according to 2011 census. It is
vacuum is developed on the 16th most populated state of India.
Ê (d) Sound waves need a medium to (a) river Krishna • Maharashtra (112,372,972) is the
travel. In vacuum, there is no medium, (b) river Tungabhadra second most populous state of India.
hence, sound waves cannot travel (c) river Godavari
• Madhya Pradesh (72,597,565) is the
through vacuum. (d) river Kavery
fifth most populous state of India.
76. Deendayal Port is located at Ê (d) The Krishna Raja Sagar Dam was • Odisha (41,947,358) is the eleventh
built across river Kavery for the Mysore largest state of India in terms of
(a) Kerala (b) Gujarat
and Mandya districts in Karnataka in population.
(c) Maharashtra (d) Goa
1932.The dam is named for the then • Uttar Pradesh (199,581,477) is the
Ê (b) Deendayal Port Trust (Kandla) is a ruler of the Mysore Kingdom, most populated state of India
seaport in Kutch District of Gujarat. Krishnaraja Wodeyar IV. whereas Sikkim (607,688) is the least
Kandla port was renamed as • The dam is the creation one of the populated state of India. According to
Deendayal Port Trust under the India greatest engineers that India had census, 2011, the total population of
Port Act of 1908. It is the largest port of produced, Sir M. Vishweshwaraiah. India is 1,210,193,422.
India by volume of cargo handled. His birthday, 15th September is
Kandla Port Trustis, India’s busiest celebrated as Engineers Day. 81. The speakers of major Indian
major port in recent years. • The reservoir is also the main source languages belong to how many
• Kochi Port is in Kerala. of drinking water for all of Mysore city language families?
• Mumbai port and Jawaharlal Nehru and almost the whole of Bangalore. (a) Two
port are the two major ports of • The water released from this dam is (b) Three
Maharashtra. further used as an important source (c) Four
• Marmugao is the major port of Goa. of water in the State of Tamil Nadu. (d) Six
• At present there are 13 major ports in • Tungabhadra dam is located at Ê (c) The Speakers of major Indian
India and nearly 187 notified Minor Tungabhadra river, a tributary of language belong to four language
and Intermediate ports. Krishna river. families.
• Nagarjuna dam, Srisailam are some Most Indians speak a language
77. Which one of the following cities is of the major dams of Krishna river. belonging to the families of the
associated with Biosafety Protocol • Gangapur dam, Jayakwadi dam , Indo-Aryan branch of Indo-European
to the Convention on Biological Ghatghor dam are some of the (c. 77%), the Dravidian (c. 20.61%), the
Diversity (2000)? famous dams across Godavari river. Austroasiatic (Munda) (c. 1.2%) or the
(a) Geneva (b) Nairobi Sino-Tibetan (c. 0.8%).
(c) Cartagena (d) Rio de Janeiro
79. Which one of the following Indian
states does not share international Article-343 of the Indian Constitution
Ê (c) Cartagena (Colombia) is associated border with two or more countries? stated that the official language of the
with Biosafety Protocol to the Union is Hindi in Devanagari script
(a) Arunachal Pradesh
Convention on Biological Diversity. The instead of the extant English. Later, a
(b) Assam
Cartagena Protocol on Biosafety to the constitutional amendment, the Official
(c) Mizoram
Convention on Biological Diversity is an Languages Act, 1963, allowed for the
(d) Tripura
international agreement which aims to continuation of English alongside Hindi
ensure the safe handling, transport and Ê (d) Tripura does not share international in the Indian Government indefinitely
use of Living Modified Organisms border with two or more countries.
until legislation decides to change it.
(LMOs) resulting from modern Tripura is bordered by the country of
The Eighth Schedule of the Indian
biotechnology that may have adverse Bangladesh to the West, North and
Constitution lists 22 languages.
effects on biological diversity, taking South; and the Indian States of Assam
also into account risks to human health. to the North-East; and Mizoram to the 82. Where did Gandhiji initially forge
It was adopted on 29th January, 2000 East. the techniques of Satyagraha?
and entered into force on 11th Arunachal Pradesh shares international (a) England (b) South Africa
September, 2003. borders with Bhutan in the West, (c) North Africa (d) India
• Nairobi (Kenya) is the headquarter of Myanmar in the East, and a disputed
border with China in the North at the Ê (b) Gandhiji first conceived Satyagraha
‘United Nations Environment in 1906 in response to a law
Programme (UNEP) which is McMohan Line.
discriminating against Asians that was
responsible for coordinating UN's Assam is bordered by Bhutan (in North)
passed by the British colonial
Environmental activities. and Bangladesh (in South).
Government of the Transvaal in South
• Geneva (Switzerland) is famous for Mizoram shares international border
Africa.
protocol which prohibits the use of with Bangladesh and Myanmar.
24 NDA/NA Solved Paper 2020 (I & II)

In 1917, the first Satyagraha campaign Readers Digest (founded in 1922) is an 90. The cost of energy to operate an
in India was mounted in the American magazine published 10 times
industrial refrigerator that
indigo-growing district of Champaran. a year.
consumes 5 kW power working 10
Satyagraha (holding onto truth or truth Life an American magazine was
force), a term coined and developed by founded in 1883.
h per day for 30 days will be (Given
Gandhiji, is a particular form of that the charge per kWh of energy
non-violent resistance or civil 85. In which of the following years = ` 4)
resistance. were passenger trains introduced in (a) ` 600 (b) ` 6000
England? (c) ` 1200 (d) ` 1500
83. Who was the Viceroy of India at the
time of Gandhiji’s Dandi March?
(a) 1823 (b) 1825 Ê (b) Given, power P = 5 kW
(c) 1848 (d) 1861
Consumption time per day, t = 10 h
(a) Lord Irwin (b) Lord Linlithgow
(c) Lord Reading (d) Lord Willingdon Ê (b) In 1825, passenger trains were Energy consumption per day
introduced in England. It used steam
= P × t = 5 kW × 10 h = 50 kWh
Ê (a) Lord Irwin was the Viceroy of India at locomotive built by George
the time of Gandhiji’s Dandi March. He Stephenson. The Liverpool and Cost per kWh = ` 4
served as Viceroy from 1926 to 1931. Manchester Railway, opened in 1830, Daily cost = 50 × 4 = ` 200
In his tenure the First Round Table was the first modern railroad. It was a Cost for 30 days = 30 × 200 = ` 6000
Conference was organised by Gandhiji public carrier of both passengers and
started Salt March on 12th March, 1930 freight. 91. Which one of the following
to break the oppressive salt taxation statements regarding magnetic field
levied on Indian. It culminated into
86. The correct relation between the is not correct?
begining of Civil- Disobedience radius of curvature R and focal (a) Magnetic field is a quantity that has
movement. length f of a spherical mirror is direction and magnitude
Lord Linlithgow was viceroy from 1936 (a) R = f (b) R = 2 f (b) Magnetic field lines are closed curves
to 1944. Cripps Mission and launch of (c) R = 3f (d) R = 4f (c) Magnetic field lines are open curves
(d) No two magnetic field lines are found
Quit India Movement were significant Ê (b) For a spherical mirror, radius of to cross each other
event of his tenure. curvature = 2 × focal length
Lord Reading was viceory from 1921 to i.e., R = 2 f Ê (c) Magnetic field lines are always
1926. In his tenurship, Swaraj Party was closed curves because magnetic
formed in India.
87. A lemon kept in water in a glass monopoles do not exist.
tumbler appears to be larger than its
Lord Willingdon was viceroy from 1931 92. Which one of the following
to 1936. Signing of Poona Pact and actual size. It is because of
statements is not correct?
communal award was significant event (a) reflection of light
(a) Buckminsterfullerence is an allotrope
of his tenure. (b) scattering of light
of carbon
(c) refraction of light
(b) Diamond is a good conductor of
84. Which one among the following (d) polarization of light
electricity
American news magazines was Ê (c) A lemon kept in water in a glass (c) Graphite is a good conductor of
highly sceptical of Gandhiji’s Dandi tumbler appears to be larger than its electricity
March initially but within a week actual size due to refraction of light. (d) In graphite, each carbon atom is
completely changed its opinion and Light gets refracted as it passes from linked to three other carbon atoms
saluted him as a Saint and water into air. In this case, the tumbler Ê (b) Statement (b) is not correct whereas
with water acts as a convex lens which statement (a), (c) and (d) are correct.
Statesman?
is curved outwards in the middle and Diamond is a poor conductor of
(a) Saturday Evening Post
can focus light rays to magnify objects. electricity because here each carbon
(b) Readers Digest
atom is bonded of four other carbon
(c) Time 88. Light enters the eye through a thin atoms forming rigid three dimensional
(d) Life membrane called structure.
Ê (c) Time was the American news (a) Retina (b) Cornea
magazine which was highly sceptical of (c) Pupil (d) Iris 93. How many covalent bonds are
Gandhiji’s Dandi March initially, but present in a Chloropropane
within a week completely changed its Ê (b) Light enters the eye through a thin
membrane called ‘cornea’. Cornea is the molecule having molecular formula,
opinion and saluted him as a Saint and
front part of the eye. It forms a thin C 3 H 2 CI?
Statesman. It named him its 1930 ‘Man
membrane and lets light enter into the eye. (a) 6 (b) 8
of the Year’. The wide coverage of Salt
(c) 9 (d) 10
March in International media helped in 89. Name the scientist who first used a
unravelling the true nature of British glass prism to obtain the spectrum Ê (d) Ten covalent bonds are present in
colonialism in India. It also helped in Chloropropane having molecular
of sunlight. formula C 3H 7Cl
making Mahatma Gandhi a popular
(a) CV Raman (b) Lord Rayleigh H H H
world leader.
(c) Isaac Newton (d) S. Chandrasekhar
 
 
 

2 5 8
Saturday Evening Post (founded on 4th 7
August, 1821) is an American Magazine, Ê (c) Isaac Newton was the first Scientist, 1 
HC 4 C C 
10 Cl
3 6 9
who used a glass prism to obtain
currently published 6 times a year. H H H
spectrum of sunlight.
NDA/NA Solved Paper 2020 (I & II) 25

94. Which one of the following is the 99. Which one of the following terms Strait of Malacca joins Pacific ocean
and Indian ocean.
most fundamental characteristic of describes the practice of growing
an element? two or more crops simultaneously 102. The United Nations Charter was
(a) Melting point (b) Atomic number on the same piece of land? signed by 51 original members of
(c) Colour (d) Atomic weight (a) Crop rotation (b) Mixed cropping the United Nations in 1945 at the
Ê (b) Atomic number is the most (c) Intercropping (d) Mixed farming (a) Hague Conference
fundamental characteristic of an Ê (b) Mixed cropping means the practice (b) London Conference
element. It is defined as the number of of growing two or more crops (c) San Francisco Conference
protons found in the nucleus of every simultaneously on the same piece of (d) Berlin Conference
atom of that element. land. In this system, pulses and some Ê (c) The United Nations Charter was
oilseeds are grown with maize, signed by SI original members of the
95. Neutrons were discovered by sorghum and pearl millet. It prevents
(a) James Chadwick United Nations in 1945 at the San
complete loss of crop due to Franciso conference. 850 delegates
(b) Ernest Rutherford
inadequate rain, pests invasion and from 51 nations deliberated, discussed
(c) JJ Thomson
pathogen infection on a particular crop. and finalised the charter which initially
(d) John Dalton
Crop rotation involves growing different divided into 4 sections. It was passed
Ê (a) Neutrons were discovered by James crops on the same land in the unanimously on 26th June, 1945.
Chadwick. Neutrons are uncharged
preplanned succession. London conference (officially named as
particles found within all atomic nucleus.
Intercropping is the practice of growing Declaration of St. James’ palace) was
96. Atomic mass of an element is equal more than one crop on the same field at signed on 12th June, 1941. This
to the sum of number of the same time in different row pattern. conference led to the idea of formation
(a) electrons and protons only Mixed farming involves the growing of of United Nation.
(b) protons and neutrons only crops as well as the raising of the The Hague Conference, 1954 is related
(c) electrons and neutrons only livestock. On the same piece of land. to protection of cultural property in the
(d) electrons, protons and neutrons Event of Armed Conflict.
100. Which one of the following
Ê (b) Atomic mass of an element is equal statements is correct about effects
Berlin conference of 1884-1885
to the sum of number of protons and regulated European Colonisation and
neutrons only. e.g. carbon atom, of antibiotics on viruses? trade in Africa.
proton = 6 nuetron = 6 (a) Viruses are ‘non-living’ entities but it
can interact with antibiotics 103. Land Revenue Records maintained
Atomic mass = 6 + 6 = 12 in India have categorised land-use
(b) Taking antibiotics cures viral
97. Which one of the following infections into
(c) Viruses do not possess metabolic (a) 6 categories (b) 9 categories
element’s isotope is used in the pathways on which antibiotics can
treatment of cancer? (c) 15 categories (d) 21 categories
function, whereas bacteria have such
(a) Uranium (b) Cobalt pathways Ê (b) Land Revenue Records maintained
(c) Sodium (d) Iodine (d) Viruses are resistant to antibiotics in India have categorised land-use into
9 categories. The following are the
Ê (b) Isotope of Cobalt (Co) is used in the Ê (c) Virus is an acellular structure which land-use categories in India
treatment of cancer. Cobalt therepy is is a non-living particle outside of the
the medical use of gamma rays from body of its host. Thus, its lacks (i) Forests
the radioisotope cobalt-60 to treatment protoplasm which is the site of most of (ii) Land put to non-agricultural use
of cancer. the metabolic activities, antibiotics (iii) Barren and wasteland
which are effective against bacteria (iv) Area under permanent pasture
98. Which one of the following cell (living prokaryotic cell), are ineffective and grazing land
organelles may play a role in on viruses. (v) Area under miscellaneous tree
expelling excess water and wastes crops and groves
in case of unicellular organisms? 101. The Panama Canal opened in 1914,
(vi) Culturable wasteland
(a) Lysosome links
(vii) Current fallow
(b) Vacuole (a) Red sea and Mediterranean sea
(b) Atlantic ocean and Pacific ocean
(viii) Fallow other than current fallow
(c) Golgi body
(d) Endoplasmic reticulum (c) Indian ocean and Pacific ocean (ix) Net sown area
(d) Adriatic sea and Black sea 104. Dry land farming in India is largely
Ê (b) In some unicellular eukaryotic
organisms (e.g Amoeba), cellular Ê (b) The Panama Canal opened in 1914, confined to areas with rainfall less
nitrogenous waste such as ammonia links Atlantic ocean and Pacific ocean.
than
and excess water are excreted by This canal (82 km), cuts across Isthmus
(a) 100 cm (b) 85 cm (c) 80 cm (d) 75 cm
exosmosis via contractile vacuole. of Panama and is a conduit for maritime
These are special types of vacuoles, trade. Construction of this canal started Ê (d) Dry land farming is largely confined
under French in 1881 and completed to the areas with rainfall less than 75
which regulates osmotic pressure of
under the United States in 1914. cm. These regions grow hardy and
cell through osmosis and control water
drought resistant crops such as ragi,
movement across the cell membrane. The administrative control of this canal bajra, moong, gram and guar. Farmers
When these vacuoles merge with the is under the Republic of Panama. in this areas practise various measures
cell membrane they expel wastes into Suez canal joins Red sea and of soil moisture conservation and rain
the external aquatic environment. Mediterranean sea. water harvesting.
26 NDA/NA Solved Paper 2020 (I & II)

105. For an area to be excluded from the and rise in temperature starts in early (d) The system of Collectorate
September. introduced by the Company for
drought-prone category, what exercising supervisory control on
percentage of its gross cropped area 108. Which one of the following is the zamindars failed to take off
should be under irrigation? natural vegetation of South-East
Ê (b) The main consequences of
(a) 10% or more (b) 20% or more China? Permanent settlement on rural society in
(c) 25% or more (d) 30% or more (a) Subtropical broadleaf evergreen Bengal was the rise of group of rich
Ê (d) For an area to be excluded from the forest peasants known as Jotedars. They
drought-prone category, 30% or more (b) Tropical broadleaf evergreen forest succeeded in consolidating their
of its gross cropped area should be (c) Tropical deciduous forest positions in the villages. They lived in
under irrigation. The irrigation (d) Temperate evergreen forest villages, advanced loans to peasants
commission considers both rainfall and Ê (a) The natural vegetation of South-East and resisted efforts by Zamindars to
irrigation as factors in identifying a China is subtropical broadleaf increase the Jama of the village. They
drought. It recommended that areas evergreen forests. The South-East also purchased the estates of
with more than 30% coverage of China experiences temperate monsoon Zamindars when they failed to pay
irrigation should be excluded from type of climate which is characterised timely revenue.
being listed as drought prone areas. by warm moist summer and a cool and Permanent settlement was introduced
According to the vulnerability profile of dry winter. It receives moderate rainfall by Lord Cornwallis in 1793 in Eastern
NDMA, around 68% of India’s cultivable between 25 inches to 60 inches, which parts of the country. It fixed the land
area is vulnerable to droughts. is evenly distributed throughout the revenue and recognised Zamindar as
year. The well distributed rainfall all the owners of land.
106. Which one of the following is not a year round make the regions look green
current of Pacific ocean? at all times. The flora of this region 111. What was Damin-i Koh in Rajmahal
(a) Oyashio current (b) Alaska current includes oak, camphor, camelia and area?
(c) Agulhas current (d) California current magnolia. (a) A large area of land demarcated and
Ê (c) Agulhas current is not a current of Tropical Deciduous forest, also known declared to be the land of the
Pacific ocean. It flows South along the as Monsoon forests are found in the Santhals
East coast of Africa in India ocean. The belt along the equator between Tropic (b) The land of the Paharias cultivated
source of this current is East of Cancer and Tropic of Capricorn. exclusively for paddy
Madagascar current and Mozambique (c) The British territory marked for their
Temperate Evergreen forests are found military camp
current. It acts as an oceanic in temperate areas with warmer (d) The land earmarked for locating
convergence zone which enhances its summers and cool winter. There are settled agriculturists
primary productivity as compared to mostly located in the coastal region with
surrounding waters. mild winter and drier climates and are Ê (a) Damin-i Koh in Rajmahal area was
Oyashio current is a cold current which large area of land demarcated and
dominated by large trees.
circulates counter clockwise in the declared to be the land of the Santhals.
western North Pacific ocean. 109. Which one of the following is a cold It was declared in 1832 to persuade
local wind? Santhals to do settled agriculture in
Alaska current is current of Pacific
(a) Santa Ana (b) Chinook previously forested area. This region
ocean, which circulate alongside the
(c) Mistral (d) Loo was enclosed by boundary pillars and
West coast of the North American
was separated from the world of settled
continent. Ê (c) Mistral is a cold local wind. It blows agriculturists of plains and Pahariyas of
California current is cold water current around Alps mountain in France. It the hills.
of Pacific ocean which moves brings good health, sunshine and
Southward along the Western coast of dryness to the adjoining regions. 112. Which one of the following
North America. Santa Ana is a hot local wind which statements about the Revolt of 1857
blows in South California. is correct?
107. The duration of monsoon in India
Chinook is a hot and dry local wind (a) It was a Revolt carefully organised
extends for an average period of which blows in the Rocky Mountains of and planned by the Rajas, Nawabs
(a) 80–140 days (b) 100–120 days USA and Canada. and Taluqdars
(c) 90–130 days (d) 100–140 days (b) Rumours and prophecies did not play
Loo is a hot and dry local wind which
Ê (a) The duration of monsoon in India blows from West in summer season of
any role in its outbreak and spread
extends for an average period of 80-140 (c) The rebel proclamations in 1857
Northern India.
days. The rainfall received from the repeatedly appealed to all sections of
South-West monsoon is seasonal in 110. What was the consequence of the population irrespective of their
character, which occurs between June caste and creed.
Permanent Settlement on rural
and September. (d) The British succeeded in quickly and
society in Bengal? easily controlling the rebels
Generally, the monsoon arrives at the (a) The zamindars invested capital and
southern tip of Indian Peninsula by the enterprise to improve agriculture Ê (c) The Rebel proclamations in 1857
first week of June. Subsequently, it along lines of British yeoman farmers repeatedly appealed to all sections of
divided into two branches, viz the (b) A group of rich peasants known as the population irrespective of their caste
Arabian sea branch and the Bay of Jotedars succeeded in consolidating and creed. During the course of revolt,
Bengal branch. The retreat of monsoon their position in the villages many proclamations and notifications
which is characterised by clear skies (c) The ryots prospered as a result of were issued by the rebels. Many of this
fixed revenue levy imposed on them proclamations were issued by Muslim
NDA/NA Solved Paper 2020 (I & II) 27

princes or in their names but they took Shyamji Krishna Verma was acid and nitric acid fall on land with
care of Sentiments of every revolutionary freedom fighter who precipitation.
communities of the area. Famous organised India House in London. The Eastern region of Canada and it’s
proclamations were Delhi proclamation He also published a monthly magazine neighbour USA is highly industrialised
and Azamgarh proclamations. Indian sociologist. zone. This alongwith other causes such
This revolt was poorly organised and Har Dayal was revolutionary freedom as Vehicular emissions are the main
planned by the Rajas, Nawabs and fighter who founded Gadar Party in 1913. reason for Acid rain in Eastern Canada.
Taluqdars which was greatest Groundwater depletion is the depletion
weakness of Revolt of 1857. 115. The principle that the framing of of ground water level caused by
Rumours and prophecies played huge the new Constitution for sustained ground water pumping.
role in inciting the sentiments of masses independent India should be • Land degradation is the decline in the
against the Britishers. primarily (though not solely) the productive capacity of land for some
Britishers found very difficult to supress responsibility of Indians time or permanently.
the revolt due to huge participation of themselves, was for the first time • Desertification is the persistent
masses alongside rebels in Awadh area. degradation of dryland ecosystems
conceded in the by climate variation and human
113. Which one among the following (a) Government of India Act, 1935 activities.
was not a Panch sheel principle? (b) August Offer of Viceroy Linlithgow
(a) Peaceful Co-existence
(c) Cripps Proposals 117. Which one of the following
(d) Cabinet Mission mountains lies in between Caspian
(b) Mutual respect for territorial integrity
(c) Nuclear deterrence Ê (b) The principle that the framing of new sea and Black sea ?
(d) Non-interference in internal affairs Constitution for Independent India (a) Caucasus (b) Carpathians
should be primarily (though not solely) (c) Apennine (d) Elburz
Ê (c) Nuclear deterrence was not a Panch the responsibility of Indians themselves
Sheel principle. It is a theory which
was for the first time conceded in the Ê (a) Caucasus mountain lies in between
provides that a nuclear power nation August offer of Viceroy Linlithgow. Caspian sea and Black sea. This
however weak, it can successfully mountain ranges adjoins Vast Eurasian
Made on 8th August, 1940, it also
protect itself against much powerful landmass stretching from Russia to
offered the expansion of Viceroy’s
nation state. Turkey. Mount Elburz the highest peak
executive council besides
Panch Sheel principles were agreed establishment of an advisory war in Europe lies in this mountain range.
between India and China under the council. However, Congress Party Carpathians is a mountain system
leadership of Jawaharlal Nehru and rejected the offer as it provided extending South-eastwards from
Zhou Enlai respectively in 1954 to dominion status. Southern Poland and Slovakia into
establish close relations between the Government of India Act, 1935 provided Romania.
two countries. They are for establishment of All India federation, Apennine is a mountain range
• Mutual respect for each other’s division of subjects into three lists and extending in North-South direction in
territorial integrity and sovereignty. provincial autonomy. Italian Peninsula.
• Mutual non-aggression Cripps proposals were made in March, Mount Elburz is the highest peak of
• Mutual non-interference in each 1942 it promised dominion status, Europe. It is a type of dormant volcano
other’s internal affairs. Constituent Assembly besides control which lies in territory of Russia.
• Equality and mutual benefit of defence under the British.
• Peaceful co-existence Cabinet Mission, 1946 proposed a very 118. Bagalihar, Dulhasti and Salal hydro
loose confederation with centre power projects have been
114. Who among the following had controlling merely defence, foreign developed on which of the
organised, in 1904, a secret society affairs and communication.
following rivers?
of revolutionaries named Abhinav 116. Which one of the following is a (a) Chenab and Jhelum
Bharat? major environmental issue in (b) Chenab and Indus
(a) Khudiram Bose (c) Ravi
Eastern Canada ?
(b) Shyamji Krishna Verma (d) Chenab only
(a) Acid precipitation
(c) Har Dayal Ê (d) Bagalihar, Dulhasti and Salal hydro
(b) Groundwater depletion
(d) VD Savarkar power projects have been developed
(c) Land degradation
Ê (d) VD Savarkar organised in 1904, a (d) Desertification on Chenab.
secret society of revolutionaries named Bagalihar hydro power project with
Abhinav Bharat. Based in Pune, it
Ê (a) Acid precipitation or rain is a major
environmental issue in eastern Canada. installed capacity of 900 MW is situated
prepared youths for revolutionary Acid rain occurs when gaseous in the Ramban district of UT of Jammu
activities and promoted the cause of emissions of combustion of coal and and Kashmir.
national independence. other fossil fuels like Sulphur dioxide Dulhasti is a 390 MW hydroelectric
Khudiram Bose was revolutionary and nitrogen oxide transform into their power plant in Kishtwar district of UT of
freedom fighter who attempted to Secondary pollutants by combining with Jammu and Kashmir. It is a run off river
assassinate a British Judge, Douglas atmospheric moisture (H 2O). These type hydro project build on Chandra
Kingsford in Muzaffarpur. secondary pollutants such as sulphuric river a tributary of Chenab.
28 NDA/NA Solved Paper 2020 (I & II)

Salal hydroelectric power station is a Azerbaijan is a country in the Caucasus Ê (d) Amongst the given options,
run off the river project on Chenab region of Eurasia. It is bounded by the 2
kg-metre/sec is not the unit of energy.
rivers in the Reasi district of UT of caspian sea to the East. kg-(metre) 2/sec 2 is the unit of energy.
Jammu and Kashmir. All other options are the units of energy.
121. In an incandescent electric bulb, the
119. Which of the following statements filament of the bulb is made up of 124. Which one of the following
with regard to Biosphere Reserve which metal? statements is not correct?
is/are correct? (a) Aluminium (b) Copper (a) Biomass is a renewable source of
1. The idea of Biosphere Reserve (c) Tungsten (d) Silver energy
(b) Gobar gas is produced - when
was initiated by UNESCO in Ê (c) In an incandescent electric bulb, the cowdung, crop residues, vegetable
1973-74. filament of the bulb is made up of
waste and sewage are allowed to
2. There are 18 designated tungsten because it has very high
decompose in the absence of oxygen
Biosphere Reserves in India. melting point (3400°C). (c) Biogas generation reduces soil and
3. All Biosphere Reserves in India 122. Two equal resistors R are connected water pollution
(d) Heating capacity of biogas is very low
have been included in world in parallel, and a battery of 12 V is
network of Biosphere Reserves connected across this combination. Ê (d) The heating capacity or calorific
of UNESCO. value of biogas is high due to which it is
A dc current of 100 mA flows
Select the correct answer using the used for lighting.
through the circuit as shown below:
codes given below. R 125. In prokaryotic organisms, nuclear
(a) Only 1 (b) Only 2 region is not surrounded by a
(c) 1 and 2 (d) 1, 2 and 3
membrane. This undefined nuclear
Ê (b) The concept of Biosphere Reserve 100 mA R region is known as
was initiated by UNESCO in 1971, when
(a) Nucleic acid (b) Nucleoid
it launched Man and Biosphere
12 V (c) Nucleolus (d) Nucleosome
Programme (MAB). It aims to establish
a Scientific basis for the improvement of The value of R is Ê (b) Prokaryotic organisms such as
relationship between people and their bacteria lack true nucleus, as their
(a) 120 Ω (b) 240 Ω
environments. nuclear region is not bound by outer
(c) 60 Ω (d) 100 Ω
In India, there is 18 Biosphere reserves R nuclear membrane. It is known as
recognised by the government to
Ê (b) Nucleoid. It has a circular DNA without
histone proteins, which is the main
protect man and environment. Nilgiri
genetic material of bacteria.
Biosphere Reserve in Tamil Nadu and 100 mA R
Kerala, was the first to be included in 126. Which one of the following plant
this list. tissues has dead cells ?
12 V
In India, 11 out of 18 Biosphere (a) Epidermis (b) Parenchyma
Reserves have been included in world In the above circuit, both resistance of
(c) Collenchyma (d) Sclerenchyma
network of Biosphere Reserves of RR are connected in parallel
UNESCO. combination. Ê (d) Sclerenchyma is a dead permanent
tissue. Its cells These have heavily
Hence, Equivalent resistance of circuit
120. Which one of the following is,
thickened secondary walls containing
countries does not have direct lignin. The cells are rigid,
R.R
Req = non-stretchable and lack protoplasma.
access to the sea/ocean ? R+R They provide protection and
(a) Syria (b) Jordan R
(c) Azerbaijan (d) Armenia Req = mechanical support to plant. They are
2 usually found in non-growing regions of
Ê (d) Armenia does not have direct Here, I = 100 mA = 100 × 10 −3 A = 0.1A plant bodies such as the bark protective
access to the sea/ocean. It is layering of seeds and fruits or mature
landlocked country in the western Asia. V = 12 V
stems. Rest epithelium, paranchyma
It is bordered by Turkey to the West, Using ohm’s law and collenchyma have living tissues.
Georgia to the North, Artsakh and V = I Req
Azerbaijan to the East and Iran to the V 127. Cartilage is not found in
⇒ Req = (a) larynx (b) nose
South. Yerevan is the capital of I
Armenia. R 12 (c) ear (d) urinary bladder
⇒ =
Syria is a country in Western Asia with 2 01. Ê (d) Cartilage is not found urinary
its capital at Damascus. It is bordered bladder. Urinary bladder consists of
⇒ R = 240 Ω
by the Mediterranean sea to the West. epithelium on the lumen surrounded by
Jordan is a country in Western Asia with 123. Which one of the following is not a collagen rich connective tissue and
its capital at Amman. It has 26 km the unit of energy? muscle layer. Hyaline Cartilage is the
coastline on the Red sea in its extreme (a) Joule (b) Watt-hr type of cartilage found in areas such as
South-West. (c) Newton-metre (d) kg-metre/sec 2 nose, ears and larynx of the human
body.
NDA/NA Solved Paper 2020 (I & II) 29

128. Two planets orbit the Sun in A proton enters a magnetic field at Ê (c) Maintaining sacred groves is not a
circular orbits, with their radius of right angles to it, as shown above. reason of decreasing biodiversity. It is
orbit as R1 = R orbit as R 2 = 4 R. The direction of force acting on the an in-situ conservation method which
proton will be provide protection to the organisms in
Ratio of their periods (T1 / T2 ) (a) to the right (b) to the left some area in its natural habitat. Rest
around the Sun will be (c) out of the page (d) into the page three are the causes of decrease of
(a) 1/16 (b) 1/8 (c) 1/4 (d) 1/2 biodiversity.
Ê (c) According to Fleming’s left hand
Ê (b) According to Kepler’s 3rd law, rule, the direction of force on conductor 137. Which one of the following is not a
T2 ∝ R3 ⇒ T ∝ R 3/ 2 is perpendicular to the direction of
cause of depletion in groundwater?
magnetic field and current. The
∴As per question, (a) Afforestation
direction of current is taken same as the
3/ 2 (b) Loss of forests
T1  R1  direction of motion of positive charge
=  (c) Excessive pumping of groundwater
T2  R 2  (proton), therefore, the force on proton (d) Construction of large scale concrete
3/ 2 3/ 2 is directed out of the page. buildings
T1  R 
=  
1
⇒ = 
T2  4R   4 133. Which one of the following Ê (a) Groundwater will not be depleted in
1/ 2 statements about sound is not case afforestation. Afforestation refers
T1  1  1
⇒ =  = correct? to planting of various types of trees over
T2  64  8 a particular area.
(a) Sound travels at a speed slower than
129. A metallic wire having resistance of the speed of light This allows rain and surface water to
percolate with in the soil and prevents
20 Ω is cut into two equal parts in (b) Sound waves are transverse waves
(c) Sound waves are longitudinal waves surface run off of water. It also helps in
length. These parts are then (d) Sound travels faster in moist air than recharging or refilling the groundwater
connected in parallel. The in dry air level. Hence, the water table rises up
resistance of this parallel and clear uncontaminated water is
Ê (b) Sound waves are longitudinal waves,
combination is equal to but they are not transverse waves. In available.
(a) 20 Ω (b) 10 Ω (c) 5 Ω (d) 15 Ω sound waves, the individual particles of But in all other options, groundwater will
the medium move in a direction parallel be depleted without getting recharge.
Ê (c) As, Resistance ∝ length
to the direction of propagation of the
∴When wire of resistance 20 Ω is cut 138. Which one of the following types of
disturbance.
into two equal parts in length, the radiations has the smallest
resistance of each part will be 134. When the short circuit condition
R 20 wavelength ?
= = 10 Ω occurs, the current in the circuit (a) Microwaves
2 2
(a) becomes zero (b) Infra-red
Equivalent resistance of parallel (b) remains constant (c) Visible light
combination. (c) increases substantially (d) X-rays
1 1 1 (d) keeps on changing randomly
= + ⇒ Req = 5Ω Ê (d) X-rays have the smallest wavelength
Req 10 10
Ê (c) In short circuit condition the amongst the other three options.
resistance of the circuit is zero. The range of wavelengths for these
130. Light of uniform intensity impinges
So, as per ohm’s law waves are :
perpendicularly on a totally
reflecting surface. If the area of the Ishort =
V
=
V
=∞ Microwaves : 10 −3 to 3 × 10 −1 m
R 0
surface is halved, the radiation Infra-red : 8 × 10 −7 to 5 × 10 −2 m
So, current increases substantially in
force on it will become short circuit condition. Visible light : 4 × 10 −7 to 7 × 10 −7m
(a) double (b) half
135. Which one of the following is not a X-rays : 10 −13 to 3 × 10 −8 m
(c) four times (d) one fourth
Ê (b) As, Radiation force ∝ Area of surface component of human male 139. The instrument used for detecting
so, if area of the surface is halved, reproductive system ? the presence of electric current in a
radiation force will also get halved. (a) Cervix circuit is
(b) Urethra
131. The part of the human eye on (c) Seminal vesicle (d) Vas deferens
(a) Refractometer (b) Galvanometer
which the image is formed is (c) Viscometer (d) Diffractometer
Ê (a) Cervix is a part of female
(a) pupil (b) cornea
reproductive system. Rest Urethra, Ê (b) Galvanometer is an instrument used
(c) retina (d) iris for detecting the electric current in a
seminal vesicle and Vas deferens are
Ê (c) Retina is the light sensitive surface of circuit. Galvanometer is used to
parts of male reproductive system. It is
eye on which the image is formed. measured small amount of current in
the lower part of uterus having marrow
the circuit.
132. Consider the following image: tube-like structure forming vegina.
Proton 136. Which one of the following is not a 140. Which one of the following is the
reason of decrease in biodiversity? largest composition in biogas?
(a) Carbon dioxide
(a) Large scale deforestation
Magnetic field (b) Methane
(b) Exploitation of forest produce
(c) Hydrogen
(c) Maintaining sacred groves
(d) Hydrogen sulphide
(d) Encroachment in forest areas
30 NDA/NA Solved Paper 2020 (I & II)

Ê (b) Biogas is generally composed of 144. Number of molecules of water of 147. Which one of the following is the
methane (55-65%), carbondioxide crystallisation in copper sulphate, best example of desiccant?
(35-45%), nitrogen (0-3%), hydrogen
sodium carbonate and gypsum are (a) Silica gel
(0-1%) and hydrogen sulfide (0-1%). (b) Polystyrene
(a) 5, 10 and 2 respectively
Thus, methane has the largest (c) Sodium chloride
(b) 10, 2 and 5 respectively
composition in biogas. (d) Sodium carbonate
(c) 5, 2 and 10 respectively
141. The Sun appears reddish during (d) 2. 5 and 10 respectively Ê (a) Silica gel is the best example of
sunrise and sunset. The desiccant. It is hygroscopic in nature.
Ê (a) Number of molecules of water of
phenomenon in optics which is crystallisation of copper sulphate, It absorbs water from the surrounding.
responsible for this appearance of sodium carbonate and Gypsum are 5,
10 and 2 respectively.
148. Which one of the following was the
the Sun is first mineral acid discovered?
(a) Reflection Complete chemical formula are as
(a) Sulphuric acid (b) Hydrochloric acid
(b) Total internal reflection follows
(c) Nitric acid (d) Phosphoric acid
(c) Scattering Copper sulphate — CuSO 4 . 5H 2O
(d) Interference Sodium Carbonate — Na 2CO 3 . 10H 2O Ê (a) Sulphuric acid (H 2SO 4 ) was the first
mineral acid discovered.
Ê (c) The Sun appears reddish during Gypsum — CaSO 4 .2H 2O
sunrise and sunset due to scattering of 149. The refractive index of fused quartz
light. 145. Which one of the following is the
is 1·46 and that of sapphire is 1·77.
correct sequence of change in
142. A lens has a power of +2·0 Dioptre. colours when a turmeric stain on
If Vq is the speed of light in quartz
Which one of the following and vs is the speed of light in
white clothes is scrubbed by soap
statements about the lens is true? sapphire, then which one of the
and then washed with water?
(a) The lens is concave and has a focal following relations is correct?
(a) Yellow-pink-blue
length of 0.5 m (a) v q > v s (b) v s > v q
(b) Yellow-reddish brown-yellow
(b) The lens is convex and has a focal v
(c) Yellow-reddish brown-blue (c) v s = v q (d) v s = q
length of 2.0 m 2
(c) The lens is convex and has a focal (d) Yellow-blue-pink
length of 0.5 m Ê (b) When a turmeric stain appear on Ê (a) As, Refractive index
Speed of light in vacuum
(d) The lens is concave and has a focal white clothes it becomes reddish brown =
length of 2.0 m when it come in contact with soap, Speed of light in medium
Ê (c) Power, P = + 2 D which is basic in nature it turns yellow i.e., µ =
c
⇒µ ∝
1
1 1 again when the cloth is washed with v v
Focal length, f = = = 0.5 m
p 2 plenty of water. As, µ quartz < µ sapphire ⇒ vq > v s
The converging (or convex) lens has 146. Which one of the following 150. In case of a concave mirror, if an
positive focal length. statements regarding bleaching object is kept between principal
So, the lens is convex with focal length of powder and DDT is correct?
0.5 m.
focus F and pole P of the mirror,
(a) Both are inorganic compounds then which one of the following
143. At nearly 70°C, sodium bicarbonate (b) Both are organic compounds
statements about the image is not
shows the property of gradually (c) Both contain chlorine
(d) Both contain calcium correct?
decomposing, which makes it (a) The image will be virtual
usable in bakery products. The Ê (c) Both bleaching powder (CaOCl 2) (b) The image will be enlarged or
product of decomposition and Dichloro Diphenyl Tricholoroethane magnified
responsible for this use of sodium (DDT) contain chlorine. Bleaching (c) The image will be formed at infinity
bicarbonate is powder is inorganic compound, but (d) The image will be erect
DDT is an organic halogen compound.
(a) Carbon dioxide Ê (c) If the object is placed between F
(b) Hydrogen structures are as follows and P of concave mirror, the image is
(c) Water vapour Cl formed behind the mirror. The image
(d) Oxygen formed is virtual, erect and magnified.
Cl Cl A
Ê (a) Sodium bicarbonate (NaHCO 3) at
70°C gradually decomposes to give A
Cl
sodium carbonate (Na 2CO 3), water Ca
(H 2O) and carbon dioxide (CO 2). O-Cl
Cl Cl C FB P B
Complete reaction is as follows
CaOCl2 DDT
Heat
2NaHCO 3 → Na 2CO 3 + H 2O + CO 2
NDA /NA
National Defence Academy/Naval Academy

SOLVED PAPER 2019 (II)


PAPER I : Mathematics
1. If both p and q belong to the set 3. If A , B and C are subsets of a given Ê (c) Let α, β be the roots of equation.
{1, 2, 3, 4 }, then how many equations set, then which one of the following 2 x 2 − 2( k − 2 ) x − ( k + 1) = 0
of the form relations is not correct? 2( k − 2 )
Qα + β = = k − 2,
px 2 + qx + 1 = 0 will have real roots? (a) A ∪ ( A ∩ B) = A ∪ B 2
(b) A ∩ ( A ∪ B) = A − ( k + 1)
(a) 12 (b) 10 αβ =
(c) ( A ∩ B) ∪ C = ( A ∪ C ) ∩ ( B ∪ C )
(c) 7 (d) 6 2
(d) ( A ∪ B) ∩ C = ( A ∩ C ) ∪ ( B ∩ C )
Ê (d) Equation px + qx + 1 = 0, has real
2
We know that
Ê (a) Let U be the set and A, B andC are the
roots, where p and q belong to the set α 2 + β 2 = (α + β )2 − 2αβ
subset of U.
{1, 2, 3, 4}. We know that, A ∪ ( A ∩ B) = A, k+1
= ( k − 2 )2 + 2 ×
∴ q 2 − 4p ≥ 0 So option (a) is not correct. 2
[Q for real roots of a quadratic equation A ∩ ( A ∪ B) = A, so option (b) is correct. = k 2 + 4 − 4k + k + 1
( A ∩ B) ∪ C = ( A ∪ C ) ∩ ( B ∪ C ), = k 2 − 3k + 5
b 2 − 4ac ≥ 0]
so option (c) is correct. 9 9
= k 2 − 3k + − + 5
It is possible if value of and ( A ∪ B) ∩ C = ( A ∩ C ) ∪ ( B ∩ C ) 4 4
( p, q ) = (1, 2 ), (1, 3), (1, 4), (2, 3), (2, 4) so option (d) is correct. 2
 3 11
and ( 3, 4) = k −  +
4.  2 4
Hence, the number of equations are 6.
If the sum of first n terms of a series
is (n + 12), then what is its third  3
α 2 + β 2 is minimum, if  k −  = 0
2.  2
What is the value of term? 3
1 − 2 + 3 − 4 + 5 − ...... + 101? (a) 1 (b) 2 ⇒ k=
2
(a) 51 (b) 55 (c) 3 (d) 4
6. If the roots of the equation
(c) 110 (d) 111 Ê (a) Sum of first n term of a series = n + 12
⇒ a1 + a2 + a3 + .........+ an = n + 12 a( b − c )x 2 + b(c − a )x + c( a − b ) = 0
Ê (a) Given series,
= 1 − 2 + 3 − 4 + 5 − ...... + 101 Put n = 1, a1 = 1 + 12 = 13 are equal, then which one of the
= (1 + 3 + 5 + ..... + 101) Put n = 2, a1 + a2 = 2 + 12 ⇒ a1 + a2 = 14 following is correct?
(a) a, b and c are in AP
− (2 + 4 + 6 + ..... + 100) ⇒ 13 + a2 = 14 ⇒ a2 = 1 (b) a, b and c are in GP
= (1 + 3 + 5 + .... 51 terms) Put n = 3 (c) a, b and c are in HP
− (2 + 4 + 6 + ...... 50 terms) a1 + a2 + a3 = 3 + 12 (d) a, b and c do not follow any regular
51 ⇒ 13 + 1 + a3 = 15 pattern
= [2 + ( 51 − 1) × 2 ]
⇒ a3 = 15 − 14 = 1
2 Ê (c) The roots of the equation

50
[4 + ( 50 − 1) × 2 ] 5. What is the value of k for which the a( b − c ) x 2 + b(c − a ) x + c( a − b ) = 0
2 are equal.
sum of the squares of the roots of
[Q both series are AP and
2x 2 − 2(k − 2)x − (k + 1) = 0 is ∴b 2(c − a )2 − 4a( b − c ). c( a − b ) = 0
n
S n = [2 a + ( n − 1)d ] minimum? [Q ax 2 + bx + c = 0 of roots are real if
2
51 50 (a) − 1 (b) 1 b 2 − 4ac ≥ 0
= × 102 − × 102 (c)
3
(d) 2
2 2 ⇒ b 2(c 2 + a 2 − 2ca ) − 4ac ( ab − b 2
2
= 2601 − 2550 = 51 − ac + bc ) = 0
2 NDA/NA Solved Paper 2019 (II)

⇒ b 2c 2 + a 2b 2 – 2 ab 2c − 4a 2bc 9. Ê (d) Given equation,


If a set A contains 3 elements and
+ 4ab 2c + 4a 2c 2 − 4abc 2 = 0 another set B contains 6 elements, x 0 2 3x 0 2
then what is the minimum number 2x 2 1 + x2 2 1 = 0
⇒ b c + a b + 2 ab c
2 2 2 2 2

of elements that ( A ∪ B ) can have? 1 1 1 0 1 1


− 4a 2bc − 4abc 2 + 4a 2c 2 = 0
(a) 3 (b) 6 ⇒ x(2 − 1) − 0 + 2(2 x − 2 ) + 3 x(2 − 1)
⇒ b 2(c 2 + a 2 + 2 ac ) − 4abc ( a + c ) (c) 8 (d) 9
− 0 + 2 ( x 2 − 0) = 0
+ 4a 2c 2 = 0 Ê (b) n( A ) = 3, n( B) = 6
[expression w.r.t. first row]
⇒ b (c + a ) − 4abc ( a + c ) + (2 ac ) = 0
2 2 2 ∴The minimum number of elements in
⇒ x + 4 x − 4 + 3 x + 2 x2 = 0
⇒ [b(c + a ) − 2 ac ]2 = 0 A∪B= 6
⇒ 2 x2 + 8 x − 4 = 0
i.e n( A ∪ B) = 6
⇒ b (c + a ) − 2 ac = 0
(because max n( A ∩ B) = 3 ⇒ x2 + 4 x − 2 = 0
2 ac
⇒ b(c + a ) = 2 ac ⇒ b = −4 ± 16 − 4 (1) ( −2 )
c+ a 10. What is the number of diagonals of ⇒ x=
2
So, a, b and c are is HP. an octagon?
−4 ± 24 −4 ± 2 6
(a) 48 = =
7. If | x 2 − 3x + 2| > x 2 − 3x + 2, then (b) 40 2 2
which one of the following is (c) 28 = −2 ± 6
correct? (d) 20
13. If x + a + b + c = 0, then what is the
(a) x ≤ 1 or x ≥ 2 (b) 1 ≤ x ≤ 2 Ê (d) The number of vertices of an octagon
(c) 1 < x < 2 =8 x +a b c
(d) x is any real value except 3 and 4 ∴The number of points in a plane = 8 value of a x +b c ?
Ê (c)| x − 3 x + 2| > x − 3 x + 2
2 2
∴ Total number of straight line form by 8
points = 8C 2
a b x +c
⇒ − ( x2 − 3 x + 2 ) > x2 − 3 x + 2
[Q 1 straight line form by 2 (a) 0 (b) ( a + b + c )2
[if x 2 − 3 x + 2 < 0, and x 2 − 3 x + 2 > 0 (c) a 2 + b 2 + c 2 (d) a + b + c − 2
points]
not possible]
=
8!
=
8×7
= 28
Ê (a) Given, x + a + b + c = 0
⇒ − 2( x − 3 x + 2 ) > 0
2
x+ a b c
2 ! 6! 2
⇒ x2 − 3 x + 2 > 0 x+ b
∴ The number of diagonals of an octagon a c
⇒ x2 − 2 x − x + 2 > 0 = Total number a b x+c
⇒ ( x − 2 )( x − 1) > 0 of straight line form by 8 points − number x+ a+ b+c b c
∴ 1 < x < 2 is correct. of sides of octagon
= x+ a+ b+c x+ b c
= 28 − 8 = 20
8. A geometric progression (GP) x+ a+ b+c b x+c
consists of 200 terms. If the sum of 11. What is the value of the determinant [byC1 → C1 + C 2 + C 3]
odd terms of the GP is m, and the 1! 2! 3! 1 b c
sum of even terms of the GP is n, = ( x + a + b + c) 1 x + b c
2! 3! 4! ?
then what is its common ratio? 1 b x+c
(a) m / n (b) n / m 3! 4 ! 5!
(c) m + ( n / m) (d) n + ( m / n) [ x + a + b + c common from C1] = 0
(a) 0 (b) 12
2 [Q x + a + b + c = 0]
Ê (b) Let a, ar, ar ...... 200 terms be a (c) 24 (d) 36
geometric progression. Ê (c) Given determinant 14.  1 −1
1! 2 ! 3 ! 1 2 6 1 0 0 If A =  , then the expression
−1 1 
Where, a is the first terms and r be the
common ratio. = 2 ! 3! 4! = 2 6 24 = 2 2 6
GP of odd terms a, ar 2, ar 4 , ..... 100 terms. A 3 − 2A 2 is
3! 4! 5! 6 24 120 6 12 48
GP of even terms ar, ar 3, ar 5, …… 100 (a) a null matrix (b) an identity matrix
[by C 2 → C 2 − 2C1, C 3 → C 3 − 3C 2] (c) equal to A (d) equal to − A
terms. = 1( 96 − 72 ) − 0 + 0  1 −1
∴Sum of odd terms of the GP = m [expression w.r.t. first row]
Ê (a) A =  −1 1 
 
a{ r 200 − 1} = 24
⇒ =m …(i)  1 −1  1 −1
r−1 ∴ A2 = A ⋅ A =  ⋅ 
12. What are the values of x that satisfy  −1 1   −1 1 
Sum of even terms of the GP = n
the equation  1 + 1 −1 − 1  2 −2 
ar( r 200 − 1} = = 
⇒ =n …(ii) x 0 2 3x 0 2  −1 − 1 1 + 1   −2 2 
r −1
2x 2 1 + x 2
2 1 = 0?  2 −2   1 −1
Dividing of Eq. (i) by Eq. (ii), and A 3 = A 2 ⋅ A =  . 
1 1 1 0 1 1  −2 2   −1 1 
1 m n
⇒ = ⇒r =  2 + 2 −2 − 2   4 −4
(a) −2 ± 3 = =
r n m

n (b) −1 ± 3  −2 − 2 2 + 2   −4 4 
Hence, the common ratio of the GP is . (c) −1 ± 6
m (d) −2 ± 6
NDA/NA Solved Paper 2019 (II) 3

Now, 17. What is the value of 19. In a school, 50% students play cricket
 4 −4 2 −2 
A3 − 2 A2 =   − 2  −2 i + 3 
2019
i − 3 
2019 and 40% play football. If 10% of
 − 4 4   2 
  +  ? students play both the games, then
 4 −4  −4 4   2   2  what per cent of students play
= +  
 −4 4   4 −4 (a) 1 neither cricket nor football?
 4 − 4 −4 + 4  0 0 (b) − 1 (a) 10% (b) 15% (c) 20% (d) 25%
= =
 −4 + 4 4 − 4   0 0 (c) 2 i Ê (c) Students, who play cricket = 50%
(d) − 2 i Students, who play football = 40%
= a null matrix
i + 3
2019
i − 3
2019
Students who play both games = 10%
15. Let m and n(m < n ) be the roots of the Ê (c)   +   Students who play only cricket
 2   2 
equation x 2 − 16x + 39 = 0. If four 2019 2019
= 50 − 10 = 40%
 3 1   3 1  Students who play only football
terms p , q , r and s are inserted = + i − − i
between m and n to form an AP, then  2 2   2 2  = 40 − 10 = 30%
2019
what is the value of p + q + r + s ? = cos
π π
+ i sin 
∴Total students who play any game
(a) 29 (b) 30  6 6  = 40 + 30 + 10 = 80%
2019
(c) 32 (d) 35 π π ∴ Students who play neither cricket nor
– cos − i sin 
Ê (c)2 m and n be the roots of the equation  6 6  football = 100 − 80 = 20%
x − 16 x + 39 = 0 ( m < n). 2019 π 2019 π
= cos + i sin 20. If A = { x : 0 ≤ x ≤ 2} and B = {y : y is
∴ m + n = 16 …(i) 6 6 a prime number}, then what is
and mn = 39 …(ii) 2019 π 2019 π A ∩ B equal to?
− cos + i sin
We know that, n − m = ( m + n)2 − 4mn 6 6 (a) φ (b) {1} (c) {2} (d) {1, 2}
[De-moivre’s theorem
(Q m < n) Ê (c) A = { x : 0 ≤ x ≤ 2} = { 0, 1, 2}
(cos θ ± i sin θ)n = cos nθ ± i sin n θ]
= 256 − 156 = 100 and B = { y : y is a prime number}
2019 π
n − m = 10 …(iii) = 2 i sin = {2, 3, 5, 7, 11, ..... }
6
Solving the Eqs. (ii) and (iii), n = 13, m = 3  3π  ∴ A ∩ B = { 0, 1, 2} ∩ {2, 3, 5, 7, 11, ......}
= 2 i sin  168 × 2 π + 
Four terms p, q , r and s are inserted  6  = {2}
between m and n to form an AP. 3π
= 2 i sin 21. If x = 1 + i , then what is the value of
∴ AP is 3, p, q , r, s, 13 6
Here, a = 3, l = 13, n = 6 [Q sin (2 nπ + θ) = sin θ, n is an integer] x 6 + x 4 + x 2 + 1?
π (a) 6i − 3 (b) −6i + 3
∴ l = a + ( n − 1)d = 2 i sin = 2 i
2 (c) −6i − 3 (d) 6i + 3
13 = 3 + ( 6 − 1)d
⇒ d =2 18. If α and β are the roots of Ê (c) Given, x = 1 + i
∴ p = a + d = 3 + 2 = 5,  1 i 
= 2  + 
x 2 + x + 1 = 0, then what is  2 2
q = a + 2d = 3 + 4 = 7 3
 π π
r = a + 3 d = 3 + 6 = 9,
d = a + 4 d = 3 + 8 = 11
∑ (α j + β j ) equal to? = 2  cos
 4
+ i sin 
4
j =0
Now, p + q + r + s = 5 + 7 + 9 + 11 Now, x 6 + x 4 + x 2 + 1
(a) 8 (b) 6
= 32 = x 4 ( x 2 + 1) + 1( x 2 + 1)
(c) 4 (d) 2
16. Under which one of the following = ( x 2 + 1) ( x 4 + 1)
Ê (d) α and β are the roots of the equation  2

conditions will the quadratic x2 + x + 1 = 0  π π
= ( 2 )2  cos + i sin  + 1
equation   4 4  
∴ α + β = −1
x 2 + mx + 2 = 0 always have real and αβ = 1  4 π π
4

( 2 )  cos + i sin  + 1
roots? 3   4 4  
∑ (α + β j ) = (α 0 + β 0 )
j
(a) 2 3 ≤ m2 < 8 (b) 3 ≤ m2 < 4 Now,
j=0   2π 2π 
(c) m ≥ 8
2
(d) m ≤2
3 = 2  cos + i sin  + 1
+ (α1 + β ) + (α 2 + β 2 ) + (α 3 + β 3 )
1
  4 4  
Ê (c) The quadratic equation   4π 4π  
x 2 + mx + 2 = 0, = (1 + 1) + ( −1) + {α 2 + β 2 + 2αβ − 2αβ}  4 cos 4 + i sin 4  + 1
 
have real roots. + (α + β ) (α 2 + β 2 − αβ )
[Q (cos θ + i sinθ)n = cos nθ + i sin nθ]
∴ m2 − 4(1)(2 ) ≥ 0 = 2 − 1 + {(α + β ) − 2αβ} + ( −1)
2
  π π 
[quadratic equation ax 2 + bx + c = 0 = 2  cos + i sin  + 1
{α 2 + β 2 + 2αβ − 3αβ}   2 2  
have real roots if b 2 − 4ac ≥ 0] = 1 + {( −1)2 − 2(1)} − {(α + β )2 − 3(1)} [4(cos π + i sin π ) + 1]
⇒ m2 − 8 ≥ 0
= 1 − 1 − {( −1)2 − 3} = [2( 0 + i ) + 1] [4( −1 + 0) + 1]
⇒ m2 ≥ 8 = (2 i + 1) ( −4 + 1) = − 6i − 3
= − (1 − 3) = 2
4 NDA/NA Solved Paper 2019 (II)

22. What is the value of Ê (d) Given expansion, 27. If n ! has 17 zeros, then what is the
1 (1 + 2 x + x 2 )5 + (1 + 4 y + 4 y 2 )5 value of n?
2+ ?
1 = [(1 + x )2 ]5 + [(1 + 2 y)2 ]5
2+ (a) 95 (b) 85
1 (c) 80
2+ = (1 + x )10 + (1 + 2 y)10 (d) No such value of n exists
2 + ... ∞ ∴ Total number of terms in given Ê (b) We know that each interval of 5!is one
(a) 2 − 1 (b) 2 + 1 (c) 3 (d) 4 expansion. zero.
1 = (10 + 1) + (10 + 1) = 22
Ê (b) Let, x = 2 + 1
i.e. 5! has one zero.
2 + [Q total number of terms in expansion of 10! has two zeros.
1
2 + (1 + x )n = n + 1] ∴85! has 17 zeros.
2 + ... ∞
1 Hence, the value of n is 85.
⇒ x=2 + ⇒ x2 = 2 x + 1 25. If the middle term in the expansion
x
 1
2n 28. Let A ∪ B = { x |( x − a )( x − b ) > 0,
⇒ x2 − 2 x − 1 = 0 of  x 2 +  is 184756x 10 , then where a < b }. What are A and B equal
 x
2± ( −2 )2 − 4(1) ( −1) to?
⇒ x= what is the value of n?
2 (a) A = { x| x > a} and B = { x| x > b}
2 ± 8 2 ±2 2 (a) 10 (b) 8 (b) A = { x| x < a} and B = { x| x > b}
= = = 1± 2 (c) 5 (d) 4 (c) A = { x| x < a} and B = { x| x < b}
2 2
(d) A = { x| x > a} and B = { x| x < b}
= 2+1 (Q x > 2 ) Ê (a) The middle term in the expansion of
 2 1
2n
Ê (c) Let A ∪ B = { x :( x − a)( x − b ) > 0,
23. If P (n, r ) = 2520 and C (n, r ) = 21, x +  where a < b}.
 x
then what is the value of  2n  It is possible if x − a < 0 and x − b < 0
= + 1 th term [Q 2 n is even]
C (n + 1, r + 1) ?  2  or x < a and x < b
(a) 7 (b) 14 = ( n + 1)th term. ∴ A = { x : x < a} and B = { x : x < b}
(c) 28 (d) 56
According to the question,
Ê (c) If P( n, r ) = 2520 and C ( n, r ) = 21,
29. If the constant term in the expansion
Value of middle term = 184756 x10 10
∴ n
pr = 2520  k 
of  x − 2  is 405, then what can
n
2 2n − n  1
n! ⇒ 2n
Cn ( x )   = 184756 x
10
 x 
⇒ = 2520 …(i)  x
( n − r )!
[Q Tr +1 = nC r x n − r a r in expansion be the values of k?
and C r = 21
n
(a) ±2 (b) ±3
of ( x + a )n]
n! (c) ±5 (d) ±9
⇒ = 21 …(ii)
r ! ( n − r )! ⇒ C n( x )4 n − 2n − n = 184756 x10
2n
Ê (b) Let ( r + 1)th term in the expansion of
From Eqs. (i) and (ii), we get ⇒ C n( x )n = 184756 x10
2n
 k
10
 x − 2  is constant.
2520 Comparing the power of x both sides  x 
= 21
r! n = 10  −k 
r
2520 ∴ Tr +1 = 10C r ( x )10 − r  2 
⇒ r! = = 120 26. 1 2  x 
21
  1 2 = nC r x n − r a r in expansion
⇒ r! = 5 ! If A = 2 3 and B =   , then [Q Tr +1
3 4 2 1
∴ r=5   of ( x + a )n]
Putting the value of r in Eq. (i), 10 − r
which one of the following is − 2r
n! ⇒ 405 = 10C r ( x ) 2 ⋅ ( − k )r
= 2520 correct?
( n − 5)! 10 − 5r
(a) Both AB and BA exist ⇒ 405 = 10
C r ( x) 2 ⋅ ( − k )r …(i)
⇒ n( n − 1) ( n − 2 ) ( n − 3)( n − 4) (b) Neither AB nor BA exists
= 7 × 6× 5× 4× 3 (c) AB exists but BA does not exist For constant term
(d) AB does not exist but BA exists 10 − 5r
∴ n=7 = 0 ⇒ 10 − 5r = 0
 1 2 2
Now, C ( n + 1, r + 1) = n + 1C r + 1    1 2
Ê (c)We have, A =  2 3 and B =   ∴ r=2
7 +1  3 4  2 1
= C 5 + 1 = 8C 6   Putting the value of r, in Eq. (i),
8! 8×7 order of A = 3 × 2 and order of B = 2 × 2 405 = 10C 2.( − k )2
= =
6 !2 ! 2 Q Number of column of A = Number of 10 !
⇒ 405 = × k2
= 28 row of B 2 ! 8!
∴ AB exists. 10 × 9 2
24. How many terms are there in the ⇒ 405 = .k
and number of column of B ≠ Number of 2
expansion of 405
raw of A ⇒ k2 =
( 1 + 2x + x 2 ) 5 + ( 1 + 4y + 4y 2 ) 5 ? ∴BA does not exist. 45
(a) 12 (b) 20 ⇒ k2 = 9
Hence, AB exists but BA does not exist.
(c) 21 (d) 22 ⇒ k=± 3
NDA/NA Solved Paper 2019 (II) 5

30. What is C ( 47, 4 ) + C (51, 3) + C (50, 3) 33. Let Sn be the sum of the first n terms 36. If x log 7 x > 7 where x > 0, then which
+ C ( 49, 3) + C ( 48, 3) + C ( 47, 3) equal of an AP. If S 2n = 3n + 14n 2 , then one of the following is correct?
to? 1 
what is the common difference? (a) x ∈ ( 0, ∞ ) (b) x ∈  , 7 
(a) C( 47, 4) (b) C( 52, 5) 7 
(a) 5 (b) 6
(c) C( 52, 4) (d) C( 47, 5)  1
(c) 7 (d) 9 (c) x ∈  0,  ∪ (7, ∞ )
Ê (c) C ( 47, 4) + C ( 51, 3) + C ( 50, 3)  7
Ê (c) S 2n = 3n + 14n (S n be the sum of first
2
1 
+ C ( 49, 3) + C ( 48, 3) + C ( 47, 3) n terms of an AP) (d) x ∈  , ∞ 
7 
= 47C 4 + 51C 3 + 50C 3 + 49C 3 3 7
⇒ S 2n = .(2 n) + (2 n)2
Ê (b) x 7 > 7 where x > 0.
log x
+ 48
C3 + 47
C3 2 2
Put 2 n = n Taking log on base 7 both sides
= C3 +
51 50
C3 + C3 +
49 48
C3
3n 7 n 2 log 7 x. log 7 x > log 7 7
we get, S n = +
+ 47
C3 + 47
C4 2 2 [Q log a mn = nlog a m]
= C3 +
51 50
C3 + C3 +
49
C3 +
48 48
C4 ∴ Tn = S n − S n −1
⇒ (log 7 x )2 > 1 [Q log a a = 1]
n +1
[Q nC r + nC r − 1 = Cr ] 3 7 n2 3 7
= n+ − ( n − 1) − ( n − 1)2 ⇒ log 7 x > ( ± 1)
2 2 2 2
= C3 +
51 50
C3 + C3 +
49 49
C4 ∴ x > 71 ⇒ x > 7
3 7 3 3 7 7 7
= C3 +
51 50
C3 + 50
C4 = n + n2 − n + − n2− + .2 n and x < 7 −1 ⇒ x <
1
2 2 2 2 2 2 2 7
= C3 +
51 51
C4 Tn = 7 n − 2 1 
= C 4 = C( 52, 4)
52 Hence, x ∈  , 7 
Put n = 1, 2, .... 7 
T1 = 7(1) − 2 = 5
31. Let a, b, c be in AP and k ≠ 0 be a real 37. How many real roots does the
T2 = 7(2 ) − 2 = 12
number. Which of the following are equation x 2 + 3| x | + 2 = 0 have?
∴ d = T2 − T1 = 12 − 5 = 7
correct? (a) Zero (b) One
1. ka, kb, kc are in AP 34. If 3rd, 8th and 13th terms of a GP are (c) Two (d) Four
2. k − a, k − b, k − c are in AP p , q and r respectively, then which Ê (a) Given equation, x + 3| x| + 2 = 0
2

a b c one of the following is correct? Case I x 2 + 3 x + 2 = 0 (when x > 0)


3. , , are in AP
k k k (a) q 2 = pr (b) r 2 = pq
(c) pqr = 1 (d) 2q = p + r ⇒ x2 + x + 2 x + 2 = 0
Select the correct answer using the
code given below. Ê (a) Let first term and common ratio of a ⇒ x( x + 1) + 2( x + 1) = 0
(a) 1 and 2 only (b) 2 and 3 only GP be a and R. ⇒ ( x + 1) ( x + 2 ) = 0
(c) 1 and 3 only (d) 1, 2 and 3 ∴ T3 = aR 2 = p …(i) ∴ x = − 1, − 2
Ê (d) a, b,c are in AP. T8 = aR 7 = q …(ii) Hence, no real roots because x > 0.
We know that equal number addition, T13 = aR12 = r …(iii) Case II x 2 − 3 x + 2 = 0 (when x < 0)
subtraction and multiply, divide, by equal ⇒ x2 − 2 x − x + 2 = 0
number of each term of an AP, the Multiplying of Eqs. (i) and (iii)
( aR 2 ) ( aR12 ) = pr ⇒ x( x − 2 ) − 1( x − 2 ) = 0
resultent, series be an AP.
⇒ ( x − 2 )( x − 1) = 0
∴ka, kb, kc are in AP (multiplying by k). ⇒ 2 14
a R = pr
∴ x = 1, 2
k − a, k − b, k − c are in AP (subtraction ⇒ ( aR ) = pr
7 2
a b c Hence, no real roots because x < 0.
from k) and , , are in AP (divide by k)
k k k ⇒ q 2 = pr [from Eq. (ii)] ∴ The number of real roots of given
equation is zero.
Hence, option (d) is correct answer.
35. What is the solution of x ≤ 4, y ≥ 0
32. How many two-digit numbers are 38. Consider the following statements in
and x ≤ − 4, y ≤ 0?
divisible by 4? respect of the quadratic equation
(a) x ≥ − 4, y ≤ 0 (b) x ≤ 4, y ≥ 0
(a) 21 (b) 22 (c) x ≤ − 4, y = 0 (d) x ≥ − 4, y = 0 4( x − p )( x − q ) − r 2 = 0,
(c) 24 (d) 25
Ê (c) Given inequalities where p , q and r are real numbers.
Ê (b) Series of two-digit number that x ≤ 4, y ≥ 0 …(i) 1. The roots are real.
divisible by 4 is and x ≤ − 4, y ≤ 0 …(ii) 2. The roots are equal, if p = q and
12, 16, 20, ........., 96 Possible value of x and y. r = 0.
This series is an AP
x = { 4, 3, 2, 1, 0, − 1, − 2, − 3, − 4, − 5, ...} Which of the above statements is/are
Here, A = 12,d = 4, l = 96 correct?
y = { 0, 1, 2, 3, 4 .....} …(i)
Let total number of terms be n. (a) Only 1 (b) Only 2
and x = { −4, − 5, − 6, − 7, ...}, (c) Both 1 and 2 (d) Neither 1 nor 2
∴ l = a + ( n − 1)d y = { 0, − 1, − 2, − 3, − 4 ...} …(ii)
⇒ 96 = 12 + ( n − 1) 4 Ê (c) Given quadratic equation,
Take combine (i) and (ii), 4( x − p) ( x − q ) − r = 0
2

⇒ 84 = ( n − 1)4 x = { −4, − 5, − 6, − 7 ... }, y = 0 ⇒ 4 x 2 − ( 4q + 4 p) x + 4 pq − r 2 = 0


⇒ n − 1 = 21 x ≤ − 4, y = 0.
or Comparing it Eq. by ax 2 + bx + c = 0
⇒ n = 21 + 1 = 22
6 NDA/NA Solved Paper 2019 (II)

a = 4, b = − 4 ( p + q ), c = 4 pq − r 2 and tanθ =
P 21k 21
= = Ê (c) Given, radius of circle = 1 unit
b − 4ac = 16 ( p + q ) − 4 × 4 ( 4 pq − r )
2 2 2 B 20k 20
Angle subtends at the centre of circle by
Now, 4 sec θ + 4 tan θ
= 16 p2 + 16 q 2 + 32 pq − 64 pq + 16r 2 29 21 chord = θ
=4× + 4×
= 16 p2 + 16 q 2 − 32 pq + 16r 2 20 20 We know that, length of chord
50 θ θ θ
= 16( p − q )2 + 16r 2 =4× = 10 = 2 r sin = 2 × 1 sin = 2 sin
20 2 2 2
Q b 2 − 4ac will be positive.
42. Consider the following statements 45.   1 
So, the roots are real. What is tan 2 tan −1    equal to?
1. cos θ + sec θ can never be equal to   3 
If p = q and r = 0, then b 2 – 4ac = 0
1.5. 2 3 3
So, the roots are equal. 2. tan θ + cot θ can never be less (a) (b) (c)
3 4 8
Hence, the statements both 1 and 2 are than 2. 1
correct. (d)
Which of the above statements is/are 9
39. Let S = {2, 4, 6, 8, ......,20}. correct?  −1  1  
(a) Only 1 (b) Only 2 Ê (b) tan 2 tan   
What are the maximum number of (c) Both 1 and 2 (d) Neither 1 nor 2  3 
subsets of S? 
Ê (b) We know that, − 1 ≤ cos θ ≤ 1 and 1 
(a) 10 (b) 20  2 × 
 3 
(c) 512 (d) 1024 − 1 ≤ sec θ ≤ ∞ but cos θ = sec θ ifθ = 0 = tan  tan−1 2
and θ = 180°   1
Ê (d) S = {2, 4, 6, 8, ......., 20} 1−   
∴ − 2 ≤ cos θ + sec θ ≤ ∞   3  
Here, number of elements of set 
S = 10 ( n) So, cos θ + sec θ = 15 . is possible.  −1 −1 2 x 
∴Maximum number of subsets of set and again 0 ≤ tan θ ≤ ∞ and Q 2 tan x = tan 
 1 − x2 
S = 2 n = 2 10 = 1024 0 ≤ cot θ ≤ ∞ , but tan θ = cot θ
2
if θ = 45°   2 ×9 3
−1
40. A binary number is represented by = tan tan  3  = =
∴ 2 ≤ tanθ + cot θ ≤ ∞  8 3 × 8 4
(cdccddcccddd )2 , where c > d . What So, tan θ + cot θ can never be less than 2.  9
is its decimal equivalent? Hence, only the Statement 2 is correct.
(a) 1848 (b) 2048 46. What is the scalar projection of
(c) 2842 (d) 2872 43. A ladder 9 m long reaches a point 9 m a = $i − 2$j + k$ on b = 4 i$ − 4 $j + 7 k$ ?
Ê (d) Binary number = (cdccddcccddd )2 below the top of a vertical flagstaff.
6 919 6
where, c > d . We know that only two bit From the foot of the ladder, the (a) (b) (d)(c)
9 19 9 19
(digits) 0 and 1 be any binary number. elevation of the flagstaff is 60°. What
Ê (b) a = i − 2 j + k, b = 4 i − 4 j + 7k
$ $ $ $ $ $
∴Given binary number is the height of the flagstaff?
= (101100111000)2 (a) 9 m (b) 10.5 m Projection of a on b
(c) 13.5 m (d) 15 m a ⋅b
= (1 × 2 11 + 0 × 2 10 + 1 × 2 9 + 1 × 2 8 + =
Ê (*) Let AP be a ladder and QR be a vertical |b|
+ 0 × 27 + 0× 26 + 1 × 25 + 1 × 24 flagstaff. P is a point 9 m below the top on ( $i − 2 $j + k$ ) . ( 4$i − 4$j + 7k$ )
+ 1 × 2 3 + 0 × 2 2 + 0 × 2 1 + 0 × 2 0 )10 flagstaff. A is the foot of ladder and h is the =
height of point P from the ground. 16 + 16 + 49
= (2048 + 512 + 256 + 32 + 16 + 8)10 4+ 8+7 19
∴ AP = 9 m, PR = 9 m, PQ = hm = =
= (2872 )10 81 9
PQ
In ∆ AQP, sin θ =
41. If cosec θ = 29 , where 0 < θ < 90°, AP
47. If the magnitude of the sum of two
21 h 3 h
⇒ sin 60° = ⇒ = non-zero vectors is equal to the
then what is the value of 9 2 9
4 sec θ + 4 tan θ? magnitude of their difference, then
9 3
⇒ 9 3 = 2h ⇒ h = which one of the following is
(a) 5 (b) 10 (c) 15 (d) 20 2
9 × 173
. 15.57
correct?
29 = = = 7.7 m
Ê (b) Given, cosec θ = 2 2
(a) The vectors are parallel
21 (b) The vectors are perpendicular
where, 0 < θ < 90° ∴Height of flagstaff
(c) The vectors are anti-parallel
H 29 = h + 9 = 7.7 + 9 (d) The vectors must be unit vectors
Qcosec θ = = = k (let) = 167 . m
P 21
Ê (b) Let a and b are the two non-zero
∴ H = 29k, P = 21 k 44. What is the length of the chord of a vectors.
∴ B = ( H )2 − ( P )2 = (29k )2 – (21k )2 unit circle which subtends an angle θ According to the question,
at the centre? |a + b| = |a − b|
= 841k 2 − 441k 2 θ θ
(a) sin   (b) cos   ⇒ a 2 + b 2 + 2a ⋅ b = a 2 + b 2 − 2a ⋅ b
= 400k 2 = 20k 2 2
θ θ ⇒ 4a ⋅ b = 0 ⇒ a ⋅ b = 0
H 29k 29 (c) 2 sin   (d) 2 cos  
∴sec θ = = = 2 2 So, a and b are perpendicular.
B 20k 20
NDA/NA Solved Paper 2019 (II) 7

48. Consider the following equations for Ê (b) Given,|a| = 1,|b| = 1 [Q Distance between two points ( x1, y1 )
We know that, and ( x2, y2 )
two vectors a and b.
|a − b|2 = |a|2 + |b|2 − 2a ⋅ b = | ( x2 − x1 )2 + ( y2 − y1 )2|
1. (a + b) . (a − b) = | a| 2 − | b| 2
⇒|a − b| = 1 + 1 − 2 |a||b|cos θ
2
m2 cos 2 2β + m2 cos 2 2α
2. (| a + b| ) (| a − b| ) = | a| 2 − | b| 2
⇒|a − b|2 = 2 − 2 cos θ
= | − 2 m2 cos 2β cos 2α + m2 sin2 2β
3. | a ⋅ b| + | a × b| = | a 2 | b| 2 ⇒|a − b|2 = 2 (1 − cos θ)
+ m2 sin2 2α − 2 m2 sin 2β sin2α|
Which of the above statements are  θ
⇒|a − b|2 = 2  1 − 1 + 2 sin2 
correct?  2 (cos 2 2β + sin2 2β )
(a) 1, 2 and 3 (b) Only 1 and 2  θ
= | m + (cos 2α + sin 2α )
2 2
⇒|a − b|2 = 2 ⋅  2 sin2 
(c) Only 1 and 3 (d) Only 2 and 3  2 − 2 (cos 2β cos 2α
Ê (c) I : (a + b ) ⋅ (a − b ) ⇒ sin2
θ |a − b|2
= + sin 2β sin 2α )|
= a ⋅ a − a ⋅ b + b⋅ a − b⋅ b 2 4
= | m 1 + 1 − 2 cos (2α − 2β )|
|a|2 − a . b + a . b − |b|2 51. The equation ax + by + c = 0 = | m 2 [1 − cos 2(α − β )]|
[Q a ⋅ b = b ⋅ a ] represents a straight line 2(α − β )
= | m 2 [1 − 1 + 2 sin2 |
= |a|2 − |b|2 (a) for all real numbers, a, b and c 2
(b) only when a ≠ 0
So, Statement 1 is correct. (c) only when b ≠ 0 = | m 2 × 2 sin2 (α − β )|
2. (|a + b|) (|a − b|) = |a + b||a − b| (d) only when at least one of a and b is = | 2 m sin (α − β )|
≠ |a|2 − |b|2 non-zero.
54. An equilateral triangle has one
Ê (d) The equation ax + by + c = 0
So, Statement 2 is not correct. vertex at ( −1, − 1) and another vertex
represents a straight line only when at
3.|a ⋅ b|2 + |a × b|2 = ||a||b|cos θ|2 at( − 3, 3 ). The third vertex may lie
least one of a and b is non zero.
+ ||a||b|sin θ|2 on
52. What is the angle between the lines
= |a | |b| cos θ + |a| |b|2 sin2 θ
2 2 2 2 (a) ( − 2 , 2 ) (b) ( 2 , − 2 )
x cos α + y sin α = a and (c) (1, 1) (d) (1, − 1)
= |a | |b| (cos θ + sin θ)
2 2 2 2
x sin β − y cos β = a ?
(a) β − α (b) π + β − α Ê (c) Consider two vertices of an equilateral
= |a |2 |b|2 [Q cos 2 θ + sin2 θ = 1] ( π + 2β + 2α ) ( π − 2β + 2α ) triangle are A( −1, − 1) and B( − 3, 3 ).
(c) (d) Let third vertex x be C ( x, y).
So, statement 3 is correct. 2 2
Hence, only Statements 1 and 3 are correct. Q ∆ABC is equilateral
Ê (d) The equations of given lines
x cos α + y sin α = a … (i) ∴ AC = AB ( x + 1)2 + ( y + 1)2
49. Consider the following statements. and x sin β − y cos β = a … (ii)
1. The magnitude of a × b is same as − cos α = ( − 3 + 1)2 + ( 3 + 1)2
Slope of Eq. (i), m1 = = − cot α
the area of a triangle with sides a sin α ⇒ x + 1 + 2 x + y2 + 1 + 2 y
2

and b π  = 3 + 1− 2 3 + 3 + 1 + 2 3
= tan  + α 
2. If a × b = 0, where a ≠ 0 ,b ≠ 0, 2 
⇒ x 2 + y2 + 2 x + 2 y + 2 = 8
then a = λ b. Slope of Eq. (ii), m2 =
− sin β
= tan β
Which of the above statements is/are − cos β ⇒ x 2 + y2 + 2 x + 2 y = 6
correct? Let θ be the angle between the lines, then From option only point (1, 1) is satisfying
(a) Only 1 (b) Only 2 m − m2 of it equation. Hence, the third vertex may
(c) Both 1 and 2 (d) Neither 1 nor 2 tan θ = 1 lie on (1, 1).
1 + m1m2
Ê (b) 1. We know that, π  55. If the angle between the lines joining
Area of triangle with sides a and b tan  + α  − tan β
2  the end points of minor axis of the
1
= |a × b| =
1 + tan ( π − α ) tan β x2 y2
2 ellipse 2 + 2 = 1 with one of the
π 
So, statement 1 is not correct. tan θ = tan  + α − β  a b
2. a × b = 0, where a ≠ 0, b = 0, 2  π
π π + 2α − 2β its foci is , then what is the
So, a and b are parallel. θ= + α −β= 2
⇒ a = λb 2 2 eccentricity of the ellipse?
So, Statement 2 is correct. 1 1 3 1
53. What is the distance between the (a) (b) (c) (d)
Hence, only statement 2 is correct. points P (m cos 2α, m sin 2α ) and 2 2 2 2 2
50. If a and b are unit vectors and θ is the Q (m cos 2β, m sin 2 β )? x2 y2
Ê (b) Equation of the ellipse, 2 + 2 = 1
angle between them, then what is (a)|2 m sin (α − β )| (b)|2 m cos (α − β )| a b
(c)| m sin (2α − 2β )|
 θ End points of minor axis are
sin 2   equal to? (d)| m sin (2α − 2β )| ( 0, b ), ( 0, − b ) and one foci is ( ae, 0)
 2
Ê (a) Given points, p( m cos 2α, m sin 2α ) 0−b b
|a + b|2 |a − b|2 Slope of line BS = =− ( m1 )
(a) (b) and Q ( m cos 2β, m sin 2β ) ae − 0 ae
4 4 ( m cos 2β − mcos 2α )2 +
∴PQ = | 0+ b b
|a + b|2 |a − b|2 Slope of line B′ S = = ( m2 )
(c) (d) ( m sin 2β − m sin 2α )2| ae − 0 ae
2 2
8 NDA/NA Solved Paper 2019 (II)

According to the question, angle between From option, point (2, 5, 5) is satisfying 3 3 1
π ∴ l1 = , m1 = , n1 =
BS and B′ S is . the given equation of line. 4 4 2
2 Q 2 − 1 = 5 − 3 = 5 + 2 ⇒ 1 = 1 = 1 3 3 1
  and l 2 = − , m2 = − , n2 =
i.e. BS and B′ S are perpendicular, 1 2 7 4 4 2
∴ m1m2 = − 1 Hence, the coordinates of required point We know that,
−b b (2, 5, 5). cos θ = | l1l 2 + m1m2 + n1n2|
× = − 1 ⇒ b 2 = a 2e 2 … (i) 3 9 1
ae ae ⇒ cos θ = − − +
b2
58. If the line x − 4 = y − 2 = z − k lies 16 16 4
We know that, e = 1 − 2 1 1 2 −3 − 9 + 4 −8
a = =
on the plane 2x − 4y + z = 7, then 16 16
⇒ ae =a −b
2 2 2 2
what is the value of k? 1
⇒ a 2e 2 = a 2 − a 2e 2 [from Eq. (i)] cos θ = = cos 60°
(a) 2 (b) 3 2
⇒ 2 a 2e 2 = a 2 ⇒ e 2 =
1 (c) 5 (d) 7 ∴ θ = 60°
2 Ê (d) Equation of line
1 60. If the points ( x , y , − 3), (2, 0, − 1) and
⇒ e = x− 4 y−2 z−k
2 = = = =r C ( 4, 2, 3) lie on a straight line, then
1 1 2 what are the values of x and y
56. A point on a line has coordinates ∴ ( r + 4, r + 2, 2 r + k ) point lies on the respectively?
( p + 1, p − 3, 2p ) where p is any line. (a) 1, − 1 (b) −1, 1
This line lies on the plane (c) 0, 2 (d) 3, 4
real number. What are the direction
cosines of the line? 2 x − 4y + z = 7 Ê (a) Points, A( x, y, − 3), B (2, 0, − 1) and
1 1 1 1 1 1 Then, the point ( r + 4, r + 2, 2 r + k ) lies ( 4, 2, 3). These points lie on a straight
(a) , , (b) , ,
2 2 2 2 2 2 on the plane, we get line, then direction ratios of AB = λ
1 1 1 2( r + 4) − 4 ( r + 2 ) + (2 r + k ) = 7 (direction ratios of BC )
(c) , ,−
2 2 2 ⇒ 2 r + 8 − 4r − 8 + 2 r + k = 7 ∴(2 − x, 0 − y, − 1 + 3)
(d) Cannot be determined due to ⇒ k=7 = ( 4 − 2, 2 − 0, 3 + 1)
insufficient data
Hence, the value of k is 7. ⇒ (2 − x, − y, 2 ) = (2, 2, 4)
Ê (d) Coordinate of a point on a line is ⇒ (2 − x, − y, 2 ) = 2(1, 1, 2 )
( p + 1, p − 3, 2 p), p is any real 59. A straight line passes through the
Comparing both sides,
number. point (1, 1, 1) makes an angle 60°
with the positive direction of Z -axis, 2 − x =1 ⇒ x = 1
Equation of a line, whose direction ratios
are a, b and c and passing through the and the cosine of the angles made by and − y = 1 ⇒ y = − 1
point ( x1, y1, z1 ) it with the positive directions of the 61. What is the minimum value of
x − x1 y − y1 z − z1 Y -axis and the X -axis are in the ratio
= = =r a2 b2
a b c 3 : 1. What is the acute angle 2
+ where a > 0 and
∴ ( ar + x1, br + y1, cr + z1 ) any point on cos x sin 2 x
between the two possible positions
the line. b > 0?
of the line?
According to the questions, (a) ( a + b )2 (b) ( a − b )2
(a) 90° (b) 60° (c) a 2 + b 2 (d)| a 2 + b 2|
( ar + x1, br + y1, cr + z1 ) (c) 45° (d) 30°
a2 b2
= ( p + 1, p − 3, 2 p) Ê (b) Let the straight line makes the angle Ê (*) Let p = +
cos x sin2 x
2
∴ ar = p + 1 − x1 … (i) with X-axis, Y-axis and Z-axis be α, β
and γ. = a 2 sec 2 x + b 2 cosec 2 x
br = p − 3 − y1 … (ii)
cos β 3 − 2 ab sec x cosec x
cr = 2 p − z1 …(iii) ∴γ = 60° and =
cos α 1 + 2 ab sec x cosec x
Squaring and adding of (i), (ii) and (iii)
= ( a sec x − b cos ec x )2
( a 2 + b 2 + c 2 )r 2 = ( p + 1 − x1 )2 If l, m and n are the direction cosines of
the lines, then + 2 ab sec x cosec x
+ ( p − 3 − y1 ) + ( 2 p − z1 )
2 2
1
n = cos γ = cos 60° = For minimum value of p,
We can not find the values of a, b and c. 2
a sec x − b cos ec x = 0
Hence, the direction cosines of the line m cos β 3
and = = ⇒ a sec x = b cos ec x
can not be determined due to insufficient l cos α 1
sec x b
data. ⇒ =
m 3
⇒ = = k (Let) cos ec x a
57. A point on the line l 1 b
x −1 y −3 z +2 ∴ m = 3k, l = k ⇒ tan x =
= = a
1 2 7 We know that, l 2 + m2 + n2 = 1 b a
∴ sin x = , cos x =
has coordinates 1
k 2 + 3k 2 + = 1 a + b
2 2
a + b2
2

(a) (3, 5, 4) (b) (2, 5, 5) 4 ∴Minimum value of p


(c) ( − 1, − 1, 5) (d) (2, − 1, 0) 1 3
⇒ 4k 2 = 1 − = a 2 ( a 2 + b 2 ) b 2( a 2 + b 2 )
Ê (b) Equation of the line 4 4 = +
a2 b2
x−1 y− 3 z+ 2 3 3
= = ⇒ k2 = ⇒k=± = 2( a + b )
2 2
1 2 7 16 4
NDA/NA Solved Paper 2019 (II) 9

A A
62. If the angles of a triangle ABC 65. If 2 tan A = 3 tan B = 1, then what is cos 2 − sin2
2 cos A
2 2
are in AP and b : c = 3 : 2, then tan ( A − B ) equal to? =
A A
=
A A
1 1 sin cos 2 sin cos
what is the measure of angle A? (a) (b) 2 2 2 2
5 6 2 cos A
(a) 30° (b) 45° = = 2 cot A
1 1 sin A
(c) 60° (d) 75° (c) (d)
Ê (d) Angles of a triangle ABC are in AP, 7 9 69. What is cot A + cosec A equal to?
then 2 B = A + C Ê (c) Given, 2 tan A = 3 tan B = 1  A
(a) tan  
 A
(b) cot  
We know that, A + B + C = 180° 1 1 2 2
∴tan A = , tan B =
⇒ 3B = 180° ⇒ B = 60° 2 3  A  A
(c) 2 tan   (d) 2 cot  
sin A sin B sin C Now, tan ( A − B) 2 2
By sine rule, = =
a b c tan A − tan B Ê (b) cot A + cos ec A
=
sin A sin 60° sin C 1 + tan A tan B cos A + 1
⇒ = = =
cos A
+
1
=
a b c 1 1 3−2 sin A sin A sin A
sin 60° sin C −
= 2 3 1
Take II and III, = = 6 = A
b c  1  1 6+1 7 2 cos 2 − 1+ 1
1+     = 2
sin 60° b 3 /2 3  2   3 6 A A
⇒ = ⇒ = 2 sin cos
sin C c sin C 2 2 2
66. What is cos 80° + cos 40° − cos 20°
[Q Given, b : c = 3 : 2 ] equal to? 2 cos 2
A
cos
A
2 2  A
3 2 1 = = = cot  
⇒ sin C = × = (a) 2 (b) 1 A A A 2
2 3 2 (c) 0 (d) −19 2 sin cos sin
2 2 2
⇒ sin C = sin 45° ⇒ C = 45° Ê (c) cos 80° + cos 40° − cos 20°
∴ A = 180° − ( B + C ) 70. What is tan 25° tan 15° + tan 15°
= 180° − ( 60° + 45° ) = 75° 80° + 40° 80° − 40° tan 50° + tan 25° tan 50° equal to?
= 2 cos . cos
2 2 (a) 0 (b) 1
63. If tan A − tan B = x and − cos20° (c) 2 (d) 4
cot B − cot A = y , then what is the Ê (b)Q tan 50° = tan( 90° − 40° )
= 2 cos 60° cos 20° − cos 20°
value of cot ( A − B )? 1
= 2 × cos 20° − cos 20° ⇒ tan 50° = cot 40°
1 1 1 1
(a) + (b) − 2 1
x y y x ⇒ tan 50° =
xy 1 = cos 20° − cos 20° = 0 tan 40°
(c) (d) 1 + 1
x + y xy 67. If angle C of a triangle ABC is a right ⇒ tan 50° =
tan (25° + 15° )
Ê (a) Given, tan A − tan B = x … (i) angle, then what is tan A + tan B
1 − tan25° tan15°
andcot B − cot A = y … (ii)
equal to ? ⇒ tan 50° =
a −b 2 2
a 2 tan25° + tan15°
From Eq. (i), tan A − tan B = x (a) (b)
ab bc ⇒ tan25° tan 50°+ tan15° tan 50°
1 1
⇒ − = x b2 c2 = 1 − tan25° tan15°
cot A cot B (c) (d)
ca ab ⇒ tan25° tan15° + tan15° tan 50°
cot B − cot A
⇒ = x Ê (d) In ∆ABC , ∠C = 90° + tan25° tan 50° = 1
cot A cot B
∴ c 2 = a2 + b 2
y 71. What is the area of the region
⇒ cot A cot B = [from Eq. (ii)] [by Pythagoras theorem] … (i)
x a b bounded by | x | < 5, y = 0 and y = 8?
cot A cot B + 1 tan A = , tan B = (a) 40 sq units (b) 80 sq units
Now, cot ( A − B) = b a
(c) 120 sq units (d) 160 sq units
cot B − cot A Now, tan A + tan B = +
a b
y b a Ê (b) Given curve y = 0 and y = 8 and
+1 a2 + b 2 c 2 | x| < 5
y+ x 1 1 = = [from Eq. (i)]
= x = = +
y xy x y ab ab Case I When x < 0, then

64. What is 68. What is cot  A  − tan  A  equal area of the region bounded
 2  2 0 0
= ∫−5 0 dx − ∫−5 8 dx = 0 − 8 [ x]−5
0
sin (α + β ) − 2 sin α cos β +
sin (α − β ) equal to? to?
= − 8 [0 + 5] = − 40
(a) 0 (b) 2 sin α (a) tan A (b) cot A
(c) 2 tan A (d) 2 cot A = 40 sq units
(c) 2 sin β (d) sin α + sin β
A A [Qarea will not be negative]
Ê (a) sin (α + β ) − 2 sin α cos β Ê (d) cot − tan
2 2 Case II when x > 0, then
+ sin (α − β ) A A Area of the region bounded
cos sin
= sin α cos β + cos α sin β 5 5
= 2 − 2 = ∫0 0 dx − ∫0 8 dx = 0 − 8 [ x]0
5
− 2 sin α cos β + sin α cos β A A
sin cos
− cos α sin β = 0 2 2 = − 8[5 − 0] = − 40 = 40 sq units
10 NDA/NA Solved Paper 2019 (II)

∴Required area = 40 + 40 = 80 sq units 74. What is the degree of the differential Ê Solutions (Q. Nos. 76-78)
Y Given curve y = me mx where m > 0
equation
y=8
2 d y 
2
d 3y  dy  4 QCurve intersects Y-axis at a point P, then
+   − x   = 0? x=0
dx 3  dx   dx 4 
Case I Case II ∴ y = me 0 ⇒ y = m
(a) 1 (b) 2
(c) 3 (d) 4 ∴Point P ( 0, m)
Now, differentiation w.r.t x of given curve,
X¢ y=0 X Ê (a) Given differential equation, dy
d 3y  dy 
2
 d 4 y = m.e mx . m
+   − x2  4  = 0 dx
x=–5 x=5 3
dx  dx   dx  dy
= m2e mx
Y¢ 1  d 3y
2
d4y 1  dy  dx
⇒ −  3 − 2   = 0
72. Consider the following statements in dx 4
x  dx 
2
x  dx  at point P( 0, m),
dy
= m2e 0 = m2
respect of the function dx
We know that power of the heighest order
 1 Ê 76. (b) Slope of the curve at the point
f ( x ) = sin   for x ≠ 0 andf (0) = 0 : of differentiation is the degree of
x differential equation. P( 0, m)
1. lim f ( x ) exists  dy 
So, the degree of it equation is 1. =   at point P ( 0, m) = m2
x→ 0  dx 
2. f ( x ) is continuous at x = 0 75. Which one of the following is the
second degree polynomial function Ê 77. (c) Let the tangent makes the angle
Which of the above statement is/are with X-axis be θ, then
correct? f ( x ) where, f (0) = 5, f ( −1) = 10 and  dy 
(a) Only 1 (b) Only 2 f (1) = 6? tan θ =   at P ( 0, m)
 dx 
(c) Both 1 and 2 (d) Neither 1 nor 2 (a) 5x 2 − 2 x + 5 (b) 3x 2 − 2 x − 5
sin 1 , x ≠ 0 (c) 3x 2 − 2 x + 5 (d) 3x 2 − 10x + 5 ⇒ tan θ = m2 ⇒ θ = tan−1 m2

Ê (d) Given, f( x) =  x Ê (c) From the option (c), Now, the tangent will make the angle with
 0 ,x=0 Y-axis
f( x ) = 3 x 2 − 2 x + 5
 1  1  π π
LHL = lim sin  = lim sin  f( 0) = 3( 0)2 − 2( 0) + 5 = −θ = − tan−1 m2
x→ 0−  x  h→ 0  0 − h  2 2
=5 π
 1
= lim − sin  = − sin ∞ = cot −1 m2 Q tan−1 x + cot −1 x = 
h→ 0  h f( −1) = 3( −1)2 − 2( −1) + 5  2 
= − (a rational number) = 3 + 2 + 5 = 10  1 
[Q sin θ lies between −1to1] = sin−1  
and f(1) = 3(1)2 − 2(1) + 5  1 + m4 
 1  1   
RHL = lim sin  = lim sin  = 3−2 + 5= 6
x→ 0 +  x  h → 0  0 + h   
Hence, the required polynomial  1 
 1 Q cot −1 x = sin
= lim sin  = sin ∞ f( x ) = 3 x 2 − 2 x + 5.   1 + m2  
h→ 0  h   
= a rational number Ê 78. (d) Equation of tangent to curve at P is
[Q sin θ lies between − 1to1] Directions (Q. Nos. 76-78) Read the  dy 
y − y1 =   ( x − x1 )
Q LHL ≠ RHL following information and answer the  dx  ( x
1 , y1 )
So, f( x ) does not exists. three items that follow .
A curve y = memx where m > 0 intersects ⇒ y − m = m2 ( x − 0)
Q f( x ) = 0 at x = 0
⇒ y = m2 x + m
∴LHL ≠ RHL ≠ f( 0) Y-axis at a point P.
So, f( x ) is not continuous. 76. What is the slope of the curve at the
Hence, the statements neither 1 nor 2 Directions (Q. Nos. 79 and 80) Read
point of intersection P? the following information and answer
correct.
(a) m (b) m2 the two items that follow.
73. What is the value of lim sin x ° ? (c) 2 m (d) 2 m2
Let f ( x ) = x 2, g( x ) = tan x and
x→ 0 tan 3x ° 77. How much angle does the tangent at h( x ) = log x.
1 1 1
(a) (b) (c) (d) 1 P make with y-axis? 79. π
4 3 2 (a) tan−1 m2 For x = , what is the value of
 sin x  2
sin x °  x×  (b) cot −1(1 + m2 )
=  x  [ho( gof )]( x )?
Ê (b) lim
x→ 0 tan 3 x °
lim
x→ 0 tan 3 x    π π
(c) sin−1  
3x × 1 (a) 0 (b) 1 (b) (d)
 3x   1 + m4  4 2
 
 sin x 
  (d) sec −1 1 + m4 80. What is [ fo( fof )](2) equal to ?
1  x  1
= lim =
3 x→ 0  tan 3 x  3 (a) 2 (b) 8
  78. What is the equation of tangent to
 3x  (c) 16 (d) 256
the curve at P ?
Q lim sin θ = 1 and lim tanθ = 1 Ê Solutions (Q. Nos. 79 and 80) Given,
 (a) y = mx + m (b) y = − mx + 2 m f( x ) = x , g ( x ) = tan x and h( x ) = log x
2
 θ → 0 θ θ→0 θ
(c) y = m2x + 2 m (d) y = m2x + m
NDA/NA Solved Paper 2019 (II) 11

Ê 79. (a) ( gof ) ( x) = g { f( x)} = tan x


2 Differentiation w.r.t. x, we get
Ê (d) Suppose,
dx  1
Now, [ho( gof )] ( x ) = h {( gof )( x )} I= ∫ x (1 + ln x)n (where n ≠ 1)
dy
(1 + log x ).1 − x  0 + 
 x
= log (tan x 2 ) =
π Let 1 + ln x = t
dx (1 + log x )2
for x =
dy 1 + log x − 1
2 Diff. w.r.t. x, we get =
 π  π 1 dt dx dx (1 + log x )2
[ho( gof )]   = log tan   0+ = ⇒ = dt
 2   4 x dx x dy log x
=
= log 1 = 0 dt t −n + 1 dx (1 + log x )2
∴ I=∫ n= +c
t −n+1
Ê 80.(d) ( fof ) ( x) = f{ f( x)} dy
Ê 84. (a)Q =
log x
= (x ) = x
2 2 4
=−
1
+c
dx (1 + log x )2
( n − 1) t n −1 dy log 1
Now, [fo( fof )] ( x ) = f{( fof ) ( x )} At x = 1, =
1 dx (1 + log 1)2
= ( x 4 )2 = x 8 =− +c
( n − 1) (1 + log x )n − 1 0
∴[fo( fof )] (2 ) = 2 = 256
8 =0 = [Q log 1 = 0]
1
83. Which one of the following is the dy log x
81. What is Ê 85. (b)Q =

dx
equal to? differential equation that represents dx (1 + log x )2
2x 2 − 2x + 1 1
the family of curves y = 2 , Differentiation. w.r.t. x, we get
tan−1(2 x − 1) 2x − C
(a) +c 1
2 (1 + log x )2. − (log x ).
−1 where C is an arbitrary constant? x
(b) 2 tan (2 x − 1) + c
dy dy 1  1
tan−1 (2 x + 1) (a) = 4 xy 2 (b) = 2 (1 + log x )  0 + 
(c) +c 
dx dx y d 2y x
−1
2 =
(d) tan (2 x − 1) + c (c)
dy
= x 2 y (d)
dy
= − 4 xy 2 dx 2 (1 + log x )4
dx dx dx
Ê (d) Let I = ∫ 2
1
(1 + log x ) (1 + log x − 2 log x )
2x −2x + 1 Ê (d) The differential equation of family of = x
1 dx 1 (1 + log x )4
= ∫ curves y = … (i)
2 x2 − x + 1 2 x2 − C 1
(1 + log x ) (1 − log x )
2 (where, C is any arbitrary constant) = x
1 dx (1 + log x )4
= ∫ Differentiation w.r.t. x of Eq. (i)
2 x2 − x + 1 − 1 + 1 d 2y ( + 0) (1 − 0)
11
dy 1 d At x = 1 = =1
4 4 2 =− (2 x 2 − C )
1 dx dx (2 x − C )
2 2
dx dx 2 (1 + 0)4
= ∫ 2
2  1 1 dy
x−  + ⇒ = − y 2. ( 4 x − 0)
 2 4 dx Directions (Q.Nos. 86-88) Read the
1 dx dy following information and answer the
= ∫ ⇒ = − 4 xy 2, it is required differential
2  2 2 dx three items that follow.
1  1
x−  +   equation. Consider the function
 2 2 f ( x ) = g( x ) + h( x )
 1   x
  x − 2  
−1 
Directions (Q. Nos. 84 and 85) Read where, g( x ) = sin  
1
= .2 tan  +c the following information and answer  4
2  1   4x
 2 
the two items that follow. and h( x ) = cos  
Consider the equation x y = ex − y  5
 dx 1 x
Q ∫ 2 = tan−1  dy 86. What is the period of the function
 a + x2 a a 84. What is at x = 1 equal to?
= tan−1(2 x − 1) + c
dx g ( x )?
(a) 0 (b) 1 (a) π (b) 2 π
82. What is (c) 2 (d) 4 (c) 4 π (d) 8 π
∫ x (1 + ln x )n
dx
equal to 2
d y
85. What is at x = 1 equal to? 87. What is the period of the function
dx 2
(n ≠ 1)? (a) 0 (b) 1 h ( x )?
1 (c) 2 (d) 4 4π
(a) +c (a) π (b)
( n − 1) (1 + ln x )n − 1 Solutions (Q. Nos 84 and 85) 5
1− n Given equation, x y = e x − y 5π 3π
(b) +c (c) (d)
2 2
(1 + ln x )1 − n On taking log both sides, we get
n+1 y log x = ( x − y) log e 88. What is the period of the function
(c) +c
(1 + ln x )n + 1 ⇒ y log x = x − y[Q loge e = 1] f (x ) ?
1 (a) 10 π (b) 20 π
(d) − +c ⇒ (1 + log x ) y = x ⇒ y =
x
( n − 1) (1 + ln x )n − 1 (1 + log x ) (c) 40 π (d) 80 π
12 NDA/NA Solved Paper 2019 (II)

Ê 89. (b) f( x) is increasing, if f ′( x) ≥ 0 Ê 92. (d) 1. f[g ( x)] = 25 x + 310 x + 955


2
Ê Solutions (Q. Nos. 86-88)
Given, f( x ) = g ( x ) + h( x ), 12 x − 60 x − 24 x + 288 ≥ 0
3 2
f[g ( x )] is a polynomial of degree 2.
 x
where, g ( x ) = sin   and ⇒ x 3 − 5 x 2 − 2 x + 24 ≥ 0 So, Statement 1 is not correct.
 4
⇒ ( x + 2 )( x 2 − 7 x + 12 ) ≥ 0 2. g [g ( x )] = 25 x + 180
 4x
h( x ) = cos   ⇒ ( x + 2 ) ( x − 3) ( x − 4) ≥ 0
 5 g [g ( x )] is a polynomial of degree 1.
∴ x ≤ − 2, x ≥ 3, x ≥ 4 So, Statement 2 is not correct.
 x
Ê 86. (d) g ( x) = sin   Hence, f( x ) is increasing the interval Hence, the Statement neither 1 nor 2
4
( 3, 4). correct.
 x + 8π 
g ( x + 8 π ) = sin  
 4  Ê 90. (a) f( x) is decreasing, if f ′( x) ≤ 0 Ê 93. (b) Given, h( x) = 5f( x) − xg ( x)
 x 12 x 3 − 60 x 2 − 24 x + 288 ≤ 0 = 5( x 2 + 2 x − 5) − x ( 5 x + 30)
= sin  2 π + 
 4 ⇒ x 3 − 5 x 2 − 2 x + 24 ≤ 0 = 5 x 2 + 10 x − 25 − 5 x 2 − 30 x
 x ⇒ ( x + 2 ) ( x 2 − 7 x + 12 ) ≤ 0 = − 20 x − 25
= sin   = g ( x )
 4
⇒ ( x + 2 ) ( x − 3) ( x − 4) ≤ 0 Differentiation w.r.t. x, we get
∴ Period of the function g ( x ) = 8 π ∴ x ≥ − 2, x ≤ 3, x ≤ 4 h′ ( x ) = − 20
 4x Hence f( x ) is decreasing the interval Hence, derivative of h( x ) is −20.
Ê 87. (c) h( x) = cos   ( −2, 3).
5
 5π  4 5π 
Directions (Q.Nos. 94 and 95) Read
hx+  = cos  x +  Directions (Q.Nos. 91-93) Read the the following information and answer
 2  5 2 
following information and answer the the questions given below.
 4x
= cos  2 π +  two items that follow . Consider the integrals
 5 π
Let f ( x ) = x 2 + 2x − 5 I1 = ∫
xdx
and
 
4 x 0 1 + sin x
= cos   = h( x ) and g( x ) = 5x + 30
 5 π (π − x )dx
5π 91. What are the roots of the equation I2 = ∫
∴ Pperiod of the function h( x ) = 0 1 − sin (π + x )
2 g [( f ( x )] = 0?
Ê 88. (c) f( x) = g ( x) + h( x) (a) 1, − 1 (b) −1, − 1 94. What is the value of I 1?
 x  4x (c) 1, 1 (d) 0, 1 π
= sin   + cos   (a) 0 (b) (c) π (d) 2 π
 4  5 92. Consider the following statements. 2
 x + 40 π  4
f( x + 40 π ) = sin   + cos 1. f [ g ( x )] is a polynomial of degree 95. What is the value of I 1 + I 2 ?
 4  5
3. π
( x + 40 π ) (a) 2 π (b) π (c) (d) 0
2. g [ g ( x )] is a polynomial of degree
 x  4x 2
= sin 10 π +  + cos  32 π +  2.
 4  5
Which of the above statements is/are Ê Solutions (Q.Nos. 94 and 95)
 x  4x π xdx
= sin 5 × 2 π +  + cos  16 × 2 π +
 4  5
 correct? Given, I1 = ∫0 1 + sin x,
(a) Only 1 (b) Only 2
 x  4x ( π − x ) dx
= sin   + cos   = f( x ) (c) Both 1 and 2 (d) Neither 1 nor 2 π
 4  5 I2 = ∫0 1 − sin ( π +
∴Period of the function f( x ) = 40 π
93. If h( x ) = 5 f ( x ) − xg ( x ), then what x)
π xdx
is the derivative of h ( x )?
(a) −40 (b) −20
I1 = ∫0 1 + sin x
…(i)
Directions (Q. Nos. 89 and 90) Read (c) −10 (d) 0 π ( π − x ) dx
the following information and answer
Ê Solutions (Q. Nos. 91-93) Given,
= ∫0 1 + sin ( π − x)
the two items that follow.
f( x ) = x + 2 x − 5, g ( x ) = 5 x + 30
2
Q f( x ) dx = f( a − x ) dx 
a a
 ∫0 ∫0
consider the function
∴g [f( x )] = 5( x 2 + 2 x − 5) + 30 
f ( x ) = 3x 4 − 20 x 3 − 12x 2 + 288 x + 1
= 5 x 2 + 10 x + 5 π ( π − x ) dx
I1 = ∫ … (ii)
89. In which one of the following f[g ( x )] = ( 5 x + 30)2 + 2( 5 x + 30) − 5 0 1 + sin x

intervals is the function increasing? = 25 x 2 + 900 + 300 x + 10 x + 60 − 5 Adding Eqs. (i) and (ii), we get
(a) ( −2, 3) (b) ( 3, 4) π ( x + π − x ) dx π π dx
= 25 x 2 + 310 x + 955 2 I1 = ∫ =∫
(c) ( −3, − 2 ) (d) ( −4, − 3)
and g [g ( x )] = 5( 5 x + 30) + 30
0 1 + sin x 0 1 + sin x
90. In which one of the following = 25 x + 180 π (1 − sin x ) dx
intervals is the function decreasing?
= π ∫0 (1 − sin2 x )
Ê 91. (b) The equation, g [f( x)] = 0
(a) ( −2, 3) (b) (3, 4) 5 x + 10 x + 5 = 0
2 π (1 − sin x ) dx
(c) (4, 6) (d) (6, 9)
⇒ x2 + 2 x + 1 = 0
= π ∫0 cos 2 x
Ê Solutions (Q. Nos. 89 and 90) π
= π ∫0 (sec x − sec x tan x ) dx
2
Given function, ⇒ ( x + 1)2 = 0
f( x ) = 3 x 4 − 20 x 3 − 12 x 2 + 288 x + 1 ∴ x = − 1, − 1 = π [tan x − sec x ]π0
Differentiation w.r.t. x, we get Hence, the roots of this equation are = π [(tan π − sec π )
f ′ ( x ) = 12 x 3 − 60 x 2 − 24 x + 288 −1, − 1. − (tan 0 − sec 0)
NDA/NA Solved Paper 2019 (II) 13

= π [( 0 + 1) − ( 0 − 1)] 98. For what value of k is the function 100. x 3 5x 2


2 I1 = π (1 + 1) = 2 π  If f ( x ) = − + 6x + 7 increases
1 3 2
⇒ I1 = π 2x + , x < 0 in the interval T and decreases in the
π ( π − x ) dx  4
Now, I2 = ∫  interval S, then which one of the
0 1 − sin ( π + x ) f ( x ) = k , x = 0 continuous?
following is correct?
π ( π − π + x ) dx  2
= ∫0 1 − sin ( π − π −   x + 1 , x > 0 (a) T = ( −∞, 2 ), ∪ ( 3, ∞ ) and S = (2, 3)
x)
  (b) T = φ and S = ( −∞, ∞ )
2
(c) T = ( −∞, ∞ ) and S = φ
Q af( x ) dx = a
x ) dx 
 ∫0 ∫0 f ( a − 1 1 (d) T = (2, 3) and S = ( −∞, 2 ) ∪ ( 3, ∞ )
 (a) (b) (c) 1 (d) 2
4 2 x3 5 x2
π xdx Ê (a) Given, f( x) = − + 6x + 7
= ∫0 1 − sin ( − x ) Ê (a) Given, 3 2
π
 1 Differentiating w.r.t x, we get
xdx 2 x + , x < 0
= ∫0 1 + sin x [Q sin (− θ) = sin θ]  4 f ′( x) =
3 x2 5
− . 2 x + 6 = x2 − 5 x + 6
f( x ) =  k , x = 0 continuous. 3 2
= I1 = π  2 Q f( x ) is increases in interval T,
π xdx   x +  , x > 0
1
Ê 94. (c) I1 = ∫0 1 + sin x = π   2
∴ f ′( x) ≥ 0
⇒ x2 − 5 x + 6 ≥ 0
 1
Ê 95. (a) I1 + I2 = π + π = 2 π LHL = lim f( x ) = lim  2 x +  ⇒ x2 − 3 x − 2 x + 6 ≥ 0
x → 0− h → 0−  4
⇒ x( x − 3) − 2 ( x − 3) ≥ 0
96. The differential equation which
= lim 2( 0 − h) +
1
⇒ ( x − 3) ( x − 2 ) ≥ 0 ⇒ x ≤ 2, x ≥ 3
represents the family of curves given h→ 0  4 
∴ T = ( − ∞,2 ) ∪ ( 3, ∞ )
by tan y = C (1 − e x ) is  1 1
= lim  −2 h +  = Again, f( x ) is decreases in interval S.
h → 0 4 4
(a) e tan ydx + (1 − e ) dy = 0
x x
∴ f ′( x) ≤ 0
(b) e x tan ydx + (1 − e x ) sec 2 ydy = 0 and f( 0) = k ⇒ x2 − 5 x + 6 ≤ 0
(c) e x (1 − e x ) dx + tan ydy = 0 Q The function f( x ) is continuous at x = 0
⇒ x2 − 3 x − 2 x + 6 ≤ 0
(d) e x tan ydy + (1 − e x ) dx = 0 ∴ LHL = f( 0) = RHL
1 ⇒ x( x − 3) − 2( x − 3) ≤ 0
Ê (b) The equation of the family of curves. ⇒ LHL = f( 0) ⇒ =k
4 ⇒ ( x − 3) ( x − 2 ) ≤ 0
tan y = C (1 − e ) x
… (i)
1 ⇒ 2 < x < 3 ⇒ x ∈ (2, 3)
Hence, k =
Differentiation w.r.t. x, we get 4 ∴ S = (2, 3)
dy
sec 2 y . = C(0 − e x ) 99. What is the area of the region
dx 101. A coin is biased so that heads comes
⇒ sec 2 y .
dy
= − Ce x … (ii)
enclosed between the curve y 2 = 2x up thrice as likely as tails. For three
dx and the straight line y = x ? independent tosses of a coin, what is
EleminatingC from Eqs. (i) and (ii), we get 2 4 the probability of getting at most
(a) sq unit (b) sq unit
dy tan y 3 3 two tails?
sec 2 y. = − e x 1
dx (1 − e x ) (c) sq unit (d) 1 sq unit (a) 0.16 (b) 0.48
3 (c) 0.58 (d) 0.98
⇒ (1 − e ) sec y dy = − e tan y. dx
x 2 x

Ê (a) Equations of curves, Ê (d) Let X be a random variable, it


⇒ e x tan y dx + (1 − e x ) sec 2 y dy = 0 y2 = 2 x … (i) represents of the number tail comes of
three tosses of a coin.
97. What is the derivative of 2(sin x ) 2
and y= x … (ii)
∴Possible value of X are 0, 1, 2, 3.
with respect to sin x? From Eqs. (i) and (ii),
According to the question, the coin is
2 y 2 = 2 y ⇒ y( y − 2 ) = 0
(a) sin x 2 (sin x ) ln 4 biased in which the probability to comes
(b) 2 sin x 2 (sin x ) ln 4
2 ∴ y = 0, 2 head is thrice as likely as tails.
2  3 1
2
y2 ∴P(H) = and P(T) =
(c) ln (sin x ) 2 (sin x )
2
∴Required area = ∫0  2 − y dy

4 4
(d) 2 sin x cos x 2 (sin x ) 3
 3 27
(sin x )2
2 P(X = 0) = P({HHH}) =   =
Ê (a) Derivative of 2 with respect to  1 y3 y2   4 64
= . − 
sin x 2 3 2 0 P( X = 1) = P (2 heads and 1 tail)
d (sin x )2 2 d
2 2 (sin x ) log 2 (sin x )2 1 (2 )2  = P({HHT}) + P ({HTH}) + P({THH})
= dx = dx =  (2 )3 − −0 3 3 1 3 1 3 1 3 3
d cos x  6 2  = ⋅ ⋅ + ⋅ ⋅ + ⋅ ⋅
(sin x ) 4 4 4 4 4 4 4 4 4
dx 8 4 8 − 12
2 = − = 27
2 (sin x ) log 2. 2 sin x.cos x 6 2 6 =
= 64
cos x 4 −2 2
2
=− = = sq unit P( X = 2 ) = P (1 head and 2 tails)
= 2 log 2. (sin x ). 2 (sin x ) 6 3 3
= P({HTT}) + P({THT}) + P({TTH})
2 (∴area will not be negative)
= sin x. 2 (sin x ) .log 4
14 NDA/NA Solved Paper 2019 (II)

3 1 1 1 3 1 1 1 3 3 1
= ⋅ ⋅ + ⋅ ⋅ + ⋅ ⋅ 104. Arithmetic mean of 10 observations and r=− or −
4 4 4 4 4 4 4 4 4 6 2
is 60 and sum of squares of
=
9 = − 0.5
deviations from 50 is 5000. What is
64
the standard deviation of the 107. The class marks in a frequency table
∴Required probability
observations? are given to be 5, 10, 15, 20, 25, 30, 35,
= P( X = 0) + P( X = 1) + P( X = 2 )
(a) 20 (b) 21 40, 45, 50. The class limits of the first
27 9 27
= + + (c) 22.36 (d) 24.70 five classes are
64 64 64
63 Ê (a) Arithmetic mean of 10 observations (a) 3-7, 7-13, 13-17, 17-23, 23-27
= = 0.98 = 60 (b) 2.5-7.5, 7.5-12.5, 12.5-17.5,
64
∴Σxi = 60 × 10 = 600 Q x = Σxi  17.5-22.5, 22.5-27.5
 n  (c) 1.5-8.5, 8.5-11.5, 11.5,-18.5,
102. A bag contains 20 books out of
If, A = 50, then Σd i = 5000
2 18.5-21.5, 21.5-28.5
which 5 are defective. If 3 of the (d) 2-8, 8-12, 12-18, 18-22, 22-28
books are selected at random and Q d i = xi − A
∴ Σd i = Σ( xi − A ) = Σxi − AΣ1 Ê (b) Given, class marks in a frequency
removed from the bag in succession table are
without replacement, then what is = 600 − 50 × n [Q Σ1 = n]
5, 10, 15, 20, 25, 30, 35, 40, 45, 50.
the probability that all three books = 600 − 50 × 10 = 100
Let L1 and L2 be the lower limit and upper
are defective? 2
Σd i2  Σd i  limit of first interval.
Now, SD = − 
(a) 0.009 (b) 0.016 n  n  L + L2 L + L2
(c) 0.026 (d) 0.047 Q Class mark = 1 5= 1
2 2 2
5000  100 
Ê (a) Total books in bag = 20 = −  ⇒ L1 + L2 = 10 … (i)
10  10 
Defective books = 5 and L2 − L1 = Class interval
= 500 − 100 = 400 = 20
∴Undefective books = 20 − 5 = 15 or L2 − L1 = 5 … (ii)
∴ Probability to selected three books are 105. If p and q are the roots of the Solving Eq. (i) and (ii),
defective without replacement
equation x 2 − 30x + 221 = 0, what is L2 = 7.5 and L1 = 2.5
5 4 3
= × × the value of p 3 + q 3 ? ∴Class limit of first classes is 2.5 − 7.5
20 19 18
Similarly find class limit of other classes.
6 (a) 7010 (b) 7110
= Hence, class limits of the first five classes
684 (c) 7210 (d) 7240
are
= 0.0087 = 0.009 Ê (b) Since, p and q are the roots of the
equation 2.5 − 7.5, 7.5 − 12.5, 12.5 − 17.5,
103. The median of the observations 22, x 2 − 30 x + 221 = 0 17.5 − 22.5, 22.5 − 27.5.
24, 33, 37, x + 1, x + 3, 46, 47, 57, 58 in
∴ p + q = 30 and pq = 221 108. The mean of 5 observations is 4.4 and
ascending order is 42. What are the
Now, p3 + q 3 = ( p + q ) ( p2 + q 2 − pq ) variance is 8.24. If three of the five
values of 5th and 6th observations
respectively? = 30 [ p2 + q 2 + 2 pq − 3 pq ] observations are 1, 2 and 6, then what
are the other two observations?
(a) 42, 45 (b) 41, 43 = 30 [( p + q )2 − 3 pq ]
(c) 43, 46 (d) 40, 40 (a) 9, 16 (b) 9, 4
= 30 [( 30)2 − 663] (c) 81, 16 (d) 81, 4
Ê (b) The observations in ascending order
are = 30 [900 − 663] Ê (b) Let x1, x2, x3, x4 and x5 are five
22, 24, 33, 37, x + 1, x + 3, 46, 47, 57, 58 = 30 × 237 = 7110 observations.
Here, n = 10 ∴ x1 = 1, x2 = 2 , and x3 = 6
106. For the variables x and y, the two x + x2 + x3 + x4 + x5
∴Median
regression lines are 6x + y = 30 and ∴ x= 1
N 5
Value of the observations + 3x + 2y = 25. What are the values of
2 x1 + x2 + x3 + x4 + x5
x , y and r respectively? ⇒ 4.4 =
N  5
Values of  + 1 th observations 20 35
, − 0.5
20 35
2  (a) , (b) , , 0.5 ⇒ x1 + x2 + x3 + x4 + x5 = 22
= 3 9 3 9
2
(c)
35 20
, , − 0.5 (d)
35 20
, , 0.5 ⇒ 1 + 2 + 6 + x4 + x5 = 22
Value of 5th observations + 9 3 9 3 ⇒ x4 + x5 = 22 − 9
⇒ 42 =
Value of 6th observation Ê (c) Given lines, 6 x + y = 30 …(i) ⇒ x4 + x5 = 13 …(i)
2
and 3 x + 2 y = 25 …(ii) and variance,
⇒ 84 = x + 1 + x + 3 where, x and y are two variables. ( x1 − x )2 + ( x2 − x )2 + ( x3 − x )2 +
⇒ 2 x = 84 − 4 Solving these equations, ( x4 − x )2 + ( x5 − x )2
80 σ2 =
⇒ x= = 40 35
x = , and y =
20 5
2 9 3
∴ 5th observation = x + 1 = 40 + 1 = 41 (1 − 4.4)2 + (2 − 4.4)2 + ( 6 − 4.4)2 +
These lines are regression,
and 6th observation ( x4 − 4.4)2 + ( x5 − 4.4)2
35 20 . =
⇒ 824
= x + 3 = 40 + 3 = 43 Then, x = , y = 5
9 3
NDA/NA Solved Paper 2019 (II) 15

⇒ 824
. × 5 = 1156. + 576 . + 2.56 111. Two Ê (c) Let E1, E 2 and E 3 represent the events
dice are thrown
+ ( x4 − 4.4)2 + (13 − x4 − 4.4)2 simultaneously. What is the of two-headed coin, a fair coin and
[from Eq. (i)] probability that the sum of the biased coin respectively.
1 1 1
⇒ 4120
. = 19.88 + ( x4 − 4.4)2 + ( 8.6 − x4 )2 numbers appearing on them is a ∴P( E1 ) = , P( E 2 ) = , P( E 3 ) =
prime number? 2 2 4
⇒ 4120
. − 19.88 = x42 + 19.36 − 8.8 x4  E 1  E 1  E 1
5 1 P  = , P  = , P  =
+ 73.96 + x42 − 17.2 x (a) (b)
12 2  E1  2  E 2  2  E 3  4
⇒ 2132
. = 2 x42 − 26 x4 + 93.32 7 2
(c) (d) Apply Baye’s theorem,
⇒ 2 x42 − 26 x4 + 72 = 0 12 3
 E
⇒ x42 − 13 x4 + 36 = 0 Ê (a) Total number of sample space of two P( E1 ) ⋅ P  
dice are thrown, n( s ) = 6 × 6 = 36 E   E1 
⇒ x42 − 9 x4 − 4 x4 + 36 = 0 P 1  =
 E  E  E
⇒ x4 ( x4 − 9) − 4 ( x4 − 9) = 0 Total number of favourable outcomes the P( E1 ) ⋅ P   + P( E 2 ) ⋅ P  
sum of numbers appearing on them is a  E1   E2 
⇒ ( x4 − 9) ( x4 − 4) = 0
∴ x4 = 4, 9 prime number.  E
+ P( E 3 ) ⋅ P  
From Eq. (i), x5 = 9, 4 (1, 1), (1, 2), (1, 4), (1, 6), (2, 1), (2, 3),  E3 
Hence, other two observations are 9 (2, 5), (3, 2), (3, 4), (4, 1), (4, 3), (5, 2), 1 1 1
and 4. (5, 6), (6, 1), (6, 5) .
= 2 2 = 4
∴ n( E ) = 15 1 1 1 1 1 1 1 1
. + . + . + +
1
109. If a coin is tossed till the first head
n( E ) 2 2 2 2 4 4 4 4 16
appears, then what will be the ∴Required probability =
n(S ) 1
sample space? 4
15 5 = 4 =
(a) {H} (b) {TH} = = 4+ 4+1 9
(c) {T, HT, HHT, HHHT, ………} 36 12
16
(d) {H, TH, TTH, TTTH, ………} 112. If 5 of a Company’s 10 delivery
Ê (a) A coin is tossed till the first head trucks do not meet emission
114. Consider the following statements:
appears, then the sample space will be 1. If A and B are mutually exclusive
standards and 3 of them are chosen
= {H}
for inspection, then what is the events, then it is possible that
110. Consider the following discrete probability that none of the trucks P ( A ) = P ( B ) = 06.
.
frequency distribution. chosen will meet emission 2. If A and B are any two events
standards? such that P ( A / B ) = 1, then
x 1 2 3 4 5 6 7 8 1 3 P ( B / A ) = 1.
(a) (b)
f 3 15 45 57 5 36 25 9 8 8
0
Which of the above statement is/are
1 1
(c) (d) correct?
What is the value of median of the 12 4 (a) Only 1 (b) Only 2
distribution? Ê (c) Total trucks of a company’s = 10 (c) Both 1 and 2 (d) Neither 1 nor 2
(a) 4 (b) 5 (c) 6 (d) 7 Number of trucks that do not meet Ê (b) Statement 1 : A and B are mutually
emission standards = 5 exclusive events, then P( A ∩ B) = 0
Ê (b)
x f C
Number of trucks that are chosen for ∴P( A ∪ B) = P( A ) + P( B)
inspection = 3 = 0.6 + 0.6
1 3 3 5
C
2 15 18 ∴Required probability = 10 3 = 1.2 , it is not possible
C3
3 45 63 So, Statement 1 is not correct.
5!
4 57 120 Statement 2 : A and B are any two events
5 !7 !
5 50 170 = 3 !2 ! = such that
10 ! 10 !2 !
6 36 236  A
7 25 261 3 !7 ! P  = 1
5⋅ 4⋅ 3 1  B
8 9 270 = =
10 ⋅ 9 ⋅ 8 12 P( A ∩ B)
N = 270 ⇒ = 1 ⇒ P( A ∩ B) = P( B) … (i)
P( B)
Here, N = 270 113. There are 3 coins in a box. One is a
∴Median ∴P( A ∪ B) = P( A ) + P( B) − P( A ∩ B)
two-headed coin; another is a fair
N coin; and third is biased coin that P( A ∪ B) = P( A ) + P( B) − P( B)
Value of th term + value of
2 comes up heads 75% of time. When [from Eq. (i)]
N  one of the three coins is selected at ⇒ P( A ∪ B) = P( A ) … (ii)
 + 1 th term
2  random and flipped, it shows heads.  B  P( B ∩ A ) P( A ∪ B)
= Now, P   = =
2 What is the probability that it was  A P( A ) P( A )
Value of 135th term + Value of the two-headed coin? 1 − P ( A ∪ B) 1 − P( A )
136th term 2 1 = = =1
= (a) (b) 1 − P( A ) 1 − P( A )
2 9 3
5+ 5 4 5 So, Statement 2 is correct.
= =5 (c) (d)
2 9 9 Hence, only the Statement 2 is correct.
16 NDA/NA Solved Paper 2019 (II)

115. If a fair die is rolled 4 times, then Q Y = 40 + 3 X 119. A car travels first 60 km at a speed of
∴ Ymin = 40 + 3 Xmin 3 v km/h and travels next 60 km at 2
what is the probability that there are
exactly 2 sixes? = 40 + 3( 0) [Q Xmin = 0] v km/h. What is the average speed of
5 25 = 40 the car?
(a) (b)
216 216 and Ymax = 40 + 3 Xmax (a) 2.5 v km/h
125 175 = 40 + 3(25) [Q Xmax = 25] (b) 2.4 v km/h
(c) (d) (c) 2.2 v km/h
216 216 = 40 + 75 = 115 (d) 2.1 v km/h
Ê (b) Let X be a random variable that Now, R Y = Ymax − Ymin Ê (b) Time taken for first 60 km with speed 3
represents to appearing 6 of rolled a die.
= 115 − 40 = 75 v km/h
Probability of to get 6 to rolled a die, 60 20  Distance 
1 118. If V is the variance and M is the mean = = h Q Time = Speed 
p= 3v v  
6 of first 15 natural numbers, then
Time taken for next 60 km with speed
∴Probability of not get 6 to rolled a die, what is V + M 2 equal to? 2v km/h
1 1 5
q = 1− = 1− = 124 148 60 30
p 6 6 (a) (b) = = h
3 3 2v v
Here, n = 4, r = 2 (c)
248
(d)
124
Total distance
∴Average speed =
∴Required probability = nC r prq n − r 3 9
Total time
[By Bernoulli distribution] Ê (c) Mean of first 15 natural numbers, M 60 + 60
=
2 2 1+ 2 + 3 + 4 + 5 + 6 + 7 + 8 + 9 20 30
 1  5 +
= 4C 2     + 10 + 11 + 12 + 13 + 14 + 15 v v
 6  6 =
15 120 v
4! 1 25 =
= × × 15(15 + 1) 50
2 !2 ! 36 36 =
2 × 15 = 2.4 v km/h
4⋅ 3 1 25 25
= × × = Q 1 + 2 + 3 + ... + n = n( n + 1)
2 ⋅ 1 36 36 216   120. The mean weight of 150 students in a
2
116. Mean of 100 observations is 50 and certain class is 60 kg. The mean
=8 weight of boys is 70 kg and that of
standard deviation is 10. If 5 is added Variance of first 15 natural numbers, V girls is 55 kg. What are the number
to each observation, then what will 1
be the new mean and new standard = [(1 − 8)2 + (2 − 8)2 + ( 3 − 8)2 of boys and girls respectively in the
15 class?
deviation respectively? + ( 4 − 8)2 + ( 5 − 8)2 + ( 6 − 8)2 (a) 75 and 75
(a) 50, 10 (b) 50, 15
(c) 55, 10 (d) 55, 15 + (7 − 8)2 + ( 8 − 8)2 + ( 9 − 8)2 (b) 50 and 100
(c) 70 and 80
Ê (c) Mean of 100 observations = 50 + (10 − 8)2 + (11 − 8)2 + (12 − 8)2 (d) 100 and 50
and standard deviation = 10 + (13 − 8)2 + (14 − 8)2 + (15 − 8)2 ] Ê (b) Let number of boys and girls be x and
We know that, if k is added to each 1 y respectively.
= [( − 7 )2 + ( − 6)2 + ( − 5)2 + ( − 4)2 + ( − 3)2
observation, then new mean will be more 15 ∴ x + y = 150 … (i)
than k and standard deviation no change. + ( − 2 )2 + ( − 1)2 + 0 + (1)2 + (2 )2 + ( 3)2 Mean weight of 150 students = 60 kg
∴After 5 added to each observation. ∴Total weight of 150 students
+ ( 4)2 + ( 5)2 + ( 6)2 + (7 )2 ]
mean = 50 + 5 = 55 2 2 = 60 × 150
= [1 + 2 2 + 3 2 + 4 2 + 5 2 + 6 2 + 7 2 ]
and standard deviation = 10 15 = 9000kg.
2 7(7 + 1) (14 + 1) Mean weight of boys = 70 kg
117. If the range of a set of observations = ×
15 6 ∴Total weight of boys = 70 x kg
on a variable X is known to be 25
and if Y = 40 + 3X , then what is the Q 12 + 2 2 + 3 2 + ... + n2  and mean weight of girls = 55 kg
 
range of the set of corresponding  = n ( n + 1) (2 n + 1)  ∴Total weight of girls = 55 y kg
observations on Y ?  6  ∴Total weight of 150 students = 9000 kg
(a) 25 (b) 40 2 7 × 8 × 15 56
= × = ⇒ 70 x + 55 y = 9000
(c) 75 (d) 115 15 6 3
⇒ 14 x + 11y = 1800 … (ii)
Ê (c) Range of set of observations on a Now, V + M =2 56
+ 64
variable, X = 25 Solving Eqs. (i) and (ii), we get
3
x = 50
We know that 56 + 192
= y = 100
Range, R X = Xmax = Xmin 3
⇒ 25 = Xmax − 0 [Q Xmin = 0] 248 Hence, the number of boys and girls are
= 50 and 100 respectively.
⇒ Xmax = 25 3
PAPER II English Language and General Studies
Part A (English Language)
1. He is always anxious. 7. His language is political and 13. He wrote an incisive article on
(a) worried (b) dispassionate vitriolic. corruption in politics.
(c) sluggish (d) torpid (a) imaginative (b) sprightly (a) vague (b) penetrating
Ê (a) ‘Worried’ is the correct synonym of (c) vivacious (d) abusive (c) trenchant (d) precise
‘anxious’. Both words mean ‘feeling Ê (d) ‘Abusive’ is the correct synonym of Ê (a) The word ‘incisive’ means ‘something
uneasy about something with an the word ‘vitriolic’. Both words mean that is sharp or direct’. Its antonym is
uncertain outcome.’ ‘violent hate and anger expressed ‘vague’ which means ‘uncertain or indistinct’.
Some other synonyms are - uneasy, through severe criticism’. Some other
agitated, etc. synonyms are - acrimonious, bitter, etc.
14. Rakesh is an eccentric prodigal.
(a) extravagant (b) profligate
2. The poems of Kabir are ecstatic in 8. The Managing Director of the (c) wasteful (d) thrifty
nature. company declared that he is broke Ê (d) The word ‘prodigal’ means ‘spending
(a) efficacious (b) eerie and there is a need to seek support money or using resources freely and
(c) rapturous (d) reverential from the government. recklessly’. Its antonym is ‘thrifty’, which
means ‘using money and other
Ê (c) ‘Rapturous’ is the correct synonym of (a) bankrupt (b) rich
‘ecstatic’. Both words mean ‘feeling or (c) making profit (d) having liabilities resources carefully’.
expression overwhelming happiness’. 15. Friendship is always reciprocal.
Ê (a) ‘Bankrupt’ and ‘broke’ are synonyms.
Some other synonyms are - joyful, Both words mean ‘having completely run (a) mutual (b) one-sided
overjoyed, etc. out of money.’ Some other synonyms are (c) shared (d) corresponding
3. Ravi loves seclusion. Therefore, he - insolvent, penniless, etc.
Ê (b) The word ‘reciprocal’ means ‘mutual
lives in the mountain. 9. He loves doing nasty things. or complementary’. Its antonym is
(a) nature (b) scripture (a) nice (b) fastidious ‘one-sided’ meaning ‘having or
(c) seafaring (d) solitariness (c) foul (d) finicky occurring on side of something only’.
Ê (d) ‘Solitariness’ is the correct synonym Ê (c) ‘Foul’ is the correct synonym of 16. The debate was highly stimulating.
of ‘seclusion’. Both words mean ‘being ‘nasty’. Both words mean ‘something (a) arousing (b) invigorating
separate and apart from others’. unpleasant or disagreeable’. (c) boring (d) stirring
Some other synonyms are - solitude, Some other synonyms are - awful,
isolation, etc. disgusting, etc. Ê (c) The word ‘stimulating’ means ‘exciting
or arousing’. Its antonym is ‘boring’
4. Hitler was a despot. 10. He could not muster courage to which means ‘not interesting’.
(a) conservative (b) dictator speak the truth before his friend. 17. Malaria is a widespread disease.
(c) passionate (d) monstrous (a) injure (b) spoil (a) endemic (b) pervasive
Ê (b) ‘Dictator’ is the correct synonym of (c) gather (d) maim (c) common (d) rare
‘despot’. Both words refer to ‘a ruler or
Ê (c) ‘Gather’ is the correct synonym of Ê (a) The word ‘widespread’ means ‘extensive
other person who holds absolute power.’ ‘muster’. Both words mean ‘assemble or or distributed over a large area’.Its
Some other synonyms are - autocrat, bring together’. antonym is ‘endemic’ which means
authoritarian, etc. Some other synonyms are - gather, ‘native and restricted to a certain place’.
5. The imagery used in the poem is assemble, etc.
18. The bill received a vehement
vivid. 11. His conversations are always absurd. resistance from the opposition party
(a) lively (b) inert (a) farcical (b) foolish in the Parliament.
(c) ebullient (d) caustic (c) preposterous (d) rational (a) animated (b) apathetic
Ê (a) ‘Lively’ is the correct synonym of Ê (d) The word ‘absurd’ means (c) fervent (d) vigorous
‘vivid’. Both words refer to ‘something
which is very clear and true to life.’
‘unreasonable or inappropriate’. Its Ê (b) The word ‘vehement’ means ‘showing
antonym is ‘Rational’ which means ‘in strong feelings or shown with great
Some other synonyms are - realistic, accordance with reason or logic.’ energy or force’. Its antonym is
lifelike, etc. ‘apathetic’ which means ‘showing no
12. The politician was a dissident. enthusiasm or concern’.
6. A human being is always
(a) Agitator (b) Rebel
vulnerable to other human beings. (c) Conformist (d) Revolutionary 19. He has been obstinate since his
(a) resilient (b) elastic childhood.
(c) defenceless (d) crude Ê (c) The word ‘dissident’ means ‘a person (a) stubborn (b) dogged
who opposes official policy’.
(c) amenable (d) rigid
Ê (c) ‘Defenceless’ is the correct synonym Its antonym from the given option is
of ‘vulnerable’. Both words mean ‘open ‘conformist’, which means ‘someone Ê (c) The word ‘obstinate’ means ‘stubborn
to attack or damage.’ who conforms to the accepted or determined’. Its antonym is
Some other synonyms are - unprotected, behaviour or established practices’. ‘amenable’ which means ‘responsive to
unguarded, etc. suggestion’.
18 NDA/NA Solved Paper 2019 (II)

20. Mahatma Gandhi was one of the most prominent leaders of 28. Many a man has succumbed to this temptation. No error
his times. (a) (b) (c) (d)
(a) renowned (b) distinguishable Ê (d) The given sentence has no error.
(c) eminent (d) unknown
29. A time slot of fifteen minutes are allowed
Ê (d) The word ‘prominent’ means ‘readily noticeable.’ Its antonym is
‘unknown’ which means ‘not known or familiar’. (a) (b)
to each speaker. No error
21. He had arrived at Cairo a few months before (c) (d)
(a) (b) Ê (b) Here part (b) has an error. As ‘time slot’ given in part (a) is
protests shook the Arab world. No error singular noun, so the verb used with it, should also be singular.
(c) (d) Hence, ‘is allowed’ should be used in place of ‘are allowed’ to make
the given sentence grammatically correct.
Ê (a) Here part (a) has an error. As sentence is in past tense, so we
should use Simple Past tense ‘arrived’ without the use of had. 30. He asked whether either of the brothers
22. Most of us who are older competitive runners (a) (b)
(a) were at home. No error
are not able to race anywhere at the same speed (c) (d)
(b) Ê (c) Here part (c) has an error. ‘Were at home’ is incorrect.
as we do when we were 30. No error It should be replaced with ‘was at home’, as either is always
(c) (d) followed by the singular verb.

Ê (b) Here part (b) has an error. ‘At the same speed’ is incorrect. It 31. Be over the hill
should be ‘with the same speed’, to make the given sentence
grammatically correct. Because ‘at’ is used to show only a definite (a) To be on a mountain top
speed; e.g. at 80mph etc. (b) To travel in the jungle
(c) To be too old to do things
23. Work hard lest you do not fail. No error (d) To do something in the most complete way
(a) (b) (c) (d)
Ê (c) Idiom ‘Be over the hill’ means ‘to be too old to do things’. Hence,
Ê (b) Here part (b) has an error. With ‘lest’ we should always use option (c) is a suitable choice.
‘should’. Replace ‘do not’ with ‘should’ to make the given sentence
grammatically correct. 32. Bite your tongue
(a) To stop yourself from saying something because it would be
24. The Eastern Ghats are home of 2600 plant species better not to
(a) (b) To bite off your tongue while eating
and this habitat fragmentation can pose a serious (c) To feel sorry
(b) (c) (d) To ask someone something that you want
threat to endemic plants. No error
Ê (a) Idiom ‘Bite your tongue’ means ‘to stop yourself from saying
(d) something because it would be better not to’. So option (a) is a
Ê (a) Here part (a) has an error. ‘Home of’ is incorrect expression. suitable choice.
It should be replaced with ‘home to’ as it refers to the place where
you live or from where you belonged.
33. Turn a blind eye
(a) To run away
25. Turbidity current is a fast-moving current (b) To begin to be have in a more positive manner
(a) (c) To change a situation
that sweeps down submarine canyons, (d) To choose to ignore behaviour that you know is wrong
(b) Ê (d) The given idiom ‘Turn a blind eye’ means ‘to ignore something
carrying sand and mud into the deep sea. No error that you know is wrong’. So, option (d) is the correct choice.
(c) (d)
34. Yellow journalism
Ê (c) Here part (c) has an error. Replace ‘carrying sand’ with ‘that (a) Paid storytelling
carries sand’ as the reference is to a particular thing (Turbidity
current). (b) Government reports published on yellow coloured papers
(c) Writings in newspapers that try to influence people’s opinion by
26. Every one of the boys love to ride. No error using strong language and false information
(a) (b) (c) (d) (d) Journalism which agrees completely with the policies of the
government and capitalists
Ê (c) Here part (c) has an error. With ‘everyone’ singular verb should
be used. So, use ‘loves’ in place of ‘love’ to make the given Ê (c) Idiom ‘Yellow Journalism’ means ‘writing in newspapers that try
sentence grammatically correct. to influence people’s opinion by using strong language and false
information’. So, option (c) is a suitable choice.
27. Neither praise nor blame seem to affect him.
(a) (b) (c) 35. To be in seventh heaven
No error (a) To fly in the ail
(d) (b) To be extremely happy
Ê (b) Here part (b) has an error of use of verb. ‘Seem’ should (c) To be completely alone
be replaced with ‘seems’ as the sentence is in Simple Present (d) To visit a place which is in the lit of the seven wonders of the
tense. world.
NDA/NA Solved Paper 2019 (II) 19
Ê (b) Idiom ‘To be in seventh heaven’ means ‘to be extremely happy’. particulars, one by one; but the general counsels and the plots and
So, option (b) is the correct choice. marshalling of affairs, come best from those that are learned.
36. closely monitor the situation by the disturbing To spend too much time in studies is sloth; to use them too much
for ornament, is affectation; to make judgement wholly by their
P Q
allegations we are deeply concerned rules, is the humor of a scholar. They perfect nature and are
R perfected by experience: for natural abilities are like natural
and will continue to plants, that need pruning, by study; and studies themselves do
S give forth directions too much at large, except they be bounded in
(a) R Q S P (b) S P Q R by experience.
(c) S P R Q (d) P R S Q Crafty men condemn studies, simple men admire them and wise
Ê (a) RQSP is the correct sequence. men use them; for they teach not their own use; but that is a
wisdom without them and above them, won by observation. Read
37. relieve nausea, pain and stress aromatherapy may not to contradict and confute; nor to believe and take for granted;
P Q nor to find talk and discourse; but to weigh and consider.
also help using lavender oil on their skin
Q R 41. According to the author, why should one study?
but patients are cautioned against (a) To gather information about the world
(b) To pass the time in a creative way
S (c) For pleasure, enhance capability and holistic growth
(a) R Q P S (b) S Q P R
(d) To become only worldly wise
(c) Q P S R (d) R P S Q
Ê (c) As stated in the passage, studies should be done for pleasure,
Ê (c) QPSR is the correct sequence. enhancing capability and for holistic growth of a mind.
38. without any mandate of law the plea filed 42. Why does the author not recommend too much of studies?
P (a) Because it reflects idleness
(b) Because it requires too much money
through the advocate online is illegal and submitted (c) Because it is dangerous for the health of the mind
(d) Because it makes men crafty
Q R
that the sale of drugs and prescription medicines Ê (a) The author does not recommend too much of studies as it can
make a person lazy or idle.
S
(a) R Q S P (b) Q S R P
43. According to the author, how can the studies be perfected?
(c) Q R S P (d) P R S Q (a) By reflection (b) By application
(c) By conversation (d) By experience
Ê (b) Q S R P is the correct sequence.
Ê (d) According to the author, studies can be perfected via
39. humans and machines have co-existed dating back experience.
P 44. What kind of human beings denounce studies?
to the invention of the potter’s wheel in ancient
(a) Innocent (b) Cunning
Q R
(c) Stupid (d) Wise
Mesopotamia at the physical level for millennia.
S Ê (b) Crafty or Cunning human beings denounce studies.
(a) R S P Q (b) Q S P R 45. According to the author, one should read to
(c) Q R P S (d) P S Q R
(a) only argue (b) only believe
Ê (d) P S Q R is the correct sequence. (c) evaluate and understand (d) preach and believe
40. a sell-off as US Treasury yields Ê (c) As stated in the passage, a person should read to ‘evaluate and
P understand’.
surged to multi-year highs on robust economic data
Q R
46. The Election Commission on Saturday …… that the
global markets also witnessed Assembly elections in the five States will be held from
S November 12th to December 7th.
(a) R S Q P (b) P Q S R (a) pronounced (b) announced
(c) Q R P S (d) S P Q R (c) promulgated (d) issued
Ê (d) SPQR is the correct sequence. Ê (b) ‘Announced’ is the appropriate word to fill the given blank.
Announce means ‘make a formal public statement about a fact,
occurrence or intention’.
Passage 1 47. The victims were fruit vendors and they were going in an
Studies serve for delight, for ornament and for ability. Their chief auto when they … an accident on the way.
use for delight, is in privateness and retiring; for ornament, is in (a) met with (b) ran into (c) experienced (d) heard
discourse; and for ability, is in the judgement and disposition of Ê (a) According to the given sentence, ‘Met with’ is the correct
business. For expert men can execute and perhaps judge of alternative to fill the blank as it means ‘to experience something,
such as an accident’.
20 NDA/NA Solved Paper 2019 (II)

48. Scores of villagers are ………… a sit-in 49. It is common for patients to stop 50. A four-year-old girl got a new lease
protest against the consruction of a ……… medicine as soon as they start of life after doctors at a hospital
new underpass. feeling better. ……… a cancerous tumour from one
(a) performing (b) sitting (a) earing (b) gulping of her kidneys.
(c) staging (d) standing (c) swallowing (d) taking (a) rejuvenated (b) removed
(c) displaced (d) replaced
Ê (c) ‘Staging’ is the appropriate word for Ê (d) According to the given sentence,
the given blank. Staging means ‘the option (d) ‘taking’ is the correct Ê (b) Here, ‘removed’ is the correct
method of presenting a play or other alternative to fill the blank as ‘taking alternative to fill the given blank as
dramatic performance.’ medicine’ refers to swallow medicine. ‘removed’ means ‘take something off or
Other words are irrelevant. away’. Other words do not fit in the blank.

Part B (General Studies)


51. A thin disc and a thin ring, both have Select the correct answer using the Ê (d) Statement (d) is not correct, because
mass M and radius R. Both rotate code given below: in strong electric field, heating effect will
about axes through their centre of (a) Only 1 (b) Only 2 dominant and here is deviations of curve,
mass and are perpendicular to their (c) Both 1 and 2 (d) Neither 1 nor 2 i.e. it does not obey Ohm’s law.
surfaces at the same angular
velocity. Which one of the following Ê (a) A chemical reaction is an internal 55. When a ball bounces off the ground,
process, so there is no change in which of the following changes
statements is correct? momentum and mass (since, mass is
(a) The ring has higher kinetic energy. suddenly? (Assume no loss of energy
conserved in a chemical reaction).
(b) The disc has higher kinetic energy. But in a chemical reaction, the kinetic
to the floor)
(c) The ring and the disc both have the energy of the particle is changes with (a) Its speed
same kinetic energy. respect to centre of mass of whole body. (b) Its momentum
(d) Kinetic energies of both the bodies (c) Its kinetic energy
Hence, statement 1 is correct are
are zero, since they are not in linear (d) Its potential energy
stakines 2 is incorrect.
motion. Ê (b) When a ball bounces off the ground,
Ê (a) Given, a thin disc and a thin ring have 53. In which of the following phenomena there will be sudden change in the
equal mass and radius. do heat waves travel along a straight momentum of the ball.
Angular velocity of thin disc line with the speed of light? ● It includes the mass and the velocity of

= Angular velocity of thin ring (a) Thermal conduction the object.


ωring = ωdisc (b) Thermal convention ● It shows the sudden change of the

velocity of the object according to its


∴ Kinetic energy of ring (c) Thermal radiation
mass.
= Kinetic energy of disc (d) Both (a) and (c)
● If the mass is large, the change will be
1 1
Iringωring = Idiscω 2disc
2
Ê (c) Thermal radiations are heat waves greater and if it is small, then the
2 2 (infrared waves) which are velocity will be lesser.
1 1  MR 2  2 electromagnetic waves. Electromagnetic
( MR 2 ) ωring
2
=   ωdisc 56. Which one of the following
2 2  2  waves travel in straight line with the
speed of light. It depends on the compounds does not exhibit a
1
(Q Iring = MR 2 or Idisc = MR 2) temperature of the body and nature of different oxidation number of the
2
ω2 radiating surface of the body. same element?
ωring
2
= disc , 2ωring
2
= ωdisc
2
Hence, option (c) is correct. (a) Pb 3O 4 (b) Fe 3O 4
2
(c) Fe 2O 3 (d) Mn3O 4
Hence, KEring > KE.disc 54. Which one of the following
So, the ring has higher kinetic energy. statements regarding Ohm’s law is Ê (c) Among the given species/
compounds, only Fe 2O 3 does not exhibit
52. Let there be an object having not correct? different oxidation number of Fe, in this
some chemicals in it. It starts moving (a) Ohm’s law is an assumption that species can be calculated as :
with a uniform velocity v and a current through a conductor is always
Let x be the O.N. of Fe.
chemical reaction starts happening. directly proportional to the potential
difference applied to it. ∴ 2 x + 3( − 2 ) = 0, x = + 3
In this case, which of the following
(b) A conducting device obeys Ohm’s Hence, Fe is in only + 3 oxidation state in
statement(s) is/are correct?
law when the resistance of a device is Fe 2O 3, whereas in rest of the species, the
1. Chemical reactions happening in independent of magnitude and polarity same metal exhibits different oxidation
the system cannot change the of applied potential difference. states.
velocity (v) of the centre of mass of (c) A conducting material obeys Ohm’s
the object. law when the resistance of material is 57. Which one of the following statements
independent of the magnitude and about glass is not correct?
2. Chemical reactions happening in
direction of applied electric field. (a) Glass is often said to be a
the system cannot change kinetic supercooled liquid.
(d) All homogeneous materials obey
energy of the particles inside with Ohm’s law irrespective of whether the (b) Glass has no definite melting point.
respect to the centre of mass of field is within range or strong. (c) Soda glass is harder than pyrex glass.
object. (d) Boron is present in pyrex glass.
NDA/NA Solved Paper 2019 (II) 21

1
Ê (c) Pyrex glass is harder than soda lime 61. Which one of the following Idisc
MR 2
5
glass. This is because, pyrex glass, statements about water is not true? So, = 2 = … (i)
(a particular blend of borosilicate glass) Isphere 2 2 4
(a) Hydrogen bonds are present in liquid MR
has a higher proportion of silicone dioxide 5
water.
than soda lime glass. Due to which, Hence, from Eq. (i), we get
(b) Water has a high boiling point.
pyrex glass does not expand on heating, Idisc > Isphere
(c) Water has a high heat of fusion. So, the disc has higher moment of inertia.
while soda lime glass is not likely to be
(d) Water is a non-polar molecule.
directly and strongly heated.
Thus, the high heating tolerance of pyrex Ê (d) The option (d) contains incorrect
65. Two substances of densities ρ1 and
glass makes it harder than soda lime glass. statement. Water is a polar molecule in ρ2 are mixed in equal volume and
Hence, option (c) is incorrect. which central oxygen atom is their relative density is 4. When
Rest of the given statements are correct. sp3-hybridised. they are mixed in equal masses,
Out of four sp3-hybrid orbit, two for
relative density is 3. The values of ρ1
58. Which one of the following and ρ2 respectively, are
σ bond with H-atom and rest two are
statements is correct? (a) 6, 2 (b) 3, 5 (c) 12, 4 (d) 9, 3
occupied by lone pair of electrons.
(a) Both boiling and evaporation are Ê (a) When substances are mixed in equal
surface phenomena. Thus, due to V-shaped structure
volume.
(b) Boiling is a surface phenomenon, but of H 2O is a polar molecule (µ = 1840).
Then, relative density,
evaporation is a bulk phenomenon. 62. A sample of ‘soft soap’ contains Vρ + Vρ 2
4= 1
(c) Both boiling and evaporation are bulk
(a) caesium (b) potassium V + V
phenomena.
(c) calcium (d) magnesium [Q volume(V) x density (ρ) = mass (M)]
(d) Boiling is a bulk phenomenon, but
evaporation is a surface phenomenon. Ê (b) Soaps are the sodium or potassium ⇒ ρ1 + ρ2 = 8 …(i)
salts of long chain fatty acid (RCOONa). Similarly, when substances are mixed in
Ê (d) Boiling is a bulk phenomenon, but e.g. stearic acid (C 17H 35COOH), oleic equal mass.
evaporation is a surface phenomenon is
acid (C 17H 33COOH) and palmitic acid Relative density,
the correct statement. This is because,
(C 15H 31COOH). Therefore, a sample of
during evaporation, the high energy M + M 2ρ1ρ2
soft soap contains sodium or potassium. 3= = … (ii)
molecules from the liquid go into the M
+
M ρ1 + ρ2
gaseous phase only through the surface. Thus, the option (b) is correct.
ρ1 ρ2
While, in boiling, the vaporisation occurs 63. If an object moves at a non-zero Hence, from Eqs. (i) and (ii), we get
throughout the bulk of the liquid and the constant acceleration for a certain
vapours expand freely into the surrounding. ρ1 = 6 and ρ2 = 2
interval of time, then the distance it
59. Which one of the following covers in that time 66. A car starts from Bengaluru, goes 50
statements about diamond and (a) depends on its initial velocity km in a straight line towards South,
graphite is not correct? (b) is independent of its initial velocity immediately turns around and
(a) Diamond has a tetrahedral structure, (c) increases linearly with time returns to Bengaluru. The time
whereas graphite has a hexagonal (d) depends on its initial displacement taken for this round trip is 2h. The
planar structure. magnitude of the average velocity of
Ê (a) If object moving with non-zero constant the car for this round trip
(b) Both physical and chemical properties acceleration, then the displacement
of diamond and graphite are different. (a) is zero (b) is 50 km/h
covered by it, is given by equation
(c) Graphite is soft, but diamond is hard. 1 (c) is 25 km/h
s = ut + at 2 (d) cannot be calculated without knowing
(d) Graphite is a good conductor of 2
acceleration
electricity, while diamond is not. where, s = displacement, u = initial
Ê (b) Statement given in option (b) is velocity, a = acceleration and t = time. Ê (a) According to question, a round trip
incorrect. The correct statement is, the is mentioned by a car. But as we
Hence, from above equation, displacement
physical properties of both diamond and know, for round trip the displacement is
is dependent to initial velocity. Distance
graphite are same. But, as the both zero. So,
covered is a non-linear curve of time.
displacement 0
diamond and graphite are allotropes of vav = = = 0 ms − 1
same element (carbon). 64. A solid disc and a solid sphere have time 2
Thus, they both exhibits similar chemical the same mass and same radius. Hence, option (a) is correct.
properties. Which one has the higher moment
So, option (b) is the correct answer. of inertia about its centre of mass? 67. “The sum of emf’s and potential
(a) The disc differences around a closed loop
60. The raw materials used for the (b) The sphere equals zero” is a consequence of
manufacture of Portland cement are (c) Both have the same moment of inertia (a) Ohm’s law
(a) lime, silica and sulphur dioxide (d) The information provided is not (b) conservation of charge
(b) lime, silica and carbon dioxide sufficient to answer the question (c) conservation of momentum
(c) lime, silica and alumina (d) conservation of energy
Ê (a) As we know, moment of the inertia
(d) lime, silica and boric acid
1 2 Ê (d) It is Kirchhoff’s second law also
Idisc =MR 2 and Isphere = MR 2 known as loop’s law.
Ê (c) The raw materials used for the 2 5
manufacture of Portland cement are i.e. ΣE = iR
lime, clay (provides alumina and silica) Q Given, Mdisc = Msphere and
It is based on conservation of energy,
and gypsum. Rdisc = Rsphere
since work done in a closed path is zero.
Thus, the correct option is (c).
22 NDA/NA Solved Paper 2019 (II)

68. A student measures certain lengths 71. Which one of the following 75. Which one of the following
using a meter scale having least count statements regarding lenses is not substances is not a mixture?
equal to 1 mm. correct? (a) Tin (b) Sea water
Which one of the following (a) A convex lens produces both real and (c) Soil (d) Air
virtual images.
measurement is more precise? Ê (a) A mixture is a type of matter which has
(a) 0.50 mm (b) A concave lens produces both real variable composition throughout.
(b) 29.07 mm and virtual images. Among the given options, only Sn(tin) is
(c) 0.925 mm (c) A convex lens can produce images not a mixture, while rest are the examples
(d) 910 mm equal, greater and smaller than the of mixture. Sn is an element which
size of the object.
Ê (c) Using instrument of higher precision (d) A concave lens always produces
contains only one kind of particles or
improving experimental techniques, etc., atoms.
images smaller than the size of the
we can reduce the least count error.
Here, option (c) 0.925 mm is more
object. 76. Which one of the following is
precise than other. Ê (b) A concave lens always formes virtual termed as ‘dry ice’?
image, whereas convex lens forms both (a) Ice present in ice-cream
69. If the work done on the system or by real and virtual images. (b) Solid water at Antarctica
the system is zero, then which one of Hence, option (b) is incorrect. (c) Solid state of carbon dioxide
the following statement for a gas (d) Solid water of ionosphere
kept at a certain temperature is
72. 10 g of ice at − 10° C is mixed with 10
g of water at 0°C. The amount of heat Ê (c) When liquefied CO 2 is allowed to
correct? expand rapidly, it gets converted in the
required to raise the temperature of
(a) Change in internal energy of the form of solid CO 2 which is also called
system is equal to flow of the heat in or
mixture to 10°C is dry ice. Thus, dry ice is solid state of the
out of the system. (a) 400 cal (b) 550 cal carbon dioxide.
(b) Change in internal energy of the (c) 1050 cal (d) 1200 cal
77. Net movement of water from a dilute
system is less than heat transferred. Ê (c) Heat require by 10 g of water at
(c) Change in internal energy of the − 10° C to 0°/C is to a concentrated solution through a
system is more than the heat flow. H1 = ms∆θ selectively permeable membrane is
(d) Cannot be determined. = 10 × ( 0.50) ( 0 − ( − 10) = 50 cal
called
Ê (a) For first law of thermodynamics, (a) Diffusion (b) Dispersion
Now, latent heat of melting of 10 g of ice
∆Q = ∆U + ∆W (c) Osmosis (d) Absorption
H2 = mL = 10 × 80 = 800 cal
where, ∆Q = heat, Now, there is 20 g of water at 0°C, so heat
Ê (c) The net movement of water from a
dilute to concentrated solution through a
∆U = change in internal energy required to reach the temperature of selectively permeable membrane is
and ∆W = work done. + 10° C. called osmosis. This term is used
Since, ∆W = 0 H3 = ms∆θ = 20 × 1 × (10 − 0) = 200 cal specifically to refers to the diffusions of
So, ∆Q = ∆U Total heat required, water across semi or selectively
Hence, there is change in internal energy H = H1 + H2 + H3 permeable membrane.
of the system is equal to flow of heat in or = 50 + 800 + 200 = 1050 cal Water moves from a region of higher
out of the system, option (a) is correct. concentration to a region of lower
73. Which one of the following does not concentration until an equilibrium is
70. Consider the following part of an represent the salt, calcium reached.
electric circuit. carbonate?
2W 78. Blood is a type of
(a) Lime water (b) Limestone
(a) epithelial tissue
(c) Chalk (d) Marble
4W 1W
(b) muscular tissue
Ê (a) Calcium carbonate is also known as (c) nervous tissue
limestone and has a molecular formula is (d) connective tissue
8W CaCO 3. It is found in nature in the form of
chalk, marble etc. Thus, limestone. chalk Ê (d) Blood is a type of connective tissue,
which forms 30-32% of total extracellular
The total electrical resistance in the and marble represents calcium
fluid. The connective tissues have a
given part of the electric circuit is carbonate. Whereas, lime water is the
15 15 special function of linking and supporting
(a) Ω (b) Ω aqueous solution of Ca(OH)2.
other tissues/organs of the body.
8 7 Thus, the correct option is (a).
17 Blood is a fluid, which helps in the
(c) 15 Ω (d) Ω
3 74. Soaps clean surfaces on the principle transportation of various substances in
of based on the body. It is red because it contains a
Ê (b) There are three resistances (2 Ω, 4Ω red pigment called haemoglobin in its
and 8Ω) are in parallel. (a) viscosity (b) floatation
(c) elasticity (d) surface tension red blood cells.
1 1 1 1 8
So, = + + ⇒ R1 = Ω
R1 2 4 8 7 (d) Soaps clean surfaces on the principle 79. Mitochondria are able to produce
of based on surface tension. Soap is a their own
Now, R1 is in series with 1 Ω resistance.
surface active agent or surfactant. It (a) nucleus
8 15
So, Req = 1 Ω + R1 = 1 + = Ω helps to reduce the surface tension of (b) proteins
7 7 water and thereby speed up the cleaning (c) chloroplasts
Hence, option (b) is correct. process of soaps. (d) digestive enzymes
NDA/NA Solved Paper 2019 (II) 23

H = I   Rt = IVt
V
Ê (b) Mitochondria are able to produce their So, G(2 M ) (2 M ) 16 GM 2
own protein. It acts as the cell’s power  R F′ = 2
= … (ii)
 R R2
producer and also called as powerhouse Hence, option (a) is correct.  
of the cell. 2
It possesses single circular DNA 83. A pendulum clock is lifted to a From Eqs. (i) and (ii), we get
molecules, a few RNA molecules, height where the gravitational F ′ = 16 F
ribosomes and the components required acceleration has a certain value of g. Hence, option (a) is correct.
for the synthesis of proteins. It provides Another pendulum clock of same
energy for energy requiring processes of length, but of double the mass of the 86. Light rays move in straight lines.
the cell like muscle contraction, nerve bob is lifted to another height where But through an optical fibre, they
impulse, conduction, etc. It may store g can move in any type of zig-zag path
and release calcium when required. the gravitational acceleration is . because
2
80. Which one of the following The time period of the second (a) the holes through the fibres are
statements is correct? pendulum would be extremely fine
(b) light rays are absorbed at the entry
(a) Xylem consists of tracheids, vessels, (in terms of period T of the first end and relieved at the exit end of the
xylem parenchyma and xylem fibres pendulum) fibre
(b) Flexibility in plants is due to 1
(a) 2 T (b) T (c) scattering of light occurs inside the
sclerenchyma 2 fibre
(c) Parenchyma have no intercellular (c) 2 2 T (d) T (d) successive total internal reflections
spaces
(d) Xylem consists of sieve plate, sieve Ê (a) Time period of a pendulum, occur as a ray moves through the fibre
l Ê (d) The working of optical fibre is based
tube and companion cells T = 2π ...(i)
g on the total internal reflection. Its inner
Ê (a) The statement given in option (a) is When acceleration due to gravity part is core of higher refractive index
correct. Rest of the statements are
g surrounded by another layer of glass of
incorrect and can be corrected as changed to , then time period,
2 lower refractive index. It is surrounded by
(b) Flexibility in plants is due to the
l  l  plastic jacket.
permanent tissue collenchyma. T′ = 2 π = 2 2 π 
g /2  g When light enters from the one end of the
Sclerenchyma provides mechanical
core and moves towards cladding, then
support to organs. [from Eq. (i)]
total internal reflection takes place again
(c) Parenchyma cells are thin walled ⇒ T′ = 2 T
and again and light propagate through it.
with small intercellular spaces. Hence, option (a) is correct.
Optical fibres are used in decorative
(d) Phloem consists of sieve plates,
84. The loudness of sound depends table lamps.
sieve tube and companion cells.
upon the 87. Which one of the following
81. Who among the following popularised (a) velocity of sound waves in the statements regarding cathode rays
the use of embryological characters medium
is not correct?
in taxonomy? (b) amplitude of the sound waves
(a) Cathode ray particles are electrons.
(a) Carl Linnaeus (c) frequency of the sound waves
(b) Cathode ray particles start from
(b) Panchanan Maheshwari (d) frequency and velocity of the sound
anode and move towards cathode.
(c) Birbal Sahni waves
(c) In the absence of electrical and
(d) Bentham and Hooker Ê (b) Loudness of sound wave is depend magnetic fields, cathode rays travel in
(b) Panchanan Maheshwari popularised on its amplitude, while sharpness of straight lines.
the use of embryological characters in sound wave is depend on its frequency. (d) Television picture tubes are cathode
taxonomy. He is also known as ‘The Hence, option (b) is correct. ray tubes.
Father of Indian Plant Embryology’.
85. Two bodies of mass M each are Ê (b) Cathode rays start from cathode
82. Water is heated with a coil of placed R distance apart. In another and move towards anode, because
resistance R connected to domestic system, two bodies of mass 2M each they are made of negatively charged
electrons.
supply. The rise of temperature of R
are placed distance apart. If F be Hence, option (b) is incorrect statement.
water will depend on 2
1. supply voltage the gravitational force between the 88. A very large volume of hydrogen can
2. current passing through the coil bodies in the first system, then the be accommodated by making
3. time for which voltage is supplied gravitational force between the (a) non-metallic hydrides
Select the correct answer using the bodies in the second system will be (b) hydrogen peroxide
codes given below. (c) non-stoichiometric hydrides
(a) 16 F (b) 1 F
(a) 1, 2 and 3 (b) 1 and 2 (d) alkali metal hydrides
(c) 4F (d) None of these
(c) Only 1 (d) 2 and 3 Ê (c) A very large volume of hydrogen can
Ê (a) Situation I, be accommodated by making
Ê (a) Heat generated by a resistance coil is Force between the masses
non-stoichiometric hydrides.
given by GMM GM 2
H = I 2Rt F = = … (i) Such hydrides are formed by some of the
R2 R2 metals (e.g. Pd, Pt). This property of
V
From Ohm’s law, I = Situation II, hydrogen has high potential for
R Force between the masses hydrogen storage .
24 NDA/NA Solved Paper 2019 (II)

89. Which one of the following is not a Ê (c) Salmonella typhi is a bacterium which (Take, acceleration due to
monoatomic element? causes typhoid fever in the human body. gravity, g = 9.8 m /s2 )
This fever is contracted by the ingestion (a) 980 ms − 1 and 980 J
(a) Copper (b) Helium
(c) Iodine (d) Barium
of contaminated food and water.Varicella (b) 980 ms − 1 and 980 J
zoster is a virus causing chickenpox. (c) 980 ms − 1 and 980 J
Ê (c) Among the given species, only iodine Trypanosoma gambiense and (d) 980 ms − 1 and 980 J
is not a monoatomic element. It exists in Plasmodium falciparum are protozoans
the form of I 2 (diatomic), while rest occur which cause sleeping- sickness and Ê (d) Given, M = 2 kg and height = 50 m
as monoatomic element. malaria, respectively in the human body. Speed of body at the ground,
90. Which one of the following v 2 − u 2 = 2 gH
94. Which one of the following statements
represents the correct order of regarding viruses is not true? v = 2 gH [Qu = 0]
electron releasing tendency of (a) Viruses need living cells to reproduce = 2 × 9.8 × 50
metals? (b) All viruses are parasites ⇒ v = 980 ms –1
(a) Zn > Cu > Ag (b) Ag > Cu > Zn (c) Viruses can synthesise their food
(c) Cu > Zn > Ag (d) Cu > Ag > Zn Hence, the total energy = kinetic energy
through photosynthesis
1
Ê (a) The correct order of electron releasing (d) Viruses are similar to chemical ⇒ TE = Mv 2
substances outside the host 2
tendency of given metals is
1
Zn> Cu > Ag. Ê (c) The statement given in option (c) is = × 2 × ( 980 )2 = 980 J
2
This can be explained on the basis of not true and can be corrected as
Viruses do not perform photosynthesis Hence, option (d) is correct.
standard reduction potentials of given
° to synthesise their food. They are inert,
elements. More negative be the Ered 98. The temperature of a place on one
crystalline structure outside the living
value, greater is the tendency of element
cell.
sunny day is 113 in Fahrenheit scale.
to release electron. The Kelvin scale reading of this
° Once they infect a cell, they takeover the
Zn has highly negative Ered value, while
machinery of the host cell to replicate temperature will be
°
Cu has least negative Ered value. themselves, killing the host. (a) 318 K (b) 45 K
(c) 62.8 K (d) 335.8 K
Thus, the correct order is Rest of the statements are correct.
Zn > Cu > Ag Ê (a) Given, temperature,
95. Which one of the following diseases Tf = 113 F
91. Which one of the following is the may be caused by the deficiency of 10
major constituent of biogas? vitamin-C? As, we know Tk = 273 + (Tf − 32 )
18
(a) Carbon dioxide (b) Nitrous oxide (a) Rickets (b) Rabies 10
(c) Methane (d) Oxygen (c) Hepatitis (d) Scurvy ⇒ Tk = 273 + (113 − 32 )
18
Ê (c) Biogas usually contains 45-70% Ê (d) The deficiency of vitamin-C (ascorbic ⇒ Tk = 318 K
methane and 30-45% carbon dioxide. It acid) causes scurvy. This condition is
also contains small amounts of nitrogen, characterised by general weakness, 99. Radiations coming from the sun are
hydrogen sulphide, halogenated compound anaemia, gingivitis and skin haemorrhage. mostly in the form of
and organic silicon compounds. Thus, (a) light only
among the given options, methane is the
96. Compared to audible sound waves,
(b) light and long wavelength infrared
major constituent of biogas. ultrasound waves have
(c) light and short wavelength infrared
(a) higher speed (b) higher frequency
92. Thyroid gland produces a hormone (d) Both short and long wavelength
(c) Both (a) and (b (d) long wavelength
infrared
called ‘thyroxine’ which
Ê (b) Ultrasound waves have higher
(a) controls blood glucose levels frequency than audible sound waves. Ê (b) The mode of transfer of heat from one
(b) controls ovulation body to another without any actual
The audible range of sound for human movement the particles involved and
(c) controls metabolic rate
beings is form 20 to 20,000 Hz. Children without heating the intervening medium,
(d) maintains pregnancy
under the age of 5 and some animals is called radiation. For radiation, there is
Ê (c) Thyroid gland produces a hormone such as dogs can hear upto 25,000 Hz. no need of any medium because it is of
called thyroxine, which controls the As people grow older, their ears become
metabolic rate of the body. It acts to electromagnetic nature.
less sensitive to higher frequency.
increase the basal metabolic rate and So, radiation coming from the sun are
affects protein, fat and carbohydrate Ultrasonic sound of frequency higher mostly in form of light and long
metabolism affecting how human cells than 20,000 Hz, which cannot be heard wavelength infrared.
use energetic compounds. by human beings.
100. Which of the following are the
93. Which one of the following is a 97. A rigid body of mass 2 kg is dropped characteristics of electromagnetic
bacterium that causes disease in the from a stationary balloon kept at a waves?
human body? height of 50 m from the ground. The
1. They are elastic waves.
(a) Varicella zoster speed of the body when it just
2. They can also move in vacuum.
(b) Trypanosoma gambiense touches the ground and the total
(c) Salmonella typhi 3. They have electric and magnetic
energy when it is dropped from the
(d) Plasmodium falciparum components which are mutually
balloon are respectively. perpendicular.
NDA/NA Solved Paper 2019 (II) 25

4. They move with a speed equal to 3 straight, with very few tributaries and all of expansion of the universe which
lakh meters per second. flow in the same direction. continues to a day.
Select the correct answer using the Radial drainage patterns form when According to Planetesimal Hypothesis,
codes given below. streams and rivers flow off a central peak when a planetary system was forming,
(a) 1, 2, 3 and 4 (b) 1, 2 and 4 or dome like a volcano. there was a proto planetary disc with
(c) 2 and 3 (d) 3 and 4 In a trellis pattern, the river forms a net materials form the nebula from which the
Ê (c) Electromagnetic waves or EM waves like system and the tributaries flow system came.
are waves that are created as a result of roughly parallel to each other.
vibrations between an electric field and a
107. Which among the following
magnetic field. They are formed when an
104. Which one of the following ocean struggles, based on Gandhi’s
electric field comes in contact with a
currents is a cold current? philosophy of Satyagraha, involved
magnetic field. The electric field and (a) South Atlantic Drift the industrial working class?
magnetic field of an electromagnetic (b) Mozambique Current (a) Champaran (b) Kheda
wave are perpendicular (at right angles) (c) East Australian Current (c) Ahmedabad (d) Bardoli
to each other. They are also (d) Caribbean Current
Ê (c) Ahmedabad Mill Strike, 1918 was
perpendicular to the direction of the EM Ê (a) South Atlantic Drift/Current is an based on Gandhi’s philosophy of
wave. eastward flowing cold ocean current, fed Satyagraha, that involved the industrial
These waves travel with a constant by the Brazil Current. It is continuous with working class. The industrial dispute for
velocity of 3.0 × 10 8m / s in vacuum. They the northern edge of the Antarctic wage hike was between the owners and
are deflected neither by the electric field Circumpolar Current. workers of a cotton mill in Ahmedabad.
nor by the magnetic field. The Mozambique current is a warm The result of peaceful strike led by
Hence-statement 2 and 3 are correct. ocean current flowing along the east Gandhiji was successful and workers got
coast of Africa in Indian ocean. It flows a 35% wage increase.
101. In the Manusmriti which form of between Mozambique and Madagascar
marriage results from the ‘Voluntary islands. 108. Consider the following statements
union of a maiden and her lover’? The East Australian Current (EAC) is a
related to the Virashaiva tradition of
(a) Eighth form (b) Fifth form warm ocean current that flows along the Karnataka.
(c) Seventh form (d) Sixth form eastern cost of Australia. 1. The Virashaiva Movement was led
Ê (b) In the Manusmriti, fifth (Gandharva The Caribbean Current is a warm ocean by a Brahmana named Basavanna,
form of marriage) results from the current that flows northwestward through a minister in the court of the
‘Voluntary union of a maiden and her the Caribbean sea along the coast of Kalachuri ruler in the twelfth
love’. In Gandharva Marriage, a groom South America and into the Gulf of century.
and his bride could getmarried without Mexico.
their parents knowledge or sanction.
2. The Virashaivas encouraged
Hence, option (d) is the correct answer. 105. Paradip Port is located on the delta practices, approved by the
of river Dharmashastras, like post-puberty
102. Which one of the following is a Rabi marriage and remarriage of
(a) Rihand (b) Ganga
crop in the Northern States of India? (c) Mahanadi (d) Bitarani widows.
(a) Rice (b) Bajra Which of the statement(s) given
(c) Barley (d) Ragi Ê (c) It is a natural, deep-water port on the above is/are correct?
East coast of India in Jagatsinghpur
Ê (c) Rabi crops are winter crops which are district of Odisha. Paradip Port is (a) Only 1 (b) Only 2
sown in October-November and situated at confluence of the Mahanadi (c) Both 1 and 2 (d) Neither 1 nor 2
harvested in March. The Rabi crops in river and the Bay of Bengal. Ê (a) The twelfth century witnessed the
Northern states of India includes Barley, emergence of a new movement called
wheat, oats, gram, linseed and mustard. 106. Which one of the following Virashaiva Movement in Karnataka. The
hypothesis/theory explains the movement was led by a Brahmana
103. The rivers of North-West Europe origin of the Universe? named Basavanna (1106-68) who was
are good examples of
(a) Nebular Hypothesis initially a Jaina and a minister in the court
(a) parallel pattern of drainage (b) Binary Theory of a Kalachuri ruler (Chalukya, king of
(b) radial pattern of drainage (c) Big Bang Theory Gujarat). His followers were known as
(c) barbed pattern of drainage (d) Planetesimal Hypothesis Virashaivas (heroes of Shiva) or
(d) trellis pattern of drainage. Lingayats (wearers of the linga). The
Ê (a) Nebular Hypothesis was given by
Ê (c) The rivers of North-West Europe are Immanuel Kant explain about the origin of Chalukyas were contested for the
good examples of barbed drainage Universe. According to this theory, plants suzerainty by the kalachuris of Southern
pattern. Barbed drainage pattern is a were formed out of a cloud (nebula) of India in that period. Hence, statement (1)
rare kind of drainage pattern which is hydrogen, helium & dust surrounding the is correct.
formed when the tributaries flow in sun. Binary theory explain about the Also, the Dharmashashtras did not
opposite direction to their master existence of companion star of the sun. permit widow remarriages and
streams. post-puberty marriages. But the
According to Big Bang theory, all matter
A parallel pattern of drainage system is Virashaivas undertook them as a sign of
or substance forming this universe
a pattern of river caused by steep slopes refusal of the sacred Brahmanical
existed at one place as a tiny ball. This
with some relief. Because of the steep scriptures. Hence, statement (2) is in
tiny ball had infinite density and
slopes, the streams are swift and correct.
temperature. At the Big Bang Theory the
tiny ball exploded and started a process
26 NDA/NA Solved Paper 2019 (II)

109. Consider the following statements necessary in a repressive society like remains like forests & swamps
related to Wellesley’s Tsarist Russia. (carbonaceous rocks). Some examples
Hence, statement (1) is correct. of organically formed sedimentary rocks
administration.
are chalk, limestone, peat, lignite & coal.
1. In the six years of Wellesley’s On the other hand Mensheviks believed
in power of masses and advocated Shale is a mechanically formed
administration, the army sedimentary rock. Halite & chert are
liberal thoughts like entry of common
accounted for 42.5% of the masses in the party. Hence, statement chemically formed sedimentary rocks.
Company’s total expenditure. (2) is also correct.
2. Wellesley’s administrative 114. According to the Koppen climatic
measures were restricted to the 111. Which of the following was/were classification, the letter code Cfa
affairs to the Company and had among the decisions taken by the denotes.
nothing to do with the commercial First Continental Congress held in (a) Tropical wet climate
Philadelphia in September, 1774? (b) Humid subtropical climate
and military affairs of the Indian
(c) Tundra Climate
ruling families. 1. It rejected a plan for a colonial
(d) Tropical wet and dry climate
union under the British authority.
Which of the statement(s) given
2. It drew up a statement of Ê (b) Koeppen’s climate classification is
above is/are correct? based on annual & monthy average
(a) Only 1 (b) Only 2 grievances. temperatures and precipitation.
(c) Both 1 and 2 (d) Neither 1 nor 2 3. It agreed upon a scheme to stop According to this classification Cfa
Ê (c) During Lord Wellesley’s tenure, army trade between the Colonies and denotes Humid subtropical climate.
was an expensive machinery of the England. This type of climate is characterised by
British East India Company. He had Which of the statement(s) given warm summer and absence of dry
waged war on many Princely States and above is/are correct? season. Tropical wet climate is denoted
native rulers like Mysore, Marathas, (a) 1 and 3 (b) Only 2 by Af; Tundra climate is denoted by ET
Nizams etc. Hence, to sustain such a (c) 1 and 2 (d) 1, 2 and 3 and Tropical wet & dry climate is denoted
larger army, he spent huge amounts by Aw.
(42.5% of the compary’s total
Ê (d) The First Continental Congress held in
Philadelphia in September, 1774 is a
expenditure) on army.He also devised 115. In which one of the following folds
major landmark in the history of USA. It
the infamous Subsidiary Alliance System was attended by representatives of 12
in the axial plane found to be
which required to maintain British army in out of 13 British Colonies in USA, at the virtually horizontal?
native ruler’s courts. Hence, statement Carpenter’s Hall. It was held in response (a) Isoclinal (b) Anticlinal
(1) is correct. to British impasses over the historic (c) Recumbent (d) Monoclinal
Wellesley was an excellent administrator. Boston Tea Party of December, 1773. Ê (c) In recumbent fold, aecial plane found
He used his Subsidiary Alliance System The major outcomes of this Congress to be horizontal. The axial plane of a fold
to interfere with the commercial and were is the plane or surface that divides the
military affairs to Indian rulers by making ● Rejection of Union of colonies under fold as symmetrically as possible.
them mandatorily consult him on British Crown. In Isoclinal fold, the axial surface and
important decisions. ● Scheme for boycott of trade between
limbs slope in the same direction and at
Hence, statements (2) is also correct. the colonies and the Britishers. approximately the same angle.
● Mechanism of grievance discussion.
110. Consider the following statements An Anticlinal fold is an arch like fold
Hence, all the statements are correct.
concerning the Russian Revolution. which is convex upward.
1. The Bolsheviks believed that in a
112. Which one of the following land Monoclinal fold is a step-like fold
territories of Indonesia is not consisting of a zone of steeper dip within
repressive society like Tsarist
touched by the Equator? or otherwise horizontal topography.
Russia, the party should be
(a) Sumatra (b) Sulawesi
disciplined and should control the 116. How many Ramsar convention
(c) Java (d) Kalimantan
number of its members. sites have been enlisted from India
2. The Mensheviks believed that the Ê (c) Java island is located to the South of so far?
equator at around 5°S latitude in a West
party should be opened to all (as in to East orientation, so Java is not
(a) 12 (b) 16 (c) 27 (d) 62
Germany). touched by the equator. However, Ê (c) Has 27 Ramsar sites like Dal lake,
Selects the correct answer using the equator passes through the Sumatra, Pong Dam, Harike Wetland, Chilika lake
codes given below. Sulawesi and Kalimantan portion of etc.
(a) Only 1 (b) Only 2 Borneo island in Indonesia. Ramsar Convention on Wetlands is an
(c) Both 1 and 2 (d) Neither 1 nor 2 international treaty on conservation and
113. Which one of the following sustainable use of Wetlands. It was
Ê (c) The Bolsheviks and Mensheviks were sedimentary rocks is organically signed in Ramsar, a city in Iran in 1971.
the two factions of Russian Social
formed? India Chilika lake was first to be
Democratic Labour Party formed under
leaderships of Vladimir Lenin and Julius (a) Shale (b) Chert designated as a Ramsar site in India.
(c) Halite (d) Chalk
Martov respectively.
117. The Munda language group belongs
Bolsheviks believed in limiting the Ê (d) Organically formed sedimentary rocks
are formed from the remains of living to which family?
membership of party to a core
organisms such as corals & shell fish (a) Austric (b) Dravidian
professional member body as it was
(calcareous rock); or from vegetative (c) Sino-Tibetan (d) Indo-European
easier to manage them which was
NDA/NA Solved Paper 2019 (II) 27

Ê (a) Munda language belongs to the anyone could be arrested. Hence, Ê (b) Article 46 in part IV of the Constitution
Austroasiatic or Austric group of option (c) is not correct about the Reign of India is related to the promotion of
language. It is a group of language with of Terror. The reign came to an end on educational and economic interests of
more than 9 million user from central and 27th July, 1794 with the execution of the Scheduled Castes, Scheduled Tribes
eastern India. Robespierre by guillotine. and other weaker sections. Article 46
Dravidian languages are Tamil, Telugu, Hence, option (c) is incorrect. also provides that the state shall protect
Kannada and Malyalam. Sion-Tibetan the weaker sections of the society from
languages includes chinese and the 120. Who among the following was the social in justice and all forms of exploitation.
Tibeto-Busman languages. first Indian to qualify for the Indian
Indo-European languages are dialects Civil Service? 123. Who was the Education Minister in
in Asia and Europe. (a) Satyendranath Tagore the first Cabinet of Independent India?
(b) Surendranath Banerjee (a) Sarvepalli Radhakrishnan
118. Which one of the following (c) RC Dutt (b) Sardar Vallabhbhai Patel
statements about the Suez Canal is (d) Subhas Chandra Bose (c) Maulana Abul kalam Azad
not correct? (d) Acharya Narendra Dev
Ê (a) Satyendranath Tagore was the first
(a) The Suez Canal was constructed in Indian to join the Indian Civil Service. He Ê (c) Maulana Abul Kalam Azad was the
1869. was selected in 1863 and posted in Education Minister in the first Cabinet of
(b) It links the Mediterranean Sea and the Bombay Presidency as a Civil Servant. Independent India.
Read Sea. National Education Day is celebrated on
(c) It has a six lock system and ships 121. Which among the following 11th November every year to
cross the different levels through was/were among the factors for commemorate the birth anniversary of
these locks. England to experience the Industrial Maulana Abul Kalam Azad.
(d) It has given Europe a new gateway to Revolution first?
the Indian Ocean. 124. Who among the following died in exile?
1. The scientific inventions had paved
Ê (c) Option, (c) is not the correct statement the way for Industrial Revolution.
(a) Rani Laxmibai
about the Suez Canal because. It has no (b) Bahadur Shah Zafar
lock system as the water level in 2. It had favourable social and (c) Tantia Tope
Mediterranean Sea and Red Sea through political structures in place. (d) Nana Saheb
Gulf of Suez is nearly same. The Suez 3. Navigable rivers had made inland Ê (b) Bahadur Shah Zafar → Died in
Canal in Egypt connects the Red Sea transport easier. Rangoon while in exile in 1862.
with Mediterranean Sea. It was 4. It had seen the ascendancy of Rani Laxmibai→ Died on the battlefield in
constructed between 1859 to 1869 and capitalist practices. Gwalior on 18th June, 1858.
officially opened in 1869.
Select the correct answer using the Tantia Tope → Executed by Britishers in
It gives Europe a new gateway to the codes given below. 1859 at Shivpuri.
Indian Ocean allowing huge trade and (a) 1 and 3 (b) 1, 3 and 4 Nana Saheb → Escaped to Nepal and is
cargo movement between the Europe (c) Only 4 (d) All of these thoughts to have died in 1859.
and Asian Countries.
Ê (d) Industrial Revolution was the
125. The normal lapse rate of temperature
119. Which among the following technological implementation in mass
statements is not correct about the production during the second half of of Earth’s atmosphere drops to 0°C
Reign of Terror? 18th century in England. It was a period at the
of transition with new machines (a) upper part of ionosphere
(a) The Jacobins dashed out the
technologies and steam power driving (b) upper boundary of the tropopause
Girondists with the help of
Sans-Culottes. the production capacity. The causes/ (c) lower part of mesosphere
factors for England to experience the (d) upper boundary of stratopause
(b) The Jacobins were in league with the
French clergy. Industrial Revolution first were (b) The normal lapse rate of temperature
(c) The arrest for anti-revolutionary 1. Scientific innovations. of Earth’s atmosphere drops to 0°C at the
activities was by law restricted to the 2. Favourable Social and Political upper boundary of the tropopause. The
political class. Structure lapse rate is the rate at which
(d) The Reign of Terror came to an end 3. Navigable rivers temperature in Earth’s atmosphere
with the execution of Robespierre by 4. Higher Capital in hands for setting changes with altitude.
guillotine in July, 1794. up enterprise. 126. The headquarters of National Research
Ê (c) Reign of Terror was the period in Hence, all the statements are correct. Laboratory for Conservation of Cultural
French revolution in 1793 to 1794,
122. Article-46 of the Constitution of Property (NRLC) is located at
marked by widespread violence and
executions by the Jacobins, in India refers to the promotion of (a) New Delhi (b) Lucknow
educational and economic interests (c) Kolkata (d) Bhopal
association with the Sans-Culottes.
The Jacobins wre in league with the of (b) The National Research Laboratory for
French clergy. Conservation of Cultural Property
(a) religious minorities
(NRLC) is a scientific institution of the
Jacobins dashed out Girondists with (b) Scheduled Castes, Scheduled
Ministry of Culture, Government of India.
help of Sans–Culottes and were actively Tribes and other weaker sections
It is responsible for the Conservation of
conspiring against Aristocracy. The laws (c) displaced persons from large Cultural Heritage. The NRLC was
for arresting were not limited to political irrigation projects established in 1976 with its headquarters
class instead it was vaguely defined and (d) the economically deprived located at Lucknow.
28 NDA/NA Solved Paper 2019 (II)

127. Loktak lake is situated in the State of 132. The principal component(s) of Ê (c) Both the statements are correct as
(a) Sikkim (b) Manipur Nehru-Mahalanobis strategy of Dayanand was critical of the ancient
(c) Odisha (d) Mizoram economic development was/were caste system. He criticised it to its length
and breadth. He envisioned a homogenous
Ê (b) Loktak Lake is located at Moirang in 1. Restructuring economic dependency society where the profession/function
Manipur, India. It is the largest fresh on metropolitan capitalism into performed by the person was determinant
water lake of North-East India & is independent economic development. of their merit. Also, Dayanand’s ‘robust
famous for phumdis (floating vegetative
2. Transition from semi-feudal Vedic counterpart’ challenged the masculine
mass). Keibul Lamjao National Park, one West that had enslaved the Aryavarta.
of the phumdis of the Lake, is the only
agricultural to capitalist farming.
floating national park in the world. It is Select the correct answer using the 135. Consider the following statements
also the last natural habitat of endagered codes given below. concerning the American War of
Sangai deer. (a) Only 1 (b) Only 2 Independence.
(d) Both 1 and 2 (d) Neither 1 nor 2
128. Stalactite, stalagmite and pillars are 1. The Americans raised the slogan, ‘No
Ê (c) The Nehru-Mahalanobis strategy was taxation without representation’.
the depositional landforms of developed by PC Mahalanobis, under
(a) running water (b) wind the guidance of Jawaharlal Nehru in
2. The American merchants resorted
(c) glacier 1953. The Principal components of this to the boycott of the British products.
(d) underground water development strategy were Select the correct answer using the
codes given below.
Ê (d) Stalactite, stalagmite and pillars are (i) Restructuring economic dependency
(a) Only 1 (b) Only 2
the depositional landforms of on metropolitan capitalism into
underground water. Stalactite and (c) Both 1 and 2 (d) Neither 1 nor 2
independent economic development.
stalagmite are formed when the calcium (ii) State capitalist and capitalist develop- Ê (c) During the American War of
carbonate dissolved in groundwater. ments in the urban sector and Independence, in 1774, the first
(iii) Transition from semi-feudal Continental Congress was held in
129. A deep valley characterised by steep Philadelphia which actively advocated
step like slope is known as agricultural to capitalist farming.
three agendas—
Hence, Statements 1 and 2 both are correct.
(a) U-shaped valley (b) Blind valley 1. Rejection of Union of Colonies
(c) Gorge (d) Canyon 133. The provisions of the Fifth Schedule under British authority.
Ê (d) Canyon is a deep valley characterised of the Constitution of India shall not 2. Scheme of boycott of trade with
by steep step like slopes. A U-shaped apply to the administration and England and Britain.
valley is a glaciated landform having a control of the Scheduled Areas and 3. No taxation without representation
characteristic U-shape with steep Scheduled Tribes in and redressal of grievances.
straight sides & a flat or rounded bottom. 1. Chhattisgarh, Jharkhand and Odisha So, both statements are correct. Hence,
A blind valley is a deep, narrow & flat 2. Assam and Tripura option (c) is correct.
bottomed valley with an abrupt ending. 3. Meghalaya and Mizoram
A Gorge is a deep, narrow valley with
Select the correct answer using the
136. The first Utkrisht Double Decker
steep sides. Air-Conditioned Yatri (UDAY)
codes given below.
Express train runs between
130. Which one of the following is a (a) Only 1 (b) 1 and 2
(a) Delhi and Kalka
major plate? (c) 2 and 3 (d) 1, 2 and 3
(b) Bengaluru and Coimbatore
(a) Pacific Plate (b) Cocos Plate Ê (c) The provisions of the Fifth Schedule of (c) Chennai and Visakhapatnam
(c) Arabian Plate (d) Philippine Plate the Constitution of India apply to the (d) Chennai and Bengaluru
administration and control of the
Ê (a) Among the given options, Pacific plate Scheduled Areas and Scheduled Tribes Ê (b) The first Utkrisht Double Decker
is a major plate. The outer shell of Earth, Air-Conditioned Yatri (UDAY) Express
Lithosphere is broken into tectonic in 10 states namely: Andhra Pradesh, was a specially designed trains which
plates. Chhattisgarh, Gujarat, Himachal started service between Coimbatore and
Pradesh, Jharkhand, Madhya Pradesh, Bengaluru in June, 2018. Second, such
There are seven major tectonic plates
Maharashtra, Odisha, Rajasthan and service was launched in September,
which include Pacific Plate, North
Telangana. 2019 that runs between Visakhapatnam
American Plate, Eurasian Plate, African
and Vijayawada.
Plate, Antarctic Plate, Indo-Australian 134. Consider the following statements
Plate and South American Plate. There related to Dayanand’s idea of social 137. Who among the following were
are about 20 minor plates including reform selected for the Rajiv Gandhi Khel
Cocos Plate, Nazca plate Arabian Plate 1. Dayanand’s reform envisaged a Ratna Award, 2019?
Philippine plate and Caroline Plate etc. social order, where different castes (a) Vijay Kumar and Yogeshwar Dutt
performed functions suitable to (b) Sakshi Malik and Jitu Rai
131. Lithosphere consists of
their status determined by merit. (c) Virat Kohli and Mirabai Chanu
(a) upper and lower mantle (d) Bajrang Punia and Deepa Malik
(b) crust and core 2. Dayanand’s ‘robust Vedic counter-
(c) crust and uppermost solid mantle part’ challenged the masculine West Ê (d) Rajiv Gandhi Khel Ratna Award is the
(d) mantle and core highest sporting honour of India. It is
that had enslaved the Aryavarta.
annually awarded to players by the
(c) Lithosphere is the solid outer part of Which of the statement(s) given Ministry of Youth Affairs and Sports.
the Earth. It consists of the crust and solid above is/are correct? Bajrang Punia (Wrestling) and Deepa
outermost layer of the upper mantle. (a) Only 1 (b) Only 2 Malik (Parathlete) were awarded with this
Hence option (c) is correct answer. (c) Both 1 and 2 (d) Neither 1 nor 2 award in 2019.
NDA/NA Solved Paper 2019 (II) 29

138. Betel leaf of which one of the with him. This led to ruthless extration of 146. World Humanitarian Day is
revenue from poor peasants. The celebrated on
following states has been accorded a
revenue was not fixed and varied upon
GI tag recently? the production. Hence, statement (c) is
(a) 26th June (b) 17th July
(a) Andhra Pradesh (b) Madhya Pradesh (c) 19th August (d) 27th September
incorrect.
(c) Kerala (d) Mizoram Ê (c) World Humanitarian Day is an
142. During the Swadeshi movement, a international day celebrated on 19th
Ê (c) The Tirur Vettila (Betel leaf) from National College was started in
Kerala has got its GI tag. The leaf given August every year to recognise the
its unique component i.e., Eugenol in Calcutta under the principalship of humanitarian personnel and those who
leaves along with high protein and (a) Rabindranath Tagore laid down their lives for the causes of
chlorophyll content has several medicinal (b) Aurobindo Ghosh humanity. It was designated by the
properties. Apart from this Arnamula (c) Rajani Kant Sen United Nation General Assembly for the
Kannadi, Payyannur Pavithra Ring, (d) Syed Abu Mohammad first time in 2009 on 19th August.
Navara Rice, Alleppey Coir also gets GI Ê (b) Aurobindo Ghosh founded the Bengal 147. Which one of the following is the
tag in Kerala. National College at Calcutta on 15th name given to the lander of
August, 1906 and became its first Chandrayaan-2?
139. SAMARTH is a flagship skill Principal. But following his trial in the
development scheme of which one (a) Vigyan (b) Anusandhan
cases like Bande Mataram (Sedition
of the following Ministries? (c) Pragyan (d) Vikram
case), 1907 and Alipore Bomb
(a) Ministry of Textiles Conspiracy Case, 1908, he resigned Ê (d) Chandrayaan-2 was the second lunar
(b) Ministry of Agriculture and Farmers from the post. exploration mission of Indian Space
Welfare Research Organisation (ISRO). It consisted
(c) Ministry of Skill Development and 143. The ruins of Vijayanagar (Hampi) of a lunar orbiter, lander named ‘Vikram’
were brought to public light in the and lunar rover called ‘Pragyan’. The
Entrepreneurship
year 1800 by the following British Mission’s Lander was named after Vikram
(d) Ministry of Human Resources Sarabhai, who is widely regarded as the
Development surveyor and engineer
founder of Indian Space Programme.
(a) James Prinsep (b) Colin Mackenzie
Ê (a) SAMARTH is a flagship skill (c) James Rennell (d) Charles Metcalfe 148. Along with Pranab Mukherjee, how
development scheme of Ministry of
Textiles, Government of India. The Ê (b) Colin Mackenzie an engineer and many more persons were conferred
objective of the scheme is to provide skill antiquarian in East Indian Company the Bharat Ratna Award in the year
development to the youth for gainful and found the ruins of Vijayanagar (Hampi) in 2019?
sustainable employment in the textile 1800. He prepared the first survey map (a) One (b) Two
sector. of Hampi. James Prinsep, deciphered (c) Three (d) Four
Kharoshti and Brahmi scripts. Ê (b) In the year 2019, along with Pranab
140. CORAS is a separate Commando James Rennell, produce first accurate Mukherjee, two personalities were
Unit of maps of Bengal and the outlines of India. awarded Bharat Ratna, the highest civilian
(a) National Security Guard award of India. The awardees were:
Charles Metcalfe, appointed as acting
(b) Central Industrial Security Force
Governor-General of India in 1835. 1. Pranab Mukherjee →
(c) Railway Protection Force Former President of India.
(d) Central Reserve Police Force 144. Which country among the 2. Nanaji Deshmukh →
(c) The Ministry of Railways had following was not a part of the Social Activist from Maharashtra.
launched the Commando for Railway Third Coalition against Napoleon? 3. Bhupen Hazarika →
Security (CORAS) of Indian railways in (a) Russia (b) Prussia Assamese Composer, Musician
August, 2019. CORAS is a separate (c) Sweden (d) France and Lyricist.
Commando Unit of Railway Protection
Force and are raised keeping in mind the Ê (d) The third coalition was an alliance of 149. Aishwarya Pissay excels in which
Russia, Britain, Prussia, Austria and one of the following sports?
threat from disruptive forces. The first
Sweden against the Napoleon I of (a) Badminton (b) Boxing
deployment of the unit will be in the
France. Hence, France was a party to (c) Motorsports (d) Chess
naxalism affected Chhattisgarh state.
war, but not a coalition member.
141. Which one of the following state- Ê (c) Aishwarya Pissay is an Indian circuit
145. Which one of the following pairs of and off road motor cycle racer. She
ments is not true about Ijarah system? travellers and their nationality is became the first ever Indian athlete to
(a) It was a system of revenue farming. won a world title in motorsports.
not correctly matched?
(b) It encouraged the role of middlemen. (a) Ibn Battuta-Moroccan 150. Which one of the following films
(c) Under this system, land revenue was (b) Duarte Barbosa-Portuguese was adjudged as the Best Feature
fixed for the peasant. (c) Jean Baptiste Tavernier-Dutch Film in the 66th National Film Awards?
(d) The Ijaradar paid a fixed amount to the (d) Niccolao Manucci-Venetian (a) Bulbul can Sing (b) Hellaro
state treasury.
Ê (c) Option (c) is not correctly matched (c) Badhaai Ho (d) Uri
Ê (c) The Ijarah system was developed because Jean Baptiste Tavernier was a
during sultanate period and gained Ê (b) The movie ‘Hellaro’ won the National
French gem merchant and traveller. He Film ward for the Best Feature Film in the
prominence in Mughals after Akbar. It
made 6 voyages to Persia and India in 66th National Film Awards. ‘Hellaro’ is a
was a system of revenue farming. In this
his lifetime from 1630 to 1668 and has Gujarati language period drama,
system, the revenue was collected by a
left valuable accounts of Mughal empire, co-written and directed by Abhishek
middlemen called Ijaradars who had to
Nizams of Hyderabad, etc. Shah.
pay a certain sum to ruler and keep rest
NDA /NA
National Defence Academy/Naval Academy

SOLVED PAPER 2019 (I)


PAPER I : Mathematics
1. What is the nth term of the sequence ∴R is symmetric. 4. If A = { x ∈ Z : x 3 − 1 = 0} and
25, − 125, 625, − 3125, …? Hence, R is neither reflexive nor
B = { x ∈ Z : x + x + 1 = 0}, where, Z
2
(a) ( −5)2n−1 (b) ( −1)2n 5 n +1 transitive but symmetric.
is set of complex numbers, then
(c) ( −1)2n−1 5 n +1 (d) ( −1)n−1 5 n +1 3. A relation R is defined on the set N of what is A ∩ B equal to?
Ê (d) Given, sequence 25, natural numbers as (a) Null set
−125, 625, − 3125 xRy ⇒ x 2 − 4 xy + 3y 2 = 0. Then,  −1 + 3i −1 − 3i 
T T (b)  , 
Here, 2 = 3 = ........... which one of the following is
T1 T2  2 2 
correct?
 −1 + 3i −1 − 3i 
So, this sequence in GP whose common (a) R is reflexive and symmetric, but not (c)  , 
ratio is −5. transitive  4 4 
then a = 25, r = −5 (b) R is reflexive and transitive, but not  1 + 3i 1 − 3i 
(d)  ,
∴n th term of sequence = ar n−1
symmetric
 2 2 
= 25( − 5)n − 1 (c) R is reflexive, symmetric and transitive
Ê (b) We have, A = { x ∈ Z : x − 1 = 0}
3
(d) R is reflexive, but neither symmetric
= ( − 1)n − 1 5 2 × 5 n − 1 = ( − 1)n − 1 5 n + 1 nor transitive
and B = { x ∈ Z : x 2 + x + 1 = 0}
Ê (d) Given, xRy ⇒ x − 4 xy + 3 y = 0
2 2
2. Suppose X = {1, 2, 3, 4 } and R is a  − 1 + 3i − 1 − 3i 
For reflexive A = 1, , 
relation on X. If  2 2 
R = {(1, 1), (2, 2), (3, 3), (1, 2), (2, 1), xRx ⇒ x 2 − 4 x 2 + 3 x 2 = 0
 − 1 + 3i − 1 − 3i 
So, ( x, x ) ∈ R, ∀ x ∈ N B= , 
(2,3), (3, 2)}, then which one of the 2 2
 
following is correct? Hence, R is reflexive.
− 1 + 3i − 1 − 3i 
(a) R is reflexive and symmetric, but not For symmetric A∩B=  , 
 2 2 
transitive xRy ⇒ x 2 − 4 xy + 3 y 2 = 0
(b) R is symmetric and transitive, but not
∴ yRx ⇒ y 2 − 4 xy + 3 x 2 5. Consider the following statements
reflexive
(c) R is reflexive and transitive, but not for the two non-empty sets A and B.
It is not clear, that y − 4 xy + 3 x is equal
2 2
symmetric 1. ( A ∩ B ) ∪ ( A ∩ B ) ∪ ( A ∩ B )
to zero or not.
(d) R is neither reflexive nor transitive, but
i.e. ( x, y)∈ R but ( y, x ) ∉R ⋅ ∀ x, y ∈ N
=A∪B
symmetric 2. ( A ∪ ( A ∩ B )) = A ∪ B
Hence, R is not symmetric.
Ê (d) We have, X = {1, 2, 3, 4} For transitive
Which of the above statements
R = { (1,1), (2, 2), (3, 3),
is/are correct?
xRy ⇒ x 2 − 4xy + 3 y 2 = 0 (a) Only 1 (b) Only 2
(1, 2), (2, 1), (2, 3), (3, 2)}
yRz ⇒ y 2 − 4 yz + 3 z2 = 0 (let) (c) Both 1 and 2 (d) Neither 1 nor 2
Since, ( 4, 4) ∉ R,
xRz ⇒ x 2 − 4xz + 3 z2 Ê (a) We have,
Hence, R is not reflexive.
It is not clear, that x 2 − 4xz + 3 z2 is 1. ( A ∩ B) ∪ ( A ∩ B ) ∪ ( A ∩ B) = A ∪ B
Since, (1, 2 ) ∈ R, (2, 3) ∈ R but
equal to zero or not. LHS ≡ ( A ∩ B) ∪ ( A ∩ B ) ∪ ( A ∩ B)
(1, 3) ∉ R, R is not transitive. So, ( x , y) ∈ R ,( y, z)∈ R = { A ∩ ( B ∪ B )} ∪ ( A ∩ B)
(1, 2), (2, 3) ∈ R ⇒( x , z) ∉ R ∀ x , y, z ∈ N [by distributive property]
and also (2, 1), (3, 2) ∈ R Hence, R is not transitive.
2 NDA/NA Solved Paper 2019 (I)

= ( A ∩ U ) ∪ ( A ∩ B) B31 = 0, B32 = 0, B33 = 8 10. x − 3i


1
[Q B ∪ B = U ]
= A ∪ ( A ∩ B)
 B11 B12 B13  ′ If y i  = 6 + 11i , then what
1
adj B =  B21 B22 B23   
= ( A ∪ A ) ∩ ( A ∪ B)    0 2i −i 
= U ∩ ( A ∪ B)= A ∪ B = RHS  B31 B32 B33 
are the values of x and y
Hence, 1 is correct.  0 0 − 2 ′  0 0 0 respectively?
2. A ∪ ( A ∩ B) = A ∪ B =  0 0 −1 =  0 0 0 (a) −3, 4 (b) 3, 4
   
LHS ≡ A ∪ ( A ∩ B )  0 0 8   − 2 − 1 8  (c) 3, − 4 (d) −3, −4
= ( A ∪ A) ∩ ( A ∪ B) x − 3i 1
= U ∩ ( A ∪ B) 8. i = 6 + 11 i
What are the roots of the equation Ê (a) We have, y 1
= A∪B ≠ A ∪ B
Hence, 2 is false.
| x 2 − x − 6| = x + 2? 0 2i −i
∴Only 1 is correct. (a) −2, 1, 4 (b) 0, 2, 4 ⇒ x ( − i + 2 ) − y ( − 3 − 2 i ) = 6 + 11 i
(c) 0, 1, 4 (d) −2, 2, 4
6. ⇒ 2 x + 3 y + ( − x + 2 y) i = 6 + 11 i
Let X be a non-empty set and let
Ê (d) We have, On equating real and imaginary parts, on
A , B, C be subsets of X . Consider the | x 2 − x − 6| = x + 2 both sides,
following statements. we get 2 x + 3 y = 6
⇒ | ( x − 3) ( x − 2 )| = x + 2 ...(i)
1. A ⊂ C ⇒ ( A ∩ B ) ⊂ (C ∩ B ), Case I x < 2 and − x + 2 y = 11 ...(ii)
( A ∪ B ) ⊂ (C ∪ B ) x2 − x − 6 = x + 2 On solving Eqs. (i) and (ii), we get
2. ( A ∩ B ) ⊂ (C ∩ B ) for all sets x2 − 2 x − 8 = 0
x = − 3 and y = 4
B ⇒A ⊂C 11. The common roots of the equations
 x2 − 4 x + 2 x − 8 = 0 
3. ( A ∪ B ) ⊂ (C ∪ B ) for all sets  
B ⇒A ⊂C  x ( x − 4) + 2 ( x − 4) = 0 z 3 + 2z 2 + 2z + 1 = 0
Which of the above statements are ( x − 4) ( x + 2 ) = 0 and z 2017 + z 2018 + 1 = 0 are
correct? x = − 2 but x ≠ 4 [Q x < 2] (a) −1, ω (b) 1,ω 2
(a) Only 1 and 2 (b) Only 2 and 3 (c) −1, ω 2 (d) ω, ω 2
Case II 2 ≤ x < 3
(c) Only 1 and 3 (d) 1, 2 and 3
x2 − x − 6 = − ( x + 2 ) Ê (d) We have, z + 2 z + 2 z + 1 = 0
3 2

Ê (d) Let X = {1,2, 3, 4}


x − x−6+ x+2 =0
2
( z + 1) ( z2 + z + 1) = 0
A = {1, 2}, B = {2, 3, 4}, C = {1, 2, 3}
x2 − 4 = 0 ⇒ z + 1 = 0 or z2 + z + 1 = 0
A⊂C
A ∩ B = {2}, C ∩ B = {2, 3} x=±2 z = −1
Clearly, ( A ∩ B) ⊂ (C ∩ B) x = 2 but x ≠ − 2 [Q x ∈ (2, 3)] −1 ± 1 − 4
or z=
A ∪ B = {1, 2, 3, 4}, (C ∪ B) = {1, 2, 3, 4} Case III x ≥ 3 2
( A ∪ B) ⊂ (C ∩ B) x2 − x − 6 = x + 2 −1 + i 3 −1 − i 3
= , = ω, ω 2
Hence, Statement 1 is correct. x2 − 2 x − 8 = 0 2 2
2.( A ∩ B) ⊂ (C ∩ B) for all sets B ⇒ A ⊂ C Now, z2017 + z2018 + 1 = 0
( x + 2 ) ( x − 4) = 0
Hence, Statement 2 is also correct.
x = 4 but x ≠ − 2 [Q x ≥ 3] Put z = − 1,
3. ( A ∪ B) ⊂ (C ∪ B) for all sets LHS = ( −1)2017 + ( −1)2018 + 1
∴ x = − 2, 2, 4
B⇒A⊂ C
= − 1+ 1+ 1
Hence, Statement 3 is also correct. 9. 0 1
If A =  , then the matrix A is = 1 ≠ 0 (RHS)
7. 3 2 0 1 0 ∴ z = − 1 is not a root of equation.
If B = 2 4 0, then what is adjoint a/an Put z = ω,
  (a) singular matrix
1 1 0 LHS = (ω )2017 + (ω )2018 + 1
(b) involutory matrix
of B equal to? (c) nilpotent matrix = (ω 3 )672.ω + (ω 3 )672.ω 2 + 1
 0 0 0  0 0 −2  (d) idempotent matrix = ω + ω2 + 1 [Q ω 3 = 1]
(a)  0 0 0 (b)  0 0 −1  0 1 [Q1 + ω + ω 2 = 0]
    Ê (b) We have, A =  1 0 
 −2 −1 8  0 0 8    = 0 = RHS
| A| = − 1 ∴ z = ω is a root of equation.
0 0 2 
put z = ω 2,
(c)  0 0 1 (d) It does not exist Since,| A | ≠ 0
 
 0 0 0 Hence, A is not singular. LHS = (ω 2 )2017 + (ω 2 )2018 + 1
 3 2 0  0 1  0 1 = ω 4034 + ω 4036 + 1
A2 = A ⋅ A =  ⋅ 
Ê (a) We have, B =  2 4 0   1 0  1 0 = (ω 3 )1344 .ω 2 + (ω 3 )1345.ω + 1
 1 1 0   1 0 
=  = ω 2 + ω + 1 = 0 RHS
Co-factor of B,  0 1 ∴ z = ω 2 is a root of equation.
B11 = 0, B12 = 0, B13 = − 2 A =I
2
Hence, ω, ω 2 are the common roots of
B21 = 0, B22 = 0, B23 = − 1 Hence, A is involutory matrix.
these equations.
NDA/NA Solved Paper 2019 (I) 3

12. If C (20, n + 2) = C (20, n − 2), then Directions (Q. Nos. 16 and 17) Read 20. From 6 programmers and 4 typists,
the information carefully and answer an office wants to recruit 5 people.
what is n equal to
the given questions. What is the number of ways this can
(a) 8 (b) 10 In a school, all the students play atleast one
(c) 12 (d) 16 be done so as to recruit atleast one
of three indoor games– chess, carrom and
typist?
Ê (b) We have, C (20, n + 2 ) = C (20, n − 2 ) table tennis. 60 play chess, 50 play table
tennis, 48 play carrom, 12 play chess and (a) 209 (b) 210
⇒ 20
Cn + 2 = 20
Cn − 2 (c) 246 (d) 242
carrom, 15 play carrom and table tennis, 20
⇒ n + 2 + n − 2 = 20 play table tennis and chess. Ê (c) We have,
[Q nC x = nC y ⇒ x + y = n] 6 programmers and 4 typists
16. What can be the minimum number of Number of ways of 5 recruit people such
∴ n = 10
students in the school? that atleast one typist
13. There are 10 points in a plane. No (a) 123 (b) 111 (c) 95 (d) 63 = 4C1 6C 4 + 4C 2 6C 3 + 4C 3 6C 2
three of these points are in a straight Ê (b) Let + 4C 4 6C1
line. What is the total number of A = Student play chess = 4 × 15 + 6 × 20 + 4 × 15 + 1 × 6
straight lines which can be formed B = Student play table tennis = 60 + 120 + 60 + 6 = 246
by joining the points? C = Student play carrom
(a) 90 (b) 45 Given, n( A ) = 60, n( B) = 50, n(C ) = 48
21. What is the number of terms in the
(c) 40 (d) 30 n( A ∩ B) = 20, n( B ∩ C ) = 15 expansion of [(2x − 3y )2 (2x + 3y )2 ]2 ?
Ê (b) Given, 10 points in a plane where no n( A ∩ C ) = 12 (a) 4 (b) 5
three of these points are in straight line. (c) 8 (d) 16
For minimum number of students in
Total number of straight line formed from
Ê (b) Given, [(2 x − 3 y) (2 x + 3 y) ]
school 2 2 2
10 points is
n ( A ∩ B ∩ C ) must be zero. = [4x 2 − 9 y 2 ]4
10 ! 10 × 9
10
C2 = = = 45 ∴n( A ∪ B ∪ C ) = n( A ) + n( B) + n(C ) ∴ Total number of terms = 4 + 1 = 5
2 ! 8! 2
− n( A ∩ B) − n( B ∩ C )
14. The equation px 2 + qx + r = 0 − n( A ∩ C ) + n( A ∩ B ∩ C )
22. In the expansion of (1 +ax )n , the first
(where p , q , r , all are positive) has = 60 + 50 + 48 − 20 − 15 − 12 + 0 = 111 three terms are respectively 1, 12x
distinct real roots a and b. Which one and 64 x 2 . What is n equal to?
of the following is correct? 17. What can be the maximum number of
(a) 6 (b) 9
(a) a > 0, b > 0 students in the school? (c) 10 (d) 12
(b) a < 0, b < 0 (a) 111 (b) 123
(c) a > 0, b < 0 (c) 125 (d) 135 Ê (b) Given, first three terms of expansion
(1 + ax ) is 1, 12 x, 64 x ,
n 2
(d) a < 0, b > 0 Ê (b) For maximum number of students in Now,
Ê (b) Given, px + qx + r = 0,
2
where school n( A ∩ B ∩ C ) must be 12. n( n − 1) 2 2
∴ n( A ∪ B ∪ C ) (1 + ax )n = 1 + nax + a x +K
p, q , r > 0 and a and b are distinct roots. 2
−q = 60 + 50 + 48 − 20 − 15 − 12 + 12 On equating first three terms, we get
∴ a+ b= and ab = r
p = 123 n ( n − 1) 2
na = 12 and a = 64
Now, r> 0 18. If A is an identity matrix of order 3, 2
On putting the value of a in
∴ ab > 0 then its inverse ( A −1 ) n ( n − 1) 2
⇒ a > 0, b > 0 ...(i) a = 64, we get
(a) is equal to null matrix 2
or a < 0, b < 0 ...(ii) (b) is equal to A n ( n − 1)  12 
2
−q
< 0 q, p > 0
(c) is equal to 3 A (d) does not exist   = 64
Now, 2  n
p  1 0 0
144 ( n − 1)
∴ a+ b< 0 Ê (b) Given, A =  0 1 0  ⇒
2n
= 64
a < 0, b < 0 ...(iii)  0 0 1  ∴ n=9
From Eqs. (i), (ii) and (iii), we get  1 0 0
∴ a < 0 and b < 0 ∴ A −1 =  0 1 0  = A
23. The numbers 1, 5 and 25 can be three
  terms (not necessarily
15. If A = { λ , { λ ,µ }}, then the power set  0 0 1 
consecutive)of
of A is 19. A is a square matrix of order 3 such (a) only one AP
(a) { φ, { φ}, { λ}, { λ,µ }} that its determinant is 4. What is the (b) more than one but finite numbers of
(b) {φ, { λ},{ λ,µ },{ λ,{ λ,µ }}} APs
determinant of its transpose? (c) infinite number of APs
(c) { φ, { λ},{ λ,µ },{ λ,{ λ,µ }}} (a) 64 (b) 36 (d) finite number of GPs
(d) {{ λ},{ λ,µ },{ λ,{ λ ,µ }}} (c) 32 (d) 4
Ê (d) We have, 1, 5, 25 be three terms.
Ê (b) We have, A = { λ, { λ, µ }} Ê (d) Given, | A | = 4 Clearly, 1, 5, 25 are finite number of GPs.
P( A ) = { φ, { λ}, {{ λ, µ }}, { λ, { λ, µ }}} ∴ | A′ | = 4 [Q | A | = | A ′ |]
4 NDA/NA Solved Paper 2019 (I)

24. The sum of ( p + q )th and ( p − q )th 26. What is the least value of Directions (Q. Nos. 29 and 30) Read
terms of an AP is equal to 25 cosec2 x + 36sec2 x ? the information carefully and answer
the given questions.
(a) (2 p) th term (b) (2q ) th term (a) 1 A complex number is given by
(c) twice the p th term (b) 11
(d) twice the q th term 1 + 2i
(c) 120 z= .
Ê (c) Let a is first term and d is common (d) 121 1 − (1 − i )2
difference of AP.
Ê (d) Given, 25 cosec x + 36 sec x
2 2

ap + q = a + ( p + q − 1) d
29. What is the modulus of z?
= 25 (1 + cot 2 x ) + 36 (1 + tan2 x ) 1
and ap − q = a + ( p − q − 1) d (a) 4 (b) 2 (c) 1 (d)
= 25 + 25 cot x + 36 + 36 tan x
2 2 2
Sum of ( p + q ) th and ( p − q ) th terms 1 + 2i
= ap + q + ap − q = 2 a + (2 p − 2 ) d = 25 + 36 + 25 cot 2 x + 36 tan2 x Ê (c) We have, z =
1 − (1 − i )2
= 2 ( a + ( p − 1) d ) = 2 ap 1 + 2i
= 61 + ( 5 cot x − 6 tan x )2 + 2 × 5 × 6 z=
= twice of p th term 1 − (1 − 1 − 2 i )
≥ 61 + 60 = 121 [Qminimum value of
25. If A is a square matrix of order n > 1, 1 + 2i
( 5 cot x − 6 tan x )2 = 0] = =1
then which one of the following is 1 + 2i
∴ Minimum value of
correct? ∴ | z| = 1
25 cosec 2 x + 36 sec 2 x = 121
(a) det ( − A ) = det A 30. What is the principal argument of z?
(b) det ( − A ) = ( −1)n det A
(c) det ( − A ) = − det A Directions (Q. Nos. 27 and 28) Read π π
(a) 0 (b) (c) (d) π
(d) det ( − A ) = n det A the information carefully and answer 4 2
the given questions. −1  lm( z) 
Ê Sol. (b) We know that if A is a square Ê (a) arg ( z) = tan  
matrix of order n > 1, then Let A and B be 3 × 3 matrices with det A = 4  Re ( z)
det( − A ) = ( −1)n det A and det B = 3 .  0
= tan−1   = tan−1 0 = 0
2 3 27. What is det (2AB ) equal to?  1
For example If A =  ,
 4 5 (a) 96
31. What is the value of
 −2 −3 (b) 72
then − A =   (c) 48 sin 34 ° cos 236° − sin 56° sin 124 °
 −4 −5 ?
(d) 36 cos 28° cos 88° + cos 178° sin 208°
2 3
∴det A =  = 10 − 12 = − 2 …(i) Ê (a) A and B be ( 3 × 3) matrices with (a) −2 (b) −1 (c) 2 (d) 1
4 5 det A = 4 and det B = 3
−2 −3 Ê (a) We have,
and det( − A ) =   = 10 − 12 = − 2 We know that, sin 34° cos 236° − sin 56° sin 124°
−4 −5 det ( KAB) = K ndet( A ) × det( B) cos 28° cos 88° + cos 178° sin 208°
= ( −1)2( −2 ) [Q here n = 2] where, n is the order or A and B, K is a real sin 34° cos (180° + 56° )
= ( −1)2 det A [from Eq. (i)] number. − sin 56° sin ( 90° + 34° )
=
∴det(2 AB) = (2 )3detA × detB cos 28° cos 88° + cos ( 90° + 88° )
1 2 3 
[Q n = 3 and k = 2] sin (180° + 28° )
if A =  3 1 0 
  =8×4×3 − sin 34° cos 56° − sin 56° cos 34°
 4 3 −2  =
= 96 cos 28° cos 88° + sin 88° sin 28°
 −1 −2 −3
28. What is det (3AB −1 ) equal to? − sin ( 56° + 34° ) − sin 90°
Then, − A =  −3 −1 0  = =
  cos ( 88° − 28° ) cos 60°
(a) 12 (b) 18
 −4 −3 2  −1
(c) 36 (d) 48 = = −2
1 2 3 1
∴ det A = 3 1 0
Ê (c) A and B be ( 3 × 3) matrices with
2
det A = 4 and det B = 3
4 3 −2
We know that, 32. tan54° can be expressed as
= 1( −2 − 0) − 2( −6 − 0) + 3( 9 − 4) 1 sin 9° + cos 9° sin 9° − cos 9°
det( KAB−1 ) = K ndet( A ) × , (a) (b)
= − 2 + 12 + 15 = 25 det( B) sin 9° − cos 9° sin 9° + cos 9°
 −1 −2 −3 cos 9° + sin 9° sin 36°
where n is the order of A and B, K is a (c) (d)
and det ( − A ) =  −3 −1 0 cos 9° − sin 9° cos 36°
  real number]
 −4 −3 2
∴det ( 3 AB−1 ) = ( 3)3det( A ) ×
1 Ê (c) We have, tan 54° = tan ( 45° + 9° )
= − 1( −2 − 0) + 2( −6 − 0) − 3( 9 − 4) det B tan 45° + tan 9° 1 + tan 9°
1 = =
= 2 − 12 − 15 = − 25 = 27 × 4 × 1 − tan 45° tan 9° 1 − tan 9°
= ( −1)325 [here n = 3] 3
cos 9° + sin 9°
= 36 =
= ( −1)3det A [from Eq. (i)] cos 9° − sin 9°
NDA/NA Solved Paper 2019 (I) 5

Directions (Q. Nos. 33-35) Read the Directions (Q. Nos. 36 and 37) Read 39. What is the value of
given information carefully and answer the given information carefully and −1 4 −1 5 π
the given questions. answer the given questions. sin + sec − ?
5 4 2
If p = X cosθ − Y sin θ, It is given that cos(θ − α) = a,
π π
cos(θ − β ) = b. (a) (b)
q = X sin θ + Y cos θ and 4 2
p2 + 4 pq + q2 = AX 2 + BY 2 , 36. What is cos(α − β ) equal to ? (c) π (d) 0
π Ê (d) We have,
0 ≤θ ≤ . (a) ab + 1 − a2 1 − b 2
2 4 5 π
sin−1 + sec −1 −
5 4 2
33. What is the value of θ? (b) ab − 1 − a 2
1− b 2
4 4 π
π π = sin−1 + cos −1 −
(a) (b) 5 5 2
2 3 (c) a 1 − a2 − b 1 − a2
  1 
π π Q sec −1 x = cos −1  
(c) (d)   x  

4 6 (d) a 1 − b 2 + b 1 − a2
π π  π
Ê (c) We have, = − = 0 Q sin−1 x + cos −1 x =
 2 
p = X cos θ − Y sin θ ...(i)
Ê (a) Given cos (θ − α ) = a 2 2
cos (θ − β ) = b
q = X sin θ + Y cos θ ...(ii) 40. 2p 1 −q 2
cos (α − β ) = cos {(θ − β ) − (θ − α )} If sin −1 − cos −1
and p2 + 4 pq + q 2 = AX 2 + BY 2 ...(iii) 1+ p2 1+q2
= cos (θ − β ) cos (θ − α ) + sin (θ − β )
From Eqs. (i) and (ii), we get
sin (θ − α ) −1 2x
p2 + q 2 = ( X cos θ − Y sin θ)2 = tan , then what is x equal
= ab + 1 − a 2 1 − b 2 1−x2
+ ( X sin θ + Y cos θ) 2
to?
⇒ p2 + q 2 = X 2 + Y 2 37. What is sin 2 (α − β ) + 2ab cos(α − β) p+ q p−q
(a) (b)
and pq = ( X cos θ − Y sin θ) equal to? 1 + pq 1 + pq
( X sin θ + Y cos θ) (a) a2 + b 2 (b) a2 − b 2
⇒ pq = ( X 2 − Y 2 )sin θ cos θ (c) b 2 − a2 (d) −(a2 + b 2 ) pq p+ q
(c) (d)
+ XY cos 2θ 1 + pq 1 − pq
Ê (a) sin (α − β ) + 2 ab cos (α − β )
2
∴ p2 + 4 pq + q 2 = X 2 + Y 2 + 2
= 1 − cos 2 (α − β ) + 2 ab cos (α − β ) Ê (b) Given,
( X 2 − Y 2) 2p 1− q2
sin−1 − cos −1  
2
sin 2θ + 4 XY cos 2θ = 1 − ( ab + 1 − a2 1 − b 2 )2 + 2 ab 1+ p 2
1 + q 
Given, p2 + 4 pq + q 2 = AX 2 + BY 2 ( ab + 1 − a2 1 − b2 ) = tan−1
2x
∴ X + Y + 2 (X − Y )
2 2 2 2 1 − x2
= 1 − [a 2b 2 + (1 − a 2 ) (1 − b 2 ) + 2 ab
⇒ 2 tan−1 p − 2 tan−1 q = 2 tan−1 x
sin 2 θ + 4 XY cos 2θ = AX + 2 BY 2 2
1 − a2 1 − b 2 ] + 2 a 2b 2 + 2 ab  p−q 
Coefficient of XY = 0 ⇒ tan−1  −1
 = tan x
 1 + pq 
∴ cos 2θ = 0 ( 1 − a 2 ) ( (1 − b 2 )
π p−q
⇒ 2θ = = 1 − a 2b 2 − 1 + a 2 + b 2 − a 2b 2 ∴ x=
2 1 + pq
π − 2 ab 1 − a 2 1 − b 2 + 2 a 2b 2 + 2 ab 1 1
∴ θ= 41. If tanθ = and tan φ = , then what
4 1− a 2
1− b 2
2 3
34. What is the value of A? is the value of (θ + φ )?
= a2 + b 2
(a) 4 π
(a) 0 (b)
(b) 3 38. If sin α + cos α = p, then what is 6
π π
(c) 2 cos 2 (2α ) equal to? (c) (d)
(d) 1 4 2
π (a) p2 (b) p2 − 1 1 1
Ê (b) X + Y + 2 ( X − Y ) sin
2 2 2 2
(c) p2 (2 − p2 ) (d) p2 + 1 Ê (c) Given, tan θ = , tan φ =
2 2 3
tan θ + tan φ
= AX 2 + BY 2 Ê (c) We have, sin α + cos α = p tan (θ + φ) =
1 − tan θ tan φ
⇒ X 2 + Y 2 + 2 X 2 − 2 Y 2 = AX 2 + BY 2 sin2 α + cos 2 α + 2 sin α cos α = p2 1 1
+
⇒ 3 X − Y = AX + BY
2 2 2 2
⇒ 1 + sin 2 α = p2 ⇒ tan (θ + φ) = 2 3
1 1
∴ A = 3, B = − 1 ⇒ sin2α = p2 − 1 1− ×
2 3
35. What is the value of B ? ⇒ sin2 2α = ( p2 − 1)2 3+2 5
⇒ tan (θ + φ) = = =1
(a) −1 (b) 0 ⇒ 1 − cos 2 2α = p4 − 2 p2 + 1 6−1 5
(c) 1 (d) 2 ⇒ tan (θ + φ) = 1
⇒ cos 2 2α = 2 p2 − p4
Ê (a) B = − 1 π
∴ cos 2 α = p (2 − p )
2 2 2 ⇒ θ + φ = tan−1 1 =
4
6 NDA/NA Solved Paper 2019 (I)

3
42. If cos A = , then what is the value Ê (b) Given, sin 2θ = cos 3θ 49. If p = cosecθ − cot θ and
4 q = (cosecθ + cot θ )−1, then which
⇒ 2 sin θ cos θ = 4 cos 3 θ − 3 cos θ
 A   3A 
of sin   sin   ? ⇒ 2 sin θ = 4 cos 2 θ − 3 one of the following is correct?
 2  2 
[Qcosθ ≠ 0] (a) pq =1 (b) p = q
5 5 5 7 ⇒ 2 sin θ = 4(1 − sin2 θ) − 3 (c) p + q = 1 (d) p + q = 0
(a) (b) (c) (d)
8 16 24 32
⇒ 4 sin2 θ + 2 sinθ − 1 = 0 Ê (b) Given, p = cosec θ − cot θ
3
Ê (b) Given, cos A = −2 ± 4 + 16 q = (cosec θ + cot θ)−1
4 ⇒ sinθ = ⇒
A 3A 2 ×4
Now, sin sin  1   cosec θ − cot θ 
2 2 − 2 ± 2 5 − 1± 5 q =  
⇒ sin θ = =  cosec θ + cot θ   cosec θ − cot θ 
1 2 A 3A 2 ×4 4
=  sin sin  ⇒ q = cosec θ − cot θ
2  2 2  5 −1   π 
∴ sin θ = Q θ ∈ 0, ∴ q = p
 3A    2  

cos  −
1 A 3A A 4
=  − cos  + 
    2   50. If the angles of a triangle ABC are in
2  2 2 2
46. If the roots of the equation the ratio 1 : 2 : 3, then the
1
= [cos A − cos 2 A ] x 2 + px + q = 0 are tan19° and
2 corresponding sides are in the ratio
1 tan26°, then which one of the (a) 1 : 2 : 3 (b) 3 : 2 : 1
= (cos A − 2 cos 2 A + 1)
2 following is correct ? (c) 1 : 3 : 2 (d) 1 : 3 : 2
1 3 9  (a) q − p = 1 (b) p − q = 1 Ê (c) We have, angle of triangle ABC are in
= −2 × +1
2  4 16  (c) p + q = 2 (d) p + q = 3 the ratio 1 : 2 : 3
1 3 9  1  6 − 9 + 8 5 Ê (a) Given, tan 19° and tan 26° are roots of ∴ x + 2 x + 3 x = 180°
= − +1 = =
2  4 8  2  8  16 x 2 + px + q = 0. ⇒ x = 30°
∴tan 19° + tan 26° = − p ∴ Angles of triangle are 30° , 60° , 90°.
43. What is the value of tan 75° + cot 75° ? We know that, sine rule
tan 19° ⋅ tan 26° = q
(a) 2 (b) 4 tan 19° + tan 26° a b c
tan (19° + 26° ) = = =
(c) 2 3 (d) 4 3 1 − tan 19° tan 26° sin A sin B sinC
a b c
Ê (b) We have, tan 75° + cot 75° −p −p = =
⇒ tan 45° = ⇒1 = sin 30° sin 60° sin 90°
= tan ( 90° − 15° ) + cot ( 90° − 15° ) 1− q 1− q
a b c
= cot 15° + tan 15° ⇒ = =
⇒ 1− q = − p 1 3 1
cos 15° sin 15°
= + ∴ q − p=1 2 2
sin 15° cos 15°
cos 2 15° + sin2 15° 47. What is the fourth term of an AP of n ∴ a : b : c = 1: 3 : 2
=
sin15° cos 15° terms whose sum is n (n + 1)? 51. Consider the following statements
1 (a) 6 (b) 8 1. For an equation of a line,
= (c) 12 (d) 20
sin 15° cos 15° x cos θ + y sin θ = p , in normal
2 Ê (b) Given, form, the length of the
=
2 sin 15° cos 15° Sum of n terms of an AP perpendicular from the point
2 2 i.e. S n = n ( n + 1) ( α, β ) to the line is
= = =4
sin 30° 1 a4 = S 4 − S 3 [Q an = S n − S n−1] | α cos θ + β sin θ + p |.
2 a4 = 4( 4 + 1) − 3 ( 3 + 1)
The length of the perpendicular
a4 = 20 − 12 = 8 from the point (α,β ) to the line
44. What is the value of
∴ Fourth term of an AP = 8 aα + bβ − ab
cos 46° cos 47 ° cos 48° cos 49 ° x y
cos 50° .....cos 135°? 48. What is − sec 2 α sec 2 β equal to? + = 1 is .
 a +b 
a b 2 2
(a) −1 (b) 0
(a) 0 (b) 1
(c) 1 (d) Greater than 1
(c) 2 (d) 4
Which of the above statements
Ê (b) We have, is/are correct?
Ê (a) We have, (a) 1 only (b) 2 only
cos 46° cos 47 ° cos 48° (1 + tan α tan β )2 + (tanα − tanβ )2
cos 49° cos 50° … cos 90° … cos 135° (c) Both 1 and 2 (d) Neither 1 nor 2
− sec 2α sec 2β
=0 [Qcos 90° = 0] Ê (d) 1. Equation of line
π = 1 + tan2 α tan2 β + 2 tanα tanβ x cos θ + y sin θ = p
45. If sin 2θ = cos 3θ, where 0 < θ < , + tan2 α + tan2 β Perpendicular distance from (α,β ) to the
2
then what is sinθ equal to ? − 2 tanα tanβ − sec 2α sec 2β given line is
= 1 + tan2 α tan2 β + tan2 α + tan2 β α cos θ + β sin θ − p
5 +1 5 −1
(a) (b) − sec 2α sec 2 β
4 4 cos 2 θ + sin2 θ
5 +1 5 −1 = (1 + tan α ) (1 + tan β ) − sec α sec β
2 2 2 2
= (α cos θ + β sinθ − p)
(c) (d)
16 16 = sec 2αsec 2β − sec 2αsec 2β = 0 Hence, statement 1 is incorrect.
NDA/NA Solved Paper 2019 (I) 7

2. Length of the perpendicular from the 54. The equation 2x 2 − 3y 2 − 6 = 0 Ê (c) The lines 3 y + 4 x = 1, y = x + 5 and
x y
point (α,β ) to the line + = 1 is 5 y + bx = 3 are concurrent.
a b represents
 3 4 − 1
α β  (a) a circle
 1 − 1 − 5 = 0
 + − 1  ∴
a b αb + aβ − ab (b) a parabola  
 = 
 
 (c) an ellipse  5 b − 3
 1
+
1   a2 + b 2 
2 (d) a hyperbola ⇒ 3 ( 3 + 5b ) − 4( − 3 + 25) −1( b + 5) = 0
 a b 2
Ê (d) Given, 2 x − 3 y − 6 = 0
2 2
Hence, statement 2 is incorrect. ⇒ 9 + 15b + 12 − 100 − b − 5 = 0
∴ Neither 1 nor 2. ⇒ 2 x 2 − 3 y2 = 6 ⇒ 14b = 84
∴ b=6
52. A circle is drawn on the chord of a x2 y2
⇒ − =1
circle x + y = a as diameter. The
2 2 2 3 2 58. What is the equation of the straight
Which represents the equation of a line which is perpendicular to y = x
chord lies on the line x + y = a. hyperbola. and passes through (3, 2)?
What is the equation of the circle?
55. The two parabolas y 2 = 4ax and (a) x − y=5
(a) x 2 + y2 − ax − ay + a2 = 0 (b) x + y=5
(b) x 2 + y2 − ax − ay = 0 x = 4ay intersect
2
(c) x + y=1
(c) x 2 + y2 + ax + ay = 0 (a) at two points on the line y = x (d) x − y=1
(d) x 2 + y2 + ax + ay − 2 a2 = 0 (b) only at the origin Ê (b) Equation of line perpendicular to y = x
(c) at three points one of which lies on is x + y = λ.
Ê (b)2 Given, equation of circle is y+ x =0
x + y 2 = a 2. Since, this line is passes through (3, 2)
(d) only at (4a, 4a)
x + y = a is chord of a circle. ∴ 3 + 2 = λ⇒λ = 5
Ê (a) Given, y = 4ax
2
Hence, equation of required line is
(0, a) and x 2 = 4ay x + y = 5.
The graph of given curve is clearly from 59. The straight lines x + y − 4 = 0,
graph the given curve is intersect at two
points on the line y = x 3x + y − 4 = 0 and x + 3y − 4 = 0
y=x form a triangle, which is
(a

(a) isosceles (b) right angled


,0
)

(c) equilateral (d) scalene


Ê (a) Given, equation of line
x+ y=4 ...(i)
∴End points of diameter of required circle 3x + y = 4 ...(ii)
is (a,0) and ( 0, a ). x + 3y = 4 ...(iii)
∴Equation of circle is A (0, 4)
x ( x − a) + y ( y − a) = 0
⇒ x 2 + y 2 − ax − ay = 0 56. The points (1, 3) and (5, 1) are two
opposite vertices of a rectangle. The
4

53. The sum of the focal distances of a 3x+y=4


y=

other two vertices lie on the line


x+

point on an ellipse is constant and y = 2x + c . What is the value of c?


equal to (a) 2 (b) −2
(a) length of minor axis (c) 4 (d) −4
(b) length of major axis
(c) length of latusrectum Ê (d) The points (1, 3) and (5, 1) are two B (4, 0) x+3y=4 C (1, 1)
opposite vertex of rectangle. The other
(d) sum of the lengths of semi major On solving Eqs. (i) and (ii), we get
two vertices lie on the line y = 2 x + c.
and semi minor axes
∴ The mid point of vertices lie on the line x = 0, y = 4, A = ( 0, 4)
Ê (b) The sum of the focal distance of a  1 + 5 3 + 1 On solving Eqs. (i) and (iii), we get
point on a ellipse is constant and equal i.e.  ,  ≡ ( 3, 2 ) lie on the line
 2 2  x = 4, y = 0, B = ( 4, 0)
to the length of major axis.
y = 2x + c On solving Eqs. (ii) and (iii), we get
We know that, PS + PS ′ = 2 a
∴ 2 = 2( 3) + c x = 1, y = 1, C = (1, 1)
P ⇒ c=−4 Clearly, AC = BC
57. If the lines 3y + 4 x = 1, y = x + 5 and ∴Triangle is an isosceles.
5y + bx = 3 are concurrent, then 60. The circle x 2 + y 2 + 4 x − 7y + 12 = 0,
S S′ what is the value of b?
(a) 1 (b) 3
cuts an intercept on Y -axis equal to
1 (a) 1 (b) 3
(c) 6 (d)
2 (c) 4 (d) 7
8 NDA/NA Solved Paper 2019 (I)

Ê (a) Given, x + y + 4 x − 7 y + 12 = 0
2 2 ⇒ 1 + λ ( − 3) = 0 67. If the position vectors of points A
For intercept on Y-axis put x = 0, we get 1
⇒ λ= and B are 3$i − 2$j + k$ and 2$i + 4 $j − 3k$
y 2 − 7 y + 12 = 0 3
1 respectively, then what is the length
( y − 4) ( y − 3) = 0 On putting λ = in Eq. (i), we get
y = 3, y = 4 3 of AB ?
Length of intercept on Y-axis (2 x + y + 2 z − 9) (a) 14 (b) 29
1 (c) 43 (d) 53
= | y2 − y1 | = | 3 − 4 | = 1 + ( 4 x − 5 y − 4 z − 1) = 0
Ê (d) We have , OA = 3 i − 2 j + k
$ $ $
3
61. The centroid of the triangle with ⇒ 6 x + 3 y + 6 z − 27 + 4 x − 5 y − 4 z OB = 2 $i + 4$j − 3k$
vertices A(2, − 3, 3), B(5, − 3, − 4 ) and
− 1= 0 ∴| AB | = | OB − OA |
C(2, − 3, − 2) is the point
⇒ 10 x − 2 y + 2 z − 28 = 0
(a) (−3, 3, − 1) = | (2 $i + 4$j − 3k$ ) − ( 3$i − 2 $j + k$ )|
(b) (3, − 3, − 1) ∴ 10 x − 2 y + 2 z = 28
= | ( − $i + 6$j − 4k$ |
(c) (3, 1, − 3) 64. The distance between the parallel
(d) (−3, − 1, − 3) = ( − 1)2 + ( 6)2 + ( − 4)2
planes 4 x − 2y + 4z + 9 = 0 and
Ê (b) Given vertices of triangle ABC are 8x − 4y + 8z + 21 = 0 = 1 + 36 + 16 = 53
A(2, − 3, 3), B( 5, − 3, − 4) 1 1
(a) (b) 68. If in a right angled triangle ABC,
and C(2, − 3, − 2 ) 4 2
∴ Centroid of ∆ABC 3 7 hypotenuse AC = p , then what is
(c) (d)
2 + 5 + 2 − 3 − 3 − 3 3 − 4 − 2 2 4 AB ⋅ AC + BC ⋅ BA + CA ⋅ CB equal
= , , 
 3 3 3  to?
Ê (a) Given equation of planes
9 −9 −3  (a) p2 (b) 2 p2
=  , ,  = ( 3, − 3, − 1)
4x − 2 y + 4z + 9 = 0 ...(i)
p2
3 3 3  (c) (d) p
and 8 x − 4 y + 8 z + 21 = 0 2
62. What is the radius of the sphere 21
⇒ 4x − 2 y + 4z + = 0 ...(ii) Ê (a) In right angled ∆ABC, we have
2 ∠ABC = 90°
x 2 + y 2 + z 2 − 6x + 8y − 10z + 1 = 0?
Distance between parallel planes (i) C
(a) 5 (b) 2 and (ii)
(c) 7 (d) 3
 21 − 9 
 
Ê (c) Given, equation of sphere = 2  p
x 2 + y 2 + z2 − 6 x + 8 y − 10 z + 1 = 0 ( 4)2 + ( − 2 )2 + ( 4)2
On comparing with
3 3 θ
x 2 + y 2 + z2 + 2u x + 2 vy + 2 wz A B
2 1
+ d = 0, we get = = 2 = Let ∠BAC = θ
16 + 4 + 16 6 4
2 u = − 6, 2 v = 8, 2 w = − 10, d = 1 Then, ∠ACB = ( 90° − θ)
⇒ u = − 3, v = 4, w = − 5, d = 1 65. What are the direction cosines of ∴AB ⋅ AC + BC ⋅ BA + CA ⋅ CB
∴Radius of sphere = u 2 + v 2 + w 2 − d Z -axis? = | AB || AC | cos θ + | BC || BA |
(a) <1, 1, 1> (b) <1, 0, 0> cos 90°+ | CA || CB | cos ( 90° − θ)
= ( − 3)2 + ( 4)2 + ( − 5)2 − 1
(c) <0, 1, 0> (d) <0, 0, 1> = | AB || AB | + 0 + | CB || CB |
= 9 + 16 + 25 − 1 = 49 = 7
Ê (d) Direction cosines of Z-axis are = | AB |2 + | CB |2
63. The equation of the plane passing < cos 90° , cos 90° , cos 0°> < 0, 0,1 >
= | AC |2 = p2
through the intersection of the 66. If a = i$ − 2$j + 5k$ and b = 2$i + $j − 3k$ ,
planes 2x + y + 2z = 9,
69. The sine of the angle between
then what is ( b − a ) ⋅ (3a + b) equal vectors
4 x − 5y − 4z = 1 and the point
to ? a = 2 $i − 6$j − 3k$ and b = 4$i + 3$j − k$ is
(3, 2, 1) is
(a) 106 (b) −106 1 5
(a) 10x − 2 y + 2 z = 28 (c) 53 (d) −53 (a) (b)
(b) 10x + 2 y + 2 z = 28 26 26
Ê (b) We have, a = i − 2 j + 5k
$ $ $ 5 1
(c) 10x + 2 y − 2 z = 28 (c) (d)
(d) 10x − 2 y − 2 z = 24 b = 2 $i + $j − 3k$ 26 26

Ê (a) Equation of the plane passing through ∴b − a = (2 $i + $j − 3k$ ) − ( $i − 2 $j + 5k$ ) Ê (b) Let θ be the angle between vectors a
the intersection of plane and b
= $i + 3$j − 8k$ a ⋅b
2 x + y + 2 z = 9, 4 x − 5 y − 4 z = 1 is ∴ cos θ =
and 3 a + b = 3( $i − 2 $j + 5k$ ) | a || b |
(2 x + y + 2 z − 9)
+ (2 $i + $j − 3k$ ) Since,
+ λ ( 4 x − 5 y − 4 z − 1) = 0 ...(i)
= 5 i − 5 j + 12k$
$ $ a ⋅ b = (2 $i − 6$j − 3k$ ) ⋅ ( 4$i + 3$j − k$ )
Since, plane (i) passes through the point
∴(b − a ) ⋅ (3a + b ) = 8 − 18 + 3 = − 7
(3, 2, 1)
= ( $i + 3$j − 8k$ ) ⋅ ( 5$i − 5$j + 12k$ ) | a | = 2 2 + ( − 6)2 + ( − 3)2
∴ (2 × 3 + 2 + 2 × 1 − 9)
+ λ ( 4 × 3 − 5 × 2 − 4 × 1 − 1) = 0 = 5 − 15 − 96 = − 106 = 49 = 7
NDA/NA Solved Paper 2019 (I) 9

| b| = 4 2 + 3 2 + ( − 1)2 = 26 73. A function f defined by 76. For r > 0, f (r ) is the ratio of


−7 1 perimeter to area of a circle of radius
∴ cos θ = =− f ( x ) = ln( x + 1 − x ) is
2
7 × 26 26 r . Then, f (1) + f (2) is equal to
5 (a) an even function
∴ sin θ = 1 − cos 2 θ = (b) an odd function
(a) 1 (b) 2
26 (c) both even and odd function (c) 3 (d) 4
70. What is the value of λ for which the (d) neither even nor odd function Ê (c) We have,
Perimeter of a circle with radius r
vectors 3i$ + 4 $j − k$ and f( r )=
Ê (b) We have, f( x) = log ( x + 1 − x)
2 Area of a circle with radius r
−2i + λ j + 10k are perpendicular?
$ $ $
⇒ f( r ) =
2 πr 2
=
∴ f( − x ) = log ( x 2 + 1 + x ) πr 2 r
(a) 1 (b) 2
(c) 3 (d) 4  ( x 2 + 1 + x) ( x 2 + 1 − x) ∴
2 2
f(1) = = 2 ⇒ f(2 ) = = 1
= log  
1 2
Ê (d) Since, given vectors are  x2 + 1 − x 
  ∴ f(1) + f(2 ) = 2 + 1 = 3
perpendicular.
 x2 + 1 − x2 
∴ ( 3 $i + 4$j − k$ ) ⋅ ( − 2 $i + λ $j + 10 k$ ) = 0 = log   77. If f ( x ) = 31 + x , then f ( x ) f (y ) f (z )
 x2 + 1 − x 
⇒ − 6 + 4λ − 10 = 0   is equal to
⇒ 4λ − 16 = 0   (a) f( x + y + z) (b) f( x + y + z + 1)
1
= log   (c) f( x + y + z + 2 ) (d) f( x + y + z + 3)
∴ λ=4  x + 1 − x 
2
 1 +x
Ê (c) We have, f( x) = 3
71. What is the derivative of
= − log ( x + 1 − x )
2
Similarly, f( y) = 31 + y
sec 2 (tan −1 x ) with respect
= − f( x ) and f( z) = 31 + z
to x?
∴ f( x ) f( y) f( z) = 31 + x +1 + y +1 + z
(a) 2x (b) x 2 + 1 74. The domain of the function f 1 +2 +x +y +z
=3
(c) x + 1 (d) x 2 defined by
−1 f ( x ) = log x 10 is = f(2 + x + y + z)
Ê (a) Let y = sec (tan x)
2

(a) x > 10 78. The number of real roots for the


On differentiating both sides w.r.t x, we get (b) x > 0 excluding x = 10
dy d (c) x ≥ 10
equation x 2 + 9| x | + 20 = 0 is
= sec 2 (tan−1 x )
dx dx (d) x > 0 excluding x = 1 (a) zero
= 2 sec (tan−1 x ) ⋅ sec (tan−1 x ) (b) one
Ê (d) We have, f( x) = log x 10 (c) two
d
tan (tan−1 x ) (tan−1 x ) log 10 1 (d) three
dx = =
log x log x
Ê (a) Given, x + 9| x | + 20 = 0
2
1
= 2 sec 2(tan−1 x ) ⋅ x ⋅ ∴ f( x ) is define when x > 0 and x ≠ 1.
1 + x2 ⇒ x 2 + 9 x + 20 = 0

= 2 (1 + tan2 (tan−1 x )) ⋅
x 1 − cos 4 x 3
or x 2 − 9 x + 20 = 0
1+ x 2 75. lim is equal to
x→ 0 x2 ⇒ x 2 + 4 x + 5 x + 20 = 0
x (a) 0 (b) 12
= 2 (1 + x ) ⋅
2
= 2x or x 2 − 4 x − 5 x + 20 = 0
1 + x2 (c) 24 (d) 36
1 − cos 3 4 x  0  ⇒ x ( x + 4) + 5 ( x + 4) = 0
72. If f ( x ) = log10 (1 + x ), then what is Ê (c) xlim  0
form
 or x ( x − 4) − 5 ( x − 4) = 0
→0 x2
4 f ( 4 ) + 5 f (1) − log10 2 equal to? ⇒ ( x + 4) ( x + 5) = 0
On apply L′ Hospital rule we get
or ( x − 4) ( x − 5) = 0
(a) 0 − 3 cos 2 ( 4 x ) ( − sin 4 x ) ( 4)
lim ⇒ x = − 4, − 5, or 4, 5
(b) 1 x→ 0 2x
(c) 2 But these values of x does not satisfy the
12 cos 2 4 x sin 4 x  0
(d) 4 = lim form given equation.
x→ 0 2x  0 
Ê (d) We have, f( x) = log10 (1 + x) Again, apply’s L′ Hospital rule, we get
Hence, number of real roots of the given
equation is zero.
∴ 4f( 4) = 4 log10 (1 + 4) = 4 log10 5 12 [2 cos ( 4 x ) ( − sin 4 x ) ( 4)
5f(1) = 5 log10 (1 + 1) = 5 log10 2 79. If f ( x ) = sin(cos x ), then f ′( x ) is
sin 4 x + cos 2 4 x (cos 4 x ) ( 4)] equal to
∴ 4f( 4) + 5f(1) − log10 2 lim
x→ 0 2 (a) cos(cos x )
= 4 log10 5 + 5 log10 2 − log10 2 12 [− 8 cos 4 x sin2 4 x (b) sin( − sin x )
= 4 log10 5 + 4 log10 2 + 4 cos 3 4 x ] (c) (sin x )cos(cos x )
= lim
= 4 (log10 5 × log10 2 ) x→ 0 2 (d) (− sin x )cos(cos x )
= 4 log10 ( 5 × 2 ) = 6 ( − 8 × 0 + 4) Ê (d) Given, f( x) = sin (cos x)
= 4 log10 10 = 4 × 1 = 4 = 24 ⇒ f ′ ( x ) = cos (cos x ) ( − sin x )
10 NDA/NA Solved Paper 2019 (I)

π π − h (π + 2)
80. The domain of the function = [sin x + cos x ]04 + [− cos x − sin x ]π2 ⇒ 2r =
π
f ( x ) = (2 − x )( x − 3) is 4

2r − (π + 2)
= ⇒
h
=
− π
 1 1   h π 2r π+2
(a) ( 0, ∞ ) (b) [0, ∞ ) =  +  − ( 0 + 1)
(c) [2, 3] (d) (2, 3)  2 2  Neglecting − sign as r and h can not be
  1 1  negative.
Ê (c) We have, f( x) = (2 − x) ( x − 3) + ( − 0 − 1) − − − 
  2 2   ∴
h
=
π
f( x ) will be define if (2 − x ) ( x − 3) ≥ 0 2r π+2
2 2 4
⇒ ( x − 2 ) ( x − 3) ≤ 0 = − 1− 1+ = −2
2 2 2 85. π
∴ 2 ≤ x≤ 3
∫02 e
sin x
= 2 2 − 2 = 2 ( 2 − 1) cos xdx is equal to
81. The solution of the differential (a) e + 1 (b) e − 1
83. If y = a cos 2x + b sin 2x , then
equation (c) e + 2 (d) e
2 2 π
d y d y
dy (a) + y=0 (b) + 2y = 0
= cos(y − x ) + 1 is I= ∫02 e
sin x
dx 2 dx 2 Ê (b) Let cos x dx
dx
d 2y d 2y sin x = t
(a) e x [sec( y − x ) − tan( y − x )] = c (c) − 4y = 0 (d) + 4y = 0 Let
dx 2 dx 2 ⇒ cos x dx = dt
(b) e x [sec( y − x ) + tan( y − x )] = c
(c) e x sec( y − x ) tan( y − x ) = c
Ê (d) Given, y = a cos 2 x + b sin 2 x ...(i) π
When x = , t = 1
dy 2
(d) e x = c sec( y − x ) tan( y − x ) ⇒ = − 2 a sin 2 x + 2 b cos 2 x
dy dx x = 0, t = 0
Ê (a) Given, = cos ( y − x) + 1 ...(i) d 2y 1 t
dx ⇒
dx 2
= − 4a cos 2 x − 4b sin 2 x ∴ I= ∫0 e dt = [et ]10
Let y− x = t
d 2y = e1 − e 0 = e − 1
dy dt dy dt ⇒ = − 4 ( a cos 2 x + b sin 2 x )
⇒ − 1= ⇒ = 1+
dx dx dx dx dx 2 86. If f ( x ) = x − 2 , x ≠ −2, then what is
From Eq. (i), 1 +
dt
= cos t + 1 d 2y x +2
⇒ = − 4y [using Eq. (i)]
dx dx 2 −1
dt f ( x ) equal to ?
⇒ = cos t d 2y 4( x + 2 ) x+2
dx ∴ + 4y = 0
dx 2 (a) (b)
x −2 4( x − 2 )
⇒ sec t dt = dx
84. A given quantity of metal is to be x +2 2(1 + x )
On integrating both sides, we get (c) (d)
cast into a half cylinder x −2 1−x
∫ sec t dt = ∫ dx (i.e. with a rectangular base and x−2 x−2
log (sec t + tan t ) = x + a Ê (d) Given, f( x) = ⇒ y=
semicircular ends). If the total x+2 x+2
+a
⇒ sec t + tan t = e x surface area is to be minimum, then ⇒ x − 2 = xy + 2 y
⇒ sec t + tan t = e x ⋅ e a the ratio of the height of the half ⇒ x − xy = 2 y + 2
cylinder to the diameter of the ⇒ x (1 − y) = 2 y + 2
ex
⇒ = e −a semicircular ends is 2 ( y + 1)
sec t + tan t ⇒ x=
(a) π :( π + 2 ) (b) ( π + 2 ) : π 1− y
e (sec t − tan t )
x
(c) 1 : 1 (d) None of these
⇒ = e −a 2 ( y + 1)
(sec t + tan t ) (sec t − tan t ) ⇒ f −1( y) =
Ê (a) Let r be the radius and h be the height 1− y
of the half cylinder,
e x (sec t − tan t ) 2 ( x + 1)
⇒ = e −a Then, surface area, S = πrh + πr 2 + 2 rh ∴ f −1( x ) =
sec 2 t − tan2 t 1− x
⇒ e x (sec t − tan t ) = e − a
87. What is ∫ ln( x 2 )dx equal to?
⇒ e [sec ( y − x ) − tan ( y − x )] = e − a
x

(a) 2 x ln( x ) − 2 x + C
∴ e x [sec ( y − x ) − tan ( y − x )] = c, 2
(b) + C
[where, c = e − a] h x
(c) 2 x ln( x ) + C
82. π


2 ln( x )
2 |sin x − cos x | dx is equal to (d) −2x + C
0 x
(a) 0 (b) 2( 2 − 1) Ê (a) Let I = ∫ ln ( x 2 ) dx = ∫ 2 ln x dx
(c) 2 2 (d) 2( 2 + 1)
π dS = ln x ∫ 2 dx
∴ = πh + 2 πr + 2 h
Ê (b) ∫02| sin x − cos x | dx d
dr − ∫  (ln x ) ∫ (2 dx ) dx
π dS  dx
=0
= ∫04 (cos x − sin x) dx On putting
dr 1
π ( πh + 2 h) = ln x ⋅ 2 x − ∫ ⋅ 2 x dx
⇒ 2r = − x
+ ∫ π2(sin x − cos x) dx π
= 2 x ln x − 2 x + C
4
NDA/NA Solved Paper 2019 (I) 11

88. The minimum distance from the 90. Consider the following in respect of c2 
⇒ z = a2 x + b 2   ...(ii)
point (4, 2) to y 2 = 8x is equal to the differential equation :  x
2 On differentiability Eq. (ii) both sides, we
(a) 2 (b) 2 2 d 2y  dy 
+ 2  + 9 x = x get
(c) 2 (d) 3 2 dx 2  dx  dz b 2c 2
= a2 − ...(iii)
Ê (b) Let ( x, y) be any point on the curve 1. The degree of the differential dx x2
y 2 = 8 x. equation is 1. d 2z 2 b 2c 2
Then, the distance between ( x, y) and ⇒ = ...(iv)
2. The order of the differential dx 2
x3
( 4, 2 ) is equation is 2. dz
For maxima and minima, we put =0
D 2 = ( x − 4)2 + ( y − 2 )2 Which of the above statements dx
2
 y2  is/are correct ? b 2c 2
⇒ D2 =  − 4 + ( y − 2 )2 ...(i) ∴ a2 − =0
(a) Only 1 x2
 8  (b) Only 2 b 2c 2
[Q y 2 = 8 x] (c) Both 1 and 2 ⇒ = a2
x2
dD 2 y 2   2 y (d) Neither 1 nor 2
bc
⇒ =2  − 4   + 2 ( y − 2 ) ⇒ x=±
dy  8  8 Ê (c) The order of highest order derivative a
 2 y3 occuring in the differential equation is 2 bc d 2 z 2 a 3
2 y At x = = >0
=2  −4×  + 2 (y − 2) and its degree is 1. ,
 64 8 a dx 2 bc
y3 y3 91. What is the general solution of the ⇒ Gives minimum value
=
− 2y + 2y − 4= −4
16 16 differential equation
dy x
+ = 0? − bc d 2 z 2 a3
At x = , 2 =− <0
d 2D 2 3 y 2 dx y a dx bc
⇒ 2 =
d y 16 (a) x 2 + y 2 = C (b) x 2 − y 2 = C Gives maximum value
bc
dD 2 y3 (c) x 2 + y 2 = Cxy (d) x + y = C ∴ Minimum value of z at x = is
On putting =0 ⇒ −4=0 a
dy 16 Ê (a) Given differential equation, abc + abc = 2 abc.
⇒ y = 64 ⇒ y = 4
3 dy x
+ =0
d D 2 2 dx y 94. What is ∫ e x ln(a )dx equal to?
At y = 4, >0 dy − x
d 2y ⇒ = ax ex
So, it is point of minima. dx y (a) +C (b) +C
ln(a) ln(a)
∴Minimum D = (2 − 4)2 + ( 4 − 2 )2 ⇒ ydy = − xdx
ex ae x
Integrating both sides, we get (c) +C (d) +C
= 4+ 4= 8 =2 2 ln(ae ) ln(a)
y2 − x 2
= + C1 x
89. The differential equation of the 2 2 Ê (a) Let I = ∫ e
x ln a
dx = ∫ elna dx
system of circles touching theY -axis ∴ x 2 + y2 = C [where,C = 2C1] ax
= ∫a dx = +C
x
at the origin is
92. The value of k which makes ln a
dy
(a) x 2 + y 2 − 2 xy =0
sin x , x ≠ 0 95. What is the area of one of the loops
dx
f (x ) =  continuous at
(b) x 2 + y 2 + 2 xy
dy
=0  k, x =0 between the curve y = c sin x and
dx
dy x = 0, is X -axis ?
(c) x 2 − y 2 + 2 xy =0 (a) c (b) 2c
dx (a) 2 (b) 1
dy (c) −1 (d) 0 (c) 3c (d) 4c
(d) x 2 − y 2 − 2 xy =0 π
dx Ê (d) Given, f( x) is continuous at x = 0. Ê (d)∴ Required area = 2 ∫0 c sin x dx
Ê (c) The system of circles touching the ∴ lim f( x ) = f( 0) Y
Y-axis at the origin is x→ 0

( x − a) + y = a
2 2 2 ⇒ lim sin x = k y = csin x
x→ 0
∴ k=0 X′ X
⇒ x + a − 2 ax + y = a
2 2 2 2
O π π
93. What is the minimum value of 2
⇒ x 2 − 2 ax + y 2 = 0 ...(i)
On differentiating Eq. (i) w.r.t. x, we get a 2 x + b 2y where xy = c 2 ? Y′
π
= 2c ∫0 sin x dx = 2c[− cos x ]π0
2 x −`2 a + 2 yy′ = 0 (a) abc (b) 2 abc
⇒ x + yy′ = a (c) 3abc (d) 4abc = 2c [− (cos π − cos 0)]
= 2c (2 ) = 4c sq units
Ê (b) Let z = a x + b y
Put value of a in Eq. (i), we get 2 2
...(i)
x 2 − 2 ( x + yy′ ) x + y 2 = 0 Since, xy = c 2 96. If sin θ + cos θ = 2 cos θ, then what
⇒ x 2 − 2 x 2 − 2 xyy′ + y 2 = 0 c2 is (cos θ − sin θ ) equal to ?
⇒ y=
⇒ − x 2 − 2 xyy′ + y 2 = 0 x
(a) − 2 cosθ (b) − 2 sinθ
c2
dy On putting y = in Eq. (i), we get
∴ x − y + 2 xy
2 2
=0 x (c) 2 sinθ (d) 2sinθ
dx
12 NDA/NA Solved Paper 2019 (I)

Ê (c) Given, sin θ + cos θ = 2 cos θ 100. The angle of elevation of a tower of 103. A problem is given to three students
⇒ sin θ = ( 2 − 1) cos θ ...(i) height h from a point A due South of A , B and C whose probabilities of
Now, cos θ − sin θ it is x and from a point B due East of solving the
= cos θ − ( 2 − 1) cos θ [from Eq. (i)] A is y. If AB = z , then which one of
= cos θ [1 − ( 2 − 1)] = cos θ [2 − 2 ] 1 3 1
the following is correct ? problem are , and respectively.
= cos θ ⋅ 2 ( 2 − 1) 2 4 4
(a) h2 (cot 2 y − cot 2 x ) = z2
= 2 sin θ [using Eq. (i)] What is the probability that the
97. In a circle of diameter 44 cm, the (b) z2 (cot 2 y − cot 2 x ) = h2
problem will be solved if they all
length of a chord is 22 cm. What is solve the problem independently ?
(c) h2 (tan2 y − tan2 x ) = z2
the length of minor arc of the chord? 29 27
(a) (b)
484 242 (d) z (tan y − tan x ) = h
2 2 2 2 32 32
(a) cm (b) cm 25 23
21 21 (c) (d)
(c)
121
cm (d)
44
cm Ê (a) P 32 32
21 7 1 1
Ê (a) We have, P( A ) = , P( A ) =
2 2
Ê (a) Given, diameter of a circle be 44 cm. h 3 1
P( B) = , P( B ) =
4 4
y 1 3
and P(C ) = , P(C ) =
O B 4 4
O
∴ Required probability
cm x
22

22 = 1 − P( A ) P( B ) P(C )
cm

1 1 3 29
A 22 cm B A = 1− × × =
Here, OP be the tower, 2 4 4 32
⇒ 2 r = 44 OA = h cot x 104. A pair of fair dice is rolled. What is
⇒ r = 22 OB = h cot y the probability that the second dice
⇒ ∆OAB is an equilateral triangle. In right-angled ∆OAB, lands on a higher value than does the
⇒ ∠AOB = 60°
h2 cot 2 y = z2 + h2 cot 2 x first?
∴ Length of minor arc
1 1
60°  ∴ z2 = h2 (cot 2 y − cot 2 x ) (a) (b)
=   × 2 π × 22 4 6
 360°  5 5
1 22
101. From a deck of cards, cards are taken (c) (d)
= ×2 × × 22 out with replacement. What is the 12 18
6 7
484 probability that the fourteenth card Ê (c) Total number of possible outcomes
= cm = 36
21 taken out is an ace?
1 4 1 1 Favourable outcomes
1 1 (a) (b) (c) (d)
98. If sinθ = − and tanθ = , then in 51 51 52 13 = (1, 2 ), (1, 3), (1, 4), (1, 5), (1, 6),
2 3 (2, 3), (2, 4), (2, 5),(2, 6) ( 3, 4),
which quadrant does θ lie? Ê (d) Total number of possible outcomes ( 3, 5), ( 3, 6), ( 4, 5), ( 4, 6), ( 5, 6)
= 52
(a) First ∴Total number of favourable outcomes
And number of favourable outcomes = 4
(b) Second = 15
4 1 15 5
(c) Third ∴Required probability = = ∴Required probability = =
(d) Fourth 52 13 36 12
(c) We know that, if θ lies in third quadrant 102. If A and B are two events such that
then, sinθ < 0 and tanθ > 0.
P ( A ) = 0.5, P ( B ) = 06
. and 105. A fair coin is tossed and an unbiased
99. How many three digit even numbers P ( A ∩ B ) =0. 4, then what is dice is rolled together. What is the
can be formed using the digits 1, 2, 3, P ( A ∪ B ) equal to ? probability of getting a 2 or 4 or 6
4 and 5 when repetition of digits is not (a) 0.9 (b) 0.7 along with head?
allowed? (c) 0.5 (d) 0.3 1 1 1 1
(a) (b) (c) (d)
2 3 4 6
(a) 36 (b) 30 Ê (d) P( A ∪ B) = 1 − P( A ∪ B)
(c) 24 (d) 12
We have, Ê (c) Total number of possible outcomes
= 2 × 6 = 12
Ê (c) Here, unit digit can be filled by 2 or 4. P( A ∪ B) = P( A ) + P( B) − P( A ∩ B)
so number of ways is 2. Since repetition And favourable outcomes
is not allowed therefore hundred place = 0.5 + 0.6 − 0.4 = (H, 2), (H, 4), (H, 6)
and ten place can be fill in 4 C 2 × 2 ways = 11
. − 0.4 ∴Total number of possible outcomes = 3
∴ Total number of three digits even = 07. ∴Required probability =
3
=
1
number = 4 × 3 × 2 = 24 ∴ P( A ∪ B) = 1 − 07 . = 0.3 12 4
NDA/NA Solved Paper 2019 (I) 13

106. If A , B and C are three events, then 109. The mean of 100 observations is 50 111. Two symmetric dice flipped with
what is the probability that atleast and the standard deviation is 10. If 5 each dice having two sides painted
two of these events occur together ? is subtracted from each observation red, two painted black, one painted
(a) P( A ∩ B) + P( B ∩ C ) + P(C ∩ A ) and then it is divided by 4, then what yellow and the other painted
(b) P( A ∩ B) + P( B ∩ C ) + P(C ∩ A ) will be the new mean and the new white. What is the probability that
− P( A ∩ B ∩ C ) standard deviation respectively ? both land on the same colour ?
(c) P( A ∩ B) + P( B ∩ C ) + P(C ∩ A ) (a) 45, 5 3 2
(a) (b)
− 2 P( A ∩ B ∩ C ) (b) 11.25, 1.25 18 9
(c) 11.25, 2.5 5 1
(d) P( A ∩ B) + P( B ∩ C ) + P(C ∩ A ) (c) (d)
(d) 12.5, 2.5
− 3P( A ∩ B ∩ C ) 18 3
Ê (c) Given, mean ( x ) = 50 2 2
Ê (c) If A, B and C are three events, then Ê (c) P (two sides painted red) = ×
50 − 5 6 6
atleast two events occur i.e. The new mean =
4 2 2
( A ∩ B ∩ C ′ ) ∪ ( A ∩ B′ ∩ C ) P (two sides painted black) = ×
45 6 6
∪ ( A′ ∩ B ∩ C ) ∪ ( A ∩ B ∩ C ) = = 11.25
4 1 1
P (one side painted yellow) = ×
∴Required probability And standard deviation ( σ ) = 10 6 6
= P( A ∩ B) + P( B ∩ C ) + P(C ∩ A ) ∴The new standard deviation 1 1
and P (other side painted white) = ×
−2 P( A ∩ B ∩ C ) 10 6 6
=
107. If two variables X and Y are 4 ∴ Required probability that both land on
= 2.5 the same colour
independent, then what is the
Since, addition and subtraction does not 2 2 2 2 1 1 1 1
correlation coefficient between them? = × + × + × + ×
effect standard deviation. 6 6 6 6 6 6 6 6
(a) 1 (b) −1
110. If two fair dice are rolled, then what 4 + 4 + 1+ 1
(c) 0 (d) None of these =
is the conditional probability that 36
Ê (c) Correlation coefficient between two
the first dice lands on 6, given 10 5
independent variables is zero. = =
that the sum of numbers on the 36 18
108. Two independent events A and B are dice is 8?
2 112. There are n socks in a drawer, of
such that P( A ∪ B ) = and (a)
1
which 3 socks are red. If 2 of the
3 3
1 1 socks are chosen randomly and the
P ( A ∩ B ) = . If P ( B ) < P ( A ), then (b)
probability that both selected socks
6 4
1 1
what is P ( B ) equal to ? (c) are red is , then what is the value
5 2
1 1 1
(a) (b) (d) of n ?
4 3 6
1 1 (a) 3
(c) (d)
2 6 Ê (c) Let E1 = Event of first dice on 6 (b) 4
2 E 2 = Event of the sum of numbers on (c) 5
Ê (b) Given, P( A ∪ B) = dices 8 (d) 6
3
and P( A ∩ B) =
1 ∴ Total number of sample space of two Ê (b) Total number of socks = n
6 dices are rolled, n( s ) = 36
3
⇒ P( A ∪ B) = P( A ) + P( B) − P( A ∩ B) Possible outcomes of E1 (6, 2) P (first socks is red) =
2 1 n
⇒ = P( A ) + P( B) − Possible outcomes of E 2 (2, 6) (3, 5)
3 6 2
(4, 4) (5, 3) (6, 2) P (second socks is red) =
2 1 n−1
⇒ P( A ) + P( B) = + 1
3 6 ∴ P( E1 ∩ E 2 ) = According to the question,
5 36
⇒ P( A ) + P( B) = ...(i) 5
3
×
2
=
1
6 and P( E 2 ) = n n−1 2
1 36
And also, P( A ∩ B) = ⇒ n2 − n = 12
6 E 
∴Required probability = P  1 
1  E2  ⇒ n − n − 12 = 0
2
⇒ P( A ) P( B) = ...(ii)
6 P( E1 ∩ E 2 ) ⇒ n − 4n + 3n − 12 = 0
2
= ,
From Eqs. (i) and (ii), we get P( E 2 )
1 1 ⇒ n ( n − 4) + 3 ( n − 4) = 0
P( A ) or P( B) = or when P( E 2 ≠ 0)
2 3 ⇒ ( n − 4) ( n + 3) = 0
1 ∴ n = 4, − 3
Also, given P( B) < P( A )
36 1
1 = =
∴ P( B) = 5 / 36 5
3
14 NDA/NA Solved Paper 2019 (I)

113. Two cards are chosen at random Ê (b) Given, P( A ∩ B ) = 0 118. If all the natural numbers between 1
from a deck of 52 playing cards. ⇒ P( A ∪ B) = 0 and 20 are multiplied by 3, then what
What is the probability that both of ⇒ 1 − P( A ∪ B) = 0 is the variance of the resulting
them have the same value ? series?
⇒ P( A ∪ B) = 1
1 3 (a) 99.75 (b) 199.75
(a) (b) We know that,
17 17 (c) 299.25 (d) 399.25
5 7 P( A ∪ B) = P( A ) + P( B) − P( A ∩ B)
(c) (d) Ê (c) Variance of first n natural number
17 17 ⇒ 1 = 0.5 + 0.6 − P( A ∩ B) n2 − 1 20 2 − 1
4
C 2 × 13 = =
Ê (a)∴ Required probability = ⇒ P( A ∩ B) = 01. 12 12
52
C2 ( A ∩ B)
P   =
A P 399
4 × 3 × 13 ∴ = = 33 .25
=  B P( B) 12
52 × 51 If all the natural number between 1 and 20
01
. 1
1 = =
= 0.6 6 multiplied by 3, then
17 ∴Required variance = 9 × 33 .25
116. Consider the following statements = 299 .25
114. In eight throws of a die, 5 or 6 is
considered a success. The mean 1. The algebraic sum of deviations
of a set of values from their 119. What is the probability that an
and standard deviation of total interior point in a circle is closer to
number of successes is respectively arithmetic mean is always zero.
2. Arithmetic mean > Median > the centre than to the
given by circumference?
8 16 8 4
Mode for a symmetric
(a) , (b) , distribution. 1 1
(a) (b)
3 9 3 3 4 2
4 4 4 16 Which of the above statements 3
(c) , (d) , is/are correct? (c)
3 3 3 9 4
(a) Only 1
1 1 1 (d) It cannot be determined
Ê (b) We have, p (success) = + = (b) Only 2
6 6 3 (c) Both 1 and 2 Ê (a) Let radius of circle be r, then the points
2
∴ q = 1− p= (d) Neither 1 nor 2 closer to centre if circumference will lie
3 r
Ê (a) We know that, the algebraic sum of within radius of .
Given, n = 8 2
1 8 deviations of a set of values from their
∴ Mean = np = 8 × = arithmetic mean is always zero. So, the favourable outcome would be the
3 3 points inside the area of circle with radius
Standard deviation = npq 117. Let the correlation coefficient r
whereas the total possible outcomes
1 2 between X and Y be 0.6. Random 2
= 8× ×
3 3 variables Z and W are defined as could be all the points inside the area of
Y circle with radius r.
=
16 4
= Z = X + 5 and W = . What is the 2
π  
9 3 3 r
correlation coefficient between Z 2 1
∴ Required probability = =
115. A and B are two events such that A andW ? πr 2 4
and B are mutually exclusive. If (a) 0.1 (b) 0.2 120. If A and B are two events, then what
P ( A ) = 0.5 and P ( B ) = 06,
. then what (c) 0.36 (d) 0.6
is the probability of occurrence of
is the value of P ( A / B ) ? Ê (d) Since, the correlation coefficient is either event A or event B ?
1 1 independent of change of origin and
(a) (b) (a) P( A ) + P( B) (b) P( A ∪ B)
scale. It is given that correlation
5 6 (c) P( A ∩ B) (d) P( A ) P( B)
2 1 coefficient between X and Y be 0.6.
(c) (d) Ê (b) If A and B are two events, then the
5 3 So, correlation coefficient between Z
and W be 0.6. probability of occurrence of either event
A or event B is P( A ∪ B).
PAPER II English Language and General Studies
Part A (English Language)
1. Opening his letters, (a)/ reading grammatically correct. In conditional 13. A glance at a beautiful object gives
them carefully and sending for his sentences, condition is expressed by ‘If’ us delight.
clerk, (b)/ he dictated answers with clause and Simple Present tense is used (a) wisdom (b) happiness
them. (c)/ No error (d) to show a future result of that activity. (c) purity (d) peace
Ê (c) Here, the use of preposition ‘with’ is 7. My brother goes (a)/ to the office Ê (b) ‘Happiness’ is the appropriate
incorrect here. Replace it with ‘to’, to (b)/ five day week. (c) No error (d) synonym of word ‘delight’. Both words
make the given sentence grammatically mean ‘a feeling of great pleasure and
correct. ‘Dictated to’ refers ‘to say Ê (c) Here, the use of ‘Five days a week’ is satisfaction.’ Some other synonyms
more appropriate in place of ‘five day are-pleasure, joy, etc.
something aloud so that somebody else
week’ to make the given sentence
can write or type it.’ 14. It is terrible for people to die of
grammatically correct. Five is plural, so
2. He was my school-friend, (a)/ but days should be used and ‘a’ is used for starvation.
becoming a great man, (b)/ he has denoting one week. (a) starch (b) staple
grown proud enough to forget his old (c) plenty (d) hunger
8. If you lend Mohan a pen (a)/ he will
friends. (c)/ No error (d) lend it to someone else (b)/ and Ê (d) ‘Hunger’ is the correct synonym of
word ‘starvation’. ‘Starvation’ means ‘the
Ê (b) Here ‘but becoming’ is incorrect in never you will get it back. (c)/ No
state of having no food for a long period,
part (b). It should be replaced with ‘but error (d) often causing death’.
after becoming’ as preposition ‘after’
indicates ‘at a later time’ which makes Ê (c) Here, part (c) of the given sentence has Some other synonyms are-lack of food,
an error. ‘And never you will get it back’ is famine, deprivation, etc.
the given sentence grammatically
incorrect because adverb should be followed
correct.
by helping verb. So, the correct phrase will
15. The university has constituted a
be - ‘and you will never get it back’.
grievance redressal committee to
3. Rabindranath Tagore, (a)/ a Nobel look into the matter.
laureate and the author of the 9. One of most widely spread (a)/ bad (a) depression (b) complaint
National Anthem, (b)/ found habits (b)/ (c) abrasion (d) gratefulness
Shantiniketan. (c)/ No error is the use of tobacco. (c) No error (d)
(d) Ê (b) ‘Complaint’ means the same as the
word ‘grievance’. ‘Grievance’ means ‘an
Ê (a) Here part (a) has an error. Article ‘the’
Ê (c) As the given sentence is in Simple should be used before superlative official statement of a complaint over
Past Tense, verb ‘found’ should be degree (most). ‘One of the most’ is the something believed to be wrong or
replaced with its past form ‘founded’ to correct phrase to make the given unfair’. Some other synonyms are -
make the given sentence grammatically sentence grammatically correct. grudge, grumble, etc.
correct.
10. A great part (a)/ of Arabia (b)/ is 16. Rakesh delivered a slanderous speech.
4. The art of printing was introduced desert. (c) No error (d) (a) abusive (b) praiseworthy
into England (a)/ during the reign of (c) moderate (d) inspiring
Edward IV (b)/ by William Caxton, a Ê (d) There is no error. The sentence is
grammatically correct. Ê (a) ‘Abusive’ is similar in meaning to the
native of Kent. (c)/ No error (d) word ‘slanderous’. ‘Slanderous’ means ‘a
Ê (a) Here part (a) has an error. Use of
11. Some people complain when they false spoken statement about someone
preposition ‘into’ is incorrect. Replace it encounter a small misfortune in the that damages their reputation’. Some
with preposition ‘in’ as it is used to course of their thoroughly happy life. other synonyms of ‘slanderous’ are –
indicate a location or place while malicious, damaging, derogatory, etc.
(a) run into (b) run away
preposition ‘into’ is used to indicate (c) run down (d) run with 17. Suddenly, the sky was darkened by
movement toward the inside of a place. a gigantic bird.
Ê (a) ‘Run into’ means the same as ‘encounter’.
5. From thirty years (a)/ he devoted Both words mean ‘meet someone (a) winged (b) small
himself to public affairs (b)/without unexpectedly’. Some other synonyms (c) tiny (d) enormous
are-‘collide with’ or ‘bump into.’
taking a holiday. (c)/ No error (d) Ê (d) ‘Enormous’ means the same as
12. This world is full of miseries. ‘gigantic’. The word ‘gigantic’ means
Ê (a) Here, use ‘for’ in place of ‘from’ to start ‘something of very great size or extent’.
the sentence. Use of preposition ‘for’ (a) indifferent love
Some other synonyms of the word are –
shows time period, which is (b) perfect happiness
huge, vast, expansive, etc.
grammatically correct. (c) great suffering
(d) moderate sympathies 18. To abolish poverty would be to
6. If Ramesh will be promoted (a)/ he destroy the soil upon which mankind
will get (b)/ a higher salary. (c) No Ê (c) ‘Great suffering’ is a perfect synonym
of word ‘miseries’. ‘Miseries’ means ‘a produces the virtues conducive to
error (d) state or feeling of great physical or higher civilisation.
mental distress’. Some other synonyms
Ê (a) Here use of ‘is promoted’ in place of are-unhappiness, distress, etc.
(a) detest (b) eradicate
‘will be promoted’ make the sentence (c) nurture (d) assimilate
16 NDA/NA Solved Paper 2019 (I)

Ê (b) ‘Eradicate’ is similar word in meaning 25. He had deeper hostility towards Ê (c) Idiom ‘A show-stopper’ means ‘a
to the word ‘abolish’. Both words mean Mohan. performance in a show that is extremely
‘to remove forever’. good’. So, option (c) expresses the
(a) animosity (b) belligerence
Some other synonyms are – eliminate, correct meaning of the given idiom.
(c) malice (d) friendship
wipe out, terminate, get rid of, etc.
Ê (d) ‘The word ‘hostility’ means ‘malice or 33. A jack of all trades
19. The Arabs who are not in the cities unfriendliness or opposition’. Its (a) Someone who has many skills
live in the desert throughout the antonym is ‘friendship’ which means (b) A confident and not very serious
year, shifting from one oasis to ‘close association or bond’. young man
another. (c) Someone who has hit the jackpot
26. His life is rather monotonous. (d) a great businessman
(a) sandbank (b) mound (a) exciting (b) dreary
(c) dune (d) spring (c) tedious (d) uneventful Ê (a) Idiom ‘A jack of all trades’ means
‘someone who has many skills’. So,
Ê (d) ‘Spring’ is the most suitable synonym Ê (a) The word ‘monotonous’ means ‘boring option (a) is a suitable choice.
from the given options. ‘Oasis’ means ‘a or unexciting’. Its antonym is ‘exciting’
fertile spot in a desert where water is which means ‘thrilling or stimulating’. 34. Fight tooth and nail
found’. (a) To quarrel with someone
Some other synonyms can be– watering 27. Macbeth is a morally repulsive (b) To attack someone with a lot of force
hole, watering place, etc. character. (c) To try hard to prevent something from
(a) abominable (b) attractive happening
20. The various facets of life can be (d) To try very hard to achieve something
(c) obnoxious (d) ugly
found reflected in large city.
(a) several (b) similar Ê (b) The word ‘repulsive’ means ‘arousing Ê (d) Idiom ‘Fight with tooth and nail’ means
intense distaste’. Its antonym is ‘attractive’ ‘to try very hard to achieve something’.
(c) valuable (d) singular
which means ‘pleasing or agreeable’. So, option (d) is a suitable choice.
Ê (a) ‘Various’ means ‘several or different’.
So, ‘several’ is a perfect synonym of 28. The serene beauty of Kashmir had a 35. Fair and square
word ‘various’. soothing effect on his mind. (a) in an honest way
Some other synonyms are-varied, (a) placid (b) pleasing (b) in a critical way
varying, etc. (c) tranquil (d) turbulent (c) neither very good nor very bad
(d) in a foolish way
21. Ramesh is a very dubious character. Ê (d) ‘The word ‘serene’ means ‘peaceful or
(a) shady (b) suspicious calm’. Its antonym is ‘turbulent’ which Ê (a) Idiom ‘Fair and square’ means ‘in a
means ‘unstable, chaotic or stormy’. honest way and according to the rules’.
(c) trustworthy (d) doubtful
So, option (a) expresses the correct
Ê (c) ‘The word ‘dubious’ means ‘of 29. Life is transient in nature. meaning of the given idiom.
doubtful character’. Hence, its antonym (a) brief (b) momentary
is ‘trustworthy’ which means ‘reliable or (c) eternal (d) short-lived
36. S1 : We do not know what to do
dependable’. with our knowledge.
Ê (c) The word ‘transient’ means
S6 : In the course of time they may
22. Do not indulge in unmindful ‘temporary or short lived’. Its antonym is
‘eternal’, which means ‘everlasting or rule over us altogether.
activities, please.
immortal, perpetual’. P : For example, we are unable to
(a) vigilant (b) careless
manage our machines.
(c) stupid (d) fatuous 30. Sohan is a vain person. Q : We already find it difficult to do
Ê (a) The word ‘unmindful’ means ‘paying (a) modest (b) arrogant without machines.
no heed or being careless’. Its antonym (c) conceited (d) proud R : Machines should be fed
is ‘vigilant’, which means ‘watchful or properly and waited upon
observant, alert’ etc. Ê (a) The word ‘vain’ means ‘proud or
arrogant’. Its antonym is ‘modest’ which attentively; otherwise they refuse to
23. He is suffering from a curable means ‘humble or unpretentious’. work or cause destruction.
disease. S : Science has given us
31. A dark horse
(a) remediable (b) treatable superhuman powers, which we do
(a) A black coloured horse
(c) terminal (d) operable not use properly.
(b) A person who wins a race or
The proper sequence should be
Ê (c) The word ‘curable’ means ‘treatable competition although no one
(a) S P R Q (b) P S Q R
or remediable’. Its antonym is ‘terminal’ expected him to
(c) A person who keeps secrets (c) Q R P S (d) S R P Q
which means ‘incapable of being treated
or untreatable’. (d) An ignorant person Ê (a) SPRQ is the correct sequence of the
given sentences to form a meaningful
24. He was born on a very auspicious Ê (b) Idiom ‘a dark horse’ means ‘an unknown passage.
and unexpected winner of a race or
day. contest’. Hence, option (b) expresses 37. S1 : The British rule in India has
(a) propitious (b) fortunate the correct meaning of given idiom. brought about moral, material,
(c) ominous (d) opportune
32. A show-stopper cultural and spiritual ruination of
Ê (c) ‘Auspicious’ means ‘favourable or this great country.
conducive to success’. Its antonym is (a) Someone who stops the show
‘ominous’ which means ‘something sinister (b) Someone who organises the show S6 : We are not to kill anybody but it
or inauspicious or doomy, unfavourable’. (c) A performance that is extremely good is our dharma to see that the curse of
(d) A fashionable person this government is blotted out.
NDA/NA Solved Paper 2019 (I) 17

P : I regard this rule as a curse. people with an idea of nationality in a (a) One need not visit the Ganges to take
Q : Sedition has become my manner unknown in other parts of the holy bath
world. And we Indians are one as no two (b) The Ganges had been polluted, so
religion.
Englishmen are. Only you and I and others one should bath at
R : Ours is a non-violent battle. home
who consider ourselves civilised and
S : I am out to destroy this system (c) One should take a holy dip in the
superior persons imagine that we are many
of government. Ganges to purify one’s heart
nations. It was after the advent of railways (d) The purity of heart is superior to
The proper sequence should be that we began to believe in distinctions and
(a) S P R Q (b) P S Q R observance of any ritual
you are at liberty now to say that it is
(c) Q R P S (d) S R P Q
through the railways that we are beginning Ê (d) According to the author, if one’s heart
(b) PSQR is the correct sequence of the is pure, there is no need to observe any
to abolish those distinctions. rituals like going on pilgrimages or
given sentences to form a meaningful
An opium-eater may argue the advantage visiting different temples and other
passage.
of opium-eating from the fact that he prayer structures. God resides in a pure
38. the urban local body elections began to understand the evil of the opium heart.
(P)/unidentified gunmen (Q)/and habit after having eaten it. I would ask you
injured another during (R)/ shot to consider well what I had said on the 45. The passage is written in a
dead two workers (S) railways. (a) dialogic style (b) prescriptive style
(c) descriptive style (d) analytical style
(a) Q S R P (b) P Q S R 41. According to the author, India
(c) S P R Q (d) R P S Q
(a) has never been one nation
Ê (c) The passage is written in a descriptive
style. In descriptive writing style, the author
Ê (a) QSRP is the correct sequence. (b) has been an aggregate of several specifies an event, an object or a thing.
nations
39. both intense political and (P)/ this
state has a history of (Q)/of
(c) has always been one nation along 46. .......... forest prevent erosion.
with differences (a) Lean (b) Dense (c) Sparse (d) Tidy
syncretic accomplishments (R)/ (d) became a nation after the British
religious contestation and (S) came Ê (b) ‘Dense’ is the appropriate word for the
blank as it means ‘containing a lot of
(a) S Q P R (b) P Q S R Ê (c) The author has stated in the passage things (trees) close together.’ Other
(c) S Q R P (d) Q P S R that India has always been one nation
words do not fit in the blank.
Ê (d) QPSR is the correct sequence. along with diversities or differences.

42. Why did the great sages of India 47. Three people were arrested and an
40. the father also (P)/in his quest for illegal arms unit was ........ by the
justice (Q)/ by the system (R)/ feels establish pilgrimages in the
police in a raid.
let down (S) different corners of the country?
(a) revealed (b) searched
(a) S Q P R (b) P S R Q (a) Because they wanted to push people
(c) discovered (d) busted
(c) S Q R P (d) P Q R S to travel to different places.
(b) Because they could observe the Ê (d) ‘Busted’ is the appropriate word for
Ê (b) PSRQ is the correct sequence. underlying unity of the country as the blank. It means ‘raid or search by the
made by nature. police where illegal activity is suspected.’
(c) Because they themselves had Other words do not fit in the blank to
Passage 2 travelled to these places. make the sentence meaningful.
(d) Because they wanted people to be
I do not wish to suggest that because we 48. A woman got into the car and ... .
religious everywhere.
were one nation, we had no differences, (a) drove off (b) broke down
but it is submitted that our leading men Ê (b) The great sages of India established (c) rode in (d) drove in
travelled throughout India either on foot pilgrimages in the different corners of the
or in bullockcarts. They learned one country because they could observe the Ê (a) ‘Drove off’ is the correct phrase to
underlying unity of the country as made make the sentence meaningful. It means
another languages and there was no
by nature. For them, India was one ‘force someone to go away and to stop
aloofness amongst them. What do you
undivided nation. attacking or threatening you.’
think could have been the intention of
those farseeing ancestors of ours who 43. In the passage, the author’s attitude 49. The lecture was not very interesting.
established Setubandha (Rameshwar) in towards the railways is Infact I ....... in the middle of it.
the South, Jagannath in the East and (a) critical (b) sympathetic (a) showed off (b) put off
Haridwar in the North as places of (c) indifferent (d) apathetic (c) dozed off (d) plugged off
pilgrimage? You will admit they were no
fools. They knew that worship of God Ê (a) The author’s attitude towards the Ê (c) ‘Dozed off’ is correct phrasal verb to
railways is critical. According to the be filled in the blank. It means ‘to fall into
could have been performed just as well at author, due to railways people have been a light sleep unintentionally.’
home. They taught us that those whose able to see the diversities of nature,
hearts were aglow with righteousness had people, food, etc. But railways have
50. The cops ......... murder by kin.
the Ganges in their own homes. But they also helped to unite people of different (a) suspect (b) afford
saw that India was one undivided land so states. (c) manage (d) administer
made by nature. They, therefore, argued
44. What does the author mean when Ê (a) ‘Suspect’ is the appropriate word for
that it must be one nation. the blank to make the sentence
Arguing thus, they established holy places he says that ‘‘Whose hearts were meaningful. Suspect means ‘to have
in various parts of India and fired the aglow with righteousness had the doubts or mistrust.’
Ganges in their own homes’’? Other words are not relevant here.
18 NDA/NA Solved Paper 2019 (I)

Part B (General Studies)


51. Which one of the following cell 56. Spring tides refer to (a) gravitational potential energy → heat
organelles does not possess nucleic (a) greatest difference in the sea level at energy to air → kinetic energy → heat
acid? high and low tides energy to ground and apple→ sound
(b) lowest difference in the sea level at energy
(a) Nucleolus (b) Chloroplast
high and low tides (b) gravitational potential energy →
(c) Ribosome (d) Plasma membrane sound energy → kinetic energy →
(c) no difference in the sea level at high
Ê (d) The cell organelle given in option (d), and low tides heat energy to air → heat energy to
i.e. plasma membrane does not contain (d) counteraction of gravitational pull of ground and apple
nucleic acids. It is made up of lipids and the Sun to that of Moon (c) gravitational potential energy →
proteins. Chloroplast contains deoxyrib- kinetic energy → heat energy to air →
onucleic acid whereas ribosomes are Ê (a) The periodic rise & fall of sea level, heat energy to ground and apple →
once or twice a day, is called a Tide. sound energy
made up of ribonucleic acids and proteins.
When the Sun, the Moon and the Earth (d) gravitational potential energy →
Nucleolus is made up of proteins and are in a straight line, the height of tides will kinetic energy → sound energy →
ribonncleic acids (RNA) and the site for be higher than normal. These are called heat energy to air → heat energy to
the synthesis of ribosoural RNA. spring tides, which occur on new Moon or ground and apple
52. Which one of the following cell full Moon day. At these times, high tides
are very high & low tides are very low. Ê (c) When apple is on highest position,
organelles does not possess its own then it has only gravitational potential
Thus, spring tides refer to the greatest
genetic material encoding proteins? energy. When, it falls to the ground the
difference in the sea level at high and low
(a) Ribosome (b) Nucleus first its gravitational potential energy
tides.
(c) Mitochondria (d) Chloroplast starts converting into kinetic energy.
57. Which one of the following energy is Due to air friction some kinetic energy
Ê (a) Ribosome is the cell organelles, which stored in the links between the atoms? losses and converts into heat energy to
does not possess its own genetic
(a) Nuclear energy (b) Chemical energy air. When apple falls on the ground, then
material (DNA) encoding proteins.
Nucleus, mitochondria and chloroplast (c) Potential energy (d) Thermal energy its remaining energy converts into heat
energy to ground and sound energy.
contain DNA which code for proteins Ê (b) Chemical energy is stored in the
needed by these cell organelles. bonds of atoms and molecules. Hence, option (c) is correct.
Nuclear energy is stored in the nucleus 60. Which one of the following mineral
53. Which one of the following is not a of an atom.
component of conducting tissue in is used as a fuel in nuclear power
Potential energy is the energy of
plants? position or gravitational potential energy.
stations?
(a) Fibres (b) Tracheids Thermal energy or heat is the vibration (a) Bauxite (b) Quartz
(c) Pericycle (d) Sieve tubes or movement of atoms and molecules in (c) Feldspar (d) Pitchblende
substances. (d) Uraninite, formerly known as
Ê (c) Pericycle is not the component of pitchblende is a radioactive uranium rich
conducting tissue. It is a part of the 58. The light energy escaping from the mineral which is used as a fuel in nuclear
innermost layer of cortex. sun can be spread by power stations. It is an ore with a
Xylem and phloem both contain fibres. (a) a shower of rain drops chemical composition that is largely
Tracheids are the part of xylem whereas (b) a plane mirror UO 2, but due to oxidation the mineral
sieve tubes are found in phloem. (c) a convex lens typically contains variable proportions of
(d) a combination of a convex lens and a U 3O 8.
54. Which one of the following organisms concave lens
has vascular tissues? 61. Which one of the following is not a
(a) Cladophora (b) Penicillium Ê (a) The light energy escaping from the synthetic detergent?
sun can be spread by a shower of rain
(c) Marsilea (d) Anabaena
drops. The sunlight shines on a water (a) CH 3(CH 2 )10CH 2OSO −3Na +

Ê (c) Marsilea is pteridophytic plant, which droplet. (b) [CH 3(CH 2 )15 N (CH 3 )3 ] + Br −
has primitive vascular tissues. Cladophora (c) CH 3(CH 2 )16COO −Na +
As the light passes into the droplet, the
is an algae, Penicillium is a fungi.
light bends or refracts, a little because (d) CH 3(CH 2 )16COO(CH 2CH 2O)n ⋅
Anabaena is Blue-Green Algae (BGA).
light travels slower in water than in air CH 2CH 2OH
55. Which one of the following (because water is denser). Then, the light
Ê (d) Synthetic detergents are chemically
organisms represents the primary bounces off the back of water droplet
alkyl sulphate or sulphonate or
consumer category in an and goes back the way it came, bending
ammonium salt of long chain fatty acids.
again as its speeds up when it exist the
ecosystem? Thus, CH 3(CH 2 )16COO(CH 2CH 2O)n ⋅
water droplet.
(a) Caterpillar (b) Crabapple tree CH 2CH 2OH, option (d) is not a synthetic
Hence, the light energy escaping from detergent, while the remaining options
(c) Frog (d) Sparrowhawk
the sun can be spread. A disperse the contain synthetic detergents.
Ê (a) Caterpillar is primary consumer in an light into VIBGYOR and spread it.
ecosystem. It is because it feeds on 62. Which one of the following is an
leaves and is a herbivore. 59. The correct sequence of energy example of a clean fuel?
Crabapple trese is a producer. Frog is a transfer that occurs when an apple (a) Coke (b) Propane
secondary consumer. Sparrowhawk is a falls to the ground is (c) Petrol (d) Wax
top consumer.
NDA/NA Solved Paper 2019 (I) 19

Ê (b) Clean fuels are fuels that have a lower volume of air defined by its temperature wine region is a famous economic region
carbon intensity than the standard for the and water vapour content. Air masses of country for production of wine from
fuel it replaces. Examples of clean fuels cover many hundreds or thousands of grapes.
include ethanol, biodiesel, natural gas, miles, and adapt to the characteristic of
biogas, propane and hydrogen. the surface below them. They are 70. ‘Shamal’ warm and dry wind is a
classified according to latitude and their Local wind found in
63. Which one of the following metals (a) East Asia
continental or maritime source regions.
does not react with cold water? (b) West Coast of Africa
When air masses reaches a new region,
(a) Calcium (Ca) (b) Potassium (K) it might clash with another air mass that (c) Sahara of Africa
(c) Magnesium (Mg) (d) Sodium (Na) has a different temperature and humidty. (d) Mesopotamia
Ê (c) Magnesium (Mg) does not react with There are four type of air masses-Polar, Ê (d) ‘Shamal local’ winds are found in
cold water, but reacts with hot water, tropical, continental and maritime. Mesopotamia. Mesopotamia is a region
Mg + 2H 2O → Mg(OH)2 + H 2 ↑ covering Iraq, Iran and the Arabian
67. National Water Academy, a centre Peninsula. Shamal is a hot, dusty and dry
Mg + H 2O → MgO + H 2 ↑ of excellence in training and wind which blows from the North or
On the other hand, calcium (Ca), capacity building in water resource, North-West in Mesopotamia. Other
potassium (K) and sodium (Na) are react is located at important warm local winds are Chinook
with cold water.
(a) New Delhi (b) Kolkata in North America, Foehn in the Alps,
Thus, option (c) is correct. (c) Pune (d) Chennai Khamsin in Egypt, Sirocco in North Africa
64. In which of the following pairs are and Harmattan in West Africa.
Ê (c) National Water Academy (NWA), a
the isoelectronic ions? centre of excellence in training and 71. ‘Inversion of Rainfall’ is associated
(a) Mg 2 + , Ar (b) Na + , O 2− capacity building in water resource, is
with
(c) Al 3 + , Cl − (d) K + , Ne located in Pune (Maharashtra). It was
set-up under Central Water Commission (a) Orographic rainfall
Ê (b) According to question, by the Ministry of Water Resources in (b) Convectional rainfall
(c) Cyclonic rainfall (Tropical)
Ion Atomic Number of 1988.
(d) Cyclonic rainfall (Temperate)
Number Electrons The NWA is invisaged to function as
Mg 2 + 12 12 − 2 = 10 ‘Centre of Excellence’ for in-service Ê (a) ‘Inversion of Rainfall’ is associated
training of water resources engineering with orographic rainfall. The orographic
Ar 18 18 rainfall occurs due to the ascent of air
Na + 11 11 − 1 = 10 personnel.
forced by a mountain barrier. The
O 2− 8 8 + 2 = 10 It is addressing the wider training needs
mountain barrier should be across the
Al 3 + 13 13 − 3 = 10 of water resources engineers of State
wind direction, so that the moist air is
Cl − 17 17 + 1 = 18 and Central Agencies in the field of
forced in obstruction to move upward
K+ 19 19 − 1 = 18 planning, design, evaluation,
and get cooled. The amount of the
Ne 10 10 construction, operation and monitoring rainfall increases with increasing height
of water resources projects. of the barrier, but this is up to certain
The species containing same number of
electrons are isoelectronic. Thus, Na + 68. ‘Campos’ and ‘Llanos’, Tropical Savanna limit. After that there is a marked
and O 2− are isoelectronic species. grasslands are generally found in decrease due to lesser moisture content
Hence, option (b) is correct. of the air and this phenomenon is called
(a) Australia (b) Central Africa
‘Inversion of Rainfall’.
65. Which one of the following is used (c) South America (d) East Asia
as a binder in paints? Ê (c) ‘Campos’ and ‘Llanos’ are the tropical
72. Who was the author of the book
savanna grasslands found in South ‘History of British India’?
(a) Titanium dioxide (b) Novolac
(c) Phthalocyanine (d) Silicones America. Campos is located in Brazilian (a) Charles Grant (b) John Stuart Mill
Highlands and Llanos in Orinoco basin. (c) James Mill (d) William Jones
Ê (d) Binder refers to the substances that Tropical Grasslands are located near the
hold the particles of pigment together in
Equator, between the tropic of Cancer
Ê (c) James Mill (1773-1836) was the
paints. author of the book ‘History of British
and the tropic of Capricorn. They are India’. This book is a history of company
Silicones are used as a binder in paints.
also known as ‘Savannas’. It is a rule in India. Hence, option (c) is correct
They are synthetic organo-silicon polymers
transitional zone found between the
containing repeated R 2SiO units. 73. The Azamgarh Proclamation of 25th
equatorial rainforests and the hot
66. Which one of the following is NOT deserts. They includes savannas of August, 1857 stressed on which one
true in reference to Air mass? Africa and Campos and Llanos of South of the following issues?
America. (a) Hindu-Muslim divide
(a) Air mass forms either in tropical or in
polar region 69. ‘Viticulture’ is a common feature of (b) Support to the English government
(b) Air mass develops on continents as (c) The return of the Badshahi
which one of the following
well as over ocean (d) The imposition of heavy Jumas
(c) Air mass develops in a cyclonic
Australian cities? (revenue demand)
condition (a) Adelaide (b) Darwin
(c) Hobart (d) Brisbane Ê (a) The Azamgarh Proclamation of 25th
(d) Air mass changes the weather August, 1857 stressed on Hindu-Muslim
condition Ê (a) Viticulture is a common feature of divide. It was published in Delhi Gazette in
Ê (c) “Air mass develops in a cyclonic Adelaide, which is the capital of South the midst of the Sepoy Mutiny (1857). The
condition” this option are not true in Australia. Viticulture is the cultivation and author was most probably Firoz Shah
reference of air mass. Air mass is a harvesting of grapes. The Adelaide Hills (grandson of Bahadur Shah Zafar), who
20 NDA/NA Solved Paper 2019 (I)

fought against the British in Lucknow and Ê (b) Static electricity can charge an 82. Which one of the following statements
Awadh (Oudh). The original document insulator. Charging of insulator by is not correct for the given reaction?
was published in Urdu. induction process, static charge is Fe(s ) + CuSO 4 (aq )
required.
74. Which Viceroy had made the → FeSO 4 ( aq ) + Cu(s)
A static electric charge can be created
observation, ‘It’s a beautiful world if (a) Iron is the reducing agent
whenever two surfaces contact and
it wasn’t for Gandhi ...’? (b) The solution turns green in colour
separate at least one of the surfaces has
(a) Lord Irwin (b) Lord Wavell a high resistance to electric current. after the reaction
(c) Lord Mountbatten (d) Lord Willingdon (c) Copper is a more reactive metal than
Ê (d) Lord Willingdon was 22nd Viceroy of
78. At 20°C, the speed of sound in water iron
India from 1931 to 1936. He mentioned is approximately (d) The reaction is an example of a redox
reaction
Mahatma Gandhi in a letter written in (a) 330 m/s (b) 800 m/s
1933 by saying, ‘‘It’s a beautiful world if it (c) 1500 m/s (d) 5000 m/s Ê (c) A more reactive metal displaces a less
was not for Gandhi who is really a perfect (c) The speed of sound in water at 20°C is reactive metal from its solution.
nuisance.’’ Hence option (d) is corrrect. approximately 1500 m/s. Fe( s ) + CuSO 4 ( aq ) → FeSO 4 ( aq )
The important events of his period +Cu( s )
include Second and Third Round Table 79. Which one of the following could be In the above given reaction, iron (Fe)
Conferences, relaunching of Civil the melting point of iron? displaces copper (Cu) from its solution
Disobedience Movement, Communal (a) 25°C (b) 37°C
(CuSO 4 ), thus copper is less reactive
Award by Ramsay MacDonald (1932), (c) 500°C (d) 1500°C
than iron. Therefore, option (c) is
Poona Pact (1932), the Government of (d) Iron (Fe) is a chemical element with incorrect, while the remaining options are
India Act, 1935 and Separation of Burma atomic number 26. It is a metal that correct.
from India. belongs to the first transition series and
group 8 of the periodic table. Its melting 83. Which one of the following is an
75. Which Indian businessman favoured point is 1538°C (~1500°C) or 1811 K. organic acid?
‘healthy capitalism’ in helping Gandhiji (a) Hydrochloric acid (b) Nitric acid
to work towards a ‘common object’? 80. Let us consider a copper wire (c) Acetic acid (d) Sulphuric acid
(a) Ghanshyam Das Birla having radius r and length l. Let its
resistance be R. If the radius of Ê (c) An organic acid is an organic
(b) Ambalal Sarabhai compound with acidic properties. The
(c) Sir Biren Mukherjee another copper wire is 2r and the most common organic acids are the
(d) TT Krishnamachari length is l/2, then the resistance of carboxylic acids. Among the given
this wire will be options, acetic acid (CH 3COOH) is an
Ê (a) Ghanshyam Das Birla, an Indian (a) R (b) 2 R organic acid. The remaining options
businessman favoured ‘healthy contains inorganic acids.
(c) R/4 (d) R/8
capitalism’ in helping Gandhiji to work
towards a ‘common object.’ Birla was a Ê (d) Given, length of copper wire = l and 84. Dinitrogen (N2 ) and dioxygen (O 2 )
close associate and a steady supported radius of copper wire = r are the main constituents of air, but
of Mahatma Gandhi, whom he met for ∴Resistance, they do not react with each other to
the first time in 1916. Gandhiji stayed at l l form oxides of nitrogen because,
R = ρ = ρ ⋅ 2 (Q A = πr 2 )…(i)
Birla’s home in New Delhi during the last A πr (a) the reaction requires initiation by a
four months of his life. In 1957, GD Birla For second wire, catalyst
was awarded India’s second highest l (b) oxides of nitrogen are unstable
civilian honour, the Padma Vibhushan by length, l1 = and radius, r1 = 2 r
2 (c) the reaction is endothermic and
the Government of India. Resistance of wire, requires very high temperature
l l /2 (d) the stoichiometry of N 2 and O 2 in air
76. The art piece ‘In Memoriam’ was a R1 = ρ ⋅ 1 2 = ρ ⋅ is not ideal for the reaction to take
creation of which one of the πr1 π(2 r )2
place
following European painters? l 1 l
= ρ⋅ = ⋅ρ⋅ 2 Ê (c) The reaction of dinitrogen (N 2) and
(a) Thomas Jones Barker 2 × π ⋅ 4r 2 8 πr dioxygen (O 2) is endothermic and
(b) Joseph Noel Paton 1
= ⋅R [from Eq. (i)] requires very high temperature.
(c) Thomas Daniell 8
(d) Charles D’Oyly R N 2 + O 2 —→ 2NO
=
It is because, the nitrogen-nitrogen triple
Ê (b) The art piece ‘In Memoriam’ was a 8
creation of the European painter, Joseph bond (N ≡≡ N) is very strong.
Noel Paton. He was a Scottish artist,
81. Basic scientific principle behind a The energy cost to break that bond as
illustrator and sculptor. In 1844, Paton’s nuclear reactor is well as theO == O is not compensated by
first painting, Ruth Gleaning, was (a) nuclear fusion the formation of nitrogen-oxygen
exhibited at the Royal Scottish Academy. (b) controlled nuclear fusion (N == O) double bond.
(c) uncontrolled nuclear fission Thus, they do not react with each other.
77. Which one of the following can (d) controlled nuclear fission
charge an insulator? (d) Basic scientific principle behind a 85. Who among the following has
(a) Current electricity nuclear reactor is controlled nuclear explained the phenomenon of
(b) Static electricity fission, in which a heavy atomic nucleus photoelectric effect?
(c) Magnetic field splits into two smaller nuclei and a large (a) Max Planck (b) Albert Einstein
(d) Gravitational field amount of energy is released in this (c) Neils Bohr
process. (d) Ernest Rutherford
NDA/NA Solved Paper 2019 (I) 21

Ê (b) The experiment of photoelectric effect 90. ‘Mission Indradhanush’ is related to Directions (Q. Nos. 93-95) The
is performed by Heinrich Hertz, but the (a) Bullet train project following questions items consist of two
phenomenon was explained by Albert (b) Agriculture development statements, statement I and statement
Einstein. It is the phenomenon of ejection (c) Women empowerment II. Examine these two statements
of electrons from the structure of a metal (d) Full immunisation carefully and select the correct answer
when light of suitable frequency strikes it.
Ê (d) ‘Mission Indradhanush’ is related to using the codes given below.
86. The equivalent weight of oxalic acid full immunisation. This health mission Codes
in C 2H2O 4 ⋅ 2H2O is was launched by the Ministry of Health (a) Both the statements are individually
and Family Welfare on 25th December, true and statement II is the correct
(a) 45 (b) 63 2014. explanation of statement I.
(c) 90 (d) 126
The scheme seeks to drive towards 90% (b) Both the statements are individually
Ê (b) Equivalent weight of an acid full immunisation coverage of India and true and statement II is not the correct
Molecular weight sustain same by year 2020. Vaccination explanation of statement I.
=
Basicity is being provided against eight vaccine (c) Statement I is true, but statement II is
Molecular weight of C 2H 2O 4 ⋅ 2H 2O preventable diseases, i.e. false.
Diphtheria, Pertussis, Tetanus, Polio, (d) Statement I is false, but statement II is
= 2 × 12 + 2 × 1 + 4 × 16 + 2 × 18 = 126
Measles, Tuberculosis, Hepatitis B, true.
∴ Equivalent weight of C 2H 2O 4 ⋅ 2H 2O
Meningitis, Rotavirus, Diarrhea and 93. Statement I Abul Fazl shaped,
126
= = 63 Japanese Encephalitis. represented and articulated the
2
91. Which of the following is/are ideas associated with the reign of
87. Which one of the following is not a environmental effects of Rotation of Akbar.
west flowing river? the Earth? Statement II The qualities of Abul
(a) Periyar (b) Bharathapuzha
1. Daily or diurnal rhythm in Fazl impressed Akbar who found
(c) Pamba (d) Tamraparni the former suitable as an adviser
day-light and air temperature
Ê (d) Tamraparni is not a West flowing river and spokesperson for his policies.
while others are west flowing rivers.
2. Flow path of both air and water
are turned consistently in a Ê (a) Abul Fazl shaped, represented and
Tamraparni river originates from Pothigai articulated the ideas associated with the
hills of Western Ghats (Tamil Nadu). It sideward direction.
reign of Akbar because the qualities of
flows through Tamil Nadu and falls into 3. The movement of the tides Abul Fazl impressed Akbar, who found
Gulf of Mannar. It has been historically Select the correct answer using the the former suitable as an adviser and
Known as ‘Podhigai’. It is mentioned in codes given below: spokesperson for his policies.
the ancient Sangam and Tamil texts. (a) 1 and 2 (b) 1 and 3 So, Both the given statements are
88. Which one of the following rivers (c) 1, 2 and 3 (d) Only 3 individually true and statement II is the
was earlier known as ‘Vitasta’? Ê (c) Statement 1, 2 and 3 all are correct. correct explanation of statement I.
(a) Tista (b) Jhelum The rotation of the earth causes day and
night. This results in the diurnal changes
94. Statement I The Kisan Manifesto
(c) Tungabhadra (d) Bharathapuzha adopted by the All India Kisan Sabha
in day-light and air temperature. So,
Ê (b) ‘Jhelum river’ was earlier known as statement 1 is correct.
in August, 1936 contained radical
‘Vitasta’, while Tista was known as demands.
‘Trisrota’, Tungabhadra as ‘Pampa’ and Earth’s rotation results in the Coriolis
effect. This deflects winds and streams of
Statement II The All India Kisan
Bharathapuzha as ‘Pratichi’. Sabha was a part of the Congress
Jhelum river rises from northern slopes water to the right in Northern Hemisphere
and to the left in Southern Hemisphere. and maintained close relationship
of Pir Panjal ranges at Verinag Spring. It
So, statement 2 is correct. with the Provincial Congress
flows through Srinagar and Wular lake
Tides are caused by the gravitational
Committees.
before entering Pakistan. It is a tributary
of the Indus river. attractive forces of the moon and the sun Ê (d) The kisan manifesto released in
as well as the centrifugal force due to the August 1936, demanded abolition of the
89. River Sharda drains in the Northern Earth’s rotation. So, statement 3 is also zamindari system and cancellation of
Plains of Uttar Pradesh. Before entering correct. rural debts.
in the Plains, Sharda is known as All India Kisan Sabha was separate
(a) Saraswati (b) Bhagirathi
92. Who among the following entity. It was founded at Lucknow in 1936
(c) Kali (d) Pindar historians have described the Quit with Swami Sahajanand as its first
India Movement as a ‘spontaneous President and NG Ranga as its General
Ê (c) The Sharda river originates from the revolution’?
greater Himalayas at Kalapaani in the Secretary. So, statement I is false, but
Pithoragarh district of Uttarakhand. The (a) Gordon Johnson statement II is true. Hence, option (d) is
river is named after the Goddess Kali (b) David Arnold correct.
whose temple is situated in Kalapaani. It (c) FG Hutchins
(d) Peter Robb 95. Statement I The British ruled India
drains the Northern plains of Uttar
through a modern bureaucracy
Pradesh and Uttarakhand. It is known as Ê (c) FG Hutchins described the Quit India headed by the Indian Civil Service,
‘River Sharda’, when it reaches the plains Movement as a ‘spontaneous
of Uttarakhand and Uttar Pradesh. It revolution’. Quit India Movement, 1942 whose members were recruited
forms India’s continous border between was an important event of the Indian through merit based on open
India and Nepal. freedom struggle. competition.
22 NDA/NA Solved Paper 2019 (I)

Statement II The Indian Civil 3. The magnetic field strength in a Work done by non-conservative forces
Service was based on the whole solenoid depends upon the depends on the path of the body moving
hearted participation of Indians. diameter of the solenoid. from one point to another point.
Therefore, frictional force is a non-central
Ê (c) The British ruled India through a Which of the statements given
modern bureaucracy headed by the and non-conservative force.
above are correct?
Indian Civil Service, whose members (a) 1, 2 and 3 (b) 1 and 3 101. On exposure to moist air, copper
were recruited through merit based on (c) 2 and 3 (d) 1 and 2 gains a green coat on its surface due
open competition. Hence, statement (1)
Ê (d) Magnetic field strength in a solenoid is to formation of which one of the
is correct.
given by following compounds?
The Indian Civil Services was not based
B = µ 0ni (a) Copper carbonate
on the whole hearted participation of (b) Copper oxide
Indians. Hence, statement (2) is not where, µ 0 = permeability of free space,
(c) Copper sulphate
currect. So, the statement I is true. But, n = number of turns per unit length (d) Copper nitrate
the statement II is not true. Thus, and i = current flowing through solenoid.
option(c) is correct here.
Ê (a) On exposure to moist air, copper gains
So, it is clear that magnetic field strength a green coat on its surface due to
96. Two metallic wires A and B are B in a solenoid depends on n and i only, it formation of copper carbonate.
does not depends on diameter of
made using copper. The radius of 102. Which one of the following will not
solenoid. Therefore, statements 1 and 2
wire A is r while its length is l. A DC are correct. produce carbon dioxide on reacting
voltage V is applied across the wire with an aqueous solution of
A, causing power dissipation P. The 98. Light year is a unit of measurement hydrochloric acid?
radius of wire B is 2r and its length is of (a) Limestone (b) Quicklime
2l and the same DC voltage V is (a) very large distances (c) Chalk (d) Marble
applied across it causing power (b) time interval in years
(c) amount of light received on earth in a
Ê (b) Calcium carbonate occurs in nature in
dissipation P1. Which one of the several form like limestone,chalk,
year
following is the correct relationship marble, etc. It reacts with aqueous
(d) mass of atoms
between P and P1? solution of hydrochloric acid (HCl) to
(a) P = 2 P1 (b) P = P1/2 Ê (a) Light year is a unit of measurement of liberate carbon dioxide.
very large distances. CaCO 3 + 2HCl → CaCl 2 + H 2O
(c) P = 4P1 (d) P = P1
It is a distance travelled by light in
+ CO 2 ↑
Ê (b) For metallic wire A, vacuum in one year.
l 1 light year = 9.46 × 1015 m Quicklime on reaction with HCl does not
resistance, R A = ρ ⋅
πr 2 liberate CO 2.
Therefore, power dissipation in wire A,
99. The focal length of the objective CaO + 2HCl → CaCl 2 + H 2O
lens of a telescope is 50 cm. If the Quicklime
when a DC voltage V is applied,
magnification of the telescope is 103. Which one of the following
V2 V2 V 2 ⋅ πr 2
P= = = 25, then the focal length of the substances is not a mixture?
RA ρ ⋅ l ρl eyepiece is (a) Ice (b) Ice-cream
πr 2 (a) 12.5 cm (b) 5 cm (c) Air (d) Honey
π r 2V 2
P= …(i) (c) 2 cm (d) 10 cm Ê (a) Mixtures are obtained by mixing two or
ρl more substances in any proportions.
Ê (c) Given, focal length of objective lens,
For metallic wire B, fo = 50 cm and magnification of Thus, among the given options, ice is not
l B = 2 l, rB = 2 r, then resistance of wire B, telescope m = 25 a mixture as it contains only water
l 2l 2l (liquid). The other substances, i.e.,
RB = ρ ⋅ B 2 = ρ ⋅ = ρ⋅ If fe be the focal length of eyepiece, then
ice-cream, air and honey are mixtures.
πrB π(2 r )2 4 πr 2 m=−o
f

∴Power dissipation in wire B,


fe 104. Which one of the following is an
⇒ 25 = −
50 example of Salt-Crystal growth?
V2 V2 2 π r 2V 2
P1 = = = = 2P fe (a) Chemical weathering
RB ρ⋅2l ρl 50 (b) Physical weathering
4 πr 2 ⇒ fe = − = −2 cm
[from Eq. (i)] 25 (c) Biological weathering
P1 (d) Bio-chemical weathering
∴ P= | fe| = 2 cm
2 Ê (b) Physical weathering is an example of
100. Which one of the following force is Salt-Crystal growth. Physical weathering
97. Consider the following statements non-central and non-conservative? is a term used in science that refers to the
about a solenoid. (a) Frictional force geological process of rocks breaking
1. The magnetic field strength in a (b) Electric force apart without changing their chemical
solenoid depends upon the (c) Gravitational force composition. Over time, movements of
the earth and environment can break
number of turns per unit length (d) Mechanical force
apart rock formation, causing physical
in the solenoid. Ê (a) A force on body which does not weathering. It can also refers to other
2. The magnetic field strength in a always act towards a fixed point, is called things in the environment, breaking
solenoid depends upon the non-central force. down, like soil and mineral, pressure,
current flowing in the wire of the The forces that do not store energy are warm temperature, water and ice can
solenoid. called non-conservative forces. cause physical weathering.
NDA/NA Solved Paper 2019 (I) 23

105. Which one of the following is the and Falkland ocean currents. Hence, 111. The sun is seen little before it rises
correct sequence of proved coal option (c) is correct.
and for a short while after it sets.
reserves in the Indian States in 108. What is the time gap in occurrence This is because of
decreasing order? of two successive tides at a given (a) total internal reflection
(a) Jharkhand, Chhattisgarh, Odisha, place on the ocean surface? (b) atmospheric refraction
West Bengal
(a) 12 hr (b) 12 hr 26 min (c) apparent shift in the direction of sun
(b) Jharkhand, Odisha, Chhattisgarh,
(c) 24 hr (d) 24 hr 52 min (d) dispersion
West Bengal
(c) Odisha, West Bengal, Jharkhand, (b) A lunar day has a length of 24 hours Ê (b) The sun is seen little before it rises and
Chhattisgarh and 52 minutes. Earth rotates through for short while after it sets due to
(d) Odisha, Chhattisgarh, West Bengal, two tidal ‘bulges’ every lunar day. So, atmospheric refraction.
Jharkhand coastal areas experience two high and When light enters from vacuum to earth’s
two low tides in every 24 hours and 52
Ê (b) Coal is the main source of energy in atmosphere, it basically enters from rarer
India as it fulfils about 67% of the total minutes. There are two high tides occur in to denser medium and bends towards
commercial energy consumed in the the interval of 12 hours and 26 minutes. horizon.
country. Hence option (b) is correct answer. During sunrise, when the sun is just
The decreasing order of proved coal 109. Tooth enamel is made up of which one below the horizon, our atmosphere
reserve is given below causes the light rays to bend and we see
of the following calcium compounds?
State Proved Coal Reserve the sun early.
(a) Calcium carbonate
(million tonnes) (b) Calcium sulphate Similarly, at sunset, the apparent position
Jharkhand 45563 (c) Calcium hydroxide of the sun is visible to us and not the
(d) Calcium phosphate actual position due to the same bending
Odisha 37391
(d) Enamel is the hardest substance in of light rays effect.
Chhattisgrah 20428
the human body. It contains the highest 112. The figure shown below gives the
West Bengal 14156
percentage of minerals, 96% with water time (t ) versus position ( x ) graph of
Hence option (b) is correct answer. and organic material composing the rest.
three objects A , B and C. Which one
The primary mineral present in enamel is
106. Consider the following statements hydroxyapatite which is a crystalline
of the following is the correct
relating to Richter scale calcium phosphate. relation between their speeds v A , v B
1. It was devised in 1935 by Charles and v C , respectively at any instant
F Richter. 110. Suppose there are two planets, (t > 0)?
2. It describes the quantity of energy 1 and 2, having the same density, but
released by a single earthquake. their radii are R1 and R 2 respectively, Time C
3. Richter scale has no upper limit. where R1 > R 2 . The accelerations (t) B
due to gravity on the surface of A
Which of the statement(s) given
above is/are correct? these planets are related as
(a) Only 1 (b) 1 and 2 (a) g 1 > g 2 (b) g 1 < g 2 O Position (x)
(c) 2 and 3 (d) All of these (c) g 1 = g 2 (a) v A < v B < vC
(d) Cannot say anything > v B > vC
Ê (b) Richter scale is a scale of number (b) v A
used to indicate the magnitude of Ê (a) Acceleration due to gravity on the (c) v A = v B = vC ≠ 0
earthquakes. Charles Francis Richter surface of first planet, (d) v A = v B = vC = 0
developed the Richter scale in 1935. This GM1
scale describes the quantity of energy g1 = 2
…(i) Ê (b) From figure,
R1 t
released by earthquake. Numbers for the
Richter scale range from 0 to 9. So, here where, M1 is the mass of first planet. C2 C
statements (1) and (2) are correct. 4 B2 B
But, M1 = πR13 ⋅ ρ1
3 θC
107. Which one of the following ocean (Q mass = volume × density) A2 A
currents is not a cold ocean current? θB
where, ρ1 = density of the first planet.
(a) Canary current (b) California current
(c) Kuroshio current(d) Oyashio current
4
G ⋅ πR13 ⋅ ρ1 O θA C1 B1 A1 x
g1 = 3 [from Eq. (i)]
Ê (c) Ocean current is a continuous R1
2 Velocity of object A,
movement of ocean water from one OA1
4 vA = = cot θ A
place to another. It is created by wind, g 1 = Gπρ1R1 …(ii) AA1
water temperature, salt content and 3
gravitational force of Moon. It is of two Similarly, acceleration due to gravity on Velocity of object B,
types; cold and warm ocean currents. OB1
the surface of second planet, vB = = cot θ B
Examples of warm ocean current are 4 BB1
Kuroshio, Alaskan, El Nino, East g 2 = Gπρ2R 2 …(iii)
3 OC1
Australian, Florida, Gulf stream, Brazilian Velocity of object C, vC = = cot θC
∴From Eqs. (ii) and (iii), we get CC1
and Agulhas ocean currents. Examples
of cold ocean current are Humboldt or g1 ρR R Since, θC > θ B > θ A
= 1 1 = 1 (Q ρ1 = ρ2 )
Peruvian, Kuril or Oyashio, California, g 2 ρ 2R 2 R 2 ⇒ cot θ A > cot θ B > cot θC
Antarctica, Okhotsk, Labrador, Canary ⇒ v A > v B > vC
Since, R1 > R 2, hence g 1 > g 2.
24 NDA/NA Solved Paper 2019 (I)

113. 1 dyne (a unit of force in CGS the glass walls stops heat entering or 120. A fuse wire must be
system) equals to leaving the flask by conduction.
(a) conducting and of low melting point
(a) 10 3 g-cm/s 2 (b) 10 −3 g-cm/s 2 Hence, option (c) is not a correct (b) conducting and of high melting point
(c) 10 5 kg-cm/s 2 (d) 10 −5 kg-m/s 2 statement regarding a thermo flask. (c) insulator and of high melting point
(d) insulator and of low melting point
Ê (d) 1 dyne = 1 g-cm/s 116. Black hole is a
2

1g ⋅ 1cm 10 −3kg ⋅ 10 −2m (a) huge black star which has zero Ê (a) Fuse wire should have high resistance
= = and low melting point. It is because when
1s 2
1s 2 acceleration due to gravity on its
surface high current pass through the circuit, at
= 10 −5 kg-m/s 2 (b) star which has moderate acceleration first, it will resist the high current.
114. In the given velocity (v ) versus time due to gravity on its surface A fuse wire protects our electric
(c) star which has collapsed into itself appliances from damaging due to
(t ) graph, accelerated and decelerated
and has large acceleration due to sudden increase in electric current.
motions are respectively represented gravity on its surface
by line segments (d) star which has collapsed into itself 121. Kamarajar Port was commissioned
and has zero acceleration due to in 2001 to handle thermal coal
A D
gravity on its surface requirements. It is situated along
Velocity
(v) Ê (c) A black hole is a star which has the coast of which Indian State?
B C
collapsed into itself and has large (a) Andhra Pradesh (b) Odisha
Time (t) acceleration due to gravity on its surface. (c) Tamil Nadu (d) Karnataka
(a) CD and BC (b) BC and AB A black hole forms when the center of a (c) Kamarajar (Ennore) Port is situated
(c) CD and AB (d) AB and CD very massive star collapses in upon itself. along the coast of Tamil Nadu. It is the
The region in space of a black-hole, the 12th major port of India. This port was
Ê (c) The slope of velocity-time graph gives pulling force of gravity is, so strong that commissioned in 2001 to handle thermal
acceleration produced in the moving body. light is not able to escape. coal requirement.
A D
117. The formula for conversion 122. Which one of the following Union
Velocity
(v) B C
between Fahrenheit and Celsius is Territories of India is the smallest in
° F = X + (1.8 × ° C). terms of geographical area?
θ2 θ1 (a) 32 (b) 22 (a) Daman and Diu
Time (t) (c) 98 (d) 42 (b) Chandigarh
(c) Dadra and Nagar Haveli
From graph, slope of line segment Ê (a) The formula for conversion between
Fahrenheit and Celsius is (d) Lakshadweep
AB = tanθ 2
°C °F − 32 (d) Lakshadweep is the smallest Union
which is negative because =
5 9 Territories of India in terms of
90°< θ 2 < 180°. 9° geographical area. According to Census
Therefore , line segment AB represents ⇒ C = ° F − 32
5 2011, Lakshadweep has an area of 32
decelerated motion. ⇒ 1.8 °C = ° F − 32 km 2. It is an archipelago consisting of 36
Slope of line segment CD = tanθ1 ⇒ °F = 32 + 1.8 °C islands and having the capital at
which is positive because Comparing with given relation of Kavaratti. Among others, UTs Daman
0°< θ1 < 90°. °F = X + (1.8 × °C) and Diu has an area of 112 km 2 whereas
We get, X = 32 Chandigarh and Dadra and Nagar Haveli
Therefore, line segment CD represents
has an area of 114 km 2 and 491 km 2
accelerated motion.
118. When a beam of white light passes respectively.
115. Which one of the following through a glass prism, the colour of
light beam that deviates the least is 123. Which one of the following can be
statement regarding a thermo flask
(a) blue (b) red (c) green (d) violet said to be essentially related to
is not correct?
‘Polar Front Theory’?
(a) The walls of flask are separated by Ê (b) The wavelength of red colour of light is
maximum, therefore when a beam of white (a) Anticyclone
vacuum and made of glass which is a
light passes through a glass prism, then (b) Tropical Cyclone
poor conductor of heat. (c) Temperate Cyclone
(b) The glass walls themselves have the light of red colour deviates the least.
(d) Inter Tropical Convergence
shiny surfaces. 119. LIGO stands for Ê (c) The ‘Polar Front Theory’ of cyclogenesis
(c) The surface of inner wall radiates (a) Laser Interferometer Gravitational was developed to explain the formation
good amount of heat and the surface wave Observatory of mid-latitudes cyclone or Temperate
of outer wall absorbs some of the heat (b) Light Interferometer Gravitational cyclone. According to this theory, the
that is radiated from the inner wall. wave Observatory warm-humid air masses from the topics
(d) The cork supports are poor (c) Light Induced Gravity Observatory meet the dry-cold air masses from poles,
conductors of heat. (d) Laser Induced Gaseous Optics and thus a polar front is formed. The cold
Ê (c) The thermo flask has the silver coating Ê (a) LIGO stands for Laser Interferometer air mass pushes the warm air mass
on the inner surface which prevents heat Gravitational wave Observatory. It is a upwards, forming a low pressure
transfer by radiation. The vacuum large scale physics experiment and cyclonic system. A mid-latitude cyclone
between its double wall prevents heat observatory to detect cosmic gravitational is a large-scale low pressure system that
moving by convection. The thinness of waves and to develop gravitational waves is developed. Hence, option (b) is correct.
observations as an astronomical tool.
NDA/NA Solved Paper 2019 (I) 25

124. Brahmaputra and Indus rivers are Ê (c) The treaty of Schonbrunn (1809) or Select the correct answer using the
antecedent rivers. Which one of the treaty of Vienna was signed after the codes, given below.
Battle of Wagram. This treaty was (a) Only 1 (b) Only 2
following may be the true definition
signed between Austria and France. (c) Both 1 and 2 (d) Neither 1 nor 2
of an antecedent drainage?
Battle of Austerlitz (2nd Dec-1805), the
(a) Which follows the initial slope of the Ê (b) On 4th Jan, 1948 Burma became an
first engagement of the war of the third independent republic, under the terms of
Himalaya coalition and one of Napoleon’s greatest
(b) Which existed before the Himalayan the Burma Independence Act, 1947. The
victories. new country was named the Union of
range came into existence
(c) Which followed the dip or rock beds Battle of Tilsit fought between French Buma. A pant from this, in 1948, burma
of the Himalaya and Russia on 14th June, 1807. joired the United Nations but refused to
(d) Which followed the strikes of rock The battle of Lisbon (1st July to 25th denounce China as the aggressor in the
beds of the Himalaya Oct, 1147) was the military action that Korean war. Hence option (b) is correct
(b) An antecedent drainage is one whose brought the city of Lisbon under definitive answer.
path of flow neithin a valley was portuguese control and explelled its
established before the mountainous Moorish overloads. 130. Who among the following
structure was uplifted. Antecedent presented ‘The April Thesis’ to the
127. Which of the following statements
drainage existed before the Himalayan about the New Model Unions is/are Russian people in 1917?
range came into existence. The rivers (a) Stalin (b) Trotsky
correct?
that existed before the upheaval of the (c) Bukharin (d) Lenin
Himalayas and cut their courses 1. The New Model Unions were
southward by making gorges in the formed in the 1850s. Ê (d) Vladimir Lenin presented ‘The April
Thesis’ to the Russian people in 1917. It
mountains are known as the antecedent 2. The New Model Unions were
was the series of ten directives issued by
rivers. The Indus, Sutlej, Ganga, Kali, formed in the 1880s.
Arun, Tista and Brahmaputra are some of Lenin. It was aimed at fellow Bolsheviks
3. The New Model Unions comprised in Russia and returning to Russia from
the important antecedent rivers originated a Labour Party idea.
from beyond the Greater Himalayas. exile. The April Thesis influenced the
4. The New Model Unions excluded October Revolution.
125. The Karachi Resolution of Congress women in the 1920s.
in 1931 advocated which one of the Select the correct answer using the 131. An Election Commissioner can be
following issues? codes given below. removed from office on the
(a) State shall not own or control key (a) Only 1 (b) Only 2 recommendation of
industries and services. (c) 3 and 4 (d) Only 3 (a) the Chief Justice of India
(b) State shall handover the key (b) the Chief Election Commissioner
Ê (a) New Model Union or New Model (c) the President of India
industries and services to the Indian Trade Union (NMTU) were a variety of
business groups. Trade Union prominent in the United (d) the Parliament
(c) State should allow the Indian business Kingdom. These were formed in the Ê (b) An Election Commissioner can be
group to invest 50% of the capital. 1850s. These were generally tended to removed from office on the
(d) State shall own or control key be restricated to individual trades. These recommendation of the Chief Election
industries and services. union were not comprised by a Labour Commissioner (CEC) by President of
Ê (d) The Karachi Resolution of Congress Party idea and it also did not excluded India. Article-324 (5) of the Constitution
in 1931 advocated that, ‘State shall own women. So, here is only statement (1) is safeguards the CEC from arbitrary
or control key industries and services’. correct. Hence, option (a) is correct.
removal. CEC can be removed only by
The Karachi Resolution was passed by 128. The Truman Doctrine of 1947 was the President, just like Judge of the
the INC at its Karchi Session in 1931. announced to achieve which one of Supreme Court.
The session was conducted in the the following?
shadow of three major events— 132. Which one of the following
(a) Containment of the USSR
First, Mahatma Gandhi had just been
statements regarding the Rajya
(b) Increasing agricultural production in
released from prison following his Salt the USA Sabha is not correct?
Satyagraha. (c) Offering friendship to Europe (a) Its members are elected by the
Second, the Gandhi-Irwin pact had just (d) Strengthening the UNO elected members of the Legislative
been concluded which had brought the Assembly of a State.
Ê (a) The Truman Doctrine of 1947 was (b) The election follows the system of
Civil Disobedience Movement to an end. announced to achieve containment of
And third, the British Government had, a the USSR. It was announced by Harry S proportional representation by
week before the session, executed Truman on 12th March, 1947 and further means of a single transferable vote.
Bhagat Singh and two of his associates developed on 12th March, 1948. (c) 1/3rd of its members retire after every
in connection with the kakori Conspiracy two years.
Case.
129. Which of the following statements (d) It is a permanent body, but can be
is/are correct? dissolved earlier by the President.
126. The treaty of Schonbrunn (1809) 1. In 1948, Burma was admitted to
was signed after which one of the Ê (d) Option (d) is incorrect because Rajya
the United Nations and immediately Sabha (Article-80) is a permanent body
following battles? supported the USA in the Cold War. and not the subject to dissolution. Its
(a) Battle of Austerlitz 2. In 1948, Burma joined the United maximum strength is 250. Its members
(b) Battle of Tilsit Nations, but refused to denounce are elected by the elected member of the
(c) Battle of Wagram China as the aggressor in the Legislative Assembly of the State.
(d) Battle of Lisbon Korean War.
26 NDA/NA Solved Paper 2019 (I)

The election of Rajya Sabha members However, it is not correctly considered as 138. Who among the following was
follows the system of proportional type of marriage because it does not awarded The Hindu Prize in Fiction
representation by means of a single have the consent of the parents. category for the year 2018?
transferable vote. One-third (1/3rd) of its According to Manusmriti there are eight
(a) Neelum Saran Gour
members retire after every two years. types of Hindu marriages. These are -
(b) N. Kalyan Raman
Brahma, Daiva, Arsha, Prajapatya, (c) Manoranjan Byapari
133. Fundamental Rights guaranteed under Gandharva, Asura, Rakshasa and
which one of the following Articles (d) Arunav Sinha
Paischaca.
of the Constitution of India is Ê (a) Neelum Saran Gour was awarded The
available only to the citizens of India? 136. Consider the following description Hindu Prize in Fiction category for the
of the Samadhi of a former Prime year 2018. Neelum Gour got this award
(a) Article-19 (b) Article-20 for her novel ‘Requiem in Raga Janki’.
(c) Article-21 (d) Article-22 Minister of India.
Central Samadhi Platform She is the author of five novels, four
Ê (a) Under Article-19 of the Constitution of comprises nine square black collection of short stories and one work of
India, Fundamental Rights are literary non-fiction. She has also edited a
guaranteed only to the citizens of India. polished granite solid stone blocks,
pictorial volume on the history and
There are five fundamental articles which capped with a ‘Diya’ in the centre.
culture of Allahabad.
are only to Indian citizens i.e. Articles-15, The number nine holds significance
16, 19, 29, 30. This article guarantees the and represents the Navarasas, 139. Who among the following was
six rights. These are Navaratras and Navagrahas. Then appointed as Head of the seven
1. Right to freedom of speech and placement of the Nine-Square Members committee to look into
expression Samadhi is in a circular lotus shaped revenue shortfall being faced by the
2. Right to assemble peacefully and pattern. The Nine-Square platform States after the GST rollout in India
without arms. is accessed in four cardinal and suggest steps for augmenting
3. Right to form associations or directions by pathways made in collections?
unions or co-operations. white composite tiles so that the (a) Himanta Biswa Sharma
4. Right to move freely throughout the
floor does not get heated. (b) Thomas Isaac
territory of India. Identify the Samadhi. (c) Sushil Modi
(d) Capt. Abhimanyu
5. Right to reside and settle in any (a) Shakti Sthal (b) Shantivan
part of the territory of India. (c) Sadaiv Atal (d) Veer Bhumi Ê (c) Sushil Modi was appointed as Head
6. Right to practice any profession or of the seven members committee to look
Ê (c) The Samadhi, which is mentioned in into revenue shortfall being faced by the
to carry on any occupation, trade above question, is Sadaiv Atal. It is the
or business. States after the GST rollout in India and
crematorium (Samadhi) of former Prime
suggest steps for augmenting collections
While Article 20 deals with the protection Minister of India, Atal Bihari Vajpayee
by the States, since July, 2017.
of certain rights incase of conviction for (1924-2018). He served three terms as
offences. Article 21 deals with the the Prime Minister of India. He was the As per the term of reference of the Group
protection of life and personal liberty, and first Indian Prime Minister who was not a of Ministers, the Modi led committee will
Article 22 grants protection to persons member of the Indian National Congress undertake data analysis using
who are arrested or detained. party to have served a full five year term econometric and statistical tools and
in office. suggest suitable measures for course
134. The treaty of Yandabo was signed in Shakti Sthal is a memorial of former correction for revenue augmentation.
(a) 1826 (b) 1825 Prime Minister Indira Gandhi.
(c) 1824 (d) 1823
140. The 15th Pravasi Bharatiya Divas,
Shantivan is the Samadhi or Cremation 2019 was held in
Ê (a) The treaty of Yandabo was signed on spot of Prime Minister Jawaharlal Nehru (a) New Delhi (b) Gandhi Nagar
24th February, 1826 between the and the Veer Bhumi is a memorial (c) Prayagraj (d) Varanasi
Britishers and Burmese. It brought peace dedicated to the former Prime Minister
between East India Company and the Rajiv Gandhi. Ê (d) The 15th Pravasi Bharatiya Divas,
king of Ava that ended the first 2019 was held in Varanasi (Uttar Pradesh)
Anglo-Burmese war. This treaty started 137. IMBEX is a joint exercise conducted from 21st to 23rd January, 2019. It was
the British Rule in Assam. regularly between Armies of India inaugurated by the Prime Minister
and Narendra Modi and Prime Minister of
135. In the Manusmriti which form of (a) Malaysia (b) Maldives Mauritius, Pravind Jugnauth.
marriage results from the “Voluntary (c) Mauritius (d) Myanmar The theme of this Pravasi Bharatiya Divas
union of a maiden and her lover”? was ‘Role of Indian Diaspora in building a
Ê (d) India-Myanmar Bilateral Army
new India’. On 9th January, 2020, 16th
(a) Eighth form Exercise (IMBEX) is a joint exercise
(b) Fifth form conducted regularly between Armies of Pravasi Bhartiya Divas was celebrated.
(c) Seventh form India and Myanmar. The first edition of
(d) Sixth form 141. Which one of the following cities
this exercise was held in 2017 and 2nd
was named by UNESCO as World
Ê (b) “Voluntary union of maiden and her edition was conducted on 14th January
Capital of Architecture for 2020?
lover” are the fifth form (Gandharava 2019 in Haryana.
(a) Tokyo
Marriage) of marriage result in the This joint exercise is also the first of its
(b) Johannesburg
Manusmriti. In this marriage a groom and kind military training exercise on United
(c) Rio-de-Janerio
his bride will wed without their parents Nations Peacekeeping Operations
(d) New Delhi
knowledge. between India and Myanmar.
NDA/NA Solved Paper 2019 (I) 27

Ê (c) Rio-de-Janerio (Brazil) was named by 145. Who among the following was 148. Which one of the following statements
UNESCO as World Capital of Architecture posthumously conferred with the regarding penicillin is correct?
for 2020. Rio-de-Janerio will be hosting Ashoka Chakra, India’s highest (a) Penicillin resistant bacteria can store
the World Congress of the International
peacetime gallantry award, in 2019? this antibiotic in vacuole
Union of Architects in July, 2020.
(a) Jyoti Prakash Nirala (b) Penicillin resistant bacteria can
142. Who among the following was (b) Nazir Ahmad Wani degrade this antibiotic by an enzyme
named ICC's emerging player of the (c) Hangpan Dada called β-lactamase
year 2018? (d) Mohan Nath Goswami (c) Penicillin resistant bacteria can
degrade this antibiotic by an enzyme
(a) Rishabh Pant (b) Josh Hazlewood Ê (b) Lance Naik Nazir Ahmad Wani was called lactic acid dehydrogenase
(c) Hasan Ali posthumously conferred with the Ashoka
(d) Mustafizur Rahman Chakra, India’s Highest Peacetime (d) Penicillin is not absorbed by bacteria,
Gallantry Award, in 2019. He is the first so most bacteria are resistant
Ê (a) India's fast-rising wicket keeper
batsman Rishabh Pant was on Tuesday recipient of the Ashoka Chakra award Ê (b) Statement (b) regarding penicillin is
named the International Cricket Counils from Kashmir region of the Indian State correct. The bacteria which produce
emerging player of the year in the world Jammu and Kashmir. He was from enzyme β-lactamase can degrade
body's annuel a wards. The 21 year old Jammu and Kashmir Light Infantry of the penicillin. Most of the bacteria are
Pant was picked by the ICC's voting Indian Army. sensitive for penicillin as it interferes cell
academy for a stellar show in his break wall synthesis.
146. As per the code of the nomenclature,
through test you in 2018. 149. Which one of the following
which one of the following is the correct
143. India’s first private sector Howitzer way of writing a biological name? organelles of mammalian cell is rich
gun-making unit is located at (a) Amoeba Proteus in hydrolytic enzymes?
(a) Jamshedpur (b) Kolkata (b) Amoeba proteus (a) Mitochondria (b) Ribosomes
(c) Hazira (d) Gwalior (c) amoeba proteus (c) Lysosome (d) Nucleus
(d) Amoeba Proteus
Ê (c) India’s first private sector Howitzer Ê (c) Cell organelle lysosome is rich in
gun-making unit is located at Hazira in Ê (b) Name written in option (b), i.e. hydrolytic enzymes. These enzymes
Surat (Gujarat). It was inaugurated by Amoeba proteus is the correct way of degrade the cell debris and recycle its
the Prime Minister Narendra Modi on writing biological name of Amoeba. components within the cells.
19th January, 2019 as India’s first The first name represents genus and its Mitochondria is powerhouse of the cell.
Armoured System Complex (ASC). It is first letter is capital. The second name Ribosomes are called protein factory.
built by a private company-Larsen and represents species and its first letter is Nucleus is referred as head of the cell
Toubro (L & T). small. Both names are underlined which controls all cellular activities.
separately when hand written and written 150. Which one of the following
144. Which of the following pairs of old in Italics when typed.
names and new names of islands in statements regarding cholera is
India is/are correctly matched? 147. Which one of the following statements correct?
1. Ross Island-Shaheed Dweep regarding electrocardiogram is correct? (a) Cholera is a disease that causes loss
(a) Electrocardiogram is graphical of memory
2. Neil Island-Netaji Subhas (b) Cholera is a disease of muscles due
representation of electrical activity of
Chandra Bose Dweep cornea to consumption of alcohol
(c) Cholera is a disease due to the
3. Havelock Island-Swaraj Dweep (b) Electrocardiogram is graphical
consumption of contaminated food
representation of activity of kidney
Select the correct answer using the or water
(c) Electrocardiogram is graphical (d) Cholera is a genetic disease
codes given below.
representation of activity of brain
(a) 1, 2 and 3 (b) 2 and 3 (d) Electrocardiogram is graphical Ê (c) Statement (c) regarding cholera
(c) 1 and 2 (d) Only 3 representation of electrical activity of is correct. It is a waterborne disease
Ê (d) Ross, Neil and Havelock Islands have heart and also spread through contaminated
been renamed on 75th anniversary of food. It is caused by bacterium Vibrio
Ê (d) Statement (d) is correct regarding cholerae.
freedom fighter Subhash Chandra Bose. electrocardiography. Electrocardiogram
The new names of these islands are as is graphical representation of electrical Cholera does not cause loss of memory.
follows: activity of heart. The consumption of alcohol does not
Ross Island — Netaji Subhash Chandra cause cholera and it does not affect the
Electrodes are placed on patient’s limbs
Bose Dweep functioning of muscles. This is also not a
and on the surface of the chest and
hereditary disease.
Neil island — Shaheed Dweep connected to a machine called electrocar-
diograph. It detects any abnormality in Hence, statement (c) is correct, resl of
Havelock Island — Swaraj Dweep
the functioning of the heart. the steta are incorrect.
Hence option (d) is correct.
NDA /NA
National Defence Academy/Naval Academy

SOLVED PAPER 2018 (II)


PAPER I : Mathematics
1. What is the value of log 7 log 7 x 5. IfA, B and C are subsets of a
⇒ − 1< − <1
5 universal set, then which one of the
7 7 7 equal to ? ⇒ −2< − < 0
x following is not correct?
(a) 3 log 2 7 (b) 1 − 3 log 2 7 5 (a) A ∪ ( B ∩ C ) = ( A ∪ B) ∩ ( A ∪ C )
7 ⇒ − 10 < − x < 0 (b) A ′∪( A ∪ B) = ( B′∩ A )′ ∪ A
(c) 1 − 3 log 7 2 (d)
8 ⇒ 10 > x > 0 (c) A ′∪( B ∪ C ) = (C ′∩ B)′ ∩ A ′
(d) ( A ∩ B) ∪ C = ( A ∪ C ) ∩ ( B ∪ C )
Ê (c) We have, 3. Consider the following expressions
1 1 1 7
1 where A ′ is the complement of A
7 7 7 = 72 ⋅ 74 ⋅ 78 = 78 1. x + x 2 −
x Ê (c) Let A, B and C are subsets of a
Now, log 7 log 7 7 7 7 universal set.
d e
7 2. ax 2 + bx + x − c + − Let A = { 1 } , B = { 2 } , C = { 3 }
= log 7 log 7 (7 )8 x x2
∪ = {1, 2, 3} , A′ = {2, 3}, B′ = {1, 3},
3. 3x 2 − 5x + ab C ′ = {1, 2}
= log 7  
7
 8 2 by checking options, we get
4. 2
= log 7 7 − log 7 8 x − ax + b 3 LHS = A ′∪ ( B ∪ C )
m 1 2 = { 2, 3 } ∪ { }
[Qlog = log m − log n] 5. −
n x x +5 = {2, 3}
= log 7 7 − log 7 2 3 = 1 − 3 log 7 2 Which of the above are rational RHS = (C ′ ∩ B)′ ∩ A ′
[Qlog b a n = n log b a ] expressions? = ( { 1, 2 } ∩ {1, 3} ) ∩ { 2, 3 }
(a) 1, 4 and 5 (b) 1, 3, 4 and 5 = ( { 2 } )′ ∩ { 2, 3 }
2. If an infinite GP has the first term x (c) 2, 4 and 5 (d) 1 and 2 = { 1, 3 } ∩ { 2, 3 }
and the sum 5, then which one of the
Ê (a) We know that, rational expressions = { 3}
following is correct? are those expression which can be write LHS ≠ RHS
(a) x < −10 p( x )
(b) −10 < x < 0 in the form of , q ( x) ≠ 0 So, option (c) is wrong
q ( x)
(c) 0 < x < 10
(d) x > 10 So, 1, 4, 5 are rational expressions 6. Let x be the number of integers
lying between 2999 and 8001 which
Ê (c) Given that first term of an infinity GP is 4. A square matrix A is called have at least two digits equal.Then x
x and sum = 5 orthogonal if is equal to
x
∴ =5 (a) A = A 2 (b) A ′ = A −1 (a) 2480 (b) 2481
1− r (c) A = A −1 (d) A = A ′ (c) 2482 (d) 2483
a
[Q sum of infinity GP = ] where A′ is the transpose of A
Ê (b) We have, x be the number lying
1− r
Ê (b) A square matrix is called an between 2999 and 8001

x
= 1− r orthogonal matrix if AA ′ = I if repetition allowed
5 multiply. by A − 1 total numbers = 5 × 10 × 10 × 10 = 5000
x
⇒ r = 1− ∴ A − 1 ( AA ′ ) = A − 1I ⇒ IA ′ = A − 1 if repetition not allowed
5
⇒ A′ = A− 1 ∴total numbers = 5 × 9 × 8 × 7 = 2520
Where,| r | < 1
2 NDA/NA Solved Paper 2018 (II)

So, x = atleast two digit repeated ⇒ n ( A ∩ B) + n ( B ∩ C ) + n (C ∩ A ) 11. What is the coefficient of the middle
= 5000 − 2520 + 1 = 32 + 3n ( A ∩ B ∩ C ) ...(ii) term in the binomial expansion of
= 2481 From Eqs. (i) and (ii), we get ( 2 + 3x ) 4 ?
[Q add 1 because of number 8000] 60 + n ( A ∩ B ∩ C ) = 32 (a) 6 (b) 12
7. The (c) 108 (d) 216
sum of the series + 3n ( A ∩ B ∩ C )
Ê (d) We have, (2 + 3 x)
4
1 1 ⇒ 2 n ( A ∩ B ∩ C ) = 28
3 − 1 + − +.... is equal to
3 9 ⇒ n ( A ∩ B ∩ C ) = 14 Here, n = 4, so middle term is
20 9 th
(a) (b)
9. How many students like to play  4 + 1 = 3 rd term
9 20  
9 4 2 
(c) (d) exactly only one game?
4 9 (a) 196 (b) 228 T3 = 4C 2 × 2 2 × ( 3 x )2
(c) 254 (d) 268
Ê (c) Given series [Tr +1 = nC r a r b n − r ]1
1 1
3 − 1 + − + ..... are in GP Ê (c) Number of students like to play
exactly one 4×3
3 9 = × 4 × 9 x2
−1 game = n ( A ) + n ( B) + n (C ) 2 ×1
∴ r=
3 − 2 [n ( A ∩ B) + n ( B ∩ C ) + n (C ∩ A )] T3 = 216 x 2
3  a  + 3n ( A ∩ B ∩ C )
Sn = Q S n =  Hence, coefficient of middle term is 216.
 1  1 − r = 125 + 145 + 90 − 2 [32 + 3 × 14]
1 − − 
 3 + 3 × 14 12. For a square matrix A, which of the
3 9 = 360 − 106
following properties hold?
= =
4 4 = 254 1. ( A −1 )−1 = A
3 1
10. If α and β ( ≠ 0) are the roots of the 2. det ( A −1 ) =
Directions (Q. Nos. 8 and 9) det A
quadratic equation x 2 + αx − β = 0,
Consider the information given below then the quadratic expression 3. ( λA )−1 = λA −1, where λ is a scalar
and answer the two items that follow.
− x 2 + αx + β, where x ∈ R has Select the correct answer using the
A survey was conducted among 300
1 code given below.
students. It was found that 125 students (a) Least value −
4 (a) 1 and 2 (b) 2 and 3
like to play cricket, 145 students like to
9 (c) 1 and 3 (d) 1, 2 and 3
play football and 90 students like to play (b) Least value −
tennis. 32 students like to play exactly two 4 Ê (d) For a square matrix A
1
games out of the three games. (c) Greatest value Statement 1
4
( A − 1 )− 1 = A
8. How many stdudents like to play all (d) Greatest value
9
the three games ? 4 Statement 1 is true
(a) 14
(c) 28
(b) 21
(d) 35
Ê (d) α and β are the roots of quadratic Statement 2
1
equation. det ( A − 1 ) =
Ê (a) Let, x + αx − β = 0
2 det A
A be the set of students like to play cricket Statement 2 is correct
So, (αβ = − β ) ⇒ αβ + β = 0
B be the set of students like to play Statement 3
⇒ β (α + 1) = 0
football.
α = −1 [Qβ ≠ 0] ( λA )− 1 = λA − 1, where λ is a scalar.
C be the set of students like to play tennis.
α + β = −α So, Statement 3 is correct.
We have,
⇒ 2α + β = 0 13. Which one of the following factors
n ( A ∪ B ∪ C ) = 300
⇒ β =2 does the expansion of the
n ( A ) = 125
∴ − x 2 + αx + β [Qα = − 1, β = 2 ] determinant
n ( B) = 145
n (C ) = 90 = − x2 − x + 2 x y 3
2 3
n ( A ∪ B ∪ C ) = n( A ) + n ( B) + n (C )  ( − 1)2  x 5y 9 contain ?
Greatest value = 2 −  
− [n ( A ∩ B) + n ( B ∩ C ) + n (C ∩ A )]  4 ( − 1) x 3 10y 5 27
+ n (A ∩ B ∩ C) 2
 b2  (a) x − 3
⇒ 300 = 125 + 145 + 90 [Q Greatest value = c −   ]
 4a  (b) x − y
− [n ( A ∩ B) + n ( B ∩ C ) + n (C ∩ A )] (c) y − 3
+ n (A ∩ B ∩ C)  1  (d) x − 3 y
=2 −  
⇒ n ( A ∩ B) + n ( B ∩ C ) + n (C ∩ A )  − 4
Ê (a) We have,
= 60 + n ( A ∩ B ∩ C ) ...(i) 1
=2 +  x y 3
Again, 4  x 2 5 y3 9 
n ( A ∩ B) + n ( B ∩ C ) + n (C ∩ A ) 9  3 
= 5
 x 10 y 27 
− 3 n ( A ∩ B ∩ C ) = 32 4
NDA/NA Solved Paper 2018 (II) 3

[C1 → C1 − C 3 mes,] 16. There are 17 cricket players, out of − x − x − x 


⇒  c b− x a = 0
 x−3 y 3 which 5 players can bowl. In how  
=  x 2 − 9 5 y3 9  many ways can a team of 11 players  b a c − x
 3 
 x − 27 10 y 27 
5
be selected so as to include 3 1 1 1 
bowlers? ⇒ ( − x ) c b − x a = 0
 1 y 3  
(a) C (17, 11) c − x
= ( x − 3) x+ 3 5 y3 9  (b) C (12, 8) b a
 2  ⇒ x=0
 x + 9 + 3 x 10 y 27 
5
(c) C (17, 5) × C ( 5, 3)
(d) C ( 5, 3) × C (12, 8) Hence, x = 0 is a solution
14. What is the adjoint of the matrix Ê (d) There are 17 cricket players, out of 20. What should be the value of x, so
 cos ( −θ ) − sin ( −θ ) which 5 players can bowl.  2 4
 ? required number of ways = C 8 × 5C 3
12 that the matrix   does not
 − sin ( −θ ) cos( −θ )  −8 x 
= C (12, 8) × C ( 5, 3)
 cos θ − sin θ  have an inverse?
(a)   (b) −16
 − sin θ cos θ 17. What is the value of (a) 16
log 9 27 + log 8 32? (c) 8 (d) −8
 cos θ sin θ  7 19  2 4
(b)  
 sin θ cos θ
(a) (b) (c) 4 (d) 7 Ê (b) Let, A =  − 8 x 
2 6  
 cos θ sin θ  Ê (b) We have, Matrix does not have any solution if
(c)  
 − sin θ cos θ log 9 27 + log 8 32 | A| = 0
= log 32 3 3 + log 2 3 2 5 2 x + 32 = 0
 cos θ − sin θ
(d)   2 x = − 32
 sin θ cos θ  3 5
= log 3 3 + log 2 2 x=−
32
2 3
Ê (a) We have, 2
Q log b n = n log b  x = − 16
 cos ( − θ) − sin ( − θ)   a 
A= 
am
m 
 − sin ( − θ) cos ( − θ)  21. The system of equations
3 5 19
 cos θ sin θ  = + = 2x + y − 3z = 5
A= 
2 3 6
 sin θ cos θ  3x − 2y + 2z = 5
18. If A and B are two invertible square and 5x − 3y − z = 16
Now, C11 = cos θ matrices of same order, then what is (a) is inconsistent
C12 = − sin θ ( AB )−1 equal to? (b) is consistent, with a unique solution
C 21 = − sin θ (a) B−1 A −1 (b) A −1B−1 (c) is consistent, with infinitely many
C 22 = cos θ (c) B−1 A (d) A −1B solutions
(d) has its solution lying along X-axis in
 cos θ − sin θ
T
Ê (a) If A and B are two invertible square three-dimensional space
adj A =   matrices of same order, then
 − sin θ cos θ  Ê (b) The system of equations
( AB)− 1 = B− 1 A − 1
 cos θ − sin θ  2 x + y − 3z = 5
= 
 − sin θ cos θ  19. If a + b + c = 0, then one of the 3x − 2 y + 2 z = 5
solutions of and 5x − 3y − z = 0
15. What is the value of 2 1 −3 
a−x
A=  3 −2 2 
3a 3n c b
 −1 + i 3   −1 − i 3   
  +  c b−x a = 0 is  5 − 3 −1 
 2   2  b a c −x | A | = 2 [− 2 ( − 1) − 2 ( − 3)] − 1 [3 ( − 1) − 2 ( 5)]
where i = −1 ? (a) x = a + ( − 3) [3 ( − 3) − ( − 2 )( 5)]
(a) 3 (b) 2 3 (a2 + b 2 + c 2 ) = 2 ( 8) − 1 ( − 13) − 3(1)
(b) x =
(c) 1 (d) 0 2 = 16 + 13 − 3 = 26 ≠ 0
2 (a2 + b 2 + c 2 ) So, System is consistent with unique
Ê (b) We have, (c) x =
solution.
3n 3n 3
 − 1+ i 3  − 1− i 3 (d) x = 0
  +   22. Which one of the following is correct
 2   2 
Ê (d) We have, in respect of the cube roots of unity?
= (ω )3n + (ω 2 )3n a − x c b  (a) They are collinear
 c b− x a = 0 (b) They lie on a circle of radius 3
 − 1 + i 3 2  − 1 − i 3   (c) They form an equilateral triangle
Q ω = ,ω =  
 b a c − x
 2  2   (d) None of the above
R1 → R1 + R 2 + R 3
= (ω 3 )n + (ω 3 )2n Ê (c) We know that, cube roots of unity is
 a+ b +c − x a+ b +c − x a+ b +c − x − 1+ i 3
⇒ b− x = 0 1, ω, ω 2, where ω= and
(1)n + (1)2n [Qω 3 = 1] 
c a
 2
 b a c− x  −1 − i 3
= 1 + 1= 2 ω2 =
2
4 NDA/NA Solved Paper 2018 (II)

ω Y
(a) α > β + γ (b) α < β + γ
(–½, √3 ) Ê (c) Let first term of an AP is a and
2
(c) α = β + γ (d) α = βγ common difference is d
Ê (c) We have,(1 + x)
2n
B Given that,
th 1
Middle term =  + 1 Tm =
2n
120º 120º term n
X´ X  2 
120º A (1,2) 1
= ( n + 1)th term a + ( m − 1) d = ...(i)
ω1 n
(–½, √3 ) C Coefficient of ( n + 1)th term = 2n
Cn 1
2 and Tn =
α = 2n
Cn m
Y´ 1
Again, we have binomial expansion of a + ( n − 1) d = ...(ii)
They form an equilateral triangle. (1 + x )2n − 1 coefficient of middle terms m
Subtracting Eq. (ii) from Eq. (i), we get
23. If u, v and w (all positive) are the p th , are,
2n− 1 1 1
q th and r th terms of a GP, then the ∴ β= Cn ( m − 1) d − ( n − 1) d = −
n m
2n − 1
determinant of the matrix and γ = Cn − 1 m−n
⇒ ( m − n) d =
 ln u p 1 Now, β + γ = 2n − 1
Cn + 2n − 1
Cn − 1 mn
  1
 ln v q 1 is [Q C r + C r − 1 =
n n n +1
Cr ] ⇒ d =
 ln w r 1 mn
  = 2n
Cn Put in Eq. (i),
(a) 0 1 1
(b) 1 25. Let A = [ x ∈ R :−1 ≤ x ≤ 1], a + ( m − 1) =
mn n
(c) ( p − q )(q − r ) ( r − p)
(d) ln u × ln v × ln w B = [y ∈ R : − 1 ≤ y ≤ 1] and S be the ⇒ a+ −
1 1
=
1
th th subset of A × B, defined by n mn n
Ê (a) Given that u, v and w are the p ,q 1
and r th term of GP S = [( x , y ) ∈ A × B : x 2 + y 2 = 1]. ⇒ a=
mn
∴u = aR p − 1, v = aRq −1
[Qan = aR n−1
] Which one of the following is Now, Tmn = a + ( mn − 1) d
r −1 correct ?
and w = aR 1 1
= + ( mn − 1)
(a) S is a one-one function from A into B mn mn
 ln u p 1
(b)S is a many-one function from A into B
We have,  ln v q 1 =
1
+ 1−
1
  (c)S is a bijective mapping from A into B mn mn
 ln w r 1 (d) S is not a function
Tmn = 1
 ln a R p − 1 p 1 Ê (d) Given that,
=  ln a R q − 1 q 1 A = { x ∈ R : − 1≤ x ≤ 1 } ,
27. Suppose f ( x ) is such a quadratic

 
 expression that it is positive for all
ln a R r − 1 r 1 B = { y ∈ R : − 1≤ y < 1 }
  real x.
+ − and S = { ( x, y) ∈ A × B : x 2 + y 2 = 1 }
 ln a p 1 ln R p 1
If g ( x ) = f ( x ) + f ′( x ) + f ′′ ( x ),
=  ln a + q − 1 ln R q 1 Y
  then for any real x
 ln a + r − 1 ln R r 1 1 (a) g ( x ) < 0 (b) g ( x ) > 0
 ln a p 1  ( p − 1) ln R p 1 (c) g ( x ) = 0 (d) g ( x ) ≥ 0
=  ln a q 1 +  (q − 1) ln R q 1 Ê (b) Given that f( x) is a quadratic
   
 ln a r 1  ( r − 1) ln R r 1 X′ X expression
-1 1
 1 p 1 p−1 p 1 Let f( x ) = ax 2 + bx + c, a > 0
= ln a  1 q 1 + ln R  q − 1 q 1 ∴ b 2 − 4ac < 0 [∴ f( x ) > 0]
    -1
 1 r 1 r−1 r 1 ⇒ b 2 < 4ac
Y′
C2 → C2 − C3 Now, f ′ ( x ) = 2 ax + b
 p − 1 p − 1 1 By vertical line test. when we draw a and f ′ ′ ( x ) = 2 a
= 0 + ln R  q − 1 q − 1 1 vertical line, then line cuts the circle in two We have,
  points. Hence, S is not a function.
 r − 1 r − 1 1 g ( x ) = f( x ) + f ′ ( x ) + f ′ ′ ( x )
=0 26. Let Tr be the r th term of an AP for = ax 2 + bx + c + 2 ax + b + 2 a
r = 1, 2, 3, .... If for some distinct = ax 2 + ( b + 2 a ) x + 2 a + b + c
24. Let the coefficient of the middle term
positive integers m and n we have
of the binomial expansion of Now, ( b + 2 a )2 − 4a (2 a + b + c )
Tm = 1 / n and Tn = 1 / m, then what
(1 + x )2n be α and those of two is Tmn equal to ? = b 2 + 4ab + 4a 2 − 8a 2 − 4ab − 4ac
middle terms of the binomial (a) ( mn)−1 = b 2 − 4ac − 8a 2 < 0
expansion of (1 + x )2n −1be β and γ. (b) m−1 + n−1
[Q b 2 − 4ac < 0]
Which one of the following relations (c) 1
(d) 0 ⇒ g ( x) > 0
is correct?
NDA/NA Solved Paper 2018 (II) 5

28. Consider the following in respect of Ê (c) We have, 34. If A, B and C are the angles of a
matrices A, B and C of same order. ( 0.2 )x = 2 triangle and
1. ( A + B + C )′ = A ′ + B ′ + C ′ taking log10 both side 1 1 1
2. ( AB )′ = A ′ B ′ . = log10 2
x log10 02 I + sin A 1 + sin B 1 + sin C = 0,
3. ( ABC )′ = C ′ B ′ A ′ SinA + sin 2 A sin B + sin 2 B sin C + sin 2 C
x log10   = log10 2
2

Where A ′ is the transpose of the  10 
matrix A. Which of the above are then which one of the following is
⇒ x [log10 2 − log10 10] = log10 2
correct? correct?
(a) 1 and 2 (b) 2 and 3 ⇒ x [0.3010 − 1] = 0.3010 [Q log a a = 1] (a) The triangle ABC is isosceles
(c) 1 and 3 (d) 1, 2 and 3 0.3010
⇒ x=− ≈ − 0.43 (b) The triangle ABC is equilateral
Ê (c) Given that A, B and C are matrices of 0.6990 (c) The triangle ABC is scalene
same order 32. The total number of 5-digit numbers (d) No conclusion can be drawn with
Statement 1 that can be composed of distinct regard to the nature of the triangle
( A ′ + B + C )′ = A ′ + B′ + C ′ digits from 0 to 9 is Ê (a) We have,
[Q ( A + B)′ = A ′ + B′ ] (a) 45360 (b) 30240
So, Statement 1 is correct (c) 27216 (d) 15120  1 1 1 
 1 + sin A 1 + sin B 1 + sin C 
Statement 2 Ê (c) 5-digit number that can be  
We know that, composed by distinct digits from 0 to 9 is sin A + sin 2
A sin B + sin 2
B Sin C + Sin 2
C 
( AB)′ = B′ A ′ given as =0
Hence, Statement 2 is incorrect R1 → R1 − R 2, R 3 → R 3 − R 2
Statement 3
9 9 8 7 6  − sin A − sin B − sin C 
( ABC )′ = C ′ B′ A ′ [Q ( AB)′ = B′ A ′ ]  1 + sin A 1 + sin B 1 + sin C  = 0
required number = 9 × 9 × 8 × 7 × 6  2 
 sin A − 1 sin B − 1 sin C − 1
Hence, Statement 3 is correct. 2 2
= 27216
29. The sum of the binary numbers R 2 → R 2 + R1
(11011)2 , (10110110)2 and 33. What is the determinant of the
 − sin A − sin B − sin C 
(10011x 0y )2 is the binary numbers matrix  = 0
1 1 1
(101101101)2 . What are the values of x y y + z  
− cos 2
A − cos 2
B − cos 2
C
x and y ?    
z x z + x ?
(a) x = 1, y = 1 (b) x = 1, y = 0 R3 → R3 + R2
y z x + y 
(c) x = 0, y = 1 (d) x = 0, y = 0   sin A sin Bsin C 
Ê (b) Sum of the binary number (a) ( x − y) ( y − z) ( z − x ) ⇒  1 1 1 = 0
(11011)2, (10110110)2 and (10011xoy)2 is (b) ( x − y) ( y − z)  
 1 − cos 2
A 1 − cos 2
B 1 − cos 2
C 
(101101101)2 (c) ( y − z) ( z − x )
So, (101101101) (d) ( z − x )2 ( x + y + z)  sin A sin B sin C 
− 10110110 ⇒ 1 1 1 = 0
Ê (d) We have,  2 2 2 
10110111 x y y + z  sin A sin B sin C
− 11011 z x z + x [C1 → C1 − C 2 C 2 → C 2 − C 3]
 
10011100
y z x + y
 sin A − sin B sin B − sin C sin C 
Compare with (10011x o y)2
R1 → R1 + R 2 + R 3 ⇒ 0 0 1 
We get, x = 1 and y = 0  2 
x+ y + z x + y + z 2 ( x + y + z)   sin A − sin B sin B − sin C sin C 
2 2 2 2

30. Let matrix B be the adjoint of a  z x z+ x 


  =0
square matrix A, I be the identity  y z x+ y 
matrix of same order as A. If k( ≠ 0) is ⇒ (sin A − sin B) (sin B − sin C )
1 1 2   1 1 sin C 
the determinant of the matrix A,
= ( x + y + z) z x z + x   0 0 0 
then what is AB equal to ?    
(a) l (b) kl (c)k 2 l (d) (1 / k )l  y z x + y sin A + sin C sin B + sin C sin C 
2

C 2 → C 2 − C1 and C 3 → C 3 − 2C1
Ê (b) Given, ∴ Sin A − sin B = 0
B = adjA, l = identity Matrix 1 0 0 
or sin B − sin C = 0
= ( x + y + z) z x − z x − z
( A) = K   ⇒ sin A = sin B
∴ AB = A (adjA ) = ( A )l = kl y z− y x− y
and sin B = sin C
= ( x + y + z) [1 ( x − z) ( x − y)
. )x = 2 and log10 2 = 03010
31. If (02 . , then ⇒ A=B
− ( x − z) ( z − y)]
what is the value of x to the nearest and B=C
= ( x + y + z) [( x − z) ( x − y − z + y)]
tenth? So, ABC is an isosceles triangle
(a) −10.0 (b) −0.5 = ( x + y + z) ( x − z) ( x − z)
(c) −0.4 (d) −0.2 = ( x + y + z) ( z − x )2
6 NDA/NA Solved Paper 2018 (II)

2 2 cos θ cos α
35. Consider the following in respect of ⇒ = 39. A balloon is directly above one end
cos θ cos 2 θ − sin2 α of a bridge. The angle of depression
matrices A and B of same order
1. A 2 − B 2 = ( A + B ) ( A − B ) ⇒ cos 2 θ cos α = cos 2 θ − sin2 α of the other end of the bridge from
the ballon is 48º. If the height of the
2. ( A − I ) ( I + A ) = O ⇔ A 2 = I ⇒ sin2 α = cos 2 θ − cos 2 θ cos α
balloon above the bridge is 122 m,
where I is the identity matrix and O ⇒ sin2 α = cos 2 θ (1 − cos α ) then what is the length of the
is the null matrix. sin2 α bridge?
⇒ cos 2 θ =
1 − cos α (a) 122 sin 48º m
Which of the above is/are correct?
(b) 122 tan 42 º m
(a) 1 only (b) 2 only 1 − cos 2 α (1 + cos α ) (1 − cos α )
(c) Both 1 and 2 (d) Neither 1 nor 2 = = (c) 122 cos 48º m
1 − cos α 1 − cos α (d) 122 tan 48º m
Ê (b) ⇒ cos 2 θ = 1 + cos α
C
Statement 1
Ê (b) 48º

RHS = ( A + B) ( A − B) ⇒ 1 − sin2 θ = 1 + cos α


122 m
= A 2 − AB + BA − B2 [Q AB ≠ BA ] [Q sin2 θ + cos 2 θ = 1]
≠ A 2 − B2 ⇒ 1 − sin2 θ = 1 + cos α 48º
A B
Statement 1 is not correct ⇒ sin θ + cos α = 0
2
BC
Statement 2 tan 48° =
( A − I) ( I + A ) = 0 38. If A + B + C = 180º, then what is AB

⇒ AI + A 2 − I 2 − IA = 0[Q AI = IA = A ] sin 2A − sin 2B − sin 2C equal to? tan 48° =


122
(a) −4sin A sin Bsin C AB
⇒ A+ A −I − A=0
2 2
122
(b) −4cos A sin B cos C AB = = 122 cot 48°
⇒ A2 = I [Q I 2 = I ] (c) −4 cos A cos Bsin C tan 48°
Statement 2 is correct (d) −4sin A cos Bcos C = 122 cot ( 90° − 42 ° )
2 tan θ Ê (d) We have, [Q cot ( 90° − θ) = tan θ]
36. What is equal to ?
sin 2 A − sin 2 B − sin2 C
1 + tan θ 2 = 122 tan 42 °
(a) cos2θ (b) tan2θ = sin 2 A − [sin 2 B + sin 2C ] 40. A is an angle in the fourth quadrant
2 B + 2C 
= sin 2 A − [2 sin 
(c) sin2θ (d) cosec 2θ it satisfies the trigonometric

Ê (c) We have,  2  equation 3(3 − tan 2 A − cot A )2 = 1.
2 sin θ 2 B − 2C  
2 tanθ cos θ cos  
Which one of the following is a value
=  2  of A ?
1 + tan2 θ sin2 θ
1+  C + D 
(a) 300º (b) 315º (c) 330º (d) 345º
cos 2 θ Q Sin C + Sin D = 2 Sin 2   Ê (a) We have,
2 sin θ cos θ  
= [Q sin2 θ + cos 2 θ = 1] Cos  C − D   3 ( 3 − tan2 A − cot A )2 = 1
cos 2 θ + sin2 θ    
  2   Checking through options
= 2 sin θ cos θ = sin 2θ
= 2 sin A cos A − [2 sin ( B + C ) A = 300
37. If sec (θ − α ), sec θ and sec (θ + α ) are cos ( B − C )] So, 3 [3 − tan2 300° − cot ( 300)° ]2
in AP, where cosα ≠ 1, then what is [Q Sin 2 A = 2 Sin A cos A ] = 3 [3 − tan2 ( 360° − 60° )
the value of sin 2 θ + cos α ? = 2 sin A cos A − [2 sin (180° − A ) − cot ( 360° − 60)° ]2
(a) 0 (b) 1
1 cos ( B − C )] = 3 [3 − tan 60° + cot 60° ]2
2
(c) –1 (d)
2 [Q A + B + C = 180] 2
 1  1
= 2 sin A cos A − 2 sin A cos ( B − C ) = 3 3 − 3 + = 3× =1
Ê (a) If sec (θ − α ), sec θ and sec (θ + α )  3  3
are in AP, then [Q sin (180° − θ) = sinθ]
So, option (a) is correct
2 sec θ = sec (θ − α ) + sec (θ + α ) = 2 sin A [cos A − cos ( B − C )]

2
=
1
+
1 41. The top of a hill observed from the
= 2 sin A [cos (180 − ( B + C )
cos θ cos (θ − α ) cos (θ + α ) top and bottom of a building of
cos (θ + α ) + cos (θ − α ) − cos ( B − C )] π
2
⇒ = height h is at angles of elevation
cos θ cos (θ − α ) (cos (θ + α ) = 2 sin A [− cos ( B + C ) − cos ( B − C )] 6
⇒ = − 2 sin A [cos ( B + C ) + cos ( B − C )] π
and respectively. What is the
 θ + α + θ − α  θ + α − θ + α = − 2 sin A [2 cos B cos C ] 3
2 cos   cos  
2  2   2  Q cos C + cos D height of the hill?
=
cos θ cos 2 θ − sin 2 α C + D C − D 3h

= 2 cos   (a) 2 h (b)

 cos   2
 C + D C − D   2   2 
Q cos C + cos D = 2 cos  2  cos  2 

h
 = − 4 sin A cos B cos C
(c) h (d)
2
[and cos ( A + B) cos ( A − B) = cos 2 A − sin 2 B]
NDA/NA Solved Paper 2018 (II) 7

Ê (b) height of building be h and let height 3 3


⇒ sin x = So, cosα =
of hill is H 2 2
D π 2π − 3
∴ x= , and cosβ =
3 3 2
30º π 2  − 2 − 4
B E
43. If θ = , then what is the value of Now, secα ⋅ secβ = ×  =
H
8 3  3 3
h h
(2 cos θ + 1)10 (2 cos 2θ − 1)10 (2 cos θ − 1)10 45. Consider the following values of x :
A
60º
C (2 cos 4θ − 1) ? 10 1. 8 2. −4
1 1
X
(a) 0 (b) 1 3. 4. −
In ∆ACD (c) 2 (d) 4 6 4
π Which of the above values of x is/are
tan 60° =
H Ê (b) If θ = , then
x 8 the solution (s) of the equation
H (2 cos θ + 1)10 (2 cos 2θ − 1)10 π
⇒ 3= tan −1(2x ) + tan −1(3x ) = ?
x (2 cos θ − 1)10 (2 cos 4 θ − 1)10 4
H
⇒ x= ...(i) ⇒ (2 cos θ + 1)10 (2 cos θ − 1)10 (a) 3 only (b) 2 and 3
3 (c) 1 and 4 (d) 4 only
(2 cos 2θ − 1) (2 cos 4θ − 1)
10 10
In ∆BDE Ê (a) We have,
H−h ⇒ ( 4 cos 2 θ − 1)10 (2 cos 2 θ − 1)10 π
tan 30° = tan− 1(2 x ) + tan− 1( 3 x ) =
x (2 cos 4θ − 1)10 4
1 H−h  2 x + 3x  π
⇒ = ⇒ [2 (2 cos 2 θ) − 1]10 (2 cos 2θ − 1)10 tan− 1   =
3 x  1 − 2 x ⋅ 3x 4
(2 cos 4θ − 1) 10
⇒ x= 3 ( H − h)
 a + b
H ⇒ [2 (1 + cos 2 θ) − 1] (2 cos 2 θ − 1)10
10
[Q tan− 1 a + tan−1 b = tan− 1  ,
⇒ = 3 ( H − h)  1 − ab 
3 (2 cos 4 θ − 1)10
⇒ H = 3 ( H − h) ab ≤ 1
⇒ (2 cos 2 θ + 1)10 (2 cos 2 θ − 1)10
⇒ H = 3H − 3h and tan− 1 a + tan− 1 b = π + tan− 1
(2 cos 4 θ − 1)10
⇒ 2 H = 3h  a + b
10 10   , ab > 1
π  2 cos π − 1
⇒ H= h
3 ⇒ 2 cos + 1    1 − ab 
2  4   4 
5x π
10 ⇒ = tan
42. What is/are the solution (s) of the  2 cos π − 1 Q θ = π  1 − 6 x2 4
  
trigonometric equation  2  8  5x
⇒ =1
cosec x + cot x = 3 where ⇒ 1 − 6 x2
10 10
0 < x < 2π ?  1   1  ⇒ 1 − 6 x2 = 5 x
5π π 2 × 2 + 1 2 ⋅
 2
− 1

( 0 − 1)10
(a) only (b) only ⇒ 6x + 5x − 1 = 0
2
3 3
π 5π ⇒ ( 2 + 1)10 ( 2 − 1)10 × 1
(c) π only (d) π, , ⇒ 6 x2 + 6 x − x − 1 = 0
3 3 ⇒ (( 2 )2 − (1)2 )10 × 1
⇒ ( 6 x − 1) ( x + 1) = 0
Ê (b) We have, ⇒ (2 − 1)10 1
⇒ x = − 1,
cosecx + cot x = 3 ...(i) 6
⇒ 1
Now, x = − 1 is not possible
(cosecx + cot x ) (cosecx − cot x ) 44. If cosα and cosβ (0 < α < β < π) are 1
⇒ = 3 So, x=
cosecx − cot x the roots of the quadratic equation 6
cosec 2 x − cot 2 x 4 x 2 − 3 = 0, then what is the value of 46. If the second term of a GP is 2 and
⇒ = 3
cosecx − cot x sec α × sec β ? the sum of its infinite terms is 8, then
4 4
1 (a) − (b) the GP is
⇒ = 3 3 3
cosecx − cot x (a) 8, 2,
1 1
, ,…
3 3
1 (c) (d) − 2 8
⇒ cosecx − cot x = ...(ii) 4 4 2 2
3 (b) 10, 2, , ,…
Ê (a) Given, 5 25
1 1
Adding Eqs. (i) and (ii), we get
4 x2 − 3 = 0 (c) 4, 2, 1, , 2 …
1 2 2
2 cosecx = 3 +
3 ⇒ 4 x2 = 3 3. 3
(d) 6, 3, , ,…
4 3 2 4
⇒ 2 cosecx = ⇒ x2 =
3 4 Ê (c) Let first term of a GP is a and common
2 3 ratio is r
⇒ cosecx = ⇒ x=±
3 2 ∴ a2 = 2
8 NDA/NA Solved Paper 2018 (II)

⇒ ar = 2 ...(i) 49. The ratio of roots of the equations Ê (b) We have


and Sn = 8 ax 2 + bx + c = 0 and px 2 + qx + r = 0 A = sin2 θ + cos 4 θ
a
⇒ =8 are equal. If D1 and D 2 are respective = sin2 θ + (1 − sin2 θ)2
1− r
D Let sin2 θ = x
From Eq. (i)
discriminants, then what is 1 equal
D2
2 = x 2 − x + 1, 0 ≤ x ≤ 1
⇒ =8 to ?
r (1 − r ) = x2 − x + 1
a2 b2
⇒ 8r − 8r = 22 (a) (b) 2
p2 q2  x − 1 + 3
⇒  
⇒ 4r − 4r + 1 = 0
2
c2  2 4
(c) (d) None of these
⇒ 4r 2 − 2 r − 2 r + 1 = 0 r2 Now, A( 0) = A(1) = 1
⇒ 2 r (2 r − 1) − 1 (2 r − 1) = 0 Ê (b) Let α, β are the roots of quadratic So,
3
≤ A≤1
⇒ (2 r − 1) (2 r − 1) = 0 equation. 4
1 ax 2 + bx + c = 0 51. The equation of a circle whose end
⇒ r=
2 b c points of a diameter are ( x 1, y1 ) and
So, α +β=− and αβ =
∴Put in Eq. (i) a a ( x 2 , y 2 ) is
a   = 2
1 and let γ , δ are the roots of quadratic (a) ( x − x 1 ) ( x − x 2 ) + ( y − y1 ) ( y − y2 )
2 equation
= x 2 + y2
⇒ a=4 px 2 + qx + r = 0 (b) ( x − x 1 ) + ( y − y1 ) = x 2 y2
2 2
1
∴GP is 4, 2, 1, ... γ +δ=−
q (c) x 2 + y 2 + 2 x 1x 2 + 2 y1 y2 = 0
2 p (d) ( x − x 1 ) ( x − x 2 ) + ( y − y1 ) ( y − y2 ) = 0
47. If a, b, c are in AP or GP or HP, then and γδ =
r
Ê (d) Equation of circle if end points of
a −b p
diameter are ( x1, y1 ) and ( x2, y2 ) are
is equal to
b −c Acoording to the question,
α γ
=
b b c c
(a) or 1 or (b) or or 1
a c a b β δ
A B
a a a c
(c) 1 or or (d) 1 or or according to componendo and dividendo (x1, y1) (x2, y2)
α +β γ +δ
b c b a
=
Ê (c) a, b, c are in AP, then α −β γ −δ
∴ a− b= b−c α +β γ +δ
⇒ = ( x − x1 ) ( x − x2 ) + ( y − y1 ) ( y − y2 ) = 0
a−b
⇒ =1 (α + β ) − 4αβ
2
( γ + δ )2 − 4γδ
b−c 52. The second degree equation
b q
− −
If a, b, c are in GP, then
a p x 2 + 4y 2 − 2x − 4y + 2 = 0
a b a−b ⇒ =
∴ = = − b − 4
2
c  q  r
2 represents
b c b−c      −  − 4  (a) A point
 a  a  p  p
If a, b, c are in HP, then (b) An ellipse of semi-major axis 1
2 ac b q 3
∴ b= p
(c) An ellipse with eccentricity
a+c ⇒ a = 2
b 2 − 4ac q 2 − 4 pr (d) None of the above
⇒ ab + bc = 2 ac
⇒ ab − ac = ac − bc a p Ê (d) Given that,
⇒ a (b − c ) = c (a − b ) ⇒
b
=
q x 2 + 4 y2 − 2 x − 4 y + 2 = 0
a−b a b − 4ac
2
q − 4 pr
2
Compare with
⇒ =
b−c c b q ax 2 + by 2 + 2 hxy + 2 gx + 2 yx + 2 fy + c = 0
⇒ =
D1 D2
48. What is the sum of all three digit Then,
numbers that can be formed using all ⇒
D1
=
b a = 1, b = 4, h = 0, g = − 1, f = − 2
the digits 3, 4 and 5, when repetition D2 q and c =2
of digits is not allowed?
D1 b2 ∆ = abc + 2 fhg − af 2 − bg 2 − ch2
(a) 2664 (b) 3382 ⇒ = 2
(c) 4044 (d) 4444 D2 q
= 1 × 4 × 2 + 2 × ( − 2 ) × ( − 1) × 0
Ê (a) Three digit number. that can be 50. If A = sin 2 θ + cos 4 θ, then for all real −1 × ( − 2 )2 − 4 × ( − 1)2 − 2( 0)
formed using 3, 4 and 5 when repetition θ, which one of the following is
not allowed are 543, 534, 453, 435, 354, =8+0−4−4=0
correct?
345 3 =8−8
(a) 1 ≤ A ≤ 2 (b) ≤ A ≤ 1
Sum 4 =0
= 543 + 534 + 453 + 435 + 354 + 345 13 3 13 Equation, represent a pair of straight line.
(c) ≤ A≤1 (d) ≤ A ≤
= 2664 16 4 16
NDA/NA Solved Paper 2018 (II) 9

53. The angle between the two lines Statement 2 for ax + by + c1 = 0 56. What is the distance of the point
lx + my + n = 0 and and ax + by + c 2 = 0 (2, 3, 4) from the plane
l ′ x + m ′ y + n ′ = 0 is given by both lines are parallel so,
3x − 6y + 2z + 11 = 0 ?
tan −1 θ. What is θ equal to?  c −c 
(a) 1 unit (b) 2 units
∴ D =

1 2

lm′ − l ′ m  lm′ + l ′ m  2
(a) (b)   a 2
+ b 
(c) 3 units (d) 4 units
ll ′ − mm′  ll ′ + mm′ 
Statement 3 x = c1 Ê (a) Distance of point ( x1, y1, z1 ) from plane
lm′ − l ′ m, lm′ + l ′ m ax + by + cz + d = 0 is
(c) (d) x = c2
ll ′ + mm′ ll ′ − mm′  ax + by + cz +
D = | c1 − c 2 | d
d =

1 1 1


Ê (c) Given straight lines, Statement 3 is correct  a 2
+ b 2
+ c2 
lx + my + n = 0 ……(i) Y
l P (2, 3, 4)
m1 = −
m
and l ′ x + m′ y + n′ = 0
l′ X′ X
m2 = − ′
m′
M 3x–6y+2z+11=0
angle between both lines is θ
l l′
− − x=c1 x=c2  3(2 ) − 6 ( 3) + 2 ( 4) + 1
m m′ − Y′ PM = 
 
2 
ll ′  ( 3) + ( − 6) + (2 ) 
2 2
∴ tanθ = 1 + ′
mm′ 55. What is the equation of straight line
passing through the
point of 6 − 18 + 8 + 11 7
= =
intersection of the lines
x y
+ =1  9`+ 36 + 4  49
  m1 − m2  2 3
Q tanθ =   PM = 1 unit
  1 + m1m2 x y
and + = 1 and parallel to the line
− lm′ − l ′ m 3 2 57. Coordinates of the points O , P , Q and
= ′
mm′ 4 x + 5y − 6 = 0 ? R respectively: (0, 0, 0), (4, 6, 2m), (2,
mm′ + ll ′ lm′ − l ′ m (a) 20x + 25 y − 54 = 0 0, 2n) and (2, 4, 6) L , M , N and K
′ =  (b) 25x + 20 y − 54 = 0
mm′ ll ′ + mm′ (c) 4x + 5 y − 54 = 0
OR, OP , PQ and QR respectively such
(d) 4x + 5 y − 45 = 0
that LMNK is a parallelogram whose
54. Consider the following statements
two adjacent sides LK and LM are
1. The distance between the lines Ê (a) Given lines, each of length 2 ?
y = mx + c 1 and y = mx + c 2 is x y
+ =1
| c1 − c 2 | 2 3
(a) 6, 2
x y (b) 1, 3
1 + m2 and + =1 (c) 3, 1
3 2
2. The distance between the lines (d) None of the above
ax + by + c 1 = 0 and l1=0 Ê (c) LMNK is a parallelogram we know
| c1 − c 2 | that, if we join mid- point of any
ax + by + c 2 = 0 is .
quadrilateral we get a parallelogram
a2 + b2
R Q
3. The distance between the lines l2=0 (2, 4, 6) K (2, 0, 3n)
x = c 1 and x = c 2 is | c 1 − c 2 |. l1+λl2=0
Which of the above statements are
correct? Equation of line passing through the point L N
of intersection are l1 + λl 2 = 0
(a) 1 and 2 (b) 2 and 3
(c) 1 and 3 (d) 1, 2 and 3 ( 3 x + 2 y − 6) + λ (2 x + 3 y − 6) = 0 ...(i)
Slope of line O M P
Ê (b) Statement 1 Given lines, (0, 0, 0) (4, 6, 2m)
− (3 + 2 λ ) 4
y = mx + c1 ∴ =−
2 + 3λ 5
and y = mx + c 2 So. M , N, K , L are mid- points of
both lines are parallel ⇒ + 15 + 10λ = 8 + 12 λ OP, PQ, QR and RO respectively
Q ⇒ 7 = 2λ ∴Coordinate of M
7
⇒ λ= 0 + 4 0 + 6 0 + 2 m
2 =  , , 
 2 2 2 
7
P ( 3 x + 2 y − 6) + (2 x + 3 y − 6) = 0
2 = (2, 3, m)
c − c
PQ = 

1 2 ⇒ 2 ( 3 x + 2 y − 6) + 14 x + 21y − 42 = 0 [Q Coordinate of mid-point
 1 + m 2
⇒ 6 x + 4 y − 12 + 14 x + 21y − 42 = 0 x + x2 y1 + y2 
=  1 , ]
So, Statement 1 is correct ⇒ 20 x + 25 y − 54 = 0  2 2 
10 NDA/NA Solved Paper 2018 (II)

Coordinate of L = (1, 2, 3) Which of the above statement is/are Ê (d) Statement I If the line segment joining
Coordinate of K = (2, 2, 3 + n) correct? the point P( m, n) and Q( r, s ) subtends
Now, LM = 2 (a) 1 only (b) 2 only angle α at origin, then
(c) Both 1 and 2 (d) Neither 1 nor 2 mr + ns
⇒ (2 − 1)2 + ( 3 − 2 )2 + ( m − 3)2 = 2 cosα =
Ê (c) Statements 1 Given, m + n2 r 2 + s 2
2

⇒ 1 + 1 + ( m − 3)2 = 2 2 x − y + z = 1 and x + y + 2 z = 3 So, Statement I is not correct


Squaring both side, Here, a1 = 1, b1 = −1, c1 = 1 Statement II
⇒ 2 + ( m − 3)2 = 2 and a2 = 1, b 2 = 1, c 2 = 2 In any triangle ABC
a1a2 + b1b 2 + c1c 2 a 2 = b 2 + c 2 − 2 bc cos A
⇒ ( m − 3)2 = 0 cos θ =
a1 + b12 + c12 a22 + b 22 + c 22
2
⇒ m=3 Statement II is correct
LK = 2 2 × 1 + ( − 1 × 1) + 1 × 2
Again, = 61. What is the area of the triangle with
⇒ 2 2 + ( − 1)2 + (1)2 12 + 12 + 2 2 vertices
(2 − 1)2 + (2 − 2 )2 + ( 3 + n − 3)2 = 2 4 −1 
= =
3
=
3 1  1  1
4 + 1+ 1 4 + 1+ 1 6 6 6  x 1,  ,  x 2 ,  ,  x 3 ,  ?
⇒ 1 + 0 + n2 = 2  x1   x2  x3
1
Squaring both side, = (a)| ( x 1 − x 2 ) ( x 2 − x 3 ) ( x 3 − x 1 )|
2
⇒ 1 + n2 = 2 π
(b) 0
∴ θ=  (x − x 2 ) (x 2 − x 3 ) (x 3 − x1 ) 
⇒ n2 = 1 3 (c) 1 
 x 1x 2x 3 
⇒ n=1 So, Statement 1 is correct.
 (x1 − x 2 ) (x 2 − x 3 ) (x 3 − x1 ) 
x −1 y −2 z −3 Statement 2 Distance between two (d) 
58. The line = = is planes  2 x 1x 2x 3 
2 2 4
given by ax + by + cz + d 1 = 0 Ê (d) Area of ∆ABC
ax + by + cz + d 2 = 0  1  1  1
(a) x + y + z = 6, x + 2 y − 3 z = − 4 A  x1,  , B  x2,  , C  x3, 
(b) x + 2 y − 2 x = − 1, 4x + 4 y − 5 z  d1 − d 2   x1   x2   x3 
distance S=  
−3=0 2  x1 y1 1
 a + b +c 
2 2
(c) 3x + 2 y − 3 z = 0, 3x − 6 y + 3 z = − 2 1
(d) 3x + 2 y − 3 z = − 2, 3x − 6 y + 3 z = 0 2 A= x2 y2 1
d1 = , d 2 = 4 2 
3  x3 y3 1
Ê (d) Given lines,
x−1 y−2 z−3  4−
2   10  1 
= = = λ (let)    3 x 1
2 3 4 distance = 3 =  1 x1 
 2 2 + 12 + 2 2
   9
  
x−1 1 1
∴ =λ     =  x2 1
2 2 x2 
10 10
= =  1
⇒ x = 2λ + 1
3×3 9  x3 1
y−2  x3 

3 60. Consider the following statements :  1 
x 1
⇒ y = 3λ + 2 Statement I : If the line segment  1 x1 
z−3 1  1 
and =λ joining the points P (m, n ) and Q (r , s ) =  x2 1
4 subtends an angle α at the origin, 2 x2 
⇒ z = 4λ + 3 ms − nr 
 x3
1
1
then cos α = .  x3 
by checking options if 3 x + 2 y − 3 z, then
(m + n 2 ) (r 2 + s 2 )
2
3 (2 λ + 1) + 2 ( 3λ + 2 ) − 3 ( 4λ + 3)  x12 1 x1 
= 6λ + 3 + 6λ + 4 − 12 λ − 9 Statement II. In any triangle ABC, ⇒
1  x2 1 x 
2 x1 x2 x3  
2 2
= −2 it is true that  2 
x3 1 x3
a 2 = b 2 + c 2 − 2bc cos A .  
3 (2 λ + 1) − 6 ( 3λ + 2 ) + 3 ( 4λ + 3)
 ( x − x2 ) ( x2 − x3 ) ( x3 − x1 ) 
⇒ 6λ + 3 − 18λ − 12 + 12 λ + 9 = 0 Which one of the following is = 1 
So, option (d) is correct correct in respect of the above two  2 x1 x2 x3 
statements? 62 If Y-axis touches the circle
59. Consider the following statements (a) Both Statement I and Statement II are
1. The angle between the planes true and Statement II is the correct x 2 + y 2 + gx + fy +
c
= 0, then the
2x − y + z = 1 and x + y + 2z = 3 explanation of statement I. 4
π (b) Both Statement I and Statement II are normal at this point intersects the
is true, but Statement II is not the correct
3 circle at the point.
explanation II is not the correct
(a)  − ,  (b)  − g , − 
2. The distance between the planes g f f
explanation of Statement I
6x − 3y + 6z + 2 = 0 and  2 2  2
(c) Statement I is true, but Statement II is
10 (c)  − , f 
false g
2x − y + 2z + 4 = 0 is (d) ( − g , − f )
(d) Statement I is false, but Statement II is  2 
9
true
NDA/NA Solved Paper 2018 (II) 11
r r r r r
Ê (b) Now, r ⋅ ( $i + $j + k$ ) $i . $i = 1 C = a − b− c
y r r r
= ( xi$ + yj$ + zk$ ) ⋅ ( $i + $j + k$ ) | a | = | b| = | c | = 1
r r r r r r
= ( x + y + z) | a ⋅ b| = | b ⋅ c | = c ⋅ a = 0
-1 r
( 0, 2 ( A 65. A unit vector perpendicular to each
B Now, | A | = a 2 + b 2 + c 2
-g , -f
( 2 2( of the vectors 2i$ − $j + k$ and = 1+ 1+ 1 = 3
3i$ − 4 $j − k$ is
x′ r
x | B| = a2 + b 2 + c 2 = 1 + 1 + 1 = 3
O 1 r
$i + 1 $j − 1 $
y′ (a) k |C| = a + b + c = 1+ 1+ 1 =
2 2 2
3
3 3 3 r r r
The equation of circle is 1 ⇒ | A | =| B| =| C |
(b) $i + 1 $j + 1 k$
c
x 2 + y 2 + gx + fy + =0 2 2 2 r r r r
4 1 1 $ 1 $ 68. What is ( a − b) × ( a + b) equal to?
(c) i$ − j − k r r r
y −f
Q centre =  − ,  and 3 3 3 (a) 0
r r
(b) a × b
r
 2 2  1 1 $ 1 $ (c) 2 ( a × b)
r
(d)| a |2 − | b|2
(d) i$ + j + k
g 2 f2 c 3 3 3
radius = + − Ê (c) We have,
4 4 4 Ê (a) We know that,r r
r r r r
( a − b) × ( a
+ b)
Since, circle touches Y-axis, then r a × b r r r r r r
n = r r r r
=a ×a + a × b− b× a − b× b
AC = radious | a × b|
r r r r
 $i = a × b− b× a
g g 2 f2 −c $j k$ 
⇒ = + r r   r r r r
2 4 4 4 a × b= 2 −1 1 [Q a × a = 0, b × b = 0]
 3 − 4 − 1
  r r r r r r r r
f2 c = a × b + a × b [Q a × b = − b × a ]
⇒ =  
4 4 r r r
r
a × b = 5$i + 5 $j − 5k$ = 2 ( a × b)
∴circle touches Y-axis at r r
| a × b| = 5 3 69. A spacecraft located at i$ + 2 $j + 3k$ is
f2
y 2 + fy + =0
4 r 5 ( $i + $j − k$ ) subjected to a force λk$ by firing a
∴ n =
2 5 3
⇒ y+ f  = 0 rocket. The spacecraft is subjected to
  $i + $j − k$
 2 = a moment of magnitude
−f 3 (a) λ (b) 3λ
⇒ y= r r r r
2 66. If | a | = 3, | b | = 4 and | a − b | = 5, (c) 5λ (d) None of these
 −f  r r Ê (c) Werhave,
∴ A  0,  then what is the value of | a + b | ?
 2  r = $i + 2 $j + 3k$
(a) 8 (b) 6 (c) 5 2 (d) 5 r
∴ Normal at A will pass through centre C and F = λk$
and intersect circle again at B. Ê (d) Given that, r
| a| = 3 We know that,
−f
∴Coordinates of B are  − g ,  r r r
Moment = r × F
 2  | b| = 4
r r r r r r r r = ( i$ + 2 $j + 3k$ ) × ( λk$ )
63. Let | a | ≠ 0, | b | ≠ 0. ( a + b) ⋅ ( a + b) and | a − b| = 5
r r  ∴ $i × k$ = − $j 
r → ∴ a⋅ b= 0  $ $ $ 
= | a | 2 + | b | 2 holds if and only if r r r r = − λ$j + 2 λi$  j ×k=i 
r r | a + b|2 = | a |2 | + | b|2 + 0 = 25  k$ × k$ = 0 
(a) a and rb are perpendicular r
r r
| a + b| = 5  
(b) a andrb are parallel
r
(c) a and br are inclined at an angle of 45º r r r Magnitude of moment = ( − λ )2 + (2 λ )2
r
(d) a and b are anti-parallel
67. Let a, b and c be three mutually
perpendicular vectors each of unit = λ2 + 4λ2 = 5 λ2 = 5λ
Ê (a) Given, magnitude.
r r r Ifr r r r r
r r
| a | ≠ 0,| b| ≠ 0 70. In a triangle ABC, if taken in order,
r r r r r r r r A = a + b + c, B = a − b + c and consider the following statements
( a + b) ⋅ ( a + b) = | a |2 + | b|2 + 2 a ⋅ b r r r r
C = a − b − c , then which one of the −→ −→ −→ −→
r r r r
a ⋅ b = b⋅ a 1. AB + BC + CA = 0
following
r ris correct?
r −→ −→ −→ −→
r r
a⋅ b= 0 (a)| A | > | B | > | C |
r r r
2. AB + BC − CA = 0
r −→ −→ −→ −→

r
a ⊥ b (b)| A | = | B | ≠ | C |
r r r 3. AB − BC + CA = 0
r (c)| A | = | B | =| C | −→ −→ −→ −→
64. If r = xi$ + yj$ + zk$, then what is r r r
(d)| A | ≠ | B | ≠| C | 4. BA − BC + CA = 0
r
r ⋅ (i$ + $j + k$ ) equal to ? How many of the above statement
Ê (c) Givenrthat,
(a) x (b) x + y r r r are correct?
A =a + b+ c
(c) − ( x + y + z) (d) ( x + y + z) r r (a) One (b) Two
r
r B=a − b+ c (c) Three (d) Four
Ê (d) r = $xi + yj + zk
$ $
12 NDA/NA Solved Paper 2018 (II)

1 1
Ê (a) We know that, Ê (a) We have, (a) − (b)
24 24
In a triangle ABC  sin 2 x
,x ≠ 0 1 1
−→ −→ −→ −→  (c) − (d)
f( x ) =  5 x
AB + BC + CA = O [Q by triangle law] 4 3 4 3
 2 ,x = 0
 15
So, only first statement is correct. Ê (a) We have,
at x = 0
f( x ) = 25 − x 2
71. Let the slope of the curve lim f( x )
sin 2 x × 2 2 f( x ) − f(1)
y = cos − 1 (sin x ) be tanθ. Then the = lim
x → 0 x→ 0 5 x × 2
=
5
Now, lim
x→1 x−1
value of θ in the interval (0, π ) is 2
π 3π π π f( 0) = lim 25 − x 2 − 24
(a) (b) (c) (d) 5 =
6 4 4 2 x→ 1 x−1
∴ lim f( x ) ≠ f( 0)
Ê (b) We have, x→ 0 − 2x
−0
y = cos − 1 (sin x ) at x = 0 function is discontinuous. 2 25 − x 2
= lim
differentiation w.r.t. x, we get 74. For the function f ( x ) = | x − 3,| x→1 1− 0
dy −1
= cos x which one of the following is not [by L-Hosptial]
dx 1 − sin2 x 1
correct? =−
− cos x (a) The function is not continuous at 24
= [Q sin2 x + cos 2 x = 1]
x = −3  5 − 2 tan
cos x
x
dy (b) The function is continuous at x = 3 77. If y = tan − 1   , then
= −1 (c) The function is differentiable at x = 0  2 + 5 tan x
dx
(d) The function is differentiable at x = −3
Slope of the curve = tan θ what is
dy
equal to?
∴ tan θ = − 1 Ê (a) We have, dx
π f( x ) =| x − 3 | 1
⇒ tanθ = − tan (a) − (b) 1
4 We know that, modulus function is 2 x
π 1
tanθ = tan  π − 
 continuous in R. (c) − 1
⇒ (d)
 4 So, option (a) is incorrect. 2 x
[Q θ ∈ ( 0, π )] 2x − sin − 1 x Ê (a) We have,
3π 75. If the function f ( x ) = −1  5 − 2 tan x 
⇒ θ= 2x + tan x y = tan− 1  
4
 2 + 5 tan x 
is continuous at each point in its
x −1  5 − tan x 
72. If f ( x ) = , defines a function domain, then what is the value of  
x−4 f (0)? y = tan −1
 2 
5
 1 + tan x 
on R, then what is its domain ? 1 1 2
(a) − (b) (c) (d) 2  2 
(a) ( − ∞, 4) ∪ ( 4, ∞ ) 3 3 3
(b) [4, ∞ ) Let,
5
= tan A
(c) (1, 4) ∪ ( 4, ∞ ) Ê (b) We have, 2
(d) [1, 4) ∪ ( 4, ∞ ) 2 x − sin− 1 x  tan A − tan x 
f( x ) = = tan− 1  
Ê (d) We have, 2 x + tan− 1 x
 1 + tan A tan x 
x−1 Function is continuous at each point
f( x ) = = tan− 1 [tan( A − x )] = A − x
x−4 ∴ f( 0) = lim f( x ) 5
∴ x − 1≥ 0
x→ 0 y = tan− 1 − x
2 x − sin− 1 x 2
and x−4≠0 Now, lim f( x ) = lim
x→ 0 x→ 0 2 x + tan− 1 x differentiation w.r.t x, we get
⇒ x≥1
1 dy 1
x≠4 2− =−
1 − x2 dx 2 x
So, x ∈ [1, 4) ∪ ( 4, ∞ ) = lim
Domain = [1, 4) ∪ ( 4, ∞ ) x→ 0
2 +
1 78. Which one of the following is
1 + x2 correct in respect of the function
73. Consider the function
[by L-Hospital] 1
 sin 2x f ( x ) = x sin x + cos x + cos 2 x ?
 , if x ≠ 0 =
2 −1 1
= 2
f ( x ) =  5x 2 +1 3 π
(a) It is increasing in the interval  0, 
2
 , if x = 0 1  2
 15 ∴ f( 0) =
3 (b) It remains constant in the interval
 0, π 
Which one of the following is  
76. If f ( x ) = 25 − x 2 , then what is  2
correct in respect of the function?
π
(a) It is not continuous at x = 0 f ( x ) − f ( 1) (c) It is decreasing in the interval  0, 
(b) It is continuous at every x lim equal to?  2
(c) It is not continuous at x = π
x →1 x −1  π π
(d) It is decreasing in the interval  
(d) It is continuous at x = 0  4, 2 
NDA/NA Solved Paper 2018 (II) 13

Ê (a) We have, Ê (d) We have,


5
∫ sin
1 3
f( x ) = x sin x + cos x + cos 2 x x cos x dx
2
1 2 Let, sin x = t
f ′ ( x ) = x(cos x ) + sin x − sin x + cos x dx = dt
2 1
⋅ 2 cos x ( − sin x )
1 2 4
= ∫ t 3 dt
f ′ ( x ) = x cos x − sin x cos x
At x=2,y=1 t4
By checking options, we put = +C
At x = 1 , y = (1)2 − 4 (1) + 5 = 2 4
π
x= sin4 x
4 At x = 4 y = ( 4)2 − 4 ( 4) + 5 = 5 = +C
π π π π 4
f ′ ( x ) = cos − sin cos So, y ∈ [1, 5)
(sin2 x )2
4 4 4 4 Range = [1, 5) = +C
π 1 1 1 4
= ⋅ − ×
81. What is ∫ [ x ] dx + ∫ [ − x ] dx equal
b b
4 2 2 2 (1 − cos 2 x )2
a a = +C
π 1 4
= − >0 to, where [,] is the greatest integer
4 2 2 [Q sin2 x + cos 2 x = 1]
function?
π
So, f( x ) is increasing in the interval  0,  (a) b − a (b) a − b ln (tan x )
 2 (c) 0 (d) 2( b − a )
84. What is ∫ e dx equal to
(a) ln| tan x | + C
1 − cos θ Ê (b) We have, (b) ln| sec x | + C
79. What is lim equal to? b b
θ→ 0 θ ∫a [ x] dx + ∫a [− x ] dx (c) tan x + C
(d) e tan x + C
(a) 2 (b) 2 2 b

1 1
= ∫a ([ x ] + [− x ]) dx
Where C is the constant of
(c) (d) −
2 2 2 [Q[ x ] + [− x ] = − 1, if x ∈
/ z] integration.
b
Ê (c) We have,
= ∫a ( − 1) dx Ê (b) We have,
∫e [Q eln ( x ) = x ]
ln (tan x )
1 − cos θ = − ( x )ba = − ( b − a ) = a − b dx
lim
θ→ 0 θ 8 = ∫ tan x dx
θ 82. What is ∫ | x − 5 | dx equal to?
1 −  1 − 2 sin2  2 = log| sec x | + C
 2 (a) 2 (b) 3 (c) 4 (d) 9
= lim
θ→ 0 θ
1
d  − 1 1 
Ê (d) We have, 85. What is ∫   tan   dx equal
2θ 8
− 1
dx  x 
2 sin
2 ∫2 | x − 5| dx
= lim to ?
θ→ 0 θ 5 8

θ
= ∫2 | x − 5| dx |+ ∫5 | x − 5| dx (a) 0 (b) −
π
2 sin b c b 4
= lim 2 [Q ∫ f( x ) dx = ∫ f( x ) dx + ∫ f( x ) = dx ] π π
θ→ 0 θ a a c (c) − (d)
×2 5 8 2 2
2 =− ∫2( x − 5) dx + ∫5( x − 5) dx
=
1 Ê (d) We have,
5 8
2  x2   x2  1
d  − 1 1 
=−
 2
− 5 x + 
2  2
− 5 x
5
∫  dx  tan

 dx
x  
80. A function f : A → R is defined by −1
  25   1
the equation f ( x ) = x 2 − 4 x + 5, = −  − 25 − (2 − 10)  1 
=  tan− 1   
 2    x   −1
where A = (1, 4 ). What is the range 
 
+ ( 32 − 40) −  − 25 
25
of the function?
2  = tan− 1(1) − tan− 1 ( − 1)
(a) (2, 5) (b) (1, 5) 
25 25 π π
(c) [1, 5) (d) [1, 5] = −8−8+ =9 = +
2 2 4 4
Ê (c) We have, π
A function f : A → R is defined by 83. What is ∫ sin 3 x cos x dx equal to ? =
2
f( x ) = x 2 − 4 x + 5, (a) cos 4 x + C
(b) sin4 x + C
86. In which one of the following
Where, A = (1, 4)
(1 − sin2 x )2 intervals is the function
Let, y = x − 4x + 5
2
+C
(c)
4 f ( x ) = x 2 − 5x + 6 decreasing?
dy
= 2x − 4 (1 − cos x )
2 2
(a) ( − ∞, 2 ] (b) [3, ∞ )
dx (d) +C
4 (c) ( − ∞, ∞ ) (d) (2, 3)
dy
Now, =0
dx Where C is the constant of Ê (a) We have,
⇒ 2x − 4 = 0 integration. f( x ) = x 2 − 5 x + 6
⇒ x=2 f ′( x) = 2 x − 5
14 NDA/NA Solved Paper 2018 (II)

1 1
For decreasing −2 = + +C 91. If u = e ax sin bx and v = e ax cos bx ,
f ′( x) < 0 3 2
du dv
2x − 5< 0 5 then what is u +v equal to?
⇒ −2 = + C dx dx
5 6
x< (a) ae 2ax (b) ( a 2 + b 2 )e ax
2 −5
⇒ C = −2 (c) ab e 2ax (d) ( a + b ) e ax
x < 2.5 6
− 17 Ê (a) We have,
x ∈ ( − ∞, 2.5) ⇒ C = u = e ax sin bx
6
87. The differential equation of the Put in Eq. (i)
differentiation. w. r. t. x, we get
family of curves du
− x3 1 17 = e ax ( b cos bx ) + sin bx ( ae ax )
y = p cos (ax ) + q sin (ax ), where p ,q y= + − dx
3 2 x2 6 du
are arbitrary constants, is = e ax ( b cos bx + a sin bx ) ...(i)
d 2y − 2 x 5 + 3 − 17 x 2 dx
(a) − a2 y = 0 ⇒ y=
dx 2 6 x2 Now, v = e ax cos bx
d 2y ⇒ 6 x y = − 2 x 5 + 3 − 17 x 2
2
differentiation. w. r. t. x, we get
(b) − ay = 0
dx 2 ⇒ 6 x 2 y + 2 x 5 + 17 x 2 − 3 = 0 dv
= e ax ( − b sin bx ) + cos bx ( ae ax )
d 2y dx
(c) + ay = 0
dx 2 89. What is the order of the differential = e ax ( − b sin bx + a cos bx )
d 2y equation whose solution is Now we have,
(d) + a y=0 2
dx 2 y = a cos x + b sin x + ce − x + d , du dv
u + v
Ê (d) We have, where a, b, c and d are arbitrary dx dx
y = pcos ( ax ) + q sin ( ax ) constants? = e ax sin bx [e ax ( b cos bx + a sin bx )]
differentiation. w. r. t. x, we get (a) 1 (b) 2 (c) 3 (d) 4 + e ax cos bx [e ax ( − b sin bx + a cos bx )]
dy
= − p a sin ax + q a cos ax Ê (d) We have, = e 2ax [b sin bx cos bx + a sin2 bx ] + e 2ax
dx y = a cos x + b sin x + ce − x + d , ( − b cos bx sin bx + a cos 2 bx )
Again, differentiation. w. r. t. x, we get a, b, c and d are arbitrary constants. =e 2ax
[a sin2 bx + a cos 2 bx ]
d 2y We know that,
= − pa cos ax − qa sin ax
2 2
= ae 2ax
dx 2 order = number of arbitrary constant

d 2y
= − a 2 ( pcos ax + q sin ax ) So, order = 4 92. If y = sin (ln x ), then which one of
dx 2 the following is correct?
90. What is the solution of the
d 2y d 2y
⇒ = − a2 y differential equation (a) + y=0
dx 2
 dy  dx 2
ln   = ax + by ? d 2y

d 2y
+ a2 y = 0  dx  (b) =0
dn2
dx 2 (a) ae ax + be by = C d 2y dy
1 1 (c) x 2 +x + y=0
88. The equation of the curve passing (b) e ax + e by = C dx 2 dx
through the point ( −1, − 2), which a b 2
d y
(d) x 2 − x
dy
+ y=0
dy 1 (c) ae ax + be − by = C
= − x 2 − 3 , is dx dx
satisfies 1 1
dx x (d) e ax + e − by = C Where C is the constant of
a b
(a) 17 x 2 y − 6x 2 + 3x 5 − 2 = 0 integration
(b) 6x 2 y + 17 x 2 + 2 x 5 − 3 = 0 Ê (d) We have,
Ê (c) We have,
(c) 6xy − 2 x 2 + 17 x 5 + 3 = 0 ln   = ax + by
dy
y = sin (loge x )
(d) 17 x 2 y + 6xy − 3x 5 + 5 = 0  dx 
differentiation. w. r. t. x, we get
Ê (b) We have, dy
= e ax + by dy cos (loge x )
=
dy 1 dx
= − x2 − 3 dx x
dx x dy dy
= e ax ⋅ e by x = cos (loge x )
dy =  − x 2 − 3  dx
1 dx dx

 x  dy Again, differentiation. w. r. t. x, we get
= e ax dx
Integrating both sides e by d 2 y dy − sin (loge x )
x 2 + =
 1
∫ dy = ∫  − x − x 3  dx
2 Integrating both sides dx dx x
− by
− 2
∫e dy = ∫ e axdx
⇒ x2
d 2y
+ x
dy
= − sin (loge x )
x x 3
dx 2 dx
⇒ y=− −  +c e − by e ax
3 −2 ⇒ = +C d 2y dy
−b a ⇒ x2 + x =− y
− x 1 3 dx 2 dx
⇒ y=
+ +C ...(i) e ax e − by
3 2 x2 ⇒ + +C =0 d 2y dy
a b ⇒ x2 + x + y=0
given curve is passing through ( − 1, − 2 ) dx 2 dx
NDA/NA Solved Paper 2018 (II) 15

π d

93. A flower-bed in the form of a sector 2 6 ( 6 ln 2 + 3π) = (sin x − cos x )
has been fenced by a wire of 40 m (c) dx
length. If the flower-bed has the π
6 Q π < x < π ∴ sin x > cos x ]
 4

greatest possible area, then what is 2 6 ( 6 ln 2 − 3π) 2
(d)
the radius of the sector? 6 = cos x − ( − sin x )
(a) 25 m (b) 20 m = cos x + sin x
Ê (a) We have,
(c) 10 m (d) 5 m
y = | sin x || x |
∫ a 2 sin 2 x + b 2 cos 2 x
dx
97. What is
Ê (c) Let radius of sector be r and angle π
subtended at centre be θ. ∴ x=− ,
6 equal to ?
tan− 1 
sin x < 0, x < 0 1 a tan x 
(a) C + 
r rθ  b 
y = ( − sin x )− x ab
tan− 1  
1 a tan x
θ y = f( x )g ( x ), So (b) C − 
r
Q ab  b 
tan− 1 
dy  g ( x)  1 b tan x 
∴ r + r + rθ = 40 = f( x )g ( x )  f ′ ( x ) + log| f( x )| ⋅ g ′ ( x ) (c) C + 
dx  f( x )  ab  b 
⇒ 2 r + rθ = 40 dy (d) None of the above
−x
40 − 2 r = ( − sin x )
⇒ θ= dx Where C is the constant of
r
 ( − x)  integration.
1 2  ⋅ ( − cos x ) + log| ( − sin x )| ( − 1)|
Area of the sector = r θ −
2  ( sin x )  Ê (a) We have,
dx
A= r 
1 2  40 − 2 r 


dy   −
π
π 6 ∫ a 2 sin2 x + b 2 cos 2 x
2   x = − π / 6 =  − sin  
 6  
r
dx   Divide numerator and denominator by
1
A=r ( 40 − 2 r ) 
cos 2 x
2  π/6 π 
1  − cos  −   I=∫ 2
sec 2 x dx
A = ( 40r − 2 r 2 )  − sin ( − π / 6)  6
2 a tan2 x + b 2
π 
differentiating. w. r. t. r, − log| − sin  −   Let a tan x = t
 6 a sec 2 x dx = dt
dA 1
= ( 40 − 4r )
dr 2 π − π  dt
 6  3 sec 2 x dx =
π
=  sin 
6  1 
Now,
dA
=0  1   − log   a
 6  2  2 1 dt
dr   I= ∫ 2
 2  a t + b2
⇒ 40 − 4r = 0
π
× tan− 1   + C
⇒ r = 10m − 3π 1 1 1 t
=  
1 6
− log  =
2  a b  b
94. What is the minimum value of  6 2 
 1 1 x 
1
 3π  Q ∫ 2 dx = tan− 1 + C 
[ x ( x − 1) + 1]3 , where a ≤ x ≤ 1? = 2 − π / 6 log 2 −   x + a 2
a a 
 6 
1 1 − 1  a tan x 
π = tan   +C
(a)  
3 3 −  6 log 2 − 3π   b 
(b) 1 =2 6 ab
 4  
1/ 3  6 
(c)
1
(d)  
3 98. Let f ( x + y ) = f ( x ) f (y ) and
2  8 d 1 − sin 2x
96. What is equal to, f ( x ) = 1 + xg ( x ) φ ( x ), where
Ê (a) We have, dx
lim g ( x ) = a and lim φ ( x ) = b.
π π
where < x < ?
1
x→ 0 x→ 0
y = ( x 2 − x + 1)3 4 2 Then, what is f ′( x ) equal to?
1 (a) cos x + sin x (a) 1 + abf ( x ) (b) 1 + ab
y =  x 2 − x + − + 1
1 1 3 (c) ab (d) ab f ( x )
(b) − (cos x + sin x )
 4 4  (c) ± (cos x + sin x )
1
Ê (d) Let, f( x + y) = f( x) f( y)
(d) None of the above and f( x ) = 1 + x g ( x ) ⋅ φ( x )
at x = , y is minimum y =  
1 3 3
2  4 Ê (a) We have, Thus, f( y) = 1 + yg ( y) φ ( y)
d π π Where lim g ( x ) = a and lim φ( x ) = b
1 − sin2 x, < x <
95. If y = | sin x || x | , then what is the dx 4 2 x→ 0 x→ 0

dy π d using first principal


value of at x = − ? = cos 2 x + sin2 x − 2 sin x cos x
6 dx f( x + y) − f( x )
dx
π
f ′ ( x ) = lim
y→ 0
−  Q sin2 x + cos 2 x = 1  y
2 6 ( 6 ln 2 − 3π)  
(a) f( x ) ⋅ f( y) − f( x )
6  and sin2 x = 2 sin x cos x  = lim
π y→ 0 y
d
26 ( 6 ln 2 + 3π) = | cos x − sin x | f( x )[f( y) − 1]
(b) dx = lim
6 y→0 y
16 NDA/NA Solved Paper 2018 (II)

 1 + yg ( y)φ( y) − 1 (sin x + 1) (2 sin x − 1) and in group of women, number of


lim f( x )  = lim
y→ 0  π (sin x − 1) (2 sin x − 1) women = b (let)
 y  x→
6
= f( x ) lim g ( y) lim φ( y) sin x + 1 X 2 = 21 yr
y→ 0 y→ 0 = lim `
x→
π sin x − 1 and combined mean X = 25
= f( x ) ⋅ a ⋅ b 6
= ab f( x ) n1 X 1 + n2 X 2
1 3 Now, X =
+1 n1 + n2
99. What is the solution of the differential = 2 = 2
1 1 26a + 21b
dx x + y + 1 −1 − ⇒ 25 =
equation = ? 2 2 a+ b
dy x + y − 1 =−3 ⇒ 25a + 25b = 26a + 21b
(a) y − x + 4 ln ( x + y) = C
(b) y + x + 2 ln ( x + y) = C 101. If two dice are thrown and atleast ⇒ 4b = a
(c) y − x + ln ( x + y) = C one of the dice shows 5, then the ⇒
a 4
=
(d) y + x + 2 ln ( x + y) = C probability that the sum is 10 or b 1
Where C is an arbitrary constant. more is By checking options (d) is correct.
1 4
dx x+ y+1 (a) (b) 104. If sinβ is the harmonic mean of sinα
Ê (c) We have, = 6 11
dy x+ y−1 3 2 and cosα and sinθ is the arithmetic
(c) (d)
Let x+ y=u 11 11 mean of sinα and cosα, then which
dx du of the following is/are correct?
+ 1= Ê (c) Let A be event of dice shows 5 and B
dy dy be the event that the sum is 10 or more  π
1. 2 sin α +  sin β = sin 2α

dx du
= −1 Here, n(S ) = 36  4
dy dy n( A ) = { (1, 5), (2, 5), ( 3, 5), ( 4, 5), ( 5, 5),
u +1  π

du
− 1= ( 6, 5),( 5, 1), ( 5, 2 ), ( 5, 3), ( 5, 4), ( 5, 6) } 2. 2 sin θ = cos α − 
u −1  4
dy n( B) = { ( 5, 5), ( 6, 4), ( 4, 6), ( 6, 5),
du u + 1 ( 5, 6), ( 6, 6) } Select the correct answer using the
⇒ = +1
dy u − 1 n( A ∩ B) = { ( 5, 5), ( 6, 5), ( 5, 6) code given below.
du u + 1+ u − 1 3 (a) 1only (b) 2 only
⇒ =
dy u −1 
P  =
B 36 (c) Both 1 and 2 (d) Neither 1 nor 2
 A 11

dy
=
2u Ê (c) Given that,
36
dy u − 1 sinβ is HM of sinα and cosα
  B  P ( B ∩ A ) 2 sin α cos α
⇒  u − 1 du = 2 dy Q P  A  = P( A )  So, sin β =
   sin α + cos α
 u 
3
Integrating both sides, = . ⇒ sin β (sin α + cos α ) = sin 2α
11
⇒ sin β × 2  
 1 1 1
sinα + cos α 
∫  1 − u  du = ∫ 2 dy 102. The correlation coefficient computed  2 2 
from a set of 30 observations is 0.8. = sin 2α
⇒ u − log u = 2 y + C
Then the percentage of variation not π π
[Q u = x + y] explained by linear regression is ⇒ sin β × 2  cos sinα + sin cos α 

 4 4 
⇒ x + y − log ( x + y) = 2 y + C (a) 80% (b) 20%
⇒ x − log ( x + y) = y + C (c) 64% (d) 36% = sin 2α
 π 
⇒ y − x + log ( x + y) = C Ê (b) Given that, ⇒ 2 sin β sin  α +   = sin 2α
  4
2 sin x + sin x − 1
2 correlation coefficient = 0.8 = 80%, if the
100. What is lim relation is 80% explained, then 20% of  π
π 2 sin x − 3 sin x + 1
2 ⇒ 2 sin  α +  sin β = sin 2α
x→ variation will not explained by near  4
6
regression. Statement I is true.
equal to?
1 1 103. The average age of a combined Now, sinθ is AM of sin α and cosα
(a) − (b) −
2 3 group of men and women is 25 yr. If sin α + cos α
sinθ =
(c) − 2 (d) − 3 the average age of the group of men 2
is 26 yr and that of the group of ⇒ 2 sinθ = sin α + cos α
Ê (d) We have,
women is 21 yr, then the percentage
⇒ 2 sinθ = 2  
1 1
2 sin2 x + sin x − 1 sinα + cos α 
lim
π
of men and women in the group is  2 2 
x→ 2 sin2 x − 3 sin x + 1
6 respectively ⇒
2 sin2 x + 2 sin x − sin x − 1 (a) 20, 80 (b) 40, 60 π π
= lim
π (c) 60, 40 (d) 80, 20 2 sinθ = 2  sin sinα + cos cos α 
x→ 2 sin x − 2 sin x − sin x + 1
2
 4 4 
6
Ê (d) In group of men, let number of men π
= lim
2 sin x (sin x + 1) − (sin x + 1)
=a ⇒ 2 sin θ = cos  α − 
π 2 sin x (sin x − 1) − 1 (sin x − 1)  4
x→
6
X 1 = 26 yr and n1 = a (let)
Hence, Statement II is also correct.
NDA/NA Solved Paper 2018 (II) 17

P   = 4% =
E 4
105. Let A , B and C be three mutually Ê (a) In first group, 3 girls and 1 boy
 B 100 3 1
exclusive and exhaustive events P(G ) = and P( B) =
P   = 5% =
associated with a random E 5 4 4
experiment. If P ( B ) = 15 . P ( A ) and C  100 In second group, 2 girls and 2 boys
P (C ) = 05
. P ( B ), then P ( A ) is equal to Probability of defective bulb that was 2 1
P(G ) = =
3 4 manufactured by machine X, 4 2
(a) (b)
4 13 2 1
P( A ) ⋅ P   and P( B) = =
E
2 1
(c) (d)  A  A 4 2
3 2 P  =
 E
P( A ) ⋅ P   + P( B) ⋅ P  
E E One child is selected at random from
Ê (b) We have,  A  B each group and consists of 1 girl and 2
3 boys are GBB or BGB or BBG.
P( B) = 15
. P( A ) =
+ P(C ) ⋅ P  
P( A ) E
2 So, required probability
C 
1 3 1 3 1 1 3
and P(C ) = 0.5 P( B) = P( B) 25 2 = × × + × ×
2 × 4 2 4 4 2 4
1 3 = 100 100
= × P( A ) 1 1 1
25 2 35 4 40 5 + × ×
2 2 × + × + × 4 2 4
100 100 100 100 100 100
3 9 3 1 13
= P( A ) 25 × 2 = + + =
4 = 32 32 32 32
25 × 2 + 35 × 4 + 40 × 5
Now, A, B and C are mutually exclusive
50 109. Consider the following statements :
and exhaustive events =
So, 50 + 140 + 200 1. If 10 is added to each entry on a
P( A ) + P( B) + P(C ) = 1 50 5 list, then the average increases by
= =
3 3 390 39 10.
⇒ P( A ) + P( A ) + P( A ) = 1
2 4 2. If 10 is added to each entry on a
107. 8 coins are tossed simultaneously. list, then the standard deviation
⇒ 
P( A ) 1 + +
3 3
=1 The probability of getting atleast 6
 2 4  increases by 10.
heads is 3. If each entry on a list is doubled
P( A )   = 1
13
⇒ 7 57
 4 (a) (b) then the average doubles.
64 64
4 Which of the above statements are
⇒ P( A ) = 37 229
13 (c) (d) correct?
256 256
(a) 1, 2 and 3 (b) 1 and 2
106. In a bolt factory, machines X ,Y , Z Ê (c) We have, (c) 1 and 3 (d) 2 and 3
manufacture bolts that are 8 coins are tossed simultaneously i.e
respectively 25%, 35% and 40% of the 1 Ê (c) Statement 1 The average is affected
n = 8 probability of getting head p = , by the change of the origin.
factory’s total output. The machines 2
X , Y , Z respectively produce 2%, 4% 1 So, if 10 is added to each entry on list then
so q = 1 − p = . Probability of getting average increase by 10. Statement 1 is
and 5% defective bolts. A bolt is 2
correct.
drawn at random from the product atleast 6 heads.
and is found to be defective. What is 6 2 7 Statement 2 Standard deviation is
= 8C 6     + 8C 7  
1 1 1
independent on change in origin.
the probability that it was 2 2 2
Statement 2 is incorrect.
manufactured by machine X ? 1 8
 1  + 8C  1  Statement 3 The average is affected by
(a)
5
(b)
14   8  
2 2 change in scale in same ratio as each
39 39
20 34 8 entry is changes.
=   ( 8C 6 + 8C 7 + 8C 8 )
(c) (d) 1
39 39 2 So, Statement 3 is correct.

Ê (a) Let 8
110. The variance of 25 observations is 4.
=   (28 + 8 + 1) =
1 37
A : bolt manufactured from machine 2 256 If 2 is added to each observation,
B : bolt manufactured from machine then the new variance of the
C : bolt maufactured from machine 108. Three groups of children contain resulting observations is
and E : bolt is defective 3 girls and 1 boy; 2 girls and 2 boys; (a) 2 (b) 4
25 1 girl and 3 boys. One child is (c) 6 (d) 8
∴ P( A ) = 25% =
100 selected at random from each group.
Ê (b) We know that,
35 The probability that the three
P( B) = 35% = Variance is independent on change in
100 selected consist of 1 girl and 2 boys is origin.
40 13 9
P(C ) = 40% = (a) (b) So, if 2 is added to each observation, then
100 32 32
variance is remain same
and P   = 2% =
E 2 3 1
(c) (d) Hence, variance = 4
 A 100 32 32
18 NDA/NA Solved Paper 2018 (II)

111. If x i > 0, yi > 0 (i = 1, 2, 3,..., n ) are the 113. If the regression coefficient of Y on (1, 9), (10, 2 ), (2, 10), ( 9, 0), ( 0, 9),
values of two variables X andY with X is −6 and the
correlation (10, 1), (1, 10) , (10, 0), ( 0, 10) }
1 n( E ) = 30
geometric means P and Q coefficient between X and Y is − ,
respectively, then the geometric 2 So, required probability =
30
X then the regression coefficient of X 121
mean of is
Y on Y would be 116. An analysis of monthly wages paid
1 1
(a)
P (a) (b) − to the workers in two firms A and B
Q 24 24
1 1 belonging to the same industry gives
(b) antilog   (c) − (d)
P
Q 6 6 the following result
(c) n (log P − log Q ) Ê (b) Given that, Firm A Firm B
(d) n (log P + log Q ) b yx = − 6
Number of 500 600
Ê (a) We have, xi > 0, yi > 0, and rxy = −
1
workers
(i = 1, 2, 3,...., n) 2
1 We know that, Average ` 1860 ` 1750
P = ( x1 × x2 × x3 × ..... xn )n monthly wage
rxy = b yx × b xy
1 1 Variance of 81 100
⇒ − = − 6 × b xy
and Q = ( y1 × y2 × y3 × ......× yn )n 2 distribution of
x wages
Now, geometric mean of Squaring both sides,
y 1
⇒ = − 6 × b xy The average of monthly wage and
1 4
x x x n 1 variance of distribution of wages of
=  1 × 2 × ....× n  ⇒ b xy = −
 y1 y2 yn  24 all the workers in the firms A and B
P taken together are
= 114. The set of bivariate observations (a) ` 1860, 100
Q
( x 1, y1 ), ( x 2 , y 2 )....( x n , yn ) are such (b) ` 1750, 100
112. If probability of simultaneous that all the values are distinct and all (c) ` 1800, 81
occurrence of two events A and B is the observations fall on a straight (d) None of the above
p and the probability that exactly line with non-zero slope. Then the Ê (d) For firm A
one of A , B occurs is q, then which of possible values of the correlation n1 = 500, X 1 = 1860
the following is/are correct? coefficient between x and y are
and variance = σ 12 = 81
(a) 0 and 1 (b) 0 and −1
1. P ( A ) + P ( B ) = 2 − 2p − q (c) 0, 1 and −1 (d) −1 and 1 So, S.D. = σ 1 = 9
2. P ( A ∩ B ) = 1 − p − q
Ê (d) Given that, For firm B
Select the correct answer using the All the observations fall on a straight line n2 = 600, X 2 = 1750
code given below. with non-zero, slope then if slope is
and variance σ 22 = 100
(a) 1 only positive then r = 1
(b) 2 only and if slope is negative then r = −1 So, S.D. = σ = 10
(c) Both 1 and 2 So, values of the correlation coefficient n1 X 1 + n2 X 2
Now, combined mean X =
(d) Neither 1 nor 2 between x and y are −1 and 1. n1 + n2
Ê (c) Given that, 115. Two integers x and y are chosen =
500 × 1860 + 600 × 1750
P( A ∩ B) = P 500 + 600
with replacement from the set
and P( A ) + P( B) − 2 P ( A ∩ B) = q
[0, 1, 2, ...., 10]. The probability that X = 1800
Now, Statement 1 | x − y | > 5 is Combined variance
Given, 6 35
(a) (b) n1 ( σ 12 + d 12 ) + n2 ( σ 22 + d 22 )
P( A ) + P( B) − 2 P ( A ∩ B) = q 11 121 =
30 25 n1 + n2
⇒ 1 − P( A ) + 1 − P( B ) − 2 p = q (c) (d)
⇒ P( A ) + P( B ) = 2 − 2 p − q
121 121 500 [81 + ( − 60)2 ] + 600 [100 + ( 50)2 ]
=
Statement 1 is correct Ê (c) Given that, 500 + 600
S = { 0, 1, 2, ..., 10 }
For Statement 2 Q d 1 = X − X1 = 1800 − 1860
n(S ) = 11 × 11 = 121
P( A ∩ B ) = 1 − P( A ∪ B) = − 60
Now, x − y > 5
= 1 − [P( A ) + P( B) − P( A ∩ B)] and d 2 = 1800 − 1750 = 50
Now, E is the set of element such that
= 1 − [P( A ) + P( B) − 2 P( A ∩ B) 500 ( 81 + 3600) + 600 (100 + 2500)
| x − y| > 5 =
+ P( A ∩ B)] E = { ( 6, 0), ( 0, 6), (7, 1), (1, 7 ), ( 8, 2 ),
1100
= 1 − [q + p] 500 ( 3681) + 600 (2600)
(2, 8), ( 9, 3), ( 3, 9), (10, 4), ( 4, 10), (7, 0), =
= 1− q − p ( 0, 7 ), ( 81
, ), (18
, ), ( 9, 2 ), (2, 9), (10, 3), 1100
Statement 2 is also correct. ( 3, 10), ( 8, 0), ( 0, 8), ( 9, 1), = 309136
.
NDA/NA Solved Paper 2018 (II) 19

117. Three dice having digits 1, 2, 3, 4, 5 Case III If x = 4 which one of the following is
and 6 on their faces are marked I, II, Then, possible ways correct?
and III and rolled. Let x, y and z = { ( 4, 3, 2 ), ( 4, 3, 1), ( 4, 2, 1) (a) y ≥ x (b) y ≤ x
(c) x = y (d) x < y
represent the number on die-I, die-II So, possible ways = 3
and die-III, respectively. What is the Case IV If x = 3 Ê (d) We know that,
number of possible outcomes such 4
MD =S.D.
Then, possible ways = 1 ( 3, 2, 1)
that x > y > z ? 5
(a) 14 (b) 16 So, required possible outcomes ⇒ 5MD = 4.S.D.
(c) 18 (d) 20 = 10 + 6 + 3 + 1 ⇒ 5x = 4y
Ê (d) Three dice having digit 1, 2, 3, 4, 5 and = 20 [Q MD = x and SD = y]
6 and given that x > y > z. ∴ x< y
118. Which one of the following can be
So, possiblities are obtained from an ogive? 120. In which one of the following cases
Case I If x = 6 (a) Mean would you expect to get a negative
x> y> z (b) Median
(c) Geometric Mean correlation?
Possible ways (a) The ages of husbands and wifes
(d) Mode
= { ( 6, 5, 1), ( 6, 5, 2 ), ( 6, 5, 3), ( 6, 5, 4) (b) Shoe size and intelligence
( 6, 4, 1), ( 6, 4, 2 ), ( 6, 4, 3) Ê (b) We know that, median is always (c) Insurance companies profits and the
calculated from less than or more than
( 6, 3, 2 ), ( 6, 3, 1) ( 6, 2, 1) number of claims they have to pay
type ogive curve where both curve cuts
So, possible ways = 10 (d) Amount of rainfall and yield of crop
each other that point median.
Case II If x = 5
119. In any discrete series (when all Ê (c) In negative correlation, if x is
Then, possible ways increases then y is decreases by
values are not same), if x represents checking options Insurance companies
= { ( 5, 4, 3), ( 5, 4, 2 ), ( 5, 4, 1) mean deviation about mean and y profits and the number of claims they
( 5, 3, 2 ), ( 5, 3, 1), ( 5, 2, 1)} represents standard deviation, then have to pay are negatively correlated.
So, possible ways = 6

PAPER II English Language and General Studies


Part A (English Language)
Directions (Q.Nos. 1-5) Each item 3. This view has been taken Ê (c) Part (c) has error of correct use of
in this section has a sentence with (a) verb. As noun clause is in present perfect
three underlined parts labelled (a), (b) by one of the ablest persons tense, use of ‘grow up’ is incorrect. Will
and (c). Read each sentence to find out be used Past participle of ‘grow’ ie
(b)
whether there is any error in any ‘grown up’.
who has written on this subject.
underlined parts. If you find no error,
your response should be indicated as
(c) Directions (Q.Nos. 6-10) Each item
No error in this section consists of a sentence
(d). (c) with an underlined word followed by
1. He appears to be a honest man. Ê (c) Use of verb ‘has’ should be replaced four words or groups of words. Select
(a) (b) (c) by plural ‘have’ as to agree with the the word or group of words that is
subject ‘persons’.
No error nearest in meaning to the
(c) 4. Slow and steady owns the race, underlined word.
Ê (c) Before honest article ‘an’ should be (a) (b) 6. The Industrial Revolution saw a
used as ‘h’ is mute here and the word as the wise would say. No error massive rise in the population of
gives us vowel sound. (c) (d) Europe.
(a) enormous (b) erroneous
2. One of the members expressed doubt Ê (b) Part (b) has error as owns will be
replaced by wins. If is a popular proresh. (c) hazardous (d) perilous
if
(a) (b) Ê (a) ‘enormous’ is nearest in meaning to
5. We often hear people say the word ‘massive’ which means large
the Minister was an atheist. (a) and heavy.
(c) that most human beings have not
No error evovled 7. I had some deepest convictions
(d) reflected in my work.
(b) (a) ideas and opinions
Ê (b) In place of ‘if’ we should use ‘that’ as it or grow up enough to stick to truth (b) firm beliefs
is a noun clause. or non-violence. No error (c) prejudices
(c) (d) (d) biases
20 NDA/NA Solved Paper 2018 (II)

Ê (b) ‘firm beliefs’ is nearest meaning to the Ê (c) ‘permit’ means to allow whereas prohibit ‘‘He-has-a-hen- up-his-sleeve,’’ and all the
word ‘conviction’ as both words mean a means to disallow or deny. So, it is the people whispered it on. ‘‘He-has-a-lot-of-
firmly held belief or opinion. Other options antoyism of permit. hens-up-his- sleeve’’.
are not suitable here.
15. None but the brave deserves the fair. The egg trick was ruined.
8. This boy is very timid. (a) ugly (b) coward It went on like that all through. It transpired
(a) courageous (b) shy (c) jealous (d) weak from the whispers of the Quick Man that the
(c) clever (d) dull
Ê (b) ‘coward’ is correct antonym of the word conjuror must have concealed up his sleeve, in
Ê (b) ‘shy’ is nearest in meaning to the word ‘brave’. ‘Brave’ means courageous or ready addition to the rings, hens and fish, several
‘timid’ which means showing lack of to face danger whereas ‘coward’ means a
courage or confidence.
packs of cards, a loaf of bread, a doll’s Cradle, a
person who lacks courage and turns away
live guinea pig, a fifty-cent piece and a rocking
from danger.
9. My friend is as stubborn as a mule. chair.
(a) observant (b) obnoxious
(c) obstinate (d) corpulent Directions (Q.Nos. 16-25) In this 16. ‘‘The brow of the conjuror was
section, you have two short passages. clouded with a gathering frown.’’
Ê (c) ‘obstinate’ is nearest in meaning to the
word ‘stubborn’ which means a person who After each passage, you will find some The sentence means that the conjuror
is determined not to change one’s atitude or items based on the passage. First, (a) was very pleased (b) was very sad
who is headstrong or uncompromising. read a passage and answer the items (c) was rather angry (d) was very afraid
10. His behaviour was deliberately based on it. You are required to select Ê (c) ‘was rather angry’ is correct option. All
provocative. your answers based on the contents of tricks of conjuror were ruined due to the
(a) exciting desire (b) infuriating the passage and opinion of the author Quick man’s remarks.
(c) pitiable only. 17. ‘‘The egg trick was ruined’’. This
(d) creating frustration
Passage-1 means that
Ê (b) ‘infuriating’ is nearest in meaning to the (a) eggs were all broken
word ‘provocative’ which means causing ‘‘Now, ladies and gentlemen,’ said the
(b) people were unconvinced
anger. conjuror, ‘having shown you that the cloth is
(c) conjuror was disappointed
absolutely empty. I will proceed to take from it
(d) the trick could not be performed
Directions (Q.Nos 11-15) Each item a bowl of goldfish. Presto !’’
in this section consists of a sentence All around the hall people were saying, ‘‘Oh, Ê (b) The egg trick was ruined means that
people were unconvinced.
with an underlined word followed by how wonderful ! How does he do it?’’
four words or groups of words. Select But the Quick Man on the front seat said in a 18. According to the Quick Man, the
the word or group of words that is big whisper to the people near him, conjuror
opposite in meaning to the ‘‘He-had-it-up-his-sleeve.’’ (a) had everything bought for production
underlined word. Then the people nodded brightly at the Quick (b) produced things with the magic he
knew
Man and said, ‘Oh, of course’; and everybody
11. Too many cooks spoil the broth. whispered round the hall, ‘‘He-had-it-
(c) had things in the large sleeves of his
(a) tarnish (b) wreck coat
(c) embellish (d) upset up-his-sleeve.’’
(d) created an illusion of things with his
‘‘My next trick,’’ said the conjuror, ‘‘is the magic
Ê (c) The word spoil means to diminish or
famous Hindostanee rings. You will notice
destroy. ‘Embellish’ means to improve or
that the rings are apparently separate; at Ê (c) According to the Quick man, the conjurer
enhance. So, it is correct antonym of ‘spoil’. had things in large sleeves of his coat.
a blow they all join (clang, clang, clang)—
12. He is biased against the students Presto !’’ 19. The author believes that the Quick
from cities. Man was really
(a) open (b) prejudiced
There was a general buzz of stupefaction till
(a) foolish (b) clever
(c) liked (d) impartial the Quick Man was heard to whisper,
(c) wrong (d) right
‘‘He-must-have-had-another-lot-up-his-
Ê (d) The word ‘biased’ means inclined to
sleeve’’. Ê (a) ‘foolish’ as last sentence of the author
favour one over another. The word
Again everybody nodded and whispered, shows that the Quick Man was not
‘impartial’ means treating everyone equally.
‘‘The-rings-were-up-his-sleeve.’’ intelligent to presume that the conjuror must
13. It is easy to be an orthodox. The brow of the conjuror was clouded with a
be having a live hen up his sleeve who on
(a) idolatrous (b) intelligent demand will give 17 eggs.
(c) malignant (d) heterodox gathering frown.
‘‘I will now,’’ he continued, ‘‘show you a most 20. The conjuror extracted seventeen
Ê (d) ‘Orthodox’ means following or
amusing trick by which I am enabled to take eggs from the hat of
conforming to the traditions whereas its
antonym is ‘heterodox’ which means a any number of eggs from a hat. Will some (a) the Quick Man
person who does not conform to accepted gentleman kindly lend me his hat?’ Ah, thank (b) his own
standards or traditions. you—Presto !’’ (c) one gentleman from the audience
14. Permit me to present you with a He extracted seventeen eggs and for (d) None of the above
book. thirty-five seconds the audience began to Ê (c) The conjuror extraeted sevouteen eggs
(a) allow (b) enclose think that he was wonderful. Then the Quick from the hat of one gentleman from the
(c) prohibit (d) persuade Man whispered along the front bench, audience.
NDA/NA Solved Paper 2018 (II) 21

(d) pursue her studies 33. He had ………… spoken for two
Passage-2
Ê (d) Magda’s sister was coming to pursue minutes when there was a
Magda Good morning, Smiles. Mrs. It’s wet, her studies. commotion in the crowd.
isn’t it? (a) even (b) hardly
Mrs. Smiles Yes, it is, isn’t it? How are you 25. Mrs. Smiles and Magda are (c) often (d) little
(a) classmates (b) school friends
today? All right? You haven’t been studying (c) neighbours (d) sisters-in-law Ê (b) ‘hardly’ is correct here. The word means
too hard, have you? You look a bit pale. ‘not’ much, or ‘scarcely’.
Magda I don’t, do I? Well, I haven’t been out Ê (c) From the conversation shown in the
passage between Mrs. Smiles and Magda, 34. I would not commit myself ………
much lately. I don’t like this weather a bit. it appears that both are neighbours. that course of action.
Why, we haven’t had a dry day for weeks,
(a) of (b) to
have we? Directions (Q.Nos. 26-35) Each of (c) by (d) with
Mrs. Smiles No, I don’t think we have. Let’s the following sentences in this section
see, you’ll be going back home next month, Ê (b)
has a blank space and four words or
won’t you? groups of words given after the 35. The soldiers waiting to go into battle
Magda Oh no, I’m not going back yet. My sentence. Select whichever word or for the first time were full of ……… .
sister’s coming over first. group of words you consider most (a) apprehension (b) consideration
appropriate for the blank space. (c) anticipation (d) frivolity
Mrs. Smiles Oh, is she? You’re looking
forward to that, I expect. How long is it since 26. We fail to understand your reasons Ê (a)
you saw her? for ………… the college without
Magda Nearly a year now. Yes, I am looking completing the degree.
Directions (Q.Nos. 36-40) Given
forward to it very much. (a) attending (b) joining below are some idioms/phrases
Mrs. Smiles She’s coming over here to study? (c) leaving (d) refusing followed by four alternative meanings
Magda Yes, that’s right, I shan’t go back home Ê (c) to each. Choose the response (a), (b),
until she’s settled down. I don’t think I ought (c) or (d) which is the most
27. She …… her energy and started
to, ought I? shouting only when she heard the appropriate meaning.
Mrs. Smiles Well, no, she’s younger than you noise of bulldozers and cranes. 36. A red-letter day
are, is she? (a) checked (b) conserved
(a) a trivial day
Magda Yes, she’s only eighteen. (c) maintained (d) controlled
(b) a very important or significant day
21. Magda had not been out much lately Ê (b) (c) a day of bloodshed and violence
(d) a mourning day
because 28. The British ………… all over Africa
(a) she had not been keeping well and Asia collapsed in the first half of Ê (b)
(b) she was busy with her studies the twentieth century.
(c) the weather had been unpleasant
37. The gift of the gab
(a) hegemony (b) domicile
(d) her sister had come over to stay (a) ability to speak easily and confidently
(c) residence (d) inheritance
(b) ability to spoil something
Ê (c) As stated in the passage Magda had not Ê (a)
been out much lately because it has been (c) ability to sell things
raining continuously. It also implies that ‘the 29. The football match has to be ………… (d) gift from a sacred institution
weather has been unpleasant’. because of the weather. Ê (a)
(a) called off (b) continued
22. Magda was not going back home yet (c) put off (d) turned off 38. Walk a tightrope
because (a) to be forced to leave your job
(a) it had been raining heavily Ê (a)
(b) to be ready to fail
(b) her sister was coming over 30. If I were rich, ………… a lot. (c) to act very carefully
(c) her studies were not completed (a) I’ll travel
(d) she was not feeling well (d) to invite danger
(b) I can travel
Ê (b) ‘her sister was coming over’ so Magda (c) I would travel Ê (c)
was not planning to go back home. (d) I travelled 39. To be in a fix
23. Mrs. Smiles says, ‘‘You’re looking Ê (c) (a) to receive strong criticism
forward to that’’. This means, 31. They apologised ………… me for (b) to support oneself
Magda was ………… her sister’s what happened.
(c) to fix problems
arrival. (d) to be in a difficult situation
(a) to (b) at
(a) expecting with pleasure (c) for (d) with Ê (d)
(b) preparing hard for
(c) thinking greatly about Ê (a) 40. To fish in troubled waters
(d) watching with joy (a) to borrow money
32. If you are tired of swimming, just
(b) to steal belongings of
Ê (a) Magda was expecting with pleasure her ……… for a while.
sister’s arrival (a) struggle (b) stroke (c) to get benefit in bad situation
(c) float (d) streak (d) to extend a helping hand
24. Magda’s sister was coming to
(a) visit her sister (b) help Mrs. Smiles Ê (c) ‘float’ is appropriate word here as Ê (c)
(c) settle down in England floating in the water will not tire anyone.
22 NDA/NA Solved Paper 2018 (II)

Directions (Q.Nos. 41-45) Each of R. the apple falls The honeybee is a very unusual kind
the following items in this section S. on the ground of insect. ............. 46. other insects
consists of a sentence the parts of The correct sequence should be which live
which have been jumbled. These parts (a) QPRS (b) RSPQ (a) Unlike
have been labelled P, Q, R and S. (c) QSPR (d) SQRP (b) Similar to
(c) With
Given below each sentence are four Ê (b) (d) Like
sequences namely (a), (b), (c) and (d).
You are required to rearrange the
44. The knocking Ê (a)
P. were still in the house
jumbled parts of the sentence. Q. for some time
alone, the honeybee lives as a/an
R. although the echoes of it ............. 47. of a community. These
41. Domestic fires in Indian villages bees live
P. as the heat is dispersed S. ceased
Q. in unhealthy smoke accumulates in The correct sequence should be (a) group (b) individual
unventilated houses (a) SRPQ (b) SQRP (c) member (d) troop
R. are wasteful (c) PQSR (d) RSPQ Ê (c)
S. lit in Chulhas together in what is known as a bee
The correct sequence should be Ê (b)
............. 48. The head of the bees is
(a) PQSR (b) PQRS 45. The history of mankind is called the
(c) SPQR (d) SRPQ P. and steady progress
(a) house. (b) home.
Q. continuous change
Ê (d) (c) army, (d) colony
R. the history of
42. Bold rocks near at hand S. from barbarism to refinement Ê (d)
P. makes a fine forest for the The correct sequence should be queen bee. She is the ............. 49. than
imagination (a) PQRS (b) RQPS the rest of the bees. Her main task
Q. than distant Alps (c) QSPR (d) SQRP (a) heaviest (b) heavier
R. are more inspiring (c) largest (d) larger
S. and the thick fern upon a health Ê (b)
The correct sequence should be Ê (d)
(a) QRSP (b) RSQP
Directions (Q.Nos. 46-50) In the is to ............. 50. eggs.
(c) RQSP (d) SRQP following passage there are some (a) lay (b) hatch
blank spaces with four words or (c) make (d) bear
Ê (c)
groups of words given. Select
43. Newton discovered that whichever word or group of words you Ê (a)
P. due to the gravitational pull consider most appropriate for the
Q. of the earth
blank space.

Part B (General Studies)


51. Statement I The pitch of sound Statement II The provincial Ê (c) During the reign of Jahangir (1605–1627)
wave depends upon its frequency. autonomy was granted to the the Mughal painting reached its zenith. He
Statement II The loudness of the Provinces. favoured paintings of events from his own
sound wave depends upon its life rather than illustrated fiction. He
amplitude. Ê (b) Government of India Act of 1935 marked
a milestone towards a completely encouraged portraiture and scientific
Ê (b) Pitch of sound wave depends upon its responsible government in India. It was a studies of birds, flowers and animals, which
frequency because pitch of a sound is were collected in albums. Mansur and
lengthy and detailed document having 321
actually its property due, to which we can Manohar were among his famous painters.
Sections and 10 Schedules.
differentiate between a shrill and grave
The Act provided for the adoption of But during the reign of Aurangzeb
sound. The amplitude of a sound wave
dyarchy at the Centre. Consequently, the (1659–1707) the Mughal painting started to
determines its loudness or volume.
federal subjects were divided into reserved decline, although his ornate Pearl Mosque
52. Statement I Sound wave cannot subjects and transferred subjects. And (1662) at Delhi is worthy of mention. Many
propagate in vacuum. abolished dyarchy in the provinces and artists then joined Rajput courts, where their
Statement II Sound waves are introduced ‘provincial autonomy’ in its influence on Hindu painting is clearly
elastic waves and require a medium place. evident.
to propagate. 54. Statement I Mughal Painting 55. Statement I Phytoplanktons
Ê (a) Sound waves are elastic waves because reached its climax during the reign produce most of the organic carbon
their propagation depends on the material
of Jahangir. in the ocean.
media, which possess elasticity and inertia.
So it cannot propagate in vacuum. Statement II Aurangzeb’s Court Statement II Algae are produced
was adorned by some of the best in the cold water biome.
53. Statement I The Government of known artists of the Mughal School
India Act, 1935 introduced Dyarchy of Paintings. Ê (d) Algae are a diverse group of aquatic
organisms that have the ability to conduct
at the centre. photosynthesis, for example, seaweeds
NDA/NA Solved Paper 2018 (II) 23

(such as kelp or phytoplankton), pond scum 60. Which of the following kingdom(s) 64. Which one of the following is a
or the algal blooms in lakes. They are has/have only unicellular heterogeneous mixture?
produced in the cold water biome.
organisms? (a) Hydrochloric acid
Phytoplanktons are similar to terrestrial (a) Monera (b) Protista (b) Vinegar
plants. They contain chlorophyll and require (c) Milk
(c) Monera and Protista
sunlight in order to live and grow. They (d) Soda water
(d) Protista and Fungi
produce oxygen as a byproduct of
photosynthesis. Ê (c) Ê (c) A mixture that does not have uniform
composition is called heterogeneous
56. Statement I Geostrophic wind 61. Which one of the following is a mixture.
blows above a height of 600 m, waterborne disease? Out of the given compounds, milk is a
parallel to the isobars. (a) Jaundice (b) Tuberculosis heterogeneous mixture of fats,
(c) Rabies (d) Arthritis carbohydrates, proteins etc. dispersed in
Statement II Geostrophic wind is
the horizontal wind velocity, in Ê (a) Jaundice is caused by microbial water.
infection presnet in water and food items.
which the Coriolis force balances 65. What is the formula mass of
the horizontal pressure force. 62. The atomic number of an element is anhydrous sodium carbonate?
Ê (c) An air parcel initially at rest, but it moves 8. How many electrons will it gain to (Given that the atomic masses of
from high pressure to low pressure because form a compound with sodium? sodium, carbon and oxygen are 23 µ,
of the Pressure Gradient Force (PGF). (a) One (b) Two (c) Three (d) Four 12 µ and 16 µ respectively)
(a) 286 µ (b) 106 µ
However, as that air parcel begins to move, Ê (b) The atomic number of an element = 8
it is deflected by the Coriolis force to the Electronic configuration = K2 L6 (c) 83 µ (d) 53 µ
right in the Northern hemisphere (to the left ∴ Valency = 8 − 6 = 2 Ê (b) Chemical formula of (anhydrous)
on the Southern hemisphere). sodium carbonate = Na 2CO 3
Sodium (Atomic number 11) has electronic
As the wind gains speed, the deflection configuration = K2 L8 M1 ∴ Formula unit mass of Na 2CO 3
increases until the Coriolis force equals the = (2 × 23 + 12 + 16 × 3 ) µ
∴ Valency = 1
pressure gradient force. At this point, the
Thus, the given element will gain 2 electrons = 106 µ
wind will be blowing parallel to the isobars.
from two sodium atoms to complete its
When this happens, the wind is referred to
octet.
66. Which one of the following is called
as geostrophic wind. It blows above the
Formula of the compound
‘syngas’?
height of 600 m at the upper atmosphere. (a) C (s) + H 2O( g ) (b) CO( g) + H 2O( g )
= Na O (c) CO( g) + H 2 ( g ) (d) NO 2( g) + H 2 ( g )
57. Which one of the following
groups of cellular organelles 1 2 Ê (c) Syngas or synthesis gas is mixture of
contains DNA? = Na2O Carbon monoxide (CO) gas and Hydrogen
(H 2 ) gas. It may also consist of Carbon
(a) Mitochondria, nucleus, chloroplast
(b) Mitochondria, Golgi bodies, nucleus 63. A sample of oxygen contains two Dioxide (CO 2 ) gas. It is used as a fuel gas in

(c) Mitochondria, plasma membrane, isotopes of oxygen with masses 16 µ various industries.

nucleus and 18 µ respectively. The 67. The frequency of ultrasound waves


(d) Chloroplast, nucleus, ribosomes proportion of these isotopes in the is
sample is 3 : 1. What will be the (a) less than 20 Hz
Ê (a) The Mitochondria, nucleus and
average atomic mass of oxygen in (b) between 20 Hz and 2 kHz
chloroplast contain DNA. Mitochondria and
chloroplasts are known as this sample? (c) between 2 kHz and 20 kHz
semiautonomous cell organelles because (a) 17.5 µ (b) 17 µ (d) greater than 20 kHz
they can replicate themselves. (c) 16 µ (d) 16.5 µ Ê (d) Ultrasonic waves range above 20 kHz
which is above the audible range of human
58. One of the additional function of Ê (d) Average atomic mass of an element
ears.
 Atomic mass of isotope I 
Smooth Endoplasmic Reticulum  × %age of isotope I  68. The magnetic field strength of a
(SER) is = +
(a) protein synthesis  100  current carrying wire at a particular
  distance from the axis of the wire
(b) lipid synthesis  
(c) storage of biomolecules  Atomic mass of isotope II  (a) depends upon the current in the wire
(d) detoxification of toxic substances  × %age of isotope II  (b) depends upon the radius of the wire
=  (c) depends upon the temperature of the
Ê (d) Detoxification of toxic substances is the  100 
additional function of Smooth Endoplasmic   surroundings
  (d) None of the above
Reticulum (SER).
As it is given that the proportion of two
59. Damage to the apical meristem of a isotopes of oxygen with masses 16µ and18
Ê (a) The magnetic field strength of a current
carrying wire at a particular distance from
growing young plant will affect the µ is 3 : 1 respectively. So, they have the axis of the wire is given by the
(a) length of the plant percentage composition 75% and 25% Biot-Savart’s law.
(b) colour of the flower respectively. idl sin θ
Magnetic field, dB α
(c) colour of the leaves ∴ Average atomic mass of oxygen in the r2
(d) taste of the fruits
sample Where i = current in wire.
r = distance of the point from the wire.
Ê (a) Damage to the apical meristem of a  75   25 
= 16 × + 18 × = 16 . 5 µ dB ∝ i
100   100 
growing young plant will affect the length of
 Hence option (a) is correct.
the plant. It is responsible for apical growth.
24 NDA/NA Solved Paper 2018 (II)

69. A stainless steel chamber contains Ê (b) The correct sequence in order of Sunrise Ê (d) The temperature of the troposphere is
Ar gas at a temperature T and timing in the following places are as: highest near the surface of the Earth and
pressure P. The total number of Ar At Imphal, the capital city of Manipur, the decreases with height or altitude. On
atoms in the chamber is n. Now Ar sunrise time is 4:59 AM. average, the temperature gradient of the
At Itanagar, the capital city of Arunachal troposphere declines by 6.5°C per kilometre
gas in the chamber is replaced by
Pradesh, the sunrise time is 5:00 AM. on going above. However, in the
CO 2 gas and the total number of CO 2 stratosphere, temperature increases with
At Aizawal, the capital city of Mizoram, the
molecules in the chamber is n/2 at sunrise time is 5:05 AM. altitude. The reason is that the direct heat
the same temperature T . The At Agartala, the capital city of Tripura, the source for the stratosphere is the Sun.
pressure in the chamber now is P’. sunrise time is 5:10 AM.
Which one of the following 77. Which one of the following is
relations holds true? (Both the gases 74. Which one of the following is known as a zone of sharp salinity
behave an ideal gases) known as uplands of delta region? change in the vertical section of
(a) P ′ = P (b) P ′ = 2 P (a) Bef (b) Bils ocean?
(c) P ′ = P /2 (d) P ′ = P / 4 (c) Peh (d) Chars (a) Thermocline (b) Halocline
(c) Photic kone (d) Pycnocline
Ê (c) For Ar gas PV = nRT Ê (d) The Deltaic plain is the extension of the
P=
nRT
...(i)
Khadar land. It covers large area in the lower Ê (b) A halocline is a strong, vertical salinity
reaches of the Ganga River. In the delta gradient within a body of water. It is known
V
n region, the upland is called Chars, while the as the zone of sharp salinity change in the
For CO 2 gas, n ′ = , P = P ′ marshy area is known as Bils.
2 vertical section of ocean. As salinity (in
P ′ V = n ′ RT 75. Consider the following Wildlife concert with temperature) affects the
n RT density of seawater, it can play an important
P′ = Sanctuaries of India :
role in its vertical stratification.
2 V 1. Shikari Devi 2. Bhadra
P 3. Simlipal 4. Pachmarhi 78. Permanent hardness of water
From Eqn. (i), we get P ′ =
2 Which one of the following is the cannot be removed by which one of
70. Which one of the following is the correct order of the above Wildlife the following methods?
correct relation between Å and nm? Sanctuaries in terms of their location (a) Treatment with washing soda
(a) 1 nm = 10 −1 Å (b) 1 nm = 10 Å from South to North? (b) Calgon’s method
(c) 1 nm = 1 Å (d) 1 nm= 10 −2 Å (a) 1-2-3-4 (b) 2-4-3-1 (c) Boiling
(c) 2-3-4-1 (d) 3-1-2-4 (d) Ion exchange method
Ê (b) 1Å = 0.1 nm
⇒ 10Å = 1 nm Ê (c) The correct order of the given wildlife
Ê (c) Permanent hardness of water is due to
sanctuaries in terms of their location from the presence of chlorides and sulphates of
71. The full form of LED is South to North is calcium and magnesium ions. These
(a) Light Emitting Diode Bhadra Wildlife Sanctuary is located at compounds do not decompose on heating.
(b) Light Emitting Device Chikkamagaluru town in Karnataka. It was These are removed by ion exchange resin
(c) Light Enhancing Device established in the year 1951. It is a
treatment or by Calgon’s method or by
(d) Light Enhancing Diode protected area and a tiger reserve as part of
using a water softner like washing soda,
Project Tiger.
Ê (a) The full form of LED is Light Emitting
Simlipal Wildlife Sanctuary, also known as
calcium hydroxide, etc.
Diode. It is a heavily doped p − n junction
diode which converts electrical energy into Mayurbhanj Elephant Reserve, is a national 79. Which one of the following
light energy. park and a tiger reserve. It was established reactions will give NO (Nitric Oxide)
in the year 1980. It is located in the
gas as one of the products?
72. If a free electron moves through a Mayurbhanj district of Odisha.
potential difference of 1 kV, then the Pachmarhi Wildlife Sanctuary is also a
(a) 3Cu + 8HNO 3 (dilute)→
energy gained by the electron is biosphere reserve in the Satpura Range of (b) Cu + 4HNO 3 (conc)→
given by Madhya Pradesh. It was established in the (c) 4Zn + 10HNO 3 (dilute)→
. × 10 −19 J
(a) 16 . × 10 −16 J
(b) 16 year 1999. (d) Zn + 4HNO 3 (conc)→
(c) 1 × 10 −19 J (d) 1 × 10 −16 J Shikari Devi Wildlife Sanctuary is located
near the Shikari Devi Temple in Mandi Ê (a) 3Cu + 8HNO 3 → 3Cu(NO 3) 2
Ê (b) Given, Potential difference, district of Himachal Pradesh. It was (Dilute)
V = 1kV = 1 × 10 V
3

We know that, established in the year 1974. +2NO ↑ +4H2O


Energy gained by electron = e ⋅ V 76. Which one of the following Cu + 4HNO3 → Cu(NO3 )2
= 1.6 × 10 −19 × 10 3 = 1.6 × 10 −16 J statements about temperature is (Conc. )
correct? +2NO 2 ↑ +2H 2O
73. Consider the following places of
(a) Temperature decreases with height
India :
in the stratosphere.
Zn + 2HNO3 → Zn(NO3 )2 +H2 ↑
1. Itanagar 2. Imphal (b) Temperature is constant at different (Conc. )
3. Agartala 4. Aizawl heights in the stratosphere. 4Zn + 10HNO3 → 4Zn(NO3 )2
Which one of the following is the (c) Temperature increases with height in (Dilute)3
correct chronological order of the the troposphere at an average rate of
above places in terms of sunrise 6.5°C per km. + N2O ↑+ 5H2O
time? (d) Temperature decreases with height
Thus, in option (a) contains the reaction
(a) 3 − 2 − 1 − 4 (b) 2 − 1 − 4 − 3 in the troposphere at an average rate having NO gas as one of the products.
(c) 1 − 4 − 3 − 2 (d) 4 − 3 − 2 − 1 of 6.4°C per km.
NDA/NA Solved Paper 2018 (II) 25

80. Which one of the following is a In the above circuit R1 and R2 are connected
Ê (a) Photosynthesis is the process that plants
undergo in order to transform solar energy in the parallel combination. Hence,
tribasic acid? equavilant resistance,
into chemical energy which is stored in the
(a) Hydrochloric acid 1 1 1 R
bonds of sugar molecules. During the = + ⇒ R′ =
(b) Nitric acid
process, released oxygen comes from the R′ R R 2
(c) Sulphuric acid splitting of water in the presence of Now, circuit becomes
(d) Phosphoric acid light-dependent reaction. The products of
R′=R/2 R 3=R
the light-dependent reactions are ATP and
Ê (d) H 3 PO 4 , Phosphoric acid can be written A B
NADPH oxygen production is simply a
as O =P(OH) 3. It has three acidic hydrogens
byproduct of the splitting of water by R 4=R
(attached to oxygen) and is therefore
sunlight).
tribasic acid. On the other hand
The overall process of photosynthesis is
hydrochloric acid, nitric acid are monobasic In the circuit R′ and R are connected in
typically represented by the following
while sulphuric acid is dibasic. series combination, Hence
equation
R +2R
81. Which one of the following 6 CO 2 + 6 H 2 O + sunlight → C 6 H12O 6 + 6O 2 R′ ′ = R / 2 + R =
2
statements is not correct? 86. Which one of the following depicts 3R
(a) All carbons in diamond are linked by R′ ′ =
the correct circuit of a reflex arc? 2
carbon-carbon single bond.
(a) Effector→sensory neuron→spinal Now R′ ′ and R4 are connected in parallel
(b) Graphite is layered structure in which combination. Hence, equalient resistance
cord→motor neuron→receptor
layers are held together by weak van between A and B.
(b) Receptor→sensory neuron→spinal
der Waal’s forces. 1 1 1
(c) Graphite layers are formed by cord→neuron→effector = +
RAB 3 R R
hexagonal rings of carbon atoms. (c) Receptor→sensory neuron→brain→
2
(d) Graphite layers are held together by motor neuron→effector
1 2 1 1 2 +3
carbon-carbon single bond. (d) Sensory neuron→receptor→brain = + ⇒ =
RAB 3 R R RAB 3R
effector→motor neuron
Ê (d) Graphite layers are held together by van 1 5 3
der Waals forces and not by carbon-carbon Ê (b) The correct reflex arch is depicted by = ⇒ R AB = R
R AB 3R 5
single bond. option (b). It is receptor → sensory neuron
→ spinal cord → neuron → effector.
82. Which one of the following is called 89. The absolute zero temperature is 0
Dry Ice? 87. If one set of chromosomes for a Kelvin. In °C unit, which one of the
(a) Solid carbon dioxide given plant is represented as N ; in following is the absolute zero
(b) Liquid carbon dioxide case of double fertilisation, the temperature?
(c) Liquid nitrogen zygote and the endosperm nucleus (a) 0°C (b) −100°C
(d) Liquid ammonia of a diploid plant would have how (c) −273.15°C (d) −173.15°C
Ê (a) Solid carbon dioxide is called Dry Ice or many sets of chromosomes Ê (c) The absolute zero temperature is 0°K.
Cardice. It is used as a refrigerant for respectively? So, the relation between Celcius and Kelvin
ice-cream and frozen food. C K − 273.15
(a) N and 2 N (b) 2N and 2 N scale is = where,
(c) N and 3N (d) 2 N and 3N 100 100
83. The acidic, semidigested food K = 0, C = −273.15
coming out of the stomach is Ê (d) The zygote will have 2N chromosomes
neutralised by while endosperm nucleus will have 3N set of 90. Consider the following statements
(a) pancreatic juice chromosomes after double fertilisation in about visible light, UV light and
(b) duodenal secretion plant. X-rays:
(c) large intestine secretion 88. Consider the following circuit : 1. The wavelength of visible light is
(d) bile juice more than that of X-rays.
R
Ê (d) The acidic, semidigested food coming R 2. The energy of X-ray photons is
out of the stomach is neutralised by sodium A B higher than that of UV light photons.
bicarbonate present in the bile juice 3. The energy of UV light photons is
R R
secreted by liver. Here it makes the medium
less than that of visible light
of food alkaline.
photons.
Which one of the following is the
84. The oxygenated blood from the value of the resistance between Which of the statement(s) given above
lungs is received by the
(a) left auricle (b) left ventricle
points A and B in the circuit given is/are correct?
above? (a) 1, 2 and 3 (b) 1 and 2
(c) right auricle (d) right ventricle
2 3 3 (c) 2 and 3 (d) Only 1
(a) R (b) R (c) R (d) 4R
Ê (a) The oxygenated blood from the lungs is
received by left auricle through pulmonary
5 5 2 Ê (b) (1) The frequency of X-ray is greater than
vein from here it goes to left ventricle and Ê (b) visible light. So, the wavelength of visible
then it is pumped into other body parts. R 1=R light is more than that of X-rays.
R 3=R
R 2=R (2) We know that, E = hν
85. The oxygen evovled during photo- A B
synthesis comes from splitting of Energy ( E ) ∝ ν (frequency)
R 4=R
(a) water (b) carbon dioxide X-ray have higher frequency so it has higher
(c) oxygen (d) light energy than that of UV light photons.
26 NDA/NA Solved Paper 2018 (II)

91. The time period of oscillation of a 94. Consider the following statements 96. Match List I with List II and select
simple pendulum having length L about a microscope and a telescope : the correct answer using the codes
and mass of the bob m is given as T . 1. Both the eyepiece and the given below the Lists :
If the length of the pendulum is objective of a microscope are
increased to 4L and the mass of the convex lenses. List I (River Basin) List II (Town)
bob is increased to 2 m, then which 2. The focal length of the objective
one of the following is the new time A. Bhagirathi 1. Lansdowne
of a telescope is larger than the
period of oscillation? focal length of its eyepiece. B. Alaknanda 2. Narendra
T Nagar
(a) T (b) 2 T (c) 4T (d) 3. The magnification of a telescope
2
increases with the increase in C. Nayar 3. Uttarkashi
Ê (b) Time period of simple pendulum, focal length of its objective.
l D. Ganga 4. Pauri
T =2π …(i) 4. The magnification of a
g
microscope increases with the
According to the question, increase in focal length of its Code
A B C D
l′ = 4l objective.
(a) 3 1 4 2
New time period of simple pendulum, Which of the statements given (b) 3 4 1 2
4l  l  above are correct?
T′=2π = 2 × 2 π  (c) 2 4 1 3
g  g (a) 1 and 3 (b) 1 and 4
(d) 2 1 4 3
(c) 2, 3 and 4 (d) 1, 2 and 3
From equation (i), we get
Ê (b) Uttarkashi lies in Bhagirathi basin. Pauri
T ′ = 2T Ê (d) lies in Alaknanda basin. Lansdowne lies in
(1) Microscope consists of two convex Nayar basin. Narendra Nagar lies in Ganga
92. The connecting cable of electrical lenses coaxially separated by some basin.
appliances like electric iron, water distance. The lens nearer to the object is
heater or room heater contains three called the objective. The lens through 97. Match List I with List II and select
insulated copper wires of three which the final image is viewed is called the correct answer using the codes
different colours—red, green and the eyepiece. given below the Lists :
black. Which one of the following is (2) The focal length of the objective of a
the correct colour code? telescope is larger than the focal length List I (Local Wind) List II (Place)
(a) Red–live wire, Green–neutral wire, of eyepiece because objective has A. Yamo 1. Sudan
Black–ground wire much larger aperture than the eyepiece.
B. Black Roller 2. France
(b) Red–neutral wire, Green–ground (3) Magnification of telescope is directly
wire, Black–live wire proportional the focal length of obejctive C. Bise 3. Japan
(c) Red–live wire, Green–ground wire, (f0 ). Hence, the focal length of telescope D. Haboob 4. North
Black–neutral wire increases with the increase in focal America
(d) Red–ground wire, Green–live wire, length of its objective. Hence,
statements (1), (2) and (3) are the
Black–neutral wire Code
correct.
Ê (c) Red-Live wire (brings the current) A B C D A B C D
Green − Ground wire 95. A planet has a mass M1 and radius (a) 1 4 2 3 (b) 1 2 4 3
R1 . The value of acceleration due to (c) 3 4 2 1 (d) 3 2 4 1
Black − Neutral wire
gravity on its surface is g 1 . There is Ê (c) Yamo is warm and dry wind that blows
93. The graphs between current ( I ) and another planet 2, whose mass and down the steep valleys in Japan.
voltage (V ) for three linear resistors radius both are two times that of the Black roller is a very strong and dust-laden
1, 2 and 3 are given below : first planet. Which one of the wind blowing from the South–West or
following is the acceleration due to North-West in the Great Plains of North
1 America.
3
gravity on the surface of planet 2?
I (a) g 1 (b) 2 g 1 Bise is a cold, dry wind in Switzerland and
2 France.
(c) g 1 / 2 (d) g 1 /4
V A haboob is a type of intense dust storm
Ê (c) Acceleration due to gravity at planet is that blows in Sudan.
If R1 , R 2 and R 3 are the resistances of given by
these resistors, then which one of g1 =
GM1
…(i)
98. ‘Majuli’, the river island, is located
the following is correct? R1
2 in which one of the following
rivers?
(a) R1 > R 2 > R 3 (b) R1 < R 3 < R 2 For the second planet (2)
(a) Jamuna
(c) R 3 < R1 < R 2 (d) R 3 > R 2 > R1 M 2 = 2 M1 , R2 = 2 R1
(b) Padma
Now, acceleration due to gravity,
Ê (b) We know that, G(2 M1 ) (c) Ganga
1 g2 = (d) Brahmaputra
Slope of graph × (2 R1 )2
Resistance 1 GM1 Ê (d) Majuli is a river island, located in Asom. It
In the graph, g2 = × 2 is formed by river Brahmaputra. In 2016,
2 R1
slope of 1 > slope of 3 > slope of 2 Majuli became the first island to be made a
g1 district in India. It is often claimed to be the
R1 < R3 < R2 From Eqn. (i), we get g 2 =
2 world's largest river island.
NDA/NA Solved Paper 2018 (II) 27

99. Which one of the following Indian (stress, compression), and chemically 107. Which one of the following
active like fluids, solution, etc.
States has recorded negative growth statements about the law of
During metamorphism, rocks are often
of population as per Census 2011? subjected to all three metamorphic agents
conservation of mass is correct?
(a) Manipur (b) Mizoram simultaneously. However, the degree of (a) A given compound always contains
(c) Tripura (d) Nagaland metamorphism and the contribution of each exactly same proportion of elements.
agent vary greatly from one environment to (b) When gases combine in a reaction,
Ê (d) As per Census 2011, Nagaland is the
another. they do so in a simple ratio by
only state of the country which has negative
growth rate of -0.58%. Growth rate of Indian volume, provided all gases are at
103. The solution of which one of the room temperature.
population was 17.69% between 2001 and
following will have pH less (c) Matter can neither be created nor
2011.
than 7? destroyed.
100. Which one of the following types of (a) NaOH (b) KCl (d) Equal volumes of all gases at same
cloud is characterised by continuous (c) FeCl 3 (d) NaCl temperature and pressure contain
precipitation? equal number of molecules.
Ê (c) FeCl 3 is a salt of strong acid (HCl) and
(a) Cirrocumulus (b) Cumulus weak base Fe(OH) 3 . As a result FeCl 3 is an Ê (c) Law of conservation of mass states that
(c) Nimbostratus acidic salt and hence in solution form its pH matter can neither be created nor be
(d) Cumulonimbus lies below 7. destroyed.
Ê (c) Clouds are visible accumulations of tiny 104. Which one of the following is an 108. The wavelength of X-rays is of the
water droplets or ice crystals in the Earth’s
atmosphere. Nimbostratus clouds bring oxidation-reduction reaction? order of
(a) NaOH + HCI → NaCI + H 2 O (a) 1 Å (b) 1 µm (c) 1 mm d. 1 cm
continuous precipitation that can last for
many hours. These low-level clouds are full (b) CaO + H 2 O → Ca(OH) 2 Ê (a) X-Ray is a electromagnetic wave which is
of moisture. (c) 2Mg + O 2 → 2MgO discovered by German professor Rontgen.
(d) Na 2 SO 4 + BaCI 2 → BaSO 4 + 2NaCI The wavelength of X-rays is of the order of
101. Match List I with List II and select +1 −1 +1 − 1 +1 −1 +1 −2
1Å. It is used in surgery to detect the
the correct answer using the code Ê (c) (a) Na OH + HCl → NaCl + H 2 O fracture, diseased organs, stones in the
given below the Lists : (No change in oxidation states) body.
+2 − 2 +1 − 2 +2 + 2 − 1
List I (Major Port) List II (Location) (b) CaO + H 2O → Ca(OH )2 109. Consider the following velocity and
A. Kolkata 1. Land-locked
(No change in oxidation states)
time graph
area
(c)
B. Mormugao 2. In the delta Oxidation Velocity
region +2–2 (m/sec)
° g + 0° 2 → 2 MgO
2M
C. Visakhapatnam 3. On the
Reduction 8
riverside
+1 −2 + −1 +2 −2 +1 -1
D. Paradip 4. On the (d) Na 2 SO 4 + BaCI 2 → BaSO 4 + 2NaCl 4
entrance of
the estuary (No change in oxidation states)
∴It is clear from the above equations, that in 4 8 12 16 Time (s)
Code
option (c), the reaction involves both
A B C D A B C D
oxidation and reduction. Thus, it is a redox Which one of the following is the
(a) 3 1 4 2 (b) 3 4 1 2 reaction.
(c) 2 4 1 3 (d) 2 1 4 3
value of average acceleration from
105. Which one of the following is not 8sec to 12sec?
Ê (b) The Port of Kolkata is a riverine port in the used as fertilizer? (a) 8 m/sec 2 (b) 12 m/sec 2
city of Kolkata. It is the oldest operating port (c) 2 m/sec 2 (d) −1 m/sec 2
and was constructed by the British East (a) Ammonium nitrate
India Company. (b) Ammonium sulphide Ê (d)
(c) Ammonium phosphate
Mormugao is main port of Goa. It is a natural Velocity
(d) Ammonium sulphate
port and on the entrance of the estuary. (m/sec) A
8
Visakhapatnam Port is the deepest Ê (b) Ammonium sulphide is not used as
land-locked and protected port of India. It is fertilizer. Rest of the given compounds are B
4
located in Andhra Pradesh. It is India's used as fertilizers to compensate the
second largest port by volume of cargo deficiency of nitrogen, phosphorus and 4 8 12 16 Time (s)
handled. sulphur in the soil.
Paradip Port is a natural, deep-water, and Average acceleration from 8 sec to
106. Which one of the following is the 12 sec, is given by
deltaic port of India in Odisha. It is situated
chemical formula of gypsum? Velocity at B − Velocity at A
at confluence of the Mahanadi River and the aavg =
Bay of Bengal.
(a) CaSO 4 .2H 2 O (b) Ca 2 SiO 4 Time Interval
(c) 2CaSO 4 .H 2 O (d) CaSO 4 4−8 −4
102. Which one of the following is not an = = = − 1 m/sec 2
Ê (a) Gypsum has the chemical formula CaSO (12 − 8 ) 4
agent of metamorphism? 4 .2H 2 O. It can be prepared by mixing
(a) Heat (b) Compression plaster of paris with water. 110. If the focal length of a convex lens is
(c) Decomposition (d) Solution 50 cm, which one of the following is
1 1
CaSO 4⋅ H 2O + 1 H 2O → CaSO 4 ⋅ 2 H 2O
Ê (c) The literary meaning of metamorphism is 2 2 its power?
to change form. The agents of
(Plaster of Paris) (Gypsum) (a) +2 dioptre (b) +0.02 dioptre
metamorphism include heat, pressure (c) −0.5 dioptre (d) +0.5 dioptre
28 NDA/NA Solved Paper 2018 (II)

Ê (a) Given, Focal length of lens, f = 50 cm, . , which one of the following
15 116. Match List I with List II and select
= 0.5 m. statements is correct? the correct answer using the codes
We know that, (a) v1 is 15
. times v 2 (b) v 2 is 1.5 times v1 given below the Lists :
1 (c) v1 is equal to v 2 (d) v1 is 3 times v 2
Power of lens, P =
Focal length (m ) List I List II
Ê (a) We know that,
1 (Railway Zone) (Headquarters)
= = 2 dioptre Refractive index =
0.5 Velocity in vacuum (C) A. West Central 1. Jabalpur
111. A ball is released from rest and rolls Velocity in medium (V) B. South-East 2. Gorakhpur
down an inclined plane, as shown in C Central
In first medium n1 = ……(i)
the following figure, requiring 4 sec v1 C. East-Central 3. Bilaspur
to cover a distance of 100 cm along Similarly n 2 =
C
……(ii) D. North-Eastern 4. Hajipur
the plane : v2
From equation (i) and (ii)
Code
n 2 v1 A B C D A B C D
= (a) 2 3 4 1 (b) 1 4 3 2
n1 v 2
v
(c) 2 4 3 1 (d) 1 3 4 2
1.5 = 1
v2 Ê (d) Indian Railways is divided into 17 zones,
θ which are further sub-divided into divisions.
⇒ v1 = 1.5 v 2 Each zonal railway is made up of a certain
Which one of the following is the So, option (a) is correct. number of divisions, each having a
correct value of angle θ that the plane divisional headquarter. There are a total of
makes with the horizontal? 114. Which one of the following sixty-eight divisions.
(g = 1000 cm/sec 2 ) greenhouse gases is in largest
(a) θ = sin−1 (1 / 9.8) (b) θ = sin−1 (1 / 20) concentration in the atmosphere? Railway Zone Headquarters
(c) θ = sin−1 (1 / 80) (d) θ = sin−1 (1 / 100) (a) Chlorofluorocarbon West Central Jabalpur
(b) Nitrous oxide
Ê (c) Given, (c) Carbon dioxide South-East Central Bilaspur
Initial velocity of ball, u = 0 (d) Methane East-Central Hajipur
distance, s = 100 cm, time t = 4 sec
Ê (c) A greenhouse gas is a gas that absorbs North-Eastern Gokakhpur
We know that and emits radiant energy within the thermal
1 infrared range. In order, the most abundant
S = ut + at 2 117. Which one of the following is the
2 greenhouse gases atmosphere are water
1
correct descending order of Indian
100 = 0 + × a × ( 4 )2 vapour, carbon dioxide, methane, nitrous
2
States in terms of sex ratio as per
oxide, ozone, chlorofluorocarbons and
100 × 2 25 hydro fluorocarbons.
Census 2011?
a= = cm / sec 2 (a) Mizoram — Manipur — Tripura —
4×4 2
Now, a = g sin θ
115. Match List-I with List-II and select Meghalaya
a 25 / 2 the correct answer using the codes (b) Tripura — Manipur — Meghalaya —
sin θ = = Mizoram
g 1000 given below the Lists :
(c) Meghalaya — Manipur — Mizoram
 1 List I List I
⇒ θ = sin −1   — Tripura
 80 (Letter Code) (d) Manipur — Meghalaya — Tripura —
(Koppen’s Climatic
Type) Mizoram
112. The coefficient of areal expansion of
a material is 16. × 10−5 K −1 . Which A. Tropical wet 1. Af Ê (c) Sex ratio is used to describe the number
of females per 1000 of males. In India, it is
one of the following gives the value B. Mid-latitude desert 2. Cs
especially significant because the ratio is
of coefficient of volume expansion C. Mediterranean 3. Df heavily skewed towards men. In the
of this material? D. Humid Continental 4. BWk Population Census of 2011, it was revealed
−5 −1 −5 −1
(a) 0.8 × 10 K (b) 2.4 × 10 K that the population ratio of India 2011 is 943
(c) 3.2 × 10 −5 K −1 (d) 4.8 × 10 −5 K −1 Code females per 1000 of males.
Ê (b) We know that, A B C D A B C D As per final data (given by Union Minister of
Area expansion coefficient (β) (a) 1 4 2 3 (b) 1 2 4 3 Health and Family Welfare) the correct data
Volumetric expansion coefficient (X) (c) 3 2 4 1 (d) 3 4 2 1 is
=
2 Ê (a) State Sex Ratio (2011)
3 Type Type Characteristics
3 Meghalaya 989
X = Xβ Where β = 1.6 × 10 −5 k −1 Tropical Wet Af No dry season
2 Manipur 987
3 Mid-latitude BWk Mid-latitude
X = × 1.6 × 10 −5 K −1 Mizoram 976
2 arid Desert
or dry Tripura 960
X = 2 .4 × 10 −5 k −1
Mediterranean Cs Dry hot But according to the provisional data of
113. The refractive indices of two media summer Census 2011, the correct descending order
are denoted by n1 and n 2 and the is Meghalaya (989)-Manipur (987)-Mizoram
Humid- Df No dry season,
velocities of light in these two media continental severe winter (976)-Tripura (960).
are respectively v 1 and v 2 . If n 2 /n1 is Hence, option (c) is correct.
NDA/NA Solved Paper 2018 (II) 29

118. Steppe (temperate continental) (d) The size of the image becomes larger Ê (d) Chili is reported to be an inhabitant of
climate is not experienced in which than that of the object when the South America and is widely scattered in all
object is placed at a distance equal tropical and sub-tropical countries
one of the following places?
to half the focal length. including India. It was first introduced in
(a) Pretoria (b) Saskatchewan
India by the Portuguese towards the end of
(c) Perth (d) Buenos Aires Ê (a) The convex mirror will never form an
inverted image. the 15th Century.
Ê (a) Pretoria  is a city in the Northern part
of  Gauteng  province in  South Africa. 122. A circular coil of radius R having N 125. Consider the following statements
Pretoria has a  humid sub-tropical number of turns carries a steady about merchant guilds of South
climate  with long hot rainy summers and current I . The magnetic induction at India :
short cool to cold, dry winters. the centre of the coil is 0.1 tesla. If 1. Ayyavole merchant guild was
The mid latitude steppe climate spread over the number of turns is doubled and originally established in Aihole.
temperate grasslands is located in the the radius is halved, which one of 2. Manigraman merchant guild was
interiors of the continents, which come in the the following will be the correct sub-ordinated to the Anjuvannam
westerly wind belt, but because of their more value for the magnetic induction at merchant guild in the 13th
interior locations they do not get sufficient the centre of the coil? century.
rainfall and hence the grasslands are (a) 0.05 tesla (b) 0.2 tesla Which of the statement(s) given above
practically treeless. (c) 0.4 tesla (d) 0.8 tesla is/are correct?
119. Which one of the following is not a Ê (c) Magnetic field at the centers of coil. (a) Only 1 (b) Only 2
µ 0 Nl
process of chemical weathering? B= (c) Both 1 and 2 (d) Neither 1 nor 2
2r
(a) Solution (b) Carbonation µ Nl Ê (a) Southern Indian trade guilds  were
⇒ 0.1 = 0 …(i) formed by merchants in order to organise
(c) Oxidation (d) Exfoliation 2r
and expand their trading activities. The two
According to the question,
Ê (d) Chemical weathering is caused by
µ (2 N)( l ) most important merchant guilds of South
rainwater reacting with the mineral grains in B1 = 0 India were known as the Ayyavole and the
r
rock to form new minerals and soluble salts. 2× Manigraman. Ayyavole were a merchant
These reactions occur particularly when the 2
water is slightly acidic. Different types of  µ Nl guild from Aihole that provided trade links
⇒ B1 = 4  0 
chemical weathering are solution, oxidation  2r  between trading communities in  Tamil
and carbonation. These chemical Nadu, Karnataka and Andhra Pradesh.
From Eqs. (i), we get
processes occur more rapidly at higher Manigraman guild flourished in Tamil Nadu
B1 = 4 × 0.1 ⇒ B1 = 0.4 tesla
temperature. in the Pallava and Chola periods and was
Weathering  is the breakdown of rocks at 123. Which one among the following is active in South-East Asia. The Manigramam
the Earth’s surface, by the action of not a Fundamental Right under the merchants enjoyed a special position as
rainwater, extremes of temperature and Constitution of India? compared to the other merchants.
biological activity. It does not involve the (a) Right to equality Anjuvannam was another body of
removal of rock material. (b) Right to freedom merchants in South India, which almost
120. Which one of the following (c) Right to citizenship certainly represented an association of
statements is correct for a plane (d) Right against exploitation foreign merchants who traded across the
mirror? Arabian sea. It interacted both with local
Ê (c) Fundamental Rights  are the basic rights
merchants as well as the Ayyavole and
(a) Its focal length is zero. of the common people and inalienable
rights of the people who enjoy it under Manigramam organisations.
(b) The size of the image of an object
placed in front of the mirror is slightly the  charter of rights  contained in Part III
(Article-12 to 35) of  Constitution of India.
126. Which one of the following pairs is
less than that of the object. correctly matched?
(c) The image is virtual, erect and Right to citizenship is not is fundamental
right. Bhakti Saint Philosophy
laterally inverted.
All the Fundamental Rights have been
(a) Shankara : Avadhuta
(d) Its focal length is 200 cm. (b) Ramananda : Kevaladvaita
classified under the following six categories
Ê (c) In a plane mirror, image formed is virtual, Right to be Equality (Article14-18)
(c) Ramanuja : Vishishtadvaita
erect at the same distance behind the mirror (d) Chaintanya : Advaita
as the object is in front of mirror the focal Right to Freedom (Article 19-22)
length of plane mirror is infinite and power is Right against Exploitation (23-24) Ê (c) Kevaladvaita is another name for the
philosophy of advaita which was
zero. Right to Freedom of Religion
expounded by Adi Shankara. The meaning
(Article 25-28)
121. An object is placed in front of a Cultural and Educational Rights
of Advaita is non-dualism or belief in one
convex mirror. Which one of the reality.
(Article 29-30)
following statements is correct? Philosophy of Vishishtadvaita was
Right to Constitutional Remedies (Art.32)
(a) It will never form an inverted image. pioneered by Ramanujacharya during the
The Right to Property is no longer a 11th century. It literally means the Unique
(b) The image moves towards the focus Fundamental Right. Advaita,  i.e., Advaita with some
when the object moves towards the
124. Which one of the following crops amendments.
mirror.
was introduced by the Portuguese in Avadhuta is a Sanskrit term used to refer to
(c) Depending on the position of the
India? a person who has reached a stage in their
object with respect to the mirror, the
(a) Opium (b) Coffee spiritual development in which they are
image can be inverted and real.
(c) Betel leaf (d) Chili beyond worldly concerns.
30 NDA/NA Solved Paper 2018 (II)

127. When did the Stamp Act Congress 130. The Central Vigilance Commission Poverty and Un-British Rule in India was
consisting of delegates from 9 of the was established on the authored by Dadabhai Naoroji. It brought
attention to the draining of India's wealth
13 colonies of America meet in New recommendation of which one of
into Britain.
York City? the following Committees?
(a) 1763 (b) 1764 (c) 1765 (d) 1766 (a) Santhanam Committee 132. Which one of the following is the
(b) Dinesh Goswami Committee Official Mascot of the FIFA World
Ê (c) The  Stamp Act Congress, or  First
(c) Tarkunde Committee
Congress of the American Colonies, was a Cup 2018?
meeting held between October 7 and 25, (d) Narasimham Committee (a) Fuleco (b) Zakumi
1765, in  New York City. It consists of Ê (a) The Central Vigilance Commission was set (c) Pille (d) Zabivaka
representatives from some of the  British up by the government in February,1964 on
colonies in North America. At this meet, the
Ê (d) The official mascot for the FIFA World
the recommendations of the Committee on Cup 2018 tournament was Zabivaka—an
delegates discussed and united against the Prevention  of Corruption, headed by K. anthropomorphic wolf dressed in the
stamp act, which required the use of Santhanam, to advise and guide Central colours of the Russian national team.
specially stamped paper for legal government agencies in the field of vigilance.
The  2018 FIFA World Cup  was the
documents, playing cards, calendars, CVC is conceived to be the apex vigilance
21st  FIFA World Cup, an international
newspapers and dice for virtually all institution, free of control from any executive
football   tournament contested by
business in the colonies. authority, monitoring all vigilance activity
the  men's national teams  of the member
under the Central Government and advising
128. Who among the following travellers various authorities in Central Government
associations of  FIFA  once every four
was from Italy and visited years.
organizations in planning, executing,
Vijayanagar Kingdom in the reviewing and reforming their vigilance work. It 133. The Headquarters of the proposed
fifteenth century? consists of A Central Vigilance Commissioner National Sports University (as per the
(a) Nikitin (b) Fa-Hien as a chairperson; and not more than two National Sports University
(c) Bernier (d) Nicolo Conti Vigilance Commissioners as its members.
Ordinance, 2018) will be set-up in
Ê (d) Important Foreign Travellers Who Visited 131. Match List I with List II and select (a) Chhattisgarh (b) Manipur
India (c) Kerala (d) West Bengal
the correct answer using the codes
Nicolo Conti (AD 1420-1421), an Italian
traveller, visited Vijayanagar empire during
given below the Lists : Ê (b) National Sports University is a sports
university proposed to be set-up in Manipur
the reign of Devaraya-I (theTuluva dynasty List I (Author) List II (Book) in India  and three outlying campuses, one
ruler).
A. Bal 1. The Arctic each at Lucknow, Mohali and Gwalior. It
Nikitin (AD 1470–1474) was a Russian
Gangadhar Home in the was proposed to be set-up by the National
merchant,who describes the condition of
Tilak Vedas Sports University Ordinance, 2018 (the
the Bahmani kingdom under Muhammad III
Ordinance) was designed on the lines of the
(1463–82). B. Dadabhai 2. Hind Swaraj
Naoroji National Sports University Bill, 2017.
Fa-Hien was a Chinese pilgrim who visited
India during the reign of Chandra Gupta II. C. Mahatma 3. The Discovery 134. Sentosa island, is located in
His primary aim was to visit the Buddhist Gandhi of India (a) Singapore (b) China
religious places and to take with him the (c) Australia (d) Sri Lanka
D. Jawaharlal 4. Poverty and
copies of the Buddhist religious texts.
François Bernier  (September 25,
Nehru Un-British Ê (a) Sentosa, previously called  Pulau
Rule in India Blakang Mati, is a resort island in
1620–September 22, 1688) was a French
Singapore. It was once a British military
physician and traveller. He was briefly Code base and a Japanese prisoner of war camp.
personal physician to Mughal prince Dara A B C D A B C D The island was renamed Sentosa and
Shikoh (son of the Mughal emperor Shah (a) 3 4 2 1 (b) 3 2 4 1 turned into a tourist destination in 1972.
Jahan), and was attached to the court of the (c) 1 4 2 3 (d) 1 2 4 3 The island hosted the  2018 North
emperor  Aurangzeb.
Ê (c) The Arctic Home in the Vedas is a history Korea–United States summit  between the
129. Where did the French East India book on the origin of  Aryanic United States President  Donald Trump
Company first establish its factory People  by  Bal Gangadhar Tilak. It and  Kim Jong-un of North Korea on June
in India? propounded the theory that the  North 12, 2018, at the  Capella Hotel. Sentosa
(a) Calicut (b) Surat Pole  was the original home of Aryans island was gazetted as a ‘special event
(c) Pondicherry (d) Masulipatnam during the pre-glacial period which they had area’ by the Singapore government.
to leave due to the ice deluge around  8000
Ê (b) Among the Dutch, Danish, Portuguese
BC.
135. India, in June, 2018, asserted that
and French, the French East India Company any mega connectivity project must
Hind Swaraj  or  Indian Home Rule  is a
was the last to be formed. It was founded by respect sovereignty and territorial
book written by  Mohandas K. Gandhi  in
Jean-Baptiste Colbert in 1664. Under Francis integrity of the countries. The
1909. In it he expresses his views
Caron, the company established first factory project referred to above is
on  Swaraj,  modern civilisation,
at Surat in 1667 and second factory was (a) North-South Corridor Project
mechanisation  etc.
established at Masulipattanam a year later. (b) Belt and Road Initiative
In 1741, Joseph François Dupleix began to The  Discovery of India  was written by
(c) Chabahar Port
cherish the ambition of a French Empire in India’s first  Prime Minister Pt. Jawaharlal
(d) Panama Canal Expansion
India. But the problem began to emerge Nehru  during his imprisonment in 1942–46
when the conflict of the British and French at Ahmednagar fort in Maharashtra, India. It Ê (b) The One Belt One Road (OBOR) initiative
is an honour paid to the rich cultural is a Chinese effort which focusses on
started. The hopes of the French company
heritage of India, its history and its improving connectivity and cooperation
were finally ruined with the arrival of Robert
philosophy . among Asian countries, Africa, China and
Clive(a British General) in India in 1744.
NDA/NA Solved Paper 2018 (II) 31

Europe. The emphasis is on enhancing land 138. Every Judge of the Supreme Court of l
Protection and improvement of
as well as maritime routes through ‘Silk environment and safeguarding of
India is appointed by forests and wildlife.
Road Economic Belt’ and ‘Maritime Silk
Road’ initiatives respectively. With the (a) the Supreme Court Collegium l
Organisation of Village Panchayats and
exception of Bhutan, which has no (b) the Cabinet to promote cottage industry.
diplomatic ties with China, almost every (c) the President of India l
To bring about the prohibition of
other South Asian country has signed into intoxicating drinks and drugs that are
(d) the Lok Sabha
OBOR. But India has not signed it. The main injurious to health.
reason behind India’s opposition towards Ê (c) Article-124 provides for the l
Equal pay for equal work.
the policy is the  China-Pakistan Economic establishment and Constitution of Supreme l
Participation of workers in the
Corridor (CPEC), which is a part of OBOR. Court of India, which is the Apex Court of management of the industries.
Keeping this in mind, India in 2018 asserted India. The sanctioned strength of the judges
that to be a part of any mega project, it is is 31, consisting of the Chief Justice and 30 141. The word ‘socialist’ was inserted
necessary that the project must respect other judges. Every Judge of the Supreme into the Preamble to the
sovereignty and territorial integrity of the Court shall be appointed by the President Constitution of India through which
countries. by warrant under his hand and seal after one of the following Amendment
consultation with such of the Judges of the Acts?
136. Who among the following is not a (a) 41st Amendment Act
member of G-7? Supreme Court and of the High Courts in
the States as the President may deem (b) 42nd Amendment Act
(a) France (b) Germany
necessary for the purpose and shall hold (c) 43rd Amendment Act
(c) Russia (d) Japan
office until he attains the age of 65 years. (d) 44th Amendment Act
Ê (c) The  Group of Seven  (G-7) is a group of
seven largest advanced economies in the 139. The Nehru-Mahalanobis Strategy of Ê (b) The preamble to the constitution of India
is a brief introductory statement that sets out
world, consisting of Canada, France, Development was implemented for
Germany, Italy, Japan, the  United the guiding purpose, principles and
the first time by which one of the
Kingdom, and the United States. The 44th philosophy of the Constitution. Preamble
following Five-Year Plans? gives idea about the source of the
edition of G-7 summit 2018 was hosted by
Canadian Prime Minister Justin Trudeau in (a) First Five-Year Plan Constitution, nature of Indian State
Quebec (Canada) on June 8–9. The summit (b) Second Five-Year Plan (democratic, sovereign, socialist, secular,
was dubbed the G-6+1 by the French (c) Third Five-Year Plan republic) a statement of its objectives and
Government and political commentators (d) Seventh Five-Year Plan the date of its adoption. The  42nd
because of the United States withdrawal Amendment  Act amended the
from the Iran Deal along with the Paris Ê (b) At the time of the formulation of the
Preamble  and changed the description of
Agreement, rampant American Second Five Year Plan, Prof.PC
India from ‘sovereign democratic republic’
protectionism and controversies with Mahalanobis (adviser to Late Prime
to a ‘sovereign, socialist secular democratic
French President Emmanuel Macron and Minister Jawaharlal Nehru ), therefore it is
republic’, and also changed the words
Canadian Prime Minister Justin Trudeau. called as Nehru-Mahalanobis strategy. It
‘unity of the nation’ to ‘unity and integrity of
The 45th G-7 summit 2019 will be held a prepared a growth model with which he
the nation’.
France. showed that to achieve a rapid long-term
rate of growth it would be essential to 142. The palace of English East India
137. Who among the following leaders devote a major part of the investment outlay Company settlement in Madras was
started the Indian Home Rule to building of basic heavy industries. known as
League? (a) Fort William
(a) Gopal Krishna Gokhale
140. Which one of the following is not a
part of the Directive Principles of (b) Fort St. George
(b) Mahatma Gandhi
State Policy as enshrined in the (c) Elphinstone Circle
(c) Bal Gangadhar Tilak (d) Marble Palace
Constitution of India?
(d) JB Kripalani
(a) Equal justice and free legal aid Ê (b) The English East India Company was
Ê (c) Between the years 1916 and 1918, the (b) Protection of monuments and places originally known as ‘The Company of
Indian independence movement witnessed and objects of national importance Merchants of London trading into the East
the growth and spread of the home rule Indies.’ It was founded by a Royal Charter,
(c) Protection of personal law
movement started by leaders like Bal signed by Queen Elizabeth-I, on
Gangadhar Tilak and Annie Besant. The aim (d) Separation of Judiciary from
Executive 31st December, 1600. Sir Thomas Smith
of the home rule movement was the
was the Company’s first Governor.
attainment of home rule or a dominion
Ê (c) Part-IV of Indian Constitution deals with Soon, English open their trading centres at
status for India under the British Empire. Directive Principles of our State Policy
This movement was carried out through the several parts of India. In 1613, Mughal
(DPSP).  Basic aim of DPSPs is to
two home rule leagues – Indian Home Rule set-up  social and economic goals before emperor Jahangir issued a farman
League and Home Rule League. While (permission letter) to English to establish
the law makers. Some of the (other than
Tilak  launched the  Indian Home Rule their trade centre in India, hence British
given option DPSP’s) important DPSPs are
League  in April 1916 at Belgaum, Annie established their 1st trading factory at Surat
l
Uniform Civil Code for the citizens.
Besant launched the Home Rule League  in in 1613. In 1639 AD, the East India Company
September, 1916 at Madras. The two
l
Provide free and compulsory education
for children below 14 years. obtained the lease of the city of Madras from
leagues worked closely with one another. l
To promote international peace and the local king where it built Fort St. George to
However, they did not merge to avoid
amity. protect its factory.
friction between both the leaders
32 NDA/NA Solved Paper 2018 (II)

143. Which one of the following Indian Ê (a) The host country for World Blood Donor 148. Article-352 of the Constitution of
States (other than Himalayan Day 2018 events was Greece with the India contains provisions related to
or North-Eastern States) ranked first theme ‘Blood Connects us All’. The global (a) financial emergency
in the Composite Water event was held in Athens on 14th June, (b) failure of constitutional machinery in
2018. States
Management Index as per the report
Every year, on 14 June, countries around (c) suspension of the enforcement of
issued by the NITI Aayog in June,
the world celebrate World Blood Donor Day. rights conferred in Part III of the
2018? The event serves to thank voluntary, unpaid Constitution
(a) Madhya Pradesh(b) Karnataka blood donors for their life-saving gifts of (d) general emergency
(c) Gujarat (d) Maharashtra blood and to raise awareness of the need
Ê (d) The part 18 of Indian Constitution deals
Ê (c) The NITI (National Institution for for regular blood donations to ensure the with the emergency provisions. It contain
Transforming India) Aayog launched quality, safety and availability of blood and three types of emergency namely:
Composite Water Management Index to blood products for patients in need.
National Emergency : Article 352
assess and improve performance in This Article says that if the president is
efficient management of water resources. It
146. ‘Seva Bhoj Yojana’, a scheme of the
satisfied that a grave emergency exists
aims to inspire States/UTs towards efficient government of India that seeks to
whereby the security of India or any part of
and optimal utilisation of water and reimburse Central share of CGST the territory of India is threatened by war or
recycling thereof with sense of urgency. and IGST on external aggression or armed rebellion, he
The index comprises of nine parameters Food/Prasad/Langar/Bhandara may proclaim an emergency.
and 28 indicators. The states were divided offered by religious institutions, is
into two categories namely North-Eastern introduced recently by which one of
149. The Theosophical Society was led by
and Himalayan states and other States, to (a) AO Hume (b) Arthur Griffith
the following Ministries? (c) Annie Besant (d) Lord Dufferin
take into account different hydrological
conditions across these groups. The
(a) The Ministry of Culture
(b) The Ministry of Home Affairs Ê (c) The Theosophical Society was founded
ranking of States/UTs is given below : by  Madame Blavatsky and Col. Olcott  in
Gujarat topped the list followed by Madhya (c) The Ministry of Consumer Affairs, 1875 in New York. It was only in 1879, that
Pradesh, Andhra Pradesh, Karnataka and Food and Public Distribution this ideology gained its roots in the India. It
Maharashtra in category of General States (d) The Ministry of Social Justice and was crystallised in the  Madras Presidency
or Other States. Tripura has topped the list Empowerment with its headquarters in  Adayar.  The
followed by Himachal Pradesh, Sikkim and Ê (a) The Ministry of Culture, government of movement was popularised by  Annie
Asom in category of North–Eastern and India has introduced a new scheme namely Besant in India. It was based on the
Himalayan states. ‘Seva Bhoj Yojana’ to provide financial principles of Universal brotherhood and the
assistance on purchase of specific food study of comparative religion and
144. Who among the following is the items by Charitable Religious Institutions philosophy. The chief aim of the
ex-officio Chairman of the (CRIs) for free distribution among people. Theosophical Society in India was to
North-Eastern Council? Under this scheme, Centre’s share of establish the lost glory and honour of Hindu
(a) The President of India Central Goods and Services Tax (CGST) religion. Mrs. Annie Besant became its
(b) The Prime Minister of India charged on the raw food materials President and worked for the upliftment of
(c) The Union Home Minister purchased by the religious institutions will Indian mass.
(d) The Union Minister of State be refunded.
150. Which one of the following
(Independent Charge), Ministry of
Development of North-Eastern
147. Dr. Bindeshwar Pathak, who was statements about Bipin Chandra Pal
awarded Nikkei Asia Prize 2018 for is correct?
Region
Culture and Community, is the (a) He was a member of the moderate
Ê (c) The North-Eastern Council is the nodal
founder of
group of Congress.
agency for the economic and social (b) He was a member of the extremist
development of the North-Eastern Region (a) Bachpan Bachao Andolan group of Congress.
which consists of the eight states of (b) PRS Legislative Research (c) He was the Minister of Defence in the
Arunachal Pradesh, Asom, Manipur, (c) Sulabh Sanitation and Social Reform first government of independent India.
Meghalaya, Mizoram, Nagaland, Sikkim and Movement (d) He was the Chief Minister of West
Tripura. It was constituted in 1971 by an Act of Bengal.
(d) Smile Foundation
Parliament. The Union Home Minister acts as
ex-officio  Chairman of North-Eastern Ê (c) Bindeshwar Pathak, a noted social Ê (b) In the beginning of the 20th century, a
new class of national leaders emerged in
Council  (NEC). The Secretariat reformer and founder of Sulabh
India which was different from the moderate
(Headquarters) of the Council is located at International, was honoured with Japan’s
group. They were known as extremists, who
Shillong, Meghalaya. The President of India prestigious ‘Nikkei Asia Prize for Culture and
took a more aggressive stance against the
nominates the Chairman of the Council. Community’ for his significant work in
British Empire and did not believe in the
tackling poor hygiene and discrimination.
145. Which one of the following is the Launched in 1996, the award honours
soft, persuasive approach of the moderate
theme of the World Blood Donor leaders. Important extremist leaders were
people in Asia who have made significant
Day 2018? Lala Lajpat Rai, Bal Gangadhar Tilak, Bipin
contributions in one of the three
Chandra Pal (Lal-Bal-Pal) leading the
(a) Blood connects us all areas–regional growth; science, technology
extremist cause in Punjab, Bombay and
(b) Be there for someone else. Give and innovation; and culture and community.
Bengal respectively. Their goal was ‘swaraj’
blood. Share life Former Prime Minister Manmohan Singh
and did not stick to constitutional methods
(c) Give blood. Give now. Give often and Infosys Chairman Narayan Murthy are
of protest and demand rather they resorted
(d) Thank you for saving my life among the few Indians who have won the
to boycotts, strikes, etc.
prize in the past.
NDA /NA
National Defence Academy/Naval Academy

SOLVED PAPER 2018 (I)


PAPER I : Mathematics
1. If n ∈ N , then 3. In the expansion of (1 + x )43 , if the 5. Let α and β be real number and z be a
121 − 25 + 190 − ( − 4 ) is divisible
n n n n coefficients of (2r + 1)th and (r + 2) complex number. If z 2 + αz + β = 0
by which one of the following? th terms are equal, then what is the has two distinct non-real roots with
(a) 1904 (b) 2000 (c) 2002 (d) 2006 values of r (r ≠ 1) ? Re (z ) = 1, then it is necessary that
(a) 5 (b) 14 (c) 21 (d) 22 (a) β ∈ ( −1, 0) (b)|β| = 1
Ê (b) We have, (c) β ∈ (1, ∞ ) (d) β ∈ ( 0, 1)
Ê (b) We have, (1 + x)
43
121n − 25 n + 1900 n − ( − 4)n
On putting n = 1, we get
∴ General term, Tr +1 = 43
C r xr Ê (c) Let z = x + iy
Now, we have
(121)1 − (25)1 + (1900)1 − ( −4)1 Now, T2r +1 = 43
C 2r x 2r
z2 + αz + β = 0
r +1
= 121 − 25 + 1900 + 4 and Tr +2 = 43
Cr +1 x ⇒ ( x + iy)2 + α ( x + iy) + β = 0
= 2000
Now, according to the question ⇒ x 2 − y 2 + 2 ixy + αx + iαy + β = 0
Which is divisible by 2000.
Coefficients of (2 r + 1) th and ( r + 2 ) th [Q i 2 = −1]
2. If n = (2017 )! , then what is terms are equal
⇒ ( x − y + αx + β ) + (2 xy + αy) i = 0
2 2

1 1 1 ∴ 43
C 2r = 43C r + 1
+ + On comparing,
log 2 n log 3 n log 4 n ⇒ 2 r + r + 1 = 43 x 2 − y 2 + αx + β = 0 and 2 xy + αy = 0
1 [Q if nC x = nC y ⇒ x + y = n]
+…+ equal to ? ⇒ x 2 − y 2 + αx + β = 0 and (2 x + α ) y = 0
log 2017 n ⇒ 3r = 42
⇒ x 2 − y 2 + αx + β = 0 and 2 x + α = 0
n ⇒ r = 14
(a) 0 (b) 1 (c) (d) n [Q y ≠ 0]
2 4. What is the principal argument of
⇒ x 2 − y 2 + αx + β = 0 and α = − 2 x
Ê (b) We have, ( − 1 − i ) , where i = −1?
1 1 1 1 π π ⇒ x 2 − y 2 + αx + β = 0 and α = − 2
+ + + .... + (a) (b) −
log 2 n log 3 n log 4 n log 2017 n 4 4 [QRe( z) = 1 = x]
3π 3π ⇒ 1 − y 2 − 2 + β = 0 [Q x = 1, α = − 2 ]
= log n 2 + log n 3 + log n 4 + .... + log n 2017 (c) − (d)
4 4
⇒ β = y2 + 1
 1  Ê (c) Let z − 1 − i
Q log b a = log b  | b| | − 1|
⇒ β ∈ (1, ∞ )
 a  Now, tanα = = [Q y 2 ≥ 0 ⇒ y 2 + 1 ≥ 1]
= log n(2 ⋅ 3 ⋅ 4......2017 ) | a| | − 1|
[Qlog a + log b = log ab] [Q a = − 1, b = − 1] 6. Let A and B be subsets of X and
= log n(1 ⋅ 2 ⋅ 3 ⋅ 4......2017 ) π−1 C = ( A ∩ B ′ ) ∪ ( A ′ ∩ B ), where A ′
∴ α = tan (1) =
= log n(2017 )! 4 and B′ are complements of A and B
Since a, b both are negative, respectively in X . What is C equal to ?
[Q n( n − 1)( n − 2 ) K 3 ⋅ 2 ⋅ 1 = n!]
∴ arg( z) = α − π (a) ( A ∪ B′ ) − ( A ∩ B′ )
= log( 2017)! (2017 )! [Q n = 2017 !] (b) ( A ′ ∪ B) − ( A ′ ∩ B)
π −3 π
=1 [Q log a a = 1] = − π=
4 4 (c) ( A ∪ B) − ( A ∩ B)
(d) ( A ′ ∪ B′ ) − ( A ′ ∩ B′ )
34 NDA/NA Solved Paper 2018 (I)

Ê (c) We have, C = ( A ∩ B′ ) ∪ ( A ′ ∩ B) According to the question, ⇒ 15 x (1 + 3 x ) = 12 ⋅ 5 x


a+ b
= ( A − B) ∪ ( B − A ) 5 ⇒ 3 x (1 + 3 x ) = 12
2 =
[Q X ∩ Y ′ = x − y] ab 3 ⇒ y (1 + y) = 12 [where y = 3 x ]
= ( A ∪ B) − ( A ∩ B)  a+ b  ⇒ y + y − 12 = 0
2
Q A :G = 5 : 3, A = ,G = ab
[from venn diagram]  2 
a + b 10 ⇒ ( y + 4)( y − 3) = 0
⇒ = ⇒ y = − 4, 3
A B ab 3
2 ⇒ 3 x = − 4, 3
( a + b )2  10 
⇒ =  ⇒ 3x = 3 [Q3 x ≠ − 4]
(A–B) ab  3
(B–A) ⇒ x=1
a 2 + b 2 + 2 ab 100
⇒ =
U ab 9 12. How many four-digit numbers
a b 100 divisible by 10 can be formed using 1,
⇒ + +2 =
7. How many numbers between 100 b a 9 5, 0, 6, 7 without repetition of digits?
and 1000 can be formed with the 1 100 Q a = t  (a) 24 (b) 36
⇒ t + +2 =
 b  (c) 44 (d) 64
digits 5, 6, 7, 8, 9, if the repetition of t 9
digits is not allowed? t 2 + 1 + 2t 100 Ê (a) We have to form four digit numbers
5 3 ⇒ = which are divisible by 10 and using 1, 5, 0,
(a) 3 (b) 5 t 9
(c) 120 (d) 60 6, 7. Since numbers must be divisible by
⇒ 9 t − 82t + 9 = 0
2
10, so unit place must be zero.
Ê (d) Number lying between 100 and 1000 ⇒ (t − 9)( 9 t − 1) = 0 ∴ Total number of such numbers =
are of three digit. Since the numbers are 1
⇒ t = 9, Permutations of three digits using 1, 5, 6,
to be formed with 5, 6, 7, 8, 9 and
9 7
repetition is not allowed, so total number
of numbers ∴
a
= 9 or =
a 1 Q t = a  = 4 P3 =
4!
= 4! = 24
b b 9  b  ( 4 − 3)!
= 5 × 4 × 3 = 60
⇒ a : b = 9 : 1 or 1 : 9
8. The number of non-zero integral Directions (Q. Nos. 13-14) Consider
solution of the equation 10. If the coefficients of am and an is the
m +n the information given below and
| 1 − 2i | x = 5x is expansion of (1 + a ) are α and β,
answer the two items that follow
(a) Zero (no solution) then which one of the following is
In a class, 54 students are good in Hindi
(b) One correct? only, 63 students are good in Mathematics
(c) Two (a) α = 2β (b) α = β only and 41 students are good in English
(d) Three (c) 2α = β (d) α = ( m + n)β only. There are 18 students who are good
Ê (a) We have, Ê (b) We have in both Hindi and Mathematics. 10
|1 − 2 i| = 5
x x (1 + a )m + n students are good in all three subjects.
m+n
∴ Tr = C r ar 13. What is the number of students who
⇒ ( (1)2 + ( −2 )2 )x = 5 x +1

∴ Coefficient of a = m m+n
Cm [Qr = m] are good in either Hindi or
[Q a + ib = a2 + b 2 ] Mathematics but not English?
m+n
and coefficient of a =
n
Cn [Q (a) 99 (b) 107
⇒ ( 1 + 4 )x = 5 x
r = n] (c) 125 (d) 130
⇒ ( 5 )x = 5 x ∴ α = m+n
Cm 14. What is the number of students who
⇒ x/2
=5 x m+n
5 and β= Cn are good in Hindi and Mathematics
m+n

x
= x = C m + n − n[Q nC r = nC n − r ] but not in English?
2 = m + nC m = α (a) 18 (b) 12 (c) 10 (d) 8
[Q a m = a n ⇒ m = n] ∴ α =β Solution (Q. Nos. 13-14)
x 11. If x + log15 (1 + 3x ) = x log15 5 + log15 12,
⇒ =0x−
2 H M
x
where x is an integer, then what is x
⇒ =0 equal to? 54 8 63
2
(a) − 3 (b) 2 (c) 1 (d) 3 10
⇒ x =0
But x is non-zero integral. Ê (c) We have,
∴Given equation has no solution. x + log15(1 + 3 x ) = x log15 5 + log15 12 41
⇒ log15 15 x + log15(1 + 3 x ) E
9. If the ratio of AM of GM of two
positive numbers a and b is 5 : 3, then = log15 5 x + log15 12
a : b is equal to [Qlog a a = 1 and log b a m = mlog b a]
Ê (c) From given Venn-diagram
(a) 3 : 5 (b) 2 : 9 ⇒ log15[15 x (1 + 3 x )] = log15( 5 x × 12 ) n( H ∪ M ∪ E ′ ) = 54 + 8 + 63 = 125
(c) 9 : 1 (d) 5 : 3
[Qlog a + log b = log ab] Ê (d) From given Venn-diagram
Ê (c) Let a and b be two numbers. n( H ∩ M ∩ E ′ ) = 18 − 10 = 8
NDA/NA Solved Paper 2018 (I) 35

15. If α and β are different complex 2 31 [Q z = a + ib ⇒| z| = a2 + b 2 ]


numbers with | α | = 1, then what is 2 15 1
= 3 + 1= 4 = 2
α −β 2 7 1
Now, amp ( z) = tan−1  
b
equal to ? 2 3 1  a
1 − αβ
2 1 1  1  π
(a)|β| (b) 2 (c) 1 (d) 0 = tan−1  =
0 1  3 6
Ê (c) We have,  π 1 
α −β α −β ∴( 31)10 = (11111)2 Q tan =
=  6 3 
1 − αβ αα − αβ 18. What is i 1000 + i 1001 + i 1002 + i 1003 ∴ z = r (cos θ + i sinθ)
[Q|α| = 1 ⇒ |α|2 = 1 ⇒ α ⋅ α = 1] equal to (where i = −1) ? π π
= 2  cos + i sin 
α −β (a) 0 (b) i (c) − i (d) 1  6 6
= π
α (α − β ) Ê (a) We have, [Q r = | z| = 2 and θ = amp( z) = ]
6
1 α −β i 1000 + i 1001 + i 1002 + i 1003
= 21. What is the number of non-zero
|α| α − β = i 1000[1 + i + i 2 + i 3 ]
terms in the expansion of
|α − β| = i 1000[1 + i − 1 − i ]
= (1 + 2 3x )11 + (1 − 2 3x )11 (after
|α ||α − β| [Q i 2 = − 1, i 3 = − i ] simplification)?
|α − β| =0
= [Q| z | = | z|] (a) 4 (b) 5
|α ||α − β| (c) 6 (d) 11
19. What is
1
Ê (c) In ( a + b ) + ( a − b ) , number of
n n
= =1 1 1 1
|α| + + +…
log 2 N log 3 N log 4 N terms
[Q|α| = 1]
1  n + 2 , if n is even
16. The equation |1 − x | + x 2 = 5 has + equal to ( N ≠ 1) ? 
= 2
log100 N n+1
(a) a rational root and an irrational root  , if n is odd
(b) two rational roots (a)
1
(b)
1  2
(c) two irrational roots log100! N log 99! N ∴ Number of terms in
(d) no real roots
(c)
99
(d)
99 (1 + 2 3 x )11 + (1 − 2 3 x )11
Ê (a) We have, log100! N log 99! N
=
11 + 1
[Qn = 11, is odd]
|1 − x | + x 2 = 5
Ê (a) We have, 12
2
Case I When x < 1 1
+
1
+
1
+ .... +
1 = =6
2
1 − x + x2 = 5 log 2 N log 3 N log 4 N log100 N

[Q x < 1 ⇒|1 − x | = 1 − x] = log N 2 + log N 3 + log N 4 + ... + log N 100 22. What is the greatest integer among
 1  the following, by which the number
⇒x − x−4=0
2
Q log b a = log b  55 + 7 5 is divisible?
1± 1 − 4 (1) ( − 4)  a 
⇒ x= (a) 6 (b) 8 (c) 11 (d) 12
2 = log N 2 ⋅ 3 ⋅ 4 ...100
1± 17 [Qlog a + log b = log ab] Ê (d) We know that when m is odd then
= ( x m + y m ) is divisible by ( x + y).
2 = log N 1 ⋅ 2 ⋅ 3 ⋅ 4 ...100
= log N (100)! ∴5 5 + 7 5 is divisible by 5 + 7 = 12 as
1 − 17
⇒ x= [Q x < 1] m = 5 is odd.
2 [Q n! = n( n − 1) ( n − 2 ) ... 2 ⋅ 1]
Case II When x ≥ 1 =
1 23. If x = 1 − y + y 2 − y 3 … up to
log(100)! N
− (1 − x ) + x 2 = 5 infinite terms, where | y | < 1, then
[Q x ≥ 1 ⇒ |1 − x | = − (1 − x )] 20. The modulus-amplitude form of which one of the following is
⇒ − 1 + x + x2 = 5
correct?
3 + i, where i = −1 is 1 1
(a) x = (b) x =
⇒ x2 + x − 6 = 0 π π 1+ y 1− y
(a) 2  cos + i sin 
⇒ ( x + 3)( x − 2 ) = 0  3 3 (c) x =
y
(d) x =
y
 π π 1+ y 1− y
⇒ x = − 3, 2 (b) 2  cos + i sin 
 6 6
⇒ x=2 [Q x ≥ 1] Ê (a) We have,
π π
∴ Given equation has a rational root and (c) 4  cos + i sin  x = 1 − y + y 2 − y 3 + ... ∞ ,| y|< 1
an irrational root.  3 3
1
π π =
17. The binary number expression of the (d) 4  cos + i sin  1 − ( − y)
 6 6
decimal number 31 is a
[Q a + ar + ar 2 + K ∞ = , r < 1]
(a) 1111 (b) 10111 Ê (b) Let z = 3 + i 1− r
(c) 11011 (d) 11111 1
∴ | z| = ( 3 )2 + (1)2 =
Ê (d) 1+ y
36 NDA/NA Solved Paper 2018 (I)

cos θ − sinθ 0 
24. What is the inverse of the matrix Ê (d) We have,
=  sinθ cos θ 0 C ( n1r ) + 2 C ( n, r − 1) + C ( n, r − 2 )
 
 cos θ sin θ 0  0 0 1
  = nC r + 2 ⋅ nC r − 1 + nC r − 2
A =  − sin θ cos θ 0 ?
 0 25. If A is a 2 × 3 matrix and AB is a 2 × 5 = ( nC r + nC r − 1 ) + ( nC r − 1 + nC r − 2 )
 0 1 n +1 n +1
matrix, then B must be a = Cr + Cr − 1
 cos θ − sinθ 0 (a) 3 × 5 matrix (b) 5 × 3 matrix
  [Q C r + nC r − 1 =
n n +1
(a)  sinθ cos θ 0 (c) 3 × 2 matrix (d) 5 × 2 matrix Cr]
 0 1 n +1 +1
 0 Ê (a) Let order of B is m × n. = Cr
 cos θ 0 − sinθ Now, according to the question = n +2
Cr
 
(b)  0 1 0  A 2 × 3 × Bm × n = ( AB)2 × 5 = C ( n + 2, r )
 sinθ 0 cos θ 
  ∴ m = 3 and n = 5 29. Let | x | denote the greatest integer
1 0 0  ∴ Order of B is 3 × 5.
  function. What is the number of
(c)  0 cos θ − sinθ 1 2
 0 sinθ cos θ  26. If A =   and A − kA − I 2 = O ,
2 solutions of the equation
  2 3 x 2 − 4 x + [ x ] = 0 in the interval
 cos θ sinθ 0 where I 2 is the 2 × 2 identity matrix, [0, 2] ?
 
(d)  − sinθ cos θ 0 then what is the value of k ? (a) Zero (no solution) (b) One
 0 0 1 (b) − 4
 (a) 4 (c) Two (d) Three
(c) 8 (d) − 8
Ê (a) We have, Ê (b) We have,
 cos θ sinθ 0 
Ê (a) We have, x 2 − 4 x + [ x] = 0
1 2 
A =  − sinθ cos θ 0  A=  Case I x∈ [0, 1]
  2 3
 0 0 1  ∴ x2 − 4 x + 0 = 0
1 2  1 2  [Q x∈ [0, 1] ⇒ [ x ] = 0]
| A | = 1 [cos 2 θ − ( − sin2 θ)] = 1 ≠ 0 ∴ A2 = A ⋅ A =   
2 3 2 3 ⇒ x2 − 4 x = 0
cos θ 0
C11 = = cos θ  1⋅ 1 + 2 ⋅ 2 1⋅ 2 + 2 ⋅ 3 
0 1 = ⇒ x ( x − 4) = 0

− sinθ 0 2 ⋅ 1 + 3 ⋅ 2 2 ⋅ 2 + 3 ⋅ 3 ⇒ x = 0, 4
C12 =− = sinθ 5 8 
0 1 = ⇒ x=0

− sinθ cos θ  8 13 [Q x∈ [0, 1]]
C13 = =0 Case II x∈ [1, 2 ]
0 0 Now, it is given that,
A 2 − kA − I2 = 0 ∴ x2 − 4 x + 1 = 0
sinθ 0
C 21 =− = − sinθ [Q x∈ [1, 2 ] ⇒ x = 1]
0 1 5 8   k 2 k   1 0
⇒ − − =0
cos θ 0  8 13 2 k 3k   0 1 ⇒ x=
4± 16 − 4
C 22 = = cos θ 2
0 1 4 8   k 2 k
⇒ = 4±2 3
cos θ sinθ  8 12  2 k 3k  ⇒ x=
C 23 = − =0 2
0 0 ⇒ k=4
sinθ 0 ⇒ x=2 ± 3
C 31 = =0 27. What is the number of triangles that ⇒ x = 0 .268, 3.732
cos θ 0 can be formed by choosing the
No solution [Q x∈ [1,2 ] ]
cos θ 0 vertices from a set of 12 points in a
C 32 =− =0 ∴Given equation has only one solution i.e.
− sinθ 0 plane, seven of which lie on the same x = 0.
cos θ sinθ straight line?
C 33 = = cos 2 θ + sin2 θ = 1 (a) 185 (b) 175
30. A survey of 850 students in a
− sinθ cos θ University yields that 680 students
(c) 115 (d) 105
 cos θ sinθ 0
T
like music and 215 like dance. What
∴ adjA =  − sinθ cos θ 0 Ê (a) Required number of triangle is the least number of students who
  = C 3 − 7C 3
12
 0 0 1  like both music and dance?
12 × 11 × 10 7 × 6 × 5 (a) 40 (b) 45
cos θ − sinθ 0 = −
3×2 ×1 3×2 ×1 (c) 50 (d) 55
=  sinθ cos θ 0
  Ê (b) Let A be the set of students who like
 0 0 1 = 220 − 35 = 185
music and B be the set of students whose
1 28. What is like dance.
∴ A −1 = adj
| A| C (n , r ) + 2C (n , r − 1) + C (n , r − 2) ∴n( A ) = 680, n( B) = 215 and n(U ) = 850
T equal to ? We know that,
cos θ − sinθ 0  (a) C ( n + 1, r )
1  n( A ∩ B) = n( A ) + n( B) − n( A ∪ B)
A= sinθ cos θ 0  (b) C ( n − 1, r + 1)
1   ⇒
 0 0 1  (c) C ( n, r + 1)
n( A ∩ B)min = n( A ) + n( B) − n( A ∪ B)max
(d) C ( n + 2, r )
NDA/NA Solved Paper 2018 (I) 37

⇒ n( A ∩ B)min = 680 + 215 − 850 3 π π


34. If x , , z are in AP; x , 3, z are in GP; (a) π (b) (c) (d) 0
[Q n( A ∪ B)max = n( ∪ )] 2 2 4
= 45 1 1
then which one of the following will Ê (c) We have, sin x = and sin y =
be in HP? 5 10
31. What is the sum of all two-digit 1 −1 1
numbers, which when divided by 3 (a) x , 6, z (b) x , 4, z ⇒ x = sin and y = sin−1
(c) x , 2, z (d) x , 1, z 5 10
leave 2 as the remainder ? 1 1
(a) 1565 (b) 1585 Ê (a) We have, Now, x + y = sin−1 + sin−1
(c) 1635 (d) 1655 3 5 10
x, , z are in AP.
2  2 2
Ê (c) Required numbers are 11, 14, 17, ... 98 1  1  1  1  
x+ z 3 = sin−1  1−   + 1−  
which is an AP. ⇒ =  5  10  10  5 
2 2  
We know that,
⇒ x+ z=3 … (i) [Q sin−1 x + sin−1 y = sin−1 [ x 1 − y 2
an = a + ( n − 1)d
98 = 11 + ( n − 1) ( 3) Also, x, 3, z are in GP + y 1 − x2 ]
⇒ 98 = 11 + 3n − 3 ⇒ x z = 32
 1 1 1 1
= sin−1  1− + 1− 
⇒ 98 = 3n + 8 ⇒ x z=9 … (ii) 10 5
 5 10
⇒ 90 = 3n Now, from Eqs. (i) and (ii), we have
−1  1 3 1 2 
⇒ n = 30 2 xz 2 ×9 = sin  × + ×
=  5 10 10 5 
∴ Sum = 11 + 14 + 17 + .... + 98 x+ y 3
 5 
2 xz = sin−1 
[11 + 98] Q S n = ( a + l )
30 n ⇒ =6 
=
2  2  x+ z  5 × 10 
⇒ x , 6, z are in HP.  1 
= 15 × 109 = sin−1  
= 1635  2
35. What is the value of the sum
π  π 1 
32. If 0 > a < 1, the value of log10 a is 11
= Q sin 4 = 2 
negative. This is justified by
∑(in + in+1 ), where i = −1? 4
n =2
(a) Negative power of 10 is less than 1 sin 5x − sin 3x
(a) i (b) 2 i (c) − 2 i (d) 1 + i 37. What is equal to ?
(b) Negative power of 10 is between 0 cos 5x + cos 3x
and 1 Ê (c) We have,
(c) Negative power of 10 is positive 11 11 (a) sin x (b) cos x
(d) Negative power of 10 is negative
Σ (i n + i n + 1 ) = Σ i n(1 + i ) (c) tan x (d) cot x
n= 2 n= 2
sin 5 x − sin 3 x
Ê (b) Let log10 a = x 11 Ê (c) Given,
= (1 + i ) Σ i n
cos 5 x + cos 3 x
⇒ a = 10 x n= 2
5x + 3x 5x − 3x
= (1 + i ) [i 2 + i 3 + i 4 + ... + i 11 ] 2 cos . sin
It is given that = 2 2
0< a< 1  i 10 − 1 5x + 3x 5x − 3x
= (1 + i ) i 2  
2 cos . cos
 i −1 
⇒ 0 < 10 x < 1 2 2
   C + D  C − D
⇒ x must be negative n−1  r n − 1  Q sinC − sin D = 2 cos  2  ⋅ sin 2  and
Q a + ar + ar + K + ar = a
2
∴If 0 < a < 1, the value oflog10 a is negative  
  r − 1   C + D C − D 
cos C + cos D = 2 cos 
implies that negative power of 10 is
between 0 and 1. (1 + i )i 2( i 2 × 4 + 2 − 1)  2 
 cos  
 2  
=
( i − 1) 2 cos 4x sin x sin x
33. The third term of a GP is 3. What is = = = tan x
the product of the first five terms? − (1 + i ) ( i 2 − 1) 2 cos 4x cos x cos x
= [Q i 2 = − 1]
(a) 216 ( i − 1)
38. What is sin 105° + cos 105° equal to ?
(b) 226 − (1 + i )( − 1 − 1) 2 (1 + i )
(c) 243 = = (a) sin 50° (b) cos 50°
( i − 1) ( i − 1) 1
(d) Cannot be determined due to (c) (d) 0
2 (1 + i ) i +1 2
insufficient data = ×
( i − 1) ( i + 1)
Ê (c) Let a and r be the first term and Ê (c) We have, sin105° + cos 105°
common ratio of of the GP. 2 (i + 1 + i 2 + i ) = sin( 90° + 15° ) + cos 105°
=
∴ a3 = 3 i2 − 1 = cos 15° + cos 105°
⇒ ar 2 = 3 [Qan = ar n − 1] … (i) 2 (i + 1 − 1 + i ) [Qsin( 90 + θ) = cos θ]
=
Required product = a1 ⋅ a2 ⋅ a3 ⋅ a4 ⋅ a5 −1 − 1  105° + 15°   105° − 15° 
= 2 cos   cos  
= ( a )( ar )( ar 2 )( ar 3 )( ar 4 ) = − 2i  2   2 
= a 5r10 = ( ar 2 )5 1 1  C + D 
36. If sin x = , siny = , where Q cos C + cos D = 2 cos  2  
= ( 3)5
[from Eq. (i)] 5 10  
C − D
= 243 π π cos   
0< x < , 0 < y < , then what is   2  
2 2
( x + y ) equal to ?
38 NDA/NA Solved Paper 2018 (I)

= 2 cos 60° cos 45° = 2 ×


1
×
1
42. ABC is a triangle inscribed in a circle and in ∆OCD
2 x+ 6
2 with centre O. Let α = ∠ BAC, where tanα = … (ii)
 1  y+2 3
1 45° < α < 90°. Let β = ∠ BOC.
Q cos 60° = 2 , cos 45° = 2  From Eqs. (i) and (ii), we get
Which one of the following is
1 x+ 6
= correct? x
=
2 1 − tan2 α y y+2 3
(a) cos β =
39. In a ∆ABC, if a = 2, b = 3 1 + tan2 α ⇒ xy + 2 3 x = xy + 6 x
2 1 + tan2 α
and sin A = , then what is ∠ B equal (b) cos β = ⇒
x
= 3
3 1 − tan2 α y
2 tanα
to ? (c) cos β = ⇒ tanα = 3 [from Eq. (i)]
π π π π 1 + tan2 α
(a) (b) (c) (d) ⇒ α = 60°
4 2 3 6 (d) sinβ = 2 sin2 α
 1  3
Ê (b) We have, a = 2, b = 3 and sin A =
2 Ê (a) We know that angle subtended by a 44. What is tan −1   + tan −1  
3 chord at centre is always double the   4   5
Now, from sine formula angle subtended by it at any other part of equal to ?
sin A sin B the circle. π
= (a) 0 (b)
a b A 4
π π
2 (c) (d)
α 3 2
⇒ 3 = sin B
2 b
O Ê (b) We have
⇒ sin B = 1 tan−1   + tan−1  
1 3
β  4  5
π Q sin π = 1
⇒ B=
 
2 2 B C  1+ 3 
 
40. What is the principal value of = tan−1  4 5 
1 3
1 − × 
 2π  ∴ β = 2α  4 5
sin −1 sin  ?
 3 ⇒ cos β = cos 2α x+ y
[Q tan−1 x + tan−1 y = tan−1 ,
π π π 2π 1 − tan2 α 1 − xy
(a) (b) (c) (d) ⇒ cos β =
4 2 3 3 1 + tan2 α xy < 1]
−1  2π  1 + tan2 θ   5 + 12 
Ê (c) We have, sin  sin  Q cos 2θ =   
3  1 + tan2 θ 
= tan−1  20 
 π  20 − 3
= sin−1  sin  π −   43. If a flage-staff of 6 m height placed  
  3  20 
on the top of a tower throws a
= tan−1  
π 17
−1
= sin sin [Qsin( π − θ) = sinθ] shadow of 2 3 m along the ground, = tan−1 1
 17 
3
then what is the angle that the sun
π π Q tan−1 1 = π 
= makes with the ground ? =
3 4  4 
(a) 60° (b) 45°
 −1  −π π (c) 30° (d) 15°
Q sin sinθ = θ, if θ ∈  2 , 2   45. A spherical balloon of radius r
  Ê (a) Let OB and BD be the tower and subtends an angle α at the eye of an
flag-staff respectively. OA and AC be the
41. If x , x − y and x + y are the angles of shadow of tower and flag-staff observer, while the angle of
a triangle (not an equilateral respectively. elevation of its centre is β. What is
triangle) such that tan( x − y ), tan x D the height of the centre of the
and tan( x + y ) are in GP, then what 6m balloon (neglecting the height of the
is x equal to ? observer)?
π π B
r sinβ r sinβ
(a) (b) (a) (b)
4 3 α α
π π
xm sin   sin  
(c) (d) 2  4
6 2 α α
β
O A C r sin 
Ê (b) We have, ym 2 r sinα
x, x − y, x + y are the angles of a 2√3 m (c) (d)
sinα β
triangle. Since, sum of angles of a triangle Again let α be the angle that sun makes sin  
2
= π with the ground.
∴ x+ x− y+ x+ y= π ∴ ∠OAB = ∠OCD = α Ê (a) LetO bet the centre of the balloon, P be
⇒ 3x = π Now, in ∆OAB the eye of the observer and ∠APB be the
π x angle subtendd by the balloon at the eye
⇒ x = tanα = … (i)
3 y of the observer. ∠APB = α
NDA/NA Solved Paper 2018 (I) 39

 π  π
Ê (b) We have, Ê (a) Let f( x) = sin  x +  + cos  x + 
A sinα + sinβ = 0 = cos α + cos β 5 5
r  1 π π 
∴(sinα + sinβ )2 + (cos α + cos β )2 = 0 sin  x +  + cos  x +  
1
O = 2 
 2  5  2  5
⇒ sin2 α + sin2 β + 2 sinα sinβ + cos 2 α
  π π 
α/2 α B + cos 2 β + 2 cos α cos β = 0 sin  x + 5  cos 4 + 
= 2  
β ⇒ (sin α + cos 2 α ) + (sin2 β + cos 2 β )
2
 π π 
 cos  x +  sin 
P L X
+ 2 (cos α cos β + sinα sinβ ) = 0   5  4 
α  π π 
∴ ∠APO = ∠BPO = ⇒ 1 + 1 + 2 cos(α − β ) = 0 = 2 sin  x + +  
2 ⇒ 2 cos (α − β ) = − 2   5 4
In ∆OAP ⇒ cos(α − β ) = − 1  π π
= 2 sin x + + 
α OA  5 4
sin = ⇒ α −β = π
2 OP ⇒ α =β + π f( x ) attains maximum value, when
α r α
⇒ sin = ⇒ OP = r cosec … (i) 48. Suppose cos A is given. If only one π π π
2 OP z x+ + =
In ∆OPL,
 A 5 4 2
value of cos   is possible, then A π
OL  2 ⇒ x=
sinβ = 20
OP must be
51. What is the distance between the
⇒ OL = OP sinβ (a) An odd multiple of 90°
points which divide the line segment
α (b) A multiple of 90°
⇒ OL = r cosec . sinβ (c) An odd multiple of 180° joining ( 4, 3) and (5, 7 ) internally and
2
(d) A multiple of 180° externally in the ratio 2 : 3 ?
[from Eqs. (i)]
r sinβ Ê (c) We know that, (a)
12 17
(b)
13 17
∴ OL = A 5 5
α cos A = 2 cos −12
sin   2 17 6 17
2 A (c) (d)
Since, cos A is given and cos has only 5 5
sin( x + y ) a + b 2
46. If = , then what is one solution. So, A must be odd multiple Ê (a) Let P andQ be the points which divides
sin( x − y ) a − b of 180°. A( 4, 3) and B( 5, 7 ) internally and
tan x externally in the ratio 2 : 3 respectively.
equal to ? 49. If cos α + cos β + cos γ = 0, where  2 × 5 + 3 × 4 2 × 7 + 3 × 3
tan y ∴P= , 
π π π  2 +3 2 + 3 
(a)
a
(b)
b 0< α ≤ , 0 < β ≤ , 0 < γ ≤ , then
b a 2 2 2 2:3
a+ b a−b what is the value of
(c) (d) A (4, 3) P B (5, 7)
a−b a+ b sin α + sin β = sin γ ?
=  , 
(a) 0 (b) 3 22 23
Ê (a) We have, 5 2 3 2  5 5
sin( x + y) a + b (c) (d)
 2 × 5 − 3 × 4 2 × 7 − 3 × 3
= 2 2 and Q = 
sin ( x − y) a − b , 
 2 + 3 2 −3 
Ê (b) We have,
On using componendo and dividendo
cos α + cos β + cos γ = 0, 2
rule, we get
π π π
sin( x + y) + sin( x − y) a + b + a − b 0< α ≤ , 0< β ≤ ,0< γ ≤ Q A (4, 3) B (5, 7)
= 2 2 2
sin( x + y) − sin( x − y) a + b − a + b
cos α + cos β + cos γ = 0
⇒ π 3
x + y + x − y  x + y − x + y ⇒ α =β = γ =
2 sin   cos   2 = (2, − 5)
 2   2 
∴ sinα + sinβ + sin γ ∴Required distance = PQ
 x + y + x − y  x + y − x + y π π π
2 cos   sin   = sin + sin + sin
2
 2 − 22  +  − 5 − 23  = 12
2
   2  2 2 2 =     17
 5  5 5
=
2a   π π 
Q if θ ∈  0, 2  cos 2 = 0 52. What is the angle between the
2b    

sin x cos y a
= = 1 + 1 + 1= 3 straight lines
cos x sin y b (m 2 − mn )y = (mn + n 2 ) x + n 3 and
50. The maximum value of

tan x a
=  π  π (mn + m 2 )y = (mn − n 2 )x + m 3 ,
tan y b sin  x +  + cos  x +  , where
 5  5 where m > n ?
47. If sin α + sin β = 0 = cos α + cos β ,  2 mn  −1  4m n 
2 2
 π (a) tan−1  2  (b) tan  2 
x ∈ 0,  , is attained at m + n 
2
m − n 
2
where 0 < β < α < 2π, then which  2
one of the following is correct? π π π π  4m2n2 
(a) (b) (c) (d) (c) tan−1  4  (d) 45°
(a) α = π − β (b) α = π + β 20 15 10 2 m + n 
4
(c) α = 2 π − β (d) 2α = π + 2β
40 NDA/NA Solved Paper 2018 (I)

Ê (c) Given equations of lines are ∴Equation of line is Ê (c) We know that x1 distance of a point
1
( m − mn) y = ( mn + n ) x + n
2 2 3
y − (− 2 ) = ( x − 0) ( x1 y1 ) from the line Ax + By + C = 0 is
3 given as
and ( mn + m2 ) y = ( mn − n2 ) x + m3 1
⇒ y+2 = x  Ax + By1 + C 
Given equation of lines can be written as 3 Distance =  1 
mn + n2  A 2 + B2 
n3 ⇒ 3 y+2 3= x
y= 2 x+ 2 Statement I
m − mn m − mn ⇒ x − 3y − 2 3 = 0
(0, 0)
mn − n2 m3 54. What is the equation of the line
and y = x+
mn + m 2
mn + m 2 passing through the point of p
Let m1 and m2 be the slopes of given lines.
intersection of the lines
x + 2y − 3 = 0 and 2x − y + 5 = 0 and
mn + n2
∴ m1 = 2 parallel to the line y − x + 10 = 0 ? ax+by–c=0
m − mn (a) 7 x − 7 y + 18 = 0
(b) 5x − 7 y + 18 = 0 a⋅ 0 + b ⋅ 0 − c
mn − n2 ∴ p=
and m2 = (c) 5x − 5 y + 18 = 0
mn + m 2 a2 + b 2
(d) x − y + 5 = 0
c
Ifθ is the angle between these lines, then ⇒ p=
m − m2
Ê (c) Equation of line passing through
intersection point of lines x + 2 y − 3 = 0 a + b2
2
tanθ = 1
1 + m1m2 and 2 x − y + 5 = 0 is c2
⇒ p2 =
mn + n2 mn − n2 x + 2 y − 3 + λ (2 x − y + 5) = 0 … (i) a + b2
2

m2 − mn mn + m2 ⇒ (1 + 2 λ ) x + (2 − λ ) y + 5λ − 3 = 0
= (1 + 2 λ )
It is true.
mn + n2 mn − n2 ∴ Slope of above line = − Statement II
1+ . (2 − λ )
m2 − mn mn + n2 (0, 0)
Since line is parallel to y − x + 10 = 0
( mn + n2 )( mn + m2 ) − ( mn − n2 )( m2 − mn)
= − (1 + 2 λ ) − ( − 1)
( m2 − mn)( mn + m2 ) + ( mn + n2 )( mn − n2 ) = = p
(2 − λ ) 1
m n + m n + mn + m n − m n + m n
2 2 3 3 2 2 3 2 2
= ⇒ − (1 + 2 λ) = 2 − λ
m3n + m4 − m2n2 − m3n2 − m2n2 − mn3 x y
⇒ − 1 − 2λ = 2 − λ a +b
–1=0
+ m n − mn
2 2 3
⇒ −λ=3
+ mn3 − n4
⇒ λ=−3
0 0
4 m 2n 2 Putting λ = − 3 in Eq. (i), we get + −1
= a b
m4 − n 4 ∴ p=
x + 2 y − 3 − 3 (2 x − y + 5) = 0 2 2
 4m2n2   1 +  1 
∴ θ = tan−1  4  ⇒ 5 x − 5 y + 18 = 0    
 a  b
m − n 
4 Which is equation of required line.
55. Consider the following statements 1
⇒ p=
53. What is the equation of the straight 1 1
I. The length p of the perpendicular + 2
line cutting-off an intercept 2 from a2 b
from the origin to the line
the negative direction of Y -axis and
ax + by = c satisfies the relation ⇒
1
=
1 1
+ 2
inclined at 30° with the positive p2 a 2 b
c2
direction of X -axis ? p2 = 2 . It is true.
(a) x − 2 3y − 3 2 = 0 a + b2
Statement III
(b) x + 2 3y − 3 2 = 0 II. The length p of the perpendicular (0, 0)
(c) x + 3y − 2 3 = 0 from the origin to the line
(d) x − 3y − 2 3 = 0 x y
+ = 1 satisfied the relation p
Ê (d) From the given figure, it is clear that a b
1 1 1
2
= 2 + 2. y–mx–C=0
p a b
30º III. The length p of the perpendicular 0− m× 0−c
from the origin to the line ∴ p=
( − m)2 + (1)2
2 y = mx + c satisfies the relation
1 1 + m2 + c 2 ⇒ p=
c
2
= . m2 + 1
p c2
1 1 1 + m2
Slope of line = tan 30° = and line Which of the above is/are correct? ⇒ 2
=
3 (a) I, II and III (b) I only p c2
passes through the point ( 0, − 2 ). (c) I and II (d) II only It is false.
NDA/NA Solved Paper 2018 (I) 41

56. What is the equation of the ellipse 58. Let the coordinates of the points ⇒ ( x + 2 ) ( − 18) − ( y − 6) ( − 9) +
whose vertices are ( ± 5, 0) and foci A , B, C be (1, 8, 4 ), (0, − 11, 4 ) and ( z + 6) (18) = 0
are at ( ± 4, 0) ? (2, − 3, 1) respectively. What are the ⇒ 2 ( x + 2 ) − ( y − 6) − 2 ( z + 6) = 0
x2 y2 coordinates of the point D which is ⇒ 2x − y − 2z − 2 = 0
(a) + =1 ⇒ 2x − y − 2 z = 2
25 9 the foot of the perpendicular from A
x2 y2 on BC ? 60. A sphere of constant radius r
(b) + =1 (a) ( 3, 4, − 2 ) (b) ( 4, − 2, 5)
16 9 through the origin intersects the
(c) ( 4, 5, − 2 ) (d) (2, 4, 5)
x2 y2 coordinate axes in A , B and C. What
(c) + =1
25 16 Ê (c) We have, is the locus of the centroid of the ∆
x2 y2 A (1, 8, 4), B (0, − 11, 4) and C (2, − 3, 1)
(d) + =1 ABC?
9 25 ∴Equation of BC is (a) x 2 + y2 + z2 = r 2
x−0 y + 11 z−4 (b) x 2 + y2 + z 2 = 4r 2
Ê (a) We have, = =
2 − 0 − 3 + 11 1 − 4 (c) 9 ( x 2 + y2 + z 2 ) = 4r 2
Vertices = ( ± 5, 0) and Foci = ( ± 4, 0)
x y + 11 z − 4 (d) 3 ( x 2 + y2 + z2 ) = 2 r 2
∴ a = 5 and ae = 4 ⇒ = = = λ [say]
[Q vertex = ( ± a, 0) and focus ( ± ae, 0) 2 8 −3 Ê (c) Let A (a, 0, 0), B (0, b, 0) andC(0, 0,c )
4 ⇒ x = 2 λ, y = 8λ − 11, z = − 3λ + 4 ∴ Equation of sphere passing through
⇒e = A, B, C and origin is
5 A (1, 8, 4)
x 2 + y 2 + z2 − ax − by − cz = 0
b2
Now, e = 1−
a2 a2 b2 c 2
∴ Radius = r = + +
2 2 4 4 4
⇒  4 = 1 − b [Q a = 5]
  D (x, y, z) ⇒ 4r 2 = a 2 + b 2 + c 2
 5 ( 5)2 B C … (i)

16 b 2
Now, DR’s of Let (α, β, γ ) be the centroid of triangle.
⇒ = 1− a+ 0+ 0 0+ b+ 0
25 25 AD = < 2 λ − 1, 8λ − 11 − 8, − 3λ + 4 − 4 > ∴ α = ,β = ,
⇒ 16 = 25 − b 2 = < 2 λ − 1, 8λ − 19, − 3λ > 3 3
0+ 0+c
⇒ b2 = 9 Since, AD ⊥ BC γ =
3
∴2 (2 λ − 1) + 8 ( 8λ − 19) − 3 ( − 3λ ) = 0
⇒ b=3 ⇒ a = 3α, β = 3β, c + 3γ … (ii)
⇒ 4λ − 2 + 64λ − 152 + 9λ = 0
∴ Equation of ellipse is From Eqs. (i) and (ii), we have
⇒ 77 λ = 154
x2 y2 ( 3α )2 + ( 3β )2 + ( 3γ )2 = 4r 2
2
+ 2=1 ⇒ λ =2
a b ⇒ 9 (α 2 + β 2 + γ 2 ) = 4r 2
∴Coordinates of
x2 y2
⇒ + =1 D = (2 × 2, 8 × 2 − 11, − 3 × 2 + 4) ∴Locus of the centroid of ∆ABC is
25 9 = ( 4, 5, − 2 ) 9( x 2 + y 2 + z2 ) = 4r 2z
57. What is the equation of the straight 59. What is the equation of the plane 61. The coordinates of the vertices P , Q
line passing through the point (2, 3) passing through the points ( −2, 6, − 6),
and R of a triangle PQR are
and making an intercept on the ( −3, 10, − 9 ) and ( −5, 0, − 6) ?
positive Y -axis equal to twice its (1, − 1, 1), (3, − 2, 2) and (0, 2, 6)
(a) 2 x − y − 2 z = 2
intercept on the positive X -axis? (b) 2 x + y + 3 z = 3 respectively. If ∠ RQP = 9, then what
(a) 2 x + y = 5 (b) 2 x + y = 7 (c) x + y + z = 6 is ∠ PRQ equal to ?
(c) x + 2 y = 7 (d) 2 x − y = 1 (d) x − y − z = 3 (a) 30° + θ (b) 45° − θ
(c) 60° − θ (d) 90° − θ
Ê (b) Let the equation of line be Ê (a) Equation of the plane passing through
x2 y three points ( x1, y1, z1 ), ( x2, y2, z2 ) and Ê (d)
+ =1 ( x3, y3, z3 ) is P (1, –1, 1)
a b
x − x1 y − y1 z − z1
It is given that, b = 2 a and line passes
through the point (2, 3). x2 − x1 y2 − y1 z2 − z1 = 0
2 3 x3 − x1 y3 − y1 z3 − z1
∴ + =1
a 2a θ
Equation of plane is
4+ 3 Q (3, –2, 2) R (0, 2, 0)
⇒ =1 x − ( − 2) y−6 z − ( − 6)
2a
− 3 − ( − 2 ) 10 − 6 − 9 − ( − 6) = 0 DR’s of
⇒ 7 = 2a
− 5 − ( − 2 ) 0 − 6 − 6 − ( − 6) PQ = < 3 − 1, − 2 − ( − 1), 2 − 1 >
7
⇒ a= < a1, b1, c1 > = < 2, − 1, 1 >
2 x+2 y−6 z+ 6
7 and DR’s of
⇒ b = 2a = 2 × = 7 ⇒ −1 4 −3 =0 PR = < 0 − 1, 2 − ( − 1), 6 − 1 >
2 −3 −6 0
x y < a2, b 2, c 2 > = < − 1, 3, 5 >
∴Equation of line is + =1 Now, a1a2 + b1b 2 + c1c 2 =
7 /2 7 ⇒ ( x + 2 ) [4 × 0 − ( − 6) ( − 3)] − ( y − 6)
⇒ 2x + y = 7 [( − 1) ( 0) − ( − 3) ( − 3)] + ( z + 6) [( − 1) ( − 6) 2 × ( − 1) + ( − 1) × 3 + 1 × 5
− ( −3) ( 4)] = 0
42 NDA/NA Solved Paper 2018 (I)

= −2 − 3+ 5 = 0 ∴ Equation of line is
x y
+ =1 ⇒ ( x + 2 )2 + ( y − 3)2 + ( z − 4)2 = 36
∴ PQ ⊥ PR 5 5 ⇒ x 2 + 4 x + 4 + y 2 + 9 − 6 y + z2
⇒ ∠QPR = 90° ⇒ x+ y=5 + 16 − 8 z = 36
Case II ⇒ x 2 + y 2 + z2 + 4 x − 6 y − 8 z + 29 − 36 = 0
Now, by angle sum property
∠PQR + ∠QPR + ∠PRQ = 180° ⇒ x 2 + y 2 + z 2 + 4x − 6 y − 8 z = 7
(0, b) r r
⇒ θ + 90° + ∠PRQ = 180° 66. If a and b are vectors
⇒ ∠PRQ = 90° − θ r r such that
b
(2, 3) | a | = 2, | b| = 7 and
62. The perpendiculars that fall from r r
3:2 a × b = 3i + 2 j + 6k, then what is
$ $ $
any point of the straight line a (a, 0)
(0, 0) the acute angle between a and b ?
2x + 11y = 5 upon the two straight
lines 24 x + 7y = 20 and 4 x − 3y = 2 (a) 30° (b) 45° (c) 60° (d) 90°
From above figure, →
are
3a + 2 × 0 3 × 0 + 2b Ê (a) Given,|a| = 2
(a) 12 and 4 respectively = 2 and =3 →
(b) 11 and 5 respectively 3+2 3+2 |b| = 7
(c) Equal to each other ⇒ 3a = 10 and 2 b = 15 → →
(d) Not equal to each other 10 15 and a × b = 3$i + 2 $j + 6k$
⇒ a= and b =
Ê (c) Let ( − 3, 1) be a point on 2 x + 11y = 5 3 2 Q
→ → →
|a × b| = |a||b| sinθ

Now, perpendicular from ( − 3, 1) on ∴ Equation of line is → →


24 x + 7 y = 20 x y |a × b |
+ =1 ⇒ sinθ =
10 / 3 15 / 2 → →
24 ( − 3) + 7(1) − 20 |a||b|
=
⇒ 9 x + 4 y = 30
(24)2 + (7 )2 |3$i + 2 $j + 6k$|
=
64. What is the distance between the 2 ×7
− 72 + 7 − 20
= straight lines 3x + 4y = 9 and
576 + 49 32 + 2 2 + 62
6x + 8y = 15 ? = =
49
− 85 17 3 3 14 14
= = (a) (b)
7 1
25 5 2 10 ⇒ sinθ = =
(c) 6 (d) 5 14 2
Again, perpendicular from ( − 3, 1) on
Ê (b) Given equation of straight lines are ⇒ sinθ = sin 30°
4x − 3y = 2
3x + 4y = 9 … (i) ⇒ θ = 30°
4( − 3) − 3(1) − 2 r r
= and 6 x + 8 y = 15 67. Let p and q be the position vectors of
4 + ( − 3)
2 2
15
⇒ 3x + 4y = … (ii) the points P and Q respectively with
− 12 − 3 − 2 17 2 respect to origin O. The points R and
= = 15
16 + 9 5 9− S divide PQ internally and externally
2 respectively in the ratio 2 : 3. If
∴ Both perpendicular are equal to each ∴ Required distance =
→ →
other. 32 + 42 OR and OS are perpendicular, then
63. The equation of the line, when the [Q distance between two lines which one of the following is
portion of it intercepted between the ax + by = c1 and ax + by = c 2 is given correct?
axes is divided by the point (2, 3) in c 2 − c1  (a) 9 p2 = 4 q 2 (b) 4 p2 = 9 q 2
by 
the ratio of 3 : 2, is a 2 + b 2  (c) 9 p = 4 q (d) 4 p = 9 q
(a) Either x + y= 4 or 9x + y = 12 3 Ê (a) The points R and S divider s PQ
(b) Either x + y= 5 or 4x + 9 y = 30 = 2 =
3 internally and externally respectively in
(c) Either x + y= 4 or x + 9 y = 12 5 10 the ratio 2 : 3. The position vectors of R
(d) Either x + y= 5 or 9x + 4 y = 30 and S are
65. What is the equation of the sphere
Ê (d) Case I whose centre is at ( −2, 3,4 ) and radius

3p + 2 q

→ →
and 3 p − 2 q respectively.
is 6 units? 5
(0, b) (a) x 2 + y2 + z2 + 4x − 6y − 8z = 7 →

3p − 2 q

(b) x 2 + y2 + z2 + 6x − 4y − 8z = 7 OR =
b (c) x 2 + y2 + z2 + 4x − 6y − 8z = 4 5
(2, 3) → → →
(d) x 2 + y2 + z2 + 4x + 6y + 8z = 4 OS = 3 p − 2 q
2:3
(0, 0) a (a, 0) Ê (a) Given, centre = ( − 2, 3, 4) and radius →
Now, OR ⊥ OS
→

= 6 units → →
From above figure, Equation of the sphere having centre at ⇒ OR ⋅ OS = 0
2a + 3 × 0 2 × 0 + 3b (α,β, γ ) and radius r is  → →
= 2 and =3 ⇒  3 p + 2 q  . ( 3→ →
p − 2 q) = 0
2 + 3 2 + 3 ( x − α )2 + ( y − β )2 + ( z − γ )2 = r 2  
5
So, equation of sphere  
⇒ 2 a = 10 and 3b = 15
⇒ a = 5 and b = 5 ⇒ { x − ( − 2 )} 2 + ( y − 3)2 + ( z − 4)2 = 6 2 ⇒
→ → →
(3 p + 2 q ) ⋅ (3 p − 2 q ) = 0

NDA/NA Solved Paper 2018 (I) 43

→→ →→ →→ →→
⇒ 9 p. p − 6 p. q + 6 q. p − 4 q. q = 0 73. If f ( x ) = | x | + | x − 1|, then which
→ → → → → → → →
→2 →2 Now,a × b + b × c + c × a = λ (b × c ) one of the following is correct?
⇒ 9|p| − 4|q| = 0 → → → →
= 3 (b × c ) + (b × c ) + 2 (b × c )
→ → (a) f( x ) is continuous at x = 0 at x = 1
→→ → → → → → (b) f( x ) is continuous at x = 0 but not at
[Qa .a =a2 and a ⋅ b = b ⋅ a ] → →
= 6(b × c ) [from Eqs. (i) and (ii)] x =1
→2 →2
⇒ 9|p| = 4|q| (c) f( x ) is continuous at x = 1 but not at
on comparing, we get
x =0
⇒ 9 p2 = 4q 2 λ=6
r r (d) f( x ) is neither continuous at x = 0 nor
70. If the vectors K and A are parallel at x = 1
68. What is the moment about the point r to r
$i + 2$j − k$ of a force represented by each other, then what is kK × A Ê (a) We have,
equalr to ? r f( x ) = | x| + | x − 1|
3i$ + k$ acting through the point r r
 − 2 x + 1, x< 0
2
(a) k A (b) 0 2
(c) − k A (d) A 
2$i − $j + 3k$ ? → → → →
⇒ f( x ) =  x − x + 1, 0 ≤ x < 1
(a) − 3$i + 11$j + 9k$ (b) 3$i + 2 $j + 9k$ Ê (b) Since,a × b = 0, ifa and b are parallel.  x + x − 1, x≥1
→ → → →

So, k K × A = 0 if K and A are parallel to  − 2 x + 1, x< 0
(c) 3$i + 4$j + 9k$ (d) $i + $j + k$ 
each other. ⇒ f( x ) =  1, 0≤ x< 1
Ê (a) Given that, 71. When one of the following is correct  2 x − 1, x≥1


r = (2 $i − $j + 3k$ ) − ( $i + 2 $j − k$ ) in respect of the function f : R → R +
= $i − 3$j + 4k$ defined as f ( x ) = | x + 1| ? Clearly, lim f( x ) = 1 = lim f( x )
x → 0− 1 x → 0+
→ (a) f( x 2 ) = [f( x )]2 and lim f( x ) = lim f( x ).
and F = 3$i + k$
(b) f(| x|) = | f( x )| x → 1− x − 1+
→ → →
∴ Moment τ = r + F (c) f( x + y) = f( x ) + f( y) So, f( x ) is continuous at x = 0, 1.
(d) None of the above
= ( $i − 3$j + 4k$ ) × ( 3$i + k$ ) 74. Consider the function
Ê (d) Given, f( x) = | x + 1|  x 2 In | x | x ≠ 0
$i $j k$ By checking the options, we get f (x ) =  What is
= 1 −3 4  0 x =0
(a) f( x 2 ) = | x 2 + 1|
3 0 1 f ′(0) equal to ?
{ f( x )} 2 = ( x + 1)2
(a) 0 (b) 1
= $i ( − 3 − 0) − $j (1 − 12 ) + k$ ( 0 + 9) Which implies that f( x 2 ) ≠ { f( x )} 2 (c) − 1 (d) It does not exist
(b) f(| x |) = || x | + 1| Ê (a) Given function is
= − 3$i + 11$j + 9k$  x 2 ln| x|,
| f( x )| = || x + 1| = | x + 1| x≠ 0
r r r r f( x ) = 
69. If a + 2b + 3c = 0 and which implies that f(| x |) ≠ | f( x )|  0, x=0
r r r r
r r r r
a × b + b × c + c × a = λ ( b × c), (c) f ( x + y) = | x + y + 1| f ( h) − f ( 0 )
∴ f ′ ( 0) = lim
f( x ) + f( y) = | x + 1| +| y + 1| h→ 0 h
then what is the value of λ ?
which implies that f( x + y) ≠ f( x ) + f( y) h2 log h
(a) 2 (b) 3 = lim
So, option (d) is correct. h→ 0 h
(c) 4 (d) 6
72. Suppose f : R → R is defined by = lim hlog h = 0
Ê (d) Given that, h→ 0
→ → → 2
a + 2 b + 3c = 0 x 75. What is the area of the region
f (x ) = . What is the range of
→ → → 1 + x2 bounded by the parabolas
⇒ a + 2 b = − 3c
the function? y 2 = 6( x − 1) and y 2 = 3x ?
→ → → → →
⇒ (a + 2 b ) × b = − 3 c × b (a) [0, 1) (b) [0, 1] (c) (0, 1] (d) (0, 1) 6 2 6 4 6 5 6
(a) (b) (c) (d)

→ → →
a × b + 2 b × b = 3 (b × c )
→ → →
Ê (a) Let f( x) = y 3 3 3 3

→ → → → Then, y ≥ 0 and f( x ) = y Ê (c) Given,


[Q c × b = − b × c ] x2 y 2 = 6( x − 1) … (i)
∴ = y
→ → → → → →
x +1
2 and y2 = 3 x … (ii)
⇒ a × b = 3 (b × c ) [b × b = 0] … (i)
x2 + 1 1 on solving Eqs. (i) and (ii), we get
→ → →
Again 3c + a = − 2 b ⇒ 2
=
for y > 0 x = 2 and y = ± 6
x y
→ → → → →
⇒ (3 c + a ) × a = − 2 b × a 1 1− y x=y2/3 y2=6(x–1)
⇒ =
→ → → → → → x2 y y=√6
⇒ 3 c × a + a × a = 2 (a × b )
y
→ → → → ⇒ x=
⇒ 3(c × a ) = 2 (a × b ) 1− y
x=1
→ → → → y y
⇒ 3 (c × a ) = 6 (b × c ) Now, is real ⇒ ≥0
1− y 1− y
[from Eq. (i)] y=–√6
⇒ 0≤ y< 1
→ → → →
⇒ c × a = 2 (b × c ) … (ii) So, Range of f( x ) is [0, 1).
44 NDA/NA Solved Paper 2018 (I)

6
 y2 y2  ⇒ ( x + 6) ( x − 3) = 0 x −9
∴Required area = ∫ 1 +


3
 dy
⇒ x = 3, − 6
80. If f (x ) = , x ≠ 3 is
− 6
6 x − 2x − 3 2
Again, on differentiating it

6
y2  continuous at x = 3, then which one
=2 ∫  1 − 6
 dy
2
( − 6 − 4 x ) 36 − x 2 −
( 36 − 6 x − 2 x 2 )( −2 x ) of the following is correct?
0 d A
= 2 36 − x 2 (a) f( 3) = 0 (b) f( 3) = 15
.
 y 3
6
dx 2
( 36 − x ) 2 (c) f( 3) = 3 (d) f( 3) = − 15
.
= 2 y − 
 18 0 d 2A Ê (b) Since, f( x) is continuous at x = 3
At x = 3, = − 6 − 12 = − 18
6 dx 2 x2 − 9
 18 y − y 3  Therefore, f( 3) = lim
=2 ×   d A 2 x→ 3 x − 2x − 3
2

 18  0 ∴ <0
dx 2 Applying L ′ Hospital rule
 18 6 − 6 6 
=2 ×   So, at x = 3 is maximum. d 2
18 ( x − 9)
  x 3 1 dx
Now, cosα = = = f( 3) = lim
12 6 4 6 6 6 2 x→ 3 d
= = ( x 2 − 2 x − 3)
9 3 π π dx
⇒ cos α = cos ⇒ α =
3 3 2x
Directions (Q. Nos. 76-78) Consider = lim
x→ 3 2x − 2
the following information for the next 77. If the area of the trapezium is
maximum, what is the length of the 2⋅3 6
three items that follow Three sides of a = = = 1. 5
fourth side? 2⋅3 − 2 4
trapezium are each equal to 6 cm. Let
π
αε  0,  be the angle between a pair of
(a) 8 cm (b) 9 cm e
 2 (c) 10 cm (d) 12 cm 81. What is ∫ x In x dx equal to ?
adjacent sides. Ê (d) So, fourth side = x + 6 + x 1

= 3 + 6 + 3 = 12 e +1 e2 + 1
76. If the area of the trapezium is the (a) (b)
4 4
maximum possible, then what is α 78. What is the maximum area of the e −1 e2 − 1
equal to ? (c) (d)
trapezium? 4 4
π π π 2π
(a) (b) (c) (d) (a) 36 3 cm 2 (b) 30 3 cm 2 e
6 4 3 5 2 Ê (b) Let 1 = ∫1 x log xdx
(c) 27 3 cm (d) 24 3 cm 2
Ê (c) = [log x ⋅ ∫ xdx ] e1 − ∫  {log x}. ∫ xdx dx ]
e d
Ê (c) Maximum area = ( 6 + x) 36 − x
2
D 6 C 1  dx

= ( 6 + 3) 36 − 9 e
 x2  e 1 x2
6
√36 – x 2
6 = 9 × 27 = 9 × 27 = log x ⋅  −
2 1 ∫1 . dx
x 2

= 9 × 3 3 = 27 3 cm 2
α e2 1 1 2 e
x π = − × [ x ]1
A 6 x B
79. What is ∫ e x sin x dx equal to ? 2 2 2
1 e 2 [e 2 − 1]
∴ Area = A = ( 6 + 6 + 2 x ) 36 − x 2 0 = −
2 eπ + 1 eπ − 1 2 4
(a) (b) 2e − e 2 + 1 e 2 + 1
2
= ( 6 + x ) 36 − x 2 2 2 = =
eπ + 1 4 4

d( A) d
= [( 6 + x ) 36 − x 2 ] (c) e π + 1 (d) 2
dx dx 4
 
π 82. What is ∫ [ x 2 ] dx equal to (where
Ê (a) Let I = ∫ e sin x dx
x
−2 x
= ( 6 + x)   + 36 − x 2 0
 2 36 − x 2  0 [.] is the greatest integer function) ?
π
d x (a) 2 − 1 (b) 1 − 2
x ( 6 + x) = [sin x ⋅ e x ]π0 − ∫  dx {sin x} ⋅ e dx
= 36 − x 2 −  (c) 2 ( 2 − 1) (d) 3 − 1
36 − x 2 0
2
Ê (a) Let I = ∫ [ x ]dx
π 2
36 − 6 x − 2 x 2
= [sin x ⋅ e x ]π0 − ∫ cos x ⋅ e dx x
= 0
36 − x 2 0
1 2
 π  = ∫ [ x 2 ]dx + ∫[x
2
] dx
For maximum area, = 0 − [cos x ⋅ e x ]π0 + ∫ sin x ⋅ e xdx 
0 1
dA  
=0 0
1 2
dx
⇒ I = − [− e π − 1] − I = ∫ 0 dx + ∫ 1 dx
⇒ 36 − 6 x − 2 x = 0
2

I + I = eπ + 1
0 1

⇒ 2 x 2 + 6 x − 36 = 0  0 0 ≤ x < 1 
⇒ 2I = eπ + 1 Q [ x ] =  
⇒ x 2 + 3 x − 18 = 0  1 1 ≤ x < 2 
⇒ x 2 + 6 x − 3 x − 18 = 0 eπ + 1
⇒ I= = 0 + [x ]1 2 = 2 − 1
2
⇒ x ( x + 6) − 3 ( x + 6) = 0
NDA/NA Solved Paper 2018 (I) 45

83. What is the maximum value of Which is possible 89. What is the period of the the
16sin θ − 12sin θ ? 2 ∴Domain of f( x ) = ( − ∞, 0) function f ( x ) = sin x ?
3 4 16 π π
(a) (b) (c) (d) 4 86. What is the solution of the (a) (b)
4 2
4 3 3 differential equation x dy − y dx =0 ?
(c) π (d) 2 π
Ê (c) Let f( x) = 16sinθ − 12 sin θ
2
(a) xy = c
(b) y = cx Ê (d) We have,
= − 12 sin2 θ − 16 sinθ (c) x + y = c f( x ) = sin x
 12 
(d) x − y = c
= − 12 sin2 θ − sinθ
4
 3  Ê (b) Given differentiation equation
xdy − ydx = 0 –2π –π O π 2π 3π 4π
 2
2
4
= − 12   sinθ −  −  ⇒ xdy = y dx
  3  9 
2
Variable seprate on both sides
= − 12  sinθ −  +
2 16 dy dx f( x + 2 π ) = sin( x + 2 π ).
  ⇒ =
 3 3 y x = sin x
16
∴ f( x ) ≤ On integration both sides, we get = f( x )
3 ∴Period of f( x ) is 2 π.
16 dy dx
∴ Maximum value of f( x ) = ⇒ ∫ y =∫ x
90. What is ∫
3 dx
equal to ?
84. If f : R → S defined by ⇒ log y = log x + log c 2x − 1
(a) ln (2 x − 1) + C
f ( x ) = 4 sin x − 3 cos x + 1 is onto, [where logc is integrating constant]
ln (1 − 2 −x )
then what is S equal to ? ⇒ y = xc (b) +C
(a) [− 5, 5] (b) ( −5, 5) ln 2
⇒ y = cx
(c) ( −4, 6) (d) [− 4, 6] ln (2 − x − 1)
87. What is the derivative of the (c) +C
Ê (d) We have 2 ln 2
function
f( x ) = 4 sin x − 3 cos x + 1 ln (1 + 2 −x )
f ( x ) = e tan x + ln ( sec x ) − e ln x at (d) +C
We know that, ln 2
π
− 4 2 + ( − 3)2 ≤ 4 sin x − 3 cos x x= ? dx
4 Ê (b) Let I = ∫ 2 x2 − 1
≤ 4 + ( − 3)
2 2
e
(a) (b) e (c) 2e (d) 4e
2 −x
[Q − a + b ≤ a sin x + b cos x
2 2 2 = ∫ 1 − 2 − x dx
Ê (c) We have,
≤ a2 + b 2 ] F( x ) = e tan x + log(sec x ) − elog x 1 2 − x log 2
⇒ − 5 ≤ 4 sin x − 3 cos x ≤ 5
=
log 2 ∫ 1 − 2 −x
dx
f( x ) = e tan x + log(sec x ) − x
⇒ − 5 + 1 ≤ 4 sin x − 3 cos x + 1 ≤ 5 + 1 Put 1 − 2 −x = t
⇒ − 4 ≤ f( x ) ≤ 6 [Q a log a b = b] −x
⇒ 2 log 2 dx = dt
∴ f( x ) ∈ [− 4, 6] On differentiating with respect to x both 1 dt
log 2 ∫ t
since, f( x ) is onto. the sides, we get ∴ I=
∴ S = Range of f = [− 4, 6] 1
f ′ ( x ) = e tan x ⋅ sec 2 x + ⋅ sec x tan x − 1 1
85. For f to be a function, what is the sec x = . logt + C
log 2
1 = sec 2 xe tan x + tan x − 1 log (1 − 2 − x )
domain of f , if f ( x ) = ? = +C
|x | − x tan
π
π π log 2
∴[f ′ ( x )] π =e 4 ⋅ sec 2⋅ + tan − 1
(a) ( − ∞, 0) (b) ( 0, ∞ ) x= 4 4
(c) ( − ∞, ∞ ) (d) ( − ∞, 0)
4 91. The order and degree of the
= e1( 2 )2 + 1 − 1 differential equation y 2 = 4a ( x − a ),
Ê (a) We have, = e ⋅ 2 + 1 − 1 = 2e
1 where ‘a’ is an arbitrary constant, are
f( x ) =
| x| − x 88. Which one of the following respectively
differential equations has a periodic (a) 1, 2 (b) 2, 1
f( x ) is defined, if (c) 2, 2 (d) 1, 1
solution?
| x| − x > 0
d 2x d 2x Ê (a) We have,
⇒ | x| > x (a) 2
+ µx = 0 (b) 2
− µx = 0 y 2 = 4a ( x − a ) … (i)
dt dt
Case I x > 0
dx dx ⇒ y = 4ax − 4a
2 2
∴ x > x [Q| x | = x, x > 0] (c) x + µt = 0 (d) + µxt = 0
which is not possible
dt dt On differentiating both sides, we get
d 2x dy
Case II x < 0 Ê (a) + µx = 0 is the differential 2y = 4a
dt 2 dx
∴ − x > x [Q| x | = − x, x < 0]
equation of simple harmonic motion, 1 dy
⇒ 0> 2x ⇒ a= y … (ii)
which has a periodic solution. 2 dx
⇒ x< 0
46 NDA/NA Solved Paper 2018 (I)

On putting the values of a from Eq. (ii) in ⇒ b 3 − ( − b )3 = 2 [Q from Eq. (ii)] = lim
3
Eq. (i), we get h→ 0 4 2 x + 3h
⇒ 2b = 2 3

1 dy  1 dy  3 1 3
y2 = 4 × y x− y  ⇒ b3 = 1 = . =
2 dx  2 dx  4 2x + 0 4 2x
2 ⇒ b =1
− y 2  
dy dy ⇒ a= −1
⇒ y 2 = 2 xy [from Eq. (ii)] 97. If f ( x ) is an even function, where
dx  dx 
∴Order = 1 and degree = 2
1
f ( x ) ≠ 0, then which one of the
94. What is ∫ x (1 − x )9 dx equal to ? following is correct?
92. What is the value of 0
(a) f ′( x ) is an even function
π /4 1 1
(a) (b) (b) f ′( x ) is an odd function
∫ ( sin x − tan x ) dx ? 110
1
132
1
(c) f ′( x ) may be an even or odd function
−π / 4 (c) (d) depending on the type of function
1  1  (b) 1 148 240 (d) f ′( x ) is a constant function
(a) − + ln   1
 2
∫ x (1 − Ê (b) We have, f( x) is an even function.
2 2
Ê (a) Let I = x )9 dx
(c) 0 (d) 2 0 ∴ f( − x ) = f( x )
π /4
∫− π / 4 (sin x − tan x) dx
1
Ê (c) Let I = = ∫ (1 − x ) x dx 9 On differentiating both the sides, we have
− f ′( − x) = f ′( x)
Let f( x ) = sin x − tan x 0

 a a  ⇒ f ′( − x) = − f ′( x)
∴ f( − x ) = sin( − x ) − tan( − x ) Q ∫ f( x ) dx =
 0
∫ f( a − x ) dx 

∴ f ′( x ) is an odd function.
= − sin x + tan x 0 2 dy
[Qsin( − θ) = − sinθ, tan( − θ) = − tanθ]  x10
1 98. If y = e x sin 2x , then what is at
x11 
1

= − (sin x − tan x ) = ∫ ( x − x ) dx = 
9 10
− 
dx
0  10 11  0 x = π equal to ?
= − f( x )
=  −  =
2 2
1 1 1 (a) (1 + π ) e π (b) 2 π e π
∴ f( x ) is odd function.  10 11 110 2 2
π /4 (c) 2e π (d) e π
∴ I= ∫− π / 4 (sin x − tan x) dx 95. What is lim
tan x
equal to Ê (c) We have, y = e sin2 x
x2

=0 x → 0 sin 2x
On differentiating both the sides, we get
[Q ∫ f( x ) dx = 0, if f( x ) is odd]
a
1
−a (a) dy 2 2
2 = 2 cos 2 xe x + 2 xe x sin2 x
b b dx
2 (b) 1
93. If ∫ x 3 dx = 0 and ∫ x 2 dx = , then (c) 2 ∴  
dy 2 2
= 2 cos 2 πe π + 2 πe π sin2 π
3  dx  x = π
a a (d) Limit does not exist
what are the values of a and b 2 2
Ê (a) We have, = 2(1) e π + 2 πe π ( 0)
respectively? tan x 2
(a) − 1, 1 (b) 1, 1 lim
x → 0 sin 2 x
= 2e π
(c) 0, 0 (d) 2, − 2
By using L’ Hospital rule, we have 99. What is the solution of
Ê (a) We have, sec 2 x (1 + 2x )dy − (1 − 2y )dx = 0 ?
b = lim
(a) x − y − 2 xy = c
∫ x 3dx = 0 x → 0 2 cos 2 x
(b) y − x − 2 xy = c
a 1
= lim (c) y + x − 2 xy = c
b x → 0 2 cos 2 x ⋅ cos 2 x
 x4  (d) x + y + 2 xy = c
⇒   =0 1 1
 4 a = =
2 cos 0° cos 2 0° 2 × 1 × 1 Ê (a) We have,
b 4 − a4 (1 + 2 x ) dy − (1 − 2 y) dx = 0
⇒ =0 =
1
4 ⇒ (1 + 2 x ) dy = (1 − 2 y) dx
2
⇒ b = a4
4 dy dx
⇒ =
2x + 3h − 2x 1− 2y 1+ 2x
⇒ b=± a … (i) 96. What is lim equal
x→0 2h
b On integrating both the sides, we get
But ∫ x dx = 0 and x is an odd function.
3 3 to ? dy dy
a (a)
1
(b)
1 ∫1− 2y = ∫1+ 2x
∴ a=−b … (ii) 2 2x 2x
1 1
b (c)
3
(d)
3 ⇒ − log(1 − 2 y) = log(1 + 2 x ) + C ′
2 2 2
Again, ∫ x 2dx = 2 2x 4 2x
3 ⇒ − log(1 − 2 y) = log(1 + 2 x ) + 2C ′
a 2 x + 3h − 2 x
b Ê (d) xlim
→0
⇒ log(1 + 2 x ) + log(1 − 2 y) = − 2C ′
x 3
2 2h
⇒   = ⇒ log(1 + 2 x ) (1 − 2 y) = − 2C ′
By using, L’ Hospital rule, we get
 a 3
3
⇒ (1 + 2 x ) (1 − 2 y) = e − 2C ′
1
b 3 − a3 2 .3− 0 − 2C ′
2 2 x + 3h ⇒ 1 − 2 y + 2 x − 4 xy = e
⇒ =
3 3 = lim ⇒ 2 x − 2 y − 4 xy = e − 2C ′
−1
h→ 0 2
⇒ b 3 − a3 = 2
NDA/NA Solved Paper 2018 (I) 47

1 − 2C ′ Statement II
⇒ x − y − 2 xy = (e − 1) 106. Consider the following statements
2 P ( A ∩ B ) = P( B) − P( A ∩ B) is wrong as I. If the correlation coefficient
⇒ x − y − 2 xy = C
1 − 2C ′
P ( A ∩ B ) = P( A ) − P( A ∩ B). rxy = 0, then the two lines of
[where C = (e − 1)]
2 Statement III regression are parallel to each
other.
P( A ∩ B) = P( B) × P   is correct. [by
A
100. What are the order and degree, II. If the correlation coefficient
 B
respectively, of the differential rxy = 1, then the two lines of
2 conditional theorem]
 d 3y   dy 
5
regression are perpendicular to
equation  3  = y 4 +   ? Hence, Statements I and III are correct.
 dx   dx  each other.
103. If the correlation coefficient between Which of the above statements is/are
(a) 4, 5 (b) 2, 3 x and y is 0.6, covariance is 27 and
(c) 3, 2 (d) 5, 4 correct?
variance of y is 25, then what is the (a) I only
Ê (c) Given differential equation is variance of x? (b) II only
2
 d 3y  dy 
5
(a)
9
(b)
81 (c) Both I and II
 3 = y +  
4

 dx   dx  5 25 (d) Neither I nor II

d 3y
(c) 9 (d) 81 Ê (d) According to correlation condition,
Here, highest order derivative is 3
.
Ê (d) Given, σ ( y) = 25 ⇒ σ( y) = 5
2
If correlation coefficient rr ⋅y = 0, then lines
dx
C⋅V of regression are parpendicular
So, order = 3, and degree = 2 Correlation coefficient =
σx ⋅ σy And if rry = 1, then lines of regression are
101. In a Binomial distribution, the mean parallel.
27 So, both statements are wrong.
is three times its variance. What is 0.6 =
σx × 5
the probability of exactly 3 successes 107. If 4 x − 5y + 33 = 0 and
out of 5 trials ? ⇒ σx =
27
=
27
=9 20x − 9y = 107 are two lines of
(a)
80
(b)
40 0.6 × 5 3
regression, then what
243 243
∴ Variance of x = σ 2( x ) = ( 9)2 = 81 are the values of x and y respectively?
20 10
(c) (d) (a) 12 and 18 (b) 18 and 12
243 243 104. The probabilities that a student will (c) 13 and 17 (d) 17 and 13
solve Question A and Questions B
Ê (a) According to the question, Ê (c) Given lines of regression are
Mean = 3 (Variance) are 0.4 and 0.5 respectively. What is
4 x − 5 y + 33 = 0 … (i)
⇒ np = 3 npq
the probability that he solves atleast
one of the two questions? and 20 x + 9 y − 107 = 0 … (ii)
[where n = number, of trials]
(a) 0.6 (b) 0.7 on multiplying Eq. (i) by 5 and subtract
1
⇒ q = (c) 0.8 (d) 0.9 Eq. (ii) from it, we get
3
20 x − 25 y + 165 = 0
Q p+ q =1 Ê (b) Given that,
P( A ) = 0.4 20 x − 9 y − 107 = 0
1
⇒ p+ =1 − + +
3 and P( B) = 0.5
2 − 16 y = − 272
⇒ p= Q P( A ∪ B) = 1 − P ( A ′ ∩ B′ )
y = 17
3
= 1 − [(1 − 0.4) × (1 − 0.5)] on putting the value of y in Eq. (i), we get
3 2
p( X = 3) = C 3   ×  
2 1
∴ 5
= 1 − ( 0.6)( 0.5) 4 x − 85 + 33 = 0
 3  3
3 2 = 1 − ( 0.3) = 0 .7 ⇒ 4 x = 52 ⇒ x = 13
×   ×   =
5! 2 1 80
= The mean of two regression lines are the
3! 2 !  3   3 243 105. Let x be the mean of x 1, x 2 , x 3 ,… x n .
solution set at given regression lines,
If x i = a + cyi for some constants a Here, X = 13 and Y = 17
102. Consider the following statements
and c, then what will be the mean of 108. Consider the following statements
I. P ( A ∪ B ) = P ( A ) y1, y 2 , y 3 , …, yn ?
1 I. Mean is independent of change in
= P( B ) − P( A ∩ B ) (a) a + c x (b) a − x scale and change in origin.
c
II. P ( A ∩ B ) = P ( B ) − P ( A ∩ B ) 1 x −a II. Variance is independent of
(c) x −a (d)
c c change in scale but not in origin.
III. P ( A ∩ B ) = P ( B ) P ( A | B )
Ê (d) Given that, Which of the above statements is/are
Which of the above statements are
Mean of x1, x2, x3, .... xn i.e. x = x correct?
correct? (a) I only (b) II only
(a) I and II (b) I and III Now, we have
(c) Both I and II (d) Neither I nor II
(c) II and III (d) I, II and III xi = a + cyi
Ê (d) Since, mean changes with changes in
Ê (b) Here, 1
⇒ yi = ( x − a ) origin. So, Statement I is wrong.
Statement I c And variance is independent to the choice
P( A ∪ B) = P( A ) + P( B) − P( A ∩ B) is 1
⇒ y = ( x − a) of origin. So, Statement II is also wrong.
correct. c Hence, both statements are wrong.
48 NDA/NA Solved Paper 2018 (I)

109. Consider the following statements So, new mean = 5 + 5 116. A box has ten chits numbered 0, 1, 2,
I. The sum of deviations from mean = 10 3, …, 9. First, one chit is drawn at
is always zero. But standard deviation will remain same. random and kept aside. From the
II. The sum of absolute deviations is Hence, coefficient of variation remaining, a second chit is drawn at
minimum when taken around σ random. What is the probability that
= × 100
median. mean the second chit drawn is ‘‘9’’ ?
2 1 1
Which of the above statements is/are = × 100 = 20 (a) (b)
10 10 9
correct. 1
(c) (d) None of these
(a) I only (b) II only 113. A train covers the first 5 km of its 90
(c) Both I and II (d) Neither I nor II journey at a speed of 30 km/h and the
(c) By the property of deviation both Ê (a) Let E1 be the event at drawing a chit
next 15 km at a speed of 45 km/h.
statement are correct. which is not 9 and E 2 be the event of
What is the average speed of the drawing second chit bearing number 9.
110. What is the median of the numbers train? 9
C 9
4.6, 0, 9.3, −4.8, 7.6 2.3, 12.7, 3.5, 8.2, (a) 35 km/h (b) 37.5 km/h ∴ P( E1 ) = 10 1 =
(c) 39.5 km/h (d) 40 km/h C1 10
6.1, 3.9, 5.2 ?
Total distance 1
(a) 3.8 (b) 4.9 C1 1
Ê (d) Average speed = and P( E 2 ) = =
(c) 5.7 (d) 6.0 Total time 9
C1 9
5 + 15 20
Ê (b) On arranging the given number is = = ∴ Required probability = P( E1 ) ⋅ P ( E 2 )
ascending order, we have 5 15 1 1
+ + 9 1 1
− 4.8, 0, 2.3, 3.5, 3.9, 4.6, 5.2, 6.1, 7.6, 8.2, 30 45 6 3 = × =
20 10 9 10
9.3, 12.7 =
1+ 2 117. One bag contains 3 white and 2 black
Here, n = 12
6 balls, another bag contains 5 white
So, median
20 × 6 and 3 black balls. If a bag is chosen at
= = 40 km/h
Value of   th number
12
3 random and a ball is drawn from it,
 2 
what is the change that it is white?
+ Value of 
12  114. Two fair dice are rolled. What is the
+ 1 th number 3 49 8 1
 2  probability of getting a sum of 7? (a) (b) (c) (d)
= 8 80 13 2
2 1 1
(a) (b)
Value of 6th number + Value 7th number 36 6 Ê (b) Let E1 be the event of selecting the
=
2 7 5 first bag and E 2 be the event of selecting
(c) (d)
4.6 + 52 . 12 12
= the second bag. Let A be the event of
2
= 4.9
Ê (b) Here, n(S ) = 36 drawing white ball.
and E be the event of getting a sum of 7 So, by theorem at total probability.
111. In a test in Mathematics, 20% of the on two fair dice.  A  A
P( A ) = P( E1 ) × P   + P( E 2 ) × P  
students obtained ‘‘first class’’. If the = {(1, 6), (2, 5), (3, 4), (4, 3), (5, 2), (6, 1)}  E1   E2 
data are represented by a pie chart, ∴ n( E ) = 6 1 3C1 1 5C
what is the central angle n( E ) 6 1 = ×5 + × 8 1
So, required probability = = = 2 C1 2 C1
corresponding to ‘‘first class’’? n(S ) 36 6
1  3 5  1 24 + 25
(a) 20° (b) 36° = + = ×
(c) 72° (d) 144° 115. If A and B are two events such that 2  5 8  2 40
2P ( A ) = 3P ( B ), where 1 49 49
Ê (c) Pie chart contains total angle equal to = × =
360°. 0 < P ( A ) < P ( B ) < 1, then which one 2 40 80
So, central angle corresponding to “First of the following is correct?
118. Consider the following in respect of
class” (a) P( A| B) < P( B| A ) < P( A ∩ B) two events A and B
= 20% of 360° (b) P( A ∩ B) < P( B| A ) < P( A | B)
I. P ( A occurs but not
20 (c) P( B| A ) < P ( A | B) < P( A ∩ B)
= × 360° B ) = P ( A ) − P ( B ) if B ⊂ A
100 (d) P( A ∩ B) < P( A | B) < P( B| A )
II. P ( A alone or B alone occurs) =
= 72 ° Ê (b) Given that, 2 P( A ) = 3 P( B) P( A ) = P( B ) − P( A ∩ B )
112. The mean and standard deviation of ⇒2
P( A )
=
3P( B) III. P ( A ∪ B ) = P ( A ) + P ( B ) if A and
a set of values are 5 and 2 P( A ∩ B) P( A ∩ B) B are mutually exclusive
respectively. If 5 is added to each [dividing both sides by P( A ∪ B)] Which of the above is/are correct?
value, then what is the coefficient of 1 P ( A ∩ B) 1 P ( A ∩ B)
⇒ × = × (a) I only (b) I and III
variation for the new set of values? 2 P( A ) 3 P( B) (c) II and III (d) I and II

× P   = P  
(a) 10 (b) 20
(c) 40 (d) 70 ⇒
1 B 1 A Ê (b) If B ⊂ A, then P( A − B)
2  A 3  B
= P( A ) − P( A ∩ B) = P( A ) − P( B)
Ê (b) Given, mean = 5
P   < P  
B A [QB ⊂ A ⇒ A ∩ B = B]

and standard deviation ( σ ) = 2  A  B
So, Statement I is correct.
Since, 5 is added to each value.
NDA/NA Solved Paper 2018 (I) 49

P (A alone or B alone) Favourable numbers of selecting two N−1


(c) σ =
= P( A ) − P( A ∩ B) + P( B) − P( A ∩ B) members as men 12
= P( A ) + P( B) − 2 P( A ∩ B) = 4C 2 × 5C1 N2 − 1
(d) σ =
So, Statement II is wrong.
4
C 2 × 5C1 6N
So, required probability = 9
C3 1
Ê (b)Q σ = ΣX i − ( X )
If A and B are mutually exclusive, 2 2 2

then P( A ∩ B) = 0 4 ×3 5 N
× 1
⇒ P( A ∪ B) = P( A ) + P( B) 2 ×1 1 = (12 + 2 2 + ... + N 2 )
=
So, Statement III is correct. 9× 8×7 N
2
3×2 ×1
−  (1 + 2 + 3 + ... + N )
Here, Statement I and III are correct. 1
2 ×3×5 5  N 
119. A committee of three has to be = =
3 × 4 × 7 14 1 N ( N + 1) (2 N + 1)  ( N + 1)
2
chosen from a group of 4 men and 5 = × −
 2 
women. If the selection is made at 120. The standard deviation σ of the first N 6
random, what is the probability that N natural numbers can be obtained N2 − 1
exactly two members are men? =
using which one of the following 12
5 1
(a) (b) formulae?
14 21 N2 − 1
N2 − 1 σ =
3 8 (a) σ = 12
(c) (d) 12
14 21
N2 − 1
Ê (a) Total number of selecting three (b) σ =
members = C 3
9 12

PAPER II English Language and General Studies


Part A (English Language)
Directions (Q. Nos. 1-10) Each item 4. The judge told that he would analyse Ê (c) dialogist is the correct answer. An
in this section consists of a sentence the evidence and then deliver the interliocutor is the person who engages
with an underlined word/words verdict. himself in dialogues. Some other
followed by four words. Select the (a) liberate (b) surrender synonyms are interviewer, speaker, etc.
option that is nearest in meaning to the (c) transfer (d) pronounce 8. He spends his money lavishly.
underlined word/words and mark your
Ê (d) pronounce is the correct synonym of (a) hesitatingly
response in your answer sheet deliver. (b) generously
accordingly. Some other synonyms are accepted, (c) foolishly
acknowledged, assumed, etc. (d) carefully
1. I do not want you to lead a life of
sycophancy as you did during the 5. The growth and development of the Ê (b) generously is the correct answer.
Other options do not match here. Lavishly
foreign rule. peasant movement was indissolubly may also mean to spend money
(a) admiration (b) love linked with the national struggle for extravagantly.
(c) appreciation (d) flattery
freedom.
Ê (d) A sycophant is one who praises (a) firmly (b) vaguely
9. The government's new policies will
people in authority in a way that is not (c) individually (d) steadily come into force from the next fiscal
sincere, usually in order to get some year.
advantage of them. So, ‘flattery’ is the Ê (a) In the given sentence, indissolubly (a) calendar (b) academic
means incapable of being broken. So,
nearest in meaning to sycophancy. (c) financial (d) leap
‘firmly’ is the nearest in meaning to it.
2. In India, it has become easy to attack Ê Fiscal means relating to money and
6. Weather conditions have been finances. So, option (c) financial is
cultural artefacts these days.
improving over the past few days. nearest in meaning to it.
(a) beckon (b) assault
(a) mending (b) amending
(c) belch (d) appreciate 10. Abundant food was· available for the
(c) becoming better (d) advancing
Ê (b) assault is the correct synonym of soldiers in the mess.
attack. Other synonyms are beat, strike Ê (c) becoming better is the perfect (a) little
synonym of improving. Some other
at, knock down, etc. (b) plentiful
synonyms are correcting, elaborating,
(c) delicious
3. A local court granted bail to the fixing, etc. (d) wholesome
criminal on Thursday.
7. The confusion on the interlocutor’s Ê (b) plentiful is the correct answer for the
(a) confessed (b) donated
(c) allowed (d) yielded face was gratifying. word abundant which means in large
(a) government officer (b) party worker quantity. Some other synonyms are
Ê (c) is the correct answer as allowed is the (c) dialogist (d) revolutionary plenty, profuse, bounteous, etc.
synonym of granted.
50 NDA/NA Solved Paper 2018 (I)

Directions (Q. Nos. 11-20) Each item Ê (c) coarse, from the given options is Ê (c) has error of correct degree of
in this section consists of a sentence correct. Other antonyms could be adjective. Here, we will use superlative
with an underlined word/words unsatisfactory, ill, etc. degree of greater i.e. ‘greatest’ as we are
followed by four words. Select the comparing one with many people.
18. There is no likeness between him
option that is opposite in meaning to and his brother. 24. In my younger days (a)/I could run
the underlined word/words and mark (a) unlikeliness (b) unlikelihood four miles (b)/at a stretch. (c)/No
your response in your Answer Sheet (c) dissimilarity (d) disaffinity error. (d)
accordingly.
Ê (c) dissimilarity is the correct antonym as Ê (a) has error of correct syntax ‘When I was
11. The country's economy must be likeness means resemblance and young’ is better construction.
geared to wartime requirements. dissimilarity means when there is nothing
similar or resembling with the other. 25. The owner (a)/as well as his servants
(a) subordinated to
(b) related to
(b)/is honest. (c)/No error. (d)
19. Cultural diversity in the working
(c) adjusted to
place is good for business. Ê (d) The sentence is correct.
(d) unlinked to
(a) uniformity (b) conformity
Ê (d) unlinked to is correct antonym of word (c) identity (d) similarity
Directions (Q. Nos. 26-30) Given
geared which means to adjust or adapt. below are some idioms/phrases
Ê (a) diversity means the condition of followed by four alternative meanings
12. Why does fire attract insects? having or being composed of different
to each. Choose the response (a), (b), (c)
(a) discharge (b) destroy types of people or things. So, its opposite
or (d) which is the most appropriate
(c) repel (d) remove will be uniformity.
expression.
Ê (c) repel is opposite in meaning to attract 20. The company was liquidated within
as it means to force something or 26. Cry over spilt milk
five years. (a) Complaining about a loss in the past
someone to move away or stop attacking
(a) bankrupt (b) closed down (b) Too much inquisitive about something
you. (c) flourishing (d) privatised (c) When something is done badly to save
13. The party was excellent, and I would Ê (c) liquidate means to cause a business money
like to thank all the people to close, so that its assets can be sold to (d) Dealing with a problem only in an
concerned. pay its debts. So, its opposite would be emergency situation
‘flourishing’.
(a) cared (b) attentive Ê (a) Complaining about a loss in the past is
(c) dependable (d) uninvolved the correct meaning of the idiom ‘Cry over
Ê (d) uninvolved is correct antonym of
Directions (Q. Nos. 21-25) Each item spilt milk.’
concerned which means interested or in this section has a sentence with three
27. Cut the mustard
involved. underlined parts labelled (a), (b) and
(a) Prepare spices out of mustard seeds
(c). Read each sentence to find out
14. He is very serious by temperament. whether there is any error in any
(b) To come up to expectations
(a) grave (b) trivial (c) Making absurd expectations
underlined part and indicate your
(c) sober (d) stupid (d) Very enthusiastic
response in the Answer Sheet against
Ê (b) trivial is the only correct antonym from the corresponding letter i.e., (a) or (b) or Ê (b) To come up to the expectations is the
the given options. Grave and sober are correct meaning of the idiom ‘cut the
(c). If you find no error, your response
synonyms of serious. Stupid does not mustard’.
should be indicated as (d).
match. Trivial means of small value or
unserious matter. 21. The politician lost face in his 28. Devil’s advocate
(a) A dangerous person
15. There are a few miscellaneous items constituency (a)/when he broke the
(b) To present a counter argument
to discuss in this meeting. pre-election promises (b)/he made to
(c) Very argumentative person
(a) pure (b) mixed his people. (c)/No error. (d)
(d) Creating an unpleasant situation
(c) homogenous (d) discordant Ê (c) has error. As sentence is in simple past
tense, here we must use past perfect Ê (c) A devil’s advocate is a person who
Ê (c) homogenous is correct antonym of tense ‘he had made to the people’ should expresses an unpopular opinion in order
miscellaneous. Miscellaneous means of to provoke debate. So, out of the given
be used here.
mixed type whereas homogenous means options ‘very argumentative person’ best
of the same kind. 22. At the request of the Defence expresses its meaning.
16. Due to the postal strike, the outgoing Attorney, (a)/the jury were called 29. Don’t count your chickens before the
mail got delayed. (b)/and their individual verdicts eggs have hatched
(a) urgent (b) incoming
were recorded. (c)/No error. (d) (a) If you are not good at something,
(c) ordinary (d) speedy Ê (b) has error of correct use of verb with better to avoid that
jury which is taken as singular noun, verb (b) Don’t make plans for something that
Ê (b) incoming is the correct antonym of ‘was’ will be used here. might not happen
outgoing. In and out are perfect
antonyms. 23. Frank Lloyd Wright has been (c) Not to come up to expectations
acclaimed (a)/by colleagues (b)/as (d) Don’t put all your resources in one
17. He had a fine ear for music. possibility
(a) small (b) close the greater of all modern architects.
(c) coarse (d) smooth (c)/ No error. (d) Ê (b) This idiom means to not make plans
for something that might not happen.
NDA/NA Solved Paper 2018 (I) 51

30. Give the benefit of doubt 33. S1 : A mighty popular Revolt broke Directions (Q. Nos. 36-40) In this
(a) To be partial to someone out in Northern and Central India section you have few short passages.
(b) To be judgemental in 1857. After each passage, you will find some
(c) Regard someone as innocent until items based on the passage. First, read
S 6 : Millions of peasants, artisans
proven otherwise a passage and answer the items based
(d) Say something exactly right and soldiers fought heroically and on it. You are required to select your
wrote a glorious chapter. answers based on the contents of the
Ê (c) Benefit of doubt means to regard
someone as innocent until proven P : Sepoys, or the Indian soldiers of passage and opinion of the author only.
otherwise. the Company’s army Passage 1
Q : but soon engulfed wide regions The rule of the road means that in order
Directions (Q. Nos. 31-35) In this and involved the masses that the liberties of all may be preserved,
section each item consists of six sentences R : and nearly swept away the the liberties of everybody must be
of a passage. The first and sixth British rule curtailed. When the policeman, say, at a
sentences are given in the beginning as road-crossing steps into the middle of the
S : It began with a mutiny of the
S1 and S6. The middle four sentences in road and puts out his hand, he is the
each have been jumbled up and labelled The proper sequence should be symbol not of tyranny but of liberty. You
P, Q, R and S. You are required to find (a) R S P Q (b) P Q R S have submitted to a curtailment of private
(c) S R P Q (d) Q R P S liberty in order that you may enjoy a social
the proper sequence of the four sentences
and mark your response accordingly on Ê (a) RSPQ is the correct sequence. order which makes your liberty a reality.
the Answer Sheet. We 'have both liberties to preserve — our
34. S1: The Indian Civil Service Individual liberty and our social liberty.
31. S1 : The Subsidiary Alliance system gradually developed into one of the That is, we must have a judicious mixture
was extremely advantageous to the most efficient and powerful civil of both. I shall not permit any authority to
British. services in the world. say that my child must go to this school or
S 6 : They controlled the defence and S6 : though these qualities that, shall specialise in science or arts.
the foreign relations of the protected obviously served. British, and not These things are personal. But if I say that
ally. Indian interests. my child shall have no education at all,
P : and often participated in the then society will firmly tell me that my
P : They could now maintain a large child must have education whether I like it
army at the cost of Indian states. making of policy
or not.
Q : if many war occurred in the Q : independence, integrity and
hard work 36. According to the author, the ‘‘rule of
territories
R : They developed certain the road’’ implies
R : either of the British ally or of the (a) the rule regulating the traffic on the
traditions of
Britishers road
S : Its members exercised vast (b) the principle on which a road is
S : This enabled them to fight wars far power constructed to ensure safe traffic
away from their own territories (c) unrestricted freedom for all to lead a
The proper sequence should be
The proper sequence should be (a) P Q R S (b) Q R S P happy life
(a) P Q R S (c) R S Q P (d) S P R Q (d) restricted individual freedom to ensure
(b) P S Q R freedom for all
(c) Q R P S Ê (d) SPRQ is the correct sequence.
Ê (d) Rule of the road according to the
(d) S R P Q 35. S1 : The ruin of India handicrafts passage means restricted individual
Ê (b) PSQR is the correct sequence. was reflected in the ruin of the freedom to ensure freedom for all.
32. S1 : In reality, by signing a Subsidiary towns and cities which were 37. The author thinks that when a
Alliance, an Indian state virtually famous for their manufactures. policeman signals you to stop on a
signed away its independence. S6 : Centres were developed and road-crossing, he is
S6 : In fact, the Indian ruler lost all laid waste. (a) behaving in a whimsical manner
(b) interfering with your freedom to use the
vestiges of sovereignty in external P : Dacca, Surat, Murshidabad and road
matters. many other rising industrial (c) protecting the liberty of all to use the
P : of maintaining diplomatic relations Q : ravages of war and plunder, road
Q : It lost the right of self defence (d) mischievously creating hurdles in your
failed to way from some personal motive
R : with its neighbours R : survive British conquest Ê (c) The policeman is protecting the liberty
S : and of settling its disputes S : Cities which had withstood the of all to use the road.
The proper sequence should be The proper sequence should be 38. The author is of the view that we
(a) P Q R S (a) P Q R S should
(b) R S P Q (b) S Q R P (a) have absolute individual liberty without
(c) Q P S R (c) S R P Q any restrictions imposed by the
(d) Q S R P (d) Q R S P society
Ê (c) QPSR is the correct sequence. Ê (b) SQRP is the correct sequence.
(b) have everything, controlled by the
society without any kind of individual
liberty
52 NDA/NA Solved Paper 2018 (I)

(c) try to strike a sensible balance 2. The tiny bat like all bats keeps Ê (c) My Nocturnal Visitor is the most
between our individual liberty and our near the ceiling. suitable title of the passage from the
social liberty 3. It has lost direction because its given options.
(d) have more of social liberty than
individual liberty
radar has gone wrong. Directions (Q. Nos. 46-50) Each of
4. It wants to entertain the author the following sentences in this section
Ê (c) The author is of the view that we should with its skill in flying.
try to strike a sensible balance between has a blank space and four words or
our individual liberty and our social liberty. Which of the above statements may be group of words given after the sentence.
assumed to be true from the Select the word or group of words you
39. The author holds that information given in the passage? consider most appropriate for the blank
(a) educating or not educating his child is (a) 1 only (b) 1 and 3 space and indicate your response on the
a matter of personal liberty (c) 2 and 4 (d) 3 and 4 Answer Sheet accordingly.
(b) educating or not educating his child is
also a matter of social liberty Ê (b) From the given information statements 46. The tired traveller …………… on in the
1 and 3 are true. hope of finding some resting place.
(c) choosing the school for his child is a
matter of social liberty 42. The bat entered the room (a) strolled (b) scurried
(d) choosing the subject of study for his (a) because there was no alternative (c) paraded (d) plodded
child is a matter of social liberty (b) to eat the moths round the lamps Ê (d) The word ‘plodded’ means to walk
Ê (b) The author is of the opinion that (c) as it had gone mad with slow heavy steps. So, from the given
educating or not educating his child is (d) as it preferred to fly in through the open options it is most suitable to fill the given
also a matter of social liberty. door blank.

40. The most suitable title of the passage Ê (b) The bat entered the room to eat the 47. The car was damaged beyond repair
moths round the lamps.
would be in the ………… accident.
(a) The Policeman at a Road Crossing
43. After comparing the habits of the (a) outrageous (b) ghastly
(b) The Laws of the Road tiny bat with those of other bats, the (c) nasty (d) heinous
(c) Importance of Liberty author was
(d) Education of Children (a) sure that this bat had lost its direction
Ê (b) Ghastly means unpleasant and
shocking. So, it should be used to fill the
(b) not sure of its preferences
Ê The most suitable title of the passage (c) surprised to find that it was an expert
given blank.
would be option (c). ‘Importance of
flier 48. They gave a ………… dinner to
Liberty’. (d) unable to give the correct explanation celebrate the occasion, which
for its behaviour
Passage 2 impressed every guest.
My most interesting visitor comes at night, Ê (d) The author was confused and hence (a) austere (b) public
was unable to give the correct (c) sumptuous (d) summary
when the lights are still burning — a tiny
explanation for the behaviour of the bat.
bat who prefers to fly in through the open Ê (c) Sumptuous means of high quality and
door, and will use the window only if there 44. The author calls the tiny bat an often expensive. So, from the given
is no alternative. His object in entering the ‘‘interesting visitor’’. This means options it is the most suitable word to fill
house is to snap up the moths that cluster (a) the bat visits him at night the given blank.
around the lamps. All the bats I have seen (b) the bat is interested in the moths 49. Once the ………… manuscript is
fly fairly high, keeping near the ceiling; but (c) this bat has peculiar qualities
received by the publishers, it is typed
this particular bat flies in low, like a (d) this bat surprises him by zooming in
and out like a dive-bomber in double space.
dive-bomber, zooming in and out of chair (a) total (b) full
legs and under tables. Once, he passed Ê (c) The tiny bat is called an interesting (c) complete (d) filled
straight between my legs. Has his radar visitor by the author as it has peculiar
gone wrong, I wondered, or is he just plain qualities. Ê (c) complete is the correct answer.
crazy? 45. What, according to you, can be the 50. I am used to ………… in queues.
most suitable title for the passage? (a) stand (b) standing
41. Consider the following statements (c) stand up (d) standing still
(a) Someone visits me
1. The tiny bat flew in low like a dive
bomber.
(b) Night of Mysteries Ê (b) starding is the correct answer.
(c) My Nocturnal Visitor
(d) A Funny Visitor
NDA/NA Solved Paper 2018 (I) 53

Part B (General Studies)


51. Which one of the following 53. If an object moves with constant 55. What is the net force experienced by
statements is correct? velocity then which one of the a bar magnet placed in a uniform
(a) Any energy transfer that does not following statements is NOT correct? magnetic field?
involve temperature difference in (a) Its motion is along a straight line (a) Zero
some way is not heat. (b) Its speed changes with time (b) Depends upon length of the magnet
(c) Its acceleration is zero (c) Never zero
(b) Any energy transfer always requires (d) Depends upon temperature
a temperature difference (d) Its displacement increases linearly with
(c) On heating the length and volume of
time Ê (a) The net force experienced by a bar
magnet placed in a uniform magnetic
the object remain exactly the same Ê (d) Whenever an object moves with field is zero. A magnetic field is said to be
constant velocity, the motion is always in
(d) Whenever there is a temperature uniform over r a region if its magnetic field
form of straight line, which is parallel to the
difference, heat is the only way of induction ( B) has the same magnitude
x-axis. As velocity remains same, hence
energy transfer and direction at all point in the region.
acceleration of the object is zero. Also the
Thus, net force experienced by a bar
Ê (a) Practically, all the bodies around us speed of the object changes due to change
either release or absorb energy often in magnet in a uniform magnetic field is
in displacement so than in distance. Hence,
the form of heat. The form of energy zero.
option (d) is incorrect because it is not
which is exchanged among various necessary displacement increases linearly. 56. Which one of the following has
bodies or system on account of It decreases linearly for constant velocity. maximum inertia ?
temperature difference is defined as (a) An atom (b) A molecule
heat. On heating the matter, the energy
54. An object is moving, with uniform (c) A one-rupee coin
of atoms increases and hence the
acceleration a. Its initial velocity is u (d) A cricket ball
average distance between them and after time t its velocity is v. The
equation of its motion is v = u + at . Ê (d) Inertia means resistance or
increases. This results in thermal opposition offered by the body to change
expansion. Solids can expand in one The velocity (along y-axis) time (along its state. Mass of a body is the
dimension, two dimensions and three x-axis) graph shall be a straight line measurement of its inertia. A body with
dimensions while liquid and gases (a) passing through origin greater mass shows greater inertia, i.e., it
usually expand in volume. (b) with x-intercept u is more difficult to change its state of rest
(c) with y-intercept u or uniform motion as compared to that of
52. If T is the time period of an (d) with slope u a body having small mass. Hence, in the
oscillating pendulum, which one of given option a cricket ball has maximum
the following statements is NOT Ê (c) From the graph for the equation of its mass, so it has maximum inertia.
motion.
correct?
v = (u + at ) 57. Which one of the following is the
(a) The motion repeats after time T only
y=axis value of 1 kWh of energy converted
once
into joules?
(b) T is the least time after which motion B
E . × 10 6 J
(a) 18 (b) 3.6 × 10 6 J
repeats itself (c) 6.0 × 10 6 J (d) 7.2 × 10 6 J
(c) The motion repeats itself after nT,
where n is a positive integer Ê (b) 1 kilowatt hour is the unit of electrical
power.
(d) T remains the same only for small
1 kilowatt hour = 1000 watt × 3600 s
angular displacements
= 3.6 × 10 6 Joule
Velocity (v)

Ê (a) Time-period (T ) is the time taken by (Q 1 Joule = 1 watt × 1 sec)


the particle to complete one oscillation. A D
58. Which one of the following
Intercept

Time-period of the simple pendulum is y =u


given by statements about gravitational force
x=axis is NOT correct?
l O Time (t) C
T= 2π (a) It is experienced by all bodies in the
g
The object has an initial velocity u at point A universe
It is clear that the time period is and then its velocity changes at a uniform (b) It is .a dominant force between
independent of the mass of the rate from A to B in time t . In other words, celestial bodies
pendulum. This formula is valid only there is a uniform acceleration ‘a’ from A to (c) It is a negligible force for atoms
when radius of point mass of pendulum (d) It is same for all pairs of bodies in our
B and after time t its final velocity becomes ‘
is negligible compared to length of universe
v’ equal to B in the graph. Hence, the slope
pendulum. Here, motion of pendulum of a velocity-time graph is equal to Ê (d) Gravitational force is defined as a
repeats itself after a fixed interval i.e it acceleration. Thus, we can say that the non-contact force of attraction between
remains same only for small angular velocity time graph for the equation, any two bodies in universe (no matter
displacements (Q). Hence, it is v = u + at will be straight line having slope how far the bodies are). It is experienced
equal to acceleration a with y intercept u. by all bodies in the universe. The celestial
incorrect to say that this motion repeats
bodies experienced these forces, but it is
after time T only once. ∴ Option (c) is correct.
not same for all pairs of bodies in our
universe.
54 NDA/NA Solved Paper 2018 (I)

59. Whether an object will float or sink (a) 0 − 200 Hz Thus, Gastric juice gives the highest
(b) 20 − 20,000 Hz amount of H + ions.
in a liquid, depends on (c) 200 − 20,000 Hz only
(a) mass of the object only (d) 2,000 − 20,000 Hz only 66. Brine is an aqueous solution of
(b) mass of the object and density of (a) NaCl (b) NaOH
liquid only
Ê (b) 20-20,000 Hz, human ear can (c) NaHCO 3 (d) Na 2CO 3
respond to minute pressure variations in
(c) difference in the densities of the object air. The audible frequency range to which Ê (a) Brine is an aqueous solution of
and liquid ears can respond is 20-20,000 Hz. The sodium chloride (NaCl). Electrolysis of
(d) mass and shape of the object only sound frequency beyond this limit may brine produces Sodium Hydroxide
damage ear drums and cause hearing (NaOH), Chlorine (Cl 2) and Hydrogen
Ê (c) When an object is floating then (H 2). This forms the basis of the
Volume of sinking part of the object disabilities in humans.
= chlor-alkali industry.
Total volume of the body 63. The accidental touch of Nettle leaves
=
Density of the object creates a burning sensation, which is 67. Which one of the following is the
Density of the liquid due to inject of chemical formula of Washing Soda ?
Volume of sinking part of the body (a) Hydrochloric acid (b) Methanoic acid (a) NaHCO 3 (b) Na 2CO 3 ⋅ 10H 2O
(c) Citric acid (c) Na 2CO 3 ⋅ 5H 2O (d) NaOH
Mass of object
=
Density of liquid
(d) Sulphuric acid Ê (b) 2 NaCl ( aq ) + 2H 2OCl → 2NaOH( aq )
In case of sinking, if the buoyant force or Ê (b) The accidental touch of Nettle Leaves + Cl 2( g ) + H 2( g ) chemically washing
create a burning sensation. This is due to soda is sodium carbonate decahydrate
upthrust exerted by the liquid is less than
the weight of the object, the object will
the methanoic acid secreted by them. It is with formula Na 2CO 3 ⋅ 10H 2O. Its
a Herbaceous plant that grows in the wild. anhydrous form, i.e. Na 2CO 3 is called
sink in liquid. Therefore, it is clear that
The leaf of the dock plant, which often soda ash.
whether an object will float or sink in a
grows beside the nettle, is the traditional Na 2CO 3 + 10H 2O → Na 2CO 3 ⋅ 10H 2O
liquid depends on difference in the
remedy for it. Washing Soda
densities of the object and liquid. 68. Which one of the following is NOT
64. Which of the following properties is true for bleaching powder?
60. Which one of the, following devices
true for a tooth paste? (a) It is used as a reducing agent in
is non-ohmic?
(a) It is acidic (b) It is neutral chemical industries
(a) Conducting copper coil
(c) It is basic (b) It is used for bleaching wood pulp in
(b) Electric heating coil
(d) It is made up of Calcium phosphate, paper factories
(c) Semi conductor diode
the material of tooth enamel (c) It is used for disinfecting drinking water
(d) Rheostat
(d) It is used for bleaching linen in textile
Ê (c) According to Ohm’s law, the electric Ê (c) Tooth pastes are derived from industry
different variety of components namely
current flowing through a conductor is
directly proportional to the potential
abrasives, flouride and detergents. Ê (a) Bleaching powder (CaOCl 2 ) is
Abrasives includes particles of calcium produced by the action of chlorine on dry
difference applied across its ends,
hydrogen phosphates, aluminium slaked lime [Ca(OH)2 ]. The following are
provided the physical condition (such as
hydroxide, etc. Tooth pastes are basic in the uses of bleaching powder:
temperature) remain unchanged. Hence,
nature and prevents tooth decay by (i) It is used for bleaching purposes in
the device which follows Ohm’s law for all
neutralising the excess of acid. They can textile industries, paper industry and
voltage across it is called as an ohmic
neither be acidic nor normal. in laundry.
device.
(ii) It is also used as a disinfectant for
For example, a wire rheostat conducting 65. Which one of the following gives the
water to make it free of germs.
copper coil, electric heating coil, etc. The highest amount of hydrogen ions ( (iii) It is used as an oxidising agent in
device that does not follow Ohm’s law is H+ ) ? many chemical industries.
known as non-ohmic device. (a) Sodium hydroxide solution Thus among the given, option (a) is
For example, Semi conductor diode. (b) Milk of magnesia incorrect.
61. Which one of the following is the (c) Lemon juice
69. AIDS is caused by a virus whose
natural phenomenon based on which (d) Gastric juice
+
genetic material is
a simple periscope works? Ê (d) The amount of H ions produced (a) single stranded circular DNA
(a) Reflection of light depends upon the strength of an acid (b) double stranded DNA
(b) Refraction of light which further depends on the (c) single stranded RNA
(c) Dispersion of light corresponding pH values. Relation (d) double stranded RNA
(d) Total internal reflection of light between pH and concentration of H + ions
is given by Ê (c) AIDS is caused by Human
Ê (a) Periscope is an optical instrument 1
Immunodeficiency Virus (HIV). It belongs
which is based on the phenomenon of pH = − log [H + ] = log + to family retroviridae and genus lentivirus.
reflection of light. Light is reflected from [H ]
HIV genome consists of two identical
the first mirror into the second one at 45° As the pH increases, the acidic nature
copies of the SSRNA genome, reverse
angle which is then reflected to the eye of decreases and thus the amount of H +
transcriptase enzyme and other proteins
the viewer. ions. The order of pH values for the above
to code for viral proteins that are essential
given options are as follows:
62. Which one of the following during its replication. All these
Gastric Juice (1.3 − 3.0) < Lemon Juice components are: enclosed within the
frequency range is sensitive to
(2.2-2.4) < Milk of Magnesia (10.5) < core of viral particle.
human ears?
Sodium hydroxide (≈ 14)
NDA/NA Solved Paper 2018 (I) 55

70. Which one of the following 73. Which one of the following is the 76. Consider the following statements
organelle is NOT found in correct sequence of levels of about Roaring Forties
prokaryotic cells? hierarchy of classification of 1. They are strong Westerly winds
(a) Cell wall (b) Mitochondria organisms from higher to lower? found in the oceans of Southern
(c) Plasma membrane (d) Ribosome (a) Phylum - Class - Order - Family Hemisphere.
- Genus 2. The strong east to west air
Ê (b) Prokaryotic cells are simple and (b) Phylum - Class - Family - Order
primitive type of cells e.g. bacteria, - Genus currents are caused by the
methanogens, blue-green algae, etc. (c) Family - Order - Class - Species ' combination of air being displaced
These cell do not contain membrane - Genus from the Equator towards the
bound nucleus and other cellular (d) Class - Family - Order - Species South Pole and the earth’s
organelles like other than mitochondria, - Genus rotation and abundance of
prokaryotes also lack. Mitochondria is a landmasses to serve as wind
double membraned organelle, which is
Ê (a) A hierarchical system is used for
taxonomic classification of organisms. breaks.
responsible for the production of ATP Each level in this classification includes
molecules. It is only found in eukaryotic Which of the statements given above
organisms with similar characteristics.
cells. is/ are correct?
According to Linnean hierarchy,
(a) 1 only (b) 2 only
proposed by Carlous Linnaeus (Father of
71. Which one of the following parts of (c) Both 1 and 2 (d) Neither 1 nor 2
systematic Botany), the hierarchy
body does NOT take part in the involves seven obligate categories as Ê (c) The Roaring Forties are strong
process of breathing? shown below westerly winds found in the Southern
(a) Bronchi Hemisphere, generally between the
Kingdom → Phylum (in Animals)/Division
(b) Bowman's capsule latitudes of 40 and 50 degrees.The strong
(in plants) → Class → Order → Family →
west-to-east air currents are caused by
(c) Diaphragm Genus → Species
(d) Trachea the combination of air being displaced
74. Which one of the following from the Equator towards the South Pole
Ê (b) Bowman’s capsule is a part of human statements about meristematic and the Earth’s rotation, and there are few
excretory system. It is closed cup-shaped landmasses to serve as windbreaks.
and double-walled structure of
tissues in plants is correct ?
(a) These are dead tissues and form Hence, both the given statements are
Malpighian body of a kidney. correct.
wood
The human respiratory system includes (b) They provide flexibility to plant due to
nose, larynx, trachea, bronchi and 77. Consider the following statements
their thickened walls
bronchioles, alveoli and lungs. (c) These are present in the bark of a tree 1. The doldrums is a low pressure
Diaphragm is a dome-shaped sheet that only area around Equator where the
separate the thoracic cavity from (d) Growth occurs in plants due to division prevailing winds are calm.
abdominal cavity. It also serves major role of cells of these tissues 2. Chinook is a hot and dry wind
during breathing. that blows in winter and therefore
Ê (d) Growth occurs in plants due to the
Trachea or wind pipe conduct air to lungs division of meristematic cells. raises the temperature in a short
and bronchi are the branches of wind Meristematic tissues are a group of time.
pipe inside lungs. undifferentiated immature cells that are
either preparing to divide or are in
Which of the statements given above
72. Which one of the following
continuous state of division. These is/ are correct?
statements about classification of tissues contribute towards the growth of (a) 1 only (b) 2 only
plants is correct? the plants as they are present in the (c) Both 1 and 2 (d) Neither 1 nor 2
(a) Thallophytes have well differentiated actively growing regions of plants. Ê (c) Doldrums, (also called equatorial
body design Due to meristematic tissues, a plant body calms) are equatorial regions of light
(b) Funaria is a fungus retains the capacity of unlimited growth ocean currents and winds within the Inter
(c) All Pteridophytes are Phanerogams throughout their life. Tropical Convergence Zone (ITCZ), a belt
(d) Vascular system is not found among of converging winds and rising air
Bryophytes
75. Which one of the following Union encircling Earth near the Equator. It is a
Territories has the highest female low pressure area around the equator
Ê (d) Bryophytes are very small, literacy rate? where the prevailing winds are calm with
photosynthetic and first terrestrial plants. (a) Chandigarh no movement.
These are non-vascular plants too. (b) Lakshadweep Chinook are local winds that blow over the
Thyllophyta is a group of thalloid plants in (c) Andaman and Nicobar Islands Rocky mountains in USA. They are
which body is not differentiated into true (d) Puducherry essentially hot and dry wind that blow in
roots, stems and leaves. e.g. Algae.
Ê (b) Lakshadweep has a sex ratio of 946 the winter season and raises the
Funaria (Moss) is a genus of females for every 1000 males and a temperature in a short time thus removing
approximately 2010 species of class literacy rate of 92.28%. The female the frost from the crops.
Bryopsida of Bryophyta. literacy of lakshwadweep is 87.95
percent which the highest among all the 78. Which one of the following is the
Pteridophytes have hidden reproductive driest desert of the world ?
union territories in India. Daman and Diu
organs, so, these are called cryptogams. (a) Atacama (b) Gobi
comes next with 79.59% in female
While all gymnosperms and angiosperms (c) Sahara (d) Kalahari
literacy.
are phanerogams.
56 NDA/NA Solved Paper 2018 (I)

Ê (a) Atacama desert located in the Andes slightly disturbed its centre of gravity is 85. How is carbon black obtained?
mountain is the driest desert in the world, lowered. The line of action of its weight (a) By heating wood at high temperature
as well as the only true desert to receive lies outside the base of the rod. Then the in absence of air
less precipitation than the polar deserts. torque due to weight of the toppled it (b) By heating coal at high temperature in
Officially, it the driest place on Earth. It down. absence of air
receives almost no rainfall. This is due to When the centre of gravity of a body lies at (c) By burning hydrocarbons in a limited
the fact that the desert lies on the leeward the point of suspension, the body is said supply of air
side of the Andes that blocks any winds to be inventral equilibrium. (d) By heating coal at high temperature in
from reaching here. presence of air
82. Which of the following statements
79. Which of the following statements about a fluid at rest in a cup is/are Ê (c) Carbon black is obtained by burning
hydrocarbons in a limited supply of air.
about latent heat for a given correct?
substance is/are correct? CH 4 + O 2 → C + 2H 2O
1. Pressure is same-at all the points (limited Carbon
black
1. It is fixed at a given temperature. in the fluid. supply)

2. It depends upon the temperature 2. Pressure is exerted on the walls. It is used as a black pigment in black ink
and volume. and as filler in automobile tyres. Which
3. Pressure exists everywhere in the charcol and coke are obtained by heating
3. It is independent of temperature fluid. wood or coal respectively at high
and volume. temperature.
Select the correct answer 'using the
4. It depends on the temperature but codes given below
independent of volume. 86. Which one of the following
(a) 1 and 2 only (b) 2 and 3 only
(c) 1only (d) 1, 2 and 3
properties is NOT true for graphite?
Select the correct 'answer using the
(a) Hybridisation of each carbon atom is
codes given below Ê (d) When a fluid is at rest in a cup then sp3
(a) 2 (b) 1 and 3 (c) 4 only (d) 1 and 4 pressure is same at all the points in the
(b) Hybridisation of each carbon atom is
fluid, pressure is exerted on the walls and
Ê (b) The amount of heat per unit mass pressure exists everywhere in the fluid.
sp2
transferred during change of state of the (c) Electrons are delocalised over the
substance is called Latent heat of the The pressure at a point within a volume of
whole sheet of atoms
substance. It is always fixed at a given fluid means that the point to be
surrounded by a small container by the (d) Each layer is composed of hexagonal
temperature and does not changes with rings
temperature and volume of the area of the container.
substance. e.g. latent heat of ice is 83. Which one of the following devices Ê (a) A graphite crystal consists of layers of
3.34×105J. carbon atoms or sheets of carbon atoms.
is used to measure atmospheric Each carbon atom in a graphite layer is
80. Which one of the following pressure? joined to other three carbon atoms by
statements about the mass of a body (a) Ammeter (b) Barometer strong covalent bonds to form flat
is correct? (c) Potentiometer (d) Lactometer hexagonal rings. In this case, only 3 of the
(a) It changes from one place to another 4 valence electrons of C participate in
Ê (b) Barometer is used to measure the bonding. Thus, hybridisation is sp2
(b) It is same everywhere atmospheric pressure. The function of a
(c) It depends on its shape barometer is to know whether or not having covalent bonds with other 3
(d) It does not depend on its temperature atmospheric pressure is rising or falling. C-atoms in same plane.
Ê (b) Mass is a quantity of matter present Ammeter is a device which is used to The electrons are delocalised over the
inside the body which measures the measure electric current in amperes. whole sheet of atoms which makes it
inertia of a body. The mass of the body Potentiometer is used to measure the emf good conductor of electricity. Thus,
cannot be zero. Irrespective of the of a cell and. Loctometer is used to among the given options, option ‘a’ is
position of body in the universe, mass of measure the purity of milk. incorrect.
the body always remains constant
everywhere, which is independent on its 84. Which one of the following is the 87. Which one of the following is the
number of water molecules that purest form of Carbon?
temperature.
share with two formula unit CaSO 4 (a) Charcoal
81. A ball balanced on a vertical rod is an in Plaster of Paris? (b) Coke
example of (a) One (b) Two (c) Fullerene
(a) stable equilibrium (d) Carbon black
(c) Five (d) Ten
(b) unstable equilibrium Ê (d) Carbon black (lamp black) is the
(c) neutral equilibrium Ê (a) Plaster of Paris is chemically most purest form of carbon. It contains
(d) perfect equilibrium known as calcium sulphate about 98-99% of carbon. It is used for
hemihydrate  CaSO 4 ⋅ H 2O  . Thus,
1
Ê (b) When the centre of gravity of a body making printer ink, black paint, varnishes
 2 
lies below point of suspension or support, and carbon papers.
the body is said to be in stable with two formula unit of CaSO 4 , one Charcoal is the impure form of carbon
equilibrium. When the centre of gravity of molecule of water is shared. because of its porous structure and large
a body lies above the point of suspension It is prepared by heating gypsum at surface area.
or support, the body is said to be in 393 K. Coke contains 80-85% carbon. It is used
unstable equilibrium. A balanced ball on 2 (CaSO 4 ⋅ 2H 2O)  393
 K
→ as a fuel and as a reducing agent in
a vertical rod is an example of unstable 2(CaSO 4 )2 ⋅ H 2O
Gypsum metallurgy.
equilibrium because when the rod is
NDA/NA Solved Paper 2018 (I) 57

88. The Poisonous nature of Carbon 92. Which one of the following types of fulfill the need for training of highway
monoxide (CO) is due to its tissues will have contractile engineers at the entry level and during the
service. It is neither a registered society
(a) insolubility in water proteins?
nor is a collaboration between the central
(b) ability to form a complex with (a) Nervous tissue (b) Muscle tissue
haemoglobin and the State Governments.
(c) Bone tissue (d) Blood tissue
(c) ability to reduce some metal oxides 96. Which one of the following is NOT a
(d) property of having one sigma bond Ê (b) Contractile proteins are found in
muscles. These proteins are actin and tributary of Indus River?
Ê (b) Carbon monoxide (CO) is highly myosin and they are, the main (a) Beas (b) Ravi
poisonous to living beings because it has components of thin and thick filaments (c) Chenab (d) Tawi
an ability to form more stable respectively. Muscle contraction occurs
carboxyhaemoglobin. It is 300 times
Ê (d) The Tributaries of Indus include Beas,
when these filaments slide over one Chenab, Gar, Gilgit, Gomal, Hunza,
more stable than oxygen- another in a series of repetitive events. Jhelum, Kabul, Kunar, Kurram, Panjnad,
haemoglobin complex. In blood, when Ravi, Shyok, Soan, Suru, Satluj, Swat,
the concentration of 93. If by an unknown accident the acid
Zanskar, Zhob. Tawi is not one of the
carbonyhaemoglobin reaches about 3-4 secreting cells of the stomach wall of
tributaries of Indus river. It is one of the
per cent. The oxygen carrying capacity of an individual are damaged. left bank tributary of the Chenab River.
blood is greatly reduced. The oxygen Digestion of' which one of the
deficiency results into headache, weak following biomolecule will be 97. Which one among the following is
eyesight, nervousness, etc. affected to a greater extent? the largest tiger reserve of India in
89. Which one of the following elements (a) Protein only terms of area of the core/critical tiger
is needed in the human body to (b) Lipid habitat?
(c) Carbohydrate only (a) Manas
transfer electrical signals by nerve (d) Protein and Carbohydrate (b) Pakke
cells? (c) Nagarjunasagar Srisailam
(a) Lithium (b) Sodium Ê (a) Pepsinogen is an inactive form of (d) Periyar
pepsin which is secreted by oxyntic cells.
(c) Rubidium (d) Caesium
It is converted into active form called Ê (c) Nagarjunsagar-Srisailam Tiger
Ê (b) Sodium in the form of ions are found Pepsin in the presence of HCl. Pepsin Reserve is the largest tiger reserve in
in large proportions in biological fluids. It converts protein molecules into India in terms of area of core and critical
plays an important role in the proteases, peptones and ultimately into habitat. The reserve spreads over five
transmission of nerve signals. It also amino acids. Hence, proteins will not be districts, Kurnool district, Prakasam
regulates the flow of water across cell digested if acid secreting cells of district, Guntur district, Nalgonda district
membranes and in transport of sugars stomach wall of an individual get and Mahbubnagar district. The total area
and amino acids into the cells. damaged. of the tiger reserve is 3,568 km2 (1,378 sq
mi).
90. Who among the following first 94. In which one of the following places,
discovered cell ? Headquarters of a Railway Zone is 98. Which one of the, following is NOT a
(a) Robert Brown located? coastal depositional feature ?
(b) Robert Hooke (a) Kanpur (b) Lucknow (a) Tombolo (b) Sand bar
(c) Leeuwenhoek (c) Hajipur (d) New Jalpaiguri (c) Stack (d) Spit
(d) Rudolf Virchow
Ê (c) The East Central Railway (ECR) is one Ê (c) A stack or sea stack is a geological
Ê (b) Robert Hooke was the first who of the 17 railway zones in India. It is landform consisting of a steep and often
studied and discovered cell from a thin headquartered at Hajipur (Bihar) and vertical column or columns of rock in the
section of dead cork. He observed small comprises Sonpur, Samastipur, sea near a coast, formed by wave
honeycomb-like structures and named Danapur, Mughalsarai and Dhanbad erosion. Stacks are formed over time by
them as cells. divisions. wind and water, processes of coastal
Robert Brown reported the discovery of geomorphology. All the other options are
nucleus. Leeuwenhock was the first 95. Which of the following statements forms of coastal depositional landform.
person to observe living cells under about Indian Academy of Highway
Engineers is/ are correct?
99. Which of the following is/are coastal
microscope. Rudolf Virchow proposed erosional feature( s) ?
that all cells arise from pre-existing cells. 1. It is a registered society.
1. Notch 2. Sea Arch
91. Which one of the following group of 2. It is a collaborative body of both
3. Cliff 4. Hook
organisms forms a food chain? Central Government and State
(a) Grass, human and fish Governments. Select the correct answer using the
(b) Grass, goat and human codes given below
Select the correct answer using the
(c) Tree, tree cutter and tiger (a) 1, 2 and 3 (b) 2, 3 and 4
codes given below (c) 2 and 3 only (d) 1 only
(d) Goat, cow and human
(a) 1 only (b) 2 only
Ê (b) The series of organisms in which one (c) Both 1 and 2 (d) Neither 1 nor 2 Ê (a) A notch or small hollow is an
organism is being eaten by other is called indenture or a concave cut into a surface
food chain. The organisms given in the
Ê (d) Indian Academy of Highway or edge (as in a coastline). It is not an
Engineers is the apex training institute erosional feature Sea Arch and Cliff are
option (b) form the following food chain: set-up to address the training needs of examples of Coastline erosional features.
Grass (Producers) → Goat (Primary Highway and Bridge Engineers in the A sea arch is a natural opening eroded
Consumer) → Human (Secondary country. It was set-up as an Institute in the out of a cliff face by marine processes.
Consumer) year 1983 with the primary objective to
58 NDA/NA Solved Paper 2018 (I)

Some arches appear to have developed 103. Which one of the following 107. Which one of the following metals is
from surge channels, which are created statements about the refractive alloyed with sodium to transfer heat
by wave refraction causing the focussing
index of a material medium with in a nuclear reactor?
of wave fronts on the side of a headland.
A cliff is a high area of rock with a very
respect to air is correct? (a) Potassium (b) Calcium
(a) It can be either positive or negative (c) Magnesium (d) Strontium
steep side, often on a coast.
(b) It can have zero value
Ê (a) Sodium-potassium alloy (NaK) is an
100. Chemical weathering of rocks is (c) It is unity for all materials
alloy of two alkali metals sodium and
much greater in a place with (d) It is always greater than one
potassium. It is used as a heat-transfer.
(a) cold and dry climate Ê (d) Refractive index of a material is given Coolant in fast-breeder nuclear reactors
(b) hot and humid climate by the ratio of speed of light in vacuum to and experimentally in gas-turbine power
(c) hot and dry climate the speed of light in any medium. plants.
(d) cold and humid climate Refractive index is always greater than
Ê (b) Chemical Weathering results from one because by the definition of refractive 108. Which one of the following metals is
chemical reactions between minerals in index, n of a medium (e.g. glass) is the used in the filaments of
rocks and external agents like air or water. ratio of the speed of light in a vacuum C, photo-electric cells that convert light
Oxygen oxidizes minerals to alteration to the speed in the medium v and speed energy into electric energy?
products whereas water can convert of light is maximum in vaccum, which (a) Tungsten (b) Copper
minerals to clays or dissolve minerals gives the value of n greater than one. (c) Rubidium (d) Aluminium
completely. It occurs majorly in areas with
104. Which one of the following Ê (a) Photo electric cells is a device which
hot and humid climate. convert light-energy directly into electric
statements about magnetic field
101. Which of the following statements lines is NOT correct? energy. Main type of these cells are photo
about specific heat of a body is/are voltaic cell, photo emissive cell, photo
(a) They can emanate from a point
conductive cell. The filament of photo
correct? (b) They do not cross each other
(c) Field lines between. two poles cannot electric cells are made up of tungsten.
1. It depends upon mass and shape Electrons are obtained from the filament
be precisely straight lines at the ends
of the body (d) There are no field lines within a bar by thermionic emission and are
2. It is independent of mass and magnet accelerated to the anode by a potential
shape of the body difference (~ 100 kV).
Ê (c) The magnetic field lines is defined as
3. It depends only upon the the path along which the compass 109. Which of the following statements
temperature of the body needless are aligned. They cannot about Ring of Fire is/are correct?
Select the correct answer using the intersect i.e., do not cross each other. 1. It is a zone of convergence plate
Tangent drawn at any point gives the
codes given below boundaries
direction of magnetic field. Outside a
(a) 1 only (b) 2 and 3 2. It is an active seismic and volcanic
(c) 1 and 3 (d) 2 only magnet, they are directed from north to
south pole and inside a magnet they are zone
Ê (a) Every substance has a unique value directed from south to north. These lines 3. It is associated with deep trench
for the amount of heat absorbed or are always normal to the surface, which
rejected to change the temperature of
Select the correct answer using the
are precisely straight at the ends. Hence,
unit mass of it by one unit. This quantity is codes given below
statement (c) is not correct.
referred as the specific heat capacity of a (a) 1 and 2 only (b) 2 and 3 only
body. It depends on the nature as well as 105. Two convex lenses with power 2 (c) 1 only (d) 1, 2 and 3
mass of the substance and its change in dioptre are kept in contact with each Ê (d) The Ring of Fire is a ring of volcanoes
temperature. Hence, option (a) is correct. other. The focal length of the around the Pacific Ocean that result from
combined lens system is subduction of oceanic plates beneath
102. Which one of the, following is an (a) 0.10 m (b) 2m lighter continental plates. This is also
example of the force of gravity of the (c) 4m (d) 0·25m where the Earth’s deep ocean trenches
earth acting on a vibrating pendulum are and where the Earth’s deep
Ê (d) Here, P1 = P2 = 2 ∴ dioptre ( D ) earthquakes are. The trenches form
bob?
Using the formula total power, P = P1 + P2 because the downgoing plate is bent
(a) Applied force 1 1
(b) Frictional force = 2 + 2 = 4D, f = = = 0.25 m downward as it subducts. It is the zone of
P 4m
(c) Restoring force convergence plate boundaries.
(d) VIrtual force 106. Which one of the following alkali 110. Which one of the following
metals has lowest melting point?
Ê (c) An object that is vibrating is acted Himalayan vegetation species is
upon by a restoring force. The restoring (a) Sodium (b) Potassium
(c) Rubidium (d) Caesium
found between the altitudes of 1800
force causes the vibrating object to slow to 2600 metres?
down as it moves away from the Ê (d) The melting point of alkali metals (a) Saal (b) Chir
equilibrium position and to speed up as it decreases on moving down the group (c) Spruce (d) Deodar
approaches the equilibrium position. due to decrease in the extent of metallic
There is the force of gravity that acts bonding. Thus, the sequence of melting Ê (d) Deodar tree is also known as
point among the given options is Himalayan Cedar. These evergreen
downward upon the bob. And the tension coniferous trees are native of Pakistan
force results from string pulling upon the Na > K > Rb > Cs
and Himalayas as they thrive at altitudes
bob of the pendulum. Therefore, Caesium (Cs) have the least of 1,500-3,200 meters above sea level.
melting point.
NDA/NA Solved Paper 2018 (I) 59

Deodar grows to a height of 40–60 Ê (d) Blizzards are the local winds of Ê (a) The Aadi Mahotsav, a celebration of
meters (131–164 feet) with a trunk Siberia which blows out from the Siberian the spirit of Tribal Culture, Craft, Cuisine
diameter of 3 meters (10 feet). The timber region to parts of south Asia. Blizzards and Commerce was held at Dilli Haat,
of Deodar tree is used as construction are dangerous winter storms that are a New Delhi, on the of 30th November,
material for its durability. combination of blowing snow and wind 2017. The theme of this festival was ‘A
resulting in very low visibilities. Celebration of the Spirit of Tribal Culture,
111. Which one of the following rivers is
Cuisine and Commerce’. It was
NOT a tributary of river Cauvery? 115. Which one of the following centres organised by the Tribal secretary and not
(a) Hemavati (b) Arkavati is NOT known for iron and steel ministry of tourism.
(c) Indravati (d) Amravati industry ?
Ê (c) The tributaries of Kaveri include (a) Bhadravati (b) Salem
118. Light year is a unit for measurement
Amaravati, Arkavathy, Bhavani, Chinnar, (c) Vishakhapatnarn(d) Renukoot of
Hemavati, Honnuhole, Kabini, Kannika, (a) age of universe
Kollidam, Lakshmana Tirtha, Lokapavani,
Ê (d) Mysore Iron and Steel Company (b) very large distance
(MISCO) is located at Bhadravati on the (c) very small time intervals
Noyyal, Pambar, Shimsha, Sujyothi. banks of river Bhadravati in Shimoga (d) very high temperature
Indravati River is a stream of the river district of Karnataka. Salem steel plant is
Godavari and is its tributary. located in the Salem district of Tamil
Ê (b) Light year is a unit of measure of
distance travelled by light in one year. It is
112. Which of the following conditions Nadu. The plant has the advantage of rich used for measuring very large distance.
is/are essential for tea cultivation? iron ore and limestone, which is readily 1 light year = 9.46 × 1015 m.
available in the adjoining areas. It also
1. Tropical and sub-tropical climate enjoys the facilities of cheap power, 119. Which of the following statements
2. Heavy rainfall ranging from 150 charcoal and vast market. about electromagnetic waves, sound
cm to 250 cm Hindalco’s plant is located in waves and water waves is/are’’
3. Soil should contain good amount Renukoot which is involved in the correct?
of lime. smelting of Aluminium.
1. They exhibit reflection
Select the correct answer using the 116. Which of the following are essential 2. They carry energy
codes given below prerequisites for establishment of a 3. They exert pressure
(a) 1, 2 and 3 (b) 1 and 2 only thermal power station? 4. They can travel in vacuum
(c) 2 and 3 only (d) 1 only 1. Availability of fossil fuels Select the correct answer using the
Ê (b) Tea is an evergreen plant. It grows 2. Proximity to a river, lake or sea codes given below
both in tropical and subtropical regions. (a) 1, 2 and 3 (b) 2 and 4
3. Good transport network
The tea plant grows well in hot and humid (c) 1 and 3 only (d) 1 only
climate. Actually, there is an intimate 4. Proximity to an urban centre
relationship between climate, yield per
Ê (a) Electromagnetic waves, sound waves
Select the correct answer using the and water waves exhibit reflection, carry
hectare and the quality of tea. codes given below energy (EM waves), exert pressure (water
(i) The ideal temperature for growing tea (a) 1, 2 and 3 (b) 2 and 4 waves), but sound waves cannot travel in
is between 20 °C and 30 °C. (c) 2 and 3 only (d) 1 and 3 only vaccum.
(ii) It needs heavy rainfall between 150
and 250 cm. It should be Ê (a) The essential prerequisites of a 120. Thermal capacity of a body depends
thermal power plant include that it must
well-distributed throughout the year on the
and should be in the form of frequent be in proximity to the fossil fuel and
proximity to a water source like river, lake (a) mass of the body only
showers. (b) mass and shape of the body only
or sea. It should also have a good
However it does not require soils with lime (c) density. of the body
transport network for the transportation of
for its cultivation. (d) mass, shape and temperature of the
raw materials and the finished products.
body
113. Bharatmala Project is related to However proximity to an urban centre is
(a) Improving road connectivity not one of the essential conditions for Ê (d) Thermal capacity of a body can be
(b) Interlinking ports and railways establishment of a power plant. defined as the amount of heat required to
(c) Interlinking of rivers change the temperature of an object by a
(d) Interlinking major cities with gas 117. Which of the following statements certain degree. It depends upon the mass,
pipelines about ‘Aadi Mahotsav’ held recently shape and temperature of the body.
in New Delhi is/are correct? 121. Who among the following first used
Ê (a) Bharatmala project is the second
largest highways construction project in 1. The theme of the festival was ‘A the term ‘Industrial Revolution’ in
the country. It aims to build 50,000 km of Celebration of the Spirit of Tribal English to describe, the changes that
highway roads targeted across the Culture, Cuisine and Commerce’ occurred in British industrial
country. Bharatmala will look to improve 2. The festival was organised by the development between 1760 and
connectivity particularly on economic Ministry of Tourism, Government 1820?
corridors, border areas and far flung
of India (a) Karl Marx (b) Georges Michelet
areas with an aim of quicker movement of
Select the correct answer using the (c) Arnold Toynbee (d) Friedrich Engels
cargo and boosting exports.
codes given below Ê (c) The term Industrial Revolution was
114. Which one of the following is a local (a) 1 only first popularised by the English economic
wind that blows out from Siberia ? (b) 2 only historian Arnold Toynbee to describe
(a) Bora (b) Purga (c) Both 1 and 2 Britain’s economic development from
(c) Mistral (d) Blizzard (d) Neither 1 nor 2 1760 to 1820.
60 NDA/NA Solved Paper 2018 (I)

122. Which one of the following England. Badshahnama is a genre of that the landlords would have perpetual
statements about the Olympe de works written as the official visual history and hereditary rights over the land, so
of Mughal emperor, Shah Jahan’s reign. long as they pay the fixed revenue to the
Gouges (1748- 1793) is correct?
126. What is the name of the award given British Government.
(a) She declared that although citizens
should have equal rights, they are not to meritorious men in the Mughal 130. Name the rebel who fought against
entitled to the same honours by the Court in the form of a robe of honour the British in the battle of Chinhat in
State
that was once worn by the Emperor? the course of the 1857 Revolt?
(b) She was a supporter of the Jacobin
government (a) Sarapa (b) Patka (a) Ahmadullah Shah(b)Shah Mal
(c) Padma murassa (d) Khilat (c) Mangal Pandey (d) Kunwar Singh
(c) She was jailed for treason by the
National Assembly Ê (c) Padma murassa i.e., a lotus blossom Ê (*) The Battle of Chinhat between the
(d) She declared that the nation is the set with jewel was considered a special British forces and Indian rebels, at
union of woman and man gift. It was gifted by the emperor only in Ismailganj, near Chinhat. The British were
exceptional circumstances. The emperor
Ê (a) Only the first statement (a) is correct. led by The Chief Commissioner of Oude,
She declared that although citizens would receive several gifts from the Sir Henry Lawrence. The insurgent force,
should have equal rights they are not courtiers. The courtiers would not go to which consisted of mutineers from the
entitled to the same honours by the state. emperor empty-handed. East India Company’s army and retainers
127. Who among the following social of local landowners was led by Barkat
123. Who among the following built a
reformer started a society for the Ahmad, a mutineer officer of the
model steam engine in 1698 called
encouragement of widow remarriage Company’s army.
‘‘Miner’s Friend’’ to drain mines?
(a) Thomas Savery in 1866 in Maharashtra ? 131. Who among the following are the
(b) Thomas Newcomen (a) Bal Gangadhar Tilak two civil servants who assisted the
(c) James Watt (b) Jyotirao Phule Constituent Assembly in framing the
(d) Richard Arkwright (c) Vishnushastri Pandit
(d) Pandita Ramabai Constitution of India ?
Ê (a) Thomas Savery was an English (a) B. N. Rau and K. M. Munshi
inventor and engineer, born in England. Ê (b) Jyotirao Govindrao Phule was an (b) S. N. Mukherjee and Alladi
He invented the first commercially used Indian social activist, a thinker and
anti-caste social reformer from Krishnaswamy Aiyar
steam powered device in 1698, a steam (c) B. N. Rau and S. N. Mukherjee
Maharashtra. He worked for widow
pump which is often referred to as an (d) K. M. Munshi and Alladi Krishnaswamy
remarriage and started a home for
‘engine’. Savery’s ‘engine’ was a Aiyar
pregnant Brahmin widows to give birth in
revolutionary method of pumping water,
a safe and secure place in 1863. Ê (c) Sir Benegal Narsing Rao was an
which solved the problem of mine
drainage and made widespread public 128. Name the first major voluntary Indian civil servant, jurist, diplomat and
water supply practical. statesman known for his key role in
association representing primarily
drafting the Constitution of India. S.N
124. Which one of the following Indian landlord interests that was set Mukherjee was another civil servant who
statements about Renaissance up in Calcutta in 1851 ? assisted in the framing of the Constitution
Humanist culture is NOT true? (a) British Indian Association of India.
(b) Landholder’s Society
(a) It slackened the control of religion over (c) Madras Native Association 132. Which member of ‘the Constituent
human life (d) Bombay Association
(b) It believed that human nature was
Assembly proposed the resolution
many-sided Ê (a) British Indian Association was created that the National Flag of India be a
(c) It was concerned with good manners after amalgamating the “Landholders ‘‘horizontal tricolour of saffron,
(d) It criticised material wealth, power and Society” and “British India Society” on 31 white and dark green in equal
glory October 1851. This was the first political proportion’’, with a wheel in navy,
organisation that brought the Indian blue at the centre?
Ê (c) The term Renaissance Humanist was
derived from the program of studies landlords together. The President of the (a) Jawaharlal Nehru
called the ‘Studia Humanitatis’. However, first committee of this organisation was (b) B. R. Ambedkar
the idea of calling this ‘Humanism’ only Raja Radhakanta Deb. (c) Rajendra.Prasad
really arose in the nineteenth century. It (d) Sardar Vallabhbhai Patel
majorly advocated that human outlook
129. Who among the following introduced
should be based on a modern plan and
the Permanent Settlement of Bengal Ê (c) In 1947, when India gained freedom
not blindly follow a religious plan. It never in 1793 ? from the British, a committee headed by
spoke about good manners. (a) Lord Cornwallis Dr. Rajendra Prasad decided to adopt the
(b) Lord Ripon flag of the Congress as the national flag of
125. Who gifted the Badshah Nama to (c) Robert Clive India with a few modifications. With this in
King George in 1799? mind, the flag of 1931 was adopted as the
(d) John Adam
(a) Abul Fazl national flag of India, but the charkha in
(b) Abdul Hamid Lahori Ê (a) The Permanent Settlement was the middle was replaced with the Ashoka
(c) Nawab of Awadh introduced by Lord Cornwallis in 1793. It
Chakra. Thus, the Indian national flag was
(d) William Jones was an agreement between the British
born. The saffron colour on top represents
Ê (c) In 1799, the Nawab (provincial East India Company and the Landlords of
sacrifice, white represents peace and
governor) of Awadh in northern India sent Bengal to settle the Land Revenue to be
purity and green signifies the law
the Padshahnama, to King George III of raised. In this agreement it was agreed
of dharma (righteousness).
NDA/NA Solved Paper 2018 (I) 61

133. Which of the following is/are NOT 136. Who among the following was the and the conduct of, all elections shall be
historical biography/biographies? author of Humayun Nama? vested in the Election Commission.
Therefore it is the sole authority in India
1. Dipavamsa (a) Roshanara Begum
(b) Ruquaiya Sultan Begum that takes charge of elections in India.
2. Harshacharita (c) Gulbadan Begum
3. Vikramankadevacharita 140. Which of the following provision(s)
(d) Gauhara Begum
of the Constitution of India became
4. Prithvirajavijaya
Ê (c) Gulbadan begum is best known as effective from .26th November,
Select the correct answer from the the author of Humayun-Nama, the 1949?
codes given below account of the life of her half-brother,
Emperor Humayun, which she wrote on 1. Elections
(a) 1 only (b) 2 and 3 only
(c) 2, 3 and 4 only (d) 1, 2, 3 and 4 the request of her nephew, Emperor 2. Citizenship
Akbar. 3. Emergency provisions
Ê (a) Dipavamsa is one of the most
important works in Pali Literature. It 137. Which one of the following about the 4. Appointment of the Judges
details the tooth relic and Bodhi Tree’s Parliament of India is NOT correct? Select the correct answer using the
arrival in Sri Lanka. It also deals with the (a) The Parliament consists of the codes given below
arrival of Buddha’s teaching and President, the Lok Sabha and the (a) 1 only (b) 1 and 2 only
preachers in Sri Lanka. It mentions that Rajya Sabha (c) 1, 2 and 3 (d) 2 and 4
Buddha visited Kelaniya and Dighavapi in (b) There are no nominated members in
Sri Lanka. All the other options are the Lok Sabha Ê (b) The provisions of elections and
citizenship became effective from 26th
biographies about kings. (c) The Rajya Sabha cannot be dissolved
November 1949. Rest of the provisions
(d) Some members of the Rajya Sabha
134. Which of the following pairs are are nominated by the President
including the Emergency provisions and
correctly matched ? the appointment of the judges were
Ê (c) Anglo-Indians are the only community added later in the Constitution.
Traveller Country from that has its own representatives
1. Marco Polo Italy nominated to the Lok Sabha (Lower 141. Which of the following statements
2. Ibn Battuta Morocco House) in India’s Parliament. It is done regarding construction of Rohtang
3. Nikitin Russia
only if the President of India feels that the tunnel is NOT correct?
Anglo Indian community has not been (a) It is located at an altitude of 5,000 feet
4. Seydi Ali Reis Turkey adequately represented in the Lok (b) It will provide all-year connectivity to
Select the correct answer using the Sabha. Lahaul and Spiti Valley
(c) The tunnel is being built by the Border
codes given below 138. Which one of the following Roads Organisation
(a) 1, 2 and 3 only (b) 2 and 3 only statements with regard to the (d) It will reduce the length of the
(c) 1, 2, 3 and 4 (d) 1 and 4 only Comptroller and Auditor General Leh-Manali highway by approximately
Ê (c) Marco Polo was an Italian merchant, (CAG) of India is NOT correct? 50 km
explorer and writer, born in the Republic (a) He is appointed by the President of
of Venice. Ibn Battuta was a Moroccan
Ê (a) The Rohtang tunnel is located at
India 3,100 metres (10,171 ft). It is being built
scholar who widely travelled the medieval (b) He can be removed from office in the by the Border Roads organisation to
world. Afanasy Nikitin was a Russian same way as the judge of the Supreme provide all year connectivity to Lahaul
merchant of Tver and one of the first Court of India and Spiti valley. It also aims at reducing
Europeans to travel and to document his (c) The CAG is eligible for further office the distance of Manali Leh Highway by
visit to India. Seydi Ali Reis was an under the Government of India after he
Approx 50 km.
Ottoman admiral and navigator from has ceased to hold his office
Turkey. He commanded the left wing of (d) The salary of the CAG is charged upon 142. Who among the following recently
the Ottoman fleet at the naval Battle of the Consolidated Fund of India became the first woman pilot in
Preveza in 1538. So all the given options Ê (c) The CAG is not eligible for further Indian Navy?
are correct. office either under the Government of (a) Astha Segal (b) Roopa A
India or under the Government of any (c) Sakthi Maya S (d) Shubhangi Swaroop
135. Which of the following clans are
State after he has ceased to hold his
included in the Agnikula Rajputs ? office. These provisions are in order to
Ê (d) Shubhangi Swaroop has made
history by becoming the first woman to be
1. Pratiharas 2. Chaulukyas ensure the independence of CAG. All the inducted into the Indian Navy as a pilot.
3. Paramaras 4. Chahamanas other provisions mentioned are correct.
143. Who among the following Indians
Select the correct answer from the 139. The Superintendence, direction and did NOT hold the title of Miss
codes given below control of elections in India is vested World ?
(a) 1 and 3 only (b) 1, 3 and 4 only in
(c) 1, 2, 3 and 4 (d) 2 and 4 only (a) Reita Faria
(a) The Supreme Court of India (b) Sushmita Sen
Ê (c) The four Rajput clans from Agnikunda (b) The Parliament of India (c) Diana Hayden
were Chauhans, Chalukyas, Parmaras (c) The Election Commission of India (d) Yukta Mookhey
and Pratiharas. This theory comes from (d) The Chief Election Commissioner
the Prithvirajraso of Chandarbardai. The Ê (b) Sushmita Sen is the winner of the
Ê (c) According to Article 324 of the Miss Universe pageant of 1994. Sen was
Rajputs descended from Agni and this Constitution of India the the first Indian woman to win the crown.
theory is based upon the Agnikula superintendence, direction and control of All the other options are former Miss
Legend of Bhavishyapurana . the preparation of the electoral rolls for, World.
62 NDA/NA Solved Paper 2018 (I)

144. Which one of the following it is likely to result in access to modern direct tax law in consonance with current
countries has failed to qualify for the technology or for other reasons to be economic needs. Central Board of Direct
recorded. Taxes (CBDT) member Arbind Modi will
first time in 60 years for the FIFA
be the convener of task force while Chief
World Cup to be held in Russia in the 147. Which one of the following Economic Adviser Dr. Arvind
year 2018 ? countries did NOT participate in the Subramanian will be a permanent Special
(a) Mexico (b) Iran 21st edition of Exercise Malabar? Invitee. The task force will draft an
(c) Saudi Arabia (d) Italy (a) United States of America appropriate direct tax legislation keeping
(b) Japan
Ê (d) Italy has failed to qualify for first World (c) India
in view the direct tax system in various
Cup in 60 years after play-off defeat to countries, international best practices
Sweden. (d) Australia and economic needs of the country. It will
145. The Defence Technology and Trade Ê (d) The 2017 Malabar exercise was submit its report within six months.
the 21st edition of the exercise and
Initiative (DTTI) is a forum for conducted from 10 to 17 July 2017. This
150. The 5th Global Conference on Cyber
dialogue on defence partnership edition involved navies from India, USA Space (GCCS) was held in New Delhi
between India and and Japan. The exercise included a in November, 2017. Which of the
(a) Russia harbor phase at Chennai. Australia was following statements about GCCS
(b) United States of America not a participant in this exercise. is/are correct?
(c) Israel
(d) France 148. Justice Dalveer Bhandari of India 1. The 4th version of GCCS was held
was recently re-elected to the in London.
Ê (b) Defence Technology and Trade
Initiative (DTTI) originally called the International Court of Justice after 2. The main theme of GCCS 2017 is
‘Carter Initiative’ after the name of U.S. Christopher Greenwood pulled out ‘Cyber4All : A Secure and
Deputy Secretary of Defence Ashton before 12th round of voting. Inclusive Cyberspace for
Carter. Its main objective is to strengthen Christopher Greenwood was a Sustainable Development’.
defence cooperation by facilitating the nominee of
Indian companies to collaborate with US 3. ‘Bindu’ is the logo of GCCS 2017.
(a) Canada (b) Russia
partners in defense co-production, where (c) Britain (d) USA Select the correct answer using the
the US provides technology and codes given below
guidance for building modern weapon Ê (c) Sir Christopher John Greenwood is a
British judge at the International Court of (a) 1 only
systems. (b) 2 only
Justice, to which he was elected on 6th
146. As per the policy applicable in 2017, November 2008. He recently pulled out of (c) 2 and 3 only
how much Foreign Direct before the 12th round of voting. (d) 1, 2 and 3
Investment (FDI) is permitted in the 149. In order to review the Income Tax Ê (b) The Global Conference on
defence sector in India? Act, 1961 and to draft a new Direct CyberSpace (GCCS) was started in 2011
(a) 49 per cent through the automatic Tax Law in consonance with in London, second GCCS was held in
route 2012 in Budapest with focus on
economic needs of the country, the
(b) 26 per cent through the government relationship between internet rights and
route Government of India in November
internet security. The third edition of
(c) 26 per cent through the automatic 2017 has constituted a Task Force.
GCCS was held in 2013 in Seoul . The
route and beyond that up to 49 per Who among the following is made
fourth version GCCS 2015 was held on
cent through the government route the convenor of it ? April 16-17, 2015 in The Hague,
(d) 75 per cent through the automatic (a) Shri Arvind Subramanian Netherlands, which saw participation
route (b) Shri Arbind Modi from 97 countries. Hence the 1st
Ê (a) In the defence sector foreign (c) Shri Amitabh Kant
statement is incorrect. Bindu is not the
investment up to 49% is permitted under (d) Dr. Bibek Debroy
logo for this conference hence the third
the automatic route, foreign investment
Ê (b) The Union Government has statement is also incorrect.
beyond 49% and upto 100% is permitted constituted a task force to review the
through Government approval, wherever Income Tax Act 1961 and draft a new
NDA /NA
National Defence Academy/Naval Academy

SOLVED PAPER 2017 (II)


PAPER I : Mathematics
1. If x + log10 (1 + 2 x ) = x log10 5 + log10 6 6. If E is the universal set and Select the correct answer using the
then x is equal to A = B È C , then the set code given below.
(a) 2, -3 (b) 2 only (c) 1 (d) 3
E - ( E - ( E - ( E - ( E - A )))) is (a) 1 and 2 only
same as the set (b) 2 and 3 only
2. The remainder and the quotient of the (a) B¢ È C ¢ (b) B È C
binary division (101110) 2 ¸ (110) 2 are (c) 1 and 3 only
(c) B¢ Ç C ¢ (d) B Ç C (d) 1, 2 and 3
respectively
(a) (111)2 and (100)2 (b) (100)2 and (111)2 7. If A = {x : x is a multiple of 2}, 10. How many different permutation
(c) (101)2 and (101)2 (d) (100)2 and (100)2 B = {x : x is a multiple of 5} and can be made out of the letters of the
C = {x : x is a multiple of 10}, word ‘PERMUTATION’?
3. The matrix A has x rows and x + 5 then A Ç ( B Ç C ) is equal to (a) 19958400 (b) 19954800
columns. The matrix B has y rows and
(a) A (b) B (c) 19952400 (d) 39916800
11 - y columns. Both AB and BA exist.
What are the values of x and y (c) C
é 4i - 6 10i ù 1
respectively? (d) {x : x is a multiple of 100} 11. If A = ê ú and k = ,
ë 14i 6 + 4i û 2i
(a) 8 and 3 (b) 3 and 4 8. If a and b are the roots of the where i = -1, then kA is equal to
(c) 3 and 8 (d) 8 and 8 equation 1 + x + x 2 = 0, then é2 + 3i 5 ù é2 - 3i 5 ù
n (n - 1)Q (a) ê (b) ê
4. If S n = nP + , where S n the matrix product ë 7 2 - 3i úû ë 7 2 + 3i úû
2 é 1 b ù éa b ù
denotes the sum of the first n terms of an é2 - 3i 7ù é2 + 3i 5 ù
êa a ú ê 1 b ú (c) ê (d) ê
AP, then the common difference is ë ûë û ë 5 2 + 3i úû ë 7 2 + 3i úû

(a) P + Q (b) 2 P + 3Q is equal to


12. The sum of all real roots of the
é1 1ù é -1 -1ù
(c) 2Q (d) Q (a) ê ú (b) ê ú equation | x - 3| 2 + | x - 3| - 2 = 0 is
ë1 2 û ë -1 2 û
5. The roots of the equation (a) 2 (b) 3
2
é 1 -1ù é -1 -1ù
(q - r )x + (r - p )x + ( p - q ) = 0 are (c) ê ú (d) ê ú (c) 4 (d) 6
ë -1 2 û ë -1 -2 û
(r - p) 1 13. It is given that the roots of the
(a) ,
(q - r ) 2 9. If | a | denotes the absolute value equation x 2 - 4 x - log 3 P = 0 are
(p - q) of an integer, then which of the real. For this the minimum value of
(b) ,1
(q - r ) following are correct? P is
(q - r ) 1. | ab | = | a || b | 1 1
(c) ,1 (a) (b)
(p - q) 2. | a + b | £ | a | + | b | 27 64
3. | a - b | ³ | a | - | b | 1
(r - p) 1 (c) (d) 1
(d) , 81
(p - q) 2
2 NDA/NA Solved Paper 2017 (II)

14. If A is a square matrix, then the 20. A person is to count 4500 notes. Let The number of ways they can be
value of adj A T - (adj A )T is equal an denote the number of notes he seated is
to counts in the nth minute. If (a) 24 ´ 8! ´ 8! (b) (8!)3
(a) A
a1 = a 2 = a 3 = … = a10 = 150, and a10 , (c) 210 ´ 8! ´ 8! (d) 16!
(b) 2| A|I, where I is the identity matrix a11 , a12 , … are in AP with the
common difference -2, then the 27. The system of equations
(c) null matrix whose order is same as
time taken by him to count all the kx + y + z = 1, x + ky + z = k and
that of A
notes is x + y + kz = k 2 has no solution if k
(d) unit matrix whose order is same as
that of A (a) 24 minutes (b) 34 minutes equals.
(c) 125 minutes (d) 135 minutes (a) 0 (b) 1
15. The value of the product
1 1 1 1 (c) -1 (d) -2
21. The smallest positive integer n for
6 2 ´ 6 4 ´ 6 8 ´ 616 ´ … up to infinite n
28. If 13
. + 23
. + 332 3
. + ¼ + n.3n
æ1 + i ö
terms is which ç ÷ = 1, is (2 n - 1)3a + b
è1 - i ø = then a and b are
(a) 6 (b) 36 (c) 216 (d) 512 4
(a) 1 (b) 4 (c) 8 (d) 16
16. The value of the determinant respectively
½cos 2 q sin 2 q½ 22. If we define a relation R on the set (a) n, 2 (b) n, 3
½ N ´ N as (a, b ) R (c , d ) Û
2 2½ (c) n + 1, 2 (d) n + 1, 3
½ 2q 2 q½
a + d = b + c for all (a, b ), (c , d )
p
29. In DPQR, ÐR = . If tan æç ö÷ and
sin cos P
Î N ´ N, then the relation is
½ 2 2½ 2 è2ø
(a) symmetric only
for all values of q, is (b) symmetric and transitive only æQ ö
tan ç ÷ are the roots of the
(a)1 (b) cos q (c) sinq (d) cos2q (c) equivalence relation è2ø
17. The number of terms in the (d) reflexive only equation ax 2 + bx + c = 0, then
expansion of ( x + a )100 + ( x - a )100 23. If y = x + x 2 + x 3 + … up to which one of the following is
after simplification is infinite terms where x < 1, then correct?
(a) 202 (b) 101 (c) 51 (d) 50 which one of the following is (a) a = b + c (b) b = c + a

18. In the expansion of (1 + x ) , the 50 correct? (c) c = a + b (d) b = c


y y
(a) x = (b) x = 4
sum of the coefficients of odd 1+ y 1- y 30. If ½z - ½ = 2, Then the maximum
powers of x is ½ z½
1+ y 1- y
(a) 2 26
(b) 2 49
(c) x = (d) x = value of | z | is equal to
y y
(c) 2 50 (d) 2 51 (a) 1 + 3 (b) 1 + 5
24. If a and b are the roots of the (c) 1 - 5 (d) 5 - 1
19. If a, b, c are non-zero real numbers,
equation 3x 2 + 2x + 1 = 0, then the
then the inverse of the matrix 31. The angle of elevation of a
equation whose roots are a + b -1
éa 0 0ù stationary cloud from a point 25 m
and b + a -1 is
A = ê0 b 0ú above a lake is 15° and the angle of
ê ú (a) 3x 2 + 8x + 16 = 0 depression of its image in the lake is
êë 0 0 c úû 45°. The height of the cloud above
(b) 3x 2 - 8x - 16 = 0
is equal to the lake level is
(c) 3x 2 + 8x - 16 = 0
é a -1 0ù
0 (a) 25 m (b) 25 3 m
ê ú (d) x 2 + 8x + 16 = 0
(a) ê 0 b -1
0ú (c) 50 m (d) 50 3 m
ê 0 0 c -1 ú 25. The value of
ë û 32. The value of
é a -1 0
1 1 1
0ù + + + …up
1 ê -1 ú log 3 e log 3 e 2
log 3 e 4 tan 9° - tan 27° - tan 63° + tan 81°
(b) ê 0 b 0ú is equal to
abc ê -1 ú to infinite terms is
0 0 c
ë û (a) -1 (b) 0
(a) loge 9 (b) 0 (c) 1 (d) loge 3
é 1 0 0ù (c) 1 (d) 4
1 ê
(c) 0 1 0ú 26. A tea party is arranged for 16
abc ê ú
people along two sides of a long 33. The value of 3 cosec 20° - sec 20°
êë 0 0 1úû
table with eight chairs on each side. is equal to
é a 0 0ù
1 ê Four particular men wish to sit on
(d) 0 b 0ú (a) 4 (b) 2
abc ê ú one particular side and two
(c) 1 (d) -4
êë 0 0 c úû
particular men on the other side.
NDA/NA Solved Paper 2017 (II) 3

34. Angle a is divided into two parts A 40. The principal value of sin -1 x lies in 46. If a$ and b$ are two unit vectors, then the
and B such that A - B = x and the interval vector (a$ + b$ ) ´ (a$ ´ b$ ) is parallel to
tan A : tan B = p : q . The value of p p p p
(a) æç - , ö÷ (b) é - , ù (a) (a$ - b$ ) (b) (a$ + b$ )
sin x is equal to è 2 2ø êë 2 2 úû
( p + q )sin a psina (c) (2a$ - b$ ) (d) (2a$ + b$ )
p
(a)
p-q
(b)
p+ q (c) é 0, ù (d) [0, p ]
êë 2 úû ®
47. A force F = i$ + 3 $j + 2k$ acts on a
psina ( p - q )sin a
(c) (d) 41. The points (a, b ), (0, 0), ( -a, - b ) and particle to displace it from the point
p-q p+ q
2
(ab, b ) are A(i$ + 2 $j - 3k$ ) to the point
35. The value of B (3i$ - $j + 5k$ ). The work done by
(a) the vertices of a parallelogram
æ3ö æ1ö
sin -1 ç ÷ + tan -1 ç ÷ is equal to (b) the vertices of a rectangle the force will be
è5ø è7 ø
(c) the vertices of a square (a) 5 units (b) 7 units
p p p
(a) 0 (b) (c) (d) (d) collinear (c) 9 units (d) 10 units
4 3 2
®

36. The angles of elevation of the top of 42. The length of the normal from 48. For any vector a
a tower from the top and foot of a origin to the plane x + 2y - 2z = 9 is ® ® ®

equal to | a ´ i$| 2 + | a ´ $j | 2 + | a ´ k$| 2


pole are respectively 30° and 45°. If is equal to
h T is the height of the tower and h P (a) 2 units (b) 3 units ® ® ® ®
(a)|a|2 (b) 2|a|2 (c) 3|a|2 (d) 4|a|2
is the height of the pole, then which (c) 4 units (d) 5 units
of the following are correct? 49. A man running round a racecourse
43. If a , b and g are the angles which
2h h ¾® notes that the sum of the distances
1. P T = h P2 the vector OP (O being the origin)
3+ 3 of two flag-posts from him is
makes with positive direction of the always 10 m and the distance
h - hP hP coordinate axes, then which of the
2. T = between the flag-posts is 8 m. The
3 +1 2 following are correct? area of the path he encloses is
2(h P + h T ) 1. cos 2 a + cos 2 b = sin 2 g (a) 18p square metres
3. =4+ 3
hP 2. sin 2 a + sin 2 b = cos 2 g (b) 15p square metres
2 2
3. sin a + sin b + sin g = 2 2 (c) 12p square metres
Select the correct answer using the
code given below. (d) 8p square metres
Select the correct answer using the
(a) 1 and 3 only (b) 2 and 3 only code given below. 50. The distance of the point (1, 3) from
(c) 1 and 2 only (d) 1, 2 and 3 (a) 1 and 2 only the line 2x + 3y = 6, measured
parallel to the line 4 x + y = 4, is
(b) 2 and 3 only
37. In a triangle ABC, a - 2b + c = 0. (a)
5
units (b)
3
units
(c) 1 and 3 only 13 17
æAö æC ö
The value of cot ç ÷ cot ç ÷ is (d) 1, 2 and 3 17
è2ø è2ø (c) 17 units (d) units
9 3 44. The angle between the lines 2
(a) (b) 3 (c) (d) 1
2 2 x + y - 3 = 0 and x - y + 3 = 0 is a 51. If the vectors ai$ + $j + k$, i$ + bj$ + k$
and the acute angle between the
38. 1 + sin A = - æçsin
A Aö lines x - 3y + 2 3 = 0 and and i$ + $j + ck$ (a, b, c ¹ 1) are
+ cos ÷ is
è 2 2ø coplanar, then the value of
3x - y + 1 = 0 is b. Which one of
true if the following is correct? 1 1 1
3p 5p p 3p
+ +
(a) < A< only (b) < A < only (a) a = b (b) a > b 1-a 1-b 1-c
2 2 2 2
(c) a < b (d) a = 2b is equal to
3p 7p 3p
(c) < A< (d) 0 < A < ® ® (a) 0 (b) 1
2 2 2 45. Let a = i$ + 2 $j - k$, b = 2i$ - $j + 3k$
® (c) a + b + c (d) abc
39. In triangle ABC, if and g = 2i$ + $j + 6k$ be three vectors.
sin 2 A + sin 2 B + sin 2 C ® ® 52. The point of intersection of the line
=2 If a and b are both perpendicular to joining the points ( -3, 4, - 8) and
cos 2 A + cos 2 B + cos 2 C the vector d and d × g = 10, then what (5, - 6, 4 ) with XY -plane is
then the triangle is is the magnitude of d? 7 8 7 8
(a) æç , - , 0ö÷ (b) æç - , - , 0ö÷
(a) right-angled (b) equilateral (a) 3 units (b) 2 3 units è3 3 ø è 3 3 ø
7 8 7 8
(c) isosceles (d) obtuse-angled (c)
3
unit (d)
1
unit (c) æç - , , 0ö÷ (d) æç , , 0ö÷
è 3 3 ø è3 3 ø
2 3
4 NDA/NA Solved Paper 2017 (II)

53. If the angle between the lines in A, B and C respectively. The 1


3. x 2 + cos is continuous at x = 0
whose direction ratios are (2, - 1, 2) locus of the centre of the sphere x
p OABC, O being the origin, is Which of the above are correct?
and x, 3, 5 is , then the smaller x y z a b c
4 (a) + + = 1 (b) + + = 1 (a) 1 and 2 only
value of x is a b c x y z
(b) 2 and 3 only
(a) 52 (b) 4 (c) 2 (d) 1 (c)
a b c
+ + =2 (d)
x y z
+ + =2
x y z a b c (c) 1 and 3 only
54. The position of the point (1, 2) (d) 1, 2 and 3
relative to the ellipse 60. The equation of the plane passing
2x 2 + 7y 2 = 20 is through the line of intersection of 65. Consider the following statements :
the planes x + y + z = 1, dy
(a) outside the ellipse 1. at a point on the curve gives
2x + 3y + 4z = 7, and perpendicular dx
(b) inside the ellipse but not at the focus
to the plane x - 5y + 3z = 5 is given slope of the tangent at that
(c) on the ellipse
by point.
(d) at the focus
(a) x + 2 y + 3 z - 6 = 0 2. If a(t ) denotes acceleration of a
55. The equation of straight line which (b) x + 2 y + 3 z + 6 = 0 particle, then ò a(t )dt + c gives
cuts off an intercept of 5 units on
(c) 3x + 4 y + 5 z - 8 = 0 velocity of the particle.
negative direction of Y -axis and
makes and angle 120° with positive (d) 3x + 4 y + 5 z + 8 = 0 3. If s(t ) gives displacement of a
direction of X -axis is 61. The inverse of the function y = 5 ln x ds
particle at time t, then gives
(a) y + 3x + 5 = 0 is dt
(b) y - 3x + 5 = 0 1 its acceleration at that instant.
(a) x = yln 5 , y > 0 Which of the above statements
(c) y + 3x - 5 = 0
(d) y - 3x - 5 = 0 (b) x = yln 5 , y > 0 is/are correct?
1
(a) 1 and 2 only (b) 2 only
56. The equation of the line passing (c) x = yln 5 , y < 0
(c) 1 only (d) 1, 2 and 3
through the point (2, 3) and the (d) x = 5ln y, y > 0
point of intersection of lines æ x + 1ö -1 æ x - 1 ö
62. A function is defined as follows : 66. If y = sec -1 ç ÷ + sin ç ÷,
2x - 3y + 7 = 0 and 7 x + 4y + 2 = 0 è x - 1ø è x + 1ø
is ì x dy
ï- , x ¹0 then is equal to
(a) 21x + 46 y - 180 = 0 f (x ) = í x2 dx
(b) 21x - 46 y + 96 = 0 ïî 0 , x =0 (a) 0 (b) 1
(c) 46x + 21y - 155 = 0 Which one of the following is x -1 x+1
(c) (d)
(d) 46x - 21y - 29 = 0 correct in respect of the above x+1 x -1
function? -1
57. The equation of the ellipse whose 67. What is ò tan (sec x + tan x )dx
(a) f(x ) is continuous at x = 0 but not
centre is at origin, major axis is
3 differentiable at x = 0 equal to?
along X -axis with eccentricity (b) f(x ) is continuous as well as px x2 px x2
4 (a) + +C (b) + +C
differentiable at x = 0 4 4 2 4
and latus rectum 4 units is (c) f(x ) is discontinuous at x = 0
px px 2 px x 2
x2 7 y2 49x 2 7 y2 (d) None of the above (c) + + C (d) - +C
(a) + =1 (b) + =1 4 4 4 4
1024 64 1024 64 (cos x ) ¥ dy
7x 2 49 y2 x2 y2 63. If y = (cos x ) (cos x ) , then is 68. A function is defined in (0, ¥ ) by
(c) + = 1 (d) + =1 dx
1024 64 1024 64 equal to æ 1- x2 for 0< x £1
ç
58. The equation of the circle which (a) -
2
y tan x
(b)
y tan x 2
f (x ) = ç ln x for 1 < x £ 2
passes through the points (1, 0), (0, 1 - y ln(cos x ) 1 + y ln(cos x ) ç ln 2 - 1 + 05
.x for 2 < x < ¥
è
-6) and (3, 4) is y2 tan x y2 sin x
2 2 (c) (d) Which one of the following is
(a) 4x + 4 y + 142 x + 47 y + 140 = 0 1 - y ln(sin x ) 1 + y ln(sin x )
(b) 4x 2 + 4 y2 - 142 x - 47 y + 138 = 0 correct in respect of the derivative
64. Consider the following of the function, i.e., f ¢ ( x )?
(c) 4x 2 + 4 y2 - 142 x + 47 y + 138 = 0
1. x + x 2 is continuous at x = 0 (a) f ¢(x ) = 2 x for 0 < x £ 1
(d) 4x 2 + 4 y2 + 150x - 49 y + 138 = 0
1 (b) f ¢(x ) = - 2 x for 0 < x £ 1
59. A variable plane passes through a 2. x + cos is discontinuous at
x (c) f ¢(x ) = - 2 x for 0 < x < 1
fixed point (a, b, c ) and cuts the axes
x =0 (d) f ¢(x ) = 0 for 0 < x < ¥
NDA/NA Solved Paper 2017 (II) 5

69. Which one of the following is sin x cos x 81. The order and degree of the
75. If limp = l and lim = m,
correct in respect of the function x® x x®¥ x differential equation
2
f ( x ) = x ( x - 1)( x + 1) ? then which one of the following is é æ dy ö 2 ù
3
2
2
2 éd y ù
(a) The local maximum value is larger correct? ê1 + ç ÷ ú = r ê 2 ú
than local minimum value 2 êë è dx ø úû ë dx û
(a) l = 1, m = 1 (b) l = ,m= ¥
(b) The local maximum value is smaller p are respectively
than local minimum value 2
(c) The function has no local maximum (c) l = ,m= 0 (d) l = 1, m = ¥ (a) 3 and 2 (b) 2 and 2
p
(d) The function has no local minimum (c) 2 and 3 (d) 1 and 3
2p
x
70. Consider the following statements : 76. What is ò 1 + sin dx equal to? æ 2x ö dy
0
2 82. If y = cos -1 ç 2
÷, then is
1. Derivative of f ( x ) may not exist è1 + x ø dx
at some point. (a) 8 (b) 4
equal to
2. Derivative of f ( x ) may exist (c) 2 (d) 0 2
finitely at some point. (a) - for all|x| < 1
77. The area bounded by the curve 1 + x2
3. Derivative of f ( x ) may be | x | + |y | = 1 2
infinite (geometrically) at some (b) - for all|x| > 1
(a) 1 square unit 1 + x2
point.
(b) 2 2 square units 2
Which of the above statements are (c) for all|x| < 1
(c) 2 square units 1 + x2
correct?
(a) 1 and 2 only (b) 2 and 3 only (d) 2 3 square units (d) None of the above
2
(c) 1 and 3 only (d) 1, 2 and 3 x
78. If x is any real number, then 83. The set of all points, where the
ln x 1+ x4 function f ( x ) = 1 - e -x
2
is
71. The maximum value of is
x belongs to which one of the differentiable, is
1 2 following intervals?
(a) e (b) (c) (d) 1 (a) (0, ¥) (b) (-¥, ¥)
1
e e (a) (0, 1) (b) æç 0, ùú (c) (-¥, 0) È (0, ¥) (d) (-1, ¥)
è 2û
72. The function f ( x ) = | x | - x 3 is 1
(c) æç 0, ö÷ (d) [0, 1] 84. Match List-I with List-II and select
(a) odd è 2ø the correct answer using the code
(b) even 79. The left-hand derivative of given below the lists :
(c) both even and odd
f ( x ) = [x ]sin( px ) at x = k List-I List-II
(d) neither even nor odd (Function) (Maximum value)
where k is an integer and [x ] is the
73. If l 1 =
d sin x
(e ) greatest integer function, is A. sin x + cos x 1. 10
k -1
dx (a) (-1) (k - 1)p
k
(b) (-1) (k - 1)p B. 3sin x + 4cos x 2. 2
e sin( x + h ) - e sin x (c) (-1)k kp (d) (-1)k - 1 kp
l 2 = lim C. 2sin x + cos x 3. 5
x®0 h x
80. If f ( x ) = - 1, then on the interval D. sin x + 3cos x 4. 5
l 3 = ò e sin x cos xdx 2
[0, p ] which one of the following is Code
then which one of the following is correct?
A B C D
correct? (a) tan[f(x )], where [×] is the greatest
d 1 (a) 2 3 1 4
(a) l1 ¹ l2 (b) ( l3 ) = l2 integer function, and are both
dx f( x ) (b) 2 3 4 1
continuous
(c) ò l3dx = l2 (d) l2 = l3 (c) 3 2 1 4
(b) tan[f(x )], where [×] is the greatest
integer function, and f -1 (x ) are both (d) 3 2 4 1
74. The general solution of continuous 85. If f ( x ) = x ( x - x + 1 ), then f ( x )
dy ax + h (c) tan[f(x )], where [×] is the greatest
= 1 is
dx by + k integer function, and are both
f( x ) (a) continuous but not differentiable at
represents a circle only when discontinuous x=0
(a) a = b = 0 (b) a = - b ¹ 0 (d) tan[f(x )], where [×] is the greatest (b) differentiable at x = 0
integer function is discontinuous (c) not continuous at x = 0
(c) a = b ¹ 0, h = k (d) a = b ¹ 0 1
but is continuous (d) None of the above
f( x )
6 NDA/NA Solved Paper 2017 (II)

86. Which one of the following graph 92. The differential equation of 96. The value of the determinant
x
represents the function f ( x ) = , minimum order by eliminating the 1-a a -a2 a2
x arbitral constants A and C in the
x ¹ 0? equation 1 - b b - b2 b2
(a) y y = A[sin( x + C ) + cos( x + C )] is 1- g g - g2 g2
(a) y¢ ¢ + (sin x + cos x )y¢ = 1
+1 is equal to
(b) y¢ ¢ = (sin x + cos x )y¢
x (a) (a - b )(b - g )(a - g )
O
(c) y¢ ¢ = ( y¢ )2 + sin x cos x
(b) (a - b )(b - g )(g - a)
(b) y (d) y¢ ¢ + y = 0
(c) (a - b )(b - g )(g - a)(a + b + g )
+1 93. Consider the following (d) 0
x statements:
O
–1 97. The adjoint of the matrix
Statement I :
(c) y é1 0 2ù
x > sin x for all x > 0
+1 A = ê2 1 0ú is
Statement II : ê ú
êë0 3 1úû
x´ O x f ( x ) = x - sin x is an increaser
function for all x > 0 é -1 6 2 ù é1 6 -2 ù
(d) None of the above (a) ê -2 1 -4ú ê
(b) -2 1 4ú
Which one of the following is ê ú ê ú
1
87. Let f (n ) = éê +
n ù correct is respect of the above êë 6 3 1 úû êë 6 -3 1 úû
ë 4 1000 úû, where [x ]
statements? é6 1 2 ù é -6 2 1ù
denot the integral part of x. Then the (a) Both Statement I and Statement II (c) ê 4 -1 2 ú (d) ê 4 -2 1 ú
1000 ê ú ê ú
are true and Statement II is the êë 6 3 -1úû êë 3 1 -6úû
value of å f (n ) is
n =1 correct explanation of StatementI
(b) Both Statement I and Statement II æ -2 2 ö
(a) 251 (b) 250 (c) 1 (d) 0 98. If A = ç ÷, then which one of
are true and Statement II is not the è 2 -2ø
88. ò ( ln x ) -1 dx - ò ( ln x ) -2 dx is equal correct explanation of Statement I
the following is correct?
(c) Statement I is true but Statement II
to is false (a) A 2 = - 2 A (b) A 2 = - 4 A
-1 -2
(a) x(ln x ) + C (b) x(ln x ) + C (d) Statement I is false but Statement
2
(c) A = - 3 A (d) A 2 = 4 A
2
(c) x(ln x ) + C (d) x(ln x ) + C II is true
99. Geometrically Re (z 2 - i ) = 2,
89. A cylindrical jar without a lid has to 94. The solution of the differential where i = -1 and Re is the real
be constructed using a given surface equation part, represents
area of a metal sheet. If the capacity dy yf¢ ( x ) - y 2 (a) circle (b) ellipse
of the jar is to be maximum, then the = is
dx f( x ) (c) rectangular hyperbola
diameter of the jar must be k times
the height of the jar. The value of k is x (d) parabola
(a) y =
(a) 1 (b) 2 (c) 3 (d) 4 f(x ) + c
100. If p + q + r = a + b + c = 0, then the
f(x )
90. The value of (b) y = +c determinant
p p
x
f(x ) + c ½pa qb rc ½
ò 4 tan x dx + ò 4 cot x dx is equal (c) y = ½qc ra pb½equals
0 0
x ½ ½
to f(x )
p p p p (d) y = ½rb pc qa½
(a) (b) (c) (d) x+c
4 2 2 2 2 (a) 0 (b) 1
4x + x 4 (c) pa + qb + rc
91. Let g be the greatest integer 95. If f (x ) = and
1 + 4x 3 (d) pa + qb + rc + a + b + c
function. Then the function
f ( x ) = ( g ( x )) 2 - g ( x ) is æ1 + x ö 101. A committee of two persons is
g ( x ) = ln ç ÷, then what is the
discontinuous at è1 - x ø selected from two men and two
(a) all integers æe - 1ö women. The probability that the
value of fg ç ÷ equal to? committee will have exactly one
(b) all integers except 0 and 1 èe + 1ø
woman is
(c) all integers except 0 1 1 2 1 1
(a) 2 (b) 1 (c) 0 (d) (a) (b) (c) (d)
(d) all integers except 1 2 6 3 3 2
NDA/NA Solved Paper 2017 (II) 7

102. Let a die be loaded in such a way 107. Which one of the following can be 113. If P( B ) = 3 , P( A Ç B Ç C ) = 1 and
that even faces are twice likely to considered as appropriate pair of 4 3
occur as the odd faces. What is the values of regression coefficient of y 1
P( A Ç B Ç C ) = , then what is
probability that a prime number on x and regression coefficient of x 3
will show up when the die is on y? P( B Ç C ) equal to?
tossed? (a) (1, 1) (b) (-1, 1) 1 3
(a) (b)
1 2 4 5 1 1 10 12 4
(a)
3
(b)
3
(c)
9
(d)
9 (c) æç - , 2 ö÷ (d) æç , ö÷
è 2 ø è3 3 ø 1 1
(c) (d)
103. Let the sample space consist of non- 108. Let A and B be two events with 15 9
negative integers up to 50, X 1 1 114. The following table gives the
denote the numbers which are P( A ) = , P( B ) = and monthly expenditure of two
3 6
multiples of 3 andY denote the odd 1 families :
numbers. Which of the following P( A Ç B ) = . What is P( B| A )
is/are correct? 12 Expenditure (in `)
8 1 equal to?
Items Family A Family B
1. P( X ) = 2. P(Y ) = 1 1
25 2 (a) (b)
5 7 Foods 3,500 2,700
Select the correct answer using the 1 1 Clothing 500 800
(c) (d)
code given below. 8 10
Rent 1,500 1,000
(a) 1 only
109. In a binomial distribution, the mean
(b) 2 only 2 5 Education 2,000 1,800
is and the variance is . What is
(c) Both 1 and 2 3 9 Miscellaneous 2,500 1,800
(d) Neither 1 nor 2 the probability that X = 2 ?
5 25 In construction a pie diagram to the
104. For two events A and B, (a) (b) above data, the radii of the circles
36 36
1 2 are to be chosen by which one of
let P( A ) = , P( A È B ) = and 25 25
2 3 (c) (d) the following ratios?
216 54
1 (a) 1 : 1 (b) 10 : 9
P( A Ç B ) = . What is P( A Ç B )
6 110. The probability that a ship safely
1 (c) 100 : 91 (d) 5 : 4
equal to? reaches a port is . The probability
1 1 1 1 3 115. If a variable takes values 0, 1, 2, 3, …,
(a) (b) (c) (d)
6 4 3 2 that out of 5 ships, at least 4 ships n with frequencies
would arrive safely is 1, C (n, 1), C (n, 2), C (n, 3), … , C (n, n )
105. Consider the following statements : 1 10
1. Coefficient of variation depends (a) (b) respectively, then the arithmetic
243 243
on the unit of measurement of mean is
11 13
the variable. (c) (d) (a) 2n (b) n + 1
243 243
2. Range is a measure of (c) n (d)
n
dispersion. 111. What is the probability that at least 2
two persons out of a group of three
3. Mean deviation is least when persons were born in the same 116. In a multiple-choice test, an
measured about median. month (disregard year)? examinee either knows the correct
Which of the above statements are 33 17 answer with probability p, or
correct? (a) (b) guesses with probability 1 - p. The
144 72
(a) 1 and 2 only (b) 2 and 3 only 1 2 probability of answering a question
(c) (d) 1
(c) 1 and 3 only (d) 1, 2 and 3 144 9 correctly is , if he or she merely
m
106. Given that the arithmetic mean and 112. It is given that X = 10, Y = 90, guesses. If the examinee answers a
standard deviation of a sample of 15 s X = 3, s Y = 12 and r XY = 08
. . The question correctly, the probability
observations are 24 and 0 regression equation of X on Y is that he or she really knows the
respectively. Then which one of the answer is
(a) Y = 32
. X + 58
following is the arithmetic mean of mp mp
the smallest five observations in the (b) X = 32
. Y + 58 (a) (b)
1 + mp 1 + (m - 1)p
data? (c) X = - 8 + 02
. Y
(m - 1)p (m - 1)p
(a) 0 (b) 8 (c) 16 (d) 24 (d) Y = - 8 + 02
. X (c) (d)
1 + (m - 1)p 1 + mp
8 NDA/NA Solved Paper 2017 (II)

117. If x 1 and x 2 are positive quantities, 1. Variance is unaffected by 119. Five sticks of length 1, 3, 5, 7 and 9
then the condition for the change of origin and change of feet are given. Three of these sticks
difference between the arithmetic scale. are selected at random. What is the
mean and the geometric mean to be 2. Coefficient of variance is probability that the selected sticks
greater than 1 is independent of the unit of can form a triangle?
(a) x1 + x 2 > 2 x1 x 2 observations. (a) 0.5 (b) 0.4 (c) 0.3 (d) 0

(b) x1 + x2 > 2 Which of the statements given 120. The coefficient of correlation when
above is/are correct? coefficients of regression are 0.2
(c)| x1 - x 2| > 2
(a) 1 only (b) 2 only and 1.8 is
(d) x1 + x 2 < 2( x1 x 2 + 1)
(c) Both 1 and 2 (d) Neither 1 nor 2 (a) 0.36 (b) 0.2 (c) 0.6 (d) 0.9
118. Consider the following statements :

PAPER II English Language and General Studies

Part A (English Language)


Directions (Q. Nos. 1-10) Each item 7. The decision of the Union 13. The students made a generous
in this section consists of a sentence government to repeal the Urban contribution to the flood relief
with an underlined word/words Land Ceiling Act has been fun(d)
followed by four options. Select the welcomed by all. (a) niggard (b) selfish
option that is nearest in meaning to the (a) suppress (b) amend (c) spendthrift (d) indecent
underlined word/words. (c) cancel (d) withhold
14. He was just idle by temperament.
1. The discussion was wound up after 8. This is his maiden appearance on (a) employed (b) occupied
a long fruitful exchange of view. the screen. (c) industrious (d) happy
(a) postponed (b) cut short (a) first (b) last (c) girlish (d) shy
(c) interrupted (d) concluded
15. Most of the decisions taken by the
9. At the end of the marathon officer were unjust.
2. He was fully alive to the need for everybody was exhauste(d) (a) serious (b) lenient
making adjustments. (a) weakened (b) honoured (c) correct (d) imbecile
(a) concerned about (c) satisfied (d) tired
16. He is a loving father and takes great
(b) worried about 10. He gave me a counterfeit coin. delight in his children.
(c) aware of (a) rare (b) fake
(d) indifferent about (a) revolt (b) dissatisfaction
(c) unmatured (d) inferior
(c) enjoyment (d) disgust
3. The police officer tried to intimidate Directions (Q. Nos. 11-20) Each
the witness but in vain. item in this section consists of a 17. He was quite concerned about his
(a) inform (b) reward sentence with an underlined son’s career.
(c) frighten (d) persuade word/words followed by four options. (a) unrelated (b) indifferent
(c) dispassionate (d) carefree
4. We must adopt drastic measures to Select the option that is opposite in
control population growth. meaning to the underlined 18. They are confident of success.
(a) simple (b) dramatic word/words. (a) imprudent (b) impatient
(c) diffident (d) reluctant
(c) realistic (d) severe 11. My mother has been working hard
5. He is extremely meticulous in his for the last two weeks and she feels 19. We carried on the search for the
approach. run down. missing person.
(a) simple (b) careful (a) morbid (b) energetic (a) delayed
(c) fair (d) reasonable (c) exhausted (d) emotional (b) reconsidered
(c) broke up
6. The experts’ minute examination 12. The President condemned the Act (d) called off
brought to light some important of violence during the celebration
clues. of the festival. 20. This TV has many indigenous
(a) quick (b) detailed (a) reason (b) instigation components.
(c) superficial (d) prolonged (c) restraint (d) sobriety (a) Indian (b) foreign
(c) unnatural (d) genuine
NDA/NA Solved Paper 2017 (II) 9

Directions (Q. Nos. 21-30) In the (a) Q P S R (b) S Q P R Read each sentence to find out whether
following passage, at certain points you are (c) R P Q S (d) P R Q S there is any error in any underlined part
given a choice of four words marked (a), (b), 32. After our school boys had won a and mark your response to corresponding
(c) and (d), one of which fits the meaning of letter i.e. (a), (b) or (c). If you find no error,
well-contested hockey match
the passage. Choose the best word out of the your response should be indicated as (d).
four. Mark the letter, viz., (a), (b), (c) or (d), so that they might communicate the
relating to this wor(d) P 36. An electrical circuit is the complete
(a) (b)
Passage news of their victory to the
headmaster who is a keen sportsman path traversed by electric current.
After this incident I went to (c)
Nainital and returned after nearly a Q
No error
month, I had 21. ______ 22. ______ they came to school in high spirits (d)
my clothes when I saw Gangu R
and takes a very lively interest in
37. He waved us a by-by as he boarded
standing (a) (b)
23. ______a new baby. He was S
the train which disappeared into
24.______with joy. Even Nanda school games (c)
(a) Q P S R (b) S Q P R
25. ______ not have 26. ______ (c) R P Q S (d) P R Q S the tunnel. No error
such joy 27. ______ getting (d)
33. Even a leisurely game like cricket
Krishna. His face had the same 28. 38. There was great deal that had to be
demanding grace rather than
______ on that (a)
P
29. ______ face of a 30. ______ scrapped, that must be scrapped;
strength and over the rough tactics
man after a full meal but surely India could not have
Q (b)
21. (a) hardly (b) barely (c) merely of the Australian team that visited
(d) rarely taken been what she undoubtedly was,
England in 1921 as we saw in the and could not have continued a
22. (a) out (b) away (c) off (d) on R (c)
23. (a) by (b) near (c) with (d}) at controversy over body-line bowling cultured existence for thousands of
24. (a) jumping (b) bursting (c) dancing can cause much ill-will years. No error
(d) singing S (d)
25. (a) could (b) would (c) should (a) P S R Q (b) R S P Q
(d) ought (c) S R Q P (d) Q P R S 39. With regard to interior decoration,
(a)
26. (a) experimented (b) shown (c) felt 34. Scientists point out of sunspot it is the attention given to the less
(d) heard P overt aspects of using space that
27. (a) at (b) in (c) on (d) into activity that it is an aftermath that (c)
28. (a) light (b) glow (c) sense (d) hope Q give it life, an identity, a quality
29. (a) comes (b) appears (c) rices has now reached its peak of the that makes it exciting and unusual.
(d) shows R S (c)
30. (a) starved (b) starving (c) hungry eleven-year cycle No error
(d) satisfied (a) R S P Q (b) P Q S R (d)
(c) Q R P S (d) Q S P R
Directions (Q. Nos. 31-35) Each of the 40. A small parcel of novels is better
following items in this section consists of a 35. As the ship streams from San Diego as (a) (b) (c)
sentence, the parts of which have been walls of gray water from a distant than none. No error
jumble(d) These parts have been labelled P, P (d)
Q, R and S. Given below each sentence are storm in the North Pacific making
four sequences namely (a), (b), (c) and (d).
Q
Directions (Q. Nos. 41-50) Each of the
You are required to re-arrange the jumbled following sentence in this section has a
parts of the sentence and mark your the greener among us miserable blank space and four words/group of words
response accordingly. with sea sickness rock and toss the given after the sentence. Select the word or
R group/words you consider most
31. The Spirit of man has slowly and appropriate for the blank space and mark
ship those of us aboard have a
P your response accordingly.
S
painfully surmounted and his
personal demonstration of powerful 41. He looks as if he .......... weary.
Q (a) is (b) was
ocean movement
growing intelligence all the obstacles (a) P Q R S (b) S R P Q (c) would be (d) were
R (c) S P R Q (d) Q S R P 42. My house is insured .......... theft and
that have come in his way has faced fire.
S Directions (Q. Nos. 36-40) Each item in
(a) for (b) against
this section has a sentence with three
all kinds of danger underlined parts labelled as (a), (b) and (c).
(c) in (d) towards
10 NDA/NA Solved Paper 2017 (II)

43. The result of the prolonged 46. Years .......... since I saw her last. 49. I saw her when she was
discussion was...… . (a) have passed standing......the side of the old statue.
(a) disappointment (b) disappointing (b) had passed (a) by
(c) disappointed (d) to disappoint (c) had been passing (b) at
(d) have been passing (c) in
44. You are lucky .......... in the 20th (d) beyond
century. 47. When he heard the terrible noise he
(a) by being born (b) to have been born asked me what was .......... on. 50. True friends never .......... their loved
(c) for being born (d) to have born (a) happening (b) being ones in adversity.
(c) getting (d) going (a) abuse
45. Sita is true to........ . 48. Could you lend me some money ? I (b) criticise
(a) word (b) her words (c) befool
am very .......... of cash at the moment. (d) desert
(c) the words (d) words
(a) down (b) low (c) short (d) scarce

Part B (General Studies)


51. According to the Travel and Tourism (a) Major Rock Edict 13 62. Consider the following States of India
Competitiveness Index (TTCI) 2017 (b) Rock Edict 6 in terms of percentage of forest area
(c) Minor Rock Edict 1
released by the World Economic (d) Pillar Edict 4 in relation to the total area of the
Forum, among the 136 economies State.
across the World, India ranked 56. The Cabinet Mission Plan for India 1. Karnataka 2. Odisha
(a) 50th (b) 40th envisaged a
(a) Federation
3. Kerala 4. Andhra Pradesh
(c) 30th (d) 20th
(b) Confederation Which one of the following is the
52. Which one of the following is the (c) Unitary from of government correct descending order?
theme of the ‘World Health Day’, (d) Union of States (a) 1-2-4-3 (b) 3-1-2-4
2017 celebrated by the World Health (c) 3-2-1-4 (d) 2-3-1-4
57. The creation of the institution of
Organisation? Lokpal was first recommended by 63. Which one of the following States in
(a) Diabetes India has the longest coastline?
(a) Law Commission
(b) Food safety (a) Odisha (b) Tamil Nadu
(b) Santhanam Committee
(c) Depression : Let’s Talk (c) Karnataka (d) West Bengal
(c) Shah Commission
(d) Ageing and Health
(d) Administrative Reforms Commission 64. Which one of the following states in
53. Which one of the following
58. Which one of the following is a cause India has the largest area under forest
ministries has launched a new cover?
of acid rains?
programme on Interdisciplinary (a) Ozone (b) Ammonia (a) Maharashtra (b) Chhattisgarh
Cyber Physical Systems (ICPS) to (c) Sulphur dioxide (d) Carbon monoxide (c) Madhya Pradesh (d) Andhra Pradesh
foster and promote R and D?
(a) Ministry of Earth Sciences 59. The desirable range of pH for 65. Which one of the following is not an
(b) Ministry of Science and Technology drinking water is igneous rock?
(a) 6 . 5 to 8 . 5 (b) 5 . 0 to 6 . 5 (a) Gabbro (b) Granite
(c) Ministry of Information and
(c) 6 . 5 to 7 .0 (d) 7 . 0 to 8 . 5 (c) Dolomite (d) Basalt
Broadcasting
(d) Ministry of New and Renewable 60. Consider the following reaction : 66. The Coriolis effect is the result of
Energy (a) Pressure gradient
CH 4 + 2O 2 → CO 2 + 2H 2 O (b) Earth’s axis of inclination
54. Consider the following statements Which of the following about the (c) Earth’s rotation
about the Nagara style of temple (d) Earth’s revolution
reaction given above is/are correct?
architecture : 67. Where is Mekong Delta located?
1. Carbon is oxidised
1. This style of temples are (a) Thailand (b) Cambodia
2. Hydrogen is oxidised
commonly found in the areas (c) Myanmar (d) Vietnam
3. Hydrogen is reduced
between Himalayas and 68. Which one of the following pairs of
4. Carbon is reduced
Vindhyas. rivers and tributaries is not correctly
Select the correct answer using the
2. The most striking feature of this code given below :
matched?
style is its pyramidal shikhara (a) Godavari : Indravati
(a) Only 1 (b) 1 and 2
(b) Ganga : Penganga
Which of the statements given above (c) 2 and 3 (d) 2 and 4
(c) Krishna : Bhima
is/are correct? 61. Sunrise in Eastern Arunachal (d) Luni : Sukri
(a) Only 1 (b) Only 2
(c) 1 and 2 (d) Neither 1 nor 2 Pradesh would be about how many 69. Consider the following characteristics
hours before the sunrise in Western of a tropical cyclone :
55. Ashoka’s connection with Buddhism Gujarat?
is evident from which one of the 1. A warm sea temperature of>26° C
(a) One hour (b) Two hours
following edicts? (c) Three hours (d) Four hours 2. High relative humidity of
atmosphere at a height of> 700 m.
NDA/NA Solved Paper 2017 (II) 11

3. Atmospheric instability. (a) Gulf of Mannar, Nokrek, Panchmarhi happen to its weight compared to its
and Simlipal weight in air?
The above mentioned characteristics (b) Gulf of Mannar, Kanchenjunga,
are associate with which one of the (a) Increase
Nokrek and Seshachalam (b) Decrease
following cycles of its development? (c) Nilgiri, Nokrek, Panchmarhi and Panna (c) Remain exactly the same
(a) Formulation and initial stage (d) Nilgiri, Nokrek, Panchmarhi and (d) Increase or decrease cannot be
(b) Modification stage Seshachalam predicted
(c) Full maturity
(d) Decay 77. Which of the following statements 85. Light year is a measure of
about magnetite ore of iron is/are (a) time
70. In the Mesopotamian records, which correct? (b) distance
one of the following terms was used for 1. It is known as black ore. (c) total amount of light falling on the Earth
the Indus Valley (Harappans)? in a year
(a) Dilmun (b) Meluha 2. It contains 60% to 70% of pure (d) average intensity of light falling on the
(c) Magan (d) Failaka iron. Earth in a year
71. Who among the following is the 3. It possesses magnetic properties. 86. Which one of the following was set as
Chairman of interdisciplinary Select the correct answer using the a target of average growth of GDP of
committee constituted recently by codes given below: India over the plan period 2012-17 by
the Government of India to examine (a) Only 1 (b) 2 and 3
the Approach Paper to the Twelfth
(c) 1 and 3 (d) All of these
framework for virtual currencies? Five Year Plan?
(a) Secretary, Department of Financial 78. Which one of the following vitamins (a) 7% (b) 8% (c) 9% (d) 10%
Services has a role in blood clotting?
(b) Special Secretary, Department of (a) Vitamin-A (b) Vitamin-D
87. Which one of the following is not a
Revenue (c) Vitamin-E (d) Vitamin-K subject that has been devolved to the
(c) Special Secretary, Department of Panchayati Raj Institutions by the
Economic Affairs 79. The term ‘Probiotic’ is applied to 11th Schedule of the Constitution of
(d) Deputy Governor, Reserve Bank of (a) organic food (b)antacid India?
India (c) antibiotic (a) Non-conventional energy resources
(d) live microbial food supplement (b) Roads
72. SAMPADA scheme is being
implemented by the Ministry of 80. Which one of the following microbes (c) Higher education
(a) Finance causes acidification and curding of (d) Libraries
(b) Housing and Urban Affairs milk? 88. Who among the following used the
(c) Food Processing Industries (a) Lactic Acid Bacillus term Industrial Revolution for the first
(d) Earth Sciences (b) Clostridium botulinum time in English to describe the
(c) Vibrio cholerae
73. The shortest day length that occurs in (d) Saccharomyces cerevisiae
changes that occurred in the British
the Northern hemisphere is on industrial development between 1760
(a) March 21 (b)September 23 81. Who among the following shared the and 1820?
(c) November 22 (d)December 22 Nobel Prize in 1962 along with (a) Friedrich Engels (b) Eric Hobsbawm
74. The Indian Railways have gone in for Francis Crick and James Watson for (c) Arnold Toynbee (d) Georges Michelet
their discoveries concerning the
qualitative improvements since 89. Who among the following is the
independence. Which of the molecular structure of nucleic acids?
author of the book ‘The Indian
(a) Erwin Chargaff
following have taken place in recent Struggle, 1920-34’?
(b) Maurice Hugh Frederick Wilkins
years? (c) Rosalind Franklin
(a) Maulana Abul Kalam
1. Gauge conversion (b) Jayprakash Narayan
(d) Phoebus Levene
(c) Subhash Chandra Bose
2. Track electrification 82. Water boils at a lower temperature at (d) Manabendra Nath Roy
3. Automatic Signals high altitudes, because
Select the correct answer using the (a) the air pressure is less
90. Which one of the following about the
(b) outside temperature is less
Swadeshi Campaign in 1896 is not
codes given below :
(a) 1 and 2 (b) 2 and 3 (c) latent heat is less correct?
(c) 1 and 3 (d) All of these (d) None of the above (a) Its centre was Maharashtra
(b) Its main participants were students.
75. In India, maximum rainfall is 83. Concave mirror is used in headlights (c) It opposed the levy of tariff on imports.
received from of vehicles, because it (d) It publicly burnt foreign clothes.
(a) Western Disturbances (a) focuses light from the bulb onto nearby
(b) North-East Monsoon vehicles 91. Which one of the following
(c) South-West Monsoon (b) sends parallel rays associations was founded in London
(d) Retreating Monsoon (c) fits well into the shape of the headlight by Dadabhai Naoroji in 1866?
(d) is cheaper than other mirrors (a) The Bengal British India Society
76. Which set of the following biosphere (b) The East India Association
reserves in India is included in the 84. If some object is weighed when (c) The British Indian Association
World Network of Biosphere submerged in water, what will (d) The Madras Native Association
Reserves?
12 NDA/NA Solved Paper 2017 (II)

92. Mariana Trench is located in the 101. Which one of the following (c) Sodium carbonate
(d) Sodium bicarbonate
ocean floor of statements about a satellite orbiting
(a) Southern Atlantic Ocean around the Earth is correct? 110. Why is potassium permanganate used
(b) Western Pacific Ocean (a) Satellite is kept in orbit by remote for purifying drinking water?
(c) Eastern Pacific Ocean control from ground station. (a) It kills germs
(d) Northern Atlantic Ocean (b) Satellite is kept in orbit by retro-rocket (b) It dissolves the impurities
and solar energy keeps it moving (c) It is a reducting agent
93. Taklamakan Desert is situated in around the Earth. (d) It is an oxidising agent
(a) Western Asia
(c) Satellite requires energy from solar 111. Consider the following movements :
(b) Southern fringe of Sahara in Africa
panels and solid fuels for orbiting.
(c) South America
(d) Satellite does not required any energy 1. Moplah Rebellion
(d) Central Asia for orbiting. 2. Bardoli Satyagraha
94. Rudraprayag is situated at the 102. Which 3. Champaran Satyagraha
one of the following
confluence of rivers Alakananda and 4. Salt Satyagraha
(a) Bhagirathi (b) Mandakini
statements about energy is correct?
Which one of the following is the
(c) Nandakini (d) Dhauliganga (a) Energy can be created as well as
destroyed correct chronological order of the
95. Arrange the following Indian cities (b) Energy can be created but not above ascending order?
according to their locations from destroyed (a) 1-3-4-2 (b) 3-1-2-4
(c) Energy can neither be created nor (c) 2-3-1-4 (d) 4-2-1-3
West to East :
destroyed 112. Which one of the following travelogues
1. Bilaspur 2. Jodhpur
(d) Energy cannot be created but can be has given an insight on the reign of
3. Bhopal 4. Ranchi destroyed
Select the correct answer using the Muhammed-bin-Tughlaq?
code given below : 103. Step-up transformers are used for (a) Ibn Battuta’s Rihla
(a) increasing electrical power (b) Francois Bernier’s Travels in the Mogul
(a) 3-2-1-4 (b) 2-3-1-4
(c) 4-1-2-3 (d) 2-1-3-4 (b) decreasing electrical power Empire
(c) decreasing voltage (c) Niccolao Manucci’s Storia do Mogor
96. The Kashmir region receives (d) increasing voltage (d) Tavernier’s Travels in India
additional amount of precipitation
during the winter brought by
104. Which one among the following 113. Which one of the following was not a
waves carries the maximum energy Chishti Sufi saint?
(a) South-West Monsoon (a) Khwaja Moinuddin
(b) Western Disturbances per photon?
(b) Baba Fariduddin Gani-i-Shakar
(c) Retreating Monsoon (a) X-rays (b) Radio waves
(c) Nizamuddin Auliya
(d) Temperate Cyclone (c) Light waves (d) Microwaves
(d) Shaikh Bahauddin Zakariya
97. Which part of India has the Kalakot 105. How much CO 2 is produced on 114. In April, 2017, India celebrated 100
tertiary coal field? heating of 1 kg of carbon? years of Mahatma Gandhi’s
(a) Brahmaputra river basin of Assam 11 3 4 3 (a) Satyagraha in Kheda
(a) kg (b) kg (c) kg (d) kg
(b) Damodar river basin of Jharkhand and 3 11 3 4 (b) Dandi March
West Bengal (c) Satyagraha in Champaran
(c) Himalayan mountain region
106. Zinc is used to protect iron form
(d) Return from South Africa
(d) Cardamom hills in Kerala corrosion because zinc is
(a) more electropositive than iron 115. A rainbow is produced due to which
98. Tendons through which muscles are (b) cheaper than iron one of the following phenomena?
connected to bones are tightly (c) a bluish white metal (a) Dispersion of light
compacted bundles of which one of (d) a good conductor of heat and (b) Interference of light
the following long fibrous protein? electricity (c) Diffraction of light
(d) Scattering of light by atmospheric dust
(a) Fibrin (b) Collagen 107. Which one of the following gases is
(c) Elastin (d) Cellulose placed second in respect of 116. Bats detect obstacles in their path by
99. Which one of the following is the abundance in the Earth’s receiving the reflected
scientific name of the causal atmosphere? (a) Infrasonic waves (b) Ultrasonic waves
(a) Oxygen (b) Hydrogen (c) Radio waves (d) Microwaves
organism elephantiasis?
(c) Nitrogen (d) Carbon dioxide 117. The statement that ‘heat cannot flow
(a) Ascaris lumbricoides
(b) Culex pipiens 108. Which one of the following is a by itself from a body at a lower
(c) Wuchereria bancrofiti chemical change? temperature to a body at a higher
(d) Fasciola hepatica (a) Cutting of hair temperature’, is known as
(b) Graying of hair naturally (a) Zeroth law of thermodynamics
100. Melanin is the natural pigment that
(c) Swelling of resin in water (b) First law of thermodynamics
gives colour to human skin, hair and (d) Cutting of fruit (c) Second law of thermodynamics
the iris provides protection against (d) Third law of thermodynamics
(a) Ultraviolet radiation 109. Which one among the following
(b) Infrared radiation chemicals is used as washing soda? 118. Which one of the following waves
(c) X-ray radiation (a) Calcium carbonate does not belong to the category of the
(d) Short wave radio radiation (b) Calcium bicarbonate other three?
NDA/NA Solved Paper 2017 (II) 13

(a) X-rays (b) Microwaves (a) It proposed that the Indian Monuments and Sites (World Heritage
(c) Radiowaves (d) Sound waves magistrates would try Europeans in Day) 2017?
criminal cases. (a) The Heritage of Sport
119. Which one of the following statements (b) It allowed Indians to file criminal
is not correct? (b) Cultural Heritage and Sustainable
cases against Europeans. Tourism
(a) Human eye is a refracting system (c) It authorised Indian ICS officers to try
containing a diverging lens. (c) Past and Present Heritage
Europeans in courts. (d) Heritage and Science
(b) The retina of the human eye contains (d) It was an agitation led by Ilbert in
millions of light sensitive cells, called support of the nationalists. 134. In April, 2017, the USA dropped MOAB
rods and cones, which convert the light (Massive Ordinance Air Blast popularly
into electrical messages. 126. Who among the following can
attend the meetings of both Houses known as the Mother of All Bombs) in
(c) Every image that is focussed on the
retina is upside down. of Parliament while not being a the suspected hideouts of militants in
(d) We need both eyes to judge the relative member of either House? which one of the following countries?
positions of objects accurately. (a) Iran (b) Syria
(a) The Solicitor General of India
(c) Afghanistan (d) Somalia
120. Which one of the following statements (b) The Vice-President of India
is not corect? (c) The Comptroller and Auditor General 135. Intake of which one of the following
of India food components should be minimised
(a) Ultrasonic waves cannot get reflected,
(d) The Attorney General of India by patients having Gouty Arthritis due
refracted or absorbed.
(b) Ultrasonic waves are used to detect the 127. Who among the following was to elevated serum uric acid level?
presence of defects like cracks, porsity, believed to be a leader of the (a) Food fibres (b)Nucleic acids
etc in the internal structure of common (c) Lipids (d)Carbohydrates
Sanyasis and Fakirs conspiring
structure materials.
(c) Ultrasonic waves can be used for
against the British in 1857? 136. Which one of the following statements
making holes in very hard materials like (a) Mangal Pandey (b) Bahadur Shah II about microbes is not correct?
diamond. (c) Queen Zeenat Mahal (a) They are used in sewage treatment
(d) Ultrasonic waves cannot travel through (d) Nana Sahib plants.
vacuum. (b) They are used in industrial fermenters for
128. Who among the following was the the production of beverages.
121. The principal use of hydrofluoric acid founder of the Avadh Kingdom in (c) No antibiotic has been obtained from any
is the 18th century? microbe.
(a) in etching glass (a) Murshid Quli Khan (b) Saadat Khan (d) They are used to get many bioactive
(b) as a bleaching agent (c) Alivardi Khan (d)Sarfaraz Khan molecules for the treatment of diseases.
(c) as an extremely strong oxidising agent 129. Who among the following was the 137. Golden rice is a genetically
(d) in the preparation of strong organic founder of the Young Bengal
fluorine compounds -modified crop plant where the
Movement? incorporated gene is meant for
122. The species that has the same number (a) Henry Vivian Derozio biosynthesis of
35 (b) David Hare (a) Omega-3 fatty acids
of electrons as Cl is (c) Dwarkanath Tagore
17 (b) Vitamin-A
(d) Prasanna Kumar Tagore (c) Vitamin-B
32 34 40 35 2−
(a) S (b) S+ (c) Ar + (d) S 130. Which one of the following (d) Vitamin-C
16 16 18 16
statements about the Quit Indian 138. An object moves in a circular path with
123. The compound C 6 H12 O 4 contains Movement is not correct? a constant speed Which one of the
(a) 22 atoms per mole (a) It broke out in August, 1942. following statements is correct?
(b) twice the mass percent of H as (b) Ahmedabad Textile Mills went on (a) The centripetal acceleration of the object
compared to the mass percent strike for more than three months. is smaller for a gentle curve (i.e., curve of
of C (c) Muslim League and Hindu larger radius) than that for a sharp curve
(c) six times the mass percent of C as Mahasabha actively participated in (i.e., curve of smaller radius).
compared to the mass percent the movement. (b) The centripetal acceleration is greater for
of H (d) Communist Party did not support the a gentle curve than that for a sharp curve.
(d) thrice the mass percent of H as movement. (c) The centripetal acceleration is the same
compared to the mass percent for both, the gentle and sharp curves.
of O
131. Who among the following is the
(d) The centripetal acceleration causes the
winner of the Singapore Open
124. The proposition ‘equal volumes of object to slow down.
Superseries Badminton Men’s
different gases contain equal numbers Singles title 2017? 139. The force acting on a particle of mass m
of molecules at the same temperature (a) Kidambi Srikanth (b) Lin Dan moving along the x-axis is given by
and pressure’ is known as (c) B Sai Praneeth (d) Kento Momota F ( x ) = Ax 2 − Bx . Which one of the
(a) Avogadro’s hypothesis
132. Koradi Thermal Power Station is following is the potential energy of the
(b) Gay-Lussac’s hypothesis
located in particle?
(c) Planck’s hypothesis
(a) Nagpur (b) Raipur x2
(d) Kirchhoff’s theory (a) 2Ax − B (b) − (2Ax − 3B)
(c) Mumbai (d) Secunderabad 6
125. Which one of the following statements 133. Which one of the following is the (c) Ax 3 − Bx 2 (d) Zero
about the Ilbert Bill is correct?
theme of the International Day for
14 NDA/NA Solved Paper 2017 (II)

140. The symbol of SI unit of inductance is 143. How long does light take to reach the (b) conduction electrons
(c) ions
H. It stands for Earth from the Sun? (d) holes
(a) Holm (b) Halogen (a) About 4 minutes (b) About 8 minutes
(c) Henry (d) Hertz (c) About 24 minutes (d) About 24 hours 148. The ionisation energy of hydrogen
atom in the ground state is
141. In a vacuum, a five-rupee coin, a 144. Radioactivity is measured by (a) 13 . 6 Me V (b) 13 . 6 eV
feather of sparrow bird and a mango (a) GM Counter (b)Polarimeter
(d) Zero (c) 13 . 6 Joule
are dropped simultaneously from the (c) Calorimeter (d)Colorimeter
same height. The time taken by them 145. The mirrors used as rear-view 149. When pure water boils vigorously,
to reach the bottom is t 1, t 2 and t 3 the bubbles that rise to the surface are
mirrors in vehicles are
respectively. In this situation, we will (a) concave (b) convex
composed primarily of
observe that (c) cylindrical (d) plane (a) air
(a) t 1 > t 2 > t 3 (b) t 1 > t 3 > t 2 (b) hydrogen
(c) t 3 > t 1 > t 2 (d) t 1 = t 2 = t 3
146. Which one of the following waves is (c) hydrogen and oxygen
used for detecting forgery in (d) water vapour
142. Electron emission from a metallic currency notes? 150. Which compound, when dissolved in
surface by application of light is (a) Ultraviolet waves (b) Infrared waves water, conducts electricity and forms
known as (c) Radio waves (d) Microwaves
a basic solution?
(a) Thermionic emission
(b) Photo electric emission
147. The majority charge carriers in a (a) HCl (b) CH 3COOH
p-type semiconductor are (c) CH 3OH (d) NaOH
(c) High field emission
(d) Auto electronic emission (a) free electrons

ANSWERS
Paper I Elementary Mathematics
1 c 2 b 3 c 4 d 5 b 6 c 7 c 8 b 9 d 10 a
11 a 12 d 13 c 14 c 15 a 16 b 17 c 18 b 19 a 20 b
21 b 22 c 23 a 24 a 25 a 26 c 27 d 28 d 29 c 30 b
31 b 32 d 33 a 34 d 35 b 36 c 37 b 38 a 39 a 40 b
41 d 42 b 43 c 44 b 45 b 46 a 47 c 48 b 49 b 50 d
51 b 52 a 53 b 54 a 55 a 56 b 57 b 58 c 59 c 60 a
61 a 62 c 63 a 64 a 65 a 66 a 67 a 68 b 69 a 70 d
71 b 72 d 73 d 74 b 75 c 76 a 77 c 78 * 79 a 80 c
81 b 82 a 83 c 84 b 85 b 86 c 87 a 88 a 89 b 90 d
91 d 92 d 93 b 94 d 95 b 96 b 97 b 98 b 99 c 100 a
101 b 102 c 103 d 104 a 105 a 106 d 107 a 108 c 109 c 110 c
111 b 12 c 113 a 114 b 115 d 116 b 117 c 118 b 119 c 120 c

Paper II English Language and General Studies


1 d 2 c 3 c 4 d 5 b 6 b 7 c 8 a 9 d 10 b
11 b 12 d 13 a 14 c 15 c 16 d 17 b 18 c 19 d 20 b
21 b 22 a 23 b 24 b 25 a 26 b 27 b 28 a 29 a 30 b
31 b 32 c 33 c 34 c 35 c 36 c 37 a 38 d 39 b 40 d
41 a 42 b 43 b 44 b 45 b 46 a 47 d 48 c 49 a 50 d
51 b 52 c 53 b 54 a 55 a 56 d 57 d 58 c 59 c 60 a
61 b 62 c 63 b 64 c 65 c 66 c 67 d 68 b 69 c 70 b
71 c 72 c 73 d 74 d 75 c 76 a 77 d 78 d 79 d 80 a
81 b 82 a 83 b 84 b 85 b 86 b 87 c 88 c 89 c 90 d
91 b 92 b 93 d 94 b 95 b 96 b 97 c 98 b 99 c 100 a
101 d 102 c 103 d 104 a 105 a 106 a 107 a 108 b 109 c 110 d
111 b 112 a 113 d 114 c 115 a 116 b 117 c 118 d 119 a 120 a
121 a 122 c 123 c 124 a 125 a 126 d 127 b 128 b 129 a 130 c
131 c 132 a 133 b 134 c 135 b 136 c 137 b 138 a 139 b 140 c

See Detailed Solution of this paper at


Competitive Exams Section in
Downloads of arihantbooks.com
01
MATHEMATICS SET THEORY 3

SET THEORY
In NDA exam, generally 3-4 questions are asked from this chapter which are based on types of sets,
operations on sets and number of elements in a set.

A set is a collection of well defined distinct objects. By ‘well defined’ we mean that from any object
given we can find whether it is an element of the set or not. Sets are usually denoted by capital letters
A , B, C, etc. The members of a set are called its elements and are usually denoted by small letters,
a, b, c, etc. If ‘a’ is an element of a set A, then we write a ∈ A i.e. ‘a’ belongs to set A and if ‘a’ does not
belong to set A, then we write a ∈ / A.

Representation of Sets
1. Tabular or Roster method In this method, a set is described by writing elements, separated by
commas, within the braces {}.
e.g. N = { 1, 2, 3, 4, K} is a set of natural numbers.
A = { 2, 3, 5, 7, 11, 13} is a set of first six prime numbers.
W = {0, 1, 2, 3, 4, K} is a set of whole numbers.
2. Set-builder or Rule method In this method, a rule or a formula is written in the braces that defines
the sets.
e.g. A = { x : x = 2n + 1, n ≥ 1, n ∈ N}, B = { x : 6 ≤ x ≤ 12, x ∈ N}, C = { x : x = 2n, n < 8, n ∈ N}

Different Types of Sets


1. Null set A set which does not contain any element is called a null set. It is denoted by φ. A null set is
also called an empty set or a void set. Therefore, φ = { }
e.g. A = { x : x is a prime number between 90 and 96} = φ
2. Singleton set A set which contains only one element is called a singleton set.
e.g. A = {0}, B = { x : x + 10 = 0, x ∈ Z }
3. Finite set A set is called a finite set, if it is either void set or its elements can be counted or labelled by
natural numbers 1,2,3, .... and the process of counting stops at a certain natural number (say ‘n’).
e.g. A = { a, e, i, o, u} = Set of all vowels
The number of distinct elements of a finite set A is called the cardinal number of the set A and it is
denoted by n ( A ).
4 NDA/NA Pathfinder

4. Infinite set A set which has unlimited number of e.g. Let A = {1, 2, 3}; B = { 2, 4, 6, 8 }; C = {1, 3, 5, 7, 9}
elements is called infinite set. and U = set of natural numbers.
e.g. N = set of all natural numbers = {1, 2, 3, …}
Here, A , B and C are the subsets of U.
Z = set of all integers
Therefore, U is an universal set.
= {…, − 2, – 1, 0, 1, 2, …} are infinite sets.
11. Power set Let A be a set, then the set of all the possible
5. Equivalent sets Two finite sets A and B are
subsets of A is called the power set of A and is denoted
equivalent, if their cardinal numbers are same, i.e.
by P ( A ).
n ( A ) = n ( B).
i.e. P ( A ) = {S | S ⊆ A }
e.g. A = { a, b, c, d , e}; B = {1, 3, 5, 7, 9}
e.g. Let A = {1, 2, 3}. Then, subsets of A are
Here, n ( A ) = n ( B)
φ, { 1}, { 2}, { 3}, { 1, 2}, { 1, 3}, { 2, 3} and { 1, 2, 3}.
So, these sets are equivalent sets.
∴ P ( A ) = { φ, { 1}, { 2}, { 3}, { 1, 2}, { 1, 3}, { 2, 3}, {1, 2, 3}}
6. Equal sets Two sets A and B are said to be equal, if
every element of A is a member of B and every Note l Since, the empty set and the set A itself are subsets of A and
element of B is a member of A. also the elements of P ( A ). Thus, the power set of a given set
is always non-empty.
If sets A and B are equal, we write A = B l If a set A has n elements, then its power set will contain 2 n
and A ≠ B, when A and B are not equal. elements.
e.g.
(i) A = {1, 3, 4} and B = {3, 1, 4} are equal sets. Some Important Properties
(ii) A = Letters of the word MASS = {M, A, S, S} (i) A ⊆ A , ∀A (ii) φ ⊆ A , ∀A
B = Letters of the word SAM = {S, A, M} (iii) A ⊆ U , ∀A in U (iv) A = B ⇔ A ⊆ B, B ⊆ A
Here, A = B
EXAMPLE 1. Which one of the following is a null set?
Since, {M, A, S, S } = {M, A, S} = {S, A, M} a. { x| x ∈ R , 4 x2 − 1 = 0 }
Note Equal sets are equivalent but equivalent sets may or may b. { x| x ∈N , x is odd and ( x + 3) is even}
not be equal. e.g. Set A = {4, 5, 3, 2} and B = {1, 6, 8, 9}
are equivalent but are not equal. c. { x| x ∈ R , x2 < 3}
d. { x| x ∈ R , x2 < 0 }
7. Subset Let A and B be two sets. If every element of
A is an element of B, then A is called a subset of B Sol. d. Only option (d) satisfies the condition as x 2 can never
and it is denoted by A ⊆ B. be less than 0. Therefore, there is no element in it, i.e. it
is a null set.
∴ A ⊆ B, if x ∈ A ⇒ x ∈ B
e.g. If A = {1, 2, 3} EXAMPLE 2. Two finite sets having m and n elements.
and B = {1, 2, 3, 4, 5, 6}, then A ⊆ B. The total number of subsets of the first set is 56 more
than the total number of subsets of the second set. Find
The total number of subsets of a finite set the values of m and n.
containing n elements is 2n .
a. 6, 5 b. 6, 3 c. 6, 6 d. 5, 4
8. Superset Let A and B be two sets. If B contains all
elements of A, then B is called superset of A and it Sol. b. Let A and B be two sets having m and n elements
is denoted by B ⊇ A. respectively. Then, number of subsets of A = 2m ,
number of subsets of B = 2n
e.g. If B = { a, b, c, d , . . . , x, y, z} and A = {a, e, i, o, u}
It is given that, 2m − 2n = 56 ⇒ 2n(2m − n − 1) = 23(23 − 1)
Then, B ⊇ A.
⇒ n = 3 and m − n = 3
9. Proper subset A set A is said to be a proper subset
of set B, if A is a subset of B and A is not equal to ∴ n = 3 and m=6
B. It is written as A ⊂ B. The total number of
proper subsets of a finite set containing n elements EXAMPLE 3. The number of proper subsets of the set
is ( 2n − 1). e.g. If A = {1, 2, 3}, then proper subsets {1, 2, 3} is
of A are φ, {1}, {2}, {3}, {1, 2}, {1, 3}, {2, 3}. a. 6 b. 3
c. 7 d. 8
10. Universal set All the sets under consideration are
likely to be subsets of a set which is called the Sol. c. Number of proper subsets of the set {1, 2, 3}
universal set and is denoted by U. = 23 − 1 = 7
MATHEMATICS Set Theory 5

Venn Diagram 4. Difference of two sets Let A and B be two sets. The
difference of A and B, written as ( A − B) or A\B, is the
Venn diagram are used to set of all those elements of A which do not belong to B.
express relationship among
Thus, ( A − B) = { x : x ∈ A , x ∉ B}
sets. In Venn diagram, the
universal set U is represented The Venn diagram of A − B is as shown in the figure
by a rectangle and its subsets A B U
are represented by closed curves (usually circles)
within the rectangle.

Operations on Sets
Similarly, the difference ( B − A ) is the set of all those
Now, we introduce some operations on sets to
elements of B, which do not belong to A,
construct new sets from the given ones.
i.e. ( B − A ) = { x : x ∈ B, x ∉ A }.
1. Union of two sets Let A and B be two sets. The
union of A and B is the set of all those elements e.g. If A = { a, b, c, d , e} and B = { a, c, e, g, i }
which belongs to either A or B or both A and B. then, A − B = { b, d } and B − A = { g, i}
The union of A and B is denoted by A ∪ B 5. Symmetric difference of two sets Let A and B be two
(read as ‘A union B’). sets. The symmetric difference of sets A and B is the set
Thus, A ∪ B = { x : x ∈ A or x ∈ B} ( A − B) ∪ ( B − A ) and is denoted by A∆B.
A B U
e.g. If A = {1, 2, 3, 4} A B U

and B = { 3, 4, 5, 6}, (B – A)
then A ∪ B = {1, 2, 3, 4, 5, 6} (A – B)
Note If A 1, A 2, K , A n is a finite family of sets, then their union
Thus, A∆B = ( A − B) ∪ ( B − A ) = { x : x ∉ A ∩ B}
n
is denoted by ∪ A i or A1 ∪ A 2 ∪ A 3 ∪ K ∪ A n.
i =1
e.g. If A = {a, b, c, d, e} and B = {a, c, e, g, i}
2. Intersection of two sets Let A A B U Then, A∆ B = ( A − B) ∪ ( B − A ) = {b, d} ∪ {g, i} = {b, d, g, i}
and B be two sets.
The intersection of A and B is 6. Complement of a set The
U
the set of all those elements complement of a set A is the set of all
which belong to both A and B. those elements which are in universal A
The intersection of A and B is denoted by A ∩ B set but not in A. It is denoted by A ′ or
(read as ‘A intersection B’). A c or U – A.
If U is a universal set and A ⊂ U , then
Thus, A ∩ B = { x : x ∈ A and x ∈ B}
A ′ = U − A = { x : x ∈U , x ∉ A }
e.g. If A = {1, 2, 3, 4} and B = {3, 4, 5, 6}, then Clearly, x ∈ A ′ ⇔ x ∉ A
A ∩ B = {3, 4} e.g. If U = {1, 2, 3, 4, 5, 6, 7, 8, 9, 10} and A = {1, 3, 5, 7, 9},
Note If A 1, A 2, K , A n is a finite family of sets, then their
n then A ′ = {2, 4, 6, 8, 10}
intersection is denoted by ∩ A i or A 1 ∩ A 2 ∩ K ∩ A n.
i =1
EXAMPLE 4. The shaded region in the U
3. Disjoint of two sets Two sets A and B are said to given figure represents which value of A B
be disjoint, if A ∩ B = φ , i.e. they don’t have any sets A = {9, 18, 27, 36 }, B = {18, 36, 54, 72}?
common element. If A ∩ B ≠ φ, then A and B are a. {18, 36} b. {9, 18, 36, 72}
said to be intersecting or overlapping sets. c. A ∪ B d. {9, 27, 54, 72}
Sol. d. Clearly, the diagram represents the condition of
symmetric difference of two sets, i.e. ( A − B) ∪ ( B − A),
where {x : x ∉ A ∩ B}.
Now, ( A − B) = A − ( A ∩ B) = {9, 27}
( B − A) = B − ( A ∩ B) = {54, 72}
e.g. If A = {1, 2, 3} and B = { 4, 5, 6} So, ( A − B) ∪ ( B − A) = {9, 27, 54, 72}
Then, A ∩ B = φ , so A and B are disjoint sets. Thus, option (d) is correct.
6 NDA/NA Pathfinder

Laws of Algebra of Sets 5. Number of elements in exactly two of the sets A, B, C.


= n ( A ∩ B) + n ( B ∩ C ) + n (C ∩ A ) − 3n ( A ∩ B ∩ C )
1. Idempotent laws For any set A, we have
6. Number of elements in exactly one of the sets A, B, C
(i) A ∪ A = A (ii) A ∩ A = A
= n ( A ) + n ( B) + n (C ) − 2n ( A ∩ B) − 2n ( B ∩ C )
2. Identity laws For any set A, we have
− 2n ( A ∩ C ) + 3n ( A ∩ B ∩ C )
(i) A ∪ φ = A (ii) A ∩ φ = φ
7. n ( A ′ ∪ B′ ) = n ( A ∩ B)′ = n (U ) − n ( A ∩ B)
(iii) A ∩ U = A (iv) A ∪ U = U
8. n ( A ′ ∩ B′ ) = n ( A ∪ B)′ = n (U ) − n ( A ∪ B)
3. Commutative laws For any two sets A and B, we have
9. n ( A ∆ B) = n ( A ) + n ( B) − 2n ( A ∩ B)
(i) A ∪ B = B ∪ A (ii) A ∩ B = B ∩ A
10. n ( A ′ ) = n (U ) − n ( A )
4. Associative laws If A, B and C are any three sets, then
11. n ( A ∩ B′ ) = n ( A ) − n ( A ∩ B)
(i) ( A ∪ B) ∪ C = A ∪ ( B ∪ C )
(ii) A ∩ ( B ∩ C ) = ( A ∩ B) ∩ C EXAMPLE 6. In a group of 500 students, there are
475 students who can speak Hindi and 200 can speak
5. Distributive laws If A , B and C are any three sets, then Bengali. What is the number of students who can
(i) A ∪ ( B ∩ C ) = ( A ∪ B) ∩ ( A ∪ C ) speak Hindi only?
(ii) A ∩ ( B ∪ C ) = ( A ∩ B) ∪ ( A ∩ C ) a. 275 b. 300 c. 325 d. 350
6. De-Morgan’s laws If A, B and C are any three sets, Sol. b. Given, n (H) = number of students who can speak
then Hindi = 475
(i) ( A ∪ B)′ = A ′ ∩ B′ n ( B) = number of students who can speak Bengali = 200
(ii) ( A ∩ B)′ = A ′ ∪ B′ and n (H ∪ B) = total number of students = 500
7. (i) A − ( B ∪ C ) = ( A − B) ∩ ( A − C ) Now, n (H ∩ B) = number of students who speak both
Hindi and Bengali = n (H) + n ( B) − n (H ∪ B)
(ii) A − ( B ∩ C ) = ( A − B) ∪ ( A − C )
= 475 + 200 − 500 = 175
(iii) A − B = A ∩ B ′ = B ′ − A ′
Now, number of students who can speak Hindi only
(iv) A − ( A − B) = A ∩ B
= n (H) − n (H ∩ B)
(v) A − B = B − A ⇔ A = B = 475 − 175 = 300
(vi) A ∪ B = A ∩ B ⇔ A = B
EXAMPLE 7. In a class of 100 students, 70 have taken
(vii) A ∪ A ′ = U
Science, 60 have taken Mathematics, 40 have taken
(viii) A ∩ A ′ = φ both Science and Mathematics.
EXAMPLE 5. For three events A, B and C, what is the Consider the following statements
simplest form of ( A ∩ B) ∩ ( A ∪ B c ) ∩ ( A c ∪ B)? I. The number of students who have not taken
Science or Mathematics is 10.
a. A ∩ B C b. A ∪ B c. AC ∩ B d. A ∩ B
II. The number of students who have not taken both
Sol. d. We have, {( A ∩ B) ∩ ( A ∪ B c )} ∩ ( Ac ∪ B) Science and Mathematics is 10.
= {A ∩ ( B ∪ B c )} ∩ ( Ac ∪ B) Which of the above statement(s) is/are correct?
= ( A ∩ U) ∩ ( Ac ∪ B) = A ∩ ( Ac ∪ B) a. Only I b. Only II
= ( A ∩ Ac ) ∪ ( A ∪ B) = A ∩ B [Q A ∩ Ac = φ] c. Both I and I d. None of these
Sol. c. Let A and B be the set of students who have taken
Important Results Science and Mathematics, respectively.
If A, B and C are finite sets and U is the finite universal ∴ n( A) = 70, n( B) = 60
set, then and n( A ∩ B) = Number of students who have taken both
1. n ( A ∪ B) = n ( A ) + n ( B) − n ( A ∩ B) = 40
2. n ( A ∪ B) = n ( A ) + n ( B) ⇔ A , B are disjoint non-void We know that, n ( A ∪ B) = n( A) + n( B) − n( A ∩ B)
sets. = 70 + 60 − 40 = 90
3. n ( A − B) = n ( A ) − n ( A ∩ B), ∴ Number of students who have not taken Science or
Mathematics = Number of students who have not taken
i.e. n ( A − B) + n ( A ∩ B) = n ( A )
both Science and Mathematics
4. n ( A ∪ B ∪ C ) = n ( A ) + n ( B) + n (C ) − n ( A ∩ B) = 100 − n( A ∪ B)
− n (B ∩ C) − n ( A ∩ C ) + n ( A ∩ B ∩ C ) = 100 − 90 = 10
MATHEMATICS > Set Theory 7

PRACTICE EXERCISE
1. Total number of elements in the power set of A 11. In a college of 300 students, every student reads
containing 15 elements is 5 newspapers and every newspaper is read by
(a) 215 (b) 152 (c) 215 −1 (d) 215 − 1 60 students. The number of newspapers is
2. What is the number of proper subsets of a given (a) atleast 30 (b) atmost 20
finite set with n elements? (c) exactly 25 (d) None of these
(a) 2 n − 1 (b) 2 n − 2 (c) 2 n − 1 (d) 2 n − 2 12. In an examination out of 100 students, 75 passed
3. If A = P ({ 1, 2}), where P denotes the power set, in English, 60 passed in Mathematics and 45
passed in both English and Mathematics. What
then which one of the following is correct?
is the number of students passed in exactly one
(a) {1, 2} ⊂ A (b) 1∈ A (c) φ ∉ A (d) {1, 2} ∈ A
of the two subjects?
4. If the cardinality of a set A is 4 and that of a set (a) 45 (b) 60 (c) 75 (d) 90
B is 3, then what is the cardinality of the set
A∆B ? 13. If A = { 4n + 2| n is a natural number} and
(a) 1 (b) 5 B = { 3n| n is a natural number}, then what is
(c) 7 (d) Cannot be determined ( A ∩ B) equal to?
5. If P, Q and R are three non-collinear points, then (a) {12 n2 + 6n|, n is a natural number}
what is PQ ∩ PR equal to? (b) {24n − 12 |, n is a natural number}
(a) Null set (b) {P} (c) {P, Q, R} (d) {Q, R} (c) {60n + 30|, n is a natural number}
(d) {12 n − 6|, n is a natural number}
6. The shaded region in the given figure is
A 14. If X = {( 4n − 3n − 1)| n ∈ N } and
Y = { 9 ( n − 1)| n ∈ N }, then what is X ∪ Y equal
to?
B C (a) X (b) Y (c) N (d) A null set

15. Let N denotes the set of natural numbers and


(a) A ∩ (B ∪ C ) (b) A ∪ (B ∩ C ) A = { n 2 : n ∈ N } and B = { n3 : n ∈ N }. Which one of
(c) A − (B ∩ C ) (d) A − (B ∪ C ) the following is correct?
7. If A and B are subsets of a set X, then what is (a) A∪B= N
{ A ∩ ( X − B)} ∪ B equal to? (b) The complement of ( A ∪ B) is an infinite set
(a) A ∪ B (b) A ∩ B (c) A (d) B (c) A ∩ B must be a finite set
(d) A ∩ B must be a proper subset of {m6 : m ∈ N}
8. Let n(U ) = 700, n( A) = 200, n( B) = 300,
n( A ∩ B) = 100, then n( A′ ∩ B ′ ) is equal to 16. Out of 800 boys in a school, 224 played cricket,
(a) 400 (b) 600 240 played hockey and 336 played basketball. Of
(c) 300 (d) None of these the total, 64 played both basketball and hockey,
9. Consider the following Venn diagram. 80 played cricket and basketball and 40 played
cricket and hockey; 24 played all the three
E games. The number of boys, who did not play
A B any game is
(a) 128 (b) 216 (c) 240 (d) 160

If n( E ) = 42, n( A) = 15, n( B) = 12 and 17. Consider the set A of all determinants of order 3
n( A ∪ B) = 22, then the area represented by with entries 0 or 1 only. Let B be the subset of A
shaded portion in the above Venn diagram, is consisting of all determinants with value 1. Let
(a) 25 (b) 27 (c) 32 (d) 37
C be the subset of the set of all determinants
with value –1. Then,
10. If A is the set of the divisions of the number 15, (a) C is empty
B is the set of prime numbers smaller than 10 (b) B has as many elements as C
and C is the set of even numbers smaller than 9,
(c) A = B∪C
then ( A ∪ C ) ∩ B is the set
(d) B has twice as many elements as C
(a) {1, 3, 5} (b) {1, 2, 3} (c) {2} (d) {2, 5}
8 NDA/NA Pathfinder

18. If A = { x : x 2 − 3x + 2 = 0} and 25. Percentage of families having both phone and


2
B = { x : x + 2x − 8 = 0}, then ( A − B) is car, is
(a) 5% (b) 10% (c) 20% (d) 25%
(a) {1, 2} (b) {2}
(c) {1} (d) {4, 3} 26. Percentage of families having either phone or
19. Let A = { x : x ∈ R , x < 1}, B = { x : x ∈ R , x − 1 ≥ 1} car, is
(a) 10% (b) 30% (c) 35% (d) 40%
and A ∪ B = R − D, then the set D is
(a) {x : 1 < x ≤ 2} (b) {x : 1 ≤ x < 2} 27. What is the number of families in the city?
(c) {x : 1 ≤ x ≤ 2} (d) None of these (a) 30000 (b) 40000
2 2 (c) 20000 (d) 10000
20. If A = {( x , y ): x + y = 25} and
B = {( x , y ): x 2 + 9 y 2 = 144}, then A ∩ B contains Directions (Q. Nos. 28-32) In a class, 3 languages
(a) one point (b) three points (c) two points (d) four points are offered mainly Hindi,
21. If two sets A and B having 3 and 6 elements English and French. The total 23 23
2x
respectively, then which of the following is/are number of students learning H E
French is 46. In x denotes the x 153 x
correct?
I. The minimum number of elements of ( A ∪ B) = 6. number of students learning F 11
Hindi and French but bot 28
II. The maximum number of elements of ( A ∩ B) = 3.
Select the correct answer using the code given English, then answer the
below. following using adjacent Venn diagram.
(a) Only I (b) Only II 28. How many students learn precisely two
(c) Both I and II (d) Neither I nor II languages?
22. If A is any set and P ( A) is its power set, then (a) 55 (b) 40 (c) 30 (d) 13
which of the following is/are correct? 29. How many students learn atleast two languages?
I. P ( A ) ∩ P (B) = P ( A ∩ B) (a) 15 (b) 30 (c) 45 (d) 55
II. P ( A ) ∪ P (B) = P ( A ∪ B) 30. What is the total strength of the class?
Select the correct answer using the code given (a) 124 (b) 100 (c) 96 (d) 66
below. 31. How many students learn English and French?
(a) Only II (b) Only I (a) 30 (b) 43 (c) 45 (d) 73
(c) Both I and II (d) Neither I nor II
32. How many students learn atleast one language?
23. Consider the following statements (a) 45 (b) 51
I. All poets (P ) are learned (L ). (c) 96 (d) None of these
II. All learned (L ) are happy (H ).
Which one of the following Venn diagrams
correctly represents both the above statements PREVIOUS YEARS’ QUESTIONS
taken together?
H 33. If A and B are any two sets, then what is the
(c) P (d)
value of A ∩ ( A ∪ B)? e 2012 I
(a) P L (b) P L L P LH
(a) Complement of A (b) Complement of B
H H
(c) B (d) A
24. A relation between three sets is established 34. Let A = {x : x is a square of a natural number
using two expressions, ( A ∪ B) = ( A ∪ C ) and x is less than 100} and B is a set of even
and ( A ∩ B) = ( A ∩ C ), which stays valid if and natural numbers. What is the cardinality of
only if A ∩ B? e 2012 I
I. B = C II. A = B = C III. A = C (a) 4 (b) 5
Which of the above statement(s) is/are correct? (c) 9 (d) None of these
(a) Only I (b) Only II 35. Let U = { x ∈ N : 1 ≤ x ≤ 10} be the universal set,
(c) Both I and II (d) Only III N being the set of natural numbers.
Directions (Q. Nos. 25-27) In a city, 25% of the If A = {1, 2, 3, 4} and B = {2, 3, 6, 10}, then what
families have phone, 15% of the families have car, is the complement of ( A − B)? e 2012 I
(a) {6, 10} (b) {1, 4}
65% of the families have neither phone nor car and
(c) {2, 3, 5, 6, 7, 8, 9, 10} (d) {5, 6, 7, 8, 9, 10}
2000 families have both phone and car.
MATHEMATICS > Set Theory 9

36. Which one of the following is a null set? e 2013 I 44. Consider the following statements
(a) {0} (b) {{{}}} I. The number of students who have taken only
(c) {{}} (d) { x| x2 + 1 = 0, x ∈R} one subject is equal to the number of students
who have taken only two subjects.
37. If A is a subset of B, then which one of the II. The number of students who have taken atleast
following is correct? e 2013 II
two subjects is four times the number of
(a) Ac ⊆ B c (b) Bc ⊆ A c (c) Ac = B c (d) A ⊆ A ∩ B students who have taken all the three subjects.
38. If A = { 1, 3, 5, 7}, then what is the cardinality of Which of the above statement(s) is/are correct?
the power set P ( A)? e 2013 II (a) Only I (b) Only II
(a) 8 (b) 15 (c) 16 (d) 17 (c) Both I and II (d) Neither I nor II
39. Consider the following 45. In a class of 60 students, 45 students like music,
I. A ∪ (B ∩ C ) = ( A ∩ B) ∪ ( A ∩ C ) 50 students like dancing, 5 students like neither.
II. A ∩ (B ∪ C ) = ( A ∪ B) ∩ ( A ∪ C ) Then, the number of students in the class who
Which of the above statement(s) is/are correct? like both music and dancing, is e 2015 I
e 2013 II (a) 35 (b) 40 (c) 50 (d) 55
(a) Only I (b) Only II (c) I and II (d) Neither I nor II 46. Let A = { 1, 2 , 3, 4, 5, 6, 7, 8, 9, 10}. Then, the
40. In a group of 50 people, two tests were conducted, number of subsets of A containing exactly two
one for diabetes and one for blood pressure. elements is e 2015 I
30 people were diagnosed with diabetes and 40 (a) 20 (b) 40 (c) 45 (d) 90
people were diagnosed with high blood pressure. 47. If A = { x : x is a multiple of 3} and B = { x : x is a
What is the minimum number of people who were
having diabetes and high blood pressure? e 2013 II multiple of 4} and C = { x : x is a multiple of 12},
(a) 0 (b) 10 (c) 20 (d) 30
then which one of the following is a null set?
(a) (A \ B) ∪ C (b) (A \ B)\C e 2015 I
41. Which one of the following is an example of (c) ( A ∩ B) ∩ C (d) (A ∩ B) \ C
non-empty set? e 2013 II
(a) Set of all even prime numbers 48. If A = { x ∈ R : x 2 + 6x − 7 < 0} and
(b) {x : x2 − 2 = 0 and x is rational} B = { x ∈ R : x 2 + 9x + 14 > 0}, then which of the
(c) {x : x is a natural number, x < 8 and simultaneously following is/are correct?
x > 12} I. ( A ∩ B) = (− 2, 1) II. ( A \ B) = (− 7, − 2)
(d) {x : x is a point common to any two parallel lines}
Select the correct answer using the code given
Directions (Q. Nos. 42-44) Read the following information below. e 2015 II
carefully to answer the questions that follow. (a) Only I (b) Only II
(c) Both I and II (d) Neither I nor II
In a survey of 25 students, it was found that 15
have taken Mathematics, 12 have taken Physics and 49. A, B, C and D are four sets such that
11 have taken Chemistry, 5 have taken Mathematics A ∩ B = C ∩ D = φ . Consider the following
and Chemistry, 9 have taken Mathematics and I. A ∪ C and B ∪ D are always disjoint.
Physics, 4 have taken Physics and Chemistry and 3 II. A ∩ C and B ∩ D are always disjoint.
have taken all the three subjects. e 2014 I
Which of the above statement(s) is/are correct?
42. The number of students who have taken only (a) Only I (b) Only II e 2015 II
Physics, is (c) Both I and II (d) Neither I nor II
(a) 2 (b) 3 (c) 5 (d) 6
50. What is the number of natural numbers less
43. The number of students who have taken only than or equal to 1000 which are neither divisible
two subjects, is by 10 nor 15 nor 25? e 2016 I
(a) 7 (b) 8 (c) 9 (d) 10 (a) 860 (b) 854 (c) 840 (d) 824

ANSWERS
1 a 2 c 3 d 4 d 5 b 6 d 7 a 8 c 9 a 10 c
11 c 12 a 13 d 14 b 15 d 16 d 17 b 18 c 19 b 20 d
21 c 22 b 23 d 24 a 25 a 26 c 27 b 28 c 29 c 30 a
31 a 32 c 33 d 34 a 35 c 36 d 37 b 38 c 39 d 40 c
41 b 42 a 43 c 44 b 45 b 46 c 47 d 48 a 49 b 50 b
10 NDA/NA Pathfinder

HINTS AND SOLUTIONS


1. (a) If a set A has n elements, then its 13. (d) A = { 4n + 2 | n ∈ N } 19. (b) We have,
power set will contain 2n elements. = { 6, 10, 14, 18, 22, 26, 30, K } A = { x : x ∈ R, − 1 < x < 1}
∴ Total number of elements in is power and B = {3n | n ∈ N } and B = { x : x ∈ R, x − 1 ≤ −1
set of A = 215 = {3, 6, 9, 12, 15, 18, 21,
2. (c) or x − 1 ≥ 1}
24, 27, 30, K }
3. (d) Let B = { 1, 2} = { x : x ∈ R, x ≤ 0 or x ≥ 2}
⇒ A ∩ B = { 6, 18, 30, K }
Then, A = P( B ) = { φ, { 1}, { 2}, { 1, 2} ∴ A ∪ B = { x : x ∈ R, x < 1 or x ≥ 2}
or A ∩ B = { 6 + ( n − 1) 12 | n ∈ N }
Clearly, { 1, 2} ∈ A = R−D
= { 12n − 6 | n ∈ N }
4. (d) Since, the sets A and B are not where, D = { x : x ∈ R, 1 ≤ x < 2}
14. (b) X = {( 4n − 3n − 1) | n ∈ N }
known, then cardinality of the set A∆B 20. (d) Clearly, A is the set of all points on
cannot be determined. and Y = { 9( n − 1) | n ∈ N } the circle x 2 + y 2 = 25 and B is the set
5. (b) Given, P , Q and R are three ⇒ X = { 0, 9, 54, K } of all points on the ellipse
non-collinear points, then and Y = { 0, 9, 18 , 27, 36, 54, K } x 2 + 9 y 2 = 144. These two intersect at
four points P, Q, R and S.
P Q ∩ PR = { P }. ∴ X ∪ Y = { 0, 9, 18, 27, 36, 54, K } = Y Y
P 15. (d) Q A = { n2 : n ∈ N }
x 2 + y 2 = 25
and B = { n3 : n ∈ N } Q P
So, A ∩ B must be a proper subset of X′ X
{m 6 : m ∈ N }. O
Q R R S
16. (d) Given, n (C ) = 224, n( H ) = 240, x 2 + 9y 2 = 144
6. (d) 7. (a) n( B ) = 336, n( H ∩ B ) = 64,
8. (c) n( A ′ ∩ B ′ ) = n( A ∪ B )′ n( B ∩ C ) = 80, n( H ∩ C ) = 40, Y′
= n(U ) − n( A ∪ B ) n(C ∩ H ∩ B ) = 24, n(U ) = 800
Hence, A ∩ B contains four points.
n(C c ∩ H c ∩ B c ) = n [C ∪ H ∪ B ]c
= n(U ) − [ n( A ) + n( B ) − n( A ∩ B )] 21. (c) n ( A ∪ B ) = n( A ) + n( B ) − n( A ∩ B )
= n(U ) − n(C ∪ H ∪ B )
= 700 − { 200 + 300 − 100} = 300 = 3 + 6 − n ( A ∩ B)
= 800 − [ n(C ) + n( H ) + n( B )
9. (a) Shaded region = 9 − n ( A ∩ B)
− n(C ∩ H ) − n( H ∩ B )
= n ( E ) − n ( A ∪ B) + n ( A ∩ B ) As maximum number of element in
− n( B ∩ C ) + n(C ∩ H ∩ B )]
= n( E ) − n( A ∪ B ) + n( A ) + n ( B ) ( A ∩ B ) = minimum of n ( A ) and
− n( A ∪ B ) = 800 − ( 224 + 240 + 336 − 40 − 64 n ( B) = 3
−80 + 24) = 160
= 42 − 22 + 15 + 12 − 22 = 25 ∴ Minimum number of elements in
17. (b) We know that, the interchange of ( A ∪ B) = 9 − 3 = 6
10. (c) Given, A = { 15, 30, 45, 60, 75, ...}
two adjacent rows (columns) changes Hence, both statements are correct.
B = { 2, 3, 5, 7} ; C = { 2, 4, 6, 8} the value of a determinant only in sign
Now, A ∪ C = { 15, 30, 45, 60, 75,K } but not in magnitude. Hence, 22. (b) Let x ∈ P ( A ∩ B )
∪ { 2, 4, 6, 8} corresponding to every element ∆ of B ⇔ x ⊆ ( A ∩ B)
there is an element ∆′ in C obtained by ⇔ x ⊆ A and x ⊆ B
= { 2, 4, 6, 8, 15, 30, 45, 60, K }
interchanging two adjacent rows
Now, ( A ∪ C ) ∩ B (columns) in ∆ . It follows that ⇔ x ∈ P ( A ) and x ∈ P ( B )
= { 2, 4, 6, 8, 15, 30, 45, K } ∩ { 2, 3, 5, 7} n( B ) ≤ n(C ), i.e. the number of elements ⇔ x ∈ P ( A) ∩ P ( B)
= { 2} in B is less than or equal to the number ∴ P ( A ∩ B) ⊆ P ( A) ∩ P ( B)
11. (c) Let n be the number of newspapers. of elements in C.
and P ( A ) ∩ P ( B ) ⊆ P ( A ∩ B )
Then, 60 × n = 300 × 5 Similarly, n(C ) ≤ n( B )
Hence, P ( A ) ∩ P ( B ) = P ( A ∩ B )
⇒ n = 25 Hence, n( B ) = n(C ),
Now, consider sets A = { 1}, B = { 2}
12. (a) Let E and M be the set of students i.e. B has as many elements as C.
passed in English and Mathematics, ⇒ A ∪ B = { 1, 2}
18. (c) Given, A = { x : x 2 − 3x + 2 = 0}
then n( E ) = 75, n( M ) = 60 ∴ P ( A ) = { φ,{ 1}}, P ( B ) = { φ, { 2}}
= { x : ( x − 1) ( x − 2) = 0}
and n ( E ∩ M ) = 45 and P ( A ∪ B )
= { 1, 2}
∴ Required number of students = { φ,{ 1},{ 2},{ 1, 2}}
and B = { x : x 2 + 2x − 8 = 0}
= n( E ) + n( M ) − 2 n ( E ∩ M ) ≠ P ( A) ∪ P ( B)
= { x : ( x + 4) ( x − 2) = 0}
= 75 + 60 − 90 Hence, Statement I is true but
= { 2, − 4}
= 45 Statement II is false.
∴ ( A − B ) = { 1, 2} − { 2,−4} = { 1}
MATHEMATICS Set Theory 11

23. (d) We have three categories, i.e. poets, 28. (c) The number of students learn 38. (c) Given that, A = { 1, 3, 5, 7}
learned and happy for which we can precisely two languages Here, n ( A) = 4
Venn diagram as follows. = x + 2x + 3x ∴ Number of elements on power set of A
Statement I represents = 6x = 6 × 5 = 30 = 2n ( A ) = 24 = 16
L 29. (c) The number of students learn atleast ∴ Cardinality of the power set
two language P ( A ) = 16
P
= x + 2x + 3x + 15 39. (d) We know that,
= 6x + 15 = 30 + 15 = 45 In set theory, according to ‘Distribution
30. (a) Total number of students in a class law’,
Thus, P ⊆ L
= 28 + 23 + 17 + 11 + x + 2x + 3x + 15 I. A ∪ ( B ∩ C ) = ( A ∪ B ) ∩ ( A ∪ C )
Statement II represents
= 79 + 6x + 15 = 94 + 30 = 124 II. A ∩ ( B ∪ C ) = ( A ∩ B ) ∪ ( A ∩ C )
H 31. (a) The number of students learn
L
So, both statements are incorrect.
English and French
40. (c) Given, n(D ) = 30, n( B ) = 40
= 15 + 3x = 15 + 15 = 30
As, n (D ∩ B ) = n(D ) + n( B ) − n(D ∪ B )
32. (c) The number of students learn atleast = 40 + 30 − n(D ∪ B )
Thus, L ⊆ H one language = 124 − 28 = 96 = 70 − n(D ∪ B )
On combining both statements, we get 33. (d) Here, A and B are any two sets and As total number of people is 50, therefore
P ⊆ L ⊆ H and the Venn diagram of U is the universal set. maximum value of n(D ∪ B ) is 50.
both statements taken together is given U Hence, minimum value of
below. n(D ∩ B ) = 70 − 50 = 20
A B
41. (b) Let S be the set of all even prime
numbers.
P L H ∴ S = { 2 } = non-empty set
Sol. (Q. Nos. 42-44)
A ∩ (A ∪ B)
P C U
24. (a) We have, B = B ∪ ( A ∩ B ) 34. (a) Given, 12 11
= B ∪ ( A ∩ C ) [Q A ∩ B = A ∩ C ] A = { 1, 4, 9, 16, 25, 36, 49, 64, 81} 2 1 5
and B = { 2, 4, 6, K } 6 3 2
= ( B ∪ A) ∩ ( B ∪ C )
= ( A ∪ C ) ∩ (B ∪ C ) Now, A ∩ B = {4, 16, 36, 64} 4
= ( A ∩ B) ∪ C ∴ Cardinality of ( A ∩ B ) 15
M
= (A ∩C) ∪C = C = Number of elements in ( A ∩ B ) = 4
Hence, only statement I is correct 35. (c) Given that, Given that,
Total number of surveyed students = 25
Sol. (Q. Nos. 25-27) Let P and C be the set of U = { x ∈ N : 1 ≤ x ≤ 10}
families having phone and car, Number of students, who have taken all
A = { 1, 2, 3, 4} and B = { 2, 3, 6, 10} three subjects = 3
respectively.
Now, A − B = { 1, 4} Number of students, who have taken
Given, n ( P ∩ C ) = 65%
∴ Complement of ( A − B ) = ( A − B )′ Physics and Chemistry = 4
⇒ n (U ) − n ( P ∪ C ) = 65%
= U − ( A − B ) = { 2, 3, 5, 6, 7, 8, 9, 10} Number of students, who have taken
⇒ n ( P ∪ C ) = 100 − 65 = 35% Mathematics and Physics = 9
36. (d) { 0}→ Singleton set and x 2 + 1 = 0
25. (a) Percentage of families having both Number of students who have taken
⇒ x 2 = −1
phone and car, Mathematics and Chemistry = 5
⇒ x is a complex number.
n( P ∩ C ) = n( P ) + n(C ) − n( P ∪ C ) Number of students, who have taken
While { x |x 2 + 1= 0, x ∈ R}. Chemistry = 11
= 25% + 15% − 35% = 5%
So, it is a null set. Number of students, who have taken
26. (c) Percentage of families having either Physics = 12
37. (b) Let U = { 1, 2, 3, 4, 5, 6, 7, 8, 9, 10}
phone or car, n( P ∪ C ) = 35%
A = {1, 2, 3, 4, 5}, B = {1, 2, 3, 4, 5, 6} Number of students, who have taken
27. (b) If total number of families is x, then Mathematics = 15
Clearly, A ⊆ B
5% of x = 2000 42. (a) The number of students, who have
2000 Now, AC = {6, 7, 8, 9, 10},
⇒ x = × 100 = 40000 C taken only Physics
5 B = {7, 8, 9, 10}
= 12 − ( 1 + 3 + 6)
Sol. (Q. Nos. 28-32) Given that, the total ⇒ BC ⊆ A C
= 12 − 10 = 2
number of students learning French = 46 Now, we have A ∩ B = {1, 2, 3, 4, 5}
43. (c) The number of students, who have
∴ 15 + 11 + x + 3x = 46 ⇒ A = ( A ∩ B) taken only two subjects = 6 + 2 + 1 = 9
⇒ 4x = 20 ⇒ x = 5 Hence, option (b) is correct.
12 NDA/NA Pathfinder

1000 
44. (b) ∴ A ∩ B = { 12, 24, 36, 48, K } Then, n( A ) =   = 100,
I. The number of students, who have ⇒ ( A ∩ B) \ C = ( A ∩ B) − C  10 
taken only one subject = { 12, 24, 36, K } − { 12, 24, 36, ...} 1000 
n( B ) =   = 66
= 2 + 5 + 4 = 11 which is a null set.  15 
and the number of students, who 1000 
have taken only two subjects 48. (a) We have, and n(C ) =   = 40
= 6 + 2 + 1= 9 2  25 
A = { x ∈ R : x + 6x − 7 < 0}
Since, 11 ≠ 9, therefore given = { x ∈ R : ( x + 7)( x − 1) < 0} Now, n( A ∩ B ) = n (set of numbers
statement is not correct. which are divisible by both 10 and 15)
= { x ∈ R : − 7 < x < 1}
II. The number of students, who have = n (set of numbers divisible by 30)
taken atleast two subjects and B = { x ∈ R : x 2 + 9x + 14 > 0}
1000 
= 1 + 2 + 6 + 3 = 12 = { x ∈ R : ( x + 7)( x + 2) > 0} =   = 33
 30 
The number of students, who have = { x ∈ R : x < − 7 and x > −2}
taken all three subjects = 3 Similarly, n( A ∩ C )
∴ A ∩ B = { x ∈ R : − 2 < x < 1}
Since, 12 = 4 × 3, therefore given = n (set of numbers divisible by 50)
= ( −2 , 1)
statement is correct. 1000 
and ( A \ B ) = A − B =   = 20
45. (b) Let the number of students in the  50 
= { x ∈ R : − 7 < x ≤ −2 }
class be x, who like both music and n( B ∩ C )
dancing. = ( −7 , − 2 ]
Hence, only Statement I is correct. = n (set of numbers divisible by 75)
5 1000 
49. (b) Since, A ∩ B = φ and C ∩ D = φ =   = 13
45–x x 50–x  75 
∴ ( A ∩ B ) ∩ (C ∩ D ) = φ
⇒ ( A ∩ C ) ∩ (B ∩ D) = φ and n( A ∩ B ∩ C )
Music Dancing Hence, A ∩ C and B ∩ D are always = n (set of numbers divisible by
disjoint. 10, 15 and 25)
∴ ( 45 − x ) + x + (50 − x ) + 5 = 60 = n (set of numbers divisible by 150)
But A ∪ C and B ∪ D may be or not
⇒ 100 − x = 60 1000 
disjoint. See example given below =   =6
⇒ x = 100 − 60 = 40  150 
46. (c) Required number of subsets of A A B ∴ n( A ∪ B ∪ C ) = n( A ) + n( B )
containing exactly two elements C D
+ n(C ) − n( A ∩ B ) − n( B ∩ C )
10 × 9
= 10 C = − n(C ∩ A ) + n( A ∩ B ∩ C )
2 Here, ( A ∩ C ) ∩ ( B ∪ D ) = φ
2
90 = 100 + 66 + 40 − 33 − 13 − 20 + 6
= = 45 But, ( A ∪ C ) ∩ ( B ∪ D ) ≠ φ
= 146
2 Hence, only Statement II is correct.
Hence, required numbers
47. (d) Here, A = {3, 6, 9, 12, 15, K } 50. (b) Let A, B and C be the set of natural = n(U ) − n( A ∪ B ∪ C )
B = { 4, 8, 12, 16, 20, K } numbers (≤ 1000) which are divisible by
= 1000 − 146 = 854
and C = { 12, 24, 36, 48, K } 10, 15 and 25 respectively.
02
MATHEMATICS Relations and Functions 13

RELATIONS AND
FUNCTIONS
In NDA exam, generally 4-7 questions are asked from this chapter which are based on inverse
relation, types of relation, equivalence relation, domain and range of function, composition of
functions and various standard functions.

Ordered Pair Properties of Cartesian Product


Two elements a and b listed in a particular order, (i) For any three sets A , B and C, we have
is called ordered pair and it is denoted by ( a, b), (a) A × ( B ∪ C ) = ( A × B) ∪ ( A × C )
where ‘a’ is the first element and ‘b’ is the second (b) A × ( B ∩ C ) = ( A × B) ∩ ( A × C )
element. (c) A × ( B − C ) = ( A × B) − ( A × C )
It is evident from the definition that (ii) If A and B are two non-empty sets, then
( a, b) ≠ ( b, a) ⇔ a ≠ b A × B= B× A ⇔A = B
Also, if ( a, b) = ( c, d ), then a = c and b = d . (iii) If A ⊆ B, then A × A ⊆ ( A × B) ∩ ( B × A )
(iv) If A ⊆ B, then A × C ⊆ B × C, for any set C
Cartesian Product of (v) If A ⊆ B and C ⊆ D, then A × C ⊆ B × D
Two Sets (vi) For any sets A , B, C and D,
Let A and B be any two non-empty sets. The set ( A × B) ∩ (C × D) = ( A ∩ C ) × ( B ∩ D)
of all ordered pairs ( a, b) such that a ∈ A and b ∈ B (vii) For any sets A and B,
is called the Cartesian product of the sets A and B ( A × B) ∩ ( B × A ) = ( A ∩ B) × ( B ∩ A )
and is denoted by A × B. (viii) For any three sets A , B and C,
Thus, A × B = {( a, b) : a ∈ A and b ∈ B} (a) A × ( B′ ∪ C ′ )′ = ( A × B) ∩ ( A × C )
Note • A = φ or B = φ, then we define A × B = φ. (b) A × ( B′ ∩ C ′ )′ = ( A × B) ∪ ( A × C )
• If A has n elements and B has m elements, then
(ix) Let A and B be two non-empty sets having n elements in
A × B has mn elements. common, then A × B and B × A have n 2 elements in
• If A ≠ B, then A × B ≠ B × A. common.
14 NDA/NA Pathfinder

EXAMPLE 1. Let A and B be two sets such that A × B


consists of 6 elements. If three elements of A × B are
Codomain of a Relation
(1, 4), (2, 6) and (3, 6), then If R is a relation from set A to the set B, then the set B
is called the codomain of the relation R. The range of
a. ( A × B) = (B × A) b. ( A × B) ≠ (B × A)
relation is always a subset of its codomain
c. A × B = {(1, 4), (1, 6), ( 2, 4)} d. None of these
EXAMPLE 2. If R is a relation from set A = {2, 4, 5} to
Sol. b. Since, (1, 4), (2, 6) and (3, 6) are elements of A × B, set B = {1, 2, 3, 4, 6, 8} defined by xRy ⇔ x divides y,
it follows that 1, 2, 3 are elements of A and 4, 6 are then the domain and the range of R are
elements of B. It is given that A × B has 6 elements. a. Dom ( R) = { 2}, Range ( R) = { 2, 4 , 6 }
So, A = {1, 2, 3} and B = {4, 6} b. Dom ( R) = { 2, 4 }, Range ( R) = { 2, 4 , 6 , 8 }
c. Dom ( R) = { 4 }, Range ( R) = { 2, 4 , 6 }
Hence, A × B = {1, 2, 3} × {4, 6}
d. None of the above
= {(1, 4), (1, 6), ( 2, 4), ( 2, 6), ( 3, 4), ( 3, 6)} Sol. b. As set A = {2, 4, 5} is related to set B = {1, 2, 3, 4, 6, 8}
and B × A = {4, 6} × {1, 2, 3} through a relation defined as xRy ⇔ x divides y.
Thus, clearly we have, 2R 2, 2R 4, 2R6, 2R8, 4R 4 and 4R8
= {( 4, 1), ( 4, 2), ( 4, 3), (6, 1), (6, 2), (6, 3)}
∴ R = {( 2, 2), ( 2, 4), ( 2, 6), ( 2, 8), ( 4, 4), ( 4, 8)}
Now, as we know, domain is the first component of the
RELATION ordered pair. Thus, Domain (R) = {2, 4} and range is the
second components of ordered pair.
Relation is a definite manner or pattern which show
Thus, Range (R) = {2, 4, 6, 8}
how a set belongs to another one. Let A and B be two
non-empty sets, then a relation R from A to B is a
subset of A × B. Thus, R is a relation from A to B ⇔ Inverse Relation
R ⊆ A × B. If R is a relation from a non-void set A to a Let A and B be non-void sets and R be the relation
non-void set B and ( a, b) ∈ R, then we write aRb which from a set A to a set B. Then, the inverse of R, denoted
is read as ‘a is related to b’ by the relation R. If by R−1 , is a relation from B to A and is defined by
( a, b) ∉ R, then we write aRb and say that ‘a is not R−1 = {( b, a) : ( a, b) ∈ R}
related to b’ by the relation R.
Clearly, ( a, b) ∈ R ⇔ ( b, a) ∈ R−1
Note If A and B are two non-empty sets consisting of p and q
elements respectively, then total number of relations from A
Also, Domain ( R−1 ) = Range ( R)
to B is 2 pq .
and Range ( R−1 ) = Domain ( R)

Domain of a Relation EXAMPLE 3. Let N be the set of natural numbers and


Let R be a relation from a set A to a set B. Then, the set let R = {(a, b):a ∈ N, b ∈ N and 2a + b = 10 }. Then, find
of all first components of the ordered pair belonging to i. Dom ( R) ii. Range ( R) iii. R −1 iv. Dom ( R −1) v. Range ( R −1)
R is called the domain of R. Sol. Clearly R ⊆ N × N. So, R is a binary relation on A.
Thus, Domain ( R) = { a : ( a, b) ∈ R} ∴ R = {(1, 8), ( 2, 6), ( 3, 4), ( 4, 2)}
It is evident from the definition that the domain of a (i) Dom (R) = {1, 2, 3, 4} (ii) Range (R) = {8, 6, 4, 2}
relation from A to B is a subset of A. (iii) R −1 = {(8, 1), (6, 2), ( 4, 3), ( 2, 4)}
e.g. If A = {1, 3, 5, 7}, B = { 2, 4, 6, 8, 10} (iv) Dom (R −1) = {8, 6, 4, 2} (v) Range (R −1) = {1, 2, 3, 4}
and R = {(1, 8), ( 3, 6), (5, 2), (1, 4)} be a relation from A to B.
Then, dom ( R) = {1, 3, 5} Types of Relations
1. Void relation Let A be a set. Then, φ ⊆ A × A and φ
Range of a Relation is a null set, so it is a relation on A. This relation is
If R is a relation from a set A to a set B, then the set of called the void or empty relation on set A.
all second components of the ordered pair of R is called 2. Universal relation Let A be a set. Then,
the range of R. A × A ⊆ A × A , so it is a relation on A. This relation
Thus, Range ( R) = { b : ( a, b) ∈ R} the range of relation is called the universal relation on set A.
from A to B is a subset of B. e.g. If A = { x, y}, then the universal relation on A is
the set {( x, x), ( x, y), ( y, x), ( y, y)}
e.g. If A = { a, b, c}, B = {d , e, f , g, h} and
Note The void and the universal relations on a set A are
R = {( a, d ), ( b, e ), ( c, f )}, then range of R = {d , e, f }
respectively the smallest and the largest relations on set A.
MATHEMATICS Relations and Functions 15

3. Identity relation Let A be a set. Then, the relation Let R be an equivalence relation on the set A. For each
I A = {( a, a) : a ∈ A } on A is called the identity relation a ∈ A, the equivalence class of a ‘denoted by [a]’ is
on A. defined as the set of all those points of A which are
e.g. If A = {1, 4, 5}, then identity relation, related to a under the relation R.
I A = {(1, 1), ( 4, 4), (5, 5)} Thus, [ a] = { x ∈ A : xRa}
In other words, a relation I A on A is called the Any two equivalence classes are either equal or disjoint
identity relation, if every element of A is related to and hence equivalence relation partitions the set A.
itself only.
4. Reflexive relation A relation R on a set A is said to EXAMPLE 4. Let A be the set of all real numbers.
be reflexive, if every element of A is related to itself. Then, the relation R = {(a, b): 1 + ab > 0 } on A is
Thus, R is reflexive ⇔ ( a, a) ∈ R , ∀ a ∈ A. a. reflexive and symmetric but not transitive
b. reflexive and transitive but not symmetric
A relation R on a set A is not reflexive if there exists
c. symmetric, transitive but not reflexive
an element a ∈ R, such that ( a, a) ∉ R.
d. None of the above
e.g. Let A = {1, 2, 3} be a set. Then
Sol. a. Given, R = {( a, b) : 1+ ab > 0}
R = {(1, 1), ( 2, 2), ( 3, 3), (1, 3)} is a reflexive relation on
A. But R1 = {(1, 1), ( 3, 3), (1, 3), ( 2,1)} not a reflexive Reflexivity As 1+ a ⋅ a = 1+ a2 > 0 for all a ∈ A
relation on A. ∴ ( a, a) ∈ R is reflexive.
Symmetric ( a, b) ∈ R ⇒ 1+ ab > 0 ⇒ 1+ ba > 0 ⇒ ( b, a) ∈ R
5. Symmetric relation A relation R on a set A is said
Thus, R is symmetric.
to be symmetric relation,
Transitivity ( a, b) ∈ R and ( b, c) ∈ R need not imply
iff ( a, b) ∈ R ⇒ ( b, a) ∈ R , ∀ a, b ∈ A ( a, c) ∈ R.
i.e. aRb ⇒ bRa, ∀ a, b ∈ A Hence, R is not transitive.
e.g. Let A = { a, b, c, d } be a set, then
R = {( a, c ), ( a, d ), (d , a), ( c, a), ( b, b)} is a symmetric
Properties of Relation
(i) If any set has n elements, then total number of
relation. 2
reflexive relation is 2n − n .
6. Transitive relation Let A be any set. A relation R (ii) The universal relation on a non-empty set is always
on A is said to be a transitive relation, reflexive, symmetric and transitive.
iff ( a, b) ∈ R and ( b, c ) ∈ R ⇒ ( a, c ) ∈ R, ∀ a, b, c ∈ A (iii) The identity relation on a non-empty set is always
i.e. aRb and bRc ⇒ aRc, ∀ a, b, c ∈ A reflexive, symmetric and transitive.
(iv) The identity relation on a non-empty set is always
e.g. If a relation R is defined by xRy ⇒ x is less than
anti-symmetric.
y for any x, y, z ∈ N, then R is transitive relation.
(v) If R and S are two equivalence relations on set A,
7. Anti-symmetric relation Let A be any set. A
then R ∩ S is also an equivalence relation on A.
relation R on set A is said to be an anti-symmetric
(vi) The inverse of an equivalence relation is an
relation,
equivalence relation.
iff ( a, b) ∈ R and ( b, a) ∈ R ⇒ a = b, ∀ a, b ∈ A
e.g. Let R be a relation on the set of natural numbers Composition of Relations
defined by xRy ⇔ x ′ divides y ′ for all x, y ∈ N, then If R ⊆ A × B and S ⊆ B × C are two relations. Then,
R is anti-symmetric relation. compositions of the relations R and S denoted by
8. Equivalence relation and equivalence classes A SoR ⊆ A × C and is defined by ( a, c ) ∈ SoR if and only if
relation R on a set A is said to be an equivalence there is an element b ∈ B such that ( a, b) ∈ R and ( b, c ) ∈ S.
relation on A, iff
e.g. Let A = {1, 2, 3}, B = { a, b, c, d }, C = {α, β, γ }
(i) It is reflexive, i.e. ( a, a) ∈ R, ∀ a ∈ A
R ⊆ A × B = {(1, a), (1, c ), ( 2, d )}
(ii) It is symmetric, i.e. ( a, b) ∈ R
S ⊆ B × C = {( a, α ), ( a, γ ), ( a, β )}
⇒ ( b, a) ∈ R, ∀ a, b ∈ A
Then, SoR ⊆ A × C = {(1, α ), (1, γ ), (1, β )}
(iii) It is transitive, i.e. ( a, b) ∈ R and ( b, c ) ∈ R
Note Remember that SoR ≠ RoS
⇒ ( a, c ) ∈ R , ∀ a, b, c ∈ A
Also, (SoR )−1 = R −1oS −1 is the reversal rule.
16 NDA/NA Pathfinder

EXAMPLE 5. If R is a relation from A = {1, 2, 3, 4} to EXAMPLE 6. Find the domain and range of the
B = {1, 3, 5}, i.e. (a, b) ∈ R ⇔ a < b, then RoR −1 is function f (x) =
x
.
a. {(1, 3), (1, 5), ( 2, 3), ( 2, 5), ( 3, 5), ( 4 , 5)} 1 + x2
b. {( 3, 1), ( 5, 1), ( 3, 2), ( 5, 2), ( 5, 3), ( 5, 4)}  1 1  1 1
a. R,  − ,  b. R − { 0 }, − ,
 2 2  2 2 
c. {( 3, 3), ( 3, 5), ( 5, 3), ( 5, 5)} d. {( 3, 3), ( 3, 4), ( 4 , 5)}
 1 1
c. R, − , − { 0 } d. None of these
Sol. c. We have, R = {(1, 3), (1, 5), ( 2, 3), ( 2, 5), ( 3, 5), ( 4, 5)}  2 2 
⇒ R −1 = {( 3, 1), ( 5, 1), ( 3, 2), ( 5, 2), ( 5, 3), ( 5, 4)} x
Sol. c. Let y = f ( x) = ⇒ 1 + x2 ≠ 0, ∀ x ∈ R
Hence, RoR −1 = {( 3, 3), ( 3, 5), ( 5, 3), ( 5, 5)} 1 + x2
So, y is defined for all real numbers.
FUNCTION ∴ Domain ( f ) = R
A function ‘ f ’ from a set to another set is a rule or Also, y=
x
method which associates every element of first set to a 1 + x2
unique element of second set. ⇒ y + x2 y = x
OR
⇒ x y − x+ y =0
2
If A and B are two non-empty sets, then function f is a
relation such that for every element x ∈ A their exist a 1± 1 − 4y 2
∴ x=
unique element y ∈ B, written as y = f ( x) = B. 2y
A function is also termed as ‘map’ or ‘mapping’. If an Since, x ∈R, we have y ≠ 0 and 1 − 4y 2 ≥ 0
element x ∈ A is associated to an element y ∈ B, then y is
Now, 1 − 4y 2 ≥ 0
called ‘image of x’ or the value of function f at x. If for 1 1 1
any value of x, there are more than one value of f ( x), ⇒ y2 ≤ ⇒ − ≤ y ≤
4 2 2
then f ( x) is not a function but a relation. It is denoted 1 1
∴ − ≤y≤ and y ≠ 0
by f : A → B or f : A  f
→ B which is read as “ f is a 2 2
∴ Range ( f ) = − ,  − {0}
function of A to B’’. 1 1
 2 2
Domain of a Function
If f : A → B is a function, then the set A is called the Types of Functions
domain of the function f and B is called the co-domain
1. One-one function (Injection) A mapping
of f. Thus, domain is the all possible values of x for
f : A → B is said to be one-one or one-to-one, if
which f ( x) exists.
different elements in A have different f-images in B.
Range of a Function A f B
Let f be a mapping from A to B, i.e. f : A → B. Each a a
element of A has a unique image and each element of B b b
g
need not appear as the image of an element in A. We c
d
define the range of f to consist of those elements in B
which appear as the image of atleast one element in A. i.e. f ( x1 ) = f ( x 2 )
Hence, range of a function of f : A → B, denoted by f ( A ). ⇒ x1 = x 2 , ∀ x1 , x 2 ∈ A
Thus, f ( A ) = { f ( x) : x ∈ A } ⇒ f ( A ) ⊆ B 2. Many-one function A mapping f : A → B is said
⇒ Range is the subset of codomain. to be many-one, if two (or more than two) distinct
elements in A have the same f-image in B.
e.g. Let A = { a, b, c, d } and B = {α, β, γ }
i.e. f ( x1 ) = f ( x 2 ) ⇒ x1 ≠ x 2 , ∀ x1 , x 2 ∈ A
f
A B f
a α A B
b β a a
c b b
γ g
d c
d d
Domain = { a, b, c, d } = A ; Codomain = {α, β, γ } = B
Range = {β, γ }
MATHEMATICS Relations and Functions 17

3. Into function If the mapping f : A → B is such 8. Many-one-into function If the mapping f : A → B


that there is atleast one element in B which is not the is such that there is at least one element in B which is
f-image of any element in A, then f is a mapping of A not the image of any element in A and B has more
‘into’ B. element to A, then it is called many one into function.
f
A f B
A B
a a a a
b b b b
c g c g
d d

4. Onto function (Surjection) If the mapping


 x , x ∈θ x, x ∉ θ
f : A → B is such that each element in B is the f-image EXAMPLE 7. If g(x) =  and f (x) = 
of atleast one element in A, then f is a mapping of 0 , x ∉ θ 0, x ∈θ
A ‘onto’ B. then (f − g) will be
f a. one-one onto b. one-one into
A B
a a c. many-one onto d. many-one into
b b x − 0 , x ∉ θ
g Sol. a. Let h( x) = f ( x) − g( x) = 
0 − x , x ∈ θ
c
d d
 x , x ∉θ
i.e. h( x) =  ⇒ h ( x) is one-one and onto.
5. One-one-onto function (Bijective) If the mapping − x , x ∈ θ
f : A → B is such that each element in the co-domain
B is mapped to by exactly one element of the domain Some Important Functions
A i.e. the function is both one-one and onto function There are following some important functions come under
well as onto function, then it is called bijective this chapter.
functions. 1. Constant function A function f : R → R defined by
f
A B f ( x) = c, ∀ x ∈ R that assigns the same value to every
a a member of its domain, where c is a constant, is called
b b a constant function. The domain and range of this
g
function are R and { c }, respectively.
c
2. Identity function A function f : R → R defined by
6. One-one-into function If the mapping f : A → B f ( x) = x, ∀ x ∈ R
is such that different elements in A have different is called identity function. Here, domain and range
f-images in B and one or more than one element in both are equal to R.
the codomain B is not an image of any element in the 3. Reciprocal function The function defined by
domain A, then it is called one-one into function. 1
f ( x) = , ∀ x ∈ R, x ≠ 0
f x
A B
a a
is called reciprocal function. The domain and range
b b of this function are equal to R − {0}.
g
c d
4. Rational function When any function is written as
the ratio of two algebraic expressions, it is called a
7. Many-one-onto function If the mapping f : A → B rational function.
is such that two or more than two elements in A x 2 − 3x + 5
e.g. f ( x) = 3
have the same image in B and B has the less element x − 2x 2 + x − 1
to A, then it is called many one onto function.
5. Absolute or modulus function
f Y
A B A function defined by f(x
)= x
)=
a a  x, if x ≥ 0 −x f (x
b f ( x) = | x | = 
 − x, if x < 0
b
c
X′ O X
Y′
18 NDA/NA Pathfinder

is called absolute or modulus function. Domain 10. Trigonometric function The function involving
and range of absolute function are respectively R and circular functions of variable angles are known as
[0, ∞ ). trigonometric function.
Properties of modulus function
Function Domain Range
(i) | x | ≤ a ⇔ − a ≤ x ≤ a (ii) | x | ≥ a ⇔ x ≤ − a or
sin x R [−1, 1]
x≥ a
cos x R [−1, 1]
(iii) | x ± y | ≤ | x | + | y | (iv) | x ± y | > || x | − | y | |
π
6. Greatest integer function (Floor function) The tanx R − (2 n + 1) : n ∈ I

R
 2 
function f ( x) = [ x] is called the greatest integer
cot x R − {nπ : n ∈ I} R
function and means greatest integer less than or
π
equal to x i.e. [ x] ≤ x. sec x R − (2 n + 1) : n ∈ I (−∞,−1] ∪ [1, ∞ )
e.g. [ 3.12] = 3, [ −3.12] = − 4, [ −3] = − 3, [ 3] = 3,  2 
Its domain is R and range is I. cosec x R − {nπ : n ∈ I } (−∞,−1] ∪ [1, ∞ )
Y
11. Inverse trigonometric function The function
3 involving inverse trigonometric ratios are known as
2 inverse trigonometric functions.
1
Function Domain Range
X′ −3 −2 −1 1 2 3 X − π , π 
−1 sin−1 x [−1, 1]  2 2 
−2
−3 cos −1 x [−1, 1] [0, π ]
− π, π
Y′ tan−1 x R 
 2 2

Properties of greatest integer function
cot −1 x R (0, π )
(i) [ x + n] = [ x] + n, where x ∈ R and n ∈ I
π
 − [ x], if x ∈ I sec −1 x (−∞,−1] ∪ [1, ∞ ) [0, π ] −  
(ii) [ − x] =  2 
 − [ x] − 1, if x ∉ I  − π , π  − {0}
cosec −1 x (−∞,−1] ∪ [1, ∞ )  2 2 
0 , if x ∈ I
(iii) [ x] + [ − x] = 
 −1, if x ∉ I 12. Periodic function If a function f ( x) satisfies the
(iv) n 1 ≤ [ x] ≤ n2 ⇒ n 1 ≤ x < n2 + 1 condition f ( x + T ) = f ( x), ∀ x ∈ X , where T is the
7. Smallest integer function For any real number x, smallest positive real number for which the equality
we use the symbol [ x] to denote the smallest holds, then f ( x) is called a periodic function and T is
integer function greater than or equal to x.
called the period of the periodic function.
e.g. [9.7 ] = 10, [ − 5.78] = − 5
e.g. sin x is a periodic function with period 2π.
This is also known as ceiling function.
[Qsin x = sin ( 2π + x)]
8. Exponential function Let a ( ≠ 1) be a positive real 13. Signum function The function defined by
number. Then the function f : R → R defined by  −1, x < 0
| x| 
f ( x) = a x is called the exponential function. f ( x) = =  0, x = 0 is called the signum function.
x 
Domain of f = R  1, x > 0
Y
Range of f = (0, ∞ ) y=1
1
9. Logarithmic function Let a ( ≠ 1) be a positive real x>0

number. Then the function f : (0, ∞ ) → R defined by X′ O X


f ( x) = log a x is called the logarithmic function. y=−1
−1
Domain of f = (0, ∞ ) x<0
Y′
Range of f = R Domain = R, Range = { −1, 0, 1}
MATHEMATICS Relations and Functions 19

EXAMPLE 8. The domain of the function


1 Properties of Function
f (x ) = is (i) If A and B have n and m distinct elements
[x ] − [x ] − 6
2
respectively, then the number of mappings from A
a. ( −∞ , −2) ∪ [ 4 , ∞) b. ( −∞ , − 2] ∪ [ 4 , ∞) to B is equal to mn .
c. ( −∞ , − 2) ∪ ( 4 , ∞) d. None of these (ii) If A and B have n equal number of distinct
Sol. a. f ( x) is defined, if [ x ]2 − [ x ] − 6 > 0 elements, then the number of bijective functions
⇒ ([ x ] − 3]([ x ] + 2) > 0, [ x ] < −2 or [ x ] > 3 from A to B is equal to n!. But if A and B don’t
But [ x ] < −2 ⇒ [ x ] = −3, − 4, − 5, ...
have equal number of elements, then number of
bijective functions from A to B is 0.
Also, [ x ] > 3 ⇒ [ x ] = 4, 5, 6,...
∴ x≥4 (iii) The number of one-one functions that can be
Domain of f = ( −∞ ,−2) ∪ [ 4, ∞) defined from a finite set A into a finite set B is
n (B)
Pn (A ) , if n ( B) ≥ n ( A ) and 0 otherwise.
Inverse Function (iv) If A and B have n equal number of distinct
Let f : A → B be a one-one onto (bijection) mapping. elements, then number of onto functions from A to
Then, the mapping, f −1 : B → A, which associates to each B is equal to 2n − 2 .
element b ∈ B the element a ∈ A, such that f ( a) = b, is
called the inverse mapping of the mapping f : A → B.
Composition of Functions
A B
f Let A, B and C be three non-empty sets.
–1
Let f : A → B and g : B → C be two mappings or
a = f (b) b = f (a)
functions. Then, the composite of the functions f and
–1 g denoted by ( gof ), is a mapping of A → C, given by
f
( gof ) : A → C
Method to Find Inverse of Function such that ( gof ) ( x) = g { f ( x)}, ∀ x ∈ A
A B C
Let f : A → B be a bijective function. f g
(1) Put f ( x) = y.
x y = f(x) z = g{f(x)}
(2) Solve the equation y = f ( x) to obtain x is terms of y.
Interchange x and y to obtain the inverse of f.
gof
EXAMPLE 9. The inverse of the function gof exists iff the range of f is a subset of domain of g.
10 x − 10 − x Similarly, fog exists iff range of g is a subset of
y= x + 1 is
10 + 10 − x domain of y.
 x  1  x 
a. y = log10  b. y = log10  1+x
EXAMPLE 10. If f (x) = log
 
 2 − x 2  2 − x  and
1−x
1  x 
c. y = log10   d. None of these (3x + x 3 )
2  1− x g(x) = , then what is f [fg(x)] equal to?
1 + 3x 2
10x − 10−x 10x − 10− x
Sol. b. y = −x + 1 ⇒ y − 1 = a. −f ( x) b. 3[f ( x)] c. [f ( x)]3 d. − 3[f ( x)]
10 + 10
x
10x + 10− x
1+ x  ( 3x + x 3)
Using componendo and dividendo Sol. b. Given, f ( x) = log   and g( x) =
y − 1+ 1 102x − 1+ 102x + 1  1− x  1+ 3x 2
=
y − 1− 1 102x − 1− 102x − 1  1+ g( x)   1+ 3x 2 + 3x + x 3 
Now, f [ g( x)] = log   = log  
y 2(102x ) y  1− g( x)   1+ 3x 2 − 3x − x 3 
= = 102x
y−2 −2 2 − y
1+ x  3
 y  1  y  −1 = log  
2x = log10   ⇒ x = log10   = f ( y)  1− x 
 2 − y 2  2 − y
1+ x 
Hence, the inverse of the given functions is = 3log  
 1− x 
y = log10 
1 x 

2  2 − x = 3[ f ( x)]
20 NDA/NA Pathfinder

PRACTICE EXERCISE
1. If A = { 1, 2, 5, 6} and B = { 1, 2, 3}, then what is 12. The function f( x ) = log( x + x 2 + 1 ) is
( A × B) ∩ ( B × A) equal to? (a) an even function (b) an odd function
(a) {(1, 1), (2, 1), (6, 1), (3, 2 )} (b) {(1, 1), (1, 2 ), (2, 1), (2, 2 )} (c) periodic function (d) None of these
(c) {(1, 1), (2, 2 )} (d) {(1, 1), (1, 2 ), (2, 5), (2, 6)}
13. If A = { a , b, c} and R = {( a , a ),( a , b),( b, c),( b, b),( c, c),
2. Which one of the following is correct? ( c, a )} is a binary relation on A, then which one
(a) A × (B − C ) = ( A − B) × ( A − C ) of the following is correct?
(b) A × (B − C ) = ( A × B) − ( A × C ) (a) R is reflexive and symmetric, but not transitive
(c) A ∩ (B ∪ C ) = ( A ∩ B) ∪ C (b) R is reflexive and transitive, but not symmetric
(d) A ∪ (B ∩ C ) = ( A ∪ B) ∩ C (c) R is reflexive, but neither symmetric nor transitive
3. Let R = { x| x ∈ N , x is a multiple of 3 and x ≤ 100} (d) R is reflexive, symmetric and transitive
S = { x| x ∈ N , x is a multiple of 5 and x ≤ 100}. 14. The values of b and c for which the identity
What is the number of elements in f ( x + 1) − f ( x ) = 8x + 3 is satisfied, where
( R × S ) ∩ (S × R )? f ( x ) = bx 2 + cx + d , are
(a) 36 (b) 33 (c) 20 (d) 6 (a) b = 2, c = 1 (b) b = 4, c = − 1
4. If φ ( x ) = a , then [φ ( p)] is equal to
x 3 (c) b = − 1, c = 4 (d) None of these

(a) φ (3p) (ax + a− x )


(b) 3φ ( p) (c) 6φ ( p) (d) 2φ ( p) 15. If the function f ( x ) = (where, a > 2),
2
5. If f ( x ) = x 2 − x −2 , then f   is equal to
1
then f ( x + y ) + f ( x − y ) is equal to
 x f( x ) f( y)
1 (a) 2 f(x) ⋅ f( y) (b) f(x) ⋅ f( y) (c) (d)
(a) f(x) (b) − f(x) (c) (d) [f(x)]2 f( y) f( x )
f(x)
16. Let f be a function with domain [− 3, 5] and let
1+ x f (x) ⋅ f (x2 )
6. If f ( x ) = , then is equal to g( x ) =|3x + 4|, then the domain of fog ( x ) is
1− x 1 + [ f ( x )]2
(a)  − 3,  (b)  − 3, 
1 1
1 1 1 1  3  3 
(a) (b) (c) (d)
4 6 8 2
(c)  − 3, 
1
(d) None of these
 3
7. Which one of the following functions, f : R → R is
injective? αx
17. Let f( x ) = , x ≠ −1. Then, for what value of α
(a) f(x) = | x|, ∀ x ∈ R (b) f(x) = x , ∀ x ∈ R
2
x+1
(c) f(x) = 11, ∀ x ∈ R (d) f(x) = − x, ∀ x ∈ R is f [ f ( x )] = x?
(a) 2 (b) − 2
8. The domain of the function f ( x ) = x − 1 + 6 − x
(c) 1 (d) −1
is
(a) [1, ∞ ) (b) (− ∞, 6) 18. The inverse of the function f ( x ) = loga ( x + x 2 + 1 )
(c) [1, 6] (d) None of these (where, a < 0, a ≠ 1) is
1 x
9. The period of the function f( x ) =|sin x| +|cos x| is (a) ( a − a− x ) (b) not defined for all x
2
(a) π / 2 (b) π (c) 2 π (d) π / 4 (c) defined for x > 0 (d) None of these
10. The domain of the function 19. If f( x ) = 3x + 10 and g( x ) = x 2 − 1, then ( fog)−1 is
1
f( x ) = + ( x + 2) is equal to
log10(1 − x ) x − 7
1/ 2
x + 7
1/ 2
x−
1/ 2
x + 3
1/ 2
(a)  (b)  (c) 
3
   (d) 
(a) ] − 3, − 2.5 [ ∪ ] − 2.5, − 2 [ (b) [−2,0 [ ∪ ] 01
,[  3   3   7   7 
(c) ] 0,1[ (d) None of these
1 20. Let f ( x ) = ( − 1)[ x ] (where [⋅] denotes the greatest
11. The range of the function f( x ) = is integer function), then
( 2 − sin 3x )
(a) range of f is {− 1, 1} (b) f is an even function
(a)  , 1  (b)  , 1 (c)  , 1  (d) 1,
1 1 1 1
 3   3   3   (c) f is an odd function (d) f is one-one function
3 
MATHEMATICS Relations and Functions 21

21. The function f : R → R defined by f ( x ) = 4x + 4|x| is 29. If g( x ) = loge x 2, then range of the function g[ f ( x )]
(a) one-one and into (b) many-one and into is
(a)  − ∞, loge
11
(b) loge , ∞ 
(c) one-one and onto (d) many-one and onto 11
 3   3 
22. If f ( x ) satisfies the relation 2 f ( x ) + f (1 − x ) = x 2
(c)  − loge , loge 
11 11
(d) None of these
 3 
for all real x, then f ( x ) is 3
x 2 + 2x − 1 x 2 + 2x − 1
(a) (b)
6 3 Directions (Q. Nos. 30-31) The following functions
x 2 + 4x − 1 x 2 + 4x − 1 are defined for the set of variables x1, x2, K, xn
(c) (d)
3 6
 xi + j , if i + j ≤ n2
23. For real numbers x and y, define a relation R, f ( xi , xj ) =  and g ( xi , xj ) = xm
xi + j − n , if i + j > n
2
xRy if only if x − y + 2 is an irrational number.
where, m is the remainder when i × j is divided by n.
Then the relation R is
(a) reflexive (b) symmetric 30. Find the value of f [ f ( x2 , x3 ), f ( x5 , x6 )], if n = 3.
(c) transitive (d) an equivalence relation
(a) x5 (b) x10 (c) x13 (d) x8
24. Let A = {2, 3, 4, 5, …, 16, 17 18} and ‘*’ be the 31. Find the value of g [g( x2 , x3 ), g( x7 , x8 )], if n = 5.
equivalence relation on A × A defined by
( a , b) * ( c, d ) if ad = bc. Then, the number of (a) x1 (b) x2 (c) x5 (d) All of these
ordered pairs of the equivalence class of ( 3, 2) is
[x ]
(a) 5 (b) 6 (c) 7 (d) 8 Directions (Q. Nos. 32-33) Consider f ( x ) = and
x
25. Consider the following with regard to a relation g( x ) = | x | , where [⋅] denotes the greatest integer
R on a set of real numbers defined by xRy if and function.
only if 3x + 4 y = 5
1 2 3 32. What is the value of fog( −2 / 3) − gof ( −2 / 3)?
I. 0 R 1 II. 1 R III. R
2 3 4 (a) 1 (b) −1 (c) 0 (d) 2
Which of the above statement(s) is/are correct? 33. What is the value of fof ( −7 / 4) + gog ( −1)?
(a) I and II (b) I and III (c) II and III (d) I, II and III 1 1
−x
(a) 0 (b) −1 (c) (d) −
26. The function f : R → R is defined by f ( x ) = 3 4 8
I. f is one-one function.
II. f is onto function.
PREVIOUS YEARS’ QUESTIONS
III. f is a decreasing function.
Which of the above statement(s) is/are correct? 34. If f ( xy ) = f ( x ) f ( y ), then f ( t ) may be of the form
e 2012 I
(a) I and II (b) II and III (c) I and III (d) All of these
(a) t + k (b) ct + k (c) t k + c (d) t k

27. The domain of the given function where k is a constant


f ( x ) = log2 sin x is
35. Let A = { x ∈ W, the set of whole numbers and
π 2π
I. (4n + 1) II. (4n + 1) x < 3}, B = { x ∈ N , the set of natural numbers and
3 3 2 ≤ x < 4} and C = { 3, 4}, then how many elements
π
III. (4n + 1) where n ∈ N . will ( A ∪ B) × C contain? e 2012 II
2 (a) 6 (b) 8 (c) 10 (d) 12
Choose the correct option using the code given 36. Let P = {1, 2, 3} and a relation on set P is given
by the set R = {(1, 2), (1, 3), (2, 1) (1, 1) (2, 2),
below.
(a) Only I (b) Only II (3, 3), (2, 3)}. Then, R is e 2012 II
(c) Only III (d) I and II (a) reflexive, transitive but not symmetric
(b) symmetric, transitive but not reflexive
Directions (Q. Nos. 28-29) Consider the function (c) symmetric, reflexive but not transitive
f ( x ) = 3x 2 − 4x + 5 . (d) None of the above
37. The relation ‘has the same father as’ over the set
28. The domain of function f ( x ) is of children is e 2012 II
(d)  , ∞ 
2 (a) only reflexive (b) only symmetric
(a) R (b) (− ∞, 1) (c) (1, ∞ )
3  (c) only transitive (d) an equivalence relation
22 NDA/NA Pathfinder

x+5 49. If f be a function from the set of natural


38. If f : R → R be a function whose inverse is ,
3 numbers to the set of even natural numbers
then what is the value of f( x )? e 2012 II given by f( x ) = 2x. Then, f is e 2013 II
(a) f (x) = 3x + 5 (b) f (x) = 3x − 5 (a) one-one but not onto (b) onto but not one-one
(c) f (x) = 5x − 3 (d) Does not exist (c) Both one-one and onto (d) Neither one-one nor onto
39. If A = { x ∈ R|x ≥ 0} and a function f : A → A is 50. Let X be the set of all citizens of India. Elements
defined by f( x ) = x 2, then which one of the x, y in X are said to be related, if the difference
following is correct? e 2012 II
of their age is 5 yr. Which one of the following is
(a) The functions does not have inverse correct? e 2014 I
(b) f is its own inverse (a) The relation is an equivalence relation on X
(c) The functions has an inverse but is not its own inverse (b) The relation is symmetric but neither reflexive nor
transitive
(d) None of the above
(c) The relation is reflexive but neither symmetric nor
40. Consider the following statements transitive
I. If f (x) = x3 and g ( y) = y3 , then f = g. (d) None of the above
II. Identity function is not always a bijection. 51. Let S denote set of all integers. Define a relation
Which of the above statement(s) is/are correct? R on S as ‘aRb if ab ≥ 0, where a , b ∈ S . Then, R
is e 2014 I
(a) Only I (b) Only II e 2012 II
(a) reflexive but neither symmetric nor transitive relation
(c) Both I and II (d) Neither I nor II
(b) reflexive, symmetric but not transitive relation
41. If A = { x , y }, B = { 2, 3} , C = { 3, 4}, then what is the (c) an equivalence relation
number of elements in A × ( B ∪ C )? e 2013 I (d) symmetric but neither reflexive nor transitive relation
(a) 2 (b) 4 (c) 6 (d) 8
52. Consider the following relations from A to B,
42. If A is a relation on a set R, then which one of where
the following is correct? e 2013 I
A = {u, v, w, x, y, z} and B = {p, q, r, s}.
(a) R ⊆ A (b) A ⊆ R (c) A ⊆ (R × R ) (d) R ⊆ ( A × A)
I. {(u, p), (v, p), (w, p), (x, q), (y, q), (z, q)}
43. Let N be the set of natural numbers and II. {(u, p), (v, q), (w, r), (z, s)}
f : N → N be a function given by f ( x ) = x + 1 for
III. {(u, (v, r), (w, q), (u, p), (v, q), (z, q)}
x ∈ N . Which one of the following is correct?
s),
e 2013 I IV. {(u, q), (v, p), (w, s), (x, r), (y, q), (z, s)}
(a) f is one-one and onto (b) f is one-one but not onto Which of the above relations are not functions?
(c) f is only onto (d) f is neither one-one nor onto e 2014 I
(a) I and II (b) I and IV (c) II and III (d) III and IV
44. What is the range of the function
|x| 53. Let N denote the set of all non-negative integers
f(x) = ,x ≠ 0?
x e 2013 I and Z denote the set of all integers. The function
(a) Set of all real numbers (b) Set of all integers f : Z → N given by f( x ) =| x | is e 2014 I
(c) {−1, 1} (d) {−1 , 0, 1} (a) one-one but not onto (b) onto but not one-one
(c) Both one-one and onto (d) Neither one-one nor onto
45. If A = { 1, 2}, B = { 2, 3} and C = { 3, 4}, then what is
the cardinality of ( A × B) ∩ ( A × C )? e 2013 II 54. A and B are two sets having 3 elements in
(a) 8 (b) 6 (c) 2 (d) 1 common. If n( A) = 5 and n( B) = 4, then what is
n( A × B) equal to? e 2014 II
46. If A is a finite set having n elements, then the (a) 0 (b) 9 (c) 15 (d) 20
number of relations which can be defined in A is
55. If f ( x ) = ax + b and g( x ) = cx + d such that
e 2013 II
(a) 2 n
(b) n2
(c) 2 n2
(d) nn f [g( x )] = g[ f ( x )], then which one of the following
is correct? e 2014 II
47. Let A = { a , b, c, d } and B = { x , y , z }. What is the (a) f(c ) = g (a) (b) f(a) = g (c ) (c) f(c ) = g (d ) (d) f(d ) = g (b )
number of elements in A × B ? e 2013 II 56. The function f : N → N , N being the set of
(a) 6 (b) 7 natural numbers, defined by f ( x ) = 2x + 3 is
(c) 12 (d) 64 e 2014 II
48. The relation R in the set Z of integers given by (a) injective and surjective
R = {( a , b) : a − b is divisible by 5} is e 2013 II (b) injective but not surjective
(a) reflexive (b) reflexive but not symmetric (c) not injective but surjective
(c) symmetric and transitive (d) an equivalence relation (d) neither injective nor surjective
MATHEMATICS Relations and Functions 23

57. The relation S is defined on the set of integers Z 65. Consider the following functions
as xSy, if integer x divides integer y. Then I. f (x) = x3 , x ∈ R
(a) S is an equivalence relation e 2014 II II. f (x) = sin x, 0 < x < 2π
(b) S is only reflexive and symmetric
III. f (x) = ex , x ∈ R
(c) S is only reflexive and transitive
(d) S is only symmetric and transitive Which of the above functions have inverse defined
on their ranges? e 2015 I
(a) I and II (b) II and III
Directions (Q. Nos. 58-60) Read the following
(c) I and III (d) I, II and III
information carefully and answer these questions
given below. 66. Let Z be the set of integers and aRb, where a,
x −1 b ∈ Z if and only if ( a − b) is divisible by 5.
Consider the function f ( x ) = .
x +1 e 2014 II Consider the following statements
f(x) + 1
58. What is + x equal to? I. The relation R partitions Z into five equivalent
f(x) − 1 classes.
(a) 0 (b) 1 (c) 2x (d) 4x II. Any two equivalent classes are either equal or
disjoint.
59. What is f ( 2x ) equal to?
Which of the above statement(s) is/are correct?
f( x ) + 1 f( x ) + 1 3f(x ) + 1 f( x ) + 3
(a) (b) (c) (d) e 2015 I
f( x ) + 3 3f(x ) + 1 f( x ) + 3 3f(x ) + 1
(a) Only I (b) Only II
60. What is f [ f ( x )] equal to? (c) Both I and II (d) Neither I nor II
(b) −x 1
(a) x 67. The domain of the function f ( x ) = is
1 |x|– x
(c) − (d) None of these
x e 2015 II
61. Let A = { x , y , z } and B = { p, q , r , s}, what is the (a) [0, ∞ ) (b) (− ∞, 0) (c) [1, ∞ ) (d) (− ∞, 0]
number of distinct relations from B to A? e 2015 I 68. If f : R → R , g : R → R are two functions given by
(a) 4096 (b) 4094 (c) 128 (d) 126 f ( x ) = 2x − 3 and g( x ) = x3 + 5, then ( fog)−1( x ) is
62. Let X be the set of all persons living in a city. equal to e 2015 II
Persons x, y in X are said to be related as x < y, x + 7
1/ 3
x − 7
1/ 3

if y is atleast 5 yr older than x. Which one of the (a)   (b)  


 3   2 
following is correct? e 2015 I 1/ 3 1/ 3
(c)  x −  (d)  x + 
7 7
(a) The relation is an equivalence relation on X
 2  2
(b) The relation is transitive but neither reflexive nor
symmetric 69. Let X be the set of all persons living in Delhi.
(c) The relation is reflexive but neither transitive nor The persons a and b in X are said to be related,
symmetric if the difference in their ages is atmost 5 yr. The
(d) The relation is symmetric but neither transitive nor relation is e 2015 II
reflexive (a) an equivalence relation
x (b) reflexive and transitive but not symmetric
63. For each non-zero real number x, let f ( x ) = .
|x| (c) symmetric and transitive but not reflexive
The range of f is e 2015 I (d) reflexive and symmetric but not transitive
(a) a null set 1
(b) a set consisting of only one element 70. If g( x ) = and f ( x ) = x , x ≠ 0, then which one of
f(x)
(c) a set consisting of two elements
the following is correct? e 2015 II
(d) a set consisting of infinitely many elements
(a) f(f(f(g (g (f(x )))))) = g (g (f(g (f(x )))))
 1 + x 3x + x3 (b) f(f(g (g (g (f(x )))))) = g (g (f(g (f(x )))))
64. If f ( x ) = loge   , g( x ) = and
 1 − x 1 + 3x 2 (c) f(g (f(g (g (f(g (x ))))))) = g (g (f(g (f(x )))))
 e − 1 (d) f(f(f(g (g (f(x )))))) = f(f(f(g (f(x )))))
gof ( t ) = g ( f ( t )), then what is gof   equal to?
 e + 1 e 2015 I
71. f ( xy ) = f ( x ) + f ( y ) is true for all e 2015 II
(a) 2 (b) 1
1 (a) polynomial functions f (b) trigonometric functions f
(c) 0 (d)
2 (c) exponential functions f (d) logarithmic functions f
24 NDA/NA Pathfinder

72. Consider the following statements 76. Let R be a relation on the set N of natural
Statement I The function f : R → R such that numbers defined by ‘nRm ⇔ n is a factor of m’.
Then, which one of the following is correct?
f ( x ) = x3 for all x ∈ R is one-one.
e 2016 I
Statement II f ( a ) = f ( b) ⇒ a = b for all a , b ∈ R , if (a) R is reflexive, symmetric but not transitive
the function f is one-one. (b) R is transitive, symmetric but not reflexive
(c) R is reflexive, transitive but not symmetric
Which one of the following is correct in respect of
(d) R is an equivalence relation
the above statements? e 2015 II
(a) Both the statements are true and Statement II is the Directions (Q. Nos. 77-78) Let f ( x ) be the greatest
correct explanation of Statement I integer function and g( x ) be the modulus function.
(b) Both the statements are true and Statement II is not e 2016 I
the correct explanation of Statement I
77. What is ( gof )  −  − ( fog )  −  equal to?
5 5
(c) Statement I is true but Statement II is false
 3  3
(d) Statement I is false but Statement II is true
(a) −1 (b) 0
73. Suppose there is a relation * between the (c) 1 (d) 2
positive numbers x and y given by x * y if and
only if x ≤ y 2. Then which one of the following is 78. What is ( fof )  −  + ( gog ) ( −2) equal to?
9
correct? e 2016 I  5
(a) * is reflexive but not transitive and symmetric (a) −1 (b) 0
(b) * is transitive but not reflexive and symmetric (c) 1 (d) 2
(c) * is symmetric and reflexive but not transitive
(d) * is symmetric but not reflexive and transitive Directions (Q. Nos. 79-80) Consider the function
x −x  27( x 2/3 − x )
74. If f ( x1 ) − f ( x2 ) = f  1 2  for x1 , x2 ∈ ( −1, 1), then f (x ) = .
 1 − x1x2  4 e 2016 I
what is f ( x ) equal to? e 2016 I 79. How many solutions does the function f ( x ) = 1
1− x  2 + x 
(a) ln   (b) ln   have?
1 + x   1− x  (a) One (b) Two
1− x  1 + x  (c) Three (d) Four
(c) tan−1   (d) tan−1  
1 + x  1− x 
80. How many solutions does the function f ( x ) = − 1
x2 have?
75. What is the range of the function y =
1 + x2 (a) One (b) Two
(c) Three (d) Four
where x ∈ R ? e 2016 I
(a) [0, 1) (b) [0, 1] (c) (0, 1) (d) (0, 1]

ANSWERS
1 b 2 b 3 a 4 a 5 b 6 d 7 d 8 c 9 a 10 b
11 b 12 b 13 c 14 b 15 a 16 b 17 d 18 a 19 a 20 a
21 a 22 b 23 a 24 b 25 c 26 c 27 c 28 a 29 b 30 b
31 a 32 c 33 d 34 d 35 b 36 a 37 d 38 b 39 c 40 a
41 c 42 c 43 b 44 c 45 c 46 c 47 c 48 d 49 c 50 b
51 c 52 c 53 b 54 d 55 d 56 b 57 c 58 a 59 c 60 c
61 a 62 b 63 c 64 b 65 c 66 c 67 b 68 b 69 d 70 b
71 d 72 a 73 a 74 a 75 a 76 c 77 c 78 b 79 b 80 a
MATHEMATICS Relations and Functions 25

HINTS AND SOLUTIONS 


1. (b) Given, A = { 1, 2, 5, 6} ( 2 y − 1) y − 7
1/2
⇒ sin 3x = ⇒ x =  
and B = { 1, 2, 3} y  3 
A × B = {( 1, 1), (1, 2), (1, 3), (2, 1), Since, −1 ≤ sin 3x ≤ 1 x − 7
1/2
(2, 2), (2, 3), (5, 1), (5, 2), (5, 3), ( 2 y − 1) So, ( fog )−1 =  
We have, −1 ≤ ≤1 …(i)  3 
(6, 1), (6, 2), ( 6, 3)} y
B × A = {( 1, 1), (1, 2), (1, 5), (1, 6), Since, y > 0 multiplying the inequality 20. (a) f ( x ) = ( −1)[x ] = { −1, 1}, since [ x ] ∈Z
(2, 1), (2, 2), (2, 5), (2, 6), (3, 1), (3, 2), Eq. (i) by y, we obtain 21. (a) Since, for different values of x, 4x
(3, 5), (3, 6)}
− y ≤ 2 y − 1 ≤ y or 1 ≤ 3 y and y ≤ 1 and 4|x| are different positive numbers.
∴ ( A × B) ∩ ( B × A)
1
⇒ ≤ y≤1 ∴ f is one-one.
= {( 1, 1), (1, 2), (2, 1), ( 2, 2)}
3 Also, f is not onto as its range is (0, ∞)
2. (b)
12. (b) f ( − x ) = log[ − x + 1+ x2 ] and it is subset of its codomain R.
3. (a) Q R = {3, 6, 9, 12, 15, K , 99}
f ( x ) + f ( − x ) = log[ x + 1+ x2 ] 22. (b) Given, 2 f ( x ) + f ( 1 − x ) = x 2 …(i)
and S = {5, 10, 15, K , 95, 100}
Now, ( R × S ) ∩ ( S × R) Replacing x by ( 1 − x ), we get
+ log[ − x + 1+ x2 ]
2 f ( 1 − x ) + f ( x ) = ( 1 − x )2
= ( R ∩ S ) × ( S ∩ R)
= log [ 1 + x 2 − x 2 ] = log 1 = 0 ⇒ 2 f ( 1 − x ) + f ( x ) = 1 + x 2 − 2x …(ii)
= { 15, 30, 45, 60, 75, 90}
× { 15, 30, 45, 60, 75, 90} ∴ f ( −x ) = − f ( x ) Multiplying Eq. (i) by 2 and subtracting
∴ Number of elements in So, f ( x ) is an odd function of x. Eq. (ii) from Eq. (i), we get
( R × S ) ∩ ( S × R) = 6 × 6 = 36 13. (c) 3 f ( x ) = x 2 + 2x − 1
14. (b) f ( x + 1) − f ( x ) = 8 x + 3 x 2 + 2x − 1
4. (a) ⇒ f (x ) =
3
1
5. (b) f ( x ) = x − 2
2 ⇒ b [( x + 1)2 − x 2 ] + c [ x + 1 − x ]
23. (a) Clearly x R x as x − x + 2 = 2 is
x +( d − d ) = 8 x + 3 an irrational number.
2
∴ f   =   − ∴ 2bx + ( b + c ) = 8 x + 3
1 1 1
x x Thus, R is reflexive. Also, ( 2 , 1) ∈ R as
1/ x2 On comparing, 2 − 1 + 2 = 2 2 − 1 is an irrational
1
= 2 − x 2 = − f (x ) 2b = 8, b + c = 3 number but ( 1, 2 ) ∉R as
x ⇒ b = 4, c = − 1 1 − 2 + 2 = 1 is a rational number.
1+ x2 (1+ x 2 ) So, R is not symmetric. Since, 1R2 and
6. (d) f ( x ) =
2
= 15. (a) 2R 2, but 1 is not related to 2.
1− x 2
(1 + x ) (1 − x )
16. (b) fog( x ) = f [ g( x )] = f (|3x + 4|) So, R is not transitive.
f (x )⋅ f (x 2 ) Since, the domain of f is [ − 3, 5].
Now, 24. (b) The number of ordered pairs in the
1 + [ f ( x )]2 ∴ − 3 ≤ |3x + 4| ≤ 5 equivalence class of (3, 2) is the number
(1+ x ) (1+ x )
2
⇒ |3x + 4| ≤ 5 of ordered pairs ( a , b ) satisfying
× a 3
( 1 − x ) ( 1 + x )( 1 − x ) ⇒ − 5 ≤ 3x + 4 ≤ 5 ( a , b ) * (3, 2) i.e. 2a = 3b ⇒ =
= b 2
( 1 + x )2 1
1+ ⇒ − 9 ≤ 3x ≤ 1 ⇒ − 3 ≤ x ≤ Clearly, such ordered pairs are
( 1 − x )2 3
(3, 2), (6, 4), (9, 6), (12, 8), (15, 10) and
(1+ x 2 ) ∴ Domain of fog is  − 3, .
1
( 1 − x )2  3 
(18, 12)
= ∴ Number of ordered pairs = 6
( 1 − x )2 + ( 1 + x )2 17. (d)
( 1 − x )2 25. (c) The relation is defined as xRy, iff
18. (a) Let f ( x ) = y , then
1+ x2 (1+ x 2 ) 1 3x + 4 y = 5
= = = a =x+
y
x +1 2

If we take, ( x , y ) =  1, 
2 + 2x 2
2 (1+ x 2 ) 2 1
−y 1 x − x2 + 1  2
⇒a = =
x + x2 + 1 −1
and  ,  , then these pairs are
7. (d) 8. (c) 2 3
π  3 4
9. (a) f  x +  = |cos x | + |− sin x | 1
∴ a y − a − y = 2x ⇒ x = ( a y − a − y )
 2 2 satisfied by the given relation.
= |cos x | + |sin x | = f ( x ) −1 1 x −x 1 1
∴ f (x ) = (a − a ) 1R ⇔ 3⋅ 1 + 4⋅ = 5
10. (b) For f ( x ) to be defined, 2 2 2
x + 2≥ 0 ⇒ x ≥ − 2 2 3 2 3
19. (a) f ( x ) = 3x + 10 and g( x ) = x 2 − 1 and R ⇔ ⋅3 + 4⋅ = 5
and 1 − x > 0 and 1 − x ≠ 1 3 4 3 4
∴ fog = f [ g( x )] = 3[ g( x )] + 10
⇒ x < 1 and x ≠ 0 But 0 R 1 ∉R as
∴ x ∈ [ − 2, 0 [ ∪ ] 0, 1 [ = 3( x 2 − 1) + 10 = 3x 2 + 7 0 R 1 ⇔ 0 × 1+ 4 × 1 = 4 ≠ 5
y−7
11. (b) We have, 2 y − y sin 3x = 1 Let 3x 2 + 7 = y ⇒ x 2 = 26. (c) Since, f : R→ R such that
3
26 NDA/NA Pathfinder

f ( x ) = 3− x = f ( g( −2 / 3)) − g( f ( −2 / 3)) of same father and q and r be the


−2 2   [ −2 / 3 ]  children of same father.
Let y and y be two elements of f ( x )
1 2 = f  ×  − g 
such that y = y  3 3  −2 / 3  So, p and r are the children of same
1 2 father. Hence, R is transitive.
−x −x
⇒3 =3 ⇒ x =x = f  −  − g  
1 2 4 3
 9  2
1 2
Since, R have all three properties such
Since, if two images are equal, then their that reflexive, symmetry and transitive, so
elements are equal, therefore it is [ −4 / 9 ] 3 3 9 9
= − × = − =0 R is an equivalence relation.
one-one function. −4 / 9 2 2 4 4 x +5
Since, f ( x ) is positive for every value of 38. (b) f − 1 ( x ) =
33. (d) fof  −  + gog( −1)
7 3
x, therefore f ( x ) in into.  4 −1 y+5
On differentiating w.r.t. x, we get ⇒ f ( y) = ...(i)
dy = f ( f ( −7 / 4)) + g( g( −1)) 3
= − 3− x log3 < 0, for every value of x.  [ −7 / 4 ] 
dx = f  + g( −1 × 1) Q y = f (x ) ⇒ x = f − 1( y )
 −7 / 4  y+5
∴ It is decreasing function. ∴ x = ⇒ 3x = y + 5
−2 
∴ Statements I and III are true. = f  8
 + g( −1) = f   + ( −1) 3
 −7 / 4   7
27. (c) for f ( x ) to be real log (sin x ) ≥ 0 ⇒ y = 3x − 5 ⇒ f ( x ) = 3x − 5
2 [8 / 7] 7 1
⇒ sin x ≥ 20 ⇒ sin x = 1 = −1= −1= − 39. (c) Given, A = { x ∈ R|x ≥ 0}
π 8/7 8 8
⇒ x = ( 4n + 1) , n ∈ N f : A → A, f ( x ) = x 2
2 34. (d) Given that, f ( xy ) = f ( x ) f ( y )
Y
28. (a) f ( x ) is defined, if 3x 2 − 4x + 5 ≥ 0 From option (d), we take f ( t ) = t k y = x2

3 x 2 − x +  ≥ 0
4 5 Then, f ( xy ) = ( xy )k = ( x k ) ( y )k

 3 3  = f (x ) ⋅ f ( y) X′ X
 2
11
3  x −  +
2 O
⇒ ≥ 0 35. (b) Given, A = { 0, 1, 2}, B = {2, 3}
 3  9

C = {3, 4}
Which is true for all real x. Y′
∴ A ∪ B = { 0, 1, 2, 3}
∴ Domain ( f ) = ( − ∞ , ∞ ) = R From adjoining figure, f ( x ) is one-one
∴( A ∪ B ) × C = { 0, 1, 2, 3} × {3, 4} and onto, so its inverse exists.
29. (b) g[ f ( x )] = log e ( 3x 2 − 4x + 5 )2
= {(0, 3), (0, 4), (1, 3), (1, 4), (2, 3), Let f (x) = y
= log e (3x 2 − 4x + 5) −1
(2, 4), (3, 3), (3, 4)} ∴x 2 = y ⇒ x = y ⇒ f (x ) = x
Let y = log e (3x 2 − 4x + 5) So, required number of elements So, f is not its own inverse.
⇒ e y = 3x 2 − 4x + 5 containing by ( A ∪ B ) × C is 8.
⇒ 3 x 2 − 4 x + (5 − e y ) = 0 40. (a) I. Given, f ( x ) = x 3 and g( y ) = y 3
36. (a) Given, relation is
Since, both the functions are identical.
For x to be real, discriminant ≥ 0 R = {( 1, 2), ( 1, 3), ( 2, 1), ( 1, 1), ( 2, 2), ∴ f = g
∴ 16 − 12(5 − e y ) ≥ 0 (3, 3), ( 2, 3)} II. We know that, an identity function
⇒ 12e y ≥ 44 and P = { 1, 2, 3} f ( x ) = y = x is always one-one and onto
11 11 function, i.e. bijective function.
⇒ ey ≥ ⇒ y ≥ log e Reflexive In R, 1R 1, 2R 2 and 3R3, Y
3 3 where 1, 2, 3 ∈ P.
y=x
 11 
∴ Range of f = log e , ∞  So, R is reflexive.
 3 
Symmetry In R , 1R3 ⇒ / 3R1 and 2 R3 X′ X
⇒ O
30. (b) f ( x , x ) = x =x [Q 2 + 3 < 32 ] / 3 R2
2 3 2+3 5
and f ( x , x ) = x So, R is not symmetric.
5 6 5 + 6− 3
=x [Q 5 + 6 > 32 ] Transitive In R, 1R2 and 2R3 ⇒ 1R 3 Y′
8

∴ f [ f ( x , x ), f ( x , x )] = f ( x , x ) So, R is transitive. Hence, only Statement I is true.


2 3 5 6 5 8
Thus, R is reflexive, transitive but not 41. (c) Given that,
=x
5 + 8− 3 symmetric. A = { x , y }, B = { 2, 3}, C = {3, 4}
=x
[Q 5 + 8 > 32 ]
10 37. (d) R = {x : x is a set of all children of a ∴ B ∪ C = { 2, 3} ∪ {3, 4} = { 2, 3, 4}
2 ×3
31. (a) g( x , x ) = x Q → m = 1 same father} and A × ( B ∪ C ) = { x , y } × { 2, 3, 4}
1  
2 3
 5
Reflexive Let p be the children of same = {( x , 2),( x ,3),( x , 4),( y , 2),( y ,3),( y , 4)}
7 ×8
and g( x , x ) = x Q → m = 1 father. Hence, pRp is a reflexive.
∴ Number of elements in A × ( B ∪ C )
1 
7 8
 5 Symmetry Let p and q be the children i.e. n{ A × ( B ∪ C )} = 6
∴ g [ g( x , x ), g( x , x )] of same father.
2 3 7 8 42. (c) If R is a non-empty set, then a
1× 1
= g( x , x ) Q → m = 1 So, q and p are the children of same relation A from R to R is a subset of
1 1   father. Hence, R is symmetric.
5 R × R , i.e. A ⊆ ( R × R)
=x
1
Transitive Let p and q be the children
32. (c) fog( −2 / 3) − gof ( −2 / 3) 43. (b) Given that, f : N → N
MATHEMATICS Relations and Functions 27

and f ( x ) = x + 1, for x ∈ N , Considering f ( x ) and f ( x ) (I) u (II) u p


1 2 p
if x , x ∈N , then f ( x ) = f ( x ) Such that f (x ) = f (x ) v v
1 2 1 2 1 2 w q w q
⇒ x + 1= x + 1 ⇒ x = x ⇒ 2x = 2x ⇒ x = x x r x r
1 2 1 2 1 2 1 2 y
y
i.e. f ( x ) is one-one. So, f is one-one. z s z s
Range of f ( x ) ∈ N − { 1} For onto Let y = f ( x) (function) (not function)
y
∴ Range ⊆ Codomain Then, y = 2x ⇒ x = (III) u p (IV) u
2 p
v v
So, f ( x ) is into function. q q
∴ x ∈ N for every y ∈ A w w
Hence, f is one-one but not onto. x r x r
[every element of codomain has y y
|x | z s s
44. (c) Given function, f ( x ) = , x ≠ 0 pre-image in domain] z
x (not function) (function)
So, f is onto.
Redefine the given function,
Hence, f is both one-one and onto. 53. (b) Given that, f : Z → N
 x , x > 0  1, x > 0
 and f ( x ) = |x |
f ( x ) =  −xx = 50. (b) Given that,
−1, x < 0 Graph of f ( x ) =|x |
 , x<0  X = {Set of all citizens of India}
 x Y
∴ Range of f ( x ) = { −1, 1} and R = {(x , y ) : x , y ∈ X , |x − y | = 5}
y =| x |
45. (c) Given that, A = { 1, 2 }, B = { 2, 3} Reflexive |x − x | = 0 ≠ 5
and C = {3,4} ∴ xRx ∉ R
Now, ( A × B ) = { 1, 2 } × { 2, 3} So, R is not reflexive. X′ X
O
= {( 1, 2),( 1, 3), ( 2, 2),( 2, 3)} Symmetric Again, xR y
and ( A × C ) = { 1, 2 } × {3, 4} ⇒ x − y =5
= {( 1, 3), ( 1, 4), ( 2, 3), ( 2, 4)} ⇒ | y − x |= 5 ⇒ yRx Y′
∴( A × B ) ∩ ( A × C ) = {( 1, 3), ( 2, 3)} So, R is symmetric. We see that, if we draw a parallel line
So, the cardinality of Transitive Let x , y , z ∈ X along X-axis. It cuts the curve into more
( A × B ) ∩ ( A × C ) is 2. than one point. So, function f ( x ) = |x | is
Then, xRy ⇒ |x − y| = 5
not one-one.
46. (c) and yRz ⇒ | y − z| = 5 Since, f ( x ) = |x | is always
47. (c) Here, n ( A ) = 4 and n ( B ) = 3 But |x − z| ≠ 5 non-negative.
∴ n ( A × B ) = n ( A) × n ( B) So, R is not transitive. ∴ Range of f = Set of non-negative
integers = Codomain of f
= 4 × 3 = 12 Hence, the relation is symmetric but
⇒ f ( x ) is onto.
neither reflexive nor transitive.
48. (d) Given, R = {( a , b ): a − b is Thus, f ( x ) = |x | is onto but not
divisible by 5} 51. (c) Given that, S = Set of all integers. one-one.
Reflexive ( a − a ) is divisible by 5 for and R = {(a, b), a, b ∈ S and ab ≥ 0}
54. (d)
all a ∈ Z . So, R is reflexive. Reflexive a R a
55. (d) We have, f ( x ) = ax + b
Symmetric Let ( a , b ) ∈ R ⇒ ( a − b ) is ⇒ a ⋅ a = a 2 ≥ 0, ∀a ∈ S and g( x ) = cx + d
divisible by 5. So, R is reflexive. f [ g( x )] = a( cx + d ) + b
⇒( b − a ) is divisible by 5 ⇒ b − a ∈ R Symmetric a R b ⇒ ab ≥ 0, ∀ a , b ∈ S = acx + ad + b
So, R is symmetric. ⇒ b ⋅ a ≥ 0 ⇒ bRa and g [ f ( x )] = c ( ax + b ) + d
Transitive Let ( a , b ) ∈ R and (b, c) ∈R = acx + bc + d
So, R is symmetric.
Q f [ g( x )] = g [ f ( x )]
⇒( a − b ) and ( b − c ) are both divisible Transitive If aR b ⇒ ab ≥ 0
by 5. ⇒ ad + b = bc + d ⇒ f ( d ) = g( b )
and b R c ⇒ b c ≥ 0, ∀ a , b , c ∈ S
⇒ a − b + b − c is divisible by 5. 56. (b) We have, f : N → N
Then, ac ≥ 0 ⇒ aR c f ( x ) = 2x + 3 ⇒ f ′( x ) = 2 > 0
⇒ ( a − c ) is divisible by 5.
So, R is also transitive. So, f ( x ) is increasing, ∀x ∈ N .
⇒ ( a , c) ∈ R Hence, f ( x ) is injective.
Hence, R is an equivalence relation.
So, R is transitive. Let f (x ) = y
52. (c) Given that, A = {u, v, w, x, y, z}
Thus, R is reflexive, symmetric and ⇒ y = 2x + 3
transitive. and B = {p , q , r , s } y −3
We know that, a mapping f : x → y is ⇒ x =
Hence, R is an equivalence relation. 2
said to be a function, if each element in 1
49. (c) Let A = {Set of even natural the set x has its image in set y. It is also Let y=4 ⇒ x =
number} 2
possible that there are few elements in
Given, f : N → A and f ( x) = 2x set y which are not the image of any i.e. y ∈ N but x ∉ N
element in set x. Every element in set x Hence, f ( x ) is not surjective.
For one-one Let x , x ∈ R
1 2 should have one and only one image.
28 NDA/NA Pathfinder

57. (c) The relation S is defined on the set of 62. (b) We have, X = Set of all persons III. We have, f ( x ) = e x , x ∈ R
integers Z and xSy, if integer x divides living in a city ⇒ f ′( x ) = e x > 0
integer y. Let R be a relation on X , define as So, f is increasing function.
Reflexive Since, every integer divides x < y , if y is atleast 5 yr older than x. ⇒ f is one-one.
itself.
Since, x cannot be less than x itself, so R Hence, f is invertible on its range.
∴ Integer x divides integer x ⇒ xSx
is not reflexive. Now, let xRy, then
Hence, S is reflexive. 66. (c) We have, aRb, where a , b ∈ Z,
x < y , i.e. y is atleast 5 yr older than x.
Symmetric Let x , y ∈ z such that xSy if ( a − b ) is divisible by 5.
Thus, x is smaller than y. Hence, R is
i.e. integer x divides integer y. not symmetric. Thus, ( a − b ) = 0, 5, 10, 15, K
But this does not implies that integer y So, Statement I is true, since the relation
Now, let xRy and yRz, then x < y and
divides integer x. R partitions Z into five equivalent
y < z, clearly x < z.
Thus, S is not symmetric. classes, which are, 0, 1, 2, 3, 4.
Hence, R is transitive.
Transitive Let x , y , z ∈ Z such that x Also, any two equivalent classes are either
xSy and ySz 63. (c) We have, f ( x ) = , for x =/ 0 disjoint or equal.
|x |
⇒ Integer x divides integer y and Hence, both the statements are true.
integer y divides integer z.  x , if x > 0 1
67. (b) Given, f ( x ) =
⇒ Integer x divides integer z. i.e. f ( x ) =  x |x | − x
⇒ xSz x
 , if x < 0
⇒ Hence, S is transitive.  −x Here, f ( x ) is defined only when
x −1 1, if x > 0 | x | − x > 0 or | x | > x
58. (a) We have, f ( x ) = 
= which is possible only for negative values
x+1 −1, if x < 0 of x.
Applying componendo and dividendo,
we get Thus, range of f = { 1, − 1}. ∴ Domain of f is ( − ∞ , 0).
f (x ) + 1 x − 1+ x + 1 1+ x 
= 64. (b) We have, f ( x ) = log e   68. (b) Given, f ( x ) = 2x − 3
f (x ) − 1 x − 1− x − 1  1− x 
f (x ) + 1 3x + x 3 and g( x ) = x 3 + 5
⇒ = −x and g( x ) =
f (x ) − 1 1 + 3x 2 Now, fog( x ) = f ( g( x ))
 ( e − 1)  = 2( x 3 + 5) − 3
f (x ) + 1 1+ 
Now, + x = −x + x = 0  e − 1  ( e + 1) 
f (x ) − 1 ∴f   = log e = 2x 3 + 10 − 3 = 2x 3 + 7
 e + 1  ( e − 1) 
1−  Let fog ( x ) = y , then
59. (c) We have,  ( e + 1) 
1/3
y − 7
x −1 2x − 1 = log e 
e + 1 + e − 1
 y = 2x 3 + 7 ⇒ x =  
f (x ) = ⇒ f ( 2x ) =  e + 1 − e + 1  2 
x + 1 2x + 1 1/3
= log e   = log e ( e ) = 1…(i) x − 7
∴( fog )− 1 ( x ) = 
2e
2[ f ( x ) + 1]  2 
−1  2 
1− f (x ) e − 1   e − 1 
⇒ f ( 2x ) = gof   = gf   69. (d) Given, R = {( a , b ) : |a − b | ≤ 5}
2[ f ( x ) + 1]  e + 1   e + 1 
+ 1 Reflexive ( a , a ) ∈ R
1 − f (x )
= g ( 1) [using Eq. (i)] ( a , a ) = | a − a | = 0, 0 ≤ 5, ( a , a ) ∈ R
  3( 1) + ( 1)3 4
Q x = f ( x ) + 1  = = =1 Hence, R is reflexive.
 1 − f (x ) 1 + 3( 1)2 4 Symmetric ( a , b ) ∈ R
 
65. (c) I. We have, f ( x ) = x 3 , x ∈ R ⇒ ( b, a) ∈ R
3 f (x ) + 1
⇒ f ( 2x ) = ⇒ f ′( x ) = 3x 2 ≥ 0 ( a, b) = | a − b | ≤ 5 ,
f (x ) + 3
⇒ f is increasing function. ( b, a) = | b − a | ≤ 5
x −1
60. (c) We have, f ( x ) = So, f is one-one. ∴ ( a, b) ∈ R ⇒ ( b, a) ∈ R
x+1
Hence, f is invertible on its range. Hence, R is symmetric.
f (x ) − 1
⇒ f [ f ( x )] = Transitive ( a , b ) ∈ R, ( b , c ) ∈ R
f (x ) + 1 II. We have, f ( x ) = sin x , 0 < x < 2 π
π π ⇒ ( a , c) ∈ R
Clearly, f   = sin =
3
⇒ f [ f ( x )] = −
1 3 3 2 ( a , b ) = | a − b | ≤ 5,
2π π
and f  
x
 = sin  π −  ( b , c ) = |b – c |≤ 5
    3  3
Q x = −  f ( x ) + 1  π
|a − b |+ |b − c | ≤ 10, |a − c | ≤ 10
= sin   =
3
  f ( x ) − 1 ∴ ( a , c) ∉ R
 3 2
Hence, R is not transitive.
61. (a) Number of distinct relations from So, f is not one-one.
So, R is reflexive and symmetric but not
B to A = 24 × 3 = 212 = 4096 Hence, f is not invertible on its
transitive.
range.
MATHEMATICS Relations and Functions 29

70. (b) Given, g( x ) =


1
and f ( x ) = x  1 − x1 − x 2  27( x 2 / 3 − x )
  79. (b) Given function, f ( x ) =
f (x )  x −x  1− x x  4
and f  1 2  = ln  1 2

1
g( x ) = , x ≠ 0  1− x x   x −x  If f (x ) = 1
 1 2  1+ 1− x x 
1 2
x
 27( x 2 / 3 − x ) 4
1 1 1 2  ⇒ 1= ⇒ x 2/3 − x =
Clearly, g( f ( x )) = = 4 27
f (x ) x  1− x x − x + x  Let x1 / 3 = α ⇒ x = α3
1 = ln  1 2 1 2 …(ii)
g( g ( f ( x ))) = =x  1− x x + x − x  Then, α − α 3 =
2 4
 1  1 2 1 2 …(i)
  27
x
∴ From Eqs. (i) and (ii), we get ⇒ α 2 (1 − α ) = × ×
2 2 1
1 1
g( g( g( f ( x )))) = =  x −x  3 3 3
(x ) x f (x ) − f (x ) = f  1 2  2 2 1
1 1 2  1− x x  ⇒ α ⋅ α (1 − α ) = × ×
f ( g( g( g( f ( x ))))) =  1 2 3 3 3
x 2 2
1 x2 ⇒ α= ⇒ x1 / 3 =
f ( f ( g( g( g( f ( x )))))) = 75. (a) Let f ( x ) = 3 3
x 1 + x2 8
1 ∴ x =
and g( f ( x )) = 27
x Clearly, domain ( f ) = R
1 x2 Also, from Eq. (i)
f ( g( f ( x ))) = Let y = f ( x ) ⇒ y = 4
x 1+ x2 α2 − α3 =
1 ⇒ y + x y = x ⇒ x2 − x2y = y
2 2 27
g( f ( g( f ( x )))) = =x
⇒ α ⋅ α( 1 − α ) =  −  ×  −  ×  
 1 y 1 1 4
  ⇒ x 2 (1 − y ) = y ⇒ x = ±  3  3  3 
x 1− y
α = − ⇒ x 1 / 3 =  − 
1 1 1
g( g( f ( g( f ( x ))))) = Clearly, x will take real values, ⇒
3  3
x y y−0
From option (b), LHS = RHS if ≥0⇒ ≤0 1
1− y y−1 ⇒ x =−
71. (d) Let f ( x ) = log x + – + 27
–∞ ∞ Hence, f ( x ) has two solutions.
∴ f ( xy ) = log( xy ) 0 1
Now, f ( x ) + f ( y ) = log x + log y ⇒ 0 ≤ y < 1 ⇒ y ∈[ 0, 1) 27( x 2 / 3 − x )
80. (a) Given, f ( x ) =
Hence, range f ( x ) is [ 0, 1). 4
= log ( xy ) = f ( xy )
76. (c) Given, R is a relation on the set N of We have, f (x ) = − 1
Hence, f ( x ) + f ( y ) = f ( xy ) natural numbers defined by nRm ⇔ n is a 27( x 2/3
− x)
72. (a) I. is correct, since f ( x ) = x 3 is factor of m. ⇒ = −1
4
one-one, ∀ x ∈ R. Reflexive Since, n is a factor of n for
each n ∈ N , therefore, nRn, ∀ n ∈ N , i.e. 4
Q f ( a) = f ( b) ⇒ a3 = b 3 ⇒ a = b ⇒ x 2/3 − x = −
R is reflexive. 27
II. is also correct and is the correct
Symmetric Note that 2 is a factor of 4 Let x1 / 3 = α ⇒ x = α3
explanation of Statement I. but 4 is not a factor of 2, i.e. 2R4 but 4
Then, α2 − α3 = −
73. (a) Reflexive Given, xRy ⇒ x is less 4 R/ 2 . Thus, R is not symmetric. 27
than y 2 . Hence, option (c) is correct. 4 4
⇒ α3 − α2 = ⇒ α3 = α2 +
∴ xRx ⇒ x is less than x 2 .
77. (c) ( gof )  −  − ( fog )  − 
5 5 27 27
which is true. Hence, R is reflexive.  3  3
Y
Symmetric xRy is not equivalent to  5   5  y=α2 + 4/27
yRx because = g  f  −   − f  g  −   2
  3    3 
1R 2 ⇒ 1 is less than 22 . 1
 5   5 
2R 1 ⇒ 2 is less than 12 . = g   −   − f  −  X′ X
  3     3  −4 −3 −2 −1 0 1 2 3 4
Thus, it is not symmetric. Hence, option
= g( −2) − f  
5 −1
(a) is correct.
3 −2
1− x 
74. (a) Let f ( x ) = ln    5 y=α3
 1+ x  = |− 2|− = 2 − 1= 1 Y′
 3 
Now, f ( x ) − f ( x ) Since, the graph of y = α 3 and
78. (b) ( fof )  −  + ( gog ) ( −2)
1 2 9
 1− x   1− x  4
= ln  1  − ln  2   5 y = α2 + intersects at only one
 1+ x   1+ x  27
 1  2  9 
= f  f  −   + g( g( −2)) point.
 ( 1 − x )( 1 + x )    5
= ln  1 2 
∴ α3 = α2 +
4
 ( 1 − x )( 1 + x )  has only one solution.
 1   9 
2 = f   −   + g(| − 2 |) 27
 1+ x − x − x x    5   4
Hence, x − x 2 / 3 = has only one
= ln  2 1 1 2 …(i) 27
 1+ x − x − x x  = f ( −2) + g( 2) = − 2 + 2 = 0
 1 2 1 2 solution.
03
30 CDS Pathfinder

COMPLEX NUMBERS
In NDA exam, generally 3-5 questions are asked from this chapter which are based on real and
imaginary parts of complex number, modulus, argument, square root, cube root of unity etc.

If a and b are two real numbers, then any number of the form a + ib is called a complex number,
where i = − 1. In other words, we can say that a number consisting of real number and imaginary
number is called complex number. It is usually denoted by z, i.e. z = a + ib

Iota and Imaginary Quantities


There is no real number whose square is negative real number. Euler was the first Mathematician to
introduce the symbol i (iota) for the square root of −1 with the property that i 2 = − 1 . He called this
symbol as the imaginary unit. The square root of a negative real number is called an imaginary
9
quantity or an imaginary number. e.g. −3, −4, − etc., are imaginary quantities.
4

Integral Powers of i
We have, i = −1, i 2 = −1. Therefore, i 3 = i 2 × i = ( −1) × i = − i, i 4 = i 2 × i 2 = ( −1) × ( −1) = 1
Thus, i + i 2 + i 3 + i 4 = 0. In general, i r + i r + 1 + i r + 2 + i r + 3 = 0 i.e. the sum of four consecutive powers
of i is equal to zero.
2
 1 25 
EXAMPLE 1. The value of i 19 +    is
 i 
a. 2 b. −2 c. 4 d. −4
 
+    1 
= i +  = − i +  = ( − i − i) = 4i = − 4
1 1 i
Sol. d. We have, i  = i +
 
  i   i i   i 

Set of Complex Numbers


The product set R × R consisting of the ordered pairs of real numbers, called the set of complex
numbers. The set of all complex numbers is denoted by C, i.e. C = { a + ib | a, b ∈ R}.
Since, a real number ‘a’ can be written as a + 0 i, therefore every real number is a complex number.
Hence, R ⊂ C, where R is the set of all real numbers.
MATHEMATICS Complex Numbers 31

Real and Imaginary Parts Properties of Addition


of a Complex Number 1. z1 + z 2 = z 2 + z1 , ∀ z1 , z 2 ∈ C [Commutative law]
If z = a + ib is a complex number, then ‘a’ is called the 2. ( z1 + z 2 ) + z 3 = z1 + ( z 2 + z 3 ), ∀ z1 , z 2 , z 3 ∈ C
real part of z and ‘b’ is known as the imaginary part [Associative law]
of z. The real part of z is denoted by Re( z) and the 3. z + 0 = z = 0 + z, ∀ z ∈ C, where 0 = 0 + i0 is the identity
imaginary part is denoted by Im( z). element.
e.g. If z = 7 + 5i, then Re( z) = 7 and Im( z) = 5 4. z + ( − z) = 0 = ( − z) + z, ∀ z ∈ C, where z and −z are
A complex number z is purely real, if its imaginary additive inverse of each other.
part is zero, i.e. Im ( z) = 0 and is said to be purely
Subtraction of Complex Numbers
imaginary, if its real part is zero, i.e. Re ( z) = 0.
Let z1 = a1 + ib1 and z 2 = a 2 + ib2 be any two complex
The complex number 0 = 0 + i 0 is both purely real numbers. Then,
and purely imaginary.
z1 − z 2 = z1 + ( − z 2 ) = ( a1 + ib1 ) + ( − a 2 − ib2 )
 (3 + i) 2  = ( a1 − a 2 ) + i( b1 − b2 )
EXAMPLE 2. Find the Im  .
 7 −i  Multiplication of Complex Numbers
2 8 Let z1 = a1 + ib1 and z 2 = a 2 + ib2 be any two complex
a. i b. i
numbers. Then,
3 7
c. i
4
d. − i z1 z 2 = ( a1 + ib1 ) ( a 2 + ib2 )
7
= ( a1 a 2 − b1 b2 ) + i( a1 b2 + a 2 b1 )
( 3 + i) 9 + i + 6i 9 − 1 + 6i 8 + 6i Thus, z1 z 2 = [Re ( z1 ) Re ( z 2 ) − Im ( z1 ) Im ( z 2 )]
Sol. c. We have, = = =
7−i 7−i 7−i 7−i
+ i [Re ( z1 ) Im ( z 2 ) + Re ( z 2 ) Im ( z1 )]
(8 + 6i)(7 + i) 56 + 8i + 42i + 6i
= = Properties of Multiplication
(7 − i)(7 + i) 49 − i
56 − 6 + 50i 50 + 50i 1. z1 z 2 = z 2 z1 [Commutative law]
= = = 1 + i,
49 + 1 50 2. ( z1 z 2 ) z 3 = z1 ( z 2 z 3 ) [Associative law]
 ( 3 + i)  3. z ⋅ 1 = z = 1⋅ z, where 1 is multiplicative identity.
∴ Im   =i
 7−i  4. z ⋅ z1 = 1 = z1 ⋅ z, where complex number z1 is called the
multiplicative inverse or reciprocal of z and is given by
Equality of Complex Numbers a ( − b)
z1 = 2 +i 2
Two complex numbers z1 = a1 + ib1 and z 2 = a 2 + ib2 a +b 2
a + b2
are equal, 5. (i) z1 ( z 2 + z 3 ) = z1 z 2 + z1 z 3 [Distributive law]
if a1 = a 2 and b1 = b2 (ii) ( z 2 + z 3 ) z1 = z 2 z1 + z 3 z1 [Distributive law]
i.e. Re ( z1 ) = Re ( z 2 ) and Im ( z1 ) = Im ( z 2 )
e.g. If z1 = 2 − iy and z 2 = x + 3i are equal, then Division of Complex Numbers
2 − iy = x + 3i ⇒ x = 2 and y = − 3 The division of a complex number z1 by a non-zero
complex number z 2 is defined as the multiplication of z1 by
Note Two complex numbers cannot be compared, i.e. no z
greater complex number can be find in two given complex the multiplicative inverse of z 2 and is denoted by 1 .
numbers. z2
Let z1 = a1 + ib1 and z 2 = a 2 + ib 2
Addition of Complex Numbers z1  a ( − b 2 ) 
Let z1 = a1 + ib1 and z 2 = a 2 + ib 2 be any two Then, = ( a1 + ib1 )  2 2 2 + i 2 
z2  a 2 + b2 a 2 + b22 
complex numbers. Then,
z1 + z 2 = ( a1 + a 2 ) + i( b 1 + b 2 ).  a1 a 2 + b1 b 2   a 2 b1 − a1 b 2 
=  +i 
Thus, Re ( z1 + z 2 ) = Re ( z1 ) + Re ( z 2 )  a 2 + b2 
2 2
 a 22 + b22 
and Im ( z1 + z 2 ) = Im ( z1 ) + Im ( z 2 )
32 NDA/NA Pathfinder

5+i 2
EXAMPLE 3. Compute z = + . Modulus of a Complex Number
3−i 1 − 3i The modulus of a complex number z = a + ib is denoted
7 5 5 7 7 5 −7i 8 by mod ( z ) or | z | and is defined as
a. i + b. i + c. i − d. +
5 8 8 5 5 8 5 5
| z | = a 2 + b 2 = {Re ( z)} 2 + {Im ( z)} 2
5+ i 2 ( 5 + i )(1 − 3i ) + 2( 3 − i)
Sol. d. We have, z = + =
Clearly, | z | ≥ 0, ∀ z ∈ C
3 − i 1 − 3i ( 3 − i)(1 − 3i)
5 − 15i + i + 3 + 6 − 2i 14 − 16i −7 8 e.g. If z = 4 + 3i is a complex number, then
= = = i+
3 − 9i − i − 3 −10i 5 5 | z | = 42 + 32 = 16 + 9 = 25 = 5

Conjugate of a Complex Number Properties of Modulus


A pair of complex numbers z1 and z 2 are said to be If z, z1 , z 2 ∈ C, then
conjugate of each other, if the sum and product of two
(i) | z | ≥ 0 ⇒ | z | = 0, iff z = 0, i.e. Re(z) = Im( z) = 0 and
z1 and z 2 both are real.
| z | > 0, iff z ≠ 0,
Let z = a + ib be a complex number. Then, the conjugate
of z is denoted by z or z ′ and is equal to a − ib. (ii) | z | = | z | = | − z | = |− z |
(iii) − | z | ≤ Re ( z) ≤ | z |, − | z | ≤ Im ( z) ≤ | z |
Thus, z = a + ib ⇒ z = a − ib
(iv) | z | 2 = zz
Note It shows that the conjugate of a complex number is obtained (v) | z n | =| z | n
by replacing i by −i.
(vi) | z1 z 2 | = | z1 | | z 2 |
Properties of Conjugate z  |z |
(vii)  1  = 1 ; z 2 ≠ 0
If z, z1 and z 2 are complex numbers, then  z2  | z2 |
(i) ( z ) = z (ii) z + z = 2Re ( z) (viii) | z1 + z 2 | 2 = | z1 | 2 + | z 2 | 2 + 2 Re ( z1 z 2 )
(iii) z − z = 2i Im ( z) (iv) z = z ⇔ z is purely real
(ix) | z1 − z 2 | 2 = | z1 | 2 + | z 2 | 2 − 2 Re ( z1 z 2 )
(v) z = − z ⇔ z is purely imaginary
(x) | z1 + z 2 | 2 + | z1 − z 2 | 2 = 2 (| z1 | 2 + | z 2 | 2 )
(vi) zz = {Re ( z)} 2 + {Im ( z)} 2
(xi) | az1 − bz 2 | 2 + | bz1 + az 2 |=2 ( a 2 + b 2 ) (| z | 2 + | z | 2 ),
1 2
(vii) z1 + z 2 = z1 + z 2 (viii) z1 − z 2 = z1 − z 2
where a, b ∈ R
z  z (xii) | z1 ± z 2 | ≤ | z1 | + | z 2 |
(ix) z1 z 2 = z1 z 2 (x)  1  = 1 , z 2 ≠ 0
 z2  z2 (xiii) | z1 ± z 2 | ≥ || z1 | − | z 2 ||
z
(xi) ( z ) n = ( z n ) (xiv) | z1 + z 2 | 2 = | z1 | 2 + | z 2 | 2 ⇔ 1 is purely imaginary.
z2
(xii) z1 z 2 + z1 z 2 = 2 Re ( z1 z 2 ) = 2 Re ( z1 z 2 )

EXAMPLE 4. If (x − iy )(3 + 5i ) is the conjugate of Argument/Amplitude of


(− 6 − 24i ), then the values of x and y are respectively Complex Numbers
a. 3, –3 b. 3, 4 c. – 3, 4 d. – 3, – 4 Let z = x + iy be a Y
Sol. a. ( x − iy)( 3 + 5i ) = 3x + 5xi − 3yi − 5yi non-zero complex number
and can be represented in N P(x, y) = (r, θ)
= 3x + ( 5x − 3y) i + 5y [Q i = − 1]
y2

the form
+

= ( 3x + 5y) + ( 5x − 3y) i …(i)


√x 2

z = r (cos θ + i sin θ) where, r y = r sin θ


( x − iy) ⋅ ( 3 + 5i ) = ( − 6 − 24i )
r=

Given, is the modulus and θ is the θ


⇒ ( 3x + 5y) + i( 5x − 3y) = − 6 + 24i argument of z. X′ X
O y = r cos θ M
[using Eq. (i) and z = ( a + ib) ⇒ z = ( a − ib)] Then,
Comparing the real and imaginary parts on both sides, we y Y′
get tan θ =
x
3x + 5y = − 6 and 5x − 3y = 24
 y  y
Solving the above equations by substitution or elimination ⇒ θ = tan −1   or arg ( z) = tan −1  
method, we get x = 3, y = − 3.  x  x
MATHEMATICS Complex Numbers 33

Principal Value of Argument z is positive, so the point lies in IInd quadrant.


π 3π
The value of θ of the argument which satisfies the ∴ arg(z) = π − θ = π − =
4 4
inequality −π < θ ≤ π is called the principal value of the
1 3π
argument. Principal values of the argument are Hence, modulus = and arg ( z) =
θ, π − θ, − π + θ, − θ according as the complex number lies 2 4
on the Ist, IInd, IIIrd, or IVth quadrant respectively.
Square Root of a Complex Number
Properties of Argument Let (a + ib) be any complex number such that
If z, z1 and z 2 are complex numbers, then a + ib = x + iy, where x and y are real numbers.
(i) arg ( z ) = − arg ( z)
If b is positive, then
(ii) arg ( z1 z 2 ) = arg ( z1 ) + arg ( z 2 )
 1 1 
(iii) arg ( z1 z 2 ) = arg ( z1 ) − arg ( z 2 ) a + ib = ±  { a 2 + b 2 + a} + i { a 2 + b 2 − a} 
(iv) arg ( z n ) = n arg ( z)  2 2 
(v) arg ( z1 / z 2 ) = arg ( z1 ) − arg ( z 2 ) If b is negative, then
 z  1 1 
(vi) arg   = 2 arg ( z) a − ib = ±  { a 2 + b 2 + a} − i { a 2 + b 2 − a} 
 z
 2 2 
(vii) | z1 + z 2 | 2 = | z1 | 2 + | z 2 | 2 + 2 | z1 | | z 2 | cos (θ1 −θ 2 ) ,
Note If x, y ∈ R,then
and | z1 − z 2 | 2 = | z1 | 2 + | z 2 | 2 − 2 | z1 | | z 2 | cos (θ1 − θ 2 ) ,
• x + iy + x − iy = 2( x 2 + y 2 + x )
where θ1 = arg ( z1 ) and θ 2 = arg ( z 2 )
π • x + iy − x − iy = i 2( x 2 + y 2 − x )
(viii) | z1 + z 2 | = | z1 − z 2 | ⇔ arg ( z1 ) − arg ( z 2 ) =
2
EXAMPLE 6. If (−7 − 24i) = x − iy , then x 2 + y 2 is
(ix) | z1 + z 2 | = | z1 | + | z 2 | ⇔ arg ( z1 ) = arg ( z 2 )
(x) The argument of zero is not defined. equal to
a. 15 b. 25 c. −25 d. −15
(xi) If arg ( z) = 0 ⇒ z is real.
Sol. b. We have, −7 − 24i = x − iy
EXAMPLE 5. The modulus and argument of the
1 + 2i On squaring both sides, we get −7 − 24i = x − y − ( 2xy) i
complex number is On equating real and imaginary parts, we get
1 − 3i
x − y = − 7 and 2xy = 24
1 3π 1 3π 1 3π
a. , b. ,− c. , d. None of these ∴ x + y = 49 + 576 = 625 = 25
2 4 2 4 2 4
1 + 2i 1 + 2i 1 + 3i 1 + 3i + 2i + 6i Cube Roots of Unity
Sol. a. Let z= ⇒z= × =
1 − 3i 1 − 3i 1 + 3i 1 − ( 3i ) Let z = 11/3 , then z 3 = 1 [on cubing both sides]
[Q( a + b)( a − b) = a − b ] ⇒ z −1=0 3

1 + 5i + 6( − 1) ⇒ ( z − 1) ( z + z + 1) = 0
2
= [Q i = − 1]
1 − 9i
⇒ z − 1 = 0 or z 2 + z + 1 = 0
1 + 5i − 6 − 5 + 5i − 1+ i 1 1
= = = ⇒z = − + i −1 ± 1 − 4
1+ 9 10 2 2 2 ⇒ z = 1 or z =
2
z =  −  +  
1 1 −1 ± i 3
∴ [Q a + ib = a + b ] ⇒ z = 1 or z =
 2  2
2
1 1 2 1 1
= + = = = −1 + i 3 −1 − i 3
4 4 4 2 2 So, the cube roots of unity are 1, and .
1 2 2
 −  Im ( z) 
 Clearly, one of the cube roots of unity is real and the
Now, tan θ = 2 Q θ = tan   other two are complex.
1  Re ( )  
−  z
2 −1 + i 3 −1 − i 3
π π
Here, = ω and = ω2
⇒ tan θ = 1 = tan ⇒ θ = 2 2
4 4
Since, the real part of z is negative and imaginary part of
34 NDA/NA Pathfinder

Properties of Cube Root of Unity Euler or Eulerian Form of


0, if r is not a multiple of 3 Complex Number
(i) 1 + ω + ωr 2r
=
 3, if r is multiple of 3 We have, e iθ = cos θ + i sin θ and e − iθ = cos θ − i sin θ
(ii) ω = 1 or ω = 1 and 1 + ω + ω = 0
3 3r 2
These two are called Euler’s notations.
(iii) ω 3r + 1 = ω, ω 3r + 2 = ω 2 Let z be any complex number such that | z | = r and
(iv) ω and ω 2 are the roots of the equation z 2 + z + 1 = 0. arg ( z) = θ. Then, z = x + iy = r (cos θ + i sin θ) = re iθ , where
2π 4π r is modulus. This form of z is known as Euler form.
(v) arg (ω ) = , arg (ω 2 ) =
3 3
EXAMPLE 8. Let O be the origin and point A be
(vi) Cube roots of −1 are −1, − ω, − ω 2 . represented by z. If OA is rotated through an angle π/ 2
(vii) Cube roots of unity lie on the unit circle | z| = 1 and in the anti-clockwise direction keeping the length of
divides its circumference into three equal parts. OA same, then what represents the new point?
a. −iz b. | z | i c. iz d. z
IMPORTANT RELATIONS Sol. c. Let z = cos θ + i sin θ
(i) x 2 + xy + y 2 = ( x − yω)( x − yω2 ) π
Now, on rotating through an angle , z becomes
(ii) x − xy + y = ( x + yω)( x + yω )
2 2 2 2
π π
(iii) x 3 + y 3 = ( x + y )( x + yω)( x + yω2 ) z = cos  + θ + i sin  + θ
2  2 
(iv) x 3 − y 3 = ( x − y )( x − yω)( x − yω2 ) = − sin θ + i cos θ = i sin θ + i cos θ
= i (cos θ + i sin θ) = iz
i+ 3
EXAMPLE 7. If α = , then what is the value of DE-MOIVRE’S THEOREM
2
1+α +α8 16
+α 24
+ α 32 ? A simple formula for calculating powers of complex
a. 0 b. 1 c. −ω d. −ω 2 numbers in the form of cos θ and sin θ is known as
De-Moivre’s theorem. If n ∈ Z (the set of integers), then
i+ 3
Sol. c. We have, α = (i) (cos θ + i sin θ) n = cos nθ + i sin nθ
2
 i + 3 −1+ 3i ω (ii) (cos θ − i sin θ) n = cos nθ − i sin nθ
⇒ iα = i   = ⇒ iα = ω ⇒ α = = − iω
 2  2 i 1
(iii) = (cos θ + i sin θ) −1
∴1 + α + α + α + α cos θ + i sin θ
= 1+ ( − iω) + ( − iω) + ( − iω) + ( − iω) = cos θ − i sin θ
= 1+ ω + ω + ω + ω
(iv) (sin θ ± i cos θ) n ≠ sin nθ ± i cos nθ
= 1+ ω + ω + 1+ ω = 0 + 1+ ω = − ω
n
 π  π 
Trigonometrical or Polar Form of a (v) (sin θ + i cos θ) n = cos  − θ + i sin  − θ 
  2   2 
Complex Number  nπ   nπ 
= cos  − nθ + i sin  − nθ
Let z = x + iy be a complex number represented by a  2   2 
point P( x, y) in the argand plane. Then, (vi) (cos θ + i sin φ) ≠ cos nθ + i sin nφ
n
Y
z = r (cos θ + i sin θ) P(x, y) z 2n − 1
where, r = | z | = x + y 2 2 |z| EXAMPLE 9. If z = cos θ + i sinθ, then , n ∈ Z is
z 2n + 1
 y θ equal to (where n is a rational)
and θ = arg ( z) = tan −1   X′ X
 x O M a. i cot nθ b. i tan nθ c. tan nθ d. cot nθ
This form of z is called the z n
− 1 (cos θ + i sin θ) n
−1
Sol. b. We have, =
polar form of z. Y′
z n
+ 1 (cos θ + i sin θ) n
+1
In general, polar form is cos 2nθ + i sin 2nθ − 1
=
z = r [cos( 2nπ + θ) + i sin( 2nπ + θ)] cos 2nθ + i sin 2nθ + 1
where, r =| z |, θ = arg ( z) and n ∈ N [using De-Moivre’s theorem]
MATHEMATICS Complex Numbers 35

(1 − 2sin nθ) + 2i sin nθ cos nθ − 1 3. If P( z) is the mid-point of A ( z1 ) and B ( z 2 ), then


=
( 2cos nθ − 1) + 2i sin nθ cos nθ + 1 z + z2
z= 1
i sin nθ cos nθ + i sin nθ 2
= [Q i = − 1]
cos nθ + i sin nθ cos nθ 4. If z1 , z 2 and z 3 are the vertices of a triangle, then
i sin nθ(cos nθ + i sin nθ) z + z2 + z3
= = i tan nθ centroid of ∆ABC = 1
cos nθ(cos nθ + i sin nθ) 3
z1 z1 1
1
nth ROOTS OF UNITY Area of triangle = z 2 z 2 1
2
Let z = 11/n z3 z3 1
Then, z = (cos 0 ° + i sin 0 ° )1/n = (cos 2rπ + i sin 2rπ )1/n , 5. General equation of straight line is az + az + b = 0
2rπ 2rπ where a is a complex number and b is a real number.
where r ∈ Z ⇒ z = cos + i sin ,
n n 6. The general equation of circle with centre at z 0 and
where r = 0, 1, 2, K, n − 1 [using De-Moivre’s theorem] radius r is given by | z − z 0 | = r
 2π 2π 
r or zz − z 0 z − z 0 z + z 0 z 0 − r 2 = 0
⇒ z = cos + i sin  = α r ; r = 0, 1, 2, K, n − 1
 n n z −2
2π EXAMPLE 12. If = 2 represents a circle, then
2π 2π i z −3
where α = cos + i sin =e n
n n centre and radius are
Also, 1 + α + α 2 + . . . + α n −1 = 0 a. 
10  2  −10  2
, 0 , b. 
 10  2
, 0 , c.  , 1 , d. None of these
n−1 n−1
2kπ n−1 2kπ  3  3  3  3  3  3
∑ α k
= 0 ⇒ ∑ cos
n
+ i ∑ sin
n
=0
Sol. a. Let z = x + iy
k= 0 k=0 k=0
x + iy − 2
Given equation becomes, =2
EXAMPLE 10. The n th roots of unity are in x + iy − 3
a. AP b. GP ⇒ |( x − 2) + iy | = 2|( x − 3) + iy |
c. HP d. None of these ⇒ ( x − 2) + y = 2 ( x − 3) + y
Sol. b. x n = 1 = (cos 0° + i sin 0° ) = cos 2rπ + i sin 2rπ = ei rπ
⇒ ( x − 2) + y = 4[( x − 3) + y ]
⇒ x = ei rπ n
; r = 0, 1, 2, … ⇒ x + 4 − 4x + y = 4( x + 9 − 6x + y )
20 32
Then, the roots are 1, e πi n, e πi n
, … which are clearly in ⇒ 3x + 3y − 20x + 32 = 0 ⇒ x + y − x+ =0
3 3
GP with common ratio e πi n.
Standard equation of circle is x + y + 2g x + 2fy + c = 0
EXAMPLE 11. If α is an nth root of unity, then −10 32
∴ g= , f = 0, c =
1 + 2α + 3α 2 +…+ n ⋅ α n −1 equals 3 3
∴Centre of circle = ( − g , − f ) = 
10 
n −n −n n2 , 0
a. b. c. d.  3 
1− α 1− α (1 − α )2 (1 − α )2
∴Radius = g + f − c =   + (0) −
10 32 100 32
S = 1+ 2α + 3α + ... + n ⋅ α n− = −
Sol. b. Let …(i)  3 3 9 3
Then, αS = α + 2α + 3α + … + (n − 1) α n − + nα n …(ii) 100 − 96 4 2
= = =
On subtracting Eq. (ii) from Eq. (i), we get 9 9 3
αn − 1
S − αS = − nα n [Qα n = 1] EXAMPLE 13. If z 1 lies on the circle | z − 1| = 1, then
α −1  z −2
−n Re 1  is equal to
S=  z1 
1− α
a. 0 b. 2 c. −1 d. −2
GEOMETRICAL APPLICATIONS Sol. a. Since, | z − 1| = 1 represents a circle whose diameter is
OF COMPLEX NUMBERS a line segment joining the points z = 0 and z = 2, if z
z −2 z −2
1. Distance between A ( z1 ) and B( z 2 ) is given by lies on a circle, then , i.e. is purely
z −0 z
AB =| z 2 − z1 | imaginary.
2. Distance of point P( z) from origin = | z |
36 NDA/NA Pathfinder

PRACTICE EXERCISE
1. What is the value of 1 + i + i + i + K + i , (1 − i )n
12. If the number is real and positive,
where i = −1? (1 + i )n −
(a) 0 (b) 1 then n is
(c) −1 (d) None of these (a) any integer (b) any even integer
(c) any odd integer (d) None of these
 1 + i  1 − i
2.   +  is equal to 13. What is the argument of (1 − sin θ) + i cos θ?
 2   2 
(a) 1 (b) 2 (c) 3 (d) 0
( i = −1)
π θ π θ π θ π θ
(a) − (b) + (c) − (d) +
  1  2 2 2 2 4 2 4 2
3. Evaluate i +   .
  i  14. If the cube roots of unity are 1, ω , ω , then the
(a) 2 (1 − i ) (b) 7 (i − 1) (c) 2 − 7 i (d) 8i + 4 roots of equation ( x − 1) + 8 = 0 are
1 + 7i (a) −1, 1 + 2ω, 1 + 2ω2 (b) −1, 1 − 2ω, 1 − 2ω2
4. is equal to
(2 − i) (c) −1, − 1, − 1 (d) None of these
(a) ( 2 , π / 4) (b) ( 2 , π / 2 ) 15. If ω is a cube root of unity, then the value of
(c) ( 2 , 3 π / 4) (d) None of these (1 + ω − ω ) (1 − ω + ω ) is equal to
3+i (a) 1 (b) 0 (c) 2 (d) 4
5. If z = , then ( z +i ) is equal to
2 ( −1 + i 3 ) ( −1 − i 3 )
16. + is equal to
(a) z (b) z2 (1 − i ) (1 + i )
(c) z3 (d) None of these (a) 32 (b) 64
 3 i  3 i (c) −64 (d) None of these
6. If z =  +  + −  , then
 1 + 2i 
 2 2  2 2 17. What is the conjugate of   ?
(a) Re ( z) = 0 (b) Im ( z) = 0  2+ i
(c) Re ( z) > 0, Im ( z) = 0 (d) Re ( z) > 0, Im ( z) < 0 7 24 7 24
(a) + i (b) − −i
25 25 25 25
1+ i 3 7 24 7 24
7. The argument of is (c) − + i (d) −i
3+1 25 25 25 25
π π π 18. If ω is imaginary cube root of unity, then
(a) (b) (c) (d) None of these
6 4 3  π
sin (ω +ω )π +  is equal to
8. If z = 1 + i 3, then|arg ( z )| +| arg ( z )| is equal to  4
π 2π π (a) −
3
(b) −
1
(c)
1
(d)
3
(a) (b) (c) 0 (d)
3 3 2 2 2 2 2

9. If ( a + ib) ( c + id ) ( e + if ) ( g + ih ) = A + iB, then 19. If ω is the imaginary cube root of unity, then
( a + b ) ( c + d ) ( e + f ) ( g + h ) is equal to what is ( 2 − ω + 2ω ) equal to?
(a) A 2 − B2 (b) A 2 + B2 (c) A 4 + B4 (d) A 4 − B4 (a) 3 ω
27
(b) −3 ω 27 2
(c) 327 (d) −327
20. The modulus of the complex number
10. What is the value of (1 − i 3 ) (cos θ + i sin θ )
z= is
 i+ 3   i − 3 2(1 − i ) (cos θ − i sin θ )
  +  + 1?
 −i + 3  i + 3 (a)
1
(b)
1
(c)
1
(d) None of these
2 2 2 3
(a) −1 (b) 0 (c) 1 (d) 2
n + 21. If α , β are complex cube roots of unity and
11. What is the value of ( − −1 ) + (i + i− ) ,
x = a + b, y = aα + bβ, z = aβ + bα, then xyz is equal
where n ∈ N ?
to
(a) 0 (b) 1 (c) i (d) −i
(a) (a + b )3 (b) a3 + b 3 (c) (a − b )3 (d) a3 − b 3
MATHEMATICS Complex Numbers 37

22. If ( 3 + i ) =2 ( a + ib), then a + b is equal to 35. If α is an nth root of unity other than unity
(a) 2 (b) 4 itself, then the value of 1 + α + α + K + α n − is
(c) 3 (d) None of these equal to
z −1 (a) 0 (b) 1 (c) −1 (d) None of these
23. If the number is purely imaginary, then | z − 2|
z +1 36. If = 2 represents a circle, then its radius is
(a) | z| > 1 (b) | z| = 1
| z − 3|
1 3 2
(c) | z| < 1 (d) None of these (a) 1 (b) (c) (d)
3 4 3
24. If z + z − = 1, then z + z− is equal to 3π
(a) i (b) −i (c) 1 (d) −1
37. If z = 1 + i tan α, where π < α < , then what is
2
3 + 2i sin θ | z| equal to?
25. If is purely real, then θ is equal to (a) sec α (b) − sec α
1 − 2i sin θ
nπ π π (c) sec 2 α (d) −sec 2 α
(a) nπ (b) (c) nπ ± (d) 2 nπ ±
2 3 4 38. The smallest positive integral value of n for
1 + x + iy  1 − i
n
26. If x + y = 1, then what is equal to? which   is purely imaginary with positive
1 + x − iy  1 + i
(a) x − iy (b) x + iy (c) 2x (d) −2iy imaginary part is
1 + 2i 2− i (a) 1 (b) 3 (c) 4 (d) 5
27. If z = − , then what is the value of
2 − i 1 + 2i 39. The points z , z , z and z in the complex plane
z + zz ? ( i = −1) are the vertices of a parallelogram taken in order
if and only if
(a) 0 (b) –1 (c) 1 (d) 8
(a) z1 + z4 = z2 + z3 (b) z1 + z3 = z2 + z4
a + bω + cω a + bω + cω (c) z1 + z2 = z3 + z4
28. + is equal to (d) None of these
c + aω + bω b + cω + aω
40. If 2x = 3 + 5i, then what is the value of
(a) 1 (b) 0 (c) 2 (d) −1 2x + 2x − 7x + 72?
29. The number of solutions of the equation z = z is (a) 4 (b) −4 (c) 8 (d) −8
(a) 2 (b) 3 41. The smallest positive integer for which
(c) 4 (d) None of these (1 + i ) n
= (1 − i ) n
is
( 3 + i) I. n = 2 II. n = 8
30. If z = , then arg ( z ) is equal to
(1 − i ) (a) Only II (b) Only I
2π π 2π (c) Both (a) and (b) (d) Neither I nor II
(a) (b) − (c) − (d) None of these
3 3 3 1 + 2i 2+ i 1+ i
42. If z = , z = and z = in which
31. The solution of the equation| z| − z = 1 + 2i is 1− i 3− i 2i + 1
3 3 3 quadrant does the each complex number lies.
(a) − 2i (b) + 2i (c) 2 − i (d) None of these
2 2 2 I. z in second quadrant and z in second quadrant.
II. z in second quadrant and z in first quadrant.
32. The locus of the point z = x + iy satisfying
III. z in fourth quadrant and z in first quadrant.
z − 2i
= 1 is (a) Only I (b) Only II
z + 2i (c) Only III (d) Both I and III
(a) X-axis (b) Y-axis (c) y = 2 (d) x = 2
43. In the argand diagram, if Y
33. The complex number z satisfying the equations O , P and Q represent P
z − 4 = z − i − z + 5i = 0 is respectively the origin, the
(a) 3 − i , − 3 − i (b) 2 3 − 2 i , − 2 3 − 2 i complex numbers ( z ) and Q
(c) − 3 + i , − 3 − i (d) − 2 3 + 2 i , − 2 3 − 2 i ( z + iz ), then the ∠OPQ is X′ X
2π π π
34. The common roots of the equations I. II. III.
z + 2z + 2z + 1 = 0 and z +z + 1 = 0 are 3 2 3 Y′
(a) Only I (b) Only II
(a) − 1, ω (b) − 1, ω 2
(c) Only III (d) None of these
(c) ω, ω2 (d) None of these
38 NDA/NA Pathfinder

3 +i Directions (Q. Nos. 52-55) Let z = x + iy be a


Directions (Q. Nos. 44-45) If z1 = and
complex number such that | z| = 1 and z ≠ 1.
3 −i
i( 3 + i) 4
52. If arg ( z z ) = 0 and|z | =|z|, then
z2 = . Then,
(2 − i2 3) 2
(a) z1 + z = 0 (b) z1 z = 1
(c) z1 = z (d) None of these
44. amp ( z ) + amp ( z ) is equal to 1
π π π
(d) −
π 53. If z = , then z lies on
(a) (b) (c) 1− z
3 6 9 6
(a) a parabola (b) a straight line
45. Which of the following is correct? (c) a circle (d) an ellipse
(a) z1 < z2 (b) z1 > z2
1+ z
(c) z1 ≥ z2 (d) None of these 54. The complex number is equal to
1+ z
Directions (Q. Nos. 46-48) If n ∈ Z (the set of (a) z (b) z
integers), then (c) z + z (d) None of these
(cos θ + i sin θ) n = cos nθ + i sin nθ and 55. Consider the following statements
I. z lies on a unit circle with centre at the origin.
(cos θ − i sin θ) = cos nθ − i sin nθ
n
II. The reciprocal of z is z.
 1 + cos φ + i sin φ 
n
46. If   = u + iv, where u and v all
Which of the above statement(s) is/are correct?
 1 + cos φ − i sin φ  (a) Only I (b) Only II
real numbers, then u is (c) Both I and II (d) Neither I nor II

(c) cos   (d) sin  


nφ nφ
(a) n cos φ (b) cos nφ
2  2  Directions (Q. Nos. 56-60) Suppose ω and ω 2 are the
1 1 complex cube root of unity which are given as
47. If x + = 2 cosθ, then x n + n is equal to
−1 + 3i −1 − 3i
x x ω= and ω 2 = .
(a) 2cos nθ (b) 2sin nθ (c) cos nθ (d) sinnθ 2 2
Also, 1 + ω + ω 2 = 0 and ω3 = 1 .
48. If n is a positive integer, then (1 + i ) + (1 − i )n is n

equal to 56. The value of expression


nπ nπ
(a) ( 2 )n − 2 cos   (b) ( 2 )n − 2 sin    1  1  1  1
 4  4 2 1 +  1 +  + 3 2 +  2 + 
nπ nπ  ω  ω   ω  ω 
(c) ( 2 )n + 2 cos   (d) ( 2 )n + 2 sin  
 4  4
 1  1  1  1
Directions (Q. Nos. 49-51) If z1 , z 2 and z3 represent + 4  3 +   3 +  + K + ( n + 1)  n +   n + ,
 ω  ω   ω  ω 
the vertices of an equilateral triangle, then
z12 + z 22 + z32 = z1 z 2 + z 2z3 + z3z1 is equal to
n(n2 + 2 ) n(n2 − 2 )
(a) (b)
49. If z is the circumcentre of equilateral triangle 3 3
with vertices z , z and z , then z +z +z , n2 (n + 1)2 + 4n
(c) (d) None of these
equals to 4
(a) z02 (b) 3 z02 (c) 2 z02 (d) 0
57. If (1 + ω ) = A + Bω, then ( A, B) is equal to
50. The origin and the roots of the equation (a) (0, 1) (b) (1, 1)
z + pz + q = 0 form an equilateral triangle, if (c) (1, 0) (d) (−1, 1)
(a) p2 = q (b) p2 = 3 q (c) q 2 = 3 p (d) q 2 = p
58. The value of expression
51. Consider the following statements  1 i 3  1 i 3
I. The cube roots of unity form an equilateral 4 + 5 − +  + 3 − +  is equal to
 2 2   2 2 
triangle.
(a) 1 − i 3 (b) − 1 + i 3
II. z , z and z are the vertices of equilateral
1 1 1 (c) i 3 (d) − i 3
triangle, if + + = 0.
(z − z ) (z − z ) (z − z ) 59. arg (iω ) + arg ( iω ) is equal to
Which of the above statement(s) is/are correct? (a) 0 (b) π/2
(a) Only I (b) Only II (c) Both I and II (d) Neither I nor II (c) π (d) π/ 3
MATHEMATICS Complex Numbers 39

PREVIOUS YEARS’ QUESTIONS 70. Let z = x + iy, where x , y are real variables and
4 + 2i i = −1. If | 2z − 1|=| z − 2|, then the point z
60. If A + iB = , where i = −1, then what is the describes e 2014 I
1 − 2i
(a) a circle (b) an ellipse
value of A? e 2012 I
(c) a hyperbola (d) a parabola
(a) − 8 (b) 0 (c) 4 (d) 8
71. If| z + z | =| z − z |, then the locus of z is e 2014 I
61. If z = − z , then which one of the following is (a) a pair of straight lines (b) a line
correct? e 2012 I
(c) a set of four straight lines (d) a circle
(a) The real part of z is zero n +
(1 + i )
(b) The imaginary part of z is zero 72. What is equal to, where n is a natural
(c) The real part of z is equal to imaginary part of z (1 − i ) n +
(d) The sum of real and imaginary parts of z is z number and i = −1? e 2014 II

2+i (a) 2 (b) 2 i (c) −2 i (d) i


62. What is the modulus of , where i = −1?
2−i  3 + i
e 2012 II 73. What is   equal to, where i = −1?
(a) 3 (b) 1/2 (c) 1 (d) None of these  3 − i e 2014 II
63. Consider the following statements (a) 1 (b) 1/6 (c) 6 (d) 2
I. (ω + 1) + ω = 0 74. If z is a complex number such that | z| = 4 and
II. (ω + 1) = p for some prime number p, 5π
arg ( z ) = , then what is z equal to?
where, ω ≠ 1 is a cubic root of unity. 6 e 2014 II
(a) 2 3 + 2 i (b) 2 3 − 2 i (c) −2 3 + 2i (d) − 3 + i
Which of the above statement(s) is/are correct?
where, i = −1
e 2012 II
(a) Only I (b) Only II 75. If 1, ω and ω are the cube roots of unity, then
(c) Both I and II (d) Neither I nor II the value of (1 + ω )(1 + ω )(1 + ω )(1 + ω ) is

∑ (in + in +
e 2015 I
64. The value of the sum ), where i = −1
(a) −1 (b) 0 (c) 1 (d) 2
n=
is e 2012 II −2 (1 + 2i )
76. If z= , where i = −1, then the
(a) i (b) −i (c) 0 (d) i − 1 (3 + i)
argument θ ( − π < θ ≤ π ) of z is e 2015 I
65. What is the value of −i, where i = −1? e 2013 I 3π π 5π 3π
(a) (b) (c) (d) −
1− i 1+ i 1− i 1+ i 4 4 6 4
(a) ± (b) ± (c) ± (d) ±
2 2 2 2
77. What is the square root of i, where i = −1?
66. What is the argument of the complex number e 2015 I
( −1 − i ), where i = −1? e 2013 I 1+ i 1− i 1+ i
(a) (b) (c) (d) None of these
5π 5π 3π 2 2 2
(a) (b) − (c) (d) None of these
4 4 4  π  π 
67. What is one of the square roots of 3 + 4i, where  sin 6 + i 1 − cos 6  
i = −1?
78. What is   , where i = −1,
e 2013 II  sin π − i 1 − cos π  
  
(a) 2 + i (b) 2 − i (c) −2 + i (d) − 3 − i 6  6  
68. What is the argument of the complex number equal to? e 2015 I
(1 + i ) ( 2 + i ) (a) 1 (b) −1 (c) i (d) −i
, where i = −1?
3− i e 2014 I 79. What is the real part of (sin x + i cos x ) , where
π π π i = −1?
(a) 0 (b) (c) − (d) e 2015 I
4 4 2 (a) −cos 3x (b) −sin3x
69. If P and Q are two complex numbers, then the (c) sin3x (d) cos 3x
modulus of the quotient of P and Q is e 2014 I
80. ( x − 1) can be factorised as e 2015 I
(a) greater than the quotient of their moduli (a) (x − 1)(x − ω)(x + ω2 ) (b) (x − 1)(x − ω)(x − ω2 )
(b) less than the quotient of their moduli (c) (x − 1)(x + ω)(x + ω2 ) (d) (x − 1)(x + ω)(x − ω2 )
(c) less than or equal to the quotient of their moduli
where, ω is one of the cube roots of unity.
(d) equal to the quotient of their moduli
40 NDA/NA Pathfinder

81. If z and z are complex numbers with |z | =|z |, Directions (Q. Nos. 86-87) Let z be a complex number
then which of the following is/are correct?  z − 4
satisfying   = 1 and 
 z  = 3
I. z = z  z − 8   z − 2  2 e 2016 I

II. Real part of z = Real part of z 86. What is| z| equal to?
III. Imaginary part of z = Imaginary part of z (a) 6 (b) 12
(c) 18 (d) 36
Select the correct answer using the code given
 z − 6
 equal to?
below e 2015 II 87. What is
(a) Only I (b) Only II (c) Only III (d) None of these  z + 6
82. If the point z = 1 + i , where i = − 1, is the (a) 3 (b) 2
(c) 1 (d) 0
reflection of a point z = x + iy in the line
iz − iz = 5, then the point z is e 2015 II 88. Suppose, ω is a cube root of unity with ω ≠ 1.
(a) 1 + 4i (b) 4 + i (c) 1 − i (d) − 1 − i Suppose, P and Q are the points on the complex
plane defined by ω and ω . If O is the origin, then
83. zz + ( 3 − i ) z + ( 3 + i ) z + 1 = 0 represents a circle
what is the angle between OP and OQ? e 2016 I
with e 2015 II
(a) 60° (b) 90°
(a) centre (− 3, − 1) and radius 3 (c) 120° (d) 150°
(b) centre (− 3, 1) and radius 3
(c) centre (− 3, − 1) and radius 4 89. Suppose, ω and ω are two distinct cube roots of
(d) centre (− 3,1) and radius 4 unity different from 1. Then, what is (ω − ω )
equal to? e 2016 I
Directions (Q. Nos. 84-85) Let z1, z 2 and z3 be non-zero (a) 3 (b) 1
complex numbers satisfying z 2 = i z , where i = −1. (c) −1 (d) −3
e 2016 I −
 1 i 
84. What is z + z + z equal to? 90. If z = x + iy =  −  , where i = −1, then
 2 2
(a) i (b) −i (c) 0 (d) 1
z− 2
85. Consider the following statements what is the fundamental amplitude of ?
z−i 2
I. z z z is purely imaginary.
e 2016 I
II. z z + z z + z z is purely real. π
(a) π (b)
Which of the above statement(s) is/are correct? 2
(a) Only I (b) Only II π π
(c) (d)
(c) Both I and II (d) Neither I nor II 3 4

ANSWERS
1 b 2 b 3 a 4 c 5 c 6 b 7 c 8 b 9 b 10 b
11 c 12 d 13 d 14 b 15 d 16 c 17 d 18 b 19 d 20 a
21 b 22 b 23 b 24 d 25 a 26 b 27 a 28 d 29 c 30 c
31 a 32 a 33 b 34 c 35 a 36 d 37 a 38 b 39 b 40 a
41 b 42 b 43 b 44 b 45 d 46 b 47 a 48 c 49 b 50 b
51 c 52 c 53 b 54 a 55 c 56 c 57 b 58 c 59 c 60 b
61 a 62 c 63 b 64 d 65 a 66 a 67 a 68 d 69 d 70 a
71 a 72 a 73 a 74 d 75 c 76 d 77 c 78 c 79 b 80 b
81 d 82 a 83 a 84 c 85 c 86 a 87 d 88 c 89 d 90 a
MATHEMATICS Complex Numbers 41

HINTS AND SOLUTIONS 


1. (b) 2. (b) 11. (c) ( − −1)4 n + 3
+ (i 41 + i −257 )9 20. (a) Q | e iα | = 1 = | e − iα |
1 + 3 |e iθ |
9
+  i + 
3 1 2 1
 1 
25
= ( − i )4 n + 3
[Q i 4 = 1] ∴ |z |= = =
3. (a) We have, i 18 +     i 2 1 + 1|e − i θ | 2 2 2
 i  

= ( −1)4 n + 3
(i ) 4 n (i ) 3 + (i − i ) 9 21. (b) Let α = ω and β = ω2
= [ −1 + ( −i )25 ]3 = [ −1 − i ]3 = − ( 1) ( −i ) + 0 = i
= −[ 1 + i ]3 Then, xyz =( a + b )( aω + bω2 ) ( aω2 + bω)
n−2
1− i 
=− [ 1 + 3i − 3 − i ] =− [ −2 + 2i ] 12. (d) E =   ( 1 − i )2 = a3 + b3
 1+ i 
22. (b) ( 3 + i )100 = 299 ( a + ib )
= 2( 1−i ) n−2
=  − 
2i
( −2i ) = 2( −i )n − 1 ⇒ ( − 2iω2 )100 = 299 ( a + ib )
1 + 7i 1 + 7i 1 + 7i  2
4. (c) = = ⇒ 2100 ω2 = 299 ( a + ib )
( 2 − i )2 4 + i 2 − 4i 3 − 4i = 2 [( −i )2 ](n − 1 )/ 2 = 2( −1)(n − 1 )/ 2
 −1 − 3i 
( 1 + 7i )(3 + 4i ) Since, E is real and positive. ⇒ 2  = a + ib
= n− 1
(3)2 − ( 4i )2 Therefore, = 2λ  2 
2 ⇒ a = −1 and b = − 3
−25 + 25i ∴ n = 4λ + 1
= = −1+ i ∴ a 2 + b 2 = ( −1)2 + ( − 3 )2 = 4
25 i.e. odd of this type but not any odd.
z − 1  z − 1
∴ −1 + i = ( 2 , π − π / 4) =  2 ,
3π 13. (d) Let z = ( 1 − sin θ) + i cos θ 23. (b)  +  =0
  z + 1  z + 1
 4   Im(z ) 
⇒ arg(z ) = tan −1   ⇒
z−1 z −1
+ =0
−1 + i 3  Re(z ) 
5. (c) iz = =ω z+ 1 z + 1
2  1 + tan θ  (z − 1)(z + 1) + (z − 1)(z + 1)
    ⇒ =0
⇒ z =   ω = − iω  cos θ  = tan −1
1
...(i) = tan −1

2 (z + 1)(z + 1)
i   1 − sin θ  θ
   1 − tan  2( 1−|z |2 )
 2
∴ z101 + i 103 = ( −i )101 ω101 + i 103 ⇒ = 0 ⇒ 1−|z|2 = 0 ⇒ |z|= 1
 cos θ + sin θ  |1 + z|2
= − iω − i = − i ( 1 + ω )
2 2
−1  2 2 z + z −1 = 1 ⇒ z 2 − z + 1 = 0
= tan  24. (d)
= − i ( − ω) = iω θ θ
 cos − sin  z = − ω or −ω2
Now, (z101 + i 103 )105 = i 105 ω105 = i ⋅ ω3  2 2
For z = − ω , we have
= i (iz )3 = i 4z 3 = z 3 −1   π θ π θ
= tan + = +
  4 2   4 2 z100 + z −100 = ( − ω)100 + ( − ω)−100
tan
5 5
 3+i  3 −i  = ω + 1/ ω = ω + ω2 = − 1
6. (b) z =   + 
 2   2  14. (b) ( x − 1) + 8 = 0 or ( x − 1) = − 8
3 3
For z = − ω , we have
2

∴ x − 1 = ( −8)1 / 3 = − 2( 1)1 / 3 z100 + z −100 = ( − ω2 )100 + ( − ω2 )−100


= ( −iω)5 + (iω2 )5 = −iω2 + iω 1 1
= − 2, − 2ω, − 2ω2 = ω200 + 200 = ω2 + 2
= i [ω − ω2 ] = i (i 3 ) = − 3 Hence, x = − 1, 1 − 2ω, 1 − 2ω2 ω ω
7. (c) = ω2 + ω = − 1
15. (d) ( 1 + ω − ω2 )( 1 − ω + ω2 )
π π 2π 25. (a) For purely real number
8. (b) |arg (z )|+ |arg(z )|= + − = = ( − ω2 − ω2 )( − ω − ω)
3 3 3  3 + 2i sin θ  =  3 + 2i sin θ 
= ( −2ω2 ) × ( − 2ω) = 4ω3 = 4    
 1 − 2i sin θ   1 − 2i sin θ 
9. (b) Taking mode on both sides, we get
( 2e 2 πi / 3 )15 ( 2e −2 πi / 3 )15 3 + 2i sin θ 3 − 2i sin θ
| a + i b || c + i d || e + i f || g + i h | 16. (c) − πi / 4 20
+ ⇒ =
( 2e ) ( 2 e πi / 4 )20 1 − 2i sin θ 1 + 2i sin θ
=|A + iB|
= 25 [ e15 πi + e −15 π i ] ⇒3 + 8i sin θ − 4 sin 2 θ =3 −8i sin θ − 4 sin 2 θ
⇒ ( a 2 + b 2 ) ( c2 + d 2 ) ( e 2 + f 2 )
= 25 ⋅ 2 cos 15 π = 64 cos π = − 64 ⇒ 16i sinθ = 0 ⇒ θ = n π
( g 2 + h2 )
= A + B 2 2
17. (d) 26. (b)
1 + 2i 1 + 2i 2 + i
i + 3 ( 3 + i ) / 2 −iω 18. (b) Q ω13 + ω20 = ω + ω2 = − 1 27. (a) Let ω = = ×
10. (b) = = = − ω2 2 −i 2 −i 2+i
−i + 3 ( 3 − i ) / 2 iω2 π 3π
∴E = sin  − π +  = − sin
1
=− 2 + 5i − 2
 4 4 2 = =i
i − 3 ( − 3 + i ) / 2 −iω2 4+ 1
and = = =ω
i + 3 ( 3 + i) / 2 −iω 19. (d) ( 2 − ω + 2ω2 )27 1
200 200 ∴ z = i − = i + i = 2i
 i+ 3  i − 3 = { 2 − ( −1 − ω2 ) + 2ω2 }27 = {3 + 3ω2 }27 i
∴   +   + 1
 −i + 3  i + 3  = (3)27 ( 1 + ω2 )27 = (3)27 ( − ω)27 ∴ z 2 + zz = z (z + z )
= − (3)27 = 2i ( 2i − 2i ) = 0
= ( −ω2 )200 + ω200 + 1 = 0
42 NDA/NA Pathfinder

28. (d) The given expression can be written as 34. (c) We have, z 3 + 2z 2 + 2z + 1 = 0. ⇒ z lies in first quadrant.
2
ω ( a + bω + cω2 ) a + bω + cω2 ⇒ (z + 1)(z 2 + z + 1) = 0 1 + i 1 − 2i 1 − 2i + i − 2i 2 3 − i
= + z = × = =
( cω + aω 2 + b ) b + cω + aω 2 Its roots are − 1, ω and ω2 . The root
3
2i + 1 1 − 2i 1+ 4 5
( a + bω + cω2 )(ω + 1) z = − 1 does not satisfy the equation ⇒ z lies in fourth quadrant.
= z1985 + z100 + 1 = 0 but z = ω and
3
b + cω + aω 2 43. (b) Let z = r(cos θ + i sin θ)
z = ω2 satisfy it. Hence,ω and ω2 are the
( a + bω + cω2 )( −ω2 ) z + iz = r(cos θ + i sin θ)
= [Q 1 + ω = − ω2 ] common roots.
b + cω + aω 2 + ir(cos θ + i sin θ)
35. (a)
−( aω2 + b + cω) = r[(cos θ − sin θ) + i (sin θ + cos θ)]
= = −1 36. (d) | z − 2 | = 2 | z − 3 |
b + cω + aω 2 π π
⇒ |( x − 2) + iy | 2 = 4 |( x − 3) + iy |2 = 2r  cos  θ +  + i sin  θ +  
  4  4  
29. (c) z 2 = z ⇒ ( x + iy )2 = x − iy ⇒ ( x − 2)2 + y 2 = 4 [( x − 3)2 + y 2 ]
x 2 − y 2 − x + i ( 2xy + y ) = 0 ⇒ 3x 2 + 3 y 2 − 20x + 32 = 0 In ∆OPQ,
⇒ x − y 2 − x = 0 and 2xy + y = 0
2
20 32 PQ 2 = r 2 + ( 2r )2 − 2r( 2r ) cos π / 4
⇒ x2 + y2 − x + =0
Now, 2xy + y = 0 gives 3 3 = r 2 + 2r 2 − 2r 2 = r 2
y = 0 or x = − 1/ 2 2 PQ = r 2 ⇒ PQ = r
2
Radius =  − 10  − 32 = 2
when, y = 0, x 2 − y 2 − x = 0   ∴ ∆OPQ is right angle triangle.
 3 3 3
gives x 2 − x = 0 or x = 0, 1 π
∴ ∠OPQ = 90° = .
when, x = − 1/ 2, x 2 − y 2 − x = 0 gives 37. (a) Given, z = 1+ i tan α , 2
1/ 4 − y 2 + 1/ 2 = 0 ⇒ y = ± ( 3 / 2) where π < α < 3 π / 2 3+i
Sol. (Q. Nos. 44-45) Given, z =
Hence, there are four solutions. ⇒ |z|= 1 + tan 2 α ⇒ |z|= sec 2 α 1
3 −i
π 3+i 3+i
i
( 2e 6 )17 1 17 π i 25 π i ⇒ |z|= sec α Q π < α < 3 π  ⇒ z = ×
30. (c) z = π = e 6 ⋅e 2  2 
1
3 −i 3+i
(
− i
2e 4 )50 28
3 − 1 + 2 3i 1 3
17 π 25 π 46 π 1− i 
n
 ( 1 − i )( 1 − i ) 
n
= = + i
∴ amp (z ) = + = 38. (b) Let z =   =  3+ 1 2 2
6 2 3  1+ i  ( 1 + i )( 1 − i )   3 / 2
2π 2π 1 − 1 − 2i 
n amp (z ) = tan −1  
= 16 π − =− =   = ( −i )
n 1
 1/ 2 
3 3  2  π
= tan −1 ( 3 ) =
31. (a) Let z = x + iy , |z | − z = 1 + 2i For n = 3, i = ( −1)3 = i 3
x 2 + y 2 − ( x + iy ) = 1 + 2i Hence, required value of n = 3 i ( 3 + i )4 i ( 2 + 2 3i )2
Now, z = =
⇒ ( x 2 + y 2 − x ) − iy = 1 + 2i 39. (b) Let ABCD be a parallelogram such
2
(2 − i 2 3) 2
4( −2 − 2 3i )
that affixes of A,B,C,D are z , z , z , z , 3+i 3 i
⇒ x + y − x = 1 and y = −2
2 2
1 2 3 4 = = −
respectively. Then, 1 + 3i 2 2
Now, x 2 + y 2 − x = 1 AB = DC ⇒ z − z = z − z −1  − 1 / 2  π
2 1 3 4 amp (z ) = tan   =−
⇒ x2 + 4 = 1+ x [Q y = − 2] ⇒ z +z =z +z 2  3 / 2 6
2 4 1 3
π π π
⇒ x 2 + 4 = 1 + x 2 + 2x ⇒ x = 3 / 2 40. (a) ( 2x − 3) = (5i )2 = −25
2
44. (b) amp (z ) + amp (z ) = − =
1 2
3 6 6
∴ The solution is 3 / 2 − 2i. ⇒ 2x 2 − 6x + 17 = 0 45. (d) Complex numbers do not follow the
z − 2i Dividing, 2x 3 + 2x 2 − 7x + 72 by rule of inequality, i.e. inequality is
32. (a) = 1 ⇒ |z − 2i |= |z + 2i |
z + 2i 2x 2 − 6x + 17, we get quotient = x + 4 meaningless in complex numbers.
⇒ |x + i ( y − 2)|= |x + i ( y + 2)| and remainder = 4 n
 
∴ 2x 3 + 2x 2 − 7x + 72 1 + cos φ + i sin φ 
⇒ x + ( y − 2) = x + ( y + 2)
2 2 2 2
46. (b)  = u + iv
= ( x + 4)( 2x 2 − 6x + 17) + 4 = 4  1 + cos φ − i sin φ 
⇒ y2 + 4 − 4y = y2 + 4 + 4y ⇒ y = 0  
2n 2n n
∴ Locus of z lies on X-axis. 41. (b) 
1− i 
= 1 , 
1− i 1− i 
× =1  2 cos 2 φ + 2i sin φ cos φ 
  
33. (b) Let z = x + iy
 1+ i   1+ i 1− i  ⇒ 2 2 2  = u + iv
2 φ φ φ
−2i 
2n
 2 cos − 2 sin cos 
⇒ 
|x + iy| = 4, i.e. x 2 + y 2 = 16 …(i) i
 = 1 ⇒ ( − i )n = 1  2 2 2
and |x + iy − i |= |x + iy + 5i |  2  n
 2 cos φ  cos φ + i sin φ  
∴ Smallest positive value of n = 2   
or x 2 + ( y − 1)2 = x 2 + ( y + 5)2
⇒ 2 2 2   = u + iv
1 + 2i 1 + i 
i.e. y = −2 …(ii) 42. (b) We have, z = × φ  φ
 2 cos  cos − i sin   φ
Putting y = − 2 in Eq. (i), x 2 + 4 = 16 1− i
1
1+ i  2 2 2  
or x = ± 2 3. 1 + 3i − 2 −1 3i  e iφ / 2 
n
= = +
Hence, the complex numbers satisfying 1+ 1 2 2 ⇒  − i φ / 2  = u + iv ⇒ e inφ = u + iv
e 
the given equations are ⇒ z lies in second quadrant.
1
z = 2 3 − 2i and z = − 2 3 − 2i 2 + i 3 + i 6 + 2i + 3i − 1 1 + i ⇒ cos nφ + i sin nφ = u + iv
1 2 z = × = =
2
3 −i 3 + i 9+ 1 2 ⇒ u = cos nφ, v = sin φ
MATHEMATICS Complex Numbers 43

334 365
47. (a) x + 1/ x = 2 cosθ ∴ Statement I is correct.  1 i 3  1 i 3
58. (c) 4 + 5 − +  +3  − + 
⇒ x 2 − 2x cosθ + 1= 0 ⇒ x = cos θ ± i sin θ 1 1 1  2 2   2 2 
Now, + =
⇒ x n =(cos θ ± i sin θ)n = cos nθ ± i sin nθ z −z z −z z −z
1 1 2 2 3 1 3 = 4 + 5ω334 + 3ω365
⇒ = cos θ m i sin nθ
xn ⇒ zz −zz − z2 +zz = 4 + 5ω + 3ω2
1 2 1 3 2 2 3
1 = z2 + z2 − 2z z  1 i 3  1 i 3
∴ x n + n = 2 cos nθ 1 3 1 3 = 4 + 5 − +  + 3− − 
x ⇒ z2 + z2 + z2 = z z + z z + z z  2 2   2 2 
1 2 3 1 2 2 3 1 3
48. (c) ( 1 + i )n + ( 1 − i )n Thus, z , z , z are vertices of equilateral =i 3
1 2 3
n n triangle. 2π
=  2 
1 i    1 − i  59. (c) We have, arg (ω) =
+   +  2 
  2 2   2 2 ∴ Statement II is correct. 3
4π 2π
π π
n
52. (c) We have, arg (z z ) = 0 and arg (ω2 ) = or arg (ω2 ) = −
= 2n / 2  cos + i sin  1 3 3
 4 4  ⇒ arg (z ) + arg (z ) = 0
π π
n 1 ∴ arg (iω) + arg (iω2 )
+ 2n / 2  cos − i sin  ⇒ arg (z ) = − arg (z )
 4 4  1
= arg(i ) + arg(ω) + arg(i ) + arg(ω)2
⇒ arg (z ) = arg (z ) and |z | = |z | = |z |
nπ nπ π 2π π 2π π π
= 2n / 2  cos
1 1
+ i sin Hence, z = z = + + − = + = π
 4 4 1
2 3 2 3 2 2
+ cos
nπ nπ
− i sin  1 1 z2 − 1 4 + 2i 1 + 2i
53. (b) z = ⇒ z = 1− = 60. (b) Given, A + iB = ×
4 4  2
1−z z z 1 − 2i 1 + 2i
n/ 2 + 1  nπ  n+ 2  nπ  2 2
=2 cos   = ( 2 ) cos   z −1 z −1 |z − 1| 4 + 10i − 4 10i
 4   4  ∴|z | =
2
⇒1= 2 =
2 = = = 2i
z z |z | 1+ 4 5
49. (b) In an equilateral triangle 2 2 2 [Q i 2 = − 1]
circumcentre, centroid and orthocentre [Q |z | = 1] ⇒ A + iB = 0 + i ⋅ 2
are coincident.
z +z +z ⇒ |z | = |z − 1| ⇒ A = 0 and B = 2
2 2
∴z = 1 2 3 ⇒z + z + z = 3z
Let z = x + iy , |x + iy | = |( x − 1) + iy|
0
3 1 2 3 0
2 61. (a) Given that, z = − z
⇒ x 2 + y 2 = ( x − 1)2 + y 2 ⇒ x = 1/ 2
Now, z , z , z are the affixes of the Let z = x + iy ⇒( x + iy ) = − ( x + iy )
1 2 3
vertices of an equilateral triangle. ∴ z lies on straight line.
2
( x + iy ) = − ( x − iy )
∴ z2 + z2 + z2 = z z + z z + z z 54. (a) We have,|z | = 1⇒|z |2 = 1 ⇒ zz = 1
1 2 3 1 2 2 3 3 1 [Q z = x + iy , z = x − iy ]
1 + z zz + z z(z + 1)
(z + z + z )2 = 3(z z + z z + z z ) ∴ = = =z ⇒ ( x + iy ) = ( − x + iy )
1 2 3 1 2 2 3 3 1 1+ z 1+ z 1+ z
⇒ (3z )2 = 3(z z + z z + z z ) ⇒ 2x = 0 ⇒ x = 0
0 1 2 2 3 3 1 55. (c) |z|= 1 represents a unit circle with
⇒ z z + z z + z z = 3z 2 centre at origin. Hence, the real part of z is zero.
1 2 2 3 3 1 0
⇒ z 2 + z 2 + z 2 = 3z 2 Hence, Statement I is correct. 62. (c)
1 2 3 0 1 z z 63. (b) I. LHS = (ω10 + 1 )7 + ω
50. (b) Let z , z be the roots of Reciprocal of z = = = =z
z zz |z |2
1 2
= [(ω3 )3 ω + 1]7 + ω
z 2 + pz + q = 0.
Then, z + z = − p , z z = q Hence, Statement II is correct. = ( 1 + ω) 7 + ω = ( − ω 2 ) 7 + ω
1 2 1 2
Since, z , z , z ( = 0) form an equilateral 56. (c) In general, rth term of the given [Q 1 + ω + ω2 = 0, ∴ 1 + ω = − ω2 ]
1 2 3 expression
triangle. = − ω14 + ω = − (ω3 )4 ω2 + ω
T r = ( r + 1)  r +   r + 2 
1 1
∴z 2 + z2 + z2 =zz + z z + z z  = − ω 2 + ω = ( 1 + ω) + ω
1 2 3 1 2 2 3 3 1 ω  ω 
⇒ z2 + z z2 =zz [Q z = 0]
= r 3 + r 2  2 + + 1 = 1 + 2ω ≠ 0
1 2 2 1 2 3 1 1
⇒ (z + z )2 = 3z z ω ω  So, Statement I is false.
1 2 1 2
+ r  1 + 2 +  + 1

⇒ ( − p ) = 3q ⇒ p 2 = 3q
2 1 1 II. LHS = (ω105 + 1)10 = [(ω3 )35 + 1]10
 ω ω [Q ω3 = 1]
51. (c) Let P, Q, R represent 1, ω and ω2 ,
respectively. = r3 + 1 [Q 1 + ω + ω2 = 0] = ( 1 + 1) = 2 = p
10 10 10
n

PQ = |1 − ω | =
2
3 +  3
2
= 3
∴ Sn = ∑ ( r 3 + 1) which is true for prime number 2.
    So, Statement I is false and
 2  2  r =1
Statement II is true.
n ( n + 1)2
2
n2 ( n + 1)2 + 4n 13 13 13
QR = |ω − ω2 | = 3 and = + n =
∑ (i + i n +1 ) = ∑i ∑i
n +1
4 4 64. (d) n n
+
2 2
RP = |1 − ω2 | = 3 +  3 = 3 57. (b) We have, ( 1 + ω)7 = A + Bω
n =1 n =1 n =1
     1 − i 13  2  1− i 
13
 2  2  = i
⇒ ( − ω2 )7 = A + Bω ⇒ − ω14 = A + Bω  +i  
 1− i   1− i 
∴ PQ = QR = RP
⇒− ω2 = A + Bω ⇒ 1 + ω = A + Bω
1− i   1− i 
Thus, points representing 1, ω, ω2 form
On comparing, we get A = 1, B = 1 = i   −  =i −1
an equilateral triangle.  1− i   1− i 
44 NDA/NA Pathfinder

65. (a) −i = e − i π / 2 − i θ 70. (a) Given that, z = x + iy ; x , y ∈ R ∴ tan α =


1 Q tan α = b 
[Qe = cos θ − i sin θ] 3  a 
π 1 π We have, 2z − 1 = z − 2
−i × −i
=± e 2 2 =± e 4 ∴ z = − 3 + i [Q a < 0 and b > 0]
π π ⇒ 2 ( x + iy ) − 1 = x + iy − 2 75. (c) ( 1 + ω)( 1 + ω2 )( 1 + ω4 )( 1 + ω8 )
= ±  cos   − i sin   
  4   4   = ( 1 + ω + ω2 + ω3 )( 1 + ω3 ⋅ ω)
⇒ ( 2x − 1) + 2iy = ( x − 2) + iy
 1− i  [ 1 + (ω 3 ) 2 ⋅ ω 2 ]
=± 
1 i 
− =±   2 2
 2 2  2 ⇒ ( 2x − 1) + 2iy = ( x − 2) + iy = 1⋅ ( 1 + ω)( 1 + ω ) 2
 
[Q 1 + ω + ω2 = 0 and ω3 = 1]
66. (a) Let z = −1− i = r (cos θ + i sin θ) ⇒ ( 2x − 1)2 + ( 2 y )2 = ( x − 2)2 + y 2
= ( 1 + ω + ω2 + ω3 ) = 1
On comparing both sides real and ⇒ 4x 2 + 1− 4x + 4 y 2 = x 2 + 4 − 4x + y 2
−2( 1 + 2i )
imaginary parts, we get
⇒ 3x 2 + 3 y 2 − 3 = 0 ⇒ x 2 + y 2 = 1 76. (d) We have, z =
r cosθ = −1 ...(i) 3+i
which represents a circle. −2( 1 + 2i )(3 − i )
and r sinθ = −1 ...(ii) =
Hence, point z describes a circle. (3 + i )(3 − i )
On dividing Eq. (ii) by Eq. (i), we get
r sin θ −1 71. (a) Given condition, |z + z | = |z − z | −10
= = (i + 1) = − i − 1
r cos θ −1 Let z = x + iy ⇒ z = x − iy 10
⇒ tan θ = 1= tan π / 4 ⇒ θ = π / 4 ∴|x + iy + x − iy| = |x + iy − x + iy| ∴ Argument (θ) = − π + θ
π
= − π + tan −1   = − π +
Since, argument of z lies in the IIIrd ⇒ |2x | = |2iy| [Q|a + ib|= a2 + b2 ] 1
quadrant.  1 4
∴ arg (z ) = π + θ = π + π / 4 = 5 π / 4 ⇒ 4x 2 = 4y2 ⇒ x = ± y −4 π + π −3 π
= =
67. (a) We know that, which represent pair of straight lines 4 4
 1 passing through the origin. Hence, the
a + ib = ±  ( a 2 + b 2 + a ) locus of z is a pair of straight lines. 77. (c) Given, i = −1
 2 i
π
i
π

1  72. (a) We have, ∴ i = e 2 =e 4


+i ( a 2 + b 2 − a ) , b > 0
( 1 + i )4 n + 5 ( 1 + i )4n + 3 ⋅ ( 1 + i )2 π π
2  = = cos + i sin
 1 ( 1 − i )4 n + 3 ( 1 − i )4n + 3 4 4
∴ 3 + 4i = ±  ( 9 + 16 + 3) 4n + 3 1 1+ i
1+ i  i
 2 =   ⋅ ( 1 + i )2 = + =
1   1− i  2 2 2
+i ( 9 + 16 − 3)  4n + 3 3
2   ( 1 + i )( 1 + i )   sin π + i  1 − cos π  
= ⋅ ( 1 + i 2 + 2i ) 
= ± [ 4 + i 1] = ± ( 2 + i ) 
 ( 1 − i )( 1 + i )  78. (c) Let z =  6  6
π π 
68. (d) Let z =
( 1 + i )( 2 + i ) 4n + 3  sin − i  1 − cos  
 1 + i 2 + 2i   6  6 
(3 − i ) =  ⋅ 2i
 1+ 1  π
 2 sin cos π π 3
2 + 3i + i 2 2 + 3i − 1 + i 2 sin 2
= = [Q i 2 = − 1] = (i )4 n + 3 ⋅ 2i = 2(i )4 n + 4
=2  12 12 12 
3−1 3 −i =
 2 sin π cos π − i 2 sin 2 π 
1 + 3i 3 + i 3 + 10i + 3i 2  3+i 3+i 3+i 
= × = 73. (a)   = × 12 12 12 
3 −i 3+i 9 −i2  3 −i  3 −i 3+i π π 3
 cos + i sin 
3 + 10i − 3 10i 3 + i 2 + 2 3i 3 − 1 + 2 3i
= = =i [Qi 2 = − 1] = = = 2 12 
9+ 1 10 3 −i2 3+ 1  cos π − i sin π 
2( 1 + 3i ) 1  12 
∴ z = 0 + i⋅1 = = +i
3 2
π   cos π + i sin π   cos π + i sin π  
3
So, arg (z ) = tan − 1   = tan − 1 ( ∞ ) =
1 4 2 2
π
π π
 0 =  cos + i sin  = e 3  12   12  
i
2 2 12
 =
3 3  π π  π π 
69. (d) Let P = x + iy and Q = α + iβ be two   cos − i sin   cos + i sin  
 3+i
6
π   12 12   12 12  
= ei 2π
i
complex numbers. ∴   = (e 3)
6
2 3
P x + iy  3 −i   π π 
Then, its quotient = =   cos + i sin  
Q α + iβ 12 12 
= cos 2 π + i sin 2 π = π 
x + iy 2 π
P x + iy  cos + sin 2 
Now, = = = 1+ 0⋅i = 1
Q α + iβ α + iβ  12 12 
74. (d) Since, arg (z ) = 5 π / 6 6
π π
=
x2 + y2
=
|P | So, z lies in second quadrant. =  cos + i sin 
 12 12 
α2 + β2 |Q | Let z = a + ib and | z | = a 2 + b 2 = 4 π π
= cos 6 × + i sin 6 ×
5π π π
Hence, the modulus of the quotient of P Also, tan α =  π −  = ⇒α= π
12
π
12
and Q is equal to the quotient of their  6  6 6 = cos + i sin = i
2 2
moduli.
MATHEMATICS Complex Numbers 45

79. (b) (sin x + i cos x )3 we get ( x + iy )( x − iy ) + (3 − i ) ( x + iy )  z  3


π π
3
+ (3 + i ) ( x − iy ) + 1 = 0
Also,  =
=  cos  − x  + i  sin − x   
z−2

2
  2   2   ⇒ x 2 + y 2 + 3x + 3iy − ix + y + 3x
3 ⇒ 2 |z | = 3 |z − 2 |
  π − x   3i  π − x  −3iy + ix + y + 1 = 0
= e i 2 
 =e 2 
⇒ 2 x + y = 3 × ( x − 2)2 + y 2
2 2
  ⇒ x 2 + y 2 + 6x + 2 y + 1 = 0
π π ∴ Centre = ( − g , − f ) = ( − 3, − 1) ⇒ 4( x 2 + y 2 ) = 9 [( x − 2)2 + y 2 ]
= cos 3  − x  + i sin 3  − x 
2  2  and radius = g + f
2 2
−c For x = 6, 4(36 + y 2) = 9( 16 + y 2 )
3π 3π ⇒ 144 + 4 y 2 = 144 + 9 y 2 ⇒ y = 0
= cos  − 3x  + i sin  − 3x  = 9 + 1 −1 = 9 =3
 2   2  Hence, z = 6 + 0i
Sol. (Q. Nos. 84-85)
= ( − sin 3x − i cos 3x ) 86. (a) | z | = ( 6)2 + ( 0)2 = 6
We have, z2 = i z ...(i)
Hence, the real part is − sin3x. z−6
⇒ | z |2 = | iz | 87. (d) As, z − 6 = 0 ⇒  =0
80. (b) ( x 3 − 1) = ( x − 1)( x 2 + 1+ x ) z + 6
⇒ | z |2 = | z | [Q |z | = |z|]
= ( x − 1)( x 2 + x − ω − ω2 )
⇒ |z | − |z | = 0
2 88. (c) We know that, nth roots of unity
[Q 1 + ω + ω2 = 0] represent vertex of a regular polygon in
⇒ | z |(| z | − 1) = 0
= ( x − 1)( x 2 − ω2 + x − ω) argand plane.
As, z is non-zero complex number.
= ( x − 1)[( x + ω)( x − ω) + ( x − ω)] Angle between OP and OQ
∴ | z | = 1 ⇒ | z |2 = 1
= ( x − 1)( x − ω)( x + ω + 1) = arg(ω2 ) − arg(ω)
⇒ zz = 1 ⇒ z = 1/ z 4π 2π 2π
= ( x − 1)( x − ω)( x − ω ) 2
= − = = 120°
From Eq. (i), we have 3 3 3
81. (d) We have, | z | = | z |
1 2 z 2 = iz 89. (d) Given, (ω − ω )2
Let z = x + iy and z = x + iy 1 2
1 1 2 21 2 ⇒ z 2 = i 1/ z = ω2 + ω2 − 2 ω ω
∴ x2 + y2 = x2 + y2 1 2 1 2
1 1 2 2 ⇒ z3 = i ⇒ z3 − i = 0 ...(ii) Let ω = ω2 and ω = ω
⇒ (x 2 − x 2 ) + ( y2 − y2 ) = 0 If z , z and z satisfying the Eq. (ii),
1 2
1 2 1 2
1 2 3 ∴ (ω − ω ) 2 = (ω 2 ) 2 + ω 2 − 2 ω 2 ⋅ ω
⇒ x 2 − x 2 = 0 or y 2 − y 2 = 0 then 1 2
1 2 1 2 = ω4 + ω2 − 2ω3
⇒ x = ± x or y = ± y z +z +z =0
1 2 1 2 1 2 3 = ω3 ⋅ ω + ω2 − 2 ω3
e.g. Let z = 1 + i and z = − 1 − i z z + z z + z z = 0 and z z z = i
1 2 1 2 2 3 1 3 1 2 3 = ω + ω2 − 2 = − 1 − 2
|z | = 2 and | z | = 2 84. (c) We have, z + z + z = 0
1 2 1 2 3 ⇒ (ω − ω ) 2 = −3
∴ Re(z ) ≠ Re(z ) and 1 2
1 2 85. (c) We have, z z z = i 90. (a) We have,
Im (z ) ≠ Im(z ) and z ≠ z 1 2 3
−25
1 2 1 2 ⇒ z z z is purely imaginary.
z = x + iy = 
1 i 
82. (a) Given equation of line is iz − i z = 5. 1 2 3 − 
∴ Statement I is correct.  2 2
Let z = x + iy and z = x − iy and z z + z z + z z = 0 π 25
1 2 2 3 3 1 −i −25 i π
∴ i ( x − iy ) − i ( x + iy ) = 5 ⇒ 2 y = 5 ⇒ z z + z z + z z is purely real. = (e 4) =e 4
1 2 2 3 3 1
⇒ 2y − 5 = 0 ...(i) ∴ Statement II is correct.
π
i  6 π + 
π
i  
=e  4 =e  4
Since, it is given that reflection of point Sol. (Q. Nos. 87-88) Let z = x + iy
x + iy , i.e. ( x , y ) about the line (i) is π π 1 1
= cos+ i sin = +i
z− 4  x + iy − 4 
( 1 + i ), i.e. ( 1, 1). ∴  = 1 ⇒ = 1 4 4 2 2
1 − x 1 − y −2( 2 y − 5)  z −8   x + iy − 8  1
+i
1
− 2
∴ = = z− 2
∴ = 2 2
0 2 4
|x + iy − 4|  z |z |
1 − x −( 2 y − 5) ⇒ = 1 Q 1 = 1  z−i 2 1
+ i 
1 
⇒ = |x + iy − 8|  −i 2
 z2 |z |
2  2  2
0 2
1− y ( 2 y − 5) ( x − 4)2 + y 2 1+ i − 2
and =− =
2 2 ⇒ =1 1 + i − 2i
( x − 8)2 + y 2
⇒ x = 1 and y = 4 = − 1 + 0i
⇒ ( x − 4)2 + y 2 = ( x − 8)2 + y 2
Now, α = tan −1   = tan −1 ( 0)
∴ Required point is 1 + 4i. 0
⇒ ( x − 4)2 = ( x − 8)2  −1 
83. (a) Given,
⇒ ( x − 4) = ± ( x − 8) = tan −1 | tan 0 | = 0
zz + (3 − i )z + (3 + i )z + 1 = 0
∴ x =6 ∴ θ (argument) = π − 0 = π
Put z = x + iy and z = x − iy ,
04
46 CDS Pathfinder

BINARY NUMBERS
In NDA exam, generally 1-2 questions are asked from this chapter which are based on conversion
of binary to decimal and decimal to binary. In the binary system, only two symbols 0 and 1 are
used as digits, called binary digits or bits. Since, in this system only two numbers are used, so its
base or radix is 2.

DECIMAL SYSTEM
In the decimal system, we use 10 digits 0, 1, 2, 3, 4, 5, 6, 7, 8 and 9. Since, 10 basic symbols are used
in this system, so its base or radix is 10.

Decimal to Binary Conversion


1. Conversion of Integral Decimal Numbers
Step 1 Take the LCM of the given decimal number taking only 2 as divisor (as the base or radix of
binary number is 2).
Step 2 Write the remainder at each step in bracket as shown in the given example.
Step 3 Repeat this process until we obtain quotient less than the divisor.
e.g. Write ( 21)10 into binary number system.
2 21 5
2 10 1
2 5 0
2 2 1
1 0

Step 4 When we obtain 1 as quotient, start writing the number from there to upward direction, as shown
above.
∴ ( 21)10 = (10101) 2
Therefore, 21 of decimal system is equal to (10101) of binary system.
EXAMPLE 1. Conversion of 175 into a binary number is
a. (10101111)2 b. (10010011)2 c. (11000111)2 d. None of these
MATHEMATICS Binary Numbers 47

Sol. a. EXAMPLE 3. Conversion of (10010) 2 to its equivalent


2 175 decimal number is
2 87 1 a. (18)10 b. (81)10 c. (16)10 d. (34)10
2 43 1 Sol. a. (10010) 2 = 1 × 24 + 0 × 23 + 0 × 22 + 1 × 21 + 0 × 20
2 21 1
= 16 + 0 + 0 + 2 + 0 = 18
2 10 1
∴(10010) 2 = (18)10
2 5 0
2. Conversion of Fractional Binary Numbers In
2 2 1
order to convert the binary fractions to decimal
1 0 numbers, we use negative powers of 2 to the right of
∴ (175)10 = (10101111) 2 the binary point.
2. Conversion of Fractional Decimal Numbers
EXAMPLE 4. Find the decimal equivalent to (0.1101) 2 .
Fractional numbers can be converted to binary form by
successive multiplication by 2. In each step, the digit a. (81.25)10 b. (0.8125)10 c. (8.125)10 d. All of these
before the decimal point is being transfered to the binary Sol. b. 1 1 0 1
record and the process is repeated with the remaining
−1 1 −2 1 −3 1 −4 1
fractional number. 2 = 2 = 2 = 2 =
2 4 8 16
The last step is reached, if the fractional part is zero 1 1 1 1
or it is terminated, when the desired accuracy is ∴ (0.1101) 2 = × 1+ × 1+ × 0+ ×1
2 4 8 16
attained. The first bit obtained is the most significant
and the last is the least significant. = 0.5 + 0.25 + 0 + 0.0625 = 0.8125
Now, (0.1101) 2 = (0.8125)10
EXAMPLE 2. Conversion of 0.638 to binary form is
a. (0.1010101110)2 b. (1111000011)2 Arithmetic Operations
c. (0.1010001101)2 d. None of these of Binary Numbers
The arithmetic operations of binary numbers namely
Sol. c. Binary addition, subtraction, multiplication and division are
0.638 × 2 = 1.276 1 almost similar to those of decimal.
0.276 × 2 = 0.552 0
0.552 × 2 = 1.104 1 Binary Addition
0.104 × 2 = 0.208 0
0.208 × 2 = 0.416 0
The rules of binary addition are
0.416 × 2 = 0.832 0 (i) 0 + 0 = 0 (ii) 0 + 1 = 1
0.832 × 2 = 1.664 1 (iii) 1 + 0 = 1 (iv) 1 + 1 = 10 (one-zero, not ten)
0.664 × 2 = 1.328 1
0.328 × 2 = 0.656 0 Note The last rule is often written as 1 + 1 = 0 with a carry of 1.
0.656 × 2 = 1.312 1
EXAMPLE 5. Add (1101) 2 and the conversion of
Rounding off after 10 bits, the binary result is (53)10 into binary system of numbers. Also find the
(0.638)10 = (0.1010001101) 2 result.
Note The point in front of the binary is referred to as the binary a. (110010)2 b. (100101)2
point.
c. (1110011)2 d. None of these

BINARY TO DECIMAL Sol. d. Let us first convert ( 53)10 into binary system of numbers.

CONVERSION 2
2
53
26 1
1. Conversion of Integral Binary Numbers For 2 13 0
converting binary number to decimal number, we 2 6 1
start from the least significant bit, i.e. from right, by 2 3 0
multiplying them with the powers of 2 in increasing 1 1
order, i.e. with 20 , 21 , 22 and so on. This process is The required number in binary system is obtained by
repeated until the most significant bit, i.e. left bit has writing the number from left to upward direction, as
been processed. Adding all of them, we get the shown.
required decimal number. ∴ ( 53)10 = (110101) 2
48 NDA/NA Pathfinder

Now, add (1101) 2 and (110101) 2 Sol. b. Firstly, we multiply (1010) 2 and (111) 2
110101 1010
1101 ×111
1000010 1010
Hence, the answer is (1000010) 2. 1010×
1010××
Binary Subtraction 1000110
The rules of binary subtraction are Now, we convert (1000110) 2 to its decimal equivalent
(i) 0 − 0 = 0 (ii) 1 − 0 = 1 1 × 26 + 0 × 25 + 0 × 24 + 0 × 23 + 1 × 22 + 1 × 21 + 0 × 20
(iii) 1 − 1 = 0 (iv) 0 − 1 = 1 = 64 + 4 + 2 = (70) 2
∴ X = 70
EXAMPLE 6. Subtract 10001 from 10011.
a. ( 00100)2 b. (11000)2 Binary Division
c. ( 00010)2 d. None of these
The rules of binary division are
Sol. c. 10 0 1 1(19) (i) 1 ÷ 1 = 1 (ii) 0 ÷ 1 = 0
−10 0 0 1(17) (iii) 0 ÷ 0 = Not defined (iv) 1 ÷ 0 = Not defined
0 0 0 10 ( 2)
EXAMPLE 8. Divide (1111) 2 by (11) 2 .
a. (11)2 b. (101)2
Binary Multiplication
c. (10 )2 d. None of these
The rules of binary multiplication are
(i) 0 × 0 = 0 (ii) 0 × 1 = 0 Sol. b. 11 ) 1111 ( 101
(iii) 1 × 0 = 0 (iv) 1 × 1 = 1 11
11
11
EXAMPLE 7. What is the value of X, if
(1010) 2 × (111) 2 = (X)10 ? ×
a. 60 b. 70 c. 75 d. 80

PRACTICE EXERCISE
1. Binary equivalent of 182 is 6. The decimal number corresponding to the binary
(a) (10110110)2 (b) (111000110)2 number (111000.0101)2 is
(c) (10111001)2 (d) None of these (a) (56.3275)10 (b) (56.3125)10
(c) (57.4375)10 (d) (57.5625)10
2. The number 0.0011 in binary system represents
(a) rational number 3/8 in decimal system 7. What is the binary equivalent of the decimal
(b) rational number 1/8 in decimal system number 18.5625?
(c) rational number 3/16 in decimal system (a) (1000110011
. )2 (b) (1001110001
. )2
(d) rational number 5/16 in decimal system (c) (1001010010
. )2 (d) (1001010011
. )2
3. The binary number corresponding to (13.0625)10 is 8. In the binary addition, where x, y, z are binary
(a) (1011.0010)2 (b) (1110.0101)2 digits
(c) (1101.0001)2 (d) None of these x1 0 1
4. The sum of (1011.01) 2 + (1001.11) 2 is + 1 0 y 1
1 1 z0 0
(a) 111011 (b) 10001 (c) 10000 (d) 10101
the possible values of x, y and z respectively are
5. What is (1111)2 + (1001)2 − (1010)2 equal to? (a) 0, 1 and 0 (b) 1, 1 and 0
(a) (111)2 (b) (1100)2 (c) (1110)2 (d) (1010)2 (c) 0, 0 and 1 (d) 1, 0 and 1
MATHEMATICS Binary Numbers 49

9. If (10x 010)2 − (11 y1)2 = (10z11)2, then what are 17. A number in binary system is 110001. It is equal
the possible values of the binary digits x, y, z to which one of the following numbers in decimal
respectively? system? e 2013 II
(a) 0, 0, 1 (b) 0, 1, 0 (a) 45 (b) 46
(c) 1, 1, 0 (d) 0, 0, 0 (c) 48 (d) 49
10. If x = (1101)2 and y = (110)2, then what is the 18. The number 83 is written in the binary system
value of x 2 − y 2? as e 2013 II
(a) 100110 (b) 101101
(a) (1000101)2 (b) (10000101)2 (c) 1010011 (d) 110110
(c) (10001101)2 (d) (10010101)2
19. The number 251 in decimal system is expressed
11. What is the value of in binary system by e 2014 I
(0.101)(11) 2 + (0.011)(11) 2 (a) 11110111 (b) 11111011
2 2
2 − (0.101)(01) 2 (0.011)(01) 2 + (0.011)(10) 2
(c) 11111101 (d) 11111110
(0.101)(10)
2 2 2 2
(a) (0.001)2 (b) (0.01)2
20. What is the sum of the two numbers (11110)2 and
(c) (0.1)2 (d) (1)2 (1010)2? e 2014 I
(a) (101000)2 (b) (110000)2
(c) (100100)2 (d) (101100)2
PREVIOUS YEARS’ QUESTIONS 21. What is (1001)2 equal to? e 2014 I
12. The number 292 in decimal system is expressed (a) (5)10 (b) (9)10
in binary system by e 2012 I (c) (17 )10 (d) (11)10
(a) 100001010 (b) 100010001
(c) 100100100 (d) 10101000
22. If (11101011)2 is converted to decimal system,
then the resulting number is e 2015 I
13. What is the decimal number representation of (a) 235 (b) 175
the binary number (11101.001)2? e 2012 I (c) 160 (d) 126
(a) 30.125 (b) 29.025
23. The decimal number (127. 25)10, when converted
(c) 29.125 (d) 28.025
to binary number, takes the form e 2015 I
14. The decimal number ( 57. 375)10, when converted (a) (111111111
. )2
to binary number takes the form e 2012 II (b) (1111110.01)2
(a) (111001011
. )2 (b) (100111110
. )2 (c) (111011111
. )2
(c) (110011101
. )2 (d) (111011011
. )2 (d) (111111101
. )2

15. The decimal representation of the number 24. What is (1000000001)2 − ( 0.0101)2 equal to?
e 2015 II
(1011)2 in binary system is e 2012 II (a) ( 512.6775)10 (b) ( 512.6875)10
(a) 5 (b) 7 (c) 9 (d) 11 (c) ( 512.6975)10 (d) ( 512.0909)10
16. The binary representation of the decimal number 25. What is the binary equivalent of the decimal
45 is e 2013 I number 0.3125? e 2016 I
(a) 110011 (b) 101010 (a) 0.0111 (b) 0.1010
(c) 1101101 (d) 101101 (c) 0.0101 (d) 0.1101

ANSWERS
1 a 2 c 3 c 4 d 5 c 6 b 7 c 8 b 9 b 10 b
11 d 12 c 13 c 14 a 15 d 16 d 17 d 18 c 19 b 20 a
21 b 22 a 23 d 24 b 25 c
50 NDA/NA Pathfinder

HINTS AND SOLUTIONS


1 1
1. (a) 2 182 = 32 + 16 + 8 + +
4 16
2 91 0
= 56 + 0.25 + 0.0625 = (563125
. )
2 45 1 10

2 22 1 7. (c) For integer part of 18.5625 i.e. ( 18)


10
2 11 0 2 18
2 5 1 2 9 0
2 2 1 2 4 1
1 0 2 2 0
∴ ( 182) = ( 10110110) 1 0
10 2

2. (c) (. 0011) = 0 × 2−1 + 0 × 2−2 +1 × 2−3 + 1 × 2−4 For fractional part of 185625
. i.e. (.5625)
10
2
Binary
1 1 3
= + = 05625
. × 2 = 1125
. 1
8 16 16
0.125 × 2 = 2.250 0
3. (c) For integer part of ( 13. 0625)
10 0.250 × 2 = 0500
. 2
i.e. ( 13)
10
0500
. × 2 = 10. 1
∴ ( 13) = ( 1101) ∴ ( 185625
. ) = ( 10010.10010)
10 2 10 2

2 13 8. (b) ( x 101) = x × 23 + 1 × 22 + 0 +1 × 20 = 8x + 5
2

2 6 1 ( 10 y 1) = 1 × 23 + 0 + y × 21 +1 × 20 = 9 + 2 y
2
2 3 0 ( 11z 00) = 1 × 24 + 1 × 23 + z × 22 + 0 + 0 = 24 + 4z
2

2 1 1 ∴ 8x + 5 + 9 + 2 y = 24 + 4z ⇒ 8x + 2 y = 4z + 10
As x , y and z can take values 0 and 1 only
For fractional part of ( 13. 0625) i.e. (. 0625)
10 10 ⇒ x = 1, y = 1, z = 0
Binary
0. 0625 × 2 = 0.125 0 9. (b) ( 10x 010) − ( 11y 1) = ( 10z 11)
2 2 2
0.125 × 2 = 0.25 0
⇒ ( 2 × 1 + 0 + x × 2 + 0 × 2 + 1 × 21 + 0)
5 3 2
0.25 × 2 = 0.5 0
0.5 × 2 = 1.0 1 − ( 23 × 1 + 22 × 1 + y × 21 + 1 × 20 )
∴ (13.0625) = (1101.0001) = 2 × 1 + 0 + 22 × z + 21 × 1 + 20
4
10 2
⇒ (34 + 8x ) − ( 13 + 2 y ) = 19 + 4z
4. (d) 1 0 1 1.0 1
⇒ 2 + 8x − 2 y = 4z
+ 1 0 0 1.1 1
1 0 1 0 1. 0 0 ∴ x = 0, y = 1 and z = 0

5. (c) ( 1111) = 1 × 23 + 1 × 22 + 1 × 21 +1 × 20 = 15 10. (b) Given, x = ( 1101) = 1 × 23 + 1 × 22 + 0 × 21 + 1 × 20


2
2

( 1001) = 1 × 23 + 0 + 0 + 1 × 20 = 9 = 8 + 4 + 1 = 13
2
( 1010) = 1 × 23 + 0 + 1 × 21 + 0 = 10 and y = ( 110) = 1 × 22 + 1 × 21 + 0 × 20 = 4 + 2 = 6
2 2
∴ 15 + 9 − 10 = 14 ∴ x − y 2 = ( 13)2 − ( 6)2 = 169 − 36 = 133
2

Now,
2 14 Now, 2 133
2 7 0 2 66 1
2 3 1 2 33 0
1 1 2 16 1
2 8 0
∴ ( 14) = ( 1110)
10 2
2 4 0
6. (b) ( 111000.0101) = 1 × 25 + 1 × 24 + 1 × 23 + 0 × 22
2 2 2 0
+ 0 × 21 + 0 × 20 + 0 × 2−1 + 1 × 2−2 + 0 × 2−3 + 1 × 2−4 1 0
∴ 133 = ( 10000101)
2
MATHEMATICS Binary Numbers 51

11. (d) ( 11) = 1 × 21 + 1 × 20 = 3 18. (c) 19. (b)


2

( 10) = 1 × 2 + 0 × 2 = 2
1 0 20. (a)
2
11110
and ( 01) = 0 × 21 + 1 × 20 = 1
2
+1010
Thus, the given expression can be written as
101000
[( 0.101 ) ]3 + [( 0.011) ]3
2 2
[( 0.101) ]2 − ( 0.101) ( 0.011) + [( 0.011) ]2 21. (b) ( 1001)
2 2 2 2 2
 a3 + b3  = ( 23 × 1 + 22 × 0 + 21 × 0 + 20 × 1)
= ( 0.101) + ( 0.011) = ( 1) Q a 2 − ab + b 2 = a + b 10
2 2 2
  = (8 + 1) = ( 9)
10 10

12. (c) 22. (a) ( 11101011) = (?)


2 10

13. (c) ( 11101. 001) = 1 × 2 + 1 × 2 + 1 × 2 + 0 × 2 + 1


4 3 2 1
= [1 × 2 + 1 × 2 + 1 × 25 + 0 × 24
7 6
2
× 2 0 + 0 × 2 −1 + 0 × 2 −2 + 1 × 2 −3 + 1 × 23 + 0 × 22 + 1 × 21 + 1 × 20 ]
10
1 = [ 128 + 64 + 32 + 0 + 8 + 0 + 2 + 1]
= 16 + 8 + 4 + 0 + 1 + 0 + 0 + 10
8
= ( 235)
1 233 10
= 29 + = = ( 29.125)
8 8 10 23. (d) ( 127. 25) = (?)
10 2

14. (a) For integer part of 57.375 i.e. (57)


10 2 127
2 57 2 63 1
2 28 1
2 31 1
2 14 0
2 15 1
2 7 0
2 3 1 2 7 1

1 1 2 3 1
∴ (57) = ( 111001) 1 1
10 2

For after decimal part of 57.375 Here, 0. 25 × 2 = 050


. , carry = 0
i.e. (0.375) and . × 2 = 1.00, carry = 0
050
10
Now, Binary ∴ ( 127. 25) = ( 1111111. 01)
10 2
0375
. × 2 = 0.75 0
0.75 × 2 = 1.5 1 24. (b) ( 1000000001) − ( 0.0101)
2 2
0.5 × 2 = 1.0 1 = ( 29 + 20 ) − ( 1 × 2− 2 + 1 × 2− 4 )
= ( 0.011)
= (512 + 1) −  +
( 0.375) 1 1
10 2 
∴ (57.375) = ( 111001011
. )  4 16 
10 2
5
15. (d) ( 1011) = 1 × 23 + 0 × 22 + 1 × 21 + 1 × 20 = 513 − = 513 − 03125
.
2 16
= 8 + 0 + 2 + 1 = 11 = (512.6875)
10
16. (d)
25. (c) 0.3125 × 2 = 0.625, carry = 0
17. (d) ( 110001) = 1 × 25 + 1 × 24
2
0.625 × 2 = 125
. , carry = 1
+ 0 × 23 + 0 × 22 + 0 × 21 + 1 × 20
0.25 × 2 = 0.50, carry = 0
= 32 + 16 + 0 + 0 + 0 + 1 = ( 49)
10 0.50 × 2 = 100
. , carry = 1
So, the required decimal number is 49.
∴ ( 03125
. ) = ( 0.0101)
10 2
05
52 CDS P a t h f i n d e r

SEQUENCES
AND SERIES
In NDA exam, generally 2-5 questions are asked from this chapter which are based on general
term and sum of AP/GP, sum of special series and infinite terms of GP, conditions for terms to be
in AP/GP/HP and arithmetic/geometric mean.

A succession of numbers a 1 , a 2 , a 3 , . . . , a n formed according to some definite rule is called a sequence.


The different terms of a sequence are usually denoted by a1 , a 2 , a 3 , . . . or by t 1 , t 2 , t 3 , . . . . The term at
the nth place of a sequence, i.e. t n is called the general term of sequence. A series is obtained by adding
or subtracting the terms of a sequence. If a1 , a 2 , a 3 , a 4 , . . . , a n , ... is a sequence, then the expression
a1 + a 2 + a 3 + a 4 + a5 + . . . + a n + . . . is a series. A series can be finite or infinite depending on the
number of terms in the corresponding sequence is finite or infinite.

PROGRESSIONS
The sequences whose terms follow certain patterns are called progressions, but it is not necessary that
the terms of every sequence always follows a certain pattern or they are described by some explicit
formula.

Arithmetic Progression (AP)


A sequence is called an arithmetic progression if the difference of a term and the previous term is
always same, i.e. a n + 1 − a n = constant ( = d ), ∀ n ∈ N.
The constant difference, generally denoted by d is called the common difference.
In other words, if a1 , a 2 , a 3 , . . . , a n are in AP, then a 2 − a1 = a 3 − a 2 = . . . = a n − a n −1 = d
If a is the first term and d is the common difference, then AP can be written as
a, a + d , a + 2d , . . . , { a + ( n − 1) d }
e.g. (i) 1, 4, 7, 10, … is an AP whose first term is 1 and the common difference is 4 − 1 = 7 − 4 = 3.
(ii) 11, 7, 3, 1, … is an AP whose first term is 11 and the common difference is 7 − 11 = 3 − 7 = − 4.
53

General Term of an AP For y = 8, we get 2x = 8 = 23 or x = 3


When x = 2,log 3 ( 2x − 5) = log 3 ( − 1) is not defined.
Let a be the first term and d be the common difference
of an AP. Then, its nth term is a + ( n − 1) d , Therefore, x = 3 is the only solution.
Selection of Terms in an AP
i.e. Tn = a + ( n − 1) d , where d = Tn − Tn − 1
Number Terms Common
If l is the last term of a sequence, then of terms difference
l = Tn = a + ( n − 1) d 3 a − d , a, a + d d
4 a − 3d , a − d , a + d , a + 3d 2d
nth Term of an AP from the End 5 a − 2d , a − d , a, a + d , a + 2 d d
nth term from the end of an AP consisting of m terms is 6 a − 5 d , a − 3 d , a − d ,a + d , a + 3 d , a + 5 d 2d
( m − n + 1)th term from the beginning. So, nth term from
the end is EXAMPLE 3. If the sum of three numbers of an
Tm − n + 1 = a + ( m − n + 1 − 1) d = a + ( m − n) d . arithmetic sequence is 15 and the sum of their squares
is 83, then the numbers are
or l − ( n − 1) d , whene l is the last term.
a. 4, 5, 6 b. 1, 5, 9
EXAMPLE 1. The 3rd, 15th and the last term of an AP c. 3, 5, 7 d. 2, 5, 8
are 4, 8 and 18, respectively. Find the first term, the Sol. c. Let three numbers are ( a − d), a and ( a + d).
common difference and the number of terms.
∴ a − d + a + a + d = 15 ⇒ a = 5
10 1 1
a. , , 45 b. 10 , , 45 and ( a − d) 2 + a2 + ( a + d) 2 = 83
3 3 3
10 1 ⇒ a2 + d 2 − 2ad + a2 + a2 + d 2 + 2ad = 83
c. , , 50 d. None of these
3 3 ⇒ a2 + 2( a2 + d 2) = 83
Sol. a. Let the first term, common difference and the Put a = 5 in above equation
number of terms of the AP be a, d and n respectively. ⇒ 25 + 2( 25 + d 2) = 83
The nth term of an AP is given by Tn = a + (n − 1) d ⇒ 2d 2 = 8 ⇒ d = 2
Therefore, T 3 = a + 2d = 4, T15 = a + 14 d = 8 Thus, the numbers are 3, 5 and 7.
and Tn = a + (n − 1) d = 18 Trick 3 + 5 + 7 = 15 and 32 + 52 + 72 = 83
We have, T15 − T 3 = ( a + 14d) − ( a + 2d) = 8 − 4
⇒ 12d = 4 ⇒ d = 1/ 3 Sum of n Terms of an AP
2 10 The sum of n terms S n of an AP with first term ‘a’ and
Hence, a = 4 − 2d = 4 − =
3 3
common difference ‘d’ is
10 1 1
Therefore, + (n − 1) = 18 ⇒ (10 + n − 1) = 18 n n
3 3 3 S n = [ 2a + ( n − 1) d ] or S n = [ a + l ],
⇒ 9 + n = 54 2 2
∴ n = 45 where, l = last term = a + ( n − 1) d
Also, Tn = S n − S n − 1
EXAMPLE 2. If log 3 2, log 3 (2 x − 5) and
log 3 (2 x − 7/2) are in AP, then find the value of x. EXAMPLE 4. If the sum of first 10 terms of an
arithmetic progression with first term p and common
a. 2 b. 3 c. 2 or 3 d. None of these
difference q, is 4 times the sum of the first 5 terms,
Sol. b. If the given three numbers a = log 3 2, b = log 3 ( 2x − 5) then what is the ratio of p : q?
and c = log 3 ( 2x − 7 / 2) are in AP, a. 1 : 2 b. 1 : 4 c. 2 : 1 d. 4 : 1
then b − a = c − b ⇒ 2b = a + c Sol. a. Since, first term = p and common difference = q
2 log 3 ( 2x − 5) = log 3 2 + log 3 ( 2x − 7 / 2)
10 5
⇒ log 3 ( 2x − 5) 2 = log 3 [ 2 ( 2x − 7 / 2)] According to the question, [ 2 p + 9 q] = 4 × [ 2 p + 4 q]
2 2
⇒ ( 2x − 5) 2 = 2 ( 2x − 7 / 2)
⇒ 2p + 9q = 4 p + 8q ⇒ 2p = q ⇒ p : q = 1: 2
Let 2x = y, then we have ( y − 5) 2 = 2 ( y − 7 / 2)
⇒ y 2 − 10y + 25 = 2y − 7 Properties of Arithmetic Progression
⇒ y 2 − 12y + 32 = 0 (i) If a constant is added or subtracted from each term
⇒ ( y − 4) ( y − 8) = 0 of an AP, then the resulting sequence is also an AP
Hence, y = 4 or y = 8
with the same common difference.
For y = 4, we get 2x = 4 = 22 or x = 2
54 NDA/NA Pathfinder

(ii) If each term of a given AP is multiplied or divided by Geometric Progression (GP)


a non-zero constant k, then the resulting sequence is A sequence of non-zero numbers is called a geometric
d progression, if the ratio of a term and the term preceding
also an AP with common difference kd or , where d
k it is always constant. The constant ratio, generally
is the common difference of the given AP. denoted by r is called the common ratio of the GP.
(iii) A sequence is an AP if its nth term is of the form In other words, if a1 , a 2 , a 3 , . . . , a n are in GP, then
a2 a3 a
An + B, i.e. a linear expression in n. = = n = r (say ), where r is known as common
a1 a 2 a n − 1
(iv) In a finite AP the sum of the terms equidistant from
the beginning and end is always same. ratio of GP.
i.e. a1 + a n = a 2 + a n − 1 = a 3 + a n − 2 = . . . 1 1 3 9
, − , , − , . . . is a GP with first
e.g. The sequence
3 2 4 8
n Arithmetic Means between Two Numbers
term and common ratio  −    = − .
1 1 1 3
If a, A1 , A 2 , A 3 , . . . , A n , b are in AP, then we say that 3  2   3 2
A1 , A 2 , A 3 , . . . , A n are the n arithmetic means (AM)
between two numbers a and b. The common difference General Term of a GP
b− a The nth term of a GP with first term a and common
(d) of this AP is and mth arithmetic mean is given
n+1 ratio r is given by Tn = ar n − 1 or l = ar n−1 , where l is
m ( b − a) the last term.
by A m = a +
n+1 GP can be written as a, ar , ar 2 , . . . , ar n − 1
By putting m = 1, 2, 3, . . . , n, we can get the values of or a, ar , ar 2 , ar 3 , ar 4 , . . . , ar n − 1 , …
A1 , A 2 , . . . , A n . accordingly they are finite or infinite.
The sum of n arithmetic means between two given
numbers is n times the single AM between them i.e. nth Term from the End of a Finite GP
A1 + A 2 + A 3 + . . . + A n = n (single AM between a and b) The nth term from the end of a finite GP consisting of
If there is only one arithmetic mean ‘A’ between a and b, m terms is ar m − n , where a is the first term and r is the
a+b common ratio of the GP.
then A =
2
EXAMPLE 6. If x, y , z are the pth, qth and rth terms
EXAMPLE 5. If n arithmetic means are inserted of a GP, then the value of x q − r y r − p z p − q is equal
between 20 and 80 such that the ratio of first mean to to
the last mean is 1 : 3, then find the value of n. a. 0 b. 1 c. − 1 d. None of these
a. 12 b. 13 c. 11 d. 14
Sol. b. Let A be the first term and R be the common ratio
Sol. c. Let A1, A2 , A3 , ... , An be n arithmetic means between
of the GP. We have, Tp = AR p − 1 = x
20 and 80, and let d be the common difference of the
AP; 20, A1, A2 , ... , An , 80. Tq = AR q − 1 = y and Tr = AR r − 1 = z
q− r
Now, x = [ AR p − 1]q − r = Aq − r R ( p − 1) ( q − r )
80 − 20 60  b − a
Then, d= = using, d = n + 1 y r − p = [ AR q − 1] r − p = Ar − p R ( q − 1) ( r − p)
n+1 n+1  
z p − q = [ AR r − 1]p − q = Ap − q R ( r − 1) ( p − q)
60  n + 4
Now, A1 = 20 + d ⇒ A1 = 20 + = 20  
n+1  n + 1 ∴ x q − r y r − p z p − q = [ Aq − r + r − p + p − q ]
[R ( p − 1) ( q − r ) + ( q − 1) ( r − p) + ( r − 1) ( p − q) ]
60 n  4n + 1
and An = 20 + nd ⇒ An = 20 + = 20   = A0 R 0 = 1
n+1  n + 1
20 (n + 4) Selection of Terms in GP
A1 1 n+1 1 Number of terms Terms Common ratio
∴ = ⇒ = a
An 3 20 ( 4n + 1) 3 3 , a, ar r
r
n+1
a a
, , ar, ar 3 r2
n+ 4 1 4
r3 r
⇒ =
4n + 1 3 a a
5 , , a, ar, ar 2 r
r2 r
⇒ 4n + 1 = 3n + 12 ⇒ n = 11
MATHEMATICS Sequences and Series 55

6 6 6
+ + + ... ∞
EXAMPLE 7. Find the three numbers in GP, whose 2 3
Sol. d. 7 6 /7 ⋅ 7 6/7 ⋅ 7 6/7 ... ∞ = 7 7 72 73
sum is 19 and product is 216.
1 1 1   1/7 
6 
6  1/7 
a. 4, 6, 9 b. 13, 4, 2 6 + + + ... ∞  
 1 − 1/7 
7 7 7 7   6 /7 
2 3
c. 6, 5, 8 d. None of these = =7 =7 = 71 = 7
a
Sol. a. Let three numbers in GP be , a and ar. Properties of Geometric Progression
r
(i) If all the terms of a GP be multiplied or divided by
According to the given condition,
the same non-zero constant, then it remains a GP
Product of three numbers
a with same common ratio.
⋅ a ⋅ ar = 216 ⇒ a3 = 216 = 63
r (ii) The reciprocals of the terms of a given GP forms a
∴ a=6 GP.
a
and sum of three numbers = + a + ar = 19 (iii) If each term of a GP be raised to the same power,
r
then the resulting sequence also forms a GP.
a  + 1 + r  = 19
1

 r  (iv) In a finite GP the product of the terms equidistant
1 + r + r 
2 from the beginning and the end is always same and
⇒ 6  = 19 [put a = 6] is equal to the product of the first and the last term.
 r 
(v) Three non-zero numbers a, b, c are in GP, iff b 2 = ac.
⇒ 6 + 6r + 6r 2 = 19r
⇒ 6r + 6r − 19r + 6 = 0
2 (vi) If the terms of a given GP are chosen at regular
⇒ 6r 2 − 13r + 6 = 0 intervals, then the new sequence, so formed also
 forms a GP.
13 ± ( −13) 2 − 4 ⋅ 6 ⋅ 6 − b ± b2 − 4ac 
∴ r= Q r =  (vii) If a1 , a 2 , a 3 , . . . , a n , . . . is a GP of non-zero,
2⋅ 6  2a 
non-negative terms, then
13 ± 5 18 8 3 2
= = , = , log a1 , log a 2 , . . . , log a n , . . . is an AP and vice-versa.
12 12 12 2 3
3 6 3
When r = , then the numbers are , 6, 6 × n Geometric Means between
2 3/ 2 2
i.e. 4, 6, 9. Two Numbers
2 6 6
When r = , then the numbers are ,6 , If a, G1 , G2 , G3 , . . . , Gn , b are in GP, then
3 2/ 3 3/ 2
G1 , G2 , G3 , . . . , Gn are callled n geometric means
i.e. 9, 6, 4.
between two numbers a and b. The common ratio (r) of
Hence, the numbers are 4, 6, 9 or 9, 6, 4. 1 /(n + 1 )
 b
this GP is   and mth geometric mean is given by
Sum of n Terms of a GP  a
The sum of n terms of a GP with first terms ‘a’ and m
 b n + 1
common ratio ‘r’ is given by Gm = a   .
 a
 1 − r n  a − lr
Sn = a  = , for | r | < 1 By putting m = 1, 2, 3, . . . , n, we can get the values of
 1−r  1−r
G1 , G2 , . . . , Gn .
 r n − 1 lr − a The product of n geometric means is equal to the nth
and S n = a  = , for | r | > 1
 r −1 r −1 power of single geometric mean between the two
If number of terms is infinite, then sum of the terms is quantities, i.e. G1 ⋅ G2 ⋅ G3 ⋅ G4 . . . Gn = ( ab ) n .
a If there is only one geometric mean ‘G’ between a and
S= , | r | < 1.
1−r b, then
EXAMPLE 8. What is the value of G = ab
2 3
7 6/7 ⋅ 7 6/7 ⋅ 7 6/7 ... upto ∞?
Note • If A and G are the AM and GM between two positive
a. log7  
6 numbers, then the numbers are A ± A 2 − G 2.
b. ∞
 7 • If A and G are respectively AM and GM between two positive
6 numbers a and b, then the quadratic equation having a, b as
c. d. 7
7 its roots is x 2 − 2 Ax + G 2 = 0.
56 NDA/NA Pathfinder

EXAMPLE 9. If one geometric mean G and two Relation Among Arithmetic, Geometric and
arithmetic means A1 and A2 are inserted between two Harmonic Means
numbers, then (2 A1 − A2 )(2 A2 − A1 ) is equal to Let A , G and H be the arithmetic, geometric and
a. 2G b. G c. G 2 d. G 3 harmonic means between a and b, then
( b − a) 2a + b (i) A ≥ G ≥ H (ii) G 2 = AH
Sol. c. We have, G = ab, A1 = a + =
3 3
2( b − a) a + 2b EXAMPLE 10. If for positive real numbers x, y , z the
and A2 = a + =
3 3 numbers x + y , 2y and y + z are in harmonic
∴ 2A1 − A2 = a and 2A2 − A1 = b progression, then which one of the following is
⇒ ( 2A1 − A2)( 2A2 − A1) = ab = G 2 correct?
a. x, y , z are in GP b. x, y , z are in AP
Harmonic Progression (HP) c. x, y , z are in HP d. None of these
A sequence a1 , a 2 , . . . , a n of non-zero numbers is called a
2 ( x + y) ( y + z)
harmonic progression, if the sequence of reciprocal of Sol. a. Q x + y , 2y and y + z are in HP, 2y =
1 1 1 x+ y + y + z
these numbers, i.e. , , ..., is an AP. ⇒ 2y ( x + 2y + z) = 2 ( xy + xz + y 2 + yz)
a1 a 2 an
⇒ 2xy + 4y 2 + 2yz = 2xy + 2xz + 2y 2 + 2yz
1 1 1
e.g. The sequence 1, , , , . . . is a HP because the ⇒ 2y 2 = 2xz ⇒ y 2 = xz
3 5 7
Hence, x, y , z are in GP.
sequence 1, 3, 5, 7, . . . is an AP.
Their is no formula for finding the sum of HP sequence. Sum to n Terms of Special Series
The sum of first n terms of special series is given below
General Term of a HP n ( n + 1)
If the sequence a1 , a 2 , a 3 , . . . is a HP, then its nth term Σ n = 1 + 2 + 3 + ... + n =
2
will be n ( n + 1) ( 2n + 1)
1 a1 a 2 Σ n 2 = 12 + 22 + . . . + n 2 =
Tn = = 6
1  1 1  a 2 + ( n − 1)( a1 − a 2 ) 2
+ ( n − 1)  −   n ( n + 1)
a1  a 2 a1  Σ n 3 = 13 + 23 + . . . + n 3 = = ( Σn) 2
 2 
and nth term from end,
a1 a 2 a n Note If nth term of the sequence is Tn = an3 + bn2 + cn + d . Then,
1
Tn′ = = sum of n terms is given by
1  1 1 a1 a 2 − a n ( n − 1)( a1 − a 2 ) S n = ΣTn = a Σn3 + b Σn2 + c Σn + Σd .
− ( n − 1) − 
an  a 2 a1 
Arithmetico-Geometric
n Harmonic Means between Two Numbers Progression (AGP)
If a, H1 , H 2 , . . . , H n , b are in GP, then H1 , H 2 , . . . , H n A sequence of the form
are called n harmonic means between two numbers a and a, ( a + d ) r , ( a + 2d ) r 2 , . . . , { a + ( n − 1) d } r n − 1 , . . .
b. The common difference of the corresponding AP is is called an arithmetico-geometric sequence, where
a−b
and mth harmonic mean is given by { a + ( n − 1) d }r n − 1 is the nth term of the sequence.
( n + 1) ab
Sum of its n terms is
( n + 1) ab  a dr (1 − r n −1 ) { a + ( n − 1) d} r n
Hm = , m = 1, 2, . . . , n
ma + [ n − ( m − 1)] b 1 − r + (1 − r )2

1−r
, r ≠1
Sn = 
By putting m = 1, 2, . . . , n, we can get the values of  n { 2a + ( n − 1) d}, r = 1
 2
H1 , H 2 , . . . , H n . If there is only one harmonic mean H
2ab If this series has infinite terms, then
between a and b, then a, H, b are in HP. Then, H = a dr
a+b S∞ = + , |r |< 1
1 − r (1 − r ) 2
.
MATHEMATICS Sequences and Series 57

Difference Method for Finding Sol. b. Here, the difference in consecutive terms are
3, 5, 7, … which are in AP.
Sum of AGP Series
Let S n = 2 + 5 + 10 + 17 + K + t n − 1 + t n
Suppose a1 , a 2 , a 2 , … is a sequence such that the
On shifting every term one place to the right
sequence a 2 − a1 , a 3 − a 2 , … is either an AP or a GP
S n = 2 + 5 + 10 + K + t n − 1 + t n
The nth term ‘ a’n of this sequence is obtained as follows On subtracting, we get
S = a1 + a 2 + a 3 + K + a n − 1 + a n 0 = ( 2 + 3 + 5 + 7 + K to n terms) − t n
S= a1 + a 2 + K + a n − 1 + a n ⇒ t n = 2 + [ 3 + 5 + 7 + K to (n − 1) terms]
⇒ a n = a1 + [( a 2 − a1 ) + ( a 3 − a 2 ) + K + ( a n − a n − 1 )] n −1
= 2+ [ 2 × 3 + (n − 2) × 2]
Since, the terms within the brackets are either in AP or 2
n −1
in GP, in either case we can find the value of a n , the nth = 2+ ( 2n + 2) = 2 + (n − 1)(n + 1)
term, we can now find the sum of the n terms of the 2
n = 2 + (n2 − 1) = n2 + 1
sequence as S = ∑ a kn Putting n = 1, 2, 3, …, n and adding, we get
k=1 n
n(n + 1) ( 2n + 1)
Sn = ∑ k 2 + n = +n
EXAMPLE 11. Find the sum to n terms of the series k =1
6
2 + 5 + 10 + 17 + K . =
n
( 2n2 + 3n + 1 + 6)
n n 6
a. ( 2n2 + 3n + 7) b. ( 2n2 + 3n + 7)
5 6 n
= ( 2n2 + 3n + 7)
n (n + 1)( 2n + 1) 6
c. d. None of these
6

PRACTICE EXERCISE
3 + 5 + 7 +L+ n 7. The sum of n terms of an AP is an ( n − 1). The
1. If = 7, then the value of n
5 + 8 + 11 + L + 10 terms sum of the squares of these terms is equal to
is a2
(a) a2 n2 (n − 1)2 (b) n (n − 1) (2 n − 1)
(a) 35 (b) 36 (c) 37 (d) 40 6
2. If sum of n terms of an AP is 3n 2 + 5n and 2 a2 2 a2
(c) n (n − 1) (2 n − 1) (d) n (n + 1) (2 n + 1)
Tm = 164, then m is equal to 3 3
1 1
(a) 26 (b) 27 (c) 28 (d) None of these 8. If a , b, c are in GP, then + is equal to
a −b
2 2
b2
3. In a GP, if the ( m + n )th term be p and ( m − n )th
(a) 1/(c 2 − b 2 )
term be q, then its mth term is
(a) pq (b) p/q (c) q/p (d) p+ q (b) 4b 2 − c 2
(c) 1/ (c 2 − a2 )
4. The sum of the first ‘n’ terms of the series
1 3 7 15 (d) 1/(b 2 − c 2 )
+ + + + L is
2 4 8 16 9. If S be the sum to infinity of a GP, whose first
(a) 2 n − n − 1 (b) 1 − 2 − n term is a, then the sum of first n terms is
(c) n + 2 − n − 1 (d) 2 n − 1 n
 a 
n
(a) S  1 −  (b) S 1 −  1 −  
a
5. An AP consists of n (odd terms) and its middle  S  S 

term is m. Then, the sum of the AP is  a 
n
1 (c) a 1 −  1 −   (d) None of these
(a) 2mn (b) mn (c) mn (d) mn2  S 
2 
2 3 4 10. If the non-zero numbers a , b, c are in AP and
6. The sum of 1 + + + + L ∞ upto n terms is
5 52 53 tan− 1 a , tan− 1 b, tan− 1 c are also in AP, then
25 15 5 3 (a) a = b = c (b) b 2 = 2 ac (c) a2 = bc (d) c 2 = ab
(a) (b) (c) (d)
16 16 16 2
58 NDA/NA Pathfinder

1 1 3
a a 20. Which term of the sequence 20, 19 , 18 , 17 , ...
11. The harmonic mean of and is equal 4 2 4
1 − ab 1 + ab is the first negative term?
to (a) 27th (b) 28th
a a 1 (c) 29th (d) No such term exists
(a) (b) (c) a (d)
1 − a2 b 2 1 − a2 b 2 1 − a2 b 2 21. If p times the pth term of an AP is q times the qth
term, then what is the ( p + q )th term equal to?
1 1
12. If first three terms of sequence , a , b, are in (a) p + q (b) pq (c) 1 (d) 0
16 6
geometric series and the last three terms are 22. If a , 2a + 2, 3a + 3 are in GP, then what is the
in harmonic series, then the values of a and b fourth term of the GP?
will be (a) − 13.5 (b) 13.5
1 1 1 (c) − 27
(a) a = − , b = 1 (b) a = ,b= (d) 27
4 12 9
(c) Both (a) and (b) (d) None of these a n + 1 + bn + 1
23. If is the arithmetic mean between a
a n + bn
13. If a , b, c are in AP, p, q , r are in HP and ap, bq , cr and b, then n is equal to
are in GP, then
p r
+ is equal to (a) − 1 (b) − 2 (c) 0 (d) 1
r p
24. Let a, b, c be three positive real numbers such
a c a c b q b q
(a) − (b) + (c) + (d) − that their product is unity, then the least value
c a c a q b q b of (1 + a )(1 + b)(1 + c) is
14. If x = 1 + a + a 2 + a3 + L ∞ (| a|< 1) and (a) 16 (b) 8 (c) 0 (d) 3
y = 1 + b + b + b + L ∞ (| a|< 1), then
2 3
25. If one AM ‘A’ and two GM p and q be inserted
1 + ab + a 2b2 + a3 b3 + L ∞ is equal to between any two numbers, then the value of
xy x+ y p3 + q3 is
(a) (b)
x+ y−1 x+ y+1 2 pq
(a) (b) 2 Apq
x− y A
(c) (d) None of these
x− y+1
(c) 2 Ap2q 2 (d) None of these
1 1 1 1
15. + = + , then a , b, c are in 26. A square is drawn by joining mid-points of the
b− a b− c a c
sides of a square. Another square is drawn inside
(a) AP (b) GP (c) HP (d) None of these the second square in the same way and the
16. The value of x + y + z is 15, if a , x , y , z , b are in process is continued indefinitely. If, the side of
1 1 1 5 the first square is 16 cm, then what is the sum
AP while the value of + + is , if a , x , y , z , b of the areas of all the squares?
x y z 3
(a) 256 sq cm (b) 512 sq cm
are in HP. Then, a and b are
(c) 1024 sq cm (d) 512 / 3 sq cm
(a) 1, 9 (b) 3, 7
(c) 7, 3 (d) None of these 27. Let a , b and c be in an AP.
17. If ( m + 1)th, ( n + 1)th and (r + 1)th terms of an AP Consider the following statements
are in GP and m , n , r are in HP, then the ratio of 1 1 1
I. , and are in an AP.
the first term of the AP to its common difference ab ca bc
in terms of n is equal to 1 1 1
II. , and are in AP.
(a) n/2 (b) − n/2 (c) n/3 (d) − n/3 b+ c c+ a a + b
18. If the mth and nth term of a HP are n and m Which of the above statement(s) is/are correct?
respectively, then the mnth term is (a) Only I (b) Only II
1 (c) Both I and II (d) Neither I nor II
(a) 0 (b) 1 (c) 2 (d)
2
28. Let a , b, c be in AP and a 2 , b2 , c2 are in HP.
19. Two AM’s A1 and A2 , two GM’s G1 and G2 and
Consider the following statements.
two HM’s H 1 and H 2 are inserted between any
−1 I. a ≠ b ≠ c
two numbers, then H 1 + H 2−1 is equal to
II. − a / 2, b and c are in GP.
(a) A 1−1 + A 2−1 (b) G 1−1 + G 2−1
Which of the above statement(s) is/are correct?
G 1G 2 A + A2
(c) (d) 1 (a) Only I (b) Only II
A1 + A 2 G 1G 2 (c) Both I and II (d) Neither I nor II
MATHEMATICS Sequences and Series 59

29. We have two series, one is AP, i.e. 12, 18, 24, … Which of the above statement(s) is/are correct?
and another is GP, i.e. 4, 8, 16, … . Now taking (a) Only I (b) Only II
first three terms of each and adding its (c) Both I and III (d) Both II and III
respective middle term to each term. Which
series results into an HP? Directions (Q. Nos. 35-36) Sides of the right angled
I. AP II. GP triangle are in AP with common difference d.
(a) Only I (b) Only II 35. The area of the triangle is
(c) Both I and II (d) Neither I nor II 1
(a) d 2 (b) 12d 2 (c) d 2 (d) 6d 2
30. If|x|< 1, then the sum of the series 2
1 + 2x + 3x 2 + 4x3 + ... ∞ will be 36. If sides of the triangle are three arithmetic
1 1 means between two numbers such that sum of
I. II.
(1 + x)2 (1 − x)2 the 5 numbers is 15, then sides of the triangle are
3 5 9 15 3 5 9 15
Which of the above statement(s) is/are correct? (a) , 1, (b) , 3, (c) , 2, (d) , 6,
4 4 4 4 2 2 2 2
(a) Only I (b) Only II
(c) Both I and II (d) Neither I nor II Directions (Q. Nos. 37-38) Let a, b, c be in GP and
31. If a, b, c are distinct positive real numbers and x, y be the AM’s between a, b and b, c respectively.
a 2 + b2 + c2 = 1. Then, ab + bc + ca is a c
37. The value of + is
I. equal to 1. II. less than 1. x y
III. greater than 1. (a) 1 (b) 2 (c) 3 (d) 4
Which of the above statement(s) is/are correct? 38. Consider the following statements
(a) Only II (b) Only III (c) Only I (d) None of these ( a + c )2
I. The sum of x and y is .
32. If the sum of first ‘n’ natural numbers is 2
n( n + 1) (a + b)2
. Then, what will be the sum of first ‘n’ II. The sum of x and y is .
2 2
terms of the series of alternate positive and Which of the above statement(s) is/are correct?
negative numbers when ‘n’ is even? (a) Only I (b) Only II
12 − 22 + 32 − 42 + 52 − K (c) Both I and II (d) Neither I nor II

n (n + 1) n 2(n + 1) − n (n + 1)
I. II. III. Directions (Q. Nos. 39-41) Given two series
2 2 2
S1 = 1 + 2 + 4 + 8 + L 100 terms
Which of the above statement(s) is/are correct? and S 2 = 1 + 4 + 7 + 10 + L 100 terms.
(a) Only I (b) Only III (c) Only II (d) None of these
33. If general term of a GP with common ratio ‘r’ be 39. Find last term of series S1.
2100
tk + 1 and general term of another GP with (a) 2 98 (b) 2 99 (c) 2100 (d)
common ratio r be t ′ k + 1, then the series whose 2
general term tk′′ + 1 = tk + 1 + t ′ k + 1 is 40. Find the number of common terms in both of the
I. an AP with common difference ‘r’.
series.
(a) 4 (b) 5 (c) 6 (d) 7
II. a GP with common ratio ‘r’.
III. a HP. 41. Find the sum of all the terms of series S 2.
Which of the above statement(s) is/are correct? (a) 14450 (b) 14590 (c) 14950 (d) 19450
(a) Only III (b) Only I
(c) Only II (d) Both II and III Directions (Q. Nos. 42-44) Read the following
information carefully and answer the question that
34. Let ‘a’ be the first term, ‘d’ be the common follow.
difference, S n denotes the sum of ‘n’ terms and ‘an ’ Consider a sequence whose sum to n terms is given
S
denotes the nth term of an AP, then m equals by quadratic function, S n = 3 n2 + 5n.
Sn
m2 42. The nature of the given series is
to 2 , when m ≠ n for some m, n ∈ N . The ratio (a) AP (b) GP (c) HP (d) AGP
n
am : an is 43. For the given sequence the number 5456 is the
m −1 2m − 1 4S m − a (a) (153)th term (b) (932)th term
I. II. III.
n −1 2n − 1 4S n − a (c) (707)th term (d) (909)th term
60 NDA/NA Pathfinder

44. Sum of the squares of the first 3 terms of the 54. The HM of two numbers is 4 and their AM and
given series is GM satisfy the relation 2 A + G 2 = 27, then the
(a) 100 (b) 660 (c) 799 (d) 1000 number are
Directions (Q. Nos. 45-47) Four different integers (a) − 3 and 1 (b) 5 and − 25 (c) 5 and 4 (d) 3 and 6
form an increasing AP. The largest number is equal
to the sum of the squares of the other three 55. If the ratio of AM between two positive real
numbers a and b to their HM is m : n, then a : b
numbers. Then,
is equal to
45. The smallest number is m− n + n n+ m− n
(a) (b)
(a) − 2 (b) 0 (c) − 1 (d) 2 m− n − n n− m− n
46. The common difference of the four number is m+ m− n m− n + m
(c) (d)
(a) 2 (b) 1 (c) 3 (d) 4 m− m− n m− n − m

47. The sum of all the four numbers is


(a) 10 (b) 8 (c) 2 (d) 6 PREVIOUS YEARS’ QUESTIONS
Directions (Q. Nos. 48-51) The sum of n terms of an 56. What is the nth term of the sequence
AP is given by S n = pn + qn2, where p, q are 1, 5, 9, 13, 17, … ? e 2012 I
independent of n. (a) 2 n − 1 (b) 2 n + 1
(c) 4 n − 3 (d) None of these
48. The nth term of the AP is
(a) q + p(2 n − 1) (b) p + q (2 n − 1) 1
57. What does the series 1 + 3−1/ 2 + 3 + +L
(c) q + p(n − 1) (d) p + q (n − 1) 3 3
represent? e 2012 I
49. The common difference of the AP is (a) AP (b) GP (c) HP (d) None of these
(a) p + q (b) q (c) 2q (d) 2 p + 3q 1 1 1
58. What is the sum of the series 1 − + − +L?
2 4 8
50. If S1, S 2, S3 be the sum of n, 2n, 3n terms of AP, e 2012 I
then (a) 1/2 (b) 3/2 (c) 2 (d) 2/3
(a) 2S 2 − S 3 + S1 = pn (b) S 3 − (S 2 + S1 ) = 2qn2
59. If 1/4, 1/x and 1/10 are in HP, then what is the
(c) 2S 2 − S 3 + S1 = 3 pn (d) S 3 − (S 2 + S1 ) = 4qn2 value of x? e 2012 I
(a) 5 (b) 6 (c) 7 (d) 8
51. If sum of 7 terms is 77 and 10 terms is 140.
Then sum of 13 terms is 60. If the sequence { S n } is a geometric progression and
(a) 221 (b) 208 (c) 351 (d) 195 S 2S11 = S pS 8, then what is the value of p? e 2012 I
(a) 1 (b) 3
Directions (Q. Nos. 52-55) If A, G and H are (c) 5 (d) Cannot be determined
respectively arithmetic, geometric and harmonic
61. If p, q and r are in AP as well as GP, then which
means between a and b both being unequal and one of the following is correct? e 2012 I
positive, then (a) p = q ≠ r (b) p ≠ q ≠ r (c) p ≠ q = r (d) p = q = r
a+b 2 ab
A= , G = ab and H = 62. What is the sum of first eight terms of the series
2 a+b 1 1 1
The equation having a and b as its roots is 1− + − +L?
2 4 8 e 2012 II
x 2 − 2 A x + G 2 = 0. 89 57 85
(a) (b) (c) (d) None of these
52. If the geometric and harmonic means of two 128 384 128
4 63. The angles of a triangle are in AP and the
numbers are 16 and 12 , then the ratio of one
5 least angle is 30°. What is the greatest angle
number to the other is (in radian)? e 2012 II
π π π
(a) 1 : 4 (b) 2 : 3 (a) (b) (c) (d) π
(c) 1 : 2 (d) 3 : 1 2 3 4

53. Two numbers whose arithmetic mean is 34 and Directions (Q. Nos. 64-65) The sum of first 10 terms
the geometric mean is 16 are and 20 terms of an AP are 120 and 440,
(a) 16, 16 (b) 64, 4 (c) 32, 36 (d) 48, 20 respectively. e 2012 II
MATHEMATICS Sequences and Series 61

64. What is its first term?


75. What is the value of b?
(a) 2 (b) 3 (c) 4 (d) 5
(a) −1 (b) 0 (c) 1
65. What is the common difference? (d) Cannot be determined due to insufficient data
(a) 1 (b) 2 (c) 3 (d) 4
76. f ′ ( a ), f ′ ( b), f ′ ( c) are in
66. If the numbers n − 3, 4n − 2 , 5n + 1 are in AP, (a) AP (b) GP (c) HP
what is the value of n? e 2013 I (d) Arithmetico-geometric progression
(a) 1 (b) 2 (c) 3 (d) 4
67. What is the seventh term of the sequence 77. f ′ ′ ( a ), f ′ ′ ( b), f ′ ′ ( c) are
(a) in AP only (b) in GP only
0, 3, 8, 15, 24 ? e 2013 II
(c) in both AP and GP (d) Neither in AP nor in GP
(a) 63 (b) 48 (c) 35 (d) 33
68. The sum of the first five terms and the sum of 78. If log10 2, log10( 2 x − 1) and log10( 2x + 3) are
the first ten terms of an AP are same. Which three consecutive terms of an AP, then the value
one of the following is the correct statement? of x is e 2015 I

(a) The first terms must be negative e 2013 II (a) 1 (b) log 5 2 (c) log 2 5 (d) log10 5
(b) The common difference must be negative 79. What is the sum of the series
(c) Either the first term or the common difference is 0 . 5 + 0 . 55 + 0 . 555 + K + n terms? e 2015 I
negative but not both
(a)  n −  1 − n   (b)  5 −  1 − n  
5 2 1 1 2 1
(d) Both the first term and the common difference are
negative 9  9 10   9  9 10  

n −  1 − n   n −  1 − n  
1 5 1 5 1 1
69. What is 0.9 + 0.09 + 0.009 + L equal to? e 2013 II (c) (d)
9  9 10   9  9 10  
(a) 1 (b) 1.01 (c) 1.001 (d) 1.1
70. If the positive integers a, b, c and d are in AP, 80. The value of the infinite product
1 1 3 1
then the numbers abc, abd, acd and bcd are in 6 2 × 6 2 × 6 8 × 6 4 × ... is e 2015 II
e 2013 II
(a) 6 (b) 36 (c) 216 (d) ∞
(a) HP (b) AP (c) GP (d) None of these 3+ n
81. If the nth term of an AP is , then the sum
71. The sum of an infinite GP is x and the common 4
ratio r is such that |r|< 1. If the first term of of first 105 terms is e 2015 II
the GP is 2, then which one of the following is (a) 270 (b) 735 (c) 1409 (d) 1470
correct? e 2014 I
82. What is the sum of n terms of the series
(a) −1 < x < 1 (b) − ∞ < x < 1 2 + 8 + 18 + 32 + L ? e 2015 II
(c) 1< x < ∞ (d) None of these n(n − 1)
(a) (b) 2 n (n + 1)
72. The sum of the series formed by the sequence 2
n(n + 1) n(n − 1)
3, 3, 1, ... upto infinity is e 2014 I (c) (d)
2 2
3 3 ( 3 + 1) 3 3 ( 3 − 1)
(a) (b) 83. If p, q, r are in one geometric progression and
2 2
3( 3 + 1) 3( 3 − 1)
a, b, c are in another geometric progression, then
(c) (d) ap, bq, cr are in e 2015 II
2 2 (a) AP (b) GP
(c) HP (d) None of these
Directions (Q. Nos. 73-74) Let S n denotes the sum of
first n terms of an AP and 3S n = S 2n. e 2014 II Directions (Q. Nos. 84-85) Given that
log x y, log z x , log y z are in GP, xyz = 64 and
73. What is S3 n : S n equal to?
(a) 4 : 1 (b) 6 : 1 (c) 8 : 1 (d) 10 : 1 x 3, y3, z 3 are in AP. e 2016 I

74. What is S3 n : S 2n equal to? 84. Which one of the following is correct?
x, y and z are
(a) 2 : 1 (b) 3 : 1 (c) 4 : 1 (d) 5 : 1 (a) in AP only (b) in GP only
(c) in both AP and GP (d) Neither in AP nor in GP
Directions (Q. Nos. 75-77) Read the following
information carefully and answer these question 85. Which one of the following is correct?
given below. xy, yz and zx are
Let f ( x ) = ax 2 + bx + c such that f (1) = f ( −1) and a, b, c (a) in AP only (b) in GP only
are in Arithmetic Progression (AP). e 2014 II (c) in both AP and GP (d) Neither in AP nor in GP
62 NDA/NA Pathfinder

ANSWERS
1 a 2 b 3 a 4 c 5 c 6 a 7 c 8 d 9 b 10 a
11 c 12 c 13 b 14 a 15 c 16 a 17 b 18 b 19 d 20 b
21 d 22 a 23 c 24 b 25 b 26 b 27 c 28 b 29 b 30 b
31 a 32 b 33 c 34 d 35 d 36 b 37 b 38 a 39 b 40 b
41 c 42 a 43 d 44 b 45 c 46 b 47 c 48 b 49 c 50 d
51 a 52 a 53 b 54 d 55 c 56 c 57 d 58 d 59 c 60 c
61 d 62 c 63 a 64 b 65 b 66 a 67 b 68 c 69 a 70 a
71 c 72 a 73 b 74 a 75 b 76 a 77 c 78 c 79 d 80 b
81 d 82 c 83 b 84 c 85 c

HINTS AND SOLUTIONS 


1. (a) 7. (c) Let Sn = an( n − 1), then x −1 y−1
∴ a= , b=
2. (b) Q Tm = Sm − Sm − 1 x y
S = a ( n − 1)( n − 2)
n −1
⇒ 164 = 3 ( 2m − 1) + 5 ⋅ 1 ⇒ 6m = 162 ∴ Tn = Sn − S = 2a ( n − 1) ∴ 1 + ab + a 2 b 2 + … ∞
n −1
∴ m = 27 1 1
Tn 2 = 4a 2 ( n − 1) 2 = =
m + n −1
( n − 1)( n)( 2n − 1) 1 − ab 1 − ( x − 1)( y − 1)
3. (a) T = ar = p,
m + n ∴ Sum = ΣTn2 = 4a 2 xy
Tm − n = ar m − n − 1 = q 6 xy
On multiplying, we get a 2 r 2 m − 2 = pq 2a 2 n( n − 1)( 2n − 1) =
= x + y−1
∴ Tm = ar m − 1 = pq 3
1 3 7 15 15. (c)
4. (c) + + + + ... + n terms 8. (d)
2 4 8 16 9. (b) Let r be the common ratio of GP, 16. (a) Q a, x, y, z, b are in AP.
a + b  a + b
=  1 −  +  1 −  +  1 −  ∴ x + y + z = 3 
1 1 1 then  ⇒ 15 = 3 
 2  4  8 S=
a
, r = 1−
a  2   2 
1− r S
+ ... +  1 − n  ⇒ a + b = 10
1 …(i)
 2  a( 1 − r n ) a
∴ Sn = = (1 − rn ) Also, a , x , y , z , b are in HP.
1− r 1− r
= n −  + + + ... + n 
1 1 1 1 1 1 1 1 1
n ⇒ , , , , are in AP.
 a 
2 4 8 2  = S  1 −  1 −   a x y z b
  S 
1  1 − ( 1 / 2)n  a + b
+ + = 3
−n 1 1 1
= n−   = n+ 2 − 1 ⇒ 
2  1− 1/ 2  10. (a) 11. (c) x y z  2ab 
1 ⇒
5. (c) Middle term = T 12. (c) Q , a, b are in GP.
n+1 16 5 3 × 10
2 = [Q a + b = 10]
1 3 2ab
n+ 1 ⇒ a2 = × b ⇒ b = 16a 2
∴ a +  − 1 d = m
…(i)
[given]
 2  16 ⇒ ab = 9
1
2a + ( n − 1) d = 2m …(i) Also, a, b, are in HP. On solving Eqs. (i) and (ii), we get
n 6 a = 1, b = 9 or b = 1, a = 9
Now, Sn = [ 2a + ( n − 1) d ] = nm 1 1 2 1
2 ⇒ , , 6 are in AP. ⇒ =6+
a b b a 17. (b) Let the first term of AP be a and
6. (a) The given sequence is arithmetico- 2 6a + 1 common difference be d, then
1 ⇒ = ⇒ 8a ( 6a + 1) = 1
geometric series, where r = and d = 1 16a 2 a t = a + md , t = a + nd
5 m +1 n+1
−1 1 and t = a + rd
S∞ =
a
+
dr ⇒ 48a 2 + 8a − 1 = 0 ⇒ a = , r +1
1 − r ( 1 − r )2 4 12 Also, m, n, r in HP.
1 1 2mr

1 ∴ a = − and b = 1 and a = ⇒ n = …(i)
1 5 4 12 m+ r
= + 2 1
1  and b =
1 − 
1− 1 Given, t ,t ,t are in GP.
5   9 m +1 n+1 r +1
5
5 5 25 13. (b) ⇒ ( a + nd ) = ( a + md )( a + rd )
2
= + = ⇒ a 2 + n2 d 2 + 2and
4 16 16 1 1
14. (a) x = , y= = a 2 + ad (m + r ) + mrd 2
1− a 1− b
MATHEMATICS Sequences and Series 63

⇒ n2  d 2 +
2ad  2mr t = a + ( p − 1)d
 = ad × + mrd 2 G
p B
 n 
A
n ⇒ t q = a + ( q − 1)d L K
⇒ n2  d 2 +
2ad 
= mr  + d2
2ad Given,
 n   n  p [ a + ( p − 1)d ] = q [ a + ( q − 1)d ] H F

⇒ ( n − mr ) d +
2 2 2ad 
=0 ⇒ ap + p 2 d − pd = aq + q 2 d − qd
 n  ⇒ ap − aq = d [ q 2 − q − p 2 + p ] I J
2ad a n ⇒ a[ p − q ] = d [( q − p )( q + p − 1)]
⇒ n2 = mr or d 2 = − ⇒ =− D E C
n d 2 ⇒ a( q − p ) + d ( q − p )( q + p − 1) = 0
∴ Required sum
18. (b) Let a be the first term and d the ⇒ ( q − p )[ a + d ( q + p − 1)] = 0 1 1
common difference of corresponding ⇒ q − p = 0 or a + d ( q + p − 1) = 0 = 162 + ( 16)2 + ( 16)2 + …∞
2 4
AP. ⇒ t =0
= ( 16)2  1 + + + … ∞ 
p + q 1 1
mth and nth term of HP are n and m.
1 1 22. (a) a , 2a + 2, 3a + 3 are in GP.  2 4 
So, mth and nth terms of AP are and . ⇒ ( 2 a + 2 ) 2 = (3 a + 3 )a
n m  
⇒ 4a + 4 + 8a = 3a 2 + 3a
2  1 
1 = 162   = 512 sq cm
∴ = a + (m − 1)d …(i)
n ⇒ a 2 + 5a + 4 = 0 ⇒ a = −1, − 4  1 − 1
1  2
and = a + ( n − 1)d …(ii) Now, a = −1 does not satisfy the given
m series. 1 1 1
27. (c) Let , , are in AP.
On solving Eqs. (i) and (ii), we get ∴ −4, − 6, − 9 are in GP. ab ca bc
1 1 3 1 1 1 1 b−c a−b
⇒ − = − ⇒ =
d = and a = t = −4  
3
∴ ca ab bc ca abc abc
mn mn 4  2
∴mnth term of AP ⇒ b − c = a − b ⇒ 2b = a + c
= −13.5
1 1 So, a , b , c are in AP.
= + (mn − 1) ×
mn mn 23. (c) 1 1 1
Now, , , are
∴mnth term of HP = 1 b+ c c+ a a+ b
24. (b) Since, abc = 1
19. (d) Sum ofn AM’s = n × single AM in AP.
As, we know AM ≥ GM 2 1 1
a + b ∴ = +
∴ A + A = 2   = a + b …(i) ⇒
1+ a
≥ a c+ a b + c a + b
1 2  2 
2 ⇒ 2 ( b + c) ( a + b)
Product of n GM’s = (single GM) n ⇒ 1+ a≥ 2 a …(i)
= ( c + a) ( a + 2 b + c)
G G = ( ab )2 = ab …(ii) Similarly, 1 + b ≥ 2 b …(ii)
1 2 ⇒ 2b = a + c
1 1 1 1 and 1+ c≥ 2 c …(iii)
, , , are in AP. So, a , b , c are in AP.
a H H b On multiplying Eqs. (i), (ii) and (iii),
1 2 we get Hence, both the statements are correct.
1 1 1 1 a+ b a+ c
∴ + = + = ( 1 + a )( 1 + b )( 1 + c ) ≥ 8 abc 28. (b) Given, b = …(i)
H H a b ab 2
1 2 ∴ Least value of
[from Eq. (i)] 2a 2 c 2
( 1 + a )( 1 + b )( 1 + c ) = 8 b2 = 2 …(ii)
−1 −1 A + A a + c2
⇒H + H = 1 2
1 2 25. (b) A is the arithmetic mean of a and b. From Eqs. (i) and (ii), we get
GG
1 2
a+ b ( a + c )2 2a 2 c 2
[from Eqs. (i) and (ii)] ∴ A= = 2
2 4 a + c2
20. (b) Given, series can be rewritten as p and q are two geometric means between
77 37 71 ⇒ ( a + c + 2ac )( a 2 + c 2 ) = 8a 2 c 2
2 2

20, , , ,… a and b.
4 2 4 1/3 ⇒ ( a 2 + c 2 )2 + 2ac ( a 2 + c 2 )
∴ p = a  
b
= a 2 / 3 ⋅ b1 / 3 + a 2 c 2 = 9a 2 c 2
This is an AP series.  a
3 ⇒ ( a + c + ac ) = 9a c
2 2 2 2 2
Here, a = 20 and d = − 2/3
and q = a  
b
4 = a1 / 3 b 2 / 3 ⇒ a 2 + c 2 + ac = ±3ac
 a
∴ Tn = a + ( n − 1) d ∴ either a 2 + c 2 = 2ac
⇒ p 3 + q 3 = a 2 b + ab 2 = ab( a + b ) ⇒ ( a − c )2 = 0 ⇒ a = c ⇒ a = b = c
= 20 + ( n − 1)  −  =
3 83 3
− n = 2 Aab = 2 Apq
 4 4 4 [from Eq. (i)]
For first negative term, Tn < 0 26. (b) Given, AB = 16 cm or a 2 + c 2 + ac = −3ac
83 3
⇒ − n < 0 ⇒ 83 < 3n ⇒ n >
83
∴ HG = 8 2 =
16
cm ⇒ ( a + c )2 = −2ac
4 4 3 2 ⇒ 4b 2 = −2ac
So, n should be 28. 16 ⇒ 2b 2 = − ac
Similarly, KL = ( 4 2 ) 2 = cm [from Eq. (i)]
2 a
Hence, 28th term is first negative term. ∴ − , b, c are in GP.
2
21. (d) Let a be the first term and d be Thus, only Statement II is correct.
common difference of AP, then
64 NDA/NA Pathfinder

m
29. (b ) Given, 12, 18, 24 are is AP. Now, Sn = [ 2a + (m − 1)2a ] = am 2 39. (b) Last term of series
2
If 18 is added to each term, then the Sm S = 1 × 2100 − 1 = 299
series is ( 12 + 18), ( 18 + 18), ( 24 + 18) ⇒ m= 1

or 30, 36, 42 which is in AP. a 40. (b) For as S (i.e. GP) Tn = 2n − 1


1
Also, 4, 8, 16 are in GP. Sn For as S (i.e. AP)
Similarly, n = 2
If 8 is added to each term, then series is a Tm = 1 + (m − 1) 3 = 3m − 2
12, 16, 24 which is in HP am a + (m − 1)d 4S m − a They are common, if 2n − 1 = 3m − 2
∴ = = 3m
Hence, Statement II is correct. an a + ( n − 1)d 4S n − a ⇒ 2n − 2 + 1 = ≤ 150
30. (b) The given series is an AGP. 2
Let S = 1 + 2x + 3x 2 + K ∞ …(i)
35. (d) Let the sides of triangle in increasing ⇒ n ≤ 9, m ≤ 100
form be ( a − d ), a , ( a + d ). As, 2n − 1 = 3m − 2
⇒ x ⋅ S = x + 2x 2 + 3x 3 + K ∞ …(ii)
∴( a + d )2 = ( a − d )2 + a 2 ∴ ( n = 1, m = 1), ( n = 3, m = 2),
On subtracting Eq. (ii) from Eq. (i),
⇒ a 2 + d 2 + 2ad ( n = 5, m = 6), ( n = 7, m = 22),
we get
1 = a 2 + d 2 − 2ad + a 2 ( n = 9, m = 86),
(1 − x ) S = 1 + x + x 2 + K ∞ = ⇒ a 2 − 4ad = 0 4d

5d
1− x and for n = 2, 4, 6, 8 ; m is a fraction
1 ⇒ a( a − 4d ) = 0 which is not possible.
∴ S= 3d
( 1 − x )2 ⇒ a = 4d [Q a ≠ 0] Hence, number of common terms = 5
∴ Sides of triangle are 3d, 4d and 5d.
31. (a) Since, a and b are unequal. 1 41. (c) Sum of 100 terms of series S
a2 + b2 ∴ Area of triangle = × 3d × 4d = 6d 2 100
2
∴ > a2b2 2 = [ 2 × 1 + ( 100 − 1) × 3]
2 2
36. (b) Since, 3d, 4d and 5d are three
[Q AM > GM for unequal numbers] arithmetic means between two numbers = 50 [ 2 + 99 × 3] = 50 × 299 = 14950
⇒ a 2 + b 2 > 2ab …(i) say x and y. 42. (a) Q Sn = 3n2 + 5n
Similarly, b 2 + c 2 > 2ac …(ii) ∴ x, 3d, 4d, 5d, y are in AP. ∴ Tn = Sn − S
n −1
and c 2 + a 2 > 2ca …(iii) ⇒ x = 2 × (3 d ) − 4 d = 2 d = (3n2 + 5n) − [3( n − 1)2 + 5( n − 1)]
and y = 2 × (5d ) − 4d = 6d
On adding Eqs. (i), (ii) and (iii), we get = (3n2 + 5n) − [3( n2 + 1 − 2n) + 5n − 5]
Sum of AP = 15 = 3 n2 + 5 n − 3 n2 − 3 + 6 n − 5 n + 5
2( a 2 + b 2 + c 2 ) > 2( ab + bc + ca ) ⇒ 2d + 3d + 4d + 5d + 6d = 15 = ( 6n + 2)
⇒ ab + bc + ca < 1 [Q a 2 + b 2 + c 2 = 1] 3
⇒ 20d = 15 ⇒ d = The nth term is a linear function in n.
4 Hence, sequence must be an AP.
32. (b) When ‘n’ is even. 9 15
∴ Sides of triangle are , 3, .
Let n = 2m, then 4 4 43. (d) Given,
= 12 − 22 + 32 − 42 + 52 − K a+ b b+c T n = 5456
37. (b) Given, x = and y =
= ( 12 − 22 ) + (32 − 42 ) + (52 − 62 ) 2 2 ⇒ 6n + 2 = 5456
+ K + ( 2m − 1)2 − ( 2m )2 Also, a, b, c, are in GP ⇒ 6n = 5454 ⇒ n = 909
= ( 1 + 2)( 1 − 1) + (3 + 4)( − 1) ⇒ b 2 = ac ∴ The number 5456 is (909)th term.
a c a c
+ (5 + 6)( − 1) + K Now, + = + 44. (b) T 2 + T 2 + T 2
x y a+ b b+c 1 2 3
+ ( 2m − 1 + 2m )( − 1)
2 2 = (8)2 + ( 14)2 + ( 20)2
= − ( 1 + 2 + 3 + 4 + ... + 2m ) 2a 2c 2ab + 2ac + 2ac + 2bc
= + = = 64 + 196 + 400 = 660
= −2m( 2m + 1) = − n ( n + 1) a+ b b+ c ( a + b )( b + c )
2 2 Sol. (Q. Nos. 45-47) Let four integers be a − d ,
2[ ab + 2ac + bc ] a, a + d and a + 2d .
33. (c) Here, t = ar , t ′
k
= br k =
k +1 k +1 ab + ac + b 2 + bc where, a and d are integers and d > 0
∴ t ′′ =t + t′ 2[ ab + 2ac + bc ]
k +1 k +1 k +1 = = 2 [Q b 2 = ac] Q a + 2d = ( a − d )2 + a 2 + ( a + d )2
ab + ac + ac + bc
= ar + br = r ( a + b )
k k k ⇒ 2d 2 − 2d + 3a 2 − a = 0 …(i)
a+ b b+ c 1
which is also a GP. 38. (a) x + y = + ∴ d = [ 1 ± 1 + 2a − 6a ] 2 …(ii)
m 2 2 2
{ 2a + (m − 1)d } a + 2b + c a + 2 a c + c
S m2 = = Since, d is positive integer.
34. (d) m = 2n = 2 2 2
Sn { 2a + ( n − 1)d } n ∴ 1 + 2a − 6a 2 > 0
2 ( a + c )2
= 6a 2 − 2a − 1 < 0
m − 1
a + 
2
d 1− 7 1+ 7
 2  m ∴ Statement I is correct. ⇒ < a<
⇒ = b2 6 6
 n − 1 n a + 2b +
a+  d a + 2b + c Q a is an integer.
 2  Also, x + y = = a
d 2 2 ∴ a = 0 put in Eq. (ii)
⇒ a( n − m ) = {mn − m − mn + n} a 2 + 2ab + b 2 ( a + b )2 ∴ d = 1 or 0 but Eq. (ii) Q d > 0
2 = =
d 2a 2a ∴d = 1
⇒ a = ⇒ d = 2a [Qm ≠ n] ∴ Statement II is not correct.
2 So, the four numbers are − 1, 0, 1, 2.
MATHEMATICS Sequences and Series 65

45. (c) The smallest number is − 1. 55. (c) A = mλ, H = nλ 1 1 1


62. (c) Given series is 1 − + − +L
∴G 2 = AH = mnλ2 2 4 8
46. (b) Common difference = 0 − ( − 1) = 1
Qa, b are the roots of Since, it is a geometric progression.
47. (c) Sum of all the four numbers Here, first term, a = 1
x 2 − ( a + b )x + ab = 0
= − 1+ 0 + 1+ 2 = 2 and common ratio, r = − < 1
1
⇒ x 2 − 2 Ax + G 2 = 0
2
48. (b ) Q Sn = pn + qn2 ⇒ x − 2mλ + mnλ2 = 0
2
∴ Sum of first eight terms of the series,
∴ Tn = Sn − S
x = 2mλ ± 4m λ − 4mnλ
2 2 2
n −1 ∴ a (1 − r8 )
i.e. S =
= ( pn + qn ) − { p( n − 1) + q( n − 1) }
2 2 2 8
( 1 − r)
= pn + qn2 − { pn − p + qn2 + q − 2qn} = λ m( m ± m − n )  a( 1 − rn ) 
= p + q( 2n − 1) a m + m−n  by formula , Sn = , when r < 1
⇒ = 1− r
m − m−n  
b  
49. (c) Common difference = Tn − T
n −1 56. (c) 8
 1 
= p + q( 2n − 1) − [ p + q { 2( n − 1) − 1}] 57. (d) Given series is 1 1 −  −   1 − 1
= p + 2qn − q − { p + 2qn − 3q } = 2q 1 1  2
1+ +3+ +L =  = 256 = 85
3 3 3 
1 − − 
1 1+
1 128
50. (d) S = pn + qn2 , S = 2 pn + 4qn2 Here, between each two consecutive  2 2
1 2
and S = 3 pn + 9qn2 terms, no common difference and
3
common ratio are form. 63. (a) Let the angles of triangle be a,
∴ 2S − S + S = 2( 2 pn + 4qn2 ) a + d and a + 2d .
2 3 1 Hence, the given series does not form
− (3 pn + 9qn2 ) + ( pn + qn2 ) = 2 pn any series. Given, a = 30°
1 1 1 Q a + a + d + a + 2d = 180°
and S − ( S − S ) = (3 pn + 9qn2 ) 58. (d ) Given series is 1 − + − + ...
3 2 1 2 4 8 ∴ 3 a + 3 d = 180°
− ( 2 pn + 4qn2 ) − ( pn + qn2 ) = 4qn2 which form a GP with common ratio ⇒ 3 × 30° + 3 d = 180°
51. (a) Sum of 7 terms, S = 7 p + 49q = 77  − 1. ⇒ 3 d = 90° ⇒ d = 30°
 
7  2 ∴Angles of triangle are 30°, 60° and 90°.
⇒ p + 7q = 11 …(i)
∴ Sum of infinite term of GP π
Also, sum of 10 terms, Hence, the greatest angle = 90° =
a 1 1 2 2
S = 10 p + 100q = 140 = = = =
10 1 − r 1 −  − 1  1 + 1 /2 3 Sol. (Q. Nos. 64-65) Let the first term of an AP
⇒ p + 10q = 14 …(ii)  
 2 is a and common difference is d.
On solving Eqs. (i) and (ii), we get
59. (c) Given that, 1/4, 1/x and 1/10 are in Given, S = 120 and S = 440
p = 4 and q = 1 HP.
10 n 20
Q Sn = [ 2a + ( n − 1) d ]
∴ S = 13 p + 169q ⇒ 4, x and 10 are in AP, then 2
13
= 13( 4) + 169( 1) = 221 2x = 4 + 10 ⇒ x = 7 ∴ S =
10
[ 2a + ( 10 − 1) d ]
10
2
52. (a) ab = 16 ⇒ ab = 162 = 8 × 32 60. (c) We know that, in a GP the product of
2ab 4 2 × 256 64 two terms equidistant from the beginning ⇒ 120 = 5( 2 a + 9 d )
and = 12 ⇒ =
a+ b 5 a+ b 5 and end is a constant and is equal to the ⇒ 2a + 9 d = 24 ...(i)
product of first term and last term, i.e. if 20
⇒ a + b = 40 = 8 + 32 and S = [ 2a + ( 20 − 1) d ]
20
a 8 1 a , a , a ,..., a ,( a ) an are in GP, 2
∴ = = ⇒ a : b = 1: 4 1 2 2 (n − 2 ) n −1
b 32 4 then a an = a a =a a =K ⇒ 440 = 10 ( 2a + 19 d )
1 2 n −1 3 n− 2
a+ b Given that, ⇒ 2a + 19 d = 44 ...(ii)
53. (b) AM = 34 ⇒ = 34
2 S S = S S ⇒ ( p + 8) = ( 2 + 11) On solving Eq. (i) and Eq. (ii), we get
2 11 p 8
a = 3 and d = 2
⇒ a + b = 68 …(i) ∴ p = 13 − 8 = 5
and GM = 16 ⇒ ab = 16 ⇒ ab = 256 64. (b) 65. (b)
61. (d ) Given that, p, q and r are in AP.
∴ ( a − b )2 = ( a + b )2 − 4ab ∴ 2q = p + r ...(i) 66. (a) Given that, ( n − 3), ( 4n − 2), (5n + 1)
⇒ ( a − b )2 = ( 68)2 − 4 × 256 = 3600 are in AP.
As well as are in GP.
⇒ a − b = 60 …(ii) ∴( 4n − 2) − ( n − 3) = (5n + 1) − ( 4n − 2)
∴ q 2 = pr ...(ii)
On solving Eqs. (i) and (ii), we get ⇒3n + 1= n + 3 ⇒3n − n = 3 − 1 ⇒ 2n = 2
From Eqs. (i) and (ii), we get
a = 64 and b = 4 ∴ n=1
p + r = 2 pr
54. (d) We have, H = 4, G 2 = AH ⇒ ( p )2 − 2 p ⋅ r + ( r )2 = 0 67. (b) Given pattern is
⇒ 2 A + G 2 = 27 ⇒ 2 A + AH = 27 ⇒ ( p − r )2 = 0
0 3 8 15 24
9
⇒ 6 A = 27 ⇒ A = ⇒ p = r ⇒ p= r ...(iii) 3 5 7 9
2
From Eq. (ii), we get ∴ Sixth term = 24 + 11 = 35
∴ G 2 = 18 and seventh term = 35 + ( 11 + 2) = 48
q2 = r ⋅ r = r2 ⇒ q = r ...(iv)
∴ a + b = 9 and ab = 18 Now, from Eqs. (iii) and (iv), we get 68. (c) Let a be the first term and d be the
⇒ a = 3, b = 6 or a = 6, b = 3 p=q=r
common difference of an AP.
66 NDA/NA Pathfinder

10  10 −
n
Given, S =S S 6 n2 d 1
5 10 74. (a) 3n = = 2: 1 =
5n

5
×  
5 10 S 3 n2 d 9  10n
⇒ ( 2a + 4d ) = ( 2a + 9d ) 2n 9 90
 
2 2
=  n −  1 − n  
75. (b) We have, 5 1 1
⇒ a + 2 d = 2a + 9d ⇒ a + 7d = 0 9  9 10  
f ( x ) = ax 2 + bx + c
∴ a = − 7d ...(i)
f ( 1) = a + b + c 1 1 3 1
Thus, the series is and f ( −1) = a − b + c 80. (b) 6 2 × 6 2 × 6 8 × 6 4 ×... ∞
−7d , − 6d , − 5d , − 4d , − 3d ,... ∴ f ( 1) = f ( −1) 1 1 3 1 1 2 3
(1 + + +
4
+ ... )
+ + + + ... ∞
Hence, either the first term or the ⇒a+ b+ c = a− b+ c = 62 2 8 4 = 62 2 22 23
common difference is negative but not ⇒ b = 0 2 3 4
Let S = 1 + + 2 + 3 + ..., then
both. 2 2 2
76. (a) We have, f ′( x ) = 2ax + b 1 1 2 3
69. (a)
∴ f ′( a ) = 2a 2 , f ′( b ) = 2ab = 0 S = + 2 + 3 +....
2 2 2 2
70. (a)a, b, c, d are in AP. and f ′( c ) = 2ac [Qb = 0] 1 1 1 1
a b c d Now, S − S = 1 + + 2 + 3 + .....
⇒ , , , are in AP. ∴ f ′( a ) = 2a 2 , f ′( b ) = 0 2 2 2 2
abcd abcd abcd abcd
and f ′( c ) = − 2a 2 ⇒
1
S=
1
= 2⇒S = 4
1 1 1 1
⇒ , , , are in AP. [Q 2b = a + c ⇒ c = − a] 2 1
bcd acd abd abc 1−
Hence, f ′( a ), f ′( b ) and f ′( c ) are in AP. 2
⇒ bcd , acd , abd , abc are in HP. 1 1 3 1
77. (c) f ′′( x ) = 2a ∴ 6 2 × 6 2 × 6 8 × 6 4 ... ∞
71 (c) Given that,
sum of an infinite GP = x ⇒
a
=x ∴ f ′′ ( a ) = f ′′ ( b ) = f ′′ ( c ) 1
×4

1− r = 62 = 36
Hence, f ′′ ( a ), f ′′ ( b ) and f ′′( c ) are in

2
=x 3+ n
…(i) both AP and GP. 81. (d) We have, t n =
1− r 4
[Q given that, a = 2 and |r | < 1 ] 78. (c) 4 + ( n − 1) 1
⇒ tn = = 1 + ( n − 1)
Q |r | < 1 ⇒ − 1 < r < 1 . + 055
79. (d) 05 . + 0555
. + K n terms 4 4
⇒ 1> − r > − 1 ⇒ 1 + 1> 1 − r > 1 − 1 1
=
5
+
55
+
555
+ K n terms ∴First term = 1, Common difference =
⇒ 0 < 1 − r < 2 ⇒ ( 1 − r) < 2 10 100 1000 4
105 
2 + 104 ×  = 1470
5  1
+ K n terms  ∴S =
1 1 2 11 111
⇒ > ⇒ >1 = 1+ +
1− r 2 1− r 10  10 100 
105 2  4 
⇒ x>1
+ K n terms 
[from Eq. (i)] 5 1 99 999 82. (c) We have,
= × 9+ +
Hence, x ∈ ( 1, ∞ ), i.e. 1 < x < ∞. 10 9   2 + 2 2 + 3 2 + 4 2 + L n terms
10 100 
72. (a) 5  (102 − 1) = 2 [ 1 + 2 + 3 + 4+ .... n terms ]
= (10 − 1) + n ( n + 1)  n( n + 1)
73. (b) We have, 90  10 = 2 =
 2  2
Sn = Sum of first n terms of an AP (103 − 1) 
+ +K n terms 

n
Sn = [ 2a + ( n − 1) d ] 102  83. (b)
2 5  102 103  Sol. (Q. Nos. 84-85)
2n =  10 + + + K n terms 
Similarly, S = [ 2a + ( 2n − 1) d ] 90  10 102  Given, log x y , log z x , log y z are in GP.
2n
2 5  1 1 1 ⇒ (log z x )2 = log x y × log y z
3n + −1 − − −
and S =
3n
[ 2a + (3n − 1) d ] 90  10 10−2 10−3 1
2 = log x z =
3 Sn = S − K n terms  log z x
Now, 
2n 
 n
⇒ 3   [ 2a + ( n − 1) d ] 5 ⇒ (log z x )3 = 1 ⇒ log z x = 1 ⇒ x = z
 2 = [10 + 10 + 10 + K n terms]
90 Now, x 3 , y 3 , z 3 are in AP.
= 2   [ 2a + ( 2n − 1) d ]
n
+ ( −1)  1 +
5 1 1 1
 2 + + ∴ 2 y 3 = x 3 + z3 ⇒ 2 y 3 = z3 + z3
90  10 102 103
⇒ 2a = d ( n + 1) ⇒ y 3 = z3 ⇒ y = z [Qx = z ]
n
∴ Sn = [ d ( n + 1) + d ( n − 1)] = n2 d + K n terms  ∴ x = y =z

2 
n Also, xyz = 64 ⇒ xyz = 43
⇒ S = n [ d ( n + 1 + 2n − 1)] = 3n2 d   1 
1−    ⇒ x = y =z= 4
5 
2n
3n
and S = [ d ( n + 1 + 3n − 1)] = 6n2 d 5  10 
3n = × 10n −  
2 90 90  1 − 1  84. (c) x, y, z are in both AP and GP.
S 6 n2 d  10  85. (c) xy, yz and zx are in both AP and GP.
∴ 3n = 2 = 6 : 1
Sn n d
06
MATHEMATICS Decimal Fractions 67

QUADRATIC EQUATIONS
AND INEQUALITIES
In NDA exam, generally 4-6 questions are asked from this chapter which are based on nature of
roots, finding roots in different conditions, find equation when roots are given and solving in
equations etc.

When we equate quadratic polynomial of the form ( ax 2 + bx + c ) equal to zero we get a quadratic
equation, where a, b and c are real numbers and a ≠ 0.

POLYNOMIAL AND POLYNOMIAL EQUATION


An expression of the form a 0 x n + a1 x n− 2 + …+ a n−1 x + a n where a 0 , a1 , a 2 . . . a n are constant ( a ≠ 0 ) and
n is a positive integer is called a polynomial in x of degree n.
If f ( x) is a real or complex polynomial, then f ( x) = 0 is known as a polynomial equation.
e.g. If x 2 + 3x + 2 is a real polynomial, then x 2 + 3x + 2 = 0 is a polynomial equation.

QUADRATIC EQUATION
If f ( x) is a polynomial of degree 2, then f ( x) = 0 is called a quadratic equation. The general form of a
quadratic equation is ax 2 + bx + c = 0, where a, b and c are real numbers and a ≠ 0. Here, x is the
variable and a, b, c are the real coefficients.

Roots of a Quadratic Equation


The values of the variable satisfying the given quadratic equation are called roots of that equation. In
other words, x = α is a root of the equation, f ( x) = 0, if f (α ) = 0.
The set of all roots of an equation, in a given domain, is called the solution set of the equation. The
quadratic equation ax 2 + bx + c = 0, where a, b, c ∈ R and a ≠ 0 has two roots, namely
− b+ D − b− D
α= and β =
2a 2a
where, D = b 2 − 4ac is called the discriminant.

Note If one of the root of the quadratic equation is a + ib or a + b , then the other root will be a − ib or a − b.
68 NDA/NA Pathfinder

EXAMPLE 1. If x 2 + x − 6 is a factor of polynomial EXAMPLE 2. If the equation


x 3 + Px 2 + Q then the values of P and Q are respectively (3 x ) 2 + (27 × 31/p − 15) x + 4 = 0
a. 8 and 17 b. 7 and −36 c. − 9 and 4 d. −5 and 14 has equal roots, then p is equal to
1
a. 0 b. 2 c. −
d. None of these
Sol. b. Let P( x) = x 3 + Px 2 + Q 2
Sol. c. The given equation will have equal roots iff
Since, x 2 + x − 6 is a factor of P( x). discriminant = 0
∴ Roots of x 2 + x − 6 = 0 satisfy the equation P( x) = 0. ⇒ ( 27 × 31/ p − 15) 2 − 4 × 9 × 4 = 0 [Q D = b2 − 4ac]
Now, x + x −6 =0
2
⇒ ( 27 × 31/ p − 15) 2 − 144 = 0
⇒ x 2 + 3x − 2x − 6 = 0
⇒ ( 27 × 31/ p − 15) 2 = 144 ⇒ 27 × 31/p − 15 = ± 12
⇒ ( x + 3) ( x − 2) = 0
⇒ 27 × 31/p = 27 or 27 × 31/p = 3
⇒ x = − 3, 2
1 1 1
∴ P ( −3) = 0 ⇒ − 27 + 9P + Q = 0…(i) ⇒ = 3−2 ⇒
31/p = 1 or 31/p = = 0 or =−2
9 p p
and P ( 2) = 0 ⇒ 8 + 4P + Q = 0…(ii) 1 1
On solving Eqs. (i) and (ii), we get P = 7 and Q = − 36 But, cannot be zero. So, p = −
p 2

Nature of the Roots of a Relation between Roots and


Quadratic Equation Coefficients
Let the quadratic equation be ax 2 + bx + c = 0, a, b, c ∈ R
1. Quadratic equation Consider the quadratic
and a ≠ 0. The nature of the roots of a quadratic equation
equation is decided by discriminant (i.e. D = b 2 − 4ac ) ax 2 + bx + c = 0, where a, b, c ∈ R and a ≠ 0
(i) If b 2 − 4ac > 0, then the quadratic equation has two If α and β are the roots of the equation, then
real and distinct roots. − b − Coefficient of x
(ii) If b 2 − 4ac = 0, then the quadratic equation has two Sum of roots, α + β = =
a Coefficient of x 2
−b
equal roots i.e. α = β = . c Constant term
2a and product of roots, αβ = =
a Coefficient of x 2
(iii) If b 2 − 4ac < 0, then the quadratic equation has two
distinct complex roots, namely 2. Cubic equation If α, β, γ are the roots of the cubic
equation ax 3 + bx 2 + cx + d = 0, a ≠ 0, then
− b + i 4ac − b 2 − b − i 4ac − b 2
α= and β = Sum of roots, α + β + γ = −b / a
2a 2a
(iv) If a, b, c ∈ Q and D is a perfect square, then equation Sum of product of two roots, αβ + βγ + γα = c / a and
has rational roots. Product of three roots, αβγ = − d / a
(v) The roots are of the form p + q ( p, q ∈ Q) iff a, b, c EXAMPLE 3. If the roots of the equation
are rational and D is not a perfect square. k k +1
4β 2 + λβ − 2 = 0 are of the form and , then
k +1 k +2
SOME IMPORTANT POINTS
what is the value of λ?
(i) If the roots of ax2 + bx + c = 0 are both positives,
then the signs of a and c should be a like and opposite a. 2k b. 7 c. 2 d. k + 1
to the sign as b. k+1
k
(ii) If the roots of ax2 + bx + c = 0 are of opposite signs, Sol. b. Let and be the roots of the equation
k+1 k+2
then the sign of a is opposite to the sign of c .
(iii) If the roots of ax2 + bx + c = 0 are equal in k k+1 λ
4β 2 + λβ − 2 = 0, then + =− ...(i)
magnitude, but opposite in sign, then b = 0. k+1 k+2 4
(iv) If the roots of ax2 + bx + c = 0 are reciprocal of each k k+1 2
and × =−
other, then c = a . k+1 k+2 4
(v) It roots are negative, then a, b, c are of same sign. k 1
⇒ =− ⇒ 2k = − k − 2
(vi) The condition that the roots of the equation k+2 2
ax 2 + bx + c = 0 may be in the ratio m : n is 2
∴ k=−
mnb 2 = ac ( m + n ). 3
MATHEMATICS Quadratic Equations and Inequalities 69

On putting the value of k in Eq. (i), we get


2 2 2 1
Method to Find Common Root
− − +1 −
3 + 3 λ 3+ 3 =−λ To find the common root of two equations, make the
=− ⇒
2 2 4 1 4 4 coefficient of second degree terms in two equations
− +1 − +2
3 3 3 3 equal and subtract. The value of x, so obtained is the
1 λ required common root.
⇒ − 2+ = − ⇒ λ =7
4 4
Condition for Common Roots
Formation of a Quadratic
When only one root is common If α is common root
Equation from Given Roots of the equations
If α, β are the roots of a quadratic equation, then the a1 x 2 + b1 x + c1 = 0 …(i)
equation is
and a 2 x + b2 x + c 2 = 0
2
…(ii)
x 2 − (sum of the roots) x + (product of roots) = 0
Then, the required condition for one common root is
i.e. x 2 − (α + β ) x + αβ = 0
( a1 b2 − a 2 b1 )( b1 c 2 − b2 c1 ) = ( c1 a 2 − c 2 a1 ) 2
Note If α, β, γ are the roots of a cubic equation, then the equation
is x 3 − (α + β + γ ) x 2 + (αβ + βγ + γα ) x − αβγ = 0. and the value of the common root is
c a − c 2 a1 b c − b2 c1
α= 1 2 or 1 2
Equations in Terms of the Roots of a1 b2 − a 2 b1 c1 a 2 − c 2 a1
Another Equation When both roots are common If the Eqs. (i) and (ii)
If α, β are the roots of the equation ax + bx + c = 0, then
2
have both roots common, then these equations will be
identical. Thus, the required condition for both roots
(i) α ± h, β ± h are the roots of the equation
common is
a ( x m h) 2 + b ( x m h) + c = 0
a1 b1 c1
(ii) αh, βh are the roots of the equation = =
a 2 b2 c 2
ax 2 + bhx + h 2 c = 0
α β Note If two quadratic equations with real coefficients have an
(iii) and are the roots of the equation imaginary root common, then both roots will be common.
h h
ah 2 x 2 + bhx + c = 0 EXAMPLE 5. If x 2 − ax + b = 0 and x 2 − px + q = 0
(iv) − α, − β are the roots of the equation ax 2 − bx + c = 0. have a root in common and the second equation has
equal roots, then
1 1
(v) , are the roots of the equation cx 2 + bx + a = 0. ap ap
α β a. b + q = b. b + q = ap c. b + q = 2ap d. b + q =
2 4

EXAMPLE 4. If α, β are roots of x 2 − 3x + 1 = 0, then Sol. a. Given equations are x 2 − ax + b = 0 …(i)


1 1 and x − px + q = 0
2
…(ii)
the equation whose roots are , is
α −2 β −2 Let α be the common root. Then, roots, of Eq. (ii) will be
α and α.
a. x2 + x − 1 = 0 b. x2 − x − 1 = 0
Let β be the other root of Eq. (i), then
c. x2 + x + 1 = 0 d. None of these
α + β = a , αβ = b
Sol. b. α , β are the roots of the equation x 2 − 3x + 1 = 0, then From Eq. (ii), α + α = 2α = p and α 2 = q
α + β = 3 and αβ = 1. Now,
p
b + q = αβ + α 2 = α (α + β) = × a =
ap
1 1 α + β −4 3− 4 2 2
S= + = = =1
α − 2 β − 2 αβ − 2(α + β) + 4 1− 2( 3) + 4
1 1 Symmetric Function of Roots of a
and p = = =1 Quadratic Equation
(α − 2) (β − 2) αβ − 2 (α + β) + 4
1 Let α and β be the roots of a quadratic equation. An
Hence, the equation whose roots are expression in α and β which remains same when α and β
α −β
are interchanged, is known as a symmetric function in α
1
and is x 2 − Sx + p = 0, i.e. x 2 − x − 1= 0 and β, then
β−2
70 NDA/NA Pathfinder

(i) α 2 + β 2 = (α + β ) 2 − 2αβ Use following steps to solve it.


(ii) (α − β ) = (α + β ) − 4αβ
2 2
Step I Equate the given expression to y.
(iii) α − β = (α − β ) (α + β ) = (α + β ) (α + β ) − 4αβ
2 2 2
Step II Obtain quadratic equation in x by simplifying
(iv) α 3 + β 3 = (α + β ) 3 − 3αβ (α + β ) the expression in step I.
(v) α 3 − β 3 = (α − β ) 3 + 3αβ (α − β ) Step III Put discriminant ≥ 0 of the equation which we
= (α − β ) (α 2 + αβ + β 2 ) get in step II.
(vi) α 4 + β 4 = (α 2 + β 2 ) 2 − 2α 2β 2 Step IV The values of y obtained by D ≥ 0 is the
solution set for the given rational expression.
(vii) α 4 − β 4 = (α + β ) (α − β ) (α 2 + β 2 )
= (α + β ) (α − β ) [(α + β ) 2 − 2αβ ] x 2 − 3x + 4
EXAMPLE 7. The expression lies between
x 2 + 3x + 4
EXAMPLE 6. If α and β are the roots of 1 1 1 1
x β a. and 7 b. and 8 c. and 5 d. and 9
ax 2 + 2bx + c = 0, then + is equal to 7 8 5 9
β α
x2 − 3x + 4
4b2 − 2ac 4b2 − 4 ac 2b2 − 2ac 2b2 − 4 ac Sol. a. Let y =
a. b. c. d. x2 + 3x + 4
ac ac ac ac ⇒ x2( y − 1) + 3x ( y + 1) + 4( y − 1) = 0
Sol. a. α , β are the roots of equation ax + 2bx + c = 0
2
Since, x is real.
−2b c ∴ D ≥ 0 ⇒ 9 ( y + 1) 2 − 16 ( y − 1) 2 ≥ 0
α +β = and αβ =
a a
⇒ −7y 2 + 50y − 7 ≥ 0 ⇒ 7y 2 − 50y + 7 ≤ 0
α β α +β (α + β) 2 − 2αβ
2 2
∴ + = = ⇒ (7y − 1) ( y − 7) ≤ 0
β α αβ αβ
α β ( 4b / a ) − 2c / a 4b − 2ac
2 2 2 1
⇒ + = = ⇒ ≤ y ≤7
β α c/ a ac 7

Maximum and Minimum Value of INEQUATIONS


ax 2 + b x + c A statement involving one or more variables and sign
of inequality >,<,>, or ≤, is called an inequation.
 b
2
 4ac − b2 
Q ax 2 + bx + c = a  x +  +  Note For any real number a
 2a   4a  |x| ≤ a ⇔ − a ≤ x ≤ a
l

Case I If a > 0 |x| ≥ a ⇔ − x ≤ − a or x ≥ a


l

4ac − b 2
Then, minimum value of ax 2 + bx + c is Solution of Quadratic Inequations
4a
b Let f ( x) = ax 2 + bx + c, where a, b, c ∈ R and a ≠ 0.
and this value occurs when x = − . There is no
2a Then, f ( x) ≥ 0, f ( x) > 0, f ( x) ≤ 0, f ( x) < 0 are called
maximum value when a > 0. quadratic inequations.
Case II If a < 0 The set of real values of x, which satisfy the inequation,
4ac − b 2
Then, maximum value of ax 2 + bx + c is is called the solution set.
4a
b
and this value occurs when x = − . There is no Solution of Linear Inequations
2a
minimum value when a < 0. in Two Variables
In order to represent the solution set of linear
Method to Solve Fractional inequation in two variables, we follow the following
Quadratic Polynomial steps
Consider the fractional quadratic polynomial be Step I Convert the given inequation say ax + by ≤ c
a1 x 2 + b1 x + c1 into the equation ax + by = c and draw the
a 2 x 2 + b2 x + c 2 graph.
MATHEMATICS Quadratic Equations and Inequalities 71

Step II Choose a point not lying on this line ax + by = c 2x + 3y ≤ 6, we first draw the line 2x+3y=6.
substitute its coordinates in the inequation. If Let us take a point (0,0).
Y
the inequation is satisfied, then shade the
B (0, 2)
portion of the plane which contains the chosen
point, otherwise shade the portion which does A(3, 0)
X′ X
not contain the chosen point. O

Step III The shaded region obtained in step II represent


the desired solution set. Y′
Clearly, (0, 0) satisfy the given inequation, so the region
EXAMPLE 8. The number of positive integral containing the origin is represented in the figure.
solutions satisfying the inequation 2x + 3y ≤ 6 is Now, (1, 1), (2,0), (3,0), (0,1), (0,2) are positive integral
a. 2 b. 4 c. 6 d. 5 solutions of 2x + 3y ≤ 6.
∴ Number of positive integral solutions = 6
Sol. c. To represent the solution set of the inequation

PRACTICE EXERCISE
(a) acx 2 + (a2 + bc ) x + bc = 0
1. If x 2 − 2x + sin2 θ = 0, then x belongs to
(b) bcx 2 + (b 2 + ac ) x + ab = 0
(a) [−1, 1] (b) [0, 2 ] (c) [−2, 2 ] (d) [1, 2 ]
(c) abx 2 + (c 2 + ab ) x + ca = 0
2. If p and q are non-zero constants, the equation
(d) None of the above
x 2 + px + q = 0 has roots α and β, then the
equation qx 2 + px + 1 = 0 has roots 8. If the roots of the equation ax 2 + bx + c = 0 are of
1 1 k+1 k+ 2
(a) α and (b) and β the form and , then ( a + b + c)2 is equal
β α k k+1
1 1 to
(c) and (d) None of these
α β (a) b 2 − 4ac (b) b 2 − 2 ac
3. If one root is n times the other root of quadratic (c) 2 b 2 − ac (d) Σa2
equation ax 2 − bx + c = 0, then
(a) na2 = bc (n + 1)2 (b) nb 2 = ca (n + 1)2 9. If the roots of the equation, x 2 + 2ax + b = 0 are
real and distinct and they differ by at most 2m,
(c) nc 2 = ab (n + 1)2 (d) None of these
then b lies in the interval
4. The value of k for which one of the roots of (a) (a2 − m2 , a2 ) (b) [a2 − m2 , a2 )
x − x + 3k = 0 is double of one of the roots of
2
(c) (a , a + m )
2 2 2
(d) None of these
x 2 − x + k = 0 is
10. If a , b and c ∈ R and a + b + c = 0, then the
(a) 1 (b) −2 (c) 2 (d) None of these
quadratic equation 4ax 2 + 3bx + 2c = 0 has
5. If the roots of ax + bx + c = 0 are in the ratio
2
(a) one positive and one negative root
m : n , then (b) imaginary roots
(a) mna2 = (m + n) c 2 (b) mnb 2 = (m + n) ac (c) real roots
(c) mnb = (m + n) ac
2 2
(d) None of these (d) None of the above
11. If one of the roots of the equation
6. If 22x − 2x − 1 = 14, then x x is equal to a ( b − c) x 2 + b ( c − a ) x + c ( a − b) = 0 is 1, then
(a) 1 (b) 2 (c) 256 (d) 4 what is the second root?
7. If α and β are the roots of the equation b (c − a) b (c − a)
(a) − (b)
ax + bx + c = 0, then the equation whose roots
2 a( b − c ) a( b − c )
c (a − b) c (a − b)
1 1 1 (c) (d) −
are , + is equal to a( b − c ) a( b − c )
α +β α β
72 NDA/NA Pathfinder

12. The coefficient of x in the equation x 2 + px + q = 0 22. If the roots of ax 2 + bx + c = 0 are sin α and cos α
was taken as 17 in place of 13 and its roots were for some α, then which one of the following is
found to be −2 and −15. The roots of the original correct?
equation are (a) a2 + b 2 = 2 ac (b) b 2 − c 2 = 2 ab
(a) −2, 15 (b) −10,−3 (c) 2,15 (d) 10,3 (c) b − a = 2 ac
2 2
(d) b 2 + c 2 = 2 ab
13. Let α , β be the roots of the equation 23. If ( x + a ) is a factor of both the quadratic
x 2 − 3x + p = 0 and let γ , δ be the roots of the polynomials x 2 + px + q and x 2 + lx + m, where
equation x 2 − 12x + q = 0. If the numbers α , β , γ , δ p, q, l and m are constants, then which one of
(in order) form an increasing GP, then the following is correct?
(a) p = 2, q = 16 (b) p = 2, q = 32 (a) a = (m − q ) / (l − p) (l ≠ p)
(c) p = 4, q = 16 (d) p = 4, q = 32 (b) a = (m + q ) / (l + p) (l ≠ − p)
14. If α and β be the roots of the equation (c) l = (m − q ) / (a − p) (a ≠ p)
(d) p = (m − q ) / (a − l ) (a ≠ l )
( x − a ) ( x − b) = c, c ≠ 0. Then, the roots of the
equation ( x − α )( x − β ) + c = 0 are 24. Let α and γ be the roots of Ax 2 − 4x + 1 = 0 and β
(a) a, c (b) b, c (c) a, b (d) a + b, a + c and δ be the roots of Bx 2 − 6x + 1 = 0. If α , β , γ
15. If one root of the equation ax + bx + c = 0, a ≠ 0
2
and δ are in HP, then what are the values of A
is reciprocal of the other root, then which one of and B respectively?
the following is correct? (a) 3, 8 (b) −3, − 8 (c) 3, − 8 (d) −3, 8
(a) a = c (b) b = c (c) a = − c (d) b = 0 1 1 1
25. If the roots of the equation + = are
16. If p, q, r are rational numbers, then the roots of x+ p x+q r
the equation x 2 − 2 px + p2 − q 2 + 2qr − r 2 = 0 are equal in magnitude but opposite in sign, then
p + q is equal to
(a) complex (b) pure imaginary 2
(c) irrational (d) rational (a) r (b) 2r (c) r 2 (d)
r
17. If the roots of the equation 26. If a , b ∈ R, then the equation x 2 − abx − a 2 = 0 has
( a 2 + b2 ) x 2 − 2b( a + c) x + ( b2 + c2 ) = 0 are equal,
(a) one positive and one negative root
then which one of the following is correct? (b) both positive roots
(a) 2b = a + c (b) b 2 = ac (c) both negative roots
(c) b + c = 2 a (d) b = ac (d) non-real roots
18. Consider the equation ( x − p) ( x − 6) + 1 = 0 27. The number of real solutions of
having integral coefficients. If the equation has 1 +|ex − 1| = ex ( ex − 2) is
integral roots, then what values can p have? (a) 0 (b) 1 (c) 2 (d) 4
(a) 4 or 8 (b) 5 or 10
28. The product of the roots of the equation
(c) 6 or 12 (d) 3 or 6
( x − 2)2 − 3|x − 2| + 2 = 0 is
19. If α and β be the roots of the equation (a) 2 (b) − 4 (c) 0 (d) None of these
x 2 + x + 1 = 0. Then, the equation whose roots are
α19 and β7 is x + 34x − 71
2
29. If x is real, then the values of does
x 2 + 2x − 7
(a) x − x − 1 = 0
2
(b) x 2
− x + 1= 0
not lie in the interval
(c) x 2 + x − 1 = 0 (d) x 2 + x + 1 = 0
(a) [5, 9] (b) (− ∞, 5] (c) [9, ∞) (d) (5, 9)
20. The equation tan4 x − 2 sec2 x + a 2 = 0 will have
30. If the equations x + 2x + 3λ = 0 2
and
atleast one real solution, if
2x 2 + 3x + 5λ = 0 have a non-zero common root,
(a)| a| ≤ 4 (b)|a| ≤ 2
(c)| a| ≤ 3 (d) None of these then λ is equal to
(a) 1 (b) −1 (c) 3 (d) None of these
21. If α , β are the roots of the quadratic equation
x 2 − x + 1 = 0, then which one of the following is
31. If the equations ax 2 + bx + c = 0 and x 2 + x + 1 = 0
have common root, then
correct?
(a) a + b + c = 0 (b) a = b = c
(a) (α − β ) is real
4 4
(b) 2(α + β ) = (αβ )
5 5 5
(c) a = b or b = c or c = a (d) None of these
(c) (α 6 − β 6 ) = 0 (d) (α 8 + β 8 ) = (αβ )8
MATHEMATICS Quadratic Equations and Inequalities 73

32. If the quadratic equations ax 2 + 2cx + b = 0 and 42. If tan A and tan B are the roots of the quadratic
ax + 2bx + c = 0 ( b ≠ c) have a common root, then
2
equation x 2 − px + q = 0, then consider the
a + 4b + 4c is equal to following statements.
(a) −2 (b) −1 (c) 0 (d) 1 I. The value of tan ( A + B) is
p
.
33. The range of values of m for which the equation 1+ q
( m − 5) x 2 + 2( m − 10) x + m + 10 = 0 has real roots II. The value of (cot A + cot B) is .
p
of the same sign is given by q
(a) m > 10 (b) −5 < m < 5 III. Discriminant of the equation is (tan A − tan B)2.
(c) m < − 10, 5 < m ≤ 6 (d) None of these Which of the above statement(s) is/are correct?
34. The set of real values of x satisfying the (a) I and II (b) II and III (c) I and III (d) I, II and III
inequality|x + x − 6|< 6 is
2
43. If the equation x − px − q = 0 has roots u and v,
2

(a) (– 4, 3) (b) (– 3, 2) where p and q are non-zero constants, then


(c) (– 4, – 3) ∪ (2, 3) (d) (– 4, –1) ∪ (0, 3) consider the following statements
1 1
35. The value of ‘a’ for which the equation I. qx2 + px − 1 = 0 has roots and .
x − 2( a − 1) x + ( 2a + 1) = 0 has both roots
2 u v
positive is II. (x − p)(x + q) = 0 has roots u + v and uv.
(a) a > 0 (b) 0 < a < 4 (c) a ≥ 4 (d) None of these III. x2 + p2x + q2 = 0 has roots u 2 and v2.
36. If the roots of the equation x 2 − 4x − log3 N = 0 Which of the above statement(s) is/are correct?
are real, then what is the minimum value of N ? (a) I and II (b) II and III (c) I and III (d) I, II and III
1 1 1 1
(a) (b) (c) (d)
256 27 64 81 Directions (Q. Nos. 44-45) α and β are the roots of
quadratic equation x 2 + x α + β = 0. Considering
37. If a , b and c are positive and are in AP, the roots of
this statement answer the following questions.
the quadratic equation ax 2 + bx + c = 0 are real for
c a 44. The value of α and β is
(a)  − 7 > 4 3 (b)  − 7 < 4 3
a  c  (a) α = 1, β = − 1 (b) α = 1, β = − 2
(c) all a and c (d) no a and c (c) α = 2, β = 1 (d) α = 2, β = − 2

38. If the roots of equation x 2 − px + ( p − 1) = 0 are r1 45. The quadratic equation having roots α + 1 and
and r2, then minimum value of (r12 + r22 ) will be β + 1 is
(a) x 2 − x + 2 = 0 (b) x 2 − x − 2 = 0
for
(a) p = 0 (b) p = − 1 (c) p = 1 (d) p = 2 (c) x 2 + x + 2 = 0 (d) x 2 + x − 2 = 0

39. If α, β are roots of the equation 2x 2 + 6x + b = 0 Directions (Q. Nos. 46-47) Let the quadratic
α β equation ( ab − bc ) x 2 + ( bc − ca) x + (ca − ab) = 0,
( b < 0), then + is greater than
β α a, b, c ∈ R has both the roots equal.
(a) 0 (b) 1 (c) 2 (d) None of these 46. a, c, b are in
40. Let α, β be the roots of a( p + x )2 + 2bpx + c = 0, (a) AP (b) GP (c) HP (d) None of these
then consider the following statements
c 2 47. The roots of the equation are equal to
I. αβ = p2 + II. |α − β| = p2 − 4ac (a) − 1 (b) 1 (c) 2 (d)
1
a |a|
2
Which of the above statement(s) is/are correct?
(a) Only I (b) Only II Directions (Q. Nos. 48-49) Let α, β be the roots of
(c) Both I and II (d) Neither I nor II the equation ax 2 + bx + c = 0 such that α > β.
41. α, β are the roots of the equation x 2 + bx + c = 0 48. If both roots are positive, then
and S n = α n + β n . (a) a > 0 and b > 0 (b) a > 0 and b < 0
Consider the following statements (c) a > 0 and c < 0 (d) a < 0 and c > 0
I. S n + 1 + bS n + cS n − 1 = 0 II. S3 = S13 − 3S1c
Which of the above statement(s) is/are correct?
49. If α is positive and β is negative such that α >|β|,
(a) Only I (b) Only II then consider the following statements
I. c and a have opposite sign.
(c) Both I and II (d) Neither I nor II
II. b and a have same sign.
74 NDA/NA Pathfinder

Which of the above statement(s) is/are correct? 59. The value of tan−1 α + tan−1 β is
(a) Only I (b) Only II
 p   p 
(c) Both I and II (d) Neither I nor II (a) tan−1   (b) tan−1  
 q − 1  q + 1
Directions (Q. Nos. 50-52) Let a, b and c be real  p 
numbers with a ≠ 0 and let α, β be the roots of the (c) tan−1   (d) 0
1 − q 
equation ax 2 + bx + c = 0.
60. If α = p and β = q and p, q ≠ 0 then the values of
50. The roots of the equation a3 x 2 + abcx + c3 = 0 is p and q are respectively
(a) α 2β, β 2α (b) α, β 2 (a) 2 and − 2 (b) 2 and − 1 (c) 1 and − 2 (d) 1 and 2
(c) α 2β, βα (d) α 3β, β 3α

51. If α = sin θ, β = cos θ, then PREVIOUS YEARS’ QUESTIONS


(a) (a − c )2 = b 2 − c 2 (b) (a − c )2 = b 2 + c 2
61. What is the sum of the squares of the roots of
(c) (a + c )2 = b 2 − c 2 (d) (a + c )2 = b 2 + c 2 the equation x 2 + 2x − 143 = 0? e 2012 I
1 1
52. The equation, whose roots are and is (a) 170 (b) 180 (c) 190 (d) 290
aα + b aβ + b
(a) cax 2 − bx + 1 = 0 (b) cax 2 + bx + 1 = 0
62. If one of the roots of the equation x 2 + ax − b = 0
is 1, then what is the value of ( a − b)? e 2012 I
(c) cax 2 + bx − 1 = 0 (d) None of these (a) −1 (b) 1 (c) 2 (d) −2

Directions (Q. Nos. 53-56) Let α and β are the roots 63. If α and β are the roots of the equation
of the equation x + x + 1 = 0.
2
x 2 − q (1 + x ) − r = 0, then what is the value of
53. The roots of the equation x 2 − x + 1 = 0 is (1 + α )(1 + β )? e 2012 I
(a) 1− r (b) q − r (c) 1+ r (d) q + r
(a) α 7 and β13 (b) α13 and β 7
64. If the difference between the roots of
(c) α 20 and β 20 (d) None of these
ax 2 + bx + c = 0 is 1, then which one of the
54. If the ratio of the roots of the equation following is correct? e 2012 I
α (a) b 2 = a (a + 4c ) (b) a2 = b (b + 4c )
ax 2 + bx + c = 0 is equal to , then a, b, c are in
β (c) a2 = c (a + 4c ) (d) b 2 = a (b + 4c )
(a) AP (b) GP
(c) HP (d) None of these Directions (Q. Nos. 65-66) The equation formed by
multiplying each root of ax 2 + bx + c = 0 by 2 is
55. The quadratic equation whose roots are (α + 1)
x 2 + 36 x + 24 = 0. e 2012 I
and (β + 1) is
(a) x 2 + x + 1 = 0 (b) x 2 − x − 1 = 0 65. What is the value of b : c?
(a) 3 : 1 (b) 1 : 2 (c) 1 : 3 (d) 3 : 2
(c) x 2 − x + 1 = 0 (d) x 2 + x − 1 = 0
66. Which one of the following is correct?
56. The value of a and b for which the equation (a) bc = a2 (b) bc = 36 a2
( a + b)x 2 − ax + 1 = 0 has roots α 2 and β 2, are (c) bc = 72 a2 (d) bc = 108 a2
1
(a) − 1, 2 (b) − 2, 3 (c) , − 2 (d) None of these 67. If the roots of a quadratic equation are m + n
2
and m − n, then the quadratic equation will be
Directions (Q. Nos. 57-60) Let α, β be the roots of (a) x 2 + 2 mx + m2 − mn + n2 = 0 e 2012 II
the quadratic equation x 2 + px + q = 0. (b) x 2 + 2 mx + (m − n)2 = 0
57. If one root is square of the other root, then (c) x 2 − 2 mx + m2 − n2 = 0 (d) x 2 + 2 mx + m2 − n2 = 0
relation between p and q is 68. If the roots of the quadratic equation
(a) p3 − (3 p − 1)q + q 2 = 0 3x 2 − 5x + p = 0 are real and unequal, then which
(b) p3 − q (3 p + 1) + q 2 = 0 one of the following is correct? e 2012 II
(c) p3 + q (3 p − 1) + q 2 = 0 (a) p = 25 / 12 (b) p < 25 / 12 (c) p > 25 / 12 (d) p ≤ 25 / 12
(d) p3 + q (3 p + 1) + q 2 = 0 69. If α , β are the roots of x 2 + px − q = 0 and γ , δ are
58. If αβ = 3 and 1, p, q are in AP, then α + β is equal to the roots of x 2 − px + r = 0, then what is the
(a) − 4 (b) 1 value of (β + γ ) (β + δ )? e 2012 II

(c) 4 (d) − 2 (a) p + r (b) p + q (c) q + r (d) p − q


MATHEMATICS Quadratic Equations and Inequalities 75

70. What is the degree of the equation 81. If a and b are rational and b is not perfect
1 1 1 square, then the quadratic equation with
= − ? rational coefficients whose one root is 3a + b is
x−3 x+2 2 e 2013 I
(a) 0 (b) 1 (c) 2 (d) 3 e 2013 II
(a) x 2 − 6ax + 9a2 − b = 0 (b) 3ax 2 + x − b=0
71. If 4 − 6 ⋅ 2 + 8 = 0, then the values of x are
x x
(c) x 2 + 3ax + b=0 (d) bx 2 + x − 3a = 0
e 2013 I
(a) 1, 2 (b) 1, 1 (c) 1, 0 (d) 2, 2 82. If α and β are the roots of the equation
72. If the roots of the equation 3ax + 2bx + c = 0 are
2 α10 + β10
x 2 + x + 2 = 0, then what is equal to?
in the ratio 2 : 3, then which one of the following α −10 + β −10
is correct? e 2013 I e 2013 II
(a) 8 ac = 25 b (b) 8ac = 9b 2 (a) 4096 (b) 2048 (c) 1024 (d) 512
(c) 8b 2 = 9ac (d) 8b 2 = 25ac 83. The quadratic equation x 2 + bx + 4 = 0 will have
real roots, if e 2013 II
73. ( x + 1)2 − 1 = 0 has e 2013 I (a) Only b ≤ −4 (b) Only b ≥ 4
(a) one real root (b) two real roots (c) −4 < b < 4 (d) b ≤ −4, b ≥ 4
(c) two imaginary roots (d) four real roots
84. If α and β are the roots of the equation
74. If the roots of a quadratic equation
ax 2 + bx + c = 0, where a ≠ 0, then ( aα + b) ( aβ + b)
ax 2 + bx + c = 0 are α and β, then the quadratic
is equal to e 2014 I
equation having roots α 2 and β 2 is e 2013 I (a) ab (b) bc (c) ca (d) abc
(a) x 2 − (b 2 − 2 ac ) x + c = 0
85. The roots of the equation 2a x − 2abx + b2 = 0,
2 2
(b) a2 x 2 − (b 2 − 2 ac ) x + c = 0
when a < 0 and b > 0 are e 2014 I
(c) ax 2 − (b 2 − 2 ac ) x + c 2 = 0 (a) sometimes complex (b) always irrational
(d) a2 x 2 − (b 2 − 2 ac ) x + c 2 = 0 (c) always complex (d) always real

75. If the sum of the roots of a quadratic equation is 86. Every quadratic equation ax 2 + bx + c = 0, where
3 and the product is 2, then the equation is a , b, c ∈ R , a ≠ 0 has e 2014 II
e 2013 I (a) exactly one real root (b) atleast one real root
(a) 2 x2 − x + 3 = 0 (b) x 2 − 3x + 2 = 0 (c) atleast two real roots (d) atmost two real roots
(c) x 2 + 3x + 2 = 0 (d) x 2 − 3x − 2 = 0 87. If α , β are the roots of ax 2 + bx + c = 0 and α + h,
76. If α and β are the roots of the equation β + h are the roots of px 2 + qx + r = 0, then what
x 2 + bx + c = 0, then what is the value of α −1 + β −1? is h equal to? e 2014 II
1 b q 1  b q
e 2013 I (a)  −  (b) − + 
b b c c 2  a p 2  a p
(a) − (b) (c) (d) −
1 b q 1  b q
c c b b (c)  +  (d) − + 
2  p a 2  p a
77. The roots of the equation x 2 − 8x + 16 = 0
e 2013 II
88. Consider the following statements in respect of
(a) are imaginary (b) are distinct and real the given equation ( x 2 + 2)2 + 8 x 2 = 6x ( x 2 + 2).
(c) are equal and real (d) Cannot be determined I. All the roots of the equation are complex.
II. The sum of all the roots of the equation is 6.
78. How many real roots does the quadratic equation
f( x ) = x 2 + 3| x| + 2 = 0 have? e 2013 II
Which of the above statement(s) is/are correct?
e 2015 I
(a) One (b) Two (c) Four (d) No real root
(a) Only I (b) Only II
79. If α and β are the roots of the equation (c) Both I and II (d) Neither I nor II
ax 2 + bx + b = 0, then what is the value of 89. In solving a problem that reduces to a quadratic
α β b equation, one student makes a mistake in the
+ + =? constant term and obtains 8 and 2 for roots.
β α a e 2013 II Another student makes a mistake only in the
(a) −1 (b) 0 (c) 1 (d) 2 coefficient of first degree term and finds −9 and
80. What is the difference in the roots of the −1 for roots. The correct equation is e 2015 I
equation x 2 − 10x + 9 = 0? (a) x 2 − 10x + 9 = 0 (b) x 2 + 10x + 9 = 0
e 2013 II
(a) 2 (b) 3 (c) 5 (d) 8 (c) x − 10x + 16 = 0
2
(d) x 2 − 8x − 9 = 0
76 NDA/NA Pathfinder

Directions (Q. Nos. 95-96) Let α and β (α < β) be the


90. If m and n are roots of the equation
roots of the equation x 2 + bx + c = 0, where b > 0
( x + p)( x + q ) − k = 0, then roots of the equation
( x − m )( x − n ) + k = 0 are e 2015 I and c < 0. e 2016 I
(a) p and q 95. Consider the following
1 1 I. β < − α II. β <|α |
(b) and
p q
Which of the above statement(s) is/are correct?
(c) − p and −q (a) Only I (b) Only II
(d) p + q and p − q (c) Both I and II (d) Neither I nor II
91. If 2 p + 3q = 18 and 4 p2 + 4 pq − 3q 2 − 36 = 0, then 96. Consider the following
what is ( 2 p + q ) equal to? e 2015 I I. α + β + αβ > 0 II. α 2 β + β 2 α > 0
(a) 6 (b) 7
Which of the above statement(s) is/are correct?
(c) 10 (d) 20
(a) Only I (b) Only II
92. The number of real roots of the equation (c) Both I and II (d) Neither I nor II
x 2 − 3|x| + 2 = 0 is e 2015 II 97. If x 2 − px + 4 > 0 for all real values of x, then
(a) 4 (b) 3 which one of the following is correct? e 2016 I
(c) 2 (d) 1 (a)| p| < 4 (b)| p| ≤ 4
93. If the roots of the equation x 2 − nx + m = 0 differ (c)| p| > 4 (d)| p| ≥ 4
by 1, then e 2015 II 98. If one root of the equation ( l − m ) x 2 + lx + 1 = 0 is
(a) n2 − 4m − 1 = 0 double the other and l is real, then what is the
(b) n2 + 4m − 1 = 0 greatest value of m? e 2016 I
9 9 8 8
(c) m2 + 4n + 1 = 0 (a) − (b) (c) − (d)
8 8 9 9
(d) m2 − 4n − 1 = 0
Directions (Q. Nos. 99-100) Given that tan α and
94. If the sum of the roots of the equation tan β are the roots of the equation x 2+ bx + c = 0 with
ax + bx + c = 0 is equal to the sum of their
2
b ≠ 0. e 2016 I
squares, then e 2015 II
(a) a2 + b 2 = c 2
99. What is tan (α + β) equal to?
(a) b(c –1) (b) c(b –1) (c) c(b − 1)−1 (d) b(c –1) −1
(b) a2 + b 2 = a + b
(c) ab + b 2 = 2 ac 100. What is sin (α + β ) sec α sec β equal to?
(d) ab − b 2 = 2 ac (a) b (b) −b (c) c (d) −c

ANSWERS
1 b 2 c 3 b 4 b 5 c 6 d 7 b 8 a 9 b 10 c
11 c 12 b 13 b 14 c 15 a 16 d 17 b 18 a 19 d 20 c
21 c 22 c 23 a 24 a 25 b 26 a 27 b 28 c 29 d 30 b
31 b 32 c 33 c 34 d 35 c 36 d 37 a 38 c 39 d 40 a
41 c 42 b 43 a 44 b 45 b 46 c 47 b 48 b 49 a 50 a
51 d 52 a 53 d 54 b 55 c 56 a 57 a 58 d 59 a 60 c
61 d 62 a 63 a 64 a 65 a 66 d 67 c 68 b 69 c 70 c
71 a 72 d 73 b 74 d 75 b 76 a 77 c 78 d 79 b 80 d
81 a 82 c 83 d 84 c 85 c 86 d 87 a 88 b 89 a 90 c
91 c 92 a 93 a 94 c 95 c 96 b 97 a 98 b 99 d 100 b
MATHEMATICS Quadratic Equations and Inequalities 77

HINTS AND SOLUTIONS 


1. (b) Q x 2 − 2x + sin 2 θ = 0 ⇒ ( 2 y + 7)( y − 4) = 0 Given root is 1.
2 ± 4 − 4 sin 2 θ ⇒
7
y = − or y = 4 Let the other root be α.
∴ x = b( c − a )
2 2 Then, α + 1 = −
⇒ x = 1 ± cos θ 7
⇒ 2 = − or 2x = 4 ⇒ x = 2
x a ( b − c)
−1 ≤ cos θ ≤ 1 2 ( bc − ab )
Q ⇒ α = − 1−
∴ 0 ≤ 1 ± cos θ ≤ 2 ⇒ 0 ≤ x ≤ 2 ∴ x x = ( 2)2 = 4 ( ab − ac )
⇒ x ∈[ 0, 2] 1 α+β a b − ab + ac − bc + ab
7. (b) S = + =− − =
1 α+β αβ b c a ( b − c)
2. (c) Replacing x by in the first equation,
x ( ac + b 2 ) −c ( −a + b) c( a − b )
=− ⇒ α= ⇒α =
we get the second equation and hence, bc a ( b − c) a ( b − c)
1 1 1 α+β 1 a
its roots are and . P= ⋅ = = 12. (b) Let α , β be the roots of the original
α β α + β αβ αβ c
equation, then
3. (b) Let α ,nα be the roots of equation Put the values of P and S in α + β = −13 and αβ = (−2)(−15) = 30
ax 2 − bx + c = 0. x 2 − Sx + P = 0, we get
b b ∴ Original equation is
∴ α + nα = ⇒ α( 1 + n) = bcx 2 + ( b 2 + ac )x + ab = 0
a a x 2 + 13x + 30 = 0
b k+1 k+2 b
⇒ α= …(i) 8. (a) We have, + =− … (i) ⇒ x + 10x + 3x + 30 = 0
2
a( 1 + n) k k+1 a
c c k+1 k+2 c ⇒ ( x + 10)( x + 3) = 0
and α ⋅ nα = ⇒ α 2 n = and ⋅ =
a a k k+1 a ⇒ x = − 10, − 3
k+2 c 2 c c−a
 b 
2
or = ⇒ = − 1=
⇒ n=
c
[from Eq. (i)] k a k a a 13. (b) Here, β = αr , γ = αr 2 , δ = αr 3 , r > 1
 a( 1 + n)  a
∴ k=
2a α + β = 3, αβ = p , γ + δ = 12, γ δ = q
⇒ b 2 n = ac ( 1 + n)2 c−a α ( 1 + r) = 3 
Putting the value of k in Eq. (i),  ⇒ r2 = 4 ⇒ r = 2
4. (b) Let α be a root of x 2 − x + k = 0. 
we get αr 2 ( 1 + r ) = 12
Then, 2α is a root of c+ a 2c b 
+ =− ∴ α=1
x2 − x + 3 k = 0 2a c+ a a
∴ 4α − 2α + 3k = 0 and α 2 − α + k = 0
2
or ( c + a )2 + 4ac = − 2b ( a + c ) p = αβ = α 2 r = 2, q = γδ = α 2 r 5 = 32
α2 α 1 or ( a + c )2 + 2b ( a + c ) = − 4ac 14. (c) Given, quadratic equation is
⇒ = =
−2 k + 3 k 3 k − 4 k − 4 + 2 Add b 2 on both sides, ( x − a ) ( x − b ) = c, c ≠ 0
k k ⇒ x 2 − ( a + b ) x + ( ab − c ) = 0
⇒ α 2 = − and α = ( a + c + b )2 = b 2 − 4ac
2 2 The roots of this equation is (α , β ).
2 9. (b) | α − β | ≤ 2m.
α 2 = (α )2 ⇒  −  =  
k k Then,
Now, On squaring both sides, we get
 2  2 α + β = − { − ( a + b )} = a + b
⇒ k 2 + 2 k = 0 ⇒ k = 0 or −2 (α − β )2 < 4m 2 ⇒ (α + β )2 − 4αβ < 4m 2
and αβ = ab − c
Since, k ≠ 0, 4( a 2 − b ) ≤ 4m 2 ⇒ a 2 − m 2 ≤ b Now, consider the equation,
∴ k = −2 ∴ b ≥ a2 − m2 ( x − α) ( x − β) + c = 0
Also, ∆ > 0 ⇒ x 2 − (α + β ) x + (αβ + c ) = 0
−b + b 2 − 4ac
m
5. (c) Given, = ∴ a2 − b > 0 ⇒ b < a2
− b − b − 4ac 2
n ⇒ x 2 − ( a + b ) x + ( ab − c + c ) = 0
∴ b ∈[a2 − m2 , a2 ) ∴ x 2 − ( a + b ) x + ab = 0
Applying componendo and dividendo
rule, 10. (c) Let D be the discriminant of the So, the roots of this equation is ( a , b ).
−2 b m+ n given quadratic equation, then
= 15. (a) Roots are reciprocal to each other, if
2 b 2 − 4ac m − n D = 9b 2 − 32ac
product of roots is 1.
b2 (m + n)2 = 9( − a − c )2 − 32ac [Q a + b + c = 0] c
⇒ = ∴ = 1⇒a = c
b − 4ac (m − n)2
2 = 9a 2 + 9c 2 2− 14ac a
 a 
= c 2 9   − 14   + 9
a
⇒ b 2mn = ac (m + n)2   c 2  c 
16. (d) The given equation
 3a 7 32 
6. (d) We have, 2 − 2
2x
= 14 x −1 = c 2  −  +  > 0 x 2 − 2 px + p 2 − q 2 + 2qr − r 2 = 0
Let 2x = y , then  c 3  9
and p , q , r are rational numbers.
y Hence, roots are real. Now, D = B 2 − 4 AC
y 2 − = 14
2 11. (c) Given quadratic equation is D = 4 p 2 − 4 { p 2 − ( q − r )2 }
⇒ 2 y 2 − y − 28 = 0 a ( b − c )x 2 + b ( c − a ) x + c ( a − b ) = 0. D = 4 p 2 − 4 p 2 + 4( q − r )2
78 NDA/NA Pathfinder

D = 4 ( q − r )2 = Rational and positive ⇒ 2(α 5 + β 5 ) ≠ (αβ )5 Also, discriminant = a 2 b 2 + 4a 2 > 0


So, the roots of the equation will (c) α 6 − β 6 = ( − ω)6 − ( −ω2 )6 Thus, α , β ∈ R such that αβ < 0
always rational. = ω6 − ω12 = 0 Therefore, one of α and β is positive
17. (b) Hence, option (c) is correct. and other is negative.
18. (a) The given equation can be rewritten 22. (c) Since, sin α and cos α are roots of 27. (b) We have,
as ax 2 + bx + c = 0 1 + |e x − 1| = e x ( e x − 2)
x 2 − ( p + 6) x + ( 6 p + 1) = 0 −b ⇒ |e x − 1| = e 2 x − 2e x − 1
∴ sin α + cos α =
Now, b 2 − 4ac a = ( e x − 1)2 − 2
= ( p + 6)2 − 4( 6 p + 1) and sinα cos α = c / a ⇒ |e − 1| − |e x − 1| − 2 = 0
x 2

[Q equation has integral roots] For any real α, sin 2 α + cos 2 α = 1 ⇒ |e x − 1| = 2, − 1


= p 2 − 12 p + 32 ⇒ (sin α + cos α)2 − 2 sin α cos α = 1 ⇒ |e x − 1| = 2 [Q |e x − 1| > 0]
2
= ( p − 4) ( p − 8) ⇒  −b  − 2 × c = 1 ⇒ e x − 1 = ± 2 ⇒ e x = 3, − 1
 
For integral roots, b 2 − 4ac must be a  a  a ⇒ e x = 3 ⇒ x = log e 3
perfect square ⇒ b 2 − 2ac = a 2 ⇒ b 2 − a 2 = 2ac 28. (c) We have,
∴ Possible values of p are 4 or 8. 23. (a) Since, ( x + a ) is a factor of ( x − 2)2 − 3|x − 2| + 2 = 0
19. (d) The given quadratic equation is x 2 + px + q and x 2 + lx + m. ⇒ |x − 2|2 − 3|x − 2| + 2 = 0
x2 + x + 1 = 0 ∴ a 2 − ap + q = 0 ...(i) ⇒ (|x − 2| − 2)(|x − 2| − 1) = 0
−1 ± 1 − 4 −1 ± i 3 and a 2 − la + m = 0 ...(ii) ⇒ |x − 2| = 1, 2 ⇒ x − 2 = ± 1, ± 2
x = =
2 2 From Eqs. (i) and (ii), we get ⇒ x = 3, 1, 4, 0
−1 + i 3 −1 − i 3 − ap + q + la − m = 0 Hence, product of roots
⇒x= , or x = ω, ω2
2 2 ⇒ ( l − p) a = m − q =3 × 1× 4 × 0 = 0
m−q
i.e. α = ω and β = ω2 ∴ a = [Q l ≠ p ] x 2 + 34x − 71
Now, sum of roots l− p 29. (d) Let y = . Then,
x 2 + 2x − 7
= α19 + β 7 = ω′9 + (ω2 )7 = ω + ω2 = −1 24. (a) Since, α and γ are the roots of
[Q1 + ω + ω2 = 0] x 2 ( y − 1) + 2x ( y − 17) − ( 7 y − 71) = 0
Ax 2 − 4x + 1 = 0
and product of roots 4 1 Since, x is real. Therefore, the above
∴ α+ γ= and αγ = equation has real roots.
α19 ⋅ β 7 = (ω)19 (ω2 )7 = ω ⋅ ω2 = ω3 = 1 A A
[Q ω3 = 1] ∴ Discriminant ≥ 0.
and β and δ be the roots of
So, the required quadratic equation is, ⇒ 4( y − 17)2 + 4( y − 1)( 7 y − 71) ≥ 0
Bx 2 − 6x + 1 = 0
x 2 − ( − 1) x + ( 1) = 0 6 1 ⇒ y 2 − 14 y + 45 ≥ 0
∴ β+δ= and βδ =
⇒ x2 + x + 1 = 0 B B ⇒ ( y − 5)( y − 9) ≥ 0
Also, α , β , γ and δ are in HP. ⇒ y ≤ 5 or y ≥ 9 ⇒ y ∈ R − (5, 9)
20. (c) Given equation, 1 1 1 1
tan 4 x − 2 sec 2 x + a 2 = 0 ∴ , , and are in AP. Hence, y does not lie in (5,9).
α β γ δ
⇒ tan x − 2 − 2 tan 2 x + a 2 = 0
4 30. (b) Let α be the common root of the
1 1 1 1 1 1 1 1
⇒ − = − ⇒ − = − given equation, then
⇒ tan 4 x − 2 tan 2 x + a 2 = 0 β α δ γ β δ α γ
α 2 + 2α + 3λ = 0
2 ± 4 − 4( a 2 − 2) δ −β γ −α
⇒ tan 2 x = ⇒ = 2α 2 + 3α + 5λ = 0
2 βδ αγ
Now, by cross-multiplication method
2 ± 2 1 − a2 + 2 (δ + β )2 − 4βδ ( γ + α )2 − 4αγ
⇒ tan x =
2
⇒ = α2 α 1
2 = =
βδ αγ 10λ − 9λ 6λ − 5λ 3 − 4
= 1 ± 3 − a2 ⇒ 36 − 4B = 16 − 4 A α2 α
⇒ = =−1
For real values of tan 2 x, ⇒ 36 − 4B = 16 − 4 A λ λ
3 − a2 ≥ 0 ⇒ a2 − 3 ≤ 0 ⇒ 4B − 4 A = 20 ⇒ B − A = 5 ⇒ α 2 = − λ and α = − λ
⇒ | a |≤ 3 which is possible only, if A = 3, B = 8. ⇒ ( − λ )2 = − λ ⇒ λ2 + λ = 0
21. (c) Since, α and β are the roots of the 1 1 1
25. (b) Given, + = ⇒ λ( λ + 1) = 0 ⇒ λ = − 1 [Q λ ≠ 0]
equation x 2 − x + 1 = 0. x+ p x+q r
31. (b) Let x be the common root of the
∴ α + β = 1 and αβ = 1 ⇒ r[ 2x + p + q ] = x 2 + ( p + q )x + pq given equations, then
α = − ω and β = − ω2 ⇒ x 2 + ( p + q − 2r )x + pq − r( p + q ) = 0 aα 2 + bα + c = 0 , α 2 + α + 1 = 0
Now, (a) α 4 − β 4 = ( − ω)4 − ( − ω2 )4 As, if roots are equal in magnitude but α2 α 1
opposite in sign, then coefficient of x ∴ = =
= ω4 − ω8 = ω − ω2 = 3i b−c c−a a−b
will be zero. b−c c−a
⇒ α − β 4 is not real.
4
⇒ α2 = and α =
∴ p + q − 2r = 0 ⇒ p + q = 2r a−b a−b
(b) 2(α 5 + β 5 ) = 2[( − ω)5 + ( − ω2 )5 ]
26. (a) Let α , β be the roots of the given c−a
2
b−c
= 2[ − ω2 − ω] = 2 ⇒   =
equation.  a − b
and (αβ ) = [( − ω)( − ω2 )]5 = 1
5 a−b
Then, α + β = ab and αβ = − a 2
MATHEMATICS Quadratic Equations and Inequalities 79

⇒ ( c − a )2 = ( a − b ) ( b − c ) 37. (a) 2b = a + c , b 2 − 4ac ≥ 0 Thus, Statement I is correct.


2
⇒ a 2 + b 2 + c 2 − ab − bc − ca = 0
⇒  a + c  − 4ac ≥ 0 Now, Sn = α n + β n

1
[( a − b )2 + ( b − c )2 + ( c − a )2 ] = 0  2  ∴ S =α + β = − b …(i)
1
2 ⇒ a 2 − 14ac + c 2 ≥ 0 S = α3 + β3
Since, each is zero. c 3
⇒ t − 14t + 1 ≥ 0, where t =
2
= (α + β )3 − 3αβ(α + β )
∴ a=b=c a
= ( S )3 − 3S c
⇒ ( t − 7)2 ≥ 48 ⇒ | t − 7 | ≥ 4 3 1 1
32. (c) Since, the coefficient of x 2 in both the Hence, Statement II is also correct.
equations are equal, therefore common ⇒  c − 7 ≥ 4 3
root is obtained by subtracting them a  42. (b) We have, tan A + tan B = p

∴ 2x ( c − b ) + ( b − c ) = 0 ⇒ x = −
1 38. (c) Since, the roots of equation and tan A ⋅ tan B = q
2 x 2 − px + ( p − 1) = 0 are r and r . tan A + tan B p
1 2 ∴ tan ( A + B ) = =
On putting for x in any equation, we ∴ r + r = p and r r = p − 1 1 − tan A ⋅ tan B 1 − q
1 2 1 2
get ∴ Statement I is not correct.
a + 4b + 4c = 0 ∴ r 2 + r 2 = ( r + r )2 − 2r r 1 1
1 2 1 2 1 2
Now, cot A + cot B = +
33. (c) ∆ > 0 ⇒ −25m + 150 ≥ 0 = p2 − 2 p + 2 = p2 − 2 p + 1 + 1 tan A tan B
∴ m≤ 6 = ( p − 1)2 + 1 tan B + tan A p
m + 10 = =
Product of roots = = positive as So, value of r 2 + r 2 will be minimum, if tan A ⋅ tan B q
1 2
m −5 p − 1 = 0, i.e. p = 1
roots are of same sign. ∴ Statement II is correct.
b Now, discriminant = p 2 − 4q
Numerator and denominator both 39. (d) We have, α + β = − 3and αβ =
2 = (tan A + tan B )2 − 4 tan A ⋅ tan B
positive m > 5 [Q m ≠ 5]
Since, b < 0, therefore discriminant,
or both negative m ≤ − 10 D = 36 − 8b > 0 = (tan A − tan B )2
∴ m < − 10 and 5 < m ≤ 6 So, α , β are real. Thus, Statement III is also correct.
34. (d) |x 2 + x − 6|< 6 Now, 43. (a) Given, x 2 − px − q = 0,
⇒ −6 < x 2 + x − 6 < 6 α β α 2 + β 2 (α + β ) 2 1
+ = = −2 On replacing x by , we get
⇒ − 6 < x 2 + x − 6 and x 2 + x − 6 < 6 β α αβ αβ x
⇒ x 2 + x > 0 and x 2 + x − 12 < 6 =
18
− 2< 0 [Q b < 0] qx 2 + px − 1 = 0
⇒ x ( x + 1) > 0 and ( x + 4)( x − 3) < 0 b Thus, Statement I is correct.
⇒ x ∈ ( −∞ ,−1) ∪ ( 0, ∞ ) 40. (a) We have, a( p + x )2 + 2bpx + c = 0 Now, u + v + uv = p − q
and − 4 < x < 3 ⇒ ax 2 + 2( a + b ) px + ap 2 + c = 0 (u + v )uv = − pq
⇒ x ∈ ( − 4, − 1) ∪ ( 0, 3) −2( a + b ) p ∴ The equation whose roots are (u + v)
∴ α+β=
a and (uv ) is x 2 − ( p − q )x − pq = 0
35. (c) The given equation has positive real
ap 2 + c c ⇒ ( x − p )( x + q ) = 0
roots, if (i) Discriminant ≥ 0 and αβ = = p2 +
a a Thus, Statement II is correct.
(ii) Product of roots > 0 Thus, Statement I is correct. Now,
Now, (i) Discriminant ≥ 0 Now, (α − β )2 = (α + β )2 − 4αβ u 2 + v 2 = (u + v )2 − 2uv = p 2 + 2q
⇒ 4( a − 1)2 − 4( 2a + 1) ≥ 0 4( a + b )2 p 2  ap 2 + c 
= − 4 and u 2 v 2 = (uv )2 = q 2
⇒ a 2 − 4a ≥ 0 ⇒ a < 0 or a ≥ 4 
a 2
 a  ∴ The equation whose roots are u 2 and
(ii) Product of roots > 0 4
= 2 [( a + b )2 p 2 − a 2 p 2 − ac ] v 2 is x 2 − ( p 2 + 2q )x + q 2 = 0.
⇒ ( 2a + 1) > 0 a Thus, Statement III is not correct.
⇒ a > −1 / 2 4
= 2 [( 2ab + b 2 ) p 2 − ac ] 44. (b) Since, α , β are the roots of
∴ a≥ 4 a
2 x 2 + x α + β = 0, then α + β = − α
36. (d) Given equation is, ⇒ |α − β| = ( 2ab + b 2 ) p 2 − ac
|a| and αβ = β ⇒ α = 1, β = − 2
x 2 − 4x − log N = 0
3
∴ Statement II is not correct. 45. (b) Now, α + 1 = 1 + 1 = 2,
Then, ∆≥0
β + 1= −2 + 1= −1
⇒ B 2 − 4 AC ≥ 0 41. (c) Since, α, β are roots of equation
Their sum = 2 + ( −1) = 1
⇒ ( − 4)2 − 4 ( 1) ( − log N ) ≥ 0 x 2 + bx + c = 0
3 Their product = 2( −1) = − 2
⇒ 16 + 4 log N ≥ 0 ∴α 2 + bα + c = 0 and β 2 + bβ + c = 0
3 ∴ Required equation
⇒ log (3)16 + log ( N )4 ≥ 0 Also, α + β = − b, αβ = c
3 3 x 2 − ( 1) x + ( −2) = 0
⇒ log {(3)16 ⋅ ( N )4 } ≥ 0 Now, S + bSn + cS
3 n+1 n −1 ⇒ x2 − x − 2 = 0
⇒ 316 ⋅ N 4 ≥ 30 = 1
= (α n + 1 + β n + 1 ) + b(α n + β n ) 46. (c) The given equation has equal roots
1 1 1
⇒ N 4 ≥ 16 ⇒ N ≥ 4 = + c (α n − 1 + β n − 1 )
3 3 81 n −1
∴ Discriminant = 0
=α [α + bα + c ]
2
1 ⇒ ( bc − ca )2 − 4( ab − bc )( ca − ab ) = 0
So, the minimum value of N is . + β n − 1 [β 2 + bβ + c ]
81 n −1
⇒ b 2 c 2 + c 2 a 2 − 2c 2 ab − 4 [ a 2 bc − a 2 b 2
=α ⋅ 0 + βn − 1 ⋅ 0
− c 2 ab + b 2 ac ] = 0
80 NDA/NA Pathfinder

⇒ b 2 c 2 + c 2 a 2 + 4a 2 b 2 + 2c 2 ab  1 1  57. (a) Let β = α 2


x2 −  +  x
− 4a 2 bc − 4b 2 ac = 0  aα + b aβ + b ∴ α + α 2 = − p and α 3 = q
⇒ ( bc + ca − 2ab )2 = 0 1 1 ⇒ α(α + 1) = − p
+ ⋅ =0
⇒ bc + ca = 2ab ⇒ c ( a + b ) = 2ab aα + b aβ + b On cubing both sides, we get
⇒ c=
2b ⇒ { a 2αβ + ab(α + β ) + b 2 } x 2 α 3 [α 3 + 1 + 3α(α + 1)] = − p 3
a+ b ⇒ q( q + 1 − 3 p ) = − p 3
− { a(α + β ) + 2b } x + 1 = 0
∴ a, c, b are in HP. ⇒ p − (3 p − 1)q + q 2 = 0
3

−( bc − ca ) ⇒ ( ca − b 2 + b 2 )x 2 − ( 2b − b )x + 1 = 0
47. (b) The root of the equation = ⇒ cax 2 − bx + 1 = 0 58. (d) Here, α + β = − p and αβ = q
2( ab − bc )
But αβ = 3 ⇒ q = 3
−( bc − 2ab + bc ) 2( ab − bc ) Sol. (Q. Nos. 53-56) Since,α and β are the roots
= = =1 1+ q 1+ 3
2( ab − bc ) 2( ab − bc ) of the equation x 2 + x + 1 = 0 Also, p = ⇒ p= =2
2 2
48. (b) α, β are roots of equation ∴α + β = −1 and αβ = 1 ∴ α+β=−2
ax 2 + bx + c = 0 −1 + 3i −1 − 3i
Also, α = and β = 59. (a) Here, α + β = − p and αβ = q
b c
∴ α+β=− and αβ = 2 2  αβ 
a a
⇒ α = ω and β = ω2 ∴ tan −1 α + tan −1 β = tan −1  
Since, both roots are positive, i.e.  1 − αβ 
 
α > 0, β > 0 53. (d) Let a and b are the roots of equation
b  − p   
∴ α + β> 0 ⇒− > 0 x2 − x + 1 = 0 = tan −1   = tan −1  p 
a ∴ a + b = 1 and ab = 1  1 − q  q − 1
   
⇒ a > 0 and b < 0 or a < 0 and b > 0 None of the option (a), (b), (c) are
60. (c) We have, p + q = − p …(i)
49. (a) Given, α > 0 and β < 0 satisfied for the given equation.
c and pq = q …(ii)
⇒ αβ < 0 ⇒ < 0 Hence, option (d) is correct.
a From Eq. (ii), p = 1 [Q q ≠ 0]
54. (b) Let α′, β′ be the roots of From Eq. (i), q = − 2 p = − 2 [Q p = 1]
∴ c and a have opposite sign.
ax 2 + bx + c = 0 ∴ p = 1 and q = − 2
Thus, Statement I is correct.
α′ α ω
Also, α > |β| ⇒ − α < β < α ⇒ α + β > 0 ∴ = = ⇒ β ′ = α ′ω 61. (d)
β ′ β ω2
∴ b and a have opposite sign. b
Now, α ′ + β ′ = − and α ′β ′ =
c 62. (a) Since, one root of x 2 + ax − b = 0
Thus, Statement II is not correct. a a is 1.
50. (a) Dividing the equation b
⇒ α ′ + α ′ω = − and (α ′ )2 ω =
c ∴ 12 + a ⋅ 1 − b = 0
a 3 x 2 + abcx + c 3 = 0 by c 2 , we get a a ⇒ 1+ a − b = 0 ⇒a − b = −1
b c
2 ⇒ α ′ω2 = and (α ′ )2 ω = 63. (a) Given that, α and β be the roots of
a   + b   + c = 0
ax ax
a a
 c   c  the equation x 2 − q( 1 + x ) − r = 0
b c
ax ⇒ α ′ = ω and (α ′ )2 ω =
⇒ = α, β a a ⇒ x 2 − qx − ( q + r ) = 0
c 2
α+β=q
⇒  ω ω =
c c b c Then,
⇒ x = α, β ⇒ b 2 = ac
a  a and αβ = − ( q + r )
a a
⇒ x = α 2β , αβ 2 ⇒ a, b, c are in GP. Now, ( 1 + α ) ( 1 + β) = 1 + (α + β ) + αβ
Q c = αβ = product of roots  = 1 + q − ( q + r) = 1 − r
55. (c) Sum of roots = (α + 1) + (β + 1)
 a  64. (a) Let the roots of the equation
= (α + β ) + 2
Hence, α 2β and αβ 2 are the roots of ax 2 + bx + c = 0 are α and (α − 1) by
= − 1+ 2 = 1
the equation a 3 x 2 + abcx + c 3 = 0. given condition. Then,
Product of roots = (α + 1)(β + 1) b
b ⇒ α + (α − 1) = −
51. (d) sin θ + cos θ = − = αβ + (α + β ) + 1 a
a b a−b
c = 1− 1+ 1= 1 ⇒ 2α = 1 − ⇒ α=
sin θ cos θ = a 2a
a ∴ Required equation is x 2 − x + 1 = 0. c
Now, (sin θ + cos θ)2 = 1 + 2 sin θ cos θ and α (α − 1) =
56. (a) We have, α = ω and β = ω2 a
b2 2c a + 2c ⇒ α 2 = ω2 and β 2 = ω ( a − b)  a − b = c
⇒ = 1+ = ⇒ − 1
a2  
a a ∴ The equation whose roots are α 2 and 2a  2a  a
⇒ b 2 = a 2 + 2ac β 2 is x2 + x + 1 = 0 …(i) ⇒ − ( a 2 − b 2 ) = 4ac
⇒ b 2 + c 2 = a 2 + 2ac + c 2 = ( a + c )2 Given equation is ⇒ b 2 = a( a + 4c )
∴ b 2 + c 2 = ( a + c )2 ( a + b )x 2 − ax + 1 = 0 …(ii)
Sol. (Q. Nos. 65-66) Let α and β be the roots of
On comparing Eqs. (i) and (ii), we get
52. (a) α + β = −
b
, αβ =
c the equation ax 2 + bx + c = 0.
a = − 1 and a + b = 1
a a Then, α + β = − b/a
The required equation is ⇒ b = 1 − a = 1 − ( − 1) = 2
and α ⋅β = c/ a
MATHEMATICS Quadratic Equations and Inequalities 81

Also given that, the equation −2 b 80. (d) Given equation, x2 − 10x + 9 = 0
Now, sum of the roots = 2α + 3α =
x 2 + 36x + 24 = 0 is formed by 3a Let (α , β ) be the roots of the given
−2 b −2b
multiplying each root of ⇒ 5α= ⇒ α= ...(i) equation.
ax 2 + bx + c = 0 by 2. 3a 15a
c Then, α + β = 10
and product of the roots = 2α ⋅ 3α =
∴ 2α + 2β = − 36 ⇒ α + β = −18 3a and α ⋅β = 9
2
−b −2 b 
⇒ 
b c c
⇒ = − 18 ⇒ = 18 …(i) ⇒ 6α 2 =  = Now, we use the identity
a a 3a  15a  18a
(α − β )2 = (α + β )2 − 4αβ
and ( 2α )( 2β ) = 24 ⇒ αβ = 6 [from Eq. (i)]
= ( 10) − 4 ( 9) = ( 10)2 − 4 ( 9)
2
c 4b 2 c 4b 2 c
⇒ =6 ...(ii) ⇒ = ⇒ = ⇒ α −β = ± 8
a 225a 2 18a 25a 2 2a
∴ 8 b 2 = 25 ac ∴ |α − β | = 8
65. (a) Now, dividing Eq. (i) by Eq. (ii),
we get 73. (b) 81. (a) If one root of any quadratic equation
b 3
74. (d) α , β are the roots of ax + bx + c = 0
2 is in the form 3a + b , then other root of
= ⇒ b: c = 3 : 1
c 1 −b c this equation should be 3a − b .
∴ α+β = ,αβ =
a a ∴ Required equation is
66. (d) Now, multiplying Eqs. (i) and (ii), we
get We have, α 2 + β 2 = (α + β )2 − 2αβ x2 − (Sum of roots) ⋅ x
b c
× = 18 × 6
2 + (Product of roots) = 0
=  −  − 2 ⋅
b c
a a  a a ⇒ x2 − {(3a + b ) + (3 a − b }⋅ x
⇒ bc = 108 a 2
b 2 2c b 2 − 2ac + {(3a + b )(3a − b )} = 0
67. (c) = 2 − =
a a a2 ∴ x − 6 ax + 9 a − b = 0
2 2
2
68. (b) Since, the roots of the quadratic 2  c c2
and α ⋅ β = (αβ ) =   = 2
2 2
82. (c) Given that, (α , β ) are the roots of the
equation 3x 2 − 5x + p = 0 are real and  a a
unequal. equation x2 + x + 2 = 0, then
∴ Required quadratic equation whose α + β = − 1 and α ⋅ β = 2
∴ Discriminant > 0 ⇒ b 2 − 4 ac > 0 roots are α 2 and β 2 is Now, we have
⇒ ( −5)2 − 4(3)( p ) > 0 x 2 − ( α 2 + β 2 ) x + α 2β 2 = 0 α10 + β10
⇒ 25 − 12 p > 0 ⇒ 25 > 12 p = (αβ )10 = ( 2)10
( b 2 − 2ac ) c2 α −10 + β −10
25 ⇒ x2 − x+ 2 = 0
⇒ 12 p < 25 ⇒ p < a 2
a
12 = 1024
∴ a 2 x 2 − ( b 2 − 2ac )x + c 2 = 0
69. (c) Since, α and β are the roots of 83. (d) Given that, the equation
75. (b) 76. (a)
x 2 + px − q = 0 x2 + bx + 4 = 0 have real roots, if
∴ α + β = − p, αβ = − q 77. (c) Given equation is discriminate (D) = B 2 − 4 AC ≥ 0
Again, since γ,δ are the roots of x2 − 8x + 16 = 0
x 2 − px + r = 0 ⇒ ( x − 4)2 = 0 ⇒ x = 4, 4 + –
∴ γ + δ = p, γδ = r So, the roots of the equation are equal
–∞ –4 +4 +∞
(β + γ )(β + δ) and real.
78. (d) Given quadratic equation, ⇒ b − 4 ( 1) ( 4) ≥ 0
2
= β 2 + β( γ + δ) + γδ
= β 2 + β ( p ) + γδ x 2 + 3 |x | + 2 = 0 ⇒ b 2 − 16 ≥ 0
= β + β ( −α − β ) + r
2 ⇒ |x|2 + 3|x|+2 = 0 ⇒ ( b − 4) ( b + 4) ≥ 0 ∴ b ≤ − 4, b ≥ 4
= β 2 − αβ − β 2 + r = − αβ + r ⇒ (|x|+2)(|x|+1) = 0 84. (c)
= − ( − q) + r = q + r ⇒ |x|= −2,−1
85. (c) Given equation,
Hence, given quadratic equation has no
70. (b)
real roots. 2a 2 x 2 − 2abx + b 2 = 0
71. (a) Given that, When, a < 0 and b > 0
79. (b) Given quadratic equation is
4x − 6 ⋅ 2x + 8 = 0 ⇒ 22 x − 6 ⋅ 2x + 8 = 0 −( −2ab ) ± ( −2ab )2 − 4 ⋅ 2a 2 ⋅ b 2
ax2 + bx + b = 0 ∴ x =
Let 2x = z ⇒ z 2 − 6z + 8 = 0 2 ⋅ 2a 2
Let (α , β ) be the roots of given
⇒ z 2 − 4z − 2z + 8 = 0 equation. 2ab ± 4a b − 8a 2 b 2
2 2
=
⇒ z (z − 4 ) − 2 (z − 4 ) = 0 b b 4a 2
∴ α+β=− and α β =
⇒ (z − 4)(z − 2) = 0 a a 2ab ± −4a 2 b 2 2ab ± i 2ab
= =
∴ z = 2, 4 ⇒ 2x = 21 , 22 Now, we have 4a 2 4a 2
So, the required values of x are 1, 2. α β b α+β b which shows that the roots of the given
+ + = +
72. (d) Given quadratic equation is β α a αβ a equation is always complex.
3ax 2 + 2bx + c = 0 −b a b b b 86. (d)
= × + =− + =0
Let its root are 2α and 3α. a b a a a
82 NDA/NA Pathfinder

b c 288 ∴ Statement I is not correct.


87. (a) Q α + β = − and αβ = ⇒ 324 − 36 = 4 q( 18) ⇒ = 4q
a a 18 Now, α 2β + β 2α = αβ (α + β )
q
Also, α + h + β + h = − ⇒ q=4 As, α + β < 0 and αβ < 0
p
⇒ α + β + 2h = −
q ∴ 2 p + 3 × 4 = 18 ⇒ αβ (α + β ) > 0
p 18 − 12 ∴ Statement II is correct.
⇒ p= =3
b 
Qα + β = − 
q b 2
⇒ 2h = − + 97. (a) Q x 2 − px + 4 > 0
p a  a  ∴ 2 p + q = 2 × 3 + 4 = 10
Here, a > 0 and f ( x ) > 0
1 b q  92. (a) We have, x 2 − 3| x | + 2 = 0 ∴ D<0
⇒ h=  − 
2 a p ∴ p 2 − 16 < 0 ⇒ p 2 < 16 ⇒ | p | < 4
⇒ | x |2 − 3| x | + 2 = 0
88. (b) We have, ⇒ | x |2 − 2| x | − | x | + 2 = 0 98. (b) Given equation is
( x 2 + 2)2 + 8x 2 = 6x ( x 2 + 2) …(i) ⇒ ( | x | − 2) ( | x | − 1) = 0 ( l − m ) x 2 + lx + 1 = 0
Let x 2 + 2 = y , then Let α and 2α be the roots of given
⇒ | x | = 2 or | x | = 1
y 2 + 8x 2 = 6xy equation. Then, we have
⇒ x = ± 2 or x = ± 1 −l
⇒ y − 6xy + 8x 2 = 0
2
α + 2α = 3α =
∴ There are four real roots of the l −m
⇒ ( y − 4x )( y − 2x ) = 0
equation.
⇒ ( x 2 + 2 − 4x )( x 2 + 2 − 2x ) = 0 and α ⋅ 2α = 2α 2 =
1
⇒ x 2 − 4x + 2 = 0 or x 2 − 2x + 2 = 0 93. (a) We have, x 2 − nx + m = 0 l −m
Since, discriminant of x 2 − 4x + 2 = 0 ∴ α + β = n, αβ = m −l 1
is 16 − 8 > 0 and given that, α–β = 1 ⇒ α= and 2α 2 =
3( l − m ) l −m
∴ It has real solutions. So, Statement I We know that, 2
is not correct. (α + β)2 − (α − β)2 = 4αβ  −l 
⇒ 2⋅   = 1
Now, Eq. (i) can be written as ⇒ n2 − 1 = 4m  3( l − m )  l −m
x 4 − 6x 3 + 12x 2 − 12x + 4 = 0  
⇒ n − 4m − 1 = 0
2
2
( −6 ) 2l 1
∴ Sum of all roots = − =6 94. (c) We have, ax 2 + bx + c = 0 ⇒ =
1 9 ( l − m )2 l − m
b
∴ Sum of roots, i.e. α + β = −
So, Statement II is correct. a ⇒ 2l 2 = 9 ( l − m ) [Q l ≠ m ]
c
89. (a) Let the correct equation be and product of roots, i.e. αβ = ⇒ 2l 2 − 9l + 9m = 0
ax 2 + bx + c = 0 a
9 ± 81 − 72m
It is given that, α + β = α 2 + β 2 ⇒ l =
First student roots are 8 and 2. Equation 4
is x 2 − 10x + 16 = 0 ⇒ α + β = (α + β)2 − 2αβ
Q l is real.
2
b  b
a = 1, b = − 10 are correct, c = 16 is =  −  − 2  ∴ D≥0
c
⇒ −
wrong. a  a  a ⇒ 81 − 72m ≥ 0
Second student roots are − 9 and −1 b b 2 2c b b 2 − 2ac 81 9
⇒ − = 2 − ⇒− = ⇒ 81 ≥ 72m ⇒ µ ≤ ⇒µ≤
Equation is x 2 + 10x + 9 = 0 a a a a a2 72 8
⇒ − ab = b − 2ac
2 9
a = 1, c = 9 are correct, b = 10 is wrong Hence, greatest value of m is .
⇒ b 2 = 2ac − ab ⇒ b 2 + ab = 2ac 8
∴ a = 1, b = − 10 and c = 9
Sol. (Q. Nos. 95-96) 99. (d) Given, x 2 + bx + c = 0, b ≠ 0
Hence, correct equation is tan α + tan β = − b
Given, α and β are the roots of
x 2 − 10x + 9 = 0 tan α tan β = c
equation x 2 + bx + c = 0. and
90. (c) Since, m and n are roots of the ∴ α + β = − b and αβ = c tan α + tan β
Now, tan (α + β ) =
equation As b > 0 and c < 0 1 − tan α tan β
( x + p )( x + q ) − k = 0 b
So, α + β< 0 =− = b( c − 1)−1
⇒ x 2 + ( p + q )x + pq − k = 0 ⇒ β< −α 1− c
∴ m + n = − ( p + q ) and mn = pq − k and αβ < 0 100. (b) Q tan α + tan β = −b
Now, ( x − m )( x − n) + k = 0 Also, given that α < β ⇒ α < 0 sin α sin β
⇒ + = −b
⇒ x 2 − (m + n)x + mn + k = 0 and β> 0 cos α cos β
⇒ x 2 + ( p + q )x + pq − k + k = 0 95. (c) α + β < 0 ⇒ β < − α sin α cos β + cos α sin β
...(i) ⇒ = −b
⇒ x 2 + ( p + q )x + pq = 0 ∴ Statement I is correct. cos α cos β
Hence, − p and − q are the required roots. sin(α + β )
As, α < 0 and − α > β ⇒ = −b
91. (c) We have, 2 p + 3 q = 18 ⇒ |α |> β cos α cos β
and 4 p 2 + 4 p q − 3q 2 − 36 = 0 ∴ Statement II is correct. ⇒ sin (α + β ) sec α sec β = − b
⇒ ( 2 p + 3 q ) − 8 p q − 12 q = 36
2 2
96. (b) As,α + β < 0 and αβ < 0
⇒ 182 − 4 q ( 2 p + 3 q ) = 36 ⇒ α + β + αβ < 0
07
83

PERMUTATIONS AND
COMBINATIONS
In NDA exam, generally 2-3 questions are asked from this chapter which are based on
fundamental principle of counting, number of permutation/combination under certain
conditions etc.

In our day-to-day life, we are interested to find the number of ways in which a particular work can be
done. For this, we have to find all the possible ways to do that work and it can be done with the help
of permutation and combination.

FACTORIAL NOTATION
The continued product of first n natural numbers is denoted by n! or it is read as ‘n factorial’.
Thus, n ! = n ( n − 1) ( n − 2)K 3 × 2 × 1 , e.g. 5 ! = 5 × 4 × 3 × 2 × 1 = 120 and 4! = 4 × 3 × 2 × 1 = 24

Properties of Factorial Notation


(i) 0 ! = 1
(ii) Factorials of negative integers and fractions are not defined.
n! n!
(iii) = n ( n − 1) ( n − 2) K(r + 1) (iv) = n ( n − 1) ( n − 2) K ( n − r + 1)
r! ( n − r )!
(v) ( m + n)! ≠ m!+ n ! (vi) (mn)! ≠ m! n!
(vii) n ! = n( n − 1)! = n( n − 1)( n − 2)!

FUNDAMENTAL PRINCIPLES OF COUNTING


There are following fundamental principles of counting
1. Fundamental principle of multiplication “If there are two jobs such that one of them can be
completed in mdifferent ways following which a second job can be completed in n different ways,
then the two jobs in succession can be completed in m × n ways. This can be extended to any finite
number of operations.
84 NDA/NA Pathfinder

EXAMPLE 1. How many 3-digit numbers, each less than


600, can be formed from (1, 2, 3, 4, 7, 9), if repetition of
Permutation Under
digits is allowed? Certain Condition
a. 216 b. 180 c. 144 d. 120 1. The number of all permutations of n different
Sol. c. Unit’s and ten’s place each can be filled by any one of objects taking r at a time
the given numbers, i.e. by 6 × 6 ways. (i) when a particular object is always included
But hundredth place can be filled by only 4 ways (7 and 9 is r ⋅ n − 1 Pr−1 .
cannot be put here).
∴Total required number of ways = 4 × 6 × 6 = 144 (ii) when a particular object is never included is
n−1
2. Fundamental principle of addition If there are mways Pr .
for doing one work and n ways doing another work, 2. Number of permutations of n different things,
then ways of doing either of the two works = m + n. taken r at a time, when s particular things are to
Here, the two works are independent of each other. be always included in each arrangement, is
EXAMPLE 2. In a class there are 10 boys and 8 girls. s ![r − ( s − 1)] ⋅ n − s Pr − s .
The teacher wants to select either a boy or a girl to 3. Number of permutations of n different things,
represent the class in a function. In how many ways can taken all at a time, when mspecified things
the teacher make this selection?
always come together, is m! × ( n − m + 1)!.
a. 80 b. 18 c. 81 d. None of these
4. Number of permutations of n different things,
Sol. b. Here, the teacher has to perform either of the following
taken all at a time, when mspecified things never
two jobs
(i) selecting a boy among 10 boys or
come together, is n ! − m! × ( n − m + 1)!.
(ii) selecting a girl among 8 girls. 5. The number of permutations of n things, of
The first job can be performed in 10 ways and the second which p1 are alike of one kind; p 2 are alike of
in 8 ways. Therefore, by the fundamental principle of second kind; p 3 are alike of third kind; ...; p r are
addition either of the two jobs can be performed in alike of rth kind such that p1 + p 2 + K + p r = n, is
(10 + 8) = 18 ways. Hence, the teacher can make the n!
selection of either a boy or a girl in 18 ways. .
p1 ! p 2 ! p 3 ! K p r !

PERMUTATIONS EXAMPLE 3. The number of arrangements that can


Each of the arrangements which can be made by taking be made with the letters of the word ‘MATHEMATICS’
some or all of a number of things is called a permutation. in which all vowels come together, is
Thus, in permutation the order of arrangement is 8! × 4! 8! × 4! 8! 8!
a. b. c. d.
important. When the order is changed, a different 2! 2! 2! 2! 2! 2! 2! 2! 4 ! 2! 2!
permutation is obtained, e.g. The permutations of three Sol. b. There are 4 vowels viz. A, E, A, I. Considering
letters A, B and C taking all at a time are ABC, ACB, these four vowels as one letter we have 8 letters
BCA, BAC, CBA and CAB. (M, T, H, M, T, C, S and one letter obtained by
combining all vowels), out of which M occurs twice,
T occurs twice and the rest all different. These 8
SOME IMPORTANT RESULTS ON PERMUTATION 8!
letters can be arranged in ways. But, the four
(i) Let r and n be positive integers such that 0 ≤ r ≤ n . Then, 2! × 2!
the number of all permutations of n distinct things taken r 4!
vowels (A, E, A, I) can be put together in ways.
at time is given by nPr or p (n, r), where 2!
n!
n
Pr = = n ( n − 1) ( n − 2) … ( n − r + 1) Hence, the total number of arrangements in which
( n − r )!
8! 4!
where n ≥ r . vowels are always together = ×
(ii) The number of all permutations of n distinct things, taken 2! × 2! 2!
all at a time is n Pn = h ! .
(iii) The number of permutations of n different things taken r at Circular Permutations
a time, when each thing may be repeated any number of
times is n r . If we arrange the objects along the circumference of a
(iv) P0 = 1, P1 = n and Pn = n ! ,
n n n n
Pn −1 = n ! circle, then permutations are known as circular
permutations.
SOME IMPORTANT RESULTS ON
MATHEMATICS Permutations and Combinations 85

CIRCULAR PERMUTATION
Properties of n Cr
(i) The number of circular permutations of n distinct
n
objects is ( n − 1) ! . Here, anti-clockwise and clockwise (i) C r is a natural number.
order of arrangements are considered as distinct
permutations. (ii) C r = nC n− r , for 0 ≤ r ≤ n
n

(ii) If anti-clockwise and clockwise order of arrangements (iii) n


C 0 = n C n = 1, nC1 = n
are not distinct, e.g. arrangement of beads in a necklace,
arrangement of flowers in a garland etc., then number
(iv) Let n and r be non-negative integers such that r ≤ n.
n+1
of circular permutations of n distinct items is
( n − 1)!
. Then, n
C r + nC r−1 = Cr [Pascal’s law]
2 n n−1
n
Cr = ⋅ C r−1 [ 1≤ r ≤ n ]
EXAMPLE 4. There are 20 persons among whom r
two are brothers. Find the number of ways in which (v) If 1 ≤ r ≤ n, then n ⋅ n−1C r−1 = ( n − r + 1) ⋅ nC r−1
we can arrange them around a circle, so that there is
exactly one person between the two brothers. (vi) nC x = nC y ⇒ x = y or x + y = n
a. 18! b. 2 × 18 ! c. 17! d. 18 × 17 !  n − r + 1
(vii) n
C r / nC r−1 =  
Sol. b. Let B1 and B2 be two brothers among 20 persons  r 
and let M be the person. Clearly, M can be chosen (viii) n
C 0 + n C1 + n C 2 + . . . + n C n = 2n
from 18 persons (excluding B1 and B2) in 18 ways.
Considering the two brothers B1 and B2 and the person M (ix) n
C 0 + n C 2 + n C 4 + . . . = n C1 + n C 3 + n C5 . . . = 2n−1
2 n+1
as one person, we have 18 persons in all.
(x) C 0 + 2n+1 C1 + 2n+1 C 2 + . . . + 2 n+1
C n = 22n
These 18 persons can be arranged around a circle in
(18 − 1) ! = 17 ! ways. But B1 and B2 can be arranged among
them solve in 2! ways. SOME IMPORTANT RESULTS ON COMBINATIONS
Hence, total number of required ways
(i) Number of combinations of n different things taken r at
= 18 × 17 ! × 2! = 2 × 18 ! a time
(a) When p particular things are always included = n − p C r − p .
COMBINATIONS (b) When p particular things are never included = n − p C r .

Each of the different selections made by taking some or (c) When p particular things are not together in any
all of a number of objects, (irrespective of there selection =n C r −n − p C r − p .
arrangements) is called a combinations. e.g. The different (ii) If there are n person in a room and they shake their
combinations formed of two out of three letters A , B, C hand with others only once, then total number of shake
hands = nC 2 .
are AB, AC, BC.
(iii) The total number of combinations of n different things
n taken one or more at a time or the number of ways of n
Meaning of Cr different things selecting atleast one of them is
The number of combinations of n different things taken
n
C 1 + nC 2 + K + nC n = 2n − 1.
 n (iv) The number of combinations of n identical things taking
r at a time is denoted by n C r or C( n, r ) or   .
r  r ( r ≤ n ) at a time is 1.
n (v) The number of ways of selecting r things out of n alike
n! Pr
Then, n
Cr = = [0 ≤ r ≤ n] things is ( n + 1), (where r = 0, 1, 2, 3, K , n ).
r ! (n − r ) ! r! (vi) If out of ( p + q + r ) things, p are alike of one kind, q are
n ( n − 1) ( n − 2) K ( n − r + 1) alike of second kind and rest are alike of third kind,
= , n ∈ N and r ∈ W
r (r − 1) (r − 2) K 2⋅ 1 then the total number of combinations is
[( p + 1) ( q + 1) ( r + 1)] − 1.
If r > n, then n C r = 0
86 NDA/NA Pathfinder

EXAMPLE 5. From 12 books, the difference between


number of ways a selection of 5 books, when one
Geometrical Applications
specified book is always excluded and one specified of Combinations
book is always included is Some basic geometrical applications of n C r are as follows
a. 64 b. 118 c. 132 d. 330
1. Out of n non-concurrent and non-parallel straight
Sol. c. If a specified book is always included. It means lines points of intersection are n C 2 .
4 books are selected out of the remaining 11 books
2. Out of ‘n’ points the number of straight lines are
∴Number of ways = 11C 4
(when no three are collinear) n C 2 .
If a specified book is always excluded. It means 5 books
are selected out of the remaining 11 books. 3. If out of n points mare collinear, then number of
∴Number of ways = 11C 5 straight lines = nC 2 − mC 2 + 1, where m≥ 3.
Thus, Required number of ways = 11C 5 − 11C 4 4. In a polygon total number of diagonals out of n
11! 11 n ( n − 3)
= − = 132 points (no three are collinear) = nC 2 − n = .
5 !6 ! 4 !7 ! 2
5. Number of triangles formed from n points is nC 3
EXAMPLE 6. How many 5-letter words with or
without meaning can be formed out of letters of the (when no three points are collinear).
word ‘EQUATIONS’ if repetition of letters is not 6. Number of triangles out of n points in which mare
allowed? collinear are n C 3 − mC 3 , where m≥ 3 .
a. 59 b. 9 C5 7. Number of parallelogram in two system of parallel
c. C5 × 5!
9
d. 95 lines (when 1st set contains mparallel lines and 2nd
set contains n parallel lines) = nC 2 × mC 2 .
Sol. c. There are 9 letters in the word ‘EQUATIONS’. In
order to form 5-letter words, we first select 5 letters out
EXAMPLE 7. The number of parallelograms that can
of these 9 letters.
9
be formed from a set of four parallel lines intersecting
Selection of 5 letters out of 9 letter can be done in C 5 another set of three parallel lines is
ways.
a. 6 b. 18 c. 12 d. 9
Arranging of 5 selected letters to form 5-letter words can
be done in 5! ways. Sol. b. Required number of parallelograms
∴Total number of 5 letter words = 9C 5 × 5! 4! 3! 4×3 3
= 4C 2 × 3C 2 = × = × = 18
2! 2! 2!1! 2 × 1 1

PRACTICE EXERCISE
1. The product of r consecutive positive integers, 5. All the letters of the word ‘EAMCET’ are
divided by r ! is arranged in possible ways. The number of such
(a) a proper fraction (b) equal to r arrangements in which no two vowels are
(c) a positive integer (d) None of these adjacent to each other is
2. The number of 5 digit telephone numbers having (a) 360 (b) 144 (c) 72 (d) 54
atleast one of their digits repeated is 6. The number of words that can be formed from
(a) 90000 (b) 10000 (c) 30240 (d) 69760 the letters of the word ‘INDRAPRASTHA’ when
3. Four dice (six faced) are rolled. The number of the vowels are never separated, is
possible outcomes in which atleast one die (a) 727560 (b) 725760
shows 2 is (c) 752760 (d) 757260
(a) 1296 (b) 625
7. There are 10 lamp in a hall. Each one of
(c) 671 (d) None of these them can be switched on independently. The
4. In how many ways 4 boys and 3 girls can be number of ways in which the hall can be
seated in a row, so that they are alternate? illuminated is
(a) 108 (b) 144 (c) 96 (d) 72 (a) 102 (b) 1023 (c) 210 (d) 10!
MATHEMATICS Permutations and Combinations 87

2n + 1 2n − 1
8. 20 persons are invited for a party. The number 19. If Pn − 1: Pn = 3 : 5, then consider the
of ways in which they and the host can be seated following statements
at a circular table, if two particular persons be
seated on either side of the host is equal to I. The value of n is 3
(a) 2 ⋅ (18) ! (b) 18 ! ⋅ 3 ! II. P ( 5, n ) = P ( 6, n − 1)
(c) 19 ! ⋅ 2 ! (d) None of these Which of the above statement(s) is/are correct?
9. Seven women and seven men are to sit round a (a) Only I (b) Only II
circular table such that there is a man on either (c) Both I and II (d) Neither I nor II
side of every women, then the number of seating
20. Four-letter words are to be formed using the
arrangements is
letters of the word ‘FAILURE’.
(a) (17 !) 2
(b) (6!) 2
(c) 6! × 7 ! (d) 7!
n
Consider the following statements
P (n , r )
10. What is the value of ∑ r! ? I. Number of words of F is included in each word
is 6C3 × 4 !.
r =1
(a) 2 n − 1 (b) 2 n (c) 2 n − 1 (d) 2 n + 1 II. Number of words, if it contains two different
vowels and two different consonants is
11. How many times does the digit 3 appear while 3
C 2 × 4C 2 × 4 !.
writing the integers from 1 to 1000?
Which of the above statement(s) is/are correct?
(a) 269 (b) 271 (c) 300 (d) None of these
(a) Only I (b) Only II
12. The value of (c) Both I and II (d) Neither I nor II
(7C0 + 7C1 ) + (7C1 + 7C2 ) + K + (7C6 + 7C7 ) is
Directions (Q. Nos. 21-22) Find the number of
(a) 2 7 − 1 (b) 2 8 − 2 (c) 2 8 − 1 (d) 28 numbers between 400 and 4000 that can be formed
13. The solution set of Cx − 1 > 2 ⋅ Cx is10 10 with the digits 2, 3, 4, 5, 6 and 0.
(a) {1, 2, 3} (b) {4, 5, 6} (c) {8, 9, 10} (d) {9, 10, 11} 21. When number is of 3 digits
(a) 75 (b) 90
14. Given 5 line segments of lengths 2, 3, 4, 5, 6 (c) 60 (d) 80
units. Then the number of triangles that can be
formed by joining these lines are 22. When number is of 4 digits
(a) 5C 3 (b) 5C 3 − 3 (a) 120 (b) 150
(c) 180 (d) 160
(c) C 3 − 2
5
(d) 5
C3 − 1

15. The sides AB, BC , CA of a ∆ABC have 3, 5 and 6 Directions (Q. Nos. 23-24) Find the total numbers
interior points, respectively on them. The of four digits number that are greater than 3000,
number of triangles that can be constructed that can be formed using the digits 1, 2, 3, 4, 5
using these points as vertices is given by and 6.
(a) 364 (b) 333 23. If repetition is not allowed
(c) 240 (d) None of these (a) 240 (b) 24
(c) 360 (d) 480
16. Everybody in a room shakehands with everybody
else. The total number of hand shakes is 66. The 24. If repetition is allowed
total number of persons in the room is (a) 486 (b) 864 (c) 240 (d) 1296
(a) 11 (b) 12 (c) 13 (d) 14
Directions (Q. Nos. 25-27) A committee of five
17. In an examination, there are three multiple members is to be made from 4 gentlemen and
choice questions and each question has 4 choices.
6 ladies. Then, find the number of ways in which
Then, the number of ways in which a student
committee can be formed.
can fail to get all answer correct is
(a) 11 (b) 12 (c) 27 (d) 63 25. Gentlemen are in majority
(a) 60 (b) 66 (c) 120 (d) 126
18. A parallelogram is cut by two sets of m lines
parallel to its sides. The number of 26. Ladies are in majority
parallelograms thus formed is (a) 60 (b) 66 (c) 120 (d) 186
(a) m
C 2 × mC 2 (b) 2( m + 2 C 2 ) 27. Exactly 3 ladies are chosen
(c) ( m + 2 C 2 )2 (d) None of these (a) 120 (b) 126 (c) 60 (d) 40
88 NDA/NA Pathfinder

Directions (Q. Nos. 28-30) There are ten points in a PREVIOUS YEARS’ QUESTIONS
plane, of these ten points four points are in a straight
line and with the exception of these four points, no 38. What is the number of ways that 4 boys and
three points are in the same straight line. On the basis 3 girls can be seated, so that boys and girls sits
of this information answer the following questions. alternate? e 2012 I
(a) 12 (b) 72 (c) 120 (d) 144
28. The number of triangles formed by joining these
ten points is 39. The number of permutations that can be formed
from all the letters of the word ‘BASEBALL’ is
(a) 116 (b) 80 (c) 96 (d) 56 e 2012 II
29. The number of straight lines formed by joining (a) 540 (b) 1260 (c) 3780 (d) 5040
these ten points is 40. What is the number of diagonals which can be
(a) 15 (b) 40 (c) 24 (d) 39 drawn by joining the angular points of a polygon
30. The number of quadrilaterals formed by joining of 100 sides? e 2012 II
these ten points is (a) 4850 (b) 4950 (c) 5000 (d) 10000
(a) 82 (b) 48 (c) 185 (d) 64 41. If P ( 77, 31) = x and C ( 77, 31) = y, then which one
Directions (Q. Nos. 31-33) We have to choose of the following is correct? e 2013 I
(a) x = y (b) 2x = y (c) 77 x = 31y (d) x > y
Eleven players for cricket team from eight batsmen,
six bowlers, four all rounder and two wicket keepers 42. In how many ways can the letters of the word
in the following conditions. ‘GLOOMY’ be arranged so that the two O’s
should not be together? e 2013 I
31. The number of selections, when atmost one all (a) 240 (b) 480 (c) 600 (d) 720
rounder and one wicket keeper will play
(a) 4 C1 × 14
C10 + 2C1 × C10 + 4C1 × 2C1 ×
14
C9 +
14 14
C11
43. If C ( 28, 2r ) = C ( 28, 2r − 4), then what is r equal
to? e 2013 II
(b) C1 ×
4 15
C11 + 15
C11 (c) C1 ×
4 15
C10 + 15
C11 (a) 7 (b) 8 (c) 12 (d) 16
(d) None of the above
44. Out of 7 consonants and 4 vowels, words are to be
32. Number of selections, when two particular formed by involving 3 consonants and 2 vowels.
batsmen do not want to play when a particular The number of such words formed is e 2014 I
bowler will play (a) 25200 (b) 22500 (c) 10080 (d) 5040
(a) 17
C10 + 19
C11 (b) 17
C10 + C11 +
19 17
C11 45. How many different words can be formed by
(c) 17
C10 + 20
C11 (d) 19
C10 + 19
C11 taking four letters out of the letters of the word
‘AGAIN’, if each word has to start with A?
33. Number of selections when a particular batsman e 2012 I
and a particular wicket keeper do not want to (a) 6 (b) 12 (c) 24 (d) None of these
play together
(a) 218 C10 (b) 19
C11 + 18
C10 Directions (Q. Nos. 46-48) Given that, C (n, r): C (n, r + 1)
(c) 19
C10 + 19
C11 (d) None of these = 1: 2 and C (n, r + 1) : C (n, r + 2) = 2 : 3 e 2014 I
46. What is n equal to?
Directions (Q. Nos. 34-37) Different words are being (a) 11 (b) 12 (c) 13 (d) 14
formed by arranging the letters of the word
47. What is r equal to?
‘ARRANGE’
(a) 2 (b) 3 (c) 4 (d) 5
34. The number of arrangement in which two R’s are 48. What is P ( n , r ) : C( n , r ) equal to?
never together is
(a) 900 (b) 1080 (c) 1020 (d) 960 (a) 6 (b) 24 (c) 120 (d) 720

35. The number of arrangement in which two A’s are 49. What is the number of ways in which one can
post 5 letters in 7 letter boxes? e 2014 II
together but not the two R’s is
(a) 7 5 (b) 3 5 (c) 57 (d) 2520
(a) 210 (b) 240 (c) 270 (d) 180
36. The number of arrangements in which neither 50. What is the number of ways that a cricket team
the two A’s nor the two R’s are together is of 11 players can be made out of 15 players?
e 2014 II
(a) 840 (b) 540 (c) 720 (d) 660
(a) 364 (b) 1001 (c) 1365 (d) 32760
37. The number of words can be formed without 1
changing the relative order of the vowels and 51. What is ∑ n + r Cn equal to?
r=0 e 2015 I
consonants is n+ 2 n+ 2 n+ 3 n+ 2
(a) 48 (b) 36 (c) 72 (d) 27 (a) C1 (b) Cn (c) Cn (d) Cn + 1
MATHEMATICS Permutations and Combinations 89

52. How many words can be formed using all the 57. The number of ways in which 3 holiday tickets
letters of the word ‘NATION’, so that all the three can be given to 20 employees of an organisation,
vowels should never come together? e 2015 I if each employee is eligible for any one or more
(a) 354 (b) 348 (c) 288 (d) None of these of the tickets, is e 2015 II
(a) 1140 (b) 3420
53. A polygon has 44 diagonals. The number of its (c) 6840 (d) 8000
sides is e 2015 II
(a) 11 (b) 10 (c) 8 (d) 7 58. What is the number of different messages that
can be represented by three 0’s and two 1’s?
54. The number of 3-digit even numbers that can be e 2016 I
formed from the digits 0, 1, 2, 3, 4 and 5, (a) 10 (b) 9
repetition of digits being not allowed, is e 2015 II (c) 8 (d) 7
(a) 60 (b) 56 (c) 52 (d) 48
59. What is the number of four-digit decimal numbers
55. If different words are formed with all the letters of (< 1) in which no digit is repeated? e 2016 I
the word ‘AGAIN’ and are arranged alphabetically (a) 3024 (b) 4536
among themselves as in a dictionary, the word at (c) 5040 (d) None of these
the 50th place will be e 2015 II
(a) NAAGI (b) NAAIG (c) IAAGN (d) IAANG 60. What is the number of ways in which 3 holiday
travel tickets are to be given to 10 employees of
56. The number of ways in which a cricket team of an organisation, if each employee is eligible for
11 players be chosen out of a batch of 15 players,
anyone or more of the tickets? e 2016 I
so that the captain of the team is always
(a) 60 (b) 120
included, is e 2015 II
(c) 500 (d) 1000
(a) 165 (b) 364 (c) 1001 (d) 1365

ANSWERS
1 c 2 d 3 c 4 b 5 c 6 b 7 b 8 a 9 c 10 a
11 b 12 b 13 c 14 b 15 b 16 b 17 d 18 c 19 b 20 c
21 c 22 a 23 a 24 b 25 b 26 d 27 a 28 a 29 b 30 c
31 a 32 b 33 b 34 a 35 b 36 d 37 b 38 d 39 d 40 a
41 d 42 a 43 b 44 a 45 c 46 d 47 c 48 b 49 a 50 c
51 a 52 c 53 a 54 c 55 b 56 c 57 d 58 a 59 b 60 d

HINTS AND SOLUTIONS 


1. (c) Consider r consecutive positive 4. (b) Possible arrangement will be the 6. (b) Required number of ways
integers n + 1, K , n + r − 1, n + r. form B G B G B G B 9! 4!
= ×
( n + 1)K( n + r ) Boys occupy 1, 3, 5, 7 places and girls 2! 3!
Then,
r! occupy 2, 4, 6 places. = 725760
n! ( n + 1)K ( n + r ) n + r
= = Cr ∴ Four boys can be seated in 4! ways. 7. (b) Each bulb has two choices, either
n! r !
which is a positive integer. Three girls can be seated in 3! ways. switched on or off.
∴ Required number = 4! × 3! = 144 ∴ Required number = 210 − 1 = 1023
2. (d) A telephone number can be started
with zero. [Since, in one way when all are
5. (c) Arrangement is × M × C × T × , first
Therefore, the number of numbers switched off, then the hall will
we place 3 consonant in 3! ways and
with at least one digit repeated not be illuminated]
then 3 vowels.
= 105 − 10 P At four ‘×’ places (2 between them and
8. (a) Host can sit on any seat. Two
5
= 105 − ( 10 ⋅ 9 ⋅ 8 ⋅ 7 ⋅ 6) = 69760 particular guests can sit on either
2 on sides) in which one vowel E is sides of host in 2 ! = 2 ways and
3. (c) Total number of possible outcomes repeated can be placed in 4 P / 2! ways. these three persons consider a
3
= 64 single unit and remaining 18 guests
Hence, required number
Number of possible outcomes in
= 3! ⋅ 4 P / 2! = 72 in (18) ! ways. Hence, total ways
which 2 does not appear on any dice 3 = 2 ⋅ ( 18) !.
= 54 . Required number =64 − 54 = 671
90 NDA/NA Pathfinder

9. (c) 7 womens can be seated along a 14. (b) We know that, in a triangle, sum of 20. (c) There are 7 letters in the word
circular table in 6! ways. length of two sides is always greater than ‘FAILURE’.
w the third side. Number of vowels = 4
w w ∴ The triangle will not be formed if Number of consonants = 3
we select segments of lengths. Out of four letters, one letter is F and
( 2, 3, 5), ( 2, 3, 6) and ( 2, 4, 6). Hence, other three letters can be selected from
w
number of triangles formed = 5C − 3 remaining 6 letters in 6 C ways.
3 3
15. (b) Selection of 3 points from given These four letters can be arranged in
w w 14 points can be made in 4! ways.
w 14
C = 364 ...(i) Total number of words = 6C × 4!
3 3
If mens sit at the places marked by ‘×’, So, Statement I is correct.
But selection of 3 points from the
then mens will be on either side of women. Now, 2 vowels and 2 consonants can
points on one line cannot give any
∴ 7 mens can be seated is 7! ways triangle. Such selections are be chosen in 3 C × 4C ways and can
2 2
∴ Required number = 6! × 7! 3
C + 5C + 6C be arranged in 4! ways.
n n 3 3 3
P ( n, r ) 1 n! = 1 + 10 + 20 = 31
10. (a) ∑ =∑ ⋅ ...(ii) Total number of words
r =1 r! r =1 r ! ( n − r )! Hence, total number of triangles that = 3C × 4C × 4 !
2 2
can be formed = 364 − 31 = 333
 n n!  So, Statement II is correct.
Q Pr = ( n − r )!  16. (b) Let there be n persons in a room.
  ∴ Total number of handshakes
21. (c) Since, the number should be greater
n
 n n!  than 400 therefore, hundreds place can
= ∑ Q C r = r !( n − r )! 
1
n
Cr = nC = 66 ⇒ n ( n − 1) = 66 be filled up by any one of the three digits
r =1   2
2 4, 5, 6 in 3 different ways.
⇒ n2 − n − 132 = 0
= ( n C + nC + nC + ... + nC n ) Remaining two places can be filled up
1 2 3 ⇒ ( n + 11)( n − 12) = 0 by remaining five digits in 5 P ways.
= ( 1 + n C + n C + nC + ... + n C n ) − 1 ⇒ n = 12 [Q n ≠ − 11]. ∴ Required number
2
1 2 3
5!
= ( n C + nC + nC + ... + nC n ) − 1 17. (d) Each question can be answered in = 3 × 5 P = 3 × = 60
0 1 2 2
4 ways. 3!
= ( 1 + 1) − 1 = 2 − 1
n n
All questions can be answered in 1 way 22. (a) Thousands place can be filled up by
11. (b) Number of time 3 occurs = (when 3 So, the required number = 43 − 1 = 63 any of two digits 2 and 3 in 2 ways.
occurs exactly at one place) + (when 3 18. (c) The two sets of m parallel lines along Remaining three places can be filled
occurs exactly at two places) + (when 3 with two sets of two parallel lines of the up by remaining five digits in 5 P
3
occurs exactly at three places) given parallelogram will form two sets of ways.
= 3C × 9 × 9 + 3C × 9 + 3C (m + 2) parallel lines. Each parallelogram ∴ Required number
1 2 3 5!
= 243 + 27 + 1 = 271 is formed by choosing two parallel lines = 2 × 5P = 2 × = 120
3
from each of the above. 2!
12. (b) ( 7 C + 7 C ) + ( 7 C + 7 C ) 23. (a) Required number of numbers
0 1 1 2 ∴ Total number of parallelograms
= 4 × 5 × 4 × 3 = 240
+ ...+ ( 7 C + 7 C ) = m + 2C × m + 2 C = ( m + 2 C ) 2
6 7 2 2 2
2n + 1 24. (b) Required number of numbers
= 2[ 7 C + 7 C + ...7 C ] − 7 C P
n −1 3 = 4 × 6 × 6 × 6 = 864
0 1 7 0 19. (b) We have, 2 n −1
=
− C − C
7 7 Pn 5 Sol. (Q. Nos. 25-27) The committee of five
= 2⋅ 27 − 2 = 28 − 2
2 7
( 2n + 1)! ( n − 1)! 3
⇒ × = numbers can be made by choosing
( n + 2)! ( 2n − 1)! 5 gentlemen and ladies in the following
13. (c) For 10
C and 10
C x , 10 ≥ x − 1
x −1 ( 2n + 1)( 2n)( 2n − 1)! ( n − 1)! 3 manner
and 10 ≥ x ⇒ × =
( n + 2)( n + 1)n( n − 1)! ( 2n − 1)! 5 Gentlemen (4) Ladies (6)
∴ x ≤ 10 2( 2n + 1) 3 0 5
⇒ =
Q 10
C > 2 ⋅10 C x ( n + 2)( n + 1) 5 1 4
x −1
2 3
10! 10! ⇒ 10( 2n + 1) = 3( n + 2) ( n + 1)
⇒ > 2⋅ 3 2
( 11 − x )!( x − 1)! ( 10 − x )! x ! ⇒ 3n2 + 9n + 6 = 20n + 10
4 1
1 ⇒ 3n2 − 11n − 4 = 0
⇒ ⇒ ( n − 4) (3n + 1) = 0 ⇒ n = 4 25. (b) Number of ways to make the committee,
( 11 − x ) ⋅ ( 10 − x )!( x − 1)!
10! ∴ Statement I is not correct. when gentlemen are in majority
> 2⋅ = 4C 6C + 4C 6C
( 10 − x )! x ⋅ ( x − 1)! Now, for n = 4, 3 2 4 1
1 2 P(5, n) = 5 P = = 120
5! = 4 × 15 + 1 × 6 = 66
⇒ >
11 − x x 4
1! 26. (d) Number of ways to make a
6!
⇒ x > 22 − 2x [Q x ≥ 0] and P( 6, n − 1) = 6 P = = 120 committee, when ladies are in majority
3
22 1 3! = 4C 6C + 4C 6C + 4C 6C
⇒ x> ⇒x>7 ⇒ P(5, n) = P( 6, n − 1), for n = 4 0 5 1 4 2 3
3 3 = 1 × 6 + 4 × 15 + 6 × 20
∴ x = {8, 9, 10} [Q x ∈ N ] ∴ Statement II is correct. = 6 + 60 + 120 = 186
MATHEMATICS Permutations and Combinations 91

27. (a) Number of ways to make a If the particular bowler does not play, 38. (d ) The required number of ways that
committee, when exactly 3 ladies are then number of selections is 19 C . 4 boys and 3 girls can be seated, so that
11
chosen = 4C 6C = 6 × 20 = 120 If all the three players do not play, boys and girls alternate = 4! × 3!
2 3
= 24 × 6 = 144
28. (a) Total number of selections of then number of selections is 17 C .
11
3 points out of 10 points Total number of selections is 39. (d ) There are 8 letters in word
10 × 9 × 8 ‘BASEBALL’ in which 2B, 2A, 2L, 1S
= 10C = = 120 17
C + 19 C + 17 C
3
3! 10 11 11 and 1E.
When, 3 collinear points are selected 33. (b) If the particular batsman is selected. So, the number of permutations that
no triangle is formed. Then, rest of 10 players can be selected can be formed from all the letters of
in 18 C ways. the word ‘BASEBALL’
Number of selections of 3 points out 10
If particular wicketkeeper is selected. 8!
of 4 collinear points
Then rest of 10 players can be selected = = 5040
= 4C = 4 2! 2! 2!
3 in 18 C ways.
∴ Required number = 120 − 4 = 116 10
40. (a) The number of diagonals which can
If both are not selected, then number
be drawn by joining the angular points
29. (b) Total number of selections of of ways is 18 C .
2 points out of 10 points
11 of a polygon of 100 sides = 100C − 100
Hence, total number of ways is 2
10 × 9 100!
= 10C = = 45 = − 100
2
2 18 C + 18C = 19C + 18C . 2! 98!
2! 10 11 11 10
Sol. (Q. Nos. 34-37) 100 × 99 × 98!
Number of selections of two points, when = − 100
only one straight line is formed = 4C There are 7 letters in the word 2 × 98!
2
[If the 4 collinear points are ‘ARRANGE’ and there are two A’s = 50 × 99 − 100 = 4950 − 100
A , B , C , D, then AB, AC , AD, BC , and two R’s and three different letters. = 4850
BD, CD are the same line] Number of consonants = 4
41. (d) Given that, P ( 77,31) = x
∴ Required number Number of vowels = 3
i.e. 77
P =x ...(i)
Total number of arrangement when 31
= 10C − 4C + 1 = 45 − 6 + 1 = 40
2 2 there is no restriction and C ( 77,31) = y
30. (c) Total number of selections of 4 points 7! 77
C = y
= = 1260 i.e. ...(ii)
out of 10 given points 2! 2!
31
77 !
10! From Eq. (ii), = y
= 10C = 34. (a) Number of arrangement, when two 31 ! ( 77 − 31)!
4
4! 6!
10 × 9 × 8 × 7 R’s are together 77!
= = 210 ⇒ = 31! y ⇒ 77 P = 31! y
6! 2! ( 77 − 31)! 31
24 = = 360
Number of selections of 4 points, 2! 2!
⇒ x = (31! y ) [from Eq. (i)]
when no quadrilateral is formed ∴ Required number = 1260 − 360 = 900
∴ x> y
= 4C ⋅ 6C + 4C ⋅ 6 C 35. (b) Number of arrangement, when two
3 1 4 0 42. (a) First we arrange the four letters G, L,
A’s are together
= 4 × 6 + 1⋅ 1 = 25 M and Y in the alternate position = 4!
6! 2!
Hence, requ ired number = = 360 Now, rest of letters O,O arrange in 5
= 210 − 25 = 185 2! 2!
alternate positions = 5C
2
Number of arrangement, when two A’s
31. (a) When one all rounder and ten ∴ Required number of ways = 4! × 5C
and two R’s are together 2
players from bowlers and batsmen play, 5× 4
=
5! 2! 2!
= 120 = 24 ×
number of ways is 4 C × 14C . 2! 2! 2
1 10
When, one wicketkeeper and ∴ Required number = 360 − 120 = 240 = 24 × 10 = 240
10 players from bowlers and batsmen 43. (b) Given, C ( 28, 2r ) = C ( 28, 2r − 4)
36. (d) Number of arrangement, when two
play, number of ways is 2 C × 14C .
1 10 R’s are not together = 900 ⇒ 28
C = 28C
2r 2r − 4
When one all rounder, one [Q C x = nC y ⇒ x + y = n]
n
Number of arrangements, when two
wicketkeeper and nine players from R’s are not together and two A’s are ⇒ 2r + ( 2r − 4) = 28
batsmen and bowlers play, number of together = 240
ways is 4 C × 2C × 14C . ⇒ 4r = 32
1 1 9 ∴ Required number = 900 − 240 = 660
∴ r =8
When all eleven players play from
37. (b) Relative order of vowels and
bowlers and batsman, then number of 44. (a) In out of 7 consonants, 3 consonants
consonant remains unchanged,
ways is 14 C . can be selected in 7 C ways.
11 therefore, vowels will occupy only 3
Total number of selections is vowel’s place and consonants will In out of 4 vowels, 2 vowels can be
4
C × 14C + 2C × 14C + 4C occupy consonants place. selected in 4 C ways.
1 10 1 10 1 2
4! ∴ Number of such words
×2C × 14
C + 14
C . Now, 4 consonants can be arranged in
1 9 11 2! = 7C × 7C × 5 !
32. (b) If the particular bowler plays, then two 3! 3 2
and 3 vowels can be arranged in . [Since, 5 letters can be selected in 5
batsman will not play. so, rest of 10 players 2!
can be selected from 17 other players. 4! 3! ways]
∴ Required number = × = 36
17 2! 2! = 35 × 6 × 120 = 25200
Number of such selections is C .
10
92 NDA/NA Pathfinder

45. (c) The word ‘AGAIN’ has five letters 49. (a) There are 5 letters and 7 letter boxes. 55. (b) We have, word ‘AGAIN’
2A, 1G, 1I, 1N. Since, A repeat two First letter can be put any 7 letters The letter starts from AAGIN
times and A is fixed at first position boxes = 7 ways
then, we have to arrange remaining 4 = 4! = 24
Similarly, 2nd, 3rd, 4th and 5th letters
letters in three vacant position. The letter starts from GAAIN
be put in 7 ways each, respectively.
∴ Required number of ways =
4!
= 12
= 1 × 4 × 3 × 2 = 24 ∴ Required number of ways 2!
= 7 × 7 × 7 × 7 × 7 = 75
Sol. (Q. Nos. 46-48) The letters starts from IAAGN
Given that, C ( n, r ):C ( n, r + 1) = 12
: 50. (c) 4!
= = 12
1 1 2!
⇒ n
C r :n C = 1: 2
∑ ∑
n+ r n+ r
r +1 51. (a) C n= C
n+ r −n
n! r =0 r =0 Number of total letters = 48
r !( n − r )! 1 ∴ 49th word is NAAGI and 50th
⇒ = 1

n+ r n+1
n! 2 = C r = nC + C word is NAAIG.
0 1
( r + 1)!( n − r − 1)! r =0
56. (c) Total number of selection of 11 players
( r + 1)! ( n − r − 1)! 1 n + 1!
⇒ = = 1+ = 1+ n + 1= n + 2 out of 15 players in which captain is
r ! ( n − r )! 2 n! 1! included = 14C
( r + 1) ⋅ r ! ( n − r − 1)! 1 n+ 2 10
⇒ = = C
r ! ( n − r )! ( n − r − 1)! 2 1 14! 14 × 13 × 12 × 11
= =
r+ 1 1 52. (c) Required number of words 10! 4! 1× 2× 3× 4
⇒ = 6! 4! × 3!
n− r 2 = − = 1001
⇒ 2r + 2 = n − r 2! 2!
= 360 − 72 = 288 57. (d) Since, each employees is eligible for
⇒ n = 3r + 2 …(i)
and C ( n, r + 1) : C ( n, r + 2) = 2 : 3 one or more ticket(s).
53. (a) Let the number of sides of the
⇒ n
C : nC = 2:3 Hence, total number of ways = 203
r +1 r + 2
polygon be n.
n! = 8000
Then, number of diagonals = nC − n
⇒ ( r + 1)!( n − r − 1)! = 2 2
58. (a) Here, we are given three 0’s and two
n! ⇒ 44 = nC − n 1’s.
3 2
n!
( r + 2)!( n − r − 2)! ⇒ 44 = −n Hence, number of ways of different
( r + 2)!( n − r − 2)! 2 2!( n − 2)! messages
⇒ =
( r + 1)!( n − r − 1)! 3 n ( n − 1)( n − 2)! 5! 5× 4
⇒ 44 = −n = = = 10
( r + 2) ⋅ ( r + 1)!( n − r − 2)! 2 2!( n − 2)! 3!× 2! 2
⇒ =
( r + 1)!( n − r − 1)( n − r − 2)! 3 [ n( n − 1)] 59. (b) Clearly, number of four-digit decimal
⇒ 44 = −n
r+ 2 2 2 numbers that can be formed using the
⇒ =
n− r− 1 3 ⇒ n2 − 3n − 88 = 0 digits 0, 1, 2, 3, 4, 5, 6, 7, 8, 9, when no
⇒ 3 r + 6 = 2n − 2r − 2 digit is repeated, are given by
⇒ ( n − 11)( n + 8) = 0
⇒ 5 r + 8 = 2n …(ii) Number of numbers Number of numbers
⇒ n = 11, − 8 [ n ≠ −8] of the type of the type
On solving Eqs. (i) and (ii), we get
Hence, polygon has 11 sides. – 0
r=4
46. (d) Put the value of r in Eq. (i), we get 54. (c) We have, digits 0, 1, 2, 3, 4 and 5 for
10! 9! 9!
n = 3 ( 4) + 2 3-digit even number. 10
P − 9P = − = ( 10 − 1)
4 3
(i) When 0 is at unit place, then 6! 6! 6!
n = 12 + 2 ⇒ n = 14
9 × 8 × 7 × 6! × 9
47. (c) 0 =
6!
48. (b) We have, P ( n, r ) : C ( n, r ) 4 × 5 × 1 = 20 numbers
= 72 × 7 × 9 = 4536
= n Pr : nC r (ii) When 0 is not at unit place, then
n! n! 60. (d) Since, each ticket can be given to
= : 2 or 4
( n − r )! r !( n − r )! any one of the 10 employees of an
organisation.
1
= 1 : = r! : 1 4 × 4 × 2 = 32 numbers
∴ Required number of ways
r! ∴ Total even numbers
= 4! : 1 = 24 : 1 = 24 = 10 × 10 × 10 = 103 = 1000
= 20 + 32 = 52
08
BINOMIAL THEOREM
In NDA exam, generally 1-2 questions are asked from this chapter which are based on general
term, term independent of x, number/sum of terms, middle terms and combinational identities.
An algebraic expression consisting of two dissimilar terms with positive or negative sign between
them is called a binomial expression, e.g. ( x + a),
a b 3
x2 + y2 , 2 − 4 and x − etc., are called binomial expressions.
x x y

Binomial Theorem for Any Positive Integer n


The formula by which any power of a binomial expression can be expanded is known as binomial
theorem. Binomial theorem for any positive integer n is
( a + b) n = C ( n, 0 ) a n + C ( n, 1) a n − 1 b + C ( n, 2) a n − 2 b 2 + … + C ( n, n − 1) ab n − 1 + C ( n, n) b n
OR ( a + b) n = nC 0 a n b 0 + nC1 a n − 1 b + nC 2 a n − 2 b 2 + . . . + n C n−1 ab n−1 + n C n a 0 b n
n
n!
i.e. ( a + b) n = ∑ n C r ( a) n− r ( b) r where, C ( n, r ) = , 0 < r < n; n, r ∈ N
r= 0 r ! ( n − r )!
[here, the coefficients n C 0 , nC1 , …, nC n are called the Binomial coefficients.]
Special Cases
n
1. Putting a = 1 and b = x, we get (1 + x) n = nC 0 + nC1 x + nC 2 x 2 + . . . + nC n x n = ∑ n C r ⋅ x r
r= 0
n n
2. Putting a = 1 and b = − x, we get (1 − x) n = nC 0 − nC1 x + nC 2 x 2 + . . . + ( −1) n C n x r = ∑ ( −1) n nC r ⋅ x r
r= 0
General Term
In the expansion of ( a + b) n the general term is Tr + 1 = nC r ⋅ a n − r ⋅ b r .
In the binomial expansion of ( a + b) n , the rth term from the end is {( n + 1) − r + 1} = ( n − r + 2)th term from
the beginning.

Middle Term
In the expansion of ( a + b) n , the middle term is  + 1 th term if n is even. If n is odd, the two middle
n
2 
 n + 1 n + 1 
terms are   th and  + 1 th terms.
 2   2 
94 NDA/NA Pathfinder

Greatest Coefficient (v) If n is odd, then number of terms in ( x + a) n + ( x − a) n


If n is even, the greatest coefficient is n C n/2 and if n is odd,  n + 1
and ( x + a) n − ( x − a) n are equal to  .
 2 
the greatest coefficient is n C n − 1 or n C n + 1 both being equal.
2 2 (vi) If n is even, then number of terms in the expansion of
 n + 2
EXAMPLE 1. The middle term in the expansion of ( x + a) n + ( x − a) n are   and that in the
18  2 
 x − 1  is
expansion ( x + a) n − ( x − a) n are   .
n
 x
 2
a. 18
C9 b. −18 C9
c. 18
C0 d. −18 C10 EXAMPLE 3. If (1 + ax) n = 1 + 8x + 24x 2 +… , then
Sol. b. Here ‘n’ is even. find a and n.
a. 2, 8 b. 8, 24 c. 8, 4 d. 2, 4
∴ Middle term is  + 1 th term.
n
2 
Sol. d. We have,
∴ Middle term is 10th term. (1 + ax) n = nC 0 ( ax) 0 + nC1 ( ax)1 + nC 2 ( ax) 2 + …
1 9 n (n − 1) 2 2
⇒ T10 =18 C 9( x) 9  −  = − 18 C 9 = 1 + n ax + a x +…
 x 2!
Comparing with the given series, we get
EXAMPLE 2. The greatest coefficient in the
n (n − 1) 2 2
1 2n 1 + 8x + 24x2 + … = 1 + n ax + a x +…
expansion of  x +  is 2!
 x
n (n −1) 8 1 2
 8 = 24
1⋅ 3 ⋅ 5K( 2n − 1) ⋅ 2n 2! Hence, na = 8, a2 = 24 ⇒ a = , n (n −1)
a. b. 2! n 2  n
n! (n !)
n! ⇒ 32 (n − 1) = 24n ⇒ 4 (n − 1) = 3n ⇒ n = 4
c. 2
d. None of these 8 8
 n   ∴ a= = =2
 2 ! n 4
 
Sol. a. Since the middle term has greatest coefficient, Method for Finding the Independent
greatest coefficient = coefficient of the middle term Term or Constant Term
( 2n) ! 2n ( 2n − 1)( 2n − 2)( 2n − 3) K 4 ⋅ 3⋅ 2⋅ 1 In the expansion of ( a + b) n , the term which is free form
2n
Cn = =
n !n ! n !n !
x, i.e. which have x 0 is known as independent term. To
{( 2n − 1)( 2n − 3) K 3⋅ 1}{2n( 2n − 2)( 2n − 4) K 4) ⋅ 2} find independent term put the index of x obtained in
=
n !n ! general term equal to zero and find the value of r to
[1⋅ 3⋅ 5K ( 2n − 1)] 2[n (n − 1)(n − 2) K 2⋅ 1] calculate the required term.
=
n !n !
EXAMPLE 4. The coefficient of the term independent
1⋅ 3⋅ 5K( 2n − 1) 2nn ! 1⋅ 3⋅ 5...( 2n − 1) 2n 10
= =  1 
n !n ! n! of x in the expansion of  x +  is
 x
a. 10 b. 252 c. 256 d. 20
Properties of Binomial Theorem
10
Sol. b. In the expansion of  x +
(i) The number of terms in the Binomial expansion is 1
 , the general term is
n + 1 , that is one more than the index n.  x
(ii) In any term, the sum of the indices of a and b is equal  1
r
Tr +1 = 10C r ( x)10− r  
to n.  x
(iii) Tr + 1 in the expansion of (1 + x) n is equal to 5−r 5−r r

= 10C r ( x) 2 ( x− r/2 ) = 10C r x 2 2 = 10C r ( x) 5− r
n ( n − 1) ( n − 2) … ( n − r + 1) r
x . For independent term,
r!
r−5=0 ⇒ r = 5
(iv) Sum of the binomial coefficients in the expansion of
Thus, T5+ 1 =10C 5( x) 5− 5 = 10C 5 = 252
( a + b) n is 2n (Put a = b = 1).
MATHEMATICS Binomial Theorem 95

Method to Find Greatest Term (Numerically) 9. n


C1 + 2 nC 2 + 3 nC 3 + . . . + n nC n = n ⋅ 2n−1
in the Expansion of ( a + x ) n 10. n
C1 − 2 nC 2 + 3 nC 3 − . . . = 0
Method 1. 11. n
C 0 + 2 nC1 + 3 nC 2 + . . . + ( n + 1) nC n = ( n + 2) ⋅ 2n−1
I. Write Tr + 1 and Tr from the given expansion.

II. Find
Tr + 1
. III. Put
Tr + 1
>1
BINOMIAL THEOREM FOR
Tr Tr ANY INDEX
IV. Solve the inequality in step III for r to get an n ( n − 1) 2
inequality of the form r < m or r > m. For any rational number n, (1 + x) n = 1 + nx + x
1⋅ 2
Then,
(i) if m is an integer, then mth and ( m + 1)th terms are n( n − 1) ( n − 2) 3 n( n − 1) ( n − 2) … ( n − r + 1) xr
+ x + …+ +…
equal in magnitude and these two are the greatest 1⋅ 2⋅ 3 r!
terms. holds, whenever | x | < 1.
(ii) if m is not an integer, then obtain the integral part of
m, say k. In this case, ( k + 1) th term is the greatest Note For the expansion of
term. (i) (1 + x )− n replace n by −n,

Method 2. (ii) (1 − x )− n replace n by −n and x by −x


(iii) (1 + x )n replace x by −x in above expansion.
( n + 1) | x |
I. Find the value of m = .
a +| x|
General Term in the Expansion of (1 + x ) n
II. If mis an integer, then mth and ( m + 1)th terms are
General term in the expansion of (1 + x ) n is as follows
equal and both are greatest term.
[ n( n − 1 )( n − 2)…{ n − (r − 1 )}] r
III. If mis not integer, then obtain the integral part of m, T r+1 = x .
r!
say k. Thus, ( k + 1)th term is the greatest term.
Note • General term in the expansion of (1 − x )n is
EXAMPLE 5. Find the greatest term in the expansion n( n − 1)( n − 2 )… [n − ( r − 1)] r
Tr +1 = ( −1)r x .
of (1 + 2x) 9 , where x = 2/3. r!
• General term in the expansion of (1 + x )− n is
14330 1433614036 n( n + 1)( n + 2 ) + … + [n + ( r − 1)] r
a. b. c. d. None of these Tr +1 = ( −1)r x .
27 27 27 r!
• General term in the expansion of (1 − x )− n is
10 × 2  
2
(n + 1) 2x  3 40 5 n( n + 1)( n + 2 ) + … + [n + ( r − 1)] r
Sol. b. We have, m = = = = 5 ≠ Integer Tr +1 = x .
1+ 2x 1+ 2  
2 7 7 r!
 3 EXAMPLE 6. Find the coefficient of x 4 in the
The greatest term in the expansion is T5+ 1 = T6 expansion of (4 − 3x) −3/2 .
9 ⋅ 8 ⋅ 7 ⋅ 6  4 5 14336 76545 76445 76544 76542
∴Greatest term = T6 = 9 C 5( 2x) 5 =   = a. b. c. d.
4 ⋅ 3⋅ 2⋅ 1  3 27 786432 786423 786423 786432
Sol. a. We have,
Combinatorial Identities 3x −3 / 2 1  3x −3 / 2
n+1 n ( 4 − 3x) −3 / 2 = 4−3 / 2 1 −  = 1− 
1. n
C r + nC r − 1 = n+1
Cr 2. n+1
Cr + 1 = ⋅ Cr  4  8  4 
r +1 The general term is given by
3. n
C 0 + nC1 + nC 2 + . . . + nC n = 2n  − 3  − 3 − 1 …  − 3 − r + 1
4. n
C 0 − nC1 + nC 2 − . . . + ( − 1) n nC n = 0 1  2  2   2   3x r
Tr + 1 = ⋅ − 
8 1⋅ 2⋅ 3⋅…⋅ r  4
5. n
C 0 + nC 2 + nC 4 + . . . = 2n − 1
On putting r = 4, we get the coefficient of x as 4

6. n
C1 + nC 3 + nC5 + . . . = 2n − 1
2n !  − 3  − 3 − 1  − 3 − 2  − 3 − 3
7. C 0 2 + C1 2 + C 2 2 + . . . + C n 2 = = 2n
Cn 1  2  2   2   2   3 4
( n !) 2 ⋅ − 
8 1⋅ 2⋅ 3⋅ 4  4
n
Cr n−r +1 81 3 5 7 9 76545 76545
8. n
= = ⋅ ⋅ ⋅ ⋅ = =
C r−1 r 192 × 256 2 2 2 2 3072 × 256 786432
PRACTICE EXERCISE
1. The coefficient of x53 in the following expansion 10. If the rth, (r + 1)th and (r + 2)th coefficients of
100
(1 + x )n are in AP, then x is a root of the equation
∑ 100
Cm ( x − 3) 100 − m
⋅2
m
is
m=0 (a) x2 − x (4 r + 1) + 4 r 2 − 2 = 0
(a) 100
C 47 (b) 100
C 53 (c) − 100
C 53 (d) − 100
C100 (b) x2 + x (4 r + 1) + 4 r 2 − 2 = 0
2. The coefficient of x5 in the expansion of (c) x2 + x (4 r + 1) + 4 r 2 + 2 = 0
(1 + x ) 21
+ (1 + x ) 22
+ … + (1 + x ) 30
is (d) None of the above
51 9
(a) C5 (b) C5 11. The greatest term (numerically) in the expansion
(c) 31
C6 − 21
C6 (d) 30
C5 + 20
C5 1
of ( 3 − 5x )11, when x = is
5
3. The coefficient of x 4 in the expansion of (a) 55 × 39 (b) 46 × 39 (c) 55 × 36 (d) 46 × 36
(1 + x + x 2 + x3 )11 is
(a) 900 (b) 909 (c) 990 (d) 999 12. If in the expansion of (1 + x )m (1 − x )n , the
coefficients of x and x 2 are 3 and −6 respectively,
 1
n
−n
4. The coefficient of x in (1 + x ) 1 + n
 is then m is
 x
(a) 6 (b) 9
(a) 0 (b) 1 (c) 2 n (d) 2n (c) 12 (d) 24
5. What is the coefficient of x 4 in the expansion of 13. If the coefficient of the middle term in the
2
1 − x expansion of (1 + x )2n + 2 is p and the coefficients
  ?
1 + x of middle terms in the expansion of (1 + x )2n + 1
(a) − 16 (b) 16 (c) 8 (d) − 8 are q and r, then
n ( n + 1) 
2 (a) p + q = r (b) p + r = q
6. The sum of 1 + n 1 −  +
1 1
1 −  + L (c) p = q + r (d) p + q + r = 0
    x 2! x
will be 14. If the sum of the coefficients in the expansion of
n (α 2x 2 − 2αx + 1)51 vanishes, then the value of α is
(c) 1 −  (d) None of these
1
(a) xn (b) x−n
 x (a) 2 (b) −1
(c) 1 (d) −2
7. If (1 + x ) = C0 + C1x + C2x + … + Cn x ,
n 2 n
then
C0 +
C1 C2
+ +…+
Cn
is equal to
15. If (1 + x − 2x 2 )6= 1 + a1x + a2x 2 + K + a12x12, then
2 3 n +1 consider the following statements.
2 n+1
−1 2 n+1 I. a 2 + a 4 + a 6 + K + a12 = 31
(a) 2 n + 1 (b) (c) (d) 2 n+ 1 − 1
n+1 n+1 II. a1 + a3 + a5 + K + a11 = − 31

8. The term independent of x in the expansion of Which of the above statement(s) is/are correct?
(a) Only I (b) Only II
  1 
n
(1 + x )n 1 +    is (c) Both I and II (d) Neither I nor II
  x 
(a) C 02 + 2C13 + 3C 22 + … + (n + 1) C n2
16. If the independent term in the expansion of
10
 k
(b) C1 + C 2 + C 3 + … + C n  x + 2  is 405, then consider the following
(c) C 02 + C12 + C 22 + … + C n2
 x 
statements.
(d) C1 + 2C 2 + 3C 3 + … + nC n
I. The third term is independent of x.
9. What is the term independent of x in the II. The value of k is ± 3.
9
 3x −2 1 III. Total number of terms in the expansion is 10.
expansion of (1 + x + 2x )  −  ? 2
 2 3x Which of the above statement(s) is/are correct?
(a) 1/3 (b) 19/54 (a) I, II (b) II, III
(c) 1/4 (d) No such term exists in the expansion (c) I, III (d) I, II and III
MATHEMATICS Binomial Theorem 97

Directions (Q. Nos. 17-18) The last term in the 28. The value of m is
n log 3 8
 1  1 
binomial expansion of 21/3 −  is  1/3 
. (a) 10 (b) 11 (c) 12 (d) 13
 2 3 ⋅ (9)  3
29. The coefficient of x is
17. The value of n is (a) 140 (b) − 220 (c) 315 (d) 430
(a) 9 (b) 10 (c) 11 (d) 12 30. The value of independent term is
18. The value of 7th term from the end is (a) − 540 (b) − 624 (c) 350 (d) 495
(a) 420 (b) 240 (c) 210 (d) 120 31. The ratio of 6th term from the beginning and to
the 6th term from the end is
Directions (Q. Nos. 19-21) If x = ( a + 1) and 6
2 5 3 8 4 3
(a) x 6 (b) x (c) − x (d) − x
y = ( a − 1)6 , then 3 7 5

19. The number of terms in x + y is


(a) 2 (b) 3 (c) 4 (d) 5 PREVIOUS YEARS’ QUESTIONS
20. The number of terms in x − y is 32. The value of the term independent of x in the
3
(a) 1 (b) 2 (c) 3 (d) 4  1
expansion of  x 2 −  is
21. Find the value of x − y, if a = 2.  x e 2012 I
(a) 9 (b) 18 (c) 48 (d) 84
(a) 198 (b) 140 2
(c) 280 (d) None of these 33. In the expansion of (1 + x )n , what is the sum of
even Binomial coefficients? e 2012 II
Directions (Q. Nos. 22-24) The 2nd, 3rd and 4th (a) 2 n (b) 2 n−1
terms in the expansion of ( x + a) n are 240, 720 and (c) 2 n+ 1 (d) None of these
1080, respectively.
34. What is the sum of the coefficients in the
22. The value of ( x − a )n can be expansion of (1 + x )n ? e 2013 II
(a) 64 (b) − 1 (a) 2 n (b) 2 n − 1 (c) 2 n + 1 (d) n + 1
(c) − 32 (d) None of these 35. How many terms are there in the expansion of
23. The value of least term in the expansion is (1 + 2x + x 2 )10? e 2013 II
(a) 16 (b) 160 (c) 32 (d) 81 (a) 11 (b) 20 (c) 21 (d) 30
n
24. The sum of odd numbered terms is 36. What is ∑ C ( n ,r ) equal to?
(a) 1664 (b) 2376 (c) 1562 (d) 1486 r=0 e 2013 II
(a) 2 n − 1 (b) n (c) n! (d) 2 n
Directions (Q. Nos. 25-27) 37. If n be a positive integer and (1 + x )n = a0 + a1x
4n
Consider (1 + x + x 2) 2n = ∑ ar ⋅ x r , where + a2x 2 + ... + an x n , then what is
r=0 a0 + a1 + a2 + K+ an equal to? e 2013 II
a0, a1, a2, K a4 n are real numbers and n is positive (a) 1 (b) 2 n (c) 2 n − 1 (d) 2 n + 1
integer on the basis of above information, answer
the following questions. Directions (Q. Nos. 38-40) In the expansion of
n
25. The value of a2 is  3 1
 x − 2  , where n is a positive integer, the sum of
(a) 4n + 1
C2 (b) 3n + 1
C2 (c) 2n + 1
C2 (d) n+1
C2  x 
the coefficients of x 5 and x 10 is 0. e 2014 I
26. The value of a4n − 1 is
38. What is n equal to?
(a) 2n (b) 2 n2 + 4 n (c) 2 n + 3 (d) 2 n2 + 3 n
(a) 5 (b) 10 (c) 15 (d) None of these
27. The correct statement is 39. What is the value of the independent term?
(a) ar = an − r , 0 ≤ r ≤ n (b) an − r = an + r , 0 ≤ r < n (a) 5005 (b) 7200
(c) ar = a2 n − r , 0 ≤ r ≤ 2 n (d) ar = a4 n − r , 0 ≤ r ≤ 4n (c) –5005 (d) –7200

Directions (Q. Nos. 28-31) In the expansion of 40. What is the sum of the coefficients of the two
m middle terms?
 1
 x − 2  , the sum of first three terms is 55, where (a) 0 (b) 1
 x  (c) –1 (d) None of these
m is a natural number.
98 NDA/NA Pathfinder

Directions (Q. Nos. 41-45) Read the following Which of the above statement(s) is/are correct?
information carefully and answer the questions
(a) Only I (b) Only II
given below.
(c) Both I and II (d) Neither 1 nor 2
15
 1 10
46. In the expansion of  x +
Consider the expansion  x 2 +  . 1 
 x  the value of
e 2014 II  3x 2 
41. What is the independent term in the given constant term (independent of x) is e 2015 II
expansion? (a) 5 (b) 8 (c) 45 (d) 90
(a) 2103 (b) 3003 (c) 4503 (d) None of these
Directions (Q. Nos. 47-49) Read the following
42. What is the ratio of coefficient of x15 to the term information carefully and answer the questions
independent of x in the given expansion?
given below.
(a) 1 (b) 1/2 (c) 2/3 (d) 3/4
Consider the expansion of (1 + x ) 2n + 1. e 2015 II
43. What is the sum of the coefficients of the middle
terms in the given expansion? 47. The sum of the coefficients of all the terms in the
(a) C(15, 9) (b) C(16, 9) expansion is
(c) C(16, 8) (d) None of these (a) 2 2 n − 1 (b) 4n − 1
(c) 2 ⋅ 4n (d) None of these
44. Consider the following statements
I. There are 15 terms in the given expansion.
48. The average of the coefficients of the two middle
terms in the expansion is
II. The coefficient of x12 is equal to that of x3 . 2n+1 2n+1 2n+1 2n
(a) C n + 2 (b) Cn (c) C n − 1 (d) Cn + 1
Which of the above statement(s) is/are correct?
(a) Only I (b) Only II 49. If the coefficients of x r and x r + 1 are equal in the
(c) Both II and II (d) Neither 1 nor 2 expansion, then r is equal to
2n − 1 2n + 1
45. Consider the following statements (a) n (b) (c) (d) n + 1
2
2 2
I. The term containing x does not exist in the
given expansion. 50. The coefficient of x 99 in the expansion of
II. The sum of the coefficients of all the terms in (x − 1) (x − 2) (x − 3)... (x − 100) is e 2015 II
the given expansion is 215 . (a) 5050 (b) 5000 (c) − 5050 (d) − 5000

ANSWERS
1 c 2 c 3 c 4 b 5 b 6 a 7 b 8 c 9 d 10 a
11 a 12 c 13 c 14 c 15 a 16 a 17 b 18 c 19 c 20 c
21 b 22 b 23 c 24 c 25 c 26 a 27 d 28 c 29 b 30 d
31 a 32 d 33 b 34 a 35 c 36 d 37 b 38 c 39 c 40 a
41 b 42 a 43 c 44 b 45 c 46 a 47 c 48 b 49 a 50 c

HINTS AND SOLUTIONS 


1. (c) Given sigma expression can be 1 3. (c) ( 1 + x + x 2 + x 3 )11
= [( 1 + x )31 − ( 1 + x )21 ]
rewritten as x = [( 1 + x ) ( 1 + x 2 )]11
( x − 3 + 2)100 = ( x − 1)100 = ( 1 − x )100 ∴ Coefficient of x 5 in the given = ( 1 + x )11 ⋅ ( 1 + x 2 )11
53
So, x will occur in T . expansion = (11 C + 11C x + 11C x 2
54 0 1 2
∴ T = 100
C ( − x )53 = Coefficient of x 5 in + 11
C x3 + 11
C x 4 + …)
54 53 3 4
Hence, coefficient of x 53 is − 100
C .  1 {( 1 + x )31 − ( 1 + x )21 } (11 C + 11C x 2 + 11C x 4 + … )
 x 
53 0 1 2
2. (c) ( 1 + x )21 + ( 1 + x )22 +…+ ( 1 + x ) 30 ∴ Coefficient of x 4 in
 ( 1 + x )10 − 1 = Coefficient of x 6 in ( 1 + x + x 2 + x 3 )11
= ( 1 + x )21   [( 1 + x )31 − ( 1 + x )21 ] = C ⋅ C + C ⋅ 11C + 11C ⋅ 11C
11 11 11
 (1 + x ) − 1  = 990
0 2 2 1 4 0
  = 31
C − 21
C
6 6
MATHEMATICS Binomial Theorem 99

p =2n +
n n
4. (b) ( 1 + x )n  1 +  = ( 2 + x ) +
1 1 10. (a) Given, n C , nC r , nC are in AP. ∴ 2
C
r −1 r +1 n+1
 x  x  ∴ 2 ⋅ Cr = n n
C + nC In ( 1 + x ) 2n + 1
, index is 2n + 1, which
r −1 r +1
n 1 1 is odd.
= C (2 + x )
n
+ … + nC n n! n!
0
x xn ⇒2⋅ =
r !( n − r )! ( r − 1)!( n − r + 1)! So, there are two middle terms given by
∴ Coefficient of x − n = nC n = 1 n!
T
n+1
and T .
n+ 2
+ 2n + 1
5. (b) ( 1 − x )2 ( 1 + x )−2 = [ 1 − 2x + x 2 ] ( r + 1)!( n − r − 1)! ∴ q= C n and r = 2 n + 1C
n+1
2 n +1
[ 1 − 2x + 3x 2 − 4x 3 + 5 × x 4 + ...] 2 1 1 But C n + 2 n +1 C = 2 n + 2C
⇒ = + n +1 n+1
∴ Coefficient of x 4 r( n − r ) ( n − r + 1)( n − r ) ( r + 1)r ∴q + r = p
= 1 × 5 + ( −2 ) ( −4 ) + 1 × 3 2 2r 2 + n2 − 2nr + n 14. (c) (α 2 x 2 − 2αx + 1)51 = [( 1 − αx )2 ]51
⇒ =
= 5 + 8 + 3 = 16 r( n − r ) r ( r + 1)( n − r + 1)( n − r ) = ( 1 − αx )102
2 ⇒ n − n ( 4r + 1) + 4r − 2 = 0
2 2 = A + A x + A x 2 + … [say]
n ( n + 1) 
6. (a) 1 + n 1 −  +
1 1 0 1 2
 1 −  + ... So, x is a root of the equation Put x = 1, we get
 x  21  x
−n −n x 2 − x ( 4r + 1) + 4r 2 − 2 = 0. ( 1 − α )102 = A + A + A + … = 0
 1 
=  1 −  1 −   = 
1 0 1 2
= ( x )n 11
  x   x  11 
5x  [given]
11. (a) We have,(3 − 5x ) 11
= 3 1 − 
 3  ⇒ 1− α = 0 ⇒α = 1
C C Cn
7. (b) C + 1 + 2 + …+ 11
= 3  1 −  11  Q x = 1 15. (a) Given, ( 1 + x − 2x 2 )6
1
0
2 3 n+ 1 
 3 5  = 1 + a x + a x 2 + K + a x 12
n n( n − 1) 1 |x |( n + 1)
1 2 12
= 1+ + + …+ ∴ m= Q − 1 < 0 Putting x = 1, we get
2 6 n+ 1 ( | x | + 1)  
3 0 = 1+ a + a + a + K+ a …(i)
1 2 3 12
( n + 1) + ( n + 1) n   1 
Putting x = − 1, we get
1  2   −  ( 11 + 1) 64 = 1 − a + a − a + K + a …(ii)
=    3 
n + 1  + ( n + 1) n ( n − 1) +…+ 1 = =3 1 2 3 12
On adding Eqs. (i) and (ii), we get
 1 
 3!   −  + 1 64 = 2( 1 + a + a + K + a )
 3  2 4 12
1 n +1 n +1 n +1
= ( C + C + …+ C ) ∴ a + a + a + K + a = 31
n+ 1 1 2 n +1 The greatest terms in the expansion are 2 4 6 12
T and T . Thus, Statement I is correct.
2n + 1 − 1 3 4
= ∴ Greatest term (when r = 2) Subtracting Eq. (ii) from Eq. (i), we get
n+ 1
= 3 |T 11
| − 64 = 2( a + a + a + K + a )
1 3 5 11
8. (c) ( 1 + x )n[ 1 + x −1 ] n 2+1
2 ∴ a + a + a + K+ a = − 32
C  − 
1 1 3 5 11
= [ n C + n C x + n C x 2 + ... + n C x n ] = 311 11
= 55 × 39 Thus, Statement II is incorrect.
2
0 1
−1
2 n
3
[nC + n C x + n C x −2 + ... + n C x − n ] 16. (a) General term of the expansion
0 1 n
∴ Term independent of x
2
12. (c) ( 1 + x )m ( 1 − x )n 10
 x + k  is
m (m − 1) 2  
=  1 + mx + ⋅ x + …  x 
2
= n C × nC + n C × nC + n C × nC
0 0 0 0 2 2  2  r
= 10C r ( x )10 − r  2 
+ ...n C n × n C n k
 n ( n − 1) 2 T
×  1 − nx + ⋅ x + … r +1 x 
= C 2 + C 2 + C 2 + ... + C n2  2  10 − r
0 1 2
Coefficient of x = 10
C (x ) 2 (k)
r
⋅ ( x )− 2 r
9. (d) Let T be the term independent of r`
r +1
= (m − n) = 3 [given] …(i) 10 − 5 r
 3 x −2 1
9
= 10
Cr ( k )r ( x ) 2 …(i)
x in  −  . Coefficient of x 2
 2 3x  m (m − 1) n ( n − 1) Since, ( r + 1)th term is independent
9−r
= − mn +
of x.
 3 x −2  2 2
r
∴T = 9C r  − 1  10 − 5r
   = −6 [given] ∴ = 0⇒r = 2
r +1
 2   3x  2
9−r ⇒ m 2 − m − 2mn + n2 − n = − 12
Thus, 3rd term is independent of x.
= ( −1) C r  
3 1
r 9
⋅ r x −18 + 2 r − r ⇒ m 2 + n2 − 2mn − (m + n) = − 12
 2 3 So, Statement I is correct.
⇒ (m − n)2 − (m + n) = − 12
For coefficient of x 0 , x −1 and x −2 , Put r = 2 in Eq. (i), we get
we get ⇒ (3)2 − (m + n) = − 12
T = 10
C k 2 ⇒ 405 = 45k 2
− 18 + r = 0, − 18 + r = − 1 ⇒ m + n = 21 …(ii) 3 2

and − 18 + r = − 2 ⇒ r = 18, 17, 16 ⇒ k = 9⇒k = ± 3


2
On solving Eqs. (i) and (ii), we get
which is not possible. Thus, Statement II is correct.
m = 12
and total 10 number of terms in
Thus, no such term exists in the 2n + 2
13. (c) In ( 1 + x ) , index is 2n + 2,  x + k  is 11.
expansion of given expression.  
which is even, therefore there is one  x2 
middle term given by T .
n+ 2 So, Statement III is not correct.
100 NDA/NA Pathfinder

r
17. (b) The last term of 22. (b) ( x − a )n = ( 2 − 3)5 = − 1 29. (b) T = mC r ( x )m − r  − 2 
1
n
3 2 − 1  =  1  3
log 8
23. (c) ( 2 + 3)5 = 25 + 5C 24 × 3 + 5C 23 ×
r +1  x 
   3  1 2
 2 3⋅ 9  3 + C 2 ×3 + C 2×3 + C 3
2 5 2 3 5 4 5 5 = mC r ( − 1)r ( x )m − 3 r
n log 8 3 4 5
= 32 + 240 + 720 + 1080 + 810 + 243 C r ( − 1)r ( x )12 − 3 r
C n ⋅  −
1   1  3 = 12
[Q m = 12]
⇒ n
 = 3 
 2 3⋅ 9  Hence, least value of the term is 32. This general term will contain x 3 , if
n/ 2 log 8
n  1 
=  5 / 3 
1 3
24. (c) Sum of odd numbered terms is 12 − 3r = 3 ⇒ r = 3
⇒ ( − 1) ⋅  
 2 3  32 + 720 + 810 = 1562
5 5 ∴ T = C ( − 1)3 x 12 − 9
12
4 3
=
− ⋅3 log 2
3 3 3 =2 −5
=  
1 25. (c) a = Coefficient of x 2 in 12 × 11 × 10 3
 2
2
=− x = −220x 3
( 1 + x + x 2 )2 n 3×2
⇒ n = 10 = Coefficient of x in 2
∴ Coefficient of x 3 = − 220
18. (c) 7th term from the end [1 + 2n
C (x + x ) 2
= T = T = 5th term 1 30. (d) T will contain independent
r +1
10 − 7 + 2 5 10 + 2n
C ( x + x 2 )2 + K ]
Therefore, 5th term in  3 2 −
1  2 term if 12 − 3r = 0 ⇒ r = 4
 is = 2n
C + 2n
C
 2
4
1 2
∴ T = 12
C ( − 1)4 ( x )0
5 4
10 − 4  1  = 2n + 1
T =T = 10
C ( 2) 3
−  C ∴ Coefficient of independent term
5 4+1 4  2
2
4n
12 × 11 × 10 × 9
26. (a) ( 1 + x + x ) = ∑ ar x
2 2n r
1 = 12C = = 495
= 10
C ⋅ ( 4) ⋅ = 10
C = 10
C = 210 r =0
4
4×3× 2
4 4 6
4
Replace x by 1 / x, we get 31. (a) 6th term from the beginning = T
19. (c) Number of terms in r
( 1 + x + x 2 )2 n 4n 6
= ∑ ar  
1
x + y or ( a + 1)6 + ( a − 1)6 6th term from the end
6
2 2n
(x ) x
is equal to + 1 = 4 r =0 =T =T
4n 12 − 5 + 1 8
2
∑ ar ( x ) × ( x )
−r
⇒ ( 1 + x + x 2 )2 n = 4n
12
C ( − 1)5 ( x )12 − 15
20. (c) Number of terms in x − y T
6
r =0 ∴ 6 = 5
or ( a + 1)6 − ( a − 1) 6 is equal to = 3. 4n T 12
C ( − 1)7 ( x )12 − 21
∑ ar ( x )
4n − r
2 = 8 7

21. (b) x − y = ( a + 1)6 − ( a − 1)6 r =0 − 12


C ( x )− 3
7
4n 4n = = x6
∑ ar x ∑ ar ( x ) C ( x )− 9
= 2 ( 6C a 5 + 6C a 3 + 6C a ) 4n − r
1 3 5
∴ r
= − 12
7
r =0 r =0
= 2 ( 6 a 4 + 6C a 2 + 6C ) a 9
32. (d) Given,  x 2 − 
3 5
On comparing coefficients, we get 1
= 2 ( 6 × 4 + 20 × 2 + 6) 2  x
[putting a = 2] ar = a r
= 9C r ( x 2 )9 − r ⋅  −  ,
4n − r 1
= 2 2 ( 24 + 40 + 6) = 140 2 ∴ a =a T
r +1  x
4n − 1 1

Sol. (Q. Nos. 22-24) = Coefficient of x in ( 1 + x + x 2 )2 n = 9C r x 18 − 2 r ( −1)r x − r


2nd term is nC x n − 1 a = 240 …(i) = 2n
C = 2n
1 = 9C r x (18 − 3 r )( −1)r …(i)
1
3rd term is n C x n − 2 a 2 = 720 …(ii) 27. (d) For 0 ≤ r ≤ 4n, ar = a For independent term,
2 4n − r
4th term is n C x n − 3 a 3 = 1080 …(iii) m Put 18 − 3r = 0 ⇒ 3r = 18 ⇒ r = 6
28. (c)  x − 2 
3 1
(18 − 18 )
On multiplying Eqs. (i) and (iii) and  x  ∴ T = 9C x ⋅ ( −1)6
(6 + 1 ) 6
dividing by the square of Eq. (ii), we get −1 9 ⋅ 8 ⋅7
n
C × nC = mC x m + m
C x m − 1  2  T = 9C ⋅ 1 = = 84
3 = 240 × 1080 x  3⋅ 2⋅ 1
0 1 7 6
1
( n C )2 ( 720)2 2 m 33. (b)
−1 −1
+ m C x m − 2  2  + ... + C m  2 
2 m
n × n ( n − 1)( n − 2)( 2!) 2
1 2 x  x  34. (a) For the sum of the coefficients in
⇒ =
n2 ( n − 1)2 × 3! 2 the expansion of ( 1+ x )n .
= mC x m − mC x m − 3 + m
C xm − 6 Put x = 1, ( 1+ x )n = ( 1+ 1)n = 2n
⇒ 4( n − 2) = 3( n − 1) [Q n ≠ 1] 0 1 2
+ K mC m ( x )− 2 m
⇒ n=5 which is the required sum of the
On putting n = 5, from Eqs. (i) and (ii), Sum of coefficient of first three terms coefficients.
we get = 55 35. (c) Given, ( 1 + 2x + x2 )10 = {( 1 + x)2 }10
5x 4 a = 240 and 10x 3 a 2 = 270 ⇒ m
C − C + m m
C = 55 = ( 1 + x)20
0 1 2
(5x 4 a )2 ( 240)2 m(m − 1) ∴ Total terms = 20 + 1 = 21
⇒ = ⇒ x 5 = 32 ⇒ 1−m + = 55
10x 3 a 2 720 2 36. (d) 37. (b)
∴ x =2
⇒ m 2 − 3m − 108 = 0 Sol. (Q. Nos. 38-40)
240 48
∴ a = 4 = 4 =3
⇒ (m − 12) (m + 9) = 0 n
In the expansion of  x 3 − 2 
5x 2 1
Hence, x = 2, a = 3 and n = 5 ⇒ m = 12 [Q m ∈ N ∴m + 9 ≠ 0]  x 
MATHEMATICS Binomial Theorem 101

15
=− 15
C + 15
II. Now,  x 2 + 
General term, C 1
7 8
 x
r
T = nC r ( x 3 )n − r ⋅  2 
1 =− 15
C + 15
C [Q C r = C n − r ]
n n
7 7
r +1 x  =0 C ( x 2 )14  
1
= 15
C ( x 2 )15 + 15

= nC r ⋅ ( − 1)r ⋅ x (3 n − 5 r ) …(i) 15 0 1 x


41. (b) We have,  x 2 + 
1
Let T and T be the term  x  1
15
r +1 r +1 r + K+ 15
C  
1
containing x 5 and x
2
10
respectively. T = 15C r ( x 2 )15 − r  
1 15 x
r +1 x
Put x = 1 on both sides, we get
For coefficient of x , 3n − 5r = 5 = 15C r x 30 − 2 r − r = 15C r x 30 − 3 r
5
1 ( 1 + 1)15 = 15
C + 15
C + K+ 15
C
3n For independent term, 0 1 15
⇒ r = −1 …(ii) ⇒ 2 15
= 15
C + 15
C + K+ 15
C
1
5 30 − 3r = 0 ⇒ r = 10 0 1 15
Hence, Statement II is correct.
For coefficient of x 10 , 3n − 5r = 10 Put r = 10, we get
2 10
46. (a) We have,  x + 2 
3n 15! 1
⇒ r = −2 …(iii) T = 15C = = 3003
2
5 10 + 1 10
10!5!  3x 
10 − r
From Eqs. (ii) and (iii), we have 42. (a) For coefficient of x 15 , ∴T = 10
C r(x ) 2 (3 ) − r ( x ) − 2 r
r+ 1
r = r −1 …(iv) 30 − 3r = 15 ⇒ r = 5
2 1 10 − 5 r
So, the coefficient of x 15 is C r (3 ) − r x
15
Now, by condition the sum of C = 10
2
5
coefficients of x 5 and x 10 = 0 and coefficient of independent of x is
r r
2 =0
Since, the term independent of x
⇒ n
C r ( −1) 1
+ C r ( −1)
n 30 – 3 r = 0 ⇒ r = 10
10 − 5r
1 2
So, coefficient of independent of x is ∴ = 0⇒r = 2
r r −1 2
⇒ n
C r ( −1) 1
+ nC r ( −1) 1
=0 15
C .
1 2 10
15 Put r = 2, we get
r C
⇒ ( −1) 1 [ n C r − nC r ] = 0 ∴ Required ratio = 15 5 10 × 9 1
T = 10
C (3 ) − 2 = × = 5
1× 2
1 2 3 2
C 9
10
⇒ n C r = nC r ⇒ n = r + r 15
C
1 2 47. (c) To find the sum of coefficient of all
1 2
= 15 5 = 1 [Q nC r = n C n – r ]
[Q C x = C y ⇒ n = x + y ]
n n C terms, put x = 1 in the given expression
( 1 + x )2 n + 1 , we get
5
3n 3n 15
⇒ n= − 1+ − 2 ⇒ n = 15 43. (c) We have,  x 2 + 
1
5 5 22 n + 1 = 2 ⋅ 22 n = 2 ⋅ 4n
 x
38. (c) 48. (b) Since, 2n + 1 is odd.
Since, n is odd.
2n + 1 + 1
39. (c) For the independent term, So, it has two middle terms T and T . Hence,
8 9 2
put 3n − 5r = 0, [from Eq. (i)] Coefficients of (T + T ) =15 C + 15 C 2n + 1 + 3
⇒ 5r = 3n = 3 × 15 [Qn = 15]
8 9 7
n+1
8 and are two middle terms.
= 16
C [Q n C + nC r = Cr ] 2
8 r −1
∴ r = 9
= C ( 16, 8) i.e. ( n + 1) th and ( n + 2) th terms are
Now, put the value of r in Eq. (i), two middle terms.
we get 44. (b)I. We know that, ( a + b ) have total n
2 n +1
number of terms is n + 1. C n + 2 n +1 C
( 3 × 15 − 5 × 9 ) ∴ n +1
T = 15
C ( − 1) ⋅ x 9
15
9+1
So,  x 2 + 
9 1 2
15! have 16 terms. 2 n +1 +1
⇒ T = −15C ⋅ x 0 = − 15
C =−  x C
n +1 = 1 2 n + 2 C
10 9 9
6! 9! =
n+ 1
So, Statement I is false. 2 2
= − 5005 1 2n + 2 2 n + 1 2n + 1
II. For coefficient of x 12 , = ⋅ ⋅ Cn = C
So, the value of independent term is 2 n+ 1 n

− 5005. 30 − 3r = 12 ⇒ r = 6
49. (a) We have, ( 1 + x )2 n + 1
40. (a) Since, n = 15 ∴ Coefficient of x 12 = 15
C
6
General term, T = (2 n + 1 ) C r x r
∴ Total term in the expansion of and for coefficient of x 3 , r +1
15 2n + 1 2n + 1
 x 3 − 1  is 16. 30 − 3r = 3 ⇒ r = 9 We have, Cr = C
  r +1
 x 
2
∴ Coefficient of x 3 = 15
C Q 2n + 1 = r + r + 1
9
So, middle term =   th term ⇒ 2r = 2n ⇒ r = n
16 Q 15
C = 15
C
 2 6 9
50. (c) We have,
Hence, Statement II is correct.
and  + 1 th term
16
45. (c) I. For coefficient of x 2 , ( x − 1) ( x − 2) ( x − 3) ... ( x − 100)
 2 
28
30 − 3r = 2 ⇒ r = , r ∉ N = x 100 − ( 1 + 2 + 3 + ... + 100) x 99 + ...
= 8th term and 9th term 3
Now, the sum of the coefficients of the So, x 2 does not exist in the ∴ Coefficient of x 99
two middle terms expansion. 100
=− [ 2 + 99] = − 5050
Hence, Statement I is correct. 2
= 15
C ( −1)7 + 15
C ( −1)8
7 8
09
102 CDS Pathfinder

LOGARITHM
In NDA exam, 1 question can be asked from this chapter which is based on properties of
logarithm.

If a is positive real number other than 1 and x is a rational number such that a x = N, then we say that
logarithm of N to base a is x, written as log a N = x. Thus, a x = N ⇔ log a N = x, where a > 0, a ≠ 1 and
N > 0.
e.g. 10 2 = 100 ⇒ log 10 100 = 2
It is also known as fundamental logarithmic identity. When base is ‘e’ then the logarithmic function is
called natural or Napierian logarithmic function and when base is 10, then it is called common
logarithmic function.
Note• e is the base of natural logarithm (Napier logarithm) log x = loge x.
• Log of negative integers are not defined, loge 0 is not defined.
• Logarithmic function is positive as well as negative but exponential function is always positive.
• The base of a logarithm is never taken as 0, negative number and 1.

Properties of Logarithm
Let m and n be positive number such that a, b, c > 0 and a, b, c ≠ 1
(i) log a a = 1 (ii) log a 1 = 0 (iii) log c a = log b a ⋅ log c b
log b a
(iv) log c a = (v) log b a ⋅ log a b = 1 (vi) log a ( m ⋅ n) = log a m + log a n
log b c
 m p
(vii) log a   = log a m − log a n (viii) log a mn = n log a m (ix) log q ( m p ) = log n m
 n n q
(x) a loga n = n (xi) a loge x = x loge a , x > 0, e > 0
(xii) log a b = − log 1/a b

EXAMPLE 1. What is the value of (log13 625)/(log169 25) ?


1
a. b. 1 c. 2 d. 4
4
log13 625 log 625 log 25 4 log 5 2 log 5 4 log 5 2 log 13
Sol. d. We have, = ÷ = ÷ = × =4
log169 25 log 13 log 169 log 13 2 log 13 log 13 2 log 5
MATHEMATICS Logarithm 103

EXAMPLE 2. If (log 3 x) 2 + log 3 x < 2, then which one Logarithmic Inequalities


of the following is correct? 1. If a > 1, p > 1 ⇒ log a p > 0
1 1 1
a. 0 < x < b. < x < 3 c. 3 < x < ∞ d. ≤ x ≤ 3 2. If 0 < a < 1, p > 1 ⇒ log a p < 0
9 9 9
3. If a > 1, 0 < p < 1 ⇒ log a p < 0
Sol. b. Q (log 3 x) 2 + log 3 x < 2 ⇒ (log 3 x ) 2 + (log 3 x ) − 2 < 0
4. If p > a > 1 ⇒ log a p > 1
⇒ (log 3 x + 2) (log 3 x − 1) < 0 ⇒ −2 < log 3 x < 1
1 5. If a > p > 1 ⇒ 0 < log a p < 1
⇒ 3−2 < x < 31 ⇒ <x<3
9 6. If 0 < a < p < 1 ⇒ 0 < log a p < 1
7. If 0 < p < a < 1 ⇒ log a p > 1
Characteristic and Mantissa
 a > mb , if m > 1
In log 10 N the integral part of N is called the 8. If log m a > b ⇒ 
 a < m , if 0 < m < 1
b
characteristic and decimal part of N is called the
mantissa.  a < mb , if m > 1
9. If log m a < b ⇒ 
(i) (a) If N > 1, then the characteristic of log 10 N is one  a > m , if 0 < m < 1
b

less than the number of digits in integral part of N.


10. log p a > log p b ⇒ a ≥ b if base ‘ p’ is positive and > 1
(b) If 0 < N < 1 , then the characteristic of log 10 N is
or a ≤ b if base p is positive and < 1, i.e. 0 < p < 1.
one greater than the number of zeros
In other words, if base is greater than 1 then
immediately after the decimal point and the first
inequality remains same and if base is positive but
significant digit and is a negative integer.
less than 1, then the sign of inequality is reversed.
(ii) Insert decimal point in antilog of a number
11. For φ( x) > 1, log φ (x) f ( x) ≥ log φ (x) g( x)
(a) When characteristic is n, then insert the decimal ⇔ f ( x) ≥ g( x) > 0
point after ( n + 1)th digit.
12. For 0 < φ( x) < 1, log φ (x) f ( x) ≥ log φ (x) g( x)

(b) When characteristic is n, then insert the decimal ⇔ 0 < f ( x) ≤ g( x)
point such that the first significant digit is at nth
13. For φ( x) > 1, log φ (x) f ( x) ≥ a ⇔ f ( x) ≥ a φ (x)
place.
14. For 0 < φ( x) < 1, log φ (x) f ( x) ≥ a ⇔ 0 < f ( x) ≤ a φ (x)
EXAMPLE 3. If 2log 8 N = p, log 2 2N = q and q − p = 4,
then find the value of N? EXAMPLE 5. If log 0. 04 ( x − 1) ≥ log 0. 2 (x + 1), then x
a. 512 b. 536 c. 548 d. 560 belongs to the interval
a. (1, 2] b. ( −∞ , 2] c. [ 2, ∞) d. None of these
Sol. a. We are given that, 2log 8 N = p …(i)
log 2 2N = q …(ii) Sol. c. log 0. 04 ( x − 1) ≥ log 0. 2 ( x − 1) …(i)
and q− p=4 …(iii) For log to be defined x − 1 > 0 ⇒ x > 1
From Eq. (i), 8p = N 2 ⇒ 23p = N 2 …(iv) From Eq. (i), log ( 0. 2) 2 ( x − 1) ≥ log 0. 2 ( x − 1)
2 q 1
From Eq. (ii), 2N = 2q ⇒ N = = 2q − 1 …(v) ⇒ log 0. 2 ( x − 1) ≥ log 0. 2 ( x − 1) ⇒ x − 1 ≤ ( x − 1)
2 2
From Eqs. (iv) and (v), 23p = ( 2q − 1) 2 ⇒ x − 1 (1 − x − 1) ≤ 0 ⇒ 1− x − 1 ≤ 0
⇒ 23p = 22q − 2 ⇒ 3p = 2q − 2 ⇒ 2q − 3p = 2 …(vi) ⇒ x − 1 ≥ 1 ⇒ x ≥ 2 ∴ x ∈[ 2, ∞)
On solving Eqs. (iii) and (vi), we get p = 6 and q = 10
EXAMPLE 6. If log e (x 2 − 16) ≤ log e (4x − 11), then
∴ N = 210 − 1 = 29 = 512
a. 4 < x ≤ 5 b. x < − 4 or x > 4
EXAMPLE 4. If log10 2 = 0. 3010, then what is the c. −1 ≤ x ≤ 5 d. x < − 1or x > 5
number of digits in 20 64 ?
Sol. a. x − 16 ≤ 4x − 11
2
[base = e > 1]
a. 81 b. 82 c. 83 d. 84
⇒ x − 4x − 5 ≤ 0 ⇒ ( x − 5) ( x + 1) < 0 ⇒ −1 ≤ x ≤ 5 …(i)
2
Sol. d. Let x = 2064, Taking log on both sides, we get
Also, x 2 − 16 > 0 ⇒ x < − 4 or x > 4 …(ii)
log10 x = 64 log10 20 = 64 × 1.3010 = 83. 264 11
And 4x − 11 > 0 ⇒ x > …(iii)
∴ Number of digits in 2064 = 83 + 1 = 84 4
From Eqs. (i), (ii) and (iii), we get 4 < x ≤ 5
104 NDA/NA Pathfinder

PRACTICE EXERCISE
1. If log10 2 = 0.30103, then log10 50 is equal to 12. If log13/ [log4 ( x 2 − 12)] > 0, then x lies in the
interval
(a) 2.30103 (b) 2.69897 (c) 1.69897 (d) 0.69897
(a) (−∞, − 4) ∪ (4, ∞ ) (b) (− ∞, ∞ )
log8 17 log2 2 17 (c) (− ∞, − 2 ) ∪ (2, ∞ ) (d) None of these
2. − is equal to
log9 23 log3 23 13. If loga ( n − k) < loga 2 ( n − k) and a > 1, then n lies
17 23
(a) 0 (b) 1 (c) (d) in the interval
8 17
(a) [k, k + 1] (b) (k, k − 1)
3. The value of ( yz )log y − log z × ( zx )log z − log x (c) (k, k + 1) (d) None of these

× ( xy )log x − log y is equal to 14. If x ∈ [− 2π , 2π ] and log0.5 sin x = 1 − log0.5 cos x, then
(a) 2 (b) 1 (c) 4 (d) 3 consider the following statements
I. Number of solutions in the interval [− 2π , 2π ] is 1.
4. The value of 811/ log5 3 + 27log 9 36 + 34/ log7 9 is equal
II. One solution of the equation is π /4.
to
(a) 49 (b) 625 (c) 216 (d) 890 Which of the above statement(s) is/are correct?
(a) Only I (b) Only II
5. If a = log24 12, b = log36 24, c = log48 36. Then (c) Both I and II (d) Neither I nor II
1 + abc is equal to
(a) 2 ac (b) 2 bc Directions (Q. Nos. 15-16) Let a, b, c ∈ R; a ≠ b ≠ c
log x log y log z
(c) 2 ab (d) None of these and = =
b −c c − a a− b
6. log10 tan 1°+ log10 tan 2° + … + log10 tan 89° is
equal to
15. The value of x a ⋅ y b ⋅ z c is
(a) 0 (b) 1 (c) 27 (d) 81 (a) 0 (b) 1 (c) 2 (d) None of these
1 1 1
7. If log12 27 = a , then log6 16 is equal to 16. The value of + +
logxy 2 log yz 2 logzx 2
3−a 3−a
(a) 2 ⋅ (b) 3 ⋅
3+ a 3+ a (a) 0 (b) 1 (c) 2 (d) None of these
3−a
(c) 4 ⋅ (d) None of these Directions (Q. Nos. 17-19) Consider the equation
3+ a
log 5 x 2 + (log 5 x )2 − 12 1
1 1 2 x =
8. If + = , then a , b and c are in x4
loga x logc x logb x
17. The number of solutions of the equation is
(a) AP (b) GP
(a) 1 (b) 2
(c) HP (d) None of these
2 (c) 3 (d) 4
log x + log x 4 + log x 9 + … + log x n
9. is equal to 18. The set of all x satisfying the equation is
log x + log x 2 + log x 3 + … + log x n
(a) 1, 25,
1 
(b) 1, 25,
1 1 
2n + 1 2n − 1
,  
(a) (b)  125 625   625 
3 3 (c) {1, 25} (d) {1}
3 (n + 2 ) 3 (n − 1)
(c) (d)
2 2 19. The product of all values of x is
(a) 1 (b) 25
10. If x = y = z , then 3, 3 log y , 3 logz y, 7 logx z
18 21 28 x
1 1
(c) (d)
are in 25 3125
(a) AP (b) GP
(c) HP (d) None of these
PREVIOUS YEARS’ QUESTIONS
11. If x = log3 5, y = log17 25, then which one of the 1
following is correct? 20. What is the value of 2 log8 2 − log3 9?
3 e 2012 I
(a) x < y (b) x = y
(a) 0 (b) 1 (c) 2 (d) 1/3
(c) x > y (d) None of these
MATHEMATICS Logarithm 105

21. If (log3 x) ( logx 2x ) ( log2x y ) = logx x 2 , then what is 24. If log8 m + log8
1 2
= , then m is equal to
the value of y? e 2012 II 6 3 e 2015 II
(a) 4.5 (b) 9 (c) 18 (d) 27 (a) 24 (b) 18 (c) 12 (d) 4
22. What is the value of log y x 5 logx y 2 logz z3 ? 25. If loga ( ab) = x, then what is logb( ab) equal to?
e 2013 I e 2016 I
(a) 10 (b) 20 (c) 30 (d) 60 1 x
(a) (b)
23. What is log81 243 equal to? x x+1
e 2013 II x x
(a) 0.75 (b) 1.25 (c) 1.5 (d) 3 (c) (d)
1− x x −1

ANSWERS
1 c 2 a 3 b 4 d 5 b 6 a 7 c 8 b 9 a 10 a
11 c 12 a 13 c 14 b 15 b 16 a 17 c 18 b 19 c 20 a
21 b 22 c 23 b 24 a 25 d

HINTS AND SOLUTIONS 


100 6. (a) log tan 89° = log cot 1° = − log tan 1° 9. (a) Given expression
1. (c) log 50 = log
10 10
2 ∴ Given expression becomes ( 1 + 4 + 9 + … + n2 ) log x
=
= log 100 − log 2 log tan 1°+ log tan 2°+ … + log tan 44° ( 1 + 2 + 3 + … + n) log x
10 10
= 169897
. + log tan 45°− log tan 44° n ( n + 1)( 2n + 1)
1 2 − … − log tan 2° − log tan 1° Σn2 6
log 17 log 17 = =
3 2
3 2 = log tan 45° = log 1 = 0 Σn n ( n + 1)
2. (a) − =0
1 log 23 4 2
log 23 7. (c) log 16 = log 24 = 4 log 2 =
2 3
3
2n + 1
6 6 6
log 6 =
2 2 2
3
log 17 log 17 4 4
⇒ 3 2
− 3 2
=0 = = …(i) 10. (a) Let x 18 = y 21 = z 28 = k, then
log 23 log 23 log 2 + log 3 1 + log 3
3 3
2 2 2
18 log x = 21log y = 28 log z = log k
and a = log 27 = log 33 = 3 log 3 ⇒ log x =
log x
, log y =
log k
3. (b) 12 12 12
3 3 18 21
4. (d) Let T = 34 log 3 5 = 3log 3 (5 ) =54 = 625
4
= = log k
1 log 12 log 3 + log 4 and log z =
2 3 3 3 28
log 6 3
T = (3 3 ) 2 = 3log 3 6 = 216 3
3
= 3 log x 3 × 21 7
2
1 + 2 log 2 ∴ 3 log y x = = =
1 log y 18 2
4⋅ ⋅ log 7 3
T = 34 log 9 7 = 3 2 3
3 ⇒ a + 2a log 2 = 3 3 log y 3 × 28
3 3 log z y = = =4
3−a
2
= 3log 3 7 = 49 ⇒ log 2 = log z 21
3
∴T +T +T 2a 7 log z 7 × 18 9
1 2 3 2a 7 log x z = = =
= 625 + 216 + 49 = 890 ∴ log 3 = log x 28 2
2
3− a
log 12 log 24 log 36 log 12 7 9
5. (b) abc = ⋅ ⋅ = From Eq. (i), So, 3, , 4, are in AP.
log 24 log 36 log 48 log 48 2 2
4
log 48 + log 12 log 16 =
∴ 1 + abc = 6 2a 11. (c) Given, y = 2 log 5 and x = log 5
log 48 1+ 17 3
3−a 1 1
log ( 48 ⋅ 12) log 24 2
4(3 − a ) ∴ = log 17
= = = y 2 5
log 48 log 48 3+ a 1 1
and = log 3 = log 9
= 2⋅
log 24 log 24 log 36
= × 8. (b) log x a + log x c = 2 log x b x 5 2 5
log 48 log 36 log 48 ⇒ ac = b 2 1 1
Clearly, > ∴ x> y
= 2 log 24 × log 36 = 2bc i.e. a , b and c are in GP. y x
36 48
106 NDA/NA Pathfinder

12. (a) Given expression is Sol. (Q. Nos. 17-19) log y 2 log x
⇒ = [Q log a b = b log a ]
1
log x 2 + (log x ) 2 −12 log 3 log x
log [log ( x − 12)] > 0
2
We have, x 5 5 = 4
1/3 4 x ⇒ log y = 2 log 3
Q x − 12 > 4
2
Taking log to the base 5 of both the
⇒ x 2 − 16 > 0 ⇒ log y = log 32
sides, we get
⇒ | x |> 4 [Q log m = log n ⇒ m = n]
[log x 2 + (log x )2 − 12]log x
⇒ x < − 4 or x > 4 5 5 5 ⇒ log y = log 9
So, x lies in ( − ∞ , − 4) ∪ ( 4, ∞ ). = − 4 log x ∴ y=9
5

13. (c) log a ( n − k ) < log ( n − k ) ⇒ [log x 2 + (log x )2 − 8] log x 22. (c) log y x 5 ⋅ log x y 2 ⋅ log z z 3
5 5 5
a2
⇒ ( n − k )2 < n − k [Q a > 1] =0 = 5 log y x ⋅ 2 log x y ⋅ 3 log z z
⇒ n2 − 2kn + k 2 < n − k ⇒ log x = 0 [Q log a b n = n log a b ]
5
⇒ n2 − kn − ( k + 1) n + k ( k + 1) < 0 or (log x )2 + 2 log x − 8 = 0 = 5 log y x ⋅ 2 log x y ⋅ 3 × 1 [Q log a a = 1]
5 5
⇒ ( n − k ) ( n − k − 1) < 0 ⇒ x =5 0 log x log y  log b 
=5 ⋅ × 2⋅ × 3 Q log a b =
⇒ k < n< k + 1 log y log x  log a 
or (log x − 2)(log x + 4) = 0
So, n lies in ( k , k + 1). 5 5
= 5 × 2 × 3 =30
⇒ x = 1 or log x = 2, − 4
5 5
14. (b) Clearly, log sin x and log cos x 23. (b) log 243 = log (3 ) 5 = log 3
81 (3 ) 4 3
0. 5 0. 5 1 4
are defined, if x ∈ (0, π / 2) ⇒ x = 1, 25,
5 5
625 = × 1 = = 1.25
Now, log sin x = 1 − log cos x 4 4
0. 5 0. 5
⇒ log sin x + log cos x = 1 17. (c) Number of solutions are 3.
0. 5 0. 5
Q log a m = m 
18. (b) Solution set is 1, 25,
1   an n
⇒ log(sin x × cos x) = 1 .

0. 5
sin x cos x = (0.5)1  625  1 2
24. (a) We have, log m + log =
8 8
⇒ sin2x = 1 19. (c) Product of all values of x 6 3
π 3π m 2
⇒ x = , 1 1 ⇒ log =
= 1 × 25 × = 8
6 3
4 4 625 25
[Q log a m + log a n = log a (m × n)]
Hence, only Statement II is correct. 1
20. (a) 2 log 2 − log 9 m
log x log y log z 8
3 3 ⇒ = (8 ) 2 / 3 = ( 2 3 ) 2 / 3 = 2 2 = 4
15. (b) Let = = =k 6
b−c c−a a−b 1
= 2 log 2 − log (3)2 ⇒ m = 6 × 4 = 24
3
⇒ log x = k( b − c ) ⇒ x = 10k (b − c ) (2 3 ) 3
log y = k( c − a ) ⇒ y = 10k (c − a )
2 1 25. (d) Given, log a ( ab ) = x
= log 2 − ⋅ 2 log 3
log z = k( a − b ) ⇒ z = 10k (a − b ) 3 2
3 3
⇒ log a a + log a b = x
Q log b m = m log b  ⇒ 1 + log a b = x
Now, x a . y b .z c  an a

n
= 10k (b − c )a + k (c − a )b + k (a − b )c 2 2 ⇒ log a b = x − 1
= ( 1) − ( 1) Now, log ab = log a + log b
= 10k [a (b − c ) + b (c − a ) + c (a − b )] 3 3 b b b
= 100 = 1 2 2 1
= − =0 = + 1
1 1 1 3 3 log a b
16. (a) + +
log xy 2 log yz 2 log zx 2 21. (b) (log x ) (log x 2x )(log y ) = log x x 2 1
3 2x = + 1
= log xy + log yz + log zx log x log 2x log y log x 2 x −1
2 2 2
⇒ × × =
= log ( xy × yz × zx )
log 3 log x log 2x log x 1+ x− 1
=
2
 log a  x− 1
= log ( xyz ) = 2 log ( xyz )
2
Q log a =  x
2 2
=
= 2 log 1 = 0 [Q xyz = 1]  b log b 
 x− 1
2
10
107

MATRICES
In NDA exam, generally 1-2 questions are asked from this chapter which are based on equality of
matrices, product of matrices, transpose of a matrix etc.

A rectangular array of mn numbers (real or complex) in the form of m horizontal lines (called rows)
and n vertical lines (called columns), is called an m × n matrix (to be read as m by n matrix) or matrix of
order m × n.
 a11 a12 … a1n 
a a 22 … a 2n 
A m × n matrix is usually written as, A =  21 
 M M M M 
a m1 a m2 … a mn 
m× n
A matrix may be represented by the symbols [ a ij ], ( a ij ) or by a capital letter A, i.e.
A = [ a ij ] m × n or ( a ij ) m × n
The numbers a11 , a12 , . . . etc. are known as elements of the matrix A , a ij belongs to the ith row and jth
column and is called the ( i, j ) th element of matrix A = [ a ij ]
Note A matrix is not a number and it has no numerical value.

EXAMPLE 1. Construct a 2 × 2 matrix A = (a ij ), whose general element is given by a ij = (i + 2 j) 2/2.


9 25 9/2 25/2 9 5 9/2 25
a.  b.  c.  d. 
4 9  8 18  4 18  8 9 

Sol. b. a11 = (1 + 2) 2/ 2 = 9/ 2, a12 = (1 + 4) 2 / 2 = 25 / 2, a21 = ( 2 + 2) 2 / 2 = 8, a22 = ( 2 + 4) 2 / 2 = 18


 a11 a12  9/ 2 25 / 2
∴ A= =
a21 a22   8 18 

Types of Matrices
1. Row matrix A matrix having one row and n columns is called as row matrix. It is of the form
A = [ a11 a12 … a1n ]1 × n or A = [ a1 a 2 … a n ]1 × n
2. Column matrix A matrix having mrows and one column is called column matrix. It is of the form
 a11   a1 
a  a 
A =  21  or A =  2 
 M  M 
a m1  a m
m× 1 m× 1
108 NDA/NA Pathfinder

3. Zero matrix A matrix in which all the elements are y = x − y ⇒ 2y = x = 2 ⇒ y = 1


equal to zero is called zero matrix. It is also called a 2z = z + w ⇒ z = w
null matrix and is denoted by O. 3w = z − w + 3x ⇒ z = 4w − 6
On substituting w in terms of z, we get
4. Singleton matrix A matrix in which there is only
z = 4z − 6 ⇒ 3z = 6 ⇒ z = 2
one element is called singleton matrix.
Hence, we obtain x = 2, y = 1, z = 2 and w = 2
Thus, A = [ a ij ] m × n is singleton matrix, if m = n = 1 .
5. Square matrix A matrix in which the number of rows Algebra of Matrices
is equal to the number of columns, say n, is called a
2 3 Addition of Matrices
square matrix of order n, i.e. m = n. e.g. is a
9 7 If A and B be any two matrices of the same order
( m × n), then their sum A + B = [ a ij + bij ] (m × n)
square matrix of order 2.
6. Diagonal matrix A square matrix in which all its where, A = [ a ij ] (m × n) and B = [ bij ] (m × n)
elements are zero except those in the leading diagonal
Properties of Addition
is called a diagonal matrix, i.e. a ij = 0 for i ≠ j.
(i) Matrix addition is commutative, i.e. A + B = B + A .
d 0
e.g. The matrix, C = 1 is diagonal matrix of
0 d  (ii) Matrix addition is associative.
 2
order 2 and it can be rewritten as C = diag (d 1 , d 2 ) i.e. ( A + B) + C = A + ( B + C )
7. Identity or unit matrix A diagonal matrix in which (iii) A + O = A = O + A
all the diagonal elements are equal to 1 is called an Here, the null matrix O is called additive identity.
identity matrix. It is also called a unit matrix. An (iv) A + ( − A ) = O = ( − A ) + A
identity matrix of order n is denoted by I or I n .
Here, (−A) is called the additive inverse of A.
8. Scalar matrix A diagonal matrix in which all the
(v) Matrix addition follows cancellation law,
diagonal elements are equal is called a scalar matrix.
i.e. A + B = A +C ⇒B =C [left cancellation law]
9. Submatrix Any matrix obtained by omitting some
rows or columns or both from a given m × n matrix and B + A = C + A ⇒ B = C [right cancellation law]
A is called a submatrix of A. As a convention, the
Subtraction of Matrices
given matrix A is also taken as a submatrix of A.
If A and B are any two matrices of same order ( m × n),
10. Upper triangular matrix A square matrix A = [ a ij ]
then their difference A − B = [ a ij − bij ] m × n
is called upper triangular matrix, if a ij = 0 for all i > j. where, A = [ a ij ] m × n and B = [ bij ] m × n
11. Lower triangular matrix A square matrix A = [ a ij ] is
called a lower triangular matrix, if a ij = 0 for all i < j. 2 −1 4
EXAMPLE 3. If 2 A + 3B =  and
3 2 5
Equal Matrices  5 0 3 
Two matrices A = ( a ij ) m × n and B = ( bij ) p × q are said to A + 2B =  , then matrix B is
 1 6 2 
be equal, if
8 −1 2 8 1 2
(i) m = p and n = q (ii) a ij = bij , ∀ i and j a.  b. 
−1 10 −1 −1 10 −1
Thus, two matrices are said to be equal, if they are of −2
8 1 8 1 2
the same order and the corresponding elements are same. c.  d. 
−1 10 −1  1 10 1
EXAMPLE 2. Find x, y , z and w, if  2 −1 4
Sol. b. Given, 2A + 3B =  ...(i)
5
3x y   x + 4 x− y  3 2
= .
2z 3w z + w z − w + 3x 5 0 3 10 0 6
and A + 2B =  ⇒ 2A + 4B =  ...(ii)
a. 2, 1, 2, 2 b. 2, 2, 2, 2 c. 2, 1, 1, 2 d. 1, 2, 1, 2 1 6 2  2 12 4

Sol. a. The matrix on each side is of order 2 × 2. On subtracting Eq. (i) from Eq. (ii), we get
Equating the corresponding elements, we get 8 1 2
B=
3x = x + 4 ⇒ 2x = 4 ⇒ x = 2 −1 10 −1
MATHEMATICS Matrices 109

Scalar Multiplication of Matrices (iii) Multiplication is associative, i.e. if A , B and C are the
If a matrix is multiplied by the scalar k, then each matrices of order m × n , n × p and p × r respectively,
element is multiplied by k. Thus, if A = [ a ij ] m × n then then ( AB) C = A ( BC ).
kA = [ ka ij ] m × n . (iv) If A is a m × n matrix and I n , I m are the identity
matrices of order m × m and n × n, then
Properties of Scalar Multiplication of Matrices
I mA = A = AI n
Let the matrices A and B be of the same order and λ , µ
be scalars. Then, Here, I is the multiplicative identity.
(i) λ ( A + B) = λA + λB (ii) ( λ + µ ) A = λA + µA (v) If AB = O, then it is not necessary that either A or B
(iii) λ (µA ) = µ ( λA ) = ( λ ) µA (iv) ( − λ ) A =− ( λA )= λ ( − A ) is O or both are O.

EXAMPLE 4. Find the value or values of x such that 1 0  1 x 


EXAMPLE 5. Let A =   and B =  .
2  −3  −1 0 −1 0 1
x 2   + x   =  .
1  1  2 If AB = BA, then what is the value of x?
a. 0 b. 1 c. 1/2 d. Both (b) and (c) a. −1 b. 0
c. 1 d. Any real number
Sol. b. From the left hand side, we have
−3 2x  −3x 2x − 3x  1 0  1 x  1 x
2 2
2 AB =  =
x 2   + x   =  2 +  = Sol. b.
1  1  x   x  x + x
 2  0 −1 0 1 0 −1
2x 2 − 3x −1  1 x  1 0  1 − x
∴  x 2 + x  =  2 ⇒ 2x − 3x = − 1
2
…(i) and BA =  =
    0 1 0 −1 0 −1
and x2 + x = 2 …(ii) Q AB = BA
On solving the first equation 2x − 3x + 1 = 0, we get x = 1, 1/ 2
2
1 x  1 − x
The second equation x 2 + x = 2 is satisfied only when x = 1. ∴ 0 −1 = 0 −1
Hence, the solution is x = 1.
⇒ x= −x
Multiplication of Matrices
⇒ 2x = 0 ⇒ x = 0
If A and B are two matrices such that the number of
columns of A is equal to the number of rows in B, i.e. if Transpose of a Matrix
A = [ a ik ] is a m × n matrix and B = [ bkj ] be a n × p matrix, Let A be a m × n matrix. Then, the n × m matrix
then the product AB of these matrices is m × p matrix
obtained by interchanging the rows and columns of A
and is defined as
n is called the transpose of A and is denoted by A ′
( AB) ij = ∑ a ik bkj = a i1 b1 j + a i 2 b2 j + … + a in bnj or A T .
k=1
 b1 j  2 −3 −1  2 4
b2 j  e.g. If A = , then A ′ = −3 2
4 2 3 
= [ a i1 a i 2 . . . a in ]   2× 3 −1 3
3× 2
M 
bnj 
Properties of the Transpose Matrix
= Sum of the product of elements of ith row (i) ( A ′ )′ = A (ii) ( A ± B)′ = A ′ ± B′
of A with the corresponding elements of jth column of B.
(iii) ( kA )′ = kA ′ (iv) ( AB)′ = B′ A ′
Note In the matrix product AB, the matrix A is called pre multiplier
or pre factor and B is called post multiplier or post factor. Symmetric and Skew-symmetric
Properties of Multiplication of Matrices Matrices
(i) Matrix multiplication is not commutative in general A real square matrix A = ( a ij ) is said to be symmetric, if
i.e. AB ≠ BA a ij = a ji , ∀ i and j or A = A T .
(ii) Multiplication is distributive, i.e. if A , B and C are
the matrices of order m × n , n × p and n × p A real square matrix A = ( a ij ) is said to be skew-symmetric
respectively, then A ( B + C ) = AB + AC (anti-symmetric), if a ij = − a ji , ∀ i and j or A = − A T .
110 NDA/NA Pathfinder

Properties of Symmetric and Special Type of Matrix


Skew-symmetric Matrices 1. Orthogonal matrix A square matrix A is called an
(i) In a skew-symmetric matrix A, all its diagonal elements orthogonal matrix if the product of matrix A and its
are zero, i.e. a ii = 0, ∀ i. transpose A ′ (or A T ) is an identity matrix, i.e. AA ′ = I
(ii) The matrix which is both symmetric and
Note If A and B are orthogonal, then AB is also orthogonal.
skew-symmetric is a null matrix.
2. Conjugate of a matrix The matrix obtained from
(iii) For any real square matrix A ; A + A T and AA T or any given matrix A containing complex numbers as
A T A is a symmetric matrix and A − A T is a its elements, on replacing its elements by the
skew-symmetric matrix.
corresponding conjugate complex numbers is called
(iv) A real square matrix A can be expressed as the sum conjugate of A and is denoted by A.
of a symmetric matrix and a skew-symmetric matrix.
1 + 2 i 2 − 3 i
1 1
i.e. A = ( A + A T ) + ( A − A T ) e.g. If A= ,
4 − 5 i 5 + 6 i
2 2
1 − 2 i 2 + 3 i
cos α − sin α  then A=
4 + 5 i 5 − 6 i
EXAMPLE 6. If A =  , then find the
 sin α cos α 
3. Hermitian matrix A square matrix such that
value of AT A. ( A ′ ) = A, then A is known as hermitian matrix.
a. O b. I c. A d. AT
4. Skew-hermitian matrix A square matrix such that
 cos α sin α 
( A ′ ) = − A, then A is known as skew-hermitian
Sol. b. We have, AT = 
− sin α cos α  matrix.
 cos α sin α  cos α − sin α  5. Elementary matrix A square matrix is called an
Therefore, AT A = 
− sin α cos α   sin α cos α  elementary matrix if it can be obtained from identity
cos2 α + sin2 α 0  1 0 matrix I by performing single elementary row or
= 2  = 0 1 = I column operation.
 0 sin α + cos α  
2

PRACTICE EXERCISE
1 2 2 3 4. If α, β are the two roots of 1 + x + x 2 = 0, then
1. If A =   , B = 4 5 and 4 A − 3B + C = O , then
3 4   1 β  α β
matrix product    is equal to
C is equal to α α 1 β
2 −1 2 1 1 1  −1 −1
(a)   (b)   (a)  (b) 
 0 1  0 −1  2 
1 2   −1
 −2 1
 1 −1  −1 −1
(c)   (d) None of these (c)  (d) 
 0 −1 
 −1 2   −1 −2 
 1 0 1 0 1 2 a 0
2. If A =   and I = 0 1, then the value of k, 5. If A = 
−1 7    and B =  0 b, where a, b are natural
 3 4  
so that A2 = 8 A + kI is numbers, then which one of the following is
(a) 4 (b) 5 (c) 6 (d) −7 correct?
3. The value of x for which (a) There exist more than one but finite number of B’s
such that AB = BA
1 0 2 1 (b) There exist exactly one B such that AB = BA
[1 1 x ] 0 2 1 1 = 0 is (c) There exist infinitely many B’s such that AB = BA
  
2 1 0 1 (d) There cannot exist any B such that AB = BA

(a) 2 (b) −2 (c) 3 (d) −3


MATHEMATICS Matrices 111

Directions (Q. Nos. 6-8) If A and B are two matrices


of same order, then 15. A square matrix [aij ] such that aij = 0 for i ≠ j
and aij = k, where k is a constant for i = j is
6. ( AB)n = An Bn is called e 2012 II
(a) always true (b) never true (a) diagonal matrix but not scalar matrix
(c) true only when AB = BA (d) None of these (b) scalar matrix
(c) unit matrix
7. If A and B are symmetric matrices, then ( ABA)T (d) None of the above
is
 x  y z
16. If the sum of the matrices  x ,  y and 0 is the
   
(a) symmetric matrix (b) skew-symmetric matrix
(c) diagonal matrix (d) None of these
 y  z  0
8. If AB = BA, then ( A + B)2 is equal to 10
(a) A 2 + 2 AB + B2 (b) A 2 + B2 matrix  5 , then what is the value of y?
 
(c) A 2 + 2 BA − B2 (d) None of these  5  e 2012 II
(a) – 5 (b) 0 (c) 5 (d) 10
Directions (Q. Nos. 9-11) Matrix A has x rows and
x + 5 columns. Matrix B has y rows and 11− y 17. If the matrix AB is a zero matrix, then which
columns. Both AB and BA exist. one of the following is correct? e 2012 II
(a) A must be equal to zero matrix or B must be equal
9. Find the value of x. to zero matrix
(a) 1 (b) 3 (c) 5 (d) 7 (b) A must be equal to zero matrix and B must be equal
to zero matrix
10. Find the value of y. (c) It is not necessary that either A is zero matrix or B is
zero matrix
(a) 8 (b) 6 (c) 4 (d) 2
(d) None of the above
11. The order of AB is 18. Consider the following statements
(a) 3 × 3 (b) 8 × 8 (c) 3 × 8 (d) 8 × 3 I. Every zero matrix is a square matrix.
II. A matrix has a numerical value.
PREVIOUS YEARS’ QUESTIONS III. A unit matrix is a diagonal matrix.
Which of the above statement(s) is/are correct?
12. What is the order of the product e 2012 II
a h g  x (a) Only II (b) Only III
[x y z ] h b f   y? (c) Both II and III (d) Both I and III
   
 g f c  z  i 0 0 −1 0 i
e 2012 I 19. If A =   , B = 1  and C =  , where
(a) 3 × 1 (b) 1 × 1 0 − i  0  i 0
(c) 1 × 3 (d) 3 × 3 i = −1, then which one of the following is
13. The sum and product of matrices A and B exist. correct? e 2013 II
Which of the following implications are (a) AB = − C (b) AB = C
necessarily true? (c) A 2 = B2 = C 2 = I, where I is the identity matrix
I. A and B are square matrices of same order. (d) BA ≠ − C
II. A and B are non-singular matrices. 2 3  5 −2  1 −1
20. If  × =
1 17 λ 
, then what is λ
Select the correct answer using the code given 4 1 −3
below. e 2012 I equal to e 2013 II
(a) Only I (b) Only II (a) 7 (b) – 7 (c) 9 (d) – 9
(c) Both I and II (d) Neither I nor II
21. Consider the following statements e 2013 II
34 I. The product of two non-zero matrices can never
3 5 7
14. If A = 56 and B = 
  , then
be identity matrix.
4 6 8
8
7 II. The product of two non-zero matrices can never
be zero matrix.
Which one of the following is correct? e 2012 II Which of the above statement(s) is/are correct?
(a) B is the inverse of A (b) B is the adjoint of A (a) Only I (b) Only II
(c) B is the transpose of A (d) None of these (c) Both I and II (d) Neither I nor II
112 NDA/NA Pathfinder

22. If A is any matrix, then the product AA is defined x + y


y   2 3
only when A is a matrix of order m × n, where 28. Let A =   , B =   and C =  . If
 2x x − y −1 2
e 2014 I
(a) m > n (b) m < n AB = C, then what is A2 equal to? e 2015 I
(c) m = n (d) m ≤ n  6 −10  −10 5 
(a)   (b)  
 4 x + 2  4 26   4 24
23. If A =   is symmetric, then what is x
2x − 3 x + 1  −5 −6   −5 −7 
(c)   (d)  
equal to? e 2014 II  −4 −20  −5 20 
(a) 2 (b) 3
(c) −1  cos θ sin θ
(d) 5 29. If E(θ) =  , then E(α )E(β ) is equal to
− sin θ cos θ e 2015 I
24. If A and B be two matrices such that AB = A and
(a) E (αβ ) (b) E (α − β )
BA = B. Then, (c) E (α + β ) (d) − E (α + β )
Which of the following statement(s) is/are correct?
0 − 2
1
I. A 2 = A II. B2 = B 30. If A =  , then the matrix X for which
2 − 3 4 
III. ( AB)2 = AB
Select the correct answer using the code given 2X + 3 A = 0 holds true is e 2015 II

below. e 2014 II − 3 0 − 3 3 0 − 3
 2  2 
(a) I and II (b) II and III (a)   (b)  
(c) I and III (d) I, II and III − 3 − 9 − 6  3 − 9 − 6 
 2   2 
25. Which one of the following matrices is an
elementary matrix? e 2015 I  3 0 3  – 3 0 3
   
1 0 0 1 5 0 (c)  2  (d)  2 
3 9 9
(a)  0 0 0 (b)  0 1 0 6  –3 – 6
     2   2 
 0 0 1  0 0 1
1 1 − 1 − 1 − 2 − 1
0 2 0 1 0 0
(c)  1 0 0 (d)  0 1 0
31. If A = 2 − 3 4  and B =  6 12 6  , then
 
   
 0 0 1  0 5 2  3 − 2 3   5 10 5 
Which of the following is/are correct?
 0 −4 + i
26. The matrix   is I. A and B commute. II. AB is a null matrix.
4 + i 0  e 2015 I Select the correct answer using the code given
(a) symmetric (b) skew-symmetric below. e 2015 II
(c) hermitian (d) skew-hermitian (a) Only I (b) Only II (c) Both I and II (d) Neither I nor II

3 −4  p q
5 2 32. Consider the following in respect of the matrix
27. If X =  , B = −2 1 and A =  r satisfy
1 −1    s −1 1
A=  
the equation AX = B, then the matrix A is equal  1 −1
to e 2015 I I. A 2 = − A II. A3 = 4 A
 −7 26  7 26 
(a)  (b) 
−5  Which of the above statement(s) is/are correct?
1  4 17  e 2016 I
 −7 −4  − 7 26  (a) Only I (b) Only II
(c)  (d) 
 26 13   −6 23

(c) Both I and II (d) Neither I nor II

ANSWERS
1 b 2 d 3 b 4 b 5 c 6 c 7 a 8 a 9 b 10 a
11 a 12 b 13 a 14 c 15 b 16 b 17 c 18 b 19 a 20 b
21 d 22 c 23 d 24 d 25 b 26 d 27 a 28 a 29 c 30 d
31 b 32 b
MATHEMATICS Matrices

HINTS AND SOLUTIONS 


a b  1 2  2 3 a b  9. (b) On adding Eqs. (i) and (ii), we get 2x = 6 ⇒ x = 3
1. (b) Let C =  , then 4  −3  +  =0
c d 3 4 4 5 c d 10. (a) On subtracting Eq. (ii) from Eq. (i), we get 2 y = 16 ⇒ y = 8
       
 4− 6+ a 8− 9 + b   0 0 11. (a) Order of AB = (Number of rows in A)
⇒ =
12 − 12 + c 16 − 15 + d   0 0 × (Number of columns in B)
   
= x × ( 11 − y )
⇒ a = 2, b = 1, c = 0 and d = − 1 =3×3
2 1  a h g x 
∴ C =
0 −1    
  12. (b) Here, [ x y z ]
1 ×3 h b f y
g f c z 
 1 0  1 0  3 ×3   3 ×1
2. (d) Q A =   ,I =
−1 7 0 1
    Order of matrix = 1 × 3 : 3 × 3 : 3 × 1 = 1 × 3 : 3 × 1 = 1 × 1
 1 0  1 0  1 0 13. (a)
Now, A 2 =  =
−1 7  −1 7  −8 49 14. (c)
    
a = 0, for i ≠ j
 8 0  k 0 8 + k 0 15. (b) Given, [ a ] =  ij where k is a constant.
and 8 A + kI =  + = a = k , for i = j
−8 56  0 k   −8 56 + k  ij
ij

     
 k 0 0
Q A2 = 8A + k I  
∴ [ a ] of order 3 × 3 =  0 k 0 = Scalar matrix
 1 0 8 + k 0 ij
0 0 k 
∴  = ⇒ 8 + k = 1 ⇒ k = −7
−8 49  −8 56 + k   3 ×3
   
3. (b) 16. (b)
4. (b) α , β are the roots of 1 + x + x = 0
2
17. (c) For the matrix AB is a zero matrix. It is not necessary that
α + β = − 1 and αβ = 1 either A is zero matrix or B is zero matrix.
Also, 1 + α + α 2 = 0 ⇒ α + α 2 = − 1and 1 + β + β 2 = 0  1 0  0 0
e.g. Let A =  and B = 
⇒ β + β2 = − 1 0 0 0 −1
   
 1 β  α β   α + β β + β 2   −1 −1
∴ AB = 0, where A , B ≠ 0
∴    = 2 =
α α 1 β −1 2 
    α + α αβ + αβ    18. (b) I. Every zero matrix is not necessarily a square matrix.
1 2  a 0 II. A matrix does not have a numerical value while every
5. (c) Q A =  and B = 
3 4 0 b determinant have a numerical value.
    III. Unit matrix is a diagonal matrix and scalar matrix also.
1 2  a 0  a 2b 
∴ AB = 
3
=
4  0 b  3a 4b   i 0  0 −1 0 i 
19. (a) Given that, A =  , B= and C = 
     0 −i  1 0 i 0
     
a 0  1 2  a 2a 
and BA =  =  0 −i  0 i 
0 b  3 4 3b 4b  Now, AB =  =− = −C
     −i 0 i 0
   
 a 2 b   a 2a 
If AB = BA, then  = ⇒ a=b
3a 4b  3b 4b  20. (b)
   
From the above it is clear that there exist infinitely many B ’ s such 21. (d) We know that, the product of two identity matrix are always an
that AB = BA. identity matrix, which is non-zero matrices.
6. (c) We know that, ( AB )n = A n B n is true only, when AB = BA  1 0  1 0  1 + 0 0 + 0  1 0
 0 1 ×  0 1 =  0 + 0 1 + 0  =  0 1 = I = Identity matrix
7. (a) 8. (a)        
Sol. (Q. Nos. 9-11) Since, BA is defined. The product of two non-zero matrices can sometimes be zero
∴ Number of columns in B = Number of rows in A matrix.
⇒ 11 − y = x ⇒ x + y = 11 …(i)  0 c −b  a 2 ab ac 
   
Also, AB is defined. −c 0 a  ×  ab b2 bc 
∴ Number of columns in A = Number of rows in B  b −a 0  ac bc c 2
   
∴ x + 5 = y ⇒ x − y = −5 …(ii)
114 NDA/NA Pathfinder

 0 + abc − bac 0 + b2c − b2c 0 + bc 2 − bc 2   2x + 2 y − y  3   2x + y  3 


 2  ⇒  4x =  ⇒  = 
=  − a c + 0 + a 2 c − abc + 0 + abc − ac 2 + 0 + ac 2  − x + y   2 3x + y   2
       
 a 2 b − a 2 b + 0 ab 2 − ab 2 + 0 abc − abc + 0 
  ⇒ 2x + y = 3 and 3x + y = 2 ⇒ x = 2−3= −1
 0 0 0 ∴ y =5
 
=  0 0 0 = 0 = Zero matrix x + y y 2  4 54 5
∴ A2 =  =
 0 0 0 2x x − y −2 −6   −2 −6 
      
So, both the Statements are incorrect.  16 − 10 20 − 30   6 − 10 
= =
22. (c) Given that, A is any matrix. −8 + 12 −10 + 36  4 26 
   
Then, the product AA is defined only when number of columns in
 cos θ sin θ 
A is equal to number of rows in A, i.e. A must be a square matrix. 29. (c) Given, E (θ) = 
− sin θ cos θ
Thus, m = n  
 4 x + 2  4 2x − 3  cos α sin α   cos β sin β 
A = A′ ⇒  = ∴ E (α ) E ( β ) = 
2x − 3 x + 1  x + 2 x + 1 
23. (d) Q
− sin α cos α   − sin β cos β 
      
⇒ 2x − 3 = x + 2 ⇒ x = 5
 cos α ⋅ cos β − sin α ⋅ sin β cos α ⋅ sin β + sin α ⋅ cos β 
=
24. (d) We have, AB = A − sin α ⋅ cos β − sin β ⋅ cos α − sin α ⋅ sin β + cos α ⋅ cos β 
 
∴ A 2 = ( AB ) ⋅ ( AB ) = A ⋅ ( BA ) B
= ABB [QBA = B ]  cos(α + β ) sin(α + β ) 
= = E ( α + β)
= AB = A [Q AB = A ] − sin(α + β ) cos(α + β )
 
Also, B 2 = ( BA ) ⋅ ( BA ) = B ⋅ ( AB ) ⋅ A
= B⋅ A⋅ A [Q AB = A ] 30. (d )
= B⋅ A = B [QBA = B ] 1 1 − 1  − 1 − 2 − 1
   
Again, ( AB )2 = ( AB ) ⋅ ( AB ) = A ⋅ ( BA ) B 31. (b) We have, A =  2 − 3 4  and B =  6 12 6 
= A⋅B⋅B [QBA = B ] 3 − 2 3  5 10 5 
   
= AB
 − 1+ 6 −5 − 2 + 12 − 10 − 1+ 6 −5 
Hence, all statements are correct.  
∴ AB =  − 2 − 18 + 20 − 4 − 36 + 40 − 2 − 18 + 20
 1 5 0
   − 3 − 12 + 15 − 6 − 24 + 30 − 3 − 12 + 15 
25. (b)  0 1 0 is an elementary matrix. Since, it is obtained by  
 0 0 1
   0 0 0
 
performing operation C → C + 5C on identity matrix I . =  0 0 0
2 2 1 3
26. (d) A square matrix A is said to be skew-Hermitian, if A * = − A or  0 0 0
 
a = − a ij , ∀ i and j.
ij
Hence, AB is a null matrix.
Here, a = − 4 + i and a =4+i
12 21 Since, A ≠ 0, B ≠ 0 and AB = 0, therefore BA ≠ 0.
Now, a 12 = ( − 4 + i ) = −4 − i = −( 4 + i ) = − a
21 Thus, A and B do not commute.
Hence, the given matrix is skew-Hermitian matrix.
 −1 1
27. (a) ∴ AX = B 32. (b) Given, A = 
1 −1
 p q  3 −4  5 2 3 p + q −4 p − q   5 2  
∴ r s   = ⇒ =
1 −1  −2 1 3r + s −4r − s   −2 1  −1 1  −1 1  1 + 1 −1 − 1
         A2 = A × A =  =
1 −1  1 −1  −1 − 1 1 + 1
I. Now,
⇒ 3 p + q = 5 and −4 p − q = 2 ⇒ − p = 7 ⇒ p = − 7     
∴ q = 5 + 21 = 26  2 −2   1 −1
A2 =  =2
Also, 3r + 5 = −2 and −4r − s = 1 −2 2 −1 1
⇒ −r = − 1 ⇒ r = 1    
and s = − 2 − 3 = − 5  2 −2  −1 1  −2 − 2 2 + 2
II. A 3 = A 2 ⋅ A =  =
 −7 26  −2 2  1 −1  2 + 2 −2 − 2
∴ A=     
1 −5 
   −4 4  −1 1
= =4
28. (a) We have, AB = C 4 −4  1 −1
   
x + y y   2  3 
⇒ A3 = 4A
∴ =
 2x x − y   −1  2
      Hence, only II is correct.
11
MATHEMATICS Decimal Fractions 115

DETERMINANTS
In NDA exam, generally 4-7 questions are asked from this chapter which are based on
expansion/value of determinant, adjoint/inverse of a matrix, properties of determinant and
solution of system of linear equations.

Every square matrix is associated with a particular expression which is called the determinant. The
determinant of a square matrix A is denoted by det A or | A | . In determinant, horizontal lines and
vertical lines are called rows and columns respectively, as in matrix.
 a11 a12 a13 K a1n 
a a 22 a 23 K a 2n 
A determinant of order n × n is represented as follows, A = 21 


M M M M M 

 a n1 a n2 a n3 K a nn 

Note A matrix is an arrangement of numbers and it has no fixed value but a determinant is a number and it has a fixed value.

Determinant of Square Matrix Order 1


If A = [ a11 ] is a square matrix of order 1, then | A | = a11

Determinant of Square Matrix of Order 2


a a12 
If A =  11 is a square matrix of order 2, then determinant of A is defined as
a 21 a 22 
a a12
| A | = 11 = a11 a 22 − a12 a 21
a 21 a 22

Determinant of Square Matrix of Order 3


 a11 a12 a13 
If A = a 21 a 22 a 23  is a square matrix of order 3, then determinant of A is defined as
 
a 31 a 32 a 33 
a11 a12 a13
| A | = a 21 a 22 a 23
a 31 a 32 a 33
a 22 a 23 a 21 a 23 a 21 a 22
∴ | A | = a11 − a12 + a13
a 32 a 33 a 31 a 33 a 31 a 32
= a11 ( a 22 a 33 − a 23 a 32 ) − a12 ( a 21 a 33 − a 23 a 31 ) + a13 ( a 21 a 32 − a 22 a 31 )
116 NDA/NA Pathfinder

Properties of Determinants  log x log y log z 


 log x log x log x 
(i) If each entry in any row or column of a determinant  1 log x y log x z   
log x log y log z 
is 0, then the value of the determinant is zero. Sol. c.  log y x 1 log y z  = 
   log y log y log y 
(ii) If rows be changed into columns and columns into log z y 1  
 log z x log x log y log z 
rows, then the value of the determinant remains  
unchanged.  log z log z log z 
(iii) If any two adjacent rows (columns) of a
 log x log y log z 
determinant are interchanged, then the determinant 1  log x
= log y log z 
remains its absolute value but changed in sign. log x log y log z  
(iv) If a determinant have any two rows or columns  log x log y log z 
identical, then its value is zero. =0 [since, all rows are identical]
(v) If all the constituents (elements) of one row or of
one column, multiplied by the same quantity, then EXAMPLE 2. If ω is the cube root of unity, then
the value of new determinant is k times the value of what is one root of the equation
original determinant.  x 2 −2x −2ω 2 
ka11 ka12 ka13 a11 a12 a13 2 ω −ω  = 0?
 
i.e. a 21 a 22 a 23 = k a 21 a 22 a 23 ω
0 1 
a 31 a 32 a 33 a 31 a 32 a 33 a. 1 b. −2 c. 2 d. ω
(vi) If each constituent of any row or of any column be x 2
−2ω 
−2x 2

the sum of the two quantities, then the determinant Sol. c.  2 − ω = 0


ω
can be expressed as the sum of the two determinants  
0 ω 1 
of the same order.
ω −ω 2 −ω 2 ω
a11 + a a12 + b a13 + c a11 a12 a13 ⇒ x2 + 2x − 2ω 2 =0
ω 1 0 1 0 ω
i.e. a 21 a 22 a 23 = a 21 a 22 a 23 ⇒ x2 (ω + ω 2) + 2x ( 2) − 2ω 2 ( 2ω) = 0 [Q 1+ ω + ω 2 = 0]
a 31 a 32 a 33 a 31 a 32 a 33 ⇒ − x2 + 4x − 4ω 3 = 0
a b c ⇒ x2 − 4 x + 4 = 0
⇒ ( x − 2) 2 = 0 ⇒ x = 2
+ a 21 a 22 a 23
a 31 a 32 a 33 MINORS AND COFACTORS
(vii) If each element of a row or column of a determinant Minor of an Element of a Determinant
is multiplied by a constant k and then added to the
corresponding elements of some other row or If we delete the row and column passing through the
column, then value of determinant remains same. element a ij , the determinant, thus obtained is called the
a11 a12 a13 a11 a12 a13 minor of a ij and is usually denoted by M ij .
a 21 a 22 a 23 = a 21 a 22 a 23 a11 a12 a13
a 31 a 32 a 33 a 31 + ka 21 a 32 + ka 22 a 33 + ka 23 e.g. For the 3 × 3 determinant a 21 a 22 a 23
a 31 a 32 a 33
(viii) Number of elements in nth order determinant = n 2
(ix) If A and B are two determinants of order n, then a 22 a 23
The minor of a11 is M 11 = .
| AB | = | A | | B | a 32 a 33
(x) det ( kA ) = k n det ( A ), if A is of order n × n.
Cofactor of an Element of a Determinant
(xi) If AB = AC, then B = C is true only, when | A | ≠ 0.
The cofactor of an element a ij is ( −1) i + j M ij and it is
EXAMPLE 1. If x, y and z are all positive, then what is  M ij , when i+ j is even
 1 denoted by C ij . Thus, C ij = 
log x y log x z   − M ij , when i+ j is odd
 
the value of log y x 1 log y z ?  a11 a12 a13 

 

 log x log y 1  Let ∆ = a 21 a 22 a 23 
z z
 
a. 1 b. 3 c. 0 d. −2  a 31 a 32 a 33 
MATHEMATICS Determinants 117

a 22 a 23 a a 23 a a 22 Properties of Adjoint Matrix


= a11 − a12 21 + a13 21
a 32 a 33 a 31 a 33 a 31 a 32
If A , B are square matrices of order n and I n is
∴ ∆ = a11 M 11 − a12 M 12 + a13 M 13 corresponding unit matrix, then
= a11C11 + a12C12 + a13 C13 (i) A (adjA ) = | A | I n = (adj A )A (ii) | adjA | = | A | n − 1
(i) If ∆ ≠ 0 and ∆c denoted the determinant of cofactors, (iii) adj (adjA ) = | A | n − 2 A ; | A | ≠ 0
then ∆c = ∆n − 1 , where n (> 0 ) is the order of ∆. 2
(iv) | adj (adjA )| = | A | (n − 1) (v) adj ( A T ) = (adj A ) T
(ii) If ∆ = 0, then ∆c = 0.
(vi) adj( AB) = (adj B)(adj A )
(iii) The sum of the product of constituents of any row
(column) of a determinant with the cofactors of the (vii) adj ( A m ) = (adj A ) m, m ∈ N
corresponding elements of any other row (column) (viii) adj( kA ) = k n − 1 (adj A ) , k ∈ R
is zero. (ix) adj( I n ) = I n (x) adj(O) = O
i.e. a11C 31 + a12C 32 + a13C 33 = 0 (xi) A is symmetric matrix ⇒adj( A ) is also symmetric
matrix.
EXAMPLE 3. If ∆ is the determinant of the matrix
(xii) A is diagonal matrix ⇒ adj (A) is also diagonal matrix.
a b
 −b −a  and ∆ the determinant of the cofactors of
c
(xiii) A is triangular matrix ⇒ adj (A) is also triangular
  matrix.
the elements of the matrix. Then, which one of the 1 −1 1 
following is correct?  
EXAMPLE 4. If A = 0 2 −3 and B = (adj A) and
1
a. ∆c = ∆ b. ∆c = ∆2 c. ∆c = ∆3 d. ∆c = 2 1 0 

|adj B|
a b  a b C = 5 A, then is equal to
Sol. a. Let A =   , ∆ =  = − a2 + b2 |C|
−b − a  −b −a 
a. 5 b. 25 c. −1 d. 1
 − a b 1 −1
and matrix of cofactors of A =  
1
−b a  Sol. d. | A| = 0 2 −3 = 1( 3) + 1(6) + 1( −4) = 5
 −a b 
∆c =   = − a2 + b 2 ⇒ ∆ = ∆c 2 1 0
 −b a   3 1 1  5 −5 5 
B = adj A = −6 −2 3 , adj B =  0 10 −15 = 5A
 
ADJOINT OF A MATRIX    
−4 −3 2 10 5 0 
Let A = [ a ij ] be a square matrix of order n and C = [ c ij ]
and C = 5A
be its cofactor matrix. Then, matrix C T = [C ji ] , is | adj B| |5A|
called the adjoint of matrix A and is written as ∴ = =1
| C| |5A|
adj ( A ) = C T = [C ij ], 1 ≤ i, j ≤ n.
 1 2 3 INVERSE OF A MATRIX
e.g. If A = −1 0 1 , then C11 = − 3, C12 = 6, C13 = − 3, A non-singular square matrix A = [ a ij ] of order n is said
  to be invertible or has an inverse, if there exists another
 4 3 2 non-singular square matrix B of order n, such that
C 21 = 5, C 22 = − 10, C 23 = 5 AB = BA = I n
C 31 = 2, C 32 = − 4 and C 33 = 2 where, I is an identity matrix of order n. Then, we write
−3 6 −3 B = A −1 or A = B −1

∴ C = 5 −10 5 Hence, we say that A −1 is the inverse of A, if
 
 2 4 2 AA −1 = A −1 A = I
1
−3 5 2 The inverse of a matrix A is given by A −1 = adj ( A ).
|A|
Thus, adj ( A ) = C = 6 −10 −4
T 
  Note Non-singular and singular matrices A matrix A is said to be
−3 5 2 non-singular, if its determinant is non-zero, i.e. | A| ≠ 0. The
matrix whose determinant is zero, i.e. | A| = 0, is called a
singular matrix.
118 NDA/NA Pathfinder

Properties of Inverse Matrices Then , X = A −1 B


If A and B are invertible matrices of the same order, then This method is also known as the matrix method to
−1 −1
(i) ( A ) = A (ii) ( AB) −1
=B A−1 −1 solve a system of equations.
(iii) ( A k ) −1 = ( A −1 ) k , k ∈ N Criterion of Consistency
1 Let A x = B be a system of n linear equation with n variables
(iv) adj ( A −1 ) = (adj A ) −1 (v) | A −1 | = = | A | −1
| A| (i) If | A | ≠ 0 (i.e. A is non-singular), then the system of
equations is consistent and has a unique solution
Note If A is an invertible matrix, then given by X = A −1 B.
• If A is symmetric matrix, then A −1 is also symmetric matrix.
• If A is skew-symmetric matrix, then A −1 is also skew (ii) If | A | = 0 (i.e. A is singular) and [adj ( A )] B = 0, i.e.
symmetric matrix. null matrix, then the system of equations is
consistent and infinitely many solutions.
1 p q 
  (iii) If | A | = 0 and [adj ( A )] B ≠ 0, then the system of
EXAMPLE 5. If the inverse of 0 x 0  is equations is inconsistent and has no solution.
0 0 1 
Homogeneous Equations
 1 − p −q  The system of equations A X = B is said to
 
0 1 0  , then what is the value of x? homogeneous, if B = 0
0 0 1 (i) If | A | ≠ 0, then its only solution X = 0, is called
trivial solution.
1 1
a. 1 b. 0 c. −1 d. + (ii) If | A | = 0, then A X = 0 has a non-trivial solution. It
p q
will have infinitely many solutions.
 1 p q  1 − p − q
Sol. a. Let A = 0 x 0 and B = 0
  1 0 EXAMPLE 6. The equations
    x + 2y + 3z = 1, x − y + 4 z = 0 and 2x + y + 7 z = 1 have
0 0 1 0 0 1
a. only two solutions b. only one solution
 1 p q c. no solution d. infinitely many solutions
Thus, B −1
= 0 1 0  1 2 3
 
0 0 1 Sol. d. We have, | A | = 1 −1 4 = 1( − 11) − 2 ( − 1) + 3 ( 3) = 0
But B is inverse of A, therefore A = B −1 2 1 7
T
 1 p q  1 p q −11 1 3  −11 −11 11
⇒ 0 x 0  = 0 1 0  ⇒ x = 1 adj ( A) = −11 1 3  =  1 1 −1
       
0 0 1 0 0 1  11 −1 −3  3 3 −3
−11 −11 11 1 0
SOLUTION OF SYSTEM ∴ (adj A) B =  1 1 −1 0 = 0 = 0
    
OF LINEAR EQUATIONS  3 3 −3 1 0
So, the given system of equations is consistent and has
(i) Using Matrices infinitely many solutions.
Consider, the system of linear equations
(ii) Cramer’s Rule
a11 x1 + a12 x 2 + a13 x 3 = b1
Case I Let us consider a system of equations in two
a 21 x1 + a 22 x 2 + a 23 x 3 = b2 variables
a 31 x1 + a 32 x 2 + a 33 x 3 = b3 a1 x + b1 y = c1 , a 2 x + b2 y = c 2
We can write these equations in matrix form as a b c b a c
then, ∆ = 1 1 , ∆ 1 = 1 1 and ∆ 2 = 1 1
AX = B …(i) a 2 b2 c 2 b2 a2 c 2
 a11 a12 a13   b1   x1  By Cramer’s rule the solution of system of equation is
∆ ∆
where, A = a 21 a 22   
a 23 , B = b2 and X = x 2  x = 1 , y = 2 , provided ∆ ≠ 0.
      ∆ ∆
a 31 a 32 a 33  b3  x 3 
MATHEMATICS Determinants 119

(i) If ∆ ≠ 0, then the system is consistent and has a


unique solution.
DIFFERENTIATION OF
(ii) If ∆ = 0 and atleast one of the determinants ∆ 1 and DETERMINANTS
∆ 2 is non-zero, then the system is inconsistent. f1 ( x) g1 ( x)
Let ∆( x) = ,
Case II Let us consider a system of equations in three f 2 ( x) g 2 ( x)
variables where f1 ( x), f 2 ( x), g1 ( x) and g 2 ( x) are functions of x.
a1 x + b1 y + c1 z = d 1 , a 2 x + b2 y + c 2 z = d 2
f1′( x) g1′ ( x) f1 ( x) g1 ( x)
a 3 x + b3 y + c 3 z = d 3 Then, ∆ ′( x) = +
f 2 ( x) g 2 ( x) f 2′( x) g 2′ ( x)
 a1 b1 c1   d 1 b1 c1 
Then, ∆ = a 2 b2 c 2 , ∆ 1 = d 2 b2 c 2 , Also, ∆ ′( x) =
f1′( x) g1 ( x)
+
f1 ( x) g1′ ( x)
   
f 2′( x) g 2 ( x) f 2 ( x) g 2′ ( x)
 a 3 b3 c 3   d 3 b3 c 3 
 a1 d 1 c1   a1 b1 d 1  Thus, to differentiate a determinant, we differentiate one
∆ 2 = a 2 d 2 c 2 , ∆ 3 = a 2 b2 d 2  row (or column) at a time, keeping others unchanged.
   
 a3 d 3 c 3   a 3 b3 d 3 
INTEGRATION OF
By Cramer’s rule, the solution of system of equations is DETERMINANTS
∆ ∆ ∆
x = 1 , y = 2 and z = 3 f ( x) g( x)
∆ ∆ ∆ If ∆( x) = ,
(i) If ∆ ≠ 0, then the system is consistent and unique λ1 λ 2
solution exists. b b

(ii) If ∆ = 0 and atleast one of the determinants ∆ 1 , ∆ 2 then


b
∫a ∆( x) dx = ∫a f ( x) dx ∫a g( x)dx
λ1 λ2
and ∆ 3 is non-zero, then the given system is
inconsistent. Here, f ( x) and g( x) are functions of x and λ 1 , λ 2 are
(iii) If ∆ = 0 and ∆ 1 = ∆ 2 = ∆ 3 = 0, then the system is constants.
consistent and dependent and has infinitely many If the elements of more than one column or row are
solutions. functions of x, then the integration can be done only
after evalution/expansion of the determinant.
EXAMPLE 7. For what value of p, is the system of
equations p 3 x + (p + 1) 3 y = (p + 2) 3 , px + (p + 1) y = p + 2 cos x x 1
and x + y = 1 consistent? EXAMPLE 8. If f (x) = 2 sin x x 2
2 , then the value
a. p = 0 b. p = 1 c. p = − 1 d. for all p > 1 tan x x 1
Sol. c. The given system of equations is of f ′(x) at x = 0 is
p3x + ( p + 1) 3 y = ( p + 2) 3 ...(i) a. − 2 b. 2
px + ( p + 1) y = ( p + 2) ...(ii)
x+ y =1 ...(iii) c. 0 d. 1
 p3 ( p + 1) 3 ( p + 2) 3  − sin x 1 0 cos x x 1
This system is consistent, if  p ( p + 1) ( p + 2)  = 0 Sol. a. f ′ ( x) = 2 sin x x2 2 + 2cos x 2x 0
 
 1 1 1  tan x x 1 tan x x 1
p ( p + 1) − p ( p + 2) − p
3 3 3 3 3
cos x x 1
⇒ p 1 2 =0 + 2 sin x x2 2
1 0 0 sec 2 x 1 0
apply C 2 → C 2 − C1
0 1 0 1 0 1 1 0 1
 and C → C − C 
 3 3 1
⇒ f ′ (0) = 0 0 2 + 1 0 0 + 0 0 2
⇒ 2( p + 1) 3 − 2p3 − ( p + 2) 3 + p3 = 0
0 0 1 0 0 1 1 1 0
⇒2( p3 + 1+ 3p2 + 3p) − 2p3 − ( p3 + 8 + 12p + 6 p2) + p3 = 0
=0 + 0 + −2= −2
⇒ −6 − 6 p = 0
∴ p = −1
120 NDA/NA Pathfinder

Applications of Determinant EXAMPLE 9. The equation of straight line passing


through the points A(3, 1 ) and B (9, 3 ) is
in Geometry a. x − 2 = 0
1. Area of triangle If ( x1 , y1 ), ( x 2 , y 2 ) and ( x 3 , y 3 ) are b. x − 3y = 0
the vertices of a triangle, then
c. y + 2 = 0
x1 y1 1
1 d. y − 2 = 0
Area of triangle = x2 y2 1
2 Sol. b. Let P( x, y) be any point on the line joining A( 3, 1) and
x3 y3 1
B(9, 3).
1
= [ x1 ( y 2 − y 3 ) + x 2 ( y 3 − y1 ) + x 3 ( y1 − y 2 )] Then, the points A, B and P are collinear. Therefore, the
2
area of ∆ABP will be zero.
2. Condition of collinearity of three points Let three 3 1 1
points be A ( x1 , y1 ), B( x 2 , y 2 ) and C( x 3 , y 3 ), then 1
∴ 9 3 1 =0
these points will be collinear, if Area of ∆ABC = 0 2
x y 1
3. Equation of straight line passing through two points 1
Let two points be A ( x1 , y1 ) and B( x 2 , y 2 ) and ⇒ |3( 3 − y) − 1(9 − x) + 1(9y − 3x)| = 0
2
P( x, y) be a point on the line joining points A and B, ⇒ 9 − 3y − 9 + x + 9y − 3x = 0
then the equation of line is given by ⇒ 6y − 2x = 0
x y 1 ⇒ x − 3y = 0
x1 y1 1 =0 Hence, the equation of the line joining the given points is
x − 3y = 0.
x2 y2 1

PRACTICE EXERCISE
 1 sin α 1  5. If p + q + r = a + b + c = 0, then the value of
1. Let ∆ = − sin α 1 sin α , then ∆ lies in  pa qb rc 
 
 qc pb is
 −1 − sin α 1 

ra

the interval  rb pc qa 
(a) [2, 3] (b) [3, 4] (c) [1, 4] (d) [2, 4] (a) 0 (b) ap + bq + cr
2. If f( x ) = x − 4x − 5, then f ( A),
2 (c) 1 (d) None of these

1 2 2  1! 2! 3 !
where A = 2 1 2 , is equal to 6. The value of the determinant ∆ = 2 ! 3 ! 4 ! is
 
2 2 1  3! 4! 5 !
(a) 0 (b) I (c) −I (d) 2I (a) 2! (b) 3! (c) 4! (d) 5!
 x + 2x2
2x + 1 1 7. If A, B and C are the angles of a triangle, then
3. If

2x + 1 x+2

1 = ( x − 1)k , then k equals to

 −1 cos C cos B
  the value of ∆ = cos C −1 cos A is
 3 3 1
 
(a) 1 (b) 2 (c) 3 (d) 4  cos B cos A −1 

 2  (a) cos A cos B cos C (b) sin A sin B sin C


1 a a
4. If ∆ =  (c) 0 (d) None of these
cos ( n − 1) x cos nx cos ( n + 1) x , then ∆

 
 x α 1
 sin ( n − 1) x sin nx sin ( n + 1) x 
8. The roots of the equation β x 1 = 0 are
is β γ 1
(a) independent of x (b) independent of a independent of
(c) independent of n (d) None of these (a) α (b) β (c) γ (d) α, β and γ
MATHEMATICS Determinants 121

9. If 5 and 7 are the roots of the equation


x 4 5 17. If ( a1 / x ) + ( b1 / y ) = c1 , ( a2 / x ) + ( b2 / y ) = c2
7 7 = 0, then what is the third root? a b1  b c1  c a1 
∆ 1 = 1 , ∆ 2 = 1 , ∆3 = 1 ,
x
 
5 8 x  a2 b2   b2 c2   c2 a2 
(a) −12 (b) 9 (c) 13 (d) 14
then ( x , y ) is equal to which one of the following?
 1− i ω2 −ω 
(a) (∆ 2 / ∆1, ∆ 3 / ∆1 ) (b) (∆ 3 / ∆1, ∆ 2 / ∆1 )
10. What is the value of 

ω2 + i ω

−i ,
 (c) (∆1 / ∆ 2 , ∆1 / ∆ 3 ) (d) (− ∆1 / ∆ 2 , − ∆1 / ∆ 3 )
 
 1 − 2i − ω
2
ω −ω
2 i − ω 2 − x 1 1
where ω is the cube root of unity? 
18. If the matrix A =  1 3 − x 0  is singular,
(a) −1 (b) 1 (c) 2 (d) 0  −1 −3 − x
y y + z
x
then what is the solution set S?
11. If  z x + y  = 0, then which one of the
y (a) S = {0, 2, 3} (b) S = {−1, 2, 3}
 
x z + x
z (c) S = {1, 2, 3} (d) S = {2, 3}
following is correct? −1
1 − tan θ  1 tan θ a − b
19. If  =
1 − tan θ 1 ,
(a) Either x + y = z or x = y (b) Either x + y = − z or x = z
(c) Either x + z = y or z = y (d) Either z + y = x or x = y tan θ   b a
then
k b+ c b2 + c2  (a) a = 1, b = 1 (b) a = cos 2 θ, b = sin2 θ
 
12. What is the value of k, if k c+a c2 + a 2  (c) a = sin2 θ, b = cos 2 θ (d) None of these
k a+b 
 a 2 + b2  20. If a matrix A is such that 3 A3 + 2 A2 + 5 A + I = O ,
= ( a − b) ( b − c) ( c − a )? then what is A−1 equal to?
(a) 1 (b) −1 (c) 2 (d) 0 (a) − (3 A 2 + 2 A + 5) (b) 3 A 2 + 2 A + 5I

13. If T p , Tq , Tr are pth, qth and rth terms of an AP, (c) 3 A − 2 A − 5I


2
(d) − (3 A 2 + 2 A + 5I)

 Tp Tq Tr   cos x sin x 0
then p q r  is equal to 21. If A = − sin x cos x 0 = f ( x ), then A−1 equals

 
  0 0 1
 1 1 1
(a) 1 (b) −1 (c) 0 (d) p + q + r (a) f(− x ) (b) − f(x )
a1 b1 c1 (c) − f(− x ) (d) f(x )

14. If ∆ = a2 b2 c2 and A1 , B1 , C1 denote the 22. If l + m + n = 0, then the system of equations


a3 b3 c3 −2x + y + z = l, x − 2y + z = m, x + y − 2z = n has
(a) a trivial solution
cofactors of a1 , b1 , c1 respectively, then the value (b) no solution
A1 B1 C1 (c) a unique solution
(d) infinitely many solutions
of the determinant A2 B2 C2 is
A3 B3 C3 23. Consider the system of linear equations
a1x + b1 y + c1z + d1 = 0, a2x + b2 y + c2z + d2 = 0
(a) ∆ (b) ∆2 (c) ∆3 (d) 0
and a3 x + b3 y + c3 z + d3 = 0
15. What is the value of Let us denote by ∆ ( a , b, c) the determinant
 cos 15° sin 15°   cos 45° cos 15° 
  × ?  a1 b1 c1 
 45° sin 45°   sin 45° sin 15°  a b2 c2 
cos
1 3 1 3  2 
(a) (b) (c) − (d) −  a3 b3 c3 
4 2 4 4
If ∆ ( a , b, c) ≠ 0, then the value of x has a unique
16. If f ( x ) = ax 4 + bx3 + cx 2 + dx + e solution of the above equation, then
x3 + 3x x−1 x + 3 ∆ (b, c, d ) ∆ (b, c, d )
(a) (b) −
  ∆ (a, b, c ) ∆ (a, b, c )
= x+1 −2x x − 4 , then e equals

 
 ∆ (a, c, d ) ∆ (a, b, d )
(d) −
 x− 3 x+ 4 3x  (c)
∆ (a, b, c ) ∆ (a, b, c )
(a) 1 (b) −1 (c) 2 (d) 0
122 NDA/NA Pathfinder

1+ a 1 1 d
29. The ∆1 is
24. If ∆ = 1 1+ b 1 , then consider the dx
1 1 1+ c (a) ∆ 2 (b) ∆ 2 /2
(c) 3 ∆ 2 (d) None of these
following statements
I. If
1 1 1
+ + = 0, then ∆ = abc. 30. If ∆1 − x∆ 2 + a 2b = 0, then x equals to
a b c a + 2b 2a + b
(a) (b)
II. If a −1 + b−1 + c−1 = − 1, then ∆ = 0. 2 2
a+ b
Which of the above statement(s) is/are correct? (c) (d) None of these
(a) Only I (b) Only II 2
(c) Both I and II (d) Neither I nor II
Directions (Q. Nos. 31-32) Consider the determinant
a b aα + b
x 2 − 5x + 3 2x − 5 3
25. If the determinant b c bα + c is equal
f ( x ) = 3x 2 + x + 4 6x + 1 9
aα + b bα + c 0
7x − 6 x + 9 14x − 6 21
2
to zero, then consider the following statements.
I. a, b, c are in AP. 31. The value of f( 0) is
II. α is a root of the equation ax2 + bx + c = 0. (a) 93 (b) 120 (c) 141 (d) 0
Which of the above statement(s) is/are correct?
32. The value of f ′ ( 0) is
(a) Only I (b) Only II
(a) 0 (b) 7 (c) 19 (d) − 1
(c) I and II (d) Neither I nor II

26. If A and B are square matrices such that Directions (Q. Nos. 33-34) Let
B = A−1BA, then consider the following statements  sin θ i cos θ
A(θ) =   , where i = − 1
I. AB + BA = 0 II. A − B = ( A + B)( A − B) i cos θ sin θ 
2 2

Which of the above statement(s) is/are correct?


π 
(a) Only I (b) Only II 33. If B (θ) = A − θ , then AB equals
  2
(c) Both I and II (d) Neither I nor II
0 i  0 − i  1 0
(a)  (b)  (c)   (d) None of these
27. Let A be a 2 × 2 matrix with non-zero entries and 
 i 0 − i 0   0 1
let A2 = I , where I is 2 × 2 identity matrix, then
34. Consider the following statements
consider the following statements
I. A(θ ) is invertible for all θ ∈ R II. A (θ )−1 = A (− θ )
I. Sum of diagonal elements of A is 0.
II. Determinant of matrix A is 1. Which of the above statement(s) is/are correct?
(a) Only I (b) Only II
Which of the above statement(s) is/are correct?
(c) Both I and II (d) Neither I nor II
(a) Only I (b) Only II
(c) Both I and II (d) Neither I nor II Directions (Q. Nos. 35-37) Consider the determinant
α β γ
28. Which of the following determinants is purely
real? ∆= β γ α
2 1+ i 3 1 i 2 γ α β
I. 1 − i 0 2+ i II. − i 2 i
35. If α , β , γ are the roots of x3 + ax 2 + b = 0, then the
3 2−i 1 0 i −2
determinant ∆ equals
6 3−i 2 2+ i 6 (a) − a3 (b) a3 − 3b
III. 3 + i 2 0 3−i 8 (c) a − 3b
2
(d) a3
2−i 6 3+ i 8 11
36. If α, β, γ are the roots of x3 + bx + c = 0, then the
(a) I and II (b) II and III (c) I and III (d) I, II and III determinant ∆ equals
(a) − b 3 (b) b 3 − 3c
Directions (Q. Nos. 29-30) Given two determinants (c) b − 3c
2
(d) 0
x b b
x b 37. If α, β, γ are negative distinct real numbers, then
∆1 = a x b and ∆ 2 = .
a x (a) ∆ < 0 (b) ∆ ≤ 0
a a x (c) ∆ ≥ 0 (d) ∆ > 0
MATHEMATICS Determinants 123

Directions (Q. Nos. 38-40) Consider the determinant,


p q r
48. If a matrix A has inverse B and C, then which
one of the following is correct? e 2012 I
∆ = x y z ; Mij denotes the minor of an element (a) B may not be equal to C
l m n (b) B should be equal to C
in ith row and jth column; Cij denotes the cofactor (c) B and C should be unit matrices
of an element in ith row and jth column. (d) None of the above

38. The value of p ⋅ C21 + q ⋅ C22 + r ⋅ C23 is 49. If A is a square matrix such that A2 = I , where I
is the identity matrix, then what is the value of
(a) 0 (b) − ∆ (c) ∆ (d) ∆2
A−1 ? e 2012 I
39. The value of x ⋅ C21 + y ⋅ C22 + z ⋅ C23 is (a) A + i (b) Null matrix (c) A (d) Transpose of A
(a) 0 (b) − ∆ (c) ∆ (d) ∆2 1 2 0 −1
50. If A =   and B =   , then what is the
1 1 1 2
40. The value of q ⋅ M12 − y M 22 + m ⋅ M32 is value of B−1 A−1 ? e 2012 I
(a) 0 (b) − ∆ (c) ∆ (d) ∆2  1 −3  −1 3  −1 3  −1 −3
(a)   (b)   (c)   (d)  
 −1 2   1 −2   −1 −2   1 −2 
Directions (Q. Nos. 41-43) A and B are two
matrices of same order 3 × 3, where 51. If each element in a row of a determinant is
multiplied by the same factor r, then the value of
1 2 3 3 2 5 the determinant e 2012 II
A = 2 3 4 and B = 2 3 8
  (a) is multiplied by r 3 (b) is increased by 3r
5 6 8 7 2 9 (c) remains unchanged (d) is multiplied by r

41. The value of adj ( adj A) equals x2 1 y2 + z 2


(a) − A (b) 4A (c) 8A (d) 16A 52. The value of the determinant y 2
1 z 2 + x 2 is
42. The value of|adj ( B)| equals z2 1 x 2 + y2
(a) 24 (b) 242 (c) 243 (d) 82 e 2012 II
(a) 0 (b) x 2 + y2 + z2
43. The value of|( adj ( adj ( adj ( adj A))))| equals
(c) x 2 + y2 + z2 + 1 (d) None of these
(a) 2 4 (b) 2 9 (c) 1 (d) 219
 α 2 2
PREVIOUS YEARS’ QUESTIONS 53. If the matrix −3 0 4 is not invertible, then
 1 −1 1 e 2012 II
44. If two rows of a determinant are identical, then (a) α = − 5 (b) α = 5 (c) α = 0 (d) α = 1
what is the value of the determinant?
(a) 0 (b) 1 e 2012 I 54. The inverse of a diagonal matrix is a e 2012 II
(c) −1 (d) can be any real value (a) symmetric matrix (b) skew-symmetric matrix
1 2 1 0 (c) diagonal matrix (d) None of these
45. If A = and B = , then what is the
2 3 1 0 55. The determinant of a orthogonal matrix is
value of determinant of AB? e 2012 I e 2013 I
(a) 0 (b) 1 (a) ± 1 (b) 2 (c) 0 (d) ± 2
(c) 10 (d) 20
m n p
8 −5 1
56. The value of the determinant p m n
46. If 5 x 1 = 2, then what is the value of x?
n p m
6 3 1
e 2013 I
(a) 4 (b) 5 (a) is a perfect cube (b) is a perfect square
(c) 6 (d) 8 e 2012 I (c) has linear factor (d) is zero
−a 2
ab ac 1 t−1 1
47. What is the value of ab −b 2
bc ? 57. The roots of the equation t − 1 1 1 =0
ac bc − c2 1 1 t−1
e 2012 I
(a) 4abc (b) 4a 2 bc are e 2013 I
(c) 4a 2 b 2 c 2 (d) − 4a 2 b 2 c 2
(a) 1, 2 (b) −1 , 2 (c) 1 , − 2 (d) −1 , − 2
124 NDA/NA Pathfinder

58. If D is determinant of order 3 and D′ is the 66. Consider the following statements
determinant obtained by replacing the elements 1 2 1
of D by their cofactors, then which one of the I. The matrix a 2a 1 is singular.
following is correct? e 2013 I  
 b 2b 1
(a) D′ = D2 (b) D′ = D3
 c 2c 1
(c) D′ = 2 D 2
(d) D′ = 3D3
II. The matrix a 2a 1 is non-singular.
 
59. Consider the following statements  b 2b 1
I. A matrix is not a number. Which of the above statement(s) is/are correct?
II. Two determinants of different orders may have e 2013 II
the same value. (a) Only I (b) Only II
Which of the above statement(s) is/are correct? (c) Both I and II (d) Either I or II
(a) Only I (b) Only II e 2013 I x + a b c 
(c) Both I and II (d) Neither I nor II 67. One of the roots of a x+b c  = 0 is
 
60. What is the value of the minor of the element 9  a b x + c e 2014 I
10 19 2
(a) abc (b) a + b + c
in the determinant 0 13 1 ?
(c) −(a + b + c ) (d) −abc
9 24 2 e 2013 I 68. The determinant of an odd order skew-symmetric
(a) – 9 (b) – 7 (c) 7 (d) 0
matrix is always e 2014 I
61. If A and B are two non-singular square matrices (a) zero (b) one
such that AB = A, then which one of the (c) negative (d) depends on the matrix
following is correct? e 2013 I 69. If any two adjacent rows or columns of a
(a) B is an identity matrix (b) B = A −1 determinant are interchanged in position, the
(c) B = A 2 (d) Determinant of B is zero value of the determinant e 2014 I
(a) becomes zero (b) remains the same
62. The cofactor of the element 4 in the determinant (c) changes its sign (d) is doubled
1 2 3
70. Consider two matrices
4 5 6 is
1 2
1 2 −4
A = 2 1 and B =
5 8 9 e 2013 II
2 1 −4
(a) 2 (b) 4 1 1  
(c) 6 (d) − 6
Which one of the following is correct? e 2014 I
63. What is the value of the determinant (a) B is the right inverse of A
1 bc a ( b + c) (b) B is the left inverse of A
(c) B is the both sided inverse of A
1 ca b ( c + a ) ?
(d) None of the above
1 ab c ( a + b) e 2013 II
71. Consider the following statements in respect of
(a) 0 (b) abc  0 1 2
(c) ab + bc + ca (d) abc(a + b + c ) the matrix A = −1 0 −3
64. If A is a square matrix of order 3 with | A| ≠ 0, −2 3 0
then which one of the following is correct? I. The matrix A is skew-symmetric.
e 2013 II II. The matrix A is symmetric.
(a)|adj = A| | A| (b)|adj A| = | A|2 III. The matrix A is invertible.
(c)|adj A | = | A |3 (d)|adj A|2 =| A|
Which of the above statement(s) is/are correct?
2 1 −1
e 2014 I
65. If 2 A =  , then what is A equal to? (a) Only I (b) Only III (c) I and II (d) II and III
3 2 e 2013 II
72. If A and B are square matrices of second order
 2 −1 1  2 −1 such that | A| = − 1 and | B| = 3, then what is
(a)   (b)
 −3 2  2  −3 2  | 3AB| equal to? e 2014 II
1  2 −1 (a) 3 (b) −9
(c)  (d) None of these
4  −3 2  (c) −27 (d) None of these
MATHEMATICS Determinants 125

1 3 1 1 81. Consider the following in respect of two


73. If the matrix A is such that   A = 0 −1, non-singular matrices A and B of same order
0 1  
then what is A equal to? e 2014 II I. det ( A + B) = det A + det B
 1 4  1 4 II. ( A + B)−1 = A −1 + B−1
(a)   (b)  
 0 −1  0 1 Which of the above statement(s) is/are correct?
 −1 4  1 −4 (a) Only I (b) Only II e 2015 I
(c)   (d)   (c) Both I and II (d) Neither I nor II
 0 −1  0 −1
2 7 −1
74. From the matrix equation AB = AC , where A, B 82. If A =  , then what is A + 3 A equal to?
and C are the square matrices of same order, we 1 5 e 2015 I
(a) 3I (b) 5I (c) 7I (d) None of these
can conclude B = C provided e 2014 II
(a) A is non-singular (b) A is singular Where, I is the identity matrix of order 2.
(c) A is symmetric (d) A is skew-symmetric 83. If A is an invertible matrix of order n and k
75. If A is an invertible matrix, then what is det ( A−1 ) is any positive real number, then the value of
equal to? e 2014 II
[det ( kA)]−1 det (A) is e 2015 II
1 (a) k − n (b) k −1 (c) k n (d) nk
(a) det (A) (b)
det ( A) a 1 1
(c) 1 (d) None of these 84. If the value of the determinant 1 b 1 is
 6i −3i
1 1 1 c
76. If  4 −1 = x + iy , where i = −1, then
3i positive, where a ≠ b ≠ c, then the value of abc
 
e 2015 II
 20 3 i 
(a) cannot be less than 1 (b) is greater than − 8
what is x equal to? e 2014 II
(c) is less than − 8 (d) must be greater than 8
(a) 3 (b) 2
(c) 1 (d) 0 85. If a, b and c are real numbers, then the value of
 a b 0 1− a a− b− c b+ c
77. If  0 a b  = 0, then which one of the following the determinant 1 − b b − c − a c + a is
  1− c c−a−b a+b
 b 0 a e 2015 II
is correct? e 2014 II
(a) 0 (b) (a − b )(b − c )(c − a)
(a) a / b is one of the cube roots of unity (c) (a + b + c )2 (d) (a + b + c )3
(b) a / b is one of the cube roots of −1
(c) a is one of the cube roots of unity
86. Consider the following statements in respect of
α α
(d) b is one of the cube roots of unity cos2 sin2
the determinant 2 2 where α, β are
78. Consider the following statements 2β 2 β
sin cos
I. Determinant is a square matrix. 2 2
II. Determinant is a number associated with a complementary angles.
square matrix. 1  α − β
I. The value of the determinant is cos  .
Which of the above statement(s) is/are correct? 2  2 
(a) Only I (b) Only II e 2014 II 1
(c) Both I and II (d) Neither I nor II II. The maximum value of the determinant is .
2
79. If a ≠ b ≠ c all are positive, then the value of Which of the above statement(s) is/are correct?
a b c (a) Only I (b) Only II e 2015 II
determinantb c a is (c) Both I and II (d) Neither I nor II
 
c a b 1 3 2 
e 2014 II
(a) non-negative (b) non-positive 87. The matrix A = 1 x − 1 1  will have inverse

(c) negative (d) positive
2 7 x − 3
1 1 1 
for every real number x except for e 2015 II

80. The value of 1 1 + x 1  is 11 ± 5 9± 5
  (a) x = (b) x =
1 1 1+ y e 2015 I
2 2
(a) x + y (b) x − y 11 ± 3 9± 3
(c) x = (d) x =
(c) xy (d) 1 + x + y 2 2
126 NDA/NA Pathfinder

88. If A is an orthogonal matrix of order 3 and 90. What is the value of f ′ ( 0)?
 1 2 3 (a) p3 (b) 3 p3 (c) 6 p3 (d) −6 p3
B = − 3 0 2, then which of the following is/are
 2 5 0 91. What is the value of p for which f ′ ′ ( 0) = 0 ?
1 1
correct?. (a) − or 0 (b) −1 or 0 (c) − or 1 (d) −1 or 1
6 6
I. | AB| = ± 47 II. AB = BA 92. Which of the following determinants have value
Select the correct answer using the code given ‘zero’?
below. e 2015 II
 41 1 5  1 a b + c   0 c b
(a) Only I (b) Only II
I.  79 7 9    
II. 1 b c + a III. − c 0 a
(c) Both I and II (d) Neither I nor II      
 29 5 3  1 c a + b   −b −a 0 
89. If A is a square matrix, then what is Select the correct answer using the code given
adj ( A−1 ) − ( adj A)−1 equal to? e 2016 I below. e 2016 I
(a) 2| A | (b) Null matrix (a) I and II (b) II and III (c) I and III (d) I, II and III
(c) Unit matrix (d) None of these
93. The system of linear equations kx + y + z = 1,
Directions (Q. Nos. 90-91) Consider the function x + ky + z = 1 and x + y + kz = 1 has a unique
x solution under which one of the following
3
sin x cos x 
  conditions? e 2016 I
f (x ) = 6 −1 0 , where p is a constant. (a) k ≠ 1 and k ≠ − 2 (b) k ≠ 1 and k ≠ 2

 3 

p2 (c) k ≠ − 1 and k ≠ − 2 (d) k ≠ − 1 and k ≠ 2
p p 
e 2016 I

ANSWERS
1 d 2 a 3 c 4 c 5 a 6 c 7 c 8 a 9 a 10 d
11 b 12 a 13 c 14 b 15 c 16 d 17 d 18 a 19 b 20 a
21 a 22 d 23 a 24 c 25 d 26 b 27 a 28 d 29 c 30 b
31 c 32 a 33 a 34 a 35 d 36 d 37 c 38 a 39 c 40 b
41 a 42 b 43 c 44 a 45 a 46 d 47 c 48 b 49 c 50 b
51 d 52 a 53 a 54 c 55 a 56 c 57 b 58 a 59 c 60 b
61 a 62 c 63 a 64 b 65 d 66 a 67 c 68 a 69 c 70 b
71 a 72 c 73 a 74 a 75 b 76 d 77 b 78 b 79 c 80 c
81 d 82 c 83 a 84 b 85 a 86 c 87 a 88 a 89 b 90 d
91 a 92 d 93 a

HINTS AND SOLUTIONS


1. (d) By expansion ∆ = 2 + 2 sin 2α 4. (c) We have, = ( 1 − 2a cos x + a 2 )[cos nx ⋅ sin( n + 1)x
∴ ∆ ∈[ 2, 4] as sin α ∈ [ 0, 1]
2 2 − sin nx ⋅ cos ( n + 1)x ]
1 a a
2. (a) Given, f ( x ) = x 2 − 4x − 5 ∆ = cos ( n − 1)x cos nx cos ( n + 1)x = ( 1 − 2a cos x + a 2 ) sin x
∴ f ( A) = A2 − 4 A − 5 sin ( n − 1)x sin nx sin ( n + 1)x ∴ ∆ is independent of n.
Here, | A |= 5 5. (a) Since, a + b + c = 0 = p + q + r
∴ f ( A ) = 25 − 20 − 5 = 0 Since, cos ( n − 1)x + cos ( n + 1)x
= 2 cos nx ⋅ cos x ∴ a 3 + b 3 + c 3 = 3abc
3. (c) Applying R → R − R and
1 1 2 and sin ( n − 1)x + sin ( n + 1)x and p 3 + q 3 + r 3 = 3 pqr
R → R − R and taking ( x − 1)
2 2 3
= 2 sin nx ⋅ cos x Expanding the given determinant along
common from R and R , we get
1 2
Applying C → C − 2 cos x ⋅ C + C , first row, we get
x + 1 1 0 we get
1 1 2 3
∆ = pa [ qra 2 − p 2 bc ] − qb[ q 2 ac − b 2 pr ]
 
∆ = ( x − 1)2  2 1 0  = ( x − 1)3∴ + rc [ c 2 pq − r 2 ab ]
1 − 2a cos x + a 2 a a2
  ∆ = pqr ( Σa ) − abc ( Σp )
3 3
3 3 1 ∴∆ = 0 cos nx cos ( n + 1)x
  = pqr (3abc ) − abc (3 pqr ) = 0
k =3 0 sin x sin ( n + 1)x
MATHEMATICS Determinants 127

1! 2! 3! y x y+z a b c A B C
1 1 1 1 1 1
6. (c) ∆ = 2! 3 ⋅ ( 2!) 4 ⋅ (3!) 11. (b) Let ∆ = z y x + y 14. (b) Q ∆∆′ = a b c A B C
2 2 2 2 2 2
3! 4 ⋅ 3 ⋅ ( 2!) 5 ⋅ 4 ⋅ (3!) x z z+ x a b c A B C
3 3 3 3 3 3

1 1 1 Applying R → R + R + R , and Σa1 A1 0 0


1 1 2 3
= ( 2!)(3!) 2 3 4 taking ( x + y + z ) common from R = 0 Σa A 0
1 2 2
6 12 20 1 1 2 0 0 Σa A
3 3

= ( 2!)(3!)[( 60 − 48) − ( 40 − 24) ∆ = ( x + y + z) z y x + y


∆ 0 0
+ ( 24 − 18)] x z z+ x
= 0 ∆ 0 = ∆3
= ( 2!)(3!)( 2) = 4!
Applying C → C − (C + C ), we get 0 0 ∆
3 3 1 2
7. (c) In a triangle, A + B + C = π 1 1 0 ∴ ∆′ = ∆ 2
∴ cos ( A + B ) = cos ( π − C ) = − cos C ∆ = ( x + y + z) z y x − z ,
⇒ cos A cos B + cos C = sin A sin B cos 15° sin 15°
x z 0 15. (c) ∆ =
and sin ( A + B ) = sin C cos 45° sin 45°
∆ = ( x + y + z )( x − z )2
Expanding the given determinant, cos 45° cos 15°
Thus, ∆=0 ×
∆ = − ( 1 − cos 2 A ) sin 45° sin 15°
+ cos C [cos C + cos A cos B ] ⇒ x + y + z = 0 or ( x − z )2 = 0
+ cos B [cos B + cos A cos C ] ⇒ x + y = − z or x = z cos 15°⋅ cos 45° + sin 15°⋅ sin 45°
= − sin 2 A + cos C (sin A sin B ) =
k b+c b +c 2 2
cos 2 45° + sin 2 45°
+ cos B (sin A sin C )
12. (a) Let ∆ = k c + a c2 + a 2
= − sin 2 A + sin A sin ( B + C ) cos 2 15° + sin 2 15°
k a + b a2 + b2
= − sin 2 A + sin 2 A = 0
cos 45°⋅ cos 15° + sin 45°⋅ sin 15°
x α 1  1 0  0
8. (a) Given, β x 1 = 0 = k b + c a−b a−c  cos ( 45° − 15° ) 1

2 
 =
β γ 1 b + c2 a − b2
2
a + b2
2
1 cos ( 45° − 15° )
 
Applying R → R − R Applying C → C − C and 3
2 2 1 2 2 1 1
and R → R − R , we get C → C − C , we get = 2 =
3
− 1= −
1
3 3 1 3 3 2
3 4 4
x α 1  1 0 0  1
 2
β−x x −α 0 = 0 =k b+c a−b a−c

 2 
 16. (d ) Putting x = 0 in given equation,
β−x γ −α 0 b + c a −b
2 2 2
a 2 − c2
 
0 −1 3
Expanding with respect to C , = k( a − b ) ( b − c ) ( c − a )
3 we get e= 1 0 −4 = 0
(β − x ) ( γ − α) − ( x − α) (β − x ) = 0 But ∆ = ( a − b ) ( b − c ) ( c − a )
⇒ (β − x ) { − α + γ − x + α } = 0 Thus, k = 1 −3 4 0
⇒ (β − x ) ( γ − x ) = 0 13. (c) Let a be the first term and d be [since, skew-symmetric determinant of
∴ x = β, γ the common difference of given AP. odd order is zero]
So, roots of the given equation is Then, 1 1
17. (d ) Let = u , = v,
independent of α. T = a + ( p − 1) d , Tq = a + ( q − 1) d , x y
p
x 4 5 a u + b v = c and a u + b v = c
Tr = a + ( r − 1) d 1 1 1 2 2 2
9. (a) Given determinant is, 7 x 7 = 0
∴T − Tr = ( p − r )d and Tq − Tr = ( q − r )d Using the method of cross-multiplication,
p
5 8 x
Applying C → C − C and u v −1
1 1 3 = =
Expanding with respect to R , C → C − C in given determinant, b c −b c c a −c a a b −a b
1 2 2 3 1 2 2 1 1 2 2 1 1 2 2 1
we have
x ( x 2 − 56) − 4( 7x − 35) +5(56 − 5x ) = 0 1 1
⇒ x 3 − 109x + 420 = 0 ( p − r )d ( q − r )d Tr y −1
⇒ = x =
If (5, 7) are the roots of above equation, ∆= p−r q−r r  b1 c   c1 a
1   a1 b 

1
   1
then 0 0 1
b c   c2 a
2 a b

x 2 ( x − 5) + 5x ( x − 5) − 84( x − 5) = 0  2 2
  2 2
d d Tr 1 1
⇒ ( x − 5) ( x 2 + 5x − 84) = 0 −1
= ( p − r )( q − r ) 1 1 r ⇒ x =
y
=
⇒ ( x − 5) ( x − 7) ( x + 12) = 0
0 0 1 ∆ ∆ ∆
⇒ x = 5, 7, − 12 2 3 1

10. (d) =0 [Q C and C are identical]


1 2
128 NDA/NA Pathfinder

1 ∆ 1 ∆  cos x − sin x 0 Applying R → R − (αR + R ), we get


∴ =− 2 and =− 3   3 3 1 2
x ∆ y ∆ =  sin x cos x 0
1 1 a b aα + b
∆ ∆  0
⇒ x =− 1 and y=− 1 0 1 ∆= b c bα + c
∆ ∆  
2 3  cos ( − x ) sin ( − x ) 0 0 0 − ( aα + 2bα + c )
2

18. (a) For the singular matrix,  


=  − sin ( − x ) cos ( − x ) 0 = f ( − x ) = − ( aα + 2bα + c )( ac − b 2 )
2

2−x 1 1  0 0 1 Hence, ∆ is zero, if ac − b 2 = 0


  or aα 2 + 2bα + c = 0
1 3−x 0 =0
 −2 1 1 ⇒ a, b, c are in GP.
−1 −3 −x   or α is a root of ax 2 + 2bx + c = 0
22. (d) ∆ =  1 −2 1
⇒ ( 2 − x ) [ x ( x − 3)] − [ − x ] So, both statements are not correct.
 1 1 −2 
+ [ −3 + (3 − x )] = 0   26. (b) B = A −1 BA
⇒ x ( x − 3) ( x − 2) = 0 ⇒ x = 0, 2, 3
= −2( 4 − 1) − 1( −2 − 1) + 1 + 2 = 0 ⇒ AB = AA −1 BA ⇒ AB = I ( BA )
So, the solution set is, S = { 0, 2, 3}.
−1 l 1 1 l +m+ n 0 0 ⇒ AB = ( BA ) ⇒ AB − BA = 0
 1 − tan θ  1 tan θ
19. (b)  ∆ = m −2 1 = −2 1 ∴ Statement I is wrong.
tan θ 1   − tan θ 1  m
Now, ( A + B )( A − B )
    n 1 −2 n 1 −2
 a − b  = A( A − B ) + B( A − B )
= [R → R + R + R ] = A 2 − AB + BA − B 2
b a 1 1 2 3
  =0 [Q l + m + n = 0] = A 2 − AB + AB − B 2 [Q AB = BA]
 1 − tan θ 1 Similarly, ∆ = ∆ = 0 = A2 − B2
⇒ 
1  1 + tan 2 θ
2 3
tan θ Hence, the given system of equations ∴ Statement II is correct.
 
has infinitely many solutions. a b
 1 − tan θ  a − b  27. (a) Let A = 
 tan θ = c d
1  b a   
d b c
1 1 1
    d b c
1 2 2 2 a b  a b 
Then, A 2 = 
⇒ d b c c d c d
1 + tan 2 θ 23. (a) We know that, x =
3 3 3    
a b c  a 2 + bc ab + bd 
 1 − tan θ −2 tan θ   a − b 
2
1 1 1
 = = 2
a
a b c
 2 tan θ 1 − tan 2 θ  b 
2 2 2
 ac + cd bc + d 
  a
3
b
3
c
3
 1 − tan θ2
−2 tan θ  ∆ ( d, b, c ) ∆ ( b, c, d ) Given, A 2 = I
 1 + tan 2 θ 1 + tan 2 θ   a − b  = =
∆ ( a, b c ) ∆ ( a, b, c )  a 2 + bc ab + bd   1 0
⇒  = 
 2 tan θ b a ⇒  2 =  
1 − tan 2 θ    1+ a 1 1  ac + cd bc + d   0 1
 
 1 + tan θ 1 + tan θ 
2 2
24. (c) We have, ∆ = 1 1+ b 1
⇒ b( a + d ) = 0 and c ( a + d ) = 0
 cos 2 θ − sin 2 θ  a − b  1 1 1+ c
⇒  = ⇒ a+ d =0 [Q b ≠ 0, c ≠ 0]
sin 2 θ cos 2 θ  b a
    Expanding with respect to R1 , So, sum of diagonal elements of A is 0.
⇒ a = cos 2 θ, b = sin 2 θ ∆ = ( 1 + a )[( 1 + b )( 1 + c ) − 1] So, Statement I is correct.
− 1( 1 + c − 1) + ( 1 − 1 − b )
20. (a) Q 3 A + 2 A + 5 A + I = O
3 2
= ( 1 + a )( b + c + bc ) − c − b Now, | A| = ad − bc = − a 2 − bc
⇒ 3 A 3 A −1 + 2 A 2 A −1 = bc + ab + ac + abc = − ( a 2 + bc ) = − 1 [Q a 2 + bc = 1]
+ 5 AA −1 + IA −1 = O = abc  + +  + abc
1 1 1 So, Statement II is incorrect.
⇒ 3 A 2 + 2 A + 5 + A −1 = O  a b c 2 1+ i 3
⇒ A −1 = − ( 3 A 2 + 2 A + 5 ) 1 1 1
If + + = 0, 28. (d) I. Let z = 1 − i 0 2+i
cos x sin x 0 a b c 3 2−i 1
then ∆ = abc ( 0) + abc = abc
21. (a) | A | = − sin x cos x 0 ∴ Statement I is correct. 2 1−i 3
0 0 1 1 1 1
Now, + + =−1 Then, z = 1 + i 0 2−i
a b c
= cos 2 x + sin 2 x = 1 ≠ 0 ⇒ ∆ = abc( − 1) + abc = 0 3 2+i 1
Now, c = cos x , c = sin x , c =0 ∴ Statement II is correct.
11 12 13 2 1+ i 3
c = − sin x , c = cos x , c =0 25. (d) We have,
21 22 23 = 1− i 0 2+ i
c = 0, c = 0, c =1
31 32 33 a b aα + b 3 2− i 1
1
∴ A −1 = adj ( A ) = adj ( A ) ∆= b c bα + c
|A| [interchanging rows and columns]
aα + b bα + c 0
⇒ z = z ⇒ z is purely real.
MATHEMATICS Determinants 129

1 i 2 2x − 5 2x − 5 3 35. (d ) We have, α + β + γ = −a and


βγ + γα + αβ = 0
II. Let ∆ = − i 2 i 32. (a) f ′( x ) = 6x + 1 6x + 1 9
0 i −2 14x − 6 14x − 6 21 1 β γ
∴ ∆ = −a 1 γ α
= 1( − 4 − i 2 ) − i ( 2i − 0) +2( − i 2 − 0) x 2 − 5x + 3 2 3 1 α β
= −3+ 2+ 2= 1 + 3x 2 + x + 4 6 9
So, ∆ is purely real. 1 β γ
7x − 6x + 9 14 21
2

III. Let = −a 0 γ − β α − γ
= 0+ 0 = 0
6 3−i 2 2+i 6 π 0 α −β β − γ
33. (a) We have, B(θ) = A  − θ
z = 3+i 2 0 3 −i 8 2 
[C → C − C and C → C − C ]
 sin  π − θ i cos  π − θ 
2 2 1 3 3 1
2 −i 6 3 − i 8 11
     = − a[ − ( γ − β )2 − (α − γ )(α − β )]
2  2 
=  = a [α 2 + β 2 + γ 2 − (βγ + αγ + αβ )]
Then,  π   π 
3+i 2 2−i 6 i cos  − θ sin  − θ 
6  2 2  = a[(α + β + γ )2 − 3(βγ + αγ + αβ )]
z= 3−i 2 0 3+i 8  cos θ i sin θ = a [( − a )2 − 3( 0)] = a 3
=
2+i 6 3−i 8 i sin θ cos θ  36. (d) We have, α + β + γ = 0
11  
Now, 1 β γ
6 3−i 2 2+i 6
 sin θ i cos θ  cos θ i sin θ ∴ ∆ = (α + β + γ ) 1 γ α = 0
AB = 
= 3+i 2 0 3−i 8 i sin θ sin θ  i sin θ cos θ  1 α β
   
2−i 6 3+ i 8 11  sin θ cos θ + i 2 sin θ cos θ
[interchanging rows and columns] = 37. (c) On expanding, we get
 i cos 2 θ + i sin 2 θ 1
=z ∆ = − (α + β + γ )[(α − β )2 + (β − γ )2
2

i sin 2 θ + i cos 2 θ + ( γ − α )2 ]
So, z is purely real.
 Since, α, β, γ are distinct negative real
i cos θ sin θ + sin θ cos θ
2
 numbers.
1 0 0 x b b 0 i  Hence, ∆≥0
=
d i 0
29. (c) ∆ = a x b + 0 1 0   38. (a) p, q, r are the entries of first row and
dx 1 C , C ,C are cofactors of second row.
a a x a a x 34. (a) | A(θ)| = sin θ − i cos θ
2 2 2 21 22 23
∴ p ⋅C + q ⋅C + r ⋅C =0
x b b = sin 2 θ + cos 2 θ = 1 21 22 23

+ a x b ∴ A(θ) is invertible for all θ ∈ R 39. (c) x, y, z are the entries of second row
So, Statement I is correct. and C ,C ,C are cofactors of second
0 0 1 21 22 23
 sin θ − i cos θ row.
x b x b x b Now, A(θ)−1 = 
sin θ 
, ∴ x ⋅C + y ⋅C + z ⋅C =∆
= + + = 3∆ − i cos θ
a x a x a x 2   21 22 23

 − sin θ i cos θ  40. (b) q ⋅ M − y ⋅ M + m⋅M


A( − θ) =  12 22 32
x b b i cos θ − sin θ = −q ⋅ C − y ⋅C − m ⋅C
  12 22 32
30. (b) ∆ = a x b = −( q ⋅ C + y ⋅C + m ⋅ C ) = −∆
1
a a x ∴ A(θ)−1 ≠ A( − θ) 12 22 32
[Q q , y , m are entries of second
So, Statement II is not correct. column and C , C ,C are cofactor
= x ( x 2 − ab ) + b( ab − ax ) + b( a 2 − ax ) 12 22 32
= x ( x 2 − ab ) + ab( a + b ) −2abx α β γ of second column]
x b Sol. (Q. Nos. 35-37) ∆ = β γ α 41. (a) adj (adj A)
and ∆ = = x 2 − ab = | A |n − 2 A = | A | A = − A
2 a x γ α β
42. (b) |adj B | = |B |n − 1 = |B |2 = 242
Given, ∆ − x∆ + a 2 b = 0 Applying C → C + C + C , we get
1 2
1 1 2 3 43. (c) |(adj (adj (adj (adjA))))|
⇒ ab( a + b ) − 2abx + a 2 b = 0 2
2a + b α+β+ γ β γ = |adj (adj( − A ))| = |− A |(n −1 ) = |− A |4
⇒ ab[ a + b − 2x + a ] = 0 ⇒ x =
2 ∆= α+β+ γ γ α
= (| A |)4 = ( −1)4 = 1
3 −5 3 α+β+ γ α β
44. (a) 45. (a) 46. (d )
31. (c) We have, f ( 0) = 4 1 9
1 β γ − a2 ab ac
9 −6 21
= (α + β + γ ) 1 γ α 47. (c) Let ∆ = ab −b 2 bc
= 3( 21 + 54) + 5(84 − 81) + 3( − 24 − 9)
1 α β ac bc −c2
= 225 + 15 − 99 = 141
130 NDA/NA Pathfinder

Taking common a , b and c from rows 53. (a) If the matrix is not invertible, then Applying R → R − R and
2 2 1
R , R and R respectively, determinant of this matrix must be zero. R → R − R , we get
1 2 3
α 2 2 3 3 1
−a b c
i.e −3 0 4 = 0 1 t−1 1
∆ = abc a −b c
1 −1 1 ( t + 1) 0 2 − t 0 =0
a b −c
0 2−t t−2
⇒ α ( 0 + 4) − 2( −3 − 4) + 2(3 − 0) = 0
Again, taking common a , b and c from ⇒ ( t + 1)( 2 − t )( t − 2) = 0
columns C , C and C respectively, ⇒ 4α + 14 + 6 = 0
1 2 3 ⇒ ( t + 1)( t − 2)2 = 0
⇒ 4α = − 20 ⇒α = − 5
−1 1 1 ∴ t = − 1, 2
54. (c)
∆ = a 2 b 2 c2 1 −1 1 58. (a) Given that, D is determinant of order
55. (a) We know that, a square matrix ‘A’ is 3 and D′ is the determinant obtained by
1 1 −1 an orthogonal matrix, if AAT = I . replacing the elements of D by their
= a 2 b 2 c 2 ( 0 + 2 + 2) ⇒ | AAT |= |I | ⇒ | A || AT |= 1 cofactors.
⇒ | A || A |= 1 [Q | A | = | AT | ] ∴ D′ = Cofactor of D
= 4a 2 b 2 c 2 ⇒ |D ′|= |Cofactor of D |
⇒ |A| = 1 2
⇒ |D ′|= |adj (D )|
48. (b) We know that, every matrix possesses ∴ | A |= ± 1
a unique inverse. [Q|Cofactor of D |= |adj (D )|]
m n p
Hence, B and C should be equal. ⇒ |D ′|= |D |(3 −1 ) [Q|adj( A )|= | A | n −1
]
56. (c) Let ∆ = p m n ⇒ |D ′|= |D |2
49. (c) Given condition is A 2 = I n p m ∴ D ′ =D 2
−1 −1
⇒ A ⋅A = A 2
⋅I Applying C → C + C + C , we get 59. (c) I. A matrix is only an arrangement of
1 1 2 3
⇒ A −1 ( A ⋅ A ) = A −1 [Q A ⋅ I = a ] numbers, it has no definite value.
(m + n + p ) n p
⇒ ( A −1 A ) ⋅ A = A −1 Since, determinant is a value of the
−1 −1
∆ = (m + n + p ) m n matrix. Hence, two determinants of
⇒ I⋅A= A [Q A A = I]
(m + n + p ) p m different orders may have the same value.
−1
⇒ A = A
60. (b) Minor of element 9
1 2  0 −1 1 n p 19 2
50. (b) Given A =  and B =  = = 19 − 26 = − 7
1 1 1 2 = (m + n + p ) 1 m n 13 1
   
1 p m
 1 2  0 −1  2 3 61. (a) Given that, A and B are two
AB =  = Applying R → R − R
1 1  1 2   1 1 and non-singular square matrices.
      2 2 1
So, A −1 and B −1 must be exist.
R → R − R , we get
3 3 1
∴| AB|= 2 − 3 = −1 We have, AB = A
1 n p
Thus, B −1 A −1 = ( AB )−1 =
adj ( AB ) ( A −1 ) operating in left side on both
| AB| ∆ = (m + n + p ) 0 m − n n− p sides, we get A −1 ( AB ) = ( A −1 ) ( A )
0 p−n m− p ⇒ ( A −1 A ) B = ( A −1 A )
 1 −3   −1 3 
= ( −1) = [Q AA −1 = I and IB = B ]
−1 2   1 −2  = (m + n + p )
    ⇒ IB = I ⇒ B = I = Identity matrix
[(m − n)(m − p ) − ( n − p )( p − n)]
51. (d) 62. (c) Cofactor of the element 4
= (m + n + p )(m 2 + n2 + p 2
2 3
x 2
1 y +z
2 2
− mn − np − pm) = ( −1)2 + 1 = − ( 18 − 24) = 6
1 8 9
52. (a) Let ∆ = y 2
1 z2 + x 2 = (m + n + p )[(m − n)2 + ( n − p )2
2
z2 1 x2 + y2 + ( p − m )2 ] = A linear factor 1 bc a ( b + c )
1 t−1 1 63. (a) Let ∆ = 1 ca b ( c + a )
Applying C → C + C , we get
3 3 1
57. (b) Given that, t − 1 1 1 =0 1 ab c ( a + b )
x 2
1 x 2 + y 2 + z2 1 1 t−1 Applying R → R − R and
2 2 1
∆ = y2 1 x 2 + y 2 + z2 R →R −R ,
Applying C → C + C + C , we get 3 3 1
z2 1 x 2 + y 2 + z2 1 1 2 3
t+ 1 t−1 1 1 bc ab + ac
x 2
1 1 t+1 1 1 =0 ∆ = 0 c ( a − b) ( b − a) c
∆ = ( x 2 + y 2 + z2 ) y 2 1 1 =0 t+1 1 t−1 0 b ( a − c) b ( c − a )
z2 1 1 1 t−1 1 Expanding along C ,
1
⇒ ( t + 1) 1 1 1 =0 ∆ = bc ( a − b ) ( c − a ) − bc ( a − c ) ( b − a )
[Q C and C are identical]
2 3
1 1 t−1 = bc ( a − b ) ( c − a ) − bc ( a − b ) ( c − a )
=0
MATHEMATICS Determinants 131

64. (b) If A is a square matrix of order n Now, expanding along C , 76. (d) 77. (b) 78. (b)
1
with | A| ≠ 0, then |adj A | = | A |n −1 ( x + a + b + c) ⋅ 1 ⋅ ( x 2 ) = 0 a b c 1 b c
For order 3 matrix, Put n = 3, ⇒ x 2 ⋅ ( x + a + b + c) = 0 79. (c) b c a = ( a + b + c ) 1 c a
|adj A| = | A|3 −1 =| A|2 ⇒ x = 0 or x = − ( a + b + c )
c a b 1 a b
 2 1 68. (a) 69. (c)
65. (d) Given that, 2 A = 
3 2  1 2 [apply C → C + C + C and take
   
1 1 2 3
common ( a + b + c ) ]
70. (b) Given matrices, A =  2 1
1 2 1  1 1 / 2
 1 1
= ( a + b + c)
⇒ A=   = 
2  3 2 3/2 1   [ bc − a 2 − b 2 + ab + ac − c 2 ]
 
 1 2 − 4 = ( a + b + c)
Now | A| = 1 − 3 / 4 = 1 / 4 and B = 
2 1 − 4 [ −( a 2 + b 2 + c 2 − ab − bc − ca )]
∴ A −1 =
adj( A )   1
 1 2 = − ( a + b + c)
| A| 2
 1 −1 / 2   4 −2     1 2 − 4
=4 = Now, AB =  2 1  2 1 − 4 [( a − b )2 + ( b − c )2 + ( c − a )2 ]
−3 / 2 1  −6 4  1 1   2 × 3 = Negative value
   
 3 ×2
 1 2 1 5 4 − 12 1 1 1 1 1 1
    80. (c) 1 1 + x 1 = 0 x 0
66. (a) I. Let A =  a 2a 1 =  4 5 − 12
 b 2b 1 3 3 − 8  1 1 1+ y 0 0 y
   3 ×3
Now, | A| = 1 ( 2a − 2b ) − 2( a − b )  1 2  apply R2 → R2 − R1 
+ 1 ( 2ab − 2ab )  1 2 − 4    
And, BA =   and R3 → R3 − R1 
= 2a − 2b − 2a + 2b + 0 = 0 2 1 − 4  2 1
  2 × 3  1 1
i.e. A is a singular matrix.  3 ×2 = 1 ( xy − 0) = xy
 c 2c 1  1 0 [expanding along first column]
= =I
  0 1
II. Let B =  a 2a 1   2 × 2 81. (d)
 b 2b 1 Now, we observe that B is not the right  2 7
82. (c) We have, A = 
  inverse of A but B is the left inverse of A. 1 5
 
Now, |B | = c ( 2a − 2b ) − 2c ( a − b )
71. (a) The matrix A is skew-symmetric ⇒ | A | = 10 − 7 = 3
+ 1 ( 2 b − 2ab )
matrix of order 3 which is odd. Thus,
= 2ac − 2bc − 2ac + 2bc + 0 = 0 1 1  5 −7 
determinant of A is zero. Now, A −1 = adj (A) =  
which is also represent a singular Hence, matrix A is non-invertible. |A| 3  −1 2 
matrix. So, Statement I is correct and
72. (c) A and B are square matrices of order 2.  2 7 1  5 −7 
Statement II is incorrect. ∴ A + 3 A −1 =  +3× 
We know that, | kA | = k n | A | , where n 1 5 
3  −1 2 
67. (c) Given that,  
is order of matrix A.
x + a  7 0
b c ∴ |3 AB | = 32 | A || B | = = 7I
0 7
a x + b c =0 [Q | AB | = | A || B |]  
a b x + c = 9( −1) (3) = − 27 83. (a) [det ( kA )]− 1 det (A)
[Q | A | = − 1, | B | = 3] 1
Applying C → C + C + C , = × det ( A )
 1 3 det ( kA )
1 1 2 3
73. (a) Let B= , then | B | = 1
we get 0 1
  =
1 1
× det ( A ) = n = k – n
x + a+ b+ c b c k n det ( A )
 1 −3   1  k
⇒ x + a+ b+ c x + b c =0 ∴ B −1 =   Q A −1 = adj( A ) a 1 1
0 1  |A| 
   
x + a+ b+ c b x + c 84. (b) Let ∆ = 1 b 1
Now, 1 1 c
1 b c 1 1  1 1 
BA =  ⇒ B −1 BA = B −1  = a ( bc − 1) − 1 ( c − 1) + 1( 1 − b )
⇒ ( x + a + b + c) 1 x + b c =0 0 −1 0 −1
    = abc − a − b − c + 2
1 b x + c
 1 −3  1 1  1 4 Q∆ > 0
⇒ A= =
Applying R → R − R 0 1  0 −1  0 −1 ∴ abc − a − b − c + 2 > 0
2 2 1
and       ⇒ abc + 2 > a + b + c …(i)
R → R − R , we get 74. (a) Q a≠b≠c
3 3 1
75. (b) Since, A −1 A = I n ⇒ AM of a , b , c > GM of a , b , c
1 b c
∴ det ( A −1 A ) = det ( I n )
⇒ ( x + a + b + c) 0 x 0 = 0 a+ b+ c
⇒ det ( A −1 ) × det ( A ) = 1 ⇒ > ( abc )1 / 3
0 0 x 1 3
⇒ det ( A −1 ) =
det( A ) ⇒ a + b + c > 3( abc )1 / 3 …(ii)
132 NDA/NA Pathfinder

From Eqs. (i) and (ii), α −β  6x − sin x − cos x 


II. The maximum value of cos is 1.
abc + 2 > 3( abc )1 / 3 2 ⇒ f ′ ′( x ) =  6 −1 0 
∴ The maximum value of the 
 

Now, let x = ( abc )1 / 3 , then we have 1 p p2 p3
determinant is .  
x 3 + 2 > 3x 2
⇒ x − 3x + 2 > 0
3 Since, we have f ′ ′( 0) = 0
Hence, both statements are correct.
⇒ ( x − 1)2 ( x + 2) > 0 0 0 1
1 2 
f ′ ′ ( 0 ) =  6 −1 0  = 0
3
⇒ x + 2 > 0 [Q ( x − 1)2 > 0 ]   ⇒
87. (a) We have, A =  1 x − 1 1 , 
 

⇒ x>−2 p p2 p3
2 7 x − 3  
⇒ ( abc )1 / 3 > − 2   1
⇒ abc > − 8 A has inverse, if | A | ≠ 0 ⇒ 6 p + p = 0 ⇒ p = − or 0
2
6
85. (a) We have, ∴| A |= 1{( x − 1) ( x − 3) − 7}
 41 1 5  1 1 5
1− a a − b − c b+ c −3( x – 3 – 2) + 2 { 7 – 2( x – 1)} ≠ 0
⇒ x 2 − 4x − 4 − 3x + 15 92. (d) I. Now, 79 7 9 = 7 7 9 = 0
∆ = 1− b b − c − a c+ a 
 
 
 

+ 14 − 4x + 4 ≠ 0 29 5 3 5 5 3
1− c c−a−b a+ b    
⇒ x 2 − 11x + 29 ≠ 0 [QC and C are identical]
Applying C → C + C , we get 11 ± 121 − 116 11 ± 5 1 2
2 2 3 ⇒x ≠ ⇒x ≠ 1 a b + c  1 a a + b + c 
1− a a b+ c 2 2
   
∆ = 1− b b c+ a II. 1 b c + a = 1 b a + b + c
88. (a)Q A is an orthogonal matrix. 
 
  

1− c a+ b | AB | = ± | B | 1 c a+ b 1 c a+ b+ c
c    
Now, applying C →C + C ,
1 2 3 [apply C → C + C ]
3 2 3
1 1 2
∴ | B | = − 3 0 2 = 47  1 a 1
C → C + C and taking common
3 3 2
2 5 0 = ( a + b + c ) 1 b 1 = 0
a + b + c from C , we get 
 

3
| AB | = ± 47 1 c 1
1 a 1
Now,  
Also, AB ≠ BA [Q C and C are identical]
∆ = ( a + b + c) 1 b 1 1 3
89. (b)  0 c b
1 c 1
 x 3 sin x cos x  
III. − c 0 a
Q C and C are identical.  
1 3 90. (d) Given, f ( x ) =  6 −1 0  
−b −a 0

Hence, the determinant is 0. 
 2

  
p p p3
α α   = 0( 0 + a ) − c ( 0 + ab ) + b( ac − 0)
cos 2 sin 2
2 2  3x 2 cos x − sin x  = 0 − c( ab ) + b( ac )
86. (c) I. We have, ∆ =
β β = − abc + abc = 0
sin 2 cos 2 f ′( x ) =  6 −1 0 
2 2 
 
 Hence, I, II and III determinants have
p p2 p3 value ‘zero’.
α β α β  
= cos 2 cos 2 − sin 2 sin 2
2 2 2 2 0 1 0 93. (a) Given linear equations
 α β
=  cos cos + sin sin 
α β ⇒ f ′ ( 0 ) =  6 −1 0  k x + y + z = 1 ⇒ x + ky + z = 1
 
 
 x + y + kz = 1
2 2 2 2 p p2 p3
and
   k 1 1
 cos α cos β − sin α sin β 

 2 2 2 2
 = ( −1) ( 6 p 3 − 0) = − 6 p 3 For unique solution, 1 k 1  ≠ 0

 

91. (a) From question 90, 1 1 k
= cos  α − β  cos  α + β   
     3x 2 − sin x 
 2   2  cos x
⇒ k( k 2 − 1) − 1( k − 1) + 1( 1 − k ) ≠ 0
= cos 

α − β  cos 45°
 f ′( x ) =  6 −1 0  ⇒ k( k − 1) ( k + 1) − 2( k − 1) ≠ 0
 2  
 

p p2 p3 ⇒ ( k − 1) ( k − 1) ( k + 2) ≠ 0
α −β  
cos 
1
=  [Q α + β = 90°] ⇒ k ≠ 1 and k ≠ − 2
2  2 
12
MATHEMATICS Decimal Fractions 133

MEASUREMENT OF
ANGLES AND
TRIGONOMETRIC RATIOS
In NDA exam, generally 4-7 questions are asked from this chapter which are based on finding the
value of trigonometric ratios by using standard formulae/value, solving trigonometric equations etc.

ANGLE
B
Let a revolving line starting from OA revolves about its end point ‘O’ on a e
plane in clockwise or anti-clockwise direction and occupy the position ‘OB’. l s id
ina
It is said to trace out ∠AOB. r m
Te
Here, OB is called the terminal side and OA is called initial side. The fixed θ
point ‘O’ is called the vertex. O A
Vertex Initial side

Measurement of Angles
The amount of rotation from the initial side to the terminal side is called the measure of the angle.
Note If the rotation is in anti-clockwise sense, the angle measured is positive and if the rotation is in clockwise sense, the
angle measured is negative.

Systems of Measurement of Angles


There are three major types of systems of measurement of angles which are as follows
1. Sexagesimal system (Degree measure) In this system a right angle is divided into 90 equal parts,
called degrees. Each degree is divided into 60 equal parts, called minutes and each minute is further
divided into 60 equal parts, called seconds.
Thus, 1 right angle = 90 degrees = (90 °), 1° = 60 min = (60 ′ ), 1 ′ = 60 s = (60 ′ ′ )
2. Centesimal system In this system a right angle is divided into 100 equal parts, called grades. Each
grade is subdivided into 100 centesimal minutes and each minute is further divided into 100
centesimal seconds.
134 NDA/NA Pathfinder

Thus, 1 right angle = 100 grades = (100 g ) Let PMR be a right angled triangle at M.
1 grade = 100 min = (100 ′ ) Perpendicular q
sine θ = = and is written as sin θ
1 min = 100 s = (100 ′ ′ ) Hypotenuse r
3. Circular system In this system the unit of Base p
cosine θ = = and is written as cos θ
measurement is radian as defined below Hypotenuse r
(i) One radian, written as 1c , is the measure of an angle Perpendicular q
subtended at the centre of a circle by an arc of tangent θ = = and is written as tan θ
Base p
length equal to the radius of the circle.
Hypotenuse r
(ii) The number of radians in an angle subtended by an cosecant θ = = and is written as cosec θ
arc Perpendicular q
arc of a circle at the centre is equal to . Hypotenuse r
radius secant θ = = and is written as sec θ
Base p
Relation between Degrees, Grades Base p
and Radians cotangent θ = = and is written as cot θ
Perpendicular q
The relation between the three systems of measurement
Degree Grade 2 Radian
of an angle is = = Relation between
90 100 π
Trigonometric Ratios
Thus, 1 1
(i) sin θ = or cosec θ =
 180  cosec θ sin θ
(i) To convert radians into degrees multiply by  .
 π  1 1
(ii) cos θ = or sec θ =
 π  sec θ cos θ
(ii) To convert degrees into radians multiply by  .
 180  1 1
(iii) cot θ = or tan θ =
tan θ cot θ
EXAMPLE 1. Find the angle between the minute hand sin θ cos θ
of a clock and the hour hand when the time is (iv) tan θ = (v) cot θ =
cos θ sin θ
7 : 20 am.
a. 90° b. 95° c. 100° d. 105° Fundamental Trigonometric Identities
Sol. c. We know that, the hour hand completes one rotation The following are some fundamental identities
in 12 h while the minute hand completes one rotation
in 60 min. (i) sin 2 θ + cos 2 θ = 1 (ii) 1 + tan 2 θ = sec 2 θ
∴ Angle traced by the hour hand in 12 h = 360° (iii) 1 + cot 2 θ = cosec 2 θ
7 h 20 min = 7 +
20 1 22
 h=7 + = h
 60  3 3 EXAMPLE 2. If 1 + sin 2 A = 3 sin A ⋅ cos A, then what is
⇒ Angle traced by the hour hand in 7 h 20 min. the value of tan A?
h = 
22 360 22 1 1 1 1
i.e. ×  = 220° a. ,1 b. ,2 c. ,3 d. ,4
3  12 3 2 4 6 8
Also, the angle traced by the minute hand in 60 min = 360° Sol. a. We have, 1 + sin2 A = 3 sin A cos A
The angle traced by the minute hand in 20 min
On dividing both sides by cos2 A, we get
°
=  × 20 = 120°
360
 60  sec 2 A + tan2 A = 3 tan A
⇒ 1 + tan2 A + tan2 A = 3 tan A
∴ Required angle between two hands = 220°−120° = 100°
[Qsec 2 θ = 1+ tan2 θ]
TRIGONOMETRIC RATIOS ⇒ 2 tan2 A − 3 tan A + 1 = 0
P ⇒ 2 tan A − 2 tan A − tan A + 1 = 0
2
The ratios between different sides of a
⇒ 2 tan A (tan A − 1) − 1(tan A − 1) = 0
right angled triangle with respect to its ⇒ ( 2 tan A − 1) (tan A − 1) = 0
r q
acute angles are called trigonometric 1
⇒ tan A = , 1
ratios. 2
θ 90º
R p M
MATHEMATICS Measurement of Angles and Trigonometric Ratios 135

Domain and Range θ sinθ cos θ tanθ cotθ sec θ cosec θ

Trigonometric Ratios Domain Range 180° + θ − sinθ − cosθ tanθ cot θ − sec θ − cosec θ

sinθ R [−1, 1] 270°− θ − cosθ − sinθ cot θ tanθ − cosecθ − sec θ

cosθ R [−1, 1] 270° + θ − cosθ sinθ − cot θ − tanθ cosecθ − sec θ


π 360° − θ − sinθ cosθ − tanθ − cot θ sec θ − cosec θ
tanθ R − (2 n + 1) : n ∈ I  R
 2  360° + θ sinθ cosθ tanθ cot θ sec θ cosec θ
cosec θ R − { nπ : n ∈ I} R − ( −1, 1)
sec θ π R − ( −1, 1) EXAMPLE 3. What is the value of cot (−870 °)?
R − (2 n + 1) : n ∈ I 
 2 
a. 3 b. 1/ 3 c. − 3 d. −1/ 3
cot θ R − { nπ : n ∈ I} R
Sol. a. cot ( −870° ) = − cot ( 2 × 360° + 150° )
Trigonometric Ratios of Standard Angles = − cot 150° = − cot (90° + 60° ) = tan 60° = 3
π π π π
0 Sum and Difference of Two Angles
6 4 3 2
1 1 3
(i) sin ( A ± B) = sin A cos B ± cos A sin B
sin 0 1
2 2 2 (ii) cos ( A ± B) = cos A cos B m sin A sin B
3 1 1 tan A ± tan B
cos 1 0 (iii) tan ( A ± B) =
2 2 2 1 m tan A tan B
1
tan 0 1 3 Undefined cot A cot B m 1
3 (iv) cot ( A ± B) =
1 cot B ± cot A
cot Undefined 3 1 0
3 (v) sin ( A + B) sin ( A − B) = sin 2 A − sin 2 B
sec 1
2
2 2 Undefined
= cos 2 B − cos 2 A
3
2 (vi) cos ( A + B) cos ( A − B) = cos 2 A − sin 2 B
cosec Undefined 2 2 1
3 = cos 2 B − sin 2 A

Note |sinθ|≤ 1, |cos θ|≤ 1, |sec θ|≥ 1, |cosec|≥ 1 for all values of θ for Transformation Formulae
which the functions are defined.
(i) 2 sin A cos B = sin ( A + B) + sin ( A − B)
Signs of Trigonometric Ratios in Different Quadrants (ii) 2 cos A sin B = sin ( A + B) − sin ( A − B)
Quadrant sinθ cos θ tanθ cotθ sec θ cosec θ (iii) 2 cos A cos B = cos ( A + B) + cos ( A − B)
I + + + + + + (iv) 2 sin A sin B = cos ( A − B) − cos ( A + B)
− − − −
II + +
 C + D  C − D
III − − + + − − (v) sin C + sin D = 2 sin   cos  
 2   2 
IV − + − − + −
 C − D  C + D
(vi) sin C − sin D = 2 sin   cos  
 2   2 
Trigonometric Ratios of Allied Angles
 C + D  C − D
Two angles are said to be allied when their sum or (vii) cos C + cos D = 2 cos   cos  
difference is either zero or a multiple of 90°, i.e. the  2   2 
angles −θ, 90 ° ± θ, 180 ° ± θ, 270 ° ± θ and 360 ° ± θ are  C + D  D − C
(viii) cos C − cos D = 2 sin   sin  
called allied angles.  2   2 
θ sinθ cos θ tanθ cotθ sec θ cosec θ
EXAMPLE 4. If sin 2 A = λ sin 2B, then what is the
−θ − sinθ cosθ − tanθ − cot θ sec θ − cosec θ tan( A + B)
value of ?
90° − θ cosθ sinθ cot θ tanθ cosec θ sec θ tan ( A − B)
90° + θ cosθ − sinθ − cot θ − tanθ − cosec θ sec θ
λ −1 λ +1 λ2 − 1 λ −1
a. b. c. d.
180° − θ sinθ − cosθ − tanθ − cot θ − sec θ cosec θ λ +1 λ −1 λ2 + 1 λ +1
136 NDA/NA Pathfinder

sin 2A 3 tan A − tan 3 A


Sol. b. Q sin 2A = λ sin 2B ⇒ =λ (vii) tan 3A =
sin 2B
1 − 3 tan 2 A
On applying componendo and dividendo rule, 3cot A − cot 3 A
sin 2A + sin 2B λ + 1 (viii) cot 3A =
= 1 − 3cot 2 A
sin 2A − sin 2B λ − 1
2 sin ( A + B) cos ( A − B) λ + 1 tan ( A + B) λ + 1 Trigonometric Ratios of
⇒ = ⇒ =
2 cos ( A + B) sin ( A − B) λ − 1 tan ( A − B) λ − 1 Submultiples Angles
1
2 tan A
EXAMPLE 5. Determine the value of 1 1 2
cos 20 ° cos 40 ° cos 60 ° cos 80 °. (i) sin A = 2sin A cos A =
2 2 2 1
a. 1 / 2 b. 1 / 4 c. 1 / 8 d. 1 / 16 1 + tan A
2
Sol. d. cos 20° cos 40° cos 60° cos 80° 1 1 1
(ii) cos A = cos 2 A − sin 2 = 2cos 2 A − 1
2 A 2
= cos 60° (cos 20° cos 40° ) cos 80°
2 1
1 1 1 1 − tan A
= × ( 2 cos 20° cos 40° ) cos 80° Q cos 60° =  1 2
2 2  2 = 1 − 2sin 2 A =
2 1
1
= [{cos ( 40° + 20° ) + cos ( 40° − 20° )} cos 80° ]
1 + tan 2 A
4 2
[Q 2 cos A cos B = cos ( A + B) + cos ( A − B)] 1 1
2 tan A cot 2 A − 1
1 2 2
= [(cos 60° + cos 20° ) cos 80° ] (iii) tan A = (iv) cot A =
4 2 1 1
1 − tan A 2cot A
=  + cos 20° cos 80°
1 1
2 2
4  2  
1 EXAMPLE 6. What will be the value of
= (cos 80° + 2 cos 80° cos 20° )
8
1 2 + 2 + 2 + 2 cos 8θ ?
= [cos 80° + {cos (80° + 20° ) + cos (80° − 20° )}]
8 a. 2sinθ b. 2 cos θ c. sin 2θ d. cos 2θ
[Q 2 cos A cos B = cos ( A + B) + cos ( A − B)]
1 2 + 2 + 2 + 2cos 8θ
= [cos 80° + cos 100° + cos 60° ] Sol. b. We have,
8
1
= [cos 80° + cos (180° − 80° ) + cos 60° ] = 2 + 2 + 2 + 4 cos2 4θ − 2
8
1 = 2 + 2 + 4 cos2 2θ − 2
= [cos 80° − cos 80° + cos 60° ]
8 = 2 + 2cos 2θ = 2 + 4 cos2 θ − 2 = 2cos θ
1 1 1 1 1
= cos 80° − cos 80° +  = × = cot 2 15°−1
8 
2 8 2 16 EXAMPLE 7. is equal to
cot 2 15° + 1
Trigonometric Ratios of a.
1
b.
3
c.
3 3
d. 3
Multiple of an Angle 2 2 4
2 tan A cos2 15°
(i) sin 2A = 2 sin A cos A = cot 2 15° − 1 sin2 15° − 1 cos2 15° − sin2 15°
1 + tan 2 A Sol. b. = =
cot 2 15° + 1 cos2 15° cos2 15° + sin2 15°
(ii) cos 2A = cos 2 A − sin 2 A = 2cos 2 A − 1 + 1
sin 15°
2

1 − tan 2 A = cos ( 2 × 15° ) = cos ( 30° ) = 3 / 2


= 1 − 2 sin 2 A =
1 + tan 2 A
2 tan A Values of Trigonometric Ratios
(iii) tan 2A =
1 − tan 2 A of Some Specific Angles
cot 2 A − 1 3 −1 3 +1
(iv) cot 2A = (i) sin 15° = (ii) cos 15° =
2cot A 2 2 2 2
(v) sin 3A = 3 sin A − 4 sin 3 A (iii) tan 15° = cot 75° = 2 − 3
(vi) cos 3A = 4cos 3 A − 3cos A (iv) cot 15° = tan 75° = 2 + 3
MATHEMATICS Measurement of Angles and Trigonometric Ratios 137

5 −1 Solutions or Roots of a
(v) sin 18° = cos 72° =
4 Trigonometric Equation
5 +1 A value of the unknown angle which satisfies
(vi) cos 36° = sin 54° =
4 the given equation, is called a solution or root of the
10 + 2 5 equation. The trigonometric equation may have infinite
(vii) cos 18° = sin 72° =
4 number of solutions and can be classified as
10 − 2 5 (i) Principal solution The least value of unknown angle
(viii) sin 36° = cos 54° =
4 which satisfies the given equation, is called principal
1 2− 2 solution of trigonometric equation.
(ix) sin 22 ° = (ii) General solution We know that trigonometric
2 2
functions are periodic and solutions of trigonometric
1 2+ 2
(x) cos 22 ° = equations can be generalised with the help of the
2 2
1 periodicity of the trigonometric functions. The
(xi) tan 22 ° = 2 − 1 solution consisting of all possible solutions of a
2
1 trigonometric equation is called its general solution.
(xii) cot 22 ° = 2 + 1
2 Note A function f( x ) is said to be a periodic function, if a least positive
real number T is such that f ( x + T ) = f( x ), then T is known as
EXAMPLE 8. What will be the value of period of function f( x ).
cos 12° + cos 84° + cos 156 ° + cos 132°?
−1 3 Trigonometrical Equations with their General Solution
a. 2 ( 5) b. − 2 3 c. d.
2 4 Trigonometrical equation General solution
Sol. c. We have, cos 12° + cos 84° + cos 156° + cos 132° sin θ = 0 θ = nπ, n ∈ I
= (cos 12° + cos 132° ) + (cos 84° + cos 156° ) cos θ = 0 θ = (2 n + 1) π / 2 , n ∈ I
12°+132° 132°−12°
= 2 cos   cos   tan θ = 0 θ = nπ, n ∈ I
 2   2 
sin θ = 1 θ = 2 nπ + π / 2 , n ∈ I
 84°+156° 156°−84°
+ 2cos   cos  cos θ = 1 θ = 2 nπ, n ∈ I
 2   2 
sin θ = sin α π π
= 2 cos 72° cos 60° + 2 cos 120° cos 36° θ = nπ + ( − 1)nα, α ∈  − , , n ∈ I
 2 2 
= 2 sin 18° cos 60° + 2 cos 120° cos 36°
cos θ = cos α θ = 2 nπ ± α, α ∈ [0, π ], n ∈ I
 5 − 1  1  − 1  5 + 1 − 1
= 2    + 2    = tan θ = tan α π π
θ = nπ + α, α ∈  − ,  , n ∈ I
 4  2    2  4  2  2 2

Maximum and Minimum Values of sin2 θ = sin2 α θ = nπ ± α, n ∈ I

(a sin θ ± b cos θ) tan2 θ = tan2 α


cos 2 θ = cos 2 α
The maximum and minimum values of a sin θ ± b cos θ
sin θ = sin α and θ = 2 nπ + α, n ∈ I
are respectively, a 2 + b 2 and − a 2 + b 2 . cos θ = cos α

TRIGONOMETRIC EQUATION EXAMPLE 9. If sin 9 θ = sin θ, then the value of θ will be,
An equation involving one or more trigonometric if θ is even.
2m mπ mπ
ratios of unknown angles is called a trigonometric a. b. c. d. mπ
π 2 4
equation.
1 Sol. c. We have, sin 9θ = sin θ, 9θ = nπ + ( − 1) nθ, n ∈ I
e.g. 2cos θ + 3cos 2θ = 0, cos 2 θ + sin θ = etc. mπ
3 If n is even, then 9θ = 2mπ + θ ⇒ 8θ = 2mπ ⇒ θ =
4
138 NDA/NA Pathfinder

PRACTICE EXERCISE
1. What is the measure of the angle 114° 35′ 30′ ′ in 10. If an angle α is divided into two parts A and B
radian? such that A − B = x and tan A : tan B = 2 : 1, then
(a) 1 rad (b) 2 rad what is sin x equal to?
(c) 3 rad (d) 4 rad 2 sin α sin α
(a) 3 sin α (b) (c) (d) 2 sin α
3 3
2. The angle between the minute hand and the hour
hand of a clock when the time is 8:25 am is 11. What is the maximum value of 3 cos x + 4 sin x + 5?
(a) 92 ° 45′ (b) 102 ° 30′ (a) 5 (b) 7 (c) 10 (d) 12
(c) 105° (d) 107 °15′
12. If tan A − tan B = x and cot B − cot A = y, then
1
3. What is the value of sin 292 ° ? what is cot ( A − B) equal to?
2 1 1 1 1 1 1 1 1
1 1 (a) − (b) − (c) + (d) − −
(a) 2+ 3 (b) − 2− 3 y x x y x y x y
3 3
1 1
(c) 2+ 2 (d) − 2+ 2 13. If cos A + cos B = m and sin A + sin B = n , where
2 2
m , n ≠ 0, then what is cos ( A + B) equal to?
4. Which one of the following is correct? m2 − n2 2 mn
(a) b)
1 + cos 67 1°  1 + cos 112 1°  is m2 + n2 m2 + n2
 2  2 m2 + n2 m2 + n2
(c) (d)
(a) an irrational number and is greater than 1 2 mn m2 − n2
(b) a rational number but not an integer
(c) an integer 14. If A + B = 90°, then minimum and maximum
(d) an irrational number and is less than 1 values of (cos A cos B) respectively are
5. What is the value of 1 1 1 1
(a) − , (b) − ,
4 4 3 3
π π 5π 7π
cos   + cos   + cos   + cos  ? 1 1
(c) − , (d) None of these
 9  3  9  9 2 2
(a) 1 (b) −1
1 1 15. The smallest positive value of x (in degrees) for
(c) − (d) which tan( x + 100° ) = tan( x + 50° ) tan x
2 2
tan( x − 50° ) is
7π 9π 10π
6. tan , tan , tan are in (a) 60° (b) 90° (c) 45° (d) 30°
6 4 3
3 cos 2β − 1
(a) AP (b) GP 16. If α, β are acute angles and cos 2α = ,
3 − cos 2β
(c) HP (d) None of these
then the value of (tan α ⋅ cot β ) is
sin x + sin 3x + sin 5x + sin 7x
7. is equal to (a) 3 (b) 2
cos x + cos 3x + cos 5x + cos 7x
(c) 1 (d) None of these
(a) tan 16x (b) tan 8x
(c) tan 4x (d) tan2x 17. If sin x + sin2 x = 1, then the value of
(cos8 x + 2 cos6 x + cos4 x ) is
8. If tan θ + sin θ = m and tan θ − sin θ = n , then
(a) 0 (b) − 1 (c) 1 (d) 2
(a) m2 − n2 = 4mn (b) m2 + n2 = 4mn
2π 4π
(c) m2 − n2 = m2 + n2 (d) m2 − n2 = 4 mn 18. If x = y cos = z cos , then xy + yz + zx equals
3 3
a cos φ + b θ (a) −1 (b) 0 (c) 1 (d) 2
9. If cos θ = , then tan is equal to
a + b cos φ 2 19. Consider the following statements
I. If θ = 1200°, then (sec θ + tan θ )−1 is positive.
 a − b φ  a + b φ
(a)   tan (b)   cos II. If θ = 1200°, then (cosecθ − cot θ ) is negative.
 a + b 2  a − b 2
Which of the above statement(s) is/are correct?
 a − b φ (a) Only I (b) Only II
(c)   sin (d) None of these
 a + b 2 (c) Both I and II (d) Neither I nor II
MATHEMATICS Measurement of Angles and Trigonometric Ratios 139

20. Consider the following statements 28. The value of cos 2x is


I. sin 66° is greater than cos 66°. 2e 2 2e 2 e2 e2
(a) (b) (c) (d)
II. cos 26° is smaller than sin 26°. 1 + e2 1 − e2 2 − e2 2 + e2
Which of the above statement(s) is/are correct? 29. The value of sec4 x − sec2 x is
(a) Only I (b) Only II
(a) (e 2 − 2 )(e 2 − 1) (b) (e 2 + 2 )(e 2 − 1)
(c) Both I and II (d) Neither I nor II
(c) (e − 2 )(e + 1)
2 2
(d) None of these
21. If A, B, C, D are the angles of a cyclic
quadrilateral taken in order, then consider the Directions (Q. Nos. 30-33) ABC is a triangle right
following statements angled at B. The hypotenuse ( AC ) is four times the
I. cos A + cos B = − (cos C + cos D ) perpendicular (BD) drawn to it from the opposite
II. cos (π + A ) + cos (π − B) + cos (π − C ) vertex and AD < DC .
π
− sin  − D = 0 30. Which one of the following is acute angles of the
2 
triangle?
Which of the above statement(s) is/are correct? (a) 15° (b) 30° (c) 45° (d) None of these
(a) Only I (b) Only II
(c) Both I and II (d) Neither I nor II 31. What is ∠ABD?
(a) 15° (b) 30° (c) 45° (d) None of these
22. Which among the following is/are correct
statement(s)? 32. What is AD : DC equal to?
I. The general value of θ satisfying the equations (a) (7 − 2 3 ) : 1 (b) (7 − 4 3 ) : 1
sin 2 θ = sin 2 α , cos 2 θ = cos 2 α and tan 2 θ = tan 2 α is (c) 1 : 2 (d) None of these
given by θ = nπ ± α . 33. What is tan ( A − C ) equal to?
II. The general value of θ satisfying equations (a) 0 (b) 1 (c) 2 (d) None of these
sin θ = sin α and cos θ = cos α simultaneously is
given by θ = 2nπ ± α , n ∈ Z . Directions (Q. Nos. 34-37) Given, x = cosec2 θ,
1
Select the correct answer using the code given y = sec2 θ, z =
below 1 − sin θ cos2 θ
2

(a) Only I (b) Only II 1 1


34. 2
+ is equal to
(c) Both I and II (d) Neither I nor II x y2
2+ z 2− z z z
(a) (b) (c) (d)
Directions (Q. Nos. 23-24) If x, y and z are the z z 2+ z 2− z
angles of a triangle and z = 135 °.
35. xyz is equal to
23. The value of sin z + cos z is 1 x+ y
(a) (x + y + z) (b) xy + z (c) x + y − z (d)
1 3 2 z
(a) 0 (b) 2 (c) (d)
2 2
36. tan2 2θ is equal to
24. The value of (1 + tan x )(1 + tan y ) is 4y 2y 4 2
(a) (b) (c) (d)
(a) 1 (b) 2 (c) 3 (d) 4 x(2 − y)2 x(2 − y)2 y(2 − x )2 y(2 − x )2

Directions (Q. Nos. 25-26) Let cos α and cos β be the 37. Consider the following statements
roots of the equation, 8 x 2 − 2 x − 1 = 0. I.
1 1
+ =1 II.
1
+
1
+
1
=1
α β
x y xy yz zx
25. The value of sin ⋅ sin is Which of the above statement(s) is/are correct?
2 2
(a) Only I (b) Only II
1 1 5 3
(a) (b) (c) (d) (c) Both I and II (d) Neither I nor II
2 4 4 2 4 2
α β
26. The value of cos ⋅ cos is
2 2 PREVIOUS YEARS’ QUESTIONS
1 3 1 5
(a) (b) (c) (d)
2 4 2 4 4 2 38. Which one of the following is positive in the
third quadrant? e 2012 I
Directions (Q. Nos. 27-29) Let tan2 x = 1 − e 2, e is (a) sin θ (b) cos θ (c) tan θ (d) sec θ
any constant.
39. What is the value of sin (1920° )? e 2012 I
27. The value of (sec x + tan3 x cosec x ) is (a)
1
(b)
1
(c)
3
(d)
1
(a) (2 + e 2 )3 / 2 (b) (2 − e 2 )3 / 2 (c) (1 − e 2 )3 / 2 (d) (1 + e 2 )3 / 2 2 2 2 3
140 NDA/NA Pathfinder

sin θ cos θ 54. What is the value of sin2 A − sin2 B?


40. What is the value of + ?
cosec θ sec θ e 2012 I (a) 0 (b) 1/2 (c) 1 (d) 2
(a) 1 (b) 1/2 (c) 1/3 (d) 2
55. The line making an angle ( − 120° ) with X-axis is
41. What is the maximum value of sin2 x ? e 2012 I situated in the e 2012 II
(a) −1 (b) 0 (c) 1 (d) Infinity (a) first quadrant (b) second quadrant
1 (c) third quadrant (d) fourth quadrant
42. If cosec θ − cot θ = , where θ ≠ 0, then what is
3 56. What is the value of tan ( − 585° )? e 2012 II
the value of cos θ? e 2012 I
(a) 1 (b) −1 (c) – 2 (d) − 3
3 1 1
(a) 0 (b) (c) (d)
13
2 2 2 57. If sec α = , where 270° < α < 360°, then what
5
43. If cot A ⋅ cot B = 2, then what is the value of is the value of sinα? e 2012 II
cos( A + B) ⋅ sec ( A − B)? e 2012 I (a) 5/13 (b) 12/13 (c) –12/13 (d) –13/12
(a) 1/3 (b) 2/3 (c) 1 (d) − 1
58. What is the value of sin 15°? e 2012 II
44. What is the maximum value of 3 −1 3+1 3 −1 3+1
sin 3θ cos 2θ + cos 3θ sin 2θ? e 2012 I (a) (b) (c) (d)
2 2 2 2 3+1 3 −1
(a) 1 (b) 2 (c) 4 (d) 10
59. If 4 sin2 θ = 1, where 0 < θ < 2π, how many values
45. What is the value of sec2  tan−1 
5 
 ?
  11  e 2012 I
does θ take? e 2012 II
(a) 121/96 (b) 217/921 (c) 146/121 (d) 267/121 (a) 1 (b) 2 (c) 4 (d) None of these

46. What is the value of 60. What is the value of sin 18° cos 36°? e 2012 II
sin A cos A tan A + cos A sin A cot A? e 2012 I (a) 4 (b) 2 (c) 1 (d) 1/4
(a) sin A (b) cos A (c) tan A (d) 1 2 1
61. If sin A = and cos B = , where A and B
47. If tan θ + sec θ = 4, then what is the value of 5 10
sin θ? e 2012 I are acute angles, then what is the value of A + B ?
(a) 8/17 (b) 8/15 (c) 15/17 (d) 23/32 e 2012 II
(a) 135° (b) 90° (c) 75° (d) 60°
48. What is the value of
sin 420°⋅ cos 390° + cos ( −300° ) ⋅ sin( −330° )? 62. Consider the following statements
e 2012 I
(a) 0 (b) 1 (c) 2 (d) −1 I. The value of cos 46° − sin 46° is positive.
II. The value of cos 44° − sin 44° is negative.
49. If θ = 18°, then what is the value of
4 sin2 θ + 2 sin θ ? e 2012 I
Which of the above statement(s) is/are correct?
e 2012 II
(a) – 1 (b) 1 (c) 0 (d) 2
(a) Only I (b) Only II
π
50. What is the value of tan  ? (c) Both I and II (d) Neither I nor II
 12 e 2012 I
(a) 2 − 3 (b) 2 + 3 (c) 2 − 3 (d) 3 − 2
63. What is the value of tan105°? e 2013 I
3+1 3+1 3 −1 3+2
51. Consider the following statements e 2012 I (a) (b) (c) (d)
3 −1 1− 3 3+1 3 −1
I. 1° in radian measure is less than 0.02 radians.
II. 1 radian in degree measure is greater than 45°. 64. If sin θ + 2 cos θ = 1, then what is 2 sin θ − cos θ
Which of the above statement(s) is/are correct? equal to? e 2013 I
(a) Only I (b) Only II (a) 0 (b) 1 (c) 2 (d) 4
(c) Both I and II (d) Neither I nor II 65. If cosec θ + cot θ = c, then what is cosθ equal to?
e 2013 I
Directions (Q. Nos. 52-54) Let sin ( A + B) = 1 c c
π (a) (b)
and sin ( A − B) = 1/ 2, where A, B ∈ 0, . c2 − 1 c2 + 1
 2  e 2012 I
c2 − 1
52. What is the value of A? (c) (d) None of these
c2 + 1
π π π π
(a) (b) (c) (d)
6 3 4 8 66. What is tan4 A − sec4 A + tan2 A + sec2 A equal to?
53. What is the value of tan (A + 2B) ⋅ tan (2A + B)? e 2013 I
(a) −1 (b) 0 (c) 1 (d) 2 (a) 0 (b) 1 (c) 2 (d) –1
MATHEMATICS Measurement of Angles and Trigonometric Ratios 141

67. If A + B = 90° , then what is the value of cot 224° − cot 134°
80. What is the value of ?
cot 226° + cot 316°
sin A sec B − sin A cos B ? e 2013 I
e 2014 I
(a) − cosec 88° (b) − cosec 2 °
(a) sin A (b) cos A (c) tan A (d) 0
(c) − cosec 44° (d) − cosec 46°
68. If tan A = x + 1 and tan B = x − 1, then
1 + sin A 1 − sin A
x 2 tan ( A − B) has the value e 2013 I 81. What is the value of − ?
1 − sin A 1 + sin A e 2014 I
(a) 1 (b) x (c) 0 (d) 2
(a) sec A − tan A (b) 2sec A ⋅ tan A
69. What is the value of (sin θ − cos θ + 1) cosec 2 θ?
4 4
(c) 4sec A ⋅ tan A (d) 4cosec A ⋅ cot A
e 2013 I 82. If A + B + C = π, then what is cos ( A + B) + cos C
(a) –2 (b) 0 (c) 1 (d) 2
equal to? e 2014 I
cot x + cosec x − 1 (a) 0 (b) 2cos C
70. The expression is equal to
cot x − cosec x + 1 e 2013 I (c) cos C − sin C (d) 2 sin C
sin x 1 − cos x 1 + cos x sin x
(a)
1 − cos x
(b)
sin x
(c)
sin x
(d)
1 + cos x
83. The complete solution of 3 tan2 x = 1 is given by
e 2014 I
π π
x
1 − tan2 (a) x = nπ ± (b) x = nπ +
2 equal to? 3 3
71. What is π π
2 x (c) x = nπ ± (d) x = nπ +
1 + tan 6 6
2 e 2013 I
where, n ∈ Z
(a) sin x ⋅ cos x (b) tanx (c) sinx (d) cos x
72. What is tan 15° equal to? e 2013 II 84. What is the value of cos 36°? e 2014 I
(a) 2 − 3 (b) 2 + 3 (c) 1 − 3 (d) 1 + 3 5 −1 5+1 10 + 2 5 10 − 2 5
(a) (b) (c) (d)
cot 54° tan 20° 4 4 4 4
73. What is + equal to?
tan 36° cot 70° e 2013 II 85. Consider the following statements
(a) 0 (b) 1 (c) 2 (d) 3 I. sin| x | + cos| x | is always positive.
74. If cos x = 1/ 3, then what is II. sin (x 2) + cos (x 2) is always positive. e 2014 I

sin x ⋅ cot x ⋅ cosec x ⋅ tan x equal to? e 2013 II Which of the above statement(s) is/are correct?
2 3 (a) Only I (b) Only II
(a) (b) (c) 2 (d) 1
3 2 (c) Both I and II (d) Neither I nor II

75. What is sin2 20° + sin2 70° equal to? e 2013 II 86. Consider the following statements
1 I. Value of sin θ oscillates between –1 and 1.
(a) 1 (b) 0 (c) –1 (d)
2 II. Value of cos θ oscillates between 0 and 1.
76. What is (1 − sin2 θ) (1 + tan2 θ) equal to? e 2013 II Which of the above statement(s) is/are correct?
e 2014 I
(a) sin2 θ (b) cos 2 θ (c) tan2 θ (d) 1 (a) Only I (b) Only II
77. Consider the following (c) Both I and II (d) Neither I nor II
π 3π
I. tan   II. tan   87. Consider the following statements
 6  4 1° 1°
I. n sin 2 67 − sin 2 22  > 1 for all positive
5π 2π  2
III. tan   IV. tan   2
 4  3 integers n ≥ 2.
Which one of the following is the correct order? II. If x is any positive real number, then nx > 1 for
e 2013 II all positive integers n ≥ 2.
(a) I < IV < II < III (b) IV < II < I < III
Which of the above statement(s) is/are correct?
(c) IV < II < III < I (d) I < IV < III < II
(a) Only I (b) Only II e 2014 I
78. What is cos 20° + cos 100° + cos 140° equal to? (c) Both I and II (d) Neither I nor II
e 2014 I
(a) 2 (b) 1 (c) 1/2 (d) 0
88. Consider the following statements
I. If 3θ is an acute angle such that sin 3θ = cos 2θ,
79. What is sin2( 3π ) + cos2( 4π ) + tan2( 5π ) equal to? then the measurement of θ in radian equals to π / 10.
e 2014 I II. One radian is the angle subtended at the centre
(a) 0 (b) 1 (c) 2 (d) 3 of a circle by an arc of the same circle whose
length is equal to the diameter of that circle.
142 NDA/NA Pathfinder

Which of the above statement(s) is/are correct? 101. Let θ be a positive angle. If the number of degrees
(a) Only I (b) Only II e 2014 I in θ is divided by the number of radians in θ, then
(c) Both I and II (d) Neither I nor II an irrational number 180 / π results. If the
89. What is 1 + sin 2θ equal to? number of degrees in θ is multiplied by the
e 2014 II
number of radians in θ, then an irrational number
(a) cos θ − sin θ (b) cos θ + sin θ 125π
(c) 2 cos θ + sin θ (d) cos θ + 2 sin θ
results. The angle θ must be equal to
9 e 2015 I
90. If cot A = 2 and cot B = 3, then what is the value (a) 30° (b) 45° (c) 50° (d) 60°
of cot( A + B)? e 2014 II
π π π Directions (Q. Nos. 102-103) Let α be the root of the
(a) (b) π (c) (d) π
6 2 4 equation 25 cos2 θ + 5 cos θ − 12 = 0, where < α < π.
2
1° 1° e 2015 I
91. What is sin2 66 − sin2 23 equal to?
2 2 e 2014 II 102. What is tanα equal to?
(c) 2 sin 47 ° (d) 2 cos 47 ° 3 3 4 4
(a) sin 47° (b) cos 47° (a) − (b) (c) − (d) −
4 4 3 5
cos 7x − cos 3x
92. What is equal to? 103. What is sin 2α equal to?
sin 7x − 2 sin 5x + sin 3x e 2014 II
24 24 5 21
(a) tan x (b) cot x (c) tan 2x (d) cot 2x (a) (b) − (c) − (d) −
25 25 12 25
93. In a ∆ABC, if sin A − cos B = cos C , then what is 104. (1 − sin A + cos A)2 is equal to e 2015 I
B equal to? e 2014 II
(a) 2 (1 − cos A)(1 + sin A) (b) 2 (1 − sin A)(1 + cos A)
π π π
(a) π (b) (c) (d) (c) 2 (1 − cos A)(1 − sin A) (d) None of these
3 2 4
cos θ sin θ
sin ( x + y ) a + b tan x 105. What is + equal to?
94. If = , then what is equal to? 1 − tan θ 1 − cot θ e 2015 I
sin ( x − y ) a − b tan y
e 2014 II (a) sin θ − cos θ (b) sin θ + cos θ
b a (c) 2sin θ (d) 2cos θ
(a) (b) (c) ab (d) 1
a b
106. The value of sin2 5° + sin2 10° + sin2 15°
95. If sin A sin ( 60° − A) sin ( 60° + A) = k sin 3 A, then + sin2 20° + ... + sin2 90° is e 2015 II
what is k equal to? e 2014 II
(a) 7 (b) 8 (c) 9 (d) 19/2
(a) 1/4 (b) 1/2 (c) 1 (d) 4
107. If sin x + sin y = a and cos x + cos y = b, then
96. Which one of the following is one of the solutions
x + y x − y
of the equation tan 2θ ⋅ tan θ = 1? e 2014 II tan2   + tan2   is equal to
(a)
π
(b)
π
(c)
π
(d)
π  2   2  e 2015 II
12 6 4 3 a4 + b 4 + 4b 2 a4 − b 4 + 4b 2
(a) (b)
a2 b 2 + b 4 a2 b 2 + b 4
Directions (Q. Nos. 97-99) Read the following a4 − b 4 + 4a2
information carefully and answer the questions given (c) (d) None of these
a2 b 2 + a4
below.
11π  21π  283π 
Given, 16 sin5 x = p sin 5 x + q sin 3x + r sin x 108. If p = tan  –  ,q = tan   and r = cot  ,
e 2014 II  6   4   6 
97. What is the value of p? then which of the following is/are correct?
I. The value of p × r is 2.
(a) 1 (b) 2 (c) −1 (d) −2
II. p, q and r are in GP.
98. What is the value of q? Select the correct answer using the code given
(a) 3 (b) 5 (c) 10 (d) –5 below. e 2015 II
(a) Only I (b) Only II
99. What is the value of r?
(c) Both I and II (d) Neither I nor II
(a) 5 (b) 8 (c) 10 (d) −10
sin3 A + sin 3 A cos3 A − cos 3 A
100. The line y = 3 meets the graph y = tan x , where 109. On simplifying + ,
π sin A cos A
x ∈  0,  , in k points. What is k equal to? we get e 2015 II
 2 e 2014 II (a) sin 3A (b) cos 3A (c) sin A + cos A (d) 3
(a) One (b) Two (c) Three (d) Infinity
MATHEMATICS Measurement of Angles and Trigonometric Ratios 143

III. If ABC is any triangle, then


110. If A = (cos 12° − cos 36° ) (sin 96° + sin 24° ) and A + B
B = (sin 60° − sin 12° ) (cos 48° − cos 72° ), then what tan  C C
 sin   < cos   .
 2   2  2
A
is equal to? Which of the above statement(s) is/are correct?
B e 2016 I
e 2016 I
(a) −1 (b) 0 (c) 1 (d) 2 (a) Only I (b) Only II (c) I and II (d) II and III
Directions (Q. Nos. 111-112) Consider the equation
Directions (Q. Nos. 114-115) Consider a ∆ABC in
k sin x + cos 2 x = 2k − 7 e 2016 I
π
111. If the equation possesses solution, then what is which cos A + cos B + cos C = 3 sin
3 e 2016 I
the minimum value of k? A B C
(a) 1 (b) 2 (c) 4 (d) 6 114. What is the value of sin sin sin ?
2 2 2
112. If the equation possesses solution, then what is (a)
1
(b)
1
(c)
1
(d)
1
the maximum value of k? 2 4 8 16
(a) 1 (b) 2 (c) 4 (d) 6
115. What is the value of
113. Consider the following statements A + B B + C C + A
cos  cos  cos ?
I. If ABC is an equilateral triangle, then  2   2   2 
3 tan( A + B) tan C = 1. 1 1
II. If ABC is a triangle in which A = 78° , B = 66°, (a) (b)
4 2
then tan  + C < tan A.
A 1
2  (c) (d) None of these
16

ANSWERS
1 b 2 b 3 d 4 d 5 d 6 b 7 c 8 d 9 a 10 c
11 c 12 c 13 a 14 c 15 d 16 b 17 c 18 b 19 d 20 a
21 c 22 a 23 a 24 b 25 c 26 b 27 b 28 c 29 a 30 a
31 a 32 b 33 d 34 b 35 b 36 a 37 c 38 c 39 c 40 a
41 c 42 c 43 a 44 a 45 c 46 d 47 c 48 b 49 b 50 a
51 b 52 b 53 c 54 b 55 c 56 b 57 c 58 a 59 c 60 d
61 a 62 d 63 b 64 c 65 c 66 a 67 b 68 d 69 d 70 c
71 d 72 a 73 c 74 d 75 a 76 d 77 b 78 d 79 b 80 b
81 c 82 a 83 c 84 b 85 d 86 a 87 a 88 a 89 b 90 d
91 b 92 b 93 c 94 b 95 a 96 b 97 a 98 d 99 c 100 a
101 c 102 a 103 b 104 b 105 b 106 d 107 b 108 b 109 d 110 c
111 b 112 d 113 b 114 c 115 d

 
144 NDA/NA Pathfinder

HINTS AND SOLUTIONS


71 ′ π π 5π
1. (b) 35′ 30′′ =  35 +  ′ =   5. (d) cos   + cos   + cos   A + B =α
1 10. (c) Given,
 2  2  9 3  9  and A−B=x
71 ′ 1  °  71  ° 7π
⇒   =  × + cos 
71 sin A ⋅ cos B 2
 =   Also,
tan A 2
= ⇒ =
 2  2 60   120   9 
tan B 1 sin B ⋅ cos A 1
71  °
∴ 114° 35′ 30′ ′ =  114 +  = cos ( 20° ) + cos ( 60° ) + cos ( 100° ) sin( A + B ) + sin( A − B ) 2
 120  ⇒ =
+ cos ( 140° ) sin( A + B ) − sin( A − B ) 1
13751 °
=  
1
= cos 20° + + 2 cos 120° cos 20° sin α + sin x 2
 120  ⇒ =
2 sin α − sin x 1
We know that, 2 π rad = 360° 1
= cos 20° + − 2 sin 30° cos 20° Using componendo and dividendo
13751 ° 2π 2
⇒ 
13751
 = × 1 1
rule,
 120  360° 120 = cos 20° + − cos 20° = 2 sin α 2 + 1 sin α 3
2 2 = ⇒ =
= 2.0008069 rad 2 sin x 2 − 1 sin x 1
7π π π
= tan  π +  = tan =
1 1
∴ 114° 35′ 30′ ′ = 2 rad (approx) 6. (b) tan ⇒ sin x = (sin α )
6  6 6 3 3
2. (b) Angle traced by hour hand in 1 h = 30° 9π  π π 11. (c) Let f ( x ) = (3 cos x + 4 sin x ) + 5
101 tan = tan  2 π +  = tan = 1
Angle traced by hour hand in h 4  4 4 We know that,
12 10 π  4π
101 ° tan = tan  2 π +  − a 2 + b 2 < a cos x + b sin x < a 2 + b 2
=  30 ×  = 252° 30′ 3  3 
 12  π ⇒ − 32 + 42 < 3 cos x + 4 sin x
= tan 
4  = tan  π + π 

Angle traced by minute hand in 1 min = 6°  3   3 < 32 + 42
Angle traced by minute hand in 25 min = tan π / 3 = 3 ⇒ − 25 < 3 cos x + 4 sin x ≤ 25
Clearly, 1 / 3 , 1, 3 are in GP. ⇒ −5< 3 cos x + 4 sin x ≤ 5
= ( 25 × 6)° = 150°
⇒ − 5 + 5 ≤ (3 cos x + 4 sin x + 5)
∴ Required angle 7. (c) Given expression
sin x + sin 7x + sin 3x + sin 5x < 5 + 5 ⇒ 0 < f ( x ) < 10
= ( 252° 30′ − 150° ) = 102° 30′ =
cos x + cos 7x + cos 3x + cos 5x Hence, the maximum value of f ( x ) is
1 ° 585° 
3. (d) sin  292  = sin   2 sin 4x cos 3x + 2 sin 4x cos x 10.
 2  2  =
2 cos 4x cos 3x + 2 cos 4x cos x 12. (c) Given, cot B − cot A = y …(i)
1 − cos 585°
=− 2 sin 4x [cos 3x + cos x ] and tan A − tan B = x
2 = = tan 4x
2 cos 4x [cos 3x + cos x ] 1 1
1 − cos 225° 1 + cos 45° ⇒ − =x
=− =− cot A cot B
2 2 8. (d) We have, (m 2 − n2 )
= (tan θ + sin θ)2 − (tan θ − sin θ)2 cot B − cot A
1+ 1/ 2 2+1 ⇒ =x
=− =− cot A ⋅ cot B
2 2 2 = 4 tan θ sin θ
y
⇒ cot A ⋅ cot B = [using Eq. (i)]
2+1 2 1 and mn = tan 2 θ − sin 2 θ x
=− × =− 2+2
2 2 2 2 sin 2 θ cot A ⋅ cot B + 1
= − sin 2 θ ∴ cot( A − B ) =
1° 1° cos 2 θ cot B − cot A
4. (d)  1 + cos 67   1 + cos 112  y
 2  2 1 − cos 2 θ +1
= sin θ 1 1
1° = x = +
=  1 + cos 67  cos 2 θ y x y
 2
= sin θ tan θ
  1°   13. (a) m 2 − n2 = (cos A + cos B )2
1 + cos  180° − 67   ∴ (m 2 − n2 ) = 4 mn − (sin A + sin B )2
 2 
 1°   1°  a cos φ + b = cos A + cos B + 2 cos A cos B
2 2
=  1 + cos 67   1 − cos 67  9. (a) cos θ =
 2  2 a + b cos φ − sin 2 A − sin 2 B − 2 sin A sin B
1° 1° Applying componendo and dividendo = (cos A − sin 2 A ) + (cos 2 B − sin 2 B )
2
= 1 − cos 67 = sin 67
2 2
2 2 rule, + 2[cos A cos B − sin A sin B ]
1 − cos 135° 1 − cos θ a( 1 − cos φ) − b( 1 − cos φ)
= = = cos 2 A + cos 2B + 2 cos ( A + B )
2 1 + cos θ a( 1 + cos φ) + b( 1 + cos φ)
2+1 = 2 cos( A + B ) ⋅ cos( A − B )+ 2 cos( A + B )
= θ a−b φ = 2 cos ( A + B )[cos ( A − B ) + 1] …(i)
2 2 ⇒ tan 2 = tan 2
2 a+ b 2
which is an irrational number and is m 2 + n2 = cos 2 A + cos 2 B + sin 2 A
θ a−b φ
less than 1. ∴ tan = tan + sin 2 B + 2 cos A cos B + 2 sin A sin B
2 a+ b 2
MATHEMATICS Measurement of Angles and Trigonometric Ratios 145

= 1 + 1 + 2 cos ( A − B ) 17. (c) sin x + sin 2 x = 1 So, Statement II is correct.


= 2[ 1 + cos ( A − B )] …(ii) ⇒ sin x = 1 − sin 2 x = cos 2 x Hence, both Statements are correct.
On dividing Eq. (i) by Eq. (ii), we get ∴ cos 8 x + 2 cos 6 x + cos 4 x 22. (a)
m 2 − n2 = sin x + 2 sin 3 x + sin 2 x
4

cos ( A + B ) = 2 23. (a) We have, z = 135° =
m + n2 = (sin 2 x )2 + (sin x )2 + 2 sin x (sin 2 x ) 4
= (sin 2 x + sin x )2 = 1 3π 3π
1 ∴ sin z + cos z = sin + cos
14. (c) cos A cos B = ( 2 cos A cos B ) 4 4
2 18. (b) We have,  π  π
1 2π 4π = sin  π −  + cos  π − 
= [cos ( A + B ) + cos ( A − B )] x = y cos = z cos = k [say]  4  4
2 3 3 π π 1 1
1 1 1 1 cos 2 π / 3 1 cos 4 π / 3 = sin − cos = − =0
= [cos 90° + cos ( A − B )] = , = , = 4 4 2 2
2 x k y k z k
=
1
cos ( A − B ) 24. (b) We have, x + y + z = 180°
2π 4π
∴ + + =  1+ cos
1 1 1 1
2 + cos  ⇒ x + y = 180°− z = 180°−135° = 45°
− 1 ≤ cos ( A − B ) ≤ 1 x y z k  3 3 
Q ∴ tan ( x + y ) = tan 45° = 1
1 1 1 xy + yz + zx 1 1 tan x + tan y
⇒ − ≤ cos ( A − B ) ≤ ⇒ = 1− − = 0 ⇒ = 1
2 2 2 xyz 2 2 1 − tan x tan y
15. (d) We have, ⇒ xy + yz + zx = 0 ⇒ tan x + tan y = 1 − tan x tan y
⇒ tan( x + 100° ) = tan( x + 50° )
19. (d) I. (sec θ + tan θ)−1 ⇒ tan x + tan x tan y + tan y = 1…(i)
tan x tan( x − 50° )
tan( x + 100° ) = (sec 1200° + tan 1200° )−1 Now, ( 1 + tan x )( 1 + tan y )
⇒ = tan( x + 50° ) tan x
tan( x − 50° ) = [sec( 6 π + 120° ) + tan( 6 π + 120° )]−1 = 1 + tan x + tan y + tan x ⋅ tan y
sin( x + 100° ) cos( x − 50° ) = (sec 120° + tan 120° )−1
⇒ = 1+ 1= 2 [using Eq. (i)]
cos( x + 100° ) sin( x − 50° ) = ( − cosec 30° − cot 30° )−1
sin( x + 50° ) sin x 1
= = ( − 2 − 3 ) −1 = − (negative) 25. (c) We have,
cos( x + 50° ) cos x 2+ 3 8x 2 − 2x − 1 = 8( x − cos α )( x − cos β )
sin( 2x + 50° ) cos 50° II. cosec θ − cot θ Put x = 1, then
⇒ =
sin 150° − cos( 2x + 50° ) = cosec ( 6 π + 120° ) − cot ( 6 π + 120° ) 5 = 8( 1 − cos α )( 1 − cos β )
= cosec 120° − cot 120° α β
[applying componendo and dividendo]
= sec 30° + tan 30° ⇒ 5 = 8 2 sin 2   2 sin 2 
⇒ sin( 2x + 50° ) cos( 2x + 50° )  2  2
2
= − sin 150° cos 50° = + 3 (positive) α β 5 5
3 ⇒ sin ⋅ sin = =
⇒ 2 sin( 2x + 50° ) cos( 2x + 50° ) 2 2 32 4 2
Hence, both Statements are incorrect.
= − cos50°
20. (a) In first quadrant, sin θ is increasing. 26. (b) 8x 2 − 2x − 1
⇒ sin( 4x + 100° ) = sin( 270°−50° ) = 8( x − cos α )( x − cos β )
⇒ sin( 4x + 100° ) = sin 220° cos 66° = cos ( 90° − 24° ) = sin 24°
sin 24° < sin 66° ⇒ cos 66° < sin 66° Put x = − 1, then
⇒ 4x + 100° = 220°
So, Statement I is correct. 9 = 8( − 1 − cos α )( − 1 − cos β )
⇒ x = 30°
Now, cos 26° = cos( 90°−64° ) = sin 64° ⇒ 9 = 8( 1 + cos α )( 1 + cos β )
16. (b) We have,
3 cos 2β − 1 sin 64° > sin 26° ⇒ cos 26° > sin 26° α β
( 1 − cos 2α ) = 1 − ⇒ 9 = 8  2 cos 2   2 cos 2 
3 − cos 2β So, Statement II is not correct.  2  2
3 − cos 2β − 3 cos 2β + 1
= 21. (c) The opposite angles of a cyclic α β 9 3
(3 − cos 2β ) quadrilateral are supplementary. ∴ cos ⋅ cos = =
2 2 32 4 2
4( 1 − cos 2β ) ∴ A + C = 180° and B + D = 180°
=
(3 − cos 2β ) ⇒ A = 180° − C and B = 180° − D 27. (b) sec x + tan 3 x cosec x
= sec x  1 + tan 3 x
3 cos 2β − 1 cosec x 
and ( 1 + cos 2α ) = 1 + ⇒ cos A = cos ( 180° − C ) = − cos C 
 sec x 
3 − cos 2β cos B = cos ( 180° − D ) = − cos D
3 − cos 2β + 3 cos 2β − 1 ⇒ cos A + cos B = sec x ( 1 + tan x )2
=
3 − cos 2β = − (cos C + cos D ) …(i) = sec x (sec 2 x ) = sec 3 x
2( 1 + cos 2β ) So, Statement I is correct. = (sec 2 x )3 / 2 = ( 1 + tan 2 x )3 / 2
=
(3 − cos 2β ) Now, cos ( π + A ) + cos ( π − B ) = ( 2 − e 2 )3 / 2
1 − cos 2α 2( 1 − cos 2β ) π
∴ = + cos ( π − C ) − sin  − D  28. (c) cos 2x =
1 − tan 2 x
1 + cos 2α ( 1 + cos 2β ) 2 
1 + tan 2 x
2 sin 2 α 4 sin 2 β = − cos A − cos B − cos C − cos D 1− (1− e2 )
⇒ = =
2 cos 2 α 2 cos 2 β = − (cos A + cos B + cos C + cos D )
1+ (1− e2 )
= − ( − cos C − cos D + cos C + cos D )
⇒ tan 2 α cot 2 β = 2 e2
⇒ tan α cot β = 2
[using Eq. (i)] =
=0 2 − e2
146 NDA/NA Pathfinder

29. (a) sec 4 x − sec 2 x = sec 2 x (sec 2 x − 1) 1 1 43. (a) Given that, cot A ⋅ cot B = 2 …(i)
35. (b) z = =
= ( 1 + tan x )(tan x )
2 2 1 − sin 2 θ cos 2 θ  1 cos A ⋅ cos B 2
1−   ⇒ =
 xy  sin A ⋅ sin B 1
= ( 2 − e 2 )( 1 − e 2 ) = ( e 2 − 2)( e 2 − 1) xy cos A ⋅ cos B − sin A ⋅ sin B 2 − 1
⇒ z= ⇒ =
Sol. (Q. Nos. 30-33) Let BD = p , DE = x ⇒ xy − 1 cos A ⋅ cos B + sin A ⋅ sin B 2 + 1
AC = 4 p ⇒ xyz − z = xy [by componendo and dividendo rule]
cos ( A + B ) 1
Let E be the mid-point of AC, then ⇒ xyz = xy + z ⇒ =
AE = EC = BE = 2 p cos ( A − B ) 3
4
36. (a) sin 2 2θ = 4 sin 2 θ ⋅ cos 2 θ = ⇒ cos ( A + B ) ⋅ sec( A − B ) =
1
A xy 3
cos 2 2θ = ( 2 cos 2 θ − 1)2
2 44. (a) Let f (θ) = sin 3θ ⋅ cos 2θ
4p 2  ( 2 − y )2 + cos 3θ ⋅ sin 2θ
2p =  − 1 =
D x y  y2 = sin (3θ + 2θ) = sin 5θ
E [Q sin ( A + B ) = sin A ⋅ cos B
p 4 y2 4y
∴ tan 2 2θ = × = + cos A ⋅ sin B ]
α
2p 2p xy ( 2 − y )2 x ( 2 − y )2
B We know that,
β C
1 1 1 1 −1 ≤ sin 5θ ≤ 1 ⇒ −1 ≤ f (θ) ≤ 1
37. (c) I. + = +
Now, in ∆BDE , x y cos ec 2θ sec 2 θ So, the maximum value of f (θ) is 1.
( BE )2 = ( BD )2 + ( ED )2 = sin 2 θ + cos 2 θ = 1
45. (c) Given, sec 2 tan −1   
⇒ ( 2 p )2 = ( p )2 + ( x )2 5
So, Statement I is correct.
  11 
⇒ 4 p2 = p2 + x 2 ⇒ x 2 = 3 p2
1 1 2  −1  5  
⇒ x = 3⋅ p II. We have, + = 1 ⇒ x + y = xy = 1 + tan tan   
x y   11 
∴ AD = 2 p − x = 2 p − 3 p
∴ xyz = xy + z = x + y + z  5 
2
and DC = 2 p + x = 2 p + 3 p
1 1 1 = 1 +  tan tan −1   
30. (a) In ∆BAD, tan A =
p ⇒ 1= + +    11 
2p − 3p xy yz zx 2
= 1 +   = 1 +
5 25 146
1 2+ 3 So, Statement II is also correct. =
= ×  11 121 121
2− 3 2+ 3
2+ 3 38. (c) 46. (d) Let f ( A ) = sin A ⋅ cos A ⋅ tan A
⇒ tan A = = tan 75°
1 39. (c) sin ( 1920° ) = sin(360° × 5° + 120° ) + cos A ⋅ sin A ⋅ cot A
= sin 120° = sin( 90° + 30° ) sin A
⇒ A = 75° = sin A ⋅ cos A ⋅
p 3 cos A
In ∆BDC , tanC = = cos 30° = cos A
2p + 3p 2 + cos A ⋅ sin A ⋅
sin A
2− 3 [Q sin ( 90° + θ) = cos θ]
= =2− 3 = sin 2 A + cos 2 A = 1
( 2 + 3 )( 2 − 3 ) sin θ cos θ
40. (a) Given that, + [Q sin 2 θ + cos 2 θ = 1]
⇒ tan C = tan 15° ⇒ C = 15° cosec θ sec θ
AD sin θ cos θ 47. (c) sec θ + tan θ = 4 …(i)
31. (a) In ∆ABD, tan B = = +
BD ( 1 / sin θ) ( 1 / cos θ) As we know that, sec 2 θ − tan 2 θ = 1
2p − 3p ⇒ (sec θ + tan θ) (sec θ − tan θ) = 1
= = 2 − 3 = tan 15° = sin 2 θ + cos 2 θ = 1
p 1
⇒ sec θ − tan θ = …(ii)
∴ ∠ABD = 15° 41. (c) 4
1
AD 2 p − 3 p 2 − 3 2 − 3 42. (c) Given, cosec θ − cot θ = On adding Eqs. (i) and (ii), we get
32. (b) = = × 3
DC 2 p + 3 p 2 + 3 2 − 3 1 17
1 cos θ 1 2 sec θ = 4 + =
4+3−4 3 7−4 3 ⇒ − = 4 4
= = sinθ sin θ 3 17 8
4 −3 1 ( 1 − cos θ) 1 ⇒ sec θ = ⇒ cos θ =
⇒ = 8 17
⇒ AD : DC = ( 7 − 4 3 ) : 1 sin θ 3 2
θ
1 −  
33. (d) tan ( A − C ) = tan ( 75° − 15° ) 8 15
2 sin 2 ∴ sin θ = =
= tan 60° = 3 2 = 1  17  17

θ θ 3
1 1 1 1 2 sin ⋅ cos
34. (b) 2 + 2 = + 2 2 48. (b) We have, sin 420° ⋅ cos 390°
x y cosec 4θ sec 4 θ + cos ( − 300° ) ⋅ sin ( − 330° )
θ θ
= sin 4 θ + cos 4 θ ⇒ tan = tan 30° ⇒ = 30°
2 2 = sin (360° + 60° ) ⋅ cos (360° + 30° )
= (sin 2 θ + cos 2 θ)2 − 2 sin 2 θ cos 2 θ ⇒ θ = 60° + cos 300° ( − sin 330° )
= 1 − 2 sin 2 θ cos 2 θ 1
2 −z ∴ cos θ = cos 60° = = sin 60° ⋅ cos 30° − cos (360° − 60° )
= 1 − 2 1 −  =
1 2
 z  z ⋅ sin (360° − 30° )
MATHEMATICS Measurement of Angles and Trigonometric Ratios 147

π π π π
= sin 60° ⋅ cos 30° − cos 60° ⋅ ( − sin 30° ) = tan  +  ⋅ tan  +  62. (d)
2 6  2 3
= sin 60° ⋅ cos 30° + cos 60° ⋅ sin 30° 63. (b) tan 105° = tan( 60° + 45° )
π π
=  − cot   − cot  = − ( 3 ) ⋅  −  tan 60°+ tan 45°
1
= sin ( 60° + 30° ) = sin 90° = 1 =
 6  3  3 1 − tan 60°⋅ tan 45°
49. (b) Given, θ = 18° =1 3+1 3+1
Now, we have 4 sin 2 θ + 2 sin θ = =
54. (b) Now, sin 2 A − sin 2 B 1− 3 ⋅ 1 1− 3
= 4 sin 2 ( 18° ) + 2 sin ( 18° )
2 = sin 2 ( π / 3) − sin 2 ( π / 6) 64. (c) Given equation, sin θ + 2 cos θ = 1
 5 − 1  5 − 1
=4  +2   3
2 2
On squaring both sides, we get
=   −   = − = =
1 3 1 2 1
 4   4 
 2   2 4 4 4 2 (sin θ+ 2 cos θ)2 = 1
4(5 + 1 − 2 5 ) ( 5 − 1)
= + ⇒ sin θ + 4 cos θ+ 4 sin θ ⋅ cos θ = 1
2 2
16 2 55. (c)
⇒ ( 1− cos 2 θ) + 4( 1 − sin 2 θ )
3− 5 5−1 56. (b) tan( − 585° ) = − tan(585° )
= + =1 + 4 sin θ ⋅ cos θ = 1
2 2 = − tan( 180° × 3 + 45° )
π 180°  = − tan 45° = − 1 ⇒ 4 sin 2 θ + cos 2 θ− 4 sin θ ⋅ cos θ = 4
50. (a) tan   = tan   = tan 15°
 12   12°  57. (c) Q sec α =
13
⇒ cosα =
5 ⇒ ( 2 sin θ − cos θ)2 = 4 = ( 2)2
tan 45°− tan 30° 5 13 ∴ 2 sin θ − cos θ = 2
= tan ( 45°−30° ) =
1 + tan 45°⋅ tan 30° 25
⇒ sin α = 1 − cos 2 α = 1− 65. (c) Given that,
1 169
1− cosec θ + cot θ = c
= 3 = 3−1 × 3−1 144 12 cos θ 1 + cos θ
= =− ⇒
1
+ =c⇒ =c
1+
1 3+1 3−1 169 13 sin θ sin θ sin θ
3
Since, sinα is negative in fourth θ θ
( 3 − 1)2 3 + 1 − 2 3 2 cos 2 cos
= = = 2− 3 quadrant, i.e. 270° < α < 360°. ⇒ 2 =c ⇒ 2 =c
3−1 2 θ θ θ
58. (a) sin 15° = sin( 45° − 30° ) 2 sin ⋅ cos sin
51. (b) 2 2 2
= sin 45° cos 30° − cos 45° sin 30° θ θ 1
π 3.14 ⇒ cot = c ⇒ tan = ...(i)
I. 1° = radian = = 0.017 1 3 1 1 3−1
180 180 = × − × = 2 2 c
2 2 2 2 2 2
= 0.02 (approx) Now, we have
59. (c) 2
θ
1 − tan 2   1 −  
1
5−1
which is equal to 0.02. 60. (d) Q sin 18° = 2   c
180 4 cosθ = =
II. 1 radian = θ 2
π
degree
and
1
cos 36° = ( 5 + 1) 1 + tan 2   1 +  1
4 2   c
180
= = 57.32 degree  5 − 1 1
∴ sin 18° cos 36° =   ( 5 + 1) c2 − 1
3.14 =
Which is greater than 4⋅5°.  4  4 c2 + 1
(5 − 1) 4 1
Sol. (Q. Nos. 52-54) = = = 66. (a) tan 4 A − sec 4 A + tan 2 A + sec 2 A
16 16 4
Given that, sin ( A + B ) = 1, = (tan 2 A )2 − (sec 2 A )2 + (tan 2 A + sec 2 A )
π 61. (a) Given that, A and B are acute angles.
⇒ sin ( A + B ) = sin = (tan 2 A − sec 2 A )(tan 2 A + sec 2 A )
2 i.e. A < 90° and B < 90° and
π + (tan 2 A + sec 2 A )
⇒ A+B = ...(i) 2 1
sin A = , cos B = , = ( −1)(tan A + sec A )
2 2
2 5 10
and sin ( A − B ) =
1 + (tan 2 A + sec 2 A )
We know that, sin θ + cos
2
θ=1
2
2 = − (tan 2 A + sec 2 A ) + (tan 2 A + sec 2 A )
π 4 1
⇒ sin ( A − B ) = sin ∴ cos A = 1 − sin A =
2
1− = =0
3 5 5
π
⇒ A−B = …(ii) 1 3 67. (b) Given that,
3 sin B = 1 − cos 2 B = 1− = A + B = 90° …(i)
10 10
On adding Eqs. (i) and (ii), we get
2π π π Q sin ( A + B ) Now, sin A ⋅ sec B − sin A ⋅ cos B
2A = ⇒ A = and B =
3 3 6 = sin A ⋅ cos B + cos A ⋅ sin B = sin A ⋅ sec( 90°− A ) − sin A ⋅ cos( 90°− A )
2 1 1 3
52. (b) = ⋅ + ⋅ = sin A ⋅ cosec A − sin A ⋅ sin A
5 10 5 10
53. (c) Now, tan( A + 2B ) ⋅ tan( 2 A + B ) 2+3 5 5 1
= = = = sin A ⋅ − sin 2 A = 1 − sin 2 A
π π 2π π
= tan  +  ⋅ tan  +  5 10 5 10 10 sin A
3 3  3 6 1
= = sin 135° = cos 2 A = cos A
2π 5π
= tan   ⋅ tan   2
 3   6 
∴ A + B = 135°
148 NDA/NA Pathfinder

68. (d) Given that, tan A = x + 1 ...(i) 76. (d) ( 1 − sin 2 θ) ( 1 + tan 2 θ) = − cos C + cos C = 0
and tan B = x − 1 ...(ii) 1 [Q in second quadrant, cos θ < 0]
= cos θ ⋅ sec θ = cos θ ×
2 2 2
=1
Now, x 2 tan( A − B ) cos 2 θ
83. (c) Given that, 3 tan 2 x = 1
tan A − tan B  77. (b)
= x 2   π ⇒ tan 2 x =
1
⇒ tan x = ±
1
I. tan   =
 1 + tan A ⋅ tan B  1
 6 3 3
3
 ( x + 1) − ( x − 1)  ⇒ tan x = tan ( ± π / 6)
= x2  3π

 1 + ( x + 1) ⋅ ( x − 1)  II. tan   = − 1 ∴
π
x = nπ ± , where n ∈ Z
 4 
6
5π π π
III. tan   = tan  π +  = tan = 1
[from Eqs. (i) and (ii)]
84. (b)
 2 =x2 ⋅ 2 = 2  4   4 4 3π
=x  85. (d) In the interval  π ,
2
 , both sinθ
 1 + x − 1
2
x2 2π π π  2 
IV. tan   = tan  π −  = − tan
 3   3 3 and cosθ are negative.
69. (d) (sin 4 θ − cos 4 θ + 1) ⋅ cosec 2 θ
=− 3 As |x | and x 2 are positive and can
= {(sin 2 θ − cos 2 θ)(sin 2 θ + cos 2 θ) + 1} So, the correct order is IV < II < I < III. take any value which lies in the
⋅ cosec 2 θ 3π
78. (d) We have, cos 20°+ cos 100°+ cos 140° interval  π , .
= {(sin 2 θ − cos 2 θ) ⋅ 1 + 1}cosec 2 θ = (cos 140° + cos 20° ) + cos 100°  2 
= {sin 2 θ − cos 2 θ + 1 } cosec 2 θ = 2 cos (80° ) ⋅ cos 60° + cos 100° So, both statements are incorrect.
1
= {sin θ − ( 1− sin θ) + 1} cosec θ
2 2 2 = 2 cos 80°⋅ + cos 100° 86. (a)
1 2
= ( 2 sin 2 θ) ⋅ =2 = cos 80° + cos 100° 87. (a) I. We have,
sin 2 θ
135° 45° 
cos x 1 = 2 cos 90°⋅ cos 10° n  sin 2 − sin 2 
+ −1 = 2 × 0 × cos 10° = 0  2 2 
cot x + cosecx − 1 sin x sin x 135° 45° 
70. (c) = = nsin  +  135 − 45 
cot x − cosecx + 1 cos x − 1 + 1  ⋅ sin  
79. (b) We have,  2 2   2 2
sin x sin x
cos x + 1 − sin x sin 2 (3 π ) + cos 2 ( 4 π ) + tan 2 (5 π ) [Q sin 2 A − sin 2 B = sin( A + B ) ⋅ sin( A − B )]
=
cos x − 1 + sin x = sin 2 ( 2 π + π) + cos 2 ( 2 π + 2 π ) n n 2
= n sin 90°⋅ sin 45° = =
x x
2 cos 2 − 2 sin ⋅ cos
x + tan 2 ( 4 π + π) 2 2
= 2 2 2 = sin π + cos 2 π + tan 2 π
2 2
n 2
x x
− 2 sin 2 + 2 sin ⋅ cos
x ⇒ > 1 for all positive integers
= ( 0)2 + ( 1)2 + ( 0)2 2
2 2 2
= 0 + 1+ 0 = 1
x x x n≥ 2
2 cos  cos − sin  cot 224°− cot 134°
= 2 2 2 80. (b) Hence, Statement I is correct.
x x x cot 226°+ cot 316°
2 sin  cos − sin  cot (360°−136° ) − cot ( 90° + 44° ) II. Given that, x is any positive real number
2 2 2 =
x x cot (360°−134° ) + cot (360° − 44° ) i.e. x ∈ R+ ⇒ x ∈ ( 0, ∞ )
2 cos ⋅ cos If x ← ( 0, 1) i.e. 0 < x < 1
= 2 2 1 + cos x − cot 136°+ tan 44°
x x = sin x =
then 0 < nx < 1, ∀n > 0
2 sin cos − cot 134°− cot 44°
2 2 − cot ( 90° + 46° ) + tan 44° Hence, Statement II is false.
=
71. (d) − cot ( 90° + 44° ) − cot( 90°−46° ) 88. (a) I. Given that, sin 3θ = cos 2θ
π
72. (a) tan 46°+ tan 44° ⇒ sin 3θ = sin  − 2θ
= 2 
cot 54° tan 20° tan 44°− tan 46°
73. (c) + [since, 3θ is an acute angle]
tan 36° cot 70°  sin 46° + sin 44°  π π
  ⇒ 3θ = − 2θ ⇒ 5θ =
cot 54° tan 20°  
= + = cos 46° cos 44° 2 2
tan( 90° − 54° ) cot ( 90° − 20° )  sin 44° − sin 46°  π
  ⇒ θ=
 cos 44° cos 46°  10
cot 54° tan 20°
= + = 1+ 1= 2 sin 46°⋅ cos 44°+ sin 44°⋅ cos 46° Hence, Statement I is correct.
cot 54° tan 20° = II. By definition of radian system, a
1 sin 44°⋅ cos 46°− sin 46°⋅ cos 44°
radian is the angle subtended at the
74. (d) Given that, cos x = sin( 46°+44° )
3 = centre of a circle by an arc, whose
sin( 44°−46° ) length is equal to the radius of the
We have, sin x ⋅ cot x ⋅ cosec x ⋅ tan x
sin 90° 1 circle.
cos x 1 sin x = = = − cosec 2°
= sin x ⋅ ⋅ ⋅ =1 sin( −2° ) − sin 2° Hence, Statement II is false.
sin x sin x cos x
89. (b) Consider, 1 + sin 2θ
75. (a) Given, sin 2 20° + sin 2 70° 81. (c)
= sin 2 θ + cos 2 θ + 2 sin θ cos θ
= sin 2 20° + sin 2 ( 90° − 20°) 82. (a) Given that, A + B + C = π … (i)
= (sin θ + cos θ)2
= sin 2 20° + cos 2 20° = 1 Now, we have, cos ( A + B ) + cos C
= cos ( π − C ) + cos C [from Eq. (i)] = sin θ + cos θ
[Q sin 2 θ + cos 2 θ = 1]
MATHEMATICS Measurement of Angles and Trigonometric Ratios 149

sin( x + y ) a + b 97. (a) Clearly, p = 1, hence option (a) is


90. (d) We have, cot A = 2 and cot B = 3 94. (b) =
cot A cot B − 1 sin( x − y ) a − b correct.
cot( A + B ) =
cot A + cot B Applying componendo and dividendo, 98. (d) Clearly, q = − 5, hence option (d) is
6−1 5 we get correct.
= = =1
2+3 5 sin( x + y ) + sin( x − y ) 99. (c) Clearly, r = 10, hence option (c) is
π π sin( x + y ) − sin( x − y ) correct.
⇒ cot( A + B ) = cot   ⇒ A + B =
 4 4 ( a + b) + ( a − b) 100. (a) We have, y = 3 ...(i)
=
1° 1° ( a + b) − ( a − b) Also, to meet the graph of y = tan x
91. (b) Consider, sin 66 − sin 23
2 2
2 2 2 sin x ⋅ cos y 2a and y = 3, we should have
2 ⇒ =
  1°   1° 2 cos x ⋅ sin y 2b 3 = tan x ⇒ tan x = tan 60°
= sin  90° − 23  − sin 23 2
  2   2 a
⇒ tan x ⋅ cot y = Q x ∈  0, π  
° 1° ⇒ x = 60°  
2 1
= cos 23 − sin 23 2 b   2  
tan x a
2 2 ∴ =
tan y b Hence, one intersecting point possible in

= cos 2  23  given domain, i.e. k = 1.
 2 95. (a) We have, θ° 180°
[Q cos 2 A = cos 2 A − sin 2 A ] sin A ⋅ sin( 60°− A ) sin ( 60°+ A ) 101. (c) Given, =
= k ⋅ sin3 A θ° π
 47 ° 
= cos  2 ×    = cos 47° ⇒ sin A [sin 2 60°− sin 2 A ] = k ⋅ sin 3 A
⇒ θ° =
π × θ°
and θ° × θ ° =
125 π
  2 
⇒ sin A  − sin 2 A  = k ⋅ sin 3 A 180°
 3 9
 4  π × θ° 125 π
cos 7x − cos 3x ∴ θ° × = = 125 × 20°
92. (b) 3 sin A − 4 sin 3 A 180° 9
sin 7x − 2 sin 5x + sin 3x ⇒ = k ⋅ sin 2 A = 25 × 100° = (5 × 10° )2
7x + 3x   7x − 3x  4
−2 sin   ⋅ sin   sin 3 A 1 ⇒ θ° = 50°
 2   2  ⇒ = k ⋅ sin 3 A ⇒ k =
=
7x + 3x   7x − 3x  − 2 sin 5x 4 4 102. (a) 25 cos 2 θ + 5 cos θ − 12 = 0
2 sin   ⋅ cos   ⇒ 25 cos 2 θ + 20 cos θ
 2   2  96. (b) We have, tan 2θ ⋅ tan θ = 1
−2 sin 5x ⋅ sin 2x − 15 cosθ − 12 = 0
= 2 tan θ ⇒ 5 cos θ (5 cos θ + 4)
2 sin 5x cos 2x − 2 sin 5x ⇒ ⋅ tan θ = 1
−2 sin 5x ⋅ sin 2x sin 2x 1 − tan 2 θ − 3 (5 cos θ + 4) = 0
= = ⇒ (5 cos θ − 3)(5 cos θ + 4) = 0
−2 sin 5x [ 1 − cos 2x ] 1 − 1 + 2 sin 2 x ⇒ 2 tan 2 θ = 1 − tan 2 θ
3 4
⇒ 3 tan 2 θ = 1 ⇒ cosθ = or cosθ = −
[Q cos 2x = 1 − 2 sin x ]
2
2 5 5
=
2 sin x cos x
= ⇒ tan 2 θ = =
1  1 π
  α ∈  , π 
cot x
2 sin 2 x 3  3 Q
2 
2  π
93. (c) In a ∆ABC, we have, ⇒ tan θ = tan  
2
Since, α is a root of the given equation.
 6
4 3
sin A − cos B = cos C π π ∴ cosα = − ⇒ tanα = −
⇒ θ = nπ ± ⇒ θ= 5 4
⇒ sin A = cos B + cos C 6 6
A A 103. (b) From the above solution, we have
⇒ 2 sin ⋅ cos Sol. (Q. Nos. 97-99) Consider,
2 2 sin 2α = 2 sin α ⋅ cos α
 B+C  B −C  16 sin 5 x = 16(sin 2 x )2 ⋅ sin x
= 2 cos   ⋅ cos  = 2 × × −   = −
 2
3 4 24
 2   2  1 − cos 2x 
= 16   ⋅ sin x 5 5 25
A A  2 
⇒ 2 sin ⋅ cos 104. (b) ( 1 − sin A + cos A )2
2 2 = 4( 1 + cos 2 2x − 2 cos 2x ) ⋅ sin x
B −C  = 1 + sin 2 A + cos 2 A − 2 sin A
= 2 cos  90° −  ⋅ cos 
 1 + cos 4x
= 4  1 + − 2 cos 2x  ⋅ sin x
A
 − 2 sin A cos A + 2 cos A
 2  2   2 
= 2 − 2 sin A − sin 2 A + 2 cos A
[Q A + B + C = 180° ] 4
= (3 + cos 4x − 4 cos 2x ) ⋅ sin x
A A = 2( 1 + cos A ) − 2 sin A( 1 + cos A )
⇒ 2 sin ⋅ cos 2
2 2 = ( 6 + 2 cos 4x − 8 cos 2x ) sin x = 2( 1 − sin A )( 1 + cos A )
B −C 
= 2 sin ⋅ cos 
A = 6 sin x + 2 sin x cos 4x 105. (b)

2  2  −8 cos 2x ⋅ sin x
B −C  106. (d) sin 2 5° + sin 2 10° + sin 2 15°
= cos  = 6 sin x + sin 5x − sin 3x
A
⇒ cos  + ... + sin 2 90°
2  2  − 4(sin 3x − sin x )
A B −C [Q 2 sin A cos B = sin ( A + B ) = (sin 5° + sin 85° )
2 2
⇒ = ⇒ A+C = B ...(i)
2 2 + sin ( A − B )] + (sin 2 10° + sin 2 80° ) + ... +
Also, A + C = 180°− B ...(ii) = 6 sin x + sin 5x − sin 3x − 4 sin 3x sin 2 45° + ... + sin 2 90°
So, 180°− B = B ⇒ 2B = 180° + 4 sin x
= sin 2 5° + sin 2 ( 90° − 5° ) + sin 2 10°
∴ B = 90° = sin 5x − 5 sin 3x + 10 sin x
+ sin 2 ( 90° − 10° ) + ... + sin 2 90°
150 NDA/NA Pathfinder

sin 3 A + sin 3 A cos 3 A − cos 3 A 78°


= (sin 2 5°+ cos 2 5° ) 109. (d) + = tan  + 36°  = tan 75°
+ (sin 10°+ cos 10° ) + ... + sin 90°
2 2 2 sin A cos A  2 
2 sin 3 A + (3 sin A − 4 sin 3 A ) [Q ∠C = 180° − ∠A − ∠B
= 1+ 1+ 1+ 1+ 1+ 1+ 1 +1 + 
1  =
 +1 = 180° − 78° − 66° = 36° ]
 2 sin A
cos 3 A − ( 4 cos 3 A − 3 cos A ) As, tan 75° < tan 78°
=9+ =
1 19 +
2 2 cos A [Q value of tan θ increases, as θ varying
= ( − 3 sin 2 A + 3) + ( − 3 cos 2 A + 3)
107. (b) We have, sin x + sin y = a ...(i) from 0° to 90°]
= 6 − 3(sin 2 A + cos 2 A ) = 6 − 3( 1) = 3
and cos x + cos y = b ...(ii)  A 
Hence, tan  + C  < tan A
On squaring Eqs. (i) and (ii) and then 110. (c) We have, 2 
adding, we get A = (cos 12° − cos 36° ) III. In any ∆ABC ,
(sin 96° + sin 24° )
⇒ 2 + 2 cos ( x − y ) = a 2 + b 2 ∠A + ∠B + ∠C = π
a2 + b2 − 2 = − 2 sin 24° sin ( −12° )
A + B  π C 
⇒ cos ( x − y ) = ...(iii) ( 2 sin 60° cos 36° ) A + B = π − C   = − 
2  2   2 2
= 4 sin 24° sin 12° sin 60° cos 36°
Now, dividing Eq. (i) by Eq. (ii), we get A + B
sin x + sin y and B = (sin 60°− sin 12° ) Now, tan  C  C 
 sin   < cos  
=
a  2   2  2
cos x + cos y b (cos 48°− cos 72° )
π C
tan  −  sin   < cos  
C C
x+ y a = 2 cos 36° sin 24°
⇒ tan   = ...(iv)
{( −2 sin 60° sin ( −12° )}
 2 2  2  2
 2  b
cot   sin   < cos  
C C C
x+ y 2 x− y = 4 sin 24° sin 12° sin 60° cos 36°
∴ tan 2   + tan    2  2  2
 2   2  Clearly,
A
=1
cos  
B C
a2 1 − cos( x − y )
= +  2
⋅ sin   < cos  
C C
b2 1 + cos( x − y ) Sol. (Q. Nos. 111-112) Given equation is
 C  2  2
k sin x + cos 2x = 2k − 7 sin  
a + b −2 2 2
 2
1− ⇒ k sin x + 1 − 2 sin 2 x = 2k − 7
a2
cos   < cos  
= 2 + 2 C C
b a2 + b2 − 2 ⇒ −2 sin 2 x + k sin x + 8 − 2k = 0  2  2
1+
2 ⇒ 2 sin 2 x − k sin x + 2k − 8 = 0 which is clearly false.
a 2
4 − a2 − b2 k± k 2 − 16k + 64
= 2 + ∴ sin x = 114. (c) Given, in ∆ABC,
b a2 + b2 4 π
cos A + cos B + cos C = 3 sin
a 4 − b 4 + 4b 2 k ± ( k − 8) 3
= ⇒ sin x =
a2b2 + b4 4 ⇒ cos A + cos B + cos C
2k − 8 3 3
11π   11π  ⇒ sin x = = 3× =
108. (b) p = tan  −  = − tan   4 2 2
 6   6 
π and sin x = 2 [not possible] We know that, cos A + cos B + cos C
= − tan  2 π −  A B C
 6 As,−1 ≤ sin x ≤ 1 = 1 + 4 sin sin sin
π 2 2 2

= tan   =
1 2k − 8 A B C 3
 6 ⇒ −1 ≤ ≤1 ⇒ 1 + 4 sin sin sin =
3 4 2 2 2 2
 21π  π
q = tan   = tan  5 π +  ⇒ 4 ≤ 2k ≤ 12 ⇒ 2 ≤ k ≤ 6 ⇒ 4 sin
A B C 3
= − 1=
1
 4   4 sin sin
2 2 2 2 2
π 111. (b) The minimum value of k is 2.
= tan   = 1 ⇒ sin
A B
sin sin
C
=
1
 4 112. (d) The maximum value of k is 6. 2 2 2 8
 283 π 
r = cot   A + B  B + C  cos  C + A 
115. (d) cos 
113. (b)
 6   cos    
I. Since, ABC is an equilateral triangle.  2   2   2 
π
= cot  47 π +  ∴ ∠A = ∠B = ∠C = 60° = cos  90°−  cos  90°− 
C A
 6
Now, 3 tan ( A + B ) tan C  2  2
π
= cot = 3
cos  90°− 
= 3 tan 120° tan 60° B
6
 2
Since, p × r =
1
× 3 = 1 = q2 = 3 tan ( 180° − 60° ) tan 60°
3 = ( −3) tan 60° tan 60° [Q A + B + C = 180° ]
p, q and r is GP. = ( −3 ) ⋅ 3 ⋅ 3 = − 9 C A B A B C
= sin sin sin = sin sin sin
II. Consider, tan  + C 
Hence, only Statement II is correct. A 2 2 2 2 2 2
2  =
1
[from Q. No. 119]
8
13
MATHEMATICS Decimal Fractions

INVERSE TRIGONOMETRIC
FUNCTIONS

In NDA exam generally, generally 2-3 questions are asked from this chapter which are based on
finding the value of angle/solving the equation by using properties of inverse trigonometric

INVERSE FUNCTION
If f : X → Y is a function which is both one-one and onto, then its inverse function f −1 : Y → X is
defined as y = f ( x) ⇔ f −1 ( y) = x, ∀ x ∈ X , ∀ y ∈Y

INVERSE TRIGONOMETRIC FUNCTIONS


Trigonometric functions are not one-one and onto on their natural domains and ranges, so in order to
make them one-one and onto, we restrict their domain and codomain. And thus we obtain inverses of
trigonometric functions. Inverse of f is denoted by f −1 .
Let y = f ( x) = sin x be a function. Then, its inverse is x = sin −1 y.
 → sin −1 x.
sin x Inverse
Domain and Range of Inverse Trigonometric Functions
y Domain x Range y (Principal Value)
−1
(i) sin x −1 ≤ x ≤ 1 − π /2 ≤ y ≤ π /2
−1
(ii) cosec x x ≤ − 1 or x ≥ 1 − π / 2 ≤ y ≤ π / 2, y ≠ 0

(iii) cos −1 x −1 ≤ x < 1 0≤ y≤ π

(iv) sec −1 x x ≤ − 1 or x ≥ 1 0 ≤ y ≤ π, y ≠ π / 2
−1
(v) tan x − ∞< x< ∞ − π /2 < y < π /2
−1
(vi) cot x − ∞< x< ∞ 0< y< π
152 NDA/NA Pathfinder

EXAMPLE 1. The domain of sin −1 [x] is given by (e) sec −1 ( − x) = π − sec −1 x


a. [ − 1, 1] b. [ − 1, 2 ) (f) cosec −1 ( − x) = − cosec −1 x
c. { − 1, 0 , 1} d. None of these
(iv) (a) 2sin −1 x = sin −1 2x (1 − x 2 ) = cos −1 (1 − 2x 2 )
Sol. b. Domain of sin−1 x is [− 1, 1]
and domain of sin−1[x ] is {x : − 1 ≤ [x ] ≤ 1} (b) 2cos −1 x = cos −1 ( 2x 2 − 1) = sin −1 ( 2x 1 − x 2 )
= {x : [x ] = − 1, 0, 1} 2x 2x
(c) 2 tan −1 x = tan −1 = sin −1
 − 1, − 1 ≤ x < 0 1− x 2
1 + x2

But [x ] =  0, 0≤ x < 1 1 − x2
+ 1 , 1 ≤ x < 2 = cos −1
 1 + x2
−1 −1
∴ x ∈ [− 1, 2 ) (d) 3sin x = sin ( 3x − 4x ) 3

(e) 3cos −1 x = cos −1 ( 4x 3 − 3x)


Properties of Inverse
3x − x 3
Trigonometric Functions (f) 3 tan −1 x = tan −1
(i) (a) sin −1 (sin θ) = θ and sin (sin −1 x) = x, 1 − 3x 2
π π (v) (a) sin −1 x + cos −1 x = π/ 2, ( −1 ≤ x ≤ 1)
− ≤ θ ≤ , −1≤ x ≤ 1
2 2
−1 −1
(b) tan −1 x + cot −1 x = π/ 2, x ∈ R
(b) cos (cos θ) = θ and cos (cos x) = x,
(c) sec −1 x + cosec −1 x = π/ 2, x ≤ − 1 or x ≥ 1
0 ≤ θ ≤ π, − 1 ≤ x ≤ 1
−1 −1
(c) tan (tan θ) = θ and tan (tan x) = x,  1 
π π EXAMPLE 3. If cos −1   = θ, then what is the value
− < θ < , x ∈R  5
2 2 of cosec −1 ( 5)?
−1 −1
(d) cot (cot θ) = θ and cot (cot x) = x, π π π
0 < θ < π, x ∈ R a. +θ b. −θ c. d. − θ
2 2 2
−1 −1
(e) sec (sec θ) = θ and sec (sec x) = x Sol. b. Q cos−1 
1
⇒ sec−1( 5) = θ
 =θ
−1 −1  5
(f) cosec (cosec θ) and cosec (cosec x) = x
π Q sec −1 x + cosec−1 x = π 
⇒ − cosec−1( 5) = θ 
7π 2 2 
EXAMPLE 2. What is the value of cos −1  cos  ? π
 6  ∴ cosec−1( 5) = − θ
2
π 5π π 7π
a. − b. c. d.  x + y
6 6 6 6 (vi) (a) tan −1 x + tan −1 y = tan −1   , if xy < 1
7π  1 − xy 
Sol. b. Since, does not lie between 0 and π.
6  x + y
7π −1  5π   (b) tan −1 x + tan −1 y = π + tan −1   , if xy > 1
∴ cos−1  cos 
 = cos  cos  2 π −   1 − xy 
 6    6 
 x − y
= cos−1  cos
5π  5π
 = (c) tan −1 x − tan −1 y = tan −1   , if x > 0, y > 0
 6  6  1 + xy 

(ii) (a) sin −1 x = cosec −1 (1 /x) or cosec −1 x = sin −1 (1 / x) (vii) (a) sin −1 x ± sin −1 y = sin −1 { x (1 − y 2 ) ± y (1 − x 2 )}
(b) cos −1 x = sec −1 (1 /x) or sec −1 x = cos −1 (1 /x) (b) cos −1 x ± cos −1 y
(c) tan −1 x = cot −1 (1 /x) or cot −1 x = tan −1 (1 / x) , if = cos −1 { xy m (1 − x 2 ) (1 − y 2 )}
x>0
1 1 (viii) (a) sin −1 x = cos −1 ( 1 − x 2 )
(d) tan −1 x = cot −1 − π or cot −1 x = tan −1 − π,
x x  x   1 − x2 
if x < 0 = tan −1  = cot −1  
 1 − x2   x 
(iii) (a) sin −1 ( − x) = − sin −1 ( x)    
(b) cos −1 ( − x) = π − cos −1 ( x)  
1   1
= sec −1  = cosec −1  
(c) tan −1 ( − x) = − tan −1 ( x)  1 − x2   x
 
(d) cot −1 ( − x) = π − cot −1 ( x)
MATHEMATICS Inverse Trigonometric Functions 153

 1 − x2  = sec −1 ( 1 + x 2 )
(b) cos −1 x = sin −1 ( 1 − x 2 ) = tan −1  
 x   1 + x2 
 
= cosec −1  
  
x   1  x 
= cot −1  = sec −1  
 1 − x2   x
 
EXAMPLE 4. The principal value of
 1  1 5
= cosec −1  sin −1 + cos −1 is
 1 − x2  17 34
 
π π  23  −1  23 
 a. b. c. sin−1   d. cos  
x  4 2  17 2   17 2 
(c) tan −1 x = sin −1 
 1 + x2  Sol. a. sin−1
1
+ cos−1
5 1
= tan−1 + tan−1
3
  17 34 4 5
 1   1+ 3 
 1
= cos −1  = cot −1    
= tan−1  4 5  = tan−1(1) =
π
 1 + x2   x
  1 3
1 − ⋅ 
4
 4 5

PRACTICE EXERCISE
1. What is the principle value of cosec−1( − 2 )? 7. If 4 sin−1 x + cos−1 x = π , then x is equal to
π π π 1 3 1 1
(a) (b) (c) − (d) 0 (a) (b) (c) − (d)
4 2 4 2 2 2 2
π 1 
2. sin  − sin−1  −   is equal to 8. If sin  sin−1
1 
+ cos−1 x = 1, then what is x equal
3  2   5 
1 1 1 to?
(a) (b) (c) (d) 1
2 3 4 (a) 0 (b) 1
4 1
3. cos  tan−1
3 (c) (d)
 is equal to 5 5
 4
1
(a)
4
(b)
3
(c)
4
(d)
5 9. Let −1 ≤ x ≤ 1. If cos (sin−1 x ) =, then how many
5 4 3 4 2
1 values does tan (cos−1 x ) assume?
4. If cos−1 = θ, then tanθ is equal to (a) One (b) Two (c) Four (d) Infinite
x
1
(x2 − 1)  2x  1 − x 2
(a) (b) 10. If 3 sin−1   − 4 cos−1  
2
(x2 − 1) 1 + x 
2
1 + x 
(c) (1 − x2 ) (d) (x2 + 1)  2x  π
+ 2 tan−1   = , then x is equal to
5π 2  1 − x2  3
5. If (tan−1 x )2 + (cot−1 x )2 = , then x is equal to
8 (a) 3 (b) 1 / 3 (c) 1 (d) −1
(a) −1 (b) 1 (c) 0 (d) 2 −1 −1
11. sec (tan 2
2) + cosec (cot2
3) is equal to
6. If sin−1 x + cot−1 (1/ 2) = π / 2, then what is the (a) 5 (b) 13
value of x? (c) 15 (d) 6
1
(a) 0 (b) π
5 12. If tan− 1 2x + tan− 1 3x = , then x is equal to
4
2 3
(c) (d) 1
5 2 (a) − 1 (b) 1/6 (c) − 1, (d) None of these
6
154 NDA/NA Pathfinder

13. Consider the following statements 18. (α + β) is equal to


 1  2
I. The value of tan − 1   + tan − 1   is equal to
(b) sin−1 
3 
(a) tan−1  
17
 4  9 
 6  13 
−1 1 
sin   .
(d) cos −1 
3 
(c) sin−1  
3
 5 
 5  13 
8  3  77
II. sin − 1   + sin − 1   = sin − 1  
19. cos−1 
 17  5  85  44 
 is equal to
 125
Which of the above statement(s) is/are correct?
(a) 2 α (b) 3α (c) π − 3α (d) π − 2α
(a) Only I (b) Only II
−1 −1
(c) Both I and II (d) Neither I nor II 20. sin [cot [tan {cos ( 3 / 13 )}]] is equal to
14. Consider the following statements (a) sin β (b) cos β (c) tan β (d) sec β
I. If 2 tan − 1 (cos x) = tan − 1 (2 cosec x), then the value
π Directions (Q. Nos. 21-23) Consider the equation
of x is .
4 sin− 1( ax ) + cos− 1(y) + cos− 1( bxy) = π / 2.
 1 − x 1
II. The value of x for which tan − 1   = tan − 1 x 21. If a = 1 and b = 0, then ( x , y ) satisfies the equation
 1 + x 2
(a) x 2 + y2 = 1 (b) x 2 + y2 = 2
1
(x > 0) is . (c) x 2 + y2 = 1 / 2 (d) y = x
3
Which of the above statement(s) is/are correct? 22. If a = 1, b = 1, then ( x , y ) is a solution of the
(a) Only I (b) Only II equation
(c) Both I and II (d) Neither I nor II (a) (x 2 + 1) ( y2 − 1) = 0 (b) (x 2 − 1) ( y2 + 1) = 0

15. Consider the following statements (c) (x − 1) ( y − 1) = 0


2 2
(d) x 2 + y2 = 2
  23. If a = 2, b = 2, then ( x , y ) satisfies the equation
I. The simplest form of tan − 1   , | x| < a is
x
 a 2 − x2  (a) (x 2 − 1) (4 y2 − 1) = 0 (b) (4x 2 − 1) ( y2 − 1) = 0
 
− 1  x
(c) (4x − 1) ( y − 2 ) = 0
2 2
(d) (x 2 − 2 ) (4 y2 − 1) = 0
sin   .
 a
Directions (Q. Nos. 24-27) Let A = cos− 1 x,
−13 4 
II. The simplest form of cos  cos x + sin x , B = cos− 1 y and C = cos− 1 z
5 5 
 π π  4 24. If A + B + C = π, then x 2 + y 2 + z 2 is equal to
where x ∈ − ,  is tan − 1   .
 2 4   3 (a) 1 (b) 0 (c) 1 − 2 xyz (d) 2 xyz
III. The value of cos(sec x + cosec− 1x), | x| > 1 is 0.
−1
25. If A + B + C = 3π, then x( y + z ) + y( z + x ) + z( x + y )
Which of the above statement(s) is/are correct? is equal to
(a) I, II (b) II, III (c) I, III (d) I, II and III (a) 0 (b) 1 (c) 6 (d) 12
−1 −1
26. If A + B = 2π / 3, then sin x + sin y is equal to
Directions (Q. Nos. 16-17) Let cos− 1 x = α, (0 < x < 1)
(a) 2 π / 3 (b) π / 3 (c) π / 6 (d) π
 1  2π
and sin− 1(2 x 1 − x 2 ) + sec− 1 2  = −1 −1
 2 x − 1 3 27. If A + C = π / 2 and tan x − tan z = 0, then
x 2 + xz + z 2 is equal to
16. tan− 1( 2x ) is equal to 1 3 1
(a) π / 6 (b) π / 4 (c) π / 3 (d) π / 2 (a) 0 (b) (c) (d)
2 2 8
1  1 − y
17. If xy = , then tan− 1   is equal to Directions (Q. Nos. 28-32) Read the following
2  1 + y
information carefully and answer the questions
(a) π / 4 (b) π / 6 (c) π / 3 (d) π / 12 given below.
 1  1
Directions (Q. Nos. 18-20) Read the following infor- If α = tan −1   + tan −1  
mation carefully and answer the questions given below.  2  3
 2  5
 4  2 β = cos −1   + cos −1  
If α = cos −1   and β = tan −1   , where  3
 5  3  3
π   2π  1   2π 
0 < α, β < . Then, γ = sin −1 sin    + cos −1 cos   , then
2   
3  2   3 
MATHEMATICS Inverse Trigonometric Functions 155

3π 
28. cos (α + β + γ ) is equal to 40. What is sin−1  sin  equal to?
5π   7 π  (c) cos  π  (d) cos  11π   5
(a) cos 
e 2014 I
 (b) cos       3π 2π π
 12   12   12   12  (a) (b) (c) (d) None of these
5 5 5
β γ
29. tan α − tan  + 3 tan   is equal to 41. If x and y are positive and xy > 1, then what is
 2  4
(a) 4 (b) 3 (c) 2 (d) 1 tan−1 x + tan−1 y equal to? e 2014 I
−1 −1 −1  x + y −1  x + y
30. sin [cot [tan {cos (sin γ )}]] is equal to (a) tan   (b) π + tan  
 1 − x y  1 − x y
γ
(b) sin  
1
(a) sinγ (c) sinγ (d) cos γ  x + y  x− y
2 2 (c) π − tan−1  (d) tan−1 
 
 1 − x y  1 + x y
31. cos α + cos β + cos γ is equal to
42. What is sin−1   + sin−1   equal to?
3 4
2 −1 2 +1 2 + 3 3− 2
(a) (b) (c) (d)  5  5 e 2014 II
2 2 2 2
π π π π
32. If 4 (sin α + sin β + sin γ ) = a , then a is equal to
2 2 2 2 (a) (b) (c) (d)
2 3 4 6
(a) ±1 (b) ±2 (c) ±3 (d) ±4 π
43. The equation tan−1(1 + x ) + tan−1(1 − x ) = is
2
satisfied by e 2015 I
PREVIOUS YEARS’ QUESTIONS (a) x = 1 (b) x = − 1 (c) x = 0 (d) x = 1/2
33. If tan−1 2, tan−1 3 are two angles of a triangle,  1
then what is the third angle? e 2012 I Directions (Q. Nos. 44-46) Consider x = 4 tan−1  ,
5
(a) tan−1 2 (b) tan−1 4 (c) π / 4 (d) π / 3
 1  1
34. What is the value of cos cos −1
4 12
+ cos − 1  ? y = tan −1   and z = tan −1   .
 70   99 
 5 13 e 2015 I
e 2012 I 44. What is x equal to?
(a) tan−1 
60 
(b) tan−1 
(a) 63/65 (b) 33/65 (c) 22/65 (d) 11/65 120 
 
4  119   119 
35. If sin−1 1 + sin−1 = sin−1 x, then what is the
(c) tan−1 
90 
(d) tan−1 
5 170 
 
value of x? e 2012 I
 169   169 
(a) 3/5 (b) 4/5 (c) 1 (d) 0 45. What is x − y equal to?
 4 
36. What is the value of sin sin−1   + sin−1   ?
3
(a) tan−1 
828 
(b) tan−1 
8287 
 
  5  5   845   8450 
e 2012 II
(c) tan−1 
8281
(d) tan−1 
8287 
 
(a) 0 (b) 1/2 (c) 1 (d) 2  8450   8471 
 2a   2b  46. What is x − y + z equal to?
37. If sin−1   + sin −1   = 2 tan−1 x , then x
1 + a  1 + b 
2 2
(a) π / 2 (b) π / 3
is equal to e 2013 I (c) π / 6 (d) π / 4
a−b a−b a+b 1 π
47. The value of tan  2 tan−1
2ab
(a) (b) (c) (d) −  is
1 + ab 1 − ab a+ b 1 − ab  5 4 e 2015 II

38. What is tan−1   + tan−1   equal to?


1 1 7 5
(a) − (b)
 2  3 e 2013 II 17 16
π π π π 5 7
(a) (b) (c) (d) (c) (d)
2 3 4 6 4 17

39. Consider the following statements 48. Consider the following statements
−1 −1 4 3 π
I. tan 1 + tan (0. 5) = π / 2 I. sin −1 + sin −1 =
5 5 2
II. sin −1 (1 / 3) + cos − 1 (1 /3) = π / 2
II. tan −1 3 + tan −1 1 = − tan −1 (2 + 3 )
Which of the above statement(s) is/are correct?
e 2013 II Which of the above statement(s) is/are correct?
(a) Only I (b) Only II (a) Only I (b) Only II e 2015 II
(c) Both I and II (d) Neither I nor II (c) Both I and II (d) Neither I nor II
156 NDA/NA Pathfinder

49. Consider the following statements 50. Consider the following statements
 π π  1
I. There exists θ ∈  − ,  for which I. tan −1 x + tan −1   = π.
 2 2  x
tan −1 (tan θ ) ≠ θ. II. Their exist, x, y ∈ [−1, 1] , where x ≠ y such that
π
 1  1 −1  2 2 ( 3 − 1 ) sin −1 x + cos −1 y = .
II. sin −1   − sin −1   = sin  .
 3  5  15 
2
Which of the above statement(s) is/are correct?
Which of the above statement(s) is/are correct? (a) Only I (b) Only II e 2016 I
(a) Only I (b) Only II e 2016 I
(c) Both I and II (d) Neither I nor II
(c) Both I and II (d) Neither I nor II

ANSWERS
1 c 2 d 3 a 4 b 5 a 6 b 7 a 8 d 9 b 10 b
11 c 12 b 13 c 14 c 15 c 16 c 17 d 18 a 19 c 20 b
21 a 22 c 23 b 24 c 25 c 26 b 27 c 28 b 29 d 30 a
31 a 32 c 33 c 34 b 35 a 36 c 37 d 38 c 39 b 40 b
41 b 42 a 43 c 44 b 45 c 46 d 47 a 48 a 49 b 50 d

HINTS AND SOLUTIONS 


π 3 ⋅ 2 tan −1 x − 4 ⋅ 2 tan −1 x
6. (b ) sin −1 x + cot −1   = ⇒
1. (c) 1
 2 2 π
π + 2 ⋅ 2 tan −1 x =
2. (d ) sin  − sin −1  −  
1
π 3
cot −1   = − sin −1 x
 3  2   1
⇒ π
 2 2 ⇒ ( 6 − 8 + 4) tan −1 x =
π π
= sin  + sin −1    = sin
1 3
=1
cot −1   = cos −1 x
 3 1 π
 2   2 ⇒ ∴ tan −1 x = ⇒ x =
1
 2 6 3
3. (a) cos  tan −1 
3
Q sin −1 x + cos −1 x = π 
 4  2  11. (c) Let tan − 1 2 = α ⇒ tanα = 2
  and cot − 1 3 = β ⇒ cot β = 3
−1 1 −1  1  −1  x 
= cos  cos  4 ⇒ cot   = cot   Now, sec 2 (tan − 1 2) + cosec 2 (cot − 1 3)
 =  2  1− x2 
3 
2
 = sec 2 α + cosec 2β
 1+    5 1 x
  4  ⇒ = ⇒ 1 − x 2 = 2x = 1 + tan 2 α + 1 + cot 2 β = 15
2 1− x2 π
12. (b) tan ( 2x ) + tan − 1 (3x ) =
−1

4. (b) cos −1   = θ
1 On squaring both sides, we get 4
x 1  
1 − x 2 = 4x 2 ⇒ 5x 2 = 1 ⇒ x = −1 2x + 3x  π
 1 − ( 1 / x )2  5 = tan =
⇒ tan −1   =θ  1 − ( 2 x ) (3 x )  4
 
 1/ x  7. (a) 8. (d )
 5x 
1
∴ tanθ = tan[tan −1 x 2 − 1] = x2 − 1 9. (b) cos (sin −1 x ) = ⇒ tan − 1   = tan − 1 ( 1)
2  1 − 6x 2 
5. (a) Let tan −1 x = y 1 1  
Then, cot −1 x = − y
π ⇒ cos(cos −1 1 − x 2 ) = ⇒ 1 − x 2 = 5x
2 2 ⇒ = 1 ⇒ 6x 2 + 5x − 1 = 0
2 1 3 1 − 6x 2
π
2
5 π2 ⇒ 1− x2 = ⇒ x =±
∴ y +  − y  =
2 
4 2 ⇒ x = − 1,
1
2  8 6
Hence, tan (cos −1 x ) have two values.
⇒ 16 y 2 − 8 πy − 3 π 2 = 0 But, − 1 does not hold.
⇒ ( 4 y − 3 π) ( 4 y + π) = 0  2x   2 
10. (b) 3 sin −1   − 4 cos −1  1 − x  13. (c) I. Consider, tan − 1   + tan − 1  
1 2
⇒ y = tan −1 x = − π /4 1+ x 
2
 1 + x2   4  9
 π π    
rejected,since it is greater than 
3
 1+ 2 
 4 2   2x  π    9+8
∴ x = tan ( − π/4) = − 1 + 2 tan −1   = = tan − 1  4 9  = tan − 1  
 1 − x2  3 1 2  36 − 2 
  1− ×   
 4 9
MATHEMATICS Inverse Trigonometric Functions 157

= tan − 1   = tan − 1  
17 1 Sol. (Q. Nos. 16-17) This gives, x = − ( 1− y2 )
 34   2 Given, cos − 1 x = α ⇒ x = cosα
or x2 + y2 = 1
= sin −1
1/ 2
= sin −1 
1 
 ∴ sin − 1 ( 2 cos α 1 − cos 2 α )
2  15  22. (c) When a = 1, b = 1, then equation is

1+  
1    2π sin − 1 x + cos − 1 y + cos − 1 xy = π / 2
 2 −1 1  =
+ sec
 2 cos α − 1
2
3 π / 2 − cos −1 x + cos −1 y + cos −1 xy = π / 2
∴ Statement I is true.   ⇒ cos −1 xy = cos −1 x − cos −1 y

II. LHS = sin − 1   + sin − 1  
8
 17 
3
5
⇒ sin − 1 (sin 2α ) + sec − 1 (sec 2α ) =
2π π
3 [
cos −1 xy = cos −1 xy + 1− x2 1− y2 ]
8 2 2 ⇒ 4α = ⇒α = ⇒ xy = xy + 1− x2 1− y2
1 −   + 1 −   
3 3 8
= sin − 1  3 6
 17 5 5  17   π 3 ⇒ ( x 2 − 1)( y 2 − 1) = 0
 ∴ x = cos =
 32 + 45  6 2 23. (b) When a = 2, b = 2, the equation
77
= sin − 1   = sin
−1
= RHS  3 becomes
85  85 16. (c) tan − 1 ( 2x ) = tan − 1  2 × π
   sin − 1 ( 2x ) + cos − 1 y + cos − 1 ( 2xy ) =
 2  2
Thus, Statement II is also true. π
= tan − 1 ( 3 ) = ⇒ π / 2 − cos −1 ( 2x ) + cos −1 y
14. (c) I. 2 tan − 1 (cos x ) = tan − 1 ( 2 cosec x ) 3
1 + cos −1 ( 2xy ) = π / 2
⇒ tan −1 
2 cos x  −1 17. (d ) Given, xy =
 = tan ( 2 cosec x ) ⇒ cos ( 2xy ) = cos −1 ( 2x ) − cos −1 y
−1
 1 − cos 2 x  2
2 cos x ⇒
3
× y= ⇒y=
1 1 ⇒ cos −1 ( 2xy ) = cos −1
⇒ = 2 cosec x ⇒ 2 cos x = 2 sin x
sin 2 x 2 2 3 [ 2xy + 1 − 4x 2 1 − y 2 ]
π 1− 1 
⇒ cot x = 1 ⇒ x =
4  1− y   3 ⇒ 2xy = 2xy + 1 − 4x 2 ⋅ 1 − y 2
∴ Statement I is true. ∴ tan − 1   = tan 
−1
 1+ y  1  ⇒ ( 4x − 1) ( y − 1) = 0
2 2
1+ 
1− x 1  3
π π π 24. (c) A + B + C = π
II. tan − 1   = tan − 1 x = − =
1+ x 2 ⇒ cos − 1 x + cos − 1 y + cos − 1 z = π
  4 6 12
 1 − tan θ  1 ⇒ cos − 1 ( xy − 1− y2 1− x2 )
Sol (Q. Nos. 18-20)
⇒ tan − 1   = θ 4 3 3 = π − cos − 1 z
 1 + tan θ  2 cos α = ⇒ sin α = , tan α =
  5 5 4
2 2 3 ⇒ xy − 1− y2 1− x2
[putting x = tanθ] tan β = ⇒ sin β = , cosβ =
θ 3 13 13 = cos ( π − cos − 1 z )
⇒ tan − 1 ( 1) − tan − 1 (tan θ) = tan α + tan β
2 18. (a) tan (α + β ) = ⇒ xy − 1− y2 1− x2 = − z
π θ π 1 − tan α tan β
⇒ −θ= ⇒ θ=
4 2 6 3 2 ⇒ xy + z = 1− y2 1− x2
π 1 +
⇒ tan − 1 x = ⇒ x = 17
6 3 = 4 3 = On squaring both sides, we get
3 2 6
∴ Statement II is also true. 1− ⋅ x 2 y 2 + z 2 + 2xyz
4 3 = 1 − x2 − y2 + x2 y2
15. (c) I. Let x = a sinθ, then sin − 1   = θ
x
(α + β ) = tan −1  
17
 a ⇒ ⇒ x + y 2 + z 2 = 1 − 2xyz
2
x  6
tan − 1 25. (c) Given, A + B + C = 3 π
a2 − x 2 19. (c) cos 3α = 4 cos 3 α − 3 cos α
64 4 44 ⇒ cos − 1 x + cos − 1 y + cos − 1 z = 3 π
 a sin θ  −1  sin θ 
=4× −3× = −
= tan −1   = tan   125 5 125 Q 0 ≤ cos − 1 x ≤ π, 0 ≤ cos − 1 y ≤ π
 a 2 − a 2 sin 2 θ   cos θ  44 and 0 ≤ cos − 1 z ≤ π
and cos ( π − 3α ) = − cos 3α =
= tan − 1 (tan θ) = θ = sin − 1  
x 125 ∴ cos − 1 x = cos − 1 y = cos − 1 z = π
 a ⇒ x = y = z = cos π = − 1
  3   
So, Statement I is correct. 20. (b) sin  cot −1  tan cos −1  
    13    Thus, x ( y + z ) + y (z + x ) + z ( x + y )
II. Let cosα =
3
⇒ sinα = 1 −
9
=
4 = 2( xy + yz + zx ) = 6
= sin [cot −1 [tan {cos −1 (cos β )}]]
5 25 5 2π
4 = sin {cot −1 (tan β)} 26. (b) Given, cos − 1 x + cos − 1 y =
and tanα = 3
π
3 = sin  cot −1 cot  − β   π
⇒ − sin − 1 x +
π
− sin − 1 y =

 2  
Now, cos − 1  cos x + sin x 
3 4  2 2 3
5  π 2π π
= sin  − β  = cos β
5 ⇒ sin − 1 x + sin − 1 y = π − =
= cos − 1 [cos α cos x + sin α sin x ] 2  3 3
= cos − 1 [ cos(α − x )] 27. (c) Given, tan − 1 x − tan − 1 z = 0 ⇒ x = z
4 21. (a) When a = 1 and b = 0, the given
= α − x = tan − 1 − x π
equation reduces to Also, A+C =
3 π
sin − 1 x + cos − 1 y = 0 Q cos ( 0) = 
−1 2
∴ Statement II is not correct.  2  ⇒ cos − 1 x + cos − 1 z = π / 2
π
III. cos(sec − 1 x + cosec − 1 x ) = cos( π / 2) = 0
−1 −1 −1
sin x = − cos y = − sin ( 1 − y ) 2 ⇒ 2 cos − 1 x =
2
∴ Statement III is correct.
158 NDA/NA Pathfinder

π 1
⇒ 4  + 1 +  = a 2
1 3
45. (c) x − y = tan −1 
⇒ cos − 1 x = ⇒x = 120  −1  1 
2  − tan  
4 2 4  119   70 
Hence, x 2 + xz + z 2 4 ( 2 + 4 + 3)  120 − 1 
2 ⇒ = a2  
= 3x 2 = 3
1 
= tan −1  119 70  = tan −1 
3 8281 
 = 4 
 2 2 ⇒ a2 = 9 ⇒ a = ± 3 120 1  8450 
1+ × 
 119 70 
Sol. (Q. Nos. 28-32) 33. (c) Let α be the third angle of the
46. (d ) x − y + z = tan −1 
8281  −1  1 
α = tan   + tan −1  
−1  1 1 triangle, then
 + tan  
 2 3 In ∆ABC , tan −1 2 + tan −1 3 + α = 180°  8450   99 
2+3 
 1+ 1  5 ⇒ tan −1   + α = 180° −1  49 
= tan   + tan   −1  1 
−1  2  −1  6 
 1− 2⋅3  50   99 
= tan  3 = tan  
1 1 5 ⇒ tan −1 ( −1) + α = 180°  49 + 1 
1− ⋅    3π π
 2 3  6 ∴ α = 180°− tan −1 ( −1) = π − =  
π 4 4 = tan −1  50 99 
= tan −1 ( 1) =  1−
49 1
× 
4 34. (b)  50 99 
 5 π
β = cos −1   + cos −1  
2 4
35. (a) Given, sin −1 1 + sin −1 = sin −1 x = tan −1 ( 1) =
3  3  5 4
⇒ sin −1  1⋅ 1 − ( 4 / 5)2 + 1 − ( 1)2 
 5 4
⇒ β = cos −1   + sin −1  1 − 
2 47. (a)
3  5 
 9
48. (a) I. LHS = sin − 1   + sin − 1  
−1
4 3
−1  2  −1  2  π = sin x 5 5
⇒ β = cos   + sin   =  16 4 
3 3 2 ⇒ sin −1  1 1 − + 1 − 1 = sin −1 x
= sin − 1   + cos − 1  
4 4
2π  25 5 
and γ = sin −1  sin  
5 5
  3   ⇒ sin −1   = sin −1 x ⇒ x =
3 3 π
= = RHS
2π 5 5
+ cos −1  cos 
1 2
  II. LHS = tan − 1 3 + tan − 1 1
2  3   36. (c) 37. (d ) 38. (c)
 3 + 1 1+ 3
π 2π
= sin −1  sin  + cos −1  cos = π + tan − 1  
1
39. (b) I. tan − 1 1 + tan −1 ( 0.5) ×
 3 2  3   1− 3 1+ 3
⇒ tan − 1 1 + tan − 1  
1  
π 1 2π 2π  2
= + ⋅ = 3 + 1+ 2 3 
3 2 3 3
= tan − 1 1 + cot − 1 
1 
 = π + tan − 1  
28. (b) cos (α + β + γ )  1/ 2  1 − 3 

−1 −1
π π 2π  7π = tan 1 + cot 2 ≠ π / 2
= cos  + +  = cos ∴ Statement I is incorrect.
= π − tan − 1 ( 2 + 3 ) ≠ RHS
4 2 3 12 Hence, only Statement I is correct.
II. sin − 1   + cos − 1   = π / 2 π π
β γ 1 1
29. (d ) tan α − tan + 3 tan 3 3 49. (b) tan −1 (tan θ) = θ, − < θ <
2 4 2 2
π π π [Q sin − 1 x + cos − 1 x = π / 2, ∀ x ∈ R] ∴ Statement I is incorrect.
= tan − tan + 3 tan ∴ Statement II is correct. Now, sin −1   − sin −1  
1 1
4 4 6
1 3π 3 5
= 3⋅ =1 40. (b) Since, does not lie between − π / 2
3 5 1 2
1 
2
1 −   − 1 −   
1 1
−1 −1 and π / 2. = sin −1 
30. (a) sin [cot {tan (cos (sin γ ))}]  3 5 5 3 
3π 2π 
 
= sin  cot −1 tan  cos −1  sin
2 π    ∴ sin −1  sin  = sin −1 sin  π − 
    5   2 2 ( 3 − 1) 
    3    5 
= sin −1  
2π 2π
   3   = sin −1 sin =


15 

= sin  cot −1 tan  cos −1    5  5
   2   ∴ Statement II is correct.
π 41. (b) 42. (a)
= sin  cot −1 tan  = sin  cot −1
1 50. (d ) We know that,
π π
  6   3  43. (c) tan −1 ( 1 + x ) + tan −1 ( 1 − x ) = tan −1 x + cot −1 x =
π 2π 2 2
= sin = sin  π − 
 −1  1+ x + 1− x  π π
tan −1 x + tan −1   =
⇒ tan  1
3  3   = ∴
2π  1 − ( 1 + x )( 1 − x )  2 x 2
= sin = sin γ 2 π 2 1
3 ⇒ = tan ⇒ 2 = ∴ Statement I is incorrect.
31. (a) cos α + cos β + cos γ x2 2 x 0
Let x = y
π π 2π ⇒ x2 = 0 ⇒ x =0 π
= cos + cos + cos Given that, sin −1 x + cos −1 y =
4 2 3
44. (b) Given, x = 4 tan   = 2 ⋅ 2 tan −1  
−1  1 1 2
1 1 2−1 5 5 π
= + 0− = ⇒ sin −1 ( x ) + cos −1 ( x ) =
2 2 2
= 2 ⋅ tan −1 
2/5  −1 2
 = 2 tan (5 / 12)
32. (c) 4 (sin 2 α + sin 2 β + sin 2 γ ) = a 2  24 / 25  This is true for x ∈ [ −1, 1].
 π π 2π
= tan −1 
⇒ 4  sin 2 + sin 2 + sin 2  =a
2 10 / 12  −1  120 
 3   = tan   ∴ Statement II is not correct.
4 2  1 − 25 / 144   119 
14
MATHEMATICS Decimal Fractions 159

HEIGHT AND DISTANCE

In NDA exam, generally 1-2 questions are asked from this chapter which are based on finding
required term length/height/distance etc., by using angle of elevation/depression in triangle.

Height and distance is one of the most important application of trigonometry which helps us to
find the height of any object and distance of objects between points which are not directly measurable.

Line of Sight
When the eye of a person at a point O looks at an object P, then the line OP is called the line of sight.

Angle of Elevation
Let P be an object and OX be a horizontal line, an observer at O to perceive the object P has to
elevate his eye from direction OX to the direction OP. As such, we define ∠XOP as the angle at
elevation of P at O.
P (Object)

ht
sig
of
ne
Li

Angle of
elevation Q
(Eye of observer) O X
Horizontal line

EXAMPLE 1. The angle of elevation of a ladder leaning against a wall is 60° and the foot of the
ladder is 4.6 m away from the wall. The length of the ladder is
a. 2.3 m b. 9.2 m c. 4.6 m d. None of these
Sol. b. Let AB be the wall and BC be the ladder. Then,
B
∠ACB = 60° and AC = 4.6 m
AC 1 4.6
In ∆ACB, cos 60° = ⇒ =
BC 2 BC
⇒ BC = ( 4.6 × 2) m = 9.2 m
60º
C 4.6 m A
160 NDA/NA Pathfinder

EXAMPLE 2. What should be the height of a flag, Sol. d. Let AD be the F (Tower)
where a 20 ft long ladder reaches 20 ft below the flag. building and BF be
the tower.
The angle of elevation of the top of the flag at the h
Draw DC ⊥ BF. Then, (Building)
foot of the ladder is 60°? 60º
a. 20 ft b. 30 ft c. 40 ft d. 20 2 ft ∠FDC = 60° D C

and ∠DBA = 30°


Sol. b. Let BD be a flag
AD = BC = 20 m 20 m 20 m
and BD = (h + 20) ft
AD
BD In ∆ABD, tan 30° =
In ∆ABD, tan 60° = AB
30º
AB A B
h + 20 ⇒ AB = 20 3 m
⇒ 3=
AB h
In ∆DCF, tan 60° =
(h + 20) DC
⇒ AB = …(i)
3 ⇒ h = 3 × 20 × 3 [Q AB = DC = 20 3]
Now, in ∆ABC, ⇒ h = 60 m
(h + 20) 2 ∴ Height of tower, BF = 60 + 20 = 80 m
AC = AB + BC ⇒ 20 =
2 2 2 2
+ h2
3
(h + 20) 2 + 3h2 m-n Theorem C
⇒ 400 =
3
If D divides AB in the ratio m : n, α β
⇒ 1200 = h2 + 40h + 400 + 3h2
(i) ( m + n)cot θ = n cot A − mcot B
⇒ 4h2 + 40h − 800 = 0 ⇒ h2 + 10h − 200 = 0 A θ B
⇒ (h + 20) (h − 10) = 0 (ii) ( m + n)cot θ = mcot α − n cot β A m D n B
⇒ h = 10 [Qh ≠ − 20] EXAMPLE 4. A man observes that when he moves
∴ Height of flag = BD = BC + CD = 10 + 20 = 30 ft up a distance C metres on a slope, the angle of
depression of a point on the horizontal plane from the
Angle of Depression base of the slope is 30°, and when he moves up
Let P be an object and OX be a horizontal line, an further a distance C metres the angle of depression of
that point is 45°. The angle of inclination of the slope
observer at O to perceive the object P has to depress his
with the horizontal is
eye from the direction OX to direction OP. As such,
we define ∠XOP as the angle of depression of P at O. a. 60° b. 45° c. 75° d. 30°
Horizontal line Sol. c. Applying m-n theorem, we get
(Eye of observer) O X
Angle of depression
C
Lin
e
of

c
sig
ht

B
P (Object)
º 0º
3 c
15
Note θ− θ
• Any perpendicular line to a plane is perpendicular to all 30º
lines lying in the plane. P A
• Angle of elevation and depression are always acute
( c + c) cot (θ − 30° ) = c cot 15° − c cot 30°
angles.
1 sin ( 30° − 15° )
⇒ cot (θ − 30° ) =  
EXAMPLE 3. Looking from the top of a 20 m high  2 sin 15° sin 30°
building, the angle of elevation of the top of a tower 1
is 60° and the angle of depression of its bottom is 30°. ⇒ cot (θ − 30° ) = = 1 = cot 45°
2sin 30°
What is the height of the tower?
⇒ θ − 3θ = 45° ⇒ θ = 45° + 30° = 75°
a. 50 m b. 60 m c. 70 m d. 80 m
MATHEMATICS Height and Distance 161

PRACTICE EXERCISE
1. A 30 m long ladder is placed against a wall 15 m 9. A man standing on the bank of a river observes
high such that it just reaches the top of the wall. that the angle of elevation of the top of a tree
The angle made by the ladder with the just on the opposite bank is 60°. The angle
horizontal is of elevation is 30° from a point at a distance
(a) 30° (b) 45° (c) 60° (d) 90° y m from the bank. What is the height of the
tree?
2. What is the angle subtended by 1 m pole at a
distance 1 km on the ground in sexagesimal 3y y
(a) y m (b) 2y m (c) m (d) m
measure? 2 2
9 9 10. Two poles of equal heights are standing opposite
(a) degree (b) degree
50π 5π to each other on either side of a road, which is
(c) 3.4 min (d) 3.5 min 30 m wide. From a point between them on the
3. The angle of elevation of the top of a flag post road, the angles of elevation of the tops are 30°
from a point 5 m away from its base is 75°. What and 60°. The height of each pole is
is the approximate height of the flag post? 15
(a) 3m (b) 15 3 m
(a) 15 m (b) 17 m (c) 19 m (d) 21 m 2
(c) 10 3 m (d) None of these
4. The angle of elevation of the top of two vertical
towers as seen from the middle point of the 11. A man on the top of a tower, standing on the
line joining the foot of the towers are 60° and sea-shore finds that a boat coming towards
30°, respectively. The ratio of the heights of him takes 10 min for the angle of depression
the tower is to change from 30° to 60°. The time taken by
(a) 2 : 1 (b) 3 :1 (c) 3 : 2 (d) 3 : 1 the boat to reach the shore from this position
will be
5. Two verticals poles AL and BM of heights 20 m 1
and 80 m, respectively stand apart on a (a) 5 min (b) 15 min (c) 7 min (d) 245 s
2
horizontal plane. If A and B be the feet of the
poles and AM and BL intersect at P, the height 12. The angles of elevation of the top of a tower from
of P is equal to two points at distances a and b metres from the
(a) 50 m (b) 18 m (c) 16 m (d) 15 m base and in the same straight line with it, are
complementary. The height of the tower (in
6. A tower 100 m tall stands on the top of a mount. metres) is
From a point on the ground, the angles of
a
elevation of the top and the bottom of the tower (a) a + b (b) a − b (c) ab (d)
b
are found to be 75° and 45°, respectively. The
height of the mount in metres is 13. The shadow of a pole standing on a horizontal
(a) 100 ( 3 − 1) m (b) 50 ( 3 − 1) m plane is d metre longer when the Sun’s altitude
(c) 50 ( 3 + 1) m (d) None of these is α, then when it is β. What is the height of the
7. A chimney 20 m high standing on the top of a pole?
cos α cos β sin α cos β
building subtends an angle whose tangent is 1/6 (a) d ⋅ (b) d ⋅
at a distance 70 m from the foot of the building. cos (α − β ) sin (α − β )
sin α sin β sin β cos α
The height of the building is (c) d ⋅ (d) d ⋅
(a) 50 m (b) 40 m (c) 60 m (d) 20 m sin (β − α ) cos (α + β )

8. An aeroplane flying at a height of 300 m above 14. The foot of a tower of height h m is in a direct
the ground passes vertically above another plane line between two observers A and B. If the angles
at an instant when the angles of elevation of two of elevation of the top of the tower as seen from
plane from the same point on the ground are 60° A and B are α and β, respectively and if AB = d
and 45°, respectively. What is the height of the m, what is h/d equal to?
lower plane from the ground? tan (α + β ) cot (α + β )
100 (a) (b)
(a) 500 m (b) m cot α cot β − 1 cot α cot β − 1
3 tan (α + β ) cot (α + β )
(c) 100 3 m (d) 150( 3 + 1) m (c) (d)
cot α cot β + 1 cot α cot β + 1
162 NDA/NA Pathfinder

15. AB is a vertical pole. The end A is on the ground, PREVIOUS YEARS’ QUESTIONS
C is the middle point of AB and P is a point on
the level ground. The portion BC subtends an 22. The angle of elevation of a tower at a level
angle α at P. If AP = n. AB, then tanα is ground is 30°. The angle of elevation becomes θ
n n n  n − 1
2 when moved 10 m towards the tower. If the
(a) (b) (c) (d)  2  height of tower is 5 3 m, then what is the value
(n2 − 1) (n2 + 1) (2 n2 + 1)  n + 1
of θ? e 2012 I
16. Let PT be a tower of height 2x m, P being the (a) 45° (b) 60° (c) 75° (d) 90°
foot, T being the top of the tower, A, B are points 23. From the top of a lighthouse 70 m high with its
on the same line with P. If AP = 2x + 1 m, base at sea level, the angle of depression of a
BP = 192 m and the angle of elevation of the boat is 15°. The distance of the boat from the
tower as seen from B is double the angle of the foot of the lighthouse is e 2012 II
elevation of the tower as seen from A, then (a) 70 (2 − 3 ) m (b) 70 (2 + 3) m
I. The value of x is 8. (c) 70 (3 − 3 ) m (d) 70 (3 + 3) m
π
II. The value of θ is . 24. The top of a hill observed from the top and
4
bottom of a building of height h is at angles of
Which of the above statement(s) is/are correct? elevation α and β, respectively. The height of the
(a) Only I (b) Only II hill is e 2012 II
(c) Both I and II (d) Neither I nor II h cot β h cot α
(a) (b)
17. A round balloon of radius r subtends an angle α cot β − cot α cot α − cot β
at the eye of the observer while the angle of h tan α
(c) (d) None of these
elevation of its centre is β. Then height of the tan α − tan β
centre of the balloon is
25. The shadow of a tower standing on a level plane
I. h = r sin α cosec β II. h = r sin β sin α
is found to be 50 m longer when the Sun’s
α
III. h = r cosec   sin β elevation is 30° than when it is 60°. The height
 2
of the tower is e 2013 I
(a) Only I (b) Only II (a) 25 m (b) 25 3 m (c) 50 m (d) 50 3 m
(c) Only III (d) None of these
26. A man walks 10 m towards a lamp post and
Directions (Q. Nos. 18-19) The angle of elevation of a notices that the angle of elevation of the top of
cloud from a point 200 m above a lake is 30° and the the post increases for 30° to 45°. The height of
angle of depression of its reflection in the lake is 60°. the lamp post is e 2013 I
(a) 10 m (b) (5 3 + 5) m
18. The height of the cloud above the lake is (c) (5 3 − 5) m (d) (10 3 + 10) m
(a) 240 m (b) 500 m (c) 300 m (d) 400 m
27. The angle of elevation of the top of a tower of
19. The horizontal distance of the cloud from point of height H from the foot of another tower in the
observation is same plane is 60° and the angle of elevation of
(a) 240 3 m (b) 400 3 m the top of the second tower from the foot of the
(c) 200 3 m (d) None of these first tower is 30°. If h is the height of the other
tower, then which one of the following is correct?
Directions (Q. Nos. 20-21) An aeroplane flying e 2013 I
horizontally 900 m above the ground is observed at (a) H = 2 h (b) H = 3h (c) H = 3 h (d) H = h
an elevation of 60°. After 10 s, the elevation
changes to 30°.
28. A person standing on the bank of a river
observes that the angle subtended by a tree on
20. The uniform speed of the aeroplane (in km/h) is the opposite of bank is 60°. When he retires 40 m
50 from the bank, he finds the angle to be 30°.
(a) (b) 180 3
3 What is the breadth of the river? e 2013 II
(c) 216 3 (d) None of these (a) 60 m (b) 40 m (c) 30 m (d) 20 m
21. The angle of elevation after 40 s is 29. The angles of elevation of the top of a tower from
− 1 1  − 1 1  two places situated at distances 21 m and x m
(a) tan   (b) tan  
 5 3  3 3 from the base of the tower are 45° and 60°,
 3  3 respectively. What is the value of x? e 2013 II
(c) tan− 1   (d) tan− 1  
 5   7  (a) 7 3 m (b) (7 − 3) m (c) 7 + 3 m (d) 14 m
MATHEMATICS Height and Distance 163

30. From an aeroplane above a straight road the 33. The angles of elevation of the top of a tower
angles of depression of two positions at a standing on a horizontal plane from two points
distance 20 m apart on the road are observed to on a line passing through the foot of the tower at
be 30° and 45°. The height of the aeroplane distances 49 m and 36 m are 43° and 47° ,
above the ground is e 2014 I respectively. What is the height of the tower?
(a) 10 3 m (b) 10( 3 − 1) m e 2015 I
(c) 10( 3 + 1) m (d) 20 m (a) 40 m (b) 42 m (c) 45 m (d) 47 m
31. A lamp post stands on a horizontal plane. From 34. Two poles are 10 m and 20 m high. The line
a point situated at a distance 150 m from its joining their tops makes an angle of 15° with the
foot, the angle of elevation of the top is 30°. horizontal. The distance between the poles is
What is the height of the lamp post? e 2014 II approximately equal to e 2015 II
(a) 50 m (b) 50 3 m (a) 36.3 m (b) 37.3 m (c) 38.3 m (d) 39.3 m
50
(c) m (d) 100 m 35. A vertical tower standing on a levelled field is
3
mounted with a vertical flag staff of length 3 m.
32. The angle of elevation of the top of a tower from From a point on the field, the angles of
a point 20 m away from its base is 45°. What is elevation of the bottom and tip of the flag staff
the height of the tower? e 2015 I are 30° and 45°, respectively. Which one of the
(a) 10 m (b) 20 m following gives the best approximation to the
(c) 30 m (d) 40 m height of the tower? e 2015 II
(a) 3.90 m (b) 4.00 m (c) 4.10 m (d) 4.25 m

ANSWERS
1 a 2 a 3 c 4 d 5 c 6 b 7 a 8 c 9 c 10 a
11 a 12 c 13 c 14 b 15 c 16 a 17 c 18 d 19 c 20 c
21 b 22 b 23 b 24 b 25 b 26 b 27 c 28 d 29 a 30 c
31 b 32 b 33 b 34 b 35 c

HINTS AND SOLUTIONS 


1. (a) Let AB be the wall and BC be the BC 1 AB h
tanθ = = rad In ∆ABC , tan 75° = =
ladder and ∠ACB = θ AB 1000 BC 5
Then, AB = 15 m B ⇒ tan θ = 0.001 rad tan 45° + tan 30° h
⇒ =
BC = 30 m ⇒ θ = tan −1 ( 0.001) rad 1 − tan 45° tan 30° 5
m

15 m

AB
Then, sinθ =
30

= 9.999 × 10−4 rad 1+ 3 h


BC ⇒ =
15 1 = 9.999 × 10−4 ×
180
degree 3− 1 5
= = θ
π
30 2 C A ( 3 + 1)2
9 ⇒ h= ×5
⇒ θ = sin − 1 ( 1/ 2) = 30° = degree ( 3 )2 − ( 1)2
50 π
2. (a) Let BC be the pole and A be the 3. (c) Let h be the height of the flag post. 3 + 1 + 2 3 
point of elevation. ⇒ h =5 
A  3− 1 
Let ∠CAB = θ  
C
= 5 ( 2 + 3 ) = 5 × 3.732
= 18.660 m = 19 m (approx)
1 m (Pole) h
4. (d) Let h and h be the heights of two
1 2
towers.
θ 75º h
A 1 km B C tan 60° = 1 ⇒ h = a tan 60°
5m B a 1
164 NDA/NA Pathfinder

h h + 100 300 300 3 300 3


tan 30° = 2 ⇒ =2+ 3 ⇒ y= ⇒x = × =
a h 3 3 3 3
P tan A + tan B ∴ x = 100 3 m
Q tan ( A + B ) = [from Eq. (i)]
1 − tan A tan B
9. (c) Let the height of tree = h
Q ⇒ h (2 + 3 − 1) = 100
h1
100 100 3−1 Given, AC = y
h2 ⇒ h= = ×
30º 60º 3+ 1 3+ 1 3−1 P
B a C a A
100( 3 − 1)
= = 50 ( 3 − 1) m
⇒ h = a tan 30° 3−1 h
2
h a tan 60° 3
∴ 1 = = 7. (a) Let AB = H m be the building and 60º 30º
h a tan 30° 1 CB be the chimney. y A
2 B C
1 H + 20
5. (c) Let h be the height of point P from tan θ = , tan α = , h
In ∆PBC , tan 60° = = 3
horizontal plane. 6 70 BC
H
In ∆ABM and ∆AQP tanβ =
70 ⇒ BC = h/ 3 …(i)
80 h
tanθ = = C Now, in ∆PBA,
AB QA
20 m h h 1
AB tan30° = = =
⇒ AQ = h ⋅ BA BC + AC 3
80 B
M ⇒ 3 h = { h/ 3 + y }
L H
θ α [from Eq. (i)]
P β
80 m A ⇒ 3h = h + y 3
P 70 m
20 m
1 1 3
h Now, tanθ = ⇒ tan (α − β ) = ⇒ 2h = y 3 ⇒ h= ym
φ θ 6 6 2
B Q A (tan α − tan β ) 1
⇒ = 10. (a) Let AB and CD be the poles and let
Now, in ∆BAL and ∆BQP 1 + tan α tan β 6 O be the point of observation.
20 h
tan φ = = H + 20 H 
AB BQ ⇒ 6  − 
Let AB = CD = h m
 70 70  ∠AOB = 30°, ∠COD = 60°.
H + 20 H 
= 1 + 
AB
∴ BQ = h ⋅ ⋅  Let OA = x .
20  70 70 
Then, OC = (30 − x ) m
Now, AB = AQ + BQ 20 4900 + 20H + H 2
⇒ 6× = B D
= h ⋅ AB  +
1 1 70 70 × 70

 80 20 
⇒ H 2 + 20H + 4900 − 8400 = 0
1+ 4
⇒ 1 = h   ⇒ h = 16 m ∴ H 2 + 20H − 3500 = 0 h
 80 
⇒ ( H − 50)( H + 70) = 0
6. (b) Let QR = h m be the height of the ∴ H = 50 m 30º 60º
mount. O
8. (c) Let the height of the lower plane A x (30– x) C
Then, PR = ( 100 + h ) m from the ground = x and PA = y x
P In ∆BAO, = cot 30° = 3
h
100 m
C ⇒ x =h 3 …(i)
30 − x 1
B In ∆DCO, = cot 60° =
Q h 3
300 m
h
x ⇒ 30 − x =
º 3
75 h
60°  h 
45° ⇒ x =  30 −
45º P A 
y  3
O x R
Now, in ∆BAP, h
In ∆ORQ, ∴ h 3 = 30 − [from Eq. (i)]
AB x 3
h h tan 45° = = = 1 ⇒ x = y …(i)
Then, = tan 45° ⇒ =1  h 3 + h  = 30
x x
PA y ⇒  
 3
⇒ h=x …(i) Again, in ∆CAP,
15
100 + h tan 60° =
AC 300
= = 3 ⇒ 4h = 30 3 ⇒ h = 3m
In ∆ORP , = tan 75° AP y 2
x
MATHEMATICS Height and Distance 165

11. (a) Let AB be the tower and C and D be ⇒ h (cot α − cot β ) = d In ∆PTA, tanθ =
PT
the two positions of the boat. d sin α sin β AP
∴ h= T
Let AB = h, CD = x and AD = y . cos α sin β − cos β sin α
X B
d sin α sin β 2x m
30° =
sin (β − α )
60°
θ 2θ
h 14. (b) Let AD = x A B P
2x+1
∴ DB = d − x
2x 1
30º 60º C ⇒ tanθ = x +1
=
2 2
C x D y A PT 2x
y 1 h Now, in ∆PTB, tan 2θ = =
In ∆BAD, = cot 60° = ⇒y= h PB 192
h 3 3 1
x+ y 2×
In ∆BAC , = cot 30° = 3 a b 2 tan θ 2 x
2 = 2
x

h A x D d–x
B ⇒ = ⇒
d 1 − tan θ 192
2
1−
1 192
⇒ x + y = 3h h 4
h 2h In ∆ADC , tanα = ⇒ x = h cot α …(i) 4
∴ x = (x + y) − y = 3 h − = x ⇒ × 192 = 2x ⇒ 2x = 256
3 3 3
h
2h
is covered in 10 min. In ∆CDB, tanβ = ⇒ 2x = 28 ⇒ x = 8
3 d −x
TP 28 1
h  3 h  ⇒ d − x = h cot β …(ii) tanθ = = =
will be covered in  10 × ×  AP 29 2
3  2h 3 On adding Eqs. (i) and (ii), we get π
θ = tan −1   ≠
1
= 5 min d = h (cot α + cot β )  2 4
We know that, 17. (c) Let C be the centre of the balloon
12. (c) Let AB be the tower and C and D be
cot α cot β − 1
the points of observation. cot (α + β ) = and AC and BC be radii, where OA and
cot β + cot α OB are tangents from O.
Then, AC = a, AD = b,
cot α cot β − 1 Draw CD ⊥ OD.
∠ACB = θ, ∠ADB = 90° − θ ⇒ cot β + cot α =
AB cot (α + β )
= tanθ A r C
AC   r
cot α cot β − 1
B ⇒ d =h 
 cot (α + β )  B
 
h cot(α + β ) α
∴ =
h d cot α cot β − 1 β
O D
α
∠AOB = α, ∠BOC =
θ 90º−θ 15. (c) Let ∠APC = θ. Then, 2
AB 1 OC α
C D b A tan(θ + α ) = = and ∠COD = β, = cosec
a AP n AC 2
⇒ h = a tanθ α CD
tanθ =
AC
= ⋅
1 AB
=
1 ⇒ OC = r cosec , = sinβ
AB
and = tan( 90° − θ) = cot θ AP 2 AP 2n 2 OC
AD tan θ + tan α ⇒ CD = OC sinβ
⇒ h = b cot θ , h 2 = ab tan(θ + α ) = = r sin β cosec (α /2)
Hence, h = ab 1 − tan θ tan α
Sol. (Q. Nos. 18-19) Let AB be the upper layer
13. (c) Let CD be the pole, whose length is h.  1 + tan α  of the lake, C be the cloud, C ′ its
 
CD h 1   reflection and O be the point of
In ∆BCD, tanβ = ⇒ tanβ = ⇒ = 2n
BC x n  1 − 1 tan α  observation. Draw OD ⊥ CC ′. Then,
⇒ x = h cot β   CB = C ′ B = x , ∠COD = 30°,
…(i)  2n 
B ∠C ′OD = 60°, AO = BD = 200 m
D CD
tan30° =
Sun OD
1 CB − BD x − 200
h ⇒ = =
C 3 AB AB
C
α B β α
A θ
d x C
P A O 30º x
CD D
and in ∆ACD, tan α = n 60º
AC ⇒ tan α = 200 m
h ( 1 + 2 n2 ) B
⇒ tan α = ⇒ x + d = h cot α A
x+ d 16. (a) Q PT = 2x m , AP = 2x + 1 m x
⇒ h cot β + d = h cot α [from Eq. (i)] and BP = 192 m C′
166 NDA/NA Pathfinder

AB 22. (b) Given, height of tower From Eqs. (i) and (ii),
⇒ x − 200 = …(i)
3 x cot α = ( x + h ) cot β
= CD = 5 3 m
C ′D
tan 60° = CD 5 3 ⇒ x (cot α − cot β ) = h cot β
OD In ∆BCD, tan θ = = ...(i)
C ′ B + BP x + 200
BC x h cot β
⇒ 3= = D ⇒ x = …(iii)
AB AB cot α − cot β
⇒ x + 200 = AB 3 …(ii) ∴ Required height
18. (d) On dividing Eq. (i) by Eq. (ii), we get 5√3 m CE = CD + DE = x + h
x − 200 1 h cot β
= = + h
x + 200 3 30º θ cot α − cot β
⇒ 3x − 600 = x + 200 A 10 m B xm C h cot α
=
⇒ x = 400 m Again, in ∆ACD, cot α − cot β
Height of the cloud above the lake CD 5 3 1
tan 30° = = = 25. (b) Let shadow of a tower made by a Sun
= x = 400 m AC 10 + x 3 at an angle 60° is x m. Then, by given
19. (c) We have, AB = ( x − 200) 3 ⇒ 10 + x = 15, x = 5 m condition, the shadow of a tower made
= ( 400 − 200) 3 = 200 3 From Eq. (i), by a Sun at angle 30° is 50 m longer than
Horizontal distance = AB = 200 3 m 5 3 at angle 60°, i.e. (50 + x ) m.
tan θ = = 3 = tan 60° ⇒ θ = 60°
Sol. (Q. Nos. 20-21) Let A and B be the two 5
A
positions of the aeroplane and O be the AB Sun
23. (b) In ∆ABC , tan 15° =
point of observation. BC
OD
= cot 30° = 3 ⇒ OD = 900 3 m A (Lighthouse) h
BD 15º
OC 1 900 60° 30° D
= cot 60° = ⇒ OC = m B x C 50 m
70 m

AC 3 3
B Now, in ∆ABC ,
h h
A 15º (Boat) tan 60° = = 3 ⇒ x = m
B x C x 3
h 1
70 and in ∆ABD, tan30° = =
900 m ⇒ tan( 45° − 30° ) = x + 50 3
x h
Q tan( A − B ) = tan A − tan B  ⇒ 3 h = x + 50 ⇒ 3 h = + 50
30º  3
60º 1 + tan A tan B 
⇒  3 −
1
O C D 1  h = 50
1−  3
∴ AB = CD = OD − OC 3 = 70 ⇒ 3 − 1 70
⇒ = (3 − 1)
=  900 3 −
900  1800 ⇒ h = 50 ⇒ 2h = 50 3
 = m 1+
1 x 1 + 3 x
 3 3 3 3
Distance  1800      ∴ h = 25 3 m
20. (c) Speed = = 3 + 1  3 + 1 
 m/s
 3 × 10  ⇒ x = 70  × 26. (b) Let BL = x m and PL = h m
Time  3 − 1  3 + 1
   
= 
180 18  P
×  = 216 3 km/h
 3 5 70(3 + 1 + 2 3 )
= = 70( 2 + 3 )
21. (b) Let E be the position of the 3−1
Hence, the required distance is h
aeroplane after 40 s and θ be its elevation.
A B E 70 (2 + 3) m.
30° 45°
24. (b) Let AD = BE = y and AB = DE = h A 10 m B x L
(Hill) h
C In ∆PLB, tan 45° = = 1, x = h ...(i)
x
θ (Building) x h 1
O C D F A α Now, in ∆PLA , tan30° = =
D 10 + x 3
∴ AE = Speed × Time y
⇒ 3h = 10 + x
=  × 40 m =
1800 7200 h h
m
 3 × 10  3 β ⇒ 3h = 10 + h [from Eq. (i)]
B E
OF = OC + CF = OC + AE y ⇒ ( 3 − 1) h = 10
x
900 7200 8100 Now, in ∆ACD, tanα = 10 3+1
= + = m y ∴ h= ×
3 3 3 3− 1 3+ 1
⇒ y = x cot α …(i)
EF 900 × 3 1 10( 3 + 1) 10( 3 + 1)
tanθ = = = x+h = =
Now, in ∆BCE, tanβ =
OF 8100 3 3
y 3− 1 2
∴ θ = tan − 1 
1  = 5( 3 + 1) = (5 3 + 5) m
 ⇒ y = ( x + h ) cot β …(ii)
3 3 
MATHEMATICS Height and Distance 167

27. (c) Let AB and CD are two towers. 30. (c) Let the height of the aeroplane above and in ∆ABD, tan 43° = h / 49 …(ii)
ln ∆DCB , tan 60° =
H the ground is h and QB = x m. 1 h
⇒ =
BC A cot 43° 49
H H 49
⇒ 3= ⇒ BC = ...(i) ⇒ =h
BC 3 30° cot 43°
D 45°
49
⇒ h=
h cot ( 90°− 47° )
A 49
H ⇒ h=
30° 45° tan 47°
P B 49
h Q ⇒ h= [using Eq. (i)]
20 m x  h
60° 30°  
B C Given that, PQ = 20 m, ∠APB = 30°  36 
In ∆ABC , tan30° =
h

1
=
h and ∠AQB = 45° ⇒ h 2 = 49 × 36
BC 3 BC Now, in ∆ABQ, ⇒ h = 7 × 6 = 42 m
⇒ BC = h 3 ...(ii) AB h h 34. (b) Here, CB = CN − BN
tan 45° = = ⇒ 1= ⇒ x = h…(i)
From Eqs. (i) and (ii), we get QB x x = 20 − 10 = 10 m
H AB h
=h 3 and in ∆ABP, tan30° = = C
3 PB PQ + QB
∴ H =3 h 1 h

20 m
⇒ = A
28. (d) Let x be the breadth of the river and 3 20 + x 15° B

10 m

10 m
h be the height of the tree. ⇒ 3 h = 20 + x = 20 + h
A [from Eq. (i)]
M N
⇒ ( 3 h − h ) = 20
tan 15° = ( 45°− 30° ) 1
h ⇒ h ( 3 − 1) = 20 1−
3+ 1 tan 45°− tan 30° 3 = 3−1
∴h =
20
⋅ [rationalisation] = =
3− 1 3+ 1 1 − tan 45°⋅ tan 30° 1 − 1 3+1
30° 60° 3
B 40 m C x D 20 ( 3 + 1) 20 ( 3 + 1)
= = Now, in ∆ABC , tan 15° =
BC
h (3 − 1) 2
Now, in ∆ADC , tan 60° = AB
x = 10( 3 + 1) BC 10( 3 + 1)
h ⇒ AB = =
⇒ 3= ⇒ h = 3x ...(i) Hence, the required height is tan 15° ( 3 − 1)
x 10 ( 3 + 1) m. 10( 3 + 1) 3+1
h 1 = ×
Again, in ∆ADB, tan30° = =
x + 40 3 31. (b) Let AB be the lamp post of height h 3−1 3+1
⇒ h 3 = x + 40 and C be a point situated at a distance of = 10( 2 + 3 ) = 37.3 m
150 m from its foot B.
⇒ 3 ⋅ x ⋅ 3 = x + 40 [from Eq. (i)] A 35. (c) Let AB be tower of height h m and
⇒ 3x − x = 40 BC is the flag.
∴ x = 20 m C
So, the breadth of the river is 20 m. h 3m
B
29. (a) Let h be the height of the tower.
A 30º h
B 150 m C 45°
30°
In ∆ABC , we have D x A
h h 1 h
tan 30° = ⇒ = h
150 3 150 In ∆BDA, tan 30° =
x
150 150 × 3
60° 45° ∴ h= m= = 50 3 m ⇒ x = 3h …(i)
B 3 3 h+3
C 21 D
32. (b) Now, in ∆CDA, tan 45° =
x x
h h 33. (b) In ∆ACD, tan 47° = h / 36 …(i) ⇒x=h+3
In ∆ADC , tan 45° = ⇒1=
21 21 A ⇒ h = x − 3 = 3 h − 3 [from Eq. (i)]
⇒ h = 21m ...(i) ⇒ h( 1 − 3 ) = − 3
Again, in ∆ADB, 3 3( 3 + 1)
h h h ∴ h= =
tan 60° = ⇒ 3 = 3–1 2
x x
21 43° 47° = 15
. × 2.732 = 4.098 m
∴ x= =7 3m B
3 D
36 m
C = 4.10 m(approx)
[from Eq. (i)] 49 m
15
168 NDA/NA Pathfinder

PROPERTIES OF
TRIANGLES
In NDA exam, generally 1 question is asked from this chapter which is based on area of
triangle, sine/cosine rule, etc.

A triangle has six components, three sides and three angles. The three angles of a A
∆ABC are denoted by the letters A , B and C and the sides opposite to these angles by A
letters a, b and c respectively, i. e. a = BC, b = CA and c = AB. Semi-perimeter of the c b
a+ b+ c
∆ABC is , which is denoted by s and its area denoted by ∆.
2 B C
B a C
Also, sum of angles of a triangle is 180°, i.e. ∠ A + ∠ B + ∠C = 180 °

Sine Rule
sin A sin B sin C
In any ∆ABC, = = , i. e. the sines of the angles are proportional to the lengths of the
a b c
opposite sides.
a b c
Let = = = K (say)
sin A sin B sin C
1 1 1
Then, a = K sin A, b = K sin B, c = K sin C or sin A = a, sin B = b, sin C = c
K K K
EXAMPLE 1. The angles of a triangle are in the ratio 1 : 5 : 6. The ratio of its sides is
a. ( 3 − 1) : ( 3 + 1) : 2 2 b. ( 3 − 1) : 2 2 : ( 3 + 1) c. 2 2 : ( 3 − 1) : ( 3 + 1) d. None of these
Sol. a. Let the angles be x°, 5x°, 6x°. Then, x + 5x + 6x = 180 ⇒ x = 15
∴ Angles are 15°, 75°, 90°.
( 3 − 1) ( 3 + 1)
∴ Ratio of sides = sin 15° : sin 75° : sin 90° = : : 1= ( 3 − 1) : ( 3 + 1) : 2 2
2 2 2 2

Cosine Rule
In any ∆ABC,
b2 + c 2 − a 2 a 2 + c 2 − b2
(i) a 2 = b 2 + c 2 − 2 bc cos A or cos A = (ii) b 2 = c 2 + a 2 − 2ac cos B or cos B =
2bc 2ac
a 2 + b2 − c 2
(iii) c = a + b − 2ab cos C or cos C =
2 2 2
2ab
MATHEMATICS > Properties of Triangles 169

EXAMPLE 2. The sides of triangle are 3x + y, 4x + 3y, Some Other Formulae


and 5x + 5y units, where x, y > 0. The triangle is
In any ∆ABC, by sin 2 θ = 2sin θ ⋅ cos θ
a right angled b. equilateral
2 2∆
c. obtuse angled d. None of these (i) sin A = s ( s − a)( s − b)( s − c ) =
bc bc
Sol. c. Let a = 3x + 4y, b = 4x + 3y, and c = 5x + 5y. 2 2∆
(ii) sin B = s ( s − a)( s − b)( s − c ) =
Clearly, c is the largest side and thus the largest angle C is ac ac
given by
2 2∆
a2 + b2 − c2 (iii) sin C = s ( s − a)( s − b)( s − c ) =
cos C = ab ab
2ab
( 3x + 4y) 2 + ( 4x + 3y) 2 − ( 5x + 5y) 2 where, ∆ = area of scalene triangle, when a ≠ b ≠ c.
=
2 ( 3x + 4y) ( 4x + 3y)
Area of Triangle
−2xy
= <0 [since, x, y > 0] In a ∆ABC, if the sides of the triangle are a , b, c and
2 (12x2 + 25xy + 12y 2) corresponding angles are A , B and C respectively, then
⇒ C is an obtuse angle. area of triangle
1 1 1
Projection Formulae (i) ∆ = ab sin C, ∆ = bc sin A, ∆ = ca sin B
2 2 2
In any ∆ABC,
c 2 sin A sin B a 2 sin B sin C b 2 sin C sin A
(i) a = b cos C + c cos B (ii) b = c cos A + a cos C (ii) ∆ = ,∆= ,∆=
2 sin C 2 sin A 2sin B
(iii) c = a cos B + b cos A
(iii) ∆ = s ( s − a) ( s − b) ( s − c )
Note Any side of a triangle is equal to the sum of the projections
of other two sides on it. EXAMPLE 4. The area of the ∆ABC, in which a = 1,
b = 2, ∠C = 60 °, is
Trigonometrical Ratios of 1 3
a. 4 sq units b. sq unit c. sq unit d. 3 sq units
Half Angles of a Triangle 2 2
Sol. c. Given, a = 1, b = 2, ∠C = 60°
In any ∆ABC, let s = ( a + b + c ) / 2, then
1 1
A ( s − b)( s − c ) B ( s − a)( s − c ) ∴ Area of triangle = ab sin C = × 1 × 2 × sin 60°
(i) sin = (ii) sin = 2 2
2 bc 2 ac 3
= sin 60° = sq unit
C ( s − a)( s − b) A s ( s − a) 2
(iii) sin = (iv) cos =
2 ab 2 bc Tangent Formulae or Napier’s
B s ( s − b) C s (s − c )
(v) cos = (vi) cos = Analogy or Law of Tangents
2 ac 2 ab
In any ∆ABC,
A ( s − b)( s − c ) B ( s − a)( s − c )
(vii) tan = (viii) tan =
2 s ( s − a) 2 s ( s − b)  B − C  b − c A
(i) tan   =  cot
 2   b + c 2
C ( s − a)( s − b)
(ix) tan =
2 s (s − c )  C − A   c − a B
(ii) tan   =  cot
 2   c + a 2
A C B
EXAMPLE 3. If in ∆ABC, sin sin = sin and 2s is  A − B  a − b C
2 2 2 (iii) tan   =  cot
 2   a + b 2
the perimeter of the triangle, then s is
a. 2b b. b c. 3b d. 4b
EXAMPLE 5. If in ∆ABC, a = 6, b = 3 and
A C B 4
Sol. a. Given, sin sin = sin cos ( A − B) = , then its area will be
2 2 2 5
( s − b)( s − c) ( s − a)( s − b) ( s − a)( s − c) s−b
∴ = ⇒ =1 a 7 sq units b. 8 sq units
bc ab ac b c. 9 sq units d. None of these
⇒ s = 2b
170 NDA/NA Pathfinder

4 The centres of the escribed circles are called excentres.


Sol. c. We have, a = 6, b = 3, cos ( A − B) =
5
 A − B

le
Let t = tan   , then

rc
 2 

ci
Ex
Excircle
1− t2 4 1− t
2
1 l3
A
r2
cos ( A − B) = 2 ⇒ = ⇒ t= r3
1+ t 5 1+ t2 3 l2
 A − B 1 C
So, tan   =
 2  3 B r1

 A − B a − b C l1
Then, tan   = cot
 2  a+ b 2
Excircle
1 6−3 C
⇒ = cot ⇒ C = 90° In any ∆ABC,
3 6 +3 2
∆ A A B C
Hence, ∆ =
1
(6) ( 3) sin 90° = 9 sq units
(i) r1 = = s tan = 4Rsin cos cos
2 s−a 2 2 2 2
∆ B B C A
Circumcircle of a Triangle (ii) r 2 = = s tan = 4Rsin cos cos
A s−b 2 2 2 2
A circle passing through the vertices
of a ∆ABC is called circumcircle and ∆ C C A B
(iii) r 3 = = s tan = 4Rsin cos cos
its radius is called circumradius (R) c b s−c 2 2 2 2
and its centre is known as O (iv) r1 + r 2 + r 3 = 4R + r
circumcentre. r1r 2r 3
Circumradius of triangle is given by B a C (v) r1r 2 + r 2r 3 + r 3r1 = s 2 =
r
a b c abc
R= = = = a cos B/ 2 cos C/ 2
2sin A 2sin B 2sin C 4∆ (vi) r1 =
cos A/ 2
Incircle of a Triangle b cos C / 2cos A / 2
(vii) r 2 =
The circle which can be inscribed within the triangle so cos B / 2
as to touch each of its sides is called its inscribed circle c cos A / 2cos B / 2
(viii) r 3 =
or incircle and its centre is called incentre and its radius cos C / 2
is called inradius (r).
1 1 1 s 1
In any ∆ABC, (ix) + + = =
r1 r 2 r 3 ∆ r

(i) r = 2
s  A B C
and r1r 2r 3 = r 2 cot cot cot 
(ii) r = ( s − a) tan
A
= ( s − b) tan
B
= ( s − c ) tan
C  2 2 2
2 2 2
A B C EXAMPLE 6. If r1 = r2 + r3 + r , then triangle is
(iii) r = 4 R sin sin sin
2 2 2 a. right angled b. obtuse angled
a sin B/ 2 sin C/ 2 A c. equilateral d. None of these
(iv) r =
cos A/ 2 Sol. a. Q r1 − r = r2 + r3
b sin A/ 2 sin C/ 2 c b
∆ ∆ ∆ ∆ s−s+ a s−c+ s−b
= O r ⇒ − = + ⇒ =
cos B/ 2 s−a s s−b s−c s( s − a) ( s − b)( s − c)
r
c sin B/ 2 sin A/ 2 a a
= B a C ⇒ = ⇒ s2 − ( b + c) s + bc = s2 − as
cos C/ 2 s( s − a) ( s − b)( s − c)
( b + c + a)( b + c − a)
Escribed Circles of a Triangle ⇒ s( − a + b + c) = bc ⇒ = bc
2
The circle which touches the sides BC and two sides ⇒ ( b + c) 2 − a2 = 2bc ⇒ b2 + c2 + 2bc − a2 = 2bc
AB and AC produced of a ∆ABC, is called the escribed ⇒ b2 + c2 = a2
circle opposite to the ∠A. Its radius is denoted by r1 . ∴ ∠A = 90°
Similarly, r 2 and r 3 denote the radii of the escribed
Thus, the triangle is right angled triangle.
circles opposite to the angles B and C, respectively.
MATHEMATICS > Properties of Triangles 171

PRACTICE EXERCISE
1 + cos( A − B) cos C 11. If A, B, C are in AP and b : c = 3 : 2, then what
1. is equal to
1 + cos( A − C ) cos B is the value of sinC?
a2 + b 2 b2 + c 2 (a) 1 (b) 1/ 3 (c) 3 (d) 1/ 2
(a) (b)
a +c
2 2
b −c
2 2
12. In a ∆ABC, if 2R + r = r1, then
c 2 − a2
(c) (d) None of these (a) ∠A = π / 2 (b) ∠B = π / 2
a2 + b 2 (c) ∠C = π / 2 (d) None of these
c − a cos B sin A sin( A − B)
2. In any ∆ABC, is equal to 13. If = , then the sides of ∆ABC are in
a − c cos B sin C sin( B − C )
cos A cos C sin A sin C (a) AP (b) GP (c) HP (d) None of these
(a) (b) (c) (d)
cos C cos A sin C sin A
14. If a cos A = b cos B, then ∆ABC is
A b+ c
3. If cos = , then (a) isosceles (b) right angled
2 2c (c) equilateral (d) Either (a) or (b)
(a) a2 + b 2 = c 2 (b) b 2 + c 2 = a2 cos A cos B cos C
(c) c 2 + a2 = b 2 (d) None of these 15. If = = and the side a = 2, then
a b c
4. If a = 16, b = 24, c = 20, then cos   is equal to area of triangle is
B
 2 3
(a) 1 (b) 2 (c) (d) 3
(a) 3/4 (b) 1/4 (c) 1/2 (d) 1/3 2
16. If in a ∆ABC, a + c = 2b, then the value of
5. The area of the ∆ABC, in which a = 1, b = 2, A C
∠C = 60°, is cot ⋅ cot is equal to
1
2 2
(a) 4 sq units (b) sq unit (a) 4.5 (b) 3 (c) 1.5 (d) 1
2
(c)
3
sq unit (d) 3 sq units
17. If a = 4, b = 3 and ∠A = 60°, then c is the root of
2 the equation
6. If c2 = a 2 + b2, then 4s ( s − a )( s − b)( s − c) is equal (a) c 2 − 3c − 7 = 0 (b) c 2 + 3c + 7 = 0
to (c) c − 3c + 7 = 0
2
(d) c 2 + 3c − 7 = 0
(a) s 4 (b) b 2c 2 (c) c 2 a2 (d) a2 b 2 18. The straight roads intersect at an angle of 60°. A
b +c −a
2 2 2 bus on one road is 2 km away from the
7. In any ∆ABC, equals intersection and a car on the other road is 3 km
4 cot A
away from the intersection. Then, the direct
(a) ∆ (b) 1/ ∆ (c) 2 ∆ (d) None of these distance between the two vehicles is
8. In a ∆ABC , a + b = 3 (1 + 3 ) cm and (a) 1 km (b) 2 km (c) 4 km (d) 7 km
a − b = 3 (1 − 3 ) cm. If ∠A is 30°, then what is 19. In a ∆ABC , a + b + c = 2( a + c )b , then the ∠B is
4 4 4 2 2 2

the measure of ∠B? (a) 45° , 135° (b) 45° , 45°


(a) 120° (b) 90° (c) 75° (d) 60° (c) 45° , 90° (d) None of these
9. ABC is a triangle in which BC = 10 cm, 20. ABC is a triangle in which AB = 6cm , BC = 8 cm
CA = 6 cm and AB = 8 cm. Which one of the and CA = 10 cm. What is the value of cot ( A/ 4)?
following is correct? (a) 5 −2 (b) 5+2 (c) 3 −1 (d) 3+1
(a) ABC is an acute angled triangle
a−b
(b) ABC is an obtuse angled triangle 21. In a ∆ABC, is equal to
a+b
(c) ABC is a right angled triangle
sin( A − B) cos( A − B)
(d) None of the above (a) (b)
sin( A + B) cos( A + B)
10. In a ∆ABC , if cos B = (sin A)/( 2 sin C ), then the A − B
tan  
triangle is  2 
(c) (d) None of these
A + B
tan 
(a) isosceles triangle (b) equilateral triangle

(c) right angled triangle (d) scalene triangle  2 
172 NDA/NA Pathfinder

22. In a ∆ABC, if cot A, cot B, cotC are in AP, then Directions (Q. Nos. 32-33) In a ∆ ABC,
2 2 2 b +c c + a a+ b
a , b , c are in = =
11 12 13
(a) AP (b) GP (c) HP (d) None of these
32. The ratio sin A : sin B : sin C is
23. In a ∆ABC, ( a + b + c)  tan + tan  is equal to
A B
 2 2 (a) 9 : 10 : 11 (b) 7 : 19 : 25
A B C (c) 7 : 6 : 5 (d) 13 : 12 : 11
(a) 2 acot (b) 2 b cot (c) 2c cot (d) None of these
2 2 2 33. The value of cos A is
24. In ∆ABC, if a = 7, b = 8 and c = 9, then the length (a) 1/5 (b) 19/35 (c) 5/7 (d) None of these
of line segment joining B to the mid-point of AC is
(a) 5 (b) 6 (c) 7 (d) 8
Directions (Q. Nos. 34-35) In a ∆ABC, a = 5, b = 4
31
A B C and cos ( A − B) = . Then,
25. In ∆ABC, if cot : cot : cot = 3 : 5 : 7, then 32
2 2 2
a : b : c is equal to 34. Find the value of cos C.
(a) 4 : 5 : 6 (b) 5 : 6 : 4 (c) 6 : 5 : 4 (d) 6 : 4 : 5 1 4 6 1
(a) (b) (c) (d)
2 3 7 8
26. In any ∆ABC, a cos A + b cos B + c cos C is equal to
(a) 4 R (b) sin A sin B sin C
35. Find the value of side c.
(a) 2 (b) 6 (c) 10 (d) 12
(c) R sin A sin B sin C (d) 4 R sin A sin B sin C
27. In a ∆ABC, consider the following statements. Directions (Q.Nos. 36-38) In any ∆ABC, tan
A 5
=
I. rr1r2r3 = ∆2 2 6
II. r1r2 + r2r3 + r3 r1 = s2 B 20
and tan =
Which of the above statement(s) is/are is correct? 2 37
(a) Only I (b) Only II C
36. What is the value of tan ?
(c) Both I and II (d) Neither I nor II 2
4 3 2
28. Consider the following statements (a) (b) (c) (d) None of these
5 5 5
2C
B
I. In ∆ABC, b cos + c cos =s 2
2 2 37. The sides of the triangle a , b, c are in
A b+ c (a) AP (b) GP (c) HP (d) None of these
II. In ∆ABC, cot = ⇒ B = 90°
2 2
38. Consider the following statements
Which of the above statement(s) is/are correct? I. sin A , sin B, sin C are in HP.
(a) Only I (b) Only II II. Altitudes are in HP.
(c) Both I and II (d) Neither I nor II
Which of the above statement(s) is/are correct?
29. In a triangle, the greatest angle is 120°, when (a) Only I (b) Only II
sides of triangle are (c) Both I and II (d) Neither I nor II
I. x, y and x2 + xy + y2, (x, y > 0)
II. a − b, a + b and 3a 2 + b2, (a , b > 0)
Directions (Q. Nos. 39-42) Given that
∆ = 6, r1 = 2, r2 = 3, r3 = 6.
III. 2, 6 and 3 −1
39. Circumradius R is equal to
Which of the above statement(s) is/are correct? (a) 2.5 (b) 3.5 (c) 1.5 (d) 4.2
(a) Only I (b) Only II (c) Only III (d) All of these
40. Inradius is equal to
Directions (Q. Nos. 30-31) In a ∆ABC , (a) 2 (b) 1 (c) 1.5 (d) 2.5
2 cos A cos B 2 cos C a b 41. Difference between the greatest and the least
+ + = + . Then,
a b c bc ac angle is
(a) cos −1   (b) tan−1  
4 3
30. b + c is equal to
2 2  5  4

(c) cos −1  
3
(a) a2 (b) ac (c) bc (d) ab (d) None of these
 5
31. ∆ABC is
(a) an equilateral triangle (b) a right angled triangle 42. The value of sin2 A + sin2 B + sin2 C is
(c) an isosceles triangle (d) Cannot be determined (a) 0 (b) 1 (c) −1 (d) 2
MATHEMATICS > Properties of Triangles 173

Directions (Q. Nos. 43-46) If p1, p 2, p3 are altitudes


50. Consider the following statements e 2014 I
of a ∆ABC from the vertices A,B,C respectively and
∆ is the area of the triangle and s is semi-perimeter I. There exists no ∆ABC for which
of the triangle. sin A + sin B = sin C.
II. If the angles of a triangle are in the ratio 1 : 2 : 3,
43. The value of p1−2 + p2−2 + p3−2 is then its sides will be in the ratio 1 : 3 : 2.
(Σa)2 (Πa)3 Σ a2 Πa2 Which of the above statement(s) is/are correct?
(a) (b) (c) (d)
4∆ 2
8∆ 3
4∆2
8∆2 (a) Only I (b) Only II
(c) Both I and II (d) Neither I nor II
44. In the ∆ABC, the altitudes are in AP, then
(a) a, b, c are in AP (b) a, b, c are in HP 51. In a ∆ABC, c = 2, A = 45° and a = 2 2, then what
(c) a, b, c are in GP (d) angles A, B, C are in AP is C equal to? e 2014 II
sin A sin B sin C (a) 30° (b) 15°
45. The value of + + is
p1 p2 p3 (c) 45° (d) None of these
1 a2 + b 2 + c 2 1 1 52. In a ∆ABC, if a = (1 + 3 ) cm, b = 2 cm and
(a) (b) (c) (d)
R 4∆R ∆ r
∠ C = 60°, then the other two angles are e 2015 I
1 1 1 1
46. If + + = , then the least value of p1 p2 p3 is (a) 45° and 75° (b) 30° and 90°
p1 p2 p3 2 (c) 105° and 15° (d) 100° and 20°
(a) 8 (b) 27 (c) 125 (d) 216
Directions (Q. Nos. 53-54) Consider a ∆ABC
satisfying 2 a sin2   + 2c sin2   = 2 a + 2c − 3b
C A
PREVIOUS YEARS’ QUESTIONS 2 2 e 2015 II
47. In a ∆ABC, if the angles A, B, C are in AP, then 53. The sides of the triangle are in
which one of the following is correct? e 2012 I (a) GP (b) AP (c) HP
(a) c = a + b (b) c 2 = a2 + b 2 − ab (d) Neither in GP nor AP nor in HP
(c) a2 = b 2 + c 2 − bc (d) b 2 = a2 + c 2 − ac
54. sin A, sin B, sin C are in
48. In any ∆ABC, a = 18, b = 24 and c = 30, then (a) GP (b) AP (c) HP
what is sin C equal to? e 2013 I (d) Neither in GP nor in AP nor in HP
(a) 1/4 (b) 1/3 (c) 1/2 (d) 1
55. If a, b and c are the sides of a ∆ABC, then
49. If the angles of a triangle are 30° and 45° and 1 1 1
the included side is ( 3 + 1) cm, then what is the a + b − c , where p > 1, is
p p p

area of the triangle? e 2013 II (a) always negative


3 +1
(a) cm2 (b) 2 ( 3 + 1) cm2 (b) always positive
2 (c) always zero
3 +1 3 −1
(c) cm2 (d) cm2 (d) positive, if 1 < p < 2 and negative, if p > 2
3 2

ANSWERS
1 a 2 a 3 a 4 a 5 c 6 d 7 a 8 d 9 c 10 a
11 d 12 a 13 a 14 d 15 d 16 b 17 a 18 d 19 a 20 b
21 c 22 a 23 c 24 c 25 c 26 d 27 c 28 a 29 d 30 a
31 b 32 c 33 a 34 d 35 b 36 c 37 a 38 b 39 a 40 b
41 c 42 d 43 c 44 b 45 b 46 d 47 d 48 d 49 a 50 c
51 a 52 a 53 b 54 b 55 b
174 NDA/NA Pathfinder

HINTS AND SOLUTIONS


1 + cos( A − B ) cos C 9. (c) 14. (d) k sin A cos A = k sin B cos B
1. (a)
1 + cos( A − C ) cos B sin A [by sine rule]
10. (a) Given, cos B =
⇒ sin 2 A = sin 2B
= 1 − cos( A − B ) cos( A + B ) 2 sin C
1 − cos( A − C ) cos( A + C ) a 2 + c2 − b 2 a ⇒ 2 A = 2B or ( π − 2B )
⇒ = [by sine rule]
[in a triangle,cos C = − cos( A + B ) 2ac 2c So, A = B , i.e. ∆ABC is isosceles.
and cos B = − cos ( A + C )] π π
⇒ a 2 + c2 − b 2 = a 2 or A + B = , so that C =
2 2
= 1 − (cos 2 − sin 2 B )
2 A 2
⇒ b 2 = c2 ⇒ b = c
1 − (cos A − sin C ) i.e. ∆ABC is right angled.
Since, two sides of angle are equal, so
sin 2 A + sin 2 B a 2 + b 2 triangle is isosceles. 15. (d) By sine rule,
= = 2 cos A cos B cos C
sin 2 A + sin 2 C a + c2 11. (d) Given, A , B and C are in AP. = =
K sin A K sin B K sin C
⇒ 2B = A + C ...(i)
2. (a) Using c = a cos B + b cos A and
a = b cos C + c cos B from the projection and in ∆ABC , A + B + C = π ...(ii) ⇒ tan A = tan B = tan C
π
formula, we get the given expression, From Eqs. (i) and (ii), B = = 60° ⇒ Triangle is equilateral.
a cos B + b cos A − a cos B cos A 3 1
= By sine formula, ∴ ∆ = a ⋅ a sin 60° = 3 as a = 2
b cos C + c cos B − c cos B cos C 2
sin B sin C c
A b+c = ⇒ sinC = ⋅ sin B 16. (b)
3. (a) Given, cos = b c b
2 2c 2  b 3 17. (a) By cosine rule,
b+c = ⋅ sin 60°  given, =
⇒ cos 2 A
= 3  c 2  cos A =
b 2 + c2 − a 2
2 2c 2bc
2 3 1
s( s − a ) b + c = × =
⇒ = 3 2 2 (3 ) 2 + c 2 − ( 4 ) 2
bc 2c ∴ cos 60° =
12. (a) Given, 2R + r = r 2 ×3 × c
⇒ 2s ( s − a ) = b( b + c ) 1
1 9 + c 2 − 16
⇒ ( a + b + c )( b + c − a ) = 2b( b + c ) ⇒ 2R = r − r ⇒ =
1
A B C 2 2 ×3 × c
⇒ ( b + c )2 − a 2 = 2b 2 + 2bc ⇒ 2R = 4R sin cos cos
2 2 2 ⇒ 3c = c 2 − 7
⇒ c2 = a 2 + b 2 A
−4R sin sin sin
B C ⇒ c − 3c − 7 = 0
2

4. (a) 5. (c) 2 2 2 18. (d) a = 2, b = 3, ∠C = 60°


6. (d) Q c 2 = a 2 + b 2 ⇒ ∠C = 90° A
⇒ 2R = 4R sin Bus
2
∴ 4s ( s − a )( s − b )( s − c ) = 4∆2
2  cos B cos C − sin B sin C 
= 4  ab  = a 2 b 2
1  2 2 2 2  2 km
2 
B+C
⇒ 2R = 4R sin cos 
A

b 2 + c2 − a 2 b 2 + c2 − a 2 2  2 
7. (a) = sin A 60°
B+C π A
1 = 2 sin 2 Q = − 
4 cot A 4 cos A A
⇒ Car 3 km
b 2 + c2 − a 2 2  2 2 2 
= × 2bc × sin A
4 ( b 2 + c2 − a 2 ) A 1 π
⇒ sin = ⇒ ∠A = ∴ c 2 = a 2 + b 2 − 2ab cos 60°
2 2 2
1 1
= × bc × sin A = ∆ 13. (a) We know that, = 13 − 12 ⋅ = 7 ⇒ c = 7 km
2 2
sin C = sin{ π − ( A + B )} = sin( A + B )
c + a −b 
2 2 2
8. (d) a + b = 3 ( 1 + 3 ) ...(i) and sin A = sin( B + C ) 19. (a) cos B =  
a − b = 3(1− 3) ...(ii) sin A sin( A − B )  2ca 
∴ =
On solving Eqs. (i) and (ii), we get sin C sin( B − C )  a 4 + b 4 + c 4 − 2b 2 c 2 
a = 3 and b = 3 3 sin( B + C ) sin( A − B )  
⇒ =  − 2a 2 b 2 + 2a 2 c 2 
By sine rule,
sin A
=
sin B sin( A + B ) sin( B − C ) ⇒ cos B =
2  
a b ⇒ sin 2 B − sin 2 C = sin 2 A − sin 2 B 4a 2 c 2
sin 30° sin B 1 sin B 1
⇒ = ⇒ = [Q sin ( A + B ) ⋅ sin ( A − B ) ⇒ cos 2 B =
3 3 3 2 3 2
3 = sin 2 A − sin 2 B] ⇒ cos B = ±
1
⇒ sin B = ⇒ sin B = sin 60° ⇒ b − c = a2 − b2
2 2 2
2
∴ B = 60° ⇒ 2b 2 = a 2 + c 2 ∴ B = 45° or 135°
So, a, b and c are in AP. 20. (b) Here, a = 8, b = 10 and c = 6 cm
MATHEMATICS Properties of Triangles 175

8 + 10 + 6 ( s − c) [ 2s − ( a + b )] 27. (c) We have,


s = = 12 = ( 2s ) ⋅
2 s ( s − a ) ( s − b) ∆ ∆ ∆ ∆
r rr r = × × ×
A ( 12 − 10)( 12 − 6) 1 2 3
s −a s −b s −c
∴ tan = 1 s( s − c) C
s
2 12( 12 − 8) = 2( s )c ⋅ ⋅ = 2c cot ∆4
s ( s − a) ( s − b) 2 = = ∆2
=
1
=
1
⇒ cot
A
=2 ∆2
4 2 2 24. (c) Let D be the mid-point of AC . Then, So, Statement I is true.
AD = DC = 4 Now, r r + r r + r r
Now, cot   = cot  + 
A A A 1 2 2 3 31
 2 4 4 Let BD = K ∆2 ∆2
= +
2 A A From ∆ABC , we have ( s − a )( s − b ) ( s − b )( s − c )
cot − 1 cot 2 − 1
= 4 = 4 b 2 + c 2 − a 2 ( 64 + 81 − 49) 2 ∆2
A A cos A = = = +
2 cot 2 cot 2bc ( 2 × 8 × 9) 3 ( s − c )( s − a )
4 4
A = s( s − c) + s( s − a ) + s( s − b )
 A
Let cot   = x
 4 4 = s( s − a + s − b + s − c) = s 2
x2 − 1 9 D So, Statement II is also true.
∴ 2= ⇒ x 2 − 4x − 1 = 0
2x 4 C B
28. (a) We have, b cos 2 + c cos 2
4 ± 16 + 4 2 2
⇒ x = = 2± 5 B 7 C
2 From ∆ABD, we have s( s − c) s( s − b )
=b + c
Since, A is an acute angle. 92 + 42 − K 2 ab ac
cos A = s
= ( 2s − b − c ) = s
∴ cot   = 2 + 5
A
2×9×4 a
 4
2 97 − K 2
a − b K sin A − K sin B ⇒ = ⇒ K 2 = 49, So, Statement I is true.
21. (c) = 3 72
a + b K sin A + K sin B A b+c
i.e. K =7 Now, let cot =
(sin A − sin B ) 2 a
= Hence, BD = 7
(sin A + sin B ) cos
A
A + B A − B A B C
25. (c) cot : cot : cot = 3 : 5 : 7 ⇒ 2 = sin B + sin C
2 cos   sin   2 2 2 A
 2   2  sin sin A
= s( s − a ) s( s − b ) s( s − c) 2
A + B A − B ⇒ = 3:5: 7
2 sin   cos   : :
 2   2  ∆ ∆ ∆ A 2 sin  B + C  cos  B − C 
cos
⇒ ( s − a ):( s − b ):( s − c ) = 3 : 5 : 7 2 =  2   2 
A − B ⇒
tan   ( b + c − a ) ( c + a − b ) ( a + b − c) A A A
 2  ⇒ : : sin 2 sin cos
= 2 2 2 2 2 2
A + B
tan   = 3:5: 7 B −C 
 2  A
⇒ cos = cos   
⇒ ( b + c − a ): ( c + a − b ) : ( a + b − c ) 2  2 
22. (a) Since, cot A, cot B, cot C are in AP.
= 3:5: 7 A B −C
⇒ cot B − cot A = cot C − cot B ⇒ =
cos B cos A cos C cos B Let b + c − a = 3 K , c + a − b = 5K 2 2
⇒ − = −
sin B sin A sin C sin B and a + b − c = 7K ⇒A+C =B
sin( A − B ) sin( B − C ) Adding, ( a + b + c ) = 15K ⇒ 2B = 180°
⇒ =
sin A sin C ∴ 2a = 12K , 2b = 10K , 2c = 8K ⇒ B = 90°
sin( A − B ) sin( B − C ) ⇒ a : b : c = 6:5: 4 A b+c
⇒ = But, Statement II is cot =
sin[ π − ( B + C )] sin[ π − ( A + B )] 26. (d) a cos A + b cos B + c cos C 2 2
⇒ sin( A − B ) sin( A + B ) = 2R sin A cos A + 2R sin B cos B So, Statement II is not true.
= sin( B − C ) sin( B + C ) + 2R sin C cos C 29. (d) I. Clearly, x 2 + xy + y 2
⇒ sin 2 A − sin 2 B = sin 2 B − sin 2 C [Q a = 2 R sin A , b = 2 R sin B
⇒ a 2 − b 2 = b 2 − c2 is the greatest side.
and c = 2 R sin C ]
⇒ a , b , c 2 are in AP.
2 2 x 2 + y 2 − ( x 2 + xy + y 2 )
= R (sin 2 A + sin 2 B + sin 2C ) ∴ cos θ =
2xy
23. (c) Given expression
= R [ 2 sin ( A + B ) cos ( A − B )
 ( s − b )( s − c ) ( s − a )( s − c )  −1
= ( 2s )  + + 2 sin C cos C ] =
 s( s − a ) s ( s − b )  = 2 R [sin ( π − C ) cos ( A − B ) 2
−1
( s − c)  ( s − b ) ( s − a)  + sin C cos { π − ( A + B )}] ⇒ θ = cos −1   = 120°
= ( 2s )  2
 ( s − a) + ( s − b ) 
= 2 R [sin C {cos ( A − B )
s 
− cos ( A + B )}] So, Statement I is true.
= 4 R sin A sin B sin C
176 NDA/NA Pathfinder

II. 3a 2 + b 2 is the greatest side. a−b C 1 ∆ 6 6


⇒ ⋅ cot = 40. (b) r = = = =1
a+ b 2 63 s 3+ 4+5 6
( a − b )2 + ( a + b )2 
   A − B a−b C 2
 − (3 a 2 + b 2 )  Q tan   = cot 
  a+ b 41. (c) Since, the triangle is right angled,
∴ cos θ =    2 2
 greatest angle is 90°.
2( a − b )( a + b ) 1 C 1 C 7
2a 2 + 2b 2 − 3a 2 − b 2 ⇒ cot = ⇒ tan = Also, the least angle is opposite to side a,
= 9 2 63 2 3
2( a 2 − b 2 ) C 7 which is sin −1  .
3
1 − tan 2 1− 5
−( a 2 − b 2 ) −1 Now, cos C = 2 = 9 = 1
= =
∴ Difference = 90° – sin −1  
2C 7 8 3
2( a 2 − b 2 ) 2 1 + tan 1+
2 9 5
− 1  − 1
∴ θ = cos   = 120° = cos −1  
3
 2 35. (b) Q c 2 = a 2 + b 2 − 2ab cos C
1 5
So, Statement II is also true. ∴ c 2 = 25 + 16 − 40 × = 36 ⇒ c = 6
8 42. (d) Since, side c of ∆ABC is greatest.
III. We have, ∴ ∠C = 90° and ∠B = 90° – A
A 5 B 20
( 2)2 + ( 3 − 1)2 − ( 6 )2 36. (c) We have, tan = and tan =
cos θ = 2 6 2 37 ∴ sin 2 A + sin 2 B + sin 2 C
2( 2)( 3 − 1) 5 20
+ = sin 2 A + sin 2 ( 90°− A ) + sin 2 90°
4 + 4 − 2 3 − 6 −1 A + B
⇒ tan 
= = 305
 = 6 37 = = sin 2 A + cos 2 A + 1 = 2
4( 3 − 1) 2  2  5 20 122
1− × Sol. (Q. Nos. 43-46) AD = p ,
− 1  − 1 6 37 1
⇒ θ = cos   = 120°
 2  π C  305 C 305 BE = p and CF = p
⇒ tan  −  = ⇒ cot = 2 3
So, Statement III is also true. 2 2  122 2 122 A
C 122 2
30. (a) We have, ⇒ tan = = E
2 cos A cos B 2 cosC a b 2 305 5 F
+ + = +
A C 5 2 1 b
a b c bc ac 37. (a) tan ⋅ tan = × = c
Multiplying both sides by abc, 2 2 6 5 3
2bc cos A + ac cos B +2ab cos C = a 2 + b 2 ∆ ∆ 1
⇒ × =
( c2 + a 2 − b 2 ) s( s − a ) s( s − c) 3 B D a C
⇒ ( b 2 + c2 − a 2 ) +
2 s −b 1 1 1 1
⇒ = Q ∆ = ⋅ a⋅ p = ⋅ b⋅ p = c⋅ p
1 2 3
+ ( a 2 + b 2 − c2 ) = a 2 + b 2 s 3 2 2 2
⇒ c 2 + a 2 − b 2 = 2a 2 − 2b 2 ⇒ 2s = 3b ⇒ a + c = 2b 2∆ 2∆ 2∆
∴ p = ,p = ,p =
⇒ b 2 + c2 = a 2 ∴ a , b , c are in AP. 1
a 2 b 3
c
−2 −2 −2
31. (b) Since, b 2 + c 2 = a 2 38. (b) Since, a , b , c are in AP. 43. (c) p + p + p
1 2 3

Hence, ∆ABC is a right angled triangle. ⇒ sin A , sin B , sin C are in AP. −2 −2 −2
2∆  2∆  2∆ 
So, Statement I is not correct. =   +   +  
 a   b   c 
Sol. (Q. Nos 32-33) Let P , P , P be the altitudes of ∆ABC .
b+ c c+ a
= =
a+ b
= k. Then,
1 2 3
a 2 + b 2 + c 2 Σa 2
Let Q a , b , c are in AP. = =
11 12 13 2∆ 2∆ 2∆ 4∆2 4∆2
b + c = 11k, c + a = 12k, a + b = 13k ⇒ , , are in AP.
P P P 44. (b) Q p , p , p are in AP.
On adding, we get 2( a + b + c ) = 36k 1 2 3 1 2 3

⇒ a + b + c = 18k ⇒ P , P , P are in HP. 2∆ 2∆ 2∆


1 2 3 ⇒ , , are in AP.
∴ a = 7k, b = 6k and c = 5k So, Statement II is correct. a b c
sin A sin B sin C Sol. (Q. Nos. 39-42) We have, ∴ a , b , c are in HP.
32. (c) Since, = =
a b c ∆ ∆ sin A sin B sin C
r = = 2, r = = 3, 45. (b) + +
⇒ sin A : sin B : sin C = a : b : c = 7 : 6 : 5 1
s −a 2
s −b p p p
1 2 3
b 2 + c2 − a 2 ∆
33. (a) cos A = r = =6 2∆ 2∆ 2∆
2bc
3
s −c
Given, ∆ = 6 = bc + ca + ab
36k 2 + 25k 2 − 49k 2 12k 2 1 p p p
= = = 1 2 3
2 ⋅ ( 6 k ) (5 k ) 60k 2 5 ∴ s − a =3 ⇒ b+ c−a = 6 …(i)
 1 1 1 
s − b = 2 ⇒ a+ c−b = 4 …(ii) = 2∆  + +
A − B  p bc p ca p ab 
34. (d) We know that, tan   s − c = 1 ⇒ a + b − c = 2 …(iii)  1 
 2 
2 3
On solving Eqs. (i), (ii) and (iii), we get
= 2∆ 
31 a b c 
1− + + 
1 − cos( A − B ) 32 = 1 a = 3, b = 4, c = 5.  2∆bc 2∆ca 2∆ab 
= =
1 + cos ( A − B ) 31 abc 60
1+ 63 39. (a) R = = = 2.5 abc  a 2 + b 2 + c 2  a 2 + b 2 + c 2
32 4∆ 24 =   =
2R  2∆abc  4∆R
MATHEMATICS Properties of Triangles 177

1
+
1
+
1
=
1 ( 3 + 1) 2 2 4 + 6 − ( 1 + 3 )2
46. (d) Given, ⇒ AC = × = 2 =
p
1
p p
2
2
3 2 ( 3 + 1) 2×2× 6
AM > GM ∴ Area of ∆ ABC 3− 3 3 3 −1
Q
1 = =
1
+
1
+
1 = × BC × AC × sin 105° 2 6 2 6
1/3 2
p p p  1 1 1 3−1
1 2 3 ≥  . .  1 ( 3 + 1) 3+1 =
= ×2× 2 × = cm 2
3  p1 p2 p3  2 2 2 2 2 2
3 = cos 75°
 1 50. (c) I. Given that, sin A + sin B = sin C
 2 1
or   ≥ ( aK ) + ( bK ) = ( cK ) ∴ A = 75°,
3 p p p
1 2 3 ⇒ a+ b=c B = 45°
 
∴ p p p ≥ 216 It is not possible, because the sum of and C = 60°
1 2 3
the length of two sides of a triangle
47. (d) Given angles A , B and C are in AP. is always greater than the length of 53. (b) We have,
Thus, 2B = A + C the third side. C A
2a sin 2 + 2 c sin 2
In ∆ABC , ∠A + ∠B + ∠C = 180° Hence, there exists no ∆ABC for 2 2
2B + B = 180° ⇒ B = 60° which sin A + sin B = sin C . = 2a + 2c − 3b
According to cosine rule, II. Given that, the ratio of the angles of ⇒ a ( 1 − cos C ) + c ( 1 − cos A )
a 2 + c2 − b 2 a triangle are A : B : C = 1 : 2 : 3. = 2a + 2c − 3b
cos B = = cos 60°
2ac Let A = α, B = 2α and C = 3α
⇒ ( a + c ) – ( a cos C + c cos A )
1 a 2 + c2 − b 2 We know that, A + B + C = 180°
⇒ = = 2a + 2c – 3b
2 2ac ⇒ α + 2α + 3α = 180°
⇒ a + c − b = 2a + 2c − 3b
⇒ ac = a 2 + c 2 − b 2 ⇒ 6α = 180° ⇒ α = 30°
[Q b = a cos C + c cos A]
⇒ b 2 = a 2 + c 2 − ac ∴ A = 30°, B = 60° and C = 90°
⇒ − a − c = − 2b
48. (d) Given that, a = 18, b = 24 and c = 30 ∴ a : b : c = sin A : sin B : sin C
[by sine rule] ⇒ a + c = 2b
Now, by cosine law,
= sin 30° : sin 60° : sin 90° Hence, a, b, c are in AP.
 −c + a + b 
2 2 2
cos C =   1 3 54. (b) Since, a, b, c are in AP.
 2ab  = : : 1 = 1: 3 : 2
2 2 So, K sin A, K sin B, K sin C also in
(18) + (24) − (30)
2 2 2
= Hence, Statement II is correct. AP, where K is an any non-zero
2 × 18 × 24 constant.
324 + 576 − 900 900 − 900 51. (a) By sine rule,
= = = 0 a b c ∴ sin A, sin B, sin C are in AP.
864 864 = =
⇒ cos C = cos 90° ⇒ ∠ C = 90° sin A sin B sin C 55. (b) Since, a, b and c are sides of a triangle
∴ sin C = sin 90° = 1 c ⋅ sin A 2 ⋅ sin 45°
⇒ sin C = = ∴ a, b and c are positive numbers
49. (a) Let ∠A = 30° , ∠B = 45° and a 2 2
1 1 1 and a + b>c
AB = 3 + 1 = ⋅ = = sin 30°
2 2 2 Therefore, for p> 0
Then, ∠C = 180° − ( ∠A + ∠B )
∴ C = 30° ( a + b )1 / p > c1 / p ...(i)
⇒ ∠C = 180° − (30° + 45° ) = 105°
sin 30° sin 105° 52. (a) Here, a = 1 + 3, b = 2, ∠C = 60° Now, for any two positive numbers
By sine rule, = and p > 1
BC 3+1 a + b −c
2 2 2
∴ cosC = ( a )1 / p + ( b )1 / p > ( a + b )1 / p
 2 2  1 ...(ii)
⇒ BC = ( 3 + 1) ×  × = 2 2ab
 3 + 1 2 ( 1 + 3 )2 + ( 2)2 − c 2 From Eqs. (i) and (ii), we get
B ⇒ cos 60° =
2( 1 + 3 ) ⋅ 2 ( a )1 / p + ( b )1 / p > ( c )1 / p
45° (√3 + 1) ⇒ ( a )1 / p + ( b )1 / p − ( c )1 / p > 0
⇒ 2 + 2 3 = 4 + 2 3 + 4 − c2
⇒ c2 = 8 − 2 = 6 ⇒ c = 6 ⇒ a1 / p + b1 / p > c1 / p
105°
30° b 2 + c2 − a 2
A ∴ cos A =
C 2bc
sin 45° sin 105°
Again, by sine rule, =
AC 3+1
16
178 CDS Pathfinder

COORDINATE SYSTEM AND


STRAIGHT LINES
In NDA exam, generally 5-7 questions are asked from this chapter which are based on section
formula, area of triangle, equation of straight line in different conditions, angle between two
lines, distance of a point from a line, etc.

COORDINATE SYSTEM
A system in which points are defined as the ordered pair ( x, y) is called the coordinate system. Here, x
and y are called abscissa and ordinate, respectively.

Cartesian Coordinates
In coordinate system, there are two mutually perpendicular straight lines
XOX ′ and YOY ′, called X and Y-axes, respectively.
These axes divide the coordinate plane into four parts called quadrants. OX
and OX ′ are called positive and negative directions, respectively of X-axis
and similarly OY and OY ′ are called positive and negative directions,
respectively of Y-axis.

Distance Formula
The distance between any two points in the plane is the length of the line segment joining them.

Let P ( x1 , y1 ) and Q ( x 2 , y 2 ) be any two points in the XY-plane, then

| PQ | = ( x 2 − x1 ) 2 + ( y 2 − y1 ) 2

or | PQ | = (Difference of abscissas) 2 + (Difference of ordinates) 2

Note Distance between the origin O(0, 0) and the point P ( x , y) is OP = x 2 + y2 .


MATHEMATICS > Coordinate System and Straight Lines 179

EXAMPLE 1. Find a point P, which is equidistant from Sol. Let A = (1, − 2), B = ( 3, 6), C = ( 5, 10) and D = ( 3, 2)
the three points A (0, 1), B (1, 0) and C (4, 3). Now, using distance formula
a. (1, 2) b. ( 2, 2)
AB = (1 − 3) 2 + ( − 2 − 6) 2 = 2 17
c. ( 2, 3) d. ( 3, 3)
BC = ( 3 − 5) 2 + (6 − 10) 2 = 2 5
Sol. . Let P ( a, b) be the required point.
CD = ( 5 − 3) 2 + (10 − 2) 2 = 2 17
Then, PA = PB = PC or PA2 = PB 2 = PC 2.
Now, PA2 = PB 2 ⇒ a2 + ( b − 1) 2 = ( a − 1) 2 + b2 AD = (1 − 3) 2 + ( − 2 − 2) 2 = 2 5
⇒ a2 + b2 − 2b + 1 = a2 − 2a + 1+ b2 ⇒ b = a
AC = (1 − 5) 2 + ( − 2 − 10) 2 = 4 10
and PA2 = PC 2
⇒ a + ( b − 1) 2 = ( a − 4) 2 + ( b − 3) 2
2 and BD = ( 3 − 3) 2 + (6 − 2) 2 = 4
⇒ a + b2 − 2b + 1 = a2 − 8a + 16 + b2 − 6b + 9
2
Clearly, we have
⇒ 8a + 4b − 24 = 0 AB = CD, BC = AD and AC ≠ BD
On substituting b = a, we get 12a − 24 = 0 Thus, ABCD is a parallelogram.
⇒ a=2
Hence, the required point is P ( 2, 2). Section Formulae
Let A ( x1 , y1 ) and B ( x 2 , y 2 ) be the two given points
Applications of Distance Formula
and the point P ( x, y) divide the line joining the points
1. Collinear Points A and B in the ratio m : n, then
Three points in α − y plane will be collinear, if sum of
distances between two pairs of points is equal to
third pair of points.
2. Three Points In A Plane
Any three non-collinear points always form a
(i) Point of internal division
triangle. Given three points would form a
 mx 2 + nx1 my 2 + ny1 
(i) Isosceles triangle If lengths of any two sides ( x, y) =  , 
 m+ n m+ n 
are equal.
(ii) Equilateral triangle If lengths of all three (ii) Point of external division
sides are equal.  mx 2 − nx1 my 2 − ny1 
( x, y) =  , 
(iii) Right angled triangle If sum of squares of  m− n m− n 
lengths of two sides is equal to length of third
side.
3. Four points in a plane
Any four non-collinear points form a quadrilateral.
Given four points would form
x + x 2 y1 + y2 
(i) Square If all four sides are equal and diagonals Note If P is the mid-point of AB, then P( x , y) =  1 , .
 2 2 
are also equal.
(ii) Rhombus If all four sides are equal. EXAMPLE 3. The coordinates of the point dividing
(iii) Rectangle If opposite sides are equal and internally the lines joining the points (4, − 2) and (8, 6)
diagonals are equal. in the ratio 7 : 5 will be
c.  ,  d.  , 
19 8 8 19
(iv) Parallelogram If opposite sides are equal. a. (16, 18) b. (18, 16)
 3 3 3 3 
Note In a parallelogram, diagonals cut each other at mid-point.
l
Sol. c. Let the coordinates of the required point be ( x, y).
l In a rhombus, diagonals bisect each other at right angles.
4 × 5+ 8 ×7 19 − 2× 5+ 6 ×7 8
Then, x = = and y = =
EXAMPLE 2. Figure formed from the four 7+ 5 3 7+ 5 3
coordinates (1, − 2), (3, 6), (5, 10) and (3, 2) is  19 8
∴ Required coordinates =  , 
a. Square b. Rectangle  3 3
c. Rhombus d. Parallelogram
180 NDA/NA Pathfinder

Area of a Triangle Incentre of a Triangle


Let A ( x1 , y1 ), B ( x 2 , y 2 ) and C ( x 3 , y 3 ) be the vertices of If (x, y) is the incentre of the ∆ABC, where A ( x1 , y1 ),
a ∆ ABC, then
B ( x 2 , y 2 ), C ( x 3 , y 3 ) are coordinates and a, b, c are the sides
x1 y1 1
1  ax + bx 2 + cx 3 ay1 + by 2 + cy 3 
Area of ∆ ABC = x 2 y 2 1 of the triangle, then  1 , 
2  a+ b+ c a+ b+ c 
x3 y3 1
1 will be incentre of a triangle.
∆ = | x1 ( y 2 − y 3 ) + x 2 ( y 3 − y1 ) + x 3 ( y1 − y 2 ) |
2 Circumcentre of a Triangle
Note The three points A ( x 1, y1 ), B ( x 2, y2 ) and C ( x 3, y3 ) are collinear,
If P ( x, y) is the circumcentre of ∆ABC, then
if the area of ∆ABC is zero. PA = PB = PC …(i)
We can determine the coordinates of P by simplifying
EXAMPLE 4. For what value of k are the points Eq. (i) for x and y.
(k , 2 − 2k ), (− k + 1, 2k ) and (− 4 − k , 6 − 2k ) are collinear?
1 1 Note The circumcentre of a right angled triangle is the mid-point of
a. b. 1 c. or − 1 d. − 1 its hypotenuse.
2 2
Sol. c. Let three given points be A = ( x1, y1) = ( k , 2 − 2k) Orthocentre of a Triangle
B = ( x 2 , y 2) = ( − k + 1, 2k) The coordinates of the orthocentre of a triangle,
and C = ( x 3 , y 3) = ( −4 − k , 6 − 2k). whose vertices are A ( x1 , y1 ), B ( x 2 , y 2 ) and C ( x 3 , y 3 )
If the given points are collinear, then ∆ = 0. is given by
⇒ x1( y 2 − y 3 ) + x 2 ( y 3 − y1) + x 3 ( y1− y 2 ) = 0  x1 tan A + x 2 tan B + x 3 tan C
 ,
⇒ k(2k − 6 + 2k) + ( − k + 1)(6 − 2k − 2 + 2k)  tan A + tan B + tan C
+ ( − 4 − k)(2 − 2k − 2k) = 0 y1 tan A + y 2 tan B + y 3 tan C 
⇒ k(4 k − 6) − 4( k − 1) + (4 + k)(4 k − 2) = 0 
tan A + tan B + tan C 
⇒ 4 k 2 − 6 k − 4 k + 4 + 4 k 2 + 14 k − 8 = 0
⇒ 8k 2 + 4 k − 4 = 0 ⇒ 2k 2 + k − 1 = 0
SOME IMPORTANT POINTS
⇒ (2k − 1)( k + 1) = 0
1 (i) Orthocentre of the right angled ∆ABC , right angled at A,
⇒ k = or − 1 is A.
2
1 (ii) Orthocentre of a triangle will lie inside, if triangle is
Hence, the given points are collinear for k = or k = − 1. acute and in case of obtuse triangle, orthocentre will lie
2
outside.
Area of Quadrilateral (iii) In a ∆ABC , the circumcentre S, centroid G and
orthocentre O are collinear and SG : GO = 1 : 2
Area of quadrilateral ABCD with vertices A ( x1 , y1 ),
(iv) In equilateral triangle, orthocentre, centroid, circumcentre
B( x 2 , y 2 ), C( x 3 , y 3 ) and D( x 4 , y 4 ) taken in order is and incentre coincide.
1 x1 − x 3 y1 − y 3
Area =
2 x2 − x4 y2 − y4 Excentre of Triangle
There are three excentres in a triangle. Coordinate of
DIFFERENT POINTS RELATED each can be obtained by changing the sign of a, b, c
respectively in the formula of incentre as given by I1 , I 2
TO TRIANGLE and I 3 . The coordinates of I1 are given by
Centroid of a Triangle  − ax1 + bx 2 + cx 3 − ay1 + by 2 + cy 3 
The coordinates of the centroid G of a triangle, whose  , 
 − a+ b+ c − a+ b+ c 
vertices are A ( x1 , y1 ), B ( x 2 , y 2 ) and C ( x 3 , y 3 ) is given
 x + x 2 + x 3 y1 + y 2 + y 3  EXAMPLE 5. The vertices of a triangle are
by  1 , . A (5, − 1), B (−1, 5) and C (6, 6). Find the coordinates of
 3 3 
the circumcentre.
 10 10   23 23  28 28 
Note If D, E and F are the mid-points of sides BC, CA and AB of a a.  ,  b.  ,  c.  ,  d. None of these
∆ABC, then centroid of ∆ABC = centroid of ∆DEF.  3 3  8 8  3 3
MATHEMATICS > Coordinate System and Straight Lines 181

Sol. . Let P ( x, y ) be the circumcentre of the ∆ABC. Slopes in Different Conditions


Then, PA = PB = PC or PA = PB = PC
2 2 2
(i) Slope of X-axis or a line parallel to X-axis is
Now, PA2 = PB 2 , m = tan 0 ° = 0
we get ( x − 5) 2 + ( y + 1) 2 = ( x + 1) 2 + ( y − 5) 2
(ii) Slope of Y-axis or a line parallel to Y-axis is
⇒ x + y 2 − 10x + 2y + 26 = x 2 + y 2 + 2x − 10y + 26
2
π
⇒ x − y =0 …(i) m = tan = ∞
Now, PB 2 = PC 2, we get 2
( x + 1) 2 + ( y − 5) 2 = ( x − 6) 2 + ( y − 6) 2 (iii) The slope of the line joining the points ( x1 , y1 ) and
⇒ x + y 2 + 2x − 10y + 26 = x 2 + y 2 − 12x − 12y + 72
2 ( x 2 , y 2 ) is obtained as
y − y1 Difference of ordinates
⇒ 7x + y − 23 = 0 …(ii) m= 2 =
On solving Eqs. (i) and (ii), we get x = 23 / 8 and y = 23 / 8 x 2 − x1 Difference of abscissas
Hence, the coordinates of the circumcentre are (23/8, 23/8).
Angle between Two Lines
LOCUS OF A POINT Let θ be the angle between two lines of slopes m1 and
m − m1
The curve described by a point which moves under m2 , then tan θ = ± 2
1 + m1 m2
given condition or conditions is called its locus. The
equation of the locus of a point is the relation which is Above lines are parallel, if m1 = m2
satisfied by the coordinates of every point on the locus and lines are perpendicular, if m1 m2 = −1
of the point.
Some Facts about General
EXAMPLE 6. The sum of the squares of the distances Equation of Line ax + by + c = 0
of a moving point from two fixed points (p, 0) and
(i) The slope of the line ax + by + c = 0 is
(−p, 0) is equal to a constant quantity 2λ 2 , find the
a Coefficient of x
equation of the locus. m= − = −
b Coefficient of y
a. x2 + y2 = λ2 − p2 b. x2 + y2 = λ2 + p2
c. x2 − y2 = λ2 − p2 d. x2 − y2 = λ2 + p2 (ii) Let two straight lines be given by
a1 x + b1 y + c1 = 0 and a 2 x + b2 y + c 2 = 0
Sol. . Let P (h, k) be any position of the moving point and
Then, the lines are
A ( p, 0), B ( − p, 0) be the given points.
a1 b1 c1
Then, PA2 + PB 2 = 2λ2 (a) coincident or identical, if = =
⇒ (h − p) + ( k − 0) + (h + p) + ( k − 0) 2 = 2λ2
2 2 2 a 2 b2 c 2
⇒ h − 2ph + p2 + k 2 + h2 + 2ph + p2 + k 2 = 2λ2
2
(b) parallel, if their slopes are equal, i.e.
⇒ 2h2 + 2 k 2 + 2p2 = 2λ2 a a a1 a 2
m1 = m2 or − 1 = − 2 or =
⇒ h2 + k 2 = λ2 − p2 b1 b2 b1 b2
Hence, the locus of (h, k) is x + y = λ2 − p2.
2 2
(c) perpendicular, if the product of their slopes is
−1, i.e. m1 m2 = − 1
STRAIGHT LINES  a  a 
A straight line is the locus of all those points which are or  − 1   − 2  = − 1 or a1 a 2 + b1 b2 = 0
collinear with two given points.  b1   b2 
The equation of a straight line l is a linear relationship a1 b1
(d) intersecting, if ≠
in x and y, i.e. it is a first degree polynomial equation in a 2 b2
x and y. An equation of the form ax + by + c = 0, where (iii) The equation of a line which is
a, b and c are constants, represents a straight line in (a) parallel to ax + by + c = 0 can be written as
general form.
ax + by + k = 0 [Q m1 = m2 ]
Slope or Gradient of a Line (b) perpendicular to ax + by + c = 0 can be written
The trigonometrical tangent of the angle α (α ≠ π / 2) as bx − ay + k = 0 [Q m1 m2 = − 1]
which a given line makes with the positive direction of where, k is an arbitrary constant to be
X-axis, is called the slope or gradient of the line. It is determined using given condition.
generally denoted by m, i.e. m = tan α.
182 NDA/NA Pathfinder

x y
Equations of Line in Different Forms Sol. Let the equation of the line be
a b
+ =1 …(i)
1. Slope-intercept form y = mx + c, is called the slope 3 2
This passes through (3, 2), therefore + = 1 …(ii)
form or slope intercept form or the tangent form a b
of the equation of the line, the constant c is called It is given that a + b = 12 or b = 12 − a
the intercept made by the line on the Y-axis. To find Putting b = 12 − a in Eq. (ii), we get
the slope and the intercept on theY-axis of a line given 3 2
+ = 1 ⇒ a2 − 13a + 36 = 0 ⇒ a = 9, 4
in the form ax + by + c = 0, we write it in the form a 12 − a
a c For a = 9, b = 12 − 9 = 3 For a = 4, b = 12 − 4 = 8
y=− x− Putting the values of a and b in Eq. (ii), we get the
b b x y x y
equations of the lines + = 1 and + = 1
Therefore, the slope of the line is m = − a / b and the 9 3 4 8
intercept of the line on the Y-axis is − c / b. i.e. x + 3y = 9 and 2x + y = 8
2. Slope-point form The equation of a line passing 5. Normal or perpendicular form If p is the normal
through the point ( x1 , y1 ) and having the slope ‘m’ is from the origin to the line AB and if the normal
makes an angle α with Y
y − y1 = m ( x − x1 )
X-axis, then the equation of A
the line in the form,
EXAMPLE 7. What will be the equation of a line that p
has y-intercept 4 and is perpendicular to the line x cos α + y sin α = p is called a
X′ B X
joining (2, − 3) and (4, 2)? the normal form of the line. O
Y′
a. 2x + 5y − 20 = 0 b. 5x + 2y − 20 = 0
Note In the normal form of a straight line the sum of the squares
c. 5x − 2y + 20 = 0 d. 2x − 5y + 20 = 0 of the coefficient of x and y is equal to one.

Sol. . Let m be the slope of the required line.


Since, the required line is perpendicular to the line
FAMILY OF LINES
joining A( 2, − 3) and B( 4, 2). Let L1 = a1 x + b1 y + c1 = 0 and L2 = a 2 x + b2 y + c 2 = 0 be
2+ 3 two given lines. Then, the equation, L1 + λL2 = 0
Therefore, m × slope of AB = − 1⇒ m × = −1
4−2
i.e. a1 x + b1 y + c1 + λ ( a 2 x + b2 y + c 2 ) = 0 …(i)
−2
⇒ m= represents the equation of a line, passing through the
5
The required line cuts-off an intercept 4 on Y-axis. So, c = 4. point of intersection of the given lines, where λ ∈ R.
Hence, the equation of the required line is EXAMPLE 9. Find the equation of the line joining
2
y = − x + 4 ⇒ 2x + 5y − 20 = 0 the points (3, − 1) and (2, 3). Also, find the equation of
5 another line perpendicular to this line and passing
3. Two points form The equation of a line passing through the point (5, 2).
through the points ( x1 , y1 ) and ( x 2 , y 2 ) is a. 4 x + y − 11 = 0 , x − 4 y + 3 = 0
y − y1 b. 4 x + y + 11 = 0 , x − 4 y + 3 = 0
( y − y1 ) = 2 ( x − x1 )
x 2 − x1 c. x + 4 y − 11 = 0 , 4 x − y + 3 = 0
4. Intercept form If a line cuts
Y d. x − 4 y + 11 = 0 , 4 x − y + 3 = 0
x
the intercepts a and b on the a+ y
b= Sol. Equation of line passing through ( 3, − 1) and ( 2, 3).
b 1
axes, then the equation of 3+ 1
line is y + 1= ( x − 3)
X′
O a X 2− 3
x y
+ =1 Y′ ⇒ y + 1 = − 4 ( x − 3)
a b
⇒ 4x + y − 11 = 0
EXAMPLE 8. What will be the equation of the line which is the required equation.
passes through the point (3, 2) and the sum of its Equation of line perpendicular to it x − 4y + k = 0
intercepts on the axes is 12? But it passes through the point ( 5, 2). So, it will satisfy this
I. 2x + y = 8 II. x + 3y = 9 equation.
a. Only I b. Only II ∴ 5 − 4 ( 2) + k = 0 ⇒ k = 3
c. Both I and II d. None of these Hence, the another required equation is x − 4y + 3 = 0.
MATHEMATICS > Coordinate System and Straight Lines 183

EXAMPLE 10. Find the equation of the line which Perpendicular Distance of
passes through the point of intersection of the lines a Point from a Line
x − 2y + 3 = 0 and 2x + y − 4 = 0 and
The perpendicular distance d from P ( x1 , y1 ) to the line
(i) has slope 2.
a. x + y + 2 = 0 b. 2 x − y = 0 ax1 + by1 + c
ax + by + c = 0 is given by d =
c. 2 x + y = 0 d. x + 2 y + 2 = 0 a 2 + b2
(i) is parallel to the line 3x + 5y − 1 = 0. The perpendicular distance from the origin to the line
a. 3x + 5 y + 1 = 0 b. 3x + y + 13 = 0 c
c. 3x + 5 y + 13 = 0 d. None of these ax + by + c = 0 is d =
a + b2
2
Sol. (i) → b (ii) → d
The equation of the line which passes through the point EXAMPLE 11. The intercepts of a straight line upon
of intersection of the lines the coordinate axes are a and b. If the length of the
x − 2y + 3 = 0 and 2x + y − 4 = 0 perpendicular on this line from the origin be 1, then
can be written as ( x − 2y + 3) + λ ( 2x + y − 4) = 0 which one of the following relations is correct?
⇒ (1 + 2λ ) x + ( −2 + λ ) y + 3 − 4λ = 0 …(A) a.
1
+
1
=2 b.
1
+
1
=
1
where λ is an arbitrary constant to be determined. a2 b2 a2 b2 2
− (1+ 2λ ) 1 1 1 1 1
c. + = d. + =1
(i) Slope of the line (A) is m = =2 a2 b2 2 a2 b2
−2 + λ
⇒ −1 − 2λ = − 4 + 2λ ⇒ 4λ = 3 ⇒ λ = 3/ 4 Sol. The equation of line passing through A and B is
x y x y Y
The equation of the required line becomes + = 1 ⇒ + − 1 = 0 ...(i)
 3  3 a b a b B (0, b)
 1 +  x +  −2 +  y + 3 − 3 = 0
 2  4 Length of perpendicular from
5 5 (0, 0) to the Eq. (i) b
⇒ x − y = 0 ⇒ 2x − y = 0 1 A (a, 0)
2 4 0 0 X′ X
+ −1 O a
(ii) The line (A) is parallel to the line 3x + 5y − 1 = 0 a b
= Y′
Therefore, their slopes are equal. 1 1
2 + 2
− (1 + 2λ ) 3 a b
Hence, = − ⇒ 5 + 10λ = − 6 + 3λ 1 1 1
−2 + λ 5 Given that, 1 = ⇒ 2 + 2 =1
1 1 a b
⇒ 7λ = − 11 ⇒
λ = − 11/ 7 +
a2 b2
The equation of the
line becomes
 22
1 −  x +
 11
 −2 −  y + 3 +
44
=0 Distance between Two Parallel Lines
 7  7 7 The distance between the parallel lines ax + by + c1 = 0
15 25 65 c1 − c 2
⇒ − x− y+ = 0 ⇒ 3x + 5y − 13 = 0
7 7 7 and ax + by + c 2 = 0 is given by d =
a 2 + b2
Concurrency of Three Lines
To find, whether three lines are concurrent or not, we Image of a Point about a
find the point of intersection of any two lines. If this Straight Line or Mirror
point lies on the third line, i.e. the coordinates of this
Let AB be a given line and P be any point in the plane.
point of intersection satisfy the equation of the third
line, then the given straight lines are concurrent. Draw PM perpendicular to AB. Extend PM to Q such
that PM = MQ. P(x1, y1)
The condition that the three lines B
a1 x + b1 y + c1 = 0 …(i) Then, the point Q is called the image
a 2 x + b2 y + c 2 = 0 …(ii) of the point P about the straight line M
and a 3 x + b3 y + c 3 = 0 …(iii) AB, where M is the mid-point of PQ.
A
a1 b1 c1 The image (or reflection) of the point Q
are concurrent, is a 2 b2 c 2 = 0 P( x1 , y1 ) in the given line AB,
a3 b3 c3 ax + by + c = 0, is ( h, k), where
h − x1 k − y1 −2( ax1 + by1 + c )
Note Three or more straight lines are said to be concurrent lines, if = =
they meet at a point.
a b a 2 + b2
184 NDA/NA Pathfinder

Note The foot of the perpendicular ( h, k ) from ( x1, y1 ) to the line a1 x + b1 y + c1 a 2 x + b2 y + c 2


ax + by + c = 0 is given by =±
h − x1 k − y1 −( ax1 + by1 + c ) a12 + b12 a 22 + b22
= =
a b a2 + b 2
Conditions Acute angle bisector Obtuse angle bisector
a1a2 + b1b 2 > 0 − +
EXAMPLE 12. Find the image of the point P (2, 1)
about the straight line x − y + 5 = 0. a1a2 + b1b 2 < 0 + −

a. ( − 4, 7 ) b. (4, 7 ) c. (4, − 7 ) d. (− 4, − 7 )
Note A line which is equally inclined to given two lines is parallel
Sol. The slope of the given line is 1. Therefore, slope of to the angle bisectors of the given lines.
the perpendicular line from P to the line is −1. Hence,
the equation of the line PM, passing through P ( 2, 1) and EXAMPLE 13. Find the equations of the bisector of
having slope −1, is the angles between the straight lines 3x + 4y + 7 = 0
y − 1 = − 1( x − 2) or x + y − 3 = 0
and 12x + 5y − 8 = 0.
a. 7 x + 9 y + 17 = 0, 99x + 77 y + 51 = 0
On solving the equations, x − y + 5 = 0 and x + y − 3 = 0,
we obtain the coordinates of the point M as ( −1, 4). b. 7 x − 9 y − 17 = 0, 99x + 77 y − 51 = 0
Let the coordinates of the point Q be (h, k). Since, Q is c. 7 x − 9 y + 17 = 0, 99x + 77 y + 51 = 0
the image of P, M is the mid-point of PQ. d. None of the above
We have, (h + 2) / 2 = −1and ( k + 1) / 2 = 4, Sol. Equations of the bisectors of the angles between the
⇒ h = − 4 and k = 7 given straight lines are given by
Therefore, ( −4, 7) is the image point of ( 2, 1) about the line 3x + 4y + 7 12x + 5y − 8

x − y + 5 = 0. 9 + 16 144 + 25
⇒ 13 ( 3x + 4y + 7) = ± 5 (12x + 5y − 8)
Equations of the Bisectors of
⇒ 39x + 52y + 91 = ± (60x + 25y − 40)
Angles between Two Lines Taking positive signs, 39x + 52y + 91 = 60x + 25y − 40
Let AB and CD be two given lines represented by ⇒ − 21x + 27y + 131 = 0
a1 x + b1 y + c1 = 0 and a 2 x + b2 y + c 2 = 0 ⇒ 21x − 27y − 131 = 0
Then, equation of the bisector of the angle between them Taking negative signs, ( 39x + 52y + 91) = − (60x + 25y − 40)
are ⇒ 99x + 77y + 51 = 0

PRACTICE EXERCISE
1. The coordinates of the middle points of the sides 4. If t1 ≠ t2 and the points A( a , 0), B( at12 , 2at1 ) and
of a triangle are (4, 2), (3, 3) and (2, 2), then find C( at22 , 2at2 ) are collinear, then t1t2 is equal to
the coordinates of its centroid are
(a) 1 (b) 2 (c) − 1 (d) − 2
(a)  3, 
7
(b) ( 3, 3) 5. The area of quadrilateral ABCD whose vertices in
 3
(c) (4, 3) (d) None of these order are A(1, 1), B( 7, − 3), C(12, 2) and D( 7, 21) is
(a) 66 sq units (b) 132 sq units
2. The co-ordinates of incentre of ∆ABC with (c) 124 sq units (d) 86.5 sq units
vertices A( 0, 6), B( 8, 12) and C( 8, 0) is
6. The distance between the lines 4x + 3 y = 11 and
(a)  , 0
16
(b) (8, 11) (c) (− 4, 3) (d) (5, 6) 8x + 6 y = 15 is
 3 
(a) 7/2 (b) 7/3 (c) 7/5 (d) 7/10
3. The middle point of the segment of the straight
line joining the points ( p, q ) and ( q , − p) is 7. The equation of line parallel to the line
(r/ 2, s / 2). What is the length of the segment? 2x + 3 y + 5 = 0 and sum of whose intercepts on
the axes is 15 is
(a) [(s 2 + r 2 )1 / 2 ]/ 2 (b) [(s 2 + r 2 )1 / 2 ]/ 4
(a) 2 x + 3 y = 15 (b) 3x − 2 y + 2 = 0
(c) (s 2 + r 2 )1 / 2 (d) s + r (c) 3x − 2 y + 8 = 0 (d) 2 x + 3 y − 18 = 0
MATHEMATICS > Coordinate System and Straight Lines 185

8. The line x + y = 4 divides the line joining the 19. If ( a cos θ1 , a sin θ1), ( a cos θ2 , a sin θ2 ) and
points ( −1, 1) and ( 5, 7) in the ratio ( a cos θ3 , a sin θ3 ) represents the vertices of an
(a) 2 : 3 (b) 1 : 2 (c) 1 : 1 (d) 4 : 3 equilateral triangle inscribed in a circle, then
consider the following statements.
9. Foot of perpendicular drawn from (0, 5) to the
line 3x − 4 y − 5 = 0 is I. cos θ1 + cos θ 2 + cos θ3 = 0
(a) (1, 3) (b) (2, 3) (c) (3, 2) (d) (3, 1) II. sin θ1 + sin θ 2 + sin θ3 = 0
Which of the above statement(s) is/are correct?
10. A point P ( h , k) lies on the straight line (a) Only I (b) Only I
x + y + 1 = 0 and is at a distance 5 units from the (c) Both I and II (d) Neither I nor II
origin. If k is negative, then h is equal to 20. Consider the following statements
(a) −3 (b) 3 (c) −4 (d) 4
I. The area of equilateral triangle is a rational
11. The area of a triangle is 5 and two of its vertices number.
are A ( 2, 1), B ( 3, − 2). Then, the third vertex, in II. In a triangle, the centroid divides the joint of
Ist quadrant which lies on the line y = x + 3 is circumcentre and orthocentre in the ratio 1 : 2.
(a)  ,  (b)  ,  (c)  , 
7 13 5 5 3 3 III. Circumcentre is the only point which is
(d) (0, 0)
2 2  2 2 2 2 equidistant from all the three vertex of a
12. The diagonals of a quadrilateral ABCD are along triangle.
the lines x + 3 y = 4 and 6x − 2 y = 7. Then, ABCD Which of the above statement(s) is/are correct?
must be a (a) I and III are correct (b) I, II and III are correct
(a) rectangle (b) parallelogram (c) II and III are correct (d) I and II are correct
(c) cyclic quadrilateral (d) rhombus
Directions (Q. Nos. 21-22) The sides of a rectangle
13. The equation of straight line passing through the are represented by lines x = a, x = a ′, y = b and y = b ′.
point of intersection of the straight line
3x − y + 2 = 0 and 5x − 2 y + 7 = 0 and having
21. The point of intersection of the diagonals of the
rectangle is
infinite slope is
a + a′ b + b ′  a + b a′ + b ′ 
(a) x = 2 (b) x + y = 3 (c) x = 3 (d) x = 4 (a)  ,  (b)  , 
 2 2   2 2 
14. If ( −5, 4) divides the line segment between the (c) 
a + a′ a′ + b 
,  (d) None of these
coordinate axes in the ratio 1 : 2, then what is its  2 2 
equation? 22. Area of the rectangle is
(a) 8x + 5 y + 20 = 0 (b) 5x + 8 y − 7 = 0 (a) a − b a′ − b ′ (b) b ′ − b a′ − a
(c) 8x − 5 y + 60 = 0 (d) 5x − 8 y + 57 = 0 (c) a − b ′ a′ − b (d) None of these
15. What is the locus of a point which is equidistant
from the point ( m + n , n − m ) and the point Directions (Q. Nos. 23-24) Let ABCD be a
( m − n , n + m )? parallelogram whose equations for the consecutive
(a) mx = ny (b) nx = − my (c) nx = my (d) mx = − ny sides AB and AD are 4x + 5y = 0 and 7x + 2y = 0. The
equation of one of the diagonal is 11x + 7y = 9.
16. If x cos θ + y sin θ = 2 is perpendicular to the line
x − y = 3, then what is one of the value of θ? 23. The equation of the other diagonal is
(a) π/6 (b) π/4 (c) π/2 (d) π/3 (a) x + y = 0 (b) x − y = 0
(c) 2 x + y = 0 (d) x − 3 y = 0
17. The two points ( 2, 1) and ( 3, − 1) with respect to 24. Area of parallelogram ABCD is
the line 3x − 5 y + 7 = 0 7
(a) 8 sq units (b) sq units (c) 3 sq units (d) 6 sq units
(a) on the line (b) on same side of the line 2
(c) on opposite side of the line (d) None of these
18. ABC is a triangle formed by the lines xy = 0 and Directions (Q. Nos. 25-26) Consider the line segment
x + y = 1. Consider the following statements L : x sec θ + y tan θ = p.
I. Orthocentre of ∆ABC is at the origin. 25. The locus of mid-point of the portion of the line L
II. Circumcentre of ∆ABC is at the point intercepted between the axes is
(1 / 2, 1 / 2). p2 p2 p2 p2
(a) 2
− 2
=1 (b) 2
+ =1
4x 4y 4x 4 y2
Which of the above statement(s) is/are correct?
p2 p2
(a) Only I (b) Only II (c) − =1 (d) None of these
(c) Both I and II (d) None of these 2 x2 2 y2
186 NDA/NA Pathfinder

26. The equation of line perpendicular to line L and 36. Area of ∆PQR : Area of ∆OAB is
passing through ( p cot θ, 2 p cos3 θ) is (a) 2 : 3 (b) 2 : 9 (c) 3 : 4 (d) 1 : 3
(a) x tan θ − y sec θ + p cos 2θ = 0
(b) x tan θ − y sec θ − p cos 2 θ = 0
37. If S is the mid-point of PR, then QS is equal to
1 2 3
(c) x tan θ + y sec θ + p sin2 θ = 0 (a) OQ (b) AB (c) OQ (d) AB
2 3 2
(d) None of the above
38. Consider the following statements
Directions (Q. Nos. 27-28) The equation of an  5 13
altitude of an equilateral triangle is 3x + y = 2 3 I. Centriod of ∆PQR is  ,  .
 18 9 
and one of the vertices is (3, 3). II. ∆OAP is right angled triangle.
27. The possible number of triangles is Which of the above statement(s) is/are correct?
(a) 1 (b) 2 (c) 3 (d) 4 (a) Only I (b) Only II
(c) Both I and II (d) Neither I nor II
28. The area of equilateral triangle is (in sq units)
(a) 3 3 (b) 3
(c) 6 3 (d) 2 3
Directions (Q. Nos. 39-43) Let L be the line
belonging to the family of the straight lines
( a + 2 b) x + ( a − 3b)y + a − 8 b = 0, a, b ∈ R which is
Directions (Q. Nos. 29-31) The lines
farthest from the point (2, 2).
L1 : 4x − 3y + 7 = 0 and L2 : 3x − 4y + 14 = 0, intersect
the line L3 : x + y = 0 at P and Q, respectively. The 39. The equation of line L is
bisectors of the acute angle between L1 and L2 (a) x + 4 y + 7 = 0 (b) 2 x + 3 y + 4 = 0
intersect L3 at R. (c) 4x − y − 6 = 0 (d) 3x − 2 y + 7 = 0
29. The equation of the bisector of acute angle is 40. Area formed by the line L with coordinate axes
(a) x + y + 3 = 0 (b) x − y − 3 = 0 is
(c) x − y + 3 = 0 (d) 3x − y − 7 = 3 (a)
4
sq units (b)
9
sq units
3 2
30. The ratio PR : RQ equals to 49 5
(c) sq units (d) sq units
(a) 2 2 : 5 (b) 2 : 1 (c) 1 : 1 (d) 5 : 2 8 6
31. Area of triangle formed by lines L1 , L2 and L3 is 41. If L is concurrent with the lines x − 2 y + 1 = 0
(a)
13 7 9
sq units (b) sq units (c) sq units (d) 8 sq units and 3x − 4 y + λ = 0, then the value of λ is
2 2 2 (a) 2 (b) 1 (c) − 4 (d) 5

Directions (Q. Nos. 32-34) Given two points 42. The image of the point (2, 2) in the line L is
A ( −2, 0) and B (0, 4), M is a point with coordinates (a) (1, − 2 ) (b) (0, − 6) (c) (2, − 4) (d) (4, − 7 )
( x, x), x ≥ 0. P divides the joining of A and B in the 43. The distance of point ( 2, 2) from the line L is
ratio 2 : 1. C and D are the mid-points of BM and (a) 5 units (b) 17 units
AM, respectively. (c) 19 units (d) None of these
32. Area of the ∆AMB is minimum, if the coordinates
of M are
(a) (1, 1) (b) (0, 0) (c) (2, 2) (d) (3, 3)
PREVIOUS YEARS’ QUESTIONS
33. Ratio of the areas of the ∆’s APM and BPM is 44. What is the distance between the lines
(a) 2 : 1 (b) 1 : 2 (c) 2 : 3 (d) 1 : 3 3x + 4 y = 9 and 6x + 8 y = 18 ? e 2012 I
(a) 0 (b) 3 units (c) 9 units (d) 18 units
34. Perimeter of the quadrilateral ABCD is
(a) 2 5 (b) 3 + 3 5 (c) 2 + 5 (d) None of these 45. What is the equation of line passing through
( 0, 1) and making an angle with the Y -axis equal
Directions (Q. Nos. 35-38) O(0, 0), A (1, 1,) B(0, 3) are to the inclination of the line x − y = 4 with
the vertices of a ∆OAB. P divides OB in the ratio X-axis? e 2012 I
1: 2, θ is the mid-point of AP, R divides AB in the (a) y = x + 1 (b) x = y + 1
ratio 2 : 1. (c) 2 x = y + 2 (d) None of these

35. If α(QR )2 = β( PR )2, then 46. What is the perpendicular distance of the point
( x , y ) from X-axis? e 2012 I
α 68 α 17
(a) = (b) = (c) α = β (d) α = 3β (a) x (b) y (c) |x| (d) | y|
β 65 β 19
MATHEMATICS > Coordinate System and Straight Lines 187

47. The equation of a straight line which makes 58. A straight line passes through the points ( 5, 0)
an angle 45° with the X-axis with y-intercept and ( 0, 3). The length of the perpendicular from
101 units is e 2012 II the point ( 4, 4) on the line is e 2013 I
(a) 10x + 101y = 1 (b) 101x + y = 1 17 17 15 17
(c) x + y − 101 = 0 (d) x − y + 101 = 0 (a) (b) (c) (d)
2 2 34 2
48. The line y = 0 divides the line joining the points 59. What is the equation of a straight line which
( 3, − 5) and ( −4, 7) in the ratio e 2012 II passes through ( 3, 4) and the sum of whose x and
(a) 3 : 4 (b) 4 : 5 (c) 5 : 7 (d) 7 : 9 y-intercepts is 14? e 2013 I
49. What is the value of λ , if the straight line (a) 4x + 3 y = 24 (b) x + y = 14
( 2x + 3 y + 4) + λ ( 6x − y + 12) = 0 is parallel to (c) 4x − 3 y = 0 (d) 3x + 4 y = 25
Y-axis? e 2012 II
(a) 3 (b) −6 (c) 4 (d) −3
60. What is the angle between the line x + y = 1 and
x − y = 1? e 2013 II
50. From the point ( 4, 3) a perpendicular is dropped π π π π
(a) (b) (c) (d)
on the X-axis as well as on the Y -axis. If the 6 4 3 2
lengths of perpendiculars are p and q 61. What is the equation of the straight line passing
respectively, then which one of the following is through ( 5, − 2) and ( −4, 7)? e 2013 II
correct? e 2012 II (a) 5x − 2 y = 4 (b) − 4x + 7 y = 9
(a) p = q (b) 3 p = 4q (c) 4 p = 3q (d) p + q = 5
(c) x + y = 3 (d) x − y = − 1
51. If p, q , r and s be the distances from origin of the 62. The equation of the line, the reciprocals of whose
points ( 2, 6), ( 3, 4), ( 4, 5) and ( −2, 5), respectively. intercepts on the axes are m and n, is given by
Which one of the following is a whole number? e 2013 II
(a) nx + my = mn (b) mx + ny = 1
e 2012 II
(c) mx + ny = mn (d) mx − ny = 1
(a) p (b) q (c) r (d) s
52. What is the perpendicular distance between the 63. The equation of the locus of a point which is
parallel lines 3x + 4 y = 9 and 9x + 12 y + 28 = 0? equidistant from the axes is e 2013 II
(a) y = 2x (b) x = 2 y (c) y = ± x (d) 2 y + x = 0
. e 2012 II
(a) 7/3 units (b) 8/3 units (c) 10/3 units (d) 11/3 units 64. A points P moves such that its distances from
53. The points ( 5, 1), (1, − 1) and (11, 4) are (1, 2) and ( − 2, 3) are equal. Then, the locus of P is
e 2012 II
(a) collinear e 2013 II
(a) straight line (b) parabola (c) ellipse (d) hyperbola
(b) vertices of right angled triangle
(c) vertices of equilateral triangle 65. What angle does the line segment joining ( 5, 2)
(d) vertices of an isosceles triangle and ( 6, − 15) subtend at ( 0, 0)? e 2013 II
π π π 3π
54. The equation to the locus of a point which is (a) (b) (c) (d)
6 4 2 4
always equidistant from the points (1, 0) and
( 0, − 2) is e 2012 II 66. The value of k for which the lines 2x + 3 y + a = 0
(a) 2 x + 4 y + 3 = 0 (b) 4x + 2 y + 3 = 0 and 5x + ky + a = 0 represent family of parallel
(c) 2 x + 4 y − 3 = 0 (d) 4x + 2 y − 3 = 0 lines is e 2013 II
55. The locus of a point equidistant from three (a) 3 (b) 4.5 (c) 7.5 (d) 15
collinear points is e 2012 II 67. What is the equation of the line which passes
(a) a straight line (b) a pair of points through ( 4, − 5) and is perpendicular to
(c) a point (d) the null set 3x + 4 y + 5 = 0? e 2013 II
(a) 4x − 3 y − 31 = 0 (b) 3x − 4 y − 41 = 0
56. Two straight line paths are represented by the
equations 2x − y = 2 and − 4x + 2 y = 6. Then, the (c) 4x + 3 y − 1 = 0 (d) 3x + 4 y + 8 = 0
paths will e 2013 I 68. For what value of k are the two straight lines
(a) cross each other at one point 3x + 4 y = 1 and 4x + 3 y + 2k = 0, equidistant from
(b) not cross each other the point (1, 1)? e 2013 II
(c) cross each other at two points (a) 1/2 (b) 2 (c) −2 (d) −1 / 2
(d) cross each other at infinitely many points
69. If the three vertices of the parallelogram ABCD
57. What is the inclination of the line 3x − y − 1 = 0? are A (1, a ), B ( 3, a ), C ( 2, b), then D is equal to?
e 2013 I e 2013 II
(a) 30° (b) 60° (c) 135° (d) 150° (a) (3, b ) (b) (0, b ) (c) (4, b ) (d) (5, b )
188 NDA/NA Pathfinder

70. What is the equation of the line through (1, 2), 81. A line passes through ( 2, 2) and is perpendicular
so that the segment of the line intercepted between to the line 3x + y = 3, its y-intercept is e 2015 I
the axes is bisected at this point? e 2014 I 3 4 1
(a) 2 x − y = 4 (b) 2 x − y + 4 = 0 (a) (b) (c) (d) 3
4 3 3
(c) 2 x + y = 4 (d) 2 x + y + 4 = 0
82. The area of a triangle, whose vertices are ( 3, 4),
71. What is the equation of straight line passing ( 5, 2) and the point of intersection of the lines
through the point (4, 3) and making equal
x = a and y = 5, is 3 square units. What is the
intercepts on the coordinate axes? e 2014 I
value of a? e 2015 I
(a) x + y =7 (b) 3x + 4 y =7 (c) x − y =1 (d) None of these
(a) 2 (b) 3 (c) 4 (d) 5
72. What is the equation of the line midway between 83. The length of perpendicular from the origin to a
the lines 3x − 4 y + 12 = 0 and 3x − 4 y = 6? e 2014 I
line is 5 units and the line makes an angle 120°
(a) 3x − 4 y − 9 = 0 (b) 3x − 4 y + 9 = 0
with the positive direction of X-axis. The
(c) 3x − 4 y − 3 = 0 (d) 3x − 4 y + 3 = 0
equation of the line is e 2015 I
73. Consider the following points (a) x+ 3 y = 5 (b) 3x+ y =10
I. (0, 5) II. (2, –1) III. (3, –4) (c) 3x− y =10 (d) None of these

Which of the above lie on the line 3x + y = 5 and 84. The equation of the line joining the origin to the
x y
at a distance 10 from (1, 2)? e 2014 I point of intersection of the lines + = 1 and
(a) Only I (b) Only II (c) I and II (d) I, II and III a b
x y
+ = 1 is
74. Which one of the following is correct in respect of b a e 2015 I
x−1 y− 2 (a) x − y = 0 (b) x + y = 0 (c) x = 0 (d) y = 0
the equations = and 2x + 3 y = 5?
2 3 e 2014 II
(a) They represent two lines which are parallel 85. If a line is perpendicular to the line 5x − y = 0
(b) They represent two lines which are perpendicular and forms a triangle of area 5 sq units with
(c) They represent two lines which are neither parallel coordinate axes, then its equation is e 2015 II
nor perpendicular (a) x + 5 y m 5 2 = 0 (b) x − 5 y ± 5 2 = 0
(d) The first equation does not represent a line (c) 5x + y ± 5 2 = 0 (d) 5x − y ± 5 2 = 0
75. A( 3, 4) and B ( 5, − 2) are two points and P is a 86. The area of the figure formed by the lines
point such that PA = PB. If the area of ∆PAB is ax + by + c = 0, ax − by + c = 0, ax + by − c = 0 and
10 sq units, then what are the coordinates of P? ax − by − c = 0 is e 2015 II
e 2014 II c2 2c 2 c2 c2
(a) (b) (c) (d)
(a) Only (1, 0) (b) Only (7, 2) ab ab 2 ab 4ab
(c) Either (1, 0) or (7, 2) (d) Neither (1, 0) nor (7, 2)
87. The three lines 4x + 4 y = 1, 8x − 3 y = 2, y = 0 are
76. What is the product of the perpendiculars drawn (a) the sides of an isosceles triangle e 2015 II
from the points ( ± a 2 − b2 , 0) upon the line (b) concurrent
bx cos α + ay sin α = ab? e 2014 II (c) mutually perpendicular
(a) a2 (b) b 2 (c) a2 + b 2 (d) a + b (d) the sides of an equilateral triangle
88. The line 3x + 4 y − 24 = 0 intersects the X-axis at
Directions (Q. Nos. 77-79) Consider the ∆ABC with A and Y-axis at B. Then, the circumcentre of the
vertices A ( −2, 3), B (2, 1) and C (1, 2). e 2015 I ∆OAB, where O is the origin, is e 2015 II
77. What is the circumcentre of the ∠ABC ? (a) (2, 3) (b) (3, 3)
(a) (−2, − 2 ) (b) (2, 2 ) (c) (−2, 2 ) (d) (2, − 2 ) (c) (4, 3) (d) None of these

78. What is the centroid of the ∆ABC ? 89. The product of the perpendiculars from the two
points ( ± 4, 0) to the line 3x cos φ + 5 y sin φ = 15 is
(a)  , 1 (b)  , 2  (c)  1,  (d)  , 3
1 1 2 1
3  3   3 2  e 2015 II
(a) 25 (b) 16 (c) 9 (d) 8
79. What is the foot of the altitude from the vertex A
of ∆ABC? 90. Two straight lines passing through the point
(a) (1, 4) (b) (−1, 3) (c) (−2, 4) (d) (−1, 4) A(3, 2) cut the line 2 y = x + 3 and X-axis
perpendicularly at P and Q, respectively. The
80. The perpendicular distance between the straight equation of the line PQ is e 2015 II
lines 6x + 8 y +15= 0 and 3x + 4 y + 9= 0 is e 2015 I (a) 7 x + y − 21 = 0 (b) x + 7 y + 21 = 0
(a) 3/2 units (b) 3/10 unit (c) 3/ 4 unit (d) 2 /7 unit (c) 2 x + y − 8 = 0 (d) x + 2 y + 8 = 0
MATHEMATICS > Coordinate System and Straight Lines 189

Directions (Q. Nos. 91-92) Consider the lines y = 3x,


96. A straight line intersects X and Y-axes at P and
y = 6 x and y = 9. e 2016 I
Q, respectively. If (3, 5) is the middle point of
91. What is the area of the triangle formed by these PQ, then what is the area of the ∆OPQ?
lines? (a) 12 sq units (b) 15 sq units e 2016 I
27 27 (c) 20 sq units (d) 30 sq units
(a) sq units (b) sq units
4 2
19 19
(c) sq units (d) sq units Directions (Q. Nos. 97-98) Consider the two lines
4 2 x + y + 1 = 0 and 3x + 2y + 1 = 0. e 2016 I
92. The centroid of the triangle is at which one of 97. What is the equation of the line passing through
the following points? the point of intersection of the given lines and
(b)  , 6 (d)  , 9
3 3 parallel to X-axis?
(a) (3, 6) (c) (3, 3)
2  2  (a) y + 1 = 0 (b) y − 1 = 0
(c) y − 2 = 0 (d) y + 2 = 0
Directions (Q. Nos. 93-95) Consider a parallelogram,
whose vertices are A(1, 2), B( 4, y), C( x, 6) and D(3, 5)
98. What is the equation of the line passing through
the point of intersection of the given lines and
taken in order. e 2016 I
parallel to Y-axis?
93. What is the value of AC 2 − BD 2? (a) x + 1 = 0 (b) x − 1 = 0 (c) x − 2 = 0 (d) x + 2 = 0
(a) 25 (b) 30 (c) 36 (d) 40
99. ( a , 2b) is the mid-point of the line segment
94. What is the point of intersection of the diagonals? joining the points (10, − 6) and ( k, 4). If a − 2b = 7,
(a)  , 4 (c)  , 5
7 7 then what is the value of k? e 2016 I
(b) (3, 4) (d) (3, 5)
2  2  (a) 2 (b) 3 (c) 4 (d) 5

95. What is the area of the parallelogram? 100. What is the acute angle between the lines
7
(a) sq units (b) 4 sq units represented by the equations y − 3x − 5 = 0 and
2 3 y − x + 6 = 0? e 2016 I
11 (a) 30° (b) 45° (c) 60° (d) 75°
(c) sq units (d) 7 sq units
2

ANSWERS
1 a 2 d 3 c 4 c 5 b 6 d 7 d 8 b 9 d 10 b
11 a 12 d 13 c 14 c 15 c 16 b 17 b 18 c 19 c 20 c
21 a 22 b 23 b 24 c 25 a 26 a 27 b 28 a 29 c 30 c
31 b 32 b 33 a 34 b 35 a 36 b 37 c 38 c 39 a 40 c
41 d 42 b 43 b 44 a 45 a 46 d 47 d 48 c 49 a 50 c
51 b 52 d 53 a 54 a 55 d 56 b 57 b 58 b 59 a 60 d
61 c 62 b 63 c 64 a 65 c 66 c 67 a 68 d 69 b 70 c
71 a 72 d 73 c 74 b 75 c 76 b 77 a 78 b 79 d 80 b
81 b 82 d 83 b 84 a 85 a 86 b 87 b 88 c 89 c 90 a
91 a 92 b 93 c 94 a 95 d 96 d 97 d 98 b 99 a 100 a

HINTS AND SOLUTIONS


1. (a) Centroid of ∆ABC coincide with the Incentre is 3. (c) Mid-point of ( p , q ) and ( q , − p ) is
centroid of triangle formed by
 ax1 + bx 2 + cx 3 ay1 + by 2 + cy 3   p + q , q − p  , which is given  r , s .
mid-points of AB, BC and CA.  ,   2 2   2 2
 a+ b+ c a+ b+ c 
∴ Required coordinates p+ q r
∴ =
12 × 0 + 10 × 8 + 10 × 8
≡ 
4 + 3 + 2 2 + 3 + 2
,  ≡ (3,7 / 3) i.e.  , 2
q− p s
2
 3 3   12 + 10 + 10 and =
12 × 6 + 10 × 12 + 10 × 0  2 2
2. (d) a = BC = 02 + ( 12 − 0)2 = 12 
12 + 10 + 10  Now, length of segment
b = AC = ( 0 − 8)2 + ( 6 − 0)2 = 10  160 192  = ( p − q )2 + ( q + p )2 = s 2 + r2
= ,  = (5 , 6 )
 32 32 
c = AB = 8 + 6 = 10
2 2
190 NDA/NA Pathfinder

a 0 1 10. (b) Since, the point ( h , k ) lies on ⇒ h 2 + (m + n)2 − 2h(m + n) + k 2


1 x + y + 1 = 0. ⇒ h + k + 1 = 0 + ( n − m )2 − 2k( n − m ) = h 2 + (m − n)2
4. (c) ∆ = at 2 2at 1
2 1 1 −2h(m − n) + k 2 + ( n + m )2 − 2k(m + n)
at 2 2at 1 and h 2 + k 2 = 25
2 2
( −1 − k )2 + k 2 = 25 ⇒ −2[ h(m + n) + k( n − m )]
1 0 1 = −2[ h(m − n) + k(m + n)]
⇒ 2k 2 + 2k − 24 = 0
1
= × ( 2a ) × a × t 2 ⇒ mh + nh + nk − mk
t 1 ⇒ k 2 + k − 12 = 0
2 1 1 = mh − nh + mk + nk
t2 t 1 ⇒ k = −4 or k = 3 [rejected as k < 0]
2 2 ⇒ 2nh = 2mk ⇒ nh = mk
∴ h = −1 − ( −4 ) = 3 ∴ Required locus is nx = my
∴∆ = 0 ⇒ (t − t ) + (t 2t − t 2t ) = 0
1 2 1 2 2 1 11. (a) Let the third vertex be ( p , q ), then 16. (b) Since, slope of line
⇒ (t − t ) + t t (t − t ) = 0
1 2 1 2 1 2 q = p+3 …(i) x cos θ + y sin θ = 2 is − cot θ
⇒ (t − t ) (1 + t t ) = 0 Also, ∆ = |5 | = ± 5 and slope of line x − y = 3 is 1.
1 2 1 2
⇒ t t =−1 ∴ q + 3 p − 7 = ± 10 Also, these lines are perpendicular to
1 2
∴ 3 p + q = 17 …(ii) each other.
5. (b) Area of quadrilateral ABCD
3 p + q = −3 ∴ ( − cot θ) ( 1) = − 1
1 x1 − x 3 y1 − y 3
and …(iii)
= π π
⇒ cot θ = 1 = cot ⇒ θ =
2 x 2 − x 4 y2 − y4
On solving Eq. (i) with Eq. (ii) and Eq.
4 4
(iii), we get  ,  and  − , 
7 13 3 3
1 ( 1 − 12) ( 1 − 2) 2 2   2 2 17. (b)
=
2 ( 7 − 7) ( − 3 − 21) 18. (c) The equations of the sides of ∆ABC
∴ Required vertex =  , 
7 13
are x = 0, y = 0 and x + y = 1. Clearly,
2 2 
1 − 11 − 1 1 ∆ABC is right angled triangle with right
= = ( 264 − 0) 12. (d) Slope of line x + 3 y = 4 is m = −1 / 3 angle at the origin and coordinates the
2 0 − 24 2 1
−6 end-points of hypotenuse as ( 1, 0) and
= 132 sq units Slope of line 6x − 2 y = 7 is m = =3 ( 0, 1). So, orthocentre of the triangle
2
−2
is at the origin and circumcentre at
6. (d) Given lines are 4x + 3 y = 11 Q m m = −1
1 2 the mid-point of its hypotenuse, i.e. at
and 4x + 3 y = 15 / 2. ∴The given diagonals are perpendicular (1/2, 1/2).
Distance between them to each other.
15 So, both statements are correct.
11 − Thus, ABCD is a rhombus.
2 = 7 7
= = 19. (c) Vertices
16 + 9 2×5 10 13. (c) Required line should be ( a cos θ , a sin θ ), ( a cos θ2 , a sin θ2 )
1 1
(3x − y + 2) + λ(5x − 2 y + 7) = 0 …(i) and ( a cos θ3 , a sin θ3 ) are equidistant
7. (d) Equation of line parallel to
⇒ (3 + 5λ )x − ( 2λ + 1) y + ( 2 + 7λ ) = 0 from origin (0, 0). Hence, the origin is
2x + 3 y + 5 = 0 is 2x + 3 y = λ

x
+
y
=1 3 + 5λ 2 + 7λ circumcentre (centroid) of circumcircle.
⇒ y= x + …(ii)
( λ / 2) ( λ / 3) 2λ + 1 2λ + 1 Therefore, the coordinates of centroid
λ λ are
Given, + = 15 ⇒ λ = 18 As, the Eq. (ii) has infinite slope,
2 3  a(cos θ1 + cos θ2 + cos θ3 )
2 λ + 1 = 0 ⇒ λ = −1 / 2  ,
∴ Required line is 2x + 3 y = 18 On putting the value of λ in Eq. (i), 
 3
8. (b) Let required ratio be λ : 1. Then, the we get a(sin θ + sin θ + sin θ ) 
x =3
1 2 3

coordinates of point which divides the 
line joining ( −1, 1) and (5, 7) in the ratio
3 
14. (c) Let A ( a , 0) and B ( 0, b ) be two
5 λ − 1 7 λ + 1 But as the centroid is the origin ( 0, 0),
λ : 1, is  , . points on respective coordinate axes and
therefore cos θ + cos θ + cos θ = 0
 λ+ 1 λ+ 1 ( −5, 4) divides AB in the ratio 1 : 2. 1 2 3
1× 0 + 2 × a −15 and sin θ + sin θ + sin θ = 0.
But it lies on x + y = 4 ∴ −5= ⇒a = 1 2 3
3 2 Hence, both statements are correct.
5λ − 1 7λ + 1
∴ + =4 1× b + 2 × 0 20. (c) Area of equilateral triangle is
λ+ 1 λ+ 1 and 4 = ⇒ b = 12
3 3
⇒ 12λ = 4λ + 4 ⇒ λ = 1/ 2 (side)2 which is an irrational
Hence, equation of line joining 4
∴ Required ratio = 1 : 2
 − 15 , 0 and ( 0, 12) is number. So, Statement I is false.
9. (d) Equation of perpendicular line from  2  In a triangle, the circumcentre(S),
3x − 4 y − 5 = 0 …(i) 12 − 0  15  centroid (G), and orthocentre (O) are
( y − 0) = ⋅ x + 
15  2 collinear and
is 4x + 3 y + c = 0 0+
2 SG : GO = 1 : 2 or OG : GS = 2 : 1
Since it passes through (0, 5).
⇒ 8x − 5 y + 60 = 0 So, Statement II is true.
∴ c = −15 In a triangle, circumcentre is the only
15. (c) Let the coordinate of the moving
⇒ 4x + 3 y − 15 = 0 …(ii) point P be ( h , k ).
point which is equidistant from all the
three vertex of the triangle.
On solving Eqs. (i) and (ii), we get (3, 1) Then, [ h − (m + n)]2 + [ k − ( n − m )]2
So, Statement III is true.
= [ h − (m − n)]2 + [ k − ( n + m )]2
MATHEMATICS Coordinate System and Straight Lines 191

21. (a) Let ABCD be the rectangle the 1  35 8  3 30. (c) Let O be the points of intersection of
= − = sq units
equation of whose sides AB, BC ,CD and 2  9 9  2 lines L and L .
1 2
DA, respectively are x = a, y = b, x = a ′ Solving Eqs. (i) and (ii), we get
∴ Area of ||gm ABCD = 2 × ar( ∆ABD )
and y = b ′ O ≡ ( 2, 5)
3
y = b′ = 2 × = 3 sq units Equation of L is x + y = 0 ...(iii)
A D 2 3
Solving Eqs. (i) and (iii), we get P ≡( −1, 1)
25. (a) The equation of the given line is
x=a x = a′ x y Solving Eqs. (ii) and (iii), we get
+ =1 …(i) Q ≡ ( − 2, 2)
p cos θ p cot θ
∴ OP = ( 2 + 1)2 + (5 − 1)2 = 5
B y=b C Let the line (i) cuts X-axis and Y-axis at
Then, its vertices are the points A and B respectively, then and OQ = ( 2 + 2)2 + (5 − 2)2 = 5
A( a , b ′), B( a , b ),C ( a ′ , b ) and D( a ′ , b/ ′). A ≡ ( p cos θ, 0) and B ≡ ( 0, p cot θ)
∴ In any triangle, bisector of an angle
Since, diagonals of rectangle bisect each If P( x , y ) is the mid-point of AB, then divides the triangle into two similar
other. Therefore, coordinates of point of 2x = p cos θ and 2 y = p cot θ triangles.
a + a′ b + b′ ∴ sec θ =
p
and tan θ=
p

PR OP 5 1
= = = = 1: 1
intersection are x = and y =
2 2 2x 2y RQ OQ 5 1
22. (b) AB = ( a − a )2 + ( b ′− b )2 = |b ′ − b | Since sec 2 θ − tan 2 θ = 1 2 5 1
p2 p2 31. (b) Area of ∆OPQ =
1
−1 1 1
and BC = ( a − a ′ ) + ( b − b ) =|a ′− a |
2 2 ⇒ 2
− =1
4x 4y2 2
−2 2 1
∴ Area of rectangle ABCD = AB × BC 26. (a) Equation of line perpendicular to 1 7
= |b ′ − b ||a ′ − a | line L is x tan θ − y sec θ + k = 0 = |[ − 2 − 5 + 0]| = sq units
2 2
23. (b) Equations for sides AB and AD are Since, this line passes through
x x 1
4x + 5 y = 0 ...(i) ( p cot θ, 2 p cos 3 θ) 1
32. (b) Area of the ∆AMB = −2 0 1
and 7x + 2 y = 0 ...(ii) ∴ p cot θ ⋅ tan θ − 2 p cos 3 θ ⋅ sec θ + k = 0 2
0 4 1
The point of intersection of AB and AD ⇒ p − 2 p cos 2 θ + k = 0
is A( 0, 0), clearly A( 0, 0) does not lie on 1
⇒ k = p( 2 cos 2 θ − 1) = p cos 2θ = ( −4 x + 2 x − 8 ) = | − ( x + 4 ) |
diagonal 2
11x + 7 y = 9 ...(iii) ∴ Required equation is
which is minimum for x = 0 and thus
On solving Eqs. (i) and (iii), we get x tan θ − y sec θ + p cos 2θ = 0 the coordinates of M are (0, 0).
5 −4
B ≡  , 
27. (b) 33. (a) As P divides AB in the ratio 2 : 1.
3 3  A The base of the ∆’s APM and BPM are
On solving Eqs. (ii) and (iii), we get C(3,√3) in the ratio 2 : 1 and the length of the
perpendicular from the vertex M on the
D ≡  − , 
2 7
 3 3 √3
base is same. So, the ratio of the areas of
C x+ the ∆ APM and ∆BPM is also 2 : 1.
D y=
D 2√ Y
3 B (0, 4)
H P
B C
A′
11x + 7y=9 Two equilateral triangles ABC and X′ X
B
A D M (0, 0)
A (0,0) A ′ BC are possible with one vertices (–2, 0)
(3, 3 ) and AD as an altitude.
Since, H is the middle point of BD. Y′
28. (a)
5 − 2 − 4 + 7 34. (b) ABCD is a quadrilateral with
  29. (c) The equations of lines L and L by
∴ H =  3 3 , 3 3  ≡  , 
1 1 1 2
 making constant term positive, are AD = 1, BC = 2
 2 2  2 2
  4x − 3 y + 7 = 0 ...(i) 1 1 2
DC = AB = 2 + 42 = 5
and 3x − 4 y + 14 = 0 ...(ii) 2 2
Equation of diagonal AC is
So, the required perimeter is
1 Q 4 × 3 + ( − 3) ( − 4) = 24 > 0
0− 1+ 2 + 5 + 2 5 =3+ 3 5
y −0= 2 ( x − 0) ⇒ x − y = 0 i.e. aa + bb >0
1 2 1 2
1 Sol. (Q. Nos. 35-38)
0− So, the bisector of the acute angle is
2 given by We have, O( 0, 0), A( 1, 1) and B( 0, 3)
0 0 1 4x − 3 y + 7 3x − 4 y + 14
Coordinates of P =  ,  = ( 0, 1)
0 3
=−
4 + ( −3 )
2 2
3 2 + ( −4 ) 2  3 3
1 5 4
24. (c) Area of ∆ABD = − 1 Coordinates of Q
2 3 3 ⇒ 4x − 3 y + 7 = − 3x + 4 y − 14
1 + 0 1 + 1  1 

2 7
1 ⇒ x − y +3=0 =  ,  =  , 1
3 3  2 2  2 
192 NDA/NA Pathfinder

Coordinates of R 39. (a) ( a + 2b )x × ( a − 3b ) y + a − 8b = 0 Q θ = 45° and c = 101 unit


0+ 1 6+ ⇒ a( x + y + 1) + b( 2x − 3 y − 8) = 0 ∴ y = tan 45° x + 101
= 
1  1 7
,  = , 
 3 3  3 3 These lines are concurrent at point of ⇒ y = 1 ⋅ x + 101 ⇒ x − y + 101 = 0
2 2 intersection of the lines x + y + 1 = 0
35. (a) ( PR)2 =  0 −  +  1 − 
1 7 and 2x − 3 y − 8 = 0, which is ( 1, − 2). 48. (c) Let the line y = 0 divides the line
 3  3 joining the points (3, − 5) and ( −4, 7) in
Now, line through A( 1, − 2) which is
1 16 17 farthest from the point B( 2, 2) is the ratio n : m, then
= + = perpendicular to AB.
9 9 9 By internal section formula,
2+ 2
2 2
Slope of AB = =4 −5m + 7n
(QR) =  −  +  1 − 

2 1 1 7 = 0 ⇒ 5 m = 7n
 2 3  2−1
3 ∴ Required equation of line is
m+ n
1 16 65 −1 n 5
= + = y + 2 = ( x − 1) ⇒ x + 4 y + 7 = 0 ⇒ = or n : m = 5 : 7
36 9 36 4 m 7
Now, α(QR)2 = β( PR)2 40. (c) Also this line x + 4 y + 7 = 0 meet 49. (a) Given,
α 68 X-axis at C ( − 7, 0) and Y-axis at ( 2x + 3 y + 4) + λ( 6x − y + 12) = 0
α   = β   ⇒
65 17
⇒ = 2x + 6λx + 3 y − λy + 4 + 12λ = 0
 36   9 D  0, −  .
β 65 7
 4 2x (3λ + 1) + y(3 − λ ) + 4 + 12λ = 0
36. (b) Area of ∆OAB ...(i)
∴ Area of ∆OCD
1 3 Since, line (i) is parallel to Y -axis.
= × 1 × 3 = sq units
2 2 So, the coefficient of y must be zero.
0 0 1
1 1 49 49 ∴ 3− λ = 0⇒ λ =3
0 1 1 = −7 0 1 = × =
2 2 4 8 50. (c) 51. (b)
1 1 7
Area of ∆PQR = 1 1 0 − 1
2 2 52. (d) Given lines are,
4
1 7
1 3x + 4 y − 9 = 0 …(i)
3 3 41. (d) The point of intersection of lines and 9x + 12 y + 28 = 0
x + 4 y + 7 = 0 and x − 2 y + 1 = 0 is 28
1  −  + 1  −  
1 1 1 7 1 ⇒ 3x + 4 y + =0 …(ii)
= − ( − 3, − 1) which must satisfy the line
2   2 3  6 3   3
1 3x − 4 + λ = 0. 28
+ 9
= sq units
3 ∴ − 9+ 4+ λ = 0 ⇒λ =5 ∴ Distance between them = 3
1 9 + 16
42. (b) Let P( x , y ) be the image of ( 2, 2)
ar( ∆PQR) 3 2 with respect to the line x + 4 y + 7 = 0
∴ = = 55 1 11
ar( ∆OAB ) 3 9 = × = units
∴ Point ( 1, − 2) is the mid-point of 3 5 3
2
37. (c) Coordinates of S ( 2, 2) and ( x , y ) 53. (a) Let the points be A(5, 1), B( 1, −1)
0 + 1 1+ 7 x+2 y+2 and C ( 11, 4).
 ∴ = 1 and =−2
= 3, 3  =  1 , 10  2 2 AB = (5 − 1)2 + ( 1 + 1)2 = 2 5
 
 2 2  6 6
  ⇒ x = 0 and y = − 6 BC = ( 11 − 1)2 + ( 4 + 1)2 = 5 5
2 2 Hence, P( 0, − 6) and CA = ( 11 − 5)2 + ( 4 − 1)2 = 3 5
∴ QS =  1 − 1  +  1 − 10 
 2 6  6 43. (b) Length of perpendicular Clearly, AB + CA = BC
5 2 × 1+ 4 × 2 + 7
= = = 17 units Hence, they are collinear.
3 12 + 42 54. (a) Let A ( 1, 0) and B ( 0, − 2) are two
2
given points and P ( h , k ) be any variable
OQ =  0 −  + ( 0 − 1)2
1 44. (a) Since, both lines are coincident to
 2 each other. Hence, the distance between point.
two coincident line is zero. According to the question,
5
= PA = PB ⇒ PA 2 = PB 2
2 45. (a) Since, the line passes through the
2 point (0, 1) and making an angle with ⇒ ( h − 1)2 + ( k − 0)2 = ( h − 0)2 + ( k + 2)2
Clearly, QS = OQ
3 Y -axis which is equivalent to the slope of ⇒ 2h + 4k + 3 = 0
the line y = x − 4. Hence, locus of point P ( h , k ) is
38. (c) Centroid of ∆PQR
i.e. θ = 45° ⇒ tan θ = 1 = m 2x + 4 y + 3 = 0.
0 + 1 + 1 1+ 1+ 7
 2 3, 3  =  5 , 13  ∴ Equation of line is 55. (d) 56. (b)
=  
 3 3  18 9  ( y − 1) = m ( x − 0) = 1 ( x ) ⇒ y = x + 1 57. (b) Given equation of line,
 
46. (d) 3x − y − 1 = 0 ⇒ y = 3x − 1
So, Statement I is true.
47. (d) We know that, if the line making an On compairing with y = mx + c, we get
Q OP = 1, AP = 1 and OA = 2
angle θ with the positive direction of m= 3 [Q m = tanθ]
∴OP 2 + AP 2 = OA 2
∴OAP is right angled triangle.
X -axis with y intercept. ⇒ tan θ = 3 = tan 60° ⇒ θ = 60°
Then, equation of the line is So,the inclination of the given line is 60°.
So, Statement II is also true.
y = mx + c = tanθ ⋅ x + c
MATHEMATICS Coordinate System and Straight Lines 193

58. (b) A line which passes through the 66. (c) If the lines 2x + 3 y + a = 0 73. (c) Let S ≡ 3x + y − 5 = 0
points (5, 0) and ( 0,3) is and 5x + ky + a = 0 are parallel lines, Also, let A ≡ ( 0, 5), B ≡ ( 2, − 1)
x y
+ = 1 ⇒ 3x + 5 y − 15 = 0 ...(i) 2 3 a 15 C ≡ (3,−4)
5 3 then = ≠ ⇒ k = = 75. and
5 k a 2 S = 0 + 5 −5 = 0
Now, length of the perpendicular from A
the point ( 4,4) on the line (i) is 67. (a) Since, the required line is S = 6 − 1− 5 = 0
perpendicular to the line B
|3( 4) + 5( 4) − 15| 17 17 and S = 9 − 4 −5 = 0
= = = 3x + 4 y + 5 = 0. C
(3 ) 2 + (5 ) 2 34 2 So, the slope of required line is Thus, all three points lie on line
 −1  = 4 3x + y − 5 = 0
59. (a) The equation of line in intercept
form is x y  ( −3 / 4)  3 Now, let D ≡ ( 1,2)
+ =1 ...(i) AD = ( 1 − 0)2 + ( 2 − 5)2 = 10
a b Also, required line passing through the
Given, a + b = 14 ...(ii) point ( 4, − 5) . Then, its equation BD = ( 2 − 1) + ( −1 − 2) =
2 2
10
Since, the line (i) passes through the 4
3 4 ( y + 5) = ( x − 4) ⇒ 4x − 3 y = 31 and CD = ( 1 − 3) + ( 2 + 4) =
2 2 2
10
point (3, 4), then + = 1 3
a b Thus, only points A and B at a distance
3 4 68. (d) Perpendicular distance of the line
⇒ + = 1 [from Eq. (ii)] 3x + 4 y − 1 = 0 from the point ( 1, 1) 10 from (1, 2). Hence, correct option is
a ( 14 − a ) = Perpendicular distance of the line (c).
⇒ ( a − 6)( a − 7) = 0 ⇒ a = 6, 7 4x + 3 y + 2k = 0 from the point (1, 1) 74. (b) Given, equations of lines are
⇒ b = 8, 7 |3 × 1 + 4 × 1 − 1| |4 × 1 + 3 × 1 + 2k | x −1 y−2
⇒ = =
Hence, equations of straight lines are 9 + 16 16 + 9 2 3
1 ⇒ 3x − 2 y + 1 = 0 …(i)
4x + 3 y − 24 = 0 and x + y = 7 ⇒ 6 = 7 + 2k ⇒ 2k = − 1 ⇒ k = −
2 and 2x + 3 y = 5 .…(ii)
60. (d) 3
69. (b) ∴ Slope of line (i), m =
1
61. (c) Equation of straight line which 2
passes through the points (5, − 2) and 70. (c) Let the required equation be, 2
x y
+ =1 …(i) and slope of line (ii), m = −
7+ 2 2
3
( − 4, 7) is ( y + 2) = ( x − 5) a b
−4 − 5 ∴ mm = −1
whose intercept on X and Y-axes are a 1 2
∴ x+ y = 3 and b, respectively. Hence, two lines are perpendicular to
62. (b) We know that, the equation of Y′ each other.
straight line in intercept form is 75. (c) We have, A(3, 4) and B(5, − 2)
x y
+ =1 ...(i) (0, b)
Let P( x , y )
a b Given that, PA = PB
1 1
By given condition, a = and b =
m n (1, 2) ⇒ PA 2 = PB 2
∴ Required equation of line is (a, 0) ∴( x − 3)2 + ( y − 4)2
x y X′ X′
+ = 1 ⇒ mx + ny = 1 O Y′ = ( x − 5)2 + ( y + 2)2
( 1 / m ) ( 1 / n)
⇒ x − 3y = 1 ...(i)
63. (c) By the given condition, Area of ∆PAB = 10
a + 0 0 + b a
64. (a) Let the coordinates of point P is ( 1, 2) =  ,  ⇒ = 1⇒ a = 2 x y 1
( h, k ).  2 2  2 1
Now, by given condition, ∴ 3 4 1 = ± 10
and
b
= 2⇒b = 4 2 5 −2 1  
( h − 1)2 + ( k − 2)2 2  
= ( h + 2)2 + ( k − 3)2
Put the value of a and b in Eq. (i), we get ⇒ x ( 4 + 2) − y(3 − 5) + 1 ( −6 − 20)
x y
+ =1 = ± 20
⇒ 3h − k + 4 = 0 2 4 ⇒ 6x + 2 y − 26 = ± 20
So, the locus of P is 3x − y + 4 = 0, ⇒ 2x + y = 4 ⇒ 6x + 2 y = 46 ...(ii)
which represent a straight line. 71. (a) or 6x + 2 y = 6 ...(iii)
65. (c) Let A ≡ (5,2) and B ≡ ( −2,3) On solving Eqs. (i) and (ii), we get
2−0 2 72. (d) The given equation of lines are x = 7, y = 2
Slope of OA (m ) = =
1
5−0 5 3x − 4 y + 12 = 0 …(i) Similarly, solving Eqs. (i) and (iii),
−15 − 0 −5 and 3x − 4 y − 6 = 0 …(ii) we get x = 1, y = 0
and slope of OB (m ) = =
2
6−0 2 So, the equation of line mid way Hence, coordinates of P are (7, 2) and
between the given lines is (1, 0).
2 −5
Q m ⋅m = × =−1 12 + ( −6)
1 2
5 2 3x − 4 y + =0 76 (b) We have, equation of line is
2
i.e. angle between OA and OB is π/ 2. ⇒ 3x − 4 y + 3 = 0 bx cos α + ay sin α − ab = 0
194 NDA/NA Pathfinder

Perpendicular distance from point 2−1 Equation of line passing through point
y−2= ( x − 1) ⇒ x + y − 3 = 0
( a − b , 0) is d
2 2 1− 2 of intersection of these two lines is
1
∴ Coordinates of point is given by ( bx + ay − ab ) + λ( ax + by − ab ) = 0
 b cos α a 2 − b 2 + 0 − ab  x + 2 y − 3 −( −2 + 3 − 3) Since, it passes through origin (0, 0).
=  = =
1 1 12 + 12 ∴ − ab + λ( − ab ) = 0 ⇒ λ = −1
 b cos α + a sin α 
2 2 2 2
⇒ x + 2 = 1 and y − 3 = 1 Hence, required equation of line is
Similarly, perpendicular distance from
⇒ x = −1 and y = 4 ( bx + ay − ab ) − ( ax + by − ab ) = 0
point ( − a 2 − b 2 ,0) is Hence, the required foot of altitude is ⇒ ( b − a )x − ( b − a ) y = 0
 − b cos α a 2 − b 2 + 0 − ab  ( −1, 4).
d =  ∴ x−y=0
2
b 2 cos 2 α + a 2 sin 2 α  80. (b) We have, 6x + 8 y + 15 = 0
 85. (a) Equation of the line perpendicular to
Now, d × d and 6x + 8 y + 18 = 0 5x − y = 0 is x + 5 y = λ.
1 2
∴ Perpendicular distance between Y
λ
( b cos α a 2 − b 2 − ab )  18 − 15 3 B 0,
  them is = unit 5
36 + 64 10
( b cos α a 2 − b 2 + ab ) 
A (λ, 0)
=   81. (b) Equation of line perpendicular to the X′
O X
( b 2 cos 2 α + a 2 sin 2 α ) line 3x + y − 3 = 0 is x − 3 y + k = 0 Y′
  Since, it passes through point (2, 2). Now, area of ∆AOB = 5 sq units
( b 2 cos 2 α + a 2 sin 2 α ) 1 λ
  ∴2 − 6 + k = 0 ⇒ k = 4 ⇒ λ × =5 ⇒ λ = ± 5 2
2 5
b 2 cos 2 α( a 2 − b 2 ) − a 2 b 2 ∴ Required equation of the line is
= x y ∴Equation of the lines are
b 2 cos 2 α + a 2 sin 2 α ⇒ x − 3y + 4 = 0 ⇒ + =1 x + 5y = ± 5 2
−4 4 / 3 i.e. x + 5 y m 5 2 = 0
a 2 b 2 (cos 2 α − 1) − b 4 cos 2 α 4
= So, y-intercept is . 86. (b) The figure formed by the lines
b 2 cos 2 α + a 2 sin 2 α 3 ax + by + c = 0, ax − by + c = 0,
− b 2 [ a 2 sin 2 α + b 2 cos 2 α ] 82. (d) We have, ∆ = 3 sq units ax + by − c = 0 and ax − by − c = 0 is
= shown below
b 2 cos 2 α + a 2 sin 2 α 5 2 1
1 B 0, c
= −b 2 = b 2 ∴ 3 4 1 =3 b
2
77. (a) Let P( x , y ) is the circumcentre of a 5 1 c
– ,0 c, 0
the ∆ ABC . 1
[5( 4 − 5) − 2(3 − a ) + 1 ( 15 − 4 a )] = 3 a C A a
∴ AP 2 = PB 2 ⇒ ( x + 2)2 + ( y − 3)2 2
= ( x − 2)2 + ( y − 1)2 ⇒ 2 − a = ± 3 ⇒ a = 5 or −1 0, –
c
∴ a =5 b
⇒ 2x − y + 2 = 0 …(i) D
Also, AP 2 = PC 2 ⇒( x + 2)2 + ( y − 3)2 83. (b) Clearly, ABCD is a rhombus and its area
Y
= ( x − 1)2 + ( y − 2)2 1
= × Product of diagonals
2
⇒ 3x − y + 4 = 0 …(ii)
5 1 2c 2c 2c 2
On solving Eqs. (i) and (ii), we get = × × = sq units
60° 120° X 2 b a ab
x = − 2 and y = − 2 X′
(a, 0) 87. (b) Since, the point of intersection of
y = 0 and 4x + 4 y = 1 is  , 0 and
Hence, the required circumcentre is 1
Y′
( −2, − 2). 4 
Let the required line intercept X-axis at a.
78. (b) Given, vertices of a triangle are  1 , 0 lies on the line 8x − 3 y = 2.
Slope of the line is 4 
A( −2, 3), B( 2, 1) and C ( 1, 2).
Centroid of the ∆ABC m = tan 120° = − 3 Hence, the given lines are concurrent.
−2 + 2 + 1 3 + 1 + 2   1  5 10
=  ,  =  , 2 Also, sin 60° = ⇒ a= 88. (c) Given, 3x + 4 y − 24 = 0
 3 3  3  a 3
Y
∴ Equation of line passing through B (0, 6)
79. (d) Let D be the foot of altitude from A
 10 ,0 and having slope − 3 is
in ∆ABC and D ≡ ( x , y )  
 3  A(8, 0)
(1, 2) C X′ X
O
y − 0 = − 3  x −
10 

D  3
Y′
⇒ 3x + y = 10 Its x-intercept A = (8, 0)
A B 84. (a) We have,
x y
+ =1 …(i) and y-intercept B = ( 0, 6)
.
(–2, 3) (2, 1) a b Since, AOB is a right angled triangle.
x y
Equation of line BC is and + =1 …(ii) So, the mid-point of hypotenuse is the
b a circumcentre.
⇒ bx + ay = ab and ax + by = ab Hence, circumcentre
MATHEMATICS Coordinate System and Straight Lines 195

8+ 0 0+ 6
= , = ( 4, 3) 0 0 1 Y
2 2 1 (0, y)
Area of ∆OAB = 3 9 1 Q
89. (c) Given, 3x cos φ + 5 y sin φ = 15 2
3/ 2 9 1 (3, 5)
Lengths of perpendicular from the point
( ± 4, 0), 1 27 27
= 27 −  = sq units X′
5
X
2  2  4 O 3 P
12 cos φ − 15
p = …(i) (x,0)
1
9 cos 2 φ + 25 sin 2 φ 92. (b) Let C ( x , y ) be the centroid of the
triangle. Y′
−12 cos φ − 15 3
and p = …(ii) 0+3+ 0+ y
2
9 cos 2 φ + 25 sin 2 φ Then, x = 2 =3 and = 5 ⇒ y = 10
3 2 2
On multiplying Eqs. (i) and (ii), we get 0+ 9+ 9 1
and y = =6 ∴ Area of ∆POQ = × OP × OQ
− ( 12 cos φ − 15)( 12 cos φ + 15) 3 2
p p =
Coordinates of the centroid is  , 6 .
3 1
1 2
9 cos 2 φ + 25 sin 2 φ 2  = × 6 × 10 = 30 sq units
2
144 cos 2 φ − 225 Sol. (Q. Nos. 93-95) Since, in a parallelogram Sol. (Q. Nos. 97-98) Consider the two lines
= =9 diagonals bisect each other. x + y + 1 = 0 and 3x + 2 y + 1 = 0
9 + 16 sin 2 φ A(1, 2) B(4, y)
97. (d) Given lines are
90. (a) Equation of line passing through
x + y + 1= 0 ...(i)
(3,2) and perpendicular to line
2 y = x + 3 is and 3x + 2 y + 1 = 0 ...(ii)
O
y − 2 = − 2 ( x − 3) Point of intersection of these two lines is
⇒ 2x + y − 8 = 0 …(ii) ( 1, − 2).
On solving Eqs. (i) and (ii), we get Y

P ≡  , 
13 14 D(3, 5) C(x, 6)
X′ X
 5 5 O
∴ Mid-point of AC = Mid-point of DB
x + 1 6 + 2  4 + 3 y + 5 y = –2
Equation of line passing through (3,2) ⇒  ,  ≡ ,  –1 (1, –2)
and perpendicular to X - axis is x = 3  2 2   2 2 
Y′
Coordinate at point Q = (3, 0) x+1 7
⇒ = ⇒ x =6
Now, equation of line PQ is 2 2 ∴ Required equation of line,
0−
14 and y + 5 = 6 + 2 ⇒ y = 3 y = − 2 or y + 2 = 0
y−
14
= 5  x − 13  ∴ Coordinates of O are  , 
7 8
98. (b) Equation of line parallel to Y-axis is
5 13  5  2 2
3− x = c, since this line passes through
i.e.  , 4 .
5 7 ( 1,−2).
⇒ 7x + y − 21 = 0 2 
∴ Required equation is x − 1 = 0
91. (a) Given, lines are y = 3x ...(i) 93. (c) Now, AC 2 = ( 6 − 1)2 + ( 6 − 2)2 = 41 Y
y = 6x ...(ii) and y = 9 ...(iii) and BD = ( 4 − 3) + (3 − 5) = 5
2 2 2

Y y=6x ∴ AC 2 − BD 2 = 41 − 5 = 36
y=3x
3 ,9 94. (a) X′ X
2 O
B A y=9
(3, 9) 95. (d) Vertices of parallelogram ABCD in (1, –2)
(0, 0) order are (1,2), (4,3), (6,6), and (3,5)
X′ X x =1
O M
1 ( 1 − 6) ( 2 − 6) Y′
∴ Area of parallelogram =
2 ( 4 − 3 ) (3 − 5 ) 99. (a) Since, ( a , 2b ) is the mid-point of the
line segment joining the points ( 10, − 6)
1 −5 −4 14 and ( k , 4), therefore we have
= = = 7 sq units
Y′ 2 1 −2 2 ( a , 2b ) = 
10 + k −6 + 4 
, 
On solving Eqs. (i) and (iii), we get  2 2 
x = 3, y = 9
96. (d) Let coordinates of P and Q be ( x , 0) 10 + k
and ( 0, y ), respectively. ⇒ a=
∴ Coordinates of A = (3, 9) 2
x+0 Also, it is given that a − 2b = 7
On solving Eqs. (ii) and (iii), we get ∴ Mid-point of PQ is =3
2 10 + k
− 2  −  = 7
3 1
x =
,y=9 ⇒ x =6 ⇒
2  2
2
⇒ 10 + k + 2 = 14 ⇒ k = 2
∴ Coordinates of B =  , 9
3
2  100. (a)
17
196 NDA/NA Pathfinder

CIRCLE
In NDA exam, generally 2-3 questions are asked from this chapter which are based on finding
radius of circle, distance between centre of two circles, condition for touching two circles/a line
and a circle etc.

A circle is defined as the locus of a point which Then, the equation of circle is
moves in a plane such that its distance from a ( x − h) 2 + ( y − k) 2 = k 2
fixed point in that plane is always constant. The
fixed point is called the centre and the constant or x 2 + y 2 − 2hx − 2ky + h 2 = 0
distance is called the radius of the circle. 2. Circle touching the Y-axis Let the circle
Equation of a Circle in Standard touches theY-axis and has its centre C ( h, k).
Y
Form
Let C( h, k) be the centre of the
r=h
circle and the radius of the r P(x, y)
C(h, k)
circle be r, then equation of C(h,k)
circle in standard form X′
O
X

( x − h) + ( y − k) = r
2 2 2 Y′
Then, the equation of circle is
Note Above equation is known as the central form of the ( x − h) 2 + ( y − k) 2 = h 2
equation of a circle.
or x + y 2 − 2hx − 2ky + k 2 = 0
2

Equation of Circle in 3. Circle touching both the axes Let the circle
Particular Cases touches both the axes.
Y
1. Circle touching the X-axis Let the circle
touches the X-axis and has its centre C ( h, k).
Y
r C(h, k)
r
X′ X
C(h, k) O r=h=k
Y′
r=k
X′ X Then, the equation of circle is
O
( x − r ) 2 + ( y − r ) 2 = r 2 or
Y′
x 2 + y 2 − 2rx − 2ry + r 2 = 0
MATHEMATICS Circle 197

4. Circle centre at origin If the Y


Intercepts on the Axes
centre of the circle is at origin Y
The lengths of intercepts made B2
and radius is ‘r’, then the
r=k by the circle
equation of the circle is X′ X
C(0, 0) x 2 + y 2 + 2g x + 2 fy + c = 0 with
(x − 0 ) + ( y − 0 ) = k
2 2 2
B1
⇒ x 2 + y 2 = k2 X - axis = A1 A 2 = 2 g 2 − c X′ O
A1 A2
X
Y′ Y′
Y - axis = B1 B2 = 2 f 2 − c
General Equation of a Circle
The general second degree equation in x and y is Note Intercepts are always positive.
ax 2 + 2hxy + by 2 + 2gx + 2 f y + c = 0, represents a circle Concentric Circle
whose centre is ( − g , − f ) and radius = g 2 + f 2 − c , when Two circles having the same centre C ( h, k) but
a = b and h = 0. different radii r1 and r 2 , respectively are called
concentric circles.
EXAMPLE 1. The equation of the circle which touches Equation of concentric circle with
X-axis at (3, 0) and passes through (1, 4) is given by x 2 + y 2 + 2gx + 2 fy + c = 0 is
a. x2 + y 2 − 6 x − 5y + 9 = 0 b. x2 + y 2 + 6 x + 5y − 9 = 0
x 2 + y 2 + 2gx + 2 fy + c1 = 0
c. x2 + y 2 − 6 x + 5y − 9 = 0 d. x2 + y 2 + 6 x − 5y + 9 = 0
Therefore, the equations of concentric circles differ
5 Y only in constant term.
Sol. k 2 = 4 + ( k − 4) 2 ⇒ k =
2
(1, 4) (3, k)
Hence, the required equation of Equation of Circle if Coordinates of
5 2 5 2 End Points of Diameter are Known
circle is ( x − 3) 2 +  y −  =  
X′ X
(3, 0)
 2   2 Y′
If ( x1 , y1 ) and ( x 2 , y 2 ) are the end points of the
⇒ x 2 + y 2 − 6x − 5 y + 9 = 0 diameter of a circle, then equation of circle is
( x − x1 ) ( x − x 2 ) + ( y − y1 ) ( y − y 2 ) = 0
Nature of Circle
1. If g 2 + f 2 − c > 0, then the radius of the circle is real
C
and circle is also real. A(x1, y1) B(x2, y2)

2. If g + f − c = 0, then radius of circle is zero and


2 2

circle is known as a point circle.


3. If g 2 + f 2 − c < 0, then radius is imaginary but the Position of a Point with
centre is real and such circle is called imaginary Respect to the Circle
circle, not possible to draw such a circle.
Let S ≡ x 2 + y 2 + 2gx + 2 fy + c = 0 be the equation of a
EXAMPLE 2. Find the equation of the circle, whose circle, then a point P ( x1 , y1 ), in the XY-plane lies
centre is at (−1, 2) and which passes through the point (i) on the circle, if S1 = 0
(3, 5). (ii) outside the circle, if S1 > 0
a. x2 + y 2 + 2x + 4 y + 20 = 0 b. x2 + y 2 + 2x − 4 y + 20 = 0
(iii) inside the circle, if S1 < 0
c. x2 + y 2 − 2x + 4 y + 20 = 0 d. x2 + y 2 + 2x − 4 y − 20 = 0
where, S1 ≡ x12 + y12 + 2gx1 + 2 fy1 + c
Sol. . The centre of the circle is at ( −1, 2). Since, the point
(3, 5) lies on the circle, the distance of the centre from this Equation of Tangent
point is the radius r of the circle. Therefore, we obtain 1. Equation of tangent at the point P ( x1 , y1 ) to a
r = ( −1 − 3) 2 + ( 2 − 5) 2 = 16 + 9 = 5 circle x 2 + y 2 + 2g x + 2 fy + c = 0 is
The equation of the circle with centre at ( −1, 2) and radius 5 xx1 + yy1 + g ( x + x1 ) + f ( y + y1 ) + c = 0
is given by 2. Equation of tangent to circle x 2 + y 2 = a 2 at
( x + 1) 2 + ( y − 2) 2 = 52 or x2 + y 2 + 2x − 4y − 20 = 0
( x1 , y1 ) is given by xx1 + yy1 = a 2 .
198 NDA/NA Pathfinder

Length of Tangent Sol. Let S1 ≡ x2 + y 2 − 6x = 0


and S 2 ≡ x2 + y 2 + 6x + 2y + 1 = 0
1. The length of tangent from an external point
The coordinates of the centres of the given circles are
( x1 , y1 ) to the circle x 2 + y 2 + 2gx + 2 fy + c = 0, is
C1( 3, 0) and C 2( − 3, − 1).
x12 + y12 + 2gx1 + 2 fy1 + c . Radius of S1 = r1 = 32 + 0 − 0 = 3
2. The length of the tangent from an external point Radius of S 2 = r2 = ( − 3) 2 + ( − 1) 2 − 1 = 3
( x1 , y1 ) to the circle
Now, C1 C 2 = ( 3 + 3) 2 + (0 + 1) 2 = 37 > r1 + r2
x 2 + y 2 = a 2 is x12 + y12 − a 2
So, 4 common tangents can be drawn to the given circles.
EXAMPLE 3. The equations of the tangents to the Common Chord of Two Circles
circle x 2 + y 2 = a 2 parallel to the line 3x + y + 3 = 0
Let the two circles be
are
S1 ≡ x 2 + y 2 + 2g1 x + 2 f1 y + c1 = 0
a. 3x + y ± 2a b. 3x + y ± a = 0
S 2 ≡ x 2 + y 2 + 2g 2 x + 2 f 2 y + c 2 = 0
c. 3x + y ± 4 a = 0 d. None of these
The equation of the common chord of the above two
Sol. Equation of line parallel to the 3x + y + 3 = 0
circles
is 3x + y + k = 0.
S1 = 0 and S 2 = 0, is S1 − S 2 = 0.
But it is a tangent to the circle x2 + y 2 = a2, then
or 2( g1 − g 2 )x + 2 ( f1 − f 2 ) y + 2 ( c1 − c 2 ) = 0
k
=a ⇒ k = ± 2a
1+ 3 EXAMPLE 5. Find the length of common chord of the
Hence, the required equation is 3x + y ± 2a = 0 circles x 2 + y 2 + 2x + 6 y = 0 and x 2 + y 2 − 4x − 2y − 6 = 0.
2 3 2
Contact of Two Circles a. 106 b. 107 c. 108 d. None of these
5 5 5
Different cases at intersection at two circles Let the Sol. The equation of common chord of the circles
equation of two circles be
S1 ≡ x12 + y 2 + 2x + 6y = 0
( x − h1 ) + ( y − k1 ) = a12
2 2
...(i) and S 2 ≡ x2 + y 2 − 4x − 2y − 6 = 0
( x − h2 ) 2 + ( y − k2 ) 2 = a 22 ...(ii) is S1 − S 2 = 0 or 6x + 8y + 6 = 0 or 3x + 4y + 3 = 0
with centres C1 ( h1 , k1 ) and C 2 ( h2 , k2 ) and radii a1 and Centre of s1 is ( −1, − 3), radius = 1+ 9 = 10
a 2 , respectively, then the following cases of intersection Length of perpendicular from ( −1, − 3) to 3x + 4y + 3 = 0
at these two circles may arise |−3 − 12 + 3| 12
= = …(i)
9 + 16 5
(i) C1 C 2 > a1 + a 2 : two circles do not touch each
other. Length of common chord = 2 (radius) 2 − [by Eq. (i)]2
Number of common tangents = 4 = 2 10 −
144 2
= 106
(ii) When C1C 2 = a1 + a 2 : two circles touch each other 25 5
externally. Number of common tangents = 3
Angle of Intersection of Two Circles
(iii) When C1C 2 < a1 + a 2 : two circle intersect each other.
The angle of intersection θ of the two circles with
Number of common tangents = 2
centres at C1 and C 2 and the respective radii r1 and r 2 is
(iv) When C1C 2 = a1 − a 2 : two circles touch each given by
other internally.
r12 + r 22 − (C1C 2 ) 2 r12 + r 22 − d 2
Number of common tangents = 1 cos θ = =
2r1r 2 2r1r 2
EXAMPLE 4. How many common tangents can be where C1C 2 = d
drawn to the following circles x 2 + y 2 = 6 x and Two circles intersect orthogonally, if angle of
x 2 + y 2 + 6 x + 2y + 1 = 0? intersection between them is 90°, i.e. if
a. 4 b. 3 c. 2 d. 1
MATHEMATICS Circle 199

A
r1 r2
⇒ ( g12 + f12 − c1 ) 2 + ( g 22 + f 22 − c 2 ) 2
θ

C1 C2 = ( g12 + f12 + g 22 + f 22 − 2g1 g 2 − 2 f1 f 2 ) 2


⇒ 2g1 g 2 + 2 f1 f 2 = c1 + c 2
r12 + r 22 − (C1C 2 ) 2 Equation of a Circle in Parametric Form
cos 90 ° = 0 =
2r1r 2 Let P ( x, y) be any point on the circle with centre at
Thus, r1 + r 2 − (C1C 2 ) = 0
2 2 2
C ( h, k) and radius r, then x = h + r cos α and
y = k + r sin α. So, these are the parametric equations of
i.e. (C1C 2 ) 2 = r12 + r 22 the circle, where α is called parameter.

PRACTICE EXERCISE
1. For the equation ax 2 + by 2 + 2hxy + 2gx + 2 fy + c = 0, 8. Locus of the centre of the circle which always
where a ≠ 0, to represent a circle, the condition passes through the fixed points ( a , 0) and ( −a , 0) is
will be (a) x = 1 (b) x + y = 6 (c) x + y = 2 a (d) x = 0
(a) a = b and c = 0 (b) f = g and h = 0 9. Under which one of the following conditions does
(c) a = b and h = 0 (d) f = g and c = 0 the circle x 2 + y 2 + 2gx + 2 fy + c = 0 meet the
2. The equation of the circle passing through (4, 5) X-axis in two points on opposite sides of the
having the centre at (2, 2) is origin?
(a) x2 + y2 + 4x + 4 y − 5 = 0 (b) x2 + y2 − 4x − 4 y − 5 = 0 (a) c > 0 (b) c < 0 (c) c = 0 (d) c ≤ 0
(c) x2 + y2 − 4x = 13 (d) x2 + y2 − 4x − 4 y + 5 = 0 10. The centre of the circle ( x − α )2 + ( y − β )2 = 9 lies
3. Equation of circle which passes through the on the straight line x = y and the circle touches
points (1, − 2) and ( 3, − 4) and touch the X-axis is the circle x 2 + y 2 = 1 externally. What are the
(a) x2 + y2 + 6x + 2 y + 9 = 0 values of α , β?
(b) x2 + y2 + 10x + 20 y + 25 = 0
(a) α = ± 2 2 , β = ± 2 2 (b) α = ± 2 , β = ± 2
(c) x2 + y2 + 6x + 4 y + 9 = 0
(c) α = 0, β = 0 (d) α = 2 , β = 2
(d) None of the above
4. A square is inscribed in the circle x 2 + y 2 + 2gx 11. Consider a circle of radius R. What is the length
of a chord which subtends an angle θ at the centre?
+ 2 fy + c = 0 of radius r, then length of its side is θ θ
(a) r (b) r 2
1
(c) r (d) 2 (a) 2 Rsin   (b) 2Rsin θ (c) 2 R tan   (d) 2R tan θ
2 2
2
5. The lines 2x − 3 y = 5 and 3x − 4 y = 7 are 12. What is the equation to the circle which touches
diameters of a circle of area 154 sq units. Then, both the axes and has centre on the line
the equation of this circle is x + y = 4?
(a) x2 + y2 + 2 x − 2 y = 62 (b) x2 + y2 + 2 x − 2 y = 47 (a) x2 + y2 − 4x + 4 y + 4 = 0
(c) x + y − 2 x + 2 y = 47
2 2
(d) x + y − 2 x + 2 y = 62
2 2
(b) x2 + y2 − 4x − 4 y + 4 = 0
6. The radius of the circle passing through the (c) x2 + y2 + 4x − 4 y − 4 = 0
point P ( 6, 2), two of whose diameter are x + y = 6 (d) x2 + y2 + 4x + 4 y − 4 = 0
and x + 2 y = 4 is
(a) 10 (b) 2 5 (c) 6 (d) 4
13. A square is inscribed in a circle
x 2 + y 2 − 2x + 4 y + 3 = 0. Its sides are parallel to
7. Equation of a circle passing through ( −1, − 2) and
the coordinate axes. Then, one of the vertex of
concentric with the circle x 2 + y 2 − 3x + 4 y − c = 0 the square is
is (a) (1 + 2 , − 2 )
(a) x2 + y2 − 3x + 4 y − 1 = 0 (b) x2 + y2 − 3x + 4 y = 0 (b) (1 − 2 , − 2 )
(c) x2 + y2 − 3x + 4 y + 2 = 0 (d) None of these (c) (1, − 2 + 2 )
(d) None of the above
200 NDA/NA Pathfinder

(a) x2 + y2 = a2 (b) x2 + y2 + a2 = 0
14. Consider the following statements
I. Number of circles touching the given three (c) x + y + 2 x + 2 y = a
2 2 2
(d) x2 + y2 − 2 x − 2 y = a2
non-concurrent lines is 4. 22. How many tangents can be drawn from the
II. Number of circles passing through (1, 2), (4, 8) origin to the circle that we get in above question?
and (0, 0) is one. (a) 0 (b) 1 (c) 2 (d) 3
Which of the above statement(s) is/are correct?
(a) Only I (b) Only II Directions (Q. Nos. 23-25) Consider the two circles
(c) Both I and II (d) None of these S1 ≡ x 2 + y 2 = 16 and S 2 ≡ x 2 + y 2 − 8 x − 10y = 23
15. Consider the following statements with centres C1 and C 2, respectively and radius r1 and r2,
I. The equation x2 + y2 + 2x − 10 y + 30 = 0, respectively.
represents the equation of a circle. 23. Which of the following is true?
II. If point (0, g ) lies inside the circle (a) r1 > r2 (b) r1 < r2 (c) r1 = r2 (d) r2 > r1
x2 + y2 + 2 gx + c = 0, then c < 0. 24. Find C1 C2.
Which of the above statement(s) is/are correct? (a) 2 (b) 29 (c) 41 (d) 5
(a) Only I (b) Only II
(c) Both I and II (d) Neither I nor II
25. Find the number of common tangents drawn to
S1 and S 2.
16. Consider the following circles (a) 2 (b) 1 (c) 3 (d) 4
I. x2 + y2 + 4x − 6 y − 12 = 0
II. x2 + y2 − 12x − 14 y + 60 = 0 Directions (Q. Nos. 26-28) Consider the two circles
III. x2 + y2 − 10x + 8 y + 18 = 0 S1 ≡ x 2 + y 2 − 6 x + 4y + 11 = 0 and
Which of the above circles has equal area? S 2 ≡ x 2 + y 2 − 4x + 6y + 9 = 0.
(a) I, II (b) I, III (c) II, III (d) I, II, III
26. The equation of common chord is
Directions (Q. Nos. 17-18) Consider the circle (a) y − x = 1 (b) x − y = 1
(c) x + y − 1 = 0 (d) x + y = 1
S ≡ x + 4x + (y − 3) = 0.
2 2

27. The angle of intersection of the two circle is


17. From the point A( 0, 3) on the circle a chord AB is (a) 30° (b) 45° (c) 60° (d) 90°
drawn and extended to a point P. Such that
AP = 2 AB. The locus of P is 28. The value of k so that x + y + kx + 3 y + k = 0
2 2

(a) x + ( y − 3) = 0
2 2
(b) x + 4x + ( y + 3) = 0
2 2 and circle S1 cut orthogonally, is
8 8 5 5
(c) x2 + 8x + ( y − 3)2 = 6 (d) (x + 4)2 + ( y − 3)2 = 16 (a) (b) − (c) (d) −
3 3 4 4
18. The distance of the origin from the centre of S is
7 Directions (Q. Nos. 29-32) Consider the two circles
(a) (b) 4 (c) 13 (d) 15
2 S1 ≡ x 2 + y 2 − 4 = 0 and S 2 ≡ x 2 + y 2 − 6 x − 8y − 24 = 0.

Directions (Q. Nos. 19-20) Consider the circle 29. The two circles S1 and S 2
S ≡ x 2 + y 2 − 6 x + 12y + 15 = 0. (a) touch each other externally
(b) touch each other internally
19. The equation of circle which is concentric with (c) cuts each other at two points
circle S and has area double of its area is
(d) None of the above
(a) x2 + y2 − 6x + 12 y − 15 = 0
(b) x2 + y2 − 6x + 12 y + 40 = 0 30. The number of common tangents to the given
circles is
(c) x2 + y2 − 6x + 12 y + 45 = 0
(a) 0 (b) 1 (c) 3 (d) 4
(d) None of the above
31. The number of tangents from centre of circle S 2
20. Which of the following line is a diameter of the
to the circle S1 is
circle S?
(a) 0 (b) 1 (c) 2 (d) None of these
(a) 2 y + 3x + 3 = 0 (b) 3 y + 2 x + 7 = 0
(c) x + y − 3 = 0 (d) x − y + 9 = 0 32. Consider the following statements
I. Equation of common chord is 3x + 4 y + 10 = 0.
Directions (Q. Nos. 21-22) A circle always passes
II. Equation of common tangent is 4 y + 3x = 5.
through the fixed points (a, 0) and ( −a, 0).
Which of the above statement(s) is/are correct?
21. If given points are the ends of diameter, then the (a) Only I (b) Only II
equation of circle is
(c) Both I and II (d) None of these
MATHEMATICS Circle 201

PREVIOUS YEARS’ QUESTIONS 40. If the centre of the circle passing through the
origin is (3, 4), then the intercepts cut-off by the
33. What is the radius of the circle touching X-axis circle on X-axis and Y-axis, respectively are
at (3, 0) and Y-axis at (0, 3)? e 2012 II
e 2015 II
(a) 3 units (b) 4 units (c) 5 units (d) 6 units (a) 3 units and 4 units (b) 6 units and 4 units
34. The angle subtended at the centre of a circle of (c) 3 units and 8 units (d) 6 units and 8 units
radius 3 cm by an arc of length 1 cm is e 2012 II 41. If a circle of radius b units with centre at ( 0, b)
30° 60°
(a) (b) (c) 60° (d) 30° touches the line y = x − 2, then what is the
π π
value of b? e 2016 I
35. Which one of the following points lies inside a (a) 2 + 2 (b) 2 − 2 (c) 2 2 (d) 2
circle of radius 6 and centre at (3, 5)? e 2013 I
(a) (−2, − 1) (b) (0, 1) (c) (−1, − 2 ) (d) (2, − 1) Directions (Q. Nos. 42-43) Consider the two circles
36. The radius of the circle x + y2 + x + c = 0
2 ( x − 1) 2 + (y − 3) 2 = r 2 and x 2 + y 2 − 8 x + 2y + 8 = 0
e 2016 I
passing through the origin is e 2013 II
(a)
1
(b)
1
(c) 1 (d) 2
42. What is the distance between the centres of the
4 2 two circles?
(a) 5 units (b) 6 units (c) 8 units (d) 10 units
Directions (Q. Nos. 37-38) Read the following
information carefully and answer the questions
43. If the circles intersect at two distinct points,
then which one of the following is correct?
given below.
(a) r = 1 (b) 1 < r < 2 (c) r = 2 (d) 2 < r < 8
Consider the circles x 2 + y 2 + 2 ax + c = 0 and
x 2 + y 2 + 2 by + c = 0. e 2014 II Directions (Q. Nos. 44-45) Consider a circle passing
37. What is the distance between the centres of the through the origin and the points ( a, b) and ( − b, − a).
two circles? e 2016 I
(a) a2 + b 2 (b) a2 + b 2 44. On which line does the
(c) a + b (d) 2(a + b ) centre of the circle lie?
38. The two circles touch each other, if (a) x + y=0
1 1 1 (b) x − y=0
(a) c = a2 + b 2 (b) = + 2 (c) x + y=a+ b S=0
c a2 b L=0
1 1 1 (d) x − y = a2 − b 2 S + λL = 0
(c) c = + (d) c = 2
a2 b2 a + b2 45. What is the sum of the squares of the intercepts
cut-off by the circle on the axes?
39. A straight line x = y + 2 touches the circle 2 2
4 ( x 2 + y 2 ) = r 2. The value of r is  a2 + b 2   a2 + b 2 
e 2015 II (a)  2  (b) 2  
(a) 2 (b) 2 2  a − b2   a−b 
2
(c) 2 (d) 1  a2 + b 2 
(c) 4   (d) None of these
 a−b 

ANSWERS
1 c 2 b 3 b 4 b 5 c 6 b 7 b 8 d 9 b 10 b
11 a 12 b 13 d 14 a 15 b 16 a 17 d 18 c 19 a 20 a
21 a 22 a 23 b 24 c 25 b 26 c 27 b 28 d 29 b 30 b
31 c 32 a 33 a 34 b 35 b 36 b 37 a 38 b 39 b 40 d
41 a 42 a 43 b 44 a 45 b
HINTS AND SOLUTIONS
1. (c) The general equation of circle is 7. (b) The terms of x and y will remain ∴ Length of the chord
x 2 + y 2 + 2 gx + 2 fy + c = 0. …(i) the same as the two circles are θ
concentric. The new constant λ = 0 as it AB = 2 AD = 2R sin  
Hence, on comparing the given equation  2
passes through ( −1, − 2 ). By putting
with Eq. (i), we get point (−1, −2) in the equation. 12. (b) We know that, the equation of circle,
a = b and h = 0 which touches both the axes, is
8. (d) If ( h , k ) is centre C and A , B be the
2. (b) Here, r = Distance between (4, 5) x 2 + y 2 − 2rx − 2ry + r 2 = 0 …(i)
given points, then
and (2, 2) The centre ( r , r ) of this circle lies on the
∴ r 2 = 4 + 9 = 13 CA 2 = CB 2 ⇒ 4ah = 0 ⇒ h = 0 line x + y = 4.
⇒ ( x − 2)2 + ( y − 2)2 = 13 ∴ x =0 ∴ r + r = 4 ⇒r = 2
⇒ x + y − 4x − 4 y − 5 = 0
2 2
9. (b) The circle On putting the value of r in Eq. (i), we
get x 2 + y 2 − 4x − 4 y + 4 = 0
3. (b) Since, the circle touches X-axis. x 2 + y 2 + 2 gx + 2 fy + c = 0,
which is the required equation of the
∴ ( x − h) + ( y − k ) = k
2 2 2
…(i) meets X-axis ( y = 0) in two points on circle.
Also, it passes through given points opposite sides of origin.
13. (d) Centre of the circle is ( 1, − 2) and
( 1 − h ) 2 + ( −2 − k ) 2 = k 2 …(ii) It means x 2 + 2 gx + c = 0 sides of the inscribed square are parallel
and (3 − h )2 + ( −4 − k )2 = k 2 …(iii) − 2g ± 4 g 2 − 4c to the coordinate axes. Hence, no vertex
On subtracting Eq. (iii) from Eq. (ii), we ⇒ x = of the square can have its coordinates x
2 as 1 and y as − 2.
get h = k + 5
On solving these equations, we get =− g± g2 − c
14. (a)
k = −10,−2 and h = − 5, 3
Circle meets the X-axis in two points on 15. (b) I. The radius of this circle
By putting the values of opposite side of origin = 12 + ( −5)2 − 30 = ( −4)
( h , k ) = ( −5, − 10) or (3, − 2) in Eq. (i),
Hence, which is imaginary.
we get x 2 + y 2 + 10x + 20 y + 25 = 0
− g+ g 2 − c > 0, − g − g 2 − c < 0 So, Statement I is false.
4. (b) If a is the side of the square inscribed II. Since, ( 0, g ) lies inside
in a circle of radius r, then a 2 + a 2 = ⇒ g2 − c > g ⇒ g2 − c > g2 ⇒ c < 0
S ≡ x 2 + y 2 + 2 gx + c = 0
(Diameter)2
∴ 2a 2 = 4r 2 10. (b) The centre and radius of circle ∴ S = 0 + g2 + 0 + c < 0
1
⇒ a=r 2 ( x − α )2 + ( y − β )2 = 9 are (α , β ) and
⇒ g2 + c < 0 ⇒ c < 0 [Q g 2 > 0]
5. (c) If r is radius of the circle, 3, respectively.
So, Statement II is true.
then πr 2 = 154 Since, (α , β ) lies on the straight line
y = x. 16. (a) I. Centre = ( − 2, 3)
= 49  taking π =
7 22 
∴ r 2 = 154 × Radius = ( −2)2 + 32 + 12
22  7  ∴ α =β …(i)
= 25 = 5
⇒ r=7 Now, this circle touches the circle
∴ Area = 25 π
x 2 + y 2 = 1 externally.
Also, solving the equation of two given II. Centre = ( 6, 7)
diameters, we get the coordinates of the ∴α 2 + β 2 = 3 + 1 ⇒ α 2 + β 2 = 4 Radius = 62 + 72 − 60 = 25 = 5
centre as ( 1, − 1).
⇒ 2α 2 = 4 [using Eq. (i)] ∴ Area = 25 π
Hence, the equation of the circle is
⇒ α=± 2 III. Centre = (5, − 4)
( x − 1)2 + ( y + 1)2 = 72 = 49
∴ α=± 2 and β = ± 2 Radius = 52 + ( − 4)2 − 18 = 23
⇒ x 2 + y 2 − 2x + 2 y = 47 ∴ Area = 23 π
11. (a) In ∆ADO,
6. (b) The centre of circle is the intersection So, I and II have equal area.
θ AD AD
of equations of the diameter sin = = 17. (d) Let the coordinates of P be ( h , k ).
x + y=6 ...(i) 2 OA R
Then, B is the mid-point of AP. So, the
and x + 2y = 4 ...(ii) h k + 3
coordinates of B are  , .
On solving Eqs. (i) and (ii), we get 2 2 
x =8 O Since, B lies on the circle.
and y=−2
R R
∴ Radius of circle is the distance θ/2 θ/2
between P( 6, 2) and C (8, − 2)
A D B
Radius = (8 − 6)2 + ( 2 + 2)2 (h , k )
θ A(0, 3)
⇒ AD = R sin B P
=2 5 2
MATHEMATICS Circle 203

x 2 + 4x + ( y − 3)2 = 0 P tangent is same as equation of


2
k+3
2 common chord.
+ 4  +  − 3 = 0
h h (r2)
∴ 2 √2 (r1)
4  2  2  θ So, Statement II is false.
⇒ h 2 + 8h + ( k − 3)2 = 0 C2 C1 33. (a) Radius of the circle, AC = BC
Hence, the locus of P ( h , k ) is Y
x 2 + 8x + ( y − 3)2 = 0.
Let r and r be their radii
18. (c) We have, 1 2
C(3, 3)
(0, 3) B
S ≡ x 2 + 4x + y 2 + 9 − 6 y = 0 r = 9 + 4 − 11 = 2
1
or S ≡ x 2 + y 2 + 4x − 6 y + 9 = 0 and r = 4+ 9−9 = 2 3
2
Centre of circle, S = ( − 2, 3) Also,
O 3 A (3, 0) X
Required distance from origin C C = (3 − 2 ) 2 + ( −2 + 3 ) 2 = 2
1 2
= ( −2 ) + (3 ) =
2 2
13 AC = OB = 3 and BC = OA = 3
Suppose, two circles intersect at P.
∴ Radius = 3 units
19. (a) Equation of circle concentric to given Then,
circle is C P2 + C P2 − C C 2 34. (b) Length of arc = angle subtended at
cos ∠C PC =
1 2 1 2 l 1
S ≡ x + y − 6x + 12 y + k = 0 ...(i)
2 2 centre × radius ⇒ Angle = =
1
1 2
2C P ⋅ C P r 3
1 2
As, area of circle (S ) 2+ 4−2 1 1 180° 60°
1
= = ⇒ Angle = × =
= 2 area of (given) circle (S) 2× 2 ×2 2 3 π π
∴ Radius of circle (S ) = 2 ⇒ cos ∠C PC = cos 45° 35. (b) The equation of the circle of radius 6
1
1 2
[radius of given circle] and centre at (3, 5) is
⇒ ∠C PC = 45°
1 2 ( x − 3)2 + ( y − 5)2 = ( 6)2
⇒ 9 + 36 − k = 2 9 + 36 − 15 28. (d) The equations of the circles are Let S ≡ ( x − 3)2 + ( y − 5) 2 − 36 = 0
⇒ 45 − k = 60 ⇒ k = − 15 S ≡ x 2 + y 2 − 6x + 4 y + 11 = 0 (a) At point ( − 2, − 1),
1
Hence, the required equation of circle is S ≡ x 2 + y 2 + kx + 3 y + k = 0 S ≡ ( − 2 − 3)2 + ( −1− 5)2 − 36
3
x + y − 6x + 12 y − 15 = 0
2 2
These two circles cut orthogonally, if = 25 + 36 − 36 = 25 > 0
20. (a) Centre of the circle S is (3, − 6). which represents the point lies outside
2  − 3 × + 2 ×  = 11 + k
k 3
 the circle.
Since, centre (3, − 6) is satisfying only 2 2
(b) At point ( 0, 1),
2 y + 3x + 3 = 0 [Q 2( g g + f f ) = C C ]
1 2 1 2 1 2 S ≡ ( 0 − 3)2 + ( 1− 5)2 − 36
∴ 2 y + 3x + 3 = 0 is the required line. ⇒ − 3k + 6 = 11 + k ⇒ k = −
5 = 9 + 16 − 36 = − 11 < 0
4 which represents the point lies inside
21. (a) Since, (a, 0) and ( −a , 0) are the end
29. (b) Centre of circle S is C ( 0, 0). the circle.
points of diameter, then equation of the 1 1
Centre of circle S is C (3, 4). (c) At point ( −1, − 2),
circle is 2 2
( x − a ) ( x + a ) + ( y − 0) ( y − 0) = 0 S ≡ ( −1− 3)2 + ( − 2 − 5)2 − 36
r = 2 and r = 32 + 42 + 24 = 7
⇒ x 2 − a2 + y 2 = 0 ⇒ x 2 + y 2 = a2
1 2 = 16 + 49 − 36 = 29 > 0
Now, C C = 32 + 42 = 5 which represents the point lies outside
22. (a) Since, origin lies inside the circle 1 2 the circle.
x 2 + y 2 = a 2 . Hence, no tangent can be C C < Sum of the radii (d) At point (2, −1),
drawn. 1 2
S ≡ ( 2 − 3)2 + ( −1 − 5)2 − 36
Also, C C = Difference of the radii
23. (b) From circle S , r = 16 = 4
1 2 = 1 + 36 − 36 = 1 > 0
1 1 Thus, the two circles touch each other
which represents the point lies outside
From circle S , r = 16 + 25 + 23 = 8 internally.
2 2 the circle.
Clearly, r <r 30. (b) Since, the two circles touch each
1 2 Hence, point ( 0, 1) lies inside the circle S.
24. (d) Here, C ( 0, 0) and C ( 4, 5) other internally. Therefore, there is only
1 2 36. (b) Given equation of circle is
∴ CC = 16 + 25 = 41 one common tangent.
1 2 x2 + y 2 + x + c = 0 ...(i)
25. (b) Here, |r − r |< C C < | r + r | 31. (c) We have,
1 2 1 2 1 2 Since, the equation of circle passes
Clearly, circles cut each other at two S = (3)2 + ( 4)2 − 4 = 21 > 0 through the origin.
1
points. Hence, number of common ∴ Point (3, 4) lies outside the circle S . ∴( 0)2 + ( 0)2 + 0 + c = 0 ⇒ c = 0
1
tangents will be 2. ∴Two tangents can be drawn from (3, 4). From Eq. (i), we get
26. (c) Equation of common chord is given by 32. (a) I. Equation of common chord is x2 + y 2 + x = 0
S −S =0 given by S − S = 0
1 2 1 1
⇒ − 6x + 4 y + 11 − ( − 4x + 6 y + 9) = 0
1 2
i.e. − 4 + 6x + 8 y + 24 = 0 ⇒ x2 + y 2 + x + =
4 4
⇒ − 2x − 2 y + 2 = 0 ⇒ x + y − 1 = 0 or 3x + 4 y + 10 = 0 2 2
⇒  x +  + ( y − 0)2 =  
1 1
27. (b) Centre of circle S is C (3, − 2). So, Statement I is true.  2  2
1 1
Centre of circle S is C ( 2, − 3) II. Since, two circles touch each other 1
2 2 So, the required radius of circle is .
internally, equation of common 2
204 NDA/NA Pathfinder

37. (a) Equations of circle are Hence, the x-intercept is 6 and x 2 + y 2 + 2 gx + 2 fy + c = 0


x 2 + y 2 + 2ax + c = 0 y-intercept is 8. When x = 0, y = 0
and x 2 + y 2 + 2by + c = 0 41. (a) Here, radius of circle = b and centre ⇒ c=0 ...(i)
Q Centres of two circles are ( −a , 0) and = ( 0, b ) and the equation of line touches
( 0, − b ). When x = a, y = b
the circle is y = x − 2
∴ Distance between two centres a 2 + b 2 + 2 ga + 2 fb = 0 [Q c = 0] ...(ii)
= a2 + b2 When x = − b, y = − a
38. (b) Two circles touch each other, iff y = x – √2 b 2 + a 2 − 2 gb − 2 fa = 0 …(iii)
(0, b)
Distance between two centres A Subtracting Eq. (iii) from Eq. (ii), we get
O b
= Sum of radius of two circles 2 g( a + b ) + 2 f ( a + b ) = 0
⇒ a2 + b2 = a2 − c + b2 − c ⇒ 2( g + f )( a + b ) = 0
On squaring both sides, we get ⇒ g + f = 0 ⇒ g = − f …(iv)
∴ Perpendicular drawn from centre to
a2 + b2 = a2 − c + b2 − c Putting g = − f in Eq. (ii), we get
the line is
+ 2 ( a 2 − c )( b 2 − c ) a 2 + b 2 − 2 fa + 2 fb = 0
0−b− 2  −( b + 2 ) 
 =
b = 
⇒ c = ( a 2 − c )( b 2 − c ) 1  a2 + b2 
 2   2  ⇒ f =  
2 a−b 
Again, squaring both sides, we get ⇒ 2b = b + 2 ⇒ b( 2 − 1) = 2
Putting f = − g in Eq. (ii), we get
c2 = a 2 b 2 − a 2 c − b 2 c + c2 2 ( 2 + 1)
⇒ b= × a 2 + b 2 + 2 ga − 2 gb = 0
1 1 1 2 − 1 ( 2 + 1)
⇒ a2b2 = ( a2 + b2 ) c ⇒ = 2 + 2
c a b 1  a2 + b2 
2+ 2 ⇒ g=−  
r2 = = (2 + 2) 2 a−b 
39. (b) Given, x2 + y 2 = 2− 1
4
44. (a) From Eq. (iv), we have
We know that, the line y = mx + c meets 42. (a) Given circle are
g = − f ⇒ y = −x
the circle in unique real point or touch ( x − 1)2 + ( y − 3)2 = r 2 ...(i)
the circle x2 + y 2 = r 2 , if ∴Required equation of line is x + y = 0
and x 2 + y 2 − 8x + 2 y + 8 = 0 ...(ii)
c 45. (b) Intercept on
r= Centre of circle (i) = (1, 3)
1+ m 2 X - axis = g 2 − c = 2 g 2 [Q c = 0]
and radius r =r
1
Since, the straight line x = y + 2 Intercept on Y - axis
Centre of circle (ii) = ( 4, − 1)
touches the given circle. = 2 f 2 −c = 2 f 2
[Q c = 0]
2 r and radius ( r ) = 16 + 1 − 8 = 9 =3
Hence, = ⇒r = 2 2 2
Sum of square of intercepts
2 2 ∴ Distance between centres
= ( x -intercept)2 + ( y - intercept)2
40. (d) We have, centre = (3, 4) = (1 − 4)2 + (3 + 1)2 = 4g2 + 4 f 2
and radius = 5
= 9 + 16 = 25 = 5 units 1  a2 + b2 
2
Equation of circle having centre ( h , k ) =4 ×  
and radius a is 43. (b) Condition for two circles intersect at 4 a−b 
( x − h )2 + ( y − k )2 = a 2 two distinct points. 2
1  a2 + b2 
⇒ ( x − 3) + ( y − 4) = 25
2 2 + 4× × 
Distance between centre < Sum of radius 4  a−b 
For x-intercept 5< r + 3 ⇒ 2< r 2
a + b 
2 2
Put y = 0, we get ( x − 3)2 + 16 = 25
But r > 1 because 5 </ 1 + 3 =2 
⇒ ( x − 3)2 = 9  a−b 
⇒ x − 3 = 3 and − 3 ⇒ x = 6 and 0 Sol. (Q. Nos. 44-45) Given that circle passes
For y-intercept through (0, 0), ( a , b ) and ( − b , − a ).
Put x = 0, we get 9 + ( y − 4)2 = 25 Now, equation of circle is
⇒ y − 4 = 4 and − 4 ⇒ y = 8 and 0
18
MATHEMATICS Conic Section 205

CONIC SECTION

In NDA exam, generally 2-4 questions are asked from this chapter which are based on finding
equation of conics, eccentricity conics, various terms related to conics etc.

A conic section or conic is the locus of a point which moves in such a way that its distance from a
fixed point always bears a constant ratio to its perpendicular distance from a fixed line, all being in the
same plane. Shape of the conic section obtained depends on the position of cutting plane. This section
deals with parabola, hyperbola and ellipse etc.

Various Terms Related to Conic Section


1. Focus The fixed points is called the focus of the conic section.
2. Directrix The fixed straight line is called the directrix of the conic section.
3. Eccentricity The constant ratio of distance of point lying on conic, from focus to its perpendicular
distance from directrix is called the eccentricity of the conic section and is denoted by e.
(i) For an ellipse, e < 1 (ii) For a parabola, e = 1 (iii) For hyperbola, e > 1
(iv) For a circle, e = 0 (v) Pair of straight lines, e = ∞
4. Axis The straight line passing through the focus and perpendicular to the directrix, is called the axis
of the conic section.
5. Vertex The point of intersection of the conic section and its axis, is called vertex of the conic section.
6. Centre The point which bisects every possible chord of the conic passing through it, is called the
centre of conic.
7. Latusrectum The latusrectum of a conic is the chord passing through the focus and perpendicular to
the axis.
8. Focal chord Any chord passing through the focus of a conic is called the focal chord of the conic.
9. Double ordinate Any chord perpendicular to the axis of a conic is called the double ordinate of that
conic.
10. Focal distance or focal length The distance between the focus and a point lying on the conic, is
known as focal distance or focal length of the given point.

General Equation of Conics of Second Degree


General equation of conics of second degree viz.
ax 2 + 2hxy + by 2 + 2gx + 2 fy + c = 0
where, discriminant ∆ = abc + 2 fgh − af 2 − bg 2 − ch 2
206 NDA/NA Pathfinder

The above equation represents a non-degenerate 2. Form x 2 = 4ay In this form the focus of the
conic whose nature is as follows parabola lie in the positive side of Y-axis
S.No. Condition Nature of conic Y

1. ∆ ≠ 0, h = 0, a = b, e = 0 A circle
S (0,a)
2. ∆ ≠ 0, ab − h = 0, e = 1
2
A parabola

3. ∆ ≠ 0, ab − h2 > 0, e < 1 An ellipse

4. ∆ ≠ 0, ab − h < 0, e > 1
2
A hyperbola
X′ X
O (0, 0)
5. ∆ ≠ 0, ab − h < 0, a + b = 0,
2
A rectangular a
e = 1/ 2 hyperbola
y=–a
Y′
x = 4ay
PARABOLA 2

A parabola is the locus of a point which is equidistant 3. Form x 2 = − 4 ay In this form the focus of the
from a fixed point called focus and from a fixed straight parabola lie in the negative of Y-axis.
line called directrix. Y y=a
Directrix
Z Y a
(x, y) L(a, 2a) O (0, 0)
P X′ X
M Latusrectum

Vertex Focus axis


X′ X
O (0 , 0) S(a, 0)
S (0, – a)
a
x = –a y2 = 4ax
Y′
Z′ Y′ L′ (a,– 2a) x 2 = – 4ay

Since, point P( x, y) lies on the parabola. Some Important Results Related to Parabola
PS
∴ =e =1 [here, e = 1] y 2 = 4 ax y 2 = − 4 ax x 2 = 4 ay x 2 = − 4 ay
PM
⇒ PS 2 = PM 2 Eccentricity e =1 e =1 e =1 e =1

⇒ ( x − a) + ( y − 0 ) = ( x + a)
2 2 2 Coordinates of ( 0, 0) ( 0, 0) ( 0, 0) ( 0, 0)
vertex
⇒ y 2 = 4ax Coordinates of ( a, 0) ( −a, 0) ( 0, a ) ( 0, − a )
which is the equation of parabola in standard form. focus
Equation of x= −a x=a y= −a y=a
Note The distance between vertex of parabola and focus is equal the directrix
to perpendicular distance of vertex from directrix.
or Vertex is the mid-point of line joining focus and point of Equation of y=0 y=0 x=0 x=0
intersection of directrix and axis. the axis
Length of the 4a 4a 4a 4a
Other Standard Forms of Parabola latusrectum

1. Form y 2 = − 4 ax In this form the focus of the Focal distance


of a point x+ a a−x y+ a a− y
parabola lie in the negative side of X-axis. P( x, y)
Y
Directrix Extremities of ( a, ± 2 a ) ( − a, ± 2 a ) ( ± 2 a, a ) ( ± 2 a, − a )
latusrectum
a Parametric
X′ ( at 2, 2 at ) ( − at 2, 2 at ) (2 at , at 2 ) (2 at , − at 2 )
X coordinates
S (– a, 0) O(0,0)
x=a Parametric x = at 2, x = − at 2, x = 2 at , x = 2 at ,
equation y = 2 at y = 2 at y = at 2 y = − at 2
Y′
y 2 = – 4ax
MATHEMATICS > Conic Section 207

Parabolas with Vertex Other than (0, 0) Shifting the origin to the point (3, 4) without rotating the
axes and denoting the new coordinates with respect to
If the vertex of the parabola is at the point A ( h, k) and these new axes by X and Y , we have
its latusrectum is of length 4a, then its equation is Y y2 – 8y – x + 19 = 0
(i) ( y − k) 2 = 4a( x − h), if its axis is parallel to OX, i.e.
parabola opens rightward.
A(3, 4)
(ii) ( y − k) 2 = − 4a( x − h), if its axis is parallel to OX ′,
S 13 , 4
i.e. parabola opens leftward. 4

(iii) ( x − h) 2 = 4a ( y − k), if its axis is parallel to OY, i.e. X′


O x=3 X
Y′
parabola opens upward.
X =x−3 …(ii)
(iv) ( x − h) 2 = − 4a ( y − k), if its axis is parallel to OY ′, Y=y−4
i.e. parabola opens downward. Using these relations, Eq. (i) reduces to Y 2 = X …(iii)
This is of the form Y 2 = 4aX. On comparing, we get
EXAMPLE 1. Find the equation of the parabola, 4a = 1 or a = 1/ 4
whose focus is at (−1, − 2) and the directrix on the line (i) Vertex
x − 2y + 3 = 0. The coordinates of the vertex with respect to the new
a. x2 + 4 y 2 + 4 xy + 32x + 16 = 0 axes are ( X = 0, Y = 0).
So, the coordinates of the vertex with respect to the
b. 4 x2 + y 2 + 4 xy + 32y + 16 = 0
old axes are (3, 4). [putting X = 0, Y = 0 in Eq. (ii)]
c. 4 x2 + y 2 + 4 xy + 32x + 16 = 0 (ii) Focus
d. None of the above The coordinates of the focus with respect to the new
axes are ( X = a, Y = 0), i.e.  X = , Y = 0 .
1
Sol. Let P( x, y) be any point on the parabola, whose focus
 4 
is S( −1, − 2) and the directrix x − 2y + 3 = 0. Draw PM
perpendicular from P( x, y) on the directrix x − 2y + 3 = 0. So , the coordinates of the focus with respect to the
old axes are  , 4 . [putting X = and Y = 0 in Eq. (ii)]
Then by definition, 13 1
4  4
SP = PM ⇒ SP 2 = PM 2
2
 x − 2y + 3 (iii) Directrix
⇒ ( x + 1) 2 + ( y + 2) 2 =  
 1+ 4  The equation of the directrix with respect to the new
1
⇒ 5[( x + 1) 2 + ( y + 2) 2 ] = ( x − 2y + 3) 2 axes is X = − a i.e. X = −
4
⇒ 5( x2 + y 2 + 2x + 4y + 5)
So, the equation of the directrix with respect to the old
= ( x2 + 4y 2 + 9 − 4xy + 6x − 12y) −1 11  −1
⇒ 4x + y + 4xy + 4x + 32y + 16 = 0
2 2 axes is x = + 3⇒ x = putting x = in Eq. (ii ) 
4 4  4 
Which is the equation of the required parabola.
Position of a Point with
EXAMPLE 2. If the equation of parabola
y 2 − 8y − x + 19 = 0, find Respect to a Parabola
(i) vertex Let S ≡ y 2 − 4ax = 0 be the equation of a parabola, then
a. ( 0 , 0) b. ( 3, 4) c. ( 4 , 4) d. ( 4 , 3) a point P ( x1 , y1 ) in the XY-plane lies
(ii) focus (i) on the parabola, if S1 = 0
3   13   4 (ii) outside the parabola, if S1 > 0
a. (1, 4) b.  , 3 c.  , 4 d.  3, 
4  4   3
(iii) inside the parabola, if S1 < 0
(iii) directrix
11 11 4 4 where, S1 ≡ y12 − 4ax1
a. x = b. y = c. x = d. y =
4 4 11 11
EXAMPLE 3. The portion of point (1, 4) with respect
Sol. We have, the equation of parabola y 2 − 8y − x + 19 = 0 to the parabola y 2 + 9 − 6 y = 5x, is
y 2 − 8y − x + 19 = 0 ⇒ y 2 − 8y = x − 19 a. on curve/parabola b. outside parabola
⇒ y 2 − 8y + 16 = x − 19 + 16 ⇒ ( y − 4) 2 = ( x − 3) ...(i) c. inside parabola d. None of these
208 NDA/NA Pathfinder

Sol. . Let S = y 2 + 9 − 6y − 5x So, the PQ and AP are perpendicular to each other


 y − 2
Now, point (1, 4) lies on S. ∴  1  = − 1 ⇒ y1 − 2 = − x1 + 2 ⇒ x1 + y1 = 4…(ii)
Thus, S1 = ( 4) 2 + 9 − 6( 4) − 5(1) = 16 + 9 − 24 − 5  x1 − 2 
= 25 − 29 = − 4 < 0 From Eqs. (i) and (ii), y1 = − 4 and 2
Hence, the point lies inside the parabola. So, the coordinates of point Q are (8, −4).
Equation of Chord ∴ Required distance, PQ = (8 − 2) 2 + ( −4 − 2) 2
The equation of chord joining the points ( x1 , y1 ) and = 36 + 36 = 72 = 6 2
( x 2 , y 2 ) on the parabola y 2 = 4ax is
y ( y1 + y 2 ) = 4ax + y1 y 2
ELLIPSE
An ellipse is the locus of that point which moves in a
Condition for the Chord to be a Focal Chord plane such that the ratio of its distance from a fixed
The chord joining the points ( at 12 , 2at 1 ) and ( at 22 , 2at 2 ) point (focus) to a fixed straight line (directrix) is
constant and less than unity. This constant ratio is called
passes through focus provided t 1t 2 = −1
eccentricity and denoted by e.
Tangents and Normals to the Parabola Or
The locus of a point whose sum of distance from two
Equation of tangent to the parabola y 2 = 4ax fixed points remains constant is called an ellipse.
(i) at ( x1 , y1 ) is y y1 = 2a( x + x1 ) Y
Z' Z
(ii) at ( at 2 , 2at ) is ty = x + at 2 Minor axis
B (0, b)

A′ (–a,0)
P (x, y)
(iii) If m is the slope of tangent to the parabola y 2 = 4ax, then M
a  a 2a  A(a,0)
its equation is y = mx + . Point of contact is  2 ,  . X′ X
m  m m S′(– ae, 0) O (0, 0) S (ae, 0) Major
axis
Equation of normal to the parabola y = 4ax 2
x=
a
Directrices B′ (0, –b) Directrices e
y
(i) at ( x1 , y1 ) is y − y1 = − 1 ( x − x1 ) x = a–
e Y′
2a 2
= e, PS 2 = e2PM 2, ( x − ae) 2 + ( y − 0) 2 = e2  − x
PS a
(ii) at ( at 2 , 2at ) is y + tx = 2at + at 3
PM e 
(iii) If m is the slope of normal to the parabola y 2 = 4ax, x2 y2 x2 y 2
then its equation is given as y = mx − 2am − m3 . ⇒ + =1 ⇒ 2 + 2 =1
a 2 a2(1 − e2) a b
Where, b = a (1 − e ), b < a
2 2 2
EXAMPLE 4. Let P(2, 2) is a point on the parabola
This is the equation of ellipse in standard form. Here,
y 2 = 2x and A is its vertex. If Q is another point on the
AA′ = 2a and BB′ = 2 b are called major and minor axes,
parabola such that PQ is perpendicular to AP, then respectively.
what is the length of PQ?
a. 2 b. 2 2 c. 4 2 d. 6 2 Another Form of Ellipse
x2 y2
The another equation of the ellipse is 2 + 2 = 1
Sol. Equation of parabola is y = 2x. 2
a b
Y
[where, b > a].
) Y
2 ,2 Z y = b/e
P(
Directrix Major axis
B
X′ X
A(0, 0) S S
(0, be)
A′ A
Q (x1, y1) X′ X
Y′ (– a,0) O (0, 0) (a, 0)
So, the coordinates of vertex are A(0, 0). (0, –be) Minor axis
Let ( x1, y1) be the coordinates of the point Q. S'
∴ y12 = 2x1 …(i) B'
y −2 y = –b/e
and slope of PQ = 1 , Z′
x1 − 2 Directrix Y′
2−0 Here, AA ′ = 2a and BB′ = 2b are minor and major axes,
Also, slope of AP = =1
2−0 respectively.
MATHEMATICS > Conic Section 209

Some Important Results Related to Ellipse (ii) outside the ellipse, if S1 > 0
x12 y12
x2
+
y2
= 1, a > b
x2
+
y2
= 1, a < b (iii) inside the ellipse, if S1 < 0 where, S1 ≡ + −1
a2 b2 a2 b2 a2 b2
Coordinates of the
centre
(0, 0) (0, 0) Equation of Ellipse in
Coordinates of the ( a, 0) and ( −a, 0) ( 0, b ) and ( 0, − b ) Parametric Form
vertices
x2 y2
Coordinates of foci ( ae, 0) and ( −ae, 0) ( 0, be ) and ( 0, − be ) Parametric equation of ellipse = 1 is +
Length of major axis 2a 2b
a 2 b2
x = a cos θ, y = b sin θ and parametric coordinates of
Length of minor axis 2b 2a
point lying on it is given by (a cos θ, b sin θ). The
Equation of the major
axis
y=0 x=0 angle θ is called the eccentric angle of the point
( a cos θ, b sin θ) on the ellipse.
Equation of the minor x=0 y=0
axis
Equation of the a a b b
Equation of Chord
x= and x = − y= and y = −
directrices e e e e Q(x2, y2)
b2 a2
Eccentricity e = 1− e = 1−
a2 b2
P(x1, y1)
2 2
2b 2a
Length of latusrectum
a b
R(x3, y )
Focal distances of a b ± ey
3
a ± ex
point ( x, y)
The equation of the chord of the ellipse
 b2   a2 
Extremities of  ± ae, ±   ± , ± be  x2 y2
latusrectum  a  b  + = 1, whose mid-point be ( x1 , y1 ) is T = S1 ,
a2 b2
Note If the centre of ellipse is at (h, k) and its axis are parallel to xx1 yy1 x2 y2
the coordinate axes, then the equation of ellipse is given as
where T = + − 1, S1 = 12 + 12 − 1.
a b a b
( x − h)2 ( y − k )2
+ = 1
a 2
b2 The equation of the chord joining the points
P ( a cos θ1 , b sin θ1 ) and Q ( a cos θ 2 , b sin θ 2 ) is
EXAMPLE 5. The equation of the ellipse, whose centre is
1 x  θ + θ 2  y  θ1 + θ 2   θ1 − θ 2 
at origin (0, 0), foci (+1, 0) and eccentricity , is cos  1  + sin   = cos  
2 a  2  b  2   2 
x2 y2 x2 y2
a. + =1 b. + =1
3 4 4 3 Tangents and Normals to the Ellipse
2 2
x y x2 y2
c. + = 12 d. x 2 + y 2 = 12
3 4 Equation of tangent to the ellipse + =1
1 a2 b2
Sol. Here, foci = ( + 1, 0) and eccentricity, e = xx1 yy1
2 (i) at the point ( x1 , y1 ) is + = 1.
1 a2 b2
∴ ae = 1 and e = ⇒ a = 2
2
(ii) having slope m is y = mx + a 2 m2 +b 2 and
b = a (1 − e ) ⇒ b2 = 4 1 −  = 4 × = 3
1 3
∴ 2 2 2
 4 4 coordinates of point of contacts are
x2 y2  a2m b2 
So, the equation of required ellipse is + = 1. + ,+ .
4 3  
 a 2 m2 + b 2 a 2 m2 + b 2 
Position of a Point with x2 y2
Equation of normal to the ellipse =1 +
Respect to an Ellipse a 2 b2
x2 y2 a 2 x b2 y
If S ≡ + − 1 = 0, is the equation of an ellipse, then a (i) at the point ( x1 , y1 ) is − = a 2 − b2 .
a 2 b2 x1 y1
point P( x1 , y1 ) in the XY-plane lies m (a 2 − b2 )
(ii) having slope m is y = mx ±
(i) on the ellipse, if S1 = 0 a 2 + b 2 m2
210 NDA/NA Pathfinder

HYPERBOLA Conjugate Hyperbola


A hyperbola is the locus of that point which moves in The hyperbola whose transverse and conjugate axes are
a plane such that the ratio of its distance from a fixed respectively the conjugate and transverse axes of the
point (focus) to a fixed straight line (directrix) is given hyperbola is called the conjugate hyperbola of the
constant and greater than unity. This constant ratio is given hyperbola. The equation of conjugate hyperbola
called eccentricity and denoted by e. x2 y2 x2 y2
of the given hyperbola 2 − 2 = 1 is − 2 + 2 = 1.
Or a b a b
The locus of points difference of whose distance from Y Transverse axis
two fixed points remain constant.
S(0, be)
Y
Z′ Z Conjugate axis
B(0, b) y = b/e
M′ M
P(x, y) A′(–a, 0) A(a,0)
X′ X
Transverse axis O (0, 0) Conjugate axis
X′ X y = –b/e
S'(– ae, 0) O (0,0) A S(ae, 0) B'(0, –b)
A′

S'(0, –be)
(a ,
)
,0

0)
(–a

x = – a/e x = a/e Y′
Y′
PS Rectangular Hyperbola
= e ⇒ PS 2 = e 2 PM 2
PM It is a particular kind of hyperbola which has its
2
 a transverse and conjugate axis of equal length. It is also
⇒ ( x − ae ) 2 + y 2 = e 2  x −  known as equilateral hyperbola. The eccentricity of
 e
rectangular hyperbola is 2.
⇒ x 2 (e 2 − 1) − y 2 = a 2 (e 2 − 1)
x2 y2 x2 y2 Some Important Results Related to
⇒ − = 1 ⇒ − =1 Hyperbola and Conjugate Hyperbola
a 2 a 2 (e 2 − 1) a 2 b2
Hyperbola Conjugate hyperbola
where, b 2 = a 2 (e 2 − 1) x2 y2
− 2 =1 x2 y2
2
− + =1
This equation is called the equation of hyperbola in a b a2
b2
standard form. Coordinates of the
(0, 0) (0, 0)
centre
EXAMPLE 6. Find the equation of the hyperbola, the Coordinates of the ( a, 0) and ( −a, 0) ( 0, b ) and ( 0, − b )
length of whose latusrectum is 8 and eccentricity is 3/ 5. vertices
x 2 y2 x 2 y2 Coordinates of foci ( ± ae,0) ( 0, ± be )
a. − =1 b. − =1
25 15 25 20 Length of the
2a 2b
2 2 transverse axis
x y
c. − =1 d. None of these Length of the
20 15 conjugate axis
2b 2a
2 2
x y Equations of the a b
x=± y=±
Sol. Let the equation of the hyperbola be 2 − 2 = 1 …(i) directrices e e
a b
Then, length of the latusrectum = 8 a2 + b 2 b 2 + a2
e = 2
or e = or
2b2 Eccentricity a b2
⇒ = 8 ⇒ b2 = 4a ⇒ a2( e2 − 1) = 4a b 2 = a 2(e 2 − 1) a 2 = b 2(e 2 − 1)
a
⇒ a( e2 − 1) = 4 Length of the 2 b2 2 a2
a − 1 = 4 ⇒ a = 5
9 latusrectum
⇒ a b
5 
Equation of the y=0 x=0
On putting a = 5 in b2 = 4a, we get b2 = 20 transverse axis
Hence, the equation of the required hyperbola is Equation of the x=0 y=0
2 2
conjugate axis
x y
− = 1. Focal distance ex = ± a ey = ± b
25 20
MATHEMATICS > Conic Section 211

Note If centre of hyperbola is at point (h,k) and its axes are Let ∠QCN = θ. Here, P and Q are the corresponding
parallel to the coordinate axes, then the equation of
( x − h)2 ( y − k )2
points on the hyperbola and the auxiliary circle
hyperbola is given as 2
− = 1 (0 ≤ θ < 2π ).
a b2

Equation of Hyperbola Equation of Chord


The equation of chord joining the points
in Parametric Form P ≡ ( a sec θ1 , b tan θ1 ) and Q = ( a sec θ 2 , b tan θ 2 ) is
x2 y2
Parametric equation of the hyperbola = 1, is − x θ − θ2  y  θ1 + θ 2   θ1 + θ 2 
a 2 b2 cos  1  − sin   = cos  
a  2  b  2   2 
x = a sec θ, y = b tan θ and parametric coordinates of point
lying on it is given by (a sec θ, b tan θ).
Tangents and Normals
Position of a Point with x2 y2
Equation of tangent to the hyperbola − =1
Respect to a Hyperbola a2 b2
x2 y2 xx1 yy1
Let the equation of hyperbola be S ≡ − −1=0 (i) at the point ( x1 , y1 ) is 2
− = 1.
a2 b2 a b2
Then, the point P ( x1 , y1 ) lies (ii) having slope m is y = mx ± a 2 m2 − b 2
(i) on the hyperbola, if S1 = 0 x2 y2
Equation of normal to the hyperbola − =1
(ii) outside the hyperbola, if S1 > 0 a2 b2
(iii) inside the hyperbola, if S1 < 0 a 2 x b2 y
(i) at the point ( x1 , y1 ) is + = a 2 + b2 .
x12 y12 y1
where, S1 ≡ 2
− 2
−1 x1
a b m (a 2 + b2 )
(ii) having slope m is y = mx ±
Intersection of a Line and a Hyperbola a 2 − b 2 m2
The straight line y = mx + c will cut the hyperbola
x2 y2 EXAMPLE 7. If the eccentricity and length of
− = 1 in two points which may be real, coincident 13 10
a 2 b2 latusrectum of a hyperbola are and units
3 3
or imaginary, as c 2 > 1 = 1 < a 2 m2 − b 2 .
respectively, what is the length of the transverse axis?
7 15 15
Auxiliary Circle of Hyperbola a. units b. 12 units c. units d. units
2 2 4
Y
13 10
P (x, y) Sol. Here, e = and length of latusrectum =
3 3
Q 90º
2
2b 10 5a
θ ⇒ = ⇒ b2 =
X′ A′ (–a,0) C A (a N X a 3 3
(0,0) ,0)
= a2 
5a 13 
We know that, b = a ( e − 1) ⇒
2 2 2
− 1
3 9 

Y′ 5a 4a2 15
⇒ = ⇒ a=
2 2 3 9 4
x y
Let −
= 1 be the hyperbola, then equation of the 2 × 15 15
a 2
b2 ∴ Length of transverse axis = 2a = = units
4 2
auxiliary circle is x 2 + y 2 = a 2 .
212 NDA/NA Pathfinder

PRACTICE EXERCISE
1. The two ends of latusrectum of a parabola are 10. In an ellipse, the distance between its foci is 6
the points (3, 6) and ( −5, 6), then the focus is and its minor axis is 8. Then, its eccentricity is
(a) (1, 6) (b) (−1, 6) (a)
4
(b)
1
(c)
3
(d)
1
(c) (1, − 6) (d) (−1, − 6) 5 52 5 2

2. The parametric representation ( 2 + t 2 , 2t + 1) 11. If e1 , e2 be the eccentricities of ellipse 9x 2 + 4 y 2 = 36


represents and hyperbola 9x 2 − 4 y 2 = 36, respectively then
(a) a parabola with focus at (2,1) (a) e12 + e 22 > 3 (b) e12 + e 22 = 2
(b) a parabola with vertex at (2,1)
(c) e12 + e 22 > 4 (d) e12 + e 22 < 3
(c) an ellipse with centre at (2,1)
(d) None of the above 12. In an ellipse, the distance between the foci is 8
and the distance between the directrices is 25.
3. The equation y − 2x − 2 y + 5 = 0 represents
2
The length of major axis is
(a) circle centred at (1,1) (a) 10 2 (b) 20 2 (c) 30 2 (d) 50 2
3
(b) parabola with directrix at x =
2 13. A circle is drawn with the two foci of an ellipse
(c) parabola with focus at (1, 2) x2 y2
+ = 1 at the end of the diameter. What is
1 a 2 b2
(d) parabola with directrix at x = −
2 the equation of the circle?
4. If (0, 4) and (0, 2) are respectively, the vertex (a) x2 + y2 = a2 + b 2 (b) x2 + y2 = a2 − b 2
and focus of a parabola, then its equation is (c) x + y = 2 (a + b )
2 2 2 2
(d) x2 + y2 = 2(a2 − b 2 )
(a) x + 8 y = 32
2
(b) y + 8x = 32
2
14. A man running around a race course notes that
(c) x2 − 8 y = 32 (d) y2 − 8x = 32 the sum of the distances of two flag posts from
him is always 10 m and the distance between the
5. The coordinates of a point on the parabola
flag posts is 8 m. The area of the path, he
y 2 = 8x, whose focal distance is 4, is
encloses in square metres is
(a) (2, 4) (b) (4, 2) (c) (−2, − 4) (d) (4, − 2 ) (a) 15 π (b) 12 π (c) 18π (d) 8π
6. An equilateral triangle is inscribed in a parabola x 2
y 2
y 2 = 4ax, whose vertex is at the vertex of the 15. If the foci of the ellipse + = 1 and the
16 b2
parabola. The length of each side of the triangle x2 y2 1
is hyperbola − = coincide, then the value of
144 81 25
(a) 2 a 3 (b) 4a 3 (c) 6a 3 (d) 8a 3
b2 is
7. In the parabola y = 4ax, the length of chord
2
(a) 1 (b) 5
passing through the vertex and inclined to the (c) 7 (d) 9
 π
axis at an angle   is 16. The eccentricity of the hyperbola with
 4 latusrectum 12 and semi-conjugate axis 2 3, is
(a) 2 a 2 (b) 2a (c) 2a (d) 4a 2 3
(a) 2 (b) 3 (c) (d) 2 3
2
8. Equation of the ellipse, whose focus is (6,7), 17. If e and e′ be the eccentricities of a hyperbola
directrix is x + y + 2 = 0 and e = 1/ 3 is 1 1
(a) 5x2 + 2 x y + 5 y2 − 76x − 88 y + 506 = 0
and its conjugate, then + is equal to
e 2
e′ 2
(b) 5x2 − 2 x y + 5 y2 − 76x − 88 y + 506 = 0 (a) 0 (b) 1
(c) 5x2 − 2 x y + 5 y2 + 76x + 88 y − 506 = 0 (c) 2 (d) −1
3
(d) None of the above 18. Equation of the hyperbola with eccentricity and
2
9. The eccentricity of ellipse, if length of major axis foci at ( ± 2, 0) is 5x − 4 y = k . What is the value
2 2 2

is three times the length of minor axis, is of k?


(c)   5 (d)   5
1 1 1 2 2 4 3 4 3
(a) (b) (c) (d) (a) (b)
3 3 2 3 3 4  3  4
MATHEMATICS > Conic Section 213

x2 y2
19. Consider the following statements Directions (Q. Nos.27-29) P : y 2 = 8 x ; E : + =1
I. The area of the ellipse 2x2 + 3 y2 = 6 is more than 4 15
the area of the circle x2 + y2 − 2x + 4 y + 4 = 0. 27. Equation of a tangent common to both the
II. The equation 3x2 + 4 y2 − 18x + 16 y + 43 = k parabola P and the ellipse E is
represents an ellipse if k < 0. (a) x − 2 y + 8 = 0 (b) x ± 2 y + 8 = 0
Which of the above statement(s) is/are correct? (c) x + 2 y − 8 = 0 (d) x ± 2 y − 8 = 0
(a) Only I (b) Only II
28. Equation of the normal at the point of contact of
(c) Both I and II (c) None of these
the common tangent, which makes an acute
angle with the positive direction of X-axis, to the
Directions (Q. Nos. 20-21) The parametric equation parabola P is
of the hyperbola are x = 5 sec φ and y = 3 tan φ (a) 2 x + y = 24 (b) 2 x + y + 24 = 0
20. Distance between the foci is (c) 2 x + y = 48 (d) 2 x + y + 48 = 0
(a) 34 (b) 2 34 (c) 4 34 (d) None of these 29. Point of contact of a common tangent to P and E
21. If y = mx − 3 is a tangent to the given hyperbola, on the ellipse is
(a)  ,  (b)  ± , 
then m is 1 15 1 15
2 3 2 3 3 2 2 4   2 4
(a) (b) (c) (d)
(c)  , −  (d)  − , ± 
5 5 5 5 1 15 1 15
2 4  2 4
Directions (Q. Nos. 22-23) If a hyperbola passes
x2 y2 Directions (Q. Nos. 30-33) Consider the equation of
through the foci of the ellipse + = 1 and its
25 16 parabola 25 [( x − 2) 2 + (y − 4) 2] = ( 4x − 3y + 12) 2
transverse and conjugate axis coincide with major 30. The coordinates of the focus are
and minor axes of the ellipse and product of the (a) (3, − 2 ) (b) (2, 4) (c) (− 1, 1) (d) None of these
eccentricities is 1, then
31. Length of latusrectum is
22. The equation of hyperbola is (a)
8
(b)
4
(c)
16
(d) None of these
x2 y2 x2 y2 5 5 5
(a) − =1 (b) − =1
9 16 9 25 32. The equation of the axis is
2 2 2 2
(c) −
x
+
y
=1 (d)
x

y
=1 (a) 4x + 3 y + 15 = 0 (b) 4x + 3 y + 10 = 0
9 25 16 25 (c) 3x + 4 y − 22 = 0 (d) None of these
23. The focus of hyperbola is 33. The equation of the latusrectum is
(a) (5, 0) (b) (5 3, 0) (c) (0, 5) (d) (0, 5 3 ) (a) 4x − 3 y + 4 = 0 (b) 4x − 3 y + 8 = 0
(c) 4x + 3 y + 7 = 0 (d) None of these
Directions (Q. Nos. 24-26) Consider the equation of
hyperbola 3x 2 − 24x + 40y − 4y 2 − 88 = 0. Directions (Q. Nos. 34-37) Consider the equation of
ellipse 12 x 2 + 4y 2 + 24x − 16y + 25 = 0
24. The eccentricity of the hyperbola is
(a) 2 7 (b)
7
(c) 2 (d)
13 34. The centre of the ellipse is
2 2 (a) (4, − 1) (b) (2, 1) (c) (− 1, 2 ) (d) (− 3, 2 )
25. The equation of directrices is 35. The length of major and minor axes are
12
(a) x = 5 ± 2 (b) x = 5 ± 3 (a) 2, 4 (b) 3, 1 (c) 2, 2 3 (d) 4, 6
7
12 36. The eccentricity of the ellipse is
(c) x = 4 ± 2 (d) x = 4 ± 3
7 1 3 2 1
(a) (b) (c) (d)
3 4 3 4
26. The equation of hyperbola, whose latusrectum is
6 and eccentricity is same as eccentricity of given 37. Coordinates of the foci are
 1
(a)  0, 1 ±
hyperbola is 1 
 (b)  0, 2 ± 
x2 y2 x2 y2  3  3
(a) − =1 (b) − =1
18 10 20 16
(c)  1, 2 ±
1 
(d)  −1, 2 ±
1 
2 2  
(c)
x

y
=1 (d) None of these  2  2
16 12
214 NDA/NA Pathfinder

PREVIOUS YEARS’ QUESTIONS 47. What is the sum of the major and minor axes of
the ellipse, whose eccentricity is 4/5 and length
38. What is the eccentricity of the conic of latusrectum is 14.4 units? e 2014 I
4x + 9 y 2 = 144?
2
e 2012 I
(a) 32 units
5 5 3 2
(a) (b) (c) (d) (b) 48 units
3 4 5 3 (c) 64 units
39. The eccentricity e of an ellipse satisfies the (d) None of the above
condition e 2012 II
(a) e < 0 (b) 0 < e < 1 (c) e = 1 (d) e > 1 48. What is the equation of parabola, whose vertex
is at (0, 0) and focus is at ( 0, − 2)? e 2014 I
40. The sum of the focal distances of a point on the (a) y 2 + 8x = 0 (b) y 2 − 8x = 0
x2 y2
ellipse + = 1 is (c) x 2
+ 8y = 0 (d) x 2
− 8y = 0
4 9 e 2012 II
(a) 4 units (b) 6 units (c) 8 units (d) 10 units 49. What is the length of the latusrectum of an
ellipse 25x 2 + 16 y 2 = 400? e 2014 II
41. The difference of focal distances of any point on
a hyperbola is equal to e 2012 II (a) 25/2 (b) 25/4
(a) latusrectum (b) semi-transverse axis (c) 16/5 (d) 32/5
(c) transverse axis (d) semi-latusrectum 50. The point on the parabola y 2 = 4x nearest to
42. The equation of the ellipse, whose vertices are at focus has its abscissa e 2015 I
( ± 5, 0) and foci at ( ± 4, 0) is e 2013 I (a) x = 0 (b) x = a
x2 y2 x2 y2 a
(a) + =1 (b) + =1 (c) x = (d) x = 2a
25 9 9 25 2
x2 y2 x2 y2 x2 y2
(c) + =1 (d) + =1 51. The hyperbola −
= 1, passes through the
16 25 25 16 a 2
b2
43. The axis of the parabola y 2 + 2x = 0 is e 2013 II
point ( 3 5 , 1) and the length of its latusrectum is
(a) x = 0 (b) y = 0 (c) x = 2 (d) y = 2 4
units. The length of the conjugate axis is
3 e 2015 I
44. The length of latusrectum of the ellipse
(a) 2 units (b) 3 units
4x 2 + 9 y 2 = 36 is e 2013 II (c) 4 units (d) 5 units
4 8
(a) (b) (c) 6 (d) 12 x2 y2
3 3 52. Consider any point P on the ellipse + = 1,
25 9
45. The foci of the hyperbola 4x 2 − 9 y 2 − 1 = 0 are in the first quadrant. Let r and s represent its
e 2013 II distances from (4, 0) and (−4, 0) respectively,
 13  then (r + s) is equal to e 2015 II
(a) (± 13, 0) (b)  ± , 0
 6  (a) 10 units (b) 9 units
 13  (c) 8 units (d) 6 units
(c)  0, ±  (d) None of these
 6  53. The eccentricity of the hyperbola 16x 2 − 9 y 2 = 1, is
e 2015 II
46. A point P moves such that its distances from 3 5
(1, 2) and (−2, 3) are equal. Then, the locus of P (a) (b)
5 3
is e 2013 II 4 5
(a) straight line (b) parabola (c) (d)
5 4
(c) ellipse (d) hyperbola

ANSWERS
1 b 2 b 3 b 4 a 5 a 6 d 7 d 8 b 9 d 10 c
11 a 12 a 13 b 14 a 15 c 16 a 17 b 18 c 19 a 20 b
21 d 22 a 23 a 24 b 25 c 26 c 27 b 28 a 29 d 30 b
31 c 32 c 33 a 34 c 35 b 36 c 37 d 38 a 39 b 40 b
41 c 42 a 43 b 44 b 45 b 46 a 47 c 48 c 49 d 50 a
51 c 52 a 53 b
MATHEMATICS > Conic Section 215

HINTS AND SOLUTIONS


1. (b) Focus is the mid-point latusrectum.  3 1  5 13 137
B b , b  lies on y 2 = 4ax e2 + e2 = + = > 3 but < 4
1 2
Let the two end points of latusrectum  2 2  9 4 36
be L (3, 6) and L ( −5, 6 ). b2 12. (a) 2ae = 8 ,
2a
= 25 ⇒ 4a 2 = 200
1 2
∴ = 2 3 ab ⇒ b = 8 3 a
3 + ( −5 ) 4 e
∴ Focus, x = = −1,
2 ∴ a =5 2
7. (d) Let AP be the chord of length p,
6+ 6 ∴ 2a = 10 2
y= =6 π
2 making an angle of   with the axis. x2 y2
 4 13. (b) Q Foci of an ellipse 2 + 2 = 1 are
Focus = ( x , y ) = ( −1, 6) Y a b
2. (b) Let x = 2 + t 2 , y = 2t + 1 ( ae , 0) and ( − ae ,0). Equation of circle
P
Eliminating t, we get with centre ( 0,0) and radius ae is
( y − 1)2 = 4( x − 2) x 2 + y 2 = ( ae )2 [where, ( ae )2 = a 2 − b 2 ]
which is a parabola with vertex at (2, 1). π/4 ∴ x 2 + y 2 = a2 − b2
X′ X
A M
3. (b) Given equation can be rewritten as 14. (a) Clearly, the race course will be an
( y − 1)2 = 2( x − 2) ellipse with the flag posts as its foci. If a
or Y 2 = 4 AX , where Y = y − 1 Y′ and b are the semi-major and
semi-minor axes of the ellipse, then
X = x − 2, 4 A = 2 Then, coordinates of P are
π π 2a = 10 and 2ae = 8
i.e. 
p 
P  p cos , p sin ,
1 p
i.e. Directrix is X = − A ⇒ x − 2 = − ,  4
2  4 4  2 2 ∴ a = 5, e = and b 2 = a 2 ( 1 − e 2 ) = 9
5
∴ x =− + 2=
1 3 Since, this point lies on y = 4ax ,
2

2 2 p2 p ∴ Area of the ellipse


we have = 4a ⋅ ⇒ p = 4a 2 = πab = π ⋅5 ⋅ 3 = 15 π sq m
4. (a) AS = 2a. Vertex (0, 4) lies on Y-axis. 2 2
The parabola is of the form 15. (c) For hyperbola,
8. (b) By the definition, SP = e ⋅ PM b2
X 2 = − 4 AY (downward parabola) as e2 = 1 + 2 = 1 +
81
=
225
focus is below the vertex. {( x − 6)2 + ( y − 7)2 } a 144 144
Y 1 x+ y+ 2 15 5
= ⋅ ∴ e= = , i.e. e > 1
A (0, 4) 3 ( 1 + 1) 12 4
144
On squaring both sides, we get Also, a2 =
6 ( x 2 + y 2 − 12x − 14 y + 85) 25
S (0, 2) Hence, the foci are ( ± ae ,0)
= x 2 + y 2 + 4 + 2xy + 4 y + 4x
i.e.  ±
12 5 
∴5x + 5 y 2 − 76x − 88 y −2xy + 506 = 0
2 ⋅ , 0 = ( ± 3, 0)
 5 4 
X 9. (d) Given, 2a = 3( 2b ) Now, the foci coincide, therefore for ellipse
b2 1  b2 
∴ = ae = 3 or a 2 e 2 = 9 ⇒ a 2  1 − 2  = 9
( x − 0)2 = − 4 × 2 ( y − 4) a 2
9  a 
∴ x 2 + 8 y = 32 b2 b2 8
⇒ e = 1− 2 = 1− 2 =
2 ⇒ a 2 − b 2 = 9 ⇒ 16 − 9 = b 2
5. (a) Given, a + x = 4 or 2 + x = 4 a 9b 9 ∴ b2 = 7
∴ x =2 2 2 b2
∴ e= 16. (a) We have, 2 ⋅ = 12 and b = 2 3
On putting x = 2 in y 2 = 8x , we get 3 a
y=±4 10. (c) Distance between foci = 2ae = 6 ∴ a=2
⇒ a = 3 / e, b2
6. (d) Let ∆OAB be the equilateral ∴ e2 = 1 + 2 ⇒e=2
triangle. Minor axis = 2b = 8 ⇒ b = 4 a
9 x2 y2
Y Now, b 2 = a 2 ( 1 − e 2 ) ⇒16 = 2 ( 1 − e 2 ) 17. (b) For the hyperbola, 2 − 2 = 1
e a b
⇒ 16e 2 = 9( 1 − e 2 ) ⇒ e = 3 / 5
B b2 a2 + b2
x2 y2 x2 y2 or e = 1+ 2 =
2
11. (a) + = 1, − =1 a a2
4 9 4 9
30º C For conjugate hyperbola,
X Major axes along Y-axis, a 2 = 9, b 2 = 4 y2 y2 x2
O 30º 1 1 x2
Transverse axes along X-axis 2
− 2 = − 1 or − 2 =1
a b b2 a
a = 4, b = 9
2 2
2 2 a2 a2 + b2
A
b2 ∴ e′ = 1 + 2 =
2
4 5 b2
Then, ∠COB = 30° e2 = 1 − 1 = 1− = b
Let OA = OB = AB = b
1
a2 9 9 1 1 a2 + b2
1 ∴ + 2 = 2 =1
Then, B is B (b cos 30°, b sin 30°) b2 9 13 e2 e′ a + b2
 3 1  e2 = 1 + 2 = 1+ =
i.e. B  b, b
2
a2 4 4
2
 2 2 
216 NDA/NA Pathfinder

18. (c) Given equation of hyperbola Sol. (Q. Nos. 24-26) The given hyperbola can Similarly, the point of contact of the
other tangent is  − , −  .
x2 y2 be written as 1 15
5x 2 − 4 y 2 = k 2 ⇒ − 2 =1  2 4
k2 k 3( x − 4)2 − 4( y − 5)2 = 36
( x − 4)2 ( y − 5)2 30. (b) The given equation of parabola can
5 4 or − = 1
k k 12 9 be written as
∴ a= and b = 2 2 2
5 2 ⇒
X

Y
=1  4x − 3 y + 12 
3 ...(i) ( x − 2)2 + ( y − 4)2 =  
The eccentricity and foci are and 12 9
 4 + ( − 3) 
2 2
2 where, X = x − 4, Y = y − 5
( ± 2,0), respectively. ∴ The coordinates of focus are ( 2, 4)
3 24. (b) From Eq. (i), we have and the equation of directrix is
∴ e = and ± ae = 2
2 a 2 = 12 and b 2 = 9 4x − 3 y + 12 = 0

k 3
⋅ =2 ⇒ k=
4 a2 + b2 21 7
5 ∴ e = = = 31. (c) The distance of the focus from the
5 2 3 a 12 2 directrix
x2 y2 25. (c) Equation of the directrices is given by 4( 2) − 3( 4) + 12 8
19. (a) I. Given ellipse is + = 1, = =
3 2 a 12 4 2 + ( − 3)2 5
X =± ⇒ x −4=± ×2
whose area is = π 3 ⋅ 2 = π 6 e 7 ∴ The length of latusrectum
Circle is x 2 + y 2 − 2x + 4 y + 4 = 0 12 =2× =
8 16
⇒x =4± 2
or ( x − 1)2 + ( y − 2)2 = 1 7 5 5
Its radius is 1. Hence, area is π. x2 y2 32. (c) Axis of parabola is perpendicular to
So, Statement I is true. 26. (c) Let the equation be 2 − 2 = 1
a b the directrix.
II. The given equation can be written as 1 1
∴ Equation of line perpendicular to
3( x − 3)2 + 4( y + 2)2 = k 2b 2
We have, = 6 ⇒ b 2 = 3a 4x − 3 y + 12 = 0 is 3x + 4 y + k = 0.
So, no locus for k < 0 a
Since, 3x + 4 y + k = 0 passes through
Hence, Statement II is false. Now, b 2 = a 2 ( e 2 − 1) ( 2, 4).
3a 2
⇒ 3a = a 2  − 1 ⇒3a =
x y 7
20. (b) sec φ = , tan φ = ⇒a = 4 ∴ 3( 2) + 4( 4) + k = 0 ⇒ k = − 22
5 3 4  4
∴ Equation of axis is 3x + 4 y − 22 = 0
x2 y2 ∴ b = 3a = 3 × 4 = 12
2
∴ − =1 33. (a) Equation of the latusrectum is
25 9 Hence, the equation of the hyperbola is
This gives, a = 25, b 2 = 9
2 parallel to the equation of directrix.
x2 y2
− =1 So, equation of line parallel to
∴ a = 5 and b = 3, 16 12
a2 + b2 25 + 9 34 4x − 3 y + 12 = 0 is 4x − 3 y + k = 0
e = = = 27. (b) Equation of any tangent to the
Since, it passes through focus (2, 4)
a2 25 5 parabola P : y 2 = 8x is y = mx +
2
Distance between the focii = 2ae m ∴ 4( 2) − 3( 4) + k = 0 ⇒ k = 4
34 ∴ Required equation is 4x − 3 y + 4 = 0
= 2×5× = 2 34 where, m is the slope of tangent.
5 x2 y2 Sol. (Q. Nos. 34-37) The given equation can be
Since, it touches E : + =1 written as
21. (d) y = mx − 3 will be a tangent to the 4 15
2 12 ( x + 1)2 + 4 ( y − 2)2 = 3
x2 y2  2  = 4m 2 + 15 ⇒ m = ± 1
hyperbola − = 1, if ( x + 1)2 ( y − 2)2
25 9 m 2 or + =1
4 3/ 4
( − 3)2 = 25m 2 − 9 Equations of the tangents are
18 3 2 x ± 2 y + 8 = 0. 34. (c) Coordinates of centre of the ellipse
⇒ 9 + 9 = 25m 2 ⇒ m = = are given by
25 5 1
28. (a) When m = , the slope of the x + 1 = 0 and y − 2 = 0
22. (a) Eccentricity of ellipse 2 ⇒ x = − 1 and y = 2
normal is −2 and equation of the normal ∴ Centre of the ellipse is ( − 1, 2).
25 − 16 3
= = to the parabola is
25 5 35. (b) Let a and b the length of the
5 y = −2x − 2( 2)( −2) − 2( −2)3 semi-minor and semi-major axes, then
∴ Eccentricity of hyperbola = ⇒ 2x + y = 24 1 3
3 a 2 = and b 2 =
Foci of ellipse ( ± 3,0). 29. (d) Equation of the tangent at 4 4
x2 y2 ( 2 cos θ, 15 sin θ) on the ellipse E to ∴ Length of major axis
∴ Equation of hyperbola 2 − 2 = 1 x
the ellipse is cos θ +
y
sin θ = 1, 3
9 b = 2b = 2 × = 3
2 15 4
where, b 2 = 9( e 2 − 1) if it represents the tangent 1
Length of minor axis = 2a = 2 × =1
= 9 − 1 = 16
25 x − 2 y + 8 = 0, then 4
 9  cosθ sinθ −1
= = 1
x2 y2 2 −2 15 8
∴Equation of hyperbola is − =1 a2 4
9 16 1 15 36. (c) Eccentricity = 1 − 2 = 1 −
⇒ cos θ = − , sin θ = b 3
23. (a) Focus of hyperbola is 4 4 4
and the point of contact is  − ,  .
 ± 3 × 5 , 0 = ( ± 5, 0) 1 15
1 2
 3   2 4 = 1− =
3 3
MATHEMATICS > Conic Section 217

37. (d) We have, be =


3
×
2
=
1 ∴ Foci of the hyperbola = ( ± ae , 0) Let P be any point on the parabola, then
 equation directrix is y − 2 = 0
2 3 2 a2 + b2 
=  ± a , 0 =( ± a 2 + b 2 , 0) ∴ Equation of parabola is PS = PM
Coordinates of foci are given by  a  |y − 2|
x + 1 = 0, y − 2 = ± be ⇒ ( x − 0)2 + ( y + 2)2 =
 1 1   13  1
Thus, foci are  − 1, 2 ±
1 
. = ± + , 0 =  ± , 0
 2  4 9   6  ⇒ ( x 2 + ( y + 2)2 )2 = | y − 2|2
38. (a) The given conic equation is 46. (a) Let the coordinates of point P is
⇒ x 2 = − 8 y,
x2 y2 ( h, k ).
4x 2 + 9 y 2 = 144 ⇒ + =1 Now, according to given condition
which is the required equation of
36 16 parabola.
a = 36 ⇒ a = 6 and b = 16 ⇒ b = 4
2 2 Distance between ( h , k ) and (1, 2)
49. (d) Equation of ellipse is
Thus, represent an ellipse, as a > b. = Distance between ( h , k ) and (−2,3)
x2 y2
Now, eccentricity, b 2 = a 2 ( 1 − e 2 ) ⇒ ( h − 1)2 + ( k − 2)2 25x 2 + 16 y 2 = 400 ⇒ + =1
16 25
⇒ 16 = 36 ( 1 − e 2 ) = ( h + 2)2 + ( k − 3)2 Here, a 2 = 16 and b 2 = 25
4
⇒ = 1 − e2 ⇒ e =
5 2a 2 32
⇒ h 2 + 1 − 2h + k 2 + 4 − 4k ∴ Length of latusrectum = =
9 3 b 5
= h 2 + 4 + 4h + k 2 + 9 − 6k
39. (b) The eccentricity of ellipse lies ⇒ − 2h − 4k + 5 = 4h − 6k + 13 50. (a) Let P ( x , y ) be the required point on
between 0 and 1. the parabola which is nearest to the focus.
⇒ 6h − 2k + 8 = 0 ⇒ 3h − k + 4 = 0
40. (b) Since, the sum of focal distances of a So, the locus of P is 3x − y + 4 = 0, Then, D = ( x − a )2 + y 2
x2 y2
point on the ellipse 2 + 2 = 1 is which represent a straight line. = ( x − a )2 + 4ax
a b 47. (c) We know that, length of major axes
equal to 2b. When b > a. of an ellipse = 2a = ( x + a )2 = ( x + a )
Q a 2 = 4, b 2 = 9 ⇒ a = 2, b = 3 and length of minor axes of an ellipse Now, distance will be shortest, when
∴ Sum of the focal distances = 2b x = 0.
= 2 × 3 = 6 units Given that, 51. (c) Since, hyperbola passes through
eccentricity of an ellipse = 4 / 5 = e … (i) (3 5 , 1).
41. (c) and length of latusrectum of an ellipse
(3 5 ) 2 1
42. (a) Given that, = 14.4 units ∴ − 2 =1
Foci of an ellipse = ( ± 4, 0) = ( ± ae , 0) 2b 2 b2 a2 b
⇒ = 14.4 ⇒ = 7. 2
⇒ ae = 4 ...(i) a a ⇒ 45b 2 − a 2 = a 2 b 2 ...(i)
⇒ b = 7. 2 a
2
…(ii) 2b 2 4
and vertices of an ellipse Also, = ⇒ 6b 2 = 4a
= ( ± 5, 0) = ( ± a , 0) ⇒ a = 5 ...(ii) Since, eccentricity of an ellipse, a 3
b 2 = a 2 ( 1− e 2 ) 6b 2
From Eqs. (i) and (ii), we get
2
⇒ a= …(ii)
4  4  4
e= ...(iii) ⇒ 7. 2a = a 2  1 −    On putting the value from Eq. (ii) in
5  5 
Eq. (i), we get
Now, we have a relation [from Eqs. (i) and (ii)] 2 2
b 2 = a 2 ( 1− e 2 )  6b 2   6b 2  2
⇒ 7. 2a = a 2  1 −  ⇒ 7. 2a = a 2 ×
16 9 45b 2 −   =  b
 25   4   4 
⇒ b = 25  1−  ⇒ b 2 = 9 ⇒ b = ± 3

2 16 25
 25  36b 4 2
⇒ 9a 2 − 7. 2 × 25a = 0 ⇒ 45b 2 = [ b + 1]
∴ Required equation of an ellipse, ⇒9a 2 − 36 × 5a = 0 ⇒ 9a ( a − 20) = 0 16
x2 y2 x2 y2 ⇒ b 4 + b 2 = 20
+ = 1 ⇒ + =1 ⇒ a = 20 [Q a ≠ 0] ∴ b=2
a2 b2 25 9 Put the value of a in Eq. (ii), we get ∴ 2b = 4 = length of conjugate axis.
43. (b) Given, equation of parabola is b 2 = 7. 2 × 20 ⇒ b 2 = 72 × 2 = 144 x2 y2
y 2 + 2x = 0 ⇒ y 2 = − 2x Hence, the sum of the major and minor 52. (a) We have, an ellipse + =1
25 9
which is of the form y 2 = − 4ax. axes = 2a + 2b = 2( 20 + 12) = 64 units
Clearly, its foci are ( 4, 0) and ( − 4, 0).
So, axis of the parabola is y = 0. 48. (c) Given, vertex of the parabola = (0, 0) [ Q foci = S (ae, 0), S′ (ae, 0)]
and focus of the parabola = ( 0, − 2)
44. (b) Given equation of ellipse is ∴ PS + PS ′ = 2a = Major axis
x2 y2 Y ⇒ r + s = 2(5) = 10 units
4x2 + 9 y 2 = 36 ⇒ + =1
9 4 x2 y2
Directrix, y – 2 = 0 53. (b) Given, − =1
Here, a 2 = 9 and b 2 = 4 ; ( a > b ) M 1 / 16 1 / 9
∴ Length of latusrectum b2
2 ∴ Eccentricity, e 2 = 1 + 2
2b 2 2× 4 8 (0, 0) a
= = = X´ X
1/ 9
a 3 3 2
(0, –2) = 1+
1 / 16
45. (b) Given, equation of hyperbola is S P (x, y)
16
x2 y2 = 1+
4x 2 − 9 y 2 = 1 ⇒ − =1 9
( 1 / 4) ( 1 / 9) 5
1 1 ⇒ e=
Here, a2 = and b 2 = Y´ 3
4 9
19
218 CDS Pathfinder

THREE DIMENSIONAL
GEOMETRY
In NDA exam, generally 3-6 questions are asked from this chapter which are based on finding
coordinates of a point , direction ratios/cosine in different condition, equation of a
line/plane/sphere, angle between planes/lines, radius/centre of sphere, image of a point, etc.

To represent a point in a space we use x, y and z-coordinates, it is known as three dimensional


geometry. In 3-D, there are three mutually perpendicular straight lines XOX ′, YOY ′ and ZOZ′ called
axes.

Coordinates of a Point in a Space


Y
Three mutually perpendicular lines in space divide the space into 8 octants. Z
Let the lines be XOX ′ , YOY ′ and ZOZ′ intersecting at O. XOY is called the
XY-plane, YOZ is called the YZ-plane, ZOX is called the ZX-plane and all P (x, y, z)
three planes taken together are called the coordinate planes. If P( x, y, z) is a X′ X
O
point in space.
Then, |x | = distance from YZ-plane,
| y | = distance from ZX-plane, Z′
Y′
and |z | = distance from XY-plane.

Distance Formula
(i) The distance between two points P ( x1 , y1 , z1 ) and Q( x 2 , y 2 , z 2 ) is given by
PQ = ( x 2 − x1 ) 2 + ( y 2 − y1 ) 2 + ( z 2 − z1 ) 2
(ii) Three points P ( x1 , y1 , z1 ), Q( x 2 , y 2 , z 2 ) and R( x 3 , y 3 , z 3 ) are collinear, if PR = PQ + QR

Note Let O be the origin and P( x, y, z) be any point, then OP = x 2 + y 2 + z2 .

Section Formulae
Let P ( x1 , y1 , z1 ) and Q( x 2 , y 2 , z 2 ) be two points in space and let R be a point on the line segment
joining P and Q such that
MATHEMATICS Three Dimensional Geometry 219

(i) It divides PQ internally in the ratio m : n. Then, the x1 y1 1


1
coordinate of R are ∆z = x2 y2 1
2
 mx 2 + nx1 my 2 + ny1 mz 2 + nz1  x3 y3 1
 , , 
 m+ n m+ n m+ n 
(ii) PQ externally in the ratio m : n ( m ≠ n). Then, the Direction Cosines
coordinates of R are If a line makes angles α, β, γ with the positive directions
 mx 2 − nx1 my 2 − ny1 mz 2 − nz1  of X-axis, Y-axis and Z-axis respectively, then cos α,
 , , 
 m− n m− n m− n  cos β, cos γ are called its direction cosines. The direction
cosines are generally denoted as l, m, n i.e.
x + x2 y1 + y2 z1 + z2 
Note n
Mid-point of PQ =  1 , ,  l = cos α, m = cos β, n = cos γ
 2 2 2 
n
If the ratio in which a point divides a line comes out to be The angle α, β, γ are known as direction angles.
positive, then it divides the line internally and if it comes
out to negative, then it divides the line externally. Direction Ratios
n
If A ( x1, y1, z1 ), B ( x2, y2, z2 ) and C ( x3, y3, z3 ) represents the
vertices of a triangle, then the centroid of triangle is given by Three numbers a, b, c proportional to the direction
x + x2 + x3 y1 + y2 + y3 z1 + z2 + z3 
G =  1 , , 
cosines l, m , n of a line are known as the direction
 3 3 3  ratios of the line. Thus, a, b, c are the direction ratios of
l m n
EXAMPLE 1. Let P(a, b, c), Q(a + 2, b + 2 , c − 2) a line provided = = .
a b c
and R(a + 6, b + 6, c − 6) be collinear.
Consider the following statements SOME IMPORTANT RESULTS
I. R divides PQ internally in the ratio 3 : 2.
(i) If OP is a directed line segment with Z
II. R divides PQ externally in the ratio 3 : 2.
direction cosines l , m , n such that P
III. Q divides PR internally in the ratio 1 : 2. OP = r . Then, the coordinates of P γ r
Which of the above statement(s) is/are correct? are ( lr , mr , nr ). O β Y
a. Only I b. Only II c. I and III d. II and III (ii) If l, m, n are the direction cosines of X α
a directed line segment,
Sol. Since, P( a, b, c ), Q ( a + 2, b + 2, c − 2) and then l 2 + m 2 + n 2 = 1
R( a + 6, b + 6, c − 6) are collinear. or cos 2 α + cos 2 β + cos 2 γ = 1
3( a + 2) − 2( a) (iii) DC’s of X-axis are 1, 0, 0.
Also, =a+6
3− 2 C(x, y)
DC’s of Y-axis are 0, 1, 0.
3( b + 2) − 2b DC’s of Z-axis are 0, 0, 1.
n

=b+6
m

a b c
3− 2 (iv) l = ,m= ,n =
3( c − 2) − 2c a +b +c
2 2 2
a + b2 + c 2
2
a2 + b2 + c 2
and = c−6 B(x2, y2)
3− 2 (v) Direction ratios of a line joining the points A( x1 , y1 , z 1 )
So, R divides PQ and B ( x 2 , y 2 , z 2 ) are x 2 − x1 , y 2 − y1 and z 2 − z 1 and
externally in the ratio 3 : A(x1, y1) its direction cosines are
2. x 2 − x1 y 2 − y1 z 2 − z 1
( a + 6) + 2a , ,
Also, =a+ 2 | AB | | AB | | AB |
3
(vi) If P ( x1 , y1 , z 1 ) and Q ( x 2 , y 2 , z 2 ) are two points, such that
( b + 6) + 2b ( c − 6) + 2c
= b + 2 and =c−2 the direction cosines of PQ are l ,m ,n . The,
3 3
x 2 − x1 , = l | PQ | , y 2 − y1 = m | PQ | , z 2 − z 1 = n | PQ |
Thus, Q divides PR internally in the ratio 1 : 2.
These are projections of PQ on X ,Y and Z -axes,
Hence, Statements II and III are correct. respectively.

Area of Triangle
EXAMPLE 2. The direction cosines of a line equally
Let A ( x1 , y1 , z1 ), B( x 2 , y 2 , z 2 ) and C( x 3 , y 3 , z 3 ) be the inclined to all the three rectangular coordinate axes are
vertices of a triangle, then ∆ = ∆2x + ∆2y + ∆2z when a.
1 1 1
, , b. 1, 1, 1
1 1 1
c. , , d. None of these
3 3 3 2 2 2
y1 z1 1 x1 z1 1
1 1 Sol. a. l = m = n and l 2 + m2 + n2 = 1
∆x = y2 z2 1 , ∆ y= x2 z2 1
2 2 ⇒ 3l 2 = 1 ⇒ l = ±
1
,m=±
1
,n = ±
1
y3 z3 1 x3 z3 1 3 3 3
220 NDA/NA Pathfinder

EXAMPLE 3. If (l, m, n) are direction cosines of a line, (ii) The angle θ between two lines whose direction ratios
then what is the value of (l + m − n) 2 + (m + n − l) 2 are proportional to a1 , b1 , c1 and a 2 , b2 , c 2 ,
+ (l + m + n) 2 + (n + l − m) 2 ? respectively is given by
 a1 a 2 + b1 b2 + c1 c 2 
a. 0 b. 1 c. 4 d. 4 ( lm + mn + nl) cos θ = 
 a1 + b1 + c1 a 2 + b2 + c 2 
2 2 2 2 2 2
Sol. ( l + m − n) 2 + ( m + n − l) 2 + ( l + m + n) 2 + (n + l − m) 2
These lines are perpendicular, iff
= ( l 2 + m2 + n2 + 2ml − 2nl − 2mn)
+ ( m2 + n2 + l 2 + 2mn − 2ml − 2nl) a1 a1 + b1 b2 + c1 c 2 = 0
+ (n2 + l 2 + m2 + 2ml + 2nl + 2mn) a b c
and parallel, iff 1 = 1 = 1
+ (n2 + l 2 + m2 + 2nl − 2ml − 2mn) a 2 b2 c 2
= 1 + 2ml − 2nl − 2mn + 1 + 2mn − 2ml − 2nl
EXAMPLE 4. What is the angle between the two
+ 1 + 2ml + 2nl + 2mn + 1 + 2nl − 2ml − 2mn = 4 lines whose direction numbers are ( 3 − 1, − 3 − 1, 4)
[Q l 2 + m2 + n2 = 1] and (− 3 − 1, 3 − 1, 4)?
π π π π
a. b. c. d.
LINE 6 4 3 2
If a, b, c are the direction ratios of a line passing through Sol. Required angle is given by
the point ( x1 , y1 , z1 ), then the equation of line is a1a2 + b1b2 + c1 c2
cos θ =
x − x1 y − y1 z − z1 a12 + b12 + c12 a22 + b22 + c22
= = and equation of line in terms of
a b c −2 − 2 + 16 12 1 π
x − x1 y − y1 z − z1 = = = ⇒θ =
direction cosines l, m, n is = = 24 24 24 2 3
l m n
(i) General coordinates of line For Finding Foot of the Perpendicular
x − x1 y − y1 z − z1 from a Given Point
= = = r (say)
l m n P (u, v, w)
are ( x1 + lr , y1 + mr , z1 + nr )
(ii) Equation of line passing through the two points Q
( x1 , y1 , z1 ) and ( x 2 , y 2 , z 2 ) is A B
x − x1 y − y1 z − z1
x − x1 y − y1 z − z1 = =
= = a b c
x 2 − x1 y 2 − y1 z 2 − z1 1. Write the coordinates of Q (the foot of the
(iii) Equation of line passing through ( x1 , y1 , z1 ) and perpendicular) say ( x1 + ar , y1 + br , z1 + c r ).
parallel to line having direction ratios < a, b , c > is 2. Write the direction ratio of PQ.
x − x1 y − y1 z − z1 3. Apply condition of perpendicularity, i.e. PQ ⊥ AB.
= =
a b c 4. Get the value of r and substitute in step (1) to get Q.
(iv) Equation of line passing through ( x1 , y1 , z1 ) and
EXAMPLE 5. Find the foot of the perpendicular drawn
parallel to line passing through points P and Q,
x − x1 y − y1 z − z1 from the point P(1, 2, 3) to the line
= = , where PQ is the direction x −6 y −7 z −7
PQ PQ PQ = = .
3 2 −2
ratios of line joining P and Q. a. ( 3, 2, − 5) b. ( 3, 5, 9) c. ( 4 , 2, − 3) d. None of these
Angle between Two Lines Sol. Any point on the line is 3r + 6, 2r + 7, −2r + 7
(i) The angle θ between two lines whose direction Let it to be Q.
cosines are l1 , m1 , n1 and l 2 , m2 , n2 is given by Direction ratios of PQ are ( 3r + 6 − 1), ( 2r + 7 − 2),
( −2r + 7 − 3)
cos θ = l1 l 2 + m1 m2 + n1 n2 . i.e. ( 3r + 5), ( 2r + 5), ( −2r + 4)
These two lines are perpendicular iff Direction ratios of line are 3, 2, − 2.
l1 m1 n1 ∴ 3( 3r + 5) + 2( 2r + 5) −2( −2r + 4) = 0 [QPQ ⊥ AB]
l1 l 2 + m1 m2 + n1 n2 = 0 and parallel iff = = ⇒ 17r + 17 = 0 ⇒ r = − 1
l 2 m2 n2 ∴ Q is (3, 5, 9)
MATHEMATICS Three Dimensional Geometry 221

EXAMPLE 6. What is the equation of the plane


PLANE passing through (x1 , y 1 , z 1 ) and normal to the line with
A plane is a surface such that any two points lying on < a, b, c > as direction ratios?
it when joined by a line lies completely on it. a. ax + by + cz = ax1 + by1 + cz1
Equations of Plane in Different Form b. a( x + x1) + b ( y + y1) + c( z + z1) = 0
c. ax + by + cz = 0 d. ax + by + cz = x1 + y1 + z1
1. The general equation of the first degree in x, y, z,
i.e. ax + by + cz + d = 0, represents a plane in which Sol. The equation of the plane passing through ( x1, y1, z1) and
a, b, c are constants, and a 2 + b 2 + c 2 ≠ 0, i.e. normal to the line with < a, b, c > as direction ratios, is
a, b, c ≠ 0. a( x − x1) + b( y − y1) + c( z − z1) = 0
⇒ ax − ax1 + by − by1 + cz − cz1 = 0
2. The equation of the plane passing through the
⇒ ax + by + cz = ax1 + by1 + cz1
origin is given by ax + by + cz = 0.
which is required equation of plane.
3. Equation of a plane through point ( x1 , y1 , z1 ) is
a ( x − x1 ) + b ( y − y1 ) + c ( z − z1 ) = 0
Angle between Two Planes
Let the two planes be a1 x + b1 y + c1 z + d = 0 ...(i)
4. If l, m, n are the direction cosines of the normal to
and a 2 x + b2 y + c 2 z + d = 0 ...(ii)
the plane and p is the perpendicular distance of the
where ( a 1 , b1 , c 1 ) and ( a 2 , b2 , c 2 ) are the DR’s of normal
plane from origin, then the equation of the plane
to the planes (i) and (ii), respectively.
ABC will be
Let θ be the angle between the planes (i) and (ii), then
lx + my + nz = p a1 a 2 + b1 b2 + c1 c 2
cos θ = ±
5. Let the plane ABC cut the axes OX , OY , OZ at
a12 + b12 + c12 a 22 + b22 + c 22
A , B, C respectively.
x y z If given planes are perpendicular to each other, then
Let OA = a, OB = b, OC = c . Then, + + = 1 θ = 90 ° ⇒ cos θ = 0
a b c
Z i.e. a1 a 2 + b1 b2 + c1 c 2 = 0
C(0,0,c) If the given planes are parallel, then their normals are also
parallel, i.e. the directon cosines of the normal are
c proportional
A(a,0,0) a1 b1 c1
O
a X i.e. = =
b a 2 b2 c 2
Y B(0,b,0)
Distance of a Plane from a Point
is the required intercept form of the plane. Distance or perpendicular distance of a plane
6. Equation of XY-plane is z = 0. ax + by + cz + d = 0 from a point P( x1 , y1 , z1 ) is given by
7. Equation of YZ-plane is x = 0.  ax + by + cz + d 
P = 1 1 1

8. Equation of XZ-plane is y = 0.  a + b + c2 
2 2

9. Equation of plane parallel to YZ-plane and at a


distance a is x = a. Distance between Two Parallel Planes
10. Equation of plane parallel to ZX-plane and at a Distance between two parallel planes ax + by + cz + d 1 = 0
distance b is y = b.  d1 − d 2 
11. Equation of plane parallel to XY-plane and at a and ax + by + cz + d 2 = 0 is given by  
 a +b +c 
2 2 2
distance c is z = c.
12. Equation of plane passing through the three Intersection of Two Planes
points A ( x1 , y1 , z1 ), B( x 2 , y 2 , z 2 ) and C( x 3 , y 3 , z 3 )
If a1 x + b1 y + c1 z + d 1 = 0 and a 2 x + b2 y + c 2 z + d 2 = 0
is
represents two different planes, then equation of plane
x − x1 y − y1 z − z1 passing through the intersection of these planes is
x − x2 y − y2 z − z2 = 0 given by
x − x3 y − y3 z − z3 ( a1 x + b1 y + c1 z + d 1 ) + λ ( a 2 x + b2 y + c 2 z + d 2 ) = 0
222 NDA/NA Pathfinder

Angle between a Line and a Plane x 2 − x1 y 2 − y1 z 2 − z1


Let the equations of the a line and a plane be l1 m1 n1
x − x1 y − y1 z − z1 l2 m2 n2
= = and ax + by + cz + d = 0 d=
l m n $i $j k$
respectively, then the angle θ between them is given by
al + bm + cn l1 m1 n1
sin θ =
l2 m2 n2
a + b + c 2 l 2 + m2 + n 2
2 2

If a line is parallel to the plane, then θ = 0


⇒ al + bm + cn = 0 and if line is perpendicular to the SPHERE
a b c A sphere is the locus of a point which moves in space in
plane, then = =
l m n such a way that its distance from a fixed point always
remains constant.
EXAMPLE 7. The angle between the line 6 x = 4y = 3z
x y z 1 (i) The general equation
and the plane + − = is
4 6 4 3 x 2 + y 2 + z 2 + 2ux + 2vy + 2wz + d = 0
 32  represents a sphere with centre = ( −u, − v, − w ) and
a. 45° b. 0° c. tan− 1   d. 90°
 29 22 
radius = u 2 + v 2 + w 2 − d
Sol. . Angle between the plane and line is
(ii) Let P( x, y, z) be any point on the surface of a sphere
al + bm + cn
sin θ = whose centre is ( a, b, c ) and radius r, then by
a2 + b2 + c2 l 2 + m2 + n2
definition its equation is
x y z
Equation of line = = and equation of plane ( x − a) 2 + ( y − b) 2 + ( z − c ) 2 = r 2 ...(i)
2 3 4
3x + 2y − 3z − 4 = 0 If centre is origin, then the equation of sphere
al + bm + cn = 2 × 3 + 3 × 2 − 4 × 3 = 0 reduces to
∴ sin θ = 0 ⇒ θ = 0° x 2 + y 2 + z 2 = r 2 [putting a = b = c = 0 in Eq. (i)]
Coplanarity of Two Lines (iii) If ( x1 , y1 , z1 ) and ( x 2 , y 2 , z 2 ) be the end points of
x − x1 y − y1 z − z1 diameter of any sphere, then the equation of sphere
The two lines = = and is given by
l1 m1 n1
( x − x1 )( x − x 2 ) + ( y − y1 )( y − y 2 ) +( z − z1 )( z − z 2 ) = 0
x − x2 y − y2 z − z2
= = are coplanar, if (iv) ( x − x1 ) ( x − x 2 ) + ( y − y1 )( y − y 2 ) + ( z − z1 )
l2 m2 n2
( z − z 2 ) = 0 represents a sphere with ends of
x 2 − x1 y 2 − y1 z 2 − z1 diameter as ( x1 , y1 , z1 ) and ( x 2 , y 2 , z 2 ). Its centre is
l1 m1 n1 =0  x1 + x 2 y1 + y 2 z1 + z 2 
 , , 
l2 m2 n2  2 2 2 
and the equation of plane containing them is EXAMPLE 8. Find the equation of the sphere having the
x − x1 y − y1 z − z1 x − x2 y − y2 z − z2 centre (−2, 2, 3) and passing through the point (3, 4, − 1).
l1 m1 n1 = 0or l1 m1 n1 =0 a. x 2 + y 2 + z 2 + 4 x + 4 y + 6 z + 28 = 0
l2 m2 n2 l2 m2 n2 b. x 2 + y 2 + z 2 − 4 x − 4 y − 6 z + 28 = 0
c. x 2 + y 2 + z 2 + 4 x + 4 y + 6 z − 28 = 0
Skew-lines d. None of the above
The straight line which are not parallel and Sol. The equation of the sphere with centre ( −2, 2, 3) is
non-coplanar, i.e. non-intersecting are called skew-lines. ( x + 2) 2 + ( y − 2) 2 + ( z − 3) 2 = r 2
Shortest Distance between Two Skew-lines Radius, r = ( 3 + 2) 2 + ( 4 − 2) 2 + ( −1 − 3) 2 = 45
The shortest distance between the lines ∴Required equation of the sphere is
x − x1 y − y1 z − z 2 ( x + 2) 2 + ( y − 2) 2 + ( z − 3) 2 = ( 45) 2
= =
l1 m1 n1 ⇒ x 2 + y 2 + z 2 + 4x − 4y − 6z − 28 = 0
x − x2 y − y2 z − z2
and = = is given by Note Condition for orthogonal intersection of two spheres is
l2 m2 n2 2u1u 2 + 2 v1v 2 + 2 w1w2 = d 1 + d 2
223

PRACTICE EXERCISE
1. The ratio in which the line joining (2, 4, 5), 11. Equation of the plane through P( 2, 3, − 1) at right
( 3, 5, − 4) is divided by the YZ-plane is angle to OP is
(a) 2 : 3 (b) 3 : 2 (c) −2 : 3 (d) 4 : − 3 (a) 2 x + 3 y − z = 14 (b) 2 x + y − z = (14)
(c) 2 x + y + z + 14 = 0 (d) None of these
2. A straight line which makes an angle of 60° with
each of Y and Z-axes, is inclined with X-axis at 12. Equation of the plane that passes through the point
an angle ( 2, − 3, 1) and is perpendicular to the line joining the
(a) 45° (b) 30° (c) 75° (d) 60° points ( 3, 4,− 1) and ( 2, − 1, 5) is given by
(a) x + 5 y − 6 z + 19 = 0 (b) x − 5 y + 6 z − 23 = 0
3. The foot of the perpendicular from (0, 2, 3) to the
x+ 3 y−1 z + 4 (c) x + 5 y − 6 z = 19 (d) None of these
line = = is
5 2 3 13. Direction cosines of the line which is
(a) (−2, 3, 4) (b) (2, − 1, 3) (c) (2, 3, − 1) (d) (3, 2, − 1) perpendicular to the lines whose direction ratios
x−2 y−3 z−4 are 1, − 1, 2 and 2, 1, − 1, are given by
4. The line = = is parallel to the plane
(a)  −
3 
(b)  −
1 5 1 5 3 
3 4 5 , , ,− ,
 35 35 35   35 35 35 
(a) 2 x + y − 2 z = 0 (b) 3x + 4 y + 5 z = 7
(c) 
(c) x + y + z = 2 (d) 2 x + 3 y + 4 z = 0 1 5 3 
, ,− (d) None of these
 35 35 35 
5. The equation of line through the point (1, 2, 3)
x − 4 y + 1 z + 10 14. Under what condition do the planes bx − ay = n ,
parallel to line = = is
2 −3 8 cy − bz = l and az − cx = m intersect in a line?
x−1 y−2 z− 3 x−1 y−2 z− 3 (a) a + b + c = 0 (b) a = b = c
(a) = = (b) = =
2 −3 8 1 2 3 (c) al + bm + cn = 0 (d) l + m + n = 0
x − 4 y + 1 z + 10
(c) = = (d) None of these
1 2 3 15. A line makes the same angle α with each of the
6. The points (1, 3, 4), ( −1, 6, 10), ( −7, 4, 7) and X and Y-axes. If the angle θ, which it makes with
the Z-axis, is such that sin2 θ = 2 sin2 α, then
( −5, 1, 1) are the vertices of a
(a) rhombus (b) rectangle what is the value of α?
π π π π
(c) parallelogram (d) square (a) (b) (c) (d)
4 6 3 2
7. The angle between the lines with direction ratios
(1, 0, ± cos α ) is 60°. What is the value of α? 16. The line passing through (1, 2, 3) and having
−1  1  −1  1  direction ratios given by < 1, 2, 3 > cuts the X-axis
(a) cos   (b) cos  
 2  3 at a distance k from origin. What is the value of k?
(c) cos −1  1 (d) cos −1  1 (a) 0 (b) 1 (c) 2 (d) 3
   
 3 2
17. What is the equation of the sphere which has its
8. The equation of the plane passing through the centre at ( 6, − 1, 2) and touches the plane
point ( −2, − 2, 2) and containing the line joining 2x − y + 2z − 2 = 0?
the points (1, 1, 1) and (1, − 1, 2) is (a) x2 + y2 + z2 + 12 x − 2 y + 4 z + 16 = 0
(a) x + 2 y − 3 z + 4 = 0 (b) 3x − 4 y + 1 = 0
(b) x2 + y2 + z2 + 12 x − 2 y + 4 z − 16 = 0
(c) 5x + 2 y − 3 z − 17 = 0 (d) x − 3 y − 6 z + 8 = 0
(c) x2 + y2 + z2 − 12 x + 2 y − 4 z + 16 = 0
9. If from a point P ( a , b, c) perpendiculars PA, PB (d) x2 + y2 + z2 − 12 x + 2 y − 4 z + 25 = 0
are drawn to YZ-plane and ZX-plane
respectively, then the equation of the plane OAB 18. Consider the following relations among the
is angles α , β and γ made by a vector with the
(a) bcx + cay + abz = 0 (b) bcx + cay − abz = 0 coordinate axes.
(c) bcx − cay + abz = 0 (d) − bcx + cay + abz = 0 I. cos 2α + cos 2β + cos 2γ = − 1
10. Equation of the plane perpendicular to the plane II. sin 2 α + sin 2 β + sin 2 γ = 1
x − 2 y + 5z + 1 = 0 which passes through the Which of the above statement(s) is/are correct?
points ( 2, − 3, 1) and ( −1, 1, − 7) is given by (a) Only I (b) Only II
(a) 4x − 4 y + z + 7 = 0 (b) 4x + 7 y + 2 z + 11 = 0 (c) Both I and II (d) Neither I nor II
(c) 2 x + y − z = 0 (d) None of these
224 NDA/NA Pathfinder

19. Consider the following statements 26. What is the angle between one of the edges of the
I. Equation of plane parallel to X-axis is x = 0. cube and the diagonal of the cube intersecting the
II. Projection of P (x, y, z ) on the plane YOZ is (0, y, z ). edge of the cube?
(a) cos −1   (b) cos −1  
1 1
Which of the above statement(s) is/are correct?
2  3
(a) Only I (b) Only II
(c) cos −1 
1 
(d) cos −1 
(c) Both I and II (d) Neither I nor II 2 
 
 3  3
20. Consider the points ( a − 1, a , a + 1), ( a , a + 1, a − 1)
and ( a + 1, a − 1, a ). 27. What is the angle between the diagonal of one
of the faces of the cube and the diagonal of the
I. These points always form the vertices of an
cube intersecting the diagonal of the face of the
equilateral triangle for any real value of a.
cube?
II. The area of the triangle formed by these points
(a) cos −1 
1 
(b) cos −1 
2 
is independent of a.  
 3  3
III. The triangle formed with these vertices, have
 2  2
altitudes of length 3 / 2 units. (c) cos −1   (d) cos −1  
 3  3 
Which of the above statement(s) is/are correct?
(a) I and II (b) II and III (c) I, II and III (d) I and III
Directions (Q. Nos. 28-30) Two lines whose equations
x − 3 y − 2 z −1 x−2 y−3 z −2
Directions (Q. Nos. 21-22) Line AB in three are = = and = =
dimensional space makes angles α, β and γ with the 2 3 λ 3 2 3
coordinate axes. lie in the same plane.

21. If α = 45° and β = 120°, then the acute angle γ is 28. The value of sin −1 sin λ is equal to
equal to (a) 3 (b) π − 3
(a) 60° (b) 75° (c) 30° (d) 45° (c) 4 (d) π − 4

22. Consider the following statements 29. Point of intersection of the lines lies on
I. If α = 30° and β = 45°, then γ will be 150°. (a) 3x + y + z = 20 (b) 2 x + y + z = 25
II. If α + β = 90°, then γ will be 90°. (c) 3x + 2 y + z = 24 (d) x = y = z
Which of the above statement(s) is/are correct? 30. Angle between the plane containing both the
(a) Only I (b) Only II
lines and the plane 4x + y + 2z = 0 is equal to
(c) Both I and II (d) Neither I nor II π π
(a) (b)
3 2
Directions (Q. Nos. 23-24) Consider the sphere π 2
x 2 + y 2 + z 2 − 3x − 2y + 2 z − 15 = 0 (c) (d) cos −1
6 186
23. If one end of a diameter AB is A( −1, 4, − 3), then B is
Directions (Q. Nos. 31-33) Consider a plane
(a) (−2, 4, − 1) (b) (−4, 2, 1) (c) (4, 2, − 1) (d) (4, − 2, 1)
x + y − z = 1 and point A(1, 2, − 3). A line L has
24. The equation of the sphere concentric with the equation x = 1 + 3r, y = 2 − r and z = 3 + 4r.
given sphere and double the radius is
(a) x2 + y2 + z2 − 3x − 2 y + 2 z − 86 = 0 31. The coordinate of a point B of line L such that
AB is parallel to the plane is
(b) x2 + y2 + z2 − 3x + 2 y + 2 z − 86 = 0
(a) (10, − 1, 15) (b) (−5, 4, − 5)
(c) x2 + y2 − z2 − 3x − 2 y + 2 z + 86 = 0
(c) (4, 1, 7 ) (d) (−8, 5, − 9)
(d) None of the above
32. The equation of the plane containing line L and
Directions (Q. Nos. 25-27) The vertices of a cube are point A has the equation.
(0, 0, 0), (2, 0, 0), (0, 2, 0), (0, 0, 2), (2, 2, 0), (a) x − 3 y + 5 = 0 (b) x + 3 y − 7 = 0
(2, 0, 2), (0, 2, 2), (2, 2, 2), respectively. (c) 3x − y − 1 = 0 (d) 3x + y − 5 = 0
25. What is the angle between any two diagonals of 33. The distance between the points on the line
the cube? 4
(a) cos −1   (b) cos −1  
1 1 which are at a distance of from the plane is
2  3 3
(a) 4 26 (b) 20
(c) cos −1 
1 
(d) cos −1 
2 
 
 3  3 (c) 10 13 (d) None of these
MATHEMATICS Three Dimensional Geometry 225

Directions (Q. Nos. 34-36) The vertices of ∆ABC are


44. If the distance between the points (7, 1, –3) and
A(3, 2, 0), B(5, 3, 2) and C( −9, 6, − 3).
( 4, 5, λ ) is 13 units, then what is one of the
34. The bisector AD of ∠A meets BC at D. The values of λ? e 2012 II
coordinates of D are (a) 20 (b) 10 (c) 9 (d) 8
(a)  ,
19 57 17 
(b)  ,
19 57 17 
,  ,  45. What are the direction ratios of the line of
 4 4 4  8 16 16 
intersection of the planes x = 3z + 4 and
(c)  , , 
9 17 11 y = 2z − 3? e 2012 II
(d) None of these
2 2 2 
(a) 〈1, 2, 3〉 (b) 〈2, 1, 3〉 (c) 〈3, 2, 1〉 (d) 〈1, 3, 2 〉
35. If A, B and C are mid-points of PQ , QR and RP 46. What is the equation to the plane through
respectively, then centroid of ∆PQR is (1, 2, 3) parallel to 3 x + 4 y − 5z = 0 ? e 2012 II
−2 
(a)  , 1, (b)  0, ,
2 19 4 (a) 3x + 4 y + 5 z + 4 = 0 (b) 3x + 4 y − 5 z + 14 = 0
 
3 3  3 3 (c) 3x + 4 y − 5 z + 4 = 0 (d) 3x + 4 y − 5 z − 4 = 0
1 11 −1
(c)  − , ,  (d) None of these 47. What is the angle between the lines
 3 3 3
x − 2` y + 1 z + 2 x − 1 2y + 3 z + 5
= = and = = ?
36. The area of ( ∆ABC ) is equal to 1 −2 1 1 3 2
39 e 2012 II
(a) sq units (b) 13 sq units
2 π π π
(a) (b) (c) (d) None of these
3 5 13 5 2 3 6
(c) sq units (d) sq units
2 2 48. What are the direction cosines of a line which is
equally inclined to the positive directions of the
axes? e 2012 II
PREVIOUS YEARS’ QUESTIONS (a)
1 1 1
, , (b) −
1 1 1
, ,
3 3 3 3 3 3
37. What are the direction ratios of normal to the
plane 2x − y + 2z + 1 = 0? 1 1 1 1 1 1
e 2012 I (c) − ,− , (d) , ,
1 3 3 3 3 3 3
(a) 〈2, 1, 2 〉 (b) 1, − , 1 (c) 〈1, − 2, 1〉 (d) 〈1,1,2 〉
2 49. What is the angle between the planes
38. What is the cosine of angle between the planes 2x − y − 2z + 1 = 0 and 3x − 4 y + 5z − 3 = 0 ? e 2013 I
x + y + z + 1 = 0 and 2x − 2 y + 2z + 1 = 0? e 2012 I π π π π
1 (a) (b) (c) (d)
(a) 1/2 (b) 1/3 (c) 2/3 (d) 6 4 3 2
4 x − x0 y − y0 z − z 0
39. What is the diameter of the sphere 50. If the straight line = = is
l m n
x 2 + y 2 + z 2 − 4x + 6 y − 8z − 7 = 0? e 2012 I parallel to the plane ax + by + cz + d = 0, then
(a) 4 units (b) 5 units (c) 6 units (d) 12 units which one of the following is correct? e 2013 I
40. What is the sum of the squares of direction (a) l + m + n = 0 (b) a + b + c = 0
a b c
cosines of the line joining the points (1, 2, − 3) and (c) + + =0 (d) al + bm + cn = 0
( −2, 3, 1)? e 2012 I l m n

(a) 0 (b) 1 (c) 3 (d)


2 51. What is the distance between the planes
26 x − 2 y + z − 1 = 0 and −3x + 6 y − 3z + 2 = 0 ?
41. What is the equation of the straight line passing (a) 3 units (b) 1 unit e 2013 I
through (a, b, c) and parallel to Z-axis? e 2012 II (c) 0 (d) None of these

x− a y− b z−c x− a y− b z−c 52. What should be the value of k for which the
(a) = = (b) = = equation 3x 2 + 3 y 2 + ( k + 1) z 2 + x − y + z = 0,
1 0 0 0 0 1
x− a y− b z−c x− a y− b z−c represents the sphere? e 2013 I
(c) = = (d) = =
0 1 0 0 1 1 (a) 3 (b) 2 (c) 1 (d) − 1

42. What is the distance of the point (1, 2, 0) from 53. If a line makes 30° with the positive direction of
YZ-plane is e 2012 II X-axis, ∠ β with the positive direction of Y-axis
and ∠ γ with the positive direction of Z-axis, then
(a) 1 unit (b) 2 units (c) 3 units (d) 4 units
what is cos2 β + cos2 γ equal to? e 2013 I
43. If a line OP of length r (where, O is the origin) (a) 1/4 (b) 1/2 (c) 3/4 (d) 1
makes an angle α with X-axis and lies in the
XZ-plane, then what are the coordinates of P ? 54. What is the distance of the line 2x + y + 2z = 3
(a) (r cos α, 0, r sin α ) (b) (0, 0, r sin α ) e 2012 II from the origin? e 2013 II
(c) (r cos α, 0, 0) (d) (0, 0, r cos α ) (a) 1 unit (b) 1.5 units (c) 2 units (d) 2.5 units
226 NDA/NA Pathfinder

55. The sum of the direction cosines of Z-axis is 67. Consider the following statements
e 2013 II I. The two spheres intersect each other.
(a) 0 (b) 1/3 (c) 1 (d) 3 II. The radius of first sphere is less than that of
56. If θ is the acute angle between the diagonals of a second sphere.
cube, then which one of the following is correct? Which of the above statement(s) is/are correct?
e 2013 II (a) Only I (b) Only II
(a) θ = 30° (b) θ = 45° (c) 2 cos θ = 1 (d) 3cos θ = 1 (c) Both I and II (d) Neither I nor II
57. What is the equation of the sphere with unit
radius having centre at the origin? e 2013 II Directions (Q. Nos. 68-69) Read the following
(a) x + y + z = 0
2 2 2
(b) x + y + z = 1
2 2 2 information carefully and answer these question
given below.
(c) x + y + z = 2
2 2 2
(d) x + y + z = 3
2 2 2

The line joining the points (2, 1, 3) and ( 4, − 2, 5) cuts the


58. What is the sum of the squares of direction plane 2 x + y − z = 3. e 2014 II
cosines of X-axis? e 2013 II
1 68. Where does the line cut the plane?
(a) 0 (b) (c) 1 (d) 3 (a) (0, − 4, − 1) (b) (0, − 4, 1) (c) (1, 4, 0) (d) (0, 4, 1)
3
59. If a line passes through the points ( 6, − 7, − 1) and 69. What is the ratio in which the plane divides the
line?
( 2, − 3, 1), then what are the direction ratios of
(a) 1 : 1 (b) 2 : 3 (c) 3 : 4 (d) None of these
the line? e 2014 I
(a) 〈4, − 4, 2 〉 (b) 〈4, 4, 2 〉 (c) 〈−4, 4, 2 〉 (d) 〈2, 1, 1〉
Directions (Q. Nos. 70-71) Read the following
Directions (Q. Nos. 60-62) A straight line passes information carefully and answer the question given
through (1, − 2, 3) and perpendicular to the plane below.
2 x + 3y − z = 7. e 2014 I Consider the plane passing through the points A(2, 2, 1),
B(3, 4, 2) and C( 7, 0, 6). e 2014 II
60. What are the direction ratios of normal to plane?
(a) < 2, 3, − 1 > (b) < 2, 3, 1 > 70. Which one of the following points lies on the
(c) < −1, 2, 3 > (d) None of these plane?
(a) (1, 0, 0) (b) (1, 0, 1) (c) (0, 0, 1) (d) None of these
61. Where does the line meet the plane?
(a) (2, 3, − 1) (b) (1, 2, 3) (c) (2, 1, 3) (d) (3, 1, 2) 71. What are the direction ratios of the normal to
the plane?
62. What is the image of the point (1, − 2, 3) in the (a) <1, 0, 1> (b) < 0, 1, 0 >
plane? (c) < 1, 0, − 1 > (d) None of these
(a) (2, − 1, 5) (b) (−1, 2, − 3) (c) (5, 4, 1) (d) None of these
Directions (Q. Nos. 72-74) Read the following
Directions (Q. Nos. 63-65) The vertices of a ∆ABC information carefully and answer the question given
are A(2, 3, 1), B( −2, 2, 0) and C(0, 1, − 1). e 2014 I below.Consider a sphere passing through the origin
and the points (2, 1, − 1), (1, 5, − 4), ( −2, 4, −6).
63. What is the cosine of ∠ABC ?
1 1 2 e 2014 II
(a) (b) (c) (d) None of these
3 2 6 72. What is the radius of the sphere?
64. What is the area of the triangle? (a) 12 (b) 14 (c) 12 (d) 14
(a) 6 2 sq units (b) 3 2 sq units 73. What is the centre of the sphere?
(c) 10 3 sq units (d) None of these (a) (−1, 2, − 3) (b) (1, − 2, 3) (c) (1, 2, − 3) (d) (−1, − 2, − 3)
65. What is the magnitude of the line joining 74. Consider the following statements
mid-points of the sides AC and BC? I. The sphere passes through the point ( 0, 4, 0).
1 3
(a) unit (b) 1 unit (c) units (d) 2 units II. The point (1, 1, 1) is at a distance of 5 unit from
2 2
the centre of the sphere.
Directions (Q. Nos. 66-67) Consider the spheres Which of the above statement(s) is/are correct?
x 2 + y 2 + z 2 − 4y + 3 = 0 and (a) Only I (b) Only II
(c) Both I and II (d) Neither I nor II
x 2 + y 2 + z 2 + 2 x + 4z − 4 = 0. e 2014 I
66. What is the distance between the centres of the Directions (Q. Nos. 75-76) The projections of a
two spheres? directed line segment on the coordinate axes are
(a) 5 units (b) 4 units (c) 3 units (d) 2 units 12, 4, 3, respectively. e 2015 I
MATHEMATICS Three Dimensional Geometry 227

(a) x + y = 3 (b) x − y = −1
75. What is the length of the line segment? (c) z = 3 (d) 2 y + 3 zx = 14
(a) 19 units (b) 17 units (c) 15 units (d) 13 units
83. The lines 2x = 3 y = − z and 6x = − y = − 4z e 2015 II
76. What are the direction cosines of the line segment?
(a) are perpendicular (b) are parallel
(a)  ± , ± , ±  (b)  , − , 
12 4 3 12 4 3
 13 (c) intersect at an angle 45° (d) intersect at an angle 60°
13 13   13 13 13 

(c)  , − , −  (d)  − , − , 
12 4 3 12 4 3 84. The radius of the sphere
 13 13 13   13 13 13 
3x2 + 3 y2 + 3z 2 − 8x + 4 y + 8z − 15 = 0 is e 2015 II
(a) 2 (b) 3 (c) 4 (d) 5
Directions (Q. Nos. 77-78) From the point
P(3, − 1, 11), a perpendicular is drawn on the line L 85. The direction ratios of the line perpendicular
x y−2 z −3 to the lines with direction ratios < 1, − 2, − 2 >
given by the equation = = . Let Q be
2 3 4 and < 0, 2, 1 > are e 2015 II
the foot of the perpendicular. e 2015 I (a) < 2, – 1, 2 > (b) < – 2, 1, 2 >
(c) < 2, 1, − 2 > (d) < − 2, − 1, − 2 >
77. What are the direction ratios of the line segment
PQ? 86. What are the coordinates of the foot of the
(a) (1, 6, 4) (b) (−1, 6, − 4) (c) (−1, − 6, 4) (d) (2, − 6, 4) perpendicular drawn from the point (3, 5, 4) on
the plane z = 0 ? e 2015 II
78. What is the length of the line segment PQ ? (a) (0, 5, 4) (b) (3, 5, 0) (c) (3, 0, 4) (d) (0, 0, 4)
(a) 47 un (b) 7 un (c) 53 un (d) 8 un
* un = unit 87. The lengths of the intercepts on the coordinate
axes made by the plane 5x + 2 y + z − 13 = 0 are
Directions (Q. Nos. 79-80) A triangular plane ABC e 2015 II

(b)  , , 13 units


with centroid (1, 2, 3) cuts the coordinate axes at A, 13 13
(a) (5, 2, 1) units
B, C, respectively. e 2015 I  5 2 

(c)  , ,  units
79. What are the intercepts made by the plane ABC 5 2 1
(d) (1, 2, 5) units
on the axes?  13 13 13 
(a) 3, 6, 9 (b) 1, 2, 3 (c) 1, 4, 9 (d) 2, 4, 6
80. What is the equation of the plane ABC? Directions (Q. Nos. 88-90) A plane P passes through
the line of intersection of the planes 2 x − y + 3z = 2,
(a) x + 2 y + 3 z = 1 (b) 3x + 2 y + z = 3 x + y − z = 1 and the point (1, 0, 1). e 2016 I
(c) 2 x + 3 y + 6 z = 18 (d) 6x + 3 y + 2 z = 18
88. What are the direction ratios of the line of
Directions (Q. Nos. 81-82) A point P(1, 2, 3) is one intersection of the given planes?
vertex of a cuboid formed by the coordinate planes (a) < 2, − 5, − 3 > (b) < 1, − 5, − 3 >
and the planes passing through P and parallel to the (c) < 2, 5, 3 > (d) < 1, 3, 5 >
coordinate planes. e 2015 II 89. What is the equation of the plane P ?
81. What is the length of one of the diagonals of the (a) 2 x + 5 y − 2 = 0 (b) 5x + 2 y − 5 = 0
cuboid? (c) x + z − 2 = 0 (d) 2 x − y − 2 z = 0
(a) 10 un (b) 14 un (c) 4 un (d) 5 un
90. If the plane P touches the sphere
* un = units
82. What is the equation of plane passing through x 2 + y 2 + z 2 = r 2, then what is r equal to?
P (1,2,3) and parallel to XY -plane? 2 4 5
(a) units (b) units (c) units (d) 1 unit
29 29 29

ANSWERS
1 a 2 a 3 c 4 a 5 a 6 a 7 b 8 d 9 b 10 b
11 a 12 a 13 a 14 c 15 a 16 a 17 c 18 a 19 b 20 c
21 a 22 b 23 d 24 d 25 b 26 c 27 c 28 d 29 d 30 b
31 d 32 b 33 d 34 b 35 c 36 d 37 b 38 b 39 d 40 b
41 b 42 a 43 a 44 c 45 c 46 c 47 a 48 a 49 d 50 d
51 d 52 b 53 a 54 a 55 c 56 d 57 b 58 c 59 c 60 a
61 d 62 c 63 a 64 d 65 c 66 c 67 c 68 d 69 d 70 b
71 c 72 b 73 a 74 a 75 d 76 a 77 b 78 c 79 a 80 d
81 b 82 c 83 a 84 b 85 a 86 b 87 b 88 a 89 b 90 c
228 NDA/NA Pathfinder

HINTS AND SOLUTIONS


1. (a) Let the required ratio be λ : 1 . Then, CD = ( −5 + 7)2 + ( 1 − 4)2 + ( 1 − 7)2 It passes through A and B, so
the point is 0 p + bq + cr = 0 and ap + 0q + cr = 0
=7
 3 λ + 2 , 5 λ + 4 , −4 λ + 5  p q r
  DA = ( 1 + 5) + (3 − 1) + ( 4 − 1)
2 2 2 ∴ = = =λ [say]
 λ+ 1 λ+ 1 λ+ 1  bc ac − ab
It lies on YZ-plane, so its x-coordinate =7 So, the equation of the plane OAB is
of x = 0 AC = ( −7 − 1)2 + ( 4 − 3)2 + ( 7 − 4)2 bcx + cay − abz = 0
3λ + 2 2
i.e. =0 ⇒ λ=− = 74 10. (b) Equation of a plane through
λ+ 1 3
and ( 2, − 3, 1) is
So, the ratio is 2 : 3 externally. a( x − 2) + b( y + 3) + c (z − 1) = 0 ...(i)
BD = ( −5 + 1)2 + ( 1 − 6)2 + ( 1 − 10)2
2. (a) Let α be the angle, then the DC’s of It passes through ( −1, 1, − 7).
the line are  cos α , , 
1 1 = 122 ∴ − 3a + 4b − 8 c = 0 ...(ii)
 2 2  Q AB = BC = CD = DA It is perpendicular to the plane
∴ By the formula x − 2 y + 5z + 1 = 0
But BD ≠ AC
cos 2 α + cos 2 β + cos 2 γ = 1, we get ∴ a − 2b + 5c = 0 ...(iii)
1 1 1 So, the points A, B, C and D are the On solving Eqs. (ii) and (iii), we get
∴ cos 2 α + + = 1 ⇒ cos 2 α = vertices of a rhombus. a b c
4 4 2 = = =λ
⇒ α = 45°  1× 1+ 0 × 0  4 7 2
 + (cos α )( − cos α ) On putting these values of a , b and c
3. (c) Any point on the given line is 7. (b) cos 60° =   in Eq. (i), we get the equation of the
(5r − 3, 2r + 1, 3r − 4). If it is the foot of  12 + ( 0)2 + cos 2 α  required plane
the perpendicular from (0, 2, 3), then  2 ⋅ 2
 1 + ( 0) + ( − cos α ) 
2 4x + 7 y + 2z + 11 = 0
5(5r − 3 − 0) + 2( 2r + 1 − 2)
11. (a) Coordinates of the given point P are
+ 3(3r − 4 − 3) = 0 ⇒ =
1 1 − cos 2 α ( 2, 3, − 1).
⇒ 38r = 38 ⇒ r = 1 2 1 + cos 2 α 1 + cos 2 α OP is normal to the required plane,
So, foot of the perpendicular is 1 1 − cos 2 α 1+ 2 2 so direction ratios of the normal to
⇒ = ⇒ = the plane are 2, 3, − 1.
( 2, 3, − 1). 2 1 + cos 2 α 1 − 2 −2 cos 2 α
x −α y −β z − γ So, equation of the plane through
4. (a) Let the line = = is [applying componendo and dividendo] P ( 2, 3, − 1) is
l m n

3
=
1
⇒ cos α =
1 a( x − 2) + b ( y − 3) + c (z + 1) = 0.
parallel to the plane
−1 − cos 2 α 3 Since, the direction ratios of the
ax + by + cz + d = 0
normal to the plane are 2, 3, − 1.
α = cos −1  
Then, normal to the plane is 1
∴ a b c
perpendicular to the line.  3 So, we have = =
2 3 −1
i.e. al + bm + cn = 0 8. (d) Equation of a plane through Hence, the equation of the required
In this question, this condition is ( −2, − 2, 2) is given by
plane is
satisfied by the plane 2x + y − 2z = 0. a( x + 2) + b ( y + 2) + c (z − 2) = 0 2( x − 2) + 3( y − 3) − 1(z + 1) = 0
5. (a) Let the equation of line passing It contains the line joining the points ⇒ 2x + 3 y − z = 14
through (1, 2, 3) is (1, 1, 1) and B ( 1, − 1, 2), so these
x − 1 y − 2 z −3 points also lie in the plane. 12. (a) Direction ratios of the line joining
= = the points A(3, 4, − 1), B( 2, − 1, 5) are
a b c ∴ At (1, 1, 1) 3a + 3b − c = 0 3 − 2, 4 + 1, − 1 − 5, i.e. 1, 5, − 6.
But it is parallel to the given line and at (1, −1, 2) 3a + b + 0 ⋅ c = 0 Equation of the plane through
a b c
∴ = = ⇒
a
=
b
=
c
=r
2 −3 8
[say] P ( 2, − 3, 1) is a ( x − 2)
1 −3 −6
Hence, the required line is + b ( y + 3) + c (z − 1) = 0 ...(i)
⇒ a = r , b = − 3r , c = − 6r
x − 1 y − 2 z −3 This plane is perpendicular to the line
= = . So, equation of the plane is
2 −3 8 AB, so the direction ratios of the
6. (a) Let points A, B, C and D have x − 3 y − 6z + 8 = 0. normal to the plane (i) are 1, 5, − 6 and
coordinates (1, 3, 4), ( −1, 6, 10), ( −7, 4, 7) hence, we have
9. (b) A and B are the foot of perpendicular a b c
and ( −5, 1, 1), respectively. from P ( a , b , c ) to YZ-plane and = =
1 5 −6
∴ AB = ( −1 − 1)2 + ( 6 − 3)2 + ( 10 − 4)2 ZX -planes are given by A( 0, b , c ) and
B ( a , 0, c ). A plane through origin Therefore, the equation of the
=7 (0, 0, 0) is required plane is
BC = ( −7 + 1)2 + ( 4 − 6)2 + ( 7 − 10)2 1( x − 2) + 5( y + 3) − 6(z − 1) = 0
px + qy + rz = 0 ...(i)
=7 ⇒ x + 5 y − 6z + 19 = 0
MATHEMATICS Three Dimensional Geometry 229

13. (a) Let ( a , b , c ) be the direction ratios of given line. II. sin 2 α + sin 2 β + sin 2 γ
∴ a − b + 2c = 0 …(i) = 1 − cos 2 α + 1 − cos 2 β + 1 − cos 2 γ
2a + b − c = 0 …(ii)
= 3 − (cos 2 α + cos 2 β + cos 2 γ ) = 3 − 1 = 2 [from Eq. (i)]
From Eqs. (i) and (ii), we get
a b c So, Statement II is false.
= =
−1 5 3 19. (b) I. General equation of a plane is
So, ( −1, 5, 3) are the direction ratios of given line. ax + by + cz + λ = 0
Hence, direction cosines will be Normal to the plane will be perpendicular to X -axis
 −1 , 5 , 3  . DR of normal to plane are a , b , c.
 
 35 35 35 
DR of X -axis are 1, 0, 0.
14. (c) The given planes are ∴ a⋅ 1 + b⋅ 0 + c⋅ 0 = 0 ⇒ a = 0
bx − ay − n = 0 …(i) ∴ Required equation is by + cz + λ = 0
cy − bz − l = 0 …(ii) So, Statement I is false.
az − cx − m = 0 …(iii) II. On YOZ plane, we have x = 0
The equation of planes passing through the line of intersection ∴ Projection of P( x , y , z ) on YOZ plane is ( 0, y , z ).
of planes (i) and (ii) are
So, Statement II is true.
( bx − ay − n) + λ( cy − bz − l ) = 0
or bx + ( − a + λc ) y − ( bλ )z + ( − n − λl ) = 0 …(iv) 20. (c) Let A( a − 1, a , a + 1),
Eqs. (iii) and (iv) are identical. B ( a , a + 1, a − 1)
b − a + λc − bλ −( n + λl ) and C ( a + 1, a − 1, a ) be the vertices of a ∆ABC .
∴ = = =
−c 0 a −m ∴ AB = ( a − a + 1)2 + ( a + 1 − a )2 + ( a − 1 − a − 1)2 = 6
a − bm − cn
⇒ λ = and λ =
c cl BC = ( a + 1 − a )2 + ( a − 1 − a − 1)2 + ( a − a + 1)2 = 6
a − bm − cn
∴ = ⇒ al + bm + cn = 0
c cl and CA = ( a − 1 − a − 1)2 + ( a − a + 1)2 + ( a + 1 − a )2 = 6
15. (a) Q l + m + n = 1
2 2 2
∴ AB = BC = CA
⇒ cos 2 α + cos 2 α + cos 2 θ = 1 ...(i) Hence, given points are vertices of an equilateral triangle for
any real value of a.
Also, sin 2 θ = 2 sin 2 α ⇒ 1 − cos 2 θ = 2( 1 − cos 2 α )
 $i $j k$ 
⇒ cos 2 θ = 2 cos 2 α − 1
1 1 
From Eq. (i), 2 cos 2 α + 2 cos 2 α − 1 = 1 Now, area of a ∆ABC = | AB × AC | =  1 1 −2
2 2 
⇒ 4 cos 2 α = 2 ⇒ cos 2 α =
1 2 −1 −1
2  
1 π 3π 1
= | −3 i − 3 j − 3 k | =
$ 1
9+ 9+ 9 =
27
⇒ cos α = ± ⇒ α= , $ $ sq units
2 4 4 2 2 2
Thus, the area of triangle formed by these points is
16. (a) The equation of line which is passing through (1, 2, 3) and independent of a.
having direction ratio (1, 2, 3) is
As it is an equilateral triangle of side 6 units.
x− 1 y− 2 z−3
= = = a [say] Thus, its Altitude =
3
× side units =
3
× 6=
3
units
1 2 3 2 2 2
∴ x − 1 = a, y − 2 = 2a and z − 3 = 3a
1 1
⇒ x = a + 1, y = 2a + 2 and z = 3a + 3 21. (a) We have, l = cos 45° = ,m = cos 120° = − and n = cos γ
2 2
At X-axis, y = 0 and z = 0 ⇒ 2a + 2 = 0 and 3a + 3 = 0 1 1
∴ l 2 + m 2 + n2 = 1 ⇒ + + cos 2 γ = 1
⇒ a = − 1 and a = − 1 2 4
∴ x = ( −1) + 1 = 0 1 1
⇒ cos γ = ⇒ cos γ = [Q γ is acute]
Hence, the line cuts X-axis at a distance k = 0. 4 2
⇒ γ = 60°
17. (c) Given centre of sphere is ( 6, − 1, 2).
2( 6) − 1 ( −1) + 2( 2) − 2 15 3 1
∴ Radius = = =5 22. (b) I. l = cos30° = , m = cos 45° = , n = cos γ
4 + 1+ 4 3 2 2
∴ Equation of sphere is ( x − 6)2 + ( y + 1)2 + (z − 2)2 = 52 ∴ l 2 + m 2 + n2 = 1
3 1 −1
⇒ x 2 + y 2 + z 2 − 12x + 2 y − 4z + 16 = 0 ⇒ + + cos 2 γ = 1 ⇒ cos 2 γ =
4 2 4
18. (a) We know that, Which is not possible.
cos 2 α + cos 2 β + cos 2 γ = 1 ...(i) So, Statement I is false.
I. cos 2 α + cos 2β + cos 2γ II. Here, cos 2 α + cos 2 ( 90° − α ) + cos 2 γ = 1
= 2 cos 2 α + 2 cos 2 β + 2 cos 2 γ − 3 ⇒ cos 2 α + sin 2 α + cos 2 γ = 1
= 2(cos 2 α + cos 2 β + cos 2 γ ) − 3 = 2( 1) − 3 = − 1 ⇒ cos 2 γ = 0 ⇒ γ = 90°
So, Statement I is true. [from Eq. (i)] So, Statement II is true.
230 NDA/NA Pathfinder

23. (d) We have, 2u = −3, 2v = −2, 2w = 2 30. (b) Equation of plane contains both lines
3 x −3 y − 2 z − 1
⇒ u = − , v = −1, w = 1
2 2 3 4 =0

∴Centre is ( −u, − v, − w ), i.e.  , 1,− 1
3
3 2 3
2 
Let coordinates of B be B ( x , y ,z ). Then, centre is the ( x − 3) ( 1) + ( y − 2)( 12 − 6) + (z − 1)( 4 − 9) = 0
mid-point of AB. x + 6 y − 5z = 10
−1 + x 3 4 + y −3 + z π
∴ = , = 1, = −1 Thus, the angle is .
2 2 2 2 2
⇒ x = 4 , y = −2 , z = 1 x − 1 y − 2 z −3
31. (d) The line = = =r
∴ B( 4, − 2, 1) is the required point. 3 −1 4
2 Any point say B ≡ (3r + 1, 2 − r , 3 + 4r ) (on the line L)
24. (d) Radius of given sphere =  3  + ( 1)2 + ( −1)2 + 15 = 1 77
  AB = 3r , − r , 4r + 6
 2 2
Hence, AB is parallel to x + y − z = 1
Radius of new sphere = 77
2 ⇒ 3r − r − 4r − 6 = 0 or r = − 3, B is ( −8, 5, − 9)
∴ Required sphere is  x −  + ( y − 1)2 + (z + 1)2 = ( 77 )2
3
 2 32. (b) The equation of plane containing the line L is
⇒ x + y + z 2 − 3x − 2 y + 2z −
2 2 291
=0 A( x − 1) + B( y − 2) + C (z + 3) = 0 ...(i)
4 where 3 A − B + 4C = 0 ...(ii)
⇒ 4x + 4 y + 4z −12x − 8 y + 8z − 291 = 0
2 2 2
Put B = ( − 8, 5, − 9) in Eq. (i)
Sol. (Q. Nos. 25-27) from where, we get 3 A − B + 2C = 0 ...(iii)
Z
On solving Eqs. (ii) and (iii), we get
C (0,0, 2)
M (2,0,2) C = 0 and 3 A = B
Now, put 3 A = B in Eq (i)
(0,2,2) L P (2,2,2) A( x − 1) + 3 A( y − 2) = 0
X A { x − 1 + 3( y − 2)} = 0
O A (2,0,0)
x − 1 + 3y − 6 = 0
x + 3 y − 7 = 0, is the required equation of the plane.
B (0,2,0) N (2,2,0) 33. (d) As the point lies on the line L, the point be
4
Y ( 1 + 3r , 2 − r , 3 + 4r ) which is at a distance of from the
3
25. (b) DR of OP are 2, 2, 2 or 1, 1, 1.
plane. Then, The distance of point ( 1 + 3r , 2 − r , 3 + 4r )
DR of AL are −2, 2 and 2 or −1, 1, 1.
|1 + 3r + 2 − r − 3 − 4r − 1|
− 1 + 1 + 1
Therefore, required angle = cos −1  −1  1
 = cos   1 + 1+ 1
 3⋅ 3  3 |2r + 1| 4 3 5
= = ⇒ r= ,−
26. (c) DR of OC are 0, 0, 2 or 0, 0, 1 3 3 2 2
−13 9 −14 

Hence, the points are A  , ,  and B 
DR of OP are 1, 1, 1. 11 1 10 
, , .
0 + 0 + 1  2 2 2  2 2 2 
∴ Required angle = cos −1  −1  1 
 = cos  
 3⋅ 1   3 ⇒ AB = 292

27. (c) DR of CP are 2, 3, 0 or 1, 1, 0. 34. (b) We have, A(3, 2, 0), B(5, 3, 2) and C ( −9, 6, − 3). D divides BC
DR of OP are 1, 1, 1. in the ratio AB : AC
1+ 1+ 0 −1  2  AB = (5 − 3)2 + (3 − 2)2 + ( 2 − 0)2 = 3
∴ Required angle = cos −1   = cos  
 3 2   3 AC = ( −9 − 3)2 + ( 6 − 2)2 + ( −3 − 0)2 = 13
28. (d) Since, both lines are coplanar.
∴D divides BC is the ratio 3 : 13.
2 3 λ ∴ Coordinates of D are
∴ 3 2 3 = 0 ⇒ 2( −2 + 3) + 3 (3 + 3) + λ(−3 − 2) = 0 ⇒ λ = 4  3 × ( −9) + 13 × 5 3 × 6 + 13 × 3 3 × ( −3) + 13 × 2 
 , , 
1 −1 1  3 + 13 3 + 13 3 + 13 
sin −1 (sin 4) = sin −1 {sin( π − 4)} = π − 4 
= ,
19 57 17 
, 
 8 16 16 
x −3 y − 2 z − 1
29. (d) Let = = =r
2 3 4 1 35. (c) Centroid of ∆PQR is coincide with centroid of ∆ABC .
3 + 5 − 9 2 + 3 + 6 0 + 2 − 3
⇒ x = 3 + 2r , y = 2 + 3r z = 1 + 4r
1 1, 1 ∴ Centroid of ∆PQR =  , , 
x − 2 y −3 z − 2  3 3 3 
It will lie on = = ⇒ r =1  −1 11 −1
3 2 3 1
= , , 
So, point of intersection is (5, 5, 5).  3 3 3
MATHEMATICS Three Dimensional Geometry 231

−2 −3 
2 2 2
∴ l 2 + m 2 + n2 =  
36. (d) We have, cos A = A (3,2,0) 1   4 
 +  + 
3  26   26   26 
2
−2
1 −  
5 9 1 16 26
∴ sin A = = 3 13 = + + = =1
 3  3 26 26 26 26
Area ( ∆ABC ) 41. (b) The direction ratios of Z-axis are 0, 0,1.
1 x −a y−b z−c
= AB × AC × sin A B (5,3,2) C (−9,6,−3) ∴ Required equation of line is = = .
2 0 0 1
1 5 13 5
= × 3 × 13 × = sq units 42. (a) The distance of point P ( x , y ,z ) from YZ- plane is x-coordinate
2 3 2 ∴ Required distance = 1 unit
−1 z z Z
37. (b) The direction ratios are 2, − 1, 2 or 1, ,1. 43. (a) In ∆AOP, sinα = =
2 OP r P (x, y, z)
38. (b) Given equation of two planes, ⇒ z = r sinα B
x x
Again, in ∆AOP, cosα = =
x + y + z + 1 = 0 and 2x − 2 y + 2z + 1 = 0 OP r r z
Here, a = 1, b = 1 and c = 1 ⇒ x = r cosα and the y-coordinate in
O α
1 1 1
XZ-plane is always zero.
a = 2, b = − 2 and c = 2 x A X
2 2 2 So, the coordinate of P in X Z-plane is
Let θ be the angle between them. ( r cos α , 0, r sin α ).
|a a + b b + c c | Y
Then, cos θ =
1 2 1 2 1 2 44. (c) Given, the distance between the points
a + b + c a + b 2 + c2
2 2 2 2
(7, 1, − 3) and (4, 5, λ) = 13
1 1 1 2 2 2
|( 1)( 2) + ( 1)( −2) + ( 1)( 2)| ⇒ ( 4 − 7)2 + (5 − 1)2 + ( λ + 3)2 = 13
⇒ cos θ =
( 1) + ( 1) + ( 1)
2 2 2
( 2 ) + ( −2 ) + ( 2 )
2 2 2
⇒ ( −3)2 + ( 4)2 + ( λ + 3)2 = 13
|2 − 2 + 2| | 2| 1
⇒ cos θ = = ⇒ cos θ = ⇒ 9 + 16 + ( λ + 3)2 = 13 ⇒ 25 + ( λ + 3)2 = 13
3 12 3(2 3) 3
On squaring equation both sides, we get
39. (d) The equation of sphere, 25 + ( λ + 3)2 = 169 ⇒ 25 + λ2 + 9 + 6λ − 169 = 0
x 2 + y 2 + z 2 − 4x + 6 y − 8z − 7 = 0 ⇒ λ2 + 6λ − 135 = 0 ⇒ λ2 + 15λ − 9λ − 135 = 0
On comparing with ⇒ λ( λ + 15) − 9( λ + 15) = 0 ⇒ ( λ + 15)( λ − 9) = 0
ax 2 + by 2 + cz 2 + 2ux + 2vy + 2wz + d = 0, we get ∴ λ = 9, − 15
u = − 2, v = 3, w = − 4 and d = − 7 45. (c) Given equations of the plane x = 3z + 4
∴ Radius of sphere = u 2 + v 2 + w 2 − d x −4 z−0
⇒ = …(i)
3 1
= ( −2 ) 2 + (3 ) 2 + ( −4 ) 2 + 7 y+3 z−0
and y = 2z − 3 ⇒ = …(ii)
2 1
= 4 + 9 + 16 + 7 = 36 = 6 Therefore, the required equation of the line intersecting by two
∴ Diameter = 2 × Radius = 2 × 6 = 12 units planes (i) and (ii),
x −4 y+3 z−0
40. (b) The equation of line passing through ( x , y , z ) and = = …(iii)
1 1 1 3 2 1
( x , y , z ) is Hence, the direction ratio’s of line (iii) is
2 2 2
x −x y− y z−z < a , b , c > = < 3, 2, 1 >
1 1 1 1 1 1
= = ...(i)
x −x y − y z −z 46. (c) The equation of any plane parallel to the plane
2 1 2 1 2 1
3x + 4 y − 5z = 0 may be taken as
Here, ( x − x ), ( y − y ) and (z − z ) direction ratios of that 3x + 4 y − 5z + k = 0 ...(i)
2 1 2 1 2 1
line. If plane (i) passes through the point (1, 2, 3), we get
Then, its direction cosines are 3( 1) + 4( 2) − 5(3) + k = 0
(x − x ) (y − y ) (z − z ) 3 + 8 − 15 + k = 0
l = 2 1 ,m = 2 1 and n = 2 1
Σ( x − x ) 2
Σ( x − x ) 2
Σ( x − x )2 −4+ k = 0 ⇒k = 4 ...(ii)
2 1 2 1 2 1 On putting k = 4 in Eq. (i), we get required equation
Here, ( x , y , z ) = ( 1, 2, − 3) and ( x , y , z ) = ( −2, 3, 1) i.e. 3x + 4 y − 5z + 4 = 0
1 1 1 2 2 2
( −2 − 1) (3 − 2 ) x −2 y+ 1 z+ 2
∴ l = , m= 47. (a) Given lines are = = ...(i)
1 −2 1
( −3)2 + ( 1)2 + ( 4)2 ( −3)2 + ( 1)2 + ( 4)2
( 1 + 3) x − 1 2 y + 3 z + 5 x − 1 2( y + 3 / 2) z + 5
and n= and = = , = =
( −3) + ( 1) + ( 4) 2 2 2 1 3 2 1 3 2
x −1 y +3/ 2 z+5
−3 1 4 = = ...(ii)
⇒ l = ,m = ,n= 1 3 2
26 26 26
2
232 NDA/NA Pathfinder

If θ is the acute angle between lines (i) and (ii), then 52. (b) The given equation represents the equation of sphere, if
coefficient of x 2 , y 2 and z 2 are same, i.e. k + 1 = 3 ⇒ k = 2
1 × 1 + ( −2)   + 1( 2)
3
 2 53. (a) We know that, if a line makes an ∠α with the positive direction
cos θ = of X -axis, ∠β with the positive direction of Y -axis and ∠ γ with
2
1 + ( −2)2 + 12 12 +   + 22
3 the positive direction of Z-axis, then
 2
cos 2 α + cos 2 β + cos 2 γ = 1
Given that, α = 30°
1 + ( −3 ) + 2 0 ∴ cos 2 30° + cos 2 β + cos 2 γ = 1
= = =0
9 29 2
1+ 4 + 1 1+ + 4 6  3 3
4 4 ⇒   + cos β + cos γ = 1 ⇒ cos β + cos γ = 1−
2 2 2 2

π  2  4
∴ cosθ = 0 ⇒ θ = cos − 1 ( 0) = 1
2 ∴ cos β + cos γ =
2 2
4
48. (a) If α , β and γ are the angles that a line makes with the
54. (a) Distance of the line 2x + y + 2z = 3 from the origin
coordinate axes.
|2 × 0 + 0 + 2 × 0 − 3| |0 + 0 + 0 − 3| |−3| 3
Then, l = cos α, m = cos β and n = cos γ = = = = = 1 unit
( 2)2 + ( 1)2 + ( 2)2 4 + 1+ 4 9 3
Q l 2 + m 2 + n2 = 1 ⇒ cos 2 α + cos 2 β + cos 2 γ = 1 ...(i)
Here, α = β = γ, as line is equally inclined to axes. 55. (c) We know that, the direction cosines of Z-axis are (0, 0,1).
1
From Eq. (i), cos 2 α + cos 2 α + cos 2 α = 1 ⇒ cosα = ∴ Sum of the direction cosines of Z-axis = 0 + 0 + 1 = 1
3
[since, direction cosines of a line which is equally inclined to 56. (d) Let a be length of the edge of the cube.
the positive directions of the axis, so we take only positive sign] Y
1 1
∴ cos α = cos β = cos γ = or l = m = n = C (0, a, 0)
3 3 B (a, a, 0)
1 1 1
Hence, the required direction cosines are , , . (a, a, a)
3 3 3 (0, a, a) D P
49. (d) Given equation of planes
2x − y − 2z + 1 = 0 ...(i) X
and 3x − 4 y + 5z − 3 = 0 ...(ii) O A (a, 0, 0)
Here, a = 2, b = −1 and c = −2
1 1 1
and a = 3, b = − 4, c = 5 are direction ratios of planes (i) and (ii) E (0, 0, a)
F (a, 0, a)
2 2 2 Z
respectively.
Let θ be the angle between two planes. Let direction ratios of the diagonal OP of the cube is
aa + bb + cc < a , b , c > = < a − 0, a − 0, a − 0 > = < a , a , a >
1 1 1
Then, cos θ = 1 2 1 2 1 2
and direction ratios of the diagonal AD of the cube is
a + b2 + c2 a2 + b2 + c2
2
1 1 1 2 2 2 < a , b , c > = < 0 − a , a − 0, a − 0 > = < − a , a , a >
2 2 2
( 2)(3) + ( −1)( −4) + ( −2)(5) 6 + 4 − 10 π Let θ be the acute angle between the diagonals OP and AD of
= = = 0 = cos
4 + 1+ 4 4 + 16 + 25 9 45 2 a cube.
π a a + b b + c c
So, the required angle is .
2 ∴ cosθ = 1 2 1 2 1 2

a + b 2 + c2 a 2 + b 2 + c2
2
1 1 1 2 2 2
50. (d) Given that, equation of straight line is
x −x y − y z −z ( a )( − a ) + ( a )( a ) + ( a )( a ) −a2 + a2 + a2
0 = 0 = 0 ...(i) = =
l m n a + a + a
2 2 2
a + a + a
2 2 2
3a 2 3a 2
and equation of plane is ax + by + cz + d = 0 ...(ii)
a2 1
Since, the straight line is parallel to the plane, i.e. normal to = = ⇒ 3 cosθ = 1
the plane is perpendicular to the straight line. 3a 2 3
By perpendicularity condition, 57. (b) We know that, the standard equation of the sphere, whose
l l + m m + n n = 0 ⇒ al + bm + cn = 0 centre at ( x , y , z ) and radius is r, is
1 2 1 2 1 2 1 1 1
51. (d) The given equation of planes ( x − x ) 2 + ( y − y ) 2 + (z − z ) 2 = r 2 ...(i)
1 1 1
x − 2 y + z − 1= 0 ...(i) But given that, centre = (0, 0, 0) and radius = 1
and −3x + 6 y − 3z + 2 = 0 ⇒ x − 2 y + z − 2 / 3 = 0 ...(ii) ∴ Required equation of sphere is
Since, both planes are parallel to each other, then distance ( x − 0)2 + ( y − 0)2 + (z − 0)2 = ( 1)2 ⇒ x2 + y 2 + z 2 = 1
between them
−2 −2 58. (c) We know that, the direction cosines of X-axis is 1, 0, 0 .
− ( −1) + 1
3 1/ 3 1 ∴ Sum of squares of direction cosine
= = 3 = =
( 1)2 + ( −2)2 + ( 1)2 1+ 4 + 1 6 3 6 = ( 1)2 + ( 0)2 + ( 0)2 = 1 + 0 + 0 = 1
MATHEMATICS Three Dimensional Geometry 233

59. (c) If a line passes through the points ( x , y , z ) and ( x , y , z ), 63. (a) Now, of DR’s AB = < a , b , c > = < − 2 − 2, 2 − 3, 0 − 1 >
1 1 1 2 2 2 1 1 1
then its direction ratio is < x − x , y − y , z − z >. = < − 4, − 1, − 1 > A
2 1 2 1 2 1
∴ The direction ratio of the line which passes through the and DR’s of BC = < a , b , c >
2 2 2
points ( 6, − 7, − 1) and ( 2, − 3, 1) is = < 0 + 2, 1 − 2, − 1 − 0 >
< 2 − 6, − 3 + 7, 1 + 1 > = < − 4, 4, 2 > = < 2, − 1, − 1 > θ
Let θ be the angle between AB and BC . B C
Sol. (Q. Nos. 60-62) Given that, a line passes through the point (1, − 2, 3)
and perpendicular to the plane 2x + 3 y − z = 7, i.e. this straight (a a + b b + c c )
1 2 1 2 1 2
line is a normal to the plane. ∴ cos θ =
a + b 2 + c2 a 2 + b 2 + c2
2
1 1 1 2 2 2
60. (a) Equation of the plane, 2x + 3 y − z = 7
 −4 × 2 + ( −1)( −1) + ( −1) × ( −1) 
On comparing with ax + by + cz = d , a = 2 , b = 3 and c = − 1  
=  
Which is the required direction ratios of normal to the plane.  16 + 1 + 1 4 + 1 + 1 
i.e. < a , b , c > = < 2, 3, − 1 >  − 8 + 1 + 1
=   = −6 = −1 = 1
61. (d) The point, where the line meet the plane = Foot ( x , y , z ) of a  18 6  6 3 3 3
point (1, − 2, 3) in a plane 2x + 3 y − z = 7  
1
x− x y− y z− z −( ax + by + cz + d ) ∴ cos θ = cos ∠ABC =
∴ 1 = 1 = 1 = 1 1 1 3
a b c a + b + c
2 2 2
64. (d) Now, we find
x − 1 y + 2 z − 3 −[ 2( 1) + 3( −2) − (3) − 7]
⇒ = = = y z 1 3 1 1
2 3 −1 ( 2)2 + (3)2 + ( −1)2 1 1 1
1
∆x = y z 1 = 2 0 1
x − 1 y + 2 z − 3 −( 2 − 6 − 3 − 7) − ( −14) 2 2 2
2
⇒ = = = = =1 y z 1 1 −1 1
3 3
2 3 −1 4+ 9+ 1 14
1
x −1 = {3 ( 0 + 1) − 1 ( 2 − 1) + 1 ( − 2 − 0)}
⇒ = 1⇒ x − 1= 2 ⇒ x =3 2
2 1 1 1
y+2 = (3 − 1 − 2 ) = (3 − 3 ) = × 0 = 0
⇒ = 1 ⇒ y + 2=3⇒ y = 1 2 2 2
3 z x 1 1 2 1
z −3 1 1 1
1
⇒ = 1⇒z − 3 = − 1⇒ z = 2 ∆y = z x 1 = 0 −2 1
−1 2 2 2
2
z x 1 −1 0 1
∴ Required point = ( x , y , z ) = (3, 1, 2) 3 3

1
62. (c) We know that, the image ( x , y , z ) of a point ( x , y , z ) in a = { 1 ( − 2 − 0) − 2 ( 0 + 1) + 1 ( 0 − 2)}
1 1 1 2
plane
x −x y−y z −z 1 1
= ( − 2 − 2 − 2) = × − ( 6) = − 3
ax + by + cz + d = 0 is given by 1 = 1 = 1
2 2
a b c
−2( ax + by + cz + d ) x y 1 2 3 1
= 1 1 1 1 1 1
1
and ∆z = x y 1 = −2 2 1
a 2 + b 2 + c2 2 2 2
2
x y 1 0 1 −1
∴ Image of the point ( 1, − 2 , 3) in the plane 2x + 3 y − z − 7 = 0 3 3

x − 1 y + 2 z −3 1
=
{ 2( −2 − 1) − 3( 2 − 0) + 1( −2 − 0)}
is, = =
2 3 −1 2
1 1
−2[ 2( 1) + 3( −2) + ( −1)(3) − 7] −2[ 2 + ( −6) + ( −3) − 7] = ( − 6 − 6 − 2) = × − 14 = − 7
= = 2 2
( 2)2 + (3)2 + ( −1)2 4+ 9+ 1 ∴ Required area of ∆ABC
−2( −14)
= =2 = ∆2x + ∆2y + ∆2z = ( 0)2 + ( −3)2 + ( −7)2
14
x −1 = 0 + 9 + 49 = 58
⇒ = 2 ⇒ x − 1= 4 ⇒ x =5
2 + 0 3 + 1 1 − 1
2 65. (c) Mid-point of AC =  , , 
y+2  2 2 2 
⇒ =2 ⇒ y + 2=6 ⇒ y=4
3 P = ( 1, 2, 0)
− 2 + 0 2 + 1 0 − 1

z −3
= 2 ⇒ z−3= − 2 ⇒ z=1 and mid-point of BC =  , , 
−1  2 2 2 
Hence, the required image is (5, 4, 1). Q = ( − 1, 3 / 2, − 1 / 2)
Now, magnitude of the line joining P and Q = |PQ|
Sol. (Q. Nos. 63-65) Given that vertices of a triangle are,
= ( 1 + 1)2 + ( 2 − 3 / 2)2 + ( 0 + 1 / 2)2
let ( x , y , z ) ≡ A ( 2, 3, 1), ( x , y , z ) ≡ B ( − 2, 2, 0),
1 1 1 2 2 2
and ( x y , z ) ≡ C ( 0, 1, − 1) 3
3 3 3 = ( 2)2 + ( 1 / 2)2 + ( 1 / 2)2 = units
2
234 NDA/NA Pathfinder

Sol. (Q. Nos. 66-67) Given equations of sphere, 70. (b) We know that, equation of plane passing through three
x 2 + y 2 + z2 − 4 y + 3 = 0 …(i) non-collinear points ( x , y , z ), ( x , y , z ) and ( x , y , z ) is
1 1 1 2 2 2 3 3 3
and x + y 2 + z 2 + 2x + 4z − 4 = 0
2
…(ii)  x −x y− y z−z 
Compare with the standard equation of sphere,  1 1 1

 x 2 − x1 y − y z − z = 0
x 2 + y 2 + z 2 + 2ux + 2vy + 2wz + d = 0, x − x
2 1 2 1
y − y z −z 
we get u = 0, v = − 2, w = 0 and d = 3 [for first sphere]  3 1 3 1 3 1
1 1 1 1
and u = 1, v = 0, w = 2 and d = − 4 [for second sphere] Putting the value of ( x , y , z ), ( x , y , z )
1 1 1 2 2 2
2 2 2 2
66. (c) Centre of Ist sphere, i.e. C → ( − u , − v , − w ) → ( 0, 2, 0) and ( x , y , z ), we get
1 1 1 1 3 3 3
and centre of IInd sphere, i.e. C → ( − u , − v , − w ) x − 2 y − 2 z − 1
2 2 2 2
→ ( − 1, 0, − 2)  
 1 2 1 = 0
Now, distance between two centres, i.e.  5 −2 5 
 
C C = ( −1 − 0 ) 2 + ( 0 − 2 ) 2 + ( −2 − 0 ) 2 = 1+ 4 + 4
1 2
⇒ ( x − 2) ( 10 + 2) − ( y − 2) (5 − 5) + (z − 1) ( −2 − 10) = 0
[Q d = ( x − x )2 + ( y − y )2 + (z − z )2 ] ⇒ 12x − 12z = 12 ⇒ x −z= 1
2 1 2 1 2 1

= 9 = 3 units Hence, the equation of plane passes through (1, 0, 1).

67. (c) I. Radius of Ist sphere, 71. (c) Direction ratios of the normal to the plane x − z = 1 are
〈1, 0, − 1〉.
r = u2 + v2 + w2 − d = ( 0 ) + ( 2 ) + ( 0 ) − (3 )
2 2 2
1 1 1 1 1
72. (b) Equation of sphere passing through origin is
= 4 −3 = 1 = 1
Radius of IInd sphere, x 2 + y 2 + z 2 + 2ux + 2vy + 2wz = 0
r = u 2 + v 2 + w 2 − d = ( −1)2 + ( 0)2 + ( −2)2 + ( 4) Which passes through the points,
2 2 2 2 2

= 1+ 4 + 4 = 9 = 3 ( 2, 1, − 1), ( 1, 5, − 4) and ( −2, 4, − 6).


Now, r + r = 1 + 3 = 4 ∴ 4u + 2v − 2w = − 6 ...(i)
1 2
Q C C <r + r 2u + 10v − 8w = − 42 ...(ii)
1 2 1 2
So, both sphere intersect each other. and −4u + 8v − 12w = − 56 ...(iii)
II. We have, radius of Ist sphere ( r ) = 1 On solving above equations, we get
1
and radius of IInd sphere ( r ) = 3 u = 1, v = − 2 and w = 3
2
i.e. r <r
1 2 ∴ Radius of sphere = u 2 + v 2 + w 2
So, the radius of first sphere is less than that of second sphere.
= 1+ 4 + 9 = 14
68. (d) Equation of line passing through the points (2, 1, 3) and
( 4, − 2, 5) is 73. (a) Centre of sphere
x −2 y − 1 z −3 x − 2 y − 1 z −3 ( − u , − v , − w ) = ( −1, 2, − 3)
= = =λ ⇒ = = =λ
4 − 2 −2 − 1 5 − 3 2 −3 2 74. (a) I. Equation of sphere is
⇒ x = 2λ + 2, y = − 3λ + 1 and z = 2λ + 3
x 2 + y 2 + z 2 + 2x − 4 y + 6z = 0
Since, this line cuts the plane, 2x + y − z = 3.
Put the value (0, 4, 0)
So, ( 2λ + 2, − 3λ + 1, 2λ + 3) satisfies the equation of plane.
we get 0 + 16 + 0 + 0 − 16 + 0 = 0
∴ 2( 2λ + 2) − 3λ + 1 − 2λ − 3 = 3 ⇒ λ = − 1
Hence, points are [ 2( −1) + 2, − 3( −1) + 1, 2( −1) + 3], So, the sphere passes through the point (0, 4, 0).
i.e. ( 0, 4, 1) II. Distance between (1, 1, 1) and centre of sphere ( −1, 2, − 3)
69. (d) Let the ratio in which plane divides the line be k : 1.
= ( 1 + 1)2 + ( 1 − 2)2 + ( 1 + 3)2
= 4 + 1 + 16 = 21 ≠ 5
Hence, only Statement II is not correct.
k 1
(2,1,3) (4,–2,5) 75. (d) Length of the line segment
(0,4,1)
= a 2 + b 2 + c2 = 122 + 42 + 32
= 13 units
4k + 2
76. (a) Direction cosines of the line segment =  ±
Then, 0 = ⇒ 4k + 2 = 0 ⇒ k = −
1 12 4 3
,± ,± 
k+1 2  13 13 13 
−2 k + 1 1
⇒4= ⇒ 4 k + 4 = −2 k + 1 ⇒ k = −  ±a ±b 
±c
k+1 2 Q 2 , , 
 a + b 2 + c2 a +b +c
2 2 2
a +b +c 
2 2 2
Hence, plane divides the line in ratio 1 : 2 externally.
MATHEMATICS Three Dimensional Geometry 235

x y − 2 z −3 x y
= =
z 87. (b) Given, 5x + 2 y + z − 13 = 0
77. (b) We have, = = =λ 83. (a) Here,
1 1 −1
2 3 4 x y z
or + + =1
⇒ x = 2 λ , y = 3 λ + 2 ,z = 4 λ + 3 2 3 13 13 13
x y z
and = = 5 2
∴ Direction ratios of the line passing 1 −1 1
− 13 13
through P 6 4 ∴ Lengths of intercepts are ,
1 1 5 2
P (3, − 1, 11) is [( 2λ − 3), DR’s of the lines are < , , − 1 >
2 3 and 13.
(3λ + 2 + 1), ( 4λ + 3 − 11)] 1 1
and < , − 1, − >, 88. (a) Given equations of plane are
= 2λ − 3, 3λ + 3, 4λ − 8 6 4
2x − y + 3z = 2 ...(i)
× + ( − 1) + ( − 1)  − 
1 1 1 1
∴ ( 2 λ − 3 ) ⋅ 2 + (3 λ + 3 ) ⋅ 3 i.e.
2 6 3  4 and x + y −z= 1 ...(ii)
+ ( 4λ − 8) ⋅ 4 = 0 1 1 1
= − + =0 Let the direction ratios of line of
⇒ 4λ − 6 + 9λ + 9 + 16λ − 32 = 0 12 3 4 intersection be < a , b , c > .
Hence, both lines are perpendicular. ∴ 2a − b + 3c = 0 ...(iii)
⇒ 29λ = 29 ⇒ λ = 1
84. (b) Given, 3x2 + 3 y 2 + 3z 2 − 8x a+ b−c=0
∴ Direction ratios and ...(iv)
≡ ( 2 − 3 , 3 + 3 , 4 − 8 ) ≡ ( −1 , 6 , − 4 ) + 4 y + 8z − 15 = 0 [Q a a + b b + c c = 0]
1 2 1 2 1 2
8 4 8
78. (c) Length of the line segment PQ, when ⇒ x + y + z − x + y + z −5 = 0
2 2 2
On solving Eqs. (iii) and (iv), we get
3 3 3 a −b c
Q ≡ ( 2, 5, 7) = =
Compare it with equation of a sphere −2 −5 3
∴ PQ = ( 2 − 3)2 + (5 + 1)2 + ( 7 − 11)2
x2 + y 2 + z 2 + 2ux + 2vy a b c a b c
⇒ = = or = =
= 1 + 36 + 16 = 53 units + 2wz + d = 0, we get −2 5 3 2 −5 −3
79. (a) We have,
x
+
y
+
z
=1 8 4 8
2u = − , 2v = , 2 w = , d = – 5 ⇒ < a , b , c > = < 2, − 5, − 3 >
A B C 3 3 3
89. (b) The equation of plane P is
Since, triangular plane ABC with 4 2 4
∴ u = – ,v = ,w = ,d = −5 ( 2x − y + 3z − 2)
centroid (1, 2, 3) cuts the coordinate 3 3 3
axes, + λ ( x + y − z − 1) = 0
Now, radius of a sphere
∴ Intercepts made by X-axis, Y-axis and It passes through (1, 0, 1), we get
Z-axis are, 3, 6, 9. = u 2 + v 2 + w2 − d ( 2 − 0 + 3 − 2)
x y z 16 4 16
80. (d) We have, + + = 1 = + + + 5 + λ ( 1 + 0 − 1 − 1) = 0
A B C 9 9 9
⇒ 3−λ=0 ⇒ λ =3
Since, triangular plane ABC with = 4+5 = 9 =3
centroid (1, 2, 3) cuts the coordinate Hence, equation of plane P is
axes, 85. (a) Let DR’s of the line be a, b, c. ( 2x − y + 3z − 2) + 3( x + y − z − 1) = 0
∴ Intercepts made by X-axis, Y-axis We have, a − 2b − 2c = 0
⇒ 5x + 2 y − 5 = 0
and Z-axis are 3, 6, 9. and 0 ⋅ a + 2b + c = 0
∴ Equation of the plane ABC is a −b c 90. (c) Plane P touches the sphere
x y z ∴ = =
+ + =1 −2 + 4 1 − 0 2 − 0 x 2 + y 2 + z2 = r 2
3 6 9 a b c
⇒ = = Here, the centre of sphere is (0, 0, 0).
or 6x + 3 y + 2z = 18 2 −1 2 ∴ Radius of sphere, r = Perpendicular
81. (b) Length of diagonals of the cuboid 86. (b) The foot of perpendicular drawn distance from centre of sphere to the
from (3, 5, 4) on z = 0 is tangent plane.
= 1 + 2 +3 =
2 2 2
14 units
x −3 y −5 z − 4 4  0 + 0 + 0 −5  −5 
82. (c) The equation of the plane passing = = =− r =  = 
0 0 1 1  5 + 2   29 
2 2
through P ( 1, 2, 3) and parallel to
XY-plane is z = 3. ⇒ x = 3, y = 5, z = 0 5
⇒ r= units
Hence, ( x, y , z ) = (3, 5, 0) 29
20
236 CDS Pathfinder

LIMITS, CONTINUITY AND


DIFFERENTIABILITY
In NDA exam, generally 6-8 questions are asked from this chapter which are based on
finding limits of various functions, check continuity/differentiability of functions in
different conditions.

LIMIT OF A FUNCTION
Let y = f ( x) be a function of x. If at x = a, f ( x) takes indeterminate form, then we consider the values of
the function which are very close to a. If these values tend to a definite unique number as x tends to a,
then the unique number so obtained is called the limit of f ( x) at x = a and we can write it as lim f ( x).
x→ a

Right Hand Limit


If x approaches from the right, i.e. from larger values of x than a, then the limit of f ( x) is called the
Right Hand Limit (RHL) and is written as,
lim f ( x) or lim f ( x) or f ( a + 0 )
x → a+ x → (a + 0 )

To find RHL, put x = a + h and replace x → a + by h → 0 and then simplify lim f ( a + h) using
h→ 0
appropriate formula.

Left Hand Limit


If x approaches a from the left, i.e. from the smaller values of x than a, then the limit of f ( x) is called
the Left Hand Limit (LHL) and is written as, lim f ( x) or f ( a − 0 ).
x → a−

To find LHL, put x = a − h and replace x → a by h → 0 and then simplify lim f ( a − h) using appropriate
h→ 0
formula. When the left hand limit equals to the right hand limit, we say that the function has a limiting
values. Thus, for the existence of lim f ( x), the necessary and sufficient condition is
x→ a

lim f ( x) = lim f ( x) or f ( a − h) = f ( a + h)
x → a− x → a+

Note Left hand limit and right hand limit are specially used, when the function is defined by more than one rule, e.g.
Modulus functions.
MATHEMATICS Limits, Continuity and Differentiability 237

Sol. d. We know that, − 1 ≤ sin x ≤ 1 for all x


IMPORTANT RESULTS RELATED TO LIMIT
(i) lim [ f ( x ) ± φ ( x )] = lim f ( x ) ± lim φ ( x )
⇒ − 7 ≤ 7 sin x ≤ 7 ⇒ x − 7 ≤ x + 7 sin x ≤ x + 7
x→ a x→ a x→ a
Now, dividing throughout by ( − 2x + 13), we get
(ii) lim [c ⋅ f ( x )] = c lim f ( x ), where c is a constant.
x→ a x→ a x −7 x + 7 sin x x+7
(iii) lim [ f ( x ) ⋅ φ ( x )] = lim f ( x ) ⋅ lim φ ( x ) ≥ ≥ for all x that are large.
x→ a x→ a x→ a − 2x + 3 − 2x + 13 − 2x + 13
lim f ( x ) 7
f (x) x → a x −7 1−
(iv) lim = , provided lim φ ( x ) ≠ 0
Now, lim = lim x = 1− 0 = − 1
x→ a φ (x) lim φ ( x ) x→ a
x → ∞ − 2x + 13 x→ ∞ 13 − 2 + 0
x→ a − 2+ 2
x
(v) lim log f ( x ) = log[ lim f ( x )] , provided lim f ( x ) > 0
x→ a x→ a x→ a 7
x+7 1+
{ lim f ( x )} and lim = lim x = 1+ 0 = − 1
(vi) lim e f (x )
=e x→a
x → ∞ − 2x + 13 x→ ∞ 13 − 2 + 0 2
x→ a − 2+
f (x ) x
lim
1 /φ (x ) x → a φ (x )
(vii) lim [1 + f ( x )] =e
x→ a Limit of a Rational Function
φ (x ) lim g ( x )
(viii) lim [ f ( x )] = { lim f ( x )}x → a p( x)
x→ a x→ a Limit of a rational function f ( x), of the form lim ,
x→ a q( x)
(ix) If lim f [ φ( x )] = f [ lim φ( x )], provided ‘ f ’
x→ a x→ a can be find out using factorisation or substitution
is continuous at φ( x) ∈ R methods.

2kx + 3, if x < 1 Indeterminate Form


EXAMPLE 1. If f (x) is defined as f (x) = 
1 − kx , if x > 1
2
If the expression obtained after substitution of value of
then for what values of k does lim exist? the limit give the following forms 0 0 , 0 / 0, 1∞ , ∞ − ∞,
x→1 ∞ / ∞, 0 × ∞ and ∞ 0 , then it is known as an indeterminate
a. 0 b. 1
form.
1 2
c. d. −
2 3 x4 −1 x3 −k 3
Sol. d. We have, lim− f ( x) = lim f (1 − h) = lim [ 2k (1 − h) + 3] EXAMPLE 3. If lim = lim , then find
x→1 h→ 0 h→ 0 x→1 x −1 x→ k x2 −k 2
= [ 2k (1 − 0) + 3] = 2k + 3 the value of k.
and lim+ f ( x) = lim f (1 + h) = lim[1 − k (1 + h) 2 ] 8
a. 0 b.
x→1 h→ 0 h→ 0 3
= 1 − k (1 + 0) 2 = 1 − k c. 0 and
8
d.
3
Now, lim f ( x) exists, if lim− f ( x) = lim+ f ( x) 3 8
x→1 x→1 x→1
x4 − 1 x3 − k 3
⇒ 2k + 3 = 1 − k ⇒ 3k = − 2 ⇒ k = −
2 Sol. b. We have, lim = lim 2
x→1 x − 1 x→ k x − k
2
3
( x2 + 1) ( x + 1) ( x − 1) ( x − k) ( x2 + xk + k 2)
Sandwich Theorem ⇒ lim = lim
x→1 x−1 x→ k ( x − k) ( x + k)
(or Squeeze theorem) x + xk + k 2
2
⇒ lim( x2 + 1) ( x + 1) = lim
If f , g and h are functions such that f ( x) ≤ g( x) ≤ h( x) for x→1 x→ k x+ k
all x in some neighbourhood of the point a (except k + k ⋅ k + k2
2
possibly at x = a) and if lim f ( x) = l = lim h( x), then ⇒ (12 + 1) (1 + 1) =
x→ a x→ a k+k
lim f ( x) = l.
x→ a 3k 2
x + 7 sin x ⇒ 4= ⇒ 3 k2 − 8 k = 0
EXAMPLE 2. The value of lim , using 2k
x→ ∞ − 2x + 13 8
⇒ k ( 3 k − 8) = 0 ⇒ k = 0,
sandwich theorem, is 3
1 1 1 Since, k = 0 does not satisfy the given equation, therefore
a. b. c. 0 d.−
3 2 2 k = 8 / 3.
238 NDA/NA Pathfinder

sin x
Exponential and Logarithmic Limits and RHL = lim +
x→ 0 x
=1
For finding the limits of exponential and logarithmic Q LHL ≠ RHL
functions, following results are useful
So, the limit does not exist.
ax −1 ex −1
1. lim = log e a, a > 0 2. lim =1
x→ 0 x x→ 0 x EXAMPLE 6. Evaluate lim (sec x − tan x).
x → π/2 1
1 n x a. 0 b. 1 c. −1 d.
3. lim (1 +  1  a
x) x = lim 1 +  = e 4. lim 1 +  = e a 2
x→ 0 h →∞  n x → ∞ x
Sol. a. lim (sec x − tan x ) = lim 
1 sin x 
− 
log x x → π /2 x → π /2  cos x cos x
5. lim m
= 0, ( m> 0 )
x→ 0 x 2
cos x − sin x
 
log a (1 + x)  1 − sin x  2 2
6. lim = log a e, (a > 0, a ≠ 1) = lim   = lim
x → π / 2  cos x  x → π /2 2 x x
x→ 0 x cos − sin2
2 2
log (1 + x)
7. lim =1 8. lim (1 + λx)1/x = e λ x
cos − sin
x π
cos − sin
π 1

1
x→ 0 x x→ 0
= lim 2 2 = 4 4 = 2 2 =0
x → π /2 x x π π 1 1
log(5 + x) − log(5 − x) cos + sin cos + sin +
EXAMPLE 4. The value of lim is 2 2 4 4 2 2
x→ 0 x
a.
5
b.
1
c.
2
d. 0 L’ HOSPITAL’S RULE
2 5 5
In this method, we first check, whether the form of the
log ( 5 + x) − log ( 5 − x) 0 form 0
Sol. c. We have, lim 0  function after substituting the limit is or not.
x→ 0 x 0
 x    x 
log 5 1+   − log 5 1−   If it is not of this form, then make necessary operation in
= lim   5    5  the function otherwise we differentiate both numerator
x→ 0 x and denominator with respect to x. Differentiation can be
  x    x  done n number of times according to the problem. The
log 5 + log 1+   − log 5 + log 1−  
= lim 
5   5  above rule can be applied for other indeterminate forms
x→ 0 x such as
log 1 +  − log 1 −  ∞
x x
 5   5 , ∞ − ∞, 0 × ∞, 1∞ , 0 0 and ∞ 0 etc.
= lim ∞
x→ 0 x
log 1+  log 1−  sin −1 x − tan −1 x
x x
 4   5 1 1 1 2 EXAMPLE 7. The value of lim is
= lim + lim ⋅ = + = x→ 0 x3
x→ 0 x/5 x→ 0 − x / 5 ( −5) 5 5 5
1
a. 0 b.
Trigonometric Limits 2
1
sin x c. d. 3
(i) lim =1 (ii) lim sin x = 0 3
x→ 0 x x→ 0
tan x sin−7 x − tan−1 x
(iii) lim cos x = 1 (iv) lim =1 Sol. b. We have, lim
x→ 0 x3
x→ 0 x→ 0 x
sin −1 x tan −1 x (1+ x 2) − 1+ x 2 0
(v) lim =1 (vi) lim =1 = lim [use L’Hospital’s Rule, as
x→ 0 x x→ 0 x x→ 0 3x 2
1− x (1+ x )
2 2 0
form]
sin | x | (1+ x 2) 2 − (1− x 2) 1
EXAMPLE 5. The value of lim is = lim × [rationalise]
x→ 0 x x→ 0 3x ( 1− x )(1+ x )
2 2 2
(1+ x )( 1− x 2 )
2

a. 1 b. −1 c. ∞ d. Does not exist x 4 + 3x 2 1


= lim ×
sin | x | sin ( − x) x→ 0 3x 2
1− x 2 (1+ x 2) (1+ x 2) + ( 1− x 2 )
Sol. d. Q lim , LHL = lim−
x→ 0 x x→ 0 x x2 + 3 1 1
sin x = lim × =
= − lim = −1 x→ 0 2
x→ 0 x
3( 1 − x )(1 + x ) (1 + x ) + ( 1 − x )
2 2 2 2
MATHEMATICS Limits, Continuity and Differentiability 239

Properties of Continuous Functions


SOME IMPORTANT FORMULAE
If f ( x) and g ( x) are continuous functions at x = a, then
x2 x3
(i) e = 1 + x +
x
+ + ... ∞ (i) f ( x) ± g ( x) are also continuous at x = a.
2! 3!
x2 x3
(ii) f ( x) ⋅ g ( x) is also continuous at x = a.
(ii) e − x = 1 − x + − + ... ∞ f ( x)
2! 3! (iii) is also continuous at x = a, if g ( a) ≠ 0.
x 2 (loge a ) 2 x 3 (loge a ) 3 g ( x)
(iii) a x = 1 + x loge a + + + ... ∞
2! 3! (iv) k f ( x) is also continuous, where k is a constant.
x2 x3
(iv) log(1 + x ) = x − + − ... ∞ ,| x | < 1 (v) 1/ f ( x) is also continuous at x = a, if f ( a) ≠ 0
2 3
x3 x5 (vi) | f ( x) | is also continuous.
(v) sin x = x − + − ... ∞
3! 5!
x2 x4
Discontinuity of a Function
(vi) cos x = 1 − + − ... ∞
2! 4 ! A function f ( x), which is not continuous at a point
x 3 2x 5 x = a, is said to be discontinuous at that point.
(vii) tan x = x + + + ... ∞
3 15
sin 2x
(viii) sin −1 x = x + 12 ⋅
x3 x5  , when x ≠ 0
+ 12 ⋅ 32 ⋅ + ... EXAMPLE 8. If f (x) =  sin 3x , then at
3! 5!
 2, when x = 0
x3
(ix) cos −1 x = x − + ...
6 x = 0, function is
x3 x5 a. continuous
(x) tan −1 x = x − + −...
3 5 b. not continuous, since lim f ( x) is not defined
x→0
c. not continuous, since lim f ( x) ≠ f ( 0)
x→ 0
d. None of the above
CONTINUITY
Sol. c. We have, f (0) = 2, [given]
A function f ( x) is defined for x = a is said to be
continuous at x = a, if Also, lim f ( x) = lim
sin 2x
= lim 
sin 2x
×
3x 2
⋅ 
1. f ( a), i.e. the value of f ( x) at x = a is a definite
x→ 0 x→ 0sin 3x x → 0  2x sin 3x 3
number and 2 sin 2x 1 2 1 2
= lim × = × 1× =
3 x → 0 2x sin 3x 3 1 3
2. The limit of the function f ( x) as x → a exists and is lim
x → 0 3x
equal to the value of f ( x) at x = a.
Q lim f ( x) ≠ f (0)
or f ( x) is continuous at x = a, x→ 0

if lim f ( x) = lim f ( x) = f ( a) Hence, f ( x) is not continuous at x = 0.


x → a+ x → a−
EXAMPLE 9. A function f is defined as follows
or f ( a + 0 ) = f ( a − 0 ) = f ( a) 1
f (x) = x p cos   , x ≠ 0, f (0) = 0. What conditions should
If f ( x) is not continuous at x = a, it is said to be x
discontinuous and the point a is called a point of be imposed on p, so that f may be continuous at x = 0?
discontinuity. a. p = 0 b. p > 0
c. p < 0 d. No value of p
Continuity of a Function in Interval
(i) A function f ( x) is said to be continuous over an  p  1
x cos   , x ≠ 0
, lim f ( x) = lim x p cos  
1
Sol. b. Q f ( x) =   x
open interval ] a, b [ if it is continuous at every x→ 0 x→ 0  x
 0, x=0
point over the interval (a, b).
Since, f ( x) is continuous at x = 0.
(ii) A function f ( x) is said to be continuous over closed
lim x p cos   = 0
1
interval [a, b], if it is continuous over the open ∴
x→ 0  x
interval ] a, b [ and is continuous at the point a from
which is possible only, if p > 0.
the right and continuous at point b from the left.
240 NDA/NA Pathfinder

DIFFERENTIABILITY Relation between Differentiability


Let k denotes the open interval ( a, b) in R and c ∈ k . Then, and Continuity
a function f : k → R is said to be differentiable at c, if and Generally, a function which is differentiable at a point is
 f ( c + h) − f ( c ) necessarily continuous at that point, i.e. differentiability
only if lim 
h→ 0  h  exists and is denoted by f ′ ( c ). at a point ⇒ continuity at that point, but converse is
Let y = f ( x) not necessary true.
Then, the value f ′ ( x 0+ ) EXAMPLE 10. The function f (x) = (x − 2)1/3 is
f ( x 0 + δx ) − f ( x 0 )
= lim , δx > 0 …(i) a. differentiable at x = 2
δx → 0 δx
b. not differentiable at x = 2
is called the right hand derivative of f ( x) at x 0 and
the value c. Cannot be determined
f ( x 0 − δx ) − f ( x 0 ) d. None of the above
f ′ ( x 0− ) = lim , δx > 0 …(ii)
δx → 0 − δx Sol. b. Q f ( x) = ( x − 2)1/ 3
is called the left hand derivative of f ( x) at x 0 . Now, f ( x + 2) = ( x + 2 − 2)1/ 3 = ( x)1/ 3
∴ f ( 2) = ( x − 2)1/ 3
SOME IMPORTANT RESULTS = ( 2 − 2)1/ 3 = 0
(i) Trigonometric functions, inverse trigonometric functions, f ( 2 + h) − f ( 2)
logarithmic functions, exponential functions and modulus We have, f ′ ( 2) = lim
h→ 0 h
functions are continuous in their domain.
h1/ 3 − 0
(ii) Every polynomial is continuous at every point of the real = lim
line. h→ 0 h
(iii) The composition of differentiable functions is a 1
= lim 2 / 3
differentiable function. h→ 0 h

(iv) If a function is not differentiable but it is continuous at a which is not defined. Hence, the function is not
point, it geometrically implies there is a sharp corner or a differentiable at x = 2.
kink at that point.

PRACTICE EXERCISE
( a + 2x ) − ( 3x ) 1 − cos 3x
1. lim , a ≠ 0 is equal to 5. lim is equal to
x→a ( 3a + x ) − 2 x x→0 x sin x
1 1 2 1 3 9 9
(a) (b) (c) (d) 0 (a) (b) (c) (d)
3 3 3 3 3 2 2 2 4
2x − π
x − 3x + 2
3
6. lim is equal to
2. lim is equal to x → π/ 2 cos x
x→∞ 2x3 + x − 3
(a) 1 (b) 2 (c) −2 (d) 0
1
(a) 2 (b) (c) 0 (d) 1 2 − cos θ − sin θ
2 7. lim is equal to
2x − 1 θ → π/ 4 ( 4θ − π )2
3. lim is equal to
x → 0 (1 + x )1/ 2 − 1 (a)
1
(b)
1
(c)
1
(d)
1
1 16 16 2 8 8 2
(a) log 2 (b) 2 log 2 (c) log 2 (d) 0
2  x + sin x 
( x − 3) 8. lim   is equal to
4. lim is equal to x→∞  x − cos x 
x → 3| x − 3|
(a) 0 (b) 1 (c) −1 (d) ±1
(a) 0 (b) 1 (c) −1 (d) does not exist
MATHEMATICS Limits, Continuity and Differentiability 241

sin ( x + a ) + sin ( a − x ) − 2 sin a 


9. lim   is equal to
19. The set of all points of differentiability of the
x→0 x sin x   x 2 sin (1/ x ), x ≠ 0
1 function f( x ) =  is
(a) sin a (b) cos a (c) −sin a (d) cos a x=0
2  0,
(a) (−∞, 0) (b) (−∞, ∞) ~ {0} (c) (0, ∞) (d) (−∞, ∞)
 sin [x ]
, [x ] ≠ 0
10. If f ( x ) =  [x ] , where [x ] denotes the 20. Which one of the following is correct in respect of
 0, [x ] = 0 the function f( x ) =| x| + x 2?
greatest integer less than or equal to x, then (a) f (x) is not continuous at x = 0
lim f ( x ) equals to (b) f (x) is differentiable at x = 0
x→0 (c) f (x) is continuous but not differentiable at x = 0
(a) 1 (b) 0 (c) −1 (d) does not exist
(d) None of the above
a −x
x a
11. If lim = − 1, then 21. lim e−1/ x is equal to
x→a xx − aa x→0

(a) a = 1 (b) a = 0 (c) a = e (d) 2 (a) 0 (b) ∞ (c) e (d) does not exist

1 − cot3 x 22. What is the set of all points, where the function
12. lim is equal to x
x → π/ 4 2 − cot x − cot x 3 f( x ) = is differentiable?
1 +| x|
3 11 1 2
(a) (b) (c) (d) (a) (− ∞, ∞) only (b) (0, ∞) only
4 4 2 5
(c) (− ∞, 0) ∪ (0,∞) only (d) (−∞, 0) only
log (1 + ax ) − log (1 − bx )
13. The function is not defined  3x − 4, 0 ≤ x ≤ 2
x 23. Let f ( x ) =  .
at x = 0. The value which should be assigned to f at  2x + l , 2 < x ≤ 9
x = 0, so that it is continuous at x = 0 is If f is continuous at x = 2, then what is the value
(a) a − b (b) 1 + b of l?
(c) log a + log b (d) None of these (a) 0 (b) 2 (c) −2 (d) −1
sin ( ex − 2 − 1) 24. If f : R → R be defined as f( x ) = sin (| x|), then
14. If f ( x ) = , then lim f ( x ) is given by
log ( x − 1) x→2 which one of the following is correct?
(a) −2 (b) −1 (c) 0 (d) 1 (a) f is not differentiable only at 0
(b) f is differentiable only at 0
15. What is lim ( a 2x 2 + ax + 1 − a 2x 2 + 1 ) equal to? (c) f is differentiable everywhere except at x = 0
x→∞
1 (d) f is non-differentiable at many points
(a) (b) 1 (c) 2 (d) 0
2 25. Consider the following statements
16. What is the value of k for which the following I. f (x) = | x − 3| is continuous at x = 0.
function f( x ) is continuous for all x? II. f (x) = | x − 3| is differentiable at x = 0.
 x3 − 3x + 2 Which of the above statement(s) is/are correct?
 , for x ≠ 1
f ( x ) =  ( x − 1)2 (a) Only I (b) Only II
 , for x = 1
(c) Both I and II (d) Neither I nor II
 k
(a) 3 (b) 2 (c) 1 (d) −1 26. Consider the following statement
x +4
I. f (x) = |x| is continuous ∀ x ∈ R
 x + 6 II. f (x) = x3 + x2 − 1 is not continuous ∀ x ∈ R
17. What is the value of lim   ?
x → ∞  x + 1
Which of the above statement(s) is/are correct?
(a) e (b) e 2 (c) e 4 (d) e 5 (a) Only I (b) Only II
(c) Both I and II (d) Neither I nor II
 x 2/ a , 0≤ x < 1
 27. Consider the following statements
18. The function f ( x ) =  a , 1 ≤ x < 2 is
( 2 b2 − 4 b)/ x 2 , 2 ≤ x < ∞ tan−1 x sin−1 x
 I. lim −
x→0
continuous for 0 ≤ x < ∞ , then the most suitable
x x
values of a and b are sin−1 x tan−1 x
(a) a = 1, b = − 1 (b) a = − 1, b = 1 + 2 II. lim −
x→0 x x
(c) a = − 1, b = 1 (d) None of these
where,|x|< 1, then
242 NDA/NA Pathfinder

Which of the above statement(s) is/are correct? Directions (Q. Nos. 34-37) If lim f ( x ) = 1 and
x→a lim [ f ( x ) − 1] g ( x )
(a) Only I (b) Only II lim g ( x ) = ∞, then lim [ f ( x )] g ( x ) = e x → a ,
(c) Both I and II (d) Neither I nor II x→a x→a
then
28. Consider the following statements 2
x2  x2  34. lim (cos x )cot x
equals
I. lim exists. II.   is not continuous at x = 0. x→0
x→ 0 x  x (a) e (b) e 2 (c) e −1/ 2 (d) 1
| x| 1/ 4
 π 
35. lim  tan + x 
III. lim does not exist.
x→ 0 x equals
x→0   4 
Which of the above statement(s) is/are correct? (a) 0 (b) 1 (c) e (d) e 2
(a) I, II and III (b) I and II (c) II and III (d) I and III 1/ x
x − 1 + cos x 
36. lim   equals
Directions (Q. Nos. 29-30) Given the function, x→0  x 
 π (a) e1/ 2 (b) e −1/ 2 (c) e (d) e −1
 x + a 2 sin x, 0≤ x<
4  a x + bx + cx 
2/ x
 π π 37. lim   equals
f (x) =  2 x cot x + b, ≤ x < is continuous x→0 3 
 4 2
a cos 2 x − b sin x, π ≤ x ≤ π
(a) a2 / 3 + b 2 / 3 + c 2 / 3 (b) abc
 2 (c) (abc )2 / 3 (d) 1
in [0, π] , then
29. Find the value of a. PREVIOUS YEARS’ QUESTIONS
π π π π
(a) (b) − (c) (d) −
6 6 12 12 1+ x −1
38. What is the value of lim ?
x→0 x e 2012 II
30. Find the value of b.
π π π π (a) 0 (b) 1/2 (c) 1 (d) −1/2
(a) (b) − (c) (d) − x−2
6 6 12 12 39. What is the value of lim ?
x→2 x2 − 4 e 2012 II
Directions (Q. Nos. 31-33) If f , g , and h are function (a) 0 (b) 1/4 (c) 1/2 (d) 1
having a common domain D and h( x ) ≤ f ( x ) ≤ g ( x ), x+ 2
x ∈D and if lim h( x ) = lim g ( x ) = l, then 40. What is the value of lim  3 ?
x → −2  x + 8
lim f ( x ) = l. x → a x→a e 2012 II
x→a
(a) 1/4 (b) −1/4 (c) 1/12 (d) −1/12
This is known as sandwich theorem. Using this result,
41. What is the value of lim x 2 sin   ?
complete the following limits. 1
|x| x→ 0  x e 2012 II
31. The value of lim is (a) 0 (b) 1 (c) 1/2 (d) −1
x→0
x + 4x 2 + t
4
2 (1 − cos x )
(a) 1 (b) 0 (c)
1
(d) does not exist
42. What is the value of lim ?
2
x→0 x2 e 2012 II
(a) 0 (b) 1/2 (c) 1/4 (d) 1
 e1/ x − e−1/ x 
32. Let f ( x ) = x 2   , x ≠ 0 and f( 0) = 1, then
−1/ x 
43. Which one of the following is correct in respect of
e + e
1/ x
 x2
the function f ( x ) = for x ≠ 0 and f (0) = 0?
(a) lim f(x ) doesn’t exist |x| e 2012 II
x→ 0+
(a) f(x) is discontinuous everywhere
(b) lim f(x ) doesn’t exist
x→ 0 (b) f(x) is continuous everywhere
(c) lim f(x ) exists (c) f(x) is continuous at x = 0 only
x→ 0
(d) f(x) is discontinuous at x = 0 only
(d) f(x ) is a continuous function
44. Consider the following statements
33. Let f ( x ) = x5  3  , x ≠ 0 and f( 0) = 0, where [ ⋅ ] is
1 1
I. lim exists. II. lim e1/ x does not exist.
x  x→ 0 x x→ 0
greatest integer function, then Which of the above statement(s) is/are correct?
(a) lim f(x ) doesn’t exists (b) f is not continuous at x = 0 e 2012 II
x→ 0
(a) Only I (b) Only II
(c) lim f(x ) 1 (d) lim f(x ) = 0
x→ 0 x→ 0 (c) Both I and II (d) Neither I nor II
MATHEMATICS Limits, Continuity and Differentiability 243

45. Which one of the following functions is 54. Consider the following statements
differentiable for all real values of x? e 2012 II I. The function f (x ) = 3 x is continuous at all x
x 1 1 except at x = 0.
(a) (b) x| x| (c) (d)
| x| | x| x
II. The function f (x ) = [x ] is continuous at x = 2.99,
46. Consider the following in respect of the function where [ ] is the bracket function.
f( x ) =|x − 3| Which of the above statement(s) is/are correct?
I. f(x) is continuous at x = 3. e 2014 I
II. f(x) is differentiable at x = 0. (a) Only I (b) Only II
(c) Both I and II (d) Neither I nor II
Which of the above statement(s) is/are correct?
e 2012 II 55. Consider the following statements
(a) Only I (b) Only II I. The function f (x ) = | x | is not differentiable at
(c) Both I and II (d) Neither I nor II x = 1.
II. The function f (x ) = ex is differentiable at x = 0.
1− 1+ x
47. What is the value of lim ? Which of the above statement(s) is/are correct?
x→ 0 x e 2013 I (a) Only I (b) Only II e 2014 I
(a) 1/2 (b) −1 / 2 (c) 1 (d) −1 (c) Both I and II (d) Neither I nor II
sin x − tan x
48. Which is lim equal to? Directions (Q. Nos. 56-58) Let f ( x) be a function
x→ 0 x e 2013 I
defined in 1≤ x < ∞ by
(a) 0 (b) 1 (c) −1 (d) 1/2
2 − x, for 1≤ x ≤ 2
f ( x) = 
49. The derivative of|x| at x = 0 is e 2013 I 3 x − x 2
, for x > 2 e 2014 I
(a) 1 (b) −1 (c) 0 (d) does not exist
56. Consider the following statements
50. Consider the following statements in respect of a I. The function is continuous at every point in the
function f( x ) interval [1, ∞ ).
I. f (x) is continuous at x = a , iff lim f (x) exists. II. The function is differentiable at x = 1.5.
x→ a
1 Which of the above statement(s) is/are correct?
II. If f (x) is continuous at a point, then is also
f (x) (a) Only I (b) Only II
continuous at the point. (c) Both I and II (d) Neither I nor II
Which of the above statement(s) is/are correct?
57. What is the differentiable coefficient of f( x ) at
e 2013 I
(a) Only I (b) Only II x = 3?
(c) Both I and II (d) Neither I nor II (a) 1 (b) 2 (c) −1 (d) −3

51. Consider the following statements 58. Consider the following statements
1 1 I. f′ (2 + 0) does not exist.
I. lim sin does not exist. II. lim x sin exists.
x→ 0 x x→ 0 x II. f′ (2 − 0) does not exist.
Which of the above statement(s) is/are correct? Which of the above statement(s) is/are correct?
e 2013 I (a) Only I (b) Only II
(a) Only I (b) Only II (c) Both I and II (d) Neither I nor II
(c) Both I and II (d) Neither I nor II
(1 + x )n − 1
 x2 , x > 2 59. What is lim equal to?
x→0
52. Consider the function f( x ) =  x e 2014 I
 3x − 2 , x ≤ 2 (a) 0 (b) 1 (c) n (d) n − 1
Which one of the following statements is correct x
60. What is lim equal to?
in respect of the above function? e 2013 I x→0 1 − cos x e 2014 I
(a) f(x) is derivable but not continuous at x = 2 1
(a) 2 (b) − 2 (c) (d) does not exist
(b) f(x) is continuous but not derivable at x = 2 2
(c) f(x) is neither continuous nor derivable at x = 2
(d) f(x) is continuous as well as derivable at x = 2 Directions (Q. Nos. 61-62) Consider the function
1 − sin x π  π
f (x ) = and f   = λ .
, where x ≠
53. If f( 9) = 9 and f ′ ( 9) = 4, then what is (π − 2 x ) 2
2 2 e 2014 I
f(x ) − 3
lim equal to? 61. What is lim f ( x ) equal to?
π
x→9 x −3 e 2014 I x→
2
(a) 36 (b) 9 (c) 4 (d) None of these
(a) 1 (b) 1/2 (c) 1/4 (d) 1/8
244 NDA/NA Pathfinder

62. What is the value of λ, if the function is Which of the above statement(s) is/are correct?
π e 2015 I
continuous at x = ? (a) Only I (b) Only II
2
(a) 1/8 (b) 1/4 (c) 1/2 (d) 1 (c) Both I and II (d) Neither I nor II

Directions (Q. Nos. 63-64) Read the following Directions (Q. Nos. 72-73) Given a function
information carefully and answer the questions  −1, if x ≤ 0

given below. f ( x ) =  ax + b , if 0 < x < 1, where a, b are constants.
 x 2 − 5, x ≤ 3  1,
Consider the function f ( x) =   if x ≥1
 x + 13, x > 3 e 2014 I The function is continuous everywhere. e 2015 I
63. What is lim f ( x ) equal to? 72. What is the value of b?
x→3
(a) −1 (b) 1 (c) 0 (d) 2
(a) 2 (b) 4 (c) 5 (d) 13
64. Consider the following statements 73. What is the value of a?
I. The function is discontinuous at x = 3. (a) −1 (b) 0 (c) 1 (d) 2
II. The function is not differentiable at x = 0.
Directions (Q. Nos. 74-75) Given that,
Which of the above statement(s) is/are correct?
(a) Only I (b) Only II 2 + x 2 
lim  − Ax − B = 3.
(c) Both I and II (d) Neither I nor II x → ∞ 1 + x  e 2015 I
 tan kx
, x< 0
65. Consider the function f ( x ) =  x
74. What is the value of A?
(a) −1 (b) 1 (c) 2 (d) 3
 3x + 2k2 , x ≥ 0
What is the non-zero value of k for which the 75. What is the value of B ?
function is continuous at x = 0? e 2014 II
(a) −1 (b) 3 (c) −4 (d) −3
(a) 1/4 (b) 1/2 (c) 1 (d) 2
Directions (Q. Nos. 76-77) Consider the function
5x − 1
66. What is lim equal to?  α cos x π
x→0 x e 2014 II  , if x ≠ 2
(a) loge 5 (b) log 5 e (c) 5 (d) 1 f (x ) =  π − 2 x which is continuous at
π
1 + 2 + 3 + K+ n  3, if x =
67. What is lim equal to?  2
n→∞ 12 + 22 + 32 + K + n 2 e 2014 II π
(a) 5 (b) 2 (c) 1 (d) 0
x = , where α is a constant.
2 e 2015 I
log5 (1 + x )
68. What is lim equal to? 76. What is the value of α?
x→0 x e 2014 II (a) 6 (b) 3 (c) 2 (d) 1
(a) 1 (b) log 5 e (c) log e 5 (d) 5
77. What is lim f ( x ) equal to?
69. Consider the following statements x→0
I. The function f (x) = [x], where [ ] is the greatest (a) 0 (b) 3 (c)
3
(d)
6
integer function defined on R, is continuous at π π
all points except at x = 0. f ( x ) − f (1)
II. The function f (x) = sin| x| is continuous for all 78. If f ( x ) = 25 − x 2 , then what is lim
x →1 x−1
x ∈ R.
equal to? e 2015 II
Which of the above statement(s) is/are correct? 1 1 1
(a) Only I (b) only II e 2014 II
(a) (b) (c) 24 (d) −
5 24 24
(c) Both I and II (d) Neither I nor II
G( x ) − G(1) sin ( ex − 2 − 1)
70. If G( x ) = 25 − x 2 , then what is lim 79. If f ( x ) = , then lim f ( x ) is equal to
x→1 x−1 ln ( x − 1) x→2 e 2015 II
equal to? e 2015 I (a) − 2 (b) − 1 (c) 0 (d) 1
1 1 1 1
(a) − (b) (c) − (d) 1 − sin x + cos x
2 6 5 6 6 80. The function f ( x ) = is not defined
1 + sin x + cos x
71. Consider the following statements at x = π. The value of f( π ), so that f ( x ) is
I. f (x) = [x], where [⋅] is the greatest integer continuous at x = π, is e 2015 II
function, is discontinuous at x = n, where n ∈ Z . 1 1
(a) – (b) (c) –1 (d) 1
II. f (x) = cot x is discontinuous at x = nπ, where 2 2
n ∈ Z.
MATHEMATICS Limits, Continuity and Differentiability 245

81. Consider the following functions Directions (Q. Nos. 87-88) Consider the function
1 2x + 5, if x > 0 a[ x ] + x − 1
 , if x ≠ 0 f (x) =
I. f (x) =  x II. f (x) =  2
0 , if x = 0  x + 2x + 5, if x ≤ 0 [x] + x
Which of the above function(s) is/are derivable at where [ ⋅ ] denotes the greatest integer function.
x = 0? e 2015 II
e 2016 I
(a) Only I (b) Only II
(c) Both I and II (d) Neither I nor II 87. What is lim+ f ( x ) equal to?
x→0
82. Consider the function (a) 1 (b) ln a
 ax − 2, for − 2< x < − 1 (c) 1 − a−1 (d) Limit does not exist

f(x) =  − 1, for − 1≤ x ≤ 1
 a + 2 ( x − 1)2 , for 1< x < 2
88. What is lim− f ( x ) equal to?
 x→0
What is the value of a for which f ( x ) is continuous (a) 0 (b) In a
at x = − 1 and x = 1? e 2015 II (c) 1 − a−1 (d) Limit does not exist
(a) − 1 (b) 1 (c) 0 (d) 2
Directions (Q. Nos. 89-90) A function f ( x) is
Directions (Q. Nos. 83-84) Consider the function defined as follows
 π
 − 2 sin x , if x≤−
2

 π π  for x ∈ [− π, 0)
f ( x ) =  A sin x + B, if − < x < , which is  x + π,
2 2   π
 f (x ) =  π cos x , for x ∈ 0, 
 cos x , π  2
if x≥  π
2
 2   x −  , for x ∈  π , π
  2 
continuous everywhere. e 2015 II  2  e 2016 I
83. The value of A is 89. Consider the following statements
(a) 1 (b) 0 (c) − 1 (d) − 2
I. The function f (x) is continuous at x = 0.
84. The value of B is II. The function f (x) is continuous at x = π / 2.
(a) 1 (b) 0 (c) − 1 (d) − 2 Which of the above statement(s) is/are correct?
85. If lim φ ( x ) = a , where a ≠ 0, then what is
2
(a) Only I (b) Only II
x→0
(c) Both I and II (d) Neither I nor II
 x
lim φ   equal to? 90. Consider the following statements
x→0  a e 2016 I
(a) a 2
(b) a −2
(c) −a 2
(d) −a I. The function f (x) is differentiable at x = 0.
1 II. The function f (x) is differentiable at x = π / 2.

86. What is lim e x2 equal to? Which of the above statement(s) is/are correct?
x→0 e 2016 I (a) Only I (b) Only II
(a) 0 (b) 1
(c) Both I and II (d) Neither I nor II
(c) −1 (d) Limit does not exist

ANSWERS
1 c 2 b 3 b 4 d 5 c 6 c 7 b 8 b 9 c 10 d
11 a 12 a 13 d 14 d 15 a 16 a 17 d 18 c 19 d 20 c
21 d 22 a 23 c 24 c 25 c 26 a 27 b 28 a 29 a 30 d
31 b 32 c 33 d 34 c 35 d 36 b 37 c 38 b 39 b 40 c
41 a 42 d 43 b 44 b 45 b 46 c 47 b 48 a 49 d 50 d
51 c 52 b 53 c 54 b 55 b 56 b 57 d 58 d 59 c 60 d
61 d 62 a 63 b 64 d 65 b 66 a 67 d 68 b 69 b 70 a
71 c 72 a 73 d 74 b 75 c 76 a 77 d 78 d 79 d 80 c
81 b 82 a 83 c 84 a 85 a 86 a 87 b 88 c 89 c 90 d
246 NDA/NA Pathfinder

HINTS AND SOLUTIONS


( a + 2x )2 − ( 3x )2 π π
1. (c) lim where, y = θ − → 0 as θ → 15. (a) lim ( a 2 x 2 + ax + 1 − a 2 x 2 + 1)
4 4 x→ ∞
x→ a a + 2x + 3x
3a + x + 2 1 2 sin 2 ( y / 2)
×
x
[rationalise] = ⋅ lim After rationalisation,
a + 2x − ( 2 x )2 8 2 y→0 y2  
 ax 
(a − x ) 3a + x + 4x 1 1 1   = lim  
lim × = ⋅ = Q θlim sin θ = 0 x→ ∞
 a x + ax + 1 +
2 2
a x + 1
2 2
x → a 3( a − x ) a + 2x + 3x 8 2 2 16 2  → 0
 
1 4a + 4a  
= ⋅ = continue 8. (b)
3 3a + 3a  ax 
2 sin a(cos x − 1) = lim  
9. (c) lim x→ ∞ 1 1
2. (b) Here, degree of numerator and x→ 0 x sin x  x a2 + a + + x a2 + 2 
2
denominator being same.
( 1 − cos x )  x   x x x 
So, we divide both by x 3 , we get = lim − 2 sin a ⋅    
3 2 x→ 0 x2  sin x 
1− 2 + 3  a 
lim x x = 1 2 sin 2 ( x / 2) = lim  
1 3 = lim − 2 sin a ⋅ = − sin a x→ ∞
 a2 + a + 1 1 
x→ ∞
2+ 2 − 2 2 x→ 0 2 + a + 2
2

4   
x sin x  2
x x   x x x 
 2  x  a a a 1
2x − 1 = = = =
3. (b) lim a + a 2a 2
x → 0 ( 1 + x )1 / 2 − 1 10. (d) By the definition of the greatest a2 + 0 + a2
integer function,
( 2x − 1){( 1 + x )1 / 2 + 1} [ x ] = − 1 when −1 ≤ x < 0  x 3 − 3x + 2
= lim , for x ≠ 1
x→ 0 {( 1 + x )1/2 − 1} {( 1 + x )1 / 2 + 1} 
16. (a) f ( x ) =  ( x − 1)
2
and [ x ] = 0 when 0 ≤ x < 1
2x − 1 Hence, by the definition of the  k, for x = 1
= lim { ( 1 + x )1 / 2 + 1 } 
x → 0 1+ x − 1 greatest integer function
sin( −1) Since, the function is continuous.
2x − 1 f (x ) = = sin 1 …(i)
= lim lim { ( 1 + x )1/2 + 1 } −1 ∴ lim f ( x ) = f ( 1)
x→ 0 x x→ 0 x→1
when −1 ≤ x < 0
= (log 2) ⋅ 2 = 2 log 2 x 3 − 3x + 2
sin 0 0 ⇒ k = lim
Q lim 2 − 1  0 
x
and f ( x ) = = …(ii) x→1 ( x − 1)2
 x→ 0 x  0  0 0
when, 0 ≤ x < 1 ( x − 1)2 ( x + 2)
= lim
= log 2
2x log 2 ( x − 1)2
= lim ∴ Lf ( 0 − 0) = lim sin 1 = sin 1
x→1
x→ 0 1 
h→ 0
and Rf ( 0 + 0) = lim 0 = 0 ⇒ k = ( 1 + 2) ⇒ k =3
3 + h −3 h h→ 0 x+ 4
4. (d) RHL = lim = lim x + 6
h → 0 |3 + h − 3| h → 0 |h| Since, f ( 0 − 0) ≠ f ( 0 + 0), then the 17. (d) lim  
limit of f ( x ) at x = 0, does not exist. x→ ∞  x + 1 
x+ 4 5
h ⋅ ⋅( x + 1 )
= lim=1
= lim  1 +
11. (a) Using L’ Hospital’s rule, 5  5 x +1
h→ 0 h 
3 − h −3 ax − x a a x log e a − ax n − 1 x→ ∞  x + 1
LHL = lim =1 lim x = lim 5⋅
x+ 4
h→ 0 | 3 − h − 3 | x→ a x −a a x → a x x + x x log x x +1
e   x +1
= lim   1 +
5  5 
a a log e a − a ⋅ a n − 1 log e a − 1 
x → ∞  x + 1 
Since, RHL ≠ LHL = =
a + a log e a
a a
log e a + 1  
So, the limit does not exist. 4
1+
x
1 − cos 3x ( 1 − cos 3x )x satisfied only, when a = 1. 5 lim
5. (c) lim = lim x → ∞1 + 1
x → 0 x sin x x→ 0 x 2 sin x =e x = e5
12. (a) 13. (d)
1 − cos 3x x 14. (d) Let e x − 2 − 1 = θ, then as 18. (c) For continuity at x = 1,
= lim ⋅ lim
x→ 0 x2 x → 0 sin x x → 2, θ → 0 RHL = LHL = V
3 sin 3x 9 cos 3x 9 Q lim sin θ = θ = e x − 2 − 1 1
= lim ⋅ 1 = lim = θ→ 0 ⇒ = a ⇒ a = 1, − 1
e x −2 − 1  0
form 
x→ 0 2x x→ 0 2 2 a
∴ lim f ( x ) = lim
x→ 2 0
x → 2 log( x − 1)   For continuity at x = 2,
6. (c)
Since, log 1 is 0. RHL = LHL = V ⇒ a = b 2 − 2b
2 − 2 cos(θ − π / 4)
7. (b) lim Apply L’Hospital’s rule, When, a = 1, b 2 − 2b − 1 = 0
θ → π /4 16 (θ − π/4)2
(ex − 2 ) ∴ b = 1± 2
Limit = lim
2 ( 1 − cos y ) x → 2 1 / ( x − 1)
When, a = − 1, b 2 − 2b + 1 = 0
= lim ⋅
y→ 0 16 y2 = e 0 ⋅ ( 2 − 1) = 1 ⋅ 1 = 1 ∴ b=1
19. (d) We have, f ( 0 + 0)
MATHEMATICS Limits, Continuity and Differentiability 247

= lim ( 0 + h )2 sin
1 f ( 0 + h ) − f ( 0) 27. (b) We know that,
= lim
h→ 0 0+ h h→ 0 h tan −1 x sin −1 x
1 < 1 and > 1, x ∈ R
= lim h 2 sin =0 h
−0 x x
h→ 0
= lim 1 + h
h 1 −1 −1
= lim =1 tan x sin x
− <0
Similarly, f ( 0 − 0) = 0 h→ 0 h h→ 0 1 + h Q
x x
Since, f ( 0 + 0) = f ( 0 − 0) = f ( 0) , Q LHD = RHD sin −1 x tan −1 x
and − >0
So, the function is continuous at x = 0. So, f ( x ) is differentiable at x = 0. x x
( 0 + h )2 sin[ 1/ ( 0 + h )] − 0 Hence, f ( x ) is differentiable in ( −∞ , ∞ ). x2
Rf ′( 0) = lim 28. (a) I. lim = lim( x ) = 0
h→ 0 h 3x − 4, 0 ≤ x ≤ 2 x→ 0 x x→ 0
23. (c) Q f ( x ) =  x2
= lim h sin ( 1/h ) = 0 2x + l , 2 < x ≤ 9 II. It is true that is not continuous at
h→ 0 
Similarly, Lf ′ ( 0) = 0 x
and f ( x ) is continuous at x = 2 x = 0.
Hence, f is differentiable at x = 0 and ∴ lim− f ( x ) = f ( 2) |0 − h| h
x→ 2
III. LHL = lim = lim = −1
hence, for all x. h → 0 ( 0 − h) h → 0 −h
⇒ lim− ( 2x + l ) = 3 ( 2) − 4
20. (c) Given function, f ( x ) = | x | + x 2
x→ 2 |0 + h| h
⇒ lim { 2 ( 2 − h ) + l } = 6 − 4 RHL = lim = lim = 1
Again, defining the function f ( x ), h → 0 ( 0 + h) h→ 0h
h→ 0
 x 2 − x , x < 0 ⇒ 4+ l =2 ⇒ l = −2
Q LHL ≠ RHL So, it does not exist.
f (x ) =  2 , at x = 0, 24. (c)
x + x , x ≥ 0  x − 3, x ≥ 3
Sol. (Q. Nos. 29-30)
π π
LHL = f ( 0 − 0) = lim f ( 0 − h ) 25. (c) Qf ( x ) = | x − 3 | =  f   = b +
3 − x, x < 3  4 2
h→ 0 
= lim ( − h )2 − ( − h ) = 0  π  π
∴ LHL = lim − f ( x ) = lim f ( 0 − h ) f  + 0 = b +
h→ 0 x→ 0 h→ 0 4  2
RHL = f ( 0 + 0) = lim f ( 0 + h ) π π
h→ 0 = lim (3 + h ) = 3 
f  − 0 = + a
= lim ( h )2 + ( h ) = 0 h→ 0 4  4
h→ 0 and RHL = lim + f ( x ) = lim f ( 0 + h )
Also, f ( 0) = 0 x→ 0 h→ 0
Since, f ( x ) is continuous in [ 0, π ].
π π π
Q LHL = RHL = f ( 0) = 0 = lim (3 − h ) = 3 ∴ f  + 0 = f   = f  − 0
h→ 0 4   4 4 
So, function is continuous at x = 0. π π π
f ( 0 + h ) − f ( 0) ⇒ LHL = RHL ⇒ b+ = + a ⇒a −b = ...(i)
Now, Rf ′( 0) = lim 2 4 4
h→ 0 h So, f ( x ) is continuous at x = 0. π
Now, f   = − a − b
h( h + 1) − 0 Now, LHD  2
= lim =1 f ( 0) − f ( 0 − h ) π π
h→ 0 h = f ′( 0− ) = lim f  + 0 = − a − b ⇒ f  − 0 = b
f ( 0 − h ) − f ( 0)
h→ 0 h 2  2 
Lf ′( 0) = lim 3 − (3 − h ) π π π
h→ 0 −h = lim =1   
Again, f   = f  + 0 = f  − 0 
h→ 0 h  2 2  2 
( h2 + h) − 0 and RHD
= lim = −1 ⇒ − a − b = b ⇒ a = − 2b ...(ii)
h→ 0 −h f ( 0 + h ) − f ( 0)
= f ′( 0+ ) = lim
Lf ′ ( 0) ≠ Rf ′ ( 0) h→ 0 h From Eqs. (i) and (ii), we get
π π
Q
3 + h −3 a= ,b=−
So, f ( x ) is not differentiable at x = 0. = lim =1 6 12
1 h→ 0 h

(0 − h ) ⇒ LHD = RHD 29. (a) 30. (d)
21. (d) LHL = lim e
h→ 0 31. (b) Since, x 4 + 4x 2 + t ≥ 1
∴ f ( x ) is differentiable at x = 0 .
= lim e 1/h
=e ∞
=∞ |x |
Hence, both Statements I and II are So, 0≤ ≤ |x | ,
h→ 0 1


1 correct. x + 4x 2 + 7
4
(0 + h )
RHL = lim e = lim e h = e −∞ − x , x < 0 but lim |x | = 0
h→ 0 h→ 0
26. (a) For f ( x ) =|x | , f ( x ) =  x , x→ 0
x≥0
= does not exist  Hence, lim
|x |
=0
x→ 0
 x , x<0 For continuous x 2 + 4x 2 + 7

=  1 −x x
x
22. (a)Q f ( x ) = lim− f ( x ) = lim + f ( x ) = f ( 0) e 1/x
− e −1 / x 
1+ |x |  , x≥0 x→ 0 x→ 0 32. (c) 0 ≤ x 2  1 / x 
1+ x lim− ( − x ) = lim + f ( x ) = f ( 0) = 0 e + e −1 / x 
x→ 0 x→ 0
∴ LHD = f ′( 0− ) 0=0=0  1 − e −2 / x 
f ( 0 − h ) − f ( 0) = x2   ≤ x , ∀x > 0
2
= lim Hence, f ( x ) =|x | is continuous for all  1 + e −2 / x 
h→ 0 −h x for f ( x ) = x 3 + x 2 − 1 So, lim + f ( x ) = 0, Also,
−h x→ 0
−0 Clearly, x 3 + x 2 − 1 is defined for all  e 2 / x − 1
= lim 1 + h = lim
1
=1 value of x. lim− f ( x ) = lim x 2  2 / x  =0
h→ 0 −h h→ 0 1 + h
x→ 0 x→ 0 e + 1
Hence, f ( x ) is continuous, ∀x ∈R.
and RHD = f ′( 0+ ) Hence, lim f ( x ) = 0 and exists.
x→ 0
248 NDA/NA Pathfinder

33. (d) Since, x − 1 ≤ [ x ] ≤ x , ∀ x ∈ R 2( 1 − cos x ) f ( 0 + h ) − f ( 0)


42. (d) Given, lim Rf ′( 0) = lim
x→ 0 x2 h→ 0
− 1≤  3  ≤ 3
1 1 1 h
x3  x  x 2  1 − 1 + 2 sin 2 
x = lim
h2 − 0
 2
⇒ x 5  3 − 1 ≤ x 5  3  ≤ x 2 ∀x > 0
1 1 h→ 0 h
= lim
x   x  x→ 0 x2 Y

and x 2 ≤ x 5  3  ≤ x 5  3 − 1 ∀x < 0


1 1 x 2
4 sin 2  sin x 
 x  x  2 = lim  2 
= lim X
x→ 0 x  X′
x→ 0 x2  
So, lim x 5  3  = 0
1 O
 x   2 
x→ 0 f(x) = x |x|
 sin θ 
2 lim (cos x −1 ) cot 2 x = ( 1)2 = 1 Q θlim = 1 Y′
34. (c) lim (cos x ) cot x
= ex → 0 → 0 θ
x→ 0
  lim h=0
x2 h→0
cos x − 1 43. (b) f ( x ) = ,x ≠0 ∴ Lf ′( 0) = Rf ′( 0)
tan 2 x × x 2 = e x → 0 cos x − 1
lim lim |x |
= ex → 0 2 So, f ( x ) is differentiable for all real
x x2 As, f ( 0) = 0 (given)
 values of x.
1 − cos x 1   x2
= e −1 / 2 Q xlim =  = x, if x > 0
 →0 x
2
2  x 46. (c) Given function, f ( x ) = |x − 3 |
or f ( x ) =  x 2
π
1/x
 = − x , if x < 0 I. LHL at x = 3, f (3 − 0)
35. (d) lim  tan  + x    − x
x→ 0 
 4   = lim − f (3) = lim f (3 − h )
x→ 3 h→ 0
1 /x
lim 
1 + tan x
− 1 
1 ∴ f ( 0) = 0 = lim |3 − h − 3 | = lim |− h | = lim h = 0
1 + tan x 
= lim   =e x → 0  1 − tan x  x
Clearly, it is a modulus function and x→ 0 x→ 0 x→ 0
x → 0  1 − tan x 
modulus function is continuous RHL at x = 3, f (3 + 0)
lim 
2 tan x  1

 1 − tan x  x
everywhere. = lim + f (3) = lim f (3 + h )
= ex → 0 = e2 1 x→ 3 h→ 0
44. (b) I. Given, lim
x − 1 + cos x 
1/x
x→ 0 x = lim |3 + h − 3 |= lim |h | = 0
36. (b) lim   h→0 h→ 0
x→ 0  x  LHL = f ( 0 − 0) = lim f ( 0 − h )
1 x→ 0 Here, LHL = RHL = f (3)
1
= lim − = − ∞ So, f ( x ) is continuous at x = 3.
( 1 − cos x )  x
= lim  1 − 
x→ 0 h
x→ 0   f ( 0 − h ) − f ( 0)
x RHL = f ( 0 + 0) = lim f ( 0 + h ) II. Lf ′( 0) = lim
lim − 
1 − cos x  1  1 − cos x  1 x→ 0 x→ 0 −h
x→0 
 ×
 x
lim − 
x→0  x2 
 = lim + = + ∞
=e x
=e x→ 0 h | − h − 3 | − | − 3|
= lim
Q LHL ≠ RHL, h→ 0 −h
= e −1 / 2 Q lim 1 − cos x = 1  h + 3 −3 h
1 = lim = lim = −1
 x → 0 x2 2  So, lim does not exist.
x→ 0 x h→ 0 −h h→ 0 − h

a x + b x + cx 
2/x f ( 0 + h ) − f ( 0)
37. (c) lim   II. Given, lim e1 / x ,
x→ 0
Rf ′( 0) = lim
h→ 0
x→ 0  3  h
LHL = f ( 0 − 0) lim |h − 3 | − | − 3 |
= lim
x x x
 a + b + c −1  2 h→ 0
lim  −1  × 1
x→0   x − h→ 0
=e 3
f ( 0 − h ) = lim e h = e −∞ = 0
h
 a x −1 b x −1 c x −1  2 x→ 0 −h + 3 − 3 −h
lim  + +  × = lim = lim =− 1
=e x→0  x x x  3 RHL = f ( 0 + 0) = lim f ( 0 + h ) h→ 0 h h→ 0 h
h→ 0
2 1
= (log a + log b + log c ) × = lim e h = e ∞ = ∞ ∴ Lf ′ ( 0) = Rf ′ ( 0)
3 h→ 0
⇒ f ( x ) is differentiable at x = 0.
Q lim a − 1 = log a 
x
Q LHL ≠ RHL ⇒ Both Statements exist.
 x→ 0 x 
∴ lim e1 / x does not exist.
2 x→ 0 1− 1+ x
log ( abc ) 2 47. (b) lim
= e3 = ( abc ) 3 Hence, only Statement II is true. x→ 0 x
45. (b) Let us take the function f ( x ) = x |x | 1− 1+ x 1+ 1+ x
38. (b) 39. (b)
40. (c) = lim ×
Redefine this function, x→ 0 x 1+ 1+ x
 1
 x 2 , if x ≥ 0
2
41. (a) lim x sin   [rationalisation]
x→ 0 x f (x ) =  2
1 − x , if x < 0 = lim
1− (1+ x )
Put x = 1 / y = lim 2 ⋅ sin ( y ) x→ 0 x ( 1 + 1 + x )
y→ ∞ y f ( 0 − h ) − f ( 0)
Lf ′( 0) = lim −x −1
sin y h→ 0 −h = lim = lim
= lim x→0 x (1+ 1+ x ) x→0 (1+ 1+ x )
y→ ∞ y 2 −( − h )2 − 0
= lim −1 −1 −1
[Q − 1 ≤ sin θ ≤ 1, ∀ θ ∈ R ] h→ 0 −h = = =
sin∞ −h2 1+ 1 + 0 1+ 1 2
= = 0 × (finite value) = 0 = lim = lim ( + h ) = 0
∞ h→ 0 − h h→ 0
MATHEMATICS Limits, Continuity and Differentiability 249

sin x − tan x
Hence, lim sin   does not exist.
48. (a) lim which is of the 1 Using L’ Hospital’s rule,
x →0 x x →0 x 1
. f ′( x )
form   .
0
II. lim x sin   f ′( x ) × x
1 2 f (x )
 0 x→ 0 x lim = lim
x→ 9 1 x→ 9 f (x )
.1
Now, applying L’ Hospital’s rule, LHL = f ( 0 − 0) = lim f ( 0 − h ) 2 x
h →0
cos x − sec 2 x −1 f ′( 9) × 9 4 × 3 4 × 3
= lim = lim ( − h ) sin   = = = =4
x→0 1 h→ 0  h f ( 9) 9 3
cos 0 − sec 2 0 1 − 1 0 1
= lim h sin = 0 × (Finite value) = 0
= = = =0 54. (b)
1 1 1 h→ 0 h
I. Given, f ( x ) = 3 x , f ( x ) = ( x )1 / 3
49. (d) Let y =|x | RHL = f ( 0 + 0) = lim f ( 0 + h )
h →0 Now, we check the continuity of the
= lim ( h ) sin  
1
Redefined the given curve, function at x = 0.
h→ 0  h
 x, x > 0 LHL = f ( 0 − 0) = lim f ( 0 − h )
 = 0 × sin( ∞ ) = 0 × (Finite value) = 0 h→ 0
y =  0, x = 0 Q LHL = RHL = lim ( 0 − h )1 / 3 = ( 0 − 0)1 / 3 = 0
− x , x < 0 h→ 0
Hence, lim x sin   exists.
1
 RHL = f ( 0 + 0) = lim f ( 0 + h)
h →0 x h→ 0
f ( 0 − h ) − f ( 0) = lim ( 0 + h )1/ 3 = ( 0 + 0)1 / 3 = 0
LHD = Lf ′( 0) = lim
h→ 0 −h 52. (b) Given function, h→ 0
−( − h ) − 0 and f ( 0) = ( 0)1 / 3 = 0
= lim = lim
h
= −1  x 2 , x>2
h→ 0 −h h→ 0 − h f (x )=  Q LHL = RHL = f ( 0)
3 x − 2 , x≤2
f ( 0 + h ) − f ( 0) So, function is continuous at x = 0.
RHD = Rf ′ ( 0) = lim
h→ 0 h First, we check the continuity of f ( x )
II. Given, f ( x ) = [ x ], which is greatest
h−0 at x = 2 .
= lim =1 integer function.
h→ 0 h
LHL = f ( 2 − 0) = lim f ( 2 − h )
h →0 We know that, the greatest integer
Q Lf ′( 0) ≠ Rf ′( 0) = lim 3( 2 − h ) − 2 = lim 6 − 3 h − 2 function is continuous for all x
h →0 h →0
Hence, the derivative of |x | at x = 0 except integer values of x. So,
does not exist. = lim 4 − 3 h = 4 − 0 = 4 f ( x ) = [ x ] is continuous at x = 2.99.
h →0
RHL = f ( 2 + 0) = lim f ( 2 + h ) Alternate method
Note Curve | x| have sharp turn at x = 0 h→ 0
LHL = f ( 2.99 − 0) = lim ( 2.99 − h )
= lim ( 2 + h )2 = ( 2 + 0)2 = 4 h→ 0
h →0
50. (d) I. We know that, = lim ( 2.99 − h ) = lim 2 = 2
If f ( a ) = lim f ( x ), then f ( x ) is and f ( 2) = 3( 2) − 2 = 6 − 2 = 4 h→ 0 h→ 0
x→ a
and RHL = f ( 2.99 + 0)
continuous at x = a, while both hand Q f ( 2) = LHL = RHL = lim f ( 2.99 + h )
h→ 0
must exist. Hence, f ( x ) is continuous at x = 2.
II. If f ( x ) is continuous at a point, then it Now, we check the differentiability of = lim ( 2.99 + h ) = lim 2 = 2
1 h→ 0 h→ 0
is not necessary that is also f ( x ) at x = 2. and f ( 2.99) = ( 2.99) = 2
f (x ) f ( 2 − h ) − f ( 2)
LHD = Lf ′( 2) = lim Q LHL = RHL = f ( 2.99)
continuous at that point. h →0 −h
e.g. 3( 2 − h ) − 2 − (3 ⋅ 2 − 2) So, f ( x ) is continuous at x = 2.99.
= lim
(i) f ( x ) = x is continuous at x = 0 h →0 −h 55. (b) I. From Graph of f ( x ) = |x |, we
1
but f ( x ) = is not continuous at −3 h observe that the curve has sharp turn at
x = 0. x = lim =3 x = 0.
h→ 0 − h
(ii) f ( x ) = e x is continuous at x = 0 f ( 2 + h ) − f ( 2) So, the function f ( x ) = |x | is not
and f ( x ) = e − x is also continuous at RHD = Rf ′( 2) = lim
h →0 h differentiable only at x = 0. i.e.
x = 0. f ( x ) = |x | is differentiable at x = 1.
( 2 + h )2 − f ( 2)
51. (c) I. lim sin
1 = lim II. Given function, f ( x ) = e x .
h →0 h
x→ 0 x Now, we check the differentiability
LHL = f ( 0 − 0) = lim f ( 0 − h ) 4 + h 2 + 4 h − (3 ⋅ 2 − 2 ) of f ( x ) at x = 0.
h→0 = lim =4
1 1 h→ 0 h f ( 0 + h ) − f ( 0)
= lim sin = lim − sin = − sin( ∞ ) Rf ′( 0) = lim
h→0 ( − h) h → 0 h Q Lf ′( 2) ≠ Rf ′( 2) h→ 0 h
= −[A definite number persist between Hence, f ( x ) is not differentiable at e (0 + h ) − e 0
−1 to + 1] x = 2. = lim
h→ 0 h
RHL = f ( 0 + 0) = lim ( 0 + h ) 53. (c) Given that,
h →0 eh − 1  0 form 
1 f ( 9) = 9 and f ′ ( 9) = 4 = lim
= lim sin = sin( ∞ ) h→ 0 h  0 
h→ 0 h f (x ) − 3  0 form 
= [A definite number persist between lim Use L’ Hospital’s rule,
x→ 9 x −3  0 
− 1 to + 1] eh − 0
[Q f ( 9) = 9 ⇒ f ( x ) = x i.e., x = 9] = lim = e0 = 1
Q LHL ≠ RHL h→ 0 1
250 NDA/NA Pathfinder

f ( 0 − h ) − f ( 0) 2 − x for 1 ≤ x ≤ 2 2 
Lf ′( 0) = lim h

h→ 0 −h 57. (d) Q f ( x ) =  1 h 1  2
3x − x 2 for x > 2 = lim = lim
 2 h → 0 sin h 2 h → 0 sin  h 
e −h − e 0  
 2
= lim − 1 for 1 ≤ x ≤ 2 2
h→ 0 −h ⇒ f ′( x ) =  …(i) 1
= ×2×1
1 − e −h 3 − 2x for x > 2 2
= lim  0 form 
 θ 
h→ 0 h  0  So, the differentiable coefficient of
Q θlim = 1
f ( x ) at x = 3 is → 0 sin θ
 
Use L’ Hospital’s rule,
f ′(3) = 3 − 2(3) = 3 − 6 = − 3 = 2
e −h
= lim = e −0 = 1 [Q f ′( x ) = 3 − 2x for x > 2] Q LHL ≠ RHL
h→ 0 1
So, limit does not exist.
Since, Rf ′( 0) = L f ′( 0) 58. (d) Here, we check the existence of limit
Sol (Q. Nos. 61-62) Given function,
of f ′( x ) at x = 2.
Hence, f ( x ) = e x is differentiable at 1 − sin x
So, f ′( 2 + 0) = RHL = lim f ′( 2 + h ) f (x ) =
x = 1. h→ 0 ( π − 2x )2
= lim 3 − 2( 2 + h ) π π
Sol. (Q. Nos. 56-58) Given function,
h→ 0
[from Eq. (i)] where, x ≠ and f   = λ
 2
2 − x for 1 ≤ x ≤ 2 = lim 3 − 4 − 2h = lim − 1 − 2h = − 1
2
 1 − sin x
f (x ) =  h→ 0 h→ 0
61. (d) lim f ( x ) = lim
3x − x for x > 2
2
and f ′( 2 − 0) = LHL = lim f ′( 2 − h ) x → π /2 x → π / 2 ( π − 2x )2
h→ 0
and whole function defined in 1 ≤ x < ∞. = lim − 1 = − 1 ( 0 / 0 form )
h→ 0
Use L’ Hospital’s rule,
56. (b) I. Since, the function is polynomial, So, both f ′( 2 + 0) and f ′( 2 − 0) − cos x
so it is continuous as well as exists i.e. f ′( x ) exist at x = 2. = lim
x → π / 2 2( π − 2x )( −2)
differentiable in its domain [1, ∞) − {2}.
( 1 + x ) n − 1 cos x  0 form 
Now, we check the continuity of the 59. (c) lim   = lim
x → 0 x 
x → π / 2 4( π − 2x )  0 
function at x = 2.
n ( n − 1) 2
LHL = f ( 2 − 0) = lim f ( 2 − h ) ( 1 + nx + x + ...) − 1 Again, use L’ Hospital’s rule,
h→ 0
= lim 2! − sin x sin x
= lim 2 − ( 2 − h ) = lim h = 0 x→ 0 x = limπ = lim
h→ 0 h→ 0 x→ 4 ( − 2) x → π / 2 8
[by binomial expansion in any index] 2
RHL = f ( 2 + 0) = lim ( 2 + h ) n ( n − 1) 2 1 π 1 1
h→ 0 nx + x + ... = . sin = × 1 =
= lim 3( 2 + h ) − ( 2 + h )2 2! 8 2 8 8
= lim
h→ 0 x→ 0 x 62. (a) Since, the given function is
n ( n − 1) continuous at x = π /2.
= 3 ( 2 + 0) − ( 2 + 0) = 6 − 4 = 2
2 = lim n + x + ...
x→ 0 2! 1 − sin x
and f ( 2) = 2 − 2 = 0 = n + 0 + 0+ ... = n ∴ f ( π / 2) = lim
x→ π/2 ( π − 2x )2
Q f ( 2) = LHL ≠ RHL x 1
60. (d) lim ⇒ λ= [from upper part]
So, the function is not continuous at x → 0 1 − cos x
8
every point in the interval [1, ∞), i.e. = lim
x
not continuous at x = 2. x→ 0 x x 2 − 5, x ≤3
1 − ( 1 − 2 sin 2 ) 63. (b) We have, f ( x ) = 
II. We also check the differentiability of the 2  x + 13 , x > 3
x 1 x
function at x = 15 .. = lim = lim
x→ 0 2 x 2 x → 0 |sin x | To find lim f ( x )
f ( 1 . 5 + h ) − f ( 15
. ) 2 sin x→ 3
Rf ′( 1.5) = lim 2 2 LHL = lim − f ( x ) = lim x 2 − 5
h→ 0 h x→ 3 x → 3−
LHL = f ( 0 − 0) = lim f ( 0 − h )
2 − ( 1.5 + h ) − ( 2 − 1.5) h→ 0 = lim [(3 − h ) − 5]
2
= lim
h→ 0 1 −h x→ 3 −h
h = lim
. − h − 05 −h = lim ( 9 − 6h + h 2 − 5) = 4
sin  
05 . h 2 h→ 0
= lim = lim − = − 1 h→ 0
h→ 0 h→ 0 h  2 
h RHL = lim+ x + 13
. − h ) − f ( 15 1 h x→ 3
Lf ′( 15
. ) = lim
f ( 15 . ) =− lim
= lim 3 + h + 13 = lim 16 + h = 4
− sin  
h→ 0 −h 2 h→ 0 h
x→ 3 + h h→ 0
2 − ( 15
. − h ) − ( 2 − 15 . )  2
= lim ∴ lim − f ( x ) = lim + f ( x ) = 4
2 ⋅  
h→ 0 −h h x→ 3 x→ 3

. + h − 05 1  2 1 lim f ( x ) = 4
= lim
05 .
= lim
h
=−1 =− lim =− ×2×1 x→ 3
h→ 0 −h h → 0 −h 2 h→ 0 sin h 2
2 64. (d) For continuous,
Q Lf ′( 15
. ) = Rf ′( 15
. ) Q lim θ = 1 lim − f ( x ) = lim + f ( x ) = f (3)
So, the function is differentiable at  θ→ 0 sin θ  x→ 3 x→ 3

x = 1.5. ∴ lim − f ( x ) = lim + f ( x ) = 4


=− 2 x→ 3 x→ 3
RHL = f ( 0 + 0) = lim f ( 0 + h ) ⇒ f ( x ) is continuous at x = 4.
h→ 0
MATHEMATICS Limits, Continuity and Differentiability 251

We have, f ( x ) = x 2 − 5, x ≤ 3 G ( x ) − G ( 1)  2 + x 2 − ( Ax + Ax 2 + B + Bx ) 
Clearly, lim = G ′( 1) …(i) ⇒ lim   =3
⇒ f ′( x ) = 2x ⇒ f ′( 0) = 0 x →1 x −1 x→ ∞  1+ x 
Now, let us first find G ′( x ), which is
Hence, f ( x ) is differentiable at x = 0.  2 + x 2 − Ax − Ax 2 − B − Bx 
given by ⇒ lim 
So, neither I nor II Statement are correct. 1 1  =3
G ′( x ) = × ( −2 x ) x→ ∞  1+ x 
 tan kx , x<0 2 25 − x 2
 −x  2x − A − 2 Ax − B ) 
65. (b) We have, f ( x ) =  x = G ′( x ) = ⇒ lim   =3
3x + 2k , x ≥ 0
2
25 − x 2 x→ ∞  1 
−1 −1  
Q f ( x ) is continuous at x = 0. = G ′( 1) = =
24 2 6 [using L’ Hospital’s rule]
∴ lim− f ( x ) = lim+ f ( x ) = f ( 0)
x→ 0 x→ 0 71. (c) I. We have, f ( x ) = [ x ] ⇒ lim [ x ( 2 − 2 A ) − ( A + B )] = 3
tan kx x→ ∞
⇒ lim− = lim+ 3x + 2k 2 Clearly, lim + f ( x ) = lim f ( n + h ) For limit to exist and equal to 3, we
x→ 0 x x→ 0 x→ n h→ 0
should have
= 3( 0) + 2k 2 = lim [ n + h ] = n 2 − 2 A = 0 and A + B = − 3 …(i)
h→ 0
tan k( 0 − h )
⇒ lim and lim − f ( x ) = lim f ( n − h ) ⇒ A=1
x→ 0 − h 0−h x→ n h→ 0
= lim [ n − h ] = n − 1 75. (c) From Eq. (i), A + B = −3,
= lim 3( 0 + h ) + 2k = 2k 2 2
h→ 0
x→ 0 + h
∴ lim+ + f ( x ) =/ lim− f ( x ) for 1 + B = −3 ⇒ B = −4
tan kh k tan kh x→ n x→ n
⇒lim = 2k 2 ⇒lim = 2k 2
h→ 0 h h→ 0 kh all n ∈ Z ′  α cos x , , if x =/ π
Q lim tan x = 1 So, f is discontinuous at x = n ← Z ′  π − 2x , 2
⇒ k = 2k 2 76. (a) We have, f ( x ) = 
 x→ 0 x  π
II. We have, f ( x ) = cot x , consider, 3, if x =
∴ k = 1/ 2  2
f ( x ) = lim f ( nπ + h ) π
5x − 1 lim is continuous at x = , where α is a
66. (a) lim = log e 5 x → nπ + h→ 0
2
x→ 0 x = lim cot( nπ + h ) constant.
Q lim a − 1 = log a 
x h→ 0
= lim cot hπ which does not exist π
 x→ 0 x e 
 h→ 0 ⇒ limπ f ( x ) = f   = 3 …(i)
x→  2
67. (d) Thus, f is discontinuous at x = nπ, 2
n ∈ Z ′. α cos x
log ( 1 + x ) Now, consider limπ = limπ
68. (b) lim 5 x→ x→ π − 2x
x→ 0 x Sol. (Q. Nos. 72-73) 2 2
cos x
log e ( 1 + x ) log e b  We have = α limπ
= lim Q log a b =  x→ π − 2x
x→ 0 x log e 5  log e a   −1, if x≤0 2
 π
, f ( x ) = ax + b , if 0 < x < 1 Now, put y = x − ,
1 log e ( 1 + x ) 2
= lim log e  1, if x≥1 π
log e 5 x→ 0 x 5
 then as x → ⇒y→0
2
 log e ( 1 + x ) where a and b are constants and f ( x ) π
=1 cos  y + 
Q xlim is continuous everywhere.  2
 →0 x ∴ limπ f ( x ) = α lim
1  As f ( x ) is continuous everywhere, x→ y→ 0 π
π − 2  + y 

and log a b =  therefore f is continuous at x = 0 and
log a 
2
2 
b  x = 1 also. − sin y
= α lim
69. (b) I. The greatest integer function is 72. (a) lim + f ( x ) = lim − f ( x ) = f ( 0) y→ 0 −2 y
continuous at all points except x→ 0 x→ 0
and lim + f ( x ) = lim − f ( x ) = f ( 1) α sin y α
integer. Therefore, Statement I is x→1 x→1 = lim = …(ii)
false. Now, consider lim + f ( x ) = f ( 0) 2 y→0 y 2
x→ 0
II. Let h( x ) = sin x and g( x ) = | x | ⇒ lim f ( 0 + h ) = − 1 From Eqs. (i) and (ii), we get
hog( x ) = sin | x | h→ 0 α
= 3 ⇒α = 6
⇒ lim a( 0 + h ) + b = − 1 2
∴ f ( x ) = hog ( x ) = sin | x | h→ 0

So, g( x ) is continuous, ∀x ∈ R and ∴ b = −1 77. (d) Consider,


h( x ) is continuous, ∀x ∈ R. 6 cos x cos x
73. (d) lim f ( x ) = f ( 1) lim f ( x ) = lim = 6 lim
x → 1− x→ 0 π − 2x
x→ 0 x → 0 π − 2x
We know that, if g( x ) is continuous ⇒ lim f ( 1 − h ) = 1
and h( x ) is continuous. h→ 0  cos 0  6
=6  = [Q cos 0 = 1]
⇒ lim a ( 1 − h ) + b = 1 ⇒ a + b = 1  π − 2( 0)  π
Then, hog ( x ) is continuous. h→ 0
Hence, f ( x ) is continuous, ∀x ∈ R. ∴ a=2
78. (d) Refer Ques 70
2+ x2  sin( e x − 2 − 1)  0
70. (a) We have, G ( x ) = 25 − x 2
74. (b) lim  − Ax − B  = 3 79. (d) Consider, lim form 
x → ∞  1+ x 
G ( x ) − G ( 1) x→ 2 ln( x − 1)  0 
To find lim  2 + x 2 − ( Ax + B )( 1 + x ) 
x→1 x −1 ⇒ lim   =3 [use L’ Hospital’s rule]
x→ ∞  1+ x 
252 NDA/NA Pathfinder

cos( e x − 2 − 1) ⋅ e x – 2 1 × 1 π Hence, f ( x ) is continuous at x = 0.


= lim = =1 II. For continuity at x = − ,
x→ 2 1 2 π
1
π Statement I is correct. Now, for x =
x −1 limπ − f ( x ) = lim f ( x ) = f  −  2
x→ − π +  2
80. (c) Clearly, for f to be continuous at 2
x→ − LHL = lim f (x )
2 x → π /2−
x = π, π π
⇒ lim f  − − h  = lim  − + h  = lim π cos x = 0
1 − sin x + cos x  0  2  h→ 0  2 
form 
h→ 0 x→ π / 2 2
f ( π ) = lim π
x → π 1 + sin x + cos x   π RHL = lim f ( x ) = limπ  x − 
0 = f  −  x→ π / 2 + x→  2
− cos x − sin x  2 2
= lim π π
2
x → π cos x − sin x
⇒ lim − 2 sin  − − h  
  π =  −  = 0
h→ 0   2   2 2
[by L’ Hospital’s rule] 
 π π
π Also, f   = π cos = 0
=
− cos π − sin π = lim  A sin  − + h  + B   2 2
h→ 0    2  
cos π − sin π π
π Hence, f ( x ) is continuous at x = .
−( −1) − 0 1 = − 2 sin  − 
= = =−1 2
−1 − 0 −1  2 Statement II is correct.
81. (b) I. Consider, ⇒ 2=− A+ B=2 90. (d) For x = 0,
f ( 0 + h ) − f ( 0) 1/ h − 0 ⇒ A − B = −2 ...(ii) f ( 0 − h ) − f ( 0)
lim = lim On solving Eqs. (i) and (ii), we get LHD = lim −
h→ 0 h h→ 0 h h→ 0 −h
A = − 1 and B = 1
1 f ( − h ) − f ( 0)
= lim 2 = ∞ (not defined) 83. (c) The value of A is −1. = lim −
h→ 0h
84. (a) The value of B is 1.
h→ 0 −h
Hence, f is not differentiable at x = 0. 85. (a) Given, lim φ ( x ) = a 2 ( −h + π) − π −h
x→ 0
= lim = lim =1
II. We have, h→ 0 −h h → 0 −h
To find lim φ  
x
 2x + 5 , x>0
x→ 0  a f ( 0 + h ) − f ( 0)
f (x ) =  2 RHD = lim +
x + 2x + 5 , x ≤ 0
∴ lim φ   = a 2
h→ 0

x h
x→ 0  a f ( h ) − f ( 0) π cos h − π
 2, x>0 = lim+ = lim
⇒ f ′( x ) =  −1 / x 2 −1 / ( 0 + h ) 2 h→ 0 h h→ 0 2
2x + 2, x≤0 86. (a) lim e = lim e =0
 x→ 0 h→ 0
π [ −2 sin 2 ( h / 2)]
= lim
Q LHL of f ′( x ) = RHL of f ′( x ) h→ 0 h
a [x ] + x − 1
∴ f is differentiable at x = 0. 87. (b) Given, f (x ) =  ( h / 2) 
[x ] + x = lim π − sin × sin( h / 2) = 0
h→ 0   ( h / 2 ) 

82. (a)
a [x ] + x − 1
Sol. (Q. Nos. 83-84) Now, RHL = lim + LHD ≠ RHD
x→ 0 [x ] + x
Given, Hence, f ( x ) is not differentiable at
 − 2 sin x , π As, x → 0+ (i.e. approaches 0 from the x = 0.
if x ≤ −
 2 right), we get [ x ] = 0 ∴ Statement I is not correct.
 π π
f ( x ) =  A sin x + B ,if − < x < ax − 1 π
2 2 ∴ lim + f ( x ) = lim + For x = ,
 π x→ 0 x→ 0 x 2
 cos x , if x ≥
 2 a (0 + h ) − 1 ah − 1 f ( π / 2 − h ) − f ( π / 2)
= lim = lim = ln a LHD = lim −
π h→ 0 ( 0 + h) h→ 0 h h→ 0 −h
I. For continuity at x = ,
2 π
a [x ] + x − 1 π cos  − h  − 0
π 2 
lim− f ( x ) = lim+ f ( x ) = f   88. (c) lim− f ( x ) = lim− = lim
π π  2 x→ 0 x→ 0 [x ] + x h→ 0 −h
x→ x→
2 2 As x → 0− [i.e. approaches 0 from π sin h
π
⇒ lim f  − h  left], we get [ x ] = − 1 = lim = −π
h→ 0 2  a −1 + x − 1 h→ 0 −h
∴ lim − f ( x ) = lim −
π π x→ 0 x→ 0 −1 + x f ( π / 2 + h ) − f ( π / 2)
= lim f  + h  = f   RHD = lim +
h→ 0  2   2 −1 + ( 0 − h ) −1 h→ 0
a −1 a −1 h
= lim =
π −1 + ( 0 − h ) −1  π + h − π  − 0
2
⇒ lim A sin  − h  + B
h→ 0
h→ 0 2  = 1− a −1  2
= lim 2
h→ 0
π π h
= lim cos  + h  = cos   89. (c) LHL = lim − f ( x )
h→ 0 2   2 x→ 0
LHD ≠ RHD
= lim ( x + π ) = π
π π x→ 0
⇒ A sin + B = cos = 0 Hence, f ( x ) is not differentiable at
π
2 2 RHL = lim + f ( x ) = lim π cos x x = .
x→ 0 x→ 0 2
⇒ A +B = 0 ...(i)
= π cos ( 0) = π Statement II is not correct.
Also, f ( 0) = π cos ( 0) = π
21
MATHEMATICS > Measurements of Angles and Trigonometric Rations 253

DIFFERENTIATION
In NDA exam, generally 3-5 questions are asked from this chapter, which are based on finding
derivative of implicit, parametric and logarithmic functions, one function with respect to
another function and successive differentiation.

Differentiation is a method to compute the rate at which a dependent variable y changes with respect
to the change in the independent variable x. This rate of change is called the derivative of y with
respect to x. Differential coefficient (derivative) of a function y = f ( x), is defined as
dy f ( x + δx ) − f ( x )
= lim
dx δ x → 0 δx
The above definition of derivative is known as the first principle of derivative. Mathematically, we may
dy  dy  f ( x) − f ( a)  dy  f ( a + δx ) − f ( a )
define at x = a as   = lim . Also,   = lim
dx  
dx x = a x → a x−a  
dx x = a δ x → 0 δx

Rules of Differentiation
d (C )
(i) = 0, where C = constant
dx
d k d { f ( x)} d d { f ( x)} d { g ( x)}
(ii) { kf ( x)} = , where k is a constant. (iii) { f ( x) ± g ( x)} = ±
dx dx dx dx dx
d k d { f ( x)} k2 { g ( x)} d  f ( x) g ( x) f ′ ( x) − f ( x) g′ ( x)
(iv) { k1 f ( x) ± k2 g( x)} = 1 ± (v)  =
dx dx dx dx  g ( x)  { g ( x)} 2
d
(vi) [ f ( x) g ( x)] = f ( x) g′ ( x) + g ( x) f ′ ( x)
dx

Note Result (vi) can be generalised to find the derivative of the product of any number of differentiable functions.

Differentiation of Some Standard Function


d n d 1 d 1
(i) x = nx n − 1 (ii) (log e x) = , for x > 0 (iii) log a x = , for x > 0, a > 0, a ≠ 1
dx dx x dx x log a
d x d x d
(iv) a = a x log a, for a > 0 (v) e =ex (vi) (sin x) = cos x
dx dx dx
254 NDA/NA Pathfinder

d d
(vii) (cos x) = − sin x (viii) (tan x) = sec 2 x Chain Rule
dx dx
Let y = f (u ) be a differentiable function of u and
d
(ix) (sec x) = sec x tan x u = g ( x) be a differentiable function of x.
dx
d d Then, y = f (u ) = f [ g ( x)] is called a function of a
(x) cosec x = − cosec x cot x (xi) cot x = − cosec 2 x function or composite function.
dx dx
−1 dy dy du
d 1 d In this case, = ⋅
(xii) sin − 1 x = (xiii) cos − 1 x = dx du dx
dx 1 − x2 dx 1 − x2
This rule is called the chain rule. This rule can be
d 1
(xiv) sec − 1 x = , for x > 1 generalised for more functions.
dx x x2 −1
EXAMPLE 3. If x = 2 cos t − cos 2t, y = 2 sin t − sin 2t
d −1 π dy
(xv) cosec −1 x = then at t = , is equal to
dx x x2 −1 4 dx
d 1 d −1 a. 2+1 b. 2+ 1
(xvi) tan − 1 x = (xvii) cot − 1 x = 2+1
dx 1 + x2 dx 1 + x2 c. d. None of these
2
dy dx dy
EXAMPLE 1. If y = x 1/ 3 + x 1/ 2 , then find at x = 81. Sol. a. = −2 sin t + 2sin 2t and = 2cos t − 2cos 2t
dx dt dx
1 1 dy cos t − cos 2t
a. 3−1/3 + b. 3−10 /3 + =
18 6 dx sin 2t − sin t
1
c. 3−11/3 + d. None of these π
18 Put t = , we have
4
Sol. c. We have, y = x1/ 3 + x1/ 2  dy  cos π / 4 − cos π / 2
  =
dy d 1/ 3 d 1/ 2 dy 1 −2/ 3 1 −1/ 2  dx  t = π sin π / 2 − sin π / 4
∴ = (x ) + (x ) ⇒ = x + x 4
dx dx dx dx 3 2
 dy  1 −2 / 3 1 −1/ 2
= 2+1
∴  = (81) + (81)
 dx  x = 81 3 2
Differentiation of an Implicit Function
1 1 1 1
= [( 34) −2/ 3 ] + ⋅ [(9) 2 ]−1/ 2 = × 3−8/ 3 + ⋅ 9−1 When independent variable x and dependent variable y
3 2 3 2
occur together in an equation, then it is not
−8 / 3 − 1 1 1 −11/ 3 1
=3 + ⋅ =3 + convenient or possible to express y in terms of x. In
2 9 18
such cases, y is an implicit function of x and both are
dy connected by a relation of the form f ( x, y) = 0.
EXAMPLE 2. What will be the value of , when
dx dy
ex + e− x In order to find from f ( x, y) = 0, adopt the
y= ? dx
ex − e− x following procedure
4 4 −4 −4 (i) Differentiate each term of f ( x, y) = 0 with
a. b. 2 x − 2 x c. x − x 2 d. x − 2 x
( e x − e − x )2 e −e (e − e ) e −e respect to x, noting that y as a function of x.
ex + e− x dy d  ex + e− x  dy
Sol. c. We have, y = . Then, (ii) Simplify the equation thus obtained for .
=  x − x . dx
ex − e− x dx dx e −e 

( ex − e− x )
d x
( e + e− x ) − ( ex + e− x )
d x
( e − e− x )
dy
dx dx
EXAMPLE 4. If x 2 + 2xy + y 3 = 42, then is
= dx
( ex − e− x ) 2 equal to
( ex − e− x ) ( ex − e− x ) − ( ex + e− x ) ( ex + e− x ) −2( x + y ) −( 2x + 3y 2 )
= a. b.
( ex − e− x ) 2 2x + 3y 2 2( x + y )
( ex − e− x ) 2 − ( ex + e− x ) 2 −4 −2( x + y ) 2( x 2 + y 2 )
= = x c. d.
( ex − e− x ) 2 ( e − e− x ) 2 2y + 3x 2 2x + 3y 2
MATHEMATICS > Measurements of Angles and Trigonometric Rations 255

Sol. a. Q x 2 + 2xy + y 3 = 42
EXAMPLE 6. If y = sin x + sin x + sin x + K ∞ , then
d 2 d
∴ ( x + 2xy + y 3) = 42 dy
dx dx is equal to
 dy dx dy dx
⇒ 2x + 2  x + y  + 3y 2 =0 cos x cos x cos x
 dx dx  dx a. b. c. d. None of these
dy dy 2x − 1 2y − 1 2x + 1
⇒ 2x + 2x + 2y + 3y 2 =0
dx dx Sol. b. We have, y = sin x + sin x + sin x +K ∞
dy dy −2( x + y)
⇒ ( 2x + 3y 2) = −2( x + y) ⇒ =
dx dx 2x + 3y 2 ⇒ y = sin x + y ⇒ y 2 = sin x + y
On differentiating both sides w.r.t. x, we get
Important Substitutions dy dy dy
2y = cos x + ⇒ ( 2y − 1) = cos x
Inverse trigonometric functions involving the terms dx dx dx
given below, can be reduced to simple forms by the dy cos x
∴ =
following substitutions dx 2y − 1
S. No. Functions involving terms Substitutions

1. a2 − x 2 x = asin θ or acos θ
Differentiation of Parametric
Functions
2. a2 + x2 x = a tan θ or acot θ
2 2
If x = f (t ), y = g(t ), where t is parameter,
3. x −a x = asec θ or acosec θ
d
a+ x a− x
( g(t ))
4.
or
x = acos 2θ dy (dy / dt ) dt g'(t )
then = = =
a− x a+ x dx (dx / dt ) d f '(t )
( f (t ))
5. a2 + x 2 a2 − x 2 x2 = a2 cos 2 θ dt
2 2
or
a −x a2 + x 2 dy
We can always find in such cases, without first
6. 2x 2x x = tan θ dx
or
1+ x 2
1 − x2 dy dy/dt
eliminating the parameters by the formula = .
7. a sin x + b cos x a = r cos α , b = r sin α dx dx/dt

dy  1−x dy 3at 3at 2


EXAMPLE 5. Find , if y = sin  2 tan −1 . EXAMPLE 7. Find , when x = and y = .
dx  1+x  dx 1 + t3 1 + t3

a. 1 b. −1 c.
x
d.
−x t3 −4 t (2 − t 3) t (2 + t 3)
a. b. c. d. None of these
2
x −1 1− x 2
1 − 2t3 1 − 2t 3 1 + 2t 3
 1 − x t
Sol. d. y = sin  2 tan−1  Sol. b. We have, x = 3a ⋅
 1+ x  1+ t3
On putting x = cos 2θ , we get d d
 (1+ t 3) t −t ⋅ (1 + t 3)
 1 − cos 2θ  2sin2 θ  dx dt dt
y = sin 2 tan−1  = sin 2 tan
−1
 ∴ = 3a ⋅
 1+ cos 2θ   2cos2 θ  dt (1 + t 3) 2
= sin [ 2 tan−1 (tan θ) ] = sin 2θ = 1− cos2 2θ (1 + t 3) ⋅ 1 − t ⋅ 3t 2 1 − 2t 3
= 3a ⋅ 3 2 = 3a ⋅
(1 + t ) (1 + t 3) 2
⇒ y = 1− x 2 = (1 − x 2)1/ 2
t2
dy 1 d Again, y = 3a ⋅
∴ = (1 − x 2) −1/ 2 ⋅ (1 − x 2) 1+ t3
dx 2 dx
1 −x d 2 d
= ⋅ (0 − 2x ) = (1 + t 3) t − t2 (1 + t 3)
2 1− x 2
1− x 2 dy dt dt
∴ = 3a ⋅
dt (1 + t 3) 2
Differentiation of Infinite Series (1 + t 3) ⋅ 2t − t 2 ⋅ 3t 2 t ( 2 − t 3)
= 3a ⋅ 3 2 = 3a ⋅
Sometimes y is given as an infinite series and we have to (1 + t ) (1 + t 3) 2
find the derivative of y. To find this we use the fact that 3
dy dy / dt 3at ( 2 − t ) (1 + t )
3 2
t ( 2 − t 3)
if a term is deleted from infinite series, it remains ∴ = = ⋅ =
dx dx / dt (1 + t 3) 2 3a (1 − 2t 3) 1 − 2t 3
unaffected.
256 NDA/NA Pathfinder

x + e xK ∞ dy
Derivative of a Function with EXAMPLE 9. If y = e x + e , then is equal to
dx
Respect to Another Function 1 y y 1
a. b. c. d.
Let u and v be two functions of x such that y −1 y −1 1− y 1− y
u = f1 ( x) and v = f 2 ( x) x + ex K ∞
Sol. c. Let y = ex + e ⇒ y = ex + y
du du / dx f1 ′ ( x)
Then, = = ∴ log e y = log e ex + y
dv dv / dx f 2 ′ ( x)
⇒ log e y = ( x + y) log e e
EXAMPLE 8. What is the derivative of ⇒ log e y = x + y [Q log e e = 1]
 t   1  On differentiating both sides w.r.t. x, we get
sin −1   w.r.t. x cos −1  ?
2
 1+ t   1 +t2  1 dy dy
⋅ = 1+
y dx dx
a. 1 b. −1
c. 2 d. −2  1  dy  1 − y  dy
⇒  − 1 =1 ⇒   =1
   y  dx  y  dx
t −1  1 
Sol. a. Let u = sin−1   and v = cos   dy y
 1+ t 2  1+ t 2  ⇒ =
dx 1 − y
Again, let t = tan θ
 tan θ 
∴ u = sin−1   = sin−1 (sin θ) = tan−1 t Successive Differentiation
 sec θ 
of a Function
du 1
⇒ = dy d  dy 
dt 1 + t 2 The differential coefficient of i.e.   is called
dx dx  dx 
 1  −1  1 
and v = cos−1   = cos   the second differential coefficient or second derivative of
 1+ t 2   1+ tan2 θ 
y with respect to x which is denoted by
= cos−1 (cos θ) = tan−1 t
d2y
dv 1 or y′ ′ or y 2 or D 2 y.
⇒ = dx 2
dt 1 + t 2
Similarly, the second derivative may be differentiated to
du du dt
∴ = ⋅ =1 obtain the third derivative and so on.
dv dt dv
This process of finding the differential coefficient of a
Logarithmic Differentiation function is called the successive differentiation.
When a function contains another function as its
d 2x
exponent or the function is the product or quotient of EXAMPLE 10. If y = x + e x , then what is equal to?
number of functions, then first take logarithm and then dy 2
differentiation. ex ex ex
a. e x b. − x 3
c. − d. −
The process of differentiation after taking logarithm is (1 + e ) (1 + e x ) (1 + e x )2
known as logarithmic differentiation. Sol. b. Q y = x + ex
Important Logarithm Formulae ∴
dy
= 1 + ex ⇒
dx
=
1
(i) log ( mn) = log m + log n dx dy 1 + ex

 m d 2x ex dx
(ii) log   = log m − log n ⇒ 2 = − x 2 ⋅
 n dy (1 + e ) dy
ex 1 ex
(iii) log mn = n log m =− ⋅ = −
(1 + ex ) 2 (1 + ex ) (1 + ex ) 3
MATHEMATICS > Measurements of Angles and Trigonometric Rations 257

PRACTICE EXERCISE
dy  1 + x2 + 1 − x2
1. If y = sec tan−1 x, then is equal to
10. If y = tan−1  , then
dy
is
dx
2  1 + x 2 − 1 − x 2  dx
(a) x y/ (1 + x) (b) x y (1 + x )
2
equal to
(c) y/ (1 + x ) (d) x y /(1 + x 2 ) 1 −1 x −x
(a) (b) (c) (d)
 sin x  dy 1− x4 1− x4 1− x4 1− x4
2. If y = tan−1   , then is equal to
 1 + cos x  dx x   1 − x  
(a) 1/4 (b) 1/2 (c) 1 + cos 2 x (d) −1/ 4
11. If y = tan−1 + sin  2 tan−1   ,
1 + (1 − x 2 )   1 + x 

3. If y = { x + (1 + x 2 )} m , then (1 + x 2 ) y2 + xy1 − m 2 y then dy/dx is equal to
x 1 − 2x 1 − 2x 1
is equal to (a) (b) (c) (d)
(1 − x2 ) (1 − x 2 ) 2 (1 − x 2 ) (1 + x 2 )
(a) 2 (b) 1 (c) −1 (d) 0

4. If f ( x ) = loga (loga x ), then f ′ ( x ) is equal to  x 2 − y2  dy


12. If sin−1  2 2
 = log a , then is equal to
log a e log e a  x + y  dx
(a) (b)
x log e x x log a x x y x2 y
(a) (b) (c) (d)
log e a x y x2 y x
(c) (d)
x log e a 13. If x = a [cos t + log tan ( t/2)], y = a sin t, then
5. If x = a (1 − cos θ), y = a (θ − sin θ), then dy/dx is equal to
(a) tan t (b) cos t (c) sec t (d) cosec t
 dy 
  is equal to 14. If x = 2 log cot t and y = tan t + cot t , then
 dx  θ = π/2
dy
sin 2t + 1 is equal to
(a) −2 (b) −4 (c) 1 (d) −1 dx
6. The derivative of the function cot−1 [(cos 2x )1/ 2 ] at (a) cos 2 t (b) sin2 t (c) cos 2t (d) 2 cos 2 t
π 1 + x  dy
x= is 15. If y = log   , then is equal to
6 1 − x  dx
(a) (2 / 3)1/ 2 (b) (1/ 3)1/ 2
1 1 x
1/ 2 (a) (b) (c) (d) None of these
(c) 3 (d) 61/ 2 x (1 + x) x (1 − x) 1+ x
7. If y = tan−1 [{ (1 + x 2 ) − 1} / x ] , then dy
16. If x p y q = ( x + y ) p + q, then is equal to
(a) y′ (0) = 1 (b) y′ (0) = 1/ 2 dx
y py x qy
(c) y ′ (0) = 0 (d) None of these (a) (b) (c) (d)
x qx y px
(1 + x ) + (1 − x ) dy
8. If y = sin−1 , then is equal to  −1  4x 3  
2 dx d
17. cos  − x  is equal to
1 1 dx  27  
(a) (b) − 
2
(1 − x ) (1 − x 2 ) 3 1 −3 −1
(a) (b) (c) (d)
1 (9 − x )2
(9 − x ) 2
(9 − x )2
(9 − x 2 )
(c) − (d) None of these
2 (1 − x 2 )
 1 + x 2 − 1
9. If y = sin−1 (1 − x ) + cos−1 x, then dy/dx is equal 18. The derivative of tan−1   with respect
 x 
 
to
1 −1  2x 1 − x 2 
(a) (b) to tan−1   at x = 0, is
x (1 − x ) x(1 − x)  1 − 2x 2 
 
1
(c) (d) None of these (a) 1 / 8 (b) 1/ 4 (c) 1/2 (d) 1
x(1 + x)
258 NDA/NA Pathfinder

19. What is the derivative of sin2 x with respect to 30. If for a continuous function f , f ( 0) = f (1) = 0,
cos2 x ? f ′ (1) = 2 and g ( x ) = f ( ex ) e f (x )
, then g′ ( 0) is
(a) tan2 x (b) cot 2 x (c) −1 (d) 1 equal to
dy (a) 1 (b) 2 (c) 0 (d) 3
20. If x + y = 2, then what is at y = 1 equal to?
dx
d2y
(a) 5 (b) 4 (c) 2 (d) −1 31. If x = cos ( 2t ) and y = sin2 t, then what is
dx 2
21. What is the differentiation of logx x with respect equal to?
to log x? 1
(a) 0 (b) sin ( 2t ) (c) −cos (2t ) (d) −
(a) 0 (b) 1 (c) 1/ x (d) x 2
d 2 y d 2x
22. If y = cos x + cos x + cos x + cos x + K ∞ , 32. If y = e2x , then ⋅ is equal to
dx 2 dy 2
dy
then is equal to (a) e −2x (b) −2e −2 x (c) 2e −2 x (d) 1
dx
(a)
cos x
(b)
sinx
(c)
− sinx
(d)
− cos x 33. If y 2 = P ( x ) is a polynomial of degree 3, then
1− 2y 1− 2y 1− 2y 1− 2y
d  3 d 2 y
2 2 y  is equal to
1 1 d y dx  dx 2 
23. If x = t + , y = t − , then is equal to
t t dx 2 (a) P′ ′ ′ (x) + P′ (x) (b) P′ (x) P′ ′ ′ (x)
(a) −4t (t 2 − 1)−2 (b) −4t 3 (t 2 − 1)−3
(c) P (x) P′ ′ ′ (x) (d) a constant
(c) (t 2 + 1) (t 2 − 1)−1 (d) −4t 2 (t 2 − 1)−2 2
 log ( e/x )   3 + 2 log x 
d 2x
34. If y = tan−1  2
 + tan−1   , then
24. If y = sin x + e , then x
is equal to  log ( ex )   1 − 6 log x 
dy 2 d2y
x −1 sin x + e x is equal to
(a) (− sin x + e ) (b) dx 2
(cos x + e x )2
(a) 2 (b) 1 (c) 0 (d) −1
sin x − e x sin x + e x
(c) (d) 1/ x
(cos x + e x )3 (cos x + e x )3 35. If f ( x ) = x , then f ′ ′ ( e) is equal to
(a) e1/ e (b) e( 1 / e ) − 2 (c) 21/( e − 3)
(d) − e( 1 / e ) − 3
1
25. If φ is inverse of f and f ′ ( x ) = , then φ′ ( x )
1+ xn 36. What is the derivative of logx 5 with respect to
is equal to log5 x?
(a) 1 + x n (b) 1 + [f (x)]n (a) − (log 5 x)−2 (b) (log 5 x)−2 (c) − (log x 5)−2 (d) (log x 5)−2
(c) 1 + [φ (x)]n (d) None of these
d2y
1/ 4 1/ 2 1/ 4 dy 37. If e y + xy = e,then what is the value of at
26. If y = (1 + x ) (1 + x ) (1 − x ), then what is dx 2
dx
x = 0?
equal to?
(a) 1 (b) −1 (c) 0 (d) −2x (a) e −1 (b) e −2 (c) e (d) 1
dy
x
27. What is the derivative of x a 2 − x 2 + a 2 sin−1   ? 38. If 3x + 3 y = 3x + y
, then what is equal to?
 a dx
y
3x + − 3x 3x − y (3y − 1) 3x + 3y 3x + 3y
(a) a2 − x 2 (b) 2 a2 − x 2 (c) x 2 − a2 (d) 2 x 2 − a2 (a) y
(b) (c) x y
(d)
3 1− 3 x
3 −3 1 + 3x + y

28. If x = k (θ + sin θ) and y = k (1 + cos θ), then what 39. If f ( x ) = cos x , g ( x ) = log x and y = ( gof ) ( x ) , then
is the derivative of y with respect to x at θ = π/2 ? dy
what is the value of at x = 0?
(a) −1 (b) 0 (c) 1 (d) 2 dx
dy (a) 0 (b) 1 (c) −1 (d) 2
29. If 1 − x 2 + 1 − y 2 = a, then what is equal 2
dx dy d y
to?
40. If y = f ( x ), p = and q = , then what is
dx dx 2
1 − y2 d 2x
(a) (1 − x 2 ) (1 − y2 ) (b) 2 equal to?
1− x dy 2
1− x2 (a) −
q
(b) −
q
(c)
1
(d)
q
(c) (d) None of these
1− y 2 p2 p3 q p2
MATHEMATICS > Measurements of Angles and Trigonometric Rations 259

Directions (Q. Nos. 49-50) Consider,


41. If x = sin t − t cos t and y = t sin t + cos t , then what
dy π x − 2 x −1
is at point t = ? f ( x) = ⋅x
dx 2 x −1−1
π π 49. Find the value of f ′ (10).
(a) 0 (b) (c) − (d) 1
2 2
(a) 1 (b) 0 (c) 10 (d) −1
42. If the curve x + y = 1, then what is the value 3
50. Find the value of f ′   .
dy  1 1  2
of at  ,  ?
dx  4 4 (a) 0 (b) 1 (c) −1 (d) None of these
1
(a) (b) 1 (c) −1 (d) 2
2 Directions (Q. Nos. 51-52) If y = ( x + 1 + x 2 ) n, then
1 dy
43. If y = , then what is equal to? dy
log10 x dx 51. Find .
dx
(a) x (b) x log e 10
1 y
(log x 10)2 (log10 e ) (a) (b)
(c) − (d) x log 10 e 1+ x 2
1+ x2
x
ny
dy (c) (d) None of these
44. If y = sin−1 x + sin−1 1 − x 2 , then what is 1+ x2
dx
equal to? d2y dy
(a) cos −1 x + cos −1 1 − x 2 (b)
1
+
1 52. Find 2
(1 + x 2 ) + x .
cos x cos 1 − x 2 dx dx
π (a) y (b) x (c) n2 y (d) nx
(c) (d) 0
2
45. If f ( x ) = esin (log cos x ) and g ( x ) = log cos x , then what Directions (Q. Nos. 53-55) Consider function
is the derivative of f ( x ) g ( x )? u = e x sin x and v = e x cosx , then
(a) f (x) cos [g (x)] (b) f (x) sin [g (x)]
du dv
(c) g (x) cos [f (x)] (d) g (x) sin [f (x)] 53. v −u is equal to
dx dx
46. The first derivative of the function (a) u + v (b) v 2 (c) u 2 + v 2 (d) None of these
 −1  1+ x  d 2u
cos  sin  + x x  with respect to x at 54. is equal to
  2   dx 2
x = 1 is (a) 2u (b) 2v (c) u (d) v
3 −1 2
I. f′ (1) = II. f′ (2) = + 4 (log 2 + 1) d v
4 2 6
55. is equal to
dx 2
Select the correct answer using the codes given (a) 2u (b) 2v (c) −2v (d) −2u
below.
(a) Only I (b) Only II Directions (Q. Nos. 56-58) Let f ( x) be a polynomial
(c) Both I and II (d) None of these function ax 2 + bx + c of second degree. If f (1) = f ( −1)
2
dy  and a, b, c are in AP, then
47. If y = a sin x + b cos x, then y 2 +   is a
 dx  56. Find the value of f ′ ( a ).
I. Function of x II. Function of y III. Constant (a) 2 a2 (b) 2 b 2 (c) 2ab (d) 2ac
Select the correct answer using the codes given 57. Find the value of f ′ ( c).
below. (a) 2ab (b) 2ac (c) 2bc (d) 2c 2
(a) Only I (b) Only II
(c) Only III (d) None of these 58. f ′ ( a ), f ′( b) and f ′ ( c) are in
(a) AP (b) GP (c) HP
e 2x + e −2x dy (d) Arithmetico-Geometric Progression
48. If y = , then equals to
e 2x − e −2x
dx
−8 −4 8 Directions (Q Nos. 59-61) Let f ( x) = e x , g ( x) = sin−1 x
I. II. x III.
(e2x − e−2x )2 (e − e− x )2 (e2x − e−2x )2 and h( x) = f [ g ( x)]
Select the correct answer using the codes given h ′ (x)
59. Find the value of .
below. h( x )
−1 1 1
(a) Only I (b) Only II (a) esin x
(b) (c) sin−1 x (d)
(c) Only III (d) None of these 1− x 2 1 − x2
260 NDA/NA Pathfinder

60. Find the value of [ f ( x ) ⋅ g ( x )] ′. 70. The derivative of sec2 x w.r.t. tan2 x is e 2013 I
  (a) 1 (b) 2
1 (d) 2 sec 2 x tan x
(a) e x  + sin−1 x (b) e x ⋅ sin−1 x (c) 2sec x tan x
 1 − x2 
  d2y b
x 1 71. If y = sin( ax + b), then what is 2
at x = − ,
(c) e (d) None of these dx a
1 − x2 where a and b are constants and a ≠ 0? e 2013 I
(a) 0 (b) −1 (c) sin(a − b ) (d) sin(a + b )
61. Find the value of h ′ ( 0) ⋅ f ( 0). dy
72. If y = x x , then what is the value of at x = 1?
(a) 1 (b) −1 (c) 0 (d) None of these dx
e 2013 I
PREVIOUS YEARS’ QUESTIONS (a) 0 (b) 1 (c) − 1 (d) 2
dy
73. If 2x 3 − 3 y 2 = 7, then what is equal to
62. If f ( x ) = 2x , then what is f ′ ′ ( x ) is equal to? dx
e 2012 I ( where, y ≠ 0)? e 2013 II
(a) 2 x (loge 2 )2 (b) x (x − 1) 2 x − 2 x2 x x2
(a) (b) (c) (d) None of these
(c) 2 x + 1 (log 2 ) (d) 2 x (log10 2 )2 2y 2y y
74. What is the derivative of sin(sin x )? e 2013 II
x+1 dy
63. If y = , then what is the value of ? (a) cos(cos x) (b) cos(sin x)
x−1 dx (c) cos(sin x)cos x (d) cos(cos x)cos x
e 2012 I
(a)
−2
(b)
−2
(c)
2
(d)
2 75. What is the derivative of x 3 w.r.t. x 2? e 2013 II
x −1 2 2
(x − 1) 2 3x 3
(x − 1) (x − 1) (a) 3x (b) (c) x (d)
2 2
64. What is the rate of change of x 2 + 16 w.r.t. x 2 76. If f( x ) = 2x 2 + 3x − 5, then what is f ′( 0) + 3 f ′( −1)
at x = 3? e 2012 I equal to? e 2013 II
1 (a) −1 (b) 0 (c) 1 (d) 2
(a) 1/5 (b) 1/10 (c) 1/20 (d)
15
77. What is the derivative of x − 1 at x = 2? e 2013 II
65. If y = cos t and x = sin t , then what is the value of (a) −1 (b) 0 (c) 1 (d) 2
dy
? dz
dx e 2012 I 78. If z = fof ( x ) where f( x ) = x 2, then what is
(a) xy (b) x / y (c) − y/x (d) −x / y dx
equal to? e 2014 I
66. If f( x ) = 2sin x , then what is the derivative of f( x )? (a) x 3
(b) 2 x 3
(c) 4x 3
(d) 4x 2

e 2012 II
1 + cos x
(a) 2sin x In 2 (b) (sin x) 2sin x
−1
79. What is the derivative of ?
−1
1 − cos x e 2014 I
(c) (cos x) 2sin x (d) None of these 1 x 1 x
(a) sec 2 (b) − cosec 2
2 2 2 2
67. If y = In ( emx + e − mx ), then what is the value of x
dy (c) − cosec 2 (d) None of these
at x = 0? 2
dx e 2012 II
dy
(a) −1 (b) 0 (c) 1 (d) 2 80. If y = x ln x + xex , then what is the value of at
dx
dy x
68. If x m + y m = 1, such that = − , then what x = 1? e 2014 II
dx y (a) 1 + e (b) 1 − e (c) 1 + 2 e (d) None of these
should be the value of m? e 2012 II
(a) 0 (b) 1 (c) 2 (d) − 1 Directions (Q. Nos. 81-82) Consider the curve
69. Consider the following statements x = a(cos θ + θ sin θ) and y = a(sin θ − θ cos θ). e 2014 II
dy dy
I. If y = ln (sec x + tan x), then = sec x 81. What is equal to?
dx dx
dy (a) tan θ (b) cot θ (c) sin 2 θ (d) cos 2 θ
II. If y = ln (cosec x − cot x ), then = cosec x
dx 2
d y
82. What is equal to?
Which of the above statement(s) is/are correct? dx 2
e 2012 II (a) sec 2 θ (b) – cosec 2 θ
(a) Only I (b) Only II sec 3 θ
(c) (d) None of these
(c) Both I and II (d) Neither I nor II dθ
MATHEMATICS > Measurements of Angles and Trigonometric Rations 261

83. Eliminating the arbitrary constants B and C in 91. The derivative of ln ( x + sin x ) with respect to
2
the expression y = (Cx − 1)3/ 2 + B, we get ( x + cos x ) is e 2015 II
3C 1 + cos x 1 − cos x
e 2014 II (a) (b)
2 2 (x + sin x ) ( 1 − sin x ) (x + sin x ) (1 + sin x )
 dy  d 2 y dy d 2 y dy
(a) x 1 +    = (b) 2 x   = 1 +   (c)
1 − cos x
(d)
1 + cos x
 dx   dx 2  dx  dx 2  dx 
  (x − sin x ) (1 + cos x ) (x − sin x ) (1 − cos x )
2
dy d 2 y dy d 2y
(c)   =1 (d)   + 1 = dy y
 dx  dx2  dx  dx 2 92. If x a y b = ( x − y )a + b, then the value of − is
dx x
84. What is the differential coefficient of equal to e 2015 II
a b
 x2 − 5 , x ≤ 3 (a) (b) (c) 1 (d) 0
f ( x )=  at x = 12 ? b a
 x + 13 , x > 3 e 2014 II
 1 + sin x + 1 − sin x 
(a) 5/2 (b) 5 93. If y = cot−1   , where
(c) 1/5 (d) 1/10  1 +sin x − 1 − sin x 
π dy
0 < x < , then is equal to
Directions (Q. Nos. 85-87) Consider the parametric 2 dx e 2015 II
a(1 − t 2) 2 at 1
equations x = ,y= . (a) − (b) 2
1+ t 2
1+ t 2 2
e 2015 I
(c) sin x + cos x (d) sin x − cos x
85. What does the equation represent?
94. If y = log10 x + logx 10 + logx x + log10 10,
(a) It represents a circle of diameter a
(b) It represents a circle of radius a  dy 
then what is   equal to?
(c) It represents a parabola  dx  x = 10
e 2016 I
(d) None of the above (a) 10 (b) 2 (c) 1 (d) 0
dy
86. What is equal to? Directions (Q. Nos. 95-96) Consider the function
dx
y y x x f ( x) = | x 2 − 5 x + 6 |. e 2016 I
(a) (b) − (c) (d) −
x x y y
95. What is f ′ ( 4) equal to?
d2y (a) −4 (b) −3 (c) 3 (d) 2
87. What is equal to?
dx 2 96. What is f ′ ′ (2.5) equal to?
a2 a2 a2 a2
(a) (b) (c) − (d) − (a) −3 (b) −2 (c) 0 (d) 2
y2 x2 x2 y3
Directions (Q. Nos. 97-100) Let f : R → R be a
Directions (Q. Nos. 88-89) Given that, function such that
d 1 + x 2 + x 4  x −1
  = Ax + B (cos x + e ) . f ( x) = x 3 + x 2f ′ ( 1) + xf ′′(2) + f ′′′(3) for x ∈R.
dx  1+ x + x 2  e 2015 I e 2016 I
88. What is the value of A? 97. What is f(1) equal to?
(a) −1 (b) 1 (a) −2 (b) −1 (c) 0 (d) 4
(c) 2 (d) 4
98. What is f ′ (1) equal to?
89. What is the value of B?
(a) −6 (b) −5 (c) 1 (d) 0
(a) −1 (b) 1
(c) 2 (d) 4 99. What is f ′ ′ ′ (10) equal to?
 1 + x 2 − 1 (a) 1 (b) 5 (c) 6 (d) 8
90. What is the derivative of tan−1   with
 x  100. Consider the following
 
I. f (2) = f (1) − f (0) II. f ′ ′ (2) − 2 f ′ (1) = 12
respect to tan−1 x? e 2015 I
1 Which of the above is/are correct?
(a) 0 (b) (a) Only I (b) Only II
2
(c) 1 (d) x (c) Both I and II (d) Neither I nor II
262 NDA/NA Pathfinder

ANSWERS
1 d 2 b 3 d 4 a 5 c 6 a 7 b 8 c 9 b 10 d
11 c 12 d 13 a 14 d 15 d 16 a 17 c 18 b 19 c 20 d
21 a 22 b 23 b 24 c 25 c 26 b 27 b 28 a 29 d 30 b
31 a 32 b 33 c 34 c 35 d 36 a 37 b 38 b 39 a 40 b
41 a 42 c 43 c 44 d 45 a 46 c 47 c 48 a 49 a 50 b
51 c 52 c 53 c 54 b 55 d 56 a 57 b 58 a 59 b 60 a
61 a 62 a 63 b 64 b 65 d 66 d 67 b 68 c 69 c 70 a
71 a 72 b 73 c 74 c 75 b 76 b 77 c 78 c 79 b 80 c
81 a 82 c 83 b 84 d 85 b 86 d 87 d 88 c 89 a 90 b
91 a 92 d 93 a 94 d 95 c 96 b 97 d 98 b 99 c 100 c

HINTS AND SOLUTIONS


1. (d) y = sec tan −1 x 7. (b) Put x = tanθ 10. (d) Put x 2 = cos 2θ,
dy 1 sec θ − 1 1 + cos 2θ = 2 cos 2 θ,
⇒ = sec tan −1 x⋅ tan tan −1 x ⋅ ∴ y = tan −1  
dx 1+ x2  tan θ 
1 xy 1 − cos 2θ = 2 sin 2 θ
= y⋅x⋅ = −1  1 − cos θ  cos θ + sin θ 
1+ x2 1+ x2 = tan 
 sin θ 
 ∴ y = tan −1  
 cos θ − sin θ 
2. (b)
θ 1 1
y = tan −1  tan  = θ = tan −1 x = tan −1 tan [ π/4 + θ]
3. (d) y = m { x + ( 1 + x 2 )}m − 1 ⋅  2 2 2
1
= π/4 + θ
dy 1 1 1
⇒ = ⋅ = at x = 0 π 1
 1⋅ 2x  my dx 2 1 + x 2 2 y = + cos −1 x 2
1 + = 4 2
 2 ( 1 + x 2 )  1+ x2
8. (c) Put x = cos θ, dy 1 −1
∴ = ⋅ ⋅ 2x
∴ y 2 ( 1 + x 2 ) = m2 y 2 dx 2 ( 1 − x 4 )
1 1 + cos θ = 2 cos 2 (θ /2 )
Again, differentiating w.r.t. x, we get x
1 − cos θ = 2 sin 2 ( θ /2 ) =−
2 y y ( 1 + x 2 ) + y 2 ⋅ 2x = m 2 ⋅ 2 yy (1− x 4 )
1 2 1 1 ∴ y = sin −1 [( 1/ 2 ) (cos θ /2 + sin θ /2)]
⇒ y ( 1 + x 2 ) + xy − m 2 y = 0 cos θ 
2 1 = sin −1 sin (θ /2 + π /4) 11. (c) Put x = cosθ, y = tan −1  
 1 + sin θ 
4. (a) = θ / 2 + π /4
1 π  1 − cos θ  
5. (c) y = cos −1 x + + sin  2 tan −1  
2 4   1 + cos θ  
dy 1
6. (a) =− dy −1
dx 1 + cos 2x ∴ =  sin φ    θ
dx 2 ( 1 − x 2 ) = tan −1  −1
 + sin  2 tan tan   
1  1 + cos φ    2 
= ( −2 sin 2x )
2 (cos 2x ) π
9. (b) Put x = cos 2 θ ⇒ θ = cos −1 x [where, φ = − θ]
π 1 2
Now, put x = , cos 2x = ∴ y = sin −1 ( 1 − x ) + cos −1 ( x )
6 2 = tan −1 {tan ( φ / 2)} + sin ( 2 ⋅ θ/2)
= sin −1 ( sin 2 θ ) + cos −1 ( cos 2 θ ) = ( φ / 2) + sin θ
3
and sin 2x =
2 ⇒ y = sin −1
(sin θ) + cos −1
(cos θ) = π /4 − θ /2 + ( 1 − cos 2 θ)
∴  dy 
  π ⇒ y = θ + θ = 2θ π 1 −1
 dx  x = − cos x + ( 1 − x 2 )
=
6 ⇒ y = 2 cos −1 x 4 2
1 1 3 2 dy 2 1 dy 1 1 1
=− ⋅ ( −2 ) = ∴ =− × ∴ = + ( −2 x )
 1 + 1 1 2 3 dx 1− x 2 x dx 2 ( 1 − x 2 ) 2 ( 1 − x 2 )
  2
 2 2 dy −1
⇒ = 1 − 2x
dx x( 1 − x ) =
2 (1− x 2 )
MATHEMATICS > Measurements of Angles and Trigonometric Rations 263

x2 − y2 16. (a) On taking log both sides, we get dx


23. (b) We have,
1 dy
= 1− 2 , = 1+ 2
1
12. (d) = sin (log a ) = λ ,
x2 + y2 p log x + q log y = ( p + q ) log ( x + y ) dt t dt t
1 1 dy 2  
∴ x 2 ( 1 − λ) = y 2 ( 1 + λ) …(i) ⇒ p +q dy t + 1  2 
x y dx ∴ = 2 = 1+ 2
dy dx t − 1  t − 1
2x ( 1 − λ ) = 2 y ( 1 + λ ) …(ii)  
dx 1  dy 
= ( p + q) ⋅ 1+  d2y d  dy  dt
On dividing Eq. (ii) by Eq. (i), we get x+ y  dx  and =  ⋅
2
dx dt  dx  dx
2 2 dy p p + q  p + q q  dy
= − = −  −1 t2 −4 t 3
x y dx x x + y x + y y  dx = 2⋅ 2 2
⋅ 2t × 2 = 2
( t − 1) t − 1 ( t − 1)3
dy y py − qx py − qx dy
∴ = ⇒ = ⋅
dx x x ( x + y ) y( x + y ) dx 24. (c) We have, y = sin x + e x
dy dx
t dy y = cos x + e x ⇒ = (cos x + e x )− 1
13. (a) Given, x = a  cos t + log tan    ∴ = dx dy
  2   dx x
d 2x dx
and y = a sin t  −1  4 x 3  ⇒ 2 = − (cos x + e x )−2( − sin x + e x )
d dy dy
17. (c)  cos  − x
 t dx 27
sec 2     d 2x sin x − e x x −1
dx  2 ⇒ = . (cos x + e )
⇒ = a  − sin t + 1  4 (3 x 2 ) − dy 2 (cos x + e x )2
dt 2 tan 
t =−  1
 3 2  27  sin x − e x
 2  4x  =
dy 1−  − x
and = a cos t  27  (cos x + e x )3
dt −1
−27  4 x 2 − 1 25. (c) By given condition, φ ( x ) = f (x )
dx 1 
⇒ = a  − sin t + =   ∴ f [ φ ( x )] = x
dt  sin t  729 − ( 4x 3 − 27x )2  9 
On differentiating w.r.t. x, we get
dy = −3( 4x 2 − 9)
and = a cos t f ′ [ φ ( x )] φ′ ( x ) = 1
dt ( 4x 2 − 9)2 ( 9 − x 2 )
1
dx a cos 2 t dy −3( 4x 2 − 9) −3 or φ′ ( x ) = = 1 + [ φ ( x )]n ,
⇒ = and = a cos t = = f ′ ( φ ( x ))
dt sin t dt 2
( 4x − 9) ( 9 − x ) 2 2
(9 − x )
dy a cos t by definition of f ′ ( x ).
∴ = = tan t 18. (b) On putting x = tan θ, we have
dx cos 2 t 26. (b) y = ( 1 + x 1/4 ) ( 1 + x 1/2 ) ( 1 − x 1/4 )
a θ θ 1
sin t y = tan −1  tan  = = tan −1 x = ( 1 + x 1/ 2 ) ( 1 − x 1/ 2 ) = 1 − x
 2 2 2 dy
14. (d) x = 2 log cot t , ∴ = −1
dy 1 1 1 dx
sin 2 t + cos 2 t 2 ∴ = ⋅ = at x = 0
y= = dx 2 1 + x 2 2 dy 1
sin t cos t sin 2t 27. (b) =x⋅ ⋅ ( −2 x )
dx 2 Again, putting x = sin φ, we get dx 2 a − x2
2

= ⋅ ( − cosec 2 t )  2 sin φ cos φ  a2 1


dt cot t z = tan −1   + a2 − x 2 + ⋅
−2 −4  1 − 2 sin 2 φ  x 2 a
= = 1− 2
sin t ⋅ cos t sin 2t  sin 2φ  a
= tan −1   −x 2 a2 ⋅ a 1
dy −2 4 cos 2 t  cos 2φ  = + a −x +2 2

= ⋅ 2 cos 2t = − 2 2 2 2 a
dt sin 2 2t sin 2 2t = tan −1 (tan 2φ) = 2φ = 2 sin −1 x
a −x a −x
a2 − x 2
dy cos 2 t dz 2 = + a2 − x 2 = 2 a2 − x 2
∴ = ⇒ = = 2 at x = 0 a2 − x 2
dx sin 2t dx 1− x2
dy 28. (a) Q x = k (θ + sin θ)
⇒ sin 2 t + 1 = cos 2 t + 1 = 2 cos 2 t dy dy dz 1/ 2 1
∴ = ⋅ = = and y = k ( 1 + cos θ)
dx dx dx dx 2 4 dx dy
⇒ = k ( 1 + cos θ) and = − k sin θ
1+ x 
15. (d) Given, y = log  
19. (c) 20. (d) 21. (a) dθ dθ
 1− x  dy − k sin θ
22. (b) y = cos x + cos x + cos x +…∞ , ∴ =
y = log ( 1 + x ) − log ( 1 − x ) dx k ( 1 + cos θ)
2
dy 1 1 1  1  ∴ y= cos x + y ⇒ y = cos x + y θ θ
= . − −  −2 sin cos
dx 1 + x 2 x 1− x  2 x  On differentiating w.r.t. x, we get = 2 2 = − tan θ
dy dy 2 θ 2
1  1 1  2y = − sin x + 2 cos
= + dx dx 2
2 x  1 + x 1 − x 
dy dy π
1 1− x + 1+ x  ⇒ sin x = ( 1 − 2 y ) ⇒   = − tan = − 1
= dx  dx  θ = π 4
2 x  1− x  2
dy sin x
1 2 1 ⇒ =
= . = dx 1 − 2 y 29. (d)
2 x 1− x x(1− x ) 30. (b) g ( x ) = f ( e x ) e f (x )
264 NDA/NA Pathfinder

∴ g′ (x ) = f ′ (ex ) ⋅ ex ⋅ e f (x ) 1 dy
log y = log x 3x log 3 + 3 y log 3
x
+ f (e ) ⋅ e f (x )
⋅ f ′ (x ) x dx
dy 
On putting x = 0,
1
⋅y = ⋅ −
1 1 1
⋅ log x = 3(x + y ) log 3  1 + 
y 1 x x x2  dx 
f ( 0) = f ( 1) = 0, f ′ ( 1) = 2, we get x y dy x + y dy
1 1 − log x ⇒ 3 +3 =3 + 3(x + y )
g′ ( 0) = 2 ⋅ 1 ⋅ 1 + 0 = 2 ⇒ ⋅y = dx dx
y 1
x2 dy
31. (a) ⇒ ( −3 x + y + 3 y ) = 3 x + y − 3 x
1 − log x 
⇒ y = y   dx
32. (b) We have, y = e 2 x 1  x2  dy 3x (3 y − 1) 3x − y (3 y − 1)
⇒ = =
dy 1 − log x  dx 3 y ( 1 − 3x ) ( 1 − 3x )
⇒ = e2x ⋅ 2 ⇒ y = y   + y
dx 2 1 
x2 
d2y  2  1  39. (a) ∴ y = gof ( x ) = g { f ( x )}
⇒ = 4e 2 x x −
  x 
− ( 1 − log x )( 2x )

dx 2 = log (cos x )
 4 
1  x  dy 1
Now, log y = 2x , x =log y ∴ = ( − sin x ) = − tan x
2   dx cos x
dx 1 d 2x 1 1 dy
= , =− = − 4x =y  1 − log x  y( − x − 2x + 2x log x )
+ ⇒   = − tan 0 = 0
dy 2 y dy 2 2y2 2e 1
x2  x4  dx  x =0
d 2 y d 2x 1 = y 
1 − log x  y( 2x log x − 3x ) dy dx 1
∴ ⋅ = 4e 2 x ×  − 4 x   + 40. (b) We have, = p⇒ =
dx 2 dy 2  2e  1 
x2  x4 dx dy p
= − 2e − 2 x e1 / e ( 2 e − 3 e ) − e1 / e d 2x 1 dp 1 d  dy 
⇒ y (e) = 0 + = 3 ∴ =− 2 ⋅ =− 2 ⋅  
2
33. (c) Q y = P ( x ) …(i)
2
e4 e dy 2 p dy p dy  dx 
dy = − e (1/e ) − 3
2y = P′ (x ) …(ii) 1 d 2 y dx q
dx 36. (a) Let u = log x 5 and v = log x =− ⋅ ⋅ =− 3
5 p 2 dx 2 dy p
d2y dy
2 log e 5 log e x
and 2 y 2 + 2   = P ′ ′ ( x ) ⇒ u= and v =
41. (a) We have, x = sin t − t cos t
dx  dx  log e x log e 5
2 and y = t sin t + cos t
d2y dy On differentiating w.r.t. x, we get
⇒ 2y = P ′′ ( x ) − 2   log e 5 ⇒
dx
= cos t − 1⋅ cos t + t sin t = t

You might also like